You are on page 1of 880

EBD_7367

• Corporate Office : 45, 2nd Floor, Maharishi Dayanand Marg, Corner Market,
Malviya Nagar, New Delhi-110017
Tel. : 011-49842349 / 49842350

Typeset by Disha DTP Team

DISHA PUBLICATION
ALL RIGHTS RESERVED

© Copyright Publisher

No part of this publication may be reproduced in any form without prior permission of the author and the publisher. The
author and the publisher do not take any legal responsibility for any errors or misrepresentations that might have crept in.
We have tried and made our best efforts to provide accurate up-to-date information in this book.

For further information about the books from DISHA


Log on to www.dishapublication.com or email to info@dishapublication.co.in
CONTENTS

SECTION-A : MATHEMATICS A- 1– A- 400

1. Number System 1-20


2. HCF and LCM 21-30
3. Decimal Fractions 31-34
4. Powers & Roots 35-47
5. Percentage 48-54
6. Ratio & Proportion 55-66
7. Average 67-70
8. Simple & Compound Interest 71-77
9. Profit & Loss 78-83
10. Time , Speed & Distance 84-98
11. Time & Work 99-109
12. Logarithm 110-113
13. Basic Operation and Factorization 114-136
14. Linear Equations in One and Two Variables 137-148
15. Quadratic Equations 149-165
16. Set Theory 166-170
17. Trigonometric Ratios & Identities 171-209
18. Height & Distance 210-226
19. Area and Perimeter 227-251
20. Volume and Surface Area 252-284
21. Lines & Angles 285-296
22. Triangles and its Properties 297-331
23. Quadrilateral and Polygon 332-350
24. Circle 351-377
25. Statistics 378-400
EBD_7367
SECTION-B : ENGLISH B- 1– B- 204

26. Synonyms 1-20

27. Antonyms 21-38

28. Spotting Errors 39-62

29. Ordering of Words in a Sentence 63-95

30. Ordering of Sentence 96-141

31. Sentence Improvement 142-152

32. Selecting Words 153-163

33. Reading Comprehension 164-204

SECTION-C : GENERAL STUDIES C-1–C-272

34. Physics 1-15

35. Chemistry 16-32

36. Biology 33-52

37. History 53-96

38. Geography 97-148

39. Polity 149-193

40. Economics 194-210

41. General Knowledge 211-237

42. Miscellaneous 238-272


Section-A : Mathematics

C HA P T E R
NUMBER SYSTEM
1
1. By adding x to 1254934, the resulting number becomes (c) divisible by 2 and 7 but not divisible by 11
divisible by 11, while adding y to 1254934 makes the (d) divisible by 3, 7 and 11
resulting number divisible by 3. Which one of the 9. Which one of the following is correct?
following is the set of values for x and y? [2007-I] The sum of two irrational numbers [2009-II]
(a) x = 1, y = 1 (b) x = 1, y = – 1 (a) is always a natural or irrational
(c) x = – 1, y = 1 (d) x = – 1, y = – 1 (b) may be rational or irrational
2. A ten-digit number is divisible by 4 as well as by 5. What (c) is always a rational number
could be the possible digit at the ten’s place in the given (d) is always an irrational number
number? [2007-I] 10. If the numbers q, q + 2 and q + 6 are all prime, then what
(a) 0, 1, 2, 4 or 6 (b) 1, 2, 4, 6 or 8 can be the value of 3q + 9? [2009-II]
(c) 2, 3, 4, 6 or 8 (d) 0, 2, 4, 6 or 8 (a) Only 18 (b) Only 42
3. What least number should be subtracted from 26492518, (c) Only 60 (d) Both (b) and (c)
so that the resulting number is divisible by 3 but not by 11. Let p denotes the product 2 × 3 × 5 ... 59 × 61 of all primes
9? [2007-I] from 2 to 61. Consider the sequence p + n (2 £ n £ 59).
(a) 1 (b) 3 What is the number of primes in this sequence (where n
(c) 4 (d) 7 is a natural number)? [2009-II]
4. Which one of the following numbers is a composite (a) 0 (b) 16
(c) 17 (d) 58
numbers? [2007-I]
12. A three-digit number has digits h, t, u (from left to right)
(a) 589 (b) 571
with h > u. If the digits are reversed and the number thus
(c) 569 (d) 563
formed is subtracted from the original number, the unit’s
5. Which one of the following is correct ?
digit in the resulting number is 4. What are the other two
an + bn is divisible by a – b [2009-II]
digits of the resulting number from left to right?
(a) for all integral values of n
[2009-II]
(b) when n is an even integer
(a) 5 and 9 (b) 9 and 5
(c) when n is an odd integer
(c) 5 and 4 (d) 4 and 5
(d) for no integral value of n
13. Assertion (A) : Zero is a whole number.
6. If x and y denote respectively, the area and the sum of the Reason (R) : Every integer is a whole number.
length of diagonals of a rectangle with length 1 unit and [2009-II]
1 (a) A and R are correct but R is correct explanation of A
breadth unit, then which one on the following is (b) A and R are correct but R is not correct explanation
2
correct? of A
[2009-II] (c) A is correct but R is wrong
(a) x and y are rational (d) A is wrong but R is correct
(b) x is rational and y is irrational 14. One dividing 4996 by a certain number, the quotient is 62
(c) x is irrational and y is rational and the remainder is 36. What is the divisor? [2009-II]
(d) x and y are both irrational (a) 80 (b) 85
7. In a divisible operation, the division is 5 times the (c) 90 (d) 95
quotient and twice the remainder. If the remainder is 15, 15. Which of the following numbers is a prime? [2009-II]
then what is the dividend? [2009-II] (a) 667 (b) 861
(a) 175 (b) 185 (c) 481 (d) 331
(c) 195 (d) 250 16. A number, when divided by 987, gives a remainder 59.
8. Which one of the following is correct? When the same number is divided by 21, what is the
The number 222222 is [2009-II] remainder? [2009-II]
(a) divisible by 3 but not divisible by 7 (a) 21 (b) 19
(b) divisible by 3 and 7 but not divisible by 11 (c) 17 (d) 15
EBD_7367
2
A- Number System

17. Which one of the following statements is always correct? 28. There are four numbers forming a GP in which the third
[2009-II] term is greater than the first by 9 and the second term is
(a) The square of a prime number is prime greater than the fourth by 18. What is the first term?
(b) The sum of two square numbers is a square number [2009-II]
(c) The number of digits in a square number is even (a) 2 (b) 3
(d) The product of two square numbers is square number (c) – 2 (d) – 3
18. The set of integers is closed with respect to which one 29. The angles of a triangle are in AP and the greatest angle
is double the least. What is the ratio of angles in the
of the following ? [2009-II]
radian measure? [2009-II]
(a) Addition only
(a) 2 : 3 : 4 (b) 1 : 2 : 3
(b) Multiplication only (c) 3 : 3 : 6 (d) 4 : 5 : 7
(c) Both addition and multiplication 30. If p is an integer, then every square integer is of the form
(d) Division [2009-II]
19. What least value must be given to *, so that the number (a) 2p or (4p – 1) (b) 4p or (4p – 1)
8798546*5 is divisible by 11? [2009-II] (c) 3p or (3p + 1) (d) 4p or (4p + 1)
(a) 0 (b) 1 31. What least value must be given to Ä, so that the number
(c) 2 (d) 3 84705Ä2 is divisible by 9? [2009-II]
20. What is the sum of all prime numbers between 100 and (a) 0 (b) 1
120? [2009-I] (c) 2 (d) 3
(a) 652 (b) 650 32. When a polynomial is divided by a linear polynomial,
(c) 644 (d) 533 then what is the remainder? [2009-II]
21. What is the sum of positive integers less than 100 which (a) Constant polynomial only
leave a remainder 1 when divided by 3 and leave a (b) Zero polynomial only
remainder 2 when divided by 4? [2009-II] (c) Either constant or zero polynomial
(d) Linear polynomial
(a) 416 (b) 620
33. The remainder on dividing given integers a and b by 7
(c) 1250 (d) 1314
are, respectively 5 and 4. What is the remainder when ab
22. What is the total number of three digit three digit is divided by 7? [2010-I]
numbers with unit digit 7 and divisible by 11?[2009-II] (a) 3 (b) 4
(a) 6 (b) 7 (c) 5 (d) 6
(c) 8 (d) 9
34. If r and s are any real numbers such that 0 £ s £ 1 and
23. If we divide a positive integer by another positive integer, r + s = 1, then what is the maximum value of their product?
what is the resulting number? [2009-II] [2010-I]
(a) It is always a natural number
(b) It is always an integer 3
(a) 1 (b)
(c) It is a rational number 4
(d) It is an irrational number 1 1
24. The product of two alternate odd integers exceeds three (c) (d)
2 4
times the smaller by 12. What is the larger number? 35. Which one of the following numbers is not a square of
[2009-II] any natural numbers ? [2010-I]
(a) 3 (b) 5 (a) 5041 (b) 9852
(c) 7 (d) 9 (c) 1936 (d) 6241
25. Consider the following statements: 36. A three-digit number is divisible by 11 and has its digit
A number a1 a2 a3 a4 a5 is divisible by 9, if in the unit’s place equal to 1. The number is 297 more
I. a1 + a2 + a3 + a4 + a5 is divisible by 9. than the number obtained by reversing the digits. What
II. a1 – a2 + a3 – a4 + a5 is divisible by 9. is the number ? [2010-I]
Which of the above statements is/are correct? [2009-II] (a) 121 (b) 231
(a) Only I (b) Only II (c) 561 (d) 451
(c) Both I and II (d) Neither I nor II 37. Consider the following assumption and two statements:
26. What is the last digit in the expansion of (2457)754? Assumption: A number, ‘ABCDE’ is divisible by 11.
[2009-II] Statement I: E – D + C – B + A is divisible by 11.
(a) 3 (b) 7 Statement II: E – D + C – B + A = 0
Which one of the following is correct ? [2010-I]
(c) 8 (d) 9
(a) Only statement I can be drawn from the assumption
27. When a natural number n is divided by 4, the remainder
(b) Only statement II can be drawn from the assumption
is 3. What is the remainder when 2n is divided by 4? (c) Both the statements can be drawn from the
[2009-II] assumption
(a) 1 (b) 2 (d) Neither of the statements can be drawn from the
(c) 3 (d) 6 assumption
Number System A-3

38. What can be said about the expansion of 212n – 64n, 50. Consider the following statements:
where n is positive integer ? [2010-I] If p is a prime such that p + 2 is also a prime, then
(a) Last digit is 4 (b) Last digit is 8 I. p (p + 2) + 1 is a perfect square.
(c) Last digit is 2 (d) Last two digit are zero II. 12 is a divisor of p + (p + 2), if p > 3.
39. What is the value of x for which x, x + 1, x + 3 are all prime Which of the above statements is/are correct ? [2011-I]
(a) Only I (b) Only II
numbers? [2010-II]
(c) Both I and II (d) Neither I nor II
(a) 0 (b) 1
51. If three sides of a right angled triangle are integers in their
(c) 2 (d) 101 lowest form, then one of its sides is always divisible by
40. What is the last digit in the expansion of 34798 ? [2011-I]
[2010-II] (a) 6 (b) 5
(a) 1 (b) 3 (c) 7 (d) None of these
(c) 7 (d) 9 52. What is the number of prime factors of 30030? [2011-I]
41. If k is any even positive integer, then (k2 + 2k) is (a) 4 (b) 5
[2010-II] (c) 6 (d) None of these
(a) divisible by 24 53. ABC is a triangle and AD is perpendicular to BC. It is given
(b) divisible by 8 but may not be divisible by 24 that the lenghts of AB, BC, CA are all rational numbers.
(c) divisible by 4 but may not be divisible by 8 Which one of the following is correct? [2011-I]
(d) divisible by 2 but may not be divisible by 4 (a) AD and BD must be rational
(b) AD must be rational but BD need not be rational
42. If n is a positive integer, then what is the digit in the unit
(c) BD must be rational but AD need not be rational
place of 32n + 1 + 22n + 1 ? [2010-II]
(d) Neither AD nor BD need be rational
(a) 0 (b) 3
(c) 5 (d) 7 54. If n is a natural number, then n is [2011-II]
43. If the 14th term of an arithmetic series is 6 and 6th term (a) always a natural number
is 14, then what is the 95th term? [2010-II] (b) always a rational number
(a) – 75 (b) 75 (c) always an irrational number
(d) either a natural number or an irrational number
(c) 80 (d) – 80
55. The largest integer that divides product of any four
44. The product of a rational number and an irrational number consecutive integers is [2011-II]
is [2010-II] (a) 4 (b) 6
(a) a natural number (b) an irrational number (c) 12 (d) 24
(c) a composite number (d) a rational number 56. Which among the following is the largest four digit
45. For a positive integer n, define d (n) = The number of number that is divisible by 88 ? [2011-II]
positive divisors of n. What is the value of d [d {d (12)}]? (a) 9988 (b) 9966
[2011-I] (c) 9944 (d) 8888
(a) 1 (b) 2 57. Which one of the following is a prime number ?
(c) 4 (d) None of these [2011-II]
46. The number 2784936 is divisible by which one of the (a) 161 (b) 171
follwing numbers ? [2011-I] (c) 173 (d) 221
58. Consider the following statements:
(a) 86 (b) 87
I. The product of any three consecutive integers is
(c) 88 (d) 89
divisible by 6.
47. Consider the following numbers: II. Any integer can be expressed in one of the three
I. 247 forms 3k, 3k + 1, 3k + 2, where k is an integer.
II. 203 Which of the above statements is/are correct ?[2011-II]
Which of the above numbers is/are prime ? [2011-I] (a) Only I (b) Only II
(a) Only I (b) Only II (c) Both I and II (d) Neither I nor II
(c) Both I and II (d) Neither I nor II 59. Consider the following statements:
48. Which one of the following three-digit numbers divides I. Every composite number is a natural number.
9238 and 7091 with the same remainder in each case ? II. Every whole number is a natural number.
[2011-I] Which of the statements given above is/are correct ?
(a) 113 (b) 209 [2012-I]
(c) 317 (d) 191 (a) Only I (b) Only II
(c) Both I and II (d) Neither I nor II
49. When a positive integer n is divided by 5, the remainder
60. If a positive integer leaves remainder 28 when divided by
is 2. What is the remainder when the number 3n is divided
143, then what is the remainder obtained on dividing the
by 5 ? [2011-I] same number by 13 ? [2012-I]
(a) 1 (b) 2 (a) 0 (b) 2
(c) 3 (d) 4 (c) 9 (d) 10
EBD_7367
4
A- Number System

61. How many numbers between – 11 and 11 are multiples of 73. Which one of the following numbers is divisible by 11?
2 or 3 ? [2012-I] [2013-I]
(a) 11 (b) 14 (a) 45678940 (b) 54857266
(c) 15 (d) None of these (c) 87524398 (d) 93455120
62. What number should be added to 231228 to make it 74. If N, (N + 2) and (N + 4) are prime number, then the
exactly divisible by 33 ? [2012-I] number of possible solution for N are [2013-I]
(a) 1 (b) 2 (a) 1 (b) 2
(c) 3 (d) 4 (c) 3 (d) None of these
63. How many rational numbers are there between 1 and 75. The smallest positive prime (say p) such that 2p – 1 is not
1000? [2012-I] a prime is [2013-I]
(a) 998 (b) 999 (a) 5 (b) 11
(c) 1000 (d) Infinite (c) 17 (d) 29
64. The pair of numbers which are relatively prime to each 76. Consider the following statements:
other is [2012-I] (I) There is a finite number of rational numbers between
any two rational numbers.
(a) (68, 85) (b) (65, 91) (II) There is an infinite number of rational numbers
between any two rational numbers.
(c) (92, 85) (d) (102, 153)
(III) There is a finite number of irrational numbers
65. The number 58129745812974 is divided by [2012-I]
between any two rational numbers.
(a) 11 (b) 8
Which of the above statements is/are correct? [2013-I]
(c) 4 (d) None of these
(a) Only I (b) Only II
66. Consider the following statements:
(c) Only III (d) Both I and II
I. If n is a prime number greater than 5, then n4 – 1 is
divisible by 2400. 77. If m and n are natural number, then m n is [2013-II]
II. Every square number is of the form 5n, (5n – 1) or (a) always irrational
(5n + 1), where n is a whole number. (b) irrational unless n is the mth power of an integer
Which of the above statements is/are correct? [2012-II] (c) irrational unless m is the nth power of an integer
(a) Only I (b) Only II (d) irrational unless m and n are coprime
(c) Both I and II (d) Neither I nor II 78. If x is positive even integer and y is negative odd integer,
67. If the sum of an integer and its reciprocal is 10/3, then the then xy is [2013-II]
integer is equal to [2012-II] (a) odd integer (b) even integer
(a) 3 (b) 6 (c) rational number (d) None of these
(c) 12 (d) 13 1 3
68. Which one of the following is neither prime number nor 79. The pair of rational number that lies between and
4 4
composite number ? [2012-II]
is [2013-II]
(a) 1 (b) 2
(c) 3 (d) None of these 262 752 24 78
(a) , (b) ,
69. Which one of the following has least number of divisors? 1000 1000 100 100
[2012-II]
9 31 252 748
(a) 88 (b) 91 (c) , (d) ,
(c) 96 (d) 99 40 40 1000 1000
70. Consider the following statements: 80. What is the last digit in 7402 + 3402 ? [2013-II]
I. Every integer is a rational number. (a) 0 (b) 4
II. Every rational number is a real number. (c) 8 (d) None of the above
Which of the above sttemetns is/are correct? [2012-II] 81. What is 262 + 972 equal to ? [2013-II]
(a) Only I (b) Only II (a) 272 + 932 (b) 342 + 932
(c) Both I and II (d) Neither I nor II (c) 822 + 412 (d) 792 + 622
71. What is the harmonic mean of 10, 20, 25, 40 and 50? 82. Every prime number of the form 3k + 1 can be represented
[2012-II] in the form 6m + 1 (where, k and m are integers), when
(a) 25 (b) 30 [2013-II]
(c) 26.1 (d) 21.3 (a) k is odd
72. The two digit number, which when divided by sum of the (b) k is even
(c) k can be both odd and even
digit and product of the digits, respectively. The remainder
(d) No such form is possible
is same and the differnce of quotients is one, is
83. If k is a positive integer, then every square integer is of
[2013-I]
the form [2013-II]
(a) 14 (b) 23
(a) only 4k (b) 4k or 4k + 3
(c) 32 (d) 41
(c) 4k + 1 or 4k + 3 (d) 4k or 4k + 1
Number System 5
A-

84. If b is the largest square divisor of c and a2 divides c, 94. How many pairs of positive integers m and n satisfy the
then which one of the following is correct (where a, b and
1 4 1
c are integers) ? [2013-II] equation + = , where n is an odd integer less than
m n 12
(a) b divides a (b) a does not divide b
(c) a divided b (d) a and b are coprime 60 ? [2014-II]
85. 195 + 215 is divisible by [2013-II] (a) 7 (b) 5
(a) Only 10 (b) Only 20 (c) 4 (d) 3
(c) Both 10 and 20 (d) Neither 10 nor 20 95. Consider all those two-digits positive integers less than
86. Consider the following statements: 50, which when divided by 4 yield unity as remainder.
I. 7710312401 is divisible by 11. What is their sum ? [2014-II]
II. 173 is a prime number. (a) 310 (b) 314
Which of the statements given above is/are correct ? (c) 218 (d) 323
[2013-II] 96. How many pairs of X and Y are possible in the number
(a) Only I (b) Only II 763X4Y2, if the number is divisible by 9? [2014-II]
(c) Both I and II (d) Neither I nor II (a) 8 (b) 9
87. Consider the following statements: (c) 10 (d) 11
I. To obtain prime numbers less than 121, we have to 97. What is the remainder when 41012 is divided by 7?
reject all the multiples of 2, 3, 5 and 7. [2014-II]
II. Every composite number less than 121 is divisible (a) 1 (b) 2
by a prime number less than 11. (c) 3 (d) 4
Which of the statements given above is/are correct ? 98. What is the remainder when (1723 + 2323 + 2923) is divided
[2013-II] by 23 ? [2014-II]
(a) Only I (b) Only II (a) 0 (b) 1
(c) Both I and II (d) Neither I nor II (c) 2 (d) 3
88. Consider the following statements: 99. p,q and r are prime numbers such that p < q < r < 13. In
I. No integer of the form 4k + 3, where k is an integer, how many cases would (p + q + r) also be a prime
can be expressed as the sum of two squares. number ? [2014-II]
II. Square of an odd integer can expressed in the form (a) 1 (b) 2
8k + 1, where k is an integer. (c) 3 (d) None of these
Which of the above statements is/are correct? [2014-I] 100. What is the number of divisors of 360 ? [2014-II]
(a) Only I (b) Only II (a) 12 (b) 18
(c) Both I and II (d) Neither I nor II (c) 24 (d) None do these
89. If n is a whole number greater than 1, then n2 (n2 – 1) is
always divisible by [2014-I] 101. If 37 1 [2015-I]
= 2+
(a) 12 (b) 24 13 1
x+
(c) 48 (d) 60 1
y+
z
90. What is the remainder when 41000 is divided by 7?
[2014-I] where x, y, z are natural numbers, then what is z equal to?
(a) 1 (b) 2 (a) 1
(c) 4 (d) None of these (b) 2
91. 710 – 510 is divisible by [2014-II] (c) 3
(a) 10 (b) 7 (d) Cannot be determined due to insufficient data
(c) 5 (d) 11
5 + 10
92. The multiplication of a three–digits number XY5, with 102. What is equal to? [2015-I]
digit Z yields X 215. What is X + Y + Z equal to ? 5 5 - 2 20 - 32 + 50
[2014-II] (a) 5 (b) 5 2
(a) 13 (b) 15
(c) 17 (d) 18 (c)5 5 (d) 5
93. If N2 – 33, N2 – 31 and N2 – 29 are prime numbers, then 103. The digit in the units place of the product 81 × 82 × 83
what is the number of possible values of N, where N is × 84 × ... × 99 is [2015-I]
an integer ? [2014-II] (a) 0 (b) 4
(a) 1 (b) 2 (c) 6 (d) 8
(c) 6 (d) None of these
EBD_7367
6
A- Number System

104. A student was asked to multiply a number by 25. He instead 112. Consider the following statements for the sequence of
multiplied the number by 52 and got the answer 324 more numbers given below : [2015-I]
than the correct answer. The number to be multiplied was 11, 111, 1111, 11111, ...
[2015-I] 1. Each number can be expressed in the form (4m + 3),
(a) 12 (b) 15 where m is a natural number.
(c) 25 (d) 32 2. Some numbers are squares.
105. What is the number of possible pairs of (P, Q) if the number Which of the above statements is/are correct?
357P25Q is divisible by both 3 and 5? [2015-I] (a) 1 only (b) 2 only
(a) 7 (b) 6 (c) Both 1 and 2 (d) Neither 1 nor 2
(c) 5 (d) None of the above 113. Consider the following statements : [2015-I]
106. The difference between the squares of two consecutive 1. There exists only one prime number p such that (17p +
odd integers is always divisible by [2015-I] 1) is a square.
2. If x is the product of 10 consecutive prime numbers
(a) 3 (b) 7
starting from 2, then (x + 1) is also a prime number.
(c) 8 (d) 16
Which of the above statements is/are correct?
107. What is the maximum value of m if the number (a) 1 only (b) 2 only
N = 35 × 45 × 55 × 60 × 124 × 75 is divisible by 5m? (c) Both 1 and 2 (d) Neither 1 nor 2
[2015-I] 114. Out of 532 saving accounts held in a post office, 218 accounts
(a) 4 (b) 5 have deposits over ` 10,000 each. Further, in 302 accounts,
(c) 6 (d) 7 the first or sole depositors are men, of which the deposits
exceed ` 10,000 in 102 accounts. In how many accounts the
108. A person goes to a market between 4 p.m. and 5 p.m. When
first or sole depositors are women and the deposits are up to
he comes back, he finds that the hour hand and minute
` 10,000 only? [2015-I]
hand have interchanged their positions. For how much time
(approximately) was he out of his house? (a) 116
(b) 114
[2015-I]
(c) 100
(a) 55.25 minutes (b) 55.30 minutes
(d) Cannot be determined from the given data
(c) 55.34 minutes (d) 55.38 minutes
115. If x + y + z = 0
2 then x3 + y3 + z3 + 3xyz is equal to [2015-I]
109. When a ball bounces, it rises to of the height from which (a) 0 (b) 6 xyz
3
it fell. If the ball is dropped from a height of 36 m, how high (c) 12 xyz (d) xyz
will it rise at the third bounce? [2015-I] 116. The last digit in the expansion of 17256 is [2015-I]
(a) 9 (b) 7
1 2
(a) 10 m (b) 10 m (c) 3 (d) 1
3 3
117. What is the remainder obtained [2015-I]
1 2 when 1421 × 1423 × 1425 is divided by 12?
(c) 12 m (d) 12 m (a) 1 (b) 2
3 3
(c) 3 (d) 4
110. A light was seen regularly at an interval of 13 seconds. It
118. What is the remainder when 496 is divided by 6?
was seen for the first time at 1 hour 54 minutes 50 seconds
(a.m.) and the last time at 3 hours 17 minutes 49 seconds [2015-I]
(a.m.). How many times was the light seen? (a) 4 (b) 3
[2015-I] (c) 2 (d) 1
(a) 375 (b) 378 100
119. The number of values of x satisfying x + > 50 , where x
(c) 383 (d) 384 x
is a natural number less than or equal to l00 is [2015-II]
111. If n is a natural number and n = p1x1 p2x2 p3x3 , where p1, p2, (a) 51 (b) 53
p3 are distinct prime factors, then the number of prime factors (c) 55 (d) 57
for n is [2015-I] 120. The largest natural number which divides every natural
(a) x1 + x2 + x3 number of the form (n 3 – n)(n – 2), where n is a natural
(b) xlx2x3 number greater than 2 is [2015-II]
(c) (x1 + 1) (x2 + 1) (x3 +1) (a) 6 (b) 12
(c) 24 (d) 48
(d) None of the above
Number System A-7

121. The digit in the units place of the resulting number of the 130. Let a two-digit number be k times the sum of its digits. If the
expression (234)100 + (234)101 is [2015-II] number formed by interchanging the digits is m times the
(a) 6 (b) 4 sum of the digits, then the value of m is [2016-I]
(c) 2 (d) 0 (a) 9 – k (b) 10 – k
122. The seven digit number 876p37q is divisible by 225. The (c) 11 – k (d) k – 1
values of p and q can be respectively [2015-II] 131. Let S be a set of first fourteen natural numbers. The possible
(a) 9, 0 (b) 0, 0 number of pairs (a, b), where a, b, S and a, b such that ab
(c) 0, 5 (d) 9, 5 leaves remainder 1 when divided by 15, is [2016-I]
123. Let x and y be positive integers such that x > y. The (a) 3 (b) 5
expressions 3x + 2y and 2x + 3y when divided by 5 leave (c) 6 (d) None of the above
remainders 2 and 3 respectively. What is the remainder when 132. A clock strickes once at 1 o’clock, twice at 2 o’clock and
(x – y) is divided by 5? [2015-II] thrice at 3 o’clock and so on. If it takes 8 seconds to strike
(a) 4 (b) 2 at 5 o’clock, the time taken by it to strike at 10 o’clock is
(c) 1 (d) 0 [2016-I]
124. The sum of first 47 terms of the series (a) 14 seconds (b) 16 seconds
(c) 18 seconds (d) None of the above
1 1 1 1 1 1 1 1 1 133. What is the maximum value of m, if the number N = 90 × 42
+ - - - + + + - ..... is [2015-II]
4 5 6 4 5 6 4 5 6 × 324 × 55 is divisible by 3m? [2016-I]
(a) 8 (b) 7
1 (c) 6 (d) 5
(a) 0 (b) -
6 134. Consider the following statements: [2016-I]
1 9 1. Every natural number is a real number.
(c) (d) (b) Every real number is a rational number.
6 20
(c) Every integer is a real number.
125. A number consists of two digits, whose sum is 7. If the (d) Every rational number is a real number
digits are reversed, the number is increased by 27. The Which of the above statements are correct?
product of digits of the number is [2015-II] (a) 1, 2 and 3 (b) 1, 2 and 4
(a) 6 (b) 8 (c) 2 and 3 only (d) 3 and 4 only
(c) 10 (d) 12 135. Consider the following statements: [2016-I]
126. Consider all positive two digit numbers each of which when 1. There exists a positive real number m such that cos
divided by 7 leaves a remainder 3. What is their sum? x = 2m + 1.
[2015-II] 2. mn ³ m + n for all m, n belonging to set of natural
(a) 661 (b) 666 numbers.
(c) 676 (d) 777 Which of the above statements is/are correct?
127. Consider the following statements in respect of the (a) 1 only (b) 2 only
expression [2016-I] (c) Both 1 and 2 (d) Neither 1 nor 2
n(n + 1) 136. A person goes to a market between 4 p.m and 5 p.m. When
Sn = he comes back, he finds that the hour hand and the minute
2
hand of the clock have interchanged their positions. For
where n is an integer. how much time (approximately) was he out of his house?
1. There are exactly two values of n for which Sn = 861. [2016-I]
2. Sn = S(n + 1) and hence for any integer m, we have two (a) 55.38 minutes (b) 55.48 minutes
values of n for which Sn = m. (c) 55.57 minutes (d) 55.67 minutes
Which of the above statement is/are correct? 137. If m and n are distinct natural numbers, then which of the
(a) 1 only (b) 2 only following is/are integer/integers? [2016-I]
(c) Both 1 and 2 (d) Neither 1 nor 2
m n
128. Consider the following statements in respect of two different 1. +
non-zero integers p and q [2016-I] n m
1. For (p + q) to be less than (p – q), q must be negative. æm nö
2. For (p + q) to be greater than (p – q), both p and q must 2. mn ç + ÷ (m2 + n2 )
è n mø
be positive.
Which of the above statements is/are correct? mn
(a) 1 only (b) 2 only 3.
m + n2
2
(c) Both 1 and 2 (d) Neither 1 nor 2 Select the correct answer using the code given below:
129. 710 – 510 is divisible by [2016-I] (a) 1 and 2 (b) 2 only
(a) 5 (b) 7 (c) 2 and 3 (d) 3 only
(c) 10 (d) 11
EBD_7367
A- 8 Number System

138. Outside a meeting room, Madhukar was told by a person 144. What is the remainder when the number (4444)4444 is divided
that each meeting takes place after 13/4 hours. The last by 9 ? [2017-I]
meeting has been over just 45 minutes ago and the next (a) 4 (b) 6
meeting will take place at 2 p.m. At what time did Madhukar (c) 7 (d) 8
receive this information? [2016-I] 145. The number of prime numbers which are less than 100 is
(a) 10 : 20 a.m. (b) 11 : 30 a.m. [2017-I]
(c) 11 : 40 a.m. (d) 11 : 50 a.m. (a) 24 (b) 25
(c) 26 (d) 27
x 10 y
139. If = – and x – y = 8, then the value of xy is equal 146. Consider the following statements : [2017-I]
y 3 x 1. Of two consecutive integers, one is. even.
to [2016-I] 2. Square of an odd integer is of the form 8n + 1.
(a) 36 (b) 24 Which of the above statements is/are correct ?
(c) 16 (d) 9 (a) 1 only (b) 2 only
140. What would be the maximum value of Q in the equation (c) Both 1 and 2 (d) Neither 1 nor 2
5P9 + 3R7 + 2Q8 = 1114 ? [2016-II] 147. (NP–1 – 1) is a multiple of p, if N is prime to p and p is a
(a) 9 (b) 8 [2017-I]
(c) 5 (d) 4 (a) Prime number
141. In an examination, a student was asked to divide a certain (b) Rational number
number by 8. By mistake he multiplied it by 8 and got the (c) Real number
answer 2016 more than the correct answer. What was the (d) Composite number
number? [2016-II] 148. Three numbers which are co-prime to each other, are such
(a) 252 (b) 256 that the product of the first two is 286 and that of the last
(c) 258 (d) 260 two is 770. What is the sum of the three numbers?
142. A boy saves `4.65 daily. What is the least number of days [2018-1]
in which he will be able to save an exact number of rupees ? (a) 85 (b) 80
[2016-II] (c) 75 (d) 70
(a) 10 (b) 20 149. All odd prime numbers upto 110 are multiplied together.
(c) 21 (d) 25 What is the unit digit in this product. [2018-1]
143. What is the unit digit of 7139 ? [2016-II] (a) 0 (b) 3
(a) 9 (b) 7 (c) 5 (d) None of the above
(c) 6 (d) 3

HINTS & SOLUTIONS


1. (b) Difference = Sum of digit at odd place – Sum of digit at 3. (c) Sum of all the digits in the number (26492518)
even place =2 +6+4 +9+2 +5+1 +8
= (1 + 5 + 9 + 4) – (2 + 4 + 3) = 37
= 19 – 9 = 10 When we subtract 4 from 37 then number must be
In 10, we must add at least 1 so that it is divisible by divisible by 3.
11. So x = 1 4. (a) Here (24)2 < 589
Also, the sum of digits of 1254934 So, prime numbers less than 24 are 2, 3, 5, 7, 11, 13, 17,
= 1 + 2 + 5 + 4 + 9 + 3 + 4 = 28 19, 23.
Since, 589 is divisible by 19, then 589 is a composite
1254934 will be divisible by 3, after adding y, if the
number.
value of y is – 1.
5. (d) A number of the form an + bn then it is divisible by (a
So, x = 1 and y = – 1 is the set of values for x and y.
+ b), if only n is odd number. So that an + bn is not
2. (d) For divisible by 5:
divisible by integral value of n.
A number is divisible by 5 then its unit place must be
6. (b) 1 unit
0 or 5. D C
For divisible by 4: d2 d1
The last two digit of a number is divisible by 4, then 1 unit
Rectangle 2
the number is divisible by 4.
Possible digit at ten’s place = 0, 2, 4, 6, 8.
A B
Number System A-9

1 1 12. (a) Q Original number = h × 100 + t × 10 + u


Area = x = 1´ = = Rational Number obtained by reversing digits
2 2
= u × 100 + t × 10 + h
5 \ Required number = (h × 100 + t × 10 + u)
d1 = d2 = 12 + (1/ 2)2 = – (u × 100 + t × 10 + h) = 99 (h – u)
2
But the unit’s place digit in above number is 4, therefore
5 5 (h – u) should be 6, then number is 594. Whose digits
\ y = d1 + d2 = + are 5, 9, 4 respectively.
2 2
13. (c) Zero is a whole number and every integer is not a
2 5 whole number, because negative integers are not whole
= = 5 = Irrational
2 numbers.
7. (c) Dividend = Quotient × Divisor + Remainder Therefore, A is correct and R is wrong.
According to the question 14. (a) Let the divisor be x.
D = 5Q and D = 2R So, Divided = Quotient × Divisor + Remainder
But R = 15, D = 30, Dividend = 4996
5Q = 30, Q = 6. Quotient = 62 and remainder = 36
Dividend = DQ + R = 30 × 6 + 15 = 195 \ 4996 = 62 × x + 36
8. (d) Given number is 222222. Þ 62x = 4996 – 36
Sum of digits = 2 + 2 + 2 + 2 + 2 + 2 = 12 which is 4990
divisible by 3. Þ x= = 80
62
So, number is also divisible by 3.
15. (d) 667, 861, 481 is divided by 23, 21, 13 respectively.
Sum of odd terms of digits – Sum of even terms of
digits = 6 – 6 = 0, it is divisible by 11. But 331 is not divisible by any other numbers so 331 is
In a number a digit repeated six times, then this number a prime number.
is divisible by 7, 11 and 13. 16. (c) Number = 987 × k + 59
Hence, the given number is divisible by 3, 7 and 11 Now, 987 is completely divided by 21.

9. (b) Let two rational numbers be 3 and 2. 2


21 59
Then, ( 3 + 2) is an irrational number..
42
Let two rational numbers be 3 - 2, 2 . 17
\ (3 - 2) + 2 = 3 (rational) Now, 17 is remainder.
Therefore, sum of two irrational numbers may be 17. (d) It is always correct that the product of two square
rational or irrational. numbers is a square number.
10. (d) By considering every option e.g., 4 × 9 = 36
(a) 3q + 9 = 18 Þ q = 3 18. (c) The set of integers is closed with respect of addition
Then, numbers are 3, 5 and 9, which are not all and multiplication.
prime. e.g., Let Z = {..., – 3, – 2, – 1, 0, 1, 2, 3, ...}
(b) 3q + 9 = 42 Þ q = 11 1 + 2 = 3 and 2 + 1 = 3 (for addition)
Then, numbers are 11, 13, 17, which are all prime. 1 × 2 = 2 and 2 × 1 = 2 (for multiplication)
(c) 3q + 9 = 60 Þ q = 17 19. (d) N = 8 7 9 8 5 4 6 * 5
Then, numbers are 17, 19 and 23 which are all Sum of odd digit places = 8 + 9 + 5 + 6 + 5 = 33
prime. Sum of even digit places = 7 + 8 + 4 + * = 19 + *
Hence, option (d) is correct. Now, 33 – (19 + *) = 14 – *, it is divisible by 11, if value
11. (a) Given, p = 2 × 3 × 5 ... 59 × 61 = ... 0 of * is 3.
Also, 2 £ n £ 59 20. (d) The prime numbers between 100 and 120 are 101, 103,
Now, we check the sequence p + n 107, 109 and 113.
Since, unit digit of p is zero. Therefore, for every even \ Required sum = 101 + 103 + 107 + 109 + 113 = 533
value of n, (p + n) is always divisible. 21. (a) Required numbers are of the form of 12q – 2
For odd value of n = 2, 5, ... 59 i.e., 10, 22, 34, 46, 58, 70, 82, 94
Take n = 3 \ Total sum = 10 + 22 + 34 + 46 + 58 + 70 + 82 + 94
\ p + n = p + 3 = (2 × 3 × 5 ... 59 × 61 + 3)
= 416
= 3(2 × 5 ... 59 × 61 + 1) which is divisible.
22. (c) The total number of thee digit numbers with unit digit
Similarly, for even value of n, p + n is divisible.
7 and divisible by 11 are 187, 297, 407, 517, 627, 737,
Hence, it is clear always p + n is that divisible by any
847, 957.
number. So, there is no prime number exist in this
sequence. \ Total numbers = 8
EBD_7367
10
A- Number System

23. (c) When we divide a positive integer by another positive 30. (d) Let p be any positive number of the form 2m,
integer, the resultant will be a rational number i.e., in 2m + 1 for any whole number m.
the form of p/q, where p and q are positive integers Case I: p = 2m
and q ¹ 0 p2 = 4m2 = 4(m2) = 4q where q = m2
24. (c) Let the first odd number be x and the alternate odd Case II: p = 2m + 1
number is x + 4 p2 = (2m + 1)2 = 4m2 + 4m + 1
According to the question = 4(m2 + m) + 1
x (x + 4) = 3x + 12 = 4q + 1 where q = m2 + m
Þ x2 + 4x = 3x + 12 From above we see that square of any positive integer
Þ x2 + x – 12 = 0 is either of the form 4q or (4q + 1) for some integer q.
Þ (x + 4) (x – 3) = 0 31. (b) 84705Ä2
Þ x=3 (Q x ¹ – 4) If sum of digit of a number is divisible by 9, then it is
Hence, the larger number is x + 4 = 3 + 4 = 7 also devisible by 9.
25. (a) As we know that a number a1 a2 a3 a4 a5 is divisible Þ 84705Ä2 = 8 + 4 + 7 + 0 + 5 + Ä + 2 = 26 + Ä
by 9, if sum of the digits, i.e., a1 + a2 + a3 + a4 + a5 is Now, we replace Ä by 1 then it become 27 and divided
divisible by 9. Hence, only statement I is true. by 9.
32. (c) When a polynomial is divided by a linear polynomial,
26. (d) (2457)754
then the remainder is either constant or zero polynomial.
We know that a unit digit of (7)4 is 1.
Unit digit of (2457)754 = (2457)188 × 4 + 2 x
= (2457)188 × 4 . (2457)2 ® [(2457)4]188. 9 ® (1)188.9 2
e.g., (ax + b) ax + bx + c
® 9
27. (b) When a number n is divided by 4 then remainder is 3. _ ax 2 ± bx
Now, the number is double then remainder is also c = constant
double.
So, remainder = 6 OR
But remainder never greater than its divisor.
x
So, remainder = 6 – 4 = 2
2
28. (b) Let the GP series be a, ar, ar2, ar3, ar4 ,... (ax + b) ax + bx
By given condition, _ ax 2 ± bx
T3 = T1 + 9
o = zero
Þ ar2 = a + 9
Þ a (r2 – 1) = 9 ...(i) 33. (d) Let a = 7p + 5
and T2 = T4 + 18 and b = 7q + 4
Þ ar = ar3 + 18 where, p and q are natural numbers.
Þ ar (1 – r2) = 18 ...(ii) \ ab = (7p + 5) (7q + 4)
On dividing Eq. (i) by Eq. (ii), we get ab = 49pq + (4p + 5q) 7 + 20
= 7 (7pq + 4p + 5q) + 7 × 2 + 6
a(r 2 - 1) 9 when ab is divided by 7, we get the remainder 6.
\ 2
=
-ar (r - 1) 18 34. (d) P = r s = r (1 – r) = r – r2 (Q r + s = 1)
Þ –r=2
From Eq. (i) dP
Þ = 1 – 2r = 0 (For max. and min.)
a (4 – 1) = 9 dr
\ a=3
1 d 2P
29. (a) Let angles of a triangle in AP are a, a + d, a + 2d. Þ r= Þ =–2
2 dr 2
Also, a + 2d = 2a (given condition)
Þ a = 2d ...(i) = Negative Þ Maximum
Also, a + a + d + a + 2d = 180º 1
(Q sum of angles of triangle = 180º) \ Maximum value of rs =
4
Þ 3a + 3d = 180º 35. (b) Any number is not a square, if the unit’s place digit of
æaö number may be 2, 3, 7, 8.
Þ 3a + 3 ç ÷ = 180º [from Eq. (i)] Hence, the number 9852 is not a square of any natural
è2ø
number.
Þ 9a = 360º 36. (d) On taking option (d).
Þ a = 40º, d = 20º The reverse digit of 451 is 154.
\ Ratio of angles = 40º : 60° : 80° = 2 : 3 : 4 Now, 154 + 297 = 451 is equal to the original number.
Number System A-11

37. (c) We know that, if the difference of the sum of odd digits 46. (c) Factors of 88 = 11 × 8
and sum of even digits is either 0 or mlultiple of 11, 2784936
then the number is divisible by 11. = (Sum of odd place digit) – (Sum of even place digit)
Given number is ABCDE. = 25 – 14 = 11 (This is divisible by 11)
Here, A + C + E – (B + D) = 0 or divisible by 11 2784936:
Hence, both statements are true. Last three digit is 936 is divisible by 8.
38. (d) 212n – 64n = (212)n – (64)n = (4096)n – (1296)n So this number is also divisible by 8.
= (4096 – 1296) [(4096)n – 1 + (4096)n – 2(1296) + ... + Now, 2784936 is divisible by 88.
(1296)n – 1] = 2800(k) 47. (d) Here 247 is divisible by 13 and 203 is also divisible by
Hence, last two digits are always be zero. 7. So, 247 and 203 are not prime number.
39. (c) If x = 2, then x + 1 and x + 3 are all prime numbers. 48. (a) Difference between the digit = 9238 – 7091
40. (d) Last digit of 34798 = 2147
We know that 3 is cyclic of its unit digit at 4 times. So, 2147 is completely divided by 113.
we devide 4798 by 4 49. (a) Let n = 5q + 2
= 34798 = 31199 × 4 + 2 = (31199 × 4) . 32 = 9 3n = 3(5q + 2)
41. (b) If k is any even positive integer, then (k2 + 2k) is Þ 3n = 15q + 6 = 5 (3q + 1) + 1
divisible by 8 but may not be divisible by 24. When 3n is divided by 5, then remainder is 1.
Let k = 2m, m Î N, then 50. (c) On taking p = 11,
k2 + k.2 = 4m2 + 4m = 4m (m + 1) p + 2 = 13 (prime number)
which is divisible by 4. I. 11 × 13 + 1 = 144 (a square number)
42. (c) 32n + 1 + 22n + 1 = 3 × (3)2n + 2 × (2)2n II. 11 + 13 = 24 (12 is a divisor of 24)
= 3 × (9)n + 2 × (4)n Hence, both statements I and II are correct.
51. (b) Let the lowest sides of a right triangle be 3, 4, 5.
n Unit digit of (9)n Unit digit of (4)n By Pythagoras theorem, (3)2 + (4)2 = (5)2
1 9 4
Hence, one of its sides is always divisible by 5.

2 1 6 52. (c) 30030


3 9 4
4 1 6 2 15015

5 9 4
3 5005
M M M
Thus, when n is odd, then unit digit of (9)n = 9 and (4)n 5 1001
=4
and when n is even, then unit digit of (9)n = 1 and (4)n 11 91
=6
Hence, when n is odd positive integer, then 7 13
3 × (unit digit of 9) + 2 × (unit digit of 4)
= 3 × 9 + 2 × 4 = 35
Hence, unit digit of (3)2n + 1 + (2)2n + 1 = 5 13 1
Also, when n is even positive integer, then
3 × (unit digit of 9) + 2 × (unit digit of 4) So, prime factors of 30030 are
= 3 × 1 + 2 × 6 = 15 2, 3, 5, 11, 7, and 13
Hence, unit digit of (3)2n + 1 + (2)2n + 1 = 5 So, number of prime factors of 30030 is 6.
43. (a) Q T14 = 6 53. (c) Since, D is a point of BC. As BC is rational so BD must
Þ a + 13d = 6 be rational but AD need not to be rational.
and T6 = 14 Þ a + 5d = 14 54. (d) If n is a natural number, then n is either a natural
On solving equations (i) and (ii), we get number or an irrational number.
a = 19, d = – 1
\ T95 = a + 94d = 19 – 94 = – 75 e.g., when n = 3 Þ 3 = Irrational number
44. (b) We know that the product of a rational number and an When n = 9 Þ 9 = 3 = Rational number
irrational number is an irrational number. 55. (d) The largest integer that divides product of any four
45. (d) d [d {d (12)}] = d [d (6)] consecutive integers is 4! i.e., 24.
(Q positive integer divisor of 12 = 1, 2, 3, 4, 6, 12) e.g., 1, 2, 3, 4 are four consecutive integers.
= d (4) (Q positive integer divisor of 6 = 1, 2, 3, 6) Multiplication = 1 × 2 × 3 × 4 = 24 which is divided
= 3 (Q positive integer divisor of 4 = 1, 2, 4) by 24.
EBD_7367
12
A- Number System

56. (c) If a number is divisible by 88, it should be divisible by 64. (c) 92 = 22 × 23


8 and 11. 85 = 5 × 17
In a given option, number 9944 and 8888 is divisible Since there is no common factor in 92 and 85, therefore
by 88. they are relatively prime.
Hence, largest number is 9944. 65. (a) Divisibility Rule 11: The difference between the sum
57. (c) 161, 171, 221, are divisible by 7, 3 and 13 respectively. of digit at odd place numbers and the sum of digit at
But 173 is not divisible by any others numbers except even, place numbers is zero or multiple of 11.
1 and 173. 36
58. (c) I. The product of any three consecutive integers is
divisible by 3! i.e., 6.
II. Here, 3k = {..., -6, -3, 0,3, 6,...}

3k + 1 = {..., -5, -2,1, 4, 7,...}

and 3k + 2 = {..., -4, -1, 2,5,8,...}

\ {3k ,3k + 1,3k + 2} 5 8 1 2 9 7 4 5 8 1 2 9 7 4

= {..., -6, -5, -4, -3, -2, -1, 0,1, 2,3, 4,5, 6,...}
Hence, it is true.
59. (a) I. Composite Number: Natural numbers, which has
more than 2 distinct factors are called composite
number. It postulates that every composite
number is a natural number.
II. Every whole number is not a natural number (zero 36
is not a natural number). Difference = 36 – 36 = 0, then it is divisible by 11.
So, only statement I is true. 66. (c) I. Given, n is a prime number greater than 5.
60. (b) Quotient = 143K + 28 Now, n4 – 1 = (n2 – 1) (n2 + 1)
Now, when same number is divided by 13. So, we check = (n – 1) (n + 1) (n2 + 1)
143 is divided by 13 or not. Put n = 7 (prime number greater than 5)
Here, 143 is completely divided by 13. Hence we n4 – 1 = (7 – 1) (7 + 1)(49 + 1)
divides 28 by 13 then remainder is 2. = 6 × 8 × 50 = 2400
61. (c) Following are the numbers between – 11 and 11 which So, statements I is true.
are multiples of 2 or 3? II. Given, n Î W (whole number)
– 10, – 9, – 8, – 6, – 4, – 3, – 2, 0, 2, 3, 4, 6, 8, 9,10 i.e., n = 0, 1, 2, 3, 4, 5, ...
So ,the numbers of multiples 2 or 3, between – 11 and For n = 0,
11 are 15. 5n, (5n – 1), (5n + 1) = 0, – 1, 1 = (0)2, – 1, (1)2
Alternative Method: For n = 1,
Numbers between 0 and 11 which are multiples of 2 or 5n, (5n – 1), (5n + 1) = 5, 4, 6 = 5, (2)2, 6
3 For n = 2,
5n, (5n – 1 ), (5n + 1 ) = 10, 9, 11
11 11 11 = 10, (3)2, 11
+ – =5+3–1=7
2 3 6 For n = 3,
Numbers between 0 and – 11 5n, (5n – 1), (5n + 1) = 15, 14, 16 = 15, 14, (4)2
For n = 4,
11 11 11 5n, (5n – 1), (5n + 1) = 20, 19, 21
+ – =7
2 3 6 For n = 5,
So, the numbers are 14 and 0. 5n, (5n – 1), (5n + 1) = 25, 24, 26 = (5)2, 24, 26 ... so
62. (c) Þ 231228 = 7006 × 33 + 30 ... (1) on.
So, statements II is true.
Now, when the number divides by 33 its remainder is 67. (a) Let the integer be x.
30. Therefore, 3 must be added to 231228 to make it 10
Integer + Its reciprocal =
exactly divisible by 33. 3
63. (d) There are an infinite of rational numbers between any
two given numbers. 1 10
Þ x+ =
x 3
Number System A-13

72. (c) From options,


x 2 + 1 10
Þ = 14 14
x 3 (a) = = 4 (Rem) and
2
Þ 3x + 3 = 10x (1 + 4) 5
Þ 3x2 – 10x + 3 = 0 14 14
Þ 3x2 – 9x – x + 3 = 0 (by splitting middle term) = = 2 (Rem)
1´ 4 4
Þ 3x (x – 3) – 1(x – 3) = 0
Since, remainder is not same.
Þ (x – 3) (3x – 1) = 0
\ x=3 (since 1/3 is not an integer) 23 23
(b) = = 3 (Rem) and
Alternative Method: (2 + 3) 5
By Hook or by Crook.
From option (a), 23 23
= = 5 (Rem)
2´3 6
1 9 + 1 10
3+ = = Since, remainder is not same.
3 3 3
68. (a) Prime number: A number that is divisible by itself 32 32
(c) = = 2 (Rem) and
and one excluding one. (3 + 2) 5
So, 1 is not prime number.
32 32
Composite Number: Natural numbers, which has more = = 2 (Rem)
than 2 distinct factors are called composite numbers. (3 ´ 2) 6
1 is not composite numbers. Since, remainder is same.
69. (b) Here, 88 = 2 × 2 × 2 × 11= (2)3 × (11)1 \ Difference of quotients = 6 – 5 = 1
91 = (7)1 × (13)1 41 41
96 = 2 × 2 × 2 × 2 × 2 × 3 = (2)5 × (3)1 (d) = = 1 (Rem) and
(4 + 1) 5
and 99 = 3 × 3 × 11 = (3)2 × (11)1
So, 91 has least number of divisors. 41 41
70. (c) I. It is true that every integer is a rational number. = = 1 (Rem)
(4 ´1) 4
Since, remainder is same.
e.g., Set of integers,
But difference of quotients = 10 – 8 = 2 ¹ 1
Z = {..., -3 , -2 , -1 , 0 , 1 , 2 , 3 , ...} 73. (d) We know that,
1, 1 1 1 1 1 1 If the difference between the sum of digits at even
p places and sum of digits at odd places is (0) or multiple
which of the form , which also shown rational
q of 11, then the number is divisible by 11.
numbers. From options,
II. It is also true that every rational number is a real (a) 45678940
number because a real number does not contain i Sum of even places = 5 + 7 + 9 + 0 = 21
(iota). Sum of odd palaces = 4 + 6 + 8 + 4 = 22
So, both satements are true. Their difference = 22 – 21 = 1 ¹ 0
71. (d) Let the number are a1 = 10, a2 = 20, a3 = 25, a4 = 40 and (b) 54857266
a5 = 50. Sum of even places = 4 + 5 + 2 + 6 = 17
Number of observations
Sum of odd palaces = 5 + 8 + 7 + 6 = 26
\ Harmonic mean = Their difference = 26 – 17 = 9 ¹ 0
1 1 1 1 1
+ + + + (c) 87524398
a1 a2 a3 a4 a5
Sum of even places = 7 + 2 + 3 + 8 = 20
Sum of odd palaces = 8 + 5 + 4 + 9 = 26
5 Their difference = 26 – 20 = 6 ¹ 0
=
1 1 1 1 1 (d) 93455120
+ + + +
10 20 25 40 50 Sum of even places = 3 + 5 + 1 + 0 = 9
Sum of odd palaces = 9 + 4 + 5 + 2 = 20
5
= Difference = 20 – 9 = 11
20 + 10 + 8 + 5 + 4 It is divisible by 11.
200 74. (a) When N is a natural number, then there is only one
5 ´ 200 1000 possible case that N, (N + 2), (N + 4) are prime numbers
= = = 21.27 » 21.3 when N = 3, then N, (N + 2), (N + 4) = 3, 5, 7 all are
47 47
primes.
EBD_7367
14
A- Number System

75. (b) On taking p = 5,


2p – 1 = 25 – 1 = 31 which is prime 13
On taking p = 11,
2p – 1 = 211 – 1 = 2047 + + + +
Since, 2047 is divisible by 23, so it is not prime.
Thus , the required least positive prime number is 11. 7 7 1 0 3 1 2 4 0 1
76. (b) We know that, between any two rational numbers, + + + +
there are an infinite number of rational and irrational
numbers.
Hence, statement II is correct. 13

77. (b) If m and n are natural numbers, then m n is irrational Difference = Sum of odd place – Sum of even place
unless n is mth power of an integer. = 13 – 13 = 0, it is divisible by 11.
78. (c) If x is a positive even integer and y is negative odd Statement II:
integer, then xy is a rational number. 173 is square of approximately 13.
So, below 13 prime number are -
1 3 2, 3, 5, 7, 11
79. (d) = 0.25 and = 0.75
4 4 Now, 173 is not divisible by 2, 3, 5, 7 and 11.
So, it is a prime number.
252 748 So, both the statements I and II are correct.
Only option (d) with = 0.252 and = 0.748
1000 1000 87. (c) Both the statements given are correct. As 121 is the
lies between 0.25 and 0.75. square of 11. So, to obtain prime numbers less than
80. (c) Here 7, and 3 both are repeat its unit digit after four 121, we reject all the multiples of prime numbers less
times. than 11 i.e., 2, 3, 5 and 7. Similarly, every composite
= 7(4 × 100 + 2) + 3(4 × 100 + 2) number less than 121 is divisible by a prime number
less than 11 i.e., 2, 3, 5 or 7.
= 74 × 100 × 72 + 34 × 100 × 32
88. (a) I. f (k) = 4k + 3
=9+9 For k = 1, f (1) = 4 × 1 + 3 = 7
= 18 For k = 2, f (2) = 4 × 2 + 3 = 11
So, its unit digit = 8 For k = 3, f (3) = 4 × 3 + 3 = 15
81. (d) Here we can check the unit digit Values of f (k) for k = 1, 2, ...... cannot be expressed
Unit digit of 262 + 972 = 6 + 9 Þ 5 as sum of squares, since
12 + 22 = 5, 12 + 32 = 10, 22 + 32 = 13
Now according to option
II. f (k) = 8k + 1
Unit digit of 792 + 622 = 1 + 4 Þ 5 For k = 1, f (1) = (8 × 1) + 1 = 9 = (3)2
So, 262 + 972 = 792 + 622 For k = 2, f (2) = (8 × 2) + 1 = 17
82. (b) Every prime number of the form 3k + 1 can be For k = 3, f (3) = (8 × 3) + 1 = 25 = (5)2
represented in the form 6m + 1 only, when k is even. For k = 4, f (4) = (8 × 4) + 1 = 33
83. (d) For square integer 25, 4k + 1 mean 4 × 6 + 1 For k = 5, f (5) = (8 × 5) + 1 = 41
and for 36 f (k) = 8k + 1 is square of an odd integer only for
some values of k. So, only Statement I is correct.
4k mean 4 × 9
89. (a) Given that n is greater then 1, then n2(n2 – 1) is always
Now, if k is a positive integer, then every square integer divisible by 12.
is of the form 4k or 4k + 1
84. (c) Here b is the largest square divisor of c and a2 divides Example 1: Put n = 2, then
c then, it is sure that a divides b. n2(n2 – 1) = (2)2 (22 – 1)
85. (c) We know that an + bn where n is odd numbers then it = 4 × 3 = 12.
is divides by a + b. Hence n2(n2 – 1) is divisible by 12 but not by 24, 48
So, 195 + 215 = 19 + 21 = 40 and 60 for n = 2.

4(
Now, 40 is divided by both 10 and 20. 333 ´ 3 + 1)
So that number is also divided by 10 and 20. 90. (c) Remainder 41000 =
7
86. (c) Statement I:
Divisibility rule of 11: The difference between the ( 64)333 ´ 4
sum of even places and the sum of odd places is 0 or =
7
that number is divisible by 11.
=1×4=4
Number System 15
A-

91. (d) 710 – 510 is divisible by 11. (1, 4), (4, 1), (2, 3) (3, 2), (0, 5) and (5, 0). When sum of
92. (a) Here, three–digits number = XY5 X and Y is 11, then possible pairs are (5, 9), (9 , 5), (6, 8),
(8, 6) and (7, 7).
XY 5 Total possible pairs are 11.
´ Z
41012 43 ´ 337 +1 43 ´ 337 ´ 4
X 215 97. (d) = = =
7 7 7
So, Z can take value 1, 3, 5, 7 and 9.
337
But only 9 satisfies it, then X = 1, Y = 3 and Z = 9 (64)337 ´ 4 (9 ´ 7 + 1) ´ 4
= =
135
7 7

Q ´9 (ax + 1)n
We know that = then its Remainder is 1
1215 a
value of X + Y + Z = 1 + 3 + 9 = 13 =1× 4=4
93. (c) From option (c) By Hook and Crook N = 6
17 23 + 2323 + 2923
N2 – 33 = 62 – 33 = 36 – 33 = 3, which is prime. 98. (a)
N2 – 31 = 62 – 31 = 36 – 31 = 5, which is prime. 23
N2 – 29 = 62 – 29 = 36 – 29 = 7, which is prime.
17 23 + (0) 23 + (29) 23
So, N = 6 only. possible value =
23
1 4 1
94. (d) Given equation, + =
m n 12 17 23 + (29) 23 (17) 23 + (6)23
= =
Þ 12(n + 4m) = mn 23 23
Þ 12n + 48m = mn
Þ m(48 – n) = –12n a n + bn
Here
Þ m(n – 48) = 12n n
So remainder is always zero.
12 n
\ m= ...(i) 99. (b) The prime numbers less than 13 are 2, 3, 5, 7, 11.
n - 48 Also, using the condition, p < q < r < 13 and p + q + r
Here, as m and n are positive integers, therefore > 48. But n is a prime number
is an odd integer less than 60, therefore possible values of Hence, only two possible pairs exist i.e. (3, 5, 11) and
n = 49, 51, 53, 55, 57 and 59. (5, 7, 11).
But on putting n = 53, 55, and 59 in Eq. (i), we get the 100. (c) Q 360 = 23 × 32 × 5
non-integer values of m \ Number of divisors = (3 + 1) (2 + 1) (1 + 1)
On putting n = 49, 51 and 57 , we get the value of m = 588, = 4 × 3 × 2 = 24
204 and 76, respectively.
Hence, there are three possible pairs of m and n that satisfy
37 1
the equation. 101. (b) = 2+
95. (a) Let the two-digits numbers less than 50 which when 13 1
x+
divided by 4 yield unity as remainder be 13, 17, ..., 49. 1
y+
Here, first term, a = 13, common difference, z
d = 4 and n = 10.
37
n Þ can be expressed as
\ Required sum = [2 a + ( n – 1) d ] 13
2
1
10 = 2+
= [2 ´ 13 + (10 – 1)4] 1+1
2 1 1
= 2+ 5+
10 10 ´ 62 2 2
= [26 + 36] = = 310 1+
2 2 11
96. (d) If sum of all the digits is divisible by 9 then the number Now, this is compared by
is divisible by 9
1 1
Given number is 76 3× 4Y2. 2+ = 2+
1 1
Given number is divisible by 9. x+ 1+
So, 7 + 6 + 3 + X + 4 + Y + 2 = 9k 1 1
y+ 5+
Þ 22 + X + Y = 9k z 2
It is clear that LHS is divisible by 9, if X + Y = 5, 14.
\ z=2
Now sum of X and Y is 5, then possible pairs are
EBD_7367
16
A- Number System

107. (c) N = 35 × 45 × 55 × 60 × 124 × 75


5 + 10
102. (d) = 7 ´ 5 ´ 9 ´ 5 ´11´ 5 ´ 12 ´ 5 ´124 ´ 5 ´ 5 ´ 3
5 5 – 2 20 – 32 + 50
= 56 × 7 × 9 × 11 × 12 × 124 × 3
5 + 10 m = 6, The given number has maximum factor of 5 is 6.
= 108. (d) Let us assume that he was out of house for ‘t’ min.
5 5 – 4 5 – 4 2 +5 2
So angle formed by min. hand = 6 × t
Angle formed by hour hand = 0.5 × t
5 + 10 (5 + 10) ( 5 – 2)
= = Now, 0.5 × t + 6 × t = 360
5+ 2 ( 5 + 2) ( 5 – 2) Þ 6.5 t = 360
360
5 5 + 50 – 5 2 – 20 t= = 55.38 min
= 6.5
5–2
109. (b) When a ball is dropped
5 5 +5 2 –5 2 – 2 5 from a height = 36 m
=
3 2
1st bounce back = ´ 36 = 24 m
3
3 5
= = 5 2
3 2nd bounce back = ´ 24 = 16 m
3
103. (a) 81 × 82 × 83 × 84 × ... × 99
It can be written as 2 32 2
= 81 × 82 × 83 × 84 × ... 90 × 99 3rd bounce back ´ 16 = = 10 m.
3 3 3
When we multiply any number by multiple of 10, then 110. (d) 1st time seen
resultant number always carry zero at unit place. = 1 hour 54 min 50 sec
104. (a) Let a number be x = 3600 sec + 54 × 60 sec + 50 sec
multiply by 25 we get another number y. = 6890 sec
According to question 2nd time seen = 3 hour + 17 min + 49 sec
x × 25 = y ....(i) = 11869 sec.
Again x × 52 = 324 + y ....(ii) Interval between light seen = 11869 – 6890
Now subtract equation (i) from equation (ii) = 4979
Þ x × 52 – x × 25 = 324 + y – y Number of times light was seen
Þ 27x = 324
4979
= + 1 = 384
324 13
x= = 12
27
111. (b) n = P1x1 P2x2 P3x3
105. (a) 357P25Q
If it is divided by 3, then sum of the digit must be where P1, P2 and P3 are distinct prime numbers
divided by 3 and if it is divided by 5 then its unit digit So no. of factors = x1x2x3
must be 0 or 5. 112. (a) 11, 111, 1111, 11111, ...
=3+5+7+P+2+5+Q 1 Each number is divided by 4 then their remainder are
1st Case Q = 0 always 3.
3 + 5 + 7 + P + 2 + 5 + 0 = 22 + P So, it can be written as = 4 m + 3
Possible values of P = 2, 5, 8 So statement is true.
2 No, this type of numbers are not squares. So it is not
2nd Case when Q = 5
true.
3 + 5 + 7 + P + 2 + 5 + 5 = 27 + P
Only 1 is true.
Possible values of P = 0, 3, 6, 9
113. (c) Statement 1
Possible pairs of (P,Q) Yes, there exists only one prime number p such that
= (2, 0), (5, 0), (8, 0) (17p + 1) is a square.
(0, 5), (3, 5) (6, 5) (9, 5) Let p = 19
Total no. of pairs = 7.
106. (c) Let two consecutive odd numbers = (2x + 1) and (2x + 3) Þ (17 ´ 19 + 1) = 324 which is square of 18, so it is
According to question true.
= (2x + 3)2 – (2x + 1)2 Statement 2
= 4x2 + 12x + 9 – 4x2 –1 – 4x Product of first 10 consecutive number
= 8x + 8 = 8(x + 1) x (2, 3, 5, 7, 11, 13, 17, 19, 23, 29) = 6469693230
So, it is divisible by 8. \ x + 1 = 6469693231
Number System A-17

Divisibility by 7: The difference of the numbers upto


ì ü
thousands place and remaning part of the number if it is ï ± b + b 2 - 4ac ï
í using x = ý
divisible by 7 then the number is divisible by 7 ïî 2a ïþ
6469693 2 31
x = 25 ± 5 21
Thousand part Remaining part so, x2 – 50x + 100 > 0
Difference = 6469693 – 231
= 6469462 {x - ( 25 + 5 21)} {x - ( 25 - 5 21)} > 0
Again difference = 6469 – 462
= 6007
Again difference 7 – 6 = 1 which is not divisible by 7.
Therefore, number 6469693231 is not divisible by 7
So it is a prime number
So both statements are true. + +
114. (b) Number of account up to ` 10,000
= 532 – 218 12 3 4 46 47 48 x
= 314 accounts.
Rest of accounts of men deposits –
= 302 – 102 = 200 accounts
Number of accounts of women deposits = 314 – 200
= 114 As x £ 100 (Natural number)
115. (b) Given x + y + z = 0 So domain of x is = {1, 2, 48, 49, 50, 51------, 100}
x3 + y3 + z3 + 3xyz – 3xyz + 3xyz Hence no. of values of x is 55.
= x3 + y3 + z3 –3xyz + 6xyz Option (c) is correct.
= (x + y + z) (x2 + y2 + z2 – xy – yz –zx) + 6xyz 120. (c) (n3 – n) (n – 2)
(0) (x2 + y2 + z2 – xy – yz – zx) + 6xyz = n (n2 – 1) (n – 2)
= 6xyz = n(n – 1) (n + 1) (n – 2)
116. (d) 17256 (n – 2) (n – 1) n(n + 1)
Last digit is 7 so as n > 2 (natural number)
puting n = 3
We know that 7 repeats its unit digit after 4 times.
(n – 2) (n – 1) n(n + 1)
= (17)64×4 = (3 – 2) (3 – 1) 3(3 + 1)
Unit of 7 × 7 × 7 × 7 = 2401 = 1 × 2 × 3 × 4 = 24
Now, 256 is completely divided by 4 so unit digit = 1 So, option (c) is correct
1421´1423 ´1425 121. (d) (234)100 + (234)101
117. (c) See the pattern
12 41 = 4
When we divide 1421, 1423 and 1425 then 5,7 at 9 are 42 = 16
the remainders respectively. 43 = 64
5 ´ 7 ´ 9 315 44 = 256
= = =3 so, at odd power of 4 we get unit digit as ‘4’ and at
12 12 even power of 4 we get unit digit as ‘6’.
Þ (234)100 unit digit is 6
496 Þ (234)101 unit digit is 4
118. (a)
6 Þ (234)100 + (234)101 unit digit will be (6 + 4 = 10) zero.
When 41 is divided by 6 then remainder = 4 So, option (d) is correct.
42 is divided by 6 then remainder = 4 122. (*) 876p37q is divisible by 225 or 25 × 9. q has to be 5 and
43 is divided by 6 then remainder = 4 sum of all digits must be divided by 9.
\ p = 0 or 9 and
44 is divided by 6 then remainder = 4
Hence (c, d) are satisfying the condition.
................................................................ 123. (*) According to question.
................................................................ Þ 3x + 2y = 5k1 + 2 ----------(i)
496 divided by 6 then remainder = 4 Þ 2x + 3y = 5 k2 + 3 ----------(ii)
100 eq(i)-eq(ii)
119. (c) x∗ = 50 Þ x – y = 5(k1 – k2) – 1
x so when (x – y) is divided by 5 remainder will be
Þ x2 – 50x + 100 > 0 5 + (–1) = 4
At x = 5, y = 1, Remainder = 4
50 ± ∋50( , 4´100´1
2
At x = 6, y = 5, Remainder = 1
x< So, option (a) and (c) both are is correct
2
EBD_7367
18
A- Number System

124. (b) Given series is- n = 41, – 42


1 1 1 1 1 1 1 1 1 1 1 1 (1) is true (i.e., there are exactly two values of n for
+ - - - + + + - - - + +---- which Sn = 861)
4 5 6 4 5 6 4 5 6 4 5 6
144424443
Given Sn = Sn + 1
These 6 terms are repeating which are resulting to n ( n + 1) ( n + 1)( n + 2 )
zero. Þ =
2 2
so 1st 42 terms of this series will result is zero and n=n +2
after that series will be upto 47 terms – 2 ¹ 0 (this is not possible)
1 1 1 1 1 Þ 2 is true.
Þ + - - -
4 5 6 4 5 \ Option (c) is correct.

1 128. (c) (If p + q < p – q and if p + q > p – q


Þ -
6 Þ q + q < 0 Þq > – q
So, option (b) is correct. Þ q<0
125. (c) Let the number be (10x + y) Þ 2q > 0
According to question- 1 is true
Þ x + y= 7 -------------(i) Þ q> 0
(10y + x) – (10x + y) = 27 Þ q must be positive
Þ 9(y – x) = 27 Þ p is also positive
Þy–x =3 -------------(ii) Þ (2) is true.
From eq(i) and (ii) \ option (c) is correct
x = 2; y = 5 129. (*) The dividend is in the form an – bn
so x × y = 2 × 5 = 10 Here n is even
So, option (c) is correct. Þ 710 – 510 is divisible by both (7 + 5) and (7 – 5)
126. (c) No.s divided by 7 leaing 3 as reminder are - Þ 710 – 510 is divisible by 12 and 2
Hence, it is divisible by 12 and 2. No options mathcing
7 + 3 = 10 ü ï
ï with the given answer.
ï
14 + 3 = 17 ï ï 130. (c) Let two digit number = 10y + x
ï
21 + 3 = 24 ïïï According to question,
ï
28 + 3 = 31 ïï Ist condition, 10y + x = k(x + y) ...(1)
ï
35 + 3 = 38 ïï
ï
2nd condition, 10x + y = m(x + y) ...(2)
ï Adding (1) and (2) we get
42 + 3 = 45ï ï
ï
ï 11x + 11y = (k + m) × (x + y)
49 + 3 = 52ý Total 13 terms.
ï 11(x + y) = (k + m) (x + y)
56 + 3 = 59 ïï
ï k + m = 11
ï
63 + 3 = 66 ïï m = 11 – k
ï
ï
70 + 3 = 73ï ï \ Option (c) is correct
ï
77 + 3 = 80 ïï 131. (d) Here S be a set of 14 Natural Numbers i.e. {1, 2, 3, 4, 5.
ï
ï
84 + 3 = 87 ï ---- 14}
ï
ï
91 + 3 = 94 ï
Possible no. of pairs.
ï
ï
þ {a, b} is {(2, 8), (7, 13)}
Þ Pairs = 2
Sum = 3 + 13 + 7 [1 + 2 + 3 + -----+13] Þ Option (d) is correct.
æ 13´14 ö
= 39 + ç 7 ´ 132. (b) Time taken to reach 5’O Clock = 8 seconds
è 2 ÷ø
= 39 + 637 8
Time taken to reach 10’O Clock = seconds
= 676 5
So, option (c) is correct.
8
Time taken to reach 10’O Clock = ´ 10 =16 seconds
n ( n + 1) 5
127. (c) Given Sn =
2 \ Option (b) is correct.
Also Sn = 861 133. (b) Here N = 90 × 42 × 324 × 55
Now 90 = 3 × 3 × 10 = 32 × 10
n ( n + 1)
Þ 861 = 42 = 14 × 3 = 14 × 31
2 324 = 3 × 3 × 3 × 3 × 4 = 34 × 4
n2 + n – 1722 = 0 55 = 11 × 5
Number System A-19

N = 32 × 31 × 34 × 10 × 14 × 4 × 11 × 5
3 13 13 3 5 1
N = 37 × 10 × 14 × 4 × 11 × 5 +x = Þ x = - = = 2 hours
Maximum value of m = 7 4 4 4 4 2 2
\ Option (b) is correct. 1
134. (b) Option (b) is correct. Hence Madhukar received information 2 hours
2
135. (d) Option (d) is correct. before 2 p.m. i.e., 11 : 30 a.m.
136. (a) Since two hands are interchange their positions so
sum of the angles subtended at the centre by hour x 10 y
hand and minute hand = 360° 139. (d) = -
y 3 x
Let us suppose that she was out of house for t min.
So the sum of angles subtended at the centre by the
x 10 1
hour hand and minute hand. = -
Angle made by hour hand. y 3 y
i.e., 12h ® 360° x
360 x
Þ 1h = = 30° =z
12 Let
y
30
Þ 1min = = 0.5 min 10 1
60 \ z= - Þ 3z 2 - 10z + 3 = 0
3 z
Angle made by minute hand.
60 min = 360° 1
1 min = 60 Þ z = 3 or
3
Þ (0.5)t + (6 × t)° = 360°
Þ 6.5t = 360 Þ t = 55.38 min. x x 1
\ Option (a) is correct. Þ = 3 or =
y y 3
137. (b) Let m = 1 and n = 2 be two natural number
Þ x=9y ...(i) or 9x = y ...(ii)
m n 1 2 5 But x – y = 8 ...(iii)
(1) + = + =
n m 2 1 2 \ x> y
which is not an integer Hence eq. (ii) is rejected.
From eq. (i) and (iii),
æm n ö 2
(2) mn ç + ÷ m + n
è n mø
2
( ) 9y – y = 8, Þ y = 1 and x = 9
\ xy = 9 ´ 1 = 9
æ1 2ö 5 140. (a)
= 1´ 2 ç + ÷ (1 + 4 ) = 2 ´ ´ 5 = 25 1 2
è 2 1 ø 2
5 P 9
which is an integer
3 R 7
mn 1´ 2 2 +2 Q 8
(3) 2 2
= 2 2
=
m +n 1 +2 5
11 1 4
which is not an integer
\ Option (b) is correct. 2 + P + R + Q = 11
P + R+ Q = 9
138. (b) Let Madhukar received the informaton x hour before 2 For maximum value of Q
p.m. P and R be 0 then
Q=9
3 141. (b) Let the original number be x
hours
4 x hours ATQ
Last Madhukar Next x
meeting received meeting 8x – = 2016
over will be
8
information
held
é 63 ù
x ê ú = 2016
ë8û
13 8 by solving
hours x = 256
4
EBD_7367
20
A- Number System

142. (b) Through option 4.65 when multiplied by = 20 gives


(1)185 ´ ( -2)
whole number Þ
143. (d) (7)139 = 74(34)+3 9
unit digit of 74n is 1 and 73 = 34 is 3 1 ´ (-2) 7
than unit digit = 1 × 3 = 3 = i.e Remainder = 7
9 9
(4444)4444 145. (b) Number of prime numbers less than 100 are 25
144. (c) 146. (c) 1. Of two consecutive integer is always odd and
9
other is even. So this is true statement.
(7) 4444 2. Each odd no is in the form of (2n – 1)
when 4444 divided by 9 (2n + 1)2 = 4x2 + 1 + 4n
9
4 1111
147. (a) (N P -1 - 1) where n is prime to P then
( -2 )
= remainder will be 7 P is always a prime number.
9 148. (d) Let numbers be a, b and c respectively
According to the question
(16)1111
= or (–2) negative remainder a × b = 286
9 b × c = 770
Value of b is H.C.F of 286 and 770 = 22
(-2)1110 ´ (-2)
similarly (–2) negative \ a = 13 b = 22 c = 35
9 a + b + c = 13 + 22 + 35 = 70
149. (c) 1 × 3 × 5 × 7 .................. × 109
(-26 )185 ´ (-2) (64)185 ´ (-2)
= remainder If any odd number is multiplied by 5 it gives 5 on unit
9 9 place
Hence final unit digit is 5
HCF and LCM A-21

C HA P T E R
HCF AND LCM
2
1. What is the HCF of the polynomials x3 + 3x2y + 2xy2 and x4 9. The LCM of three different numbers is 150. Which of the
+ 6x3y + 8x2y2 ? [2007-I] following cannot be their HCF? [2008-I]
(a) x (x + 2y) (b) x (x + 3y) (a) 15 (b) 25
(c) 50 (d) 55
(c) x + 2y (d) None of these
10. What is the HCF of (x2 + bx – x – b) and [x2 + x (a – 1) – a]
2. If (x + k) is the HCf of (x2 + ax + b) and (x2 + cx + d), then ? [2008-I]
what is the value of k ? [2007-II] (a) x + b(b) x+a
(c) x + 1 (d) x–1
b+ d b+ d
(a) (b) 11. If the HCF of (x2 + x – 12) and (2x2 – kx – 9) is (x – k), then
a +c c+d what is the value of k ? [2008-I]
(a) – 3 (b) 3
a-b b-d
(c) (d) (c) – 4 (d) 4
c-d a -c 12. What is the LCM of (6x3 + 60x2 + 150x) and (3x4 + 12x3 –
3. If the HCF of x + mx – x + 2m and x2 + mx – 2 is a linear
3 2 15x2) ? [2008-I]
polynomial, then what is the value of m? (a) 6x2 (x + 5)2 (x – 1) (b) 3x2 (x + 5)2 (x – 1)
[2007-II] (c) 6x2 (x + 5)2 (x – 1)2 (d) 3x2 (x + 5) (x – 1)2
13. What is the LCM of (x + 2) (x – 2) and x2 – 4x – 12 ?
2
(a) 1 (b) 2 [2008-II]
(c) 3 (d) 4 (a) (x + 2) (x – 2)
4. A person has four iron bars whose lengths are 24 m, 36 m, 48 (b) (x + 2)3 (x – 2) (x – 6)
m and 72 m respectively. This person wants to cut pieces of (c) (x + 2) (x – 2)2
same length from each of four bars. What is the least number (d) (x + 2)2 (x – 2) (x – 6)
of total pieces if he is to cut without any wastage? 14. What is the value of k for which the HCF of 2x2 + kx – 12
[2007-II] and x2 + x – 2k – 2 is (x + 4) ? [2008-II]
(a) 5 (b) 7
(a) 10 (b) 15 (c) 10 (d) – 4
(c) 20 (d) 25 15. If (x + k) is the HCF of ax2 + ax + b and x2 + cx + d, then what
5. 5 bells start tolling together and toll at intervals of 2, 4, 6, 8 is the value of k? [2008-II]
and 10 s, respectively. How many times do the five bells toll
b+ d a+b
together in 20 min? [2007-II] (a) (b)
a +c c+d
(a) 10 (b) 11
(c) 12 (d) 15 a-b
(c) (d) None of these
6. LCM of two numbers is 16 times their HCF. The sum of LCM c-d
and HCF is 850. If one number is 50, then what is the other 16. 21 mango trees, 42 apple trees and 56 orange trees have to
number ? [2007-II] be planted in rows such that each row contains the same
(a) 800 (b) 1200 number of trees of one variety only. What is the minimum
(c) 1600 (d) 2400 number of rows in which the above trees may be planted?
7. If HCF of m and n is 1, then what are the HCF of m + n, m and [2008-II]
HCF of m – n, n, respectively ? (m > n) (a) 3 (b) 15
[2007-II] (c) 17 (d) 20
(a) 1 and 2 (b) 2 and 1 17. What is the greatest number which divides 392, 486 and 627
(c) 1 and 1 (d) Cannot be determined so as to leave the same remainder in each case? [2009-I]
(a) 47 (b) 43
8. Let p, q and r be natural numbers. If m is their LCM and n is (c) 37 (d) 34
their HCF, consider the following 18. What is the LCM of (x2 – y2 – z2 – 2yz), (x2 – y2 + z2
I. mn = pqr if each p, q and r is prime. + 2xz) and (x2 + y2 – z2 – 2xy) ? [2009-I]
II. mn = pqr if p, q and r are relatively prime in pairs. (a) (x + y + z) (x + y – z) (x – y + z)
Which of the above statement is/are correct? [2008-I] (b) (x + y + z) (x – y – z) (x – y + z)
(a) Only I (b) Only II (c) (x + y + z) (x + y – z) (x – y – z)
(c) Both I and II (d) Neither I nor II (d) (x + y – z) (x – y – z) (x – y + z)
EBD_7367
22
A- HCF and LCM

19 If (x + 2) is the HCF of x2 + ax + b and x2 + cx + d (where, a 30. If f (x) and g (x) are two polynomials with integral
¹ c and b ¹ d), then which one of the following is correct? 1
[2009-I] co-efficients which vanish at x = , then what is the factor
(a) a + c = b + d (b) 2a + b = 2c + d 2
(c) b + 2c = 2a + d (d) b – 2c = 2a – d of HCF of f (x) and g (x) ? [2010-II]
20. What are the values of c when the HCF of x3 + cx2 – x + 2c (a) x – 1 (b) x–2
and x2 + cx – 2 over the rationals is a linear polynomial ? (c) 2x – 1 (d) 2x + 1
[2009-I] 31. What is the sum of the digits of the least number which
(a) ± 1 (b) ± 2 when divided by 52, leaves 33 as remainder, when divided
(c) ± 3 (d) ± 4 by 78 leaves 59 and when divided by 117, leaves 98 as
21. For two natural numbers m and n, let gmn denote the greatest remainder ? [2010-II]
common factor of m and n. Consider the following in respect (a) 17 (b) 18
of three natural numbers k, m and n.
(c) 19 (d) 21
I. gm(nk) = g(mn)k
II. gmn gnk = gmk 32. What is the HCF of 4x + 3x y – 9xy2 + 2y3 and x2 + xy – 2y2
3 2

Which of the above statement is/are correct? [2009-II] ? [2010-II]


(a) Only I (b) Only II (a) x – 2y (b) x – y
(c) Both I and II (d) Neither I nor II (c) (x + 2y) (x – y) (d) (x – 2y) (x – y)
22. If the HCF of x3 – 27 and x3 + 4x2 + 12x + k is a quadratic 33. A number when dinded by 2, 3 or 5 given remainder 1. The
polynomial, then what is the value of k ? [2009-II] number is [2010-II]
(a) 27 (b) 9 (a) 31 (b) 47
(c) 3 (d) – 3
(c) 43 (d) 53
23. What is the LCM of 3(a3 – b3) and 11 (a4 – b4) ?
[2009-II] 34. The HCF of two polynomials p(x) and q(x) is 2x (x + 2) and
(a) 33(a3 – b3) (a2 + b2) (a + b) LCM is 24x (x + 2)2 (x – 2). If p(x) = 8x3 + 32x2 + 32x, then
what is q(x) equal to? [2011-I]
(b) (a3 – b3) (a2 + b2) (a – b)
(c) 33(a – b) (a2 + b2 – ab) (a3 – b3) (a) 4x3 – 16x (b) 6x3 – 24x
(d) 33(a3 – b3) (a4 – b4) (c) 12x3 + 24x (d) 12x3 – 24x
24. What is the least number which when divided by 42, 72 and 35. What is the least number of square tiles required to pave
84 leaves the remainders 25, 55 and 67, respectively? the floor of a room 9 m 99 cm long and 4 m 7 cm broad?
[2009-II] [2011-I]
(a) 521 (b) 512 (a) 247 (b) 277
(c) 504 (d) 487
25. For any integer n, what is the HCF of integers m = 2n + 1 and (c) 297 (d) 307
k = 9n + 4? [2010-I] 36. What is the HCF of 3.0, 1.2 and 0.06 ? [2011-I]
(a) 3 (b) 1 (a) 0.6 (b) 0.06
(c) 2 (d) 4 (c) 6.0 (d) 6.06
26. Consider the following statements: 37. If the HCF of three numbers 144, x and 192 is 12, then the
I. The HCF of x + y and x10 – y10 is x + y. number x cannot be [2011-I]
II. The HCF of x + y and x10 + y10 is x + y.
(a) 180 (b) 84
III. The HCF of x – y and x10 + y10 is x – y.
IV. The HCF of x – y and x10 – y10 is x – y. (c) 60 (d) 48
Which of the statement given above are correct ? 38. What is the HCF of (x4 – x2 – 6) and (x4 – 4x2 + 3) ?
[2010-I] [2011-I]
(a) I and II (b) II and III (a) x2 – 3 (b) x + 2
(c) I and IV (d) II and IV (c) x + 3 (d) x2 + 3
27. If the highest common factor of two positive integers is 24,
39. What is the LCM of a 3 b – ab 3 , a 3 b 2 + a 2 b 3 and
then their least common multiple cannot be [2010-I]
ab (a + b) ? [2011-II]
(a) 72 (b) 216
(c) 372 (d) 600 (a) a2b2 (a2 – b2) (b) ab (a2 – b2)
28. Consider those number between 300 and 400 such that when (c) a2b2 + ab3 (d) a3b3 (a2 – b2)
each number is divided by 6, 9 and 12, it leaves 4 as remainder 40. What is the HCF of the polynomials x4 – 3x + 2, x3 – 3x2 + 3x
in each case. What is the sum of the numbers? [2010-I] – 1 and x4 – 1? [2011-II]
(a) 692 (b) 764 (a) x – 1 (b) x+1
(c) 1080 (d) 1092 (c) x2 – 1 (d) None of these
29. What is the smallest positive integer which when divided
by 4, 5, 8 and 9 leaves remainder 3, 4, 7 and 8, respectively? 41. The least number which when divided by 5, 6, 7 and 8 leaves
[2010-II] a remainder 3 is [2011-II]
(a) 119 (b) 319 (a) 423 (b) 843
(c) 359 (d) 719 (c) 1683 (d) 2523
HCF and LCM A-23

2 7 53. The HCF of (x4 – y4) and (x6 – y6) is [2013-I]


14
42. What is the LCM of , and ? [2011-II] (a) x2 – y2 (b) x – y
3 9 15
(c) x3 – y3 (d) x4 – y4
7 14 54. The LCM of two numbers is 2376 while their HCF is 33. If
(a) (b)
3 3 one of the numbers is 297, then the other number is
[2013-I]
2 1
(c) (d) (a) 216 (b) 264
3 3
(c) 642 (d) 792
43. The product of two numbers is 6912 and their GCD is 24.
What is their LCM? [2011-II] 55. What is the LCM of x2 + 2x – 8, x3 – 4x2 + 4x and x2 + 4x ?
[2013-II]
(a) 280 (b) 286
(c) 288 (d) 296 (a) x (x + 4) (x – 2)2 (b) x (x + 4) (x – 2)
44. The sum of two numbers is 232 and their HCF is 29. What is (c) x (x + 4) (x + 2)2 (d) x (x + 4)2 (x – 2)
the number of such pairs of numbers satisfying the above 56. What is the HCF of a b + 2a2b2 and (ab)7 – 4a2b9?
2 4

condition? [2012-I] [2013-II]


(a) One (b) Two (a) ab (b) a2b3
(c) Four (d) None of these (c) a2b2 (d) a3b3
45. What is the HCF of 36(3x + 5x3 – 2x2), 9(6x3 + 4x2 – 2x) and
4
57. The HCF of two numbers is 98 and their LCM is 2352. The
54(27x4 – x)? [2012-I] sum of the numbers may be [2013-II]
(a) 9x (x + 1) (b) 9x (3x – 1) (a) 1372 (b) 1398
(c) 18x (3x – 1) (d) 18x (x + 1) (c) 1426 (d) 1484
46. The HCF and LCM of two natural numbers are 12 and 72, 58. If for integers a, b and c, if HCF (a, b) = 1 and HCF
respectively. What is the difference between the two (a, c) = 1, then which one of the following is correct?
numbers, if one of the numbers is 24 ? [2012-I] [2013-II]
(a) 12 (b) 18 (a) HCF (a, bc) = 1 (b) HCF (a, bc) = a
(c) 21 (d) 24 (c) HCF (a, bc) = b (d) None of these
47. If (x – 6) is the HCF of x2 – 2x – 24 and x2 – kx – 6, then what 59. What is the HCF of 8(x5¢ – x3 + x) and 28 (x6 + 1) ?
is the value of k ? [2012-II] [2014-I]
(a) 3 (b) 5 (a) 4(x4 – x2 + 1) (b) 2(x4 – x2 + 1)
(c) 6 (d) 8 (c) (x4– x2 + 1) (d) None of these
48. What is the greatest number that divides 13850 and 17030 60. In a fire range, 4 shooters are firing at their respective targets.
and leaves a remainder 17? [2012-II] The first, the second, the third and the fourth shooters hit
(a) 477 (b) 159 the target once is every 5 s, 6 s, 7 s and 8 s, respectively. If
all the them hit their target at 9:00 am, when will they hit
(c) 107 (d) 87 their target together again? [2014-I]
49. Three planets revolve round the Sun once in 200, 250 and (a) 9 : 04 am (b) 9 : 08 am
300 days, respectively in their own orbits. When do they all
come relatively to the same position as at a certain point of (c) 9 : 14 am (d) None of these
time in their orbits? [2012-II] 61. For any integer n, what is HCF (22n + 7, 33n + 10) equal to?
[2014-I]
(a) After 3000 days (b) After 2000 days
(a) n (b) 1
(c) After 1500 days (d) After 1200 days
(c) 11 (d) None of these
50. The product of HCF and LCM of 18 and 15 is
62. For any integers ‘a’ and ‘b’ with HCF (a, b) = 1, what is HCF
[2012-II] (a + b, a – b) equal to ? [2014-I]
(a) 120 (b) 150 (a) It is always 1 (b) It is always 2
(c) 175 (d) 270 (c) Either 1 or 2 (d) None of these
51. What is the HCF of the polynomials x3 + 8, x2 + 5x + 6 and 63.. If a and b be positive integers, then HCF
x3 + 2x2 + 4x + 8? [2013-I]
æ a b ö
(a) x + 2 (b) x+3 ç , ÷ equal to ? [2014-I]
2 è HCF( a , b ) HCF( a , b ) ø
(c) (x + 2) (d) None of these
52. The LCM of (x3 – x2 – 2x) and (x3 + x2) is [2013-I] (a) a (b) b
3
(a) x – x – 2x2 2
(b) x + x a
(c) 1 (d)
4
(c) x – x – 2x3 2
(d) x – 2 HCF (a, b)
EBD_7367
24
A- HCF and LCM

64. What is the number of integral solutions of the equations 75. Consider the following in respect of natural numbers a, b
HCF (a, b) = 5 and a + b = 65 ? [2014-I] and c: [2016-I]
(a) None (b) Infinitely many 1. LCM (ab, ac) = a LCM (b, c)
(c) Less than 65 (d) Exactly one 2. HCF (ab, ac) = a HCF (b, c)
65. The LCM of two intergers is 1237. What is their HCF ? 3. HCF (a, b) < LCM (a, b)
[2014-II] 4. HCF (a, b) divides LCM (a, b)
(a) 37 (b) 19 Which of the above are correct?
(c) 1 (d) Cannot be determined (a) 1 only (b) 3 and 4 only
(c) 1, 2 and 4 only (d) 1, 2, 3 and 4
66. There are 48 cricket balls, 72 hockey balls and 84 tennis 76. There are two numbers p and q such that their HCF is 1.
balls and they have to be arranged in several rows in such Which of the following statements are correct? [2016-II]
a way that every row contains the same number balls of 1. Both p and q may be prime.
balls of one type. What is the minimum number of rows 2. One number may be prime and the other composite.
required for this to happen ? [2014-II] 3. Both the numbers may be composite.
(a) 12 (b) 16 Select the correct answer using the code given below : ,
(c) 17 (d) 19 (a) 1 and 2 only (b) 2 and 3 only
67. The HCF of two natural numbers m and n is 24 and their (c) 1 and 3 only (d) 1, 2 and 3
product is 552. How many sets of values of m and n are 77. Consider the following statements : [2017-I]
possible ? [2014-II] 1. If a = bc with HCF (b, c) = 1, then
(a) 1 HCF (c, bd) = HCF (c, d).
(b) 2 2. If a = bc with HCF (b, c) = 1, then
(c) 4 LCM (a, d) = LCM (3, bd).
(d) No set of m and n is possible satisfying the given Which of the above statements is/are correct ? [2017- I]
conditions (a) 1 only (b) 2 only
68. The LCM of two numbers is 90 times their HCF. The sum of (c) Both 1 and 2 (d) Neither 1 nor 2
LCM and HCF is 1456. If one of the numbers is 160, then 78. What is the LCM of x3 + 8, x2 + 5x + 6 and x3 + 4x2 + 4x ?
what is the other number ? [2014-II] (a) x (x + 2)2 (x + 3) (x2 – 2x + 4)
(a) 120 (b) 136 (b) x (x – 2)2 (x – 3) (x2 + 2x + 4)
(c) 144 (d) 184 (c) (x + 2)2 (x + 3) (x2 – 2x + 4)
69. The HCF and LCM of two polynomials are (x + y) and (d) (x–2)2 (x–3) (x2 – 2x + 4)
(3x5+ 5x4y + 2x3y2– 3x2y3– 5xy4–2y5) respectively. If one of 79. The HCF of two expressions p and q is 1. What is the
the polynomials is (x2 – y2), then the other polynomial is reciprocal of their LCM ? [2017- I]
2015-I] (a) p + q (b) p – q
(a) 3x4 –8x3y+10x2y2 +7xy3 –2y4 (c) pq (d) (pq)–1
(b) 3x4 – 8x3y – 10x2y2 + 7xy3 + 2y4 80. How many numbers from 1 to 1000 are divisible by 2, 3, 4
(c) 3x4 + 8x3y+10x2y2 + 7xy3 +2y4 and 5? [2017- II]
(d) 3x4 + 8x3y – 10x2y2 + 7xy3 + 2y4 (a) 16 (b) 17
70. What is the sum of digits of the least multiple of 13, which (c) 32 (d) None of the above
when divided by 6, 8 and 12 leaves 5, 7 and 11 respectively 81. The product of two non-zero expressions is (x + y + z) p3. If
as the remainders ? [2015-II] their HCF is p2, then their LCM is [2017- II]
(a) (x + y + z) (b) (x + y + z) p2
(a) 5 (b) 6
(c) (x + y + z) p5 (d) (x + y + z) p
(c) 7 (d) 8 82. If the HCF of polynomials [2017- II]
71. A number when divided by 7 leaves a remainder 3 and the f(x) = (x – 1) (x2 + 3x + a) and
resulting quotient when divided by 11 leaves a remainder 6. g(x) = (x + 2) (x2 + 2x + b) is (x2 + x – 2),
If the same number when divided by 11 leaves a remainder then what are the values of a and b respectively?
m and the resulting quotient when divided by 7 leaves a (a) 2, 2 (b) 2, –3
remainder n. What are the values of m and n respectively? (c) –1, –3 (d) –2, –1
[2015-II] 83. How many numbers between 500 and 1000 are divisible by
(a) 1 and 4 (b) 4 and 1 13? [2017- II]
(c) 3 and 6 (d) 6 and 3 (a) 36 (b) 37
72. If (x+1) is the HCF of Ax2 + Bx + C and Bx2 + Ax + C where (c) 38 (d) 39
A ¹ B, then the value of C is [2015-II] 84. There are two numbers which are greater than 21 and their
(a) A (b) B LCM and HCF are 3003 and 21 respectively. What is the
(c) A–B (d) 0 sum of these numbers? [2018-I]
73. The LCM of two numbers is 12 times their HCF. The sum of (a) 504 (b) 508
HCF and LCM is 403. If one of the numbers is 93, then the (c) 514 (d) 528
other number is [2015-II] 85. Walls (excluding their roofs and floors) of 5 identical rooms
(a) 124 (b) 128 having length, breadth and height 6 m, 4 m and 2.5 m
respectively are to be painted. Paints are available only in
(c) 134 (d) 138 cans of 1 L and one litre of paint can be used for painting 20
74. The sum and difference of two expressions are 5x2 – x – 4 square metres. What is the number of cans required for
and x2 + 9x – 10 respectively. The HCP of the two expression painting? 2018-I]
will be [2016-I] (a) 10 (b) 12
(a) (x + 1) (b) (x – 1)
(c) 13 (d) 14
(c) (3x + 7) (d) (2x – 3)
HCF and LCM 25
A-

HINTS & SOLUTIONS


1. (a) Let f1(x) = x3 + 3x2y + 2xy2 8. (c) Let P = 10, q = 11, r = 13 (Co-prime numbers)
= x (x2 + 3yx + 2y2) LCM of (10, 11, 13) = 1430
= x (x2 + 2xy + xy + 2y2) HCF (10, 11, 13) = 1
= x [x (x + 2y) + y (x + 2y)] mn = 1430 × 1 = 1430
= x (x + y) (x + 2y) Also, pqr = 10 × 11 × 13 = 1430
and f2(x) = x4 + 6x3y + 8x2y2 So, mn = pqr
= x2 (x2 + 6xy + 8y2) 9. (d) We know that LCM is the multiple of HCF. So that 55
= x2 (x2 + 2xy + 4xy + 8y2) cannot be HCF because it is not divisor of 150.
= x2 [x (x + 2y) + 4y (x + 2y)] 10. (d) Let f1(x) = x2 + bx – x – b
= x2 (x + 2y) (x + 4y) = x (x + b) – 1 (x + b)
\ HCF of f1(x) and f2(x) = x (x + 2y) = (x – 1) (x + b)
2. (d) (x + k) is the HCF of (x2 + ax + b) and (x2 + cx + d). and f2(x) = x2 + xa – x – a
\ (– k)2 + a (– k) + b = 0
= x (x + a) – 1 (x + a)
= (– k)2 + c (– k) + d
= (x + a) (x – 1)
Þ (a – c) k = (b – d)
\ HCF of f1(x) and f2(x) = (x – 1)
b-d 11. (b) HCF of x2 + x – 12 and 2x2 – kx – 9 is (x – k),
Þ k=
a -c then x = k will be the factor of 2x2 – kx – 9
3. (a) Let f1(x) = x3 + mx2 – x + 2m \ 2k2 – k2 – 9 = 0
and f2(x) = x2 + mx – 2 Þ k2 – 9 = 0
Let m = 1 k=±3
\ f1(x) = x3 + x2 – x + 2 and factor of x2 + x – 12 are (x + 4) (x – 3).
and f2(x) = x2 + x – 2 = (x + 2) (x – 1) Hence, the value of k is 3.
When x = 1, 12. (a) Let f1(x) = 6x3 + 60x2 + 150x
f (1) = 1 + 1 – 1 + 2 ¹ 0
= 6x (x2 + 10x + 25)
When x = – 2,
= 3 × 2 × x × (x + 5)2
f (– 2) = (–2)3 + (–2)2 – (– 2) + 2 = 0
Required value of m is 1. and f2(x) = 3x4 + 12x3 – 15x2
4. (b) 24 = 12 × 2, = 3x2 (x2 + 4x – 5)
36 = 12 × 3, = 3x2 (x2 + 5x – x – 5)
48 = 12 × 4, = 3x2 (x + 5) (x – 1)
and 72 = 12 × 6 \ LCM of f1(x) and f2(x)
\ HCF (24, 36, 48, 72) = 12 = 3 × 2 × x2 × (x + 5)2 (x – 1)
Total pieces = 2 + 3 + 4 + 6 = 15 = 6x2 (x + 5)2 (x – 1)
5. (a) LCM of 2, 4, 6, 8 and 10 is 120s. 13. (d) Let f1(x) = (x + 2)2 (x – 2)
i.e., 2 min after tolling together. and f2(x) = x2 – 4x – 12 = (x – 6) (x + 2)
Total time \ LCM of f1(x), f2(x) = (x + 2)2 (x – 2) (x – 6)
Total in 20 min =
LCM t intervals 14. (a) (x + 4) is HCF, so it will be common in both expression.
x = – 4 will make each one zero.
20 min
In 20 min tolling = = 10 times. \ 2 (– 4)2 + k (– 4) – 12 = 0
2 min and (– 4)2 + (– 4) – 2k – 2 = 0
6. (a) Let first number = x, second number = y
Þ 32 – 12 = 4k
\ LCM × HCF = Product of numbers = x × y
Also, LCM = 16 HCF, LCM + HCF = 850 and and 16 – 6 = 2k
x = 50 \ k=5
\ 17 HCF = 850 15. (d) x + k is the HCF of given expression, then
Þ HCF = 50 x = – k, root of the function
Now, LCM = 16 × 50 = 800 Now, a (– k)2 – ak + b = 0 = k2 – ck + d
\ 800 × 50 = 50 × y Þ k2 (a – 1) – (a – c) k + b – d = 0
\ y = 800
7. (c) HCF of m and n is 1. (a - c) ± (a - c )2 - 4(a - 1)(b - d )
Þ k=
\ HCF (m + n, m) = 1 2(a - 1)
and HCF (m – n, n) =1
EBD_7367
26
A- HCF and LCM

21 42 56 25. (b) Here n = Integer.


16. (c) Minimum number of rows = + + = 17. m = 2n + 1 and k = 9n + 4
7 7 7
HCF of (m, k) = HCF (2n + 1, 9n + 4)
17. (a) Given numbers are 392, 486 and 627.
For same remainder
486 – 392 = 94
627 – 486 = 141 – –
627 – 392 = 235
HCF of (94, 141, 235) = 47 –
18. (b) LCM of [{x2 – (y + z)2}, {(x + z)2 – y2}, {(x – y)2 – z2}]
= LCM of [(x + y + z) (x – y – z), (x + z + y) ×
(x + z – y), (x – y + z) (x – y – z)]
= (x + y + z) (x – y – z) (x – y + z) HCF of (m, k) = 1
19. (c) Let f (x) = x2 + ax + b and g (x) = x2 + cx + d 26. (c) If we put x = y in the expression x10 – y10, then x10 – y10
\ f (– 2) = 4 – 2a + b = 0 Þ b – 2a = – 4 = 0. So (x – y) is factor of x10 – y10
and g (– 2) = 4 – 2c + d = 0 Þ d – 2c = – 4 Similarly, if we put x = – y in the expression x10 – y10 =
\ b – 2a = d – 2c Þ b + 2c = d + 2a 0. So (x + y) is a factor of x10 – y10
20. (a) Let f1(x) = x3 + cx2 – x + 2c So, statement I and IV are true.
and f2(x) = x2 + cx – 2 27. (c) In the given options, only 372 is not divisible by 24.
Let c = 1 Therefore, LCM of numbers cannot be 372.
\ f1(x) = x3 + x2 – x + 2 = (x + 2) (x2 – x + 1) 28. (a) LCM of (6, 9, 12) = 36
f2 (x) = x2 + x – 2 = (x + 2) (x – 1) Then, number is the form of 36p + 4.
Here, (x + 2) is HCF of f1(x) and f2(x) which is linear. Since, the required number between 300 and 400.
21. (d) Let 2 natural numbers m and n are 12 are 18. \ p = 9 and 10
\ HCF of 12 and 18 = gmn = 6 \ Required sum = 328 + 364 = 692
Let third natural number k is 20. 29. (c) LCM of (4, 5, 8, 9) = 360
So, gnk = HCF (18, 20) = 2 and gmk = 4 Difference between divisor and remainder.
So, gm(nk) = HCF (12, 360) = 12 = 3 – 4 = 4 – 5 = 7 – 8.
So, g(mn)k = HCF (216, 20) = 4 =8–9=–1
From I statement, gm(nk) ¹ g(mn)k Hence, Required value = 360 – 1 = 359.
From II statement, gmn gnk ¹ gmk
So, both statements are false. 1
30. (c) Q f (x) and g (x) vanish at x =
22. (b) Given that x3 – 27 = (x – 3) (x2 + 9 + 3x) 2
So, (2x – 1) is a factor of f (x) and g (x) both.
x +1 Hence, a factor of f (x) and g (x) = 2x – 1.
2 3 2
x + 9 + 3x x + 4 x + 12 x + k 31. (a) Here, 52 – 33 = 78 – 59 = 117 – 98 = 19
3
_ x ± 3x ± 9 x 2 Now, 52 = 13 × 2 × 2
78 = 13 × 2 × 3
x2 + 3 x + k 117 = 13 × 3 × 3
_ x2 ± 3x ± 9 \ LCM = 13 × 2 × 2 × 3 × 3 = 468
k -9 \ Required number = 468 – 19 = 449
Hence, the sum of digits is 17.
Hence the value of k should be 9. 32. (c) Given function
23. (a) Given expression can be written as f1(x) = 4x3 + 3x2y – 9xy2 + 2y3
3(a3 – b3) = 3(a – b) (a2 + b2 + ab) = (x – y) (4x2 + 7xy – 2y2)
and 11(a4 – b4) = 11(a – b) (a + b) (a2 + b2) = (x – y) (x + 2y) (4x – y)
\ Required LCM = 33 (a3 – b3) (a + b) (a2 + b2) and another function
24. (d) LCM of 42, 72 and 84 = 504 f2(x) = x2 + xy – 2y2 = (x – y) (x + 2y)
Difference between divisor and remainder HCF of f1(x) and f2(x) = (x – y) (x + 2y)
= 25 – 42 = 55 – 72 = 67 – 84 33. (a) LCM of 2, 3, 5 = 30
= – 17 30 is least number which is divided by 2, 3 and 5.
Required number = 504 – 17 = 487. The number of which gives reminder = 30 + 1 = 31.
HCF and LCM A-27

34. (b) Q p(x) × q(x) = LCM × HCF 44. (b) Let two numbers by 29x and 29y.
\ (8x3 + 32x2 + 32x) × q(x) \ 29x + 29y = 232 Þ x + y = 8
= 2x (x + 2) × 24x (x + 2)2 (x – 2) Þ (x, y) = (1, 7), (3, 5)
Since, one such pair is 87 and 145.
48 x 2 ( x + 2)3 ( x - 2)
Þ q(x) = Hence, the other pairs is 203 and 29.
8 x( x 2 + 4 x + 4)
45. (c) Let f1 (x) = 36 (3x4 + 5x3 – 2x2)
3
6 x( x + 2) ( x - 2) = 36x2 (3x2 + 5x – 2)
=
( x + 2)2 = 36 . x2 {3x2 + 6x – x – 2}
= 6x (x2 – 4) = 6x3 – 24x = 36 . x2 {3x (x + 2) – 1 (x + 2)}
35. (c) For the least number of tiles to pave the floor, the size = 2 × 2 × 3 × 3 × x × x × (x + 2) (3x – 1)
of tiles should be maximum. f2 (x) = 9 (6x3 + 4x2 – 2x)
\ Side of tiles = HCF of 999 and 407 = 37 cm = 9x (6x2 + 4x – 2)
999 ´ 407 = 18x (3x2 + 2x – 1)
\ Required number of tiles = = 297 = 18x (3x2 + 3x – x – 1)
37 ´ 37
36. (b) Multiply by 100 = 3 × 3 × 2 × x (3x – 1) (x + 1)
3.0 = 300 f3 (x) = 54(27x4 – x)
1 . 2 = 120 = 54x(27x3 – 1)
0.06 = 6 = 2 × 3 × 3 × 3 × x × (3x – 1) (9x2 + 3x + 1)
Now, HCF of (300, 120, 6) = 6
\ HCF of f1 (x), f2 (x), f3 (x)
So, HCF (3.0, 1.2, 0.06) = 0.06
37. (d) Here given the HCF of 144, x, 192 is 12. = 2 × 3 × 3 × x × (3x – 1)
= 18x (3x – 1)
1
144 192 LCM × HCF 72 ´ 12
46. (a) Second number = = = 36
144 First number 24
´48 x \ Difference between two numbers = 36 – 24 = 12
x can not be 48 because HCF is 12. 47. (b) Here, (x – 6) is the HCF of x2 – 2x – 24 and x2 – kx – 6
38. (a) x4 – x2 – 6 = x4 – 3x2 + 2x2 – 6 So, that (x – 6) is a factor of both expression.
= x2 (x2 – 3) + 2 (x2 – 3) According to question,
= (x2 – 3) (x2 + 2) Þ f (x1) = f (x2) at (x1 = x2 = 6)
and x4 – 4x2 + 3 = x4 – 3x2 – x2 + 3 Þ (6)2 – 2(6) – 24 = (6)2 – k(6) – 6 (By condition)
= x2 (x2 – 3) – 1 (x2 – 3)
Þ 36 – 12 – 24 = 36 – 6k – 6
= (x2 – 3) (x2 – 1)
\ HCF = (x2 – 3) Þ 0 = 30 – 6k Þ 6k = 30
39. (a) Here, a3b – ab3 = ab (a2 – b2) = ab (a – b) (a + b) \ k=5
a3b2 + a2b3 = a2b2 (a + b) and 48. (b) Here 13850 and 17030 are two numbers which leaves
ab (a + b) = ab (a + b) remainder 17.
\ LCM [(a3b – ab3), (a3b2 + a2b3), ab (a + b)] Now, 13850 – 17 = 13833
= a2b2 (a + b) (a – b) = a2b2 (a2 – b2) 17013 – 17 = 17013
40. (a) Let f1 (x) = x4 – 3x + 2 = (x – 1) (x3 + x2 + x – 2) 13833 = 159 × 3 × 29
f2 (x) = x3 – 3x2 + 3x – 1 = (x – 1)3
17013 = 107 × 159
and f3 (x) = x4 – 1 = (x – 1) (x + 1) (x2 + 1)
\ HCF = 159.
\ HCF of f1(x), f2(x), f3(x) = x – 1
41. (b) We find out the LCM of given numbers. It is the greatest number.
\ LCM (5, 6, 7, 8) = LCM (5, 2 × 3, 7, 2 × 4) 49. (a) Given that, three planets revolves the Sun once in 200,
= 2 × 5 × 3 × 7 × 4 = 840 250, 300 days,.
\ Required number = 840 + 3 = 843 \ Required time = LCM of (200, 250, 300)
= 3000 days
æ 2 7 14 ö LCM(2, 7,14) 14
42. (b) LCM ç , , ÷ = = Now, after 3000 days they all come relatively to the
è 3 9 15 ø HCF(3,9,15) 3
same position as at a certain point of time in their orbits.
43. (c) Let the numbers be 24x and 24y.
50. (d) HCF of 18 and 15 = 3
6912 LCM of 18 and 15 = 2 × 3 × 3 × 5 = 90
\ 24x × 24y = 6912 Þ xy = = 12
(24) 2 \ Product of HCF and LCM of both numbers
\ LCM is 24 × 12 = 288 = 3 × 90 = 270
EBD_7367
28
A- HCF and LCM

51. (a) Let f1 (x) = x3 + 8 60. (c) Time after which they will hit the target again together
= x3 + 23 = (x + 2) (x2 – 2x + 4) = LCM (5, 6, 7 and 8)
= (x + 2) (x – 2)2 = (x + 2) (x – 2) (x – 2) 2 5, 6, 7,8
f2(x) = x2 + 5x + 6 = x2 + 3x + 2x + 6 5,3, 7, 4
= x (x + 3) + 2 (x + 3)
= 5 × 3 × 7 × 2 × 4 = 840 s
= (x + 3) (x + 2)
and f3(x) = x3 + 2x2 + 4x + 8 840
They hit after target together = = 14 min.
= x2 (x + 2) + 4 (x + 2) 60
= (x + 2) (x2 + 4) So, next time target = 9 : 00 am + 14 min
\ HCF of [f1(x), f2(x), f3(x)] = x + 2 = 9 : 14 am
52. (c) Let f1(x) = x3 – x2 – 2x = x (x2 – x – 2) 61. (b) HCF of (22n + 7, 33n + 10) is always 1
= x {x2 – 2x + x – 2} Examples
= x {x (x – 2) + 1 (x – 2)} = x (x + 1) (x – 2) For n = 1, HCF (29, 43) Þ HCF = 1
and f2(x) = x3 + x2 = x2 (x + 1) = x . x (x + 1) For n = 2, HCF (51, 76) Þ HCF = 1
For n = 3, HCF (73, 109) Þ HCF = 1
\ LCM of [f1(x), f2(x)] = x (x + 1) . x (x – 2)
since 22n and 33n are multiples of 11, therefore
= x2 (x + 1) (x – 2) = x2 (x2 – x – 2)
22n + 7 and 33n + 10 are not the multiple of 11.
= x4 – x3 – 2x2
Hence, HCF of 22n + 7 and 33 + 10 will not be equal to
53. (a) Let f1(x) = (x4 – y4) = [(x2)2 – (y2)2] 11 or n.
= (x2 – y2) (x2 + y2) 62. (c) Given that HCF (a, b) = 1 means that a and b are co-
= (x – y) (x + y) (x2 + y2) prime numbers.
and f2(x) = (x6 – y6) = (x3)2 – (y3)2 So, HCF (a + b, a – b)
= (x3 + y3) (x3 – y3) Let a = 4, b = 3
= (x + y) (x2 – xy + y2) (x – y) (x2 + xy + y2) HCF (4, 3) = 1
= (x – y) (x + y) (x2 – xy + y2) (x2 + xy + y2) Now, HCF (3 + 4, 4 – 3) = HCF (7, 1)
\ HCF of [f1(x), f2(x)] = (x – y) (x + y) = x2 – y2 HCF is equal = 1
54. (b) From formula, Let a = 23 and b = 17
\ (HCF to two numbers) × (LCM of two numbers) HCF (23, 17) = 1
= (First number) × (Second number) HCF (23 + 17, 23 – 17) = HCF (40, 6) = 2
So, HCF (a + b, a – b) = Either 1 or 2
33 ´ 2376
\ Second number = = 264 æ a b ö
297 63. (c) HCF ç , ÷
55. (a) x2 + 2x – 8 = x2 + 4x – 2x – 8 è HCF( a, b) HCF( a , b) ø
= x (x + 4) – 2 (x + 4) = (x – 2) (x + 4) HCF (a,b) HCF (a,b)
x3 – 4x2 + 4x = x3 – 2x2 – 2x2 + 4x = = =1
LCM(HCF (a,b), HCF (a,b)) HCF (a,b)
= x2 (x – 2) – 2x (x – 2)
Example 1: a = 12 and b = 24
= (x2 – 2x) (x – 2) = x (x – 2) (x – 2)
x2 + 4x = x (x + 4) æ 12 24 ö
HCF of ç , ÷
So, LCM of (x 2 + 2x – 8), (x 3 – 4x 2 + 4x) and è HCF(12, 24) HCF(12, 24) ø
(x2 + 4x) = x (x – 2) (x + 4) (x – 2)
= x (x + 4) (x – 2)2 æ 12 24 ö
= HCF ç , ÷ = HCF (1, 2) = 1.
56. (c) a2b4 + 2a2b2 = a2b2 (b2 + 2) è 12 12 ø
and (ab)7 – 4a2b9 = a7b7 – 4a2b9 Example 2: a = 19, b = 23
= a2b2 (a5b5 – 4b7) æ 19 23 ö
HCF of [(a2b4 + 2a2b2), ((ab)7 – 4a2b9) = a2b2 HCF of ç , ÷
è HCF(19, 23) HCF(19, 23) ø
57. (a) HCF of two numbers is 98. It means that 98 is common
in both the numbers. Therefore, the sum of these two æ 19 23 ö
= HCF of ç , ÷
numbers also be multiple of 98. So, 1372 is divided by è 1 1 ø
98. = HCF of (19, 23) = 1
58. (a) For integers a, b and c, if HCF (a, b) = 1 and HCF (a, c) 64. (c) Q HCF (a, b) = 5
= 1, then HCF (a, bc) = 1
Let a = 5x and b = 5y
59. (a) Let f1 (x) = 8(x5 – x3 + x)
\ 5x + 5y = 65
= 4 × 2 × x (x4 – x2 + 1) Þ x + y = 13
and f2(x) = 28(x6 + 1) = 7 × 4[(x2)3 + (1)3] \ Number of pairs of (x, y)
= 4 × 7 × (x2 + 1) (x4 – x2 + 1) = (1, 12), (2, 11) ((3, 10), (4, 9), (5, 8), (6, 7)
\ HCF of f1(x) and f2(x) = 4(x4 – x2 + 1) Hence, total number of solution is less than 65.
HCF and LCM A- 29
65. (c) Given, LCM of two intergers is 1237, which is a prime \ N = 11 (7k + 4) + 1 i.e. m = 1
number. and q1 = 7k + 4, q2 = k and n = 4
So, their HCF is 1 \ (m, n ) = (1, 4)
66. (c) HCF of 48, 72 and 84 = 22 × 3 = 12 So, option (a) is correct.
48 72 84 72. (d) If (x + 1) is HCF then x = –1 will satisfy both the
minimum number of rows = + + equation-
12 12 12 A(–1)2 + B(–1) + C = 0
= 4 + 6 + 7 = 17 Þ A–B+C =0 ----------(a)
67. (d) HCF of two natural numbers m and n = 24 B(–1)2 + A(–1) + C = 0
m × n = 552 Þ B –A+ C = 0 ----------(b)
LCM of two natural numbers Adding equ. (a) and (b)
Product of m and n A– B + C + B –A+ C = 0
=
HCFof m and n 2C = 0

552 C<0
= = 23 So, option (d) is correct.
24
Therefore, no set of m and n is possible satisfying the 73. (a) LCM × HCF = 1st number × 2nd number
given condition. Let the 1st number be ‘A’ and 2nd number be ‘B’.
68. (c) Let the HCF of two number = x L× H=A× B
The LCM of two numbers = 90x L × 12 L = 93 × B
12 L2 = 93 B --------------(i)
According to question
L + H = 403 --------------(ii)
LCM + HCF = 1456
L + 12L = 403
90x + x = 1456
x = 16 403
L< < 31
HCF of two numbers = 16, 13
LCM of two number = 1440 12 × 31 × 31 = 93 B
LCM × HCF = Ist numbers × 2nd number
12 ´31´31
1440 ´ 16 B=
nd
Þ 2 Number = = 144 93
160 B = 4 × 31
69. (c) HCF × LCM = 1st polynomial × 2nd polynomial B = 124
HCF ´ LCM So, option (a) is correct.
Þ 2nd polynomial = st 74. (b) Let p(x) and q(x) be two expressions,
1 Polynomial
then p(x) + q(x) = 5x2 – x – 4 ...(i)
5 4 3 2 2 3 4 5
( x + y ) ´ (3 x + 5 x y + 2 x y – 3 x y – 5 xy – 2 y ) 2
and p(x) – q(x) = x + 9x – 10 ...(ii)
=
(x2 – y2 ) Solving (i) and (ii), we get
= 3x4 + 8x3y + 10x2y2 + 7xy3 + 2y4 2p(x) = 6x2 + 8x – 14 and 2q(x) = 4x2 – 10x + 6
p(x) = 3x2 + 4x – 7 Þ q(x) = 2x2 – 5x + 3
70. (d) LCM of 6, 8, 12 is 24
Number is when divided by 6, 8 and 12 leaves 5, 6 and Þ p(x) = (3x + 7) (x – 1) and q(x) = (x – 1) (2x – 3)
11 as remainders, as 6 – 5 = 1, 8 – 7 = 1, 12 – 11 = 1 H.C.F. of two expressions p(x) and q(x) is (x – 1)
So (24 k – 1) will be divisible by 13. \ Option (b) is correct.
24k ,1 75. (a)
Þ is divisible for kmin = 6
13 76. (d) All the statement are true example for each statement.
at k = 6 1. take number 2 and 3
Number is = 24 k – 1 2. take number 7 and 25
= 24 × 6 – 1 = 143 3. take number 25 and 16
Sum of its digit = 1 + 4 + 3 = 8 and many more.
So, option (d) is correct 77. (a) Putting any random number
71. (a) This is an example of successive division. Let the eg a = 6, b = 2, c = 3 and d = 6 in 1 statement
number be N. The number and successive quotients, it can be varified that this statement is correct and
the successive divisors and the corresponding statement 2 is incorrect.
remainders are tabulated below : 78. (a) x3 + 23 = (x + 2) (x2 – 2x + 4)
Quotients N q1 q2 x2 + 5x + 6 = x2 + 2x + 3x + 6
Dinsors 7 11 = (x + 2) (x + 3)
Remainder 3 6 x3 + 4x2 + 4x = x (x2 + 4x + 4)
x (x + 2) (x + 2)
One value of N is 6(7) + 3 = 45 LCM = x (x + 2)2 (x + 3) (x2 – 2x + 4)
In general, N = 77K + 45
EBD_7367
30
A- HCF and LCM

79. (d) LCM of p and q is 1


x 2 + 3x + a = 0 Þ (-2)2 + 3. - 2 + a = 0
LCM = pq Þ 4 - 6 + a = 0 Þ -2 + a = 0 Þ a = 2
1 Also, since (x – 1) is a factor of x2 + 2x + b, therefore, x
Reciprocal = = pq–1 = 1 will satisfy this polynomial. Thus,
pq

80. (a) LCM of 2, 3, 4 and 5 is 60. x 2 + 2 x + b = 0 Þ 1 + 2 + b = 0 Þ b + 3 = 0 Þ b = -3


Number of numbers divisible by 60 from 1 to 600 = Hence, a = 2, b = – 3
10 83. (c) Consider a = 507, b = 988, d = 13
Number of numbers divisible by 60 from 601 to 900
b-a
i.e. from 300 numbers = 5 n= +1
d
Number of numbers divisible by 60 from 901 to 1000 988 - 507
i.e. from 100 numbers = 1 n= +1
13
Total numbers = 10 + 5 + 1 = 16 481
n= +1
81. (d) We are given that HCF = p2 and the product of two 13
non-zero expressions = (x + y + z) p3 n = 37 + 1
We know that HCF × LCM = Product of two numbers n = 38
Therefore, 84. (a) Let numbers be 21x and 21y
p 2 ´ LCM = ( x + y + z ) p 3 Þ LCM According to the question
21x × 21y = 21 × 3003
( x + y + z ) p 3 = (x + y + z) p
= 21 ´ 3003
p2 x+y= = 143
21 ´ 21
82. (b) Since HCF of two polynomials is x2 + x – 2, therefore
splitting this polynomial by middle term, we get Possible pairs of values of x and y are (1, 143) and (11, 13)
Both numbers are greater than 21. Hence, (11, 13) is
x2 + x - 2 = x 2 + 2 x - x - 2 = x( x + 2) - ( x + 2) right pair of co-prime factors
= (x – 1)(x + 2) Sum of numbers = 11 × 21 + 13 × 21 = 231 + 273 = 504
Being the HCF of the given polynomials, we conclude 85. (c) length of each room = 6 m
that (x – 1)(x + 2) is a factor of f (x) and g (x). breadth of each room = 4 m
Therefore, dividing f (x) by the HCF and dividing g (x) height of each room = 2.5 m
by the HCF will give the values of a and b.
area of walls of each room = 2(6 + 4) × 2.5 = 50 m2
2 2
( x - 1)( x + 3 x + a ) ( x + 3 x + a ) Total area of walls of 5 rooms = 5 × 50 = 250 m 2
Now, =
( x - 1)( x + 2) ( x + 2)
According to the question
2 2
( x + 2)( x + 2 x + b) ( x + 2 x + b ) 1 l = 20 m2
and =
( x - 1)( x + 2) ( x - 1)
250
Required paint = = 12.5 l
Since (x + 2) is a factor of (x2 + 3x + a), therefore, x = – 20
2will satisfy this polynomial. Thus,
Hence, 13 cans are required.
C HA P T E R
DECIMAL FRACTIONS
3
1. Consider the following decimal numbers [2007-I] 8. What is the value of 1.34 + 4.12 ? [2010-I]
I. 1.16666666... II. 1.181181118...
III. 2.010010001... IV. 1.454545... 133 371
(a) (b)
Which of the above numbers represent(s) rational number (s)? 90 90
(a) Only IV (b) II and III 219 461
(c) I and IV (d) None of these (c) 5 (d) 5
990 990
p 9. Which one of the following is a non-terminating and
2. If 2.5252525... = (in the lowest form), then what is the value repeating decimal? [2010-II]
q
13 3
q (a) (b)
of ? [2007-II] 8 16
p
3 137
(a) 0.4 (b) 0.42525 (c) (d)
11 25
(c) 0.0396 (d) 0.396
3. Which one of the following is correct ? 10. What is the value of 2.6 - 1.9 ? [2011-I]
3.292929... is [2007-II] (a) 0.6 (b) 0.9
(a) an integer (b) a rational number
(c) 0.7 (d) 0.7
(c) an irrational number (d) not a real number
4. Consider the following statements: [2008-I] 11. What is 27 ´ 1.2 ´ 5.5262 ´ 0.6 equal to? [2011-I]
1 (a) 121.57 (b) 121.75
I. cannot be written as a terminating decimal.
22 (c) 121.75 (d) None of these
2
II. can be written as a terminating decimal. 12. What is 3.76 – 1.4576 equal to ? [2011-II]
15
(a) 2.3100191 (b) 2.3101091
1
III. can be written as a terminating decimal. (c) 2.3110091 (d) 2.3110901
16
Which of the statements given above is/are correct? 13. What is the value of 0.007 + 17.83 + 310.0202 ? [2012-I]
(a) Only I (b) Only II (a) (b) 327.86638
327.86638
(c) Only III (d) II and III
5. If 1 is subtracted from the numerator of a fraction it becomes (c) 327.86683 (d) 327.8668
(1/3) and if 5 is added to the denominator the fraction 14. What is the value of 0.242424... ? [2012-II]
becomes (1/4). Which fraction shall result, if 1 is subtracted (a) 23/99 (b) 8/33
from the numerator and 5 is added to the denominator ? (c) 7/33 (d) 47/198
[2008-I] p
15. Representation of 0.2341 in the form , where p and q are
5 7 q
(a) (b)
12 23 integers, q ¹ 0, is [2013-I]
1 2
781 1171
(a) (b)
(c) (d) 3330 4995
8 3
6. Which one of the following is correct? [2008-I] 2341 2339
(c) (d)
7 2 5 5 2 7 9990 9990
(a) – <- <- (b) – < - < - 16. Let p be a prime number other than 2 or 5. One would like to
10 3 8 8 3 10 express the vulgar fraction l/p in the form of a recurring
5 7 2 7 5 2 decimal. Then the decimal will be [2015-I]
(c) – < - < - (d) – <- <- (a) a pure recurring decimal and its period will be necessarily
8 10 3 10 8 3 (p – 1)
7. Which one is the largest among the following ? (b) a mixed recurring decimal and its period will be
[2009-II] necessarily (p – 1)
(a) 0.725 (b) 0.725 (c) a pure recurring decimal and its period will be some
factor of (p – 1)
(c) 0.725 (d) 0.725 (d) a mixed recurring decimal and its period will be some
factor of (p – 1)
EBD_7367
A- 32 Decimal Fractions

17. The value of (0.63 + 0.37) is [2015-II] 22. Which one among the following is the largest ? [2017-I]
7 11
100 (a) (b)
(a) 1 (b) 9 14
91
3 10
100 1000 (c) (d)
(c) (d) 4 13
99 999
18. If [2016-I] 12
23. What is the value of 3 4 ?
61 1 125
= 3+
19 1 3 2
x+ (a) 1 (b) 1
1 5 5
y+
z
4 2
where x, y and z are natural numbers, then what is z equal to ? (c) 1 (d) 2
(a) 1 (b) 2 5 5
(c) 3 (d) 4 24. If the points P and Q represent the real numbers 0.83 and
19. Which one of the following rational numbers has non–
terminating and repeating decimal expansion ? [2016-II] 0.62 on the number line, then the distance between P and
Q is [2017-II]
15 23
(a) (b) 21 19
1600 8 (a) (b)
90 90
35 17
(c) (d) 21 56
50 6 (c) (d)
100 90
25. Which one of the following is correct? [2018-I]
0.064 ´ 6.25 (a) Decimal expansion of a rational number is terminating.
20. What is equal to ? [2017-I] (b) Decimal expansion of a rational number is non-
0.081 ´ 4.84
terminating.
10 100 (c) Decimal expansion of an irrational number is terminating.
(a) (b)
99 99 (d) Decimal expansion of an irrational number is non-
(c) 9 (d) 99 terminating and non-repeating.
21. What number must be subtracted from both the numerator and 26. What is the difference between 0.9 and 0.9? [2018-I]
27 2 (a) 0 (b) 0.099
the denominator of the fraction so that it becomes ? (c) 0.1 (d) 0.09
35 3
[2017-I] 27. Which one of the following decimal numbers is a rational
(a) 6 (b) 8 number with denominator 37? [2018-I]
(a) 0.459459459 ¼ (b) 0.459459459
(c) 9 (d) 11
(c) 0.0459459459¼ (d) 0.00459459¼

HINTS & SOLUTIONS


1. (c) Since, 1.16666... and 1.454545... are recurring numbers 1
and we know that recurring numbers represent rational = 0.0625
16
numbers.
So, that I and IV are rational numbers. 1
can be written as terminating decimal.
p p 252 - 2 p 250 16
2. (d) = 2.52 Þ = Þ = 5. (c) Let the numerator and denominator of a fraction are x
q q 99 q 99 and y, respectively,
q 99 According to question,
\ = = 0.396 x -1 1
p 250 = Þ 3 x - 3 = y Þ 3 x - y = 3...(i )
y 3
3. (b) 3.292929... = 3.29 is a non-terminating repeating
decimal. Then, it is a rational number. x 1
and = Þ 4x - y = 5 ...(ii )
y+5 4
1
4. (c) = 0.04545... On solving eqs. (i) and (ii), we get
22
x = 2 and y = 3
2
= 0.1333... x -1 2 -1 1
15 \ Required fraction = = =
y +5 3+5 8
Decimal Fractions A-33
6. (a) By option (a),
76 4572 æ 76 4572 ö
-7 -2 -5 = 3+ -1 - = 2+ç - ÷
< < 99 9990 è 99 9990 ø
10 3 8
Here LCM of (3, 8, 10) = 120 1 æ 76 4572 ö 1 ( 84360 - 50292)
= 2+ ç - ÷ = 2+ ´
9 è 11 1110 ø 9 12210
-7 -2 -5
´ 120 < ´ 120 < ´ 120
10 3 8 1 34068 11356
= 2+ ´ = 2+
– 84 < – 80 < – 75 9 12210 36630
So this is correct. = 2 + 0.3100191 = 2.3100191
7. (d) (a) 0.725 13. (b) 0.007 + 17.83 + 310.0202
(b) 0.725 = 0.7255... 7 1783 - 17 3100202 - 310020
= + +
900 99 9000
(c) 0.725 = 0.7252525...
7 1766 2790182
= + +
(d) 0.725 = 0.725725725... 900 99 9000
Largest number is 0.725 . 770 + 1766000 + 30692002
=
99000
134 - 1 133
8. (d) Q 1.34 = = 32458772
99 99 = = 327.86638
99000
412 - 41 371 14. (b) Given that,
and 4.12 = =
90 90 24 8
0.242424... ? = 0.24 = =
99 33
133 371 1330 + 4081
\ 1.34 + 4.12 = + = Write down as many 9’s in the denominator as the
99 90 990 number of digits in the period of decimal number.
5411 461 15. (d) Let x = 0.2341
= =5
990 990 Here multiply by 10 both sides,

13 13 125 1625 10x = 2.341 ....... (i)


9. (c) Q = × = = 1.625 Now, multiply by 1000 both sides,
8 8 125 1000
10000 x = 2341.341 ...... (ii)
3 3 ´ 625 1875
= = = 0.1875 Now, substract equation (i) from equation (ii),
16 16 ´ 625 10000
9990 x = 2341 - 2 = 2339
137 137 ´ 4 548
2339
= = = 5.48 \ x= .
25 25 ´ 4 100
9990
It is clear that all of these are terminating and decimals.
• Shortcut:
3
Hence, is a non-terminating and repeating decimal. 2341 - 2 2339
11 0.2341 = =
9990 9990
6 6 6 16. (a) Pure recurring decimal:-
10. (a) 2.6 - 1.9 = 2 - 2 = (2 - 2) + = = 0.6
9 9 9 A decimal fraction in which all the figures occur
repeatedly is called a pure recurring decimal as 7.4444
2 4736 6
11. (d) 27 ´ 1.2 ´ 5.5262 ´ 0.6 = 27 ´ 1 ´ 5 ´ .., 2.666 ...., etc.
9 9000 9 Let P be prime number
11 49736 6 So P = 7, 11, 13,...
= 27 ´ ´ ´
9 9000 9 1
= .142857142857 ....
11 ´ 49736 ´ 2 1094192 7
= = = 121.577
9000 9000 1
= .09090909 ....
11
12. (a) 3.76 - 1.4576 = 3 + 0.76 - 1 - 0.4576
1
76 - 0 ö = 0.0769230769230...
= 3 + æç æ 4576 - 4 ö 13
99 ÷ - 1 - ç 9990 ÷
è ø è ø All above example are pure recurring decimal and its period
will (p –1)
EBD_7367
34
A- Decimal Fractions

17. (c) 0.63 = 0.636363 ------ 1 1


= 3+ = 3+
Let x = 0.636363 ------ 3 19
Þ 100 x = 63. 6363 ------ 4+
4 4
Þ 99x = 63
4
63 = 3+
Þ x= 19
99
\ Option (a) & (c) is correct.
0.37 = 0.373737 ------
Let y = 0.37 37 37 ------ 17
19. (d) only fraction whose dinominator is not in form of
Þ 100y = 37. 37 37 ------ 6
Þ 99y = 37 m n
2 5 when simplified
37 0.064 6.25 8 25
Þ y= 20. (b) ´ = ´
99 0.081 4.84 9 22
63 37 100
x+y= + = 100
99 99 99 =
So, option (c) is correct. 99
21. (d) ATQ
18. (*) 61 1
= 3+ 27 - x 2
19 1 =
x+ 35 - x 3
1
y+ 81 – 3x = 70 – 2x
z
x = 11
61 4 11
In real = 3+ 22. (b)
19 19 14
61 1 12 3 512 8 3
Þ = 3+ 23. (a) 3 4 = = =1
19 1 125 125 5 5
x+
1
3+ 24. (b) We have P = 0.83 and Q = 0.62
1
The distance between P and Q is
1
= 3+ 0.83 - 0.62 = 0.2 1
1
x+ Expressing this distance in the form of rational num-
4
ber, we assume 0.2 1 = x
= 3+ 1 No. of digits with bar = 1
9 1 No. of digits without bar = 1
+
2 4
Therefore, the denominator would be 90.
= 3+ 1 No. of digits after the decimal = 2
19 Therefore, the numerator would be 21 – 2 = 19
4 19
Thus, x =
4 90
= 3+ 25. (a)
19
9 9
Þ x= , y = 3, z = 1 26. (c) Value of 0.9 = =1
2 9
Also if z = 3, then 9
Value of 0.9 =
61 1 1 10
= 3+ = 3+
19 1 3 9 1
x+ x+ difference = 1 - = = 0.1
1 4 10 10
1+
3 27. (a)
Powers and Roots A- 35

C HA P T E R
POWERS AND ROOTS
4
1. Which one of the following numbers is an integer? (a) 0 (b) 1
[2007-I] (c) 2 (d) –1

(a) é
ë ( 2+ 3 / )( )
3- 2 ù+ 6
û 8. Assertion (A):
5041
is rational [2007-II]
6889
(b) é
ë ( 2+ 3 / )( )
3- 2 ù+2 6
û Reason (R): The square root of a rational number is
always rational.
(c) é
ë ( 2+ 3 / )( )
2- 3 ù+2 6
û
(a) A and R are correct and R is correct explanation of A
(b) A and R are correct but R is not correct explanation of A
(d) é
ë ( 2+ 3 / )( )
2- 3 ù+ 6
û
(c) A is correct but R is wrong
(d) A is wrong but R is correct

2.
æ x5 - 1 ö
What is the square root of çç
3 2
(
÷ + x + 2x + x ? ) 9. What is the square of 2 + 2 ? ( ) [2008-I]
x - 1 ÷
è ø (a) A rational number (b) An irrational number
[2007-I] (c) A natural number (d) A whole number
x2 + x + 1 x2 - x + 1
( ) ( )
(a) (b) 2 x -1 x-2
10. If ab-1 = ba -1 , then what is the value of x ?
(c) x2 - x - 1 (d) x2 + x - 1
[2008-I]
3. What is the square root of 9 + 2 14 ? (a) 1 (b) 2
[2007-II] (c) 3 (d) 4
(a) 1+ 2 2 (b) 3+ 6 11. What is the value of [2008-I]
(c) 2+ 7 (d) 2+ 5 7.84 + 0.0784 + 0.000784 + 0.00000784 ?
(a) 3.08 (b) 3.108
4. Which is the largest number among 2, 3 3, 6 6 and
(c) 3.1008 (d) 3.1108
12 12 ? [2007-II] 93 x
12. If 1+ = 1 + , then what does x equal to?
(a) 2 (b) 3
3 196 14
(c) 6 6 (d) 12 12 [2008-II]
5. The sum of the square of a number and the square of the (a) 1 (b) 2
reciprocal of the number, is thrice the difference of the (c) 3 (d) 4
square of the number and the square of the reciprocal of
(ax )
...¥
the number. What is the number? [2007-II]
1/ 4
13. If y = a x ( ) , then which one of the following is
(a) 1 (b) ( 2) correct? [2008-II]
(c) ( 3)1/ 3 (d) ( 4 )1/ 4 (a) log y = x y log a (b) log y = x + y log a
6. What is/are the real value(s) of (256)0.16 × (16)0.18 ? (c) log y = y + x log a (d) log y = ( y + x ) log a
[2007-II]
14. What is the value of
(a) Only – 4 (b) Only 4
(c) 4, – 4 (d) 2, – 2 1 1 1
+ + ... + ?
1 1 1 1+ 2 2+ 3 15 + 16
7. If x m = y n = z p and xyz = 1, then what is the value [2008-II]
( ) ( ) ( )
(a) 0 (b) 1
of m + n + p ? [2007-II] (c) 2 (d) 3
EBD_7367
36
A- Powers and Roots

15. A ball is dropped from a height 64 m above the ground 1 1 1 1


and every time it hits the ground it rises to a height equal 25. - + -
9- 8 8- 7 7- 6 6- 5
to half of the previous. What is the height attained after
it hits the ground for the 16th time? [2009-I] 1
+ equal to [2010-II]
(a) 2–12 m (b) 2–11 m 5- 4
(c) 2–10 m (d) 2–9 m (a) 0 (b) 1
16. If a x = c q = b and c y = a z = d , then which one of the 1
(c) 5 (d)
following is correct ? [2009-I] 3
(a) x / y = q / z (b) x + y = q + z 26. If 3 x ´ 27 x = 9 x +4 , then x equal to [2011-I]
xy = qz y z (a) 4 (b) 5
(c) (d) x =q
(c) 6 (d) 7
17. If 27 ´ (81)2n+3 - 3m = 0 , then what is m equal to? 27. What is one of the square roots of 9 – 2 14 ?
[2009-II] [2011-I]
(a) 2n + 5 (b) 5n + 6 (a) 7- 3 (b) 6- 3
(c) 8n + 3 (d) 8n + 15
(c) 7- 5 (d) 7- 2
1/3 1/3
æ 2 3ö
+ æç a - a 2 + b3 ö÷
18. If x = ç a + a + b ÷ , then what 0.324 ´ 0.64 ´129.6
è ø è ø 28. What is the square root of ?
0.729 ´1.024 ´ 36
is the value of x3 + 3bx - 2 a ? [2009-II] [2011-I]
(a) 4 (b) 3
(a) 2a 3 (b) -2a 3
(c) 2 (d) 1
(c) 1 (d) 0 29. What is the smallest number that must be added to 1780
x y to make it a perfect square? [2011-II]
19. If ( 3.7 ) = ( 0.037 ) = 10000 , then what is the value of
(a) 39 (b) 49
1 1 (c) 59 (d) 69
- ? [2009-II]
x y 30. If 2m + 21+ m = 24 , then what is the value of m ?[2011-II]
(a) 1 (b) 2
1
(c) 1/2 (d) 1/4 (a) 0 (b)
20. Out of a group of swans 7/2 times the square root of the 3
number are swimming in the pool while the two remaining (c) 3 (d) 6
are playing outside the pool. What is the total number of
0.0032
swans? [2009-II] 31. What is the value of ? [2011-II]
(a) 4 (b) 8 0.32
(c) 12 (d) 16 (a) 0.0001 (b) 0.001
(c) 0.01 (d) 0.1
21. The number 0.0001 is [2009-II]
(a) a rational number less than 0.01 5+ 3 5- 3
(b) a rational number 32. What is + equal to ? [2011-II]
5- 3 5+ 3
(c) an irrational number
(a) 16 (b) 8
(d) neither a rational number nor an irrational number
(c) 4 (d) 15
22. If a x = b y = c z and abc = 1 , then xy + yz + zx equal to
[2009-II] 33. If 3 x + y = 81 and 81x - y = 3, then what is the value of x ?
(a) xyz (b) x + y + z [2012-I]
(c) 0 (d) 1
17 17
23. What is the value of [2010-I] (a) (b)
16 8
29.16 + 0.2916 + 0.002916 + 0.00002916 ?
(a) 5.9949 (b) 5.9894 17 15
(c) (d)
(c) 5.9984 (d) 5.9994 4 4
34. What is one of the square roots of [2012-I]
24. If p x = r y = m and r w = p z = n , then which one of the
following is correct ? [2010-II] 16 x 6 - 24 x 5 + 25 x 4 - 20 x3 + 10 x 2 - 4 x + 1 ?
(a) xw = yz (b) xz = yw (a) 4 x3 - 3 x 2 + 2 x + 1 (b) 4 x3 - 3 x 2 - 2 x - 1
(c) x + y = w + z (d) x – y = w – z
(c) 4 x3 - 3 x 2 + 2 x - 1 (d) 4 x3 - 3 x 2 - 2 x + 1
Powers and Roots 37
A-

Which of the above statements is/are correct?


35. If a x = b, b y = c and xyz = 1, then what is the value of c z ?
(a) Only I (b) Only II
[2012-I] (c) Both I and II (d) Neither I nor II
(a) a (b) b 45. The product of four consecutive natural numbers plus one
a is [2014-I]
(c) ab (d) (a) a non-square
b
(b) always sum of two square numbers
36. If 196 x 4 = x 6 , then x3 is equal to which one of the (c) a square
following ? [2012-I] (d) None of these
14x4 46. The difference of cubes of two consecutive integers
(a) x 6 /14 (b)
[2014-I]
(c) x 2 /14 (d) 14x2 (a) is odd or even
(b) is never divisible by 2
2
37. If a = 2 + 3 , then what is the value of a + a
-2
? ( ) (c) is always even
[2012-II] (d) None of these
(a) 12 (b) 14 1 1 2b 4b 3 8b 7
(c) 16 (d) 18 47. What is - - - - equal
a - b a + b a 2 + b 2 a 4 + b 4 a 8 - b8
38. If 10 + 3 x = 4, then what is the value of x ? to ? [2014-II]
[2012-II] (a) a + b (b) a – b
(a) 150 (b) 216 (c) 1 (d) 0
(c) 316 (d) 450
39. The least number of four digits which is a perfect square is (0.75)3
48. The square root of + [0.75 + (0.75)2 + 1] is
[2012-II] 1 - 0.75
(a) 1204 (b) 1024 [2015-I]
(c) 1402 (d) 1420 (a) 1 (b) 2
(c) 3 (d) 4
40. What is the value of 6 + 6 + 6 + 6 + ... ? 49. What are the possible solutions for x of the equation
[2013-I] x x
=
n
x x , where x and n are positive integers?
(a) 2 (b) 3
[2015-I]
(c) 3.5 (d) 4
(a) 0, n (b) 1, n
41. If 16 ´ 8n + 2 = 2m , then m is equal to [2013-I] (c) n, n2 (d) 1, n2
2 2
(a) n + 8 (b) 2n + 10 50. If a – b = 4 and a + b = 40, where a and b are positive
(c) 3n + 2 (d) 3n + 10 integers, then a3 + b6 is equal to [2015-I]
(a) 264 (b) 280
é 2ù
2 (c) 300 (d) 324
42. The expression ê 2
ë
ú gives
û
( ) [2013-I]
51. What is 4 + 4 - 4 + 4 - ............... equal to?
(a) a natural number [2015-II]
(b) an integer and not a natural number
(c) a rational number but not an integer 13 - 1
(d) a real number but not a rational number (a) 3 (b)
2
43. Which is the smallest number among the following?
[2013-II] 13 + 1
(c) (d) 0
-2 -2 2
(a)
é
ë
( )
ê 5
-2 -2 ù
ú
û
(b)
é
ê 5
ë
( ) úû
-2 2 ù
3 1 1
52. If x = 3 + 2 , then the value of x + x + x + 3 is
x
-2 -2
é -5 -2 ù
(c) ê 2
ë
( )
ú
û
(d) ê 2
é -5 2 ù
ë
ú
û
( ) (a) (b)
[2015-II]
10 3 20 3
44. Consider the following in respect of the numbers (c) 10 2 (d) 20 2
3
2, 3 and 6
6 [2014-I] 53. Which one of the following is correct? [2015-II]
I. 6
6 is the greatest number.. (a) 2 < 46 <34 (b) 2 > 46 >34
II. 2 is the smallest number.. (c) 4
6< 2<34 (d) 4
6> 2>34
EBD_7367
38
A- Powers and Roots

1 (a) 1 only (b) 2 only


-1
54. If x = 2 3 + 2 3 , then the value of 2x3 – 6x – 5 is euqal to (c) Both 1 and 2 (d) Neither 1 nor 2
[2016-I] 63. What is the remainder when
(a) 0 (b) 1 135 + 145 + 155 + 165 is divided by 29 ? [2016-II]
(c) 2 (d) 3 (a) 8 (b) 5
55. If 4x2y = 128 and 33x32y – 9xy = 0, then the value of x + y can (c) 3 (d) 0
be equal to [2016-I] 64. What is the difference between the sum of the cubes and
(a) 7 (b) 5 that of squares of first ten natural numbers? [2016-II]
(c) 3 (d) 1 (a) 2280 (b) 2640
56. The value of the expression is equal to [2016-I] (c) 3820 (d) 4130
65. What is the square root of [2017-I]
(243 + 647)2 + (243 – 647) 2
243 ´ 243 + 647 ´ 647 (0.35) 2 + 0.70 + 1
+ 0.19 ?
(a) 0 (b) 1 2.25
(c) 2 (d) 3 (a) 1 (b) 2
(c) 3 (d) 4
6 2 + 7 2 + 82 + 9 2 + 102
57. What is equal to? [2016-II] 66. What is the value of [2017-I]
7+4 3 - 4+2 3
(443 + 547)2 + (443 - 547)2
(a) 330 (b) 340 ?
(c) 355 (d) 366 443 ´ 443 + 547 ´ 547
58. The sum of the squares of two positive integers is 208. If (a) 0 (b) 1
the square of the larger number is 18 times the smaller number, (c) 2 (d) 3
then what is the difference of the larger and smaller numbers ? 67. If a3 = 335 + b3 and a = 5 + b, then what is the value of a + b
[2016-II] (given that a > 0 and b > 0) ? [2017-I]
(a) 2 (b) 3 (a) 7 (b) 9
(c) 4 (d) 6 (c) 16 (d) 49
59. What is the remainder when 2100 is divided by 101 ? 68. If 9x 3y = 2187 and 23x 22y – 4xy = 0, then what can be the
[2016-II] value of (x + y) ? [2017-I]
(a) 1 (b) 11 (a) 1 (b) 3
(c) 99 (d) 100 (c) 5 (d) 7
60. Which one of the following is correct in respect of the 69. The values of x which satisfy the equation 51+x + 51–x = 26
are [2017-I]
number 1729 ? [2016-II]
(a) 1, 1 (b) 0, 1
(a) It cannot be written as the sum of the cubes of two
(c) 1, 2 (d) – 1, 0
positive integers
70. If each of the dimensions of a rectangle is increased by
(b) It can be written as the sum of the cubes of two positive
200%, the area is increased by
integers in one way only
(a) 300% (b) 400%
(c) It can be written as the sum of the cubes of two positive
(c) 600% (d) 800%
integers in two ways only
71. Let f(x) and g(x) be two polynomials (with real coefficients)
(d) It can be written as the sum of the cubes of two positive
having degrees 3 and 4 respectively. What is the degree of
integers in three ways only f(x) g(x)? [2017-II]
1 1 1 1 (a) 12 (b) 7
61. What is 1 + 2 + 2 + 1 + 2 + 2 + ... + [2016-II] (c) 4 (d) 3
1 2 2 3 72. Consider the following numbers : [2017-II]
1. 2222
1 1
1+ + equal to ? 2. 11664
2
2007 20082 3. 343343
4. 220347
1 1 Which of the above are not perfect squares? [2017-II]
(a) 2008 – (b) 2007 –
2008 2007 (a) 1, 2 and 3 (b) 1, 2 and 4
(c) 2, 3 and 4 (d) 1, 3 and 4
1 1
(c) 2007 – (d) 2008 –
2008 2009 73. The value of 1 + 1 + 1 + .... [2017-II]
62. Consider the following statements in respect of positive (a) Equals to 1
odd integers x and y : [2016-II] (b) Lies between 0 and 1
1. x2 + y2 is even integer. (c) Lies between 1 and 2
2. x2 + y2 is divisible by 4 (d) Is greater than 2
Which of the above statements is/are correct?
Powers and Roots 39
A-

74. If x = y1/a, y = z1/b and z = x1/c where x ¹ 1, y ¹ 1, z ¹ 1, then 78. What is the value of [2018-I]
what is the value of abc? [2018-I]
1 1 1
(a) –1 (b) 1
b-a c -a
+ a-b c -b
+ a -c
(c) 0 (d) 3 1+ x +x 1+ x +x 1+ x + x b-c
75. If 2b = a + c and y2 = xz, then what is xb–c yc–a za–b equal to? where x ¹ 0?
[2018-I]
(a) 3 (b) 2 (a) –1 (b) 0
(c) 1 (d) –1 (c) 1 (d) 3
76. The smallest integer with 4 digits which is a perfect square 79. The sum of a number and its square is 20. Then the number
is [2018-I] is [2018-I]
(a) 1000 (b) 1024
(c) 1089 (d) None of the above (a) –5 or 4 (b) 2 or 3
(c) –5 only (d) 5 or –4
5- 3 5+ 3
77. What is the value of = ? [2018-I] 80. What is the largest power of 10 that divides the product
5+ 3 5- 3
1 × 2 × 3 × 4 ................ × 23 × 24 × 25? [2018-I]
(a) -2 15 (b) 2 15 (a) 2 (b) 4
(c) 15 (d) - 15 (c) 5 (d) None of the above

HINTS & SOLUTIONS

( )
2
æ x5 - 1 ö
1. (c) (a)
2+ 3
+ 6=
2+ 3
+ 6 2. (a) ç
ç x -1 ÷
è ø
(
÷ + x3 + 2 x 2 + x . )
3- 2 3- 2
= 2+ 3+ 2 6 + 6 x5 - 1 3
+ x + 2x2 + x
= 5+3 6 x -1
It is not an integer.
x5 - 1 + x 4 + 2 x3 + x 2 - x3 - 2 x 2 - x
( )
2 =
2+ 3 2+ 3 x -1
(b) +2 6 = +2 6
3- 2 3- 2
x5 + x 4 + x3 - x 2 - x - 1
=
=2+3+2 6 +2 6 x -1
= 5+4 6
It is also not an integer.
=
( ) (
x3 x 2 + x + 1 - 1 x 2 + x + 1 )
( x - 1)
( )
2
2+ 3 2+ 3
(c) +2 6 = +2 6
2- 3 2-3 ( x 2 + x + 1)( x 3 - 1)
(
= – 2+3+ 2 6 + 2 6 ) =
x -1

( x 2 + x + 1) ( x - 1) ( x 2 + x + 1)
= –5
It is an integer.
=
( 2+ 3 )
2
( x - 1)
2+ 3
+ 6= + 6
(d)
( 2) - ( 3) ( )
2- 3 2 2 2
= x2 + x + 1

2+3+2 2 3 ìï x5 - 1 3 üï
= + 6 2
2-3 \ Square root of í x - 1 + x + 2 x + x ý
(
= - 5+ 2 6 + 6 ) îï þï

( x2 + x + 1)
2
= -5 - 6 = = x2 + x + 1
It is not an integer
EBD_7367
A- 40 Powers and Roots

( 7) + ( 2) ( )
2 x -1
( )
x-2
2 2
3. (c) 9 + 2 14 = +2 7´ 2 -1 -1
10. (a) Given that, ab = ba

( )
2
= 7+ 2 2 x -1 x-2
æaö æbö
Þ ç ÷ =ç ÷
èbø èaø
\ 9 + 2 14 = ( 7+ 2 ) 2 x -1 x- 2
æaö æaö
4. (b) 3
2, 3, 6 and 12 126 Þ ç ÷ ç ÷ =1
èbø èbø
LCM of 2, 3, 6 and 12 is 12
2 x -1+ x - 2 0
It can be written as æaö æaö
Þ ç ÷ =ç ÷
12 6
2 ,
12 4
3 ,
12 2
6 and 12 12 . èbø èbø
Base is same
So 3
3 is largest number..
So, 3x - 3 = 0 Þ x = 1
1
5. (b) Let number be x, then its reciprocal be . 11. (d) 7.84 + 0.0784 + 0.000784 + 0.00000784
x
According to question, 784 784 784 784
= + + +
1 æ 1 ö 100 10000 1000000 100000000
x2 + = 3 ç x2 - 2 ÷
x2 è x ø 28 28 28 28
= + + +
1 3 4 10 100 1000 10000
\ x2 + = 3x2 - Þ 2 x2 = = 2.8 + 0.28 + 0.028 + 0.0028 = 3.1108
2 2
x x x2
1/ 4 93 x
Þ x4 = 2 Þ x = ( 2 ) 12. (c) Given that, 1+ = 1+
196 14
0.16
(b) (256)0.16 × (16)0.18 = éê(16 ) ùú
2 0.18
6. ´ (16 ) 289 x 17 x
ë û Þ = 1+ Þ = 1+
196 14 14 14
= (16 )0.32 ´ (16 )0.18
x 17
Þ = -1 Þ x = 3
2 ù 0.5 14 14
= (16 )
0.5
= é4 = 41 = 4
ë û
(ax )
...¥

1
m
1
n
1
p
13. (a) Given, y = a ( ) x

7. (a) Let, ( x ) = ( y) = ( z) =k (say)


( )
y
m n and \ y = ax
Þ x=k ,y=k z =kp
Now multiply x, y and z Taking log on both sides

\ xyz = k m+ n + p (Q xyz = 1 , given) log y = y log a x Þ log y = x y log a

Þ 1 = k m+ n+ p = k 0 14. (d)
1
+
1
+ ... +
1
Here, base is same, 1+ 2 2+ 3 15 + 16
m+n+p=0 (on rationalisation)
5041 71 1- 2 2- 3 15 - 16
8. (c) (A) = = rational number = + + ... +
6889 83 1- 2 2-3 15 - 16
(R) Now, 2 is a rational number but
rational number. i.e., irrational number.
2 is not a
(
= -1 1 - 2 + 2 - 3 + ... + 15 - 16 )
= -1(1 - 4 ) = 3
( )
2
9. (b) Square of 2 + 2 = 2 + 2
1
15. (c) After 1st hit ball height will be = (64)
( 2)
2 2
= ( 2) + + 2.2. 2 = 4 + 2 + 4 2 2
2
= 6+4 2 æ1ö
After 2nd hit ball height will be = ç ÷ ( 64 )
6 + 4 2 is an irrational number.. è2ø
Powers and Roots A- 41
...................................................................... On squaring both sides, we get
...................................................................... 49
........................................................................ x = x2 + 4 - 4 x
4
16
æ1ö
After 16th hit ball height will be = ç ÷ ( 64 ) Þ 4 x 2 - 65 x + 16 = 0
è2ø
= 4 x 2 - 64 x - x + 16 = 0

216
=
1
( 26 ) = 2-10 m Þ 4 x ( x - 16 ) - 1( x - 16 ) = 0

16. x q y z
(c) Given, a = c = b and c = a = d Þ ( x - 16 )( 4 x - 1) = 0
Now, ax = cq æ 1ö
\ x = 16 çQ x ¹ ÷
( ) = (c z )
x q è 4ø
Þ a x× z = c q × z Þ a z
00001 1
21. (b) 0.0001 = 10000 = 100 = 0.01
( c y ) = c zq
x
Here, a z = c y Þ
So, it is a rational number.
Þ yx = zq
22. (c) Given, a x = b y = c z = k
2n+3
17. (d) Given, 27 ´ (81) m
-3 = 0 1/ z
Þ a = k1/ x , b = k1/ y and c = k
8n +12
Þ 33 ´ ( 3) = 3m 1 1 1
+ +
x y z
Þ 38 n +15 = 3m Þ m = 8n + 15 (on comparing) \ abc = k
1 1 1
1/ 3 1/ 3 + +
æ 2 3ö
+ æç a - a 2 + b3 ö÷ Þ 1= k x y z
(Q abc = 1, given)
18. (d) Given, x = ç a + a + b ÷ =k
è ø è ø
On cubing both sides, we get On comparing, we get
x3 = æç a + a 2 + b3 ö÷ + æç a - a 2 + b3 ö÷ 1 1 1
+ + =0
è ø è ø x y z
1/3 1/3 Þ xy + yz + zx = 0
+3 æç a + a2 + b3 ö÷ æ a - a 2 + b3 ö
ç ÷
è ø è ø 23. (d) 29.16 + 0.2916 + 0.002916 + 0.00002916
= 5.4 + 0.54 + 0.054 + 0.0054
ïìæ 3ö
1/ 3 1/ 3 ü
ï
2
íç a + a + b ÷ + æç a - a 2 + b3 ö÷ ý = 5.9994
ïîè ø è ø ïþ 24. (a) Given that, p x = r y = m and r w = p z = n
Þ x = 2a - 3b ( x )
3
Now, p x = r y
Þ x3 + 3bx - 2 a = 0
( px ) = (r y )
w w
Þ Þ p xw = r yw
x y
19. (c) Given, ( 3.7 ) = ( 0.037 ) = 10000
p xw = ( r w )
y
Þ ( 3.7 ) x = 104 and ( 0.037 ) y = 104 Þ ...(i)

4 z
4
+1 +3 Here, r w = p put in eq. (i)
and 37 = 10 y
Þ 37 = 10 x
( )
y
4 4 then, p xw = p z Þ p xw = p zy (Base is same)
+1 +3
y 4 4
Þ 10 x = 10 Þ +1 = + 3 \ xw = zy
x y
1 1 1
4 4 1 1 1 25. (c) - +
\ - = 3 -1 Þ - = 9- 8 8- 7 7- 6
x y x y 2
1 1
20. (d) Let the total number of swans = x - +
6- 5 5- 4
7
Number of swans swimming in the root = x (on rationalisation)
2
Remaining swans = 2 = ( 9+ 8 - ) ( 8+ 7 + ) ( 7+ 6 - ) ( 6+ 5 )
According to question,
7 7
+ ( 5+ 4 )
x +2= x Þ x = x-2
2 2 = 9 + 4 =3+2 =5
EBD_7367
42
A- Powers and Roots

34. (c) From option (c),


26. (a) 3 x ´ 27 x = 9 x + 4
\ 3x × 33x = 32(x + 4)
( 4x3 - 3x2 + 2x -1) ( )
2 2
Þ 3x + 3x = 32(x + 4) = éê 4 x3 - 3 x 2 + ( 2 x - 1) ùú
ë û
Here base is same
( ) ( ) ( 2x - 1)
2
So, x + 3x = 2 ( x + 4 ) Þ 4x - 2x = 8 = 4 x3 - 3 x 2 + ( 2 x - 1) + 2 4 x 3 - 3x 2
2

8
\ x= =4 6 4 5 2
= 16 x + 9 x - 24 x + 4 x + 1 - 4 x
2
27. (d) = +16 x 4 - 12 x3 - 8 x3 + 6 x 2
9 - 2 14 7 + 2 - 2´ 7 ´ 2
6 5 4 3 2
= 16 x - 24 x + 25 x - 20 x + 10 x - 4 x + 1
( )
2
= 7- 2 = 7- 2 35. (a) equ a x = b
Multiplying both sides by y in power
0.324 ´ 0.64 ´129.6
( ax )
28. (d) y
0.729 ´ 1.024 ´ 36 = by
324 ´ 64 ´1296
(Q b y = c )
= y
729 ´ 1024 ´ 36 Þ ax = c
18 ´ 8 ´ 36 Again multiplying both sides by z in power
= =1
27 ´ 32 ´ 6
( a xy )
z
= cz
2 2
29. (d) We know that, ( 42 ) = 1764 and ( 43) = 1849
But 1780 lies between 1764 and 1849. Þ a xyz = c z
Now, smallest number = 1849 – 1780 = 69 But xyz = 1 Given
30. (c) Given, 2m + 21+ m = 24 So a = cz
36. (d) Given, 196x 4 = x 6
\ 2 m (1 + 2 ) = 24 Þ 2 m ´ 3 = 24

( ) = (x )
2 3 2
Þ 2m = 8 = 23 Þ 14 x2 Þ 14 x 2 = x3
\ m=3
1
0.0032 0.32 1 1 37. (b) Given that, a = 2 + 3, = 2- 3
31. (d) = ´ = = 0.1 a
0.32 100 0.32 10
2
2 -2 æ 1ö
5+ 3 5- 3 Now, a + a = ç a + ÷ - 2
32. (b) + è aø
5- 3 5+ 3
( )
2
= 2+ 3 + 2- 3 -2
( ) + ( 5 - 3)
2 2
5+ 3
=
= ( 4 )2 - 2 = 16 - 2 = 14
( 5) - ( 3)
2 2

38. (b) Given, 10 + 3 x = 4


ì
( 5) + ( 3)
2 2ü
2í ý 2 5+3
þ= ( ) On squaring both sides
= î =8
5-3 2 10 + 3 x = 16
33. (b) Given that 3 x + y = 81 or 3 x + y = 34 Þ 3 x =6
Þ x+ y = 4 (i) On cubic both sides
and 81x–y = 3 or (34)x–y = 31 x = ( 6 ) = 216
3

1 39. (b) Factor of 1024 = 32 × 32 = 32


Þ x- y = (ii)
4 So, 1024 is a perfect square number.
On solving eqs. (i) and (ii), we get 40. (b) Factorisation of 6 = 2 × 3
1
x+ y = 4 Þ x- y = Given that 6 + 6 + 6 + 6 + ...
4
17 17 It has positive sign
2x = Þ x= Hence, its value will be greater number of the
4 8
factors = 3
Powers and Roots A- 43

41. (d) Given that, 16 ´ 8n + 2 = 2m (odd integer)3 – (even integer)3 = (odd integer)
Þ (2)4 × 23(n + 2) = 2m – (even integer)
Þ (2)(4 + 3n + 6) = 2m = odd integer

Þ 2(
3n +10 ) 4b 3 8b 7
= 2m 1
-
1
-
2b
- -
47. (d)
Here base is same, so a - b a + b a 2 + b 2 a 4 + b 4 a 8 - b8
3n + 10 = m
(a + b) - (a - b) 2b 4b3 8b7
Þ m = 3n + 10 = - - -
(a - b) (a + b) a 2 + b2 a 4 + b 4 a8 - b8
2
é 2ù
42. (d) ê
ë
( ) 2 ú
û
= ( 2)
2
=2
=
2b
-
2b
-
4b3
-
8b7
a2 - b2 a2 + b2 a 4 + b4 a8 - b8
-2 ù -2
43. (c)
é -2
From option (a) ê 5
ë
( ) ú
û
= 5-8 =
5
1
8 =
2b(a 2 + b2 ) - 2b(a 2 - b 2 )
-
4b3
-
8b7
(a 2 - b2 ) (a 2 + b2 ) a 4 + b4 a8 - b8
2 -2
é -2
From option (b) ê 5
ë
( ) ùúû = 58 = 58
=
2a 2 b + 2b3 - 2a 2b + 2b3
-
4b3
-
8b7
(a 4 - b 4 ) a 4 + b4 a8 - b8
-2
é
From option (c) ê 2
ë
( ) -5 -2 ù
ú
û
= 5-20 =
520
1
=
4b3 (a 4 + b4 ) - 4b3 (a 4 - b4 )
-
8b7
(a4 - b4 )(a 4 + b4 ) a8 - b8
-2
é
From option (d) ê 2
ë
( ) úû
-5 2 ù
= 520 = 520 4a 4 b3 + 4b7 - 4a 4b3 + 4b7
-
8b7
=
a8 - b8 a8 - b8
-2
é
Now, smallest number ê 2
ë
( )
-5 -2 ù
ú
û =
8b7
-
8b7
=0
a8 - b8 a8 - b8
44. (d) LCM of 2, 3 and 6 = 12
1 12
´ (.75)3
Now,
2 = 2 2 12 = 12 26 = 12 64 48. (d) + [.75 + (.75)2 + 1]
1 – 0.75
1 12
´
3
3 = 3 3 12 = 12 34 = 12 81 (.75)3 + (1 – .75) (.75 + (.75) 2 + 1)
=
(1– .75)
1 12
´ 12 2
6
6 = 6 6 12 = 6 = 12 36
(.75)3 + (1)3 – (.75) 3
So, =
2 is not smallest and 6 6 is not greatest. So .25
neither I nor II correct.
1
45. (c) Example 1: Let consider four consecutive natural = ´1 = 4
numbers = 4, 5, 6 and 7 product = 4 × 5 × 6 × 7 .25
= 840 x n
Now, 840 + 1 = 841 49. (d) x = xx
It is square of 29. 1 x
Example 2: Let other four consecutive natural
number = 13, 14, 15 and 16
Þ xx ( ) 2 = xn

Product = 13 × 14 × 15 × 16 = 43680 Take log on both the sides,


Now, 43680 + 1 = 43681
It is square of 43681 Þ log( x x ) = log( x) x / n
So this is always a square. x
46. (b) One of the two consecutive integers is even and Þ x log x = .log x
n
other is odd.
(even integer)3 – (odd integer)3 = (even integer) x
– (odd integer) Þ x log x – log x = 0
n
= odd integer
EBD_7367
44
A- Powers and Roots

x 1 3 1
Þ log x ( x – ) = 0 = x+ +x + 3
n x x
3
x æ 1ö æ 1ö æ 1ö
Þ ) =0
(log x) ( x ) (1 – = ç x + ÷ + ç x + ÷ - 3ç x + ÷
n è xø è xø è xø
Þ log x = 0 Þ x = 1, 3
= ( 3+ 2 + 3- 2 + ) ( 3+ 2 + 3- 2 )
x = 0 Þ x = 0 or x = n Þ x = n 2
Thus, x = 0, 1, n2 -3 ( 3+ 2+ 3 - 2)
x = 0 is not admissible
3
Since, logx is not defined = 2 3 + ( 2 3 ) - 3( 2 3 )
\ x = 1, n2
50. (b) Given = 2 3 + 24 3 - 6 3
a–b=4 ... (i) = 20 3
2 2
a + b = 40 So, option (b) is correct
Now, we know that
1
(a – b)2 = a2 + b2 –2ab 1 1
( )
6 12
Þ 16 = 40 – 2ab 53. (a) 2 = 2 2 = 2 12 = (64 )
2ab = 24
ab = 12 1 1 1
(a + b)2 = 40 + 2ab
= 40 + 24
( )( )
4 6 = 6 4 = 63 12 = 216 12

a+b= 64 = 8 ...(ii) 1 1 1
Now, on solving equations (i) and (ii)
2a = 12
( )( )
3 4 = 4 3 = 44 12 = 256 12

\a=6 so, correct order is-


b=2 1 1 1
a3 + b6 = 216 + 64 = 280 Þ 12 12 12
51. (c) Let the given enpsession be x ( 264) < (216) < (256)

\ x= 4+ 4-x Þ 2<46<34
x2 – 4 = 4 - x So, option (a) is correct.
(x2 – 4)2 = 4 – x 1
-
1
x4 + 16 – 8x2 = 4 – x 54. (a) Given x = 2 3 + 2 3
x + (x2 – 4)2 = 4
1
13 + 1 1
All the choices are positive. By trial, x = x = 23 +
1
2
23
14 + 2 3 7 + 13 Cubing on both sides, we get
\ x2 = =
4 2
3
æ 1ö æ ö æ 1 ö
3
13 - 1 14 - 2 3 1
x2 – 4 = and (x2 – 4 )2 = ç 1 ÷ 3 1 ç 3 1 ÷
2 4 x3 = ç 2 3 ÷ + ç ÷ + 3.2 . 1 ç2 + 1 ÷
ç ÷ 1
7 - 13 è ø çè 2 3 ÷
ø 23
ç
è 23
÷
ø
=
2
1
So, option (c) is correct. 3
Þ x = 2+ + 3( x )
2
52. (b) x = 3 + 2
3 4 + 1 + 3x ´ 2
1 1 3- 2 Þ x =
= = 2
x 3+ 2 ( 3+ 2 )( 3- 2 ) Þ 2x = 5 + 6x
3

= 3- 2 Þ 2x3 – 6x – 5 = 0
1 1 \ option (a) is correct.
3
Þ x +x+ + 3
x x
Powers and Roots A- 45
55. (b) 4x.2y = 128 and 33x.32y – 9xy = 0 y2 = 144
22x. 2y = 128 =(2)7 33x + 2y = 32xy éëQ 9 = 3 û
2ù y = 12
y – x = 12 – 8 = 4
Þ 2x + y = 7 Þ 3x + 2y = 2xy ...(2)
59. (a) 21, devided by (1 + 1) R = 0
Þ y = 7 – 2x ...(1) 22, devided by (2 + 1) R = 1
Substitute this value of y in (2) we get
23, devided by (3 + 1) R = 0
3x + 2(7 – 2x) = 2x (7 – 2x)
24, devided by (4 + 1) R = 1
3x + 14 – 4x = 14x – 4x2 Same for even paper 100
4x2 – 15x + 14 = 0 2100, is devided by (100 + 1), R = 1
60. (c) 1729 can be written as
(4x – 7) (x – 2) = 0
123 + 13, 103 + 93
either 4x – 7 = 0 or x – 2 = 0
7
Þ x = or x = 2
4 1 1 1 1 1 1
61. (a) 1+ + + 1+ + + .... 1 + +
7 2
1 2
2 2
2 2
3 2007 2
20082
x¹ Þx=2
4
y = 7 – 2(2) = 3 æ 32 ö æ7ö
2
(2007 ´ 2008 + 1) 2
Þ ç ÷ + ç ÷ ....
Þ x+ y= 2+ 3 = 5 ç2 ÷ è6ø (2007) 2 (2008) 2
è ø
\ Option (b) is correct.
3 7 13 (2007 ´ 2008 + 1)
( 243 + 647 )2 + ( 243 - 647 )2 + + ....
2 6 12 (2007)(2008)
56. (c)
243 ´ 243 + 647 ´ 647
1 1 1 1
Þ 1+ + 1+ +1 + ....1 +
( 243)2 + ( 647 )2 + 2 × ( 243) × ( 647 ) + ( 243)2 + ( 647 )2 1´ 2 2´3 3´ 4 2007 ´ 2008
- 2 × ( 243)( 647 ) 1 1 1 1 1 1 1
=
Þ 2007 + 1 - + - + - .... -
( 243)2 + ( 647 )2 2 2 3 3 4 2007 2008
2 2 1
2 ( 243 ) + 2 ( 647 ) Þ 2008 –
= 2008
( 243)2 + ( 647 )2 62. (a) Only statement 1 is correct it can be easily varified by
taking any 2 odd integer.
2 éê( 243) + ( 647 ) ùú
2 2
ë û ex ® 3 and 5
= or
( 243)2 + ( 647 )2 the two odd number be 2n + 1 and 2n + 3
=2 squaring and adding by we get
\ Option (c) is correct. Þ 4n2 + 4n + 1 + 4n2 + 12n + 9
Þ 8n2 + 16n + 10
6 2 + 7 2 + 82 + 9 2 + 102
57. (a) 8n2 + 16n + 8 + 2
7+4 3 - 4+2 3 [42n2 + 4n + 2] + 2
i.e. x2 + y2 is an odd number but not necessarily a
330 multiple of 4.
Þ
(2 + 3) 2 - (1 + 3) 2 135 + 145 + 155 + 165 135 + 165 145 + 155
63. (d) = +
29 29 29
300
Þ = 330
2 + 3 -1 - 3 135 + 165 155 + 145
Þ +
58. (c) let two no. 8 be x and y 13 + 16 15 + 14
x2 + y2 = 208 no remainder
x2 + 18x = 208 \ y2 = 18x as xn + an when n is odd is completely devisibel by x +
2
x + 18x – 208 = 0 a
x2 + 26x – 8x – 208 = 0
(x – 8) (x + 26) = 0 é n(n + 1) 2 ù æ n(n + 1)(2n + 1) ö
64. (b) Sn3 – Sn2 = ê ú -ç ÷
x= 8 ëê 2 ûú è 6 ø
y2 = 18 × 8
n = 10
EBD_7367
46
A- Powers and Roots

2
é10 [11] ù é10 (11) (21) ù (ax3 + bx2 + cx + d )(ax 4 + bx3 + cx 2 + dx + e)
êë 2 úû - êë 6 úû
= a 2 x 7 + abx 6 + acx5 ¼d 2 x + de
3025 – 385 = 2640 Thus, it is clear that degree of f (x) g(x) is 7.
(0.35)2 + (0.70) + 1 72. (d) According to the rule, the numbers having 2, 3, 7 and
65. (a) + 0.19 8 at their unit’s place are not perfect squares.
2.25
Therefore, here, we can see that options 1, 3 and 4
(0.35)2 + 2(0.35)1 + 12 + 2.25 ´ 0.19 have 2, 3 and 7, respectively at their unit’s place and
Þ so, consequently, these are not perfect squares.
2.25

(0.35 + 1)2 + 0.4275 4x + 1 +1


Þ 73. (c) If x + x + x + ....... then its value is
2.25 2
Therefore, according to the question,
1.8225 + 0.4275 2.25
Þ Þ =1
2.25 2.25 4x + 1 + 1 4.1 + 1 + 1 5 + 1 2.236 + 1
= = =
2 2 2 2
(443 + 547)2 + (443 - 547) 2
66. (c)
443 ´ 443 + 547 ´ 547 3.236
= = 1.618 which lies between 1 and 2.
2
2[(443)2 + (547) 2 ]
Þ 1
(443) 2 + (547)2 74. (b) x = y a (given)
Q (a + b)2 + (a – b)2 = 2 (a2 + b2)
=2 1
67. (b) a – b = 5, a3 – b3 = 335 y= zb (given)
(a – b)3 = 125
a3 – b3 – 3ab [a – b] = 125 1
(given)
3ab [a – b] = 335 - 125 z= xc
210 Now
ab = = 14
5´3 1

(a – b)2 + 4ab = 25 + 4 (14) y= zb


(a + b)2 = 81
1 1 1 1
a+b=9 ´
68. (c) 9x 3y = 2187, 23x 22y = 4xy x = ya = zb a (Q y = z b )
9x3y = 9135 or 92 33 or 9331 in all the condition x + y = 5 1
69. (a) 51+x + 51–x = 26 z = xc
51+x + 51–x = 52 + 50
Case I 1 1 1
´
1 + x = 2, 1–x =0 z= z ab c (Q x = ab
z )
x= 1 x= 1
1
Case II
1 + x = 0, 1– x=2 z = z abc
1
x = –1, x = –1 \ =1 \ abc = 1
abc
70. (d) length and breadth when increased by 200%
75. (c) 2b = a + c (given)
200 ´ 200 Þ b+ b= a+c
net effect in area = 200 + 200 + = 800%
100 Þ a–b=b–c
y2 = xz (given)
ì xy ü
íx + y + ý x(b–c) × yc–a × z(a–b) = xa–b × yc–a × za–b
î 100 þ
(Q a – b = b – c)
71. 3 2
(b) Let f ( x) = ax + bx + cx + d (xz)a–b × yc–a {because an × bn = (ab)n}
(y2)a–b × yc–a {Q y2 = xz}
4 3 2
and g ( x) = ax + bx + cx + dx + e y2a – 2b + c – a = ya+c–2b = y2b–2b (Q a + c = 2b)
Then, f (x).g (x) = y0 = 1
Powers and Roots A-47

76. (b) Smallest 4 digits integer = 1000


1 1 1
312 < 1000 = a + b + c
b c a c
\ smallest 4 digits integer which is a perfect square is x +x +x x +x +x x + xa + xb
322 = 1024 xa xb xc
5- 3 5+ 3
77. (a) - xa xb xc
5+ 3 5- 3 = a + +
x + x b + x c x a + x b + xc x a + x b + xc
( ) - ( 5 + 3)
2 2
5- 3
= xa + xb + xc
( 5 + 3 )( 5 - 3 ) =
xa + xb + xc
=1

79. (a) Let number be x


( 5) + ( 3) - 2´ 5 ´ 3 -
2 2
According to the question
x2 + x = 20
í( 5 ) + ( 3 ) + 2 ´ 5 ´ 3 ý
ì 2 2 ü
x2 + x – 20 = 0
=î þ
x2 + 5x – 4x – 20 = 0
( 5) - ( 3)
2 2
x(x + 5) –4 (x + 5) = 0
(x – 4) (x + 5) = 0
5 + 3 - 2 15 - 5 - 3 - 2 15 -4 15 x–4 =0 \x=4
= = = -2 15
5-3 2 x+5=0 \ x = –5
1 1 80. (d) In these type of questions number of the pair of 5 and
78. (c) + even number makes a multiple of 10
1 + x b -a + x c -a 1 + x a -b + x c-b
in the product of 1 × 2 × 3 × 4 × ............... × 24 × 25
1 Number of multiples of 5 = 5 (5, 10, 15, 20, 25)
+ a -c
1+ x + x b -c Here 25 is a pair of 5 (25 = 5 × 5)
1 1 1 Number of 5 in this product = 6
= + + Number of even numbers in this product = 12
b c a c a
x x x x x xb
1+ a + a 1+ b + b 1+ c + c Hence required power of 10 is 6
x x x x x x
EBD_7367
48
A- Percentage

C HA P T E R
PERCENTAGE
5
1. If the length of a rectangle is increased by 10% and the 9. If salary of X is 20% more than salary of Y, then by how
area is unchanged, then by how much per cent does the much percentage is salary of Y less than X? [2011-I]
breadth decrease? [2007-I] (a) 25 (b) 20
(a) 100/11% (b) 100/9%
50 65
(c) 9% (d) 10% (c) (d)
2. The population of a village increases by 20% in one year 3 4
and decrease by 20% by the next year. If at the begining 10. If the height of a cone is increased by 50%, then what is
of the third year, the population is 5184, what was the the percentage increase in the volume of the cone?
population in the first year? [2007-II] [2011-I]
(a) 5400 (b) 5500 100
(c) 5600 (d) 5800 (a) % (b) 40 %
3
3. There are some coins and rings of either gold or silver in
a box. 60% of the objects are coins, 40% of the rings are 200
(c) 50 % (d) %
of gold and 30% of the coins are of silver. What is the 3
percentage of gold articles? [2009-I] 11. 38L of milk was poured into a tub and the tub was found
(a) 16% (b) 27% to be 5% empty. To completely fill the tub, what amount
(c) 58% (d) 70% of additional milk must be poured? [2011-I]
4. if the radius of the base and the height of a right circular (a) 1 l (b) 2 l
cone are increased by 20%, then what is the approximate (c) 3 l (d) 4 l
percentage increase in volume? [2009-II]
(a) 60% (b) 68% 2
12. Water contains 14 % of hydrogen and the rest is
(c) 73% (d) 75% 7
5. 10% of the inhabitants of a certain city left that city. Later oxygen. In 350 g of water, oxygen will be [2011-I]
on 10% of the remaining inhabitants of that city again left (a) 300g (b) 250g
the city. What is the remaining percentage of population (c) 200g (d) None of these
of that city? [2009-II] 13. To an examination, a candidate needs 40% marks. All
(a) 80% (b) 80.4% questions carry equal marks. A candidate just passed by
(c) 80.6% (d) 81% getting 10 answers correct by attempting 15 of the total
6. The number of workers in the employment guarantee questions. How many questions are there in the
scheme increased by 15 which resulted into an increase examination? [2011-I]
of 20%. What was the initial number of workers? (a) 25 (b) 30
[2009-II] (c) 40 (d) 45
(a) 60 (b) 75 14. The income of 'A' is 20% higher than that of 'B'. The
(c) 80 (d) 90 income of 'B' is 25% less than of 'C'. What percent less
7. If 50% of (x – y) = 40% of (x + y), then what per cent of is A's income from C's income? [2011-I]
x is y? [2011-I] (a) 7% (b) 8%
1 1 (c) 10% (d) 12.5%
(a) 10 % (b) 11 % 15. What is the number whose 20% is 30% of 40? [2011-I]
9 9
(a) 90 (b) 80
1 1 (c) 60 (d) 50
(c) 13 % (d) 21 %
9 9 16. An employee is required to contribute 10% of his
8. A person spends 30% of monthly salary on rent, 25% on payment to General Provident Fund. If he gets `13500 as
food, 20% on children's education and 12% on electricity net pay in a month, then what is the monthly General
and the balance of ` 1040 on the remaining items. What Provident Fund contribution (assuming no other
is the monthly salary of the person? [2011-I] deductions)? [2011-II]
(a) ` 8000 (b) ` 9000 (a) ` 1215 (b) ` 1350
(c) ` 9600 (d) ` 10600 (c) ` 1500 (d) ` 1650
Percentage A-49

17. A man losses 20% of his money. After spending 25% of 26. A person could save 10% of his income. But 2 years later,
the remaining, he has `480 left. What is the amount of when his income increased by 20%, he could save the
money he originally had? [2012-I] same amount only as before. By how much percentage
(a) ` 600 (b) ` 720 has his expenditure increased? [2015-I]
(c) ` 800 (d) ` 840
2 1
18. The radius of the base of a right circular cone is increased (a) 22 % (b) 23 %
9 3
by 15% keeping the height fixed. The volume of the cone
will be increased by [2012-II]
2 2
(a) 30% (b) 31% (c) 24 % (d) 25 %
9 9
(c) 32.25% (d) 34.75%
27. A milkman claims to sell milk at its cost price only. Still he is
19. The price of an article is ` 25. After two successive cuts
making a profit of 20% since he has mixed some amount of
by the same percentage, the price becomes `20.25. If each water in the milk. What is the percentage of milk in the
time the cut was x%, then [2012-II] mixture? [2015-II]
(a) x= 9 (b) x = 10
200
(c) x = 11 (d) x = 11.5 (a) % (b) 75%
3
20. What is 5% of 50% of 500? [2012-II]
(a) 12.5 (b) 25 250
(c) 80% (d) %
3
(c) 1.25 (d) 6.25
21. X,Y and Z had taken a dinner together. The cost of the 28. 20% of a number when added to 20 becomes the number
meal of Z was 20% more than that of Y and the cost of itself, then the number is [2015-II]
the meal of X was 5/6 as much as the cost of the meal of (a) 20 (b) 25
Z. If Y paid `100, then what was the total amount that all (c) 50 (d) 80
the three of them had paid? [2013-II] 29. A's salary was increased by 40% and then decreased by
(a) `285 (b) `300 20%. On the whole A's salary is increased by [2015-II]
(c) `355 (d) None of these (a) 60% (b) 40%
22. A person's salary has increased from `7200 to ` 8100. (c) 20% (d) 12%
What is the percentage increase in his salary?
30. In an election 10% of the voters on the voter list did not
[2013-II]
cast their vote and 60 voters cast their ballot papers blank.
(a) 25% (b) 18% There were only two candidates. The winner was supported
by 47% of total voters in the voter list and he got 308 voters
2 1
(c) 16 % (d) 12 % more than his rival. The number of voters on the voter list is
3 2
[2015-II]
23. A person sold an article from ` 3600 and got a profit of
(a) 3600 (b) 6200
20%. Had he sold the article for ` 3150, how much profit
would he have got? [2013-II] (c) 6028 (d) 6400
(a) 4% (b) 5% 31. The salary of a person is increased by 10% of his original
(c) 6% (d) 10% salary. But he received the same amount even after
increment. What is the percentage of his salary he did not
24. A water pipe is cut into two pieces. The longer piece is receive?
70% of the length of the pipe. By how much percentage
[2016-I]
is the longer piece longer than the shorter peice?
[2014-I] (a) 11% (b) 10%
(c) (100/11)% (d) (90/11)%
400
(a) 140% (b) % 32. The expenditure of a household for a certain month is
3
` 20,000, out of which ` 8,000 is spent on education, ` 5,900
(c) 40% (d) None of these on food, ` 2,800 on shopping and the rest on personal care.
25. On a 20% discount sale, an article costs 596. What was What percentage of expenditure is spent on personal care?
the original price of the article? [2014-I] [2016-I]
(a) ` 720 (b) ` 735 (a) 12% (b) 16.5 %
(c) ` 745 (d) ` 775 (c) 18 % (d) 21.8 %
EBD_7367
50
A- Percentage

33. A candidate scoring x% marks in an examination fails by a 36. A fruit seller has a certain number of mangoes of which 5%
marks, while another candidate who scores y% marks gets are rotten. He sells 75% of the remainder and he is left with
b marks more than the minimum required pass marks. What 95 mangoes. How many mangoes did he have originally?
is the maximum marks for the examination ? [2016-II] [2017-II]
100(a + b) 100(a - b) (a) 500 (b) 450
(a) (b) (c) 400 (d) 350
x-y x+y
37. A student has to secure 40% of marks to pass an
100(a + b) 100(a - b) examination. He gets only 45 marks and fails by 5 marks.
(c) (d) The maximum marks are [2017-II]
y-x x-y
(a) 120 (b) 125
34. If a% of a + b% of b = 2% of ab, then what percent of a is b ?
(c) 130 (d) 150
[2017-I]
38. When prices rise by 12%, if the expenditure is to be the
(a) 50% (b) 75% same, what is the percentage of consumption to be reduced?
(c) 100% (d) Cannot be determined [2017-II]
5 th 2 2
35. part of the population in a village are males. If 30% of (a) 16 % (b) 10 %
9 3 7
the males are married, the percentage of unmarried females 3 5
in the total population is [2017-I] (c) 16 % (d) 10 %
5 7
2 2 39. If the price of wheat rises by 25%, then by how much percent
(a) 20 % (b) 27 %
9 9 must a man reduce his consumption in order to keep his
budget the same as before? [2018-I]
7 2 (a) 15% (b) 20%
(c) 27 % (d) 29 %
9 9 (c) 25% (d) 30%
Percentage A-51

HINTS & SOLUTIONS


1. (a) Area of Rectangle = length × breadth 20 ´ 20 + 20 ´ 20 + 20 ´ 20 20 ´ 20 ´ 20
Here, length is increased by 10%. But area is = 20 + 20 + 20 + +
constant so that breadth is decreased. 100 10000
1200 8
B = K (Constant Area) = 60 + + = 72.8 » 73%
L X 100 10
5. (d) Here two times decrease in the population of certain
city. so net rate of decrement in population
(+10%) (Decrease) xy
= x+ y+
100
10 ´ 10
110 b´ 100 = K (Constant Area) = -10 - 10 +
L X 100
100 110 = –19%
Rest of Remaining population = (100 - 19)%
100b = 81%.
b-
Percentage decrease by = 110 ´100 = 100 % 6. (b) Let initial number of workers = x
b 11 \ 20% of x = 15
2. (a) Change in population at begining of third year
20
xy Þ ´ x = 15
= x+ y+ 100
100 \ x = 75
( 20 ) ´ ( -20 ) 7. (b) Given that, 50% of ( x - y ) = 40% of ( x + y )
= 20 - 20 +
100
50 40
= – 4% Þ ´(x - y) = ´(x + y)
Let Initial population = x 100 100
Þ 5x - 5 y = 4 x + 4 y
(100 - 4 )
Now, x ´ = 5184 Þ x = 9y ...(i)
100
Let r % of x = y
5184 ´100
x= r
96 Þ ´9y = y [from Eq. (i)]
100
= 5400.
100 1
3. (c) Total articles = 100 \ r= = 11 %
9 9
40 60 8. (a) Let the monthly salary of person = `x.
Total spends = (30% + 25% + 20% + 12%)
gold gold = 87%.
40 ´ 40 Now,
100
16 42 æ
ç 60 -
60 ´ 30 ö
÷
è 100 ø 13
x´ = 1040
100
1040 ´100
\x = = 8000.
13
Rings Coins
Total of gold articles = 42 + 16 = 58 of gold articles ( +20% )
9. (c) X ¬¾¾¾
¾Y
58 120 100
= ´ 100 = 58% 20 50
100 Less percentage = x100 =
4. (c) Change in volume 120 3
xy + yz + zx xyz 1 2
= x+ y+ z+ + 10. (c) Volume of cone = p r h
100 10000 3
Here, x = y = z = 20% Here radius is constant, then volume is directly
proportional to height. Now height increase 50%. So
Change in volume percentage increase in volume is 50%.
EBD_7367
52
A- Percentage
11. (b) Let tub capacity xL. 9x
According to question = 13500
Tube (x) 10
13500 ´10
Q x= = 15000
9
Q General provident fund = 10% of basic pay
10 ´15000
= = 1500
(x – 38) (l empty) 38 (Milk) 100
17. (c) Let man has originally ` x
95 x ´ 80 8 x
Now, x ´ = 38 After 20% loss = =
100 100 10
x = 40 L,
8 x 75 8 x 3
Additional milk = 40 L – 38 L = 2L. After spending 25% = ´ = ´
10 100 10 4
12. (a) Water (100%) According to the question,
8x 3
´ = 480
10 4
Þ 8 x ´ 3 = 480 ´ 4 ´ 10
480 ´ 4 ´10
\ x= = 800
Hydrogen Oxygen 8´3
98 + 2 100 600 xy
7
=
7
%
7
% 18. (c) x+ y+
100
Ratio of Hydrogen and oxygen = 1 : 6 15 ´ 15
= 15 + 15 +
6 100
Now, oxygen in 350g of water = ´ 350 = 300g
7 225
13. (a) Let the number of questions in examination = x = 30 +
By given condition, 40% of x = 10 100
= 32. 25%
x x 40 19. (b) According to the question,
Þ = 10
100
æ 100 - x öæ 100 - x ö
1000 Þ 25 ´ ç ÷ç ÷ = 20.25
\ x= = 25 è 100 øè 100 ø
40
202500
14. (c) ( +20% )
A ¬¾¾¾ ¾B C Þ (100 - x )2 = 2
Þ (100 - x ) = 8100
25
120 : 100 Þ 100 - x = 90
(–25%)
75 ¾¾¾¾ ® 100 \ x = 10
Making B equal 5 50
20 (a) 5% of 50% of 500 = ´ ´ 500 = 12.5
A : B : C 100 100
21. (d) According to question.
360 : 300
300 400 5
(+ 20%) (Z)
Now, A : B : C 6
Y Z X
18 : 15 : 20
2
A income less = ´ 100 = 10% ` 100 ` 120 `100
20
15. (c) Let the number be x So total amount = 100 + 100 + 120
According to question 20% of x = 30% of 40 = ` 320.
22. (d) Percentage increase in salary
x ´ 20 40 ´ 30
Þ = 8100 - 7200
100 100 = ´100
7200
40 ´ 30
Þ x= = 60 900 1
20 = ´ 100 = 12.5% = 12 %
7200 2
16. (c) Let the net pay of employee = x
23. (b) Let the cost price of the article be ` x
90 9x 120 x
After contributing 10% = x ´ = After 20% profit Þ = 3600
100 10 100
x = 3000
Percentage 53
A-

Now, profit percentage, when the article is sold for `3150 28. (b) Let the number be x.
3150 - 3000 150 According to question-
= ´100 = ´ 100 = 5%
3000 3000 20
Þ x + 20 = x
Pipe 100
24. (b)
x
Þ + 20 = x
5
x
Þ x- = 20
5
4x
Longer part Shorter part Þ = 20
5
x = 25
So, option (b) is correct
29. (d) Let the salary of A be 100.
70% Þ A’s salary after 40% increase will be
30%
100 + 40
Percentage of longer pipe as compare to shorter pipe 100 + = 140%
100
70 - 30 40 400 Þ A’s salary after 20% decrease will be
= ´100 = ´100 = %
30 30 3
140 ´ 20
25. (c) Let the original price of article be ` x 140 – = 112
Now, After 20% discount article costs ` 596 100
On a whole A’s salary is increased by
80 596 ´100
So, 596 = ´x Þ x= = 745 æ 112 - 100 ö
100 80 =ç ÷ ´100 = 12%
\ Original price = ` 745 è 100 ø
26. (a) Let income be ` 100 So, option (d) is correct.
90 Short cut method
Expenditure amount = 100 ´ = ` 90 (40 ´ –20)
100 + 40 – 20 + =20 – 8 = 12% ­ se.
120 100
Now, income increased by 20% = 100 ´ = ` 120 30. (b) Let the number of voters on voter list is ‘x’.
100 Valid votes = (0.9 x – 60)
Votes in support of 2nd candidate
Expenditure amount = (120 – 10) = ` 110 = (0.9 x – 60) – 0.47 x
Increase in expenditure = 110 – 90 = ` 20 According to question-
20 Þ 0.47 x – [(0.9x – 60) – 0.47x] = 308
Increase in % of expenditure = ´ 100 Þ 0.94 x – 0.9 x + 60 = 308
90 Þ 0.04x = 248
200 2 Þ x = 6200
= = 22 % So, option (b) is correct.
9 9
31. (c) Let original salary of a person = x `
27. (d) Milkman is getting 20% profit by selling the milk mixed Increase in salary = 10% of x
with water.
So, the quantity of milk he is selling less is Then New Salary = x + 10% of x

æ 20 ö 10 11
=ç ÷ ´100 x+ ´x = x
è 100 + 20 ø 100 10
100 But he received same amount even after increment.
= %
6 Let y be the percentage salary he did not receive.
This quantity is the percentage of water in milk that is =
11 11 y
100 then x - x´ =x
% Percentage of milk in the mixture 10 10 100
6
100
11 é y ù
Þ x 1- =x
= 100 -
6 10 êë 100 úû

500 250 y 10
= %= % Þ 1- =
6 3 100 11
So, option (d) is correct.
EBD_7367
54
A- Percentage

y 10 1 1
Þ = 1- = Married Female = Part
100 11 11 6

100 4 1 5
unmarried female = - =
Þ y= % 9 6 18
11
\ Option (c) is correct. 5
\= ´100
32. (b) Total Expenditure = 2000 ` 18
Education Expenditure = 8000 7
\ = 27 %.
Food Expenditure = 5900 9
Shopping Expenditure = 2800 36. (c) Let the number of mangoes the fruit seller had origi-
Personal care Expenditure = 20000 – (8000 + 5900 + 2800) nally be x.
= 20000 – 16700 We are given that, 5% of the total mangoes are rotteni.e.
= 3300 ` x
Percentage of expenditure on personal care 5% of x i.e. are rotten.
20
Personal care expenditure We are also given that 75% of the remainder are sold
= ´ 100
total expenditure i.e. 75% of (x – 5% of x)
25% of 95% of x = 95
3300
= ´ 100 = 16.5% x = 400
2000
37. (b) Here, the maximum marks are 100% and according to
\ Option (b) is correct. the question, 45 + 5 = 40% i.e. 50 = 40%.
33. (c) Let maximum mark be = M
ATQ Therefore, by unitary method,
xM yM 50
+a = -b [Passing marks] 1% =
100 100 40
M 50
a+b= (y- x) ´ 100 = 125.
100 100% =
40
100(a + b) Thus, maximum marks are 125 marks.
(y - x)
=M
38. (d) If the price rises by r% and expenditure remains same
a´a b´b 2
then the percentage of consumption to be reduced is
34. (c) + = ab 100r
100 100 100 given by %.
2 2
a + b = 2ab 100 + r
a2 + b2 – 2ab = 0
(a – b)2 = 0 100r 100 ´ 12 1200 5
%= %= % = 10 %
a=b 100 + r 100 + 12 112 7
\ a is 100% of b 39. (b) Let price of 1 kg wheat be 100 and total consumption
be 100 kg
5 Total cost at starting = 100 × 100 = 10000
35. (c) Male Population = Total new cost = 10000
9
4 125
Females population = New price of wheat/kg = 100 ´ = 125
9 100
30 5 10000
Married males = ´ New consumption = = 80 kg
100 9 125
1 decrease in consumption
=
6 100 - 80
= ´ 100 = 20%
100
Ratio and Proportion A-55

C HA P T E R

RATIO AND PROPORTION


6
1. x varies inversely as the square of y in such a way that, 10. What number must be added to each of 4, 10, 12 and 24,
if x = 1, then y = 6. If y = 3, then what is the value of x? so that the resulting numbers are in proportion?
[2007-I] [2008-II]
1 1 (a) 6 (b) 8
(a) (b) (c) 10 (d) 4
3 2 11. 10 years ago, Ram was 5 times as old as Shyam but 20
(c) 2 (d) 4 years later from now he will be only twice as old as
2. 6 years hence a father’s age will be three times his son’s Shyam. How many years old is Shyam? [2008-II]
age and three years ago father was nine times as old as
his son. What is the present age of father? [2007-I] (a) 20 years (b) 30 years
(a) 48 years (b) 42 years (c) 40 years (d) 50 years
(c) 36 years (d) 30 years 12. The sum of the age of a father and the age of a son is
3. In a mixture of 80 l, the ratio of milk and water is 3 : 1. If 75 years. If the product of their ages before 5 years was
the ratio of milk and water is to be 2 : 3 the how much 750, then what is the present age of the father?
amount of water is to be further added ? [2007-II] [2008-II]
(a) 70 l (b) 80 l (a) 60 years (b) 55 years
(c) 100 l (d) 140 l (c) 52 years (d) 50 years
4. The age of a mother, before two years, was eight times the 13. Let y is equal to the sum of two quantities of which one
age of there daughter. After 1 year, mother’s age will be varies directly as x and the other inversely as x. If y = 6
five times the daughter’s age. After how many years from when x = 4 and y = 10/3, when x = 3, then what is the
now the mother’s age will become three times the relation between x and y ? [2009-I]
daughter’s age? [2007-II] (a) y = x + (4/x) (b) y = – 2x + (4/x)
(a) 6 years (b) 8 years (c) y = 2x + (8/x) (d) y = 2x – (8/x)
(c) 10 years (d) 12 years 14. A bag contains ` 114 in the form of ` 1, 50 paise and 10
5. The monthly incomes of A and B are in the ratio paise coins in the ratio 3 : 4 : 10. What is the number of
4 : 3. Each of them saves ` 600. If the ratio of their 50 paise coins? [2009-I]
expenditure is 3 : 2, then what is the monthly income of (a) 76 (b) 72
A? [2007-II] (c) 56 (d) 48
(a) ` 2400 (b) ` 1800
1 1
(c) ` 2000 (d) ` 3600 15. If a : b = 1 : 2 and b : c = 2 : 3 1 , then what is a : b
6. A bag contains 50 paise, ` 1 and ` 2 coins in the ratio 2 2 4 2
: 3 : 4. If the total amount is ` 240, what is the total number : c equal to ? [2009-I]
of coins ? [2008-I] (a) 12 : 8 : 21 (b) 8 : 21 : 12
(a) 90 (b) 150 (c) 8 : 12 : 21 (d) 21 : 8 : 12
(c) 180 (d) 200 16. If the ratio of x to y is 25 times the ratio of y to x, then
7. If 78 is divided into three parts which are proportional to what is the ratio of x to y ? [2009-II]
1 1 (a) 1 : 5 (b) 5 : 1
1, , , then what is the middle part ? [2008-I] (c) 25 : 1 (d) 1 : 25
2 6 17. If x : y = 1 : 3, y : z = 5 : k, z : t = 2 : 5 and t : x = 3 :
28 4, then what is the value of k ? [2009-II]
(a) (b) 13
3 (a) 1/2 (b) 1/3
(c) 2 (d) 3
52 117 18. A person P started a business with a capital of ` 2525 and
(c) (d)
3 5 another person Q joined P after some months with a
8. x varies directly as y and inversely as square of z. When capital of ` 1200. Out of the total annual profit of ` 1644,
y = 4 and z = 14, x = 10. If y = 16 and z = 7, then what P’s share was ` 1212. When did Q join as partners ?
is value of x ? [2008-I] [2009-II]
(a) 180 (b) 160 (a) After 2 months (b) After 3 months
(c) 154 (d) 140 (c) After 4 months (d) After 5 months
9. What is the number which has to be added to each term 19. If a quantity y varies as the sum of three quantities of
of the ratio 49 : 68, so that it becomes 3 : 4? [2008-II] which the first varies as x, the second varies as – x + x2,
(a) 3 (b) 5 the third varies as x3 – x2, then what is y equal to?
(c) 8 (d) 9 [2010-I]
EBD_7367
56
A- Ratio and Proportion

(a) kx3, where k is a constant 31. Two vessels are full with milk and water mixed in the ratio
(b) kx + lx2 + mx3, where k, l, m are constants 1 : 3 and 3 : 5 respectively. If both are mixed in the ratio
(c) kx2, where k is a constant 3 : 2, what is the ratio of milk and water in the new mixture
(d) kx, where k is a constant ? [2011-I]
20. The wages of labourers in a factory has increased in the (a) 4 : 15 (b) 3 : 7
ratio 22 : 25 and their number decreased in the ratio 3 : (c) 6 : 7 (d) None of these
2. What was the original wages bill of the factory, if the 32. A bag contains ` 112 in the form of ` 1,50 paise and 10
present bill is ` 5000? [2010-I] paise coins in the ratio 3 : 8 : 10. What is the number of
(a) ` 4000 (b) ` 6000 50 paise coins? [2011-I]
(c) ` 8000 (d) None of these (a) 112 (b) 108
21. A mixture contains milk and water in the ratio 5 : 1. On (c) 96 (d) 84
adding 5l of water, the ratio of milk and water becomes 33. If p% of ` x is equal to t times q% of ` y, then what is
5 : 2. What is the quantity of milk in the original mixture? the ratio of x to y? [2011-I]
(a) pt : q (b) p : qt
[2010-II]
(c) qt : p (d) q : pt
(a) 5 l (b) 25 l
(c) 27.5 l (d) 32.5 l 3 5 3 2
34. If P : Q = : and Q : R = : , then what is
22. The ratio between the ages of A and B is 2 : 5. After 8 5 7 4 5
years their ages will be in the ratio 1 : 2. What is the P : Q : R equal to? [2011-II]
difference between their present ages? [2010-II] 3 5 2 9 15 2
(a) 20 years (b) 22 years (a) : : (b) : :
5 7 5 20 28 7
(c) 24 years (d) 25 years
23. If x varies as the mth power of y, y varies as the nth power 3 3 2 3 5 3
(c) : : (d) : :
of z and x varies as the pth power of z, then which one 5 4 5 5 7 4
of the following is correct? [2010-II] 35. If ` 8400 is divided among A, B and C in the ratio
(a) p = m + n (b) p = m – n 1 1 1
(c) p = mn (d) None of theses : : , what is the share of A? [2011-II]
24. If (a – b) : (a + b) = 1 : 5, then what is (a2 – b2) : (a2 + b2) 5 6 10
(a) 3200 (b) 3400
equal to? [2010-II]
(c) 3600 (d) 4200
(a) 2 : 3 (b) 3 : 2
36. Two numbers are in the ratio 3 : 5. If 9 is subtracted from
(c) 5 : 13 (d) 13 : 15 each number, then they are in the ratio of
25. Three numbers are in the ratio 3 : 2 : 5 and the sum of 12 : 23. What is the second number ? [2011-II]
their squares is 1862. What are the three numbers? (a) 44 (b) 55
[2010-II] (c) 66 (d) 77
(a) 18, 12, 30 (b) 24, 16, 40 37. The fourth proportional to 7, 11, 14 is [2011-II]
(c) 15, 10, 25 (d) 21, 14, 35 (a) 16 (b) 18
26. In a class, the number of boys is more than the number (c) 20 (d) 22
of girls by 12% of the total students. What is the ratio 38. If x : y = 7 : 5, then what is the value of (5x – 2y) : (3x
of number of boys to that of girls? [2010-II] + 2y)? [2012-I]
(a) 11 : 14 (b) 14 : 11 (a) 5/4 (b) 6/5
(c) 28 : 25 (d) 25 : 28 (c) 25/31 (d) 31/42
27. The ratio of A to B is x : 8 and the ratio of B to C is 12 39. What is the mean proportional between (15 + 200 )
: z. If the ratio of A to C is 2 : 1, then what is the ratio
of x : z? [2010-II] and (27 – 648 )? [2012-I]
(a) 2 : 3 (b) 3 : 2 (a) 4 (b) 14 7
(c) 4 : 3 (d) 3 : 4
28. If ` 2600 is divided among three persons A, B and C in the (c) 3 5 (d) 5 3
40. Two numbers are in the ratio 2 : 3. If 9 is added to each
1 1 1 number, they will be in the ratio 3 : 4. What is the product
ratio : : , then how much does A get ? [2011-I]
2 3 4 of the two numbers? [2012-I]
(a) ` 600 (b) ` 800 (a) 360 (b) 480
(c) ` 1000 (d) ` 1200 (c) 486 (d) 512
29. A certain amount of money has be divided between two 41. Sex ratio is defined as the number of females per 1000
persons P and Q in the ratio 3 : 5. But it was divided in males. In a place, the total inhabitants are 1935000 out of
the ratio 2 : 3 and thereby Q loses ` 10. What was the which 935000 are females. What is the sex ratio for the
amount? [2011-I] place ? [2012-II]
(a) ` 250 (b) ` 300 (a) 935 (b) 1000
(c) ` 350 (d) ` 400 (c) 1935 (d) 9350
30. ‘X’ is twice as old as ‘Y’ 3 years ago, when ‘X’ was as old 42. A milkman bought 15 kg of milk and mixed 3 kg of water
as ‘Y’ today. If the difference between their ages as in it. If the price per kg of the mixture becomes ` 22, what
is cost price of the milk per kg? [2012-II]
present is 3 years, how old is ‘X’ at present? [2011-I]
(a) ` 28.00 (b) ` 26.40
(a) 18 years (b) 12 years
(c) ` 24.00 (d) ` 22.00
(c) 9 years (d) 8 years
Ratio and Proportion 57
A-

43. Age of X is six times that of Y. After 4 years, X is four (a) 2 : 1 (b) 7 : 3
times elder to Y. What is the present age of Y ? (c) 4 : 3 (d) 8 : 3
[2012-II] 4
(a) 4 years (b) 5 years 53. A tin of oil was full. When 6 bottles of oil were taken out
5
(c) 6 years (d) 7 years
from this tin and 4 bottles of oil were poured into it, it was
44. In a certain school, the ratio of boys to girls is 7 : 5. If
there are 2400 students in the school, then how many 3
full. Oil of how many bottles can the tin contain ? (All
girls are there ? [2013-I] 4
(a) 500 (b) 700 bottles are of equal volume) [2015-II]
(c) 800 (d) 1000 (a) 35 (b) 40
45. If a, b, c, d and e are in continued proportion, then a/e (c) 45 (d) 50
is equal to [2013-I] 54. If a : b = 3 : 5 and b : c = 7 : 8, then 2a : 3b : 7c is equal to
(a) a3 / b3 (b) a4 / b4 [2015-II]
(c) b3 / a3 (d) b4 / a4 (a) 42 : 105 : 320 (b) 15 : 21 : 35
46. If A : B = 2 : 3, B : C = 5 : 7 and C : D = 3 : 10, then what (c) 6 : 15 : 40 (d) 30 : 21 : 350
is A : D equal to ? [2014-I] 55. In a mixture of milk and water of volume 30 litre, the ratio of
(a) 1 : 7 (b) 2 : 7 milk and water is 7 : 3. The quantity of water to be added to
(c) 1 : 5 (d) 5 : 1 the mixture to make the ratio of milk and water 1 : 2 is
47. The height of a tree varies as the square root of its age [2015-II]
(between 5 to 17 yr). When the age of the tree is 9 yr, its (a) 30 (b) 32
height is 4 ft. What will be the height of the tree at the (c) 33 (d) 35
age of 16 yr ? [2014-II] 56. The annual incomes of two persons are in the ratio 9 : 7 and
(a) 5 ft 4 inch (b) 5 ft 5 inch their expenses are in the ratio 4 : 3. If each of them saves
` 2,000 per year, what is the difference in their annual
(c) 4 ft 4 inch (d) 4 ft 5 inch
income? [2016-I]
48. The ratio of ages of A and B is 2 : 5 and the ratio of ages
(a) ` 4,000 (b) ` 4,500
of B and C is 3 : 4, What is the ratio of ages of A, B and
(c) ` 5,000 (d) ` 5,500
C ? [2014-II]
57. In an office, one third of the workers are women, half of the
(a) 6 : 15 : 20 (b) 8 : 5 : 3 women are married and one third of the married women
(c) 6 : 5 : 4 (d) 2 : 15 : 4 have children. If three fourth of the men are married and one
1 1 1 third of the married men have children, then what is the
49. The sides of a triangle are in the ratio : : . If its ratio of married women to married men ? [2016-II]
2 3 4
(a) 1 : 2 (b) 2 : 1
perimeter is 52 cm, then what is the length of the smallest
(c) 3 : 1 (d) 1 : 3
side? [2014-II] 58. There are twelve friends A, B, C, D, E, F, G, H, I, J, K and L
(a) 9 cm (b) 10 cm who invested money in some business in the ratio of 1 : 2 :
(c) 11 cm (d) 12 cm 3 : 4 : 5 : 6 : 7 : 8 : 9 : 10 : 11 : 12 and the duration for which they
50. (x + y) : (x – y) = 3 : 5 and xy = positive imply that invested the money is in the ratio of 12 : 11 : 10 : 9 : 8 : 7 : 6
[2014-II] : 5 : 4 : 3 : 2 : 1 respectively. Who will get the maximum profit
(a) x and y are both positive at the end of the year ? [2016-II]
(b) x and y are both negative (a) F only (b) G only
(c) one of them is positive and one of them is negative (c) Both F and G (d) Neither F nor G
(d) no real solutions for x and y exist 59. Incomes of Mahesh and Kamal are in the ratio 1 : 2 and their
51. A milkman claims to sell milk at its cost price only, but he expenses are in the ratio 1 : 3. Which one of the following
is making a profit of 20% since he has mixed some amount statements is correct? [2016-II]
of water in the milk. What is the percentage of milk in the (a) Mahesh saves more than what Kamal saves
mixture? [2015-I] (b) Savings of both of them are equal
(c) Kamal saves more than what Mahesh saves
250 (d) It is not possible to determine who saves more
(a) 80% (b) %
3 60. X and Y entered into partnership with `700 and `600
respectively. After 3 months X withdrew 2/7 of his stock
200
(c) 75% (d) % but after 3 months, he puts back 3/5 of What he had
3 withdrawn. The profit at the end of the year is `726. How
52. 16 litres of a mixture contains milk and water in the ratio much of this should X receive ? [2016-II]
5:3. If 4 litres of milk is added to this mixture, the ratio of (a) `336 (b) `366
milk to water in the new mixture would be [2015-I] (c) `633 (d) `663
EBD_7367
58
A- Ratio and Proportion

61. The cost of a diamond varies directly as the square of its (a) 302 (b) 280
weight. A diamond broke into four pieces with their weights (c) 242 (d) 230
in the ratio of 1 : 2 : 3 : 4. If the loss in total value of the 1
diamond was ` 70,000, what was the price of the original 70. of the students who registered did not appear for the
25
diamond ? [2017-I] 11
(a) ` 1,00,000 (b) ` 1,40,000 examination, of those who appeared passed. If the
20
(c) ` 1,50,000 (d) ` 1,75,000 number of registered students is 2000, the number who
1 passed is [2018-I]
62. Leela got married 6 years ago. Today her age is 1 times (a) 1920 (b) 1056
4
(c) 1020 (d) 864
1 71. If A : B = 1 : 2, B : C = 3 : 4, C : D = 2 : 3 and D : E = 3 : 4, then
her age at the time of her marriage. Her son's age is times
10 what is B : E equal to? [2018-I]
her age. What is the present age of her son ? [2017-I] (a) 3 : 2 (b) 1: 8
(a) 1 year (b) 2 years (c) 3 : 8 (d) 4 : 1
(c) 3 years (d) 4 years 72. ` 120 is distributed among A, B and C so that A’s share is
63. In a class of 49 students, the ratio of girls to boys is 4 : 3. If ` 20 more than B’s and ` 20 less than C’s. What is B’s
4 girls leave the class, the ratio of girls to boys would be share? [2018-I]
[2017-I]
(a) `10 (b) `15
(a) 11 : 7 (b) 8 : 7
(c) `20 (d) `25
(c) 6 : 5 (d) 9 : 8
73. A gentleman left a sum of `39,000 to be distributed after his
64. The ratio of two numbers is 1 : 5 and their product is 320.
death among his widow, five sons and four daughters. If
What is the difference between the squares of these two
each son receives 3 times as much as a daughter receives,
numbers ? [2017-I]
and each daughter receives twice as much as their mother
(a) 1024 (b) 1256
receives, then what is the widow’s share? [2018-I]
(c) 1536 (d) 1640
(a) ` 1,000 (b) ` 1,200
65. 25 kg of alloy X is mixed with 125 kg of alloy Y. If the amount
of lead and tin in the alloy X is in the ratio 1 : 2 and the (c) ` 1,500 (d) None of the above
amount of lead and tin in the alloy Y is in the ratio 2 : 3, then 74. The age of a woman is a two-digit integer. On reversing this
what is the ratio of lead to tin in the mixture ? [2017-I] integer, the new integer is the age of her husband who is
(a) 1 : 2 (b) 2 : 3 elder to her. The difference between their ages is one-
(c) 3 : 5 (d) 7 : 11 eleventh of their sum. What is the differences between their
66. To maintain 8 cows for 60 days, a milkman has to spend ` ages? [2018-I]
6,400. To maintain 5 cows for n days, he has to spend ` (a) 8 years (b) 9 years
4,800. What is the value of n? [2017-II] (c) 10 years (d) 11 years
(a) 46 days (b) 50 days 75. An alloy A contains two elements, copper and tin in the
(c) 58 days (d) 72 days ratio of 2 : 3, whereas an alloy B contains the same elements
67. Five years ago, Ram was three times as old as Shyam. Four in the ratio of 3 : 4. If 20 kg of alloy A, 28 kg of alloy B and
years from now, Ram will be only twice as old as Shyam. some more pure copper are mixed to form a third alloy C
What is the present age of Ram? [2017-II] which now contains copper and tin in the ratio of 6 : 7, then
(a) 30 years (b) 32 years what is the quantity of pure copper mixed in the alloy C?
(c) 36 years (d) 40 years [2018-I]
1 1 (a) 3 kg (b) 4 kg
68. If 78 is divided into 3 parts which are proportional to 1, , , (c) 5 kg (d) 7 kg
3 6
then the middle part is [2017-II] 76. A, B, C, D and E start a partnership firm. Capital contributed
by A is three times that contributed by D. E contributes half
28 of A’s contribution, B contributes one-third of E’s
(a) (b) 13
3 contr ibution and C contributes two-thir d of A’s
52 55 contribution. If the difference between the combined shares
(c) (d) of A, D and E and the combined shares of B and C in the
3 3
total profit of the firm is `13,500, what is the combined share
69. There are 350 boys in the first three standards. The ratio of
of B, C and E? (The shares are supposed to be proportional
the number of boys in first and second standards is 2 : 3,
to the contributions) [2018-I]
while that of boys in second and third standards is 4 : 5.
What is the total number of boys in first and third standards? (a) ` 13,500 (b) ` 18,000
[2017-II] (c) ` 19,750 (d) ` 20,250
Ratio and Proportion 59
A-

HINTS & SOLUTIONS


1 k y = 6 years and x = 34 years
1. (d) Q xµ Þ x= ...(i)
y 2
y2 Let after z years mother will be three times of her
daughter.
x = 1 and y = 6
34 + z = 3 (6 + z) Þ z = 8 years
From Eq. (i),
5. (a) Let monthly incomes of A’s and B’s are 4x and 3x.
k And monthly expenditures of A’s and B’s are 3y and
\ 1= Þ k = 36
62 2y.
On putting the value of k in Eq. (i), we get Each saving = ` 600
\ Income – Expenditure = Saving
36 \ 4x – 3y = 600 ...(i)
x= ...(ii)
y2 3x – 2y = 600 ...(ii)
On putting y = 3 in Eq. (ii), we get On solving equations. (i) and (ii), we get
x = 600
36 \ A’s income = 4x = 4 × 600 = ` 2400
x= =4
9 6. (c) Let the number of coins of 50 paise, ` 1 and ` 2
2. (d) Let the age of son and father is x and 3x years coins are 2x, 3x and 4x respectively.
respectively 6 years hence. Value of 50 paise coins = ` x
\ Present age of father = (3x – 6) years Value of ` 1 coins = ` 3x
Present age of son = (x – 6) years Value of ` 2 coins = ` 8x
3 years ago, age of father = 3x – 6 – 3 = (3x – 9) According to the question,
years x + 3x + 8x = 240
Age of son = x – 6 – 3 = (x – 9) years
240
According to given condition, Þ x= = 20
3x – 9 = 9(x – 9) Þ 3x – 9 = 9x – 81 12
Þ 81 – 9 = 9x – 3x Þ 6x = 72 Hence, total number of coins = 2x + 3x + 4x
= 9x = 9 × 20 = 180
72
\ x= = 12 1 1
6 7. (d) Ratio of three parts = 1: : = 6 : 3 : 1
\ Present age of father = 3 × 12 – 6 = 30 years 2 6
3. (a) Volume of mixture of milk and water = 80 l 3
Ratio of milk and water = 3 : 1 \ Middle part = ´ 78
6 + 3+1
3 3 ´ 78 3 ´ 39
\ Volume of milk = ´ 80 117
3 +1 = = =
10 5 5
3 1
=´ 80 = 60 l 8. (b) Given, x µ y and x µ
4
z2
Volume of water = 80 – 60 = 20 l ky
y
Let amount of water added = x l Now, x µ Þx= 2
According to given condition, z2 z
Q x = 10 at y = 4 and z = 14
60 2
= k ×4 1960
20 + x 3 \ 10 = Þ k= = 490
Þ 40 + 2x = 180 196 4
Þ 2x = 180 – 40 Now, at z = 7 and y = 16
Þ 2x = 140 490 ´ 16
\ x = 70 l \ x= = 160
4. (b) Let present age of mother and daughter be x and 7´7
y. 9. (c) Let x number be added to 49 : 68, then it becomes
2 years ago, x – 2 = 8(y – 2) 3:4.
Þ x – 8y = – 14 ...(i) 49 + x 3
1 year after, x + 1 = 5(y + 1) \ =
68 + x 4
Þ x – 5y = 4 ...(ii) Þ 196 + 4x = 204 + 3x
On solving equations. (i) and (ii), we get \ x=8
EBD_7367
60
A- Ratio and Proportion

10. (d) Suppose x number is added to each term, they 14


become in proportion i.e., (4 + x), (10 + x), (12 + x) Þ l= =2
and (24 + x) are in proportion. 7
\ (4 + x) : (10 + x) :: (12 + x) : (24 + x) From Eq. (iii),
9 × 2 + m = 10 Þ m = 10 – 18 = – 8
4+ x 12 + x From Eq. (i),
Þ =
10 + x 24 + x 8
Þ (4 + x)(24 + x) = (10 + x)(12 + x) y = 2x –
Þ 96 + 28x + x2 = 120 + 22x + x2 x
Þ 28x – 22x = 120 – 96 Þ 6x = 24 14. (a) Let the number of ` 1, 50 paise and 10 paise coins
\ x=4 are 3x, 4x and 10x respectively.
11. (a) Let the age of Shyam and Ram was x and 2x years Given, total money = ` 114
respectively 20 years later from now. Þ (3x) + 0.5 (4x) + 0.1 (10x) = 114
\ Present age of Ram = (2x – 20) years Þ 3x + 2x + x = 114
and present age of Shyam = (x – 20) years Þ 6x = 114
10 years ago, the age of Ram = 2x – 20 – 10 114
= 2x – 30 \ x= = 19
6
Age of Shyam = x – 20 – 10 = (x – 30) years \ Number of 50 paise coins = 4 × 19 = 76
According to the question
2x – 30 = 5(x – 30) 3 9 7
15. (c) a : b = : and b : c = 2 :
Þ 2x – 30 = 5x – 150 2 4 2
Þ 150 – 30 = 5x – 2x Þ a : b = 6 : 9 and b : c = 4 : 7
Þ 3x = 120 LCM of 9 and 4 = 36
120 Þ a : b = 24 : 36 and b : c = 36 : 63
\ x= = 40 \ a : b : c = 24 : 36 : 63
3 Þ a : b : c = 8 : 12 : 21
\ The present age of Shyam = 40 – 20 = 20 years.
12. (b) Let the present age of father and the son is x years x æ yö
16. (b) Given, = 25 ç ÷
and (75 – x) years respectively. y èxø
5 years ago, age of father = (x – 5) years
and age of son = 75 – x – 5 = (70 – x) years x2 25 x 5
Þ = Þ =
According to the question, y2 1 y 1
(x – 5)(70 – x) = 750
17. (a) Given, x : y = 1 : 3, y : z = 5 : k, z : t = 2 : 5
Þ 70x – x2 – 350 + 5x = 750
Þ – x2 + 75x – 350 = 750 t:x=3:4
Þ x2 – 55x – 20x + 1100 = 0 x y z t
´ ´ ´ =1
Þ (x – 55)(x – 20) = 0 y z t x
\ x = 20, 55
\ Present age of father = 55 years 1 5 2 3 1
Þ ´ ´ ´ =1 Þ =k
13. (d) According to the question, 3 k 5 4 2
m 1
y = lx + ...(i) \ k=
x 2
where, l and m are proportionality constant. 18. (b) Let Q join for x month.
m \ Ratio of capital = 2525 × 12 : 1200 × x
When y = 6, x = 4, then 6 = 4l + = 2525 : 100x = 101 : 4x
4
Þ 16l + m = 24 ...(ii) 101
\ P’s profit = ´ 1644
101 + 4x
10
When y = , x = 3, then 101´1644
3 Þ 1212 =
m
101 + 4x
10
= 3l + 1212 1
3 3 Þ =
Þ 9l + m = 10 ...(iii) 101´1644 101 + 4x
From equations. (ii) and (iii), we get. 1 1
16l + m = 24 Þ =
137 101 + 4x
9l + m = 10
– – – Þ 101 + 4x = 137 Þ 4x = 36
\ x=9
7l + 0 = 14
Q joined for 9 month i.e., he joined after 3 months.
Ratio and Proportion 61
A-

19. (b) Since, first term µ x Þ First term = c1x 2a 6 3 a 3


Second term µ (– x + x2) Þ = = Þ =
2b 4 2 b 2
Þ Second term = c2 (– x + x2)
Third term µ (x3 – x2) a2 9
Þ Third term = c3 (x3 – x2) Þ 2 =
b 4
Also, y µ [c1x + c2 (– x + x2) + c3 (x3 – x2)]
Þ y = c4 [c1 – c2) x + (c2 – c3) x2 + c3x3] a 2 + b2 13
= c4 (c1 – c2) x + (c2 – c3) c4 x2 + c3c4x3 Þ 2 2 =
a -b 5
= kx + lx2 + mx3
(using componendo and dividendo rule)
where k = c4 (c1 – c2),
l = (c2 – c3) c4 and m = c3c4 a 2 - b2 5
20. (d) Let initial salary = ` 22x Þ 2 2 =
a +b 13
Final salary = ` 25x
\ (a2 – b2) : (a2 + b2) = 5 : 13
Let initial number of employees = 3y
25. (d) Let the numbers be 3x, 2x and 5x.
Final number of employees = 2y
\ (3x)2 + (2x)2 + (5x)2 = 1862
\ Present bill = Final salary × Final number of
Þ 9x2 + 4x2 + 25x2 = 1862
employees
Þ 5000 = 25x × 2y 1862
Þ x2 = = 49
5000 38
Þ = xy x=7
50
Hence, the required numbers are 3 × 7, 2 × 7 and 5
Þ xy = 100 × 7. i.e., 21, 14 and 35
\ Original bill = Initial salary × Initial number of 26. (b) Let the number of boys = x
employees = 22x × 3y The number of girls = y
= 66xy = 66 × 100 = ` 6600 According to the question,
21. (b) Let quantities of milk and water are 5x and x l.
According to the question, ( x + y) ´12
x–y=
5x 5 100
= Þ 25x – 25y = 3x + 3y
x+5 2 Þ 22x = 28y Þ x : y = 14 : 11
Þ 10x = 5x + 25 27. (c) Given, A : B = x : 8, B : C = 12 : z
Þ 5x = 25
A A B
\ x=5 Þ = ´
Hence, the quantity of milk in the original mixture C B C
= 5 × 5 = 25 l x 12 3x
22. (c) Let the ages of A and B are 2x and 5x years. = ´ =
8 z 2z
According to the question, But A : C = 2 : 1
2x + 8 1 3x 2
= Þ = Þ x:z=4:3
5x + 8 2 2z 1
Þ 4x + 16 = 5x + 8
Þ x=8 1
\ Difference between their present ages 2
28. (d) A’s part = 1 1 1 ´ 2600
= 5x – 2x = 3x = 3 × 8 = 24 years
+ +
23. (c) Q x µ ym ...(i) 2 3 4
y µ zn ...(ii) 1 12
= ´ ´ 2600 = ` 1200
x µ zp ...(iii) 2 13
On putting the values if x and y from equations (ii) 29. (d) Let the amount be ` x.
and (iii) in Eq. (i) we get In first condition,
zp µ (zn)m
5x 5
Þ zp µ zmn Q’s part = = x
\ p = mn 5+3 8
In second condition,
a -b 1
24. (c) Given, = 3x 3
a+b 5 Q’s part = = x
2+3 5
a+b 5 According to the question,
Þ =
a -b 1 5 3
x - x = 10
a +b+ a -b 5 +1 8 5
Þ =
a +b- a +b 5 -1 x
(using componendo and dividendo rule) \ = 10 Þ x = ` 400
40
EBD_7367
62
A- Ratio and Proportion

30. (c) Let present age of X = x years 15 2


Present age of Y = (x – 3) years = : ...(iv)
3 years ago, age of X = (x – 3) years 28 7
Age of Y = (x – 6) years From equations (iii) and (iv),
According to the question, 9 15 2
x – 3 = 2(x – 6) P:Q:R= : :
20 28 7
Þ x – 3 = 2x – 12
Þ 12 – 3 = 2x – x 1 1 1
35. (c) Given, A : B : C = : : =6:5:3
x = 9 years 5 6 10
31. (d) By alligation method,
6 6
\ Share of A = ´ 8400 = ´ 8400
1 3 6+5+3 14
4 8 = ` 3600
x 36. (b) Let two numbers are 3x and 5x.
According to the question,
3 2 3x - 9 12
=
3 5x - 9 23
-x 3
8 Þ 69x – 207 = 60x – 108
\ =
1 2 69x – 60x = 207 – 108
x-
4 Þ 9x = 99
\ x = 11
3 3 \ Second number = 5 × 11 = 55
Þ - 2x = 3x -
4 4 37. (d) Let fourth proportional be x.
6 3 7 : 11 : : 14 : x
Þ 5x = =
4 2 7 14
Þ =
3 11 x
\ x= Þ x = 2 × 11
10
32. (a) Let the number of ` 1,50 paise and 10 paise coins are \ x = 22
3x, 4x and x respectively. x 7
According to the question, 38. (c) = (Given that)
y 5
3x + 4x + x = 112
5x - 2 y (5 ´ 7 - 2 ´ 5) 35 - 10 25
112 \ = = =
Þ x= = 14 3x + 2 y (3 ´ 7 + 2 ´ 5) 21 + 10 31
8
\ Number of 50 paise coins = 14 × 8 = 112 39. (c) Mean proportional between (15 + 200 ) and
33. (c) p% of x = t (q% of y)
(27 – 648 )
xp yq
Þ = ´t
100 100 = (15 + 200)(27 - 648)
x qt
Þ = = (15 ´ 27) - (15 ´ 648) + 27 200 - ( 200 ´ 648)
y p
\ x : y = qt : p = 405 - (15 ´18 2) + (27 ´ 10 2) - 10 2 ´18 2)
3 5
34. (b) Given, P : Q = :
5 7
...(i)
( ) (
= 405 - 270 2 + 270 2 - 360 )
3 2 = 405 - 360 = 45 = 3 5
Q:R= : ...(ii)
4 5 40. (c) Let the two numbers are 2x and 3x,
From Eq. (i), According to the question,
3 3 5 3 2x + 9 3
P:Q= ´ : ´
5 4 7 4 =
3x + 9 4
9 15 Þ 9x + 27 = 8x + 36
= : ...(iii)
20 28 Þ 9x – 8x = 36 – 27
From Eq. (ii), \ x=9
3 5 2 5 So the numbers are 18 and 27.
Q:R= ´ : ´ Their product = 18 × 27 = 486.
4 7 5 7
Ratio and Proportion 63
A-

41. (a) Total number of inhabitants = 1935000 46. (a) Given, A : B = 2 : 3, B : C = 5 : 7 and C : D = 3 : 10
Total number of females = 935000 A A B C 2 5 3 1
\ Total number of males = 1935000 – 935000 \ = ´ ´ = ´ ´ =
= 1000000 D B C D 3 7 10 7
\ A:D=1:7
935000 47. (a) Let height of tree be h ft and age be a yr.
\ Sex ratio = ´ 1000 = 935
1000000 Now, according to the question,
42. (b) Let cost price of milk ` x per kg. hµ a
Price of 15kg of milk = ` 15x.
Now, mix 3kg of water, therefore quantity of mixture Þ h=k a
= (15 + 3) kg = 18 kg Here k is a constant.
So, price of mixture is `22 per kg Now, age = 9 yr, So, height = 4 ft
According to question. Þ From Eq. (i), we get 4 = k 9
15x = 22 × 18 Þ 4=k×3
22 ´18 4
x=
15 \ k=
3
132 16
= = 26.40 Again of age = 16 yr, then h = k
5
4 é 4ù
êQ k = 3 ú
Alternate Method : = ´4
Let CP of milk be ` x per kg. 3 ë û
By Alligation method
16 1
price of milk price of water \ h= ft = 5 ft = 5ft 4 inch
x 0 3 3
48. (a) A : B = 2 : 5 ® × 3
22 B:C=3:4®×5
Now, A : B = 6 : 15
22 x – 22 B : C = 15 : 20
\ 22 : (x – 22) = 15 : 3 So A : B : C = 6 : 15 : 20

22 15 1 1 1
49. (d) Sides of a triangle are in the ratio = : :
Þ = 2 3 4
x - 22 3
=6:4:3
22 x involves in ratio so that sides of a triangle 6x, 4x
Þ =5
x - 22 and 3x respectively.
Þ 22 = 5x – 110 Perimeter of a triangle = Sum of all sides of a triangle
Þ 5x = 132 Þ 52 = 6x + 4x + 3x Þ 52 = 13x
\ x = ` 26.40
52
43. (c) Let the age of X and Y are x years and y years \ x= =4
respectively. 13
Then, (x + 4) = 4(y + 4) \ Smallest side of a triangle = 3x = 3 × 4 = 12 cm
Þ 6y + 4 = 4y + 16 x+ y 3
Þ 2y = 12 50. (d) \ =
x- y 5
\ y=6
\ The present age of y = 6 years Apply componendo and dividendo rule, we get
44. (d) Let the number of boys and girls are 7x and 5x, ( x + y ) + ( x - y) 3 + 5
respectively. =
( x + y ) – ( x - y) 3 – 5
According to question,
Þ 7x + 5x = 2400 Þ 12x = 2400 2x 8
\ x = 200 Þ =
2 y –2
\ Number of girls = 5x
= 5 × 200 = 1000 x
45. (b) If a, b, c, d and e are continued proportion. Þ = –4
y
a b c d
\ = = = = k ( let ) \ x = – 4y
b c d e But given that, xy = Positive
Þ d = ek, c = dk = ek2, b = ek3, a = ek4 \ –4y × y = Positive
Þ –4y2 = Positive, it is not possible.
a ek 4 a4 a
\ = = k4 = 4 [Q k = ] Therefore, no real solution for x and y exist.
e e b b
EBD_7367
64
A- Ratio and Proportion

51. (b) Let milkman buy milk of `100. 56. (a) Let annual income of one person = 9x
20% profit then = `120 Let annual income of second person = 7x
Water Milk Let Expenses of Ist person = 4y
0 100
Let Expenses of 2nd person = 3y
Saving of Ist person = 2000
Saving of 2nd person = 2000
120
According to condition,
9x – 4y = 2000
7x – 3y = 2000
20 120
1 6 On solving, we get
6 –1 5 250 x = 2000
percentage of milk = ´ 100 = ´ 100 = % Annual income of 1st person = 9 × 2000 = 18000
6 6 3
Annual income of 2nd person = 7 × 2000 = 14000
52. (b) 16 litres
Difference between their incomes = 4000 `.
\ Option (a) is correct.
10 l milk 6 l water
57. (d) Men Women
If 4l milk is added in mixture then 2x x
(10 + 4) 3 3
New ratio =
6
14 7
= = = 7:3
6 3 Married
Married Unmarried Unmarried
53. (b) Let the number of bottles in the tin be 20 n. x x
[LCM of (5, 4) = 20]. Initially it had 16 n bottles. 6 3 æ 2x ö 1 æ 2x ö
bottles were removed and 4 were poured into the tin. 4 çè 3 ÷ø 4 çè 3 ÷ø 6 6

3 married women x / 6
Then it was full. = = 1: 3
4 married men x/2
16n – 6 + 4 = 15n Þ n = 2 58. (c) Multiplying ratio of investment and time we get ratio
\ 20n = 20 × 2 = 40 12 : 22 : 30 : 36 : 40 : 42 : 42 : 40 : 36 : 30 : 22 : 12
So, option (b) is correct. where F and G have equal maximum profit
59. (d) Not possible to determine.
a 3 3
54. (c) < Þa< b 60. (b) x invested = `700, y invested `600
b 5 5
2
b 7 7 8 x after 3 month = 700 – (700) = `500
< Þb< c< b 7
c 8 8 7
3
again after 3 months deposited = 500 Å (200)
3 8 5
a: b: c= b:b: b
5 7 = `620
= 21 : 35 : 40 effective money invested by x = 700 × 3 + 500 × 3 + 620
2a : 3b : 7c = 42 : 105 : 280 × 6 = 2100 + 1500 + 3720
= `7320
= 6 : 15 : 40
effective money invested by y = 600 × 12 = `7200
So, option (c) is correct
7 7320
Profit of x = ´ 726
55. (c) Milk in the mixture = 30 × = 21 litres [7320 + 7200]
10
3 7320
Water in the mixture =30 × = 9 litres ´ 726 = `366
10 14520
Let the quantity of water added be x. 61. (a) Let price of diamond be kx2 where K is constant
According to question- total price for 4 pieces
21 1 Kx2 [1 + 4 + 9 + 16] = 30 kx2
< Price of original diamond = 100 kx2
9∗x 2
difference = 70kx2 = 70000 or kx2 = 1000
Þ 9 + x = 42 original price
Þ x = 33 litres 100 × 1000 = ` 100000
So, option (c) is correct.
Ratio and Proportion A-65
62. (c) Let present age of Leela = x years
Leela's age at the time of Marriage = x – 6 M1 D1 M 2 D2 8 ´ 60 5 ´ n
= Þ =
ATQ W1 W2 6400 4800
1 3 n
x = 1 (x - 6) Þ =
4 40 960
3 ´ 960
5 Þn= = 3 ´ 24 = 72
x= (x - 6) 40
4
4x = 5x – 30 Hence, the milkman needs 72 days for maintenance.
x = 30 67. (b) Using the options and the first given condition, we
1 get x – 5 = 3(y – 5)
Child's age = ´ 30 = 3 years.
10 Using the option (a),

3 x - 5 = 3( y - 5) Þ 30 - 5 = 3( y - 5)
63. (b) No. of boys = ´ 49 = 21
7 Þ 25 = 3y – 15 Þ 40
which gives value of y as a fraction.
4
No. of girls = ´ 49 = 28 Using the option (b),
7
ATQ x - 5 = 3( y - 5) Þ 32 - 5 = 3( y - 5)
28 - 4 24 8 Þ 27 = 3(y – 5) Þ 9 = y – 5 Þ y = 14
= = Using the option (c),
21 21 7
64. (c) Let the 2 numbers be x and 5x x - 5 = 3( y - 5) Þ 36 - 5 = 3( y - 5)
ATQ Þ 31 = 3y – 15 Þ 46 = 3y
5x (x) = 320
5x2 = 320 which gives value of y as a fraction.
x2 = 64 Using the option (d),
x= 8 x - 5 = 3( y - 5) Þ 40 - 5 = 3( y - 5)
number are 8 and 40
402 - 82 = (40 + 8) (40 - 8) Þ 35 = 3y – 15 Þ 50 = 3y
48 × 32 = 1536 which also gives the value of y as a fraction.
65. (d) Alloy X Alloy Y Since only option (b) gives the final value of y as an
Total 25 125 integer, therefore option (b) is the correct answer
Ratio of lead 1 : 2 2: 3 68. (c) We are given that the 3 parts are proportional to 1,
and tin
1 1
1 2 , .
Lead ´ 25 ´ 125 3 6
3 5 LCM of the denominators is 6.
2 3 1´ 6 6 1´ 2 2 1
Tin ´ 25 ´ 125 Therefore, the ratio will be = : = : i.e.
3 5 1´ 6 6 3 ´ 2 6 6
When mixed 6 : 2 : 1.
25 50 Sum of the ratio parts is 6 + 2 + 1 i.e. 9.
Lead = + 50 ; Tin = + 75
3 3 2 2 52
Now, the middle part of 78 is ´ 78 = ´ 26 =
175 275 9 3 3
= =
3 3 69. (d) We are given that the ratio of the number of boys in
the first and the second standards is 2 : 3 and the ratio
175 of the number of boys in the second and third stan-
3 = 175 dards is 4 : 5.
Ratio = 275 275
Now, we calculate a common ratio for all the three stan-
3 dards.
= 7 : 11 2 : 3 and 4 : 5 will be 2 × 4 : 3 × 4 = 8 : 12 and 4 ×
3 : 5 × 3 = 12 : 15.
66. (d) If to maintain M1 cows for D1 days a milkman spends Therefore, the common ratio for all the three standards
»W1 and to maintain M2 cows for D2 days, a milkman is 8 : 12 : 15.
M 1D1 M 2 D2 Sum of the ratio parts = 8 + 12 + 15 = 35
spends » W2, then =
W1 W2
EBD_7367
66
A- Ratio and Proportion

8
x + y = 9x – 9y
Number of boys in first standard = ´ 350 = 80 8x = 10y \ x: y= 5 : 4
35
Hence possible values of x and y are 5 and 4
15 respectively
Number of boys in third standard = ´ 350 = 150
35 Required difference = 9 × 5 – 9 × 4 = 9 years
Total number of boys in both standards = 80 + 150 2
= 230. 75. (b) Copper in alloy A = ´ 20 = 8 kg
5
70. (b) Total number of registered students = 2000
Total number of appeared students 3
Tin in alloy A = ´ 20 = 12 kg
1 5
= 2000 - 2000 ´ = 1920
25 3
Total number of passed students Copper in alloy B = ´ 28 = 12 kg
7
11 4
= 1920 ´ = 1056 Tin in alloy B = ´ 28 = 16 kg
20 7
71. (c) A : B = 1 : 2 Let here copper x kg be mixed
B: C= 3 :4 According to the question,
C : D (2 : 3)2 = 4 : 6
D : E = (3 : 4)2 = 6 : 8 8 + 12 + x 6
=
\ B: E=3 :8 12 + 16 7
72. (c) Let share of A be x 20 + x 6
share of B = x – 20 =
28 7
share of C = x + 20
20 + x = 24 \ x = 24 – 20 = 4 kg
According to the question
x + x – 20 + x + 20 = 120 76. (b) Let contribution of D be x
Contribution of A = x × 3 = 3x
120
Þ 3x = 120 \x = = 40 1 3x
3 Contribution of E = 3x ´ =
Hence, share of B = 40 – 20 = 20 2 2
73. (a) Let share of mother be x 3x 1 x
share of each daughter = 2x Contribution of B = ´ =
2 3 2
share of each son = 2x × 3 = 6x
Total amount = 5 × 6x + 4 × 2x + x = 39x 2
Contribution of C = 3x ´ = 2x
According to the question 3
According to the question
39000
39x = 39000 \ x= = 1000
39 æ 3x ö æ x ö
çè 3x + x + ÷ø - çè + 2x ø÷ = 13500
Hence, widow’s share is 1000. 2 2
74. (b) Let unit digit of age of women be x and ten’s digit be y
Age of woman = y × 10 + x × 1 = 10y + x 11x 5x
- = 13500
Age of her husband = x × 10 + y × 1 = 10x + y 2 2
According to the question 6x
= 13500 \ x = 4500
1 2
(10x + y + 10y + x) = 10x + y – (10y + x)
11 Total share of B, C and E
1 4500 3 ´ 4500
(11x + 11y) = 9x – 9y = + 2 ´ 4500 + = 18000
11 2 2
Average A-67

C HA P T E R
AVERAGE
7
1. A batsman scores 80 runs in his sixth innings and thus (a) 6 (b) 7
increases his average by 5. What is his average after six (c) 8 (d) 9
innings?` [2011-I] 10. The mean weight of 150 students in a class is 60 kg. The
(a) 50 (b) 55 mean weight of boys is 70 kg and that of girls is 55 kg,
(c) 60 (d) 65 what is the number of boys in the class ? [2012-II]
2. An aeroplane flies along the four sides of a square at a (a) 50 (b) 60
speed of 100, 200, 300 and 400 km/h, respectively. What (c) 75 (c) 100
is the average speed of the plane in its flight around the 11. The average weight of a class of 15 boys and 10 girls is
square? [2011-I] 38.4 kg. If the average weight of the boys is 40 kg, then
(a) 196 km/h (b) 200 km/h what is the average weight of the girls? [2013-I]
(c) 250 km/h (d) None of these (a) 36.5 kg (b) 35 kg
3. How much tea at ` 9 per kg must be mixed with 100 kg (c) 36 kg (d) 34.6 kg
of superior tea at ` 13.50 per kg to give an average price 12. If the average of A and B is 30, the average of C and D
of ` 11 per kg? [2011-I] is 20, then which of the following is/are correct?
(a) 85 kg (b) 120 kg I. The average of B and C must be greater than 25.
(c) 125 kg (d) 130 kg II. The average of A and D must be less than 25.
4. The population of a state increased from 100 million to 169 Select the correct answer using the codes given below.
million in two decades. What is the average increase in [2014-I]
population per decade? [2011-I] (a) Only I (b) Only II
(a) 20% (b) 34.5% (c) Either I or II (d) Neither I or II
(c) 69% (d) 30% 13. The price of a commodity increased by 5% from 2010 to
5. A person invested part of ` 45000 at 4% and the rest at 2011, 8% from 2011 to 2012 and 77% from 2012 to 2013.
6%. If his annual income from both are equal, then what What is the average price increase (approximate) from
is the average rate of interest? [2011-I] 2010 to 2013? [2014-II]
(a) 4.6% (b) 4.8% (a) 26% (b) 32%
(c) 5.0% (d) 5.2% (c) 24% (d) 30%
6. Nine numbers are written in ascending order. The middle 14. The average of m numbers is n4 and the average of n
number is the average of the nine numbers. The average numbers is m4. The average of (m + n) numbers is
of the first five larger numbers is 68 and that of five [2015-II]
smaller numbers is 44. What is the sum of all nine (a) mn (b) m2 + n2
numbers? [2011-I] (c) mn (m2 + n2) (d) mn(m2 + n2 – mn)
(a) 450 (b) 501 15. The average weight of students in a class is 43 kg. Four new
(c) 504 (d) 540 students are admitted to the class whose weights are 42 kg,
7. In an examination, 40% of the candidates wrote their answers 36.5 kg, 39 kg and 42.5 kg respectively. Now the average
in Hindi and the others in English. The average marks of the weight–of the students of the class is 42.5 kg, The number
candidates written in Hindi is 74 and the average marks of of students in the beginning was [2015-II]
the candidates written in English is 77. What is the average (a) 10 (b) 15
marks of all the candidates ? [2011-I] (c) 20 (b) 25
(a) 75.5 (b) 75.8 16. Four years ago, the average age of A and B was 18 years.
(c) 76.0 (d) 76.8 Now the average age of A, B and C is 24 years. After 8 years,
8. The mean weight of 150 students in a certain class is 60 the age of C will be. [2015-II]
kg. The mean weight of the boys from the class is 70 kg, (a) 32 years (b) 28 years
while that of girls is 55 kg. What is the number of girls (c) 36 years (d) 40 years
in the class ? [2011-I] 17. If the mean age of combined group of boys and girls is 18
(a) 105 (b) 100 years and the mean of age of boys is 20 and that of girls is
(c) 95 (d) 60 16, then what is the percentage of boys in the group ?
9. Out of 250 observations, the first 100 observations have [2016-II]
mean 5 and the average of the remaining 150 observations (a) 60 (b) 50
25 (c) 45 (d) 40
is . What is the average of the whole group of 18. A cricketer has a certain average of 10 innings. In the eleventh
3 inning he scored 108 runs, thereby increasing his average
observations ? [2011-I] by 6 runs. What is his new average? [2016-II]
EBD_7367
A- 68 Average

(a) 42 (b) 47 (a) 70 (b) 65


(c) 48 (d) 60 (c) 60 (d) 50
19. The mean of 20 observations is 17. On checking it was 24. Let a, b, c, d, e, f, g be consecutive even numbers and j, k, l,
found that the two observations were wrongly copied as 3 m, n be consecutive odd numbers. What is the average of
and 6. If wrong observations are replaced by correct values all the numbers ? [2017-I]
8 and 9, then what is the correct mean ? [2016-II]
(a) 17.4 (b) 16.6 3(a + n) (5l + 7d)
(a) (b)
(c) 15.8 (d) 14.2 2 4
20. In a class of 100 students, there are 70 boys whose average
(a + b + m + n)
marks in a subject are 75. If the average marks of the complete (c) (d) None of the above
class is 72, then what is the average marks of the girls ? 4
[2016-II] 25. At present the average of the ages of a father and a son is
(a) 64 (b) 65 25 years. After seven years the son will be 17 years old.
(c) 68 (d) 74 What will be the age of the father after 10 years?
21. Sunil wants to spend ` 200 on two types of sweets, costing [2018-I]
` 7 and ` 10 respectively. What is the maximum number of (a) 44 years (b) 45 years
sweets he can get so that no money is left over ? [2017-I] (c) 50 years (d) 52 years
(a) 25 (b) 26 26. If the average of 9 consecutive positive integers is 55, then
(c) 27 (d) 28 what is the largest integer? [2018-I]
22. The mean of 5 numbers is 15. If one more number is the (a) 57 (b) 58
mean of the 6 numbers becomes 17. What is the included
(c) 59 (d) 60
number? [2017-I]
(a) 24 (b) 25 27. The average of the ages of 15 students in a class is 19
(c) 26 (d) 27 years. When 5 new students are admitted to the class, the
23. The mean marks obtained by 300 students in a subject are average age of the class becomes 18.5 years. What is the
60. The mean of top 100 students was found to be 80 and average age of the 5 newly admitted students? [2018-I]
the mean of last 100 students was found to be 50. The mean (a) 17 years (b) 17.5 years
marks of the remaining 100 students are [2017-I] (c) 18 years (d) 18.5 years

HINTS & SOLUTIONS


1. (b) Let the average of 5 innings = x \ x = 125 kg.
Scores in sixth inning = 80
\ Total of 5 innings = 5x 4. (b) The difference of population in two decades
According to the question, = 169 – 100 = 69 million
5x + 80 \ Increase in population in first decade =
=x+5 69%
6 = 34.5%
Þ 5x + 80 = 6x + 30 2
Þ x = 80 – 30 = 50 5. (b) Let a person invest 4% of x.
\ His average after six innings = 50 + 5 = 55 According to question
Total distance x ´ 4 (45000 - x)
2. (d) Average speed = \ = ´6
Total time 100 100
Þ 2x = 45000 × 3 – 3x
x+ x+ x+ x
= 45000 ´ 3
x
+
x
+
x
+
x Þ x= = ` 27000
5
100 200 300 400 Another part is ` 18000.
4x Let r = Average rate of interest
=
12 x + 6 x + 4 x + 3 x 27000 ´ 4
1200 Interest for 1st part in one year = = ` 1080
100
4 x ´1200 Similarly, interest for rest part in one year = 1080
= = 192 km/h Q Total interest = ` 2160
25x
3. (c) Let x kg of tea of ` 9 per kg. 45000 ´ r
\ = 2160
9 ´ x + 13.5 ´100 100
= 11 216
x + 100 Þ r= = 4.8%
Þ 9x + 1350 = 11x + 1100 45
Þ 2x = 250
Average 69
A-

6. (c) Since, 5th term = average of 9 numbers = x Total weight of girls + Total weight of boys
Sum of first five larger numbers = 68 × 5 = 340 =
Sum of first five smaller numbers = 44 × 5 = 220 No. of boys + No. of girls

340 + 220 - x 600 + 10 ´ x


Average of nine numbers = Þ 38.4 =
9 15 + 10
(since, x is subtracted because 5th term repeated 600 + 10 x
twice) Þ 38.4 =
25
560 - x Þ 38.4 × 25 = 600 + 10x
\ x=
9 \ x = 36 kg
Þ 9x + x = 560 Þ x = 56 12. (d) Average of A and B = 30
\ Sum of 9 numbers = 56 × 9 = 504 A+ B
7. (b) Let total number of candidates in Hindi = 100 Þ = 30 Þ A + B = 60
2
\ Total marks of 40 candidates in English = 40 × 74 Average of C and D = 20
Total marks of 60 candidates = 60 × 77
C+D
40 ´ 74 + 60 ´ 77 Þ = 20 Þ C + D = 40
\ Required average marks = 2
100 Here, we can’t find the avg of B and C, A and D so
2960 + 4620 7580 that Neither I or II are follows.
= = = 75.80
100 100 æ 5 + 8 + 77 ö
8. (b) Let the no. of boys = x 13. (d) Average price increase = ç ÷%
è 3 ø
No. of girls = 150 – x
90
70 x + (150 - x) ´ 55 = % = 30%
Average weight = 3
150
14. (d) Sum of ‘m’ numbers = m.n 4
70 x + (150 - x) ´ 55 Sum of ‘n’ numbers = n.m4
Þ 60 =
150 Sum of ‘m + n’ numbers = mn 4 + nm4
No. of boys (x) = 50
mn 4 + nm 4
No. of girls = 150 – x Average of ‘m + n’ numbers =
= 150 – 50 = 100 (m + n )

9. (b) Given, n1 = 100, x1 = 5 and n2 = 150, x2 =


25
3
(
mn n 3 + m3 )
=
Average of whole group of observations ( m + n)

x1 =
n1 x1 + n2 x2 (
mn (m + n ) m 2 + n 2 - mn )
n1 + n2 =
(m + n )
25 = mn(m2 + n2 – mn)
100 ´ 5 + 150 ´
= 3 = 1750 = 7 So, option (d) is correct.
100 + 150 250 15. (c) Let the no. of students in the beginning be x.
10. (a) Total weight of 150 students According to question-
= 150 × 60 = 9000 kg 43x + 42 + 36.5 + 38 + 42.5
Let total no. of boys = x Þ = 42.5
( x + 4)
No. of girls = 150 – x
x ´ 70 + (150 - x)55 43x +160
Average weight = Þ = 42.5
150
( x + 4)
Þ 43x + 160 = 42.5 x + 170
70 x + 150 ´ 55 - 55 x
Þ 60 = Þ 0.5x = 10
150 Þ x = 20
Þ 60 × 150 = 15x + 150 × 55 So, option (c) is correct.
Þ 15x = 60 × 150 – 150 × 55 16. (c) Let their age be A, B and C respectively.
750 A +B+C
\ x= = 50 Þ = 24
15 3
\ No. of boys in the class = 50 Þ A + B + C = 72 ----------(i)
11. (c) Let average weight of girls = x A - 4 + B- 4
Total weight of the boys = 40 kg × 15 = 600 kg. Þ = 18
2
Average weight
EBD_7367
70
A- Average

A + B-8 22. (d) Mean of 5 number = 15


Þ = 18 Total = 15 × 5 = 75
2
Let number included = 75 + x
A + B = 44 ----------(ii)
ATQ
eq(i) – eq (ii) -
(A + B + C) – (A + B) = 72 – 44 75 + x
= 17
C = 28 years 6
After 8 years age of C x = 102 – 75 = 27
= 28 + 8 = 36 years 23. (d) Mean marks of 300 student = 60
So, option (c) is correct. Total mark = 300 × 60
17. (b) Let number of boys and girls be x and y Mean marks of 100 students = 80
ATQ Mean marks of another 100 students = 50
let Mean mark of remaining 100 student = x
20 x + 16 y
= 18 ATQ
x+y
100x + 100 × 50 + 100 × 80 = 300 × 60
20x + 16y = 18x + 18y 100x = 5000
2x = 2y x = 50
x= y 24. (d) when a, b, c, d, e, f, g are consecutive evenno then no
i.e. boys are 50% of group or
18. (c) Let the cricketer's average be = x run per match d – 6, d – 4, d – 2, d + 2, d + 4, d + 6
ATQ Total = 7d
Similarly,
10x + 108
=x+6 When J, K, L, m and n be consecutive odd no. then
11 numbers are l – 4, l – 2, l, l + 2, l + 4
11x + 66 = 10x + 108 Total = 5l
x = 42 5l + 7d
New average = 42 + 6 = 48 Average =
12
19. (a) Mean of 20 observation = 17 25. (c) Total age of father and son at present = 2 × 25
total sum = 17 × 20 = 340 = 50 years
ATQ 7 years later total age of father and son = 50 + 2 × 7 = 64
years
340 - 3 - 6 + 8 + 9 348
= = 17.4 7 years later age of son = 17 years
20 20
7 years later age of father = 64 – 17 = 47 years
20. (b) Total student = 100, Boy = 70, Girls = 30
Present age of father = 47 – 7 = 40 years
Boys average = 75
age of father 10 years later = 40 + 10 = 50 years
Total marks = 75 × 70 = 5250
26. (c) Let integers be x, x + 1, x + 2, x + 3, x + 4, x + 5, x + 6, x +
Class average = 72 7 and x + 8 respectively
Total class marks = 72 × 100 = 7200 According to the question
7200 - 5250 x + x +1+ x + 2 + x + 3 + x + 4 + x + 5 +
Total marks of girls =
30
x + 6+ x + 7+ x +8
= 55
1950 9
= = 65
30 9x + 36 = 495
21. (b) Let the two types of sweets be x and y 9x = 495 – 36 = 459
ATQ 459
7x + 10y = 200 \ x= = 51
9
\ it is possible when Hence, largest integer = 51 + 8 = 59
7x is multiple of 10 27. (a) Total age of 15 students of the class = 15 × 19 =285
i.e. 70 and 140 ..... Total age of 20 students of the class = 18.5 × 20=370
7 × 10 + 10 × 13 = 200; 10 + 13 = 23 Total age of 5 new students = 370 – 285 = 85 years
7 × 20 + 10 × 6 = 200; 20 + 6 = 26
85
Maximum sweet went x = 20 and y = 6 i.e. Total 26. average age of new students = = 17 years
5
C HA P T E R
SIMPLE AND
8 COMPOUND INTEREST

1. A man borrowed ` 40000 at 8% simple interest per year. 9. An amount of ` x at compound interest at 20% per annum
At the end of second year, he paid back certain amount for 3 years becomes y. What is y : x ? [2008-I]
and at the end of fifth year, he paid back ` 35960 and (a) 3 : 1 (b) 36 : 25
cleared the debt. What is the amount did he pay back (c) 216 : 125 (d) 125 : 216
after the second year? [2007-I] 10. At what rate per cent per annum simple interest, will a sum
(a) ` 16200 (b) ` 17400 of money triple itself in 25 years ? [2008-II]
(c) ` 18600 (d) None of these (a) 8% (b) 9%
(c) 10% (d) 12%
1
2. A sum of ` 24000 is borrowed for 1 years at the rate 11. The compound interest on a sum for 2 years is ` 832 and
2 the simple interest on the same sum at the same rate for
of interest 10% per annum compound semi-annually. the same period is ` 800. What is the rate of interest?
What is the compound interest (x) ? [2007-I] [2009-I]
(a) x < ` 3000 (b) ` 3000 < x < ` 4000 (a) 6% (b) 8%
(c) ` 4000 < x < ` 5000 (d) x > ` 5000 (c) 10% (d) 12%
3. An amount at compound interest doubles itself in 4 years. 12. If the rate of interest is 10% per annum and is compound
In how many years will it become 8 times of itself? half-yearly, then the principle of ` 400 in 3/2 years will
[2007-I] amount to [2009-II]
(a) 8 years (b) 12 years (a) ` 463.00 (b) ` 463.05
(c) 16 years (d) 24 years (c) ` 463.15 (d) ` 463.20
4. At what rate per cent annum calculated in simple interest 13. A person borrowed ` 7500 at 16% compound interest.
will a sum of money double in 10 years ? [2007-I] How much does he have to pay at the end of 2 years to
(a) 10% (b) 2% clear the loan ? [2009-II]
(c) 12.5% (d) 13.5% (a) ` 9900 (b) ` 10092
5. A man invested ` 1000 on a simple interest at a certain rate (c) ` 11000 (d) ` 11052
and ` 1500 at 2% higher rate. The total interest in three 14. The simple interest on a certain sum of money for 3 years
years is ` 390. What is the rate of interest for ` 1000? at 8% per annum is half the compound interest on ` 4000
[2007-II] for 2 years at 10% per annum. What is the sum placed on
(a) 4% (b) 5% simple interest? [2010-I]
(c) 6% (d) 8% (a) ` 1550 (b) ` 1650
6. If P is principal amount and the rate of interest is R% per (c) ` 1750 (d) ` 2000
annum and the compound interest is calculated k times in 15. A person invested some amount at the rate of 12% simple
a year, then what is the amount at the end of n years ? interest and the remaining at 10%. He received yearly an
[2007-II] interest of ` 130. Had he interchanged the amounts
nk nk invested, he would have received an interest of ` 134.
æ r ö æ kr ö How much money did he invest at different rates?
(a) P ç1 + ÷ (b) P ç1 + ÷
è 100k ø è 100 ø [2010-I]
(a) ` 500 at the rate of 10%, ` 800 at the rate of 12%
n n
(b) ` 700 at the rate of 10%, ` 600 at the rate of 12%
æ kr ö k
(c) P æç1 + kr ö÷ k (d) P ç1 + ÷ (c) ` 800 at the rate of 10%, ` 400 at the rate of 12%
è 100 ø è 100k ø (d) ` 700 at the rate of 10%, ` 500 at the rate of 12%
7. Ram had ` 2 lakh, part of which he lent at 15% per annum 16. A sum of money lent on simple interest triples itself in 15
and rest at 12% per annum. Yearly interest accured was years and 6 months. In how many year still it be doubled?
` 27600. How much did he lent at 15%? [2008-I] [2010-II]
(a) ` 120000 (b) ` 100000 (a) 5 years and 3 months
(c) ` 80000 (d) ` 60000 (b) 7 years and 9 months
8. Out of a sum of ` 640, a part was lent at 6% simple interest (c) 8 years and 3 months
and the other at 9% simple interest. If the interest on the (d) 9 years and 6 months
first part after 3 years equal to the interest on the second 17. What is the least number of years in which a sum of
part after 6 years, then what is the second part ? money at 20% compound interest will be more than
[2008-I] doubled ? [2011-I]
(a) ` 120 (b) ` 140 (a) 7 (b) 6
(c) ` 160 (d) ` 180 (c) 5 (d) 4
EBD_7367
72
A- Simple and Compound Interest

18. A sum of money on compound interest amount to ` 9680 1 3


in 2 years and to ` 10648 in 3 years. What is the rate of (a) 1 years (b) 1 years
interest per annum ? [2011-I] 4 4
(a) 5% (b) 10% 1 3
(c) 15% (d) 20% (c) 2 years (d) 2 years
4 4
19. The sum which amounts to ` 364.80 in 8 years at 3.5% 30. An automobile financer claims to be lending money at
simple interest per annum is [2011-II] simple interest, but he includes the interest every six
(a) ` 285 (b) ` 280 months for calculating the principal. If he is charging an
(c) ` 275 (d) ` 270 interest at the rate of 10%, the effective rate of interest
20. A certain sum at simple interest amounts to ` 1350 in 5 becomes [2015-I]
years and to ` 1620 in 8 years. What is the sum? (a) 10.25% (b) 10.5%
[2011-II] (c) 10.75% (d) 11%
(a) ` 700 (b) ` 800 31. A sum of `10,000. is deposited for 1 year at the rate of
(c) ` 900 (d) ` 1000 interest 10% compounded half yearly. What will be the
21. What is the compound interest on ` 1600 at 25% per interest at the end of one year ? [2015-II]
annum of 2 years compounded annually? [2012-I] (a) ` 1000 (b) ` 1025
(a) ` 700 (b) ` 750
(c) ` 1050 (d) ` 1100
(c) ` 800 (d) ` 900
32. If a sum of money at a certain rate of simple interest per year
22. A sum of money becomes 3 times in 5 years. In how many
doubles in 5 years and at a different rate of simple interest
years will the same sum become 6 times at the same rate
per year becomes three times in 12 years, then the difference
of simple interest ? [2012-II]
in the two rates of Simple interest per year is [2016-I]
(a) 15 years (b) 12.5 years
(a) 2% (b) 3%
(c) 10 years (d) 7.5 years
23. The principal on which a simple interest of ` 55 will be 1 1
(c) 3 % (d) 4 %
2 3 3
obtained after 9 months at the rate of 3 % per annum is 33. A sum of ` 8,400 was taken as a loan. This is to be paid in
3
two equal instalments. If the rate of interest is 10% per
[2013-I]
annum, compounded annually, then the value of each
(a) ` 1000 (b) ` 1500
instalment is
(c) ` 2000 (d) ` 2500
24. When an article is sold at 20% discount, the selling price (a) ` 4,200 (b) ` 4,480
is `24. What will be the selling price when the discount (c) ` 4,840 (d) None of the above
is 30% ? [2014-II] 34. The difference between the simple and the compound
(a) ` 25 (b) ` 23 interest on a certain sum of money at 4% per annum in 2
(a) ` 21 (d) ` 20 years is ` 10. What is the sum ?
25. The difference between compound interest and simple (a) ` 5,000 (b) ` 6,000
interest for 2 yr at the rate of 10% over principal amount (c) ` 6,250 (d) ` 7,500
of ` X is `10. What is the value of X ? [2014-II] 35. The difference between the compound interest (compounded
(a) ` 100 (b) ` 1000 annually) and simple interest on a sum of money deposited
(c) ` 500 (d) ` 5000 for 2 years at 5% per annum is ` 15. What is the sum of
26. A sum of money becomes 3 times in 5 yr at simple money deposited?
interest. In how many years, will the same sum become (a) ` 6,000 (b) ` 4,800
6 times at the same rate of simple interest ? [2014-II] (c) ` 3,600 (d) ` 2,400
(a) 10 yr (b) 12 yr 36. A person borrowed `5,000 at 5% rate of interest per annum
(c) 12.5 yr (d) 10.5 yr and immediately lent it at 5.5%. After two years he collected
27. The difference between compound interest and simple the amount and settled his loan. What is the amount gained
interest at the same rate of interest R per cent per annum
by him in this transaction? [2018-I]
on ` 15,000 for 2 years is ` 96.What is the value of R?
(a) `25 (b) `50
[2015-I]
(a) 8 (c) `100 (d) `200
(b) 10 37. A merchant commences with a certain capital and gains
(c) 12 annually at the rate of 25%. At the end of 3 years he has
(d) Cannot be determined due to insufficient data `10,000. What is the original amount that the merchant
28. There is 60% increase in an amount in 6 years at simple invested? [2018-I]
interest. What will be the compound interest on ` 12,000 (a) ` 5,120 (b) ` 5,210
after 3 years at the same rate of interest? [2015-I] (c) ` 5,350 (d) ` 5,500
(a) ` 2,160 (b) ` 3,120 38. The annual income of a person decreases by `64 if the
(c) ` 3,972 (d) ` 6,240 annual rate of interest decreases from 4% to 3.75%. What is
29. In how much time would the simple interest on a principal his original annual income? [2018-I]
amount be 0.125 time the principal amount at 10% per (a) ` 24,000 (b) ` 24,000
annum? [2015-I] (c) ` 25,600 (d) ` 24,600
Simple and Compound Interest A-73

HINTS & SOLUTIONS


1. (b) Total borrowed money = ` 40000 Again, sum of money become 8 times.
Rate of interest = 8% T
æ R ö
40000 ´ 8 ´ 2 Then, 8P = P ç 1 + ÷
The interest for 2 years = = ` 6400 è 100 ø
100
Let he paid ` x at the end of second year. æ R ö
T
Interest will be calculated on ` (40000 – x + 6400). Þ 8 = ç1 + ÷ ...(ii)
è 100 ø
(46400 - x ) ´ 3 ´ 8
Interest for 3 years = æ R ö
100 On putting the value of ç 1 + ÷ in eq. (ii),
è 100 ø
6
=` (46400 - x) Þ 8 = (21/4)T [from Eq. (i)]
25 Þ 8 = 2T/4 Þ 23 = 2T/4
6 On comparing, we get
\ (46400 - x) + 46400 - x = 35960
25 T
3= Þ T = 12 years
6x 4
Þ 11136 – + 46400 – x = 35960
25 4. (a) Let principal be x, then amount = 2x
\ SI = A – P = 2x – x = x
31x
Þ = 21576 P´ R´T
25 Q SI =
100
21576 ´ 25
\ x= = ` 17400 x ´ R ´ 10
31 Þ x=
100
3
2. (b) Given, P = ` 24000, T = years and R% = 10% per \ R = 10%
2 5. (a) Let a man invest ` 1000 at a rate R%
annum According to question,
Semi-annual compounding,
1000 ´ R ´ 3 1500 ´ ( R + 2) ´ 3
ìï æ 2T üï + = 390
R ö 100 100
CI = í çP 1 + ÷ - Pý
ïî è 200 ø ïþ Þ 30R + 45R + 90 = 390
Þ 75R = 300
ì 3 ü Þ R = 4%

ïæ 10 ö 2 ï 6. (a) Given, principal amount = ` P
= 24000 íç 1 + ÷ - 1ý
ïè 200 ø ï r
%
î þ Rate of interest, r =
k
ìï 21 3 üï Time, t = nk
ì 9261 ü
= 24000 íæç ö÷ - 1ý = 24000 í - 1ý nk
20
ïîè ø ïþ î 8000 þ æ r ö
\ A = P ç1 + ÷
è 100k ø
ì 1261 ü 7. (a) Let first part be ` x, then second part be ` (200000 – x)
= 24000 í ý = 3 × 1261 = ` 3783
î 8000 þ According to question,
But CI = x x ´1´15 (200000 - x) ´1´12
\ ` 3000 < x < ` 4000 + = 27600
100 100
3. (b) Let sum of money be P and required time be t years.
Þ 15x – 12x + 2400000 = 2760000
4 Þ 3x = 2760000 – 2400000
æ R ö
2P = P ç1 + ÷ Þ 3x = 360000
è 100 ø \ x = 120000
4 Therefore he lented ` 120000 at 15%.
æ R ö
Þ 2 = ç1 + ÷ 8. (c) Let first part be ` x, then second part ` (640 – x)
è 100 ø According to question,
R x ´ 3´ 6 (640 - x) ´ 6 ´ 9
Þ 21/4 = 1 + ...(i) =
100 100 100
EBD_7367
A- 74 Simple and Compound Interest

Þ x = 1920 – 3x 3
Þ 4x = 1920 T= × 2 = 3 years
Þ x = ` 480 2
\ Second part = ` (640 – 480) = ` 160 10
9. (c) Given, P = ` x, R = 20%, T = 3 years, A = ` y P= = 5%
2
T
æ R ö æ R ö
T
Q A = P ç1 + ÷ Amount, A = P ç1 +
è 100 ø ÷
è 100 ø
3
æ 20 ö æ 5 ö
3
Þ y = x ç1 + ÷ A = 400 ç 1 +
è 100 ø ÷
è 100 ø
3
æ6ö 21 21 21
Þ y = xç ÷ = 400 × ´ ´ = ` 463.05
è5ø 20 20 20
13. (b) Note: By Tricky Formula,
y 216 Equivalent rate of interest.
Þ =
x 125
R1R2 16 ´16
y : x = 216 : 125 R1 + R2 + = 16 + 16 +
10. (a) Let principal amount = P 100 100
As amount = 3P, T = 25 years = 34.56%
\ SI = 3P – P = 2P 7500 ´ 34.56
Interest = = 2592
100 ´ SI 100 ´ 2 P 100
Q Rate = = = 8%
P ´T P ´ 25 Total amount = 7500 + 2592 = ` 10092
11. (b) Given CI = ` 832, SI = ` 800 and T = 2 years 14. (c) Let the principal amount be ` P.
From formula, 1
Given, SI = CI
ìïæ R ö
Tüï 2
CI = P íç 1 + ÷ - 1ý
îïè
100 ø 1é ù
2
ïþ P ´8´3 æ 10 ö
Þ = ê 4000 ç1 + ÷ - 4000 ú
100 2ê è 100 ø úû
ìïæ 2 üï ë
R ö
\ 832 = P íç1 + ÷ - 1ý
ïîè 100 ø ïþ 24P 1é 121 ù
Þ = ê 4000 ´ - 4000 ú
100 2ë 100 û
ïì R2 2 R ïü
Þ 832 = P í1 + + - 1ý 1
ïî 10000 100 ïþ = [ 4840 - 4000]
2
24 P 420 ´ 100
ïì R 2 R ïü
2
Þ = 420 Þ P =
Þ 832 = P í + ý ...(i) 100 24
îï10000 100 þï Þ P = ` 1750
Using SI formula, 15. (d) Let the person invest ` x and y at two different rates
P ´ R ´T 12% and 10% respectively.
SI = P ´ R ´T ö
100 x ´12 ´ 1 y ´10 ´ 1 æ
\ + = 130 çQ SI = ÷
P´R´2 100 100 è 100 ø
40000
Þ 800 = Þ P= ...(ii) Þ 12x + 10y = 13000 ...(i)
100 R After inter changing invested amount.
Now putting the value of P in Eq. (i) then,
y ´ 12 ´1 x ´10 ´1
40000 æ R 2 2R ö + = 134
832 = ç + ÷ 100 100
R çè 10000 100 ÷ø Þ 12y + 10x = 13400 ...(ii)
On solving equations (i) and (ii), we get
Þ 832 = 4R + 800 Þ 4R = 32
x = ` 500 and y = ` 700
32 16. (b) Let initial sum of money be ` P, then A = ` 3P
\ R= = 8%
4 31
3 T = 15 years and 6 months = years
12. (b) Given R = 10%, P = ` 400 and T = years 2
2 \ SI = A – P = ` 2P
Compounding is half-yearly, then, 31 R 2 ´ 2 ´100
Þ P× ´ = 2P Þ R =
2 100 31
Simple and Compound Interest A- 75
Let the sum of money doubled in T1 years. For 5 years:
So, SI = 2P – P = ` P
x´5´r
SI ´100 P ´ 100 ´ 31 1350 – x = ...... (i)
\ T1 = Þ T1 = 100
P´R P ´ 400 For 8 years:
31 x ´8´ r
= = 7 years and 9 months. 1620 – x = ...... (ii)
4 100
17. (d) Let the sum of money = ` P Now, divide equation (ii) by equation (i),
\ Amount = 2P
1620 - x 8
T T Þ =
æ R ö æ 20 ö 1350 - x 5
Þ A = P ç1 + ÷ Þ 2 P = P ç1 + ÷
è 100 ø è 100 ø Both sides subtracted (i), we get
T T 270 3
2P æ6ö æ6ö Þ =
Þ = ç ÷ Þ 2=ç ÷ 1350 - x 5
P 5
è ø è5ø
Þ x = ` 900
Putting T = 4, we get 21. (d) Given, P = ` 1600, R = 25% and n = 2 yr
4
æ6ö 1296 n 2
A = P éê1 +
r ù é 25 ù
ç5÷ = = 2 (approx.) \ ú = 1600 ê1 + úû
è ø 625 ë 100 û ë 100
\ Least number of years = 4
18. (b) Let the rate of interest per annum be R% and 5 5
= 1600 ´ ´ = 2500
principal amount be ` P, then 4 4
Amount in 2 years, \ Compound interest = 2500 – 1600 = ` 900.

æ R ö
2 P ´ R ´T
P ç1 + 22. (b) Using the formula, SI =
÷ = 9680 ...(i) 100
è 100 ø
Amount in 3 years, P ´ R ´5
Þ 2P =
3 100
æ R ö
P ç1 + ÷ = 10648 ...(ii) Þ r = 40%
è 100 ø Let required time be t1 years and rate of interest is
On dividing equation (ii) by equation (i), same.
R 10648 P ´ R ´ T1
Then, 1 + = 5P =
100 9680 100
æ 10648 ö 500 500
Þ R= ç - 1÷ ´ 100 Þ T1 = = = 12.5 years
è 9680 ø R 40
968 23. (c) Let P be the principal amount.
\ R= ´100 = 10%
9680 9
Then, SI = ` 55, time (T) = 9 months = years
19. (a) Given that, T = 8 years, R = 3.5% and A = ` 364.80 12
Let the sum = ` P
2 11
Rate (R) = 3 % = % per annum
æ RT ö 3 3
\ A = P ç1 + ÷
è 100 ø P ´ R ´T
By formula, SI =
æ 8 ö 100
Þ 364.80 = P ç1 + 3.5 ´
è 100 ÷ø SI ´100 55 ´ 100 ´ 3 ´ 12
Þ P= =
æ 35 ´ 8 ö R ´T 11 ´ 9
Þ 364.80 = P ç1 + ÷
è 1000 ø = 5 × 100 × 4 = 2000
\ Principal (P) = ` 2000
3648 æ 128 ö 24. (c) Let CP of article be ` x
Þ = Pç ÷
10 è 100 ø 80
So. SP = x ´ = 24
36480 100
\ P= = ` 285 x = ` 30
128
20. (c) Given, A1 = ` 1350, A2 = ` 1620 When given 30% discount
T1 = 5 years and T2 = 8 years 70
Let principal amount be ` x. SP = 30 ´ = `21
100
EBD_7367
76
A- Simple and Compound Interest

25. (b) Rate of interest from CI Now, for compound interest


xy t
= x+y+ æ r ö
100 A = P ç1 + ÷
è 100 ø
10 ´ 10 3
= 10 + 10 + = 21% æ 10 ö 1331
100 = 12000 ç1 + ÷ = 12000 ´ 1000
Rate of interest for SI = 2 × 10 = 20%
è 100 ø
Difference between rate of interest = 21–20 = 1% = 12 × 1331 = 15972
Compound Interest = 15972 – 12000
p ´ 1% = ` 3972
Diff =
100 29. (a) Let Principal = P
\ Interest = .125 × P
p ´ 1%
Þ 10 = 125 P P
100 = =
p = x = 1000 1000 8
26. (c) Let principal amount P be ` x. P ´ R ´T
Amount, A = ` 3x Now, SI =
100
Time, T = 5 yr
\ SI = 3x – x = 2x P P ´10 ´ T
Þ =
Suppose rate of interest be R. 8 100
PRT 5 1
By formula, SI = T= = 1 years
100 4 4
x´ R´5 10
\ 2x = 30. (a) Interest is calculating on every six month r = = 5%
100 2
So effective rate of interest
2 x ´ 100
Þ 2x × 100 = 5Rx Þ R= r1 r2
5x = r1 + r2 +
100
\ R = 40%
Required amount = 6x 5´5
= 5+5+ = 10.25%
SI = 6x – x = 5x 100
100 ´ SI 100 ´ 5 x
\ Requierd time = = = 12.5 yr
P´ R x ´ 40 10
31. (b) Rart = = 5%
p´r 2
27. (a) Difference= (Q R rate of diff between CI and SI) Time = 1 × 2 = 2 years
100
æ n
15000 ´ R r ÷ö
A = P ççç1 + ÷÷
Þ 96 = è 100 ø
100
æ 2
96 5 ÷ö
R= = .64 A = 10000 ççç1 + ÷÷
150 è 100 ø
By option (a)
Rate of Interest for 2 year (compounded) 21 21
= 10000 × ´
20 20
8´8
=8+8+ = 16.64 = 441 × 25 = 11025
100 Interest = 11025 – 10000
Rate of Interest for SI for two year = 2 × 8 = 16 = ` 1025
Diff = 16.64 – 16 = .64 So, option (b) is correct
28. (c) For simple Interest 32. (c) Let Ist P = x `
60% increase in amount so, that Ist Amount = 2x `
P RT Ist S.I. = A – P = 2x – x = x `.
Þ SI =
100 T = 5 years
60 p P ´ R ´ 6 S.I. ´ 100 x ´100
Þ = R= = = 20%
100 100 P´T x ´5
R = 10% Let 2nd P = x `
Simple and Compound Interest 77
A-

2nd Amount = 3x ` 35. (a) The difference between the compound interest (com-
2nd S.I. = A – P = 3x – x = 2x ` pounded annually) and simple interest on a sum of
T = 12 years 2
money deposited for 2 years at R% p.a. is P æç
R ö
.
S.I ´ 100 2x ´ 100 50 è 100 ÷ø
R= = =
P´T x ´ 12 3 2 2 2
æ R ö æ 5 ö æ 1ö
Difference between two rates Pç ÷ = 15 Þ P ç ÷ = 15 Þ P ç ÷
è 100 ø è 100 ø è 20 ø
50 10 1
= 20 - = =3 % P
3 3 3 = 15 Þ = 15 Þ P = 6000
400
\ Option (c) is correct.
33. (c) P = ` 8400 R = 10% n=2 36. (b) Total interest collected by person
Let installment = x 5000 ´ 2 ´ 5.5
= = 550
x x 100
P= + Total interest given by person
1 2
æ R ö æ R ö
ç1 + ÷ ç1 + ÷ 5000 ´ 2 ´ 5
è 100 ø è 100 ø = = 500
100
x x Profit = 550 – 500 = 50
8400 = + 2
æ 10 ö æ 10 ö 37. (a) Let original amount be x
çè1 + ÷ø ç 1 + ÷
100 è 100 ø 3
æ 25 ö
x ç1 + = 10000
x x è 100 ÷ø
8400 = + 2
æ 110 ö
÷ø æç
110 ö 3
çè ÷ æ 5ö
100 è 100 ø x ç ÷ = 10000
è 4ø
é10 æ 10 ö2 ù 125 10000 ´ 64
8400 = x êê 11 + çè 11 ÷ø úú x´
64
= 10000 \ x=
125
= 5120
ë û
38. (c) Let original amount be x
é110 + 100 ù Annual income at 4% rate of interest
8400 = x ê ú
ë 121 û x ´ 4 ´1 x
=
8400 ´ 121 100 25
x= = 40 × 121 = ` 4840
210 Annual income at 3.75% rate of interest
34. (c) Using formula when time (t) is 2 year x ´ 3.75 ´ 1 3x
2 = =
é R ù 100 80
D= Pê ú According to the question
ë100 û
2 x 3x
æ 4 ö - = 64
10 = P ç ÷ 25 80
è 100 ø
16x - 15x
10 ´100 ´100 = 64
=P 400
16 \ x = 64 × 400 = 25600
P = 6250
EBD_7367
78
A- Profit and Loss

C HA P T E R
PROFIT AND LOSS
9
1. A trader marked a watch 40% above the cost price and (a) ` 400 (b) ` 450
then gave a discount of 10%. He made a net profit of (c) ` 500 (d) ` 600
` 468 after paying a tax of 10% on the gross profit. What 10. A person A sells a table costing ` 2000 to a person B and
is the cost price of the watch? [2007-I] earns a profit of 6%. The person B sells it to another
(a) ` 1200 (b) ` 1800 person C at a loss of 5%. At what price did B sell the
(c) ` 2000 (d) ` 2340 table? [2009-I]
2. By giving 25% discount a trader earns 25% profit. If he (a) ` 2054 (b) ` 2050
sells the item at 10% discount, what is his profit? (c) ` 2024 (d) ` 2014
[2007-I] 11. A trader sells two cycles at ` 1188 each and gains 10%
(a) 10% (b) 40% on the first and loses 10% on the second. What is the
(c) 45% (d) 50% profit or loss per cent on the whole? [2009-II]
3. A man buys 4 tables and 5 chairs for ` 1000. If he sells (a) 1% loss (b) 1% gain
the tables at 10% profit and chairs 20% profit, he earns (c) No loss no gain (d) 2% loss
a profit of ` 120. What is the cost of one table? 12. A milk vendor bought 28 l of milk at the cost of ` 8.50 per l.
[2007-II] After adding some water, he sold the mixture at the same
(a) ` 200 (b) ` 220 price. If he gains 12.5%, how much water did he add?
(c) ` 240 (d) ` 260 [2009-II]
4. In respect of a bill of ` 10000, what is the difference (a) 5.5 l (b) 4.5 l
between a discount of 40% and two successive discounts (c) 3.5 l (d) 2.5 l
of 36% and 4%? [2007-II] 13. One saree was purchased for ` 564 after getting a
(a) ` 0 (b) ` 144 discount of 6% and another saree was purchased for
(c) ` 256 (d) ` 400 ` 396 after getting a discount of 1%. Taking both the
5. A refrigerator and a camera were sold for ` 12000 each. items as a single transaction, what is the percentage of
The refrigerator was sold at a loss of 20% of the cost and discount? [2010-I]
the camera at a gain of 20% of the cost. The entire (a) 3.5 (b) 4
transaction results in which one of the following? (c) 7 (d) 7.5
[2008-I] 14. A man bought a number of oranges at 3 for a rupee and
(a) No loss or gain (b) Loss of ` 1000 an equal number at 2 for a rupee. At what price per dozen
(c) Gain of ` 1000 (d) Loss of ` 2000 should he sell them to make a profit of 20%? [2010-I]
1 1 (a) ` 4 (b) ` 5
6. Successive discounts of 12 % and 7 % are given on (c) ` 6 (d) ` 7
2 2
the marked price of a cupboard. If the customer pays 15. By selling 8 dozen pencils, a shopkeeper gains the selling
` 2590, then what is the marked price? [2008-I] price of 1 dozen pencils. What is the gain? [2010-II]
(a) ` 3108 (b) ` 3148 1 1
(c) ` 3200 (d) ` 3600 (a) 12 % (b) 13 %
2 7
7. The marked price of a machine is ` 18000. By selling it at
a discount of 20%, the loss is 4%. What is the cost price 2 1
of the machine? [2008-II] (c) 14 % (d) 87 %
7 2
(a) ` 10000 (b) ` 12000 16. On selling an article for ` 240, a trader loses 4%. In order
(c) ` 14000 (d) ` 15000 to gain 10%, he must sell the article for [2010-II]
8. A dishonest dealer professes to sell his good at cost price (a) ` 275 (b) ` 280
but uses a false weight and thus gains 20%. For a (c) ` 285 (d) ` 300
kilogram he uses a weight of [2009-I] 17. A trader marks 10% higher than the cost price. He gives a
(a) 700 g (b) 750 g discount of 10% on the marked price. In this kind of sales
(c) 800 g (d) 850 g how much per cent does the trader gain or loss? [2011-I]
9. What the seller marked the printed price of a watch (a) 5% gain (b) 2% gain
purchased at ` 380, so that giving 5% discount, there is (c) 1% loss (d) 3% loss
25% profit? [2009-I]
Profit and Loss A-79
18. A fruit-seller buys lemons at 2 for a rupee and sells them 28. A shopkeeper sells his articles at their cost price but uses
at 5 for three rupees. What is his gain per cent? a faulty balance which reads 1000g for 800g.What is his
[2011-II] actual profit percentage ? [2014-II]
(a) 10% (b) 15% (a) 25% (b) 20%
(c) 20% (d) 25% (c) 40% (d) 30%
19. A man sold two watches, each for ` 495. If he gained 10% 29. A man buys 200 oranges for ` 1000. How many oranges
on one watch and suffered a loss of 10% on the other, for ` 100 can be sold, so that his profit percentage is
then what is the loss or gain percentage in the transaction? 25%? [2014-II]
[2011-II] (a) 10 (b) 14
(a) 1% gain (b) 1% loss
(c) 16 (d) 20
100 30. The value of a single discount on some amount which is
(c) % loss (d) No gain no loss
99 equivalent to a series of discounts of 10%, 20% and 40% on
20. A person sold an article for ` 136 and got 15% loss. Had the same amount, is equal to [2015-II]
he sold it for ` x, he would have got a profit of 15%. (a) 43.2% (b) 50%
Which one of the following is correct? [2012-I] (c) 56.8% (d) 70%
(a) 190 < x < 200 (b) 180 < x < 190 31. A cloth merchant buys cloth from a weaver and cheats him
(c) 170 < x < 180 (d) 160 < x < 170 by using a scale which is 10 cm longer than a normal metre
21. The cost of two articles are in the ratio 3 : 5. If there is scale. He claims to sell cloth at the cost price to his
30% loss on the first article and 20% gain on the second customers, but while selling uses a scale which is 10 cm
article, what is overall percentage of loss or gain? shorter than a normal metre scale. What is his gains?
[2012-I] [2016-I]
(a) 2.25% gain (b) 5.25% loss (a) 20% (b) 21%
(c) 2% loss (d) None of these
22. A person bought 8 quintal of rice for certain rupees. After 2 1
(c) 22 % (d) 23 %
a week, he sold 3 quintal of rice at 10% profit, 3 quintal 9 3
of rice with neither profit nor loss and 2 quintal at 5% 32. The cost of 2.5 kg rice is `125. The cost of 9 kg rice is equal
loss. In this transaction, what is the profit? [2012-I] to that of 4 kg pulses. The cost of 14 kg pulses is equal to
(a) 10% (b) 20% that of 1.5 kg tea. The cost of 2 kg tea is equal to that of 5 kg
(c) 25% (d) None of these
nuts. What is the cost of 11 kg nuts ? [2016-II]
23. A man buys a television set which lists for ` 5000 at 10%
(a) `2310 (b) `3190
discount. He gets an additional 2% discount (after the
first discount) for paying cash. What does he actually (c) `4070 (d) `4620
pay for the set? [2012-II] 33. A shopkeeper increases the cost price of an item by 20%
(a) ` 4410 (b) ` 4400 and offers a discount of 10% on this marked price. What is
(c) ` 4000 (d) ` 4500 his percentage gain ? [2016-II]
24. A merchant earns a profit of 20% by selling a basket (a) 15% (b) 12%
containing 80 apples which cost is ` 240 but he gives (c) 10% (d) 8%
one-fourth of it to his friend at cost price and sells the 34. Rajendra bought a mobile with 25% discount on the selling
remaining apples. In order to earn the same profit, at what price. If the mobile cost him ` 4,875, what is the original
price must he sell each apple? [2012-II] selling price of the mobile ? [2017-I]
(a) ` 3.00 (b) ` 3.60 (a) ` 6,300 (b) ` 6,400
(c) ` 3.80 (d) ` 4.80 (c) ` 6,500 (d) ` 6,600
25. A cloth store is offering buy 3, get 1 free. What is the net 35. Sudhir purchased a chair with three consecutive discounts
percentage discount being offered by the store?
of 20%, 12.5% and 5%. The actual deduction will be
[2012-II]
(a) 20% (b) 25% [2017-II]
(a) 33.5% (b) 30%
1 (c) 32% (d) 35%
(c) 30% (d) 33 %
3 36. Ram buys 4 chairs and 9 stools for ` 1,340. If he sells chairs
26. Two lots of onions with equal quantity, one costing ` 10 at 10% profit and stools at 20% profit, he earns a total profit
per kg and the other costing `15 per kg, are mixed of ` 188. How much money did he have to pay for the
together and whole lot is sold at ` 15 per kg. What is the
chairs? [2017-II]
profit or loss? [2013-II]
(a) 10% loss (b) 10% profit (a) ` 200 (b) ` 400
(c) 20% profit (d) 20% loss (c) ` 800 (d) ` 1,600
27. A person selling an article for ` 96 finds that his loss per 37. An article is sold at a profit at 32%. If the cost price is
cent is one — fourth of the amount of rupees that he paid increased by 20% and the sale price remains the same, then
for the article. What can be the cost price? [2014-II] the profit percentage becomes [2018-I]
(a) Only `160 (b) Only ` 240 (a) 10% (b) 12%
(c) Either ` 160 or ` 240 (d) Neither ` 160 nor ` 240 (c) 15% (d) 20%
EBD_7367
A- 80 Profit and Loss

HINTS & SOLUTIONS


1. (c) Let the cost price of the watch = ` x 4. (b) Two successive discounts
After 40% marked price and 10% discount 36 ´ 4
90 140 126x = 36 + 4 – = 38.56%
´ = 100
=x×
100 100 100 \ Difference between discounts
= 40% – 38.56% = 1.44%
126x 26x
Profit = -x = \ Required difference = 10000 × 1.44%
100 100
According to question, 10000 ´ 1.44
= = ` 144
10% pay tax on profit 100
26x 90 xy
= ´ = 468 5. (b) x + y +
100 100 100
468 ´ 100 ´ 100 20 ´ 20
x= = ` 2000 = +20 – 20 – = –4%
26 ´ 90 100
Total selling price of a refrigerator and a camera
2. (d) Let cost price = ` x, = 12000 + 12000 = ` 24000
Marked price = ` y Now, loss is 4%
and profit % at 10% discount = r % 96
CP × = 24000
75 y 125 x 100
\ = Þ 3 y = 5x ...(i) CP = ` 25000
100 100
Loss amount = (25000 – 24000) = ` 1000
9 y (100 + r ) x 6. (c) Let the marked price of a cupboard = ` x
And = According to question,
10 100
(100 - 12.5) æ 100 - 7.5 ö
15x (100 + r ) x \ x× ´ç ÷ = ` 2590
Þ = [Q3 y = 5x] 100 è 100 ø
10 100
2590 ´ 100 ´ 100
100 + r Þ x= 87.5 ´ 92.5
Þ 15 =
10 x = ` 3200
Þ r = 50% 7. (d) Given marked price of machine = ` 18000
Note that discount is always given on marked price and 20
\ Discount = × 18000 = ` 3600
profit is always occured on cost price. 100
3. (a) Let cost of 1 table be ` x and cost of 1 chair be ` y. \ SP = 18000 – 3600 = ` 14400
4x + 5y = 1000 ... (i) If loss of 4%, then
Table Chair 100 ´ SP 100 ´ 14400
CP 4x 5y CP = 100 - r = 100 - 4
æ 1 ö 44x æ 1ö
SP 4x ç1 + ÷ = 5y ç1 + ÷ = 6y 100 ´ 14400
è 10 ø 10 è 5ø = = ` 15000
96
Q SP – CP = Profit
8. (c) Let SP = CP = ` x
æ 44x ö
\ ç - 4x ÷ + 6y – 5y = 120 20 x
è 10 ø Gain percentage = x × =
100 5
4x
Þ + y = 120 ... (ii) x
10 Gain weight = × 1000 = 200 g
5
From equations (i) and (ii),
Uses weight = 1000 – 200 = 800 g
x = ` 200
Profit and Loss 81
A-

9. (a) Let marked price be ` x.


5
95 19 x Cost of 1 orange after mixing =
12
\ Discount = 5% of x = ´x=
100 20
5 120 1
According to question, Profit of 20% = ´ =
12 100 2
19 x
Þ 380 =
20 1
Selling price of 1 orange = `
\ x = ` 400. 2
10. (d) The cost price of table for person B 1
\ Then, SP of 12 oranges = × 12 = ` 6
2000 2
= 2000 + 6 ×
100 15. (c) Let the cost price = ` x
= 2000 + 120 = ` 2120 Profit = ` x
Cost price of 8 dozen pencil = ` 7x
2120 ´ 5
Selling price = 2120 – x
100
Gain per cent = × 100
= 2120 – 106 = ` 2014 7x

æ xy ö 100 2
= = 14 %
11. (a) Profit/loss = ç x + y + ÷% 7 7
è 100 ø
Given, x = +10%, y = –10% 16. (a) Q Selling price of an article = ` 240
\ Cost price of an article for the loss of 4%
æ 10 ´ 10 ö
\ Profit/loss = ç 10 - 10 - ÷ % = –1% 240 ´100
è 100 ø =`
96
(– sign represent, there is a loss of 1%.)
\ Selling price of an article for a profit of 10%
12. (c) Total C.P. of milk = 28 × 8.50 = ` 238
Profit = 12.5% of 238 240 ´100 110 240 ´110
= ´ = = ` 275
96 100 96
12.5
= × 238 = 29.75 17. (c) Let cost price = ` x
100
Let he added x l of water. x ´ 110 11x
Marked price = =`
\ Profit = x × 8.5 100 10
Þ 29.75 = x × 8.5
11x 90 99x
\ x = 3.5 l \ SP = ´ =
13. (b) Let marked price of two sarees be ` x and ` y 10 100 100
respectively 99x
-x
6x 94x 100
\ x- = 564 Þ = 564 \ Loss per cent = × 100 = –1%
100 100 x
Þ x = ` 600 (– sign show loss.)
y
and y – = 396 18. (c) ` 1 = 2 lemons × CP
100
5
99y `1 = lemons × SP
Þ = 396 Þ y = ` 400 3
100
\ Total M.P. amount = 600 + 400 = ` 1000 5
Total amount after discount = 564 + 396 = ` 960 Þ 2 × CP = × SP
3
1000 - 960 Þ 6 CP = 5 SP
\ Discount per cent = ´ 100
1000 SP 6
Þ = (Here SP > CP, then profit)
40 CP 5
= % = 4%
10 Both sides substract 1
1 SP - CP 1
14. (c) Cost of 1 orange of 1st variety = ` =
3 CP 5
1 1
Cost of 1 orange of 2nd variety = ` Profit per cent = × 100 = 20%
2 5
EBD_7367
82
A- Profit and Loss

æ 10 ´ 10 ö 82x
19. (b) Loss/gain per cent = ç 10 - 10 - ÷ % = –1% SP - CP -x
è 100 ø
\ Profit = × 100 = 80 × 100
(- sign indicate that there is a loss of 1%) CP x

Selling price 136 (82 - 80)x 2


20. (b) Cost Price = = = × 100 = × 100 = 2.5%
Loss% 15 80x 80
1- 1-
100 100 23. (a) Actual payment for the television set
= 98% of 90% of 5000 = ` 4410
136 ´100
= = ` 160 240
85 24. (c) Cost price of 1 apple = =`3
80
160 ´ (100 + 15) 160 ´115
Selling price (x) = = 1
100 100 No. of apples give to his friend = × 80 = 20
= ` 184 4
\ Hence, option (b) is correct because 180 < x < 190. Remaining apples = 80 – 20 = 60
21. (d) Let the CP of two articles be 3x and 5x, respectively. Cost of apples to his friend = 20 × 3 = ` 60
3x ´ 70 21x 120
\ SP of first article = = Total SP at a profit of 20% = 240 × = ` 288
100 10 100
SP of remaining 60 apples = ` (288 – 60) = ` 228
5x ´ 120
SP of second article = = 6x 228
100 SP of 1 apple = = ` 3.80
60
21x 60x + 21x 81x
\ Total SP = 6x + = = 25. (b) We know that,
10 10 10
Total CP = 3x + 5x = 8x Discount
\ Net percentage discount = × 100
Cost price
81x 81x - 80x x
\ Profit = – 8x = =
10 10 10 1
= × 100 = 25%
4
x
´ 100 26. (c) Let each lot of onion contains x kg onion, then total
10
\ Overall percentage of gain = cost price of these two lots together
8x
= 10x + 15x = 25x
x ´ 100 Selling price of whole lot = 15 × (x + x)
= = 1.25% = 15 × 2x = 30x
10 ´ 8x
22. (d) Let CP of 8 quintal rice = ` x 30x - 25x
Profit percentage = × 100
x 25x
\ CP of 1 quintal rice = `
8 5x
= × 100 = 20%
3x 3x 1 25x
\ SP of rice 3 quintal of rice at 10% profit = + ´ 27. (c) Let the cost price of an article = ` x
8 8 10
Selling price of an article = ` 96
3x 3x 33x According to the question,
= + =
8 80 80 x - 96 1
´ 100 = ´ x Þ 400x – 96 × 400 = x2
3x x 4
SP of 3 quital rice without profit or loss = ` 2
8 Þ x – 400x + 38400 = 0
Þ x2 – 160x – 240x + 38400 = 0
2x 2x 5
SP of 2 quintal rice at 5% loss = - ´ Þ x(x – 160x) – 240(x – 160) = 0
8 8 100 Þ (x – 160) (x – 240) = 0
x x 19x 19x \ x = 160 or 240
= - = = Hence, the cost price of an article is ` 160 or ` 240.
4 4 ´ 20 4 ´ 20 80
28. (a) Actual profit percentage
33x 3x 19x
\ Total SP = + + Fair weight – Unfair weight
80 8 80 = × 100
Unfair weight
33x + 30x + 19x 82x
= =
80 80 1000 - 800 200
= ´ 100 = ´ 100 = 25%
800 800
Profit and Loss 83
A-

29. (c) Cost price of 200 oranges = `1000.


1000 33. (d) Let CP = 100
Cost price of 1 orange = = ` 5. MP = 120
200 SP after giving discount = 108
125 Profit = 8%
Selling price of 1 orange = 5 ´ = ` 6.25 34. (c) Let SP of Mobile = x
100
discount = 25%
So, in ` 6.25, number of oranges can be sold = 1
In ` 100, number of oranges can be sold 75x
= 4875
1 100
= ´ 100 = 16
6.25 4875 ´ 100
x= = ` 6500
Hence, 16 oranges can be sold in ` 100 for profit 75
25% 35. (a) Considerthe two discounts (taking two at a time) 20%
30. (c) Discount equivalent to 10% and 20% and 12.5%
10 ´ 20 Single equivalent discount =
= 10 + 20 – = 28%
100 æ xy ö æ 20 ´ 12.5 ö
Discount equivalent to 28% and 40% çè x + y - ÷ø % = èç 20 + 12.5 - ÷%
100 100 ø
28´ 40 = (32.5 – 2.5)% = 30%
= 28 + 40 –
100 Considering 30% and 5% as the two discounts
= 68 – 11.2 Final reduction =
= 56.8%
So, option (c) is correct. æ xy ö æ 30 ´ 5 ö
çè x + y - ÷ø % = çè 30 + 5 - ÷%
31. (c) Let the actual C.P. of cloth = `1 100 100 ø
100 10 = (35 – 1.5)% = 33.5%
Then the effective C.P. = = 36. (c) Let the number of chairs be x and the number of stools
110 11
be y.
(Since he purchases 110 articles by paying ` 100)
Therefore, according to the question,
100 10 4x + 9y = 1340 ...(1)
Again S.P. = =
90 9 10% of 4x + 20% of 9y = 188
(Since he sells only 90 articles charging the CP of 100 40 x 180 y 4 x 18 y
articles). + = 188 Þ + = 188
100 100 10 10
10 10 Þ 4x + 18y = 1880 ...(2)
-
S.P - C.P Solving the equations (1) and (2) by elimination
\ Gain% = ´ 100 = 9 11 ´ 100
C.P 10 method, we get
11 – 9y = – 540 Þ y = 60
Therefore, equation (1) gives
20 11 200 2
= ´ ´ 100 = = 22 % 4 x = 1340 - 9 y Þ 4 x = 1340 - 9 ´ 60
99 10 9 9
Þ 4x = 1340 – 540 Þ 4x = 800
\ Option (c) is correct.
Thus, the money to be paid for the chairs is `800.
32. (d) cost of 2.5 kg rice = ` 125
37. (a) Let original cost price of artical be 100
125 Original selling price of artical
cost 9 kg rice = ` × 9 = cost of 4 kg pulse
2.5
32
125 ´ 9 = 100 + 100 ´ = 132
cost of 14 kg pulses = ´ 14 = cost of 1.5 kg Tea 100
2.5 ´ 4 Now
125 ´ 9 ´ 14 ´ 2 New cost price after increase
cost of 2 kg tea = = cost of 5 kg nuts
2.5 ´ 4 ´ 1.5 20
= 100 + 100 ´ = 120
125 ´ 9 ´14 ´ 2 ´11 100
cost of 11 kg nuts = ` New selling price = 132
2.5 ´ 4 ´ 1.5 ´ 5
Profit = 132 – 120 = 12
125 ´ 9 ´ 14 ´ 2 ´ 11´ 100
Þ 12
25 ´ 4 ´15 ´ 5 Required % = ´ 100 = 10%
Þ `4620. 120
EBD_7367
84
A- Time, Speed and Distance

C HA P T E R
TIME, SPEED AND DISTANCE
10
1. A scooterist completes a certain journey in 10 h. He 9. Two persons P and Q start at the same time from city A
covers half the distance at 30 km/h and the rest at 70 km/ for city B, 60 km away. P travels 4 km/h slower than Q. Q
h. What is total distance of the journey ? [2007-I] reaches city B and at once turns back meeting P, 12 km
(a) 210 km (b) 400 km from city B. What is the speed of P ? [2008-II]
(c) 420 km (d) 500 km (a) 8 km/h (b) 12 km/h
2. A train 200 m long fully passes a platform of 200 m length (c) 16 km/h (d) 20 km/h
in 15 s. What is the speed of the train ? [2007-II] 10. Two men P and Q start from a place walking at 5 km/h and
(a) 36 km/h (b) 48 km/h 6.5 km/h, respectively. What is the time they will take to
(c) 72 km/h (d) 96 km/h be 92 km apart, if they walk in opposite directions ?
3. A person goes from a place A to another place B at the [2009-II]
speed of 4 km/h and returns at a speed of 3 km/h. If he (a) 2 h (b) 4 h
takes 7 h in all, then what is the distance between the two (c) 6 h (d) 8 h
places ? [2007-II] 11. Two trains travel in the same direction at 50 km/h and 32
(a) 12 km (b) 8 km km/h, respectively. A man in the slower train observes
(c) 6 km (d) 5 km that 15 s elapse before the faster train completely passes
4. A bike travels a distance of 200 km at a constant speed. him. What is the length of the faster train ? [2009-II]
If the speed of the bike is increased by 5 km/h, the (a) 75 m (b) 125 m
journey would have taken 2 h less. What is the speed of
625
the bike ? [2008-I] (c) 150 m (d) m
3
(a) 30 km/h (b) 25 km/h
(c) 20 km/h (d) 15 km/h 12. The speed of a boat in still water is 11 km/h. It can go 12
5. A train crosses a telegraph post in 8s and a bridge 200 m km upstream and return downstream to the initial point in
long in 24 s. What is the length of the train ? 2 h 45 min. What is the speed of stream? [2010-I]
[2008-II] (a) 5 km/h (b) 4 km/h
(a) 100 m (b) 120 m (c) 3 km/h (d) 2 km/h
(c) 140 m (d) 160 m 13. A father and his son start a point A with speeds of 12
6. A train 110 m long is running with a speed of 60 km/h. km/h and 18 km/h respectively and reach another point B.
What is the time in which it will pass a man who starts If his son starts 60 min after his father at A and reaches B,
from the engine running at the speed of 6 km/h in the 60 min before his father, what is the distance between A
direction opposite to that of the train ? [2008-II] and B ? [2010-II]
(a) 5 s (b) 6 s (a) 90 km (b) 72 km
(c) 10 s (d) 15 s (c) 36 km (d) None of these
7. A train of length 150 m takes 10 s to cross another train 14. Two trains of lengths 100 m and 150 m are travelling in
100 m long coming from the opposite direction. If the opposite directions at speeds of 75 km/h and 50 km/h,
speed of first train is 30 km/h. What is the speed of respectively. What is the time taken by them to cross each
second train ? [2008-II] other ? [2010-II]
(a) 72 km/h (b) 60 km/h (a) 7.4 s (b) 7.2 s
(c) 54 km/h (d) 48 km/h (c) 7 s (d) 6.8 s
8. A boy walks from his house to school at 2.5 km/h and 15. A motorboat takes 2 h to travel a distance of 9 km down
arrives 12 min late. The next day he walks at 4 km/h and the current and it takes 6 h to travel the same distance
reaches the school 15 min earlier. What is the distance against the current. What is the speed of the boat in still
from his house to school ? [2008-II] water ? [2010-II]
(a) 2 km (b) 2.5 km (a) 3 (b) 2
(c) 3 km (d) 3.5 km (c) 1.5 (d) 1
Time, Speed and Distance A-85

16. AB is a straight line. C is a point whose perpendicular 25. A person travels a certain distance at 3 km/h and reaches
distance from AB is 3 cm. What are the number of points 15 min late. If he travels at 4 km/h, he reaches 15 min
which are at a perpendicular distance of 1 cm from AB and earlier. The distance he has to travel is [2013-I]
at a distance 4 cm from C? (a) 4.5 km (b) 6 km
(a) 1 (b) 2 (c) 7.2 km (d) 12 km
26. A sailor sails a distance of 48 km along the flow of a river
(c) 3 (d) 4 [2011-I] in 8 h. If it takes 12 h return the same distance, then the
speed of the flow of the river is [2013-I]
17. Two trains each 200 m long move towards each other on (a) 0.5 km/h (b) 1 km/h
parallel lines with velocities 20 km/h and 30 km/h, (c) 1.5 km/h (d) 2 km/h
respectively. What is the time that elapses when they first 27. If a body cover a distance at the rate of x km/h and
meet until they have cleared each other ? [2011-I] another equal distance at the rate of y km/h, then the
(a) 20 s (b) 24.8 s average speed (in km/h) is [2013-I]
(c) 28.8 s (d) 30 s x y
18. A man can walk uphill at the rate of 2.5 km/h and downhill (a) (b) xy
2
at the rate of 3.25 km/h. If the total time required to walk
a certain distance up the hill and return to the starting 2 xy x y
position is 4 h 36 min, what is the distance he walked up (c) (d)
x y xy
the hill ? [2011-I]
28. A man cycles with a speed of 10 km/h and reaches his
(a) 3.5 km (b) 4.5 km
office at 1 p.m. However, when he cycles with a speed of
(c) 5.5 km (d) 6.5 km
15 km/h, he reaches his office at 11 am. At what speed
19. A train 280 m long is moving at a speed of 60 km/h. What sould he cycle, so that he reaces his office at 12 noon?
is the time taken by the train to cross a platform 220 m [2013-II]
long? [2012-I] (a) 12.5 km/h (b) 12 km/h
(a) 45 s (b) 40 s (c) 13 km/h (d) 13.5 km/h
(c) 35 s (d) 30 s 29. Two cars A and B start simultaneously from a certain
20. A student moves 2x km East from his residence and place at the speed of 30 km/h and 45 km/hr, respectively.
then moves x km North. He, then goes x km North-East The car B reaches the distination 2 h earlier than A. What
and finally he takes a turn of 90° towards right and moves is the distance between the starting point and destination?
a distance x km and reaches his school. What is the [2013-II]
shortest distance of the school from his residence ? (a) 90 km (b) 180 km
[2012-I] (c) 270 km (d) 360 km
30. A train running at the speed of 72 km/h goes past a pole
(a) (2 2 1) x km (b) 3x km in 15 s. What is the length of the train ? [2013-II]
(a) 150 m (b) 200 m
(c) 2 2x km (d) 3 2x km
(c) 300 m (d) 350 m
21. A car travels along the four sides of a square at speeds 31. A train takes 9 s to cross a pole. If the speed of the train
v, 2v, 3v and 4v, respectively. If u is the average speed is 48 km/h, the length of the train is [2014-I]
of the car in its travel around the square, then which one (a) 150 m (b) 120 m
of the following is correct ? [2012-I] (c) 90 m (d) 80 m
(a) u = 2.25 v (b) u = 3 v 32. The distance between two points (A and B) is 110 km. X
(c) v < u < 2 v (d) 3 v < u < 4v starts running from point A at a speed of 60 km/h and Y
22. A wheel of radius 2.1 m of a vehicle makes 75 revolutions starts running from point B at a speed of 40 km/h at the
in 1 min. What is the speed of the vehicle ? [2012-I] same time. They meet at a point C, somewhere on the line
(a) 78 km/h (b) 59.4 km/h AB. What is the ratio of AC to BC ? [2014-II]
(c) 37.4 km/h (d) 35.4 km/h (a) 3 : 2 (b) 2 : 3
23. A car is travelling at a constant rate of 45 km/h. The
(c) 3 : 4 (d) 4 : 3
distance travelled by car from 10 : 40 am to 1 : 00 pm is
[2012-II] 33. A man rides one–third of the distance from A to B at the
(a) 165 km (b) 150 km rate of x km/h and the remainder at the rate of 2y km/h.
(c) 120 km (d) 105 km If he had travelled at a uniform rate of 6z km/h, then he
24. A train takes 10 s to cross a pole and 20 s to cross a could have ridden from A to B and back again in the same
platform of length 200 m. What is the length of the train? time. Which one of the following is correct ? [2014-II]
[2012-II] (a) z = x + y (b) 3z = x + y
(a) 400 m (b) 300 m
1 1 1 1 1 1
(c) 200 m (d) 100 m (c) (d)
z x y 2z x y
EBD_7367
86
A- Time, Speed and Distance

34. A train travels at a speed of 40 km/h and another train at The thief and the policeman run at the speed of 10 km/hr
a speed of 20 m/s. What is the ratio of speed of the first and 11 km/hr respectively. What is the distance between
train to that of the second train ? [2014-II] them after 6 minutes ? [2015-I]
(a) 2 : 1 (b) 5 : 9 (a) 100 m (b) 120 m
(c) 5 : 3 (d) 9 : 5
(c) 150 m (d) 160 m
35. A man rows downstream 32 km and 14 km upstream, and
he takes 6 hours to cover each distance. What is the 42. Two trains are moving in the same direction at 1.5 km/minute
speed of the current? [2015-I] and 60 km/ hour respectively. A man in the faster train
(a) 0.5 km/hr (b) 1 km/hr observes that it takes 27 seconds to cross the slower train.
(c) 1.5 km/hr (d) 2 km/hr The length of the slower train is [2015-II]
36. A car travels the first one-third of a certain distance with (a) 225 m (b) 230m
a speed of 10 km/hr, the next one-third distance with a (c) 240m (d) 250 m
speed of 20 km/hr and the last one-third distance with a 43. In a race A, B and C take part. A beats B by 30 m, B beats C
speed of 60 km/hr. The average speed of the car for the by 20 m and A beats C by 48 m. Which of the following is/
whole journey is [2015-I] are correct ? [2015-II]
(a) 18 km/hr (b) 24 km/hr l. The length of the race is 300 m.
(c) 30 km/hr (d) 36 km/hr 2. The speeds of A, B and C are in the ratio 50 :45 :42.
37. Two persons A and B start simultaneously from two Select the correct answer using the code given below :
places c km apart, and walk in the same direction. If A (a) 1 only (b) 2 only
travels at the rate of p km/hr and B travels at the rate of
(c) Both 1 and 2 (d) Neither 1 nor 2
q km/hr, then A has travelled before he overtakes B a
distance of [2015-I] 44. A motor boat, whose speed is 15 km/ hour in still water goes
30 km down–stream and comes back in a total of 4 hour and
qc pc 30 minutes. The speed of the stream is [2015-II]
(a) km (b) km
p q p–q (a) 4 km/ hour (b) 5 km/hour
(c) 6 km/ hour (d) 10 km/hour
qc pc
(c) km (d) km 45. By increasing the speed of his car by 15 km/hour, a person
p–q p q
covers 300 km distance by taking an hour less than before.
38. In a flight of 600 km, an aircraft was slowed down due to The original speed of the car was [2015-II]
bad weather. Its average speed for the trip was reduced (a) 45 km/ hour (b) 50 km/hour
by 200 km/hr and the time of flight increased by 30 (c) 60 km/hour (d) 75 km/ hour
minutes. The duration of the flight is [2015-I] 46. Two trains, one is of 121 m in length at the speed of 40 km/
(a) 1 hour (b) 2 hours hour and the other is of 99 m in length at the speed of 32 km/
(c) 3 hours (d) 4 hours hour are running in opposite directions. In how much time
39. With a uniform speed, a car covers a distance in 8 hours. will they be completely clear from each other from the
Had the speed been increased by 4 km/hr, the same moment they meet? [2015-II]
distance could have been covered in 7 hours and 30 (a) 10 s (b) 11 s
minutes. What is the distance covered? [2015-I] (c) 16 s (d) 21 s
(a) 420 km (b) 480 km 47. Three athletes run a 4 km race. Their speeds are in the ratio
(c) 520 km (d) 640 km 16 : l5 : 11. When the winner wins the race, then the distance
between the athlete in the second position to the athlete in
2 the third position is [2015-II]
40. A runs 1 times as fast as B. If A gives B a start of 80
3 (a) l000m (b) 800m
m, how far must the winning post from the starting point (c) 750 m (d) 600 m
be so that A and B might reach it at the same time ? 48. In a race of 100 m, A beats B by 4m and A beats C by 2 m. By
[2015-I] how many metres (approximately) would C beat B in another
(a) 200 m (b) 300 m 100 m race assuming C and B run with their respective
(c) 270 m (d) 160 m speeds as in the earlier race ? [2015-II]
41. A thief is noticed by a policeman from a distance of 200 (a) 2 (b) 2.04
m. The thief starts running and the policeman chases him. (c) 2.08 (d) 3.2
Time, Speed and Distance A-87

49. The speeds of three buses are in the ratio 2 : 3 : 4. The time 59. The speeds of three cars are in the ratio 2: 3 :4. What is the
taken by these buses to travel the same distance will be in ratio between the times taken by these cars to travel the
the ratio [2015-II] same distance ? [2016-II]
(a) 2 : 3 : 4 (b) 4 : 3 : 2 (a) 4 : 3 : 2 (b) 2 : 3 : 4
(c) 4 : 3 : 6 (d) 6 : 4 : 3 (c) 4 : 3 : 6 (d) 6 : 4 : 3
50. A man walking at 5 km/hour noticed that a 225 m long train 60. The wheels of a car are of diameter 80cm each. The car is
coming in the opposite direction crossed him in 9 seconds. travelling at a speed of 66 Km/hour. What is the number of
The speed of the train is [2016-I] complete revolutions each wheel makes in 10 minutes ?
(a) 75 km/hour (b) 80 km/hour [2016-II]
(c) 85 km/hour (d) 90 km/hour (a) 4275 (b) 4350
51. A cyclist moves non-stop from A to B, a distance of 14 km, at (c) 4375 (d) 4450
a certain average speed. If his average speed reduces by 1 61. A motorist travels to a place 150 km away at an average
km per hour, he takes 20 minutes more to cover the same speed of 50 km/hour and returns at 30 km/hour. What is the
distance. The original average speed of the cyclist is [2016-I] average speed for the whole journey ? [2016-II]
(a) 5 km/hour (b) 6 km/hour (a) 35 km/hour (b) 37 km/hour
(c) 7 km/hour (d) None of the above (c) 37.5 km/hour (d) 40 km/hour
52. A bike consumes 20 mL of petrol per kilometre, if it is diriven
at a speed in the range of 25 – 50 km/hour and consume 40 5
62. In a 100 m race, A runs at a speed of m/s. If A gives a start
mL of petrol per kilometer at any other speed. How much 3
petrol is consumed by the bike in travelling distance of 50 of 4 m to B and still beats him by 12 seconds, What is the
km, if the bike is driven at a speed of 40 km/hour for the first speed of B ? [2017-I]
10 km, at a speed of 60 km/hour for the next 30 km and at a
speed of 30 km/hour for the last 10 km? [2016-I] 5 7
(a) m/s (b) m/s
(a) 1 L (b) 1.2 L 4 5
(c) 1.4 L (d) 1.6 L
4 6
53. A passenger train takes 1 hour less for a journey of 120 km, (c) m/s (d) m/s
if its speed is increased by 10 km/hour from its usual speed. 3 5
What is its usual speed? [2016-I] 63. A passenger train departs from Delhi at 6 pm, for Mumbai.
(a) 50 km/hour (b) 40 km/hour At 9 p.m., an express train, whose average speed exceeds
(c) 35 km/hour (d) 30 km/hour that of the passenger train by 15 km/hour leaves Mumbai
54. In a race of 1000 m. A beats B by 100 m or 10 seconds. If for Delhi. Two trains meet each other mid-route. At what
they start a race of 1000 m simultaneously from the same time do they meet, given that the distance between the
point and if B gets injured after running 50 m less than half cities is 1080 km ? [2017-I]
the race length and due to which his speed gets haived, (a) 4 pm. (b) 2 am.
then by how much time will A beat B? [2016-I] (c) 12 midnight (d) 6 am
(a) 65 seconds (b) 60 seconds 64. A 225 m long train is running at a speed of 30 km/hour. How
(c) 50 seconds (d) 45 seconds much time does it take to cross a man running at 3 km/hour
55. Two men, A and B run a 4 km race on a course 0.25 km in the same direction ? [2017-I]
round. If their speeds are in the ratio 5:4, how often does (a) 40 seconds (b) 30 seconds
the winner pass the other ? [2016-II] (c) 25 seconds (d) 15 seconds
(a) Once (b) Twice 65. A thief is spotted by a policeman from a distance of 100 m.
(c) Thrice (d) Four times When the policeman starts the chase, the thief also starts
56. When the Speed of a train is increased by 20%, it takes 20 running. If the speed of the thief is 8 km/hour and that of
minutes less to cover the same distance. What is the time the policeman is 10 km/hour, then how far will the thief have
taken to cover the same distance with the original speed ? to run before he is overtaken ? [2017-I]
[2016-II] (a) 200 m (b) 300 m
(a) 140 minutes (b) 120 minutes (c) 400 m (d) 500 m
(c) 100 minutes (d) 80 minutes 66. If a train crosses a km-stone in 12 seconds, how long will it
57. A person can row downstream 20 km in 2 hours and upstream take to cross 91 km-stones completely if its speed in 60 km/
4 km in 2 hours. What is the speed of the current?[2016-II] hr? [2017-II]
(a) 2 km/hour (b) 2.5 km/hour (a) 1 hr 30 min (b) 1 hr 30 min 12 sec
(c) 3 km/hour (d) 4 km/hour (c) 1 hr 51 min (d) 1 hr 1 min 3 sec
58. A train is travelling at 48 km/hour completely crosses another 67. In a 100 m race, A runs at 6 km/hr. If A gives B a start of 8 m
train having half its length and travelling in opposite and still beats him by 9 seconds, what is the speed of B?
direction at 42 km/hour in 12 s. It also passes a railway platform [2017-II]
in 45 s. What is the length of the platform? [2016-II] (a) 4.6 km/hr (b) 4.8 km/hr
(a) 600 m (b) 400 m (c) 5.2 km/hr (d) 5.4 km/hr
(c) 300 m (d) 200 m
EBD_7367
A- 88 Time, Speed and Distance

68. A boy went to his school at a speed of 12 km/hr and returned 72. A passenger train and a goods train are running in the same
to his house at a speed of 8 km/hr. If he has taken 50 minutes direction on parallel railway tracks. If the passenger train
for the whole journey, what was the total distance walked? now takes three times as long to pass the goods train, as
[2017-II] when they are running in opposite directions, then what is
(a) 4 km (b) 8 km the ratio of the speed of the passenger train to that of the
(c) 16 km (d) 20 km goods train? [2018-I]
69. A man rows down a river 18 km in 4 hours with the stream (Assume that the trains run at uniform speeds)
and returns in 10 hours. [2017-II] (a) 2 : 1 (b) 3 : 2
Consider the following statements : (c) 4 : 3 (d) 1 : 1
73. A man can row at a speed of x km/hr in still water. If in a
1. The speed of the man against the stream is
stream which is flowing at a speed of y km/hr it takes him z
1.8 km/hr.
hours to row to a place and back, then what is the distance
2. The speed of the man is still water is 3.15 km/hr.
between the two places? [2018-I]
3. The speed of the stream is 1.35 km/hr.
Which of the above statements are correct? z x2 y2 z x2 y2
(a) 1 and 2 only (b) 2 and 3 only (a) (b)
(c) 1 and 3 only (d) 1, 2 and 3 2y 2x
70. A man travelled 12 km at a speed of 4 km/hr and further 10
km at a speed of 5 km/hr. What was his average speed? x2 y2 z x2 y2
(c) (d)
[2017-II] 2zx x
(a) 4.4 km/hr (b) 4.5 km/hr 74. A car has an average speed of 60 km per hour while going
(c) 5.0 km/hr (d) 2.5 km/hr
from Delhi to Agra and has an average speed
of y km per hour while returning to Delhi from Agra
71. A train moving with a speed of 60 km per hour crosses an (by travelling the same distance). If the average speed of
electric pole in 30 seconds. What is the length of the train in the car for the whole journey is 48 km per hour, then what is
metres? [2018-I] the value of y? [2018-I]
(a) 300 (b) 400 (a) 30 km per hour (b) 35 km per hour
(c) 500 (d) 600 (c) 40 km per hour (d) 45 km per hour

HINTS & SOLUTIONS


1. (c) Let total distance of the journey be x km. According to question
x x x x (3 4) 7 x
first part of journey = km 7= =
2 4 3 12 12
x x = 12 km
Then second part of journey = km 4. (c) Let the speed of bike = v km/h
2
According to question, Time taken to cover 200 km at a speed of v km/h
x x 200
7 x 3x = h
2 2 = 10 = 20 v
30 70 210 New speed of bike = (v + 5) km/h
x = 2 × 210 = 420 km Time taken to cover 200 km at a speed of
2. (d) Total distance travel by train 200
= length of platform + length of train (v + 5) km/h =
v 5
= 200 + 200 = 400 m
200 200
Total distance travel According to question, = 2
Required speed = v v 5
Time taken
(v 5 v)200
400 =2 500 v2 + 5v
= m/s v 2 5v
15
v2 + 5v – 500 = 0 v2 + 25v – 20v – 500 = 0
400 18
= km/h = 96 km/h v (v + 25) – 20 (v + 25) = 0
15 5
(v – 20) (v + 25) = 0 v 25
Distance
3. (a) Time = v 20km / h
Speed
Let distance travel from A to B be x km. So the original speed of a bike is 20 km/h
Time, Speed and Distance 89
A-

5. (a) Let the speed of a train be v m/s and length of the 9. (a) P Q
train be x m. city A 60 km city B
when crosses telegraph t = 8s
Distance between P and Q = 60 km
Distance x Let the speed of Q = x km/h
Time = 8=
Speed v Then the speed of P = (x – 4 ) km/h
x=8v ...(i) Distance travelled by Q till they meet P = 60 + 12
When crosses bridge
t = 24s, Speed = v = 72 km
Distance = x + 200 72
Distance Time taken by Q till they meet =
x
Time = Speed
Distance travelled by P till they meet
x 200 = 60 – 12 = 48 km
24 = 24v = x + 200
v 48
24v = 8v + 200 [from eq. (i)] Time taken by P till they meet =
x 4
25 since time taken by both will be equal,
16v = 200 v=
2
72 48
25
From eq. (i) x = 8v = 8 × = 100 m x x 4
2
6. (b) Train and man running opposite to each other. 72x – 288 = 48x
Relative speed = 60 + 6 = 66 km/h 24x = 288 x = 12 km/h
66 5 Speed of P = x – 4
= m/s = 12 – 4 = 8 km/h
18
Distance Distance 92 92
Required time = 10. (d) Time = = =8h
Speed Speed 6.5 5 11.5
110 110 18 11. (a) Relative velocity = (50 – 32) km/h = 18 km/h
= = 6s
66 5 66 5 1 1 15
18 Elapse time = 15s = 15 h h
60 60 3600
150 100
7. (b) Time taken to cross the trains = Distance 15 x
25
v Time =
3 Speed 3600 18
where v is the speed of second train. x × 3600 = 18 × 15
250 3
10 = 18 15 18 15
25 3v x= km = 1000 m = 75 m.
250 + 30v = 750 3600 3600
50 12. (a) Let speed of stream = D km/h
30v = 500 v= m/s Speed of boat in still water = B km/h = 11 km/h.
3
According to the question
50 18
v= = 60 km/h 12
3 5 B+D=
8. (c) According to the question, t1
t1 t 2 = 12 – (–15)
t1 – t2 = 27 min 12
t1 = ...(i)
x x B D
27
V1 V2
= h
60 12
x x 27 B–D= t
2
=
2.5 4 60
2x x 9 12
= t2 = ...(ii)
5 4 20 B D
3x 9 Now,
=
20 20 45 11
3x = 9 t1 t2 2
x = 3 km 60 4
EBD_7367
90
A- Time, Speed and Distance

12 12 11 Total relative distance = 200 + 200 = 400 m


Given B 11km / h
B D B D 4 400 18
Required time = = 28.8 s
2B 11 50 5
B 2
D 2 4 12 18. (d) Let a distance walked be x km.
According to question
2 11 11
11
2
D 2
4 12 x x 36
= 4
2.5 3.25 60
D2 = 25 D = 5 km/h.
13. (b) Let distance between A and B = x km 1 1 276
According to question x =
2.5 3.25 60
x x
=2 6x = 2 × 18 × 12 2 4 276
12 18 x =
5 13 60
2 18 12
x= = 72 km
6 276 65
required distance = 72 km x= × = 6.5 km
60 46
14. (b) Relative speed = 75 + 50 = 125 km/h 19. (d) To cover a distance by train = 280 + 220 = 500 m
125 5
m/s = 60 1000 50
18 Speed of train = 60 km/h = m/s
60 60 3
Total covered distance = 100 + 150 = 250 m
Time taken to cross each other Total distance
Time taken by train =
Total covered distance Speed
=
Relative speed
500
250 18 = = 30s
50
= = 7.2 s
125 5 3
15. (a) Let speed of motorboat be B km/h.
20. (b) In fig, O is residence and D is school.
Speed of water = D km/h.
In BCD,
According to question
North C
9 x x
B+D= 4.5 (i)
2
9 3 B D
B–D= 1.5 (ii)
6 2
Now, on solving eqs. (i) and (ii), we get
B = 3 km/h and D =1.5 km/h
West East
speed of boat = 3km/h. O A E
2x
C
16. (c)
m

4c
4c

3 cm South
P1 2 3 cm 2 3 cm P2 BD = BC + CD2 = x2 + x2
2 2

BD = 2x BD = AE = 2x
A B
1 cm OE = OA + AE = 2.x 2.x 2 2x
P3 BA = DE = x
in OED,
Required number of points = 3 ( P1, P2 and P3)
OD2 = OE2 + DE2
17. (c) Relative speed of trains = (20 + 30) km/h
Minimum distance,
5
= 50 km/h = 50 m/s OD = (2 2.x)2 x2 8 x2 x 2 = 3x km
18
Time, Speed and Distance 91
A-

21. (c) Let side of a square be x. 25. (b) According to question,


D x C t1 t 2 = 15 – (–15) 15 + 15

3v x x 30
30 min h
v1 v2 60
Here, v1 = 3 km/h, v2 = 4 km/h
x 4v 2v x x x 1
=
3 4 2

v 4 x 3x 1 x 1
=
12 2 12 2
A x B
12
Total Distance x= = 6 km.
Average speed (u) = 2
Total Time 26. (b) Let speed of the flow of water be v km/h and rate
( x x x x) 4 v 48v of sailing of sailer be u km/h.
= x x x x = = = 1.92v
1 1 1 25 48
1 Then, u v u v =6 ...(i)
v 2v 3v 4v 2 3 4 8
Average speed lies between v < u < 2 v
22. (b) Radius of the wheel = 2.1 m 48
and u v u v =4 ...(ii)
Distance covered in 1 revolution = 2 r 12
22 On solving eqs. (i) and (ii), we get
= 2 2.1 v = 1 km/hr
7
27. (c) Average speed of two object when distance is
Distance covered in 75 revolutions
same
22
= 2 r × 75 = 2 2.1 75 2v1v2
7 Average speed =
= 990 m = 0.99 km v1 + v2
1 Here v1 = x km/h. v2 = y km/h
Time = h
60 2 xy
= .
Distance 0.99 x y
Required speed = = km/h
Time 1 28. (b) t2 – t1 = 2h
60 x x
= 59.4 km/h 2
10 15
23. (d) Speed of a car = 45 km/hr
140 3x 2 x
h 2 Relative speed
Time taken by the car = 140 min = 30
60
So, required distance travelled by car = Speed × Time x = 60 km. = 15 – 10
= 5km 1h.
140
= 45 105 km distance
60 speed =
24. (c) Let the speed of a train be v m/s and length of train relative speed
be x m.
when cross the pole in time t = 10 s 60
= = 12 km/h.
Distance 5
Time = 29. (b) Let distance between starting and their destination
Speed
be x km.
x t2 – t1 = 2h
10 = x = 10 v ... (i)
v x x
When cross the platform in time, t = 20 s 2
30 45
x 200
20 = 20v = x + 200 3x 2 x
v 2
20v = 10v + 200 [from Eq. (i)] 90
10v = 200 x
v = 20 2
From Eq. (i), 90
x = 10v = 10 × 20 = 200 m x = 180 km,
EBD_7367
92
A- Time, Speed and Distance

30. (c) Speed of train 34. (b) speed of a train = 40 km/h


Length of train 5
= 40
m/s
Time taken to cross the stationary object 18
Length of train = Speed of train × Time taken to Speed of another train = 20 m/s
cross the stationary object Speed of first train
Required ratio
72 1000 15 Speed of second train
= = 300 m
3600
5
31. (b) Let the length of the train = x m. 40
18 2 5
48 1000 20 18
speed = 48 km/h = m/s
60 60
Length of train, (x) = Speed × Time to cross the pole 10 5
or 5 : 9
18 9
48 1000
= 9 = 120 m 35. (c) Let speed of boat be B km/h and
60 60 speed of stream be D km/h.
32. (a) Distance between two points = 110 km Downstream
Relative speed = 60 + 40 = 100 km/h
Time after which they meet 32
B+D= ...(i)
6
Total distance 110
= = = 1.10 h Upstream
Relative speed 100
14
C B–D= ...(ii)
6
A B Now on solving equations (i) and (ii)
110 km
Distance corvered by A in 1.10 h = AC = 60 × 1.10 46
we get 2 B
= 66 km 6
Remaining distance = BC = 110 – 66 = 44 km 23
Required ratio = AC : BC = 66 : 44 = 3 : 2 B km / h
6
33. (c) Let the total distance be d km
Put value of B in equation ...(i)
1
Then, time taken to cover d distance 32 23
3 D= –
6 6
1
d 9 3
t1 = 3 d 1.5 km / h
6 2
x 3x
36. (a) Let total distance = x km.
1 2 total distance
Remaining distance = d d d km.
Average speed =
3 3 total time
2 x1 x2 x3
Now, time taken to cover rdd distance,
3 v1 v2 v3
A B
2 x
d
2d
t2 = 3 x x
2y 6y x1 x2 x3
t1 t2 t3
Time taken to cover distance from A to B and B to v1 v2 v3
A.
x x x
2d x1 , x2 , x3
t= 3 3 3
6z
According to the question, t1 + t2 = t x
1 1 1 x 1 1 1
d 2d 2d
3x 3y 3z 3 10 20 60
3x 6y 6z
3 3 60
1 1 1 18km/h
6 3 1 10
x y z
60
Time, Speed and Distance A-93

c
40. (a) Let speed of B = V
37. (c) A B
5
Then speed of A = V
p km/h q km/h 3

Relative speed = (p – q) km/hr. Winning post

c A B
Time taken to cover the c km = hr
p–q 80 m xm
Distance travelled by A before he overtakes
Starting point
qc
B= km Time taken by B to reach
p–q
x
38. (a) Let average speed of flight = v winning post = ...(i)
v
600 Time taken by A to reach
Time taken by flight (t) = ... (i)
v x 80
Now, flight speed is reduced by 200 km/hr winning post = ...(ii)
v
Now, from eqs (i) and (ii)
600 30
= t ... (ii) x x 80
v – 200 60
v 5v
Now, put value of t in eqn (ii)
3
600 1 5x = 3x + 240
t x = 120 km
600 2
– 200 Distance from starting point = x + 80
t
= 120 + 80 = 200 m
600 t 1 5
t 41. (a) Relative speed = (11 – 10) m/s
600 – 200 t 2 18
600 t – 200t2 + 300 – 100t = 600 t 5
= m/s
2t2 + t – 3 = 0 18
Distance travelled by police in 6 min
–1 1 24 5
t
2 2 = × 60 × 6 = 100 m
18
–1 5 –6 4 Distance between them = 200 – 100 = 100 m
, ,
4 4 4 42. (a) Speed of 1st train = 1.5 km/minute
t = 1 hour = 1.5 × 60 km/h
Duration of flight = 1 hour = 90 km/h
39. (b) Let Initial speed = v and Relative speed of trains = 90 – 60 = 30 km/h
total distance = x km. 5 25
= 30 × = m/sec
x 18 3
Time taken by car = 8
v 25
Length of slower train = ´ 27 = 25 × 9 = 225 m
x = 8v ... (i) 3
Now, speed is increased by 4 km/hr So, option (a) is correct.
43. (c) Let the length of race be ‘x’ meters and velocity of A,
x 30 B, C be respectively vA, vB and vC.
7
v 4 60 According to question-
x x - 30
x 1 15 =
7 vA vB
v 4 2 2
2x = 15v + 60 vA x - 30
= ---------------(i)
Put value of x from equation (i) vB x
16v = 15v + 60 x x - 20
v = 60 =
vB vC
Distance covered = 8 × 60 = 480 km
EBD_7367
94
A- Time, Speed and Distance

vC x - 20 45. (c) Let the original speed of car be = x km/hour


= ---------------(ii)
vB x 300 300
- =1
x x - 48 x ( x + 15)
=
vA vC
x + 15 - x 1
=
vC
=
x - 48 ( x + 15) x 300
---------------(iii)
vA x x2 + 15x – 4500 = 0
Multiplying eq (i) & (ii)- (x + 75) (x – 60) = 0
vB vC x 30 x 20 x – 75
vA vB x x x = 60
vC x 30 x 20 So, option (c)is correct.
vA x x ---------------(iv) 5 200
46. (b) Speed of 1st train = 40 × = m/sec
From eq (iii) & eq (iv)- 18 18

x 48 x 30 x 20 5 160
Speed of 2nd train = 32 × = m/sec
x x x 18 18
2 2
x – 48x = x + 600 – 50x Relative speed of trains
2x = 600
200 160 360
= + = = 20 m/sec
x = 300 18 18 18
Putting x = 300 in equation (i) & eq (ii) Total length of both the trains = 121 + 99 = 220 m
vB 300 - 20 270 9
= = = Time required =
220
= 11 sec
vA 300 300 10 20
9 So, option (b) is correct.
vB = v ----------------(v) 47. (a) Time taken by the 1st athlete
10 A
4000
vC 300 - 20 280 28 = = 250 unit
= = = 16
vB 300 300 30
Distance covered by 2nd Athlete
28 28 9 84 = 15 × 250 = 3750 m
vC = vB = ´ v = v --------(vi)
30 30 10 A 100 A Distance covered by 3rd Athlete
From eq (v) 7 eq (vi)- = 11 × 250 = 2750 m
Distance between 2nd and 3rd Athlete
9 84
vA : vB : vC = vA : v : v = 3750 – 2750 = 1000 m
10 A 100 A So, option (a) is correct.
= 100 : 90 : 84 48. (b) In a race of 100 m-
= 50 : 45 : 42 when A covers total 100 m.
so, option (c) is correct. B will cover = 100 – 4 = 96 m
44. (b) Let the speed of stream be ‘x’ km/hour.
C will cover = 100 – 2 = 98 m
30 30 When C covers 98 m, B covers = 96 m
+ = 4.5
(15 + x ) (15 - x ) 96
When C covers 100 m, B will cover = ´100
15 - x + 15 + x 4.5 98
=
(15 + x)(15 - x) 30 4800
= m
49
30 45
=
2
225 - x 300
So C will beat B by = 100 –
4800
225 – x2 = 200 49
x2 = 25
4900 – 4800 100
x=5 2.04 m
So, option (b) is correct 49 49
So, option (b) is correct
Time, Speed and Distance A-95
49. (d) Ratio of speed = 2 : 3 : 4 52. (d) Speed of bike = 25 – 50 km/h
1 1 1 Consumption of petrol at 25-50 hm/hr = 20 ml/km
Ratio of time = : : and consumes petrol = 40 ml/km at other speed
2 3 4 Given Distance
12 12 12 50 km 10 km + 30 km + 10 km
= : :
2 3 4
=6:4:3 Speed 40 km/hr 60 km/hr 30 km/hr
So, option (d) is correct. Petrol consumed = (10 + 10) × 20 ml + 30 × 40 ml
50. (c) Let the speed of train = x km/hr = 400 ml + 1200 ml
= 1600 ml
225
Distance of train = 225 m = km 1600
1000 =L 1.6L
100
9
Time taken = 9 sec = hr Option (d) is correct.
3600 53. (d) Let usual speed of train = 4 km/hr
Speed of man = 5 km/hr Time taken by train = t hr
Relative speed = Speed of train + speed of man Distance travelled = 120 km
= (x + 5) km/hr
120 120
225 So, u t hr
t 4
But relative speed = 1000 km / hr ...(1)
9 According to question
3600 New speed = (u + 10) km/hr.
225 Distance = 120 km
= 3600 90km / hr Time taken = (t – 1) hr
9 1000
x + 5 = 90 120
So u 10
x = 85 km/hr t 1
Option (c) is correct (using (1)
51. (c) Let Average speed = x km/hr
Time taken = t hr 120 120u
u 10
Distance = 14 km 120 120 u
1
14 4
x ...(1) (u + 10) (120 – 4) =1204 1204 + 1200 – u2 – 104 = 1204
t
u2 + 10 u – 1200 = 0
According to question, (u + 40) (u – 30) = 0 u = – 40 is not possible
Now Average speed = (x – 1) km/hr u = 30 km/hr
20 1 Option (d) is correct.
Time taken = t hr t hr
60 3 100 m
Distance = 14 km 54. (a) VB 10 m/sec
10sec
14
x 1 VA 1000 10
1
t VB 900 9
3
14 14 VA 10 100
1 14 VA
t 1 x 10 9 9
t t
3
450 x 1000
14 t 42 10 5 100
t 3t 1 9
(14 – t) (3t + 1) = 42 t
42t – 3t2 + 14 – t = 42 t x
45 90
– 3t2 + 14 – t = 0 5
(3t + 7) (t – 2) = 0
x = 45 × 5 = 225
7 Now 450 + 225 = 675
t = 2 hr t
3 1000 – 675 = 325
14 14
x Stat 325
t 2 65
5
x = 7 km/hr
Option (c) is correct Hence A will beat B by 65 sec.
EBD_7367
96
A- Time, Speed and Distance

55. (c) A runs 5 rounds, B runs 4 rounds (ratio of speed) 59. (d) When speed ratio is S1 : S2 : S3 : S4
A passes B each time A has run 5 rounds or 1 1 1 1
than time ratio will be S : S : S : S ...
1 5 1 1 2 3 4
5x = km = 1 km
4 4 4
1 1 1
i.e. time ratio : :
1 2 3 4
1 km is contained in 4 km 3 times
4 6: 4:3
hence A passes B thrice
80
60. (c) Radius of wheels = cm = 40 cm
S T2 2
56. (b) By the relation 1
S2 T1
22
Time ratio Wheel covers in 1 revolution = 2 r = 2 × × 40
7
x 120 Speed of wheel = 66 km/hr
=
x 20 100 or
= 5x = 6x – 120 66 1000 100cm
x = 120 minutes. 60 min
57. (d) Let speed of boat and current be x and y
66 1000 100 10
Distance covered in 10 minutes =
20 60
x + y= = 10
2
66 1000 100 10
4 No. of revolution = × 7 = 4375
60 2 22 40
x – y= =2
2 61. (c) Since distance is same in both ride than the average
by adding both equation
2xy
2x = 12 speed = x y
x= 6
y = 10 – 6 = 4 2 50 30 3000
=
58. (b) Let length of A train = 2l 30 50 80
length of B train = l = 37.5 km/hour

5 6
effective speed = (48 + 42) × m/s 62. (c) Time taken for A to complete the race = 100 × = 60
18 10
sec.
Case I
B took 60 + 12 sec to complete 96 m
3l
96 4
5 = 12 So speed is = 1.33 i.e. m/s
90 72 3
18
63. (d) Let time taken by passenger train = t
l = 100 m
time taken by express train = t + 3
Length of A train = 200 m
when distance = 540
Case II let length of plat for m = P
ATQ
200 P
= 45 540 540 D
5 = 15 S
48 t t 3 T
18
t 3 t
48 5 540 = 15
200 + P = 45 × t2 3t
18
108 = t2 + 3t
= 600 m
t2 + 3t – 108 = 0
P = 400 m
Time, Speed and Distance A- 97

t2 + 12t – 9t – 108 Therefore, if A takes 60 sec to complete 100 m race, then B


(t – 9) (t + 12) takes (60 + 9) seconds i.e. 69 seconds to complete 92 m.
t = 9 hr D 92 92 18 24
m/ s km/ hr 4.8km/ hr
Express train takes 9 hr T 69 69 5 5
9pm + 9 hr = 6 am 68. (b) Distance travelled by the boy from house to school in
64. (b) Length of train = 225 m 1 hr. i.e. 60 minutes = 12 km
Speed of train = 30 km/h Distance travelled by the boy from house to school in
Speed of Man = 3 km/hr 12 1
1 minute = km
the man is going in same direction than relatives 60 5
speed = (30 – 3) = 27 km/h
Similarly, distance travelled by the boy from school to
Then time taken house in 60 minutes = 8 km
225 Distance travelled by the boy from school to house in
225 2
5 sec = = 30 seconds. 8 2
27 3 5 1 minute = km
18 60 15
65. (c) Difference of distance covered by Police and thief is This means, total distance travelled in 2 minutes
100 m
Speed of Police = 10 km/h Speed of theif = 8 km/hr 1 2 3 2 5 1
= km
5 15 15 15 3
let time be t
Therefore, total distance travelled in 1 minute
5 5
10 t 8 t = 100
18 18 1 1
= km
D = 8T 3 2 6

5t Thus, total distance travelled in 50 minutes


= 100
9 1
= 50 8.333... 8 km
t = 180 S 6

5 69. (d) Let the speed of the man in still water be x km/hr and
Theif ran = 8 180 = 400 m let the speed of the stream be y km/hr.
18
Speed of the man downstream = x + y km/hr
66. (b) Here, we need to find the time that will take to cross 91
km stones completely. Speed of the man upstream = x – y km/hr
Given that, in 1 hr. train travels 60 km i.e. 60 km is Therefore,
travelled in 60 min.
This means in 1 minute 1 km is travelled. 18
x+y= ...(1)
4
Therefore, 90 km is travelled in 90 minutes i.e. 1 hour
30 minutes and the remaining 1 km in 12 seconds. 18
Thus, The total time taken is 1 hr. 30 min. 12 sec. x–y= 1.8 km/hr ...(2)
10
67. (b)
Solving these equations by elimination method, we
get

We are given that A gives B a start of 8 m. This means 18


2x = 1.8 4.5 1.8 6.3 x 3.15 km/hr
B starts from the point where A finishes its 8 m. there- 4
fore B covers 92 m. ...(3)
Given that A runs at 6 km/hr.
3.15 – y = 3.15 y 1.8 y 3.15 1.8
6 1000m
i.e. A runs at 100 m/ min y 1.35 y = 1.35 km/hr ...(4)
60min
It is also given that even after giving B a start of Therefore, equations (2), (3) and (4) implies that all the
8 m, A reaches early than B by 9 seconds. given statements are correct.
EBD_7367
98
A- Time, Speed and Distance

70. (a) Time taken to travel 12 km at the speed of 4 km/hr 73. (b) Speed in still water = x km/h
12 Speed of stream = y km/h
= 3 hrs. Speed in down stream = (x + y) km/h
4
Speed in up stream = (x – y) km/h
Time taken to travel 10 km at a speed of 5 km/hr
Total time = Z hrs
10 Let distance be D km from each side
= 2 hrs.
5 According to the question
Average speed = total distance ÷ total time taken D D
Z
10 12 22 x y x y
= 4.4 km/hr..
3 2 5
Dx Dy Dx Dy
Z
71. (c) Speed of train = 60 km/h 60
5 50
m/s x2 y2
18 3
2x × D = Z(x2 – y2)
time = 30 sec
Z x2 y2
50 D
length of train 30 500 m 2x
3
74. (c) V1 = speed from Delhi to Agra = 60 km/h
72. (a) Let speed of passenger train be x km/h and speed of
goods train be y km/h V2 = Speed from Agra to Delhi = y km/h
Average speed = 48 km/h
Speed in same direction = x – y km/h
Speed in opposite direction = (x + y) km/h 2V1V2
Average speed V1 V2
Let total length of trains be 100 m
According to the question According to the question

100 100 2 60 y
3 48
x y x y 60 y
120y = 48 × 60 + 48y
100 300 120y – 48y = 48 × 60
x y x y 72y = 48 × 60
100x + 100y = 300x – 300y 48 60
y 40 km / h
200x = 400y x: y= 2 : 1 72
C HA P T E R
TIME & WORK
11
1. 42 men take 25 days to dig a pond. If the pond would complete the same work? [2009-I]
have to be dug in 14 days, then what is the number of (a) 8 days (b) 10 days
men to be employed? [2007-I] (c) 12 days (d) 16 days
(a) 67 (b) 75 10. Two taps can fill a tub in 5 min and 7 min respectively. A
(c) 81 (d) 84 pipe can empty it in 3 min. If all the three are kept open
2. A person can do a job as fast as his two sons working simultaneously, when will the tub be full? [2009-I]
together. If one son does the job in 6 days and the other (a) 60 min (b) 85 min
in 12 days, how many days does it take the father to do (c) 90 min (d) 105 min
the job? [2007-I] 11. P and Q can do a job in 2 days; Q and R can do it in 4
(a) 9 days (b) 6 days days and P and R in 12/5 days. What is the number of
(c) 4 days (d) 3 days days required for P alone to do the job? [2009-II]
3. The ratio of the radii of the taps T1 and T2 is 2:1. Water (a) 5/2 (b) 3
(c) 14/5 (d) 6
is flowing through them with the same velocity. What is
12. The mess charges for 35 students for 24 days in `6300.
ratio of the time required to completely fill two identical
In how many days will the mess charges be ` 3375 for 25
drums kept under T1 and T2 ? [2007-I] students? [2009-II]
(a) 2 : 1 (b) 1 : 2 (a) 12 (b) 15
(c) 4 : 1 (d) 1 : 4 (c) 18 (d) 21
4. A can finish a work in 8 days and B can do it in 12 days. 13. A person walks a distance in 114 days, when he rests 9h
After A had worked for 3 days, B also joins A to finish a day. How long will he take to walk twice the distance,
the remaining work. In how many days will the remaining if he walks twice as fast and rests twice as long each day
work be finished? [2007-II] as before? [2009-II]
(a) 2 days (b) 3 days (a) 57 days (b) 228 days
(c) 4 days (d) 5 days (c) 285 days (d) 324 days
5. If 18 men earn ` 1440 in 5 days, how many men can earn 14. In an army camp ration is available for 100 soldiers for 10
` 1920 in 8 days? [2007-II] days. After 2 days, 60 soldiers joined. Then, for how many
(a) 10 (b) 12 more days will the remaining ration last? [2009-II]
(c) 15 (d) 18 (a) 7 days (b) 6 days
6. Ram can do a piece of work in 6 days and Shyam can (c) 5 days (d) 4 days
finish the same work in 12 days. How much work will be 1
finished, if both work together for 2 days? [2008-I] 15. of a work is completed in half a day by 5 persons.
(a) One-fourth of the work 48
(b) One-third of the work 1
(c) Half of the work Then, of the work can be completed by 6 persons in
40
(d) Whole of the work
7. If one man or two women or three boys can do a peice how many days? [2010-I]
of work in 55 days, then one man, one woman and one (a) 1 (b) 2
boy will do it how many days? [2008-II] 1
(a) 20 days (b) 30 days (c) 3 (d)
2
(c) 40 days (d) 50 days 16. If 6 men and 8 boys can do a peice of work in 10 days
8. If m men can do a job in p days, then (m+r) men can do while 26 men and 48 boys can do the same in 2 days, what
the job in how many days? [2008-II] is the time taken by 15 men and 20 boys in doing the same
mp type of work? [2010-I]
(a) ( p + r) days (b)
m+r
days (a) 4 days (b) 5 days
(c) 6 days (d) 7 days
p m+r 17. A can do a piece of work in 24 days. If B is 60% more
(c) days (d) p
days efficient then A, then how many days does B require to
m+r
do the same work? [2010-II]
9. A and B can do a piece of work in 8 days, B and C can
(a) 12 (b) 15
do the same work in 12 days. If A,B and C can complete
(c) 16 (d) 18
the same work in 6 days, in how many days can A and C
EBD_7367
100
A- Time & Work

18. Four taps can individually fill a cistern of water in 1h, 2h, 27. 2 men and 1 woman can complete a piece of work in 14
3h and 6h , respectively. If all the four taps are opened days, while 4 women and 2 men can do the same work in
simultaneously, the cistern can be filled in how many 8 days. If a man gets ` 90 per day, what should be the
minutes? [2011-I] wages per day of a woman ? [2014-I]
(a) 20 (b) 30 (a) ` 48 (b) ` 60
(c) 35 (d) 40 (c) ` 72 (d) ` 135
19. X completes a job in 2 days and Y completes it in 3 days 28. 18 men can earn ` 360 in 5 days. How much money will
and Z takes 4 days to complete it. If they work together 15 men earn in 9 days ? [2014-I]
and get `3900 for the job, then how much amount does (a) ` 600 (b) ` 540
Y get? [2011-I] (c) ` 480 (d) ` 360
(a) ` 1800 (b) ` 1200 29. 20 workers working for 5 h per day complete a work in 10
(c) ` 900 (d) ` 800 days. if 25 workers are employed to work 10 h per day,
20. A garrison of 'n' men had enough food to last for 30 days. what is the time required to complete the work ?
After 10 days, 50 more men joined them. If the food now [2014-I]
lasted for 16 days, what is the value of n? [2011-I] (a) 4 days (b) 5 days
(a) 200 (b) 240 (c) 6 days (d) 8 days
(c) 280 (d) 320 30. A can finish a work in 15 days, B in 20 days and C in 25
21. Consider the following statements : days. All these three worked together and earned ` 4700.
I. If 18 men can earn ` 1440 in 5 days, then 10 men can The share of C is [2014-I]
earn `1280 in 6 days. (a) ` 1200 (b) ` 1500
II. If 16 men can earn `1120 in 7 days, then 21 men can (c) ` 1800 (d) ` 2000
earn ` 800 in 4 days. 31. 4 goats or 6 sheeps can graze a field in 50 days. 2 goats
and 9 sheeps can graze the field in [2014-I]
Which of the above statements is/are correct? [2011-I]
(a) 100 days (b) 75 days
(a) Only I (b) Only II (c) 50 days (d) 25 days
(c) Both I and II (d) Neither I nor II 32. Pipe A can fill a tank in 10 min and pipe B can empty it
22. 15 men complete a work in 16 days. If 24 men are in 15 min. If both the pipes are opened in an empty tank,
employed, then the time required complete that work will the time taken to make it full is [2014-I]
be [2014-I] (a) 20 min (b) 25 min
(a) 7 days (b) 8 days (c) 30 min (d) None of these
(c) 10 days (d) 12 days 33. X can complete a job in 12 days. If X and Y work together,
23. A, B and C can do a piece of work individually in 8, 12
and 15 days, respectively. A and B start working but A 2
they can complete the job in 6 days. Y alone can
quits after working for 2 days. After this, C joins B till the 3
completion of work. In how many days will the work be complete the job in [2014-I]
completed ? [2014-I] (a) 10 days (b) 12 days
(c) 15 days (d) 18 days
8 6 34. A mason can build a tank in 12 h. After working for 6 h,
(a) 5 days (b) 4 days
9 7 hetook the help of a boy and finished the work in another
7 3 5h. The time that the boy will take alone to complete the
(c) 6 days (d) 3 days work is [2014-I]
13 4
(a) 30h (b) 45h
24. A can do a piece of work in 4 days and B can complete (c) 60h (d) 64h
the same work in 12 days. What is the number of days 35. A swimming pool 70m long, 44m wide and 3 m deep
required to do the same work together ? [2014-I] is filled by water flowing from a pipe at the rate of
(a) 2 days (b) 3 days 30800 cm3/s. The time taken to fill the swimming pool is
(c) 4 days (d) 5 days [2014-I]
25. A bus starts with some passengers. At the first stop, (a) 71/2 h (b) 80 h
one–fifth of the passengers gets down and 40 passengers (c) 250/3 h (d) None of these
get in. At the second stop, half of the passengers gets 36. If 3 men and 4 boys can do a piece of work in 8 days,
down and 30 get in. The number of passengers now is 70. then 6 men and 8 boys can do the same work in
The number of passengers with which the bus started [2014-I]
was [2014-I] (a) 2 days (b) 4 days
(a) 40 (b) 50 (c) 6 days (d) 16 days
(c) 60 (d) 70 37. X can do a piece of work in 25 days. Y is 25% more
26. X can do a work in 16 days. In how many days will the efficient than X. The number of days taken by Y is
work be completed by Y, if the efficiency of Y is 60% more [2014-I]
than of X? [2014-I] (a) 15 days (b) 20 days
(a) 10 days (b) 12 days (c) 21 days (d) 30 days
(c) 25 days (d) 30 days
Time & Work A-101
38. 45 people take 18 days to dig a pond. If the pond would 48. A is thrice as efficient as B and hence completes a work
have to be dug in 15 days, then the number of people to in 40 days less than the number of days taken by B. What
be employed will be [2014-I] will be the number of days taken by both of them when
(a) 50 (b) 54 working together ? [2014-II]
(c) 60 (d) 72 (a) 22.5 days (b) 15 days
39. A and B can do a piece of work in 10 h. B and C can do
it in 15 h, while A and C take 12 h to complete the work. (c) 20 days (d) 18 days
B independently can complete the work in [2014-I] 49. If 10 persons can dig 8 ft trench in 12 days, then how
(a) 12h (b) 16h many days will 8 persons take to dig 6 ft trench ?
(c) 20h (d) 24h [2014-II]
40. There are two taps A and B to fill up a water tank. The
(a) 10 days (b) 10.25 days
tank can be filled in 40 min, if both taps are on. The same
tank can be filled in 60 min, if tap A alone is on. How much (c) 11 days (d) 11.25 days
time will tap B alone take, to fill up the same tank? 50. The efficiency of P is twice that of Q, whereas the
[2014-I] efficiency of P and Q together is three times that of R. If
(a) 64 min (b) 80 min P, Q and R work together on a job, in what ratio should
(c) 96 min (d) 120 min they share their earnings? [2015-I]
41. A stock of food is enough for 240 men for 48 days. How (a) 2 : 1 : 1 (b) 4 : 2 : 1
long will the same stock last for 160 men? [2014-I]
(a) 54 days (b) 60 days (c) 4 : 3 : 2 (d) 4 : 2 : 3
(c) 64 days (d) 72 days 51. A and B are two taps which can fill a tank individually in 10
42. A can do a piece of work in 'x' days and B can do the minutes and 20 minutes respectively. However, there is a
same work 3x days, To fininsh the work together they leakage at the bottom, which can empty a filled tank in 40
take 12 days. What is the value of 'x' [2014-I] minutes. If the tank is empty initially, how much time will
(a) 8 (b) 10 both the taps take to fill the tank with leakage ? [2015-II]
(c) 12 (d) 16 (a) 2 minutes (b) 4 minutes
43. A,B and C can do a peice of work individually in 8, 10 and (c) 5 minutes (d) 8 minutes
15 days, respectively. A and B start working but A quits 52. If 4 men working 4 hours per day for 4 days complete 4 units
after working for 2 days. After this, C joins B till the of work, then how many units of work will be completed by
completion of work. In how many days will the work be 2 men working for 2 hours per day in 2 days ? [2015-II]
completed? [2014-I] (a) 2 (b) 1
(a) 53/9 days (b) 34/7 days
(c) 85/13 days (d) 53/10 days 1 1
(c) (d) .
44. 76 ladies complete a job in 33 days. Due to some reason 2 8
some ladies did not join the work and therefore, it was 53. If m persons can paint a house in d days, how many days
completed in 44 days. The number of ladies who did not will it take for (m +2) persons to paint the same house?
report for the work is [2014-I] [2015-II]
(a) 17 (b) 18 (a) md + 2 (b) md – 2
(c) 19 (d) 20 m+2 md
45. How many men will be required to plough 100 acres of (c) (d)
land in 10 days, if 10 men require 8 days to plough 20 md m+2
acres of land? [2014-I] 54. Two pipes A and B can fill a tank in 60 minutes and 75 minutes
(a) 30 (b) 40 respectively. There is also an outlet C. If A, B and C are
(c) 50 (d) 60 opened together, the tank is full in 50 minutes. How much
46. Pipe A can fill a tank in 3 h but there is a leakage also, time will be taken by C to empty the full tank? [2016-I]
due to which it takes 3.5 h for the tank to be filled. How (a) 100 minutes (b) 110 minutes
much time will the leakage take in emptying the tank, if (c) 120 minutes (d) 125 minutes
the tank is filled initially? [2014-II] 55. A can do 50% more work than B in the same time. B alone
(a) 21 h (b) 20 h can do a piece of work in 30 hours. B starts working and had
already worked for 12 hours when A joins him. How many
(c) 18 h (d) 10.5 h hours should B and A work together to complete the
47. A, B and C can do a piece of work individually in 8, 12 remaining work ? [2016-II]
and 15 days, respectively. A and B start working together (a) 6 hours (b) 12 hours
but A quits after working for 2 days. After this, C joins (c) 4.8 hours (d) 7.2 hours
and works till completion of the work. In how many days, 56. A tank can be filled by pipe X in 2 hours and pipe Y in 6
will the work be completed ? [2014-II] hours. At 10 am. pipe X was opened. At what time will the
tank be filled if pipe Y is opened at 11 am. ? [2016-II]
8 8
(a) 3 days (b) 5 days (a) 12:45 hours (b) 5:00 pm.
9 9
(c) 11: 45 a.m. (d) 11:50 am.
2 1 57. If 15 men take 21 days of 8 hours each to do a piece of work,
(c) 5 days (d) 6 days
3 18 then what is the number of days of 6 hours each that 21
EBD_7367
A- 102 Time & Work

women would take, if 3 women would do as much work as 2 63. A work when done by 10 women is completed in 12 days.
men ? [2017-I] The same work can be completed in 8 days when done by 5
(a) 18 (b) 20 men. How many days will it take to completed when 6 women
(c) 25 (d) 30 and 3 men are employed to perform the same job? [2018-I]
58. A and B working together can finish a piece of work in 12 (a) 12 (b) 10
days while B alone can finish it in 30 days. In how many (c) 8 (d) 5
days can A alone finish the work ? [2017-I]
64. A man undertakes to do a certain work in 150 days. He
(a) 18 days (b) 20 days
employs 200 men. He finds that only a quarter of the work is
(c) 24 days (d) 25 days
59. If 5 men can do a piece of work in 10 days and 12 women can done in 50 days. How many additional men should he employ
do the same work in 15 days, the number of days required so that the whole work is finished in time? [2018-I]
to complete the work by 5 men and 6 women is [2017-I] (a) 75 (b) 85
(c) 100 (d) 120
1
(a) 7 days (b) 8 days 65. If 5 tractors can plough 5 hectares of land in 5 days, then
2 what is the number of tractors required to plough 100
1 hectares in 50 days? [2018-I]
(c) 9 days (d) 12 days (a) 100 (b) 20
2
60. 30 men can complete a job in 40 days. However, after 24 (c) 10 (d) 5
days some men out of the assigned 30 left the job. The 66. A water tank has been fitted with two taps P and Q and a
remaining people took another 40 days to complete the job. drain pipe R. Taps P and Q fill at the rate of 12 litres per
The number of men who left the job is [2017-II] minute and 10 litres per minute respectively.
(a) 24 (b) 18 Consider the following statements S1, S2 and S3 :
(c) 12 (d) 6 S1 : Pipe R drains out at the rate of 6 litres per minute.
61. 4 goats or 6 sheep can graze a field in 50 days. 2 goats and S2 : If both the taps and the drain pipe are opened
3 sheep will graze it in [2017-II] simultaneously, then the tank is filled in 5 hours
(a) 200 days (b) 150 days 45 minutes.
(c) 100 days (d) 50 days S3 : Pipe R drains out (fully) the filled tank in 15 hours
62. A tap can fill a tub in 10 hours. After opening the tap for 5
20 minutes.
hours it was found that a small outlet at the bottom of the
To know what is the capacity of the tank, which one of the
tub was open and water was leaking through it. It was then
immediately closed. It took 7 hours to fill the tub after closing following is correct? [2018-I]
the outlet. What time will be taken by the outlet to empty (a) S2 is only sufficient
the full tub of water? [2017-II] (b) S1, S2 and S3 are necessary
(a) 35 hours (b) 25 hours (c) Any two out of S1, S2 and S3 are sufficient
(c) 20 hours (d) 17 hours (d) None of the above

HINTS & SOLUTIONS


1. (b) Let the number of men be n? T1 2
Men Days 3. (d) Given that, T =
2 1
42 25
n¯ 14 ­
Area of first drum p r2 4
n 25 \ = 1 =
\ = Þ n = 75 Area of sec ond drum p r22 1
42 14
time taken will be inversely proportional to the area
1
2. (c) One day's work of first son = 1
6 then required time ratio = = 1: 4
4
1
One day's work of second son = 1
12 4. (b) Work of A in one day = th part
\ One day's work of them working together 8
1 1 2 +1 1 1
= + = = Work of B in one day = th part
6 12 12 4 12
father will finish the work in 4 days. 3
3 days work of A = th part
8
Time & Work 103
A-

3 5 1
Remaining work of A = 1 - = th part 9. (a) A's and B's one day work =
8 8 8
1 1 1
Together one day's work of A and B = + B's and C's one day work =
8 12 12
3+ 2 5 1
= = A's, B's and C's one day work =
24 24 6
5 5 1 1 1 1
\ Number of days to finish the work = ¸ = 3 days B's one day work = + - =
8 24 8 12 6 24
5. (c) According to the formula 1 1 3 1
A's and C's one day work = - = =
M1D1 M 2 D2 6 24 24 8
=
Wa1 Wa2 A and C can do the work in 8 days.
1 1 1
18 ´ 5 m2 ´ 8 10. (d) Net filling in 1 min = + -
Þ = 5 7 3
1440 1920
21 + 15 - 35 1
= =
1920 ´18 ´ 5 105 105
\ m2 = = 15
8 ´ 1440 the tub will be full in 105 min.
1 1
6. (c) \ One day's work of Ram = 11. (b) P's and Q's 1 day work =
6 2
1 1
\ One day's work of Shyam = Q's and R's 1 day work =
12 4
One day's work together, Ram and Shyam 5
P's and R's 1 day work =
1 1 12
= +
6 12 1 1 5
Þ Now, ( P + Q ) + (Q + R ) + ( P + R ) = + +
2 +1 3 1 2 4 12
= = =
12 12 4 6+3+5 14
Þ 2 P + 2Q + 2 R = Þ 2( P + Q + R) =
1 12 12
\ Two day's work together = (half of the work)
14 7
2
P + Q + R work in one day = =
7. (b) 1 man = 2 women = 3 boys 24 12
3 1
\ 1 man + 1 woman + 1 boy = 3 boys + boys + 1 boy Q + R work in one day =
2 4
\ Boys Days 7 1 7-3 4 1
P alone work in one day = - = = =
3 55 12 4 12 12 3
11 x P's can do alone in 3 days.
2 12. (c) From formula
M1D1 = M 2 D2 M1 D1 M 2 D2 35 ´ 24 M 2 ´ 25
= Þ =
11 W1 W2 6300 3375
Þ 3 ´ 55 = ´ D2
2 35 ´ 24 ´ 3375
\ M2 = = 18
3 ´ 55 ´ 2 6300 ´ 25
D2 = = 30 days 13. (c) Distance Speed Hour/Day Speed
11
1 1 15 114
8. (b) Men Days 2
¯ 2
­ 6 ­ x ¯

m 1: 2 ü
­ p ï
(m+r ) x¯
2 :1 ý114 : x
6 :15ïþ
x m
\ =
p m+r \1 ´ 2 ´ 6 ´ x = 2 ´ 1 ´ 15 ´ 114

mp 2 ´ 15 ´114
Þ x= days Þ x= = 285 days
m+r 2´6
EBD_7367
104
A- Time & Work

14. (c) Soldiers Days Þ 20n = 16n + 800


100 8
160
­ x¯ Þ 4n = 800
160 8 Þ n = 200
Þ =
100 x 21. (d) From statement I
8 ´100 M 1d1 M 2 d 2
\ x= = 5 days =
160 Wa1 Wa 2

1 Here M1 = 18, d1 = 5 Wa1 = 1440



15. (d) Q
M 1D1 M 2 D2
= Þ 2 = 6 ´ D2 M2 = 10, d2 = 6, Wa2 = ?
W1 W2 1 1
48 40 18 ´ 5 10 ´ 6
Þ =
1440 Wa2
1
´ 5 ´ 48
1 10 ´ 6 ´ 1440
Þ D2 = 2 = Þ Wa2 =
40 ´ 6 2 = ` 960
18 ´ 5
16. (a) 6M + 8B = 10 days ...(i) But Wa2 is given as 1280 so it is not correct.
26M + 48B = 2 days ... (ii) From statement II
15M + 20 B = ?
By to formula Þ 20B = 10 days M 1d1 M 2 d 2
=
M1 D1 = M2 D2 Þ (6M + 8B) × 10 = (26M + 48B) × 2 Wa1 Wa 2
Þ 60M + 80B = 52M + 96B Þ 8M = 16B Here M1= 16, d1 =7, Wa1 = 1120
M = 2B M2 = 21, d2 = 4, Wa2 = ?
Now in eq. (i), put M = 2B
6 × 2B + 8B = 10 days 16 ´ 7 21´ 4
Þ = W
12B + 8B = 10 days 1120 a2
Again
15M + 20B = 15 × 2B + 20B = 30B + 20B = 50B 21 ´ 4 ´1120
Wa2 = = 3 ´ 280
From formula, M1 D1 = M2 D2 16 ´ 7
Þ 20 × 10 = 50 × D2 = ` 840
Now, Wa2 = ` 800
20 ´ 10
D2 = = D2 = 4 days . So both statements are not correct.
50 22. (c) According to the formula,
17. (b) + 60%)x B
A (uuuuuuuuu M1D1 M 2 D2
100 160 =
efficiency is proportional to the number of days W1 W2
Þ n1D1 = n2 D2 Þ 100 ´ 24 = 160 ´ D2 Here, M1 = 15, D1 = 16, W1 = W2 = 1
M2 = 24 and D2 = ?
100 ´ 24
\ D2 = = 15 days 15 ´ 16 24 ´ D2
160 Þ =
1 1
1 6 15 ´ 16
18. (b) Required time = =
1 1 1 1 6 + 3 + 2 +1 D2 = = 10
+ + + 24
1 2 3 6 10 days are required to complete the work.
6 6 1 1 5
= h = ´ 60 min = 30 min 23. (c) Work done by A and B in 1 day = + =
12 12 8 12 24
1 1 1 10
19. (b) Ratio of work done by X,Y and Z = : : 2 days work of A and B =
2 3 4 24
=6:4:3 After 2 days A left the work.
ratio in their amount = 6 : 4 : 3
10 14
4 \ Remaining work = 1 - =
´ 3900 = ` 1200 24 24
\ Part of Y =
6+ 4+3
1 1 9
20. (a) M1 = n, d1 = 30 –10 = 20, M2 = (50 + n) d2 = 16 work of B and C together = + =
By formula, 12 15 60
M1d1 = M2d2 the number of days required by B and C to finish
work
Þ n ´ 20 = ( n + 50 ) ´16
Time & Work A- 105

14 Þ 12 M = 18 W
24 = 14 ´ 60 = 35 M 18 3
= 9 24 9 9 = =
W 12 2
60 Now, a man gets ` 90 per day
\ Total days to complete the work
2
35 53 8 \ 1 woman wages = × (wages of 1 man)
=2+ = = 5 days 3
9 9 9
2
1 = × 90 = ` 60.
24. (b) A's one day work = 3
4
M1D1 M 2 D2
1 28. (b) =
B's one day work = W1 W2
12
Þ Here M1 = 18, D1 = 5, W1 = ` 360
1 1 M2 = 15, D2 = 9, W2 = ?
One day work of A and B together = +
4 12 Þ 18 × 5 × W2 = 15 × 9 × 360
3+1 4 1 15 ´ 9 ´ 360
= = = \ W2 = = ` 540
12 12 3 18 ´ 5
No. of days together = 3 days. 29. (a) M1D1T1 = M2D2T2
25. (b) Let bus starts with x number of passengers. Here M1 = 201, D1 = 101T1 = 5h, M2 = 25, D2 = ?,
After 1st stoppage, number of passengers T2 = 10 h
x 5x - x + 200 Þ 20 × 10 × 5 = 25 × D2 × 10
= x- + 40 = 20 ´ 10 ´ 5
5 5 \ D2 = = 4 days
4 x + 200 25 ´ 10
= 1
5
30. (a) A's one day work =
After 2nd stoppage, number of passengers 15
4 x + 200 4 x + 200 1
= - + 30 B's one day work =
5 5´2 20
4 x + 200 4 x + 200 1
Þ - + 30 = 70 C's one day work =
5 10 25
4 x + 200 æ 1 ö A, B and C worked together one day work
Þ çè1 - ÷ø + 30 = 70 1 1 1
5 2
= + +
4 x + 200 æ 2 - 1ö 15 20 25
Þ ç ÷ = 40 20 + 15 + 12 47
10 è 2 ø = =
300 300
4 x + 200 1
Þ ´ = 40 time taken to complete work by A, B and C working
5 2
300
4 x + 200 together =
Þ = 40 47
10
Þ 4x + 200 = 400 Þ 4x = 200 1 300
\ Share of C = ´ ´ 4700 = ` 1200
25 47
200
\x= = 50 31. (d) Given that
4
26. (a) Efficiency is proportional to days 3
1 Goats = sheeps.
X Y 2
( +60%)
100 ¾¾¾¾ ®160 Now, 2 goats + 9 sheeps
Þ 100 × 16 = 160 × D 3
= 2 ´ sheeps + 9 sheeps
100 ´ 16 2
\D= = 10 days
160 = 12 sheeps
27. (b) M1D1 = M2D2 Here M1D1 = M2 D2
Þ ( 2 M+1W ) ´14 = ( 4 W+ 2 M ) ´ 8 Þ 6 ´ 50 = 12 ´ D2
D2 = 25 days
Þ 28 M+14 W = 32 W+16 M
EBD_7367
106
A- Time & Work

1 Þ 100 ´ 25 = 125 ´ Days


32. (c) Part filled by pipe A in 1 min =
10 100 ´ 25
1 \ Days of Y =
125
Part empty by pipe B in 1 min =
15 = 20 days.
1 1 3- 2 1 38. (b) M1 = 45, D1 = 18
\ Total tank filled in minutes = - - =
10 15 30 30 Here, M 2 = x, D2 = 15
Hence, the tank will be filled in 30 min.
By the formula, M1D1 = M 2 D2
1
33. (c) X's one day's work = M1 D1
12
\ M2 =
3 D2
( X + Y ) ' s one day's work = 20 45 ´ 18
Þ x= = 3 ´ 18
3 1 4 1 15
\ Y's one day's work = - = =
20 12 60 15 = 54
\ Number of day's taken by Y = 15 days 1
1 39. (d) A's and B's 1 h work =
34. (c) Mason work for 1h= 10
12
1
6 1 B's and C's 1 h work =
Mason work for 6h = = 15
12 2
1
1 1 A's and C's 1 h work =
Work left = 1 - = 12
2 2
Let the boy can finish the work in x h \ A ' s, B ' s and C ' s 1h work
1 1 x + 12 1æ 1 1 1 ö
Then, thier 1 h work = + = = ç + + ÷
12 x 12x 2 è 10 15 12 ø
x + 12 1
\ ´5 = 1 1 1
12 x 2 = ´ =
2 4 8
5x + 60 1
Þ = 1 1 1
12 x 2 B's work in 1 h = - =
8 12 24
Þ 10 x + 120 = 12 x
B can complete the work in 24 h
Þ 120 = 2x 40. (d) Work done by tap B in 1 min
Þ x = 60 h 1 1 3- 2 1
35. (c) Required time taken to fill the pool = - = =
40 60 120 120
Volume of the pool Total time taken by the tap B to fill the tank is 120 min.
= Part of pool filled in1s
240 ´ 48
41. (d) Required days =
70 ´ 44 ´ 3 ´ 100 ´ 100 ´ 100 160
= s
30800 = 72 days
250 1
= 300000 s = h 42. (d) 1 day work of A =
x
3
36. (b) 3 men + 4 boys = 8 days, 6 men + 8 boys = ? 1
By formula, M1D1 = M 2 D2 1 day work of B =
3x
Þ ( 3M + 4 B ) ´ 8 = ( 6 M + 8B ) ´ D2 1 1 4
Both A and B days work = + =
Þ ( 3M + 4 B ) ´ 8 = 2 ( 3M + 4 B ) ´ D2 x 3x 3x
1
( 3M + 4 B ) ´ 8 Work of both A and B =
\ D2 = 12
2 ´ ( 3M + 4B )
= 4 days 4 1
Þ = Þ 3 x = 48
37. (b) X Y 3 x 12
\ x = 16
100 (uuuuuuuuu
+ 25%r)125 efficiency is proportional to days
Time & Work 107
A-

1 47. (b) A = 8
43. (d) A work in one day = B = 12
8
8
1
B work in one day = 12 120
10 C = 15 15
1
C work in one day =
15
Here, 120 is total work
1 1 5+ 4 9
A+B work in one day = + = = 1 1 5
8 10 40 40 (A + B) starts together in 1 day work = + =
8 12 24
2´9 9
A+B work in two day = = 5 5
40 20 2 days work of (A + B) = ´2 =
24 12
9 11
Remaining work 1 - = 5 7
20 20 Remaining work = 1- =
12 12
1 1 3+ 2 5 1
B + C work in one day = + = = = Now, (B + C) work together in 1 day
10 15 30 30 6
1 1 5+ 4 3
So, ( B + C ) a work together in 6 days. = + = =
12 15 60 20
11 11 11´ 3 33
Q work together = 6 ´ = = days 7 20 35
´
8
= 3 days
20 20 10 10 (B + C) Complete in days = =
12 3 9 9
Q Total number of days =
A worked for 2 days.
33 20 + 33 53
2+ = = days 8 8
10 10 10 So A, B, C all 3 completed work in 2 + 3 = 5 days
44. (c) Here M1D1 = M2 D2 9 9
M1 = 76 Ladies, M2 = x Let 48. (b) Let efficiency of B is x
D1 = 33 days D2 = 44 days
Þ 76 ´ 33 = x ´ 44
x 3x
\x = 57 days Days
no. of ladies who did not Report
for work = 76 – 57 = 19. A B
3x x Efficiency
45. (b) Here, M1 = 10, D1 = 8,W1 = 20
Days are inversaly proportional to efficiency.
M 2 = x ( let ) , D2 = 10, W2 = 100
According to question
M1 D1 M 2 D2 Þ 3x – x = 40
Q =
W1 W2 Þ 2x = 40
\ x = 20
10 ´ 8 x ´10
\ = Þ x = 8 ´ 5 = 40 Time taken by A = x days = 20 days
20 100
Time taken by B = 3x days = 3 × 20 = 60 days
7
46. (a) Time taken by pipe A = 3h 21 20 ´ 60
(–6) Time taken by A and B together = = 15 days
80
35 æ 7 ö 49. (d) We know that
Due to leakage, time taken = 3.5 = = ç– ÷
10 è 2 ø M1D1 M2D2
=
W1 W2
7
Here LCM of 3 and is 21 Here given M1 = 10, W1 = 8, D1 = 12
2
Here 21 is full capacity of tank M2 = 8, W2 = 6, D2 = ?
then due to leakage in 1 h tank filled by A = (7–6) 10 ´12 8´ D2
= 1l. Þ =
8 6
leakage takes 21 h Þ D2 = 11.25 days
EBD_7367
108
A- Time & Work

50. (a) Earning of man is directly proportinonal to their A 100


income =
P Q P+Q R 30 150
2 : 1 3 : 1 A = 20 days
ATQ
Q : R
1 1 x 12 x
+ + =1
Ratio of efficiency = 2 : 1 : 1 A B B
é1 1ù 12
51. (d) In 1 minute all the taps will fill the part of the tank x ê + ú = 1-
ë 20 30 û 30
1 1 1
= + - é 5 ù 18
10 20 40 xê ú =
ë 60 û 30
4 + 2 -1 5 1
= = = x = 7.2 hrs
40 40 8 56. (c) X fill in 2 hr, y in 6 hr
1 ATQ
th part of tank will fill in 1 minute x x -1
8
+ =1
Full tank will fill in 1 × 8 = 8 minutes 2 6
So, option (d) is correct. 3x + x – 1 = 6
M1D1H1 M 2D 2H 2 4x = 7
52. (c) =
W1 W2 7 3
x= or 1 hr ´ 60 min
4 4
4´4´4 2´2´2 1 = 1 hr 45 min
= Þ W = unit
9 W 2 tank will be filled by 11 : 45 am
So, option (c) is correct. 57. (d) Given 3w = 2M
53. (d) M1D1 = M2D2
2M
(m) (d) = (m + 2)D2 w=
3
md M1d1h1 = M2d2h 2
D2 =
m+2
æ2ö
So, option (d) is correct. 15 × 21 × 8 = 21ç ÷ ´ 6 ´ d
54. (a) Let pipes A, B, C fill a tank in x, y and z hrs. respectively. è3ø
Then x = 60 min, y = 75 min, z = z min 15 ´ 21´ 8 ´ 3
d=
Required time to fill the tank = 50 min. 21 ´ 2 ´ 6
d = 30 days
xyz 58. (b) A and B together can do in = 12 days
But Required time =
yz + zx - xy B alone can do in = 30 days
A can do in
60 ´ 75 ´ z
50 = 1 1 3 1
75z + 60z - 60 ´ 75 = - = = i.e. 20 days
12 30 60 20
450z 59. (a) 5 Men can do work in 10 days, 1 man will do in 10 × 5.
Þ 5= 12 women can do work in 15 days, 1 women will do in
135z - 4500
12 × 15
Þ 5(135z – 4500) = 450z ATQ. 5 men and 6 women will do in
Þ 675z – 4500 × 5 = 450 z 5 6
Þ 675z – 450z = 4500 × 5 Þ +
10 ´ 5 12 ´15
4500 ´ 5 1 1 4
Þ z= = 100 Þ + =
225 10 30 30
\ Option (a) is correct.
2 15 1
efficiency of A 150 i.e. or 7 days
= 15 2 2
55. (d) 60. (b) If M1 men can do W1 work in D1 days and M2 men
efficiency of B 100
can do W2 work in D2 days (where all men work at the
Time taken by A 100
= M 1D1 M 2 D2
Time taken by B 150 same rate), then = .
W1 W2
Time & Work A-109
Therefore, here, according to the question, M1= 30,
M1D1 w
16 = 1
D1= 40, W1 = 1, M2 = x, D2 = 40, W2 = M 2 D2 w 2
40
Thus, 1
200 ´ 50
M1 D1 M 2 D2
= Þ
30 ´ 40 x ´ 40 = 4
W1 W2 1
=
16
Þ 1200 ( 200 + x ) 100 3
4
40
40 x ´ 40 1200 ´ 16 100 1
= Þx= = 12 = \ 200 + x = 300
16 40 ´ 40 200 + x 3
Thus, the number of men who left the job are (30 – 12) \ x = 300 – 200 = 100
i.e. 18 men. Hence, additional number of men = 100
æ AND ö 65. (c) M1 = 5 D1 = 5 W1 = 5
61. (d) Using the formula, ç x = given number of days,
è OR ÷ø M2 = x D2 = 50 W2 = 100
where x is the number of days we have to divide. M1D1 W
Thus, = 1
M 2 D2 W2
æ AND ö æ 2 3ö
çè ÷ø x = 50 Þ çè + ÷ø x = 50 5´5 5
OR 4 6 Þ =
x ´ 50 100
æ 1 1ö Þ x = 10
Þ ç + ÷ x = 50 Þ x = 50
è 2 2ø Hence, required number of tractors is 10.
62. (b) Let us assume the capacity of the tub is 100L. 66. (c) From S1 and S2
It is given that a tap can fill 100L in 10 hrs. volume of water filled in 1 minute by all three pipes
This means, in 1 hr. a tap can fill only 10L. together = 12 + 10 – 6 = 16 ltrs
Therefore, in 7 hrs. a tap can fill only 70L. Total time = 5 hrs 45 minutes = 5 × 60 + 45
= 345 minutes
This means in 5 hrs. a tap fills only 30L but actually the
tap should fill 50L in 5 hrs. Capacity of tank = 345 × 16 = 5520 ltrs
This means, there is a leakage of 20L which has a du- From S1 and S3
ration of 5 hrs. Water drained by pipe in 1 minute = 6 ltrs
If 20L of water is leaked in 5 hrs., then 1L water is Total time = 15 hrs 20 minutes = 15 × 60 + 20
= 920 minutes
5 1
leaked in = hrs. Capacity of tank = 920 × 6 = 5520 ltrs
20 4
From S2 and S3
1 Time required for all three pipes = 5 hrs 45 minutes =
This means 100L water is leaked in ´ 100 = 25 hrs.
4 345 minutes
63. (c) M1 = 10 D1 = 12 Time required for drain pipe = 15 hrs 20 minutes
M2 = 5 D2 = 8 = 920 minutes
10 women × 12 = 5 men × 8 Capacity of tank = 920 × 6 = 5520 ltrs.
From S2 and S3
120
1 man = women = 3 women Time required for all there pipees = 5hrs 45 minutes =
40 345 min.
Now Time required for drain pipe = 15 hrs 20 minutes =
6 women + 3 men = 6 women + 3 × 3 women = 15 920 minutes
women
Time required for first and second pipe
\ 15 women × No. of days = 10 women × 12 days
10 women ´ 12 days 1
\ Required no. of days = 15 women
1
= 8+3 =
2760
minutes
= 1 1 2760 11
= 8 days +
345 920
1
64. (c) M1 = 200 D1 = 50 w1 = 2760
4 Capacity of the tank = ´ 22 = 5520 ltrs
11
1 3
M2 = 200 + x D2 = 100 w2= 1 - = Hence, any two out of S1, S2 and S3 are sufficient.
4 4
EBD_7367
110
A- Logarithm

C HA P T E R
LOGARITHM
12
1. What is the value of 8. What is the value of [log10 (5 log10 100)]2 ? [2011-I]
(a) 4 (b) 3
æ öæ öæ
ö æ ö
(c) 2 (d) 1
ç log 1 2 ÷ ç log 1 3 ÷ ç log 1
4 ÷ ..... ç log 1 1000 ÷ ?
ç ÷ç ÷ç
÷ ç ÷
è øè 3 øè ø è 1000 ø
What is log10 æç ö÷ + log10 æç 4 ö÷ + log10 æç 5 ö÷ + ..... upto
2 4 3
9.
[2007-I] è 2ø è 3ø è 4ø
(a) 1 (b) –1 8 terms equal to? [2011-II]
(c) 1 or –1 (d) 0 (a) 0 (b) 1
2. What is the value of log100 0.1? [2008-II] (c) log10 5 (d) None of these
1 1 10. What is the logarithm of 0.0001 with respect to base 10?
(a) (b) – [2012-I]
2 2
(a) 4 (b) 3
(c) –2 (d) 2 (c) –4 (d) –3
3. What is the value of 11. If log10 a = p and log10 b = q, then what is the value of
æ5ö æ 128 ö æ5ö log10 (a ppq)? [2012-II]
2 log ç ÷ + log ç ÷ + log ç ÷ ? [2009-I] (a) p2 + q2 (b) p2 – q2
è8ø è 125 ø è 2ø
2 2
p2
(a) 0 (b) 1 (c) p q (d)
q2
(c) 2 (d) 5
ærö 1
4. If log r 6 = m and logr 3 = n, then what is log r ç ÷ 12. The value of log10 3125 – 4log10 2 + log10 32 is [2016-I]
è 2ø 5
equal to? [2009-I] (a) 0 (b) 1
(a) m – n + 1 (b) m + n – 1 (c) 2 (d) 3
(c) 1 – m – n (d) 1 – m + n 13. What is the solution of
1
éë log13 (10 )ùû -
é ì -1 ü-1 ù 2
5. What is the value of
éëlog169 (10 )ùû
? [2009-II]
(
log10 ê1 - í1 - 1 - x 2
êë î
) ý ú
þ úû
=1? [2016-II]

1 (a) x =100 (b) x =10


(a) (b) 2
2 (c) x =1 (d) x = 0
(c) 1 (d) log10 13 x
6. What is the value of x +2
14. What are the roots of the equation 2 .27 -1
x =9?
æ1 ö [2016-II]
ç log10 125 – 2 log10 4 + log10 32 + log10 1 ÷ ? [2010-II]
è 3 ø æ2ö
æ3ö
(a) 2,1 – log ç ÷ (b) 2, 1– log ç ÷
1 è2ø è3ø
(a) 0 (b)
5
æ3ö æ log 2 ö
2 (c) -2,1 - log ç ÷ (d) -2,1 ç log 3 ÷
(c) 1 (d) è2ø è ø
5 15. The number of digits in 330 is n and it is given that log10 3
7. What is the value of = 0.4771. What is the value of n ? [2016-II]
1 (a) 13 (b) 14
log 25 – 2log10 3 + log10 18 (c) 15 (d) 16
2 10
[2012-I] 16. What is the number of digits in 240 ?
(a) 2 (b) 3 (Given that log10 2 = 0.301) [2017-I]
(c) 1 (d) 0 (a) 14 (b) 13
(c) 12 (d) 11
Logarithm 111
A-

19. If log10 6 = 0.7782 and log10 8 = 0.9031, then what is the


æ 10 ö
17. What is the solution of the equation x log10 ç ÷ + log10 value of log10 8000 + log10 600? [2017-II]
è 3ø (a) 4.6813 (b) 5.5813
3 = log10 (2 + 3x) + x ? [2017-I] (c) 1.5813 (d) 6.6813
(a) 10 (b) 3 20. For 0 < m < 1, which one of the following is correct?
(c) 1 (d) 0 [2018-I]
18. Let P, Q, R be the mid-points of sides AB, BC, CA respectively (a) log10 m < m2 < m < m–1
of a triangle ABC. If the area of the triangle ABC is 5 square (b) m < m–1 < m2 < log10 m
units, then the area of the triangle PQR is [2017-I] (c) log10 m < m < m–1 < m2
5. (d) log10 m < m–1 < m < m2
5 5
(a) square units (b) square units
3 2 2
5
(c) square units (d) 1 square unit
4

HINTS & SOLUTIONS


æ öæ öæ ö æ ö 52 ´ 128 ´ 5 52 ´ 27 ´ 5
1. (b): ç log 1 2 ÷ ç log 1 3 ÷ ç log 1 4 ÷ ..... ç log 1 1000 ÷ = log = log
( 23 )
2 2
ç ÷ç ÷ç ÷ ç ÷ 8 ´ 125 ´ 2 ´ 53 ´ 2
è 2 øè 3 øè 4 ø è 1000 ø

æ öæ öæ ö æ ö 27 ´ 53 27 ´ 53
ç log 2 ÷ ç log 3 ÷ ç log 4 ÷ ç log 1000 ÷ = log = log = log 1 = 0
= ç ÷ç ÷ç ÷ ..... ç ÷ 26 ´ 53 ´ 2 27 ´ 53
çç log 1 ÷÷ çç log 1 ÷÷ çç log 1 ÷÷ ç æ 1 ö÷
è 2øè 3øè 4ø ç log ç 1000 ÷ ÷ 4. (d): Given, log r 6 = m and log r 3 = n
è è øø
Q log r 6 = log r (2 × 3)
= log r 2 + log r 3
æ log a ö
ç Q log b a = ÷ \ log r 3 + log r 2 = m
è log b ø
Þ n + log r 2 = m
æ log 2 ö æ log 3 ö æ log 4 ö æ log 1000 ö Þ log r 2 = m – n
= ç ÷ç ÷ç ÷ ..... ç ÷
è - log 2 ø è - log 3 ø è - log 4 ø è - log1000 ø ærö
\ log r ç ÷ = log r r – log r 2
= (-1) × (-1) × (-1) × ..... × (-1) è 2ø
( Q number of factors is odd) =1–m+n
= –1
log13 (10) log13 (10) æ 1 ö
1 5. (b): = ç Q log a b c = loga c ÷
2. (b): log100 0.1 = log100 log169 (10) log 2 (10) è b ø
10 13

= log100 1 – log102 10
log13 10 1
= = =2
1 1 1 1 1
= 0 – log10 10 = – × 1 = – log 10
2 2 2 2 13 2

æ5ö æ 128 ö æ5ö 1


3. (a): 2log ç ÷ + log ç ÷ + log ç ÷ 6. (c): log10 125 – 2log10 4 + log10 32 + log10 1
è8ø è 125 ø è2ø 3
2 1
æ 5ö æ 128 ö æ5ö log10 (5)3 – 2log10 (2)2 + log10 (2)5 + 0
= log ç ÷ + log ç ÷ + log ç ÷ =
3
è8ø è 125 ø è2ø
EBD_7367
A- 112 Logarithm

= log10 5 – 4log10 2 + 5log10 2 1


-1 ù - 2
= log10 5 + log10 2 = log10 5 × 2 = log10 10 = 1 13. (b) log10
é
ë
{ 2 -1
ê1 - 1 - (1 - x ) } ú
û
=1
1
7. (c): log10 25 – 2log10 3 + log10 18
2 -
1
é ì -1
= log10 251/2 – log10 32 + log10 18 1 ü ù 2
Þ log10 ê1 - í1 - ý ú =1
= log10 5 – log10 9 + log10 18 êë î 1 - x 2 þ úû

5 ´18 90 -
1
= log10 = log10 = log10 10 = 1 é ì 2 ü-1 ù 2
9 9
ê1 - ïí -x ïý ú
8. (d): [log10 (5 log10 100)]2 = [log10 (5 log10 102)]2 Þ log10 ê ï1 - x 2 ï ú = 1
ë î þ û
= [log10 (10 log10 10)]2
= [log10 10]2 ( Q log10 10 = 1) 1
-
2
=l =1 é (1 - x 2 ) ù 2
Þ log10 ê1 - ú =1
ëê - x 2 ûú
(c): log10 æç ö÷ + log10 æç 4 ö÷ + log10 æç 5 ö÷ + ..... + 8th term
3
9.
è 2ø è 3ø è 4ø -
1
é - x 2 - (1 - x 2 ) ù 2
Þ log10 ê ú =1
= log10 æç 3 ö÷ + log10 æç 4 ö÷ + log10 æç 5 ö÷ + ..... + log10 æç 10 ö÷ ëê -x 2 ûú
è 2ø è 3ø è 4ø è 9ø
1
-
æ3 4 5 10 ö é 1 ù 2
ç ´ ´ ´ ..... ´ ÷ Þ log10 ê 2ú =1
= log10 è 2 3 4 9ø ëx û
Þ log10 x = log1010
= log10 æç ö÷ log10 5.
10
x = 10
è 2ø Þ log10
10. (c): Let log10 0.0001 = a
æ x ö
ç ÷
1 14. (c) 2x+2 27è x-1ø = 9
a = log10
4
(10) æ x ö
ç ÷
4
= log10 1 – log10 (10) = 0 – 4 = –4 2x+2 27è x-1ø = 20 32
11. (a): Given that, 2x+2 = 20
x=–2
log10 a = p and log10 b = q
log10 (ap bq ) = log10 ap + log10 bq x
now taking 2x+2 , =9
= p log10 a + q log10 b 27 x -1
= p × p + q × q = p2 + q2
é æ x öù
ç ÷
1 ê
log 2 x+2
27è x -1 ø ú = log 32
12. (b) log10 3125 - 4 log10 2 + log10 32 ê ú
5 êë úû

1 5 5 3x
= log10 ( 5 ) - 4 log10 2 + log10 ( 2 ) (x + 2) log (2) + log 3 = 2 log 3
5 x -1
5 é 3x ù
= log10 5 - 4 log10 2 + 5 log10 2 = log10 5 + log10 2 (x + 2) log 2 = ê 2 - log3
5 ë x - 1úû
= log10 ( 5 ´ 2 ) é -x - 2ù
(x + 2) log 2 = ê ú log3
[log m + log n = log mn] ë x -1 û
= log10 10 = 1 log 3
\Option (b) is correct. 1–x=
log 2
Logarithm A-113

18. (c)
æ 3ö A
x = 1 – log ç ÷
è 2ø

x = –2 and 1 – log æç ö÷
3
è 2ø P R
15. (c) 30
let 3 = 10 x

30 log [3] = x log (10)

30 × 0.4771 = x B Q C
14.313 = x
x > 14 1 1 1
PQ = AC, QR = AB, PR = BC
i.e. 15 2 2 2
16. (b) let 240 = 10x AB BC CA 2
log 240 = log 10x = = =
QR PR PQ 1
40 log2 = x log10
40 (.301) = x 2
area of ABC æ 2 ö 4
12.04 = x =ç ÷ =
area of PQR è 1 ø 1
i.e. x > 12
\ x = 13 = in D ABC = 5 cm2
No. of digit in 240 is 13 5
as D PQR = cm2
4
æ 10 ö
17. (d) x log10 ç ÷ + log10 3 = log (2 + 3x ) + x 19. (d) We are given log10 6 = 0.7782 and log10 8 = 0.9031
è 3ø 10
Therefore,
xlog10 10 – x log103 + log103 = log10 (2 + 3x) + x
log10 8000 + log10 600 = log10 6 ´ 102 + log10 8 ´ 103
x – log103x + log103 = log10 (2 + 3 ) + x
x

= log10 6 + log10 10 2 + log10 8 + log10 103


log é3 ù = 0.7782 + 2log10 10 + 0.9031 + 3log10 10
10 ê ú log (2 + 3x )
ë 3x û 10
= 1.6813 + 2 + 3
Þ 31–x = 2 + 3x = 6.6813
Þ 31–x –3x = 31 – 30
20. (a)
x= 0
EBD_7367
114
A- Basic Operation and Factorisation

C HA P T E R
BASIC OPERATION AND
13 FACTORISATION
1. For what value of k, will the expression 3x3 – kx2 + 4x + 16 (a) p6 + 6p (b) p6 – 6p
æ kö (c) p6 + 6p4 + 9p2 + 2 (d) p6 – 6p4 + 9p2 – 2
be divisible by ç x - ÷ ? [2007-I]
è 2ø 10. If x + y + z = 0, then what is
(a) 4 (b) – 4
3 3 3
(c) 2 (d) 0 é (y – z – x) ù é (z – x – y) ù é (x – y – z) ù
ê ú +ê ú +ê ú
2. Which one of the following is the factor of ëê 2 ûú êë 2 ûú êë 2 ûú
x4 + xy3 + xz3 + x3 y + y4 + yz3 ? [2007-I] equal to? [2007-I]
(a) x + y + z (b) x2 + y2 + z2 (a) 24 xyz (b) –24 xyz
3 3 3
(c) x + y + z (d) x2 + y2 (c) 3 xyz (d) xyz
3. If pqr = 1, what is the value of the expression 11. If m and n are odd integers with n < m, then which is the
1 1 1 largest integer among the following which divides
+ + ? [2007-I] m2 – n 2 ? [2007-II]
-1 -1
1+ p + q 1+ q + r 1 + r + p -1
(a) 2 (b) 4
(a) 1 (b) –1
(c) 6 (d) 8
(c) 0 (d) 1/3
4. If x + y + z = 2s, then what is 1 1
12. What is the value of 2 + 2 + - ?
3 3
(s – x) + (s – y) + 3(s – x) (s – y)z 2+ 2 2- 2
equal to? [2007-I] [2007-II]
(a) z3 (b) –z3
(c) x 3
(d) y 3 (a) 2 (b) 2 - 2

5. If x2 = y + z, y2 = z + x, z2 = x + y, then what is the value (c) 4 + 2 (d) 2 2


1 1 1 13. Under constant pressure conditions, the temperature
of + + ? [2007-I]
x +1 y + 1 z +1 varies directly as volume. At a given instant, volume
(a) 1 (b) 0 = 60 m3, temperature = 300 K. If the volume is increased
(c) –1 (d) 2 to 100 m3, then what will be the temperature? [2007-II]
6. If p, q and r be such that p + q = r and pqr = 30, then (a) 200 K (b) 300 K
what is the value of p3 + q3 – r 3 ? [2007-I] (c) 400 K (d) 500 K
(a) 0 (b) 90
(c) – 90 (d) Cannot be determined 14. If one of the two factors of an expression which is the
difference of two cubes, is (x4 + x2 y + y2), then what is
x8 + 4 the other factor? [2007-II]
7. What is equal to? [2007-I]
x 4 + 2x 2 + 2 (a) x + y (b) x – y
(a) x + 2x2 – 2
4
(b) x4 – 2x2 + 2 (c) x2 + y (d) x2 – y
(c) x4 – 2x2 – 2 (d) Cannot be determined
15. Which one of the following is one of the factors of
8. If 2x – 2x – 1 = 4, then what is the value of 2x + 2x – 1? x2(y – z) + y2 (z – x) – z(xy – yz – zx)? [2007-II]
[2007-I] (a) x–y (b) x + y – z
(a) 8 (b) 10 (c) x – y – z (d) x + y + z
(c) 12 (d) 16
16. If (3x3 – 2x2y – 13xy2 + 10y3 ) is divided by (x – 2y), then
æ1ö æ 1 ö what is the remainder? [2008-I]
9. If x + ç ÷ = p, then what is x6 + ç 6 ÷ equal to?
x
è ø èx ø (a) 0 (b) y + 5
[2007-I] (c) y + 1 (d) y2 + 3
Basic Operation and Factorisation A- 115

1 11 1
17. What is the value of x in 1 + = ? 27 If x2 – 4x + 1 = 0, then what is the value of x3 + ?
ì 1 ü 7 x3
ï ï [2008-II]
+
1 íæ 1 öý
ï çè 1 +
x ÷ø ïþ
(a) 44 (b) 48
î (c) 52 (d) 64
[2008-I]
(a) 1 (b) 3 28. If x + y + z = 6 and xy + yz + zx = 11, then what is the
value of x3 + y3 + z3 – 3xyz? [2008-II]
1 7
(c) (d) (a) 18 (b) 36
2 11
(c) 54 (d) 66
18. If (x + y + z = 0), then what is (x + y) (y + z) (z + x)
equal to? [2008-I] 29. If a is a rational number such that (x – a) is a factor of
(a) – xyz (b) x2 + y2 + z3 the polynomial x3 – 3x2 – 3x + 9, then [2008-II]
(c) x3 + y3 + z3 + 3xyz (d) xyz (a) a can be any integer
(b) a is an integer dividing 9
19. If (a + b = 3), then what is the value of (a3 + b3 + 9ab)? (c) a cannot be an integer
[2008-I] (d) a can take three values
(a) 18 (b) 27
30. If x2 – 11x + a and x2 – 14x + 2a have a common factor,
(c) 81 (d) Cannot be determined then what are the values of a? [2008-II]
20. If (x3 + 5x2 + 10k) leaves remainder –2x when divided by (a) 0, 7 (b) 5, 20
(x2 + 2), then what is the value of k ? [2008-I] (c) 0, 24 (d) 1, 3
(a) –2 (b) –1 31. Which one of the following is a factor of
(c) 1 (d) 2 2x3 – 3x2 – 11x + 6? [2008-II]
21. If (5x2 + 14x + 2)2 – (4x2 – 5x + 7)2 is divided by (a) x + 1 (b) x – 1
(x2 + x + 1), what is the remainder? [2008-I] (c) x + 2 (d) x – 2
(a) –1 (b) 0 32. What should be subtracted from 27x3 – 9x2 – 6x – 5
(c) 1 (d) 2 to make it exactly divisible by (3x – 1)? [2008-II]
22. What is/are the factor(s) of (x29 – x24 + x13 – 1)? (a) –5 (b) –7
(c) 5 (d) 7
[2008-I]
(a) Only (x – 1) 33. Suppose p « q = 2P + 2q – pq, where p, q are natural
(b) Only (x + 1) numbers. If 8 « x = 4, then what is the value of x?
(c) (x – 1) and (x + 1) [2008-II]
(d) Neither (x – 1) nor (x + 1) (a) 1 (b) 2
(c) 3 (d) 4
23. What is the expression
(x + y)–1 (x–1 + y–1) (xy–1 + x–1y)–1 equal to? 34. What is the simplified form of
[2008-I] æ x 2 - 3x + 2 ö æ x 2 - 9 ö æ x 3 + 2x 2 + 4x ö
(a) x + y (b) (x2 + y2) –1 ç ÷¸ç 2 ÷´ç 2 ÷?
3
(c) xy (d) (x2 + y2 ) è x - 8 ø è x + 7x + 12 ø è x + 3x - 4 ø
[2008-II]
24. If (ab – b + 1 = 0) and (bc – c + 1 = 0), then what is
(a – ac) equal to? [2008-I] x x-2
(a) (b)
(a) –1 (b) 0 x -3 x -3
(c) 1 (d) 2 x x +3
(c) (d)
25. Let S be a set of all even integers. If the operations x +3 x+4
I. addition II. subtraction
III. multiplication IV. division é (2.3)3 - 0.027 ù
are applied to any pair of numbers from S, then for which 35. What is the value of ê 2
ú ? [2008-II]
êë (2.3) + 0.69 + 0.09 úû
operations is the resulting number in S? [2008-I]
(a) 2.6 (b) 2
(a) I, II, III and IV (b) I, II and III
(c) 1.3 (d) 1
(c) I and III (d) II and IV
26. What is the value of the polynomial r(x), so that 36. What is the simplified form of 9 2 - 8 - 4 2 ?
f (x) = g(x) q(x) + r(x) and deg r(x) < deg g(x), [2008-II]
where f(x) = x2 + 1 and g(x) = x + 1? [2008-II]
(a) 4 2 (b) 3 2
(a) 1 (b) –1
(c) 2 (d) –2 (c) 2 2 (d) 2
EBD_7367
116
A- Basic Operation and Factorisation

37. What is (a) 1 (b) 2


x(y – z) (y + z) + y(z – x) (z + x) + z(x – y) (x + y) (c) 3 (d) 4
equal to? [2009-I]
46. Which one of the following statements is correct?
(a) (x + y) (y + z) (z + x) [2009-II]
(b) (x – y) (x – z) (z – y) (a) Remainder theorem is a special case of factor theorem
(c) (x + y) (z – y) (x – z) (b) Factor theorem is a special case of remainder theorem
(d) (y – x) (z – y) (x – z) (c) Factor theorem and remainder theorem are two
independent results
38. If the remainder of the polynomial a0 + a1 x + a2 x2 + .....
+ an xn when divided by (x – 1) is 1, then which one of (d) None of the above
the following is correct? [2009-I] 2 –2
47. If x = 2 + 3, then what is (x + x ) equal to?
(a) a0 + a2 + ..... = a1 + a3 + .....
[2009-II]
(b) a0 + a2 + ..... = 1 + a1 + a3 + .....
(a) 12 (b) 13
(c) 1 + a0 + a2 + ..... = – (a1 + a3 + .....) (c) 14 (d) 15
(d) 1 – a0 – a2 – ..... = a1 + a3 + .....

39. When (x3 – 2x2 + px – q) is divided by (x2 – 2x – 3), the 1+ x 1 2a + 1


48. If a = , then what is + 2 equal to?
remainder is (x – 6). What are the values of p, q 2- x a + 1 a -1
respectively? [2009-I] [2009-II]
(a) –2, –6 (b) 2, – 6 (1 + x) (2 + x) (1 - x) (2 - x )
(c) –2, 6 (d) 2, 6 (a) (b)
2x - 1 x-2

40. If x = 1 + 4 3 2
2 , then what is the value of x – 4x + 4x ? (1 + x) (2 - x ) (1 - x )(2 - x)
(c) (d)
[2009-I] 2x -1 2x +1
(a) –1 (b) 0
49. If pq + qr + rp = 0, then what is the value of
(c) 1 (d) 2
p2 q2 r2
+ + ? [2009-II]
x z p 2 - qr q 2 - rp r 2 - pq
41. If y = w , then what is (xy + zw)2 equal to?
(a) 0 (b) 1
[2009-I] (c) –1 (d) 3
2 2 2 2
(a) (x + z ) (y + w ) (b) x2 y2 + z2 w2
50. If x + y + z = 0, then what is the value of
(c) x2 w2 + y2 z2 (d) (x2 + w2) (y2 + z2)
1 1 1
+ + ? [2009-II]
2 2 2 2 2 2
1 2 1009 x +y -z y +z -x z + x2 - y2
2
42. If + + = 1, then what is the value of
x + 1 y + 2 z + 1009 1
(a) (b) 1
x y z x + y2 + z2
2
+ + ? [2009-I]
x + 1 y + 2 z + 1009 (c) –1 (d) 0
(a) 0 (b) 2 ( x - y)3 + ( y - z )3 + ( z - x )3
(c) 3 (d) 4 51. What is equal to?
4 ( x - y ) ( y - z ) ( z - x)
15 3 [2009-II]
43. If the unit of weight is kg, then what number will
4 2 3 1
quintal represent? [2009-I] (a) – (b)
4 4
(a) 25 (b) 6
3
1 (c) (d) 0
(c) (d) None of these 4
9 52. If the expression px + 3x –3 and 2x3 – 5x + p when
3 2

divided by x – 4 leave the same remainder, then what is


44. If m and n are two integers such that m = n2 – n, then the value of p? [2010-I]
(m2 – 2m) is always divible by [2009-II] (a) –1 (b) 1
(a) 9 (b) 16 (c) –2 (d) 2
(c) 24 (d) 48 53. If x(x + y + z) = 9, y(x + y + z) = 16 and z(x + y + z) =
144, then what is x equal to? [2010-I]
45. When x40 + 2 is divided by x4 + 1, what is the remainder?
[2009-II]
Basic Operation and Factorisation A-117

9 9 (a) 1 (b) 2
(a) (b)
5 7 (c) 3 (d) 2
9 16 65. If a + b + c = 6 and a + b2 + c2 = 26, then what is
2
(c) (d) ab + bc + ca equal to? [2011-II]
13 13
54. If u, v and w are real numbers such that u3 – 8v3 – 27w3 (a) 0 (b) 2
= 18uvw, then which one of the following is correct? (c) 4 (d) 5
[2010-I] 66. If 3x3 – 2x2 y – 13xy2 + 10y3 is divided by x – 2y, then
(a) u – v + w = 0 (b) u = –v = –w what is the remainder? [2011-II]
(c) u – 2v = 3w (d) u + 2v = –3w (a) 0 (b) x
55. x4 + 4y4 is divisible by which one of the following? (c) y + 5 (d) x – 3
[2010-I] 67. What is the remainder when (x11 + 1) is divided by (x + 1)?
(a) (x2 + 2xy + 2y2) (b) (x2 + 2y2) [2011-II]
(c) (x2 – 2y2) (d) None of these (a) 0 (b) 2
56. The shadow of a pole 6 m high is 15 m long and at the (c) 11 (d) 12
same time the shadow of a tree is 25 m long. What is the 68. If (x – 3) is a factor of (x2 + 4px – 11p), then what is the
height of the tree? [2010-I] value of p? [2011-II]
(a) 21 m (b) 10 m (a) – 9 (b) – 3
(c) 35 m (d) None of these (c) –1 (d) 1

a 2 b2 c2 x2 - 3x + 2 x2 - 5x + 4
57. If a + b + c = 0, then what is the value of + + ? 69. What is ¸ equal to?
bc ca ab x2 - 5 x + 6 x 2 - 7 x + 12
[2010-II] [2011-II]
(a) –3 (b) 0 x+3
(c) 1 (d) 3 (a) (b) 1
x -3
58. If (x4 + x–4) = 322, then what is one of the value of
(x – x–1)? [2010-II] x +1
(a) 18 (b) 16 (c) (d) 2
x -1
(c) 8 (d) 4
59. If x = (b – c) (a – d), y = (c – a) (b – d) and (2.3)3 - 0.027
70. What is the value of ? [2011-II]
z = (a – b) (c – d), then what is x3 + y3 + z3 equal to? (2.3)2 + 0.69 + 0.09
[2010-II]
(a) 0.3 (b) 2.3
(a) xyz (b) 2xyz
(c) 1 (d) 2
(c) 3xyz (d) –3xyz
60. What are the factors of x2 + 4y2 + 4y – 4xy – 2x – 8? xyz
[2010-II] 71. If x + y + z = 0, then what is equal
( x + y) ( y + z) ( z + x)
(a) (x – 2y – 4) and (x – 2y + 2)
to (where, x ¹ –y, y ¹ –z, z ¹ –x)? [2011-II]
(b) (x – y + 2) and (x – 4y + 4)
(a) –1 (b) 1
(c) (x – y + 2) and (x – 4y – 4)
(c) xy + yz + zx (d) None of these
(d) (x + 2y – 4) and (x + 2y + 2)
72. What is the value of
61. If the expression (px3 + x2 – 2x – q) is divisible by
(x – 1) and (x + 1), what are the values of p and q, 1.001 ´ 1.001 ´ 1.001 + 0.999 ´ 0.999 ´ 0.999
?
respectively? [2010-II] 1.001 ´ 1.001 - 1.001 ´ 0.999 + 0.999 ´ 0.999
(a) 2, –1 (b) –2, 1
[2012-I]
(c) –2, –1 (d) 2, 1
(a) 1 (b) 2
62. If x5 – 9x2 + 12x – 14 is divisible by (x – 3), then what
(c) 3 (d) 4
is the remainder? [2011-I]
73. AB is a straight line. C is a point whose distance from AB
(a) 0 (b) 1
is 3cm. What is the number of points which are at a
(c) 56 (d) 184
distance of 1cm from AB and 5cm from C?
1 1 9 [2012-I]
63. If x - = , then what is 9x 2 + is equal to?
x 3 x2 (a) 1 (b) 2
[2011-I] (c) 3 (d) 4
(a) 18 (b) 19 74. What is the value of k that (2x – 1) may be a factor of
(c) 20 (d) 21 4x4 – (k – 1)x3 + kx2 – 6x + 1? [2012-I]
(a) 8 (b) 9
1 1 (c) 12 (d) 13
64. What is the value of + ?
1+ 2 + 3 1- 2 + 3 75. If p(x) is a common multiple of degree 6 of the polynomials
f(x) = x3 + x2 – x – 1 and g(x) = x3 – x2 + x – 1, then which
[2011-I]
one of the following is correct? [2012-I]
EBD_7367
118
A- Basic Operation and Factorisation

(a) p(x) = (x – 1)2 (x + 1)2 (x2 + 1) 85. If the expression x3 + 4x2 + 4x + k has (x + 4) as a factor,
(b) p(x) = (x – 1) (x + 1)2 (x2 + 1)2 then what is the value of k? [2012-II]
(c) p(x) = (x – 1)3 (x + 1)2 (x2 + 1) (a) –16 (b) 16
(d) p(x) = (x – 1)2 (x4 + 1) (c) 32 (d) –32
76. If x3 + 5x2 + 10k leaves remainder –2x when divided by 86. What is the value of k which will make the expression
x2 + 2, then what is the value of k? [2012-I] 4x2 + 12x + k a perfect square? [2012-II]
(a) –2 (b) –1 (a) 5 (b) 7
(c) 1 (d) 2 (c) 8 (d) 9
87. For what value of k, (x + 5) is a factor of 6x2 + kx + 10?
1 1 1 [2013-I]
77. If x + = a, then what is the value of x3 + x2 + 3 + 2 ?
x x x (a) 5 (b) 32
[2012-I] (c) 36 (d) 40
(a) a3 + a2 (b) a3 + a2 – 5a 88. One of the factors of the polynomial
(c) a3 + a2 – 3a – 2 (d) a3 + a2 – 4a – 2 x4 – 7x3 + 5x2 – 6x + 81 is: [2013-I]
78. If a = b, b = c and xyz = 1, then what is the value of cz ?
x y (a) x + 2 (b) x – 2
[2012-I] (c) x + 3 (d) x – 3
(a) a (b) b 89. (a + 1)4 – a4 is divisible by [2013-I]
(a) –2a2 + 2a – 1 (b) 2a3 – 2a – 1
a (c) 2a3 – 2a + 1 (d) 2a2 + 2a + 1
(c) ab (d)
b 90. The factor(s) of 5px – 10qy + 2rpx = 4qry is/ are
79. If 3 x + y = 81 and 81 x – y = 3, then what is the value of x? [2013-I]
[2012-I] (a) Only (5 + 2r)
(b) Only (px – 2qy)
17 17
(a) (b) (c) Both (5 + 2r) and (px – 2qy)
16 8 (d) Neither (5 + 2r) nor (px – 2qy)
17 15 91. If the expression x3 + 3x2 + 4x + p contains (x + 6) as a
(c) (d) factor, then the value of p is [2013-I]
4 4
4 3 3 3 4 3 (a) 132 (b) 141
80. x + xy + x y + xz + y + yz is divisible by (c) 144 (d) 151
[2012-I] 92. What is the value of
(a) Only (x – y)
(b) Only (x3 + y3 + z3) 725 ´ 725 ´ 725 + 371 ´ 371 ´ 371
? [2013-I]
(c) Both (x + y) and (x3 + y3 + z3) 725 ´ 725 - 725 ´ 371 + 371 ´ 371
(d) None of the above (a) 9610 (b) 1960
81. What is the value of (c) 1096 (d) 1016
3 × 0.3 × 0.03 × 0.003 × 0.0003 × 30? [2012-II] 93. Consider the following statements :
(a) (0.09)3 (b) (0.009)3 I. x + 3 is the factor of x3 + 2x2 + 3x + 8.
(c) (0.0009)3 (d) None of these II. x – 2 is the factor of x3 + 2x2 + 3x + 8.
82. What is the value of Which of the statements given above is/are correct?
[2013-II]
é(2.247)3 + (1.730)3 + (1.023)3 - 3 ´ 2.247 ù (a) Only I (b) Only II
ê ú (c) Both I and II (d) Neither I nor II
êë ´ 1.730 ´ 1.023 úû
?
é(2.247) 2 + (1.730)2 + (1.023) 2 - 2.247 ´ 1.730 ù ( x 2 + y 2 ) ( x - y ) - ( x - y )3
ê ú 94. What is equal to?
êë - 1.730 ´ 1.023 - 2.247 ´ 1.023úû x 2 y - xy 2
[2013-II]
[2012-II]
(a) 1 (b) 2
(a) 1.730 (b) 4
(c) 4 (d) – 2
(c) 5 (d) 5.247
95. If the expression x3 + 3x2 + 4x + k has a factor x + 5, then
83. If two factors of a4 – 2a3 – 9a2 + 2a + 8 are (a + 1) and
(a – 1), then what are the other two factors? what is the value of k? [2013-II]
[2012-II] (a) –70 (b) 70
(a) (a – 2) and (a + 4) (b) (a + 2) and (a + 4) (c) 48 (d) –48
(c) (a + 2) and (a – 4) (d) (a – 2) and (a – 4) 96. The quantity which must be added to (1 – x) (1 + x2) to
obtain x3 is [2013-II]
æ y-xö (a) 2x3 + 3x2 + x + 1 (b) 2x3 + x2 + x – 1
84. If 0.764 y = 1.236 x, then what is the value of ç ÷?
è y+xø (c) 2x3 – x2 + x – 1 (d) –x2 + x – 1
2 2
[2012-II] 97. If (49) – (25) = 37x, then what is x equal to? [2013-II]
(a) 0.764 (b) 0.236 (a) 64 (b) 74
(c) 2 (d) 0.472 (c) 48 (d) 42
Basic Operation and Factorisation A-119
98. For what values of k will Which of the above statements is / are correct?
4x5 + 9x4 – 7x3 – 5x2 – 4kx + 3k2 (a) Only 1 (b) Only 2
contain x –1 as a factor? [2013-II] (c) Both 1 and 2 (d) Neither 1 nor 2
1 109. If the equation x2 + 2(1 + k) x + k2 = 0 has equal roots,
(a) 3, - (b) 3, –1 then what is the value of k ? [2014-II]
2
1 1 1 1
(c) 0, (d) 1, (a) (b) –
3 3 2 2
99. x(y2 – z2) + y(z2 – x2) + z(x2 – y2) is divisible by (c) 1 (d) – 1
[2013-II] 110. What is the highest common factor of 2x3 + x2 – x – 2
(a) Only (y – z) and 3x3 – 2x2 + x –2 ? [2014-II]
(b) Only (z – x) (a) x –1 (b) x + 1
(c) Both (y – z) and (z – x) (c) 2x + 1 (d) 2x – 1
(d) Neither (y – z) nor (z – x) 111. What is the remainder when (1235 × 4523 × 2451) is
100. x3 + 6x2 + 11x + 6 is divisible by [2014-I] divided by 12 ? [2014-II]
(a) Only (x + 1) (b) Only (x + 2) (a) 1 (b) 3
(c) Only (x + 3) (d) All of these (c) 5 (d) 7
101. What should be added to be x(x + a) (x + 2a) (x + 3a),
112. For what value of k is (x – 5) a factor of x3 –3x2 + kx – 10?
so that the sum be a perfect square? [2014-I]
[2015-I]
(a) 9a2 (b) 4a2
(c) a 4 (d) None of these (a) – 8 (b) 4
102. If 3x4 – 2x3 + 3x2 – 2x + 3 is divided by (3x + 2), then the (c) 2 (d) 1
remainder is [2014-I] 113. The expression
x3q2 – x3pt + 4x2pt – 4x2q2 + 3xq2 – 3xpt is divisible by
185
(a) 0 (b) [2015-I]
27 (a) (x – 1) only
181 3 (b) (x – 3) only
(c) (d) (c) both (x – 1) and (x–3)
25 4
103. The expression 2x3 + x2 – 2x + 1 is divisible by (d) neither (x – 1) nor (x – 3)
[2014-I]
1 1 1 1
(a) x + 2 (b) 2x + 1 114. The value of + + + .... +
(c) x – 2 (d) 2x – 1 1 ´ 4 4 ´ 7 7 ´ 10 16 ´ 19
[2015-II]
æ 1 ö 17 æ 1 ö
104. If ç x 2 + = , then what is ç x 3 - ÷ equal to ?
2 ÷ 5 6
è x ø 4 è x3 ø (a) (b)
19 19
[2014-I]
75 63 8 9
(a) (b) (c) (d)
16 8 19 19
115. Consider the following in respect of the equation
95 [2015-II]
(c) (d) None of these
8
105. If (x + k) is the common factor of x2 + ax + b and x2 + (x - 1)2
y=
cx + d. of and then what is k equal to ? [2014-II] x -1
(a) (d – b) / (c – a) (b) (d – b) / (a – c) 1. y = 1 if x > 1
(c) (d + b) / (c + a) (d) (d – b) / (c + a) 2. y = –1 if x < 1
106. What is the remainder when x5 – 5x2 + 125 is divided by 3. y exists for all values of x
x + 5? [2014-II] Which of the above statements is/are correct?
(a) 0 (b) 125 (a) 1 only (b) 2 only
(c) –3125 (d) 3125 (c) 1 and 2 only (d) 1, 2 and 3
107. What is the lowest common multiple of ab (x2 + 1) + x 116. If a, b and c satisfy the equation x3 – 3x2 + 2x + 1 = 0 then
(a2 + b2) and ab (x2 – 1) + x (a2 – b2) ? [2014-II]
1 1 1
(a) (a2x2 – b2) (a + bx) what is the value of + + ? [2015-II]
(b) (a2x2 – b2) (a + bx)2 a b c
(c) (a2x2 – b2) (a – bx) 1
(d) (a2x2 – b2) (a – bx)2 (a) - (b) 2
2
108. Consider the following statements : [2014-II]
1. (a – b – c) is one of the factors of 3abc + b3 + c3 – a3. 1
(c) –2 (d)
2. (b + c – 1) is one of the factors of 3bc + b3 + c3 – 1 2
EBD_7367
120
A- Basic Operation and Factorisation

117. Which one of the following is correct? [2015-II] 126. For what value of k is (x + 1) a factor of x3 + kx2 – x + 2?
(a) (x + 2) is a factor of x4 – 6x3 + 12x2 – 24x + 32 [2016-I]
(b) (x + 2) is a factor of x4 + 6x3 – 12x2 + 24x – 32 (a) 4 (b) 3
(c) (x – 2) is a factor of x4 – 6x3 + 12x2 – 24x + 32 (c) 1 (d) –2
(d) (x – 2) is a factor of x4 + 6x3 – 12x2 + 24x – 32 127. Which of the points P(5, – 1), Q(3, –2) and R(1, 1) lie in the
1 solution of the system of inequations x + y £ 4 and x – y ³ 2?
91
118. If x = , then the value of 3 - is [2015-II] [2016-I]
216 (1 - x)1/ 3 (a) Q and R only (b) P and R only
9 5 (c) P and Q only (d) P Q and R
(a) (b) 128. If x2 = y + z, y2 = z + x and z2 = x + y, then what is the value
5 9
1 1 1
4 4 of + + ? [2016-II]
(c) (d) x +1 y +1 z +1
9 5
119. If the equations x – px + q = 0 and x2 + qx – p = 0 have a
2 (a) –1 (b) 1
common root, then which one of the following is correct? (c) 2 (d) 4
[2016-I] 129. 2122 + 462 + 842 + 464 + 2130 is divisible by which one of
(a) p – q = 0 (b) p + q – 2 = 0 the following integers ? [2016-II]
(c) p + q – 1 = 0 (d) p – q – 1 = 0 (a) 3 (b) 5
120. If (s – a) + (s – b) + (s – c) = s, then the value of (c) 7 (d) 11
130. If 4x + 3a = 0, then. what is the value of
(s – a)2 + (s – b)2 + (s – c) + s 2 , will be [2016-I] x 2 + ax + a 2 x 2 - ax + a 2
a2 + b2 + c 2 - ? [2016-II]
(a) 3 (b) 1 x3 - a3 x3 + a 3
(c) 0 (d) –1 4 7
121. If the polynomial x6 + px5 + qx4 – x2 – x – 3 is divisible by (a) - (b)
7a a
(x4 – 1), then the value of p2 + q2 is [2016-I]
(a) 1 (b) 9 32 24
(c) - (d)
(c) 10 (d) 13 7a 7a
122. Let m be a non-zero integer and n be a positive integer. Let
R be the remainder obtained on dividing the polynomial x2 1
+ m2 by (x – m). Then [2016-I] 131. If x + = 2, then what is x equal to ? [2016-II]
1
(a) R is a non zero even integer 1+
1
(b) R is odd, if m is odd 2+
3
(c) R + t3 for some integer s, if n is even
(d) R = t3 for some integer t, if 3 divides n 7 13
(a) (b)
a + 2b + a – 2b 10 10
123. If x =
a + 2b – a – 2b 11 17
then bx2 – ax + b is equal to (given that b ¹ 0) [2016-I] (c) (d)
10 10
(a) 0 (b) 1 132. (x + 4) is a factor of which one of the following expressions ?
(c) ab (d) 2ab [2017-I]
(a) x2 – 7x + 44 (b) x2 + 7 x – 44
124. If a3 = 117 + b3 and a = 3 + b, then the value of a + b is (c) x2 – 7x – 44 (d) x2 + 7x + 44
(given that a > 0 and b > 0) [2016-I]
1 t
(a) 7 (b) 9 133. If x = and y = , t > 0, t ¹, then what is the relation
(c) 11 (d) 13 t -1
t t 1
t -

between x and y ? [2017-I]


a b c
125. If = = , then which of the following is/are correct? (a) yx = x1/y (b) x1/y = y1/x
b c d (c) xy = yx (d) xy = y1/x
134. If A : B = 3 : 4, then what is the value of the expression
b3 + c 3 + d 3 d
1. = æ
a 3 + b3 + c 3 a ç
3A 2 + 4B ö÷
çè 3A - 4B2 ÷ø ? [2017-I]
2 2 2
a +b +c a
2. =
2
b +c +d 2 2 d 43 43
(a) (b) -
Select the correct answer using the code given below. [2016-I] 55 55
(a) 1 only (b) 2 only 47
(c) Both 1 and 2 (d) Neither 1 nor 2 (c) (d) Cannot be determined
55
Basic Operation and Factorisation A-121
135. If x = 2 + 22/3 + 21/3, then what is the value of x3 – 6x2 + 6x? 145. The factors of x (x + 2) (x + 3) (x + 5) – 72 are
[2017-I] (a) x, (x + 3), (x + 4) and (x – 6)
(a) 3 (b) 2 (b) (x – 1), (x + 6) and (x2 – 2x – 12)
(c) 1 (d) 0 (c) (x – 1), (x + 6) and (x2 + 5x + 12)
(d) (x + 1), (x – 6) and (x2 – 5x – 12)
x 24 y 146. a, b, c, b are non-zero integers such that (ab) divides (cd). If
136. If = + and x + y = 26, then what is the value of
y 5 x a and c are coprime, then which one of the following is
correct? [2017-II]
xy? [2017-I] (a) a is a factor of c (b) a is a factor of b
(a) 5 (b) 15 (c) a is a factor of d (d) d is a factor of a
(c) 25 (d) 30
137. If a + b = 5 and ab = 6, then What is the value of a 3 + b3 ? ' a2
(a) 35 (b) 40 147. If ab + bc + ca = 0, then what is the value of
a 2 - bc
(c) 90 (d) 125
138. What are the factors of x3 + 4x2 – 11x – 30? [2017-II] b2 c2
(a) (x – 2), (x + 3) and (x + 5) + + ? [2017-II]
b2 - ca c2 - ab
(b) (x + 2), (x + 3) and (x – 5)
(a) 3 (b) 0
(c) (x + 2), (x – 3) and (x + 5) (c) 1 (d) –1
(d) (x + 2), (x – 3) and (x – 5)
139. If x = 111 .... 1 (20 digits), y = 333 .... 3 (10 digits) and ( x - y )( y - z)( z - x )
148. What is equal to? [2017-II]
x-y 2 ( x - y)3 + ( y - z)3 + ( z - x )3
z = 222 .... 2 (10 digits), then what is equal to?
z 1 1
(a) - (b)
[2017-II] 3 3
1 (c) 3 (d) –3
(a) (b) 1 149. 517 + 518 + 519 + 520 is divisible by [2018-I]
2 (a) 7 (b) 9
(c) 2 (d) 3 (c) 11 (d) 13
140. If 5x3 + 5x2 – 6x + 9 is divided by (x + 3), then the remainder
is [2017-II] b z c x ab + xy
150. If + = 1 and + = 1 , then what is equal to?
(a) 135 (b) – 135 y c z a bx
(c) 63 (d) – 63 [2018-I]
141. The quotient of 8x3 – y3 when divided by 2xy + 4x2 + y2 is (a) 1 (b) 2
[2017-II] (c) 0 (d) –1
(a) 2x + y (b) x + 2y 6
(c) 2x – y (d) 4x – y a2 -1 a -1
151. If = 5 , then what is the value of ? [2018-I]
142. If (x + 2) is a common factor of x 2 + ax + b and a a3
x2 + bx + a, then the ratio a : b is equal to (a) 125 (b) –125
[2017-II] (c) 140 (d) –140
(a) 1 (b) 2 152. If (x + 3) is a factor of x3 + 3x2 + 4x + k, then what is the value
(c) 3 (d) 4 of k? [2018-I]
143. Let (a) 12 (b) 24
f(x) = a0xn + a1xn–1 + a2xn–2 + .... + an–1 x + an, where (c) 36 (d) 72
a0, a1, a2, ...., an are real numbers. 153. Which one of the following is a zero of the polynomial 3x3
If f(x) is divided by (ax – b), then the remainder is 4x2 – 7? [2018-I]
(a) 0 (b) 1
[2017-II]
(c) 2 (d) –1
æ bö æ bö 154. The remainder when 3x3 + kx2 + 5x – 6 is divided by
(a) fç ÷ (b) f ç- ÷ (x + 1) is –7. What is the value of k? [2018-I]
è aø è aø
(a) –14 (b) 14
æ aö æ aö (c) –7 (d) 7
(c) fç ÷ (d) f ç- ÷ 155. If f(x) and g(x) are polynomials of degree p and q respectively,
è bø è bø
then the degree of {f(x) ± g(x)} (if it is non-zero) is [2018-I]
144. The product of the polynomials (x + 2), (x – 2), (a) Greater than min (p, q)
(x3 – 2x2 + 4x – 8) and (x3 + 2x2 + 4x + 8) is [2017-II] (b) Greater than max (p, q)
(a) x8 – 256 (b) (x4 – 16)2 (c) Less than or equal to max (p, q)
(c) (x4 + 16)2 (d) (x2 – 4)4 (d) Equal to min (p, q)
EBD_7367
A- 122 Basic Operation and Factorisation

HINTS & SOLUTIONS


1. (b) The expression 3x3 – kx2 + 4x + 16 is divisible by
x 1
k Þ =
x- . x + y + z x +1
2
1 y
k Similarly, =
Then, x = satisfy the equation y +1 x + y + z
2
1 z
ækö
3
ækö ækö
2 and =
Þ 3 ç ÷ - k ç ÷ + 4 ç ÷ + 16 = 0 z +1 x + y + z
è2ø è2ø è2ø
1 1 1
3 3 \ + +
3k - 2k + 16k + 128 x + 1 y +1 z + 1
Þ = 0
8
x y z
Þ k3 + 16k + 128 = 0 = + +
Þ (k + 4) (k2 – 4k + 32) = 0 x+y+z x+y+z x +y+z
Þ k+4=0 x+ y+z
Þ k=–4 = =1
2. (c) x4 + xy3 + xz3 + x3 y + y4 + yz3 x+ y+z
= x (x3 + y3 + z3) + y (x3 + y3 + z3) 6. (c) Here, p3 + q3 – r3
= (x + y) (x3 + y3 + z3) = (p + q) (p2 + q2 – pq) – r3
Hence, (x3 + y3 + z3) is a factor of = (p + q) [(p + q)2 – 3pq] – r3
x4 + xy3 + xz3 + x3 y + y4 + yz3
é 30 ù
= r ê r 2 - 3 × ú - r3
1 1 1 ë r û
3. (a) + +
1 + p + q -1 1 + q + r -1 1 + r + p -1 (Q p + q = r given and pqr = 30)
= r3 – 90 – r3 = –90
1 1 1
= + +
1 1 1 x8 + 4 x8 + 4x 4 + 4 - 4x 4
1+ p + 1+ q + 1+ r + 7. (b) =
q r p x 4 + 2x 2 + 2 x 4 + 2x 2 + 2

q r p (x 4 + 2)2 - (2x 2 )2
= + + =
1 + pq + q r + rq + 1 p + rp + 1 x 4 + 2x 2 + 2
q r p (x 4 + 2x 2 + 2) (x 4 - 2x 2 + 2)
= + + (Q pqr = 1) =
1 + pq + q 1 1 1
+ +1 p + +1 x 4 + 2x 2 + 2
pq p q
= x4 – 2x2 + 2
q rpq pq 8. (c) Given, 2x – 2x – 1 = 4
= + +
1 + pq + q 1 + q + pq pq + 1 + q æ 1ö
Þ 2x ç1 – ÷ = 4
è 2ø
q + rpq + pq
= (Q pqr = 1) x
Þ2 =8
1 + pq + q
Þ 2x = 23
q + 1 + pq Þx=3
= =1 \ 2x + 2x – 1 = 23 + 23 – 1 = 8 + 4 = 12
1 + pq + q
4. (a) Given, x + y + z = 2s 1
9. (d) Given, x + =p
Also, (s – x) + (s – y) – z = 2s – (x + y + z) x
= 2s – 2s = 0
2
\ (s – x)3 + (s – y)3 – z3 + 3(s – x) (s – y) (z) = 0 æ 1ö
(Q a + b + c = 0 Þ a3 + b3 + c3 = 3abc) Þ çx + ÷ = p2
è xø
Þ (s – x) + (s – y)3 + 3(s – x) (s – y) (z) = z3
3

5. (a) Given, x2 = y + z 1
Þ x2 + x = x + y + z Þ x2 + + 2 = p2
x2
Basic Operation and Factorisation 123
A-

15. (c) x2 (y – z) + y2 (z – x) – z(xy – yz – zx)


1
Þ x +2
=p –2 2
... (i) = x2 y – x2 z + y2 z – y2 x – zxy + yz2 + z2 x
x2 = xy(x – y – z) – z (x2 – y2) + z2 (x + y)
3 = xy(x – y – z) – z(x + y) (x – y – z)
æ 1 ö
Þ ç x 2 + 2 ÷ = (p2 – 2)3 = (x – y – z) (xy – yz – zx)
è x ø 16. (a) Required remainder
= 3(2y)3 – 2 (2y)2 y – 13(2y)y2 + 10y3
1 æ 1 ö
Þ x6 + + 3ç x2 + = p6 – 8 – 6p2 (p2 – 2) = 24y3 – 8y3 – 26y3 + 10y3
6 2 ÷
x è x ø = 34y3 – 34y3 = 0
1 17. (b) Given that,
6 2 6 4 2
Þ x + + 3(p – 2) = p – 8 – 6p + 12p
x6 1
=
11
1+
[from equation (i)] ì 1 ü 7
ï ï
1 1 + íæ 1 öý
ïèç 1 + ÷
Þ x6 + = p6 – 6p4 + 9p2 – 2 î x ø ïþ
x6
10. (c) Given, x + y + z = 0
1 11
3 3 3 Þ 1+ =
æy–z–xö æz – x – yö æx – y–zö ì x ü 7
\ ç ÷ +ç ÷ +ç ÷ í1 + ý
è 2 ø è 2 ø è 2 ø î 1+ xþ
=
1 + x 11
3 3 3 Þ 1+ =
æ y – (z + x ) ö æ z – ( x + y) ö æ x – (y + z) ö 1 + 2x 7
çç ÷÷ + çç ÷÷ + çç ÷÷
è 2 ø è 2 ø è 2 ø 1 + 2x + 1 + x 11
Þ =
1 + 2x 7
3 3 3
æ y – ( –y ) ö æ z – ( –z ) ö æ x – ( –x ) ö 2 + 3x 11
= çç ÷÷ + çç ÷÷ + çç ÷÷ Þ =
è 2 ø è 2 ø è 2 ø 1 + 2x 7
Þ 14 + 21x = 11 + 22x
3 3 3 \ x=3
æ 2y ö æ 2z ö æ 2x ö
= ç ÷ +ç ÷ +ç ÷ 18. (a) Given, x + y + z = 0
è 2 ø è 2 ø è 2 ø
\ (x + y) (y + z) (z + x) = (–z) (–x) (–y) = –xyz
= y3 + z3 + x3 19. (b) Given, a + b = 3
= 3xyz \ (a + b)3 = 33
(since, a3 + b3 + c3 = 3abc, if a + b + c = 0) Þ a3 + b3 + 3ab(a + b) = 27
11. (d) m and n are odd and m > n. Þ a3 + b3 + 9ab = 27 (Q a + b = 3)
Let m = 3, n = 1
\ m 2 – n 2 = 32 – 1 = 8 20. (c) x +5
Again, let m = 5, n = 3 2 3 2
\ m2 – n2 = 52 – 32 = 16 x + 2 x + 5x + 10k
So, it is clear that m2 – n2 is always divisible by 8. x3 + 2x
– –
1 1
12. (a) 2 + 2 + -
2+ 2 2- 2 5x 2 – 2x + 10k
5x 2 + 10
2- 2 - 2- 2 – –
= 2+ 2 +
4-2 – 2x + 10k – 10

(- 2 2) But remainder is –2x.


= 2+ 2 + = 2+ 2 - 2 = 2
2 \ –2x = –2x + 10k – 10 Þ k = 1.
13. (d) When the volume increases upto 60 cm3, then 21. (b) Q (5x2 + 14x + 2)2 – (4x2 – 5x + 7)2
temperature 300 K. = (5x2 + 14x + 2 – 4x2 + 5x – 7)
\ Volume is increased to 100 m3, temperature (5x2 + 14x + 2 + 4x2 – 5x + 7)
2 2
300 ´ 100 = (x + 19x – 5) (9x + 9x + 9)
= = 500 K = 9(x2 + 19x – 5) (x2 + x + 1)
60
14. (d) Now, (x2)3 – y3 = (x2 – y) (x4 + y2 + x2 y) From above it is clear that (x2 + x + 1) is a factor of
{(5x2 + 14x + 2)2 – (4x2 – 5x + 7)2}, then remainder is
Hence, the other factor is x2 – y.
zero.
EBD_7367
124
A- Basic Operation and Factorisation

22. (a) For x = 1,


( ) + (2 - 3 )
3 3
(1)29 – (1)24 + (1)13 – 1 = 1 – 1 + 1 – 1 = 0 = 2+ 3
So, (x – 1) is a factor of x29 – x24 + x13 – 1.
( 3) ( ) ( 3)
3 3
+ 3 × 2 × 3 2 + 3 + (2) -
3
For x = –1, = 23 +
(–1)29 – 1(–1)24 + (–1)13 – 1 = –1 – 1 – 1 – 1 = –4
So, (x + 1) is not a factor of x29 – x24 + x13 – 1. -3×2× 3 (2 - 3 )
23. (b) (x + y)–1 (x–1 + y–1) (xy–1 + x–1y)–1
( 3) ( 3 ) - 12
3 3
-1 =8+ + 12 3 + 18 + 18 - 3 +8
æ 1 ö æ1 1ö æx yö
=ç + +
è x + y ÷ø çè x y ÷ø çè y x ÷ø = 8 + 18 + 8 + 18 = 52
If x = 2 - 3,
-1
æ 1 ö æ x + y ö æ x 2 + y2 ö 1
=ç çè xy ÷ø ç xy ÷ then x3 + = 52
è x + y ÷ø è ø x3
28. (a) Given, x + y + z = 6 and xy + yz + zx = 11
1 xy 1
= × = = (x2 + y2)–1. \ x3 + y3 + z3 – 3xyz
2 2
xy x + y x + y2
2
= (x + y + z) [(x + y + z)2 – 3 (xy + yz + zx)]
24. (c) Given, ab – b + 1 = 0 = 6 [62 – 3 (11)] = 6 × 3 = 18
Þ b(a – 1) = –1 29. (d) x3 – 3x2 – 3x + 9 = (x2 – 3) (x – 3)
1 So, the factors are 3, - 3 and 3.
Þ b= ... (i)
1- a 30. (c) Let a be the common factors of the given equations.
Also, bc – c + 1 = 0 \ a2 – 11a + a = 0 and a2 – 14a + 2a = 0
-1 + c Thus, it represents the same equation
Þ b= ... (ii)
c a2 a 1
From equations (i) and (ii), \ = =
-22a + 14a a - 2a -14 + 11
1 -1 + c (cross multiplication method)
= Þ c = (1 – a) (–1 + c)
1- a c
a2 a 1
Þ c = –1 + c + a – ac Þ a – ac = 1 Þ = =
-8a -a -3
25. (b) Let S = {... – 6, – 4, – 2, 0, 2, 4, 6, ...}
I. Now, 2 + (– 2) = 0 Î S, it is applied. a 1
Þ =-
II. Now, – 2 –2 = –4 Î S, it is applied. -a 3
III. Now, – 4 × (4) = –16 Î S, it is applied.
IV. Now, – 4 ÷ 4 = –1 Ï S, it is not applied. a a2 1
Þ a= and =-
3 -8a 3
26. (c) x -1
a2
x + 1 x2 + 1
9 =-1
x2 + x Þ
– – -8a 3
Þ a = 0, a = 24.
– x +1
31. (c) Let f(x) = 2x3 – 3x2 – 11x + 6
– x –1
+ + Put x = –2, we get
2 f(–2) = 2(–2)3 – 3(–2)2 – 11(–2) + 6
= –16 – 12 + 22 + 6 = 0
Hence, r(x) = 2. Hence, (x + 2) is a factor of f(x).
27. (c) Given, equation is x2 – 4x + 1 = 0
32. (b)
9x 2 - 2
4 ± 16 - 4 ´ 1 ´ 1 4±2 3
\ x= = =2± 3 3x - 1 27x 3 - 9x 2 - 6x - 5
2 ´1 2
27x 3 – 9x 2
If x = 2 + 3, then – +
– 6x – 5
3
æ 1 ö – 6x + 2
1
( )
3
x3 + = 2+ 3 +ç +
x 3 ç 2 + 3 ÷÷ –
è ø –7
Basic Operation and Factorisation 125
A-

33. (b) Given, p « q = 2p + 2q – pq 40. (c) Given, x = 1 + 2


Q8«x=4 \ x4 – 4x3 + 4x2 = x2 (x2 – 4x + 4) = x2 (x – 2)2
Þ 2(8) + 2(x) – 8x = 4
( ) (1 + )
Þ –6x = –12 2 2
= 1+ 2 2 -2
\x=2

( )( )
2 2
æ x 2 - 3x + 2 ö æ x 2 - 9 ö æ x3 + 2x 2 + 4x ö = ( 2 - 1) = 1
2
= 1+ 2 2 -1
34. (a) ç ÷¸ç ÷´ç ÷
ç x 3 - 8 ÷ ç x 2 + 7x + 12 ÷ ç x 2 + 3x - 4 ÷
è ø è ø è ø x z
41. (a) Given, =
æ x 2 - 3x + 2 x 2 + 7x + 12 ö x3 + 2x 2 + 4x y w
=ç ´ ÷´
ç x3 - 8 x 2
- 9 ÷ x 2 + 3x - 4 Þ xw = yz
è ø Now, (xy + zw)2 = x2 y2 + z2 w2 + 2(xy . zw)
(x - 1) (x - 2) (x + 4) (x + 3) = x2 y2 + z2 w2 + 2(yz × yz)
= ´ = x2 y2 + y2 z2 + z2 w2 + y2 z2
2
(x - 2) (x + 4 + 2x) (x - 3) (x + 3)
= y2 (x2 + z2) + z2 w2 + x2 w2
= y2 (x2 + z2) + w2 (x2 + z2)
x (x 2 + 2x + 4)
´ = (x2 + z2) (y2 + w2)
(x - 1) (x + 4)
1 2 1009
42. (b) Given, + + =1
x x + 1 y + 2 z + 1009
=
x-3
1 2 1009
é (2.3)3 - 0.027 ù Þ -1+ -1+ -1 = 1- 3
x +1 y+2 z + 1009
35. (b) ê 2
ú
ëê (2.3) + 0.69 + 0.09 ûú x y z
Þ - - - = -2
3
(2.3) - (0.3) 3 x + 1 y + 2 z + 1009
=
(2.3) + 2.3(0.3) + (0.3) 2
2
x y z
Þ + + =2
= 2.3 – 0.3 = 2 x + 1 y + 2 z + 1009
[Q a3 – b3 = (a – b) (a2 + ab + b2)] 43. (d) Q 1 quintal = 100 kg
36. (b) 9 2 - 8 - 4 2 3 3
\ quintal = 100 × = 150 kg
2 2
=9 2 -2 2 - 4 2 =3 2
37. (b) x (y – z) (y + z) + y(z – x) (z + x) + z(x – y) (x + y) 15
Given, kg = 1 unit
= x(y2 – z2) + y(z2 – x2 ) + z(x2 – y2) 4
= x(y2 – z2) + yz2 – yx2 + zx2 – zy2 4
= x(y – z) (y + z) + x2 (z – y) + yz(z – y) 1 kg = unit
15
= (y – z) (xy + xz – x2 – yz)
= (y – z) [y(x – z) + x(z – x)] 4
\ 150 kg = × 150 = 40 units
= (y – z) (z – x) (x – y) 15
= (x – y) (x – z) (z – y) 44. (c) Given, m = n2 – n
38. (d) Let f(x) = a0 + a1 x + a2 x2 + ..... + an xn \ m2 – 2m = (n2 – n)2 – 2(n2 – n)
= n(n – 1) (n2 – n – 2)
\ f(1) = a0 + a1 + a2 + ..... + an
= (n + 1) n (n – 1) (n – 2)
Þ 1 = a0 + a1 + a2 + ..... + an So, it is a product of consecutive number.
Þ 1 – a0 – a2 – ..... = a1 + a3 + ..... Therefore, it is divisible by 24.
39. (c) 45. (c) Let f(x) = x40 + 2
x
Put x4 = –1,
x 2 - 2 x - 3 x 3 - 2 x 2 - px - q f(x) = (–1)10 + 2 = 3
46. (b) Factor theorem is a special case of remainder
x 3 – 2 x 2 – 3x
– + + theorem.
( p + 3) x – q 1 æ 1ö
2
47. (c) \ x2 + =çx + ÷ – 2
2 xø
According to question, x è
(p + 3)x – q = x – 6
2
Þ p + 3 = 1, q = 6 æ 1 ö
Þ p = –2, q = 6 = ç2 + 3 + ÷ -2
ç 2 + 3 ÷ø
è
EBD_7367
126
A- Basic Operation and Factorisation

2
52. (b) Let f(x) = px3 + 3x2 – 3 and g(x) = 2x3 – 5x + p
æ 2- 3ö At x = 4,
= çç 2 + 3 + ÷ - 2 = 16 – 2 = 14 f(4) = p(4)3 + 3(4)2 – 3
è 1 ÷ø
= 64p + 48 – 3 = 64p + 45
1+ x and g(4) = 2(4)3 – 5(4) + p
48. (c) Given, a = = 128 – 20 + p = 108 + p
2-x
But f(4) = g(4) (given)
æ 1+ x ö \ 64p + 45 = 108 + p
3ç ÷ Þ 63p = 63 Þ p = 1
1 2a + 1 3a 2-x ø
\ + = = è 53. (c) Given, x(x + y + z) = 9 ... (i)
a + 1 a 2 - 1 a 2 - 1 æ 1 + x ö2 y(x + y + z) = 16 ... (ii)
ç ÷ -1 and z(x + y + z) = 144 ... (iii)
è 2-x ø
On adding (i), (ii) and (iii), we get
3(1 + x) (2 - x) (x + y + z) (x + y + z) = 9 + 16 + 144
= Þ (x + y + z)2 = 169
1 + x + 2 x - (4 + x 2 - 4 x)
2
Þ x + y + z = 13 [ Q x(x + y + z) = 9]
3(1 + x) (2 - x ) (1 + x ) (2 - x) 9
= = Þ x(13) = 9 Þ x =
6x - 3 (2 x - 1) 13
49. (b) Given, pq + qr + rp = 0 54. (c) Given, (u)3 + (–2v)3 + (–3w)3 = 3 × (–2) (–3)uvw
\ u + (–2v) + (–3w) = 0
p2 q2 r2 Þ u – 2v – 3w = 0
\ + +
p 2 - qr q 2 - rp r 2 - pq Þ u – 2v = 3w
55. (a): x4 + 4y4 = x4 + 4y4 + 4x2y2 – 4x2 y2
p2 q2 r2 = (x2 + 2y2)2 – (2xy)2
= + +
p 2 + rp + pq q 2 + pq + qr r 2 + qr + rp = (x2 + 2y2 – 2xy) (x2 + 2y2 + 2xy)
From above it is clear that x4 + 4y4 is divisible by
p q r x2 + 2y2 + 2xy.
= + + 56. (b) At the same time, ratio of height and length of
p+r +q q+ p+r r+q+ p
shadow of an object are same.
p+q+r 6 Height of tree
= =1 \ =
p+q+r 15 25
50. (d) Given, x + y + z = 0 Þ Height of tree = 10m
1 1 57. (d) Given, a + b + c = 0
Now, = ... (i) Þ a3 + b3 + c3 = 3abc
x2 + y 2 - z 2 z 2 - 2 xy - z 2
a3 b3 c3
1 Þ + + =3
=- abc abc abc
2xy
a2 b2 c2
1 1 1 Þ + + =3
\ + + bc ac ab
x2 + y 2 - z 2 y2 + z2 - x2 z2 + x2 - y2
1
58. (d) Given, x 4 + 4 = 322
1 1 1 x
= + +
-2 xy -2 yz -2 zx 2
æ 1 ö
Þ ç x 2 + 2 ÷ - 2 = 322
1æ z+ x+ yö è x ø
= ç ÷
2 è xyz ø 2
æ 2 1 ö 2
=0 [from equation (i)] Þ ç x + 2 ÷ = 324 = 18
51. (c) We know that, if a + b + c = 0 è x ø
\ a3 + b3 + c3 = 3abc 1
Here, x – y + y – z + z – x = 0 Þ x2 + = 18
x2
( x - y )3 + ( y - z )3 + ( z - x ) 3 2
\ æ 1ö
4 ( x - y ) ( y - z ) ( z - x) Þ ç x - ÷ + 2 = 18
è xø
3( x - y ) ( y - z ) ( z - x) 3
= = 1
4 ( x - y )( y - z ) ( z - x ) 4 Þ x- =4
x
Basic Operation and Factorisation 127
A-

59. (c) Given, x = (b – c) (a – d), y = (c – a) (b – d) 67. (a) Let f(x) = x11 + 1


and z = (a – b) (c – d) Put x = –1, we get
\ x + y + z = (b – c) (a – d) + (c – a) (b – d) f (–1) = (–1)11 + 1 = –1 + 1 = 0
+ (a – b) (c – d) = 0 (an + bn), where n is odd number then it is divisible
\ x3 + y3 + z3 = 3xyz by (a + b). Then remainder is zero.
60. (a) x2 + 4y2 + 4y – 4xy – 2x – 8 68. (a) Let f(x) = x2 + 4px – 11p
= (x – 2y)2 – 2(x – 2y) – 8 Since, (x – 3) is a factor of f(x).
= (x – 2y)2 – 4(x – 2y) + 2(x – 2y) – 8 \ f(3) = 0
= (x – 2y – 4)(x – 2y + 2) Þ (3)2 + 4p(3) – 11p = 0
61. (d) Here, px3 + x2 – 2x – q is divisible by (x – 1) and (x Þ p = –9
+ 1)
\ p(1)3 + (1)2 – 2(1) – q = 0 x2 - 3 x + 2 x2 - 5 x + 4
69. (b) ¸
Þ p–q=1 ... (i) x2 - 5 x + 6 x 2 - 7 x + 12
and p (–1)3 + (–1)2 – 2 (–1) – q = 0
Þ p+q=3 ... (ii) x 2 - 3x + 2 ( x 2 - 7 x + 12)
= ´
From equations (i) and (ii), x2 - 5 x + 6 ( x 2 - 5x + 4)
p = 2 and q = 1
62. (d) On putting = 3 in x5 – 9x2 + 12x – 14, we get ( x -1) ( x - 2) ( x - 4) ( x - 3)
= ´
Remainder = (3)5 – 9(3)2 + 12 × 3 – 14 ( x - 3) ( x - 2) ( x - 4) ( x - 1)
= 243 – 81 + 36 – 14 = 184 =1
1 1
63. (b) Given, x - = (2.3)3 - 0.027 (2.3)3 - (0.3) 3
x 3 70. (d) =
(2.3) 2 + 0.69 + 0.09 (2.3) 2 + 0.69 + 0.09
3
Þ 3x - = 1
x (2.3 - 0.3) [(2.3) 2 + (0.3) 2 + 2.3 ´ 0.3)]
Squaring both sides, =
(2.3) 2 + 0.69 + 0.09
9
9x2 + –2×9=1 2 [(2.3) 2 + 0.09 + 0.69]
x2 = =2
(2.3) 2 + 0.69 + 0.09
9
Þ 9x2 + = 19 71. (a) Given, x + y + z = 0
x2
xyz xyz
1 1 \ =
64. (d) + ( x + y) ( y + z ) ( z + x) (- z ) (- x) (- y)
1+ 2 + 3 1- 2 + 3
xyz
1 1 = = -1
= + - xyz
(1 + 3) + 2 (1 + 3) - 2
72. (b) Here, a = 1.001 and b = 0.999 (assume)
1+ 3 - 2 +1+ 3 + 2 2 (1 + 3)
= = =1 a 3 + b3 (a + b) (a 2 - ab + b 2 )
2
(1 + 3) - 2 2 (1 + 3) Now, = =a+b
a 2 - ab + b 2 a 2 - ab + b 2
65. (d) Q (a + b + c)2 = a2 + b2 + c2 + 2(ab + bc + ca)
(6)2 = 26 + 2(ab + bc + ca) = 1.001 + 0.999 = 2
Þ 2(ab + bc + ca) = 10 73. (d) Required number of points = 4(P1 P2 P3 P4)
Þ ab + bc + ca = 5
66. (a) 2 2 C
3 x + 4 xy - 5 y
x - 2 y 3 x 3 - 2 x 2 y - 13 xy 2 + 10 y 3
3 x3 – 6 x 2 y
– +
5 cm 5 cm
4 x 2 y – 13 xy 2
3 cm 3 cm
4 x 2 y – 8 xy 2
– +
21 cm 21 cm
– 5 xy 2 + 10 y 3 P1 P2
2 3 1 cm
– 5 xy + 10 y A B
+ – 1 cm
´ P3 P4
3 cm 3 cm
Thus, the remainder is zero.
EBD_7367
128
A- Basic Operation and Factorisation

74. (d) According to question (2x – 1) is a factor of 79. (b) Given 3x + y = 81


4x4 – (k – 1)x3 + kx2 – 6x + 1 ... (i) Þ 3x + y = 34 Þ x + y = 4 ... (i)
1 81x – y = 3 Þ (34)x – y = 31
On putting x = in Eq. (i), we get 1
2
Þ x–y= ... (ii)
4 3 2 4
æ1ö æ1ö æ1ö 1
4 ´ ç ÷ - (k - 1) ´ ç ÷ + k ´ ç ÷ - 6 ´ + 1 = 0 From equations (i) and (ii),
è2ø è2ø è2ø 2
1 17
1 ( k - 1) k then, x + y = 4 Þ x – y = Þ 2x =
Þ - + –2=0 4 4
4 8 4
17
k ( k - 1) 1 \ x=
Þ - =2– 8
4 8 4 80. (c) Given, x4 + xy3 + x3 y + xz3 + y4 + yz3
2k - ( k - 1) 8 - 1 7 = (x4 + xy3 + xz3) + (x3y + y4 + yz3)
Þ = = = x(x3 + y3 + z3) + y(x3 + y3 + z3)
8 4 4
Þ 8k – 4k + 4 = 56 = (x + y) (x3 + y3 + z3)
Þ 4k = 52 Thus, the equation is divisible by both (x + y) and
\ k = 13 (x3 + y3 + z3).
75. (a) f(x) = x3 + x2 – x – 1 81. (b) Given, 3 × 0.3 × 0.03 × 0.003 × 0.0003 × 30
g(x) = x3 – x2 + x – 1 = 3 × 3 × 10–1 × 3 × 10–2 × 3 × 10–3 × 3
p(x) = f {g(x}) = p(x) × f(x) × 10–4 × 3 × 10
6 –9 2 3 –3 3
f(x) . g(x) = (x3 + x2 – x – 1) (x3 – x2 + x – 1) = (3) × (10) = (3 ) (10 )
= x6 – x5 + x4 – x3 + x5 – x4 + x3 – x2 3
1 æ 9 ö
– x4 + x3 – x2 + x – x3 + x2 – x + 1 = (9)3 × = ç 3
÷ = (0.009) .
.
p(x) = f(x) g(x) = x6 – x4 – x2 + 1 (1000)3 è 1000 ø
p(x) = x4 (x2 – 1) – 1(x2 – 1) 82. (c) We know that,
= (x2 – 1) (x4 – 1) = (x – 1) (x + 1) [(x2)2 – 1] a3 + b3 + c3 – 3abc
= (x – 1) (x + 1) (x2 – 1) (x2 + 1) = (a + b + c)(a2 + b2 + c2 – ab – bc – ca)
= (x – 1) (x + 1) (x – 1) (x + 1) (x2 + 1)
= (x – 1)2 (x + 1)2 (x2 + 1) æ a3 + b3 + c3 - 3abc ö
Þ (a + b + c ) = ç ÷ ... (i)
ç a 2 + b 2 + c 2 - ab - bc - ca ÷
76. (c) x+5 è ø
Given that,
x + 2 x + 5 x 2 + 10k
2 3

x3 + 2 x é(2.247)3 + (1.730)3 + (1.023)3 - 3 ´ 2.247 ù


– – ê ú
êë ´ 1.730 ´ 1.023úû
5 x 2 – 2 x + 10k é(2.247)2 + (1.730)2 + (1.023)2 - ( 2.247 ´ 1.730 ) ù
2
5 x + 10 ê ú
– – êë - (1.730 ´ 1.023) - ( 2.247 ´ 1.023) úû
–2 x – 10 + 10k = Remainder = (2.247 + 1.730 + 1.023) [from Eq. (i)]
= 5.000 = 5
Given, remainder = –2x
83. (c)
\ –2x – 10 + 10k = –2x a 2 - 2a - 8
Þ 10k = 10
a 2 - 1 a 4 - 2 a3 - 9 a 2 + 2 a + 8
Þ k=1
1 a4 – a2
77. (c) Given that, x + = a – +
x
– 2 a3 – 8a 2 + 2 a + 8
1 1 æ 1 ö æ 2 1 ö
Then, x3 + x 2 + + = ç x3 + ÷ + çx + 2 ÷ – 2 a3 + 2a
x 3
x2
è x3 ø è x ø + –

3 2 – 8a 2 +8
æ 1ö æ 1ö æ 1ö
= ç x + ÷ - 3ç x + ÷ + ç x + ÷ - 2 – 8a 2
+8
è xø è xø è xø + –
= a3 – 3a + a2 – 2 = a3 + a2 – 3a – 2 ´
78. (a) Given, xyz = 1, ax = b, by = c
b = ax Þ by = axy Required factor is (a2 – 2a – 8)
Þ byz = axyz Þ cz = a Further its factorize by spilt the middle term,
Basic Operation and Factorisation A- 129
= a2 – 4a + 2a – 8 90. (c) Given expression = 5px – 10qy + 2rpx – 4qry
= a(a – 4) + 2(a – 4) = (5px + 2rpx) – (10qy + 4qry)
= (a – 4) (a + 2) = px(5 + 2r) – 2qy(5 + 2r)
Other two factors are (a + 2) and (a – 4). = (5 + 2r)(px – 2qy)
84. (b) Given, 0.764 y = 1.236 x 91. (a) Zero of (x + 6) is –6, i.e., x + 6
y 1.236 Þ x=–6
Þ = ... (i) If (x + 6) is a factor of the expression
x 0.764
Now, f(x) = x3 + 3x2 + 4x + p
Then, f(–6) = 0
y Þ (–6)3 + 3(– 6)2 + 4(–6) + p = 0
-1
y-x x Þ –216 + 108 – 24 + p = 0
=
y+x y \ p = 240 – 108
+1
x = 132
1.236 725 ´ 725 ´ 725 + 371 ´ 371 ´ 371
-1 92. (c) = 725 + 371
0.764 1.236 - 0.764 725 ´ 725 - 725 ´ 371 + 371 ´ 371
Þ =
1.236 1.236 + 0.764
+1 æ ö
0.764 a 3 + b3 (a + b) (a 2 - ab + b2 )
çQ 2 = = a +b÷
ç a - ab + b2 2
(a - ab + b )2 ÷
0.472 è ø
= = 0.236 = 1096
2.000
85. (b) Given that, expression x3 + 4x2 + 4x + k has (x + 4) 93. (d) Statement I :
as a factor, so it will satisfy that expression. When x = –3 then x3 + 2x2 + 3x + 8
Let f(x) = x3 + 4x2 + 4x + k = (–3)3 + 2(–3)2 + 3(–3) + 8
\ f(–4) = 0 (by condition) = –10 ¹ 0
Hence, (x – 3) is not the factor of x3 + 2x2 + 3x + 8
Þ (–4)3 + 4(–4)2 + 4(–4) + k = 0
Statement II :
Þ –64 + 64 – 16 + k = 0
Similarly,
\ k = 16
When x = 2, then x3 + 2x2 + 3x + 8
86. (d) Given expression is f(x) = 4x2 + 12x + k
= (2)3 + 2(2)2 + 3(2) + 8
= (2x)2 + 2.3 . (2x) + k
= 30 ¹ 0
For the value of k Hence, x – 2 is also not the factor of
2 2 x3 + 2x2 + 3x + 8.
é Coefficient of (2 x) ù æ6ö
= ê ú = ç ÷ = (3)2 = 9
ë 2 û è2ø ( x 2 + y 2 ) ( x - y ) - ( x - y )3
94. (b)
Hence, 9 is the value of k which will make the x 2 y - xy 2
expression f(x) a perfect square.
87. (b) Let f(x) = 6x2 + kx + 10 x3 + xy 2 - x 2 y - y 3 - ( x3 - y 3 - 3x 2 y + 3xy 2 )
=
If (x + 5) is a factor of f(x), then x 2 y - xy 2
f(–5) = 0
So, 6(–5)2 + k(–5) + 10 = 0 x3 + xy 2 - x 2 y - y 3 - x 3 + y 3 + 3x 2 y - 3xy 2
=
Þ 6 × 25 – 5k + 10 = 0 x 2 y - xy 2
Þ 5k = 150 + 10 = 160
\ k = 32 2 x 2 y - 2 xy 2 2( x 2 y - xy 2 )
= = =2
88. (d) Let f(x) = x4 – 7x3 + 5x2 – 6x + 81 x 2 y - xy 2 x 2 y - xy 2
By hit and trial method,
95. (b) Here, x + 5 is a factor.
x4 – 7x3 + 5x2 – 6x + 81
So,
= x3 (x – 3) – 4x2 (x – 3) – 7x(x – 3) – 27(x – 3) x+5=0
= (x – 3) (x3 – 4x2 – 7x – 27) Þ x = –5
(x – 3) is one of the factor of the given polynomial. Now, when x = –5, then
89. (d) Given, (a + 1)4 – a4 x3 + 3x2 + 4x + k = (–5)3 + 3 × (–5)2 + 4 × (–5) + k
= {(a + 1)2 – a2}{(a + 1)2 + a2} = –125 + 75 – 20 + k = –70 + k
= {(a + 1) + a}{(a + 1) – a}{a2 + 1 + 2a + a2} since (x – 5) is a factor of x3 + 3x2 + 4x + k
= (2a + 1) (1) (2a2 + 2a + 1) \ –70 + k = 0
= (2a + 1) (2a2 + 2a + 1) \ k = 70
Hence, (2a2 + 2a + 1) is a factor of {(a + 1)4 – a4}.
EBD_7367
130
A- Basic Operation and Factorisation

96. (c) (1 – x) (1 + x2) = 1 – x + x2 – x3 103. (b) Given the f(x) = 2x3 + x2 – 2x + 1


2x3 – x2 + x – 1 is added to 1 – x + x2 – x3 to obtain x3. = x2 (2x + 1) – 1(2x + 1) = (2x + 1) (x2 – 1)
97. (c) (49)2 – (25)2 = 37x = (2x + 1) (x + 1) (x – 1)
Þ 2401 – 625 = 37x Hence, expression is divisible by (2x + 1).
Þ 1776 = 37x
æ 1 ö 17
1776 104. (b) ç x 2 + 2 ÷ =
x= = 48 è x ø 4
37
98. (d) If x –1 is a factor, then x = 1, then 1 17
Þ x2 + + 2-2 =
2
4x5 + 9x4 – 7x3 – 5x2 – 4kx + 3k2 = 0 x 4
Þ 4 × (1)5 + 9 × (1)4 – 7 × (1)3 – 5 × (1)2
2
–4 × k × (1) + 3 × k2 = 0 æ 1ö 17
Þ çx- ÷ +2=
Þ 4 + 9 – 7 – 5 – 4k + 3k2 = 0 è xø 4
Þ 3k 2 – 4k + 1 = 0
Þ 3k2 – 3k – k + 1 = 0 æ 1ö 17
2
Þ 3k (k – 1) – 1(k – 1) = 0 Þ çx- ÷ = -2
è xø 4
Þ (3k – 1) (k – 1) = 0
2
1 æ 1ö 9
k= ,1 Þ çx- ÷ =
3 è xø 4
99. (c) x(y2 – z2) + y(z2 – x2) + z (x2 – y2)
If divisible by (y – z), then y – z = 0 Þ y = z æ 1ö 3
Þ çx- ÷ =
On putting y = z, we get è xø 2
x(z2 – z2) + z(z2 – x2) + z(x2 – z2) On cubing both sides, we get
= z3 – zx2 + zx2 – z3 = 0
3 3
Hence, y – z is a factor, so it is divible by (y – z). æ 1ö æ3ö
Also, if z – x is a factor, then çx- ÷ = ç ÷
è xø è2ø
z–x=0Þz=x
On putting z = x, we get 1 1 æ 1 ö 27
Þ x3 - - 3 ´ .x ç x - ÷ =
x(y2 – x2) + y(x2 – x2) + x(x2 – y2) x 3 x è xø 8
= xy2 – x3 + x3 – xy2 = 0
Hence, (z - x) is also a factor, so it is also divisible 1 27 æ3ö
by (z - x). Þ x3 - = + 3´ç ÷
x3 8 è2ø
100. (d) Let f(x) = x3 + 6x2 + 11x + 6
f(–1) = 0 1 27 9
Þ x3 - = +
\ (x + 1) is a factor of f(x). x 3 8 2
Also f(–2) = f(–3) = 0
Hence (x + 2) and (x + 3) are also factors of f(x). 1 63
\ x3 - 3
=
101. (c) x(x + a) (x + 2a) (x + 3a) x 8
= (x2 + ax) (x2 + 5ax + 6a2) 105. (a) Given, (x + k) is the common factor of x2 + ax + b
= x4 + ax3 + 5ax3 + 5a2 x2 + 6a2 x2 + 6a3 x and x2 + cx + d.
= x4 + ax(x2 + 5x2 + 5ax + 6ax + 6a2) \ k2 – ka + b = 0 ...(i) k2 – kc + d = 0 ...(ii)
= x4 + ax(6x2 + 11ax + 6a2) ... (i) Now, from equation (i) and equation (ii)
So, for terms to be perfect square, k2 – ka + b = k2 – kc + d
= (x + y)2 (x + y)2 = (x2 + 2xy + y2) (x2 + y2 + 2xy) Þ k (c – a) = d – b
= x4 + 2x3 y + x2 y2 + x2 y2 + 2xy3 + y4
d -b
+ 2x3 y + 4x2 y2 + 2xy3 \ k=
= x + xy(4x + 6xy + 4y ) + y4
4 2 2 c-a
On comparing equations. (i) and (ii), as y = a 106. (c) Let f (x) = x5 – 5x2 + 125
a4 must be added to make it a perfect square. \ Required remainder = f (–5) = (–5)2 –5 (–5)2 + 125
102. (b) Let f(x) = 3x4 – 2x3 + 3x2 – 2x + 3 = – 3125 – 125 + 125 = – 3125
107. (a) We have, ab (x2 + 1) + x (a2 + b2).
æ 2ö = abx2 + ab + a2x + b2x = ax(a + bx) + b(a + bx)
Remainder = f ç - ÷
è 3ø = (a + bx) (ax + b)
and ab (x2 – 1) + x (a2 – b2)
æ 2ö
4
æ 2 3 ö æ 2 ö2 æ 2ö
= 3 ç - ÷ - 2 çç - ÷÷ + 3 ç - ÷ - 2 ç - ÷ + 3 = 185 = abx2 – ab + a2x – b2x = ax(a + bx) + b(a + bx)
è 3 ø è 3 ø è 3 ø è 3ø 27 = (a + bx) (ax – b)
Basic Operation and Factorisation A-131
Lowest common multiple 1 1 1 1
= (a + bx) (ax + b) (ax – b) 114. (b) + + ...... +
= (a + bx) (a2x2 – b2) 1´ 4 4 ´ 7 7 ´10 16 ´19
108. (c) 1. Given, 3abc + b3 + c3 – a3 1é 3 3 3 3 ù
= – (a3 – b3 – c3 – 3abc) Þ ê + + + ...... + ú
3 ëê 1´ 4 4 ´ 7 7 ´10 16 ´19 ûú
= – [a3 + (–b)3 + (–c)3 – 3(a) (–b) (–c)]
= – (a – b – c) (a2 + b2+ c2 + ab – bc + ac) 1 é 1 æç 1 1 ÷ö æç 1 1 ÷ö ...... æç 1 1 öù
Þ ê1 - + ç - ÷ + ç - ÷ +
÷ ÷ + ç - ÷÷÷ú
Hence, (a – b – c) is a factor of 3abc + b3 + c3 – ê ç ç
3 ë 4 è 4 7 ø è 7 10 ø ç
è16 19 øúû
a3
Therefore, Statement 1 is correct. 1 æç 1ö
Þ ç1- ÷÷
Given, 3bc + b3 + c3 – 1 3 çè 19 ÷ø
= b3 + c3 – (1)3– 3bc (–1)
= (b + c – 1) [b2 + c2 + 12 – bc + c + b] 1 18 6
Þ ´ =
Hence, (b+c–1) is a factor of 3bc + b3 + c3 – 1. 3 19 19
Therefore, Statement 2 is also correct. So, option (b) is correct
109. (b) x2 + 2(1 + k) x + k2 = 0.
If it has equal roots, then D = 0 ( x -1)2
115. (c) 4=
Þ {2(1 + k)}2 - 4k2 = 0 ( x -1)
Þ 4(1 + k2 + 2k) - 4k2 = 0
when x > 1–
Þ 4 + 4k2 + 8k - 4k2 = 0 Þ 4 + 8k = 0
(x -1)
4 4= =1
Þ k= – x -1
8
when x < 1–
1
\ k= – 0
2 y= = undefined
0
110. (a) f(x) = 2x3 + x2 – x – 2
So, option (c) is correct.
= (x – 1) (2x2 + 3x + 2)
116. (c) x3 – 3x2 + 2x + 1 = 0
and g(x) = 3x3 – 2x2 + x – 2 as a, b and c are the roots-
= (x – 1) (3x2 + x + 2) Þ ab + bc + ca = 2 ------------(i)
Therefore, the highest common factor of f(x) and g(x) abc = –1 ------------(ii)
is (x–1). Dividing eq (i) by eq (ii)-
1235 ´ 4523 ´ 2451 (ab + bc + ca ) 2
111. (b) =
12 abc -1
1235, 4523 and 2451 is divided by 12 then its
remainder are 11, 11 and 3 respectively ab bc ca
+ + = -2
abc abc abc
11´ 11´ 3 æ 11 ö æ 11 ö æ 3 ö
= = ç ÷´ç ÷´ç ÷
12 è 12 ø è 12 ø è 12 ø 1 1 1
+ + = -2
= (–1) × (–1) × 3 (By Negative Concept of Remainder) c a b
= 1 × 3 = 3 (Remainder)
1 1 1
112. (a) If (x–5) is a factor of x3 –3x2 + hx –10 + + = -2
then x – 5 = 0 Þ x = 5 a b c
So, Þ (5)3– 3 × (5)2 + k × 5 – 10 = 0 So, option (c) is correct.
Þ 125 – 75 + 5k – 10 = 0 117. (c) If (x + 2) is a factor, x = –2 will satisfy the expression
Þ 5 k = – 40 Þ x4 – 6x3 + 12x2 – 24 x + 32
Þ (–2)4 – 6(–2)3 + 12 (–2)2 – 24 (–2) + 32
\ k = –8 Þ 16 + 48 + 48 + 48 + 32 ¹ 0
113. (c) x3q2 – x3pt + 4x2pt – 4x2q2 + 3xq2–3xpt Again,
If we put x = 1 Þ x4 + 6x3 – 12x2 + 24 x – 32
= q2 – pt + 4pt – 4q2 + 3q2 – 3pt = 0 Þ (–2)4 + 6(–2)3 –12(–2)2 + 24(–2) – 32
So, (x –1) is a factor of this function of we put x = 3 Þ 16 – 48 – 48 – 48 – 32 ¹ 0
Checking x = 3 also, Again if (x – 2) is a factor, x = 2 will satisfy the
= 27q2 – 27pt + 36pt – 36q2 + 9q2 – 9pt = 0 expression-
So, (x – 1) and (x – 3) both are factors. Þ x4 – 6x3 + 12x2 – 24x + 32
Þ (214 – 6(213 + 12(2)2 – 24(2) + 32
Þ 16 – 48 + 48 – 48 + 32 = 0
EBD_7367
132
A- Basic Operation and Factorisation

Again Squaring on both sides, we get


Þ x4 + 6x3 – 12x2 + 24x – 32
Þ (2)4 + 6(2)3 – 12(2)2 + 24(2) – 32 ( x + 1) 2 a + 2b
=
Þ 16 + 48 – 48 + 48 – 32 ¹ 0 ( x - 1) 2 a - 2b
So, option (c) is correct.
Again applying componendo and dividendo, we get
91
118. (a) x=
216 x 2 + 1 + 2x + x 2 + 1 - 2x 2a a
= =
2 2 4ab 2b
1 x + 1 + 2x - x - 1 + 2x
3-
Þ
( )=
1
(1 - x)3 2 x2 + 1 a
Þ
1 4x 2b
3-
1
Þ æ ö x2 + 1 a
ç1- 91 ÷÷3 Þ =
çè 216 ø÷ x b
Þ bx2 + b = ax
1 Þ bx2 – ax + b = 0
3-
1
Þ æ 125 ö÷3 \ Option (a) is correct.
çç ÷
è 216 ø÷ 124. (a) Given a3 = 117 + b3 ...(1)
and a = 3 + b
1 6 15 - 6 9 Putting the value of a in (1), we get
Þ 3- æ ö = 3- = =
5
çç ÷÷ 5 5 5 (3 + b)3 = 117 + b3
è 6 ø÷
27 + b3 + 9b2 + 27b = 117 + b3
So, option (a) is correct. 9b2 + 27b – 90 = 0
119. (a) Let a be common root of both the equations b2 + 3b – 10 = 0
x2 – px + q = 0 and x2 + qx – p = 0 (b + 5) (b – 2) = 0
so a2 – pa + q = 0 ...(i)
b ¹ – 5 (Q b > 0 )
and a + qa – p = 0
2
...(ii)
From (i) we get a2 = pa – q Þ b= 2
Putting in (ii), we get \ a=3+b=3+2
a2 + qa – p = 0 Þ a=5
Þ pa – q + qa – p = 0 Thus a + b = 5 + 2 = 7
Þ (p + q)a – (p + q) = 0 \ Option (a) is correct.
Þ (p + q)a = p + q a b c
125. (a) Let = = =k
Þ a=1 b c d
From (i) we get l2 – p.1 + q = 0 Þ a = bk, c = ck, c = dk
Þ –p + q = 0 Þ abc = bcdk3
Þ p–q=0 Þ a = dk3
\ option (a) is correct.
b3 + c3 + d 3 d
120. (b) 121. (c) 122. (c) (1) =
3 3 3 a
a +b +c
x a + 2b + a - 2b
123. (a) =
1 a + 2b - a - 2b a 2 + b 2 + c2 a
(2) =
2 2 2 d
Applying componendo and dividendo, we get b +c +d

x + 1 2 a + 2b b3 + c3 + d 3 d
= Þ =
3 3 3 3 3 3 a
x - 1 2 a - 2b b k +c k +d k

x +1 a + 2b b 2 k 2 + c2 k 2 + d 2 k 2 a
= Þ =
Þ 2
b +c +d 2 2 d
x -1 a - 2b
Basic Operation and Factorisation 133
A-

1 z
b 3 + c3 + d 3
d z2 + z = x + y + z; =
= z +1 x + y + z
Þ
( 3 3 3 3
b +c +d k a ) x y z x+ y+z
Þ + + = =1
2
(b + c + d )k 2 2 2
a x+y+z x+y+z x+y+z x+ y+z
Þ =
2
b +c +a 2 2 d 129. (d) 2122 + 462 + 842 + 464 + 2130
2122 + 2124 + 2126 + 2128 + 2130
1 d 2122 [1 + 22 + 24 + 26+ 28]
Þ =
k 3 a 2122 [1 + 4 + 16 + 64 + 256]
2122 [341]
a
Þ k2 = x 2 + ax + a 2 x 2 - ax + a 2
d 130. (c) -
Þ k – k3
2 x 3 - a3 x3 + a3

1 d x 2 + ax + a 2 x 2 - ax + 1a 2
= Þ –
Þ a a (x - a)(x 2 + ax + a 2 ) (x + a)(x 2 - ax + a 2 )
d
which is not possible 1 1
Þ -
x -a x +a
d d
Þ = x+a-x+a 2a
a a
Þ 2 2 Þ
Þ only (1) is true. x -a x - a2
2

\ Option (a) is correct. Q 4x + 3a = 0


126. (d) Given (x + 1) is a factor of x3 + kx2 – x + 2 -3a
...(1)
x=
4
\ x = –1 satisfies the equation (1), we get
-3a
(– 1)3 + k(– 1)2 – (– 1) + 2 = 0 putting x =
4
–1+k+1+2=0
k= –2 2a 2a ´16
Þ =
\ Option (d) is correct. æ -3a ö
2
2
9a 2 - 16a 2
127. (c) Inequations are ç ÷ -a
è 4 ø
x + y £ 4 and x – y ³ 2
To check point P(5, – 1), we get 32
Þ–
5 – 1 £ 4 and 5 + 1 ³ 2 7a
4 £ 4 and 6 £ 2 1
P is true. 1
To check Q(3, – 2), we get 131. (b) x + 1+ =2
1
2+
3 – 2 £ 4 and 3 + 2 ³ 2 3
1 £ 4 and 5 ³ 2
1
Q is true.
3
To check R(1, 1) we get Þ x + 1+ =2
7
1 + 1 £ 4 and 1 – 1 ³ 2
2 £ 4 and 0 ³ 2 7
Þx+ =2
R is not true. 10
\ Option (c) is correct. 13
7
128. (b) x2 = y + z, y2 = z + x, z2 = x + y x= 2– =
10 10
1 x 132. (c) Putting x = -4 in equations we find option c satisfy the
x2 + x = x + y + z; =
x +1 x + y + z equation x2 – 7x - 44 = 0
(-4)2 – 7 (–4) – 44 = 0
1 y 44 – 44 = 0
y2 + y = x + y + z; y + 1 = x + y + z
EBD_7367
134
A- Basic Operation and Factorisation

1 138. (c) The given polynomial is of the form a x3 + bx2 + cx + d


t
133. (c) x = t -1 , y =
t Let a, b and g be three zeroes of the given polyno-
t t -1 mial.
1

( )
-b
y= ( t)t t -1 Then, sum of the zeroes i.e. a + b + g =
a
= -4

1 Product of the zeroes (taken two at a time) i.e. =


y = tx, t = ( y ) x ...(i) c
ab + bg + ga = = -11
1 a
x= t -1
t Product of the zeroes (individual)
t -d
- i.e. abg = = 30
xt = t 1
t ...(ii) a
on comparing equation (i) and (ii) Now, we will check each option for the correct
y answer.
=y In option (a), we have 2, –3 and –5 as three zeroes.
xx
x = yx
y Sum of these zeroes is – 6 and product of these zeroes
134. (d) is 30.
2 1 In option (b), we have –2, –3 and 5 as three zeroes.
135. (b) x= 2 + 23 + 23 Sum of these zeroes is 0 and product of these zeroes
is 30.
2 1
In option (c), we have –2, 3 and –5 as three zeroes.
(x – 2) = 23 + 23
Sum of these zeroes is – 4 and product of these zeroes
3 is 30.
æ 2 1ö
In option (d), we have –2, 3 and 5 as three zeroes. Sum
(x – 2)3 = çç 2 + 2 ÷÷
3 3
è ø of these zeroes is 6 and product of these zeroes is 30.
Out of these options, only results of option (c) matches
2 1
with the results calculated above.
x3 – 8 – 6x2 + 12x = 4 + 2 + 3 (2) 3 (2) 3
Thus, our correct answer is (c).
x3 – 8 – 6x2 + 12x = 6 + 6 (x – 2) 139. (b) We are given x = 111 ...1(20 digits), y = 333 ...3(10
x3 – 8 – 6x2 + 12x = 6 + 6x – 12 digits) and z = 222 ...2(10 digits)
x3 – 6x2 + 6x = 2
Therefore,
x y 24
136. (c) - = , x + y = 26
x - y 2 111¼1( 20digits ) - ( 333¼3) (10digits )
2
y x 5
=
z 222¼ 2 (10digits )
x-y 24
=
111¼1( 20digits ) - ( 3) (111¼1) (10digits )
2 2
xy 5
=
222¼ 2 (10digits )
24 xy
x–y= ....(i)
5 111¼1( 20digits ) 9 (111¼1)(10 digits )
x + y = 26 ...(ii) = -
2 (111¼1)(10digits ) 2
Squaring and subtracting (i) from (ii)
2 Since 111111 ¸ 111 = 1001, therefore
æ 24 xy ö
(x + y)2 – (x – y)2 = (26)2 – çç ÷÷ 111¼1( 20digits ) 9 (111¼1)(10digits )
è 5 ø -
2 (111¼1)(10digits ) 2
576xy
4xy = 676 –
25 10000000001 - 999¼9 (10 digits )
=
137. (a) a + b = 5 ab = 6 2
(a + b)3 = 125 Now, since 1001 – 999 = 2 and 10001 – 9999 = 2,
a3 + b3 + 3ab (a+ b) = 125
a3 + b3 + 3 (6) (5) = 125 10000000001 - 999 ¼9 (10 digits ) 2
a3 + b3 = 125 - 90 = 35
therefore = =1
2 2
576xy
4xy + = 676 140. (d) Let f ( x) = 5x3 + 5 x2 - 6 x + 9 and g ( x) = x + 3
25
676xy = 25 × 676 To find the remainder, g (x) should be equal to zero.
xy = 25 Therefore, g (x) = x + 3 = 0 Þ x = – 3
Basic Operation and Factorisation 135
A-

Putting this value of f (x) in, we get Therefore, in option (a), there would be no constant
3 2
f (x) = 5.( -3) + 5.(-3) - 6.( -3) + 9 term as each term will contain a variable x.
In option (b), we get the product of the constants as –
f ( x ) = 5. - 27 + 5.9 + 18 + 9 1 × 6 × –12 i.e. 72.
= -135 + 45 + 27 = -63 In option (c), we get the product of the constants as –
Thus, the remainder is – 63. 1 × 6 × 12 i.e. –72.
141. (c) In option (d), we get the product of the constants as 1
× –6 × –12 i.e. 72.
8 x3 - y3 = (2 x)3 - ( y )3 = (2 x - y)(4 x 2 + y 2 + 2 xy)
Thus, option (c) is correct.
(2 x - y )(4 x 2 + y 2 + 2 xy ) 146. (c) Since we are given that a and c are co-prime i.e. HCF of
Quotient = = 2x - y a and c is 1, therefore we can say that a definitely
2 xy + 4 x 2 + y 2
divides d exactly. So, a is a factor of d.
142. (a) We are given that (x + 2) is a common factor of x2 + ax
147. (c) We are given that ab + bc + ca = 0
+ b and x2 + bx + a.
Let f (x) = x2 + ax + b and g (x) = x2 + bx + a. Let p(x) = Therefore, ab + bc + ca = 0 Þ ab + ca = - bc
x + 2. Similarly, bc + ca = – ab and ab + bc = – ca
This means x + 2 = 0 Þ x = – 2 and so, –2 is a zero of f Now,
(x) and g (x).
Therefore, a2 b2 c2
+ +
a 2 - bc b 2 - ca c 2 - ab
x2 + ax + b = (-2)2 - 2a + b = 4 - 2a + b and
x2 + bx + a = ( -2) 2 - 2b + a = 4 - 2b + a a2 b2 c2
= 2
+ 2 + 2
Both polynomials are same. a + ab + ca b + ab + bc c + bc + ca
Thus,
4 - 2a + b = 4 - 2b + a Þ -2a + b = -2b + a a2 b2 c2
Þ + +
3a a ( a + b + c) b ( a + b + c) c( a + b + c)
Þ b + 2b = a + 2a Þ 3a = 3b Þ =1 a b c
3b Þ + +
a+b+c a+b+c a+b+c
Þ a : b = 1:1
143. (a) According to remainder theorem, when a polynomial a+b+c
Þ =1
say p (x) is divided by (x – a), the remainder is p(a). a+b+c
Therefore, when the given polynomial f (x) is divided
by (ax – b), then the remainder is 148. (b) We know that if a + b + c = 0 then a3 + b3 + c3 = 3abc

æ bö ( x - y )( y - z )( z - x )
f (ax - b = 0) Þ f (ax = b) Þ f ç x = ÷ . Now, we have
è aø ( x - y ) 3 + ( y - z )3 + ( z - x )3

Hence, the remainder is f æç ö÷ .


b Since, here, ( x - y ) + ( y - z ) + ( z - x ) = 0 therefore,
è aø
144. (b) ( x - y)3 + ( y - z)3 + ( z - x)3

( x + 2)( x - 2)( x3 - 2 x 2 + 4 x - 8)( x3 + 2 x 2 + 4 x + 8) = 3( x - y)( y - z )( z - x)

= ( x 2 - 4)[ x 2 ( x - 2) + 4( x - 2)][ x 2 ( x + 2) + 4( x + 2)] Thus,

( x - y )( y - z )( z - x )
= ( x 2 - 4)( x - 2)( x 2 + 4)( x + 2)( x 2 + 4)
( x - y ) 3 + ( y - z )3 + ( z - x )3
= ( x 2 - 4)( x 2 - 4)( x 2 + 4)2
= ( x 2 - 4)2 ( x 2 + 4)2 ( x - y )( y - z )( z - x ) 1
= =
3( x - y )( y - z )( z - x) 3
= {( x 2 - 4)( x 2 + 4)}2
149. (d) 517 + 518 + 519 + 520
= ( x 4 - 16) 2
= 517 (1 + 51 + 52 + 53)
145. (c) We can simply check the product of the constants in = 517 (1 + 5 + 25 + 125)
each option whether it yields the constant term or not = 517 × 156 = 517 × 2 × 2 × 3 × 13
i.e. –72. Hence, this number is completely divisible by 13.
EBD_7367
136
A- Basic Operation and Factorisation

b z 3
+ = 1 (given) 1 æ 1ö 1æ 1ö
150. (a)
y c a3 - = a - ÷ + 3.a. ç a - ÷
3 çè
a a ø a è aø
b z c-z y c -c = (5)3 + 3 × 5 = 125 + 15 = 140
=1- = \ = =
y c c b c-z z-c 152. (a) x + 3 is a factor of x3 + 3x2 + 4x + k
x+3=0 \x= –3
c x Now
+ = 1 (given)
z a x3 + 3x2 + 4x + k = 0
x c z-c a z (–3)3 + 3x(–3)2 + 4 × (–3) + k = 0
=1- = \ =
a z z x z -c –27 + 27 – 12 + k = 0
k – 12 = 0
ab + xy ab xy a y
= + = + \ k = 12
bx bx bx x b
153. (b) 3x3 + 4x2 – 7 = 0
z æ -c ö z c z-c according to the option
= +ç ÷ø = - = =1
z-c è z-c z-c z-c z-c checking option (a)
x= 0
a2 -1
151. (c) =5 3 × 03 + 4 × 02 – 7 = 0 + 0 – 7 = –7
a checking option (b)
a2 1 x= 1
Þ - =5 3 × (1)3 + 4 × (1)2 – 7 = 3 + 4 – 7 = 0
a a
Hence, 1 is the zero of polynomial 3x3 + 4x2 – 7.
1 154. (d) According to the options
Þ a- =5
a If we take k = 7 then remainder is –7
a6 - 1 a6 1 1 Hence, check the options.
3
= 3
- 3
= a3 - 155. (a)
a a a a3
linear Equations in One and Two Variables A-137

C HA P T E R
LINEAR EQUATIONS IN ONE AND
14 TWO VARIABLES
1. Assertion (A) : The equations 2x – 3y = 5 and 8. What is the sum of two numbers whose difference is 45
6y – 4x = 11 cannot be solved graphically. and the quotient of the greater number by the lesser
Reason (R) : The equations given above represent number is 4 ? [2008-I]
parallel lines. [2007-I] (a) 100 (b) 90
(a) A and R are correct and R is correct explanation of A (c) 80 (d) 75
(b) A and R are correct but R is not correct explanation 9. The solution of the equations
of A 3x – y + 1 2x + y + 2 3x + 2y + 1
(c) A is correct but R is wrong = =
(d) A is wrong but R is correct 3 5 6
2. If a two-digit number is added to a number obtained by is given by which one of the following?
reversing the digits of the given number, then the sum is [2008-II]
always divisible by which one of the following numbers? (a) x = 2, y = 1 (b) x = 1, y = 1
[2007-II] (c) x = –1, y = –1 (d) x = 1, y = 2
(a) 7 (b) 9 10. A person bought 5 tickets from a station P to a station
(c) 10 (d) 11 Q and 10 tickets from the station P to a station R. He paid
3. A train started from a station with a certain number of ` 350. If the sum of a ticket from P to Q and a ticket from
P to R is ` 42, then what is the fare from P to Q?
1 [2009-I]
passengers. At the first halt, rd of its passengers got (a) ` 12 (b) ` 14
3
down and 120 passengers got in. At the second halt, half (c) ` 16 (d) ` 18
of the passengers got down and 100 persons got in. 11. Pooja started her job with certain monthly salary and gets
a fixed increment every year. If her salary was ` 4200 after
Then, the train left for its destination with 240 passengers.
3 years and ` 6800 after 8 years of service, then what are
How many passengers were there in the train when it her initial salary and the annual increment, respectively?
started ? [2008-I] [2009-I]
(a) 540 (b) 480 (a) ` 2640, ` 320 (b) ` 2460, ` 320
(c) 360 (d) 240 (c) ` 2460, ` 520 (d) ` 2640, ` 520
4. A person bought a certain number of books for `80. 12. What is the solution of the equations x – y = 0.9 and
If he had bought 4 more books for the same sum, each 11(x + y)–1 = 2? [2009-I]
book would have cost ` 1 less. What is the price of each (a) x = 3.2 and y = 2.3 (b) x = 1 and y = 0.1
book ? [2008-I] (c) x = 2 and y = 1.1 (d) x = 1.2 and y = 0.3
(a) ` 10 (b) ` 8 13. What is the value of k for which the system of equations
(c) ` 5 (d) ` 4 x + 2y – 3 = 0 and 5x + ky + 7 = 0 has no solution?
5. What is/are the solutions of the set of homogeneous [2009-I]
equations (4x + 2y = 0) and (6x + 3y = 0) ? [2008-I] 3 14
(a) Only x = 0, y = 0 (a) – (b) –
14 3
(b) Only x = 0, y = 0 and x = 1, y = 2
(c) An infinite number of solutions 1
(c) (d) 10
(d) No solution 10
6. The cost of 4 books and 3 pencils is same as that of 8 14. A number consists of two digits, whose sum is 10. If 18
books and 1 pencil. This cost will be same as that of is subtracted from the number, digits of the number are
which one of the following ? [2008-I] reversed. What is the product? [2009-II]
(a) 2 books and 6 pencils (a) 15 (b) 18
(b) 5 books and 5 pencils (c) 24 (d) 32
(c) 6 books and 2 pencils 15. A railway ticket for a child costs half the full fare but the
(d) 12 books and 4 pencils reservation charge is the same on half tickets as much as
on full ticket. One reserved first class ticket for a journey
7. If one-third of a two-digit number exceeds its one-fourth between two stations is ` 362, one full and one half
by 8, then what is the sum of the digits of the number? reserved first class tickets cost ` 554. What is the
[2008-I] reservation charge? [2009-II]
(a) 6 (b) 13 (a) ` 18 (b) ` 22
(c) 15 (d) 17 (c) ` 38 (d) ` 46
EBD_7367
138
A- linear Equations in One and Two Variables

16. Let there be three simultaneous linear equations in two 26. The system of equations x + 2y = 3 and 3x + 6y = 9 has
unknowns, which are non-parallel and non-collinear them. [2011-II]
What can be the number of solutions (if they do exist)? (a) unique solution
[2010-I] (b) no solution
(a) One or infinite (b) Only one (c) infinitely many solutions
(c) Exactly two (d) Exactly three (d) finite number of solutions
17. Under what condition do the equations kx – y = 2 and 5x – 7y + 10 3x + 2y + 1 11x + 4y – 10
6x – 2y = 3 have a unique solution? [2010-II] 27. If = = , then
1 8 9
(a) k = 3 (b) k ¹ 3 what is x + y equal to? [2011-II]
(c) k = 0 (d) k ¹ 0 (a) 1 (b) 2
18. If 1 is added to the denominator of a fraction, it becomes (c) 3 (d) –3
1 3 2 9 4
and if 1 is added to the numerator, the fraction 28. If + = 2 and – = 1, then what
2 x+y x–y x+y x–y
becomes 1. What is the fraction? [2010-II]
x
5 2 is the value of ? [2013-I]
(a) (b) y
9 3
3
4 10 (a) (b) 5
(c) (d) 2
7 11
2 1
(c) (d)
2 3 9 4 9 21 3 5
19. If + = and + = , where x ¹ 0 and y ¹ 0, 29. The graphs of ax + by = c, dx + ey = f will be:
x y xy x y xy
I. parallel, if the system has no solution.
then what is the value of x + y? [2011-I]
II. co-incident, if the system has finite number of
(a) 2 (b) 3
solutions.
(c) 4 (d) 8
III. intersecting, if the system has only one solution.
20. The sum of two numbers is 80. If the larger number Which of the above statements are correct? [2013-I]
exceeds four times the smaller by 5, what is the smaller (a) Both I and II (b) Both II and III
number? [2011-I] (c) Both I and III (d) All of these
(a) 5 (b) 15
30. The sum of two numbers is 20 and their product is 75.
(c) 20 (d) 25
What is the sum of their reciprocals? [2013-I]
21. If a and b are positive integers, x and y are non-negative 1 1
integers and a = bx + y, then which one of the following (a) (b)
15 5
is correct? [2011-I]
(a) 0 £ y < a (b) 0 < y £ b 4 7
(c) 0 < y < a (d) 0 £ y < b (c) (d)
15 15
2x – 3y + 1 x + 4y + 8 4x – 7y + 2
22. If = = , then what is 10
2 3 5 31. If the sum of a number and its reciprocal is , then the
(x + y) equal to? [2011-I] 3
(a) 3 (b) 2 numbers are [2013-I]
(c) 0 (d) –2 1 1
23. If (x, y) = (4, 1) is the solution of the pair of linear (a) 3, (b) 3, –
3 3
equations mx + y = 2x + ny = 5, then what is m + n
equal to? [2011-II] 1 1
(a) –2 (b) –1 (c) –3, (d) –3, –
3 3
(c) 2 (d) 1
24. The sum of two numbers is 10 and their product is 20. 32. A number consists of two digits. The sum of the digits
What is the sum of their reciprocals? [2011-II] is 10. On reversing the digits of the number, the number
decreases by 36. What is the product of the two digits?
1 1
(a) (b) [2013-I]
10 2 (a) 21 (b) 24
(c) 1 (d) 2 (c) 36 (d) 42
25. The sum of digits of a two-digit number is 8 and the
difference between the number and that formed by 33. If 2x + 3y £ 6, x ³ 0, y ³ 0, then one of the solutions is
reversing the digits is 18. What is the difference between [2013-I]
the digits of the number? [2011-II] (a) x = –2 and y = 3 (b) x = 1 and y = 2
(a) 1 (b) 2 (c) x = 1 and y = 1 (d) x = –1 and y = –1
(c) 3 (d) 4
linear Equations in One and Two Variables A-139
34. There are some benches in a class room having the 43. If x + y – 7 = 0 and 3x + y – 13 = 0, then what is
number of rows 4 more than the number of columns. If 4x2 + y2 + 4xy equal to? [2013-II]
each bench is seated with 5 students, there are two seats (a) 75 (b) 85
vacant in a class of 158 students. The number of rows is (c) 91 (d) 100
[2013-I]
(a) 4 (b) 8 2 3
x y
(c) 6 (d) 10 44. If + = 4 and + = 1, then what is x + y equal to?
2 3 x y
a b x a b [2013-II]
35. If – = and + = x – y, then what is the value (a) 11 (b) 10
b a y b a
(c) 9 (d) 8
of x? [2013-I]
a+b a+b 45. The average age of male employees is 52 years and that
(a) (b) of female employees is 42 years. The mean age of all
a b
employees is 50 years. The percentage of male and female
a–b employees are respectively [2014-I]
(c) (d) None of these
a (a) 80% and 20% (b) 20% and 80%
(c) 50% and 50% (d) 52% and 48%
36. A number consists of two digits whose sum is 8. If 18 is
added to the number, the digits are reversed. The number
is equal to [2013-I] 46. The present age of Ravi’s father is 4 times of Ravi’s
(a) 26 (b) 35 present age. 5 years back, Ravi’s father was seven times
(c) 53 (d) 62 as old as Ravi was at that time. What is the present age
of Ravi’s father? [2014-I]
37. The sum of the squares of two numbers is 97 and the (a) 84 years (b) 70 years
squares of their difference is 25. The product of the two (c) 40 years (d) 35 years
numbers is [2013-I] 47. A positive number, when increased by 10 equals 200 times
(a) 45 (b) 36 its reciprocal. What is number? [2014-I]
(c) 54 (d) 63 (a) 100 (b) 10
(c) 20 (d) 200
1 1 48. The sum of two positive numbers x and y is 2.5 times their
38. If x + = 2, then what is value of x – ? [2013-I]
x x difference. If the product of numbers is 84, then what is
(a) 0 (b) 1 the sum of those two numbers? [2014-I]
(c) 2 (d) –2 (a) 26 (b) 24
39. The system of equations 3x + y – 4 = 0 and 6x + 2y – 8 = 0 (c) 22 (d) 20
has [2013-I] 49. Ravi’s brother is 3 years elder to him. His father was
(a) a unique solution x = 1, y = 1 28 years of age when his sister was born while his mother
(b) a unique solution x = 0, y = 4 was 26 years of age when he was born. If his sister was
(c) no solution 4 years of age when his brother was born, the ages of
(d) infinite solution Ravi’s father and mother, respectively when his brother
was born were [2014-I]
40. The sum of two numbers is 7 and the sum of their squares (a) 32 years and 23 years
is 25. The product of the two numbers is [2013-I] (b) 32 years and 29 years
(a) 6 (b) 10 (c) 35 years and 29 years
(c) 12 (d) 15 (d) 35 years and 33 years
41. A number consists of two digits whose sum is 10. If the 50. Two chairs and one table cost `700 and 1 chair and
digits of the number are reversed, then the number 2 tables cost ` 800. If the cost m tables and m chairs is
decreased by 36. Which of the following is/are correct? ` 30,000, then what is m equal to? [2014-I]
I. The number is divisible by a composite number. (a) 60 (b) 55
II. The number is a multiple of a prime number. (c) 50 (d) 45
Select the correct answer using the codes given below: 51. A certain number of two digits is three times the sum of
[2013-I] its digits. If 45 is added to the number, then the digits will
(a) Only I (b) Only II be reversed. What is the sum of the squares of the two
(c) Both I and II (d) Neither I nor II digits of the number ? [2014-II]
(a) 41 (b) 45
42. Ten chairs and six tables together cost ` 6200, three (c) 53 (d) 64
chairs and two tables together cost ` 1900. The cost of 52. If ax + by – 2 and axby = 1, where a ¹ 0, b ¹ 0, then what
4 chairs and 5 tables is [2013-I] is (a2x + b2y) equal to ? [2014-II]
(a) ` 3000 (b) ` 3300 (a) a + b (b) 2ab
(c) ` 3500 (d) ` 3800 (c) a3 + b3 (d) a4 + b4
EBD_7367
140
A- linear Equations in One and Two Variables

53. If m and n (m > n) are the roots of the equation The linear inequations, for which the shaded area in the
7(x + 2a)2 + 3a2 = 5a (7x + 23a), where a > 0, then figure given above is the solution set, are [2016-I]
what is 3m – n equal to ? [2014-II] (a) 2x + 6y £ 21, 5x – 2y £ 10
(a) 12a (b) 14a
(b) 2x + 6y £ 21, 5x – 2y ³ 10
(c) 15a (d) 18a
54. Consider the following statements : [2015-I] (c) 2x + 6y ³ 21, 5x – 2y £ 10
1 The equation 1990x – 173y = 11 has no solution in (d) 2x + 6y ³ 21, 5x – 2y ³ 10
integers for x and y. 62. If a and b are negative real numbers and c is a positive real
2. The equation 3x – 12y = 7 has no solution in integers number, then which of the following is/are correct?
for x and y. 1. a – b < a – c [2016-I]
Which of the above statements is/are correct?
(a) 1 only (b) 2 only a b
2. If a < b then < .
(c) Both 1 and 2 (d) Neither 1 nor 2 c c
55. If k = x – y + 2z where –2 £ x £ 1, –1 £ y £ 2 and 3 £ z £ 6, then
which one of the following is correct ? [2015-II] 1 1
3. <
(a) 0 £ k £ 9 (b) 5 £ k £ 11 b c
(c) 2 £ k £ 14 (d) 2 £ k £ 11 Select the Correct answer using the code given below.
56. The number of pairs (x, y) where x, y are integers satisfying (a) 1 (b) 2 only
the equation 21x + 48y = 5 is [2015-II]
(c) 3 only (d) 2 and 3
(a) Zero (b) One
(c) Two (d) Infinity
57. Let x and y be positive integers such that x is prime and y is 63. The pair of linear equations kx + 3y + 1 = 0 and 2x + y + 3 =
composite. Which of the following statements are correct? 0 intersect each other, if [2017-I]
[2015-II] (a) k = 6 (b) k ¹ 6
1. (y – x) can be an even integer. (c) k = 0 (d) k ¹ 0
2. xy can be an even integer. 64. The system of equations 2x + 4y = 6 and 4x + 8y = 8 is
3. 0.5 (x + y) can be an even integer [2017-I]
Select the correct answer using the code given below
(a) Consistent with a unique solution
(a) 1 and 2 only (b) 2 and 3 only
(c) land 3 only (d) 1, 2 and 3 (b) Consistent with infinitely many solutions
(c) Inconsistent
p q q p x
58. If + = m and + = n, then what is equal to? (d) None of the above
x y x y y 65. What is the value of u in the system of equations
[2016-I] 3 (2u + v) = 7uv, 3 (u + 3v) = 11uv? [2017-II]
np + mq np + mq 1
(a) (b) (a) 0 (b)
mp + nq mp - nq 4
np – mq np – mq 1
(c) (d) (c) (d) 1
mp – nq mp + nq 2
59. The value of k, for which the system of equation 3x – ky –
20 = 0 and 6x – 10y + 40 = 0 has no solution, is [2016-I] a c
66. If a + b = 2c, then what is the value of + ?
(a) 10 (b) 6 a -c b-c
(c) 5 (d) 3 [2018-1]
60. There are three brothers. The sums of ages of two of them (a) –1 (b) 0
at a time are 4 years, 6 years and 8 years. The age difference (c) 1 (d) 2
between the eldest and the youngest is [2016-I]
67. If 65x – 33y = 97 and 33x – 65y = 1, then what is xy equal to?
(a) 3 years (b) 4 years
[2018-1]
(c) 5 years (d) 6 years
(a) 2 (b) 3
61. 5x –2y = 10 (c) –2 (d) –3
4
68. If x + y + z = 0, then what is (y + z – x)3 + (z + x – y)3 +
3 (x + y – z)3 equal to? [2018-1]
2 2x – 6y = 21 (a) (x + y + z)3 (b) 3(x + y) (y + z) (z + x)
1 (c) 24xyz (d) –24xyz
0 1 2 3 4
linear Equations in One and Two Variables A-141

HINTS & SOLUTIONS


1. (a): Given that, Þ 80 – y + 4x = 84 [using equation (i)]
2x – 3y = 5 ... (i) Þ 4x – y = 4
and –4x + 6y = 11 ... (ii) Þ y = 4(x – 1)
On putting this value of y in equation (i), we get
a1 2 –1
Also, = = 4(x – 1)x = 80
a 2 –4 2 Þ (x – 1)x = 20
Þ x2 – x – 20 = 0
b1 –3 –1
= = Þ x2 – 5x + 4x – 20 = 0
b2 6 2 Þ (x – 5) (x + 4) = 0
\ x=5 (Q x ¹ –4)
a1 b1 –1 c1
Þ = = ¹ Thus, the price of each is ` 5.
a 2 b2 2 c2 5. (c): Given equations are:
So, both equations are in parallel. So, it is not solved 4x + 2y = 0 ... (i)
graphically. and 6x + 3y = 0 ... (ii)
Hence, A and R are individually true and R is correct Here, a1 = 4, b1 = 2, c1 = 0
explanation of A. and a2 = 6, b2 = 3, c2 = 0
2. (d): Let a two-digit number be (10x + y) and reversing a1 b1
number be (10y + x) Here, =
\ Required sum = 10x + y + 10y + x a 2 b2
= 11x + 11y = 11(x + y)
Thus, it is divisible by 11.
4 2 2 2
3. (d): Suppose number of passengers be x in the starting. \ = Þ =
6 3 3 3
Number of passengers after 1st halt
æ xö 2x So, it has infinite solutions.
= ç x – ÷ + 120 = + 120
è 3 ø 3 6. (c): Let the cost of one book = ` x
Number of passengers after 2nd halt and the cost of one pencil = ` y
According to question,
1 æ 2x ö 4x + 3y = 8x + y
= ç + 120 ÷ + 100
2è 3 ø Þ 2y = 4x Þ y = 2x
According to question, \ Cost of 4 books and 3 pencils
Number of passengers after 2nd halt = 4x + 3y = 4x + 6x = 10x
Cost of 6 books and 2 pencils
1 æ 2x ö = 6x + 2y = 6x + 4x = 10x
= ç + 120 ÷ + 100 = 240
2è 3 ø 7. (c): Let the number be y.
2x y y
Þ + 120 = (240 – 100) × 2 \ = +8
3 3 4
2x 4y – 3y
Þ = 280 – 120 Þ =8
3 12
Þ y = 12 × 8 = 96
2x
= 160 \ Sum of digits = 9 + 6 = 15
3 8. (d): Let the greater number be x and smaller number be y.
80 \ x – y = 45 ... (i)
160 ´ 3 and x = 4y ... (ii)
x= From equations (i) and (ii),
2
1 4y – y = 45
x = 240 45
4. (c): Let the price of each book is ` x and the number of Þ y= = 15
3
books is y. On putting the value of y in equation (i), we get
\ xy = 80 ... (i) x = 4 × 15 = 60
and (y + 4) (x – 1) = 80 \ Required sum = x + y = 60 + 15 = 75
Þ xy – y + 4x – 4 = 80
EBD_7367
A- 142 linear Equations in One and Two Variables

9. (b): Given equations are: 1


3x – y + 1 2x + y + 2 3x + 2y + 1 and 1 x + 2y = 554
= = 2
3 5 6 Þ 3x + 4y = 1108 ... (ii)
st nd On solving equations (i) and (ii), we get
On taking I and II terms,
5(3x – y + 1) = 3(2x + y + 2) \ x = 340 and y = 22
Þ 9x – 8y = 1 ... (i) Reservation charge = ` 22
On taking IInd and IIIrd terms, 16. (d): If three lines are parallel, then there are infinite
solution. If all three lines are collinear, then there is
6(2x + y + 2) = 5(3x + 2y + 1)
only one solution. If all three lines are non-parallel
Þ 3x + 4y = 7 ... (ii) and non-collinear, then there is only three solutions.
On solving equations (i) and (ii), we get Hence, the given system of equation have here
y = 1 and x = 1 exactly three solutions.
10. (b): Let, 17. (b): The equations kx – y = 2 and 6x – 2y = 3 have a
fare from P ® Q = ` x unique solution. Then,
fare from P ® R = ` y
k 1
According to question, \ ¹ Þk¹3
x + y = ` 42 ... (i) 6 2
5x + 10y = ` 350 ... (ii) 18. (b): Let the numerator = x and denominator = y
According to question,
By solving equations (i) and (ii), then we get
x = 14 x 1
=
y = 28 y +1 2
\ fare from station P ® R = ` 14 Þ 2x = y + 1
11. (d): Let Pooja’s initial salary is ` x and fixed increment Þ 2x – y = 1 ... (i)
every year is ` y.
According to question, x +1
and =1Þx+1=y
x + 3y = 4200 ... (i) y
and x + 8y = 6800 ... (ii) Þ x – y = –1 ... (ii)
On solving equations (i) and (ii), we get 2x – y = 1
x = ` 2640, y = ` 520
– x m y = m1
12. (a): Given, Þ
x – y = 0.9 ... (i) x =2
and 11(x + y)–1 = 2 Putting the value of x in equation (ii),
Þ 2x + 2y = 11 ... (ii) 2 – y = –1
On multiplying equation (i) by 2 and adding equations Þ y = 3 Þ x = 2 and y = 3
(i) and (ii), we get
x 2
4x = 12.8 \ Required fraction = = .
Þ x = 3.2 y 3
From equation (i), 19. (c): Given,
y = 3.2 – 0.9 = 2.3 2 3 9
13. (d): x + 2y – 3 = 0 and 5x + ky + 7 = 0 + =
x y xy
a1 b1 c1 Þ 2y + 3x = 9 ... (i)
Then, = ¹
a 2 b 2 c2 4 9 21
and + =
1 2 –3 x y xy
\ = ¹
5 k 7 Þ 4y + 9x = 21 ... (ii)
Þ k = 10 On solving equations (i) and (ii), we get
So, it has no solution. x = 1 and y = 3
14. (c): Let the two-digit number be 10y + x. \ x+ y=1+3=4
According to question, 20. (b): Let smaller number = x
x + y = 10 ... (i) Larger number = 80 – x
and 10y + x – 18 = 10x + y According to question,
Þ 9x – 9y = –18 80 – x = 4x + 5
Þ x – y = –2 ... (ii) Þ 5x = 75 Þ x = 15
On solving equations (i) and (ii), we get 21. (c): Let a, b ³ 0 and x, y > 0
x = 4 and y = 6 Q a = bx + y
\ Required product = xy = 4 × 6 = 24. a–y a–y
15. (b): Let full fare = ` x and reservation charges = ` y Þ x= Þ >0
b b
\ x + y = 362 ... (i) Þ a–y>0Þa>y
\ 0<y<a
linear Equations in One and Two Variables A-143

2x – 3y + 1 x + 4y + 8 4x – 7y + 2
= =
(5x + 10 – 7y) + (3x + 2y + 1) – (11x + 4y – 10 )
22. (d): =
2 3 5 (1 + 8) – 9
(2x – 3y + 1) + ( x + 4y + 8) – ( 4x – 7y + 2 ) –3x – 9y + 21
=
( 2 + 3) – 5 =
0
= –x + 8y + 7 = x – 8y = 7 Þ x + 3y = 7 ... (i)
Þ x = 7 + 8y ... (i) On taking first two terms,
Now taking first two terms, 8(5x – 7y + 10) = 3x + 2y + 1
3(2x – 3y + 1) = 2(x + 4y + 8) 37x – 58y + 79 = 0 ... (ii)
4x – 17y = 13
From equation (i), on putting the value of x in
Putting the value of x from equation (i), then we get
equation (ii), we get
4(7 + 8y) – 17y = 13
37(7 – 3y) – 58y + 79 = 0
28 + 32y – 17y = 13
15y = –15 Þ 259 – 111y – 58y + 79 = 0
y = –1 Þ 169y = 338
Now putting the value of y in equation (i), then we Þ y=2
get From equation (i),
x = 7 – 8 = –1 x = 7 – 3(2) = 1
Then, x + y = –1 – 1 = –2 \ x+ y=1+2=3
Hence, x + y = –2. 28. (b): Given,
23. (a): Given,
3 2
(x, y) = (4, 1) and mx + y = 2x + ny = 5 + =2 ... (i)
\ m(4) + 1 = 2 × 4 + n = 5 x+y x–y
\ 4m + 1 = 5 and 8 + n = 5
9 4
Þ m = 1 and n = –3 and – =1 ... (ii)
\ m + n = 1 – 3 = –2 x+y x–y
24. (b): Let the two numbers be x and y.
1 1
\ x + y = 10 and xy = 20 Let = a and =b
x+y x–y
1 1 x + y 10 1
\ + = = = . \ 3a + 2b = 2 ... (iii)
x y xy 20 2 9a – 4b = 1 ... (iv)
25. (b): Let x be the first digit and y be the second digit of On multiplying equation (iii) by 2 and addition of
two digit number. equation (iv) and new one, then we get
According to question, 6a + 4b = 4
x+ y= 8 ... (i)
(10x + y) – (10y + x) = 18 9a – 4b = 1
Þ 9x – 9y = 18 15a =5
Þ x–y=2 ... (ii)
Adding (i) and (ii), 5 1
Þ a= =
15 3
x+y=8
1 1
x–y=2 \ =
x+y 3
2x = 10
Þ x+ y= 3 ... (v)
Þ x=5 On putting the value of (a) in equation (iii), we get
Þ y=8–5=3
1
Þ x = 5 and y = 3 3×+ 2b = 2
\ Required difference of digits, x – y 3
= 5 – 3 = 2. Þ 2b = 2 – 1 = 1
26. (c): Given system of equations are: 1 1 1
x + 2y = 3 and 3x + 6y = 9 Þ b= Þ =
2 x–y 2
Þ x + 2y = 3
Þ x–y=2 ... (vi)
a1 b1 c1 1
Here, = = = \ x + y =3
a 2 b 2 c2 3
x – y =2
So, it has infinitely many solutions.
2x =5
27. (c): Given,
5x – 7y + 10 3x + 2y + 1 11x + 4y – 10 5
= = Þ x=
1 8 9 2
EBD_7367
144
A- linear Equations in One and Two Variables

From equation (v), Put x = 1, y = 1


5 1 Þ 2×1+3×1£6
y=3– = Þ 5<6
2 2 x = 1 and y = 1 is only solution according to given
5 condition.
x 2 34. (b): Let the number of column be x.
\ = =5 Number of rows = x + 4
y 1
According to the question,
2
x(x + 4) × 5 – 2 = 158
29. (c) Þ 5x(x + 4) = 160
30. (c): Let the numbers be x and y.
Þ x(x + 4) = 32
According to the question,
Þ x2 + 4x – 32 = 0
x + y = 20
Þ x2 + 8x – 4x – 32 = 0
and
Þ x(x + 8) – 4(x + 8) = 0
xy = 75
Þ (x + 8) (x – 4) = 0
1 1 y+x So, x = 4 as x = –8 is not possible
Þ + =
x y xy \ Number of rows = x + 4 = 4 + 4 = 8
35. (d): Given equations are:
20 4
Þ = a b x
75 15 – = ... (i)
b a y
1
31. (a): Let the numbers are x and . Then, a b
x +
and =x–y ... (ii)
1 10 b a
x+ = From equations (i) and (ii), we get
x 3
æa bö
x 2 + 1 10 ç – ÷x – x
= a b x

b aø
x 3 + =x–
2 b a æa bö a b
Þ 3x – 10x + 3 = 0 ç – ÷ –
Þ 3x2 – 9x – x + 3 = 0 èb aø b a
Þ 3x(x – 3) – 1(x – 3) = 0
Þ (3x – 1) (x – 3) = 0 æa böæa bö æa b ö
Þ ç + ÷ ç – ÷ = x ç – – 1÷
èb aøèb aø èb a ø
1
\ x= , x=3
3 æ a 2 b2 ö æ a 2 – b2 – ab ö
32. (a): Let the unit digit of the number be x and tens digit Þ çç 2 – 2 ÷÷ = xç
ç
÷
÷
èb a ø è ab ø
be y.
\ Number = 10y + x æ a 4 – b4 ö
According to the question, ab
Þ x= ´ç ÷
x + y = 10
and 10x + y = (10y + x) – 36
... (i) ( )
a 2 – b 2 – ab çè a b ÷ø
2 2

Þ 10x + y – 10y – x = –36


Þ 9x – 9y = –36
Þ x= 2
( a 4 – b4 )
×
1 ( a – b )( a + b ) a + b
=
2 2
(
.
)
Þ x – y = –4 ... (ii)
(
a – b 2 – ab ab )
ab a 2 – b2 – ab ( )
x + y =10
36. (b): Let tens digit and unit digits are x and y, respectively.
x – y = -4 Sum of digits = 8
2x =6 x+ y= 8 ... (i)
According to question,
Þ x=3 (10x + y) + 18 = (10y + x)
From equation (i), Þ 9y – 9x = 18
3 + y = 10
Þ y–x=2 ... (ii)
Þ y=7
On solving equations (i) and (ii), we get
Þ x = 3 and y = 7
\ y = 5 and x = 3
\ Required product of two digits = 3 × 7 = 21
\ Required number = 10x + y
33. (c): Given,
2x + 3y £ 6, x ³ 0, y ³ 0 = 10(3) + 5 = 30 + 5 = 35
37. (b): Let the two numbers are x and y.
Now,
According to question,
2x + 3y £ 6
Sum of squares of two numbers = 97
x2 + y2 = 97 ... (i)
linear Equations in One and Two Variables A-145
Square of their difference = 25 42. (a): Let the cost of one chair = ` x
i.e. (x – y)2 = 25 ... (ii) Cost of one table = ` y
Þ (x2 + y2) – 2xy = 25 According to question,
Þ 97 – 2xy = 25 [from equation (i)] 10x + 6y = 6200 ... (i)
Þ 2xy = 72 and 3x + 2y = 1900 ... (ii)
\ xy = 36 ... (iv) Þ 9x + 6y = 5700 ... (iii)
1 On subtracting equation (iii) from equation (i), we get
38. (a): Given that x + =2 ... (i) 10x + 6y = 6200
x Þ –9x + 6y = 5700
Squaring both sides, we get Þ x = 500
2 \ x = ` 500
æ 1ö
çè x + ÷ø = 4 From equation (i),
x 5000 + 6y = 6200
1 Þ 6y = 1200
Þ x2 + +2=4 Þ y = ` 200
x2 \ Cost of 4 chairs and 5 tables = 4x + 5y
1 = 4 × 500 + 5 × 200 = 2000 + 1000 = ` 3000
Þ x2 + =2 ... (ii) 43. (d): We have,
x2 x+ y–7= 0
2 Þ x+ y= 7 ... (i)
Now, æç x – 1 ö÷ = æç x 2 + 1 ö÷ – 2 and 3x + y – 13 = 0
è xø è x2 ø Þ 3x + y = 13 ... (ii)
=2–2=0 [from equation (ii)] By subtracting equation (i) from equation (ii), we get
1 3x + y = 13
\ x– =0
x x+y=7
39. (d): Given equations of system – – –
3x + y = 4 ... (i) 2x = 6
6x + 2y = 8 ... (ii)
\ x=3
a1 b1 c1 1 On putting the value of x in equation (i), we get
Q = = =
a 2 b 2 c2 2 3+y=7
Therefore, the system of equation has infinite solutions Þ y=4
because it is a coincident line. Now, 4x2 + y2 + 4xy
40. (c): Let the two numbers be x and y. = 4 × (3)2 + (4)2 + 4 × 3 × 4 = 4 × 9 + 16 + 48
Given, = 36 + 16 + 48 = 100
x+ y= 7 x y
x2 + y2 = 25 44. (b): + =4
2 3
xy = ?
Þ 3x + 2y = 24 ... (i)
Now,
(x + y)2 = (x2 + y2) + 2xy 2 3
and + =1
(7)2 = 25 + 2xy x y
49 – 25 = 2xy 2y + 3x = xy ... (ii)
24 = 2xy From equations (i) and (ii), we get
x = 12 xy = 24 ... (iii)
41. (d): Let the two-digit number be 10x + y. Now, multiply by y on both the sides in equation (i),
According to the question, Þ 3xy + 2y2 = 24y
x + y = 10 ... (i) Þ y2 – 12y + 36 = 0
and 10y + x + 36 = 10x + y This can be written as (y – 6)2 = 0
Þ –9y + 9x = 36 y=6
Þ x + y = 10 Now, put y = 6 in equation (iii),
Þ x–y=4 ... (ii) x=4
on adding (i) and (ii), we get Then, x + y = 4 + 6 = 10.
Þ 2x = 14 Þ x = 7 45. (a): Let the number of male employees be x and female
On putting the value of x in equation (i), we get employees be y.
7 + y = 10 Total average age of male employees = 52x
Þ y=3 Total average age of female employees = 42y
The required number is 73 which is neither divisible 52x + 42y
by a composite number nor a multiple of a prime Mean age of all employees = x+y
number.
EBD_7367
146
A- linear Equations in One and Two Variables

According to the question, Father’s age when his sister was born
52x + 42y = 28 years ... (ii)
= 50 Sister’s age when his brother was born
x+y = 4 years ... (iii)
Þ 52x + 42y = 50x + 50y Ravi’s brother is 3 years elder to him ... (iv)
Þ 2x = 8y From (i) and (iv),
x 4 Mother’s age when brother was born
Þ = = 26 – 3 = 23 years
y 1 From (ii) and (iii),
i.e. x : y = 4 : 1 Father’s age when brother was born
4 = 28 + 4 = 32 years
Percentage of male employees = × 100 = 80% 50. (a): Let the cost of chair be x and the cost of table be y,
5
then
1 2x + y = 700 ... (i)
Percentage of female employees = × 100 = 20%
5 and x + 2y = 800 ... (ii)
46. (c): Let present age of Ravi be x. From equations (i) and (ii), we get
\ Present age of Ravi’s father = 4x
2x + y = 700
According to question,
5 years before, 2x + 4y = 1600
Ravi’s father age = 7 × Ravi’s age – – –
4x – 5 = 7(x – 5) –3y = – 900
Þ 4x – 5 = 7x – 35
Þ 3x = 30 Þ y = 300
\ x = 10 \ x = 800 – 600 = 200
\ Ravi’s present age = x = 10 years Given the number of chairs and tables are m to be
Ravi’s father’s present age purchased for ` 30000
= 4 × 10 = 40 years According to question,
47. (b): Let the positive number be x. \ 200m + 300m = 30000
According to the question, 30000
Þ m= Þ m = 60
200 500
x + 10 = 51. (c) Let the two digit number be 10x + y.
x
Þ x2 + 10x = 200 According to the question, 10x + y = 3 (x + y)
Þ x2 + 10x – 200 = 0 Þ 10x + y – 3x – 3y = 0
Þ (x – 10) (x + 20) = 0 Þ 7x – 2y = 0 ... (i)
\ x = 10, –20 According to question
But x ¹ –20, since x is a positive number Þ (10x + y) + 45 = 10y + x
The required number is 10. Þ 10x + y + 45 – 10y – x = 0
48. (d): According to the question, Þ 9x – 9y + 45 = 0
(x + y) = 2.5(x – y) Þ x–y=–5 ... (ii)
Þ x + y = 2.5x – 2.5y Now, solving equations (i) and (ii), we get
Þ 3.5y = 1.5x x = 2 and y = 7
digit = 10 × 2 + 7 = 27
x 7 \ sum of the squares of digits = (2)2 + (7)2
Þ = ... (i)
y 3 = 4 + 49 = 53
52. (a) Given, ax + by – 2 = 0
7
Þ x= y ax + by = 2
3 By Squaring both sides.
Now, xy = 84 (ax + by)2 = (2)2
7 a2x2 + b2y2 + 2axby = 4
Þ y × y = 84 Given (ax by = 1)
3
Þ a2x2 + b2y2 + 2 = 4
84 ´ 3 Þ a2x2 + b2y2 = 2
Þ y2 =
7 Þ a2x2 = 1 b2y2 = 1
Þ y2 = 12 × 3 Þ ax = 1 and by = 1
7 1 1
\ y=6Þx= × 6 = 14 Þ x= and y =
3 a b
\ Sum of numbers = x + y = 14 + 6 = 20
1 1
49. (a): Mother’s age when Ravi was born \ a2x + b2y = a2 . + b2. = a + b
= 26 years ... (i) a b
linear Equations in One and Two Variables 147
A-

53. (c) 7(x + 2a)2 + 3a2 = 5a (7x + 23a)


Þ 7(x2 + 4a2 + 4ax) + 3a2 = 35ax + 115a2 é xù
y êp + q ú
Þ x2 – ax – 12a2 = 0 ë yû m
=
Þ (x + 3a) (x – 4a) = 0 Þ é xù n
Þ x = – 3a and x = 4a y êq + p ú
ë yû
m and n are the roots of the given equation.
Let m = 4a and n = –3a
Now, 3m – n = 3(4a) – (–3a) = 12a + 3a = 15a é xù é xù
Þ n ê p + q ú = m êq + p ú
54. (c) Statement 1 ë yû ë yû
1990x – 173y = 11
x x
-11 1990 - 11 Þ np + nq = mq + mp
x = 0, y = , x = 1, y = = 11.43 y y
173 173
11 11 + 173 x
y = 0, x = y = 1, x = = 0.0924 Þ np - mq = ( mp - nq )
1990 1990 y
This equation has no integers solution for any value
of x and y. x np - mq
Þ =
Statement 2 y mp - nq
3x – 12y = 7
\ Option (c) is correct.
-7 59. (c) Given that system of equations has no solution
x = 0, y = = -0.5833, x = 1, y = -0.333
12 Þ |A| = 0
7
y = 0, x = = 2.33 y = 1, x = 6.33. 3 -k 20
3 A = B=
6 -10 -40
So statement 1 and statement 2
both are correct. |A| = – 30 + 6 k
55. (c) –2 £ x £ 1 Þ –2 £ x £ 1 ----------(i) But |A| = 0
–1 £ y £ 2 Þ –2 £ –y £ 1 ----------(ii) Þ 6k – 30 = 0
3 £ 2 £ 6 Þ 6 £ 2z £ 12 ----------(iii)
Adding equ. (i), (ii), (iii) Þ k= 5
–2 – 2 + 6 £ x - y + 2z £ 1 + 1 + 12 \ Option (c) is correct.
2 £ x - y + 2z £ 14 60. (b) Let the age of 1st brother = x years
So, option (c) is correct. The age of 2nd brother = y years
56. (a) 21x + 48y = 5 for any integer value of x, y is not a
integer value and vice-versa. The age of 3rd brother = z years
So, option (a) is correct. According to question
57. (d) Let x = 2 (even prime no.) x + y= 4 Þ y= 4 – x
y = 6 (composite no.) ...(1)
Þ 4 – x = 6 – 2 = 4, it is even integer y+ z = 6
Þ xy = 2 × 6 = 12, it is even integer ...(2)
Þ 0.5(x + y) = 0.5 (2 + 6) = 4, it is even integer.
z+x=8Þz=8–x
So, option (d) is correct. ...(3)
p q q p Substitute the values of y and z in (2) from (1) and (3)
58. (c) Given + = m and + = n
x y x y we get
x = 3 years
py + qx y = 1years
Þ =m
xy z = 5 years
Eldest Brother = 5 years
qy + px
Þ =n Youngest Brother = 1 years
xy
Difference is their ages = 5 – 1 = 4 years
Þ py + qx = mxy ....(1) \ Option (b) is correct.
Þ qy + px = nxy ...(2) 61. (a) Given equations are
Dividing (1) by (2) we get 5x – 2y = 10
2x + 6y = 21
py + qx m On solving, we get
=
qy + px n x = 3 and y = 5/2
EBD_7367
148
A- linear Equations in One and Two Variables

Their interaction point is (3, 5/2)


1 1
The shaded region shows the graph below the original Let = x and = y
region. u v
So solution set is Multiplying equation (4) by 2, we get
5x – 2y £ 10 and 2x + 6y £ 21 3x + 6y = 7
\ Option (a) is correct. 18x + 6y = 22
62. (b) Since a, b are negative numbers. Solving these equations by elimination method, we
a < 0 and b < 0 get
C is a positive real number
Þ c>0 – 15x = – 15
(1) a – b < a – c x=1
Þ –b<–c 4 2
Þ b> c Therefore, equation (3) gives y = =
6 3
It is not true as b < c 1
Thus, x = 1 Þ = 1Þ u = 1
a b u
(2) if a < b Þ < this is true.
c c 66. (c) a + b = 2c
1 1 Þ a+ b= c+ c
(3) < Þ a–c=c–b Þ b – c = –(a – c)
b c
1 1 a c a c
< this is not true. + = +
Since c > b Þ
c b a - c b - c a - c - ( a - c)
\ option (b) is correct.
a c a -c
= - = =1
63. (b) equation a -c a -c a -c
kx + 3y + 1 = 0 , and 2x + y + 3 = 0 intersect i.e. have one 67. (a) 65x – 33y = 97 .... (i) (given)
solution 33x – 65y = 1 ....(ii) (given)
From eq (i) + eq (ii)
k 3 æ a1 ¹ b1 ö
¹ ç ÷ 98x – 98y = 98
2 1 è a 2 b2 ø
98(x – y) = 98 \ x – y = 1 ....(iii)
k¹6 Now
64. (c) 2x + 4y = 6 From eq (i) × 1 – eq (iii) × 33
4x + 8y = 8
65x - 33y = 97
2 4 6 64
= ¹ -33x + 33y = -33 \x = =2
4 8 8 32
32x = 64
1 1 3
= ¹ From eq (iii)
2 2 4
x–y=1
a1 b1 c1 2–y=1 \y=2–1 =1
So the form is a = b ¹ c
2 2 2 Hence, xy = 2 × 1 = 2
\ the system of equation is inconsistant. 68. (d) x + y + z = 0
65. (d) Given that, (y + z – x)3 + (z + x – y)3 + (x + y – z)3
3(2u + v) = 7uv Here
3(u + 3v) = 11uv y+ z –x + z + x– y+ x + y–z = x + y+ z = 0
If a + b + c = 0 then value of a3 + b3 + c3 = 3abc
6u + 3v = 7uv ...(1) \ (y + z – x)3 + (z + x – y)3 + (x + y – z)3 = 3 (y + z – x)
3u + 9v = 11uv ...(2) (z + x – y) (x + y – z)
Dividing equation (1) and equation (2) by uv, we get According to the question
x + y = –z
6 3
+ =7 y + z = –x
v u
x + z = –y
3 9
+ = 11 \ 3 (–x –x) (–y –y) (–z –z) = 3 × (–2x) × (–2y) × (–2z)
v u = – 24xyz
C HA P T E R
QUADRATIC EQUATIONS
15
1. If x2 – kx – 21 = 0 and x 2 – 3kx + 35 = 0 have one common
root, then what is the value of k? [2007-I] 3abc b3 b3 3abc
(c) (d)
(a) + 4 (b) – 4 a3 a3
(c) 4 (d) 1 10. For what value of k, does the equation
2. What are the roots of the equation, [kx2 + (2k + 6)x + 16 = 0] have equal roots? [2008-I]
(a + b + x)–1 = a–1 + b–1 + x–1 ? (a) 1 and 9 (b) –9 and 1
[2007-II] (c) –1 and 9 (d) –1 and –9
(a) a, b (b) – a, b 11. If the product of the roots of x2 – 3kx + 2k2 – 1 = 0 is 7
(c) a, – b (d) – a, – b for a fixed k, then what is the nature of roots?
3. What is one of the value of x in the equation [2008-I]
(a) Integral and positive
x 1 x 13
? [2007-II] (b) Integral and negative
1 x x 6 (c) Irrational
5 7 (d) Rational but not integral
(a) (b) 12. Which one of the following is the quadratic equation
13 13
whose roots are reciprocal to the roots of the quadratic
9 11 equation 2x2 – 3x – 4 = 0? [2008-II]
(c) (d)
13 3 (a) 3x2 – 2x – 4 = 0 (b) 4x2 + 3x – 2 = 0
4. If the equations, (c) 3x2 – 4x – 2 = 0 (d) 4x2 – 2x – 3 = 0
2x2 – 7x + 3 = 0 and 4x2 + ax – 3 = 0 13. The value of y which will satisfy the equations
have a common root, then what is the value of a? 2x2 + 6x + 5y + 1 and 2x + y + 3 = 0 may be found by
[2007-II] solving which one of the following equations?
(a) –11 or 4 (b) –11 or – 4 [2008-II]
(c) 11 or – 4 (d) 11 or 4 (a) y2 + 14y – 7 = 0 (b) y2 + 8y + 1 = 0
5. If one root of px2 + qx + r = 0 is double of the other root, (c) y2 + 10y – 7 = 0 (d) y2 – 8y + 7 = 0
then which one of the following is correct? [2007-II] 14. If a polynomial equation has rational co-efficients and has
(a) 2q2 = 9pr (b) 2q2 = 9p exactly three real roots, then what is the degree of the
2
(c) 4q = 9r (d) 9q2 = 2pr polynomial? [2008-II]
6. If (2x – 3y < 7) and (x + 6y < 11), then which one of the (a) Equal to 3
following is correct? [2008-I] (b) Greater than or equal to 3
(a) x + y < 5 (b) x + y < 6 (c) Strictly greater than 3
(c) x + y 5 (d) x + y 6 (d) Less than 3
7. What is one of the roots of the equation 15. If and are the roots of ax2 + bx + c = 0, then what
2
2x 3 x 3 1 1
[2008-I] is the value of ? [2008-II]
3 x 2x 2 2 2
(a) 1 (b) 2
(c) 3 (d) 4 b2 b 2 – 4ac b b2 – 4ac
8. If and are the roots of the equation (x2 – 3x + 2 = 0), (a) (b)
then which equation has the roots ( + 1) and ( + 1)? c4 c2
[2008-I]
(a) x2 + 5x + 6 = 0 (b) x2 – 5x – 6 = 0 b2 – 4ac b2 – 4ac
(c) x2 + 5x – 6 = 0 (d) x2 – 5x + 6 = 0 (c) (d)
9. If and are the roots of the equation (ax2 + bx + c = 0), c2 c4
then what is the value of 3 + 3? [2008-I] 16. Which one of the following is one of the two consecutive
positive integers, the sum of whose squares is 761?
b3 3abc a 3 b3 [2008-II]
(a) (b) (a) 15 (b) 20
a3 3ab
(c) 24 (d) 25
EBD_7367
150
A- Quadratic Equations

17. If 3x + 27(3 –x) = 12, then what is the value of x? 27. What are the roots of the equation
[2009-I] log10 (x2 – 6x + 45) = 2?
(a) Only 1 (b) Only 2 [2010-I]
(c) 1 or 2 (d) 0 or 1 (a) 9, –5 (b) –9, 5
18. What is the magnitude of difference of the roots of (c) 11, –5 (d) –11, 5
x2 – ab + b = 0? [2009-I] 1 1 1
2 2
28. The sum of the roots of the equation
(a) a 4b (b) b 4a x a x b c
is zero. What is the product of the roots of the equation?
(c) 2 a 2 4b (d) b2 4ab [2010-I]
2
19. If a + b = 2m , b + c = 6m, a + c = 2, where m is a real (a b) (a b)
(a) (b)
number and a b c, then which one of the following 2 2
is correct? [2009-II]
(a 2 b 2 ) (a 2 b2 )
1 (c) (d)
(a) 0 m (b) –1 m 0 2 2
2 29. For what value of k, will the roots of the equation
1 kx2 – 5x + 6 = 0 be in the ratio of 2 : 3? [2010-I]
(c) m 1 (d) 1 m 2 (a) 0 (b) 1
3
(c) –1 (d) 2
20. Students of a class are made to stand in rows. If one 30. If and are the roots of the equation x2 + px + q = 0,
student is extra in a row, there would be two rows less. then – –1, – –1 are the roots of which one of the
If one student is less in a row, there would be three rows following equations? [2010-II]
more. Then, what is the number of students in the class? (a) qx2 – px + 1 = 0 (b) q2 + px + 1 = 0
[2009-II] (c) x2 + px – q = 0 (d) x2 – px + q = 0
(a) 65 (b) 55 31. If one root of the equation ax2 + x – 3 = 0 is –1, then what
(c) 60 (d) 50 is the other root? [2010-II]
21. What is the ratio of sum of squares of roots to the
product of the roots of the equation 7x2 + 12x + 18 = 0? 1 1
(a) (b)
[2009-II] 4 2
(a) 6 : 1 (b) 1 : 6 3
(c) – 6 : 1 (d) – 6 : 7 (c) (d) 1
4
x (x 1) (m 1) x 32. If the equation
22. If the roots of the equation are
(x 1) (m 1) m (a2 + b2) x2 – 2 (ac + bd)x + (c2 + d2 ) = 0
has equal roots, then which one of the following is
equal, then what is the value of m? [2009-II]
correct? [2010-II]
1 (a) ab = cd (b) ad = bc
(a) 1 (b) (c) a2 + c2 = b2 + d2 (d) ac = bd
2
33. What is the solution of the equation
1
(c) 0 (d) – x x 3 3
2 ?
23. What is the least integral value of k for which equation x 3 x 2
x2 – 2(k – 1)x + (2k + 1) = 0 has both the roots positive? [2010-II]
[2010-I] (a) 1 (b) 2
(c) 4 (d) None of these
1
(a) 1 (b) 34. What are the roots of the equation 4x – 3.2x + 2 + 32 = 0?
2 [2010-II]
(c) 4 (d) 0 (a) 1, 2 (b) 3, 4
24. If one of the roots of the equation ax2 + x – 3 = 0 is 1.5, (c) 2, 3 (d) 1, 3
then what is the value of a? [2010-I] 35. If and are the roots of the equation x2 – x – 1 = 0,
(a) 4 (b) 3 then what is the value of ( 4 + 4)? [2010-II]
(c) 2 (d) –2 (a) 7 (b) 0
25. r is a non-zero real number such that r75 > r 90. This is (c) 2 (d) None of these
possible only when [2010-I] 36. If sum as well as product of roots of a quadratic equation
(a) –1 < r < 0 (b) 0 < r < 1 is 9, then what is the equation? [2010-II]
(c) 1 < r (d) –1 < r < 1 (a) x2 + 9x – 18 = 0 (b) x2 – 18x + 9 = 0
26. When the roots of the quadratic equation ax2 + bx + c = 0 (c) x2 + 9x + 9 = 0 (d) x2 – 9x + 9 = 0
are negative of reciprocals of each other, then which one 37. If one root of the equation 2x2 + 3x + c = 0 is 0.5, then
of the following is correct? [2010-I] what is the value of c? [2011-I]
(a) b = 0 (b) c = 0 (a) – 1 (b) – 2
(c) a = c (d) a = –c (c) – 3 (d) – 4
Quadratic Equations 151
A-

38. What is the condition that the quation ax2 + bx + c = 0,


where a 0 has both the roots positive? [2011-I] x2 x 1
49. If one root of the equation 0 is reciprocal
(a) a, b and c are of same sign. a b c
(b) a and b are of same sign. of the other, then which one of the following is correct?
(c) b and c have the same sign opposite to that of a. [2012-II]
(d) a and c have the same sign opposite to that of b. (a) a = b (b) b = c
39. The equation (1 + n2)x2 + 2ncx + (c2 – a2) = 0 will have (c) ac = 1 (d) a = c
equal roots, if [2011-I] 50. If f(x) is a polynomial with all coefficients are integers
(a) c2 = 1 + a2 (b) c2 = 1 – a2 and constant term 10 having a factor (x – k), where k is
(c) c2 = 1 + n2 + a2 (d) c2 = (1 + n2 )a2 an integer, then what is the possible value of k?
40. The equation whose roots are twice the roots of the [2012-II]
equation x2 – 2x + 4 = 0 is [2011-I] (a) –20 (b) 20
(a) x2 – 2x + 4 = 0 (b) x2 – 2x + 16 = 0 (c) 8 (d) 5
(c) x2 – 4x + 8 = 0 (d) x2 – 4x + 16 = 0 51. The difference in the roots of the equation
41. If sin and cos are the roots of the equation 2x2 – 11x + 5 = 0 is
ax2 – bx + c = 0, then which one of the following is [2013-I]
correct? [2011-II] (a) 4.5 (b) 4
(a) a2 + b2 + 2ac = 0 (b) a2 – b2 + 2ac = 0 (c) 3.5 (d) 3
(c) a2 + c2 + 2ab = 0 (d) a2 – b2 – 2ac = 0 52. If one of the roots of the equation x2 – bx + c = 0 is the
42. If and are the roots of the equation x2 – 6x + 6 = 0, square of the other, then which of the following option is
what is 3 + 3 + 2 + 2 + + equal to? correct? [2013-I]
[2011-II] (a) b3 = 3bc + c2 + c (b) c3 = 3bc + b2 + b
(a) 150 (b) 138 (c) 3bc = c3 + b2 + b (d) 3bc = c3 + b3 + b2
(c) 138 (d) 124 53. Consider the following statements in respect of the
43. If 6 x 8, then which one of the following is correct? quadratic equation ax2 + bx + b = 0, where a 0.
[2011-II] I. The product of the roots is equal to the sum of the
(a) (x – 6) (x – 8) 0 (b) (x – 6) (x – 8) > 0 roots.
(c) (x – 6) (x – 8) 0 (d) (x – 6) (x – 8) < 0 II. The roots of the equation are always unequal and
44. What are the roots of the quadratic equation real.
a2b2x2 – (a2 + b2) x + 1 = 0? Which of the statements given above is/are correct?
[2011-II] [2013-II]
1 1 1 1 (a) Only I (b) Only II
(a) 2
, 2 (b) 2
, (c) Both I and II (d) Neither I nor II
a b a b2
1 1 1 1 54. If x 2 6 6 6 6 ... , then what is one of the
(c) 2
, 2 (d) 2
, 2
a b a b values of x equal to? [2013-II]
45. If the roots of the equation (a) 6 (b) 5
x2 – 2ax + a2 + a – 3 = 0 (c) 4 (d) 3
are real and less than 3, then which one of the following 55. If and are the roots of the equation x2 – x – 1 = 0,
is correct? [2012-I]
2 2
(a) a < 2 (b) 2 < a < 3 then what is equal to? [2013-II]
(c) 3 < a < 4 (d) a > 4 2 2
( )( )
46. Two students A and B solve an equation of the form
x2 + px + q = 0. A starts with a wrong value of p and 2 3
(a) (b)
obtains the roots as 2 and 6. B starts with a wrong value 5 5
of q and gets the roots as 2 and –9. What are the correct
roots of the equation? [2012-I] 4
(c) (d) 4
(a) 3 and – 4 (b) – 3 and – 4 5
(c) – 3 and 4 (d) 3 and 4 56. Which one is one of the factors of
47. If one of the roots of quadratic equation 7x2 – 50x + k = 0 1 1
is 7, then what is the value of k? [2012-I] x2 8 x 14 ?
2 x
(a) 7 (b) 1 x
[2013-II]
50 7
(c) (d) 1 1
7 50 (a) x 1 (b) x 3
48. The quadratic equation whose roots are 3 and –1, is x x
[2012-II] 1 1
(a) x2 – 4x + 3 = 0 (b) x2 – 2x – 3 = 0 (c) x 6 (d) x 7
x x
(c) x2 + 2x – 3 = 0 (d) x2 + 4x + 3 = 0
EBD_7367
152
A- Quadratic Equations

57. In solving a problem, one student makes a mistake in the 67. If an = 3– 4n, then what is a1 + a2 + a3 + ... + an equal
coefficient of the first degree term and obtains –9 and 1 to ? [2014-II]
for the roots. Another student makes a mistake in the (a) –n(4n –3) (b) –n(2n –1)
constant term of the equation and obtains 8 and 2 for the (c) –n2 (d) –n(2n +1)
roots. The correct equation was [2014-I] 68. For which value of k does the pair of equations x 2 –
(a) x2 + 10x + 9 = 0 (b) x2 – 10x + 16 = 0 y 2 = 0 and (x – k)2 + y 2 = 1 yield a unique positive
(c) x2 – 10x + 9 = 0 (d) None of these solution of x? [2015-I]
58. If m and n are the roots of the equation ax2 + bx + c = (a) 2 (b) 0
m2 1 (c) 2 (d) – 2
0, then the equation whose roots are and
m
69. If the roots of the quadratic equation x2 – 4x – log10 N = 0
2 are all real, then the minimum value of N is [2015-II]
n 1
is [2014-I]
n 1 1
(a) (b)
(a) acx2 + (ab + bc)x + b2 + (a – c)2 = 0 100 1000
(b) acx2 + (ab – bc) x + b2 + (a – c)2 = 0
1
(c) acx2 – (ab – bc)x + b2 – (a – c)2 = 0 (c) (d) 10000
(d) acx2 – (ab + bc)x + b2 – (a – c)2 = 0 10000
59. The value of x2 – 4x + 11 can never be less than 70. The difference of maximum values of the expressions (6 + 5x
[2014-I] – x2) and (y – 6 – y2) for any real values of x and y is
(a) 7 (b) 8 [2015-II]
(c) 11 (d) 22 (a) 16 (b) 17
60. The expression 2x3 + x2 – 2x – 1 is divisible by (c) 18 (d) 19
[2014-I] 71. If the roots of the equation lx2 + mx + m = 0 are in the ratio
(a) x + 2 (b) 2x + 1 p : q, then [2016-I]
(c) x – 2 (d) 2x – 1
61. If x + y = 5, y + z = 10 and z + x = 15, then which one p q m
of the following is correct? [2014-I] q p l
(a) z > x > y (b) z > y > x
is equal to
(c) x > y > z (d) x > z > y
(a) 0 (b) 1
62. If the roots of the equation
(c) 2 (d) 3
(a2 – bc)x2 + 2(b2 – ac)x + (c2 – ab) = 0
are equal, where b 0, then which one of the following 72. If 3x 2 – 7 x – 30 – 2 x 2 – 7 x – 5 x – 5 [2016-I]
is correct? [2014-I] has and as its roots, then the value of is
(a) a + b + c = abc (b) a2 + b2 + c2 = 0
(a) –15 (b) –5
(c) a3 + b3 + c3 = 0 (d) a3 + b3 + c3 = 3abc
(c) 0 (d) 5
63. If the roots of the equation Ax2 + Bx + C = 0 are –1 and 1,
73. If a2 – by – cz = 0, ax – b2 + cz = 0 and ax + by – c2 = 0, then
then which one of the following is correct? [2014-I]
(a) A and C are both zero x y z
(b) A and B are both positive the value of will be [2016-I]
a x b y c z
(c) A and C are both negative
(d) A and C are of opposite sign (a) a + b + c (b) 3
64. If one the roots of the equation px2 + qx + r = 0 is (c) 1 (d) 0
three times the other, then which one of the following 74. Let p and q be non-zero integers. Consider the polynomial
relations is correct ? [2014-II] A(x) = x2 + px + q
(a) 3q2 = 16 pr (b) q2 = 24 pr It is given that (x – m) and (x – km) are aimple factora of A(x),
(c) p = q + r (d) p + q + r = 1 where m is a non-zero integer and k is a positive integer, k
65. If m% of m + n% of n = 2% of (m × n), then what 2. Which one of the following is correct? [2016-I]
percentage of m is n? [2014-II] (a) (k + 1)2p2 = kq (b) (k + 1)2q = kp2
(a) 50% (b) 75% (c) k2q = (k + 1)p2 (d) k2p2 = (k + 1)2q
(c) 100% (d) Cannot be determined 75. If the linear factors of ax – (a2 + 1)x + a are p and q then p
2
66. If m and n are the roots of the equation x2 + ax + b = 0 + q is equal to [2016-I]
and m2, n2 are the roots of the equation x2 – cx + d = 0, (a) (x – 1) (a + 1) (b) (x + 1) (a + 1)
then which of the following is / are correct ? (c) (x – 1) (a – 1) (d) (x + 1) (a – 1)
1. 2b – a2 = c 2. b2 = d 76. If the sum of the roots of ax2 + bx + c = 0 is equal to the sum
Select the correct answer using the codes given below : of the squares of their reciprocais, then which one of the
(a) Only 1 (b) Only 2 following relations is correct? [2016-I]
(c) Both 1 and 2 (d) Neither 1 nor 2 (a) ab2 + bc2 = 2a2c (b) ac2 + bc2 = 2b2a
[2014-II] (c) ab2 + bc2 = a2c (d) a2 + b2 + c2 = 1
Quadratic Equations 153
A-

77. Under what condition on p and q, one of the roots of the 85. If and are the roots of the equation x2 + px + q = 0, then
equation x2 + px + q = 0 is the square of the other? [2016-I] what is 2 + 2 equal to ? [2017-I]
(a) 1 + q + q2 = 3pq (b) 1 + p + p2 = 3pq 2
(a) p – 2q 2
(b) q – 2p
3 2
(c) p + q + q = 3pq (d) q3 + p + p2 = 3pq (c) p2 + 2q (d) q2 – q
78. The solution of the inequation [2016-I]
86. Aman and Alok attempted to solve a quadratic equation.
1 1 Aman made a mistake in writing down the constant term
1 – 0
x x2 and ended up in roots (4, 3). Alok made a mistake in writing
is (given that x 0) down the coefficient of x to get roots (3, 2). The correct
(a) x > 0 roots of the equation are [2017-I]
(b) x < 0 (a) – 4, – 3 (b) 6, 1
(c) 4, 3 (d) – 6, – 1
–1 – 5 –1 5
(c) x 87. What is the positive value of m for which the roots of the
2 2 equation 12x2 + mx + 5 = 0 are in the ratio 3 : 2? [2017-II]
–1 – 5 –1 5 5 10
(d) x or x (a) (b)
2 2 5 10
12

79. If it is an integer and are the roots of 4x2 – 16x +


= 5 12
4 (c) (d)
12 5
0 such that 1< < 2 and 2 < < 3, then how many values can 88. If the roots of the equation
take ? [2016-II] a (b – c) x2 + b (c – a) x + c (a – b) = 0 are equal, then which
(a) 3 (b) 9 one of the following is correct? [2017-II]
(c) 14 (d) 15 (a) 2b = a + c (b) b2 = ac
80. If and are the two zeros of the polynomial 25x2 – 15x + 2,
then what is a quadratic polynomial whose zeros are (2 )– 2 1 1 1 1 1
1 and (2 )–1?
(c) (d)
[2016-II] b a c b a c
(a) x2 + 30 x + 2 (b) 8x2 – 30x + 25
7
(c) 8x2 – 30x (d) x2 + 30x 89. If k is an integer, then x 2 7x 14 k 2 0 has
2
81. If p and q are the roots of x +px +q = 0, then which one of 8
the following is correct ? [2016-II] [2017-II]
(a) p = 0 or 1 (b) p = 1 only (a) Both integral roots
(c) p = –2 or 0 (d) p = –2 only (b) At least one integral root
82. If and are the roots of the quadratic equation 2x 2 + 6x + (c) No integral root
k = 0, Where k < 0, then what is the maximum value of (d) Both positive integral roots
90. If the roots of the equation px2 + x + r = 0 are reciprocal to
? [2017-I] each other, then which one of the following is correct?
[2018-I]
(a) 2 (b) – 2 (a) p = 2r (b) p = r
(c) 9 (d) – 9 (c) 2p = r (d) p = 4r
83. If one root of 91. If and are the roots of the equation ax2 + bx + c = 0, then
(a2 – 5a + 3) x2 + (3a – 1 ) x + 2 = 0 what is the value of the expression ( + 1) ( + 1)?
is twice the other, then what is the value of 'a'? [2017-I] [2018-I]
2 2 a b c b c a
(a) (b) (a) (b)
3 3 a a
1 1 a b c a b c
(c) (d) (c) (d)
3 3 a a
92. A quadratic polynomial ax2 + bx + c is such that when it is
a b a b
84. If x = , then what is bx2 – 2ax + b equal to (b divided by x, (x – 1) and (x + 1), the remainders are 3, 6 and
a b a b 4 respectively. What is the value of (a + b)? [2018-I]
0) ? [2017-I] (a) 3 (b) 2
(a) 0 (b) 1 (c) 1 (d) –1
(c) ab (d) 2ab
EBD_7367
A- 154 Quadratic Equations

HINTS & SOLUTIONS


1. (c) Let the common root be , then 9
2
– k – 21 = 0 ... (i) x
and 2 – 3k + 35 = 0 ... (ii) 13
Solving by the rules of cross multiplication 4. (a) Given equation, 2x2 – 7x + 3 = 0
2x2 – 6x – x + 3 = 0
2
1 2x (x – 3) – 1(x – 3) = 0
35 63k 21 35 3k k (2x – 1) (x – 3) = 0
Both equation have a common root.
2
1 2
1 1 1
98 k 56 2k So, we put x 4 a 3 0
2 2 2
2
1 a a
1 3 0 2 a 4
98k 2k 2 2
Again, we put x = 3
2 56 28 4(3)2 + a(3) – 3 = 0
= 49 and
2k k 36 + 3a – 3 = 0 a = –11
a = –11 or 4.
2
28 5. (a) Given, px2 + qx + r = 0
Then, = 49 Let and are the roots of equation.
k
According to the question,
28 28 2
= k2
49 r 2
16 = k2 Product of roots ( ) 2
p
k = ± 4.
1 1 1 1 2 r
2. (d) Given, ... (i)
a b x a b x 2p

1 1 1 1 (a b) (a b) q
Sum of roots ( = 3
a b x x a b (a b x) x ab p
x2 + (a + b)x + ab = 0 (x + a)(x + b) = 0 q
Hence, x = –a, –b. ... (ii)
3p
x We put the value of in equation (i),
3. (c) Let y Now,
1 x
2
1 13 q r
y (y2 + 1) 6 = 13y
y 6 3p 2p
6y2 – 13y + 6 = 0 6y2 – 9y – 4y + 6 = 0 2q2 = 9pr
3y(2y – 3) – 2(2y – 3) = 0 6. (b) Given,
(3y – 2) (2y – 3) = 0 2x – 3y < 7 ...(i)
2 3 and x + 6y < 11 ...(ii)
y and On adding equations (i) and (ii),
3 2 3x + 3y < 18
2 x 4 x+ y< 6
When, we put y 7. (b) Given equation,
3 1 x 9
4 2x 3 x 3
9x = 4 – 4x x 3 x 2x 2
13
3 2x
When we put y = Let a
2 3 x
x 9 1 3
4x = 9 – 9x a
1 x 4 a 2
Quadratic Equations A-155
2(a2 – 1) = 3a x2 6x + 7 = 0
2a2 – 3a – 2 = 0
2a2 – 4a + a – 2 = 0 D b2 4ac (6) 2 4 7 36 28 2 2
2a (a – 2) + 1(a – 2) = 0
(2a + 1) (a – 2) = 0 D = 2 2 , so roots of equation are irrational.
If a – 2 = 0 12. (b) Given equation,
Now, put a = 2 2x2 – 3x – 4 = 0
2x 1
2 For a reciprocal roots, we replace x by , we get
3 x x
Squaring both sides, then we get 1 1
2
2x = 4(3 – x) 2 3 4 0
6x = 12 x=2 x x
If 2a + 1 = 0, – 4x2 – 3x + 2 = 0
4x2 + 3x – 2 = 0
1 1
a , a 13. (c) Given that,
2 2 2x2 + 6x + 5y + 1 = 0 ... (i)
x = 2 is the root of equation.
8. (d) Since, and are the roots of the equation y 3
and 2x + y + 3 = 0 x ... (ii)
x2 – 3x + 2 = 0 2
3 and =2 ... (i) Now, putting the value of x from (ii) in equation (i),
Now, + 1 + + 1 = +2 2
=3+2=5 3 y 3 y
2 6 5y 1 0
and ( + 1) ( + 1) = + + +1 2 2
=2+3+1=6
Required equation is 9 y 2 6y (18y 6y)
5y 1 0
x2 – ( + 1 + + 1)x + ( + 1)( + 1) = 0 2 2
x2 – 5x + 6 = 0 2
y + 10y – 7 = 0
9. (c) and are the roots of the equation 14. (a) Since, a polynomial equation has rational coefficient
ax2 + bx + c = 0, then and has exactly three real roots, then degree of the
b c polynomial must be equal to 3.
and 15. (a) and are the roots of the equation ax2 + bx + c = 0
a a
3 3 b c
( )3 3 ( ) and
a a
3
b c b 2 2 2
2
3 1
a a a
2 2 2 2
3 3
b 3bc 3abc b
3 2 ( ) 2 {( )2 4 }
a a a3
2 2 2
10. (c) Given equation is ( )
kx2 + (2k + 6)x + 16 = 0
For equal roots then D must be zero. b 2 b2 4c
(2k + 6)2 – 4k . 16 = 0 2 2 a
a a
4k2 + 24k + 36 – 64k = 0
2
4k2 – 40k + 36 = 0 c2
k2 – 10k + 9 = 0
k2 – 9k – k + 9 = 0 a2
k(k – 9) – 1(k – 9) = 0
(k – 1) (k – 9) = 0 b2
k = 1 and 9. 4
(b2 4ac)
c
11. (c) Let the roots of equations are and
x2 – 3kx + 2k2 – 1 = 0 16. (b) Let two positive integers be x and x + 1.
= 2k2 – 1 According to question,
But given, =7 x2 + (x + 1)2 = 761
2k2 – 1 = 7 2x2 + 2x – 760 = 0
2k2 = 8 k2 = 4 x2 + x – 380 = 0
k 2 (x + 20)(x – 19) = 0
On putting k = ±2 in the equation, then we get x = 19 ( x – 20)
So, two consecutive integers are 19 and 20.
EBD_7367
156
A- Quadratic Equations

17. (c) Given, 3x + 27(3 –x) = 12 144


Let 3x = y a
49
27
y 12 144 36 108
y
49 7 49
y2 – 12y + 27 = 0
y2 – 9y – 3y + 27 = 0 108
2 2
(y – 3)(y – 9) = 0 y = 3, 9 49 6
when y = 3; when y = 9 18 7
3x = 3 3x = 9 7
x=1 x=2
x = 1, 2 are value of x. x (x 1) (m 1) x
22. (d) Given,
18. (a) Given expression (x 1) (m 1) m
x2 – ax + b = 0
Here and are roots of the equation m(x2 – x – m – 1) = x(mx – x – m + 1)
a and b mx2 – mx – m(m + 1) = mx2 – x2 – mx + x
x2 – x – m(m + 1) = 0
So, ( )2 4 a 2 4b Let roots be and
= 1, = – m (m + 1)
19. (c) Given, a + b = 2m2 ... (i)
b + c = 6m ... (ii) 2
1 1
and a + c = 2 ... (iii) m (m 1)
2 2
On adding equations (i), (ii) and (iii), we get
2(a + b + c) = 2m2 + 6m + 2 4m2 + 4m + 1 = 0 (2m + 1)2 = 0
a + b + c = m2 + 3m + 1 ... (iv) 1
On subtracting equation (ii) from equation (iv), we m
2
get
23. (a) The condition for both the roots of the equation
a = m2 + 3m – 1
ax2 + bx + c = 0 are positive, if
On subtracting equation (iii) from equation (iv), we get
b = m2 + 3m – 1 b c
0 and 0
On subtracting equation (i) from equation (iv), we get a a
c = –m2 + 3m + 1 Given, equation is x2 – 2(k – 1)x + (2k + 1) = 0, whose
As a b and b c roots are positive
m2 – 3m + 1 m2 + 3m – 1 and
m2 + 3m – 1 –m2 + 3m + 1 b 2 (k 1)
0 k 1
6m 2 and 2m2 2 a 1
1 c 2k 1 1
m and 1 m 1 and 0 k
3 a 1 2
k>1
1
m 1. Hence, the least value k = 1.
3 24. (c) –1.5 is a root of ax2 + x – 3 = 0
20. (c) Let number of student in each row = x then, a(–1.5)2 + (–1.5) – 3 = 0
and number of rows = y 2.25a – 4.5 = 0
Hence total number of students = xy
According to the question, 4.5
a
xy = (x + 1) (y – 2) 2.25
xy = xy –2x + y – 2 a=2
2x – y = – 2 ...(i) 25. (b) r75 > r90 is possible only when r lies between 0 and 1
and xy = (x – 1) (y + 3) (0 < r < 1)
xy = xy + 3x – y – 3 26. (c) Let the roots of equation,
–3x + y = 3 1
3x – y = 3 ax2 + bx + c = 0 are – and .
From equation (i) and (ii)
x = 5, y = 12 1 c
Hence number students = xy ( )
a
= 5 × 12 = 60
21. (d) Let and are roots of equation. c
1 c=a
7x2 + 12x + 18 = 0 a
12 18 27. (c) Given, log10 (x2 – 6x + 45) = 2
then, and (x2 – 6x + 45) = 102 = 100
7 7
Quadratic Equations 157
A-

x2 – 6x – 55 = 0 31. (c) Given, one root of the equation ax2 + x – 3 = 0 is 1


x2 – 11x + 5x – 55 = 0 a(–1)2 + (–1) – 3 = 0 a=4
x(x – 11) + 5(x – 11) = 0 4x2 + x – 3 = 0
(x + 5) (x – 11) = 0 Let other root of the euqation is
x = 11, –5 3 3
–1 .
1 1 1 4 4
28. (c) Given,
x a x b c 32. (b) The roots of equation
(a2 + b2) x2 – 2 (ac + bd)x + (c2 + d2 ) = 0 are equal.
(x b) (x a) 1
B2 – 4AC = 0, B2 = 4AC
(x a) (x b) c 4(ac + bd)2 = 4(a2 + b2)(c2 + d)2
2cx + (a + b)c = x2 + (a + b)x + ab a2c2 + b2d2 + 2abcd = a2c2 + a2d2 + b2c2 + b2d2
x2 + (a + b – 2c)x + ab – ac – bc = 0 (ad – bc)2 = 0
Let the roots of above equation be and . ad = bc
Given, =0
x x 3 3
– a(a + b – 2c) = 0 33. (a) Given,
a + b = 2c ... (i) x 3 x 2
Now, = ab – ac – bc = ab – (a + b)c x
(a b) Let y , then
x 3
= ab – (a + b) [from equation (i)]
2
1 3
2 2 2 2 y
2ab (a b 2ab) (a b ) y 2
2 2 2y2 – 2 = – 3y
29. (b) Let the roots of the equation be and . 2y2 + 3y – 2 = 0
2y2 + 4y – y – 2 = 0
5 6
and 2y(y + 2) – 1(y + 2) = 0 (2y – 1)(y + 2) = 0
k k Either, (2y – 1) = 0 or y+2=0
2 1
Given, y y = –2, y –2
3 2
2 x
y it can not be negative.
3 x 3
2 5 2 2 6 x 1
and
3 k 3 k x 3 2
5 5 2 9 On squaring both sides,
and
3 k k x 1
3 2 9 x 3 4
and 4x = x + 3
k k
x=1
9 9 So, the solution of x = 1.
k 2
k 34. (c) Given, 4x – 3.2x + 2 + 32 = 0
k = 1 and k 0. It is not satisfy the given condition. 22x – 8.2x – 4.2x + 32 = 0
(2x – 8) (2x – 4) = 0
30. (a) and be the roots of the equation :
On comparing both sides :
x2 + px + q = 0
Either 2x = 8 x=3
= –p and =q
or 2x = 4 x=2
1 1 1 1 p 35. (a) and be the roots of the equation
q x2 – x – 1 = 0
= 1 and = –1
We know that
1 1 1 1 4
and + 4 = ( 2 + 2)2 – 2(
q = [( )2 – 2 ]2 – 2( )2 = (1 + 2)2 – 2 = 9 – 2 = 7
x2 – ( –1
– –1
)x + (– –1
)(– –1
)=0 36. (d) Let roots of equations are and
= 9 and =9
p 1
x2 x 0 Now, equation is
q q x2 – ( x+( )=0
qx2 – px + 1 = 0 x2 – 9x + 9 = 0
EBD_7367
158
A- Quadratic Equations

37. (b) Given, 2x2 + 3x + c = 0


2
and 0.5 is the root of equation. So, we put x = 0.5 a2 b2 4
2(0.5)2 + 3(0.5) + c = 0 ( )2 4
2 2 2 2
0.5 + 1.5 + c = 0 a b a b
c = –2
38. (d) a and c have the same sign opposite to that of b. (a 2 b2 )2 a2 b2
39. (d) The equation will have equal roots, if
(a 2 b 2 ) 2 a2b2
B2 – 4AC = 0
(2nc)2 – 4(1 + n 2) (c2 – a2) = 0 1 1
4n2c2 – 4(c2 – n2c2 – a2 – n2a2) = 0 On solving, we get 2
.and
–4c2 + 4a2 + 4n 2a2 = 0 b a2
c2 = a2 (1 + n 2 ) 45. (a) If the roots of the equation
40. (d) Let the roots of equation a and b x2 – 2ax + a2 + a – 3 = 0 are real and less than 3.
= 2 and =4 Then, D 0 and f(3) > 0
On taking 2 and 2 4a2 – 4(a2 + a – 3) 0
+ 2 = 4 and 2 2 = +4 × 4 = +16 and (3)2 – 2a(3) + a2 + a – 3 > 0
Thus, the new equation x2 – 4x + 16 = 0. –a+3 0
41. (b) Since, sin and cos are the roots of the equation and 9 – 6a + a2 + a – 3 > 0
ax2 – bx + c = 0. –a + 3 0 and a2 – 5a + 6 > 0
a – 3 0 and (a – 3)(a – 2) > 0
b
sin + cos ... (i) a 3 ... (i)
a and a < 2 or a > 3 ... (ii)
c From equations (i) and (ii)
and sin cos ... (ii) a < 2.
a
46. (b) Let, the roots of the quadratic equation
Now, squaring on both sides of equation (i) x2 + px + q = 0 is ( ).
b2 Given that, A starts with a wrong value of p and
2 2
sin + cos + 2 sin cos obtains the roots as 2 and 6. But this time q is
a2 correct.
Put the value of sin cos i.e., Product of roots
q= = 6 × 2 = 12 ...(i)
c b2 and B starts with a wrong value of q and gets the roots
1 2
a a2 as 2 and – 9. But this time p is correct.
i.e., Sum of roots p = = –9 + 2 = –7 ... (ii)
c b2 a 2 ( )2 = ( 2
–4
2
a a2 = (–7)2 – 4.12 = 49 – 48 = 1
2ac = b – a2
2 [From equations (i) and (ii)]
a2 – b2 + 2ac = 0 =1 ... (iii)
42. (b) + = 6 and =6 From equations (ii) and (iii),
( + ) 2 = 62 = –3 and = –4
2
+ 2 + 2 = 36 which is correct roots.
2
+ 2 = 36 – 2 (6) = 24 47. (a) Quadratic equation 7x2 – 50x + k = 0
( + 3) + ( 2 + 2) + ( + )
3 Here, a = 7, b = –50 and c = k
2
=( + 2– )+( 2+ 2
)+( b
= 6(24 – 6) + (24) + (6) Since,
a
= 6(18) + 30 = 108 + 30 = 138
43. (c) 6 x 8 50
+ =
i.e., 7
1
+ – + = ( = 7, given)
6 8 7
or x [6, 8] c k k
and or 7 k 7
(x – 6) (x – 8) 0 a 7 7
44. (a) Let roots of equation are and . 48. (b) Here, given roots of equation are 3 and –1.
a 2
b 2
1 So, expression can be written as
2 2
and 2 2
(x – 3)(x + 1) = 0
a b a b = x2 – 3x + x – 3 = 0
We know that = x2 – 2x – 3 = 0
Quadratic Equations A-159
49. (d) Given equation, 3
1
2
x x 1 c c3 1 = b3
0
a b c
bcx2 + acx + ab = 0
1 1
1
Let roots are and c c 1 3c 3 c 3 1 = b3

1 ab
Product of roots = . c(c + 1 + 3b) = b3 [from equation (iii)]
bc b3 = 3bc + c2 + c
a 53. (d) ax2 + bx + b = 0
1
c b b
x2 x 0
c a a a
50. (d) Given that, f(x) is a polynomial with constant term 10 b
Sum of roots,
and all coefficients are integer. Let k, k1, k2 ... kn-1 be a
roots of nth degree polynomial.
b
Now (x – k) is a factor of f(x), where k is an integer. and Products of roots,
a
Constant term Hence, product of roots is not equal to the sum of
Product of roots =
Coefficient of x n roots, so Statement I not correct.
Now, for roots to be real and uequal.
10 Determinant, D > 0
i.e., k.k1.k 2 ... k n 1 Coefficient of xn is 1)
1 b2 – 4ac > 0
k.k1.k2 ... kn-1 = 10 = 5.2.1 ... 1 b2 – 4a (b) > 0
Therefore, the possible value of k is 5. b2 – 4ab > 0
51. (a) Let and be the roots of this quadratic equation b2 > 4ab
2x2 – 11x + 5 = 0 b > 4a
( 11) 11 So, if b > 4a, then roots are unequal and real,
... (i) so Statement II is not always true it will depend on
2 2 values of a and b.
5
and · 54. (d) x 2 6 6 6 6 ...
2
We know that
( )2 = ( )2 – 4 x2 = 6 + x 2
2 2
x2 = 6 + x
11
4
5 121 40 81 9 x2 – x – 6 = 0
2 2 4 4 4 2 x2 + 2x – 3x – 6 = 0
x(x + 2) – 3(x + 2) = 0
9 (x – 3) (x + 2) = 0
Difference of roots = ( )= = 4.5.
2 x=3
52. (a) According to question Alternate Method :
Let one roots of equation is then others roots of Given,
equation is .
x2 6 6 6 6 ...
2 ( b)
Sum of roots =
1
+ 1) = b ... (i) x 6 6 6 6 ...

2 c Here, factor of 6 = 2 and 3


Product of roots = . Sign in expression is positive.
1
3
=c = c1/3 ... (ii) So that x = 3.
From equations (i) and (ii), ( 1)
55. (b) 1
1 1 1
c3 c3 1 =b ... (iii)
1
. 1
1
On cubing both sides, we get
Now, ( + )2 = 2 + 2 + 2
2
+ 2 = (1)2 – 2 × (–1) = 3
EBD_7367
160
A- Quadratic Equations

and 2 2
( 2 2 2
) 4 2 2 b c
Put m n and mn
a a
9 4( 1) 2 5

– = ( )2 4 m2 1 n2 1 (m 2 1) (n 2 1)
Product of roots
= m n mn
1 4( 1) 5
2 2 m2 n 2 n 2 m2 1
3 3
Now, mn
( 2 2
)( ) 5. 5 5
56. (c) We know that (mn)2 (m n) 2 2mn 1
2 mn
1 1
x 2 x2 2 2
x x2 c b c
2 1
Now, put this value in expression a a a
2 c
1 1
x 2 8 x 14 a
x x
2 c2 b22ac a 2 b 2 (a c) 2
1 1
x 8 x 12 ac ac
x x We know that, quadratic equation is of the form
x2 – (Sum of roots) x + Product of roots = 0
6+2 6×2
b (a c) b2 (a c) 2
1 x2 x 0
So factor of this expression = x 6 ac ac
x
57. (c) When mistake is done in first degree term the roots acx2 + b(a + c)x + b2 + (a – c)2 = 0
of the equation are – 9 and – 1. acx2 + (ab + bc)x + b2 + (a – c)2 = 0
Equation is (x + 1) (x + 9) = x2 + 10x + 9 ...(i) 59. (a) x2 – 4x + 11
When mistake is done in constant term, the roots of This can be written as
equation are 8 and 2. = (x – 2)2 + 7
Equation is (x – 2) (x – 8) = x2 – 10x + 16 ... (ii) Here, (x – 2)2 0
Required equation from equations (i) and (ii), we get So, given function can be not be less than 7.
x2 – 10x + 9 60. (b) f(x) = 2x3 + x2 – 2x – 1
58. (a) For the given equation ax2 + bx + c = 0, m and n are = x2 (2x + 1) – 1(2x + 1) = (2x + 1) (x2 – 1)
the roots = (2x + 1) (x + 1) (x – 1)
So given expression is divisible by 2x + 1.
b 61. (a) Given,
Sum of roots = m + n = –
a x+ y= 5
c y + z = 10
and Product of roots mn = z + x = 15
a By solving equations (i), (ii) and (iii), then
x = 5, y = 0 and z = 10
m2 1 z>x>y
Sum of roots & of required equation = and
m 62. (d) Given equation
2 (a2 – bc)x2 + 2(b2 – ac)x + (c2 – ab) = 0
n 1 The given roots are equal, then D must be zero.
n D=0
i.e., [2(b2 – ac)] 2 – 4(a2 – bc)(c2 – ab) = 0
m2 1 n2 1 m 2 n n mn 2 m 4(b4 + a2c2 – 2ab2c) – 4(a2c2 – bc3 – a3b + ab2c) = 0
m n mn 4b4 + 4a2c2 – 8ab2c – 4a2c2 + 4bc3 + 4a3b
mn (m n) (m n) (m n) (mn 1) – 4ab2c = 0
4 2 3 3
4b – 12ab c + 4bc + 4a b = 0
mn mn b3 + c3 + a3 – 3abc = 0
b c a3 + b3 + c3 = 3abc
1 63. (c) Given equation,
a a b (a c)
Ax2 + Bx + C = 0
c ac Since, the given roots are –1 and 1.
a Sum of roots = –1 + 1 = 0
Quadratic Equations 161
A-

and Product of roots = 1 × (–1) = –1 67. (b) Given, an = 3 – 4n


Standard equation is an (3 – 4n) 3– 4 n
x2 – (Sum of roots) x + Product of roots = 0
On comparing with above equation from equation (i), [n (n 1)]
3n – 4
B C 2
x2 x 0 2
A A = 3n – 2n – 2n
= n – 2n2 = –n(2n –1)
C
= Product of roots C = –A 68. (a) x2 – y2 = 0 x2 = y2
A 2 2
(x –k) + y = 1
So, A and C are of opposite signs.
x2 +k2 – 2kx + y2 – 1= 0
64. (a) Given equation px2 + qx + r = 0.
Let one root of the equation be . Here we put y2 = x2
So, other root = 3 x2 + k2 – 2kx + x2 – 1 = 0
2x2 – 2kx + k2 – 1 = 0
q
Sum of roots = +3 =– 2
p 2k 2k 4 2 2k 2 1
x
q q 2 2
... (i)
p 4p 2k 4 k 2 8k 2 8
r 4
Product of roots = ( ) . (3 ) =
p 2k 8 4k 2
Put the value of 4
2
r q r 2k 2 2 k 2
3 3
p 4p p 4
3q 2 r k 2 k2
= 3q2p = 16p2r
16 p 2 p 4
3q2 = 16pr For unique positive solution we put k = 2
65. (c) m% of m + n% of n = 2% of (m × n) 2
x 1.
m n 2 2
m n (mn) 69. (c) x2 – 4x – log10 N = 0
100 100 100
if roots are real, b2 – 4ac 0
m 2 n2 2mn (–4)2 + 4. log10N 0
100 100 100 16 + 4 log10N 0
16 log10 10 + 4 log10 N 0
m2 + n2 = 2mn
log10(10)16 + log10 N4 0
m2 + n2 – 2mn = 0
log10[(10)16.N4] log101
(m – n)2 = 0
(10)16 N4 1
m –n=0
m= n æ 1 ö÷16¸4
Though, both are equal, so 100% of m is n. N4 çç ÷
çè10 ø÷
66. (b) Here m and n are the roots of the equation
x2 + ax + b = 0.
æ 1 ö÷4
m+ n = – a ...(i) N çç ÷
mn = b ...(ii) çè10 ø÷
Also, m2 and n2 are the roots of the equation of
1
x2 – cx + d = 0. N
10000
m2 + n2 = c ...(iii)
m2n2 = d ...(iv) 1
by squaring Eq. (i) both sides, we get So, minimum value of N is .
10000
m2 + n2 + 2mn = a2 [from Eqs. (i) and (ii)] Option (c) is correct.
c + 2b = a2 c = a2 – 2b 2
2b – a2 = – c æ 5ö 25
70. (c) 6 + 5x – x2 – ççç x - ÷÷÷ + + 6
Therefore, Statement 1 is incorrect. è 2 ø 4
2. From Eq. (ii) 2
49 æç 5ö
m2n2 = b2 b2 = d = - çç x - ÷÷÷
Therefore , Statements 2 is correct. 4 è 2ø
EBD_7367
162
A- Quadratic Equations

49 3x2 – 30+ x2 + 25–10x –2x2 + 5= 2(x –5) 3x 2 1x 30


So, maximum value of 6 + 5 x – x2 is .
4
2x2 – 10x = 2(x – 5) 3x 2 7x 30
æ 1ö2 1
y– 6– y2 = – ççç y - ÷÷÷ + - 6
è 2 ø 4 2x (x – 5) = 2(x – 5) 3x 2 7x 30
2
23 æç 1ö 3x 2 7x 30
=- -çç4 - ÷÷÷ x
4 è 2ø Again squaring on both sides, we get
23 x2 = 3x2 – 7x – 30
So maximum value of y–6–42 is – . 2x2 – 7x – 30 = 0
4
Difference between the maximum values Let and be roots of this equation
49 æç 23 ö÷
-- ÷ 30
4 çè 4 ø÷
=
2
49 + 23 72 = – 15
= = = 18
4 4 Option (a) is correct.
So, option (c) is correct 73. (c) Given a2 – by – cz = 0 ...(1)
71. (a) Let , be the roots of the equation lx2 + mx + m = 0 ax – b2 + cz = 0 ...(2)
p ax + by – c2 = 0 ...(3)
Given ...(1) Adding (1) and (3), we get
q
a2 + ax – cz – c2 = 0
m
Now + (sum of roots) = a(a + x) = c(c + z)
1
c
m a x c z ...(4)
and (product of roots) = a
1
Substracting (2) from (3), we get
p q m by + b2 – c2 – cz = 0
Consider
q p l c
b(b + y) = c(c + z) b+ y= c z ...(5)
Using (1) b

m x y z
= Consider
l a x b y c z
Using (4) and (5), we get
m
= l ax by z
c c z c c z c z
m
m m m ax by z
l 0 = c c z
= m l l l c z
l [Using (3) ax + by = c2]
option (a) is correct. c2 z
= c c z c z
72. (a) 3x 2
7x 30 2x 2
7x 5 x 5

3x 2 7x 30 x 5 2x 2 7x 5 c z c z
=
c z c z c z
Squaring on both sides, we get
=1
(3x2 – 7x – 30) + (x – 5)2 – 2 (x – 5) 3x 2 7x 30 Option (c) is correct.
= 2x2 – 7x – 5 74. (b)
75. (a) Consider ax2 – (a2 +1)x + a ...(1)
3x2 – 7x – 30 + x2 + 25 – 10x – 2(x – 5) 3x 2 7x 30
ax2 – a2x – x + a
= 2x2 – 7x – 5
Quadratic Equations 163
A-

ax(x – a) – 1(x – a)
q
= (x – a) (ax – 1) 3 3 2 3 6
q ( +1+3 +3 )=–p 6
q2
Given p and q are two linear factors of (1)
p = x + a and q = ax – 1 q3 [q + 1 + 3( 2 + )] = – p3q2
p + q = x – a + ax – 1 q [q + 1 + 3(– p)] = – p3
= x(a + 1) – 1(a + 1) q2 + q – 3pq = – p3
= (x – 1) (a + 1) p3 + q2 + q = 3pq
Option (a) is correct Option (c) is correct
1 1
76. (a) Let and be two roots of the given equation 78. (d) 1 0
ax2 + bx + c = 0
x x2
x2 + x – 1 0
b c
Then &
a a 1 5 1 5
x x 0
According to question, 2 2
1 1
2 2 1 5
x
2
2 2 2
2 2 2
= 2 2 2 2 1 5
and x
2
Substitute the value of + and , we get
2
79. (d) 4x2 – 16x + 0
b 2c 4
b a a
x2 4x 0
a c 16
a2
, 4
16
2 2
b b 2ac a b 2ac
2 2 2
1 2
a a c c ,3 4
2 3
2 2
–bc = a(b – 2ac)
ab2 + bc2 – 2a2c = 0 3 4
Option (a) is correct. 16
77. (c) Let , be two roots of the equation x2 + px + q = 0 48 < 64, = 49, 50...... 63 (Noof 's values = 15)
80. (b) 25x2 – 15x + 2 = 0
According to question 25x2 – 10x – 5x + 2 = 0
= 2 5x (5x – 2) – 1 (5x – 2) = 0
Sum of roots = + 2 = – p (5x – 1) (5x – 2) = 0
... (1) 1 2
Product of roots = 3 = q x ,x
5 5
...(2)
Dividing (1) by (2) we get 1 2
,
5 5
1 p
1 5
3 q New roots = (2 )
2
1 p 5
1
2 q (2 )
2
Cubing on both sides, we get equation
q3 ( + 1)3 = (– p)3 ( 2 )3 5 5 5 5
x2 x 0
2 4 2 4
EBD_7367
164
A- Quadratic Equations

15 25 a b a b
x2 x 0 84. (a) x=
4 8 a b a b
8x2 – 30x + 25 = 0 By using componendo and dividendo
81. (b) Given p and q are roots of eq
x2 + px + q = 0 x 1 2 a b a b
i.e. p + q = – P p×q=q x 1 2 a b a b
i.e. p is 1 only
Squaring both side and using componendo and
1×q=q
dividendo
which is divisible by 11.
(x 1)2 (x 1)2 a b a b
k
82. (a) x 2 3x 0 (x 1) 2
(x 1) 2 a b a b
2
+ =–3
2[x 2 12 ] 2a
k 4x 2b
+ =
2
x2 1 a
2 2 2
( ) 2 2x b
bx2 + b – 2ax = 0
k b c
9 2 85. (a) Sum of root = product of root =
2 18 a a
2, k O
k k + =–p =q
2 ( + )2 = p 2
2+ 2+ 2 = p2
83. (a) Let the roots of equation be and 2
2 + 2 = p2 – 2q
Sum of root
86. (b) Roots of Aman (4, 3)
3a 1 equation = x2 – (4 + 3) x + (4) (3)
– (2 + ) = 2
a 5a 3 eq = x2 – ( + ) x +
3a 1 x2 – 7x + 12
–3 = 2 ....
a 5a 3 here cosntant 12 is wrong
Squaring Both the side Roots of Alok (3, 2)
equation = x2 – (3 + 2) x + (3) (2)
(3a 1) 2 = x2 – 5x + 6
9 2=
(a 2 5a 3) 2 Here – 5 is wrong
So the correct equation is
...(i) x2 – 7x + 6
Product of root Solution
2 x2 – x – 6x – 16
2 × = 2 x(x – 1) – 6 (x – 1)
a 5a 3
(x – 6)(x – 1)
1 x = 6 and x = 1
2=
2 87. (a) Let the two roots be 3x and 2x. Let = 3x and = 2x
a 5a 3
...(2) Sum of the roots, a + b =
Dividing equation (i) from (ii) m m
2 2 2
3x 2 x 5x m 60 x ...(1)
9 (3a 1) a 5a 3 12 12
2
(a 2 5a 3) 2 1 Product of the roots,
5 5 5
(3a 1)2 3x.2 x 6 x2 x2
9= 12 12 72
(a 2 5a 3)
9a2 – 45a + 27 = 9a2 + 1 – 6a 5 5 5
x
26 = 39a 72 2 2 2 3 3 6 2
2
a= 5
3 Putting this value of x in (1), we get m 60
6 2
Quadratic Equations A-165
Since we need positive value of m, therefore,
b( a c ) 2ac
5 10 5 2 a c
m 60 5 10
6 2 2 b ac
88. (c) Since the roots of the given equation are equal, there- 2 1 1
fore the discriminant of the given equation is zero. b c a
Thus,
7
b2 4ac 0 [b 2 ( c a ) 2 4.a(b c).c ( a b)] 0 89. (c) We are given that x 2 7 x 14 k 2 0 . Let us
2 2 2 2
8
[b (c a 2ac ) 4ac(ab b ac bc )] 0
check for the nature of roots.
We have,
[(ab)2 (bc )2 ( 2ac )2 2ab 2 c 4a 2 bc
7
4abc 2 ] 0 D b2 4ac 72 4.1.14 k 2
8
[(ab)2 (bc )2 ( 2ac )2 2.ab.bc 2.ab.
= 49 + 56k2 – 49 = 56
( 2ac) 2.bc.( 2ac)] 0 Now,

b D 7 56k 2 7 2 k 14
[b 2 c 2 b2 a 2 2ab 2 c 4a 2 bc 4ab 2 c x
2a 2 2
4a 2 c 2 4abc 2 ] 0
Since 14 is an irrational number, therefore any value of k
[b 2 a 2 b 2c 2 4a 2 c 2 2ab2 c 4a 2 bc
will give an irrational number only.
4ab 2 c 4abc 2 ] 0 Therefore, the given equation has no integral roots.
90. (b)
( ab bc 2ac )2 0
91. (c)
ab bc 2ac 0 92. (a)
ab bc 2ac
EBD_7367
C HA P T E R
SET THEORY
16
1.
é
{ ¢ ù
}
What is ê( A È B ) Ç A ú – ( A – B ) equal to?
ë û
6. If two sets A and B have 2n and 4n elements, respectively.
When n is a natural number. What can be the minimum
number of elements in A È B? [2007-II]
[2007-I]
(a) f (b) A (a) 2n (b) 3n
(c) 4n (d) 6n
(c) B (d) B¢ 7. If A = {x : x2 – 6x + 8 = 0} and
2. If A = {(22n – 3n – 1) | n Î N} B = {x : 2x2 + 3x – 2 = 0}
and B = {9(n – 1) | n Î N}, Then, which one of the following is correct? [2007-II]
then which one of the following is correct? [2007-I] (a) A Í B
(a) A Ì B (b) B Í A
(b) B Ì A (c) Neither A Í B nor B Í A
(c) A = B (d) A = B
(d) Neither A is a subset of B nor B is a subset of A 8. If A = {1, 2, 3, 4}, then what is the number of subsets of
3. Which one of the following is correct? [2007-I] A with atleast three elements? [2008-I]
(a) {f} Ì {{f}, {{f}}} (b) {f} Î {{f}, {{f}}} (a) 3 (b) 4
(c) f Î {{f}, {{f}}} (d) f = {{f}, {{f}}} (c) 5 (d) 10
4. Let : 9. The set {2, 4, 16, 256, ...} can be represented as which one
P = Set of all integral multiples of 3 of the following? [2008-II]

{ }
Q = Set of all integral multiples of 4
n
R = Set of all integral multiples of 6 (a) x Î N | x = 22 , n Î N
Consider the following relations:

{ }
I. P È Q = R n
II. P Ì R (b) x Î N | x = 22 , n = 0, 1, 2, ...
III. R Ì (P È Q)

{x ÎN | x = 2 }
Which of the relations given above is/ are correct? 4n
[2007-II] (c) , n = 0, 1, 2, ...
(a) Only I (b) Only II

5.
(c) Only III
M P
(d) II and III (d) {x Î N | x = 2 2n
, n = 0, 1, 2, ...}
10. Which one of the following is a correct statement?
[2008-II]
(a) f Î f (b) f Ï P(f)
(c) f = P(f) (d) f Î P(f)
11. The set of integers is closed with respect to which one
C
of the following? [2008-II]
E (a) Addition only
(b) Multiplication only
The Venn diagram given above represents four sets of (c) Both addition and Multiplication
students who have opted for Mathematics, Physics, (d) Division
Chemistry and Electronics. What does the shaded region 12. Which one of the following is not correct in respect of the
represent? [2007-II] sets A and B? [2009-II]
(a) Students who opted for Physics, Chemistry and (a) If A Í B, then B È A = B
Electronics (b) If A Í B, then A Ç (A – B) = f
(b) Students who opted for Mathematics, Physics and (c) If A Í B, then B Ç A = A
Chemistry (d) If A Ç B = f, then either A = f or B = f
(c) Students who opted for Mathematics, Physics and 13. Which one of the following is a correct statement?
Electronics only [2010-I]
(d) Students who opted for Mathematics, Chemistry and (a) {a} Î {{a}, {b}, c} (b) {a} Í {{a}, b, c}
(c) {a, b} Í {{a}, b, c} (d) a Í {{a}, b, c}
Electronics only
Set Theory A-167

14. Which one of the following is an infinite set? [2010-I] (a) A = B (b) A Í B
(a) {x : x is a whole number less than or equal to 1000} (c) B Í A (d) A Í BC
(b) {x : x is a natural number less than 1000} 24. Consider the following in respect of the sets A and B.
(c) {x : x is a positive integer less than or equal to 1000} I. (A Ç B) Í A
(d) {x : x is an integer and less than 1000} II. (A Ç B) Í B
15. In an examination, 52% candidates failed in English and III. A Í (A È B)
42% failed in Mathematics. If 17% candidates failed in Which of the above are correct? [2013-I]
both English and Mathematics, what percentage of (a) I and II (b) II and III
candidates passed in both the subjects? [2011-I] (c) I and III (d) I, II and III
(a) 18% (b) 21% 25. The set of natural numbers is closed under
(c) 23% (d) 25% I. addition II. subtraction
16. The set S = {x Î N : x + 3 = 3} is a [2011-I] III. multipliction IV. division [2013-I]
(a) null set (b) singleton set Which of the above is/are correct?
(c) infinite set (d) None of the above (a) Only I (b) Both I and III
17. Consider the following statements: (c) I, II and III (d) Both III and IV
I. Set of points of a given line is a finite set. 26. In a class of 110 students, x students take both
II. Intelligent students in a class is a set. Mathematics and Statistics, 2x + 20 students take
III. Good books in a school library is a set. Mathematics and 2x + 30 students take Statistics. There
Which of the above statements is/ are not correct? are no students who take neither Mathematics nor
[2011-II] Statistics. What is x equal to? [2013-II]
(a) Only I (b) Both II and III (a) 15 (b) 20
(c) Both I and II (d) I, II and III (c) 25 (d) 30
18. If A = {x : x is an even natural number}, 27. Out of 105 students taking an examination English and
B = {x : x is a natural number and multiple of 5} and Mathematics, 80 students pass in English, 75 students
C = {x : x is a natural number and multiple of 10}, then pass in Mathematics 10 students fail in both the subjects.
what is the value of A Ç (B È C)? [2012-I] How many students fail in only one subject? [2014-I]
(a) {10, 20, 30, ...} (b) {5, 10, 15, 20, ...} (a) 26 (b) 30
(c) {2, 4, 6, ...} (d) {20, 40, 60, ...} (c) 35 (d) 45
19. If a set A contains 60 elements and another set B contains 28. If A and B are any two non-empty subsets of a set E, then
70 elements and there are 50 elements in common, then what is A È (A Ç B) equal to? [2014-I]
how many elements does A È B contain? [2012-I] (a) A Ç B (b) A È B
(a) 130 (b) 100 (c) A (d) B
(c) 80 (d) 70 29. If A is a non-empty subset of a set E, then what is
20. Let x Î {2, 3, 4} and y Î {4, 6, 9, 10}. If A be the set of E È (A Ç f) – (A – f) equal to? [2014-I]
all order pairs (x, y) such that x is a factor of y. Then, how (a) A (b) Complement of A
many elements does the set A contain? [2012-II] (c) f (d) E
(a) 12 (b) 10 30. Let A and B be finite non-empty sets with the number of
(c) 7 (d) 6 elements in A = m and number of elements in B = n. Let
21. Which one of the following is a null set? [2012-II] m > n. If for some integer k > 1, the number of non-empty
(a) A = {x is a real number : x > 1 and x < 1} subsets of A = 2k + the number of non-empty subsets of
(b) B = {x : x + 3 = 3} B, then which one of the following is correct? [2015-I]
(c) C = {f} (a) m = n + 2
(d) D = {x is a real number : x ³ 1 and x £ 1} (b) m = n + 1
22. In a school there are 30 teachers who teach Mathematics (c) m = n + p for some odd prime number p
or Physics. Of these teachers, 20 teach Mathematics and (d) m = n + t for some composite number t
15 teach Physics, 5 teach both Mathematics and Physics. 31. Let A denote the set of quadrilaterals having two diagonals
The number of teachers teaching only Mathematics is equal and bisecting each other. Let B denote the set of
[2013-I] quadrilaterals having diagonals bisecting each other at 90°.
(a) 5 (b) 10 Then AÇB denotes [2015-II]
(c) 15 (d) 20 (a) the set of parallelograms
23. If A = {x : x is an odd integer} and B = {x : x2 – 8x + 15 = 0}. (b) the set of rhombuses
Then, which one of the following is correct? (c) the set of squares
[2013-I] (d) the set of rectangles
EBD_7367
A- 168 Set Theory

32. In a gathering of 100 people, 70 of them can speak Hindi, 60 36. Let S be a set of first ten natural numbers. What is the
can speak English and 30 can speak French Further, 30 of possible number of pairs (a, b) where a, b E S and a ¹ b such
them can speak both Hindi and English. 20 can speak both that the product ab (>12) leaves remainder 4 when divided
Hindi and French. If x is the number of people who can by 12 ? [2016-II]
speak both English and French, then which one of the (a) 4 (b) 6
following is correct? (Assume that everyone can speak at (c) 8 (d) 10
least one of the three languages) [2016-I] 37. If A = (x : x is a multiple of 7),
(a) 9 < x £ 30 (b) 0 £ x < 8 B = (x : x is a multiple of 5) and
(c) x = 9 (d) x = 8 C = (X : x is a multiple of 35),
33. A is a set of positive integers such that when divided by 2, then which one of the following is a null set ? [2017-I]
3, 4, 5 and 6 leaves the remainder 1, 2, 3, 4 and 5 respectively. (a) (A – B) È C
How many integers between 0 and 100 belong to the set A ? (b) (A– B) – C
[2016-II] (c) (A Ç B) Ç C
(a) No integer (b) One (d) (A Ç B) – C
(c) Two (d) Three 38. Which one of the following is a correct statement?
34. In the quadratic equation x2 + ax + b = 0, a and b can take [2017-II]
any value from the set {1, 2, 3, 4}. How many pairs of values (a) {x : x + 5 = 5} = f
of a and b are possible in order that the quadratic equation (b) {x : x + 5 = 5} = {0}
has real roots? [2016-II] (c) {x : x + 5 = 5} = 0
(a) 6 (b) 7 (d) {x : x + 5 = 5} = {f}
(c) 8 (d) 16 39. In an examination, 35% students failed in Hindi, 45%
35. Let A = {7, 8, 9, 10, 11, 12} and B = {7, 10, 14, 15}. What is students failed in English and 20% students failed in both
the number of elements in (A–B) and (B–A) respectively ? the subjects. What is the percentage of students passing
[2016-II] in both the subjects? [2017-II]
(a) 2 and 4 (b) 4 and 2 (a) 0 (b) 20
(c) 2 and 2 (d) 4 and 4 (c) 30 (d) 40

HINTS & SOLUTIONS


{ }
= 2n + 4n – 2n = 4n
1. (a) ( A È B)¢ Ç A – ( A – B) Hence, minimum number of elements of A È B is 4n.
7. (c): Given, A º x2 – 6x + 8 = 0
= {(U – (A È B)) Ç A} – (A – B) Þ (x – 4) (x – 2) = 0
= {(U Ç A) – {(A È B) Ç A}} – (A – B) Þ x = 4, 2
= {A – A} – (A – B) and B º 2x2 + 3x – 2 = 0
= f – (A – B) = f Þ (2x – 1) (x + 2) = 0
2. (a) Given:
A = {(22n – 3n – 1) | n Î N} 1
Þ x= , –2
= {0, 9, 54, 243, ...} 2
and B = {9(n – 1) | n Î N} Hence, neither A Í B nor B Í A.
= {0, 9, 18, 27, ...} 8. (c) Given, A = {1, 2, 3, 4}
From the above, it is clear that A Ì B. So, the required subsets are {1, 2, 3}, {1, 2, 4},
3. (b) Here, {f} is an element of {f} Î {{f}, {{f}}}. {1, 3, 4}, {2, 3, 4} and {1, 2, 3, 4}
4. (c): Here, P = {..., –6, –3, 0, 3, 6, ...} 9. (b) Let A = {2, 4, 16, 256, ...}
Q = {..., –8, – 4, 0, 4, 8, ...} This set can be re-written as :

{ }
and R = {..., –36, – 6, 0, 6, 36, ...} n
I. P È Q x Î N | x = 22 , n = 0, 1, 2, ...
= {..., – 8, – 6, – 4, –3, 0 3, 4, 6, 8, ...} ¹ R
II. Here, P Ë R 10. (d) In the given options, the correct statement is f Î P(f).
III. Here, R Ì (P È Q) is true. 11. (c) The set of integers is closed with respect to addition
5. (c): It is clear from the given Venn diagram that shaded and multiplication.
portion represent the students who opted for e.g., Let z = {..., –3, –2, –1, 0, 1, 2, 3, ...}
Mathematics, Physics and Electronics only. 1 + 2 = 3 and –2 – 1 = –3 (for addition)
6. (c): Here, n(A Ç B) = 2n 1 × 2 = 2 and 2 × 1 = 2 (for multiplication)
\ n(A È B) = n(A) + n(B) – n(A Ç B) Hence, the number of subsets is 5.
Set Theory A-169

12. (d): If A Ç B = f, then it is not necessary that either A 21. (a) From option (a),
= f or B = f A = {x is a real number : x > 1 and x < 1}.
13. (a) Here, {a} is an element of {{a}, {b}, {c}}. So, there is no element which is greater or less than 1.
14. (d) In a given option only, So, A is a null set.
{x : x is an integer and less than 1000} From option (b),
i.e., x Î (– ¥, 1000) is an infinite set.
B = {x : x + 3 = 3} = {0}
15. (c) Total number of candidates = 100%
= Singleton set
From option (c):
Maths English C = {f} = Singleton set
From option (d),
25% 17% 35% D = {x is a real number : x ³ 1 and x £ 1}
= {1} = Singleton set
22. (c) Total number of teachers = 30.

Percentage of candidates passed in both the subjects Maths Physics


= {100 – (25 + 17 + 35)}% = 23%
16. (a) Given:
15 5 10
S = x Î N : {x + 3 = 3}
S={}
Thus, S is a null set.
17. (d) I. The set of points of a given line is not a finite set.
Number of teachers who teaches only Math
II. Here, we cannot decide, which students are
intelligent. = 20 – 5 = 15.
III. Here, we cannot decide, which books are good a 23. (c) Given that,
school library. A = {x : x is an odd integer}
18. (a) We know that and B = {x : x2 – 8x + 15 = 0} = (x : x2 – 5x – 3x + 15 = 0)
A Ç (B È C) = {A Ç B) È (A Ç C) = {x : x (x – 5) – 3(x – 5) = 0}
Example:
= {x : (x – 5) (x – 3) = 0} = {3, 5}
A = Set of an even natural number
A = {2, 4, 6, 8, 10, 12, ...} Since, B has the odd elements,
B = Set of natural number and multiples of 5. \BÍA
B = {5, 10, 15, 20, 25, ...} 24. (d) From figure,
C = Set of natural number and multiple of 10.
C = {10, 20, 30, 40, 50, ...} U
A Ç B = {2, 4, 6, 8, 10, 12, ...}
Ç {5, 10, 15, 20, 25, ...}
A B
= {10, 20, 30, ...}
A Ç C = {2, 4, 6, 8, 10, 12, ...}
Ç {10, 20, 30, 40, 50, ...}
= {10, 20, 30, 40, ...}
A Ç (B È C) = (A Ç B) È (A Ç C) (A Ç B)
= {10, 20, 30, ...} È {10, 20, 30, 40, ...}
(A Ç B) Í A (true)
= {10, 20, 30, 40, ...}
19. (c) Here, n(A) = 60, n(B) = 70, n(A Ç B) = 50 and (A Ç B) Í B (true)
n(A È B) = ? and A Í (A È B) also (true)
We know that: Thus, all three statements are correct.
n(A È B) = n(A) + n(B) – n(A Ç B) Shaded region = (A È B).
= 60 + 70 – 50
25. (b) Set of natural numbers,
= 130 – 50 = 80
20. (d) Given that N = {1, 2, 3, 4, 5, 6, ...
x Î {2, 3, 4} (i) Addition: 2 + 3 = 5 is also an element of N.
and (ii) Subtraction: 2 – 3 = –1 Ï N
y Î {4, 6, 9, 10} (iii) Multiplication: 2 × 3 = 6 Î N
A=x×y
But, A is set of pairs in which 1st number is factor of 3
(iv) Division: = 1.5 Ï N (since, N contains only
second number. 2
A = {2, 3, 4} × {4, 6, 9, 10} positive integers)
= {(2, 4); (2, 6); (2, 10); (3, 6); (3, 9); (4, 4)} Therefore, the set of natural numbers is closed under
Total number of elements = 6 addition and multiplication.
EBD_7367
170
A- Set Theory

26. (b) n(M) = 2x + 20 100 = 109 – x


n(S) = 2x + 30 Þ x = 109 – 100
n(M Ç S) = x Þ x= 9
n(M È S) = 110 \ Option (c) is correct.
We know that, 33. (b) LCM of 2, 3, 4, 5, 6 = 60
n(M È S) = n(M) + n(S) – n(M Ç S) Number when divided by 2, 3, 4, 5, 6 gives
Þ 110 = 2x + 20 + 2x + 30 – x remainder 1, 2, 3, 4, 5 respectively here 2 – 1 = 1
Þ 110 = 3x + 50 3 – 2 = 1, 4 – 3, = 1, 5 – 4 = 1, 6 – 5 = 1
Þ 3x = 60 So required no. = 60 – 1 = 59
\ x = 20 other no. 59 × 2 = 118
27. (d) Number of students failing in Mathematics i.e. there is one no. below 100.
= 105 – 75 = 30 34. (b) x2 + ax + b = 0 value of (a, b)
Number of students failing in English a2 – ab ³ 0 [1, 2, 3, 4]
= 105 – 80 = 25 for real roots
\ Number of students failing in 1 subject 12 – 4 × 1 ³ 0 (1, 1)
= (25 + 30) – 10 = 45 not possible
28. (c) A and B are non-empty subsets of E. (2, 1) ® 22 – 4 × 1³ 0 possible
A B (3, 1) ® possible
(3, 2), (4, 1), (4, 2) (4, 3) (4, 4) ® possible
So ® possible values can be possible
35. (b) A = {7, 8, 9, 10, 11, 12}; B = {7, 10, 14, 15}
(A – B) = {8, 9, 11, 12} (B – A) = {14, 15}
n (A – B) = 4; n (B – A) = 2
\ A È (A Ç B) = A È (Shaded portion) 36. (c) Since numbers which and leave 4 as Remainder when
=A devided by 12 are
29. (b) E È (A Ç f) – (A – f) 16, 28, 40, 52, 64, 76, 88, 100 and 124
= E È f – A = E – A = A' 16 = 2 × 8, 8 × 2
28 = 4 × 7, 7 × 4
30. (b) A and B are non-empty set
40 = 4 × 10, 10 × 4, 5 × 5, 8 × 5
A = m elements
All remaining numbers doesn't meet the requirement
B = n elements
Aswer is 8.
m > n, A > B 37. (d) A = {7, 14, 21.............}
m=n+1 B = {5, 10, 15..............}
31. (b) Externality is a result of an economic activity which is C = {35, 70, 105..............}
realised by third one. It may be of two types - negative (A Ç B) = {35, 70, 105.......} i.e. LCM of 7 and 5 i.e.
and positive. Pollution caused by a factory is negative multiple of 35
one and increase in land price of a plot due to AÇB–C=f
construction of a road.
38. (b) Solving the given equation for x, we have
32. (c) Let n(A) be no. of people who speak Hindi Þ n(A) = 70
x +5=5Þx =5–5=0
Let n(B) be no. of people who speak English Þ n(B) = 60
Therefore, in the given set we have only one element
Let n(C) be no. of people who speak French Þ n(C) = 30 i.e. 0. Element j is a null set i.e. no elements in the set.
Given n ( A È B È C ) = 100 Hence, option (b) is the correct answer.
39. (d) Let the total number of students be 100%
n ( A Ç B) = 30 , n ( A Ç C ) = 20
Number of students failed in Hindi = 35%
n ( B Ç C) = x , n ( A Ç B Ç C) = 1 Number of students failed in English = 45%
We know that Number of students failed in both the subjects = 20%
n ( A È B È C ) = n ( A ) + n ( B) + n (C ) - n ( A Ç B) Total number of students failed = (35 + 45 – 20)% =
60%
-n(BÇ C) - n(CÇ A) - n(AÇ BÇ C) Number of students passing in both the subjects =
100 = 70 + 60 + 30 – 30 – x – 20 – 1 (100 – 60)% = 40%
C HA P T E R
TRIGONOMETRIC RATIOS
17 & IDENTITIES
1. Which one of the following is true for some value of q, 9. Which of the following expression for 0º < q < 90º is/ are
where 0º £ q £ 90º? [2007-I] independent of q? [2007-II]
(a) sin q = 2 (b) sin q + cos q = 2 I. cos q(1 – sin q)–1 + cos q(1 + sin q)–1
(c) sin q + cos q = 0 (d) sin q – cos q = 1 II. cos q(1 + cosec q)–1 + cos q(cosec q – 1)–1
2. If sin q + cosec q = 2, then what is the value of Select the correct answer unsing the codes given below :
(a) Only I (b) Only II
sin4 q + cos4 q? [2007-I]
(c) Both I and II (d) Neither I nor II
(a) 2 (b) 22
(c) 23 (d) 1 p
3. If q Î R be such that sec q > 0 and 2 sec2 q + sec q – 6 = 0. 10. If cosec q = and q is acute, then what is the value of
q
Then, what is the value of cosec q? [2007-I]
æ p2 - q 2 ö
(a) (b)
3 è ø tan q? [2007-II]
5
2
(a) p (b) q
3 2
(c) (d) (c) pq (d) p2 + q2
5 3
4. Under which one of the following conditions is the 11. If a cos q – b sin q = c, then what is the value of
a sin q + b cos q? [2007-II]
sin x (1 – cos x ) true?
trigonometrical identify = (a) ± a 2 + b2 + c2 (b) ± a 2 - b2 + c2
(1 cos x )
+ sin x
[2007-I] (c) ± a 2 + b2 - c2 (d) ± a 2 - b2 - c2
(a) x is not a multiple of 360º
(b) x is not an odd multiple of 180º 12. If 2x2 cos 60º – 4 cot2 45º – 2 tan 60º = 0, then what is the
(c) x is not a multiple of 180º value of x? [2007-II]
(d) None of the above (a) 2 (b) 3
5. If 3 sin q + 4 cos q = 5, then what is 3 cos q – 4 sin q (c) 3 -1 (d) 3 + 1
equal to? [2007-I]
13. Which one of the following statements is true in respect
(a) 0 (b) 3
of the expression sin 31º + sin 32º ? [2007-II]
(c) 4 (d) 5
(a) Its value is 0
13 2 sin q - 3 cos q (b) Its value is 1
6. If sec q = , then what is the value of ?
5 4 sin q - 9 cos q (c) Its value is less than 1
[2007-I] (d) Its value is greater than 1.
(a) 1 (b) 2 14. Assertion (A) : tan 50º > 1.
(c) 3 (d) 4 Reason (R) : tan q > 1 for 0º < q < 90º. [2007-II]
7. If p = sin10 x, then which one of the following is correct (a) A and R are correct and R is correct explanation of A.
for any value x? [2007-II] (b) A and R are correct but R is not correct explanation
(a) p ³ 1 (b) 0 £ p £ 1 of A.
(c) 1 £ p £ 2 (d) None of these (c) A is true but R is false.
8. What is the value of the expression : (d) A is true but R is false.
p p p p p 15. Which one of the following is correct? [2007-II]
cos 2 + 4 cos 2 - sec + 5 tan 2 + sin 2 ?
8 4 3 3 8 1
[2007-II] (a) sin 35º > cos 55º (b) cos 61º >
2
(a) 8 (b) 10
(c) 16 (d) 18 1
(c) sin 32º > (d) tan 44º > 1
2
EBD_7367
172
A- Trigonometric Ratios & Identities

16. What is the value of x in the equation DIRECTIONS (Q. Nos. 24-25): The following two questions
2 2
cosec 30° sec 45º consists of two statements, one labelled as the ‘Assertion (A)’
x = tan2 60° – tan2 30° ? and the other as ‘Reason (R)’. You are to examine these two
8 cos 2 45° sin 2 60º statements carefully and select the answers to these items
[2007-II] using the codes given below :
(a) x = 1 (b) x = 2 Codes :
1 3 (a) Both A and R are individually true and R is the correct
(c) x = (d) x = explanation of A.
2 2
(b) Both A and R are individually true and R is not the correct
17. The smallest side of a right angled triangle has length
explanation of A.
3 (c) A is true but R is false.
2 cm. The tangent of one acute angle is . What is the
4 (d) A is false but R is true.
hypotenuse of the triangle? [2008-I] 24. Assertion (A) : sec2 23º – tan2 23º = 1.
(a) 5 cm (b) 2.5 cm Reason (R) : sec2 q – tan2 q = 1 for all real values of q.
[2008-I]
10
(c) 1.25 cm cm(d) 25. Assertion (A) : sin 1º < cos 1º.
3 Reason (R) : sin q < cos q when 0º < q < 90º.
18. If sin x – cos x = 0, then what is the value of sin4 x + [2008-I]
cos4 x? [2008-I]
æ 1 ö
3 26. What is the value of cot2 q – ç 2 ÷ = ? [2008-II]
(a) 1 (b) è sin q ø
4
1 1 1
(c) (d) (a) (b) –1
2 4 2
tan x tan x 1 3
19. What is the expression - equal to? (c) – (d)
1 + sec x 1 - sec x 2 2
[2008-I] 27. If sin2 60º + cos2 (3x – 9º) = 1, then what is the value of x?
(a) cosec x (b) 2 cosec x [2008-II]
(c) 2 sin x (d) 2 cos x (a) 24º (b) 23º
(c) 22º (d) 21º
3
20. If tan x = , where 0º < x < 90º, then what is the value 5 sin A - cot A
4 28. If cos A = , then what is the value of ?
of sin x cos x? [2008-I] 13 2 tan A

3 4 [2008-II]
(a) (b) 395 395
5 5
(a) (b)
12 13 3644 3844
(c) (d)
25 25 395 385
(c) (d)
21. Which one of the following is correct? [2008-I] 3744 3744
(a) tan x > 1, 45º < x < 90º 29. If sin x = cos y, Ðx and Ðy are acute angle then what is
1 the relation between Ðx and Ðy = ? [2008-II]
(b) sin x > , 0º < x < 30º
2 p 3p
(a) x – y = (b) x + y =
1 2 2
(c) cos x > , 60º < x < 90º
2 p p
(d) sin x = cos x for some value of x, 30º < x < 45º (c) x + y = (d) x + y =
2 4
22. What is the expression :
(sin4 x – cos4 x + 1) cosec2 x equal to? m2 - n 2
[2008-I] 30. If sin q = , then what is the value of tan q?
m2 + n 2
(a) 1 (b) 2
[2008-II]
(c) 0 (d) –1
2 2 2mn
23. If x + y = 90º, then what is cos x cosec y - cos x sin y m +n
(a) (b)
equal to? [2008-I] m2 - n 2 m2 + n 2
(a) cos x (b) sin x
m2 - n 2 m2 + n 2
(c) cos x (d) sin x (c) (d)
2mn 2mn
Trigonometric Ratios & Identities A-173

II. sec2 x + tan2 x = sec2 x tan2 x


1 1
31. If sin (x – y) = and cos(x + y) = , then what is the III. cosec2 x + tan2 x = cot2 x + sec2 x
2 2
Which of the above statements are correct? [2009-I]
value of x? [2008-II]
(a) I and II (b) II and III
(a) 15º (b) 30º
(c) I and III (d) All of these
(c) 45º (d) 60º
40. If cos x + cos2 x = 1, then what is the value of
32. If 1 + tan q = 2 , then what is the value of cot q – 1?
sin2 x + sin4 x = 1= ? [2009-I]
[2008-II]
(a) 0 (b) 1
1 (c) 2 (d) 4
(a) (b) 2
2 1
41. If sin x cos x = , then what is the value of sin (x – cos x)?
1 2
(c) 2 (d)
2 [2009-I]
33. If sin(x + 54º) = cos x, where 0 < x, x + 54º < 90º, (a) 2 (b) 1
then what is the value of x? [2008-II] (c) 0 (d) –1
(a) 54º (b) 36º 42. If tan y cosec x – 1 = tan2 y, then which one of the
2 2

(c) 27º (d) 18º following is correct? [2009-I]


34. If clock started at noon, then what is the angle turned by (a) x – y = 0 (b) x = 2y
hour hand at 3 : 45 pm? [2008-II] (c) y = 2x (d) x – y = 1º
(a) 67.5º (b) 97.5º
(c) 112.5º (d) 142.5º cos x cos x
43. If + = 2, then which one of the
1 + cosec x cosec x - 1
4
35. If the given figure, BC = 15 cm and sin B = . What is following is one of the values of x? [2009-I]
5
the value of AB? [2008-II] p p
(a) (b)
2 3
A
p p
(c) (d)
4 6
44. If x + y = 90º and sin x : sin y = 3 :1, then what is x : y
equal to? [2009-I]
(a) 1 : 1 (b) 1 : 2
(c) 2 : 1 (d) 3 : 2
B C
cos x sin x
45. If = n and = m, then (m2 – n2 ) sin2 y is
(a) 25 cm (b) 20 cm cos y sin y
(c) 5 cm (d) 4 cm equal to [2009-I]
36. If x cos 60º + y cos 0º = 3 and 4x sin 30º – y cot 45º = 2,
(a) 1 – n2 (b) 1 + n2
then what is the value of x? [2009-I]
(c) m2 (d) n 2
(a) –1 (b) 0
(c) 1 (d) 2 p
37. What is the angle (in radian) included between the hands 46. If 0 £ x £ , then which one of the following is always
2
of a clock, when the time is 10 min past 5? [2009-I] correct? [2009-I]
17 p 19 p 1 1
(a) (b) (a) sin2 x < and cos2 x >
36 36 2 2
5p 7p
(c) (d) 1 1
9 12 (b) sin2 x > and cos2 x <
2 2
38. What is log(tan 1º) + log(tan 2º) + log(tan 3º) + ...
+ log(tan 89º) equal to? 1 1
(c) sin2 x < and cos2 x <
[2009-I] 2 2
(a) 0 (b) 1 (d) Atleast one of sin 2 x, cos2 x is less than 1
(c) 2 (d) –1 47. p = tan2 x + cot2 x, then which one of the following is
39. Consider the following equations: correct? [2009-I]
I. cosec2 x + sec2 x = cosec2 x sec2 x (a) p £ 2 (b) p ³ 2
(c) p < 2 (d) p > 2
EBD_7367
174
A- Trigonometric Ratios & Identities

48. What is the value of 55. The difference of the two angles in degree measure is 1
and their sum in circular measure is also 1. What are the
5 sin 75º sin 77º + 2 cos 13º cos 15º 7 sin 81º
- ? angles in circular measure? [2009-II]
cos 15º sin 77º cos 9º
æ1 p ö æ1 p ö
[2009-I] (a) ç - , +
(a) –1 (b) 0 è 2 360 ÷ø çè 2 360 ÷ø
(c) 1 (d) 2
49. If sin x + sin y = a and cos x + cos y = b, what is æ 1 90 ö æ 1 90 ö
(b) ç - ÷ , ç + ÷
sin x . sin y + cos x . cos y equal to? [2009-I] è2 p ø è2 p ø
(a) a + b – ab (b) a + b + ab
æ1 p ö æ1 p ö
(c) ç - , +
æ a 2 + b2 - 2 ö è 2 180 ÷ø çè 2 180 ÷ø
(c) a2 + b2 – 2 (d) ç 2
÷
(d) None of these
è ø
56. If 0 < x < 45º and 45º < y < 90º, then which one of the
50. If a is the angle of first quadrant such that following is correct? [2009-II]
cosec4 a = 17 + cot4 a, then what is the value of sin a? (a) sin x = sin y (b) sin x < sin y
[2009-I] (c) sin x > sin y (d) sin x £ sin y
1 1 57. What is the value of
(a) (b)
3 4 sin 3 60º cot 30º – 2 sec2 45º + 3 cos 60º tan2 45º – tan2 60º?
1 1 [2009-II]
(c) (d) 35 –35
9 16
(a) (b)
0 8
æ1ö
51. If x + ç ÷ = 2 cos a, then what is the value of
èxø –11 11
(c) (d)
æ 1 ö 8 8
x2 + ç 2 ÷ ? [2009-I]
èx ø p p sec q - q cosec q
58. If tan q = , then what is equal to?
(a) 4 cos2 a (b) 4 cos2 a – 1 q p sec q + q cosec q
(c) 2 cos2 a – 2 sin2 a (d) cos2 a – sin2 a [2009-II]
8 1 - cos q p-q q 2 - p2
52. If cot q = , then what is the value of
15 1 + cos q (a) (b)
p+q q 2 + p2
where, q is a positive acute angle? [2009-II]
1 2 p2 - q 2
(a) (b) (c) (d) 1
5 5 q 2 + p2
3 4 59. The value of cosec2 q – 2 + sin2 q is always [2009-II]
(c) (d) (a) less than zero (b) non-negative
5 5
53. Consider the following : (c) zero (d) 1
2xy
cos2 q - sin 2 q 60. If cot q = , then what is cos q equal to?
I. = cos2 q (1 + tan q) (1 – tan q) x - y2
2
cos2 q + sin 2 q
[2009-II]
1 + sin q
II. 1 - sin q = (tan q + sec q)2 [2009-II] x 2 - y2 x 2 + y2
(a) (b)
x 2 + y2 x 2 - y2
Which of the statements given above is/ are correct?
(a) Only I (b) Only II 2xy 2xy
(c) Both I and II (d) Neither I nor II (c) 2 2
(d)
x +y x 2 + y2
cos q cos q
54. - = 2 is satisfied by which one of the 61. For what value of q is (sin q + cosec q) = 2.5, where
1 - sin q 1 + sin q
0 < q < 90º? [2009-II]
following values of q? [2009-II] (a) 30º (b) 45º
p p (c) 60º (d) 90º
(a) (b) 62. If 0 < q < f < 90º, then which one of the following is
2 3
correct? [2009-II]
p p
(c) (d) (a) (sin q + cos q)2 > 2 (b) (sin2 q + cos2 f) £ 2
4 6 (c) (sin2 q + cos2 f)2 < 2 (d) (sin2 q + cos2 f)2 > 2
Trigonometric Ratios & Identities A-175

63. What is the angle subtended at the centre of a circle of to be vertices A, B and C respectively, then what is
radius 8 m after traversing 4p m along its circumference? tan A + tan B equal to? [2010-I]
[2009-II]
u2
p p (a) (b) 1
(a) (b) uw
3 2
w2
2p 3p (c) u + v (d)
(c) (d) uv
3 4
64. If cos 1º = p and cos 89º = q, then which one of the 72. ABC is a right triangle with right angle at A. If the value
following is correct? [2010-I] 1
(a) p is close to 0 and q is close to 1 of tan B = , then for any real k the length of the
3
(b) p < q
hypotenuse is of the form [2010-I]
(c) p = q
(a) 3 k (b) 2 k
(d) p is close to 1 and q is close to 0
(c) 5 k (d) 9 k
65. Which one of the following is correct? [2010-I]
73. What is the value of
(a) There is only one q with 0º < q < 90º such that
sin12 15º + sin 2 20º + sin 2 25º + ... + sin2 75º?
sin q = a, where a is a real number.
[2010-I]
(b) There is more than one q with 0º < q < 90º such that
(a) tan2 15º + tan 2 20º + tan 2 25º + ... + tan2 75º
sin q = a, where a is a real number.
(b) cos2 15º + cos2 20º + cos2 25º + ... + cos2 75º
(c) There is no q with 0º < q < 90º such that sin q = a,
(c) cot2 15º – cot2 20º + cot2 25º – ... + cot2 75º
where a is a real number.
(d) sec2 15º + sec2 20º + sec2 25º + ... + sec2 75º
(d) There are exactly q’s with 0º < q < 90º such that
sin q = a, where a is a real number. x -1
74. If a is an acute angle and sin a = , then what is
p 2x
66. If 7 cos2 q + 3 sin2 q = 4 and 0 < q < , then what is the
2 tan a equal to? [2010-I]
value of tan q? [2010-I]
x -1 x +1
7 (a) (b)
(a) (b) x +1 x -1
7
3
(c) x2 - 1 (d) x2 + 1
(c) 3 (d) 3
67. What is the value of [(1 – sin2 q) sec2 q + tan2 q] 1
75. If cos ³ in the first quadrant, then which one of the
(cos2 q + 1) when 0° < q < 90º? [2010-I] 2
(a) 2 (b) > 2 following is correct? [2010-II]
(c) ³ 2 (d) < 2
p p
p (a) q £ (b) q ³
68. If 0 £ q < and p = sec2 q, then which one of the 3 3
2
p p
following is correct? [2010-I] (c) q £ (d) q ³
(a) p < 1 (b) p = 1 6 6
(c) p > 1 (d) p ³ 1 76. What is the value of cos 1º cos 2º cos 3º ..... cos 90º?
69. In a DABC, ÐABC = 90º, ÐACB = 30º, AB = 5 cm. [2010-II]
What is the length of AC? [2010-I] 1
(a) 10 cm (b) 5 cm (a) (b) 0
2
(c) 5 2 cm (d) 5 3 cm (c) 1 (d) 2
77. If sin q + cos q = 1, then what is the value of sin q cos q?
p [2010-II]
70. If 0 £ q £ and cos q + 3 sin q = 2, then what is the
2 (a) 2 (b) 0
value of q? [2010-I]
1
p p (c) 1 (d)
2
(a) (b)
3 4
1 + sin q
p p 78. What is
(c) (d) 1 - sin q equal to? [2010-II]
6 2
71. If ABC is a right angled triangle at C and having u units, (a) sec q – tan q (b) sec q + tan q
v units and w units as the lengths of its sides opposite (c) cosec q + cot q (d) cosec q – cot q
EBD_7367
176
A- Trigonometric Ratios & Identities

79. Two sides of an acute angle triangle are 6 cm and 2 cm,


sin q cos q
respectively. Which one of the following represents the 90. If + = 2 with 0 < q < 90º, then what is q
correct range of the third side in cm? [2010-II] cos q sin q

(a) (4, 8) (b) (4, 2 10 ) equal to?


(a) 30º (b) 45º
[2011-II]

(c) 60º (d) 75º


(c) (4 2, 8 ) (d) (4 2, 2 10 ) p p
80. What is cot 15º cot 20º cot 70º cot 75º equal to? 91. If A = and B = , then which of the following is/ are
6 3
[2011-I]
correct?
(a) –1 (b) 0
I. sin A + sin B = cos A + cos B
(c) 1 (d) 2
II. tan A + tan B = cot A + cot B
81. If sin 3q = cos (q – 2º), where 3q and (q – 2º) are acute
Select the correct answer using the codes given below.
angles, what is the value of q? [2011-I]
[2011-II]
(a) 22º (b) 23º
(a) Only I (b) Only II
(c) 24º (d) 25º
(c) Both I and II (d) Neither I nor II
sin 6 q - cos6 q 92. The Earth takes 24 h to rotate about its own axis. Through
82. What is equal to? [2011-I] what angle will it turn in 4 h and 12 min? [2011-II]
sin 2 q - cos2 q (a) 63º (b) 64º
(a) sin q – cos4 q
4
(b) 1 – sin2 q cos2 q (c) 65º (d) 70º
2 2
(c) 1 + sin q cos q (d) 1 – 3sin2 q cos2 q 93. Consider the following :
83. Consider the following : I. sin2 1º + cos2 1º = 1
I. tan2 q – sin2 q = tan2 q sin2 q II. sec2 33º – cot2 57º = cosec2 37º – tan2 53°
II. (cosec q – sin q) (sec q – cos q) (tan q + cot q) = 1 Which of the above statements is/ are correct? [2012-I]
Which of the identities above is/ are correct? [2011-I] (a) Only I (b) Only II
(a) Only I (b) Only II (c) Both I and II (d) Neither I nor II
(c) Both I and II (d) Neither I nor II 94. If p = a sin x + b cos x and q = a cos x – b sin x, then what
1 - cos B 2 tan A is the value of p2 + q2 ? [2012-I]
84. If tan A = , then what is equal to? (a) a + b (b) ab
sin B 1 - tan 2 A
(c) a2 + b2 (d) a2 – b2
[2011-I] 95. 2 2
The expression sin x + cos x – 1 = 0 is satisfied by how
tan B many values of x? [2012-I]
(a) (b) 2 tan B (a) Only one value of x (b) Two values of x
2
(c) Infinite values of x (d) No value of x
(c) tan B (d) 4 tan B
96. Consider the following statements :
85. Assume the Earth to be a sphere of radius R. What is the
I. The angular measure in radian of a circular arc of
radius of the circle of latitude 40º S? [2011-I]
fixed length subtending at its centre decreases, if the
(a) R cos 40º (b) R sin 80º
radius of the arc increases.
(c) R sin 40º (d) R tan 40º
II. 1800° is equal to 5p radian.
86. If a and b are complimentary angles, then what is
Which of the above statements is/ are correct? [2012-I]
-
1 (a) Only I (b) Only II
æ sin a cos a ö 2 (c) Both I and II (d) Neither I nor II
cosec a × cosec b ç + ÷ equal to?
è sin b cos b ø 97. Consider the following statements :
I. There is only one value of x in the first quadrant that
[2011-II] satisfies six + cos x = 2.
(a) 0 (b) 1 II. There is only one value of x in the first quadrant that
(c) 2 (d) None of these satisfies sin x – cos x = 0.
87. If A, B, C and D are the successive angles of a cyclic Which of the statements above is/ are correct? [2012-I]
quadrilateral, then what is cos A + cos B + cos C + cos D (a) Only I (b) Only II
are equal to? [2011-II] (c) Both I and II (d) Neither I nor II
(a) 4 (b) 2
(c) 1 (d) 0 5
98. If x lies in the first quadrant and cos x = , what is the
88. A unit radian is approximately equal to [2011-II] 13
(a) 57º 17¢ 43¢¢ (b) 57º 16¢ 22¢¢ value of tan x – cot x? [2012-I]
(c) 57º 17¢ 47¢¢ (d) 57º 17¢ 49¢¢ –139 139
89. How many degrees are there in an angle which equals (a) (b)
60 60
two-third of its complement? [2011-II]
(a) 36º (b) 45º 119
(c) 48º (d) 60º (c) (d) None of these
60
Trigonometric Ratios & Identities 177
A-

99. Consider the following : 7 5


cot 30º + 1 (a) (b)
8 8
I. = 2(cos 30° + 1)
cot 30º - 1 3 1
II. 2 sin 45º cos 45º – tan 45º cot 45º = 0 (c) (d)
8 8
Which of the above identities is/ are correct? [2012-I] 109. A wheel makes 12 revolutions per min. The angle in
(a) Only I (b) Only II radian described by a spoke of the wheel in 1 s is
(c) Both I and II (d) Neither I nor II [2012-II]
100. If 3 sin x + 5 cos x = 5, then what is the value of (a) 5p/2 (b) 2p/5
(3 cos x – 5 sin x)? [2012-I] (c) 3p/5 (d) 4p/5
(a) 0 (b) 2 110. If cos A + cos2 A = 1, then what is the value of
(c) 3 (d) 5
2(sin2 A + sin4 A)? [2013-I]
3 (a) 4 (b) 2
101. If tan q = and q is acute, then what is the value of
4 1
sin q? [2012-II] (c) 1 (d)
2
3 3 111. (1 – tan A)2 + (1 + tan A)2 + (1 – cot A)2 + (1 + cot A)2
(a) - (b) is equal to [2013-I]
5 5
(a) sin 2 A cos2 A (b) sec2 A cosec2 A
4 4 (c) 2 sec2 A cosec2 A (d) None of these
(c) (d) -
5 5 DIRECTIONS (Q. Nos. 112-115) : Read the following
102. What is the value of sec (90 – q)° . sin q sec 45º? information carefully to answer the questions that follow.
[2012-II] The angles A, B, C and D of a quadrilateral ABCD are in the
ratio 1 : 2 : 4 : 5.
3 A = 30º, B = 60º, C = 120º, D = 150º
(a) 1 (b)
2 112. What is the value of cos(A + B)? [2013-I]
(c) 2 (d) 3 1
(a) 0 (b)
103. If an angle measures p degrees and q radians, then which 2
one of the following is correct? [2012-II] (c) 1 (d) None of these
(a) pp = 90q (b) pp = 360q 113. What is the value of cosec(C – D + B)? [2013-I]
(c) pp = 180q (d) pp = 180p (a) 1 (b) 2
104. If the angle q is in the first quadrant and tan q = 3, them (c) 3 (d) 4
what is the value of (sinq + cosq)? [2012-II] 114. Consider the following statements :
I. ABCD is a cyclic quadrilateral.
1 2
(a) (b) II. sin(B – A) = cos(D – C)
10 10 Which of the above statements is/ are correct? [2013-I]
(a) Only I (b) Only II
3 4
(c) (d) (c) Both I and II (d) Neither I nor II
10 10 115. What is the value of sec 2 D – tan2 D? [2013-I]
105. If 0º < q < 90º, then all the trigonometric ratios can be 1 2
obtained when [2012-II] (a) (b)
(a) only sin q is given 2 3
(b) only cos q is given (c) 1 (d) None of these
(c) only tan q is given tan A - sin A
(d) any one of the six ratios is given 116. What is the value of ? [2013-I]
sin 3 A
106. What is the value of sin A cos A tan A + cos A sin A
cot A? [2012-II] sec A sec A
(a) sin2 A + cos A (b) sin2 A + tan2 A (a) (b)
1 - cos A 1 + cos2 A
(c) sin2 A + cot2 A (d) cosec2 A – cot2 A
sec A
sin q 1 + cos q (c) (d) None of these
107. What is the value of + ? [2012-II] 1 + cos A
1 + cos q sin q
117. Consider the following statements for 0 £ q £ 90º.
(a) 2 cosec q (b) 2 sec q
(c) sec q (d) cosec q I. The value of sin q + cos q is always greater than 1.
II. The value of tan q + cot q is always greater than 1.
3 Which of the above statements is/ are correct?
108. If sin q cos q = , then sin4 q + cos4 q is equal to [2013-I]
4
[2012-II] (a) Only I (b) Only II
(c) Both I and II (d) Neither I nor II
EBD_7367
178
A- Trigonometric Ratios & Identities

3 cos 2 (45º + q) + cos 2 (45º - q)


118. If sin A = and A is an acute angle, then tan A + sec A 126. What is equal to?
5 tan (60º + q) tan (30º - q)
is equal to [2013-I]
(a) 0 (b) 1 [2013-II]
(c) 2 (d) –1 (a) –1 (b) 0
(c) 1 (d) 2
x 2 - y2 127. What is sin6 q + cos6 q + 3sin2 q cos2 q equal to?
119. If sin q = , then which one of the following is [2013-II]
x 2 + y2
(a) 0 (b) 1
correct? [2013-I] (c) 2 (d) 4
2xy 2xy (1 + sec q - tan q) cos q
(a) cos q = (b) cos q = 128. What is equal to? [2013-II]
2
x -y 2 x + y2
2
(1 + sec q + tan q) (1 - sin q)
x-y xy (x - y) (a) 1 (b) 2
(c) cos q = 2 2 (d) cos q = 2 2 (c) tan q (d) cot q
x +y x +y
129. If sin q + cos q = 3 , then what is tan q + cot q equal
1 + 2 sin q cos q
120. If a =2 , then what is the value of to? [2013-II]
1 - 2 sin q cos q
(a) 1 (b) 2
a +1
? [2013-I] (c) 2 (d) 3
a -1 130. If tan q + sec q = m, then what is sec q equal to?
(a) sec q (b) 1 [2013-II]
(c) 0 (d) tan q
121. If 5 sin q + 12 cos q = 13, then what is 5 cos q – 12 sin q m2 - 1 m2 + 1
equal to? [2013-II] (a) (b)
2m 2m
(a) –2 (b) –1
(c) 0 (d) 1 m +1 m2 + 1
(c) (d)
4sin q - cos q m m
122. If 4 tan q = 3, then what is equal to?
4 sin q + 9 cos q 131. What is cosec(75º + q) – sec(15º – q) equal to?
[2013-II]
[2013-II]
(a) 0 (b) 1
1 1 (c) 2 sin q (d) 2 cos q
(a) (b)
2 3 132. If DABC is right angled at C, then what is
cos (A + B) + sin (A + B) equal to? [2013-II]
1 1
(c) (d) 1
4 6 (a) 0 (b)
123. If sin q – cos q = 0, then what is sin4 q + cos4 q equal 2
to? [2013-II] (c) 1 (d) 2
3
3 133. If a, b and g are acute angles such that sin a = ,
(a) 1 (b) 2
4
3
1 1 cos b = and tan g = 1, then what is a + b + g equal to?
(c) (d) 2
2 4 [2013-II]
124. Consider the following statements : (a) 105º (b) 120º
I. tan q increases faster than sin q as q increases. (c) 135º (d) 150º
II. The value of sin q + cos q is always greater than 1. 134. The value of cos 25º – sin 25º is [2014-I]
Which of the statements given above is / are correct? (a) positive but less than 1
[2013-II] (b) positive but greater than 1
(a) Only I (b) Only II (c) negative
(c) Both I and II (d) Neither I nor II (d) 0
(sin q + cos q) ( tan q + cot q) 4
125. What is equal to? 135. In a right angled DABC, right angle at B, if cos A = ,
sec q + cosec q 5
[2013-II] then what is the value of sice? [2014-I]
(a) 1 (b) 2 3 4
(c) sin q (d) cos q (a) (b)
5 5
3 2
(c) (d)
4 5
Trigonometric Ratios & Identities A-179
136. If a and b are complementary angles, then what is 145. If sin(A + B) = 1, where 0 < B < 45º, then what is
cos(A – B) equal to? [2014-I]
cos a cosec b - cos a sin b equal to? [2014-I]
(a) sin 2B (b) sin B
(a) sec b (b) cos a (c) cos 2B (d) cos B
(c) sin a (d) –tan b 146. At what point of time after 3 o’clock, hour hand and the
2 cos q - sin q minute hand of a clock occur at right angles for the first
137. If 2 cot q = 3, then what is equal to? time? [2014-I]
2 cos q + sin q
[2014-I] 1
(a) 9 o’clock (b) 4 h 37 min
2 1 6
(a) (b)
3 3 8 8
(c) 3 h 30 min (d) 3 h 32 min
1 3 11 11
(c) (d) 147. If from the top of a post a string twice the length of the
2 4
post is stretched tight to a point on the ground, then what
1 angle will the string make with the post? [2014-II]
138. If sin q cos q = , then what is sin6 q + cos6 q equal to?
2 p p
[2014-I] (a) (b)
(a) 1 (b) 2 6 4
1 5p p
(c) 3 (d) (c) (d)
4 12 3
139. If sec q + tan q = 2, then what is the value of sec q ? p
[2014-I] 148. If 0 < q < , then what is 1 - 2sin q cos q equal to?
4
3 [2014-II]
(a) (b) 2
2 (a) cos q – sin q (b) sin q – cos q
5 5 (c) ± (cos q – sin q) (d) cos q sin q
(c) (d) 149. If tan q + cot q = 2, then what is sinq + cos q equal to?
2 4
140. What is cosec (75º + q) – sec (15º – q) – tan (55º + q) [2014-II]
+ cot (35º – q) equal to? 1 1
[2014-I] (a) (b)
2 3
(a) –1 (b) 0
(c) 2 (d) 1
3
(c) 1 (d)
2 sec x
150. What is equal to ? [2014-II]
p cot x + tan x
141. If sin q + 2 cos q = 1, where 0 < q < , what is (a) sin x (b) cos x
2
2 sin q – cos q equal to? [2014-I] (c) tan x (d) cot x
1 sin x - cos x + 1
(a) –1 (b) 151. What is equal to ? [2014-II]
2 sin x + cos x - 1
(c) 2 (d) 1 sin x - 1 sin x + 1
142. If cos x + sec x = 2, then what cosn x + secn x equal to, (a) (b)
where n is a positive integer? [2014-I] cos x cos x
(a) 2 (b) 2 n – 2 sin x - 1 sin x + 1
(c) 2n – 1 (d) 2 n (c)
cos x + 1
(d)
cos x + 1
143. What is sin 25º sin 35º sec 65º sec 55º equal to?
[2014-I] 152. What is (sin2 x – cos2x) (1 – sin2 x cos2x) equal to?
(a) –1 (b) 0 [2014-II]
(a) sin4x – cos4x (b) sin6x – cos6x
1 (c) cos8x – sin8x (d) sin8x – cos8x
(c) (d) 1
2 153. What is (sin x cos y + cos x sin y) (sin x cos y – cos x
sin y) equal to? [2014-II]
p
144. If tan 8q = cot 2q, where 0 < 8q < , then what is the (a) cos2 x – cos2 y (b) cos2 x – sin2 y
2 (c) sin2 x – cos2 y (d) sin2 x – sin2 y
value of tan 5q? [2014-I] 154. What is (1 + cot x – cosec x) (1 + tan x + sec x) equal
1 to ? [2014-II]
(a) (b) 1 (a) 1 (b) 2
3 (c) sin x (d) cos x
(c) 3 (d) 0
EBD_7367
180
A- Trigonometric Ratios & Identities

155. What is (cosec x – sin x) (sec x – cos x) (tan x + cot x) 164. Consider the following : [2015-I]
equal to ? [2014-II]
cos A sin A
(a) sin x + cos x (b) sin x – cos x 1. + = cos A + sin A
(c) 2 (d) 1 1 – tan A 1 – cot A
156. Consider the following statements [2014-II] 2. (1– sinA– cosA)2 = 2(1–sinA) (1 + cosA)
1. sin 1° > sin 1 2. cos 1° < cos 1 Which of the above is/are identity/identities?
Which of the above statements is / are correct ? (a) 1 only (b) 2 only
(a) Only 1 (b) Only 2 (c) Both 1 and 2 (d) Neither 1 nor 2
(c) Both 1 and 2 (d) Neither 1 nor 2 165. ABC is a triangle right angled at B and AB : BC = 3 : 4.
157. If sin x + cosec x = 2, then what is sin9 x + cosec9 x equal What is sinA + sinB + sinC equal to? [2015-I]
to ? [2014-II]
11
(a) 2 (b) 18 (a) 2 (b)
(c) 512 (d) 1024 5
158. If sin x + cos x = p and sin3x + cos3x = q, then what is 12
p3 – 3p equal to ? [2014-II] (c)
5
(d) 3
(a) 0 (b) –2q 166. The value of cosec 67° + sec257° – cot233° – tan223° is
2
(c) 2q (d) 4q [2015-I]
159. If tan(A + B) = 3 and tan A = 1, then tan(A – B) is equal (a) 2 2 (b) 2
to [2015-I]
(a) 0 (b) 1 (c) 2 (d) 0
167. Consider the following statements : [2015-I]
1
(c) (d) 1. There exists at least one value of x between 0 and
3 2
p
160. If cos A = tan B, cos B = tan C and cos C = tan A then which satisfies the equation sin4 x – 2sin2 x – 1 = 0.
sin A is equal to [2015-I] 2
2. sin 1.5 is greater than cos 1.5. Which of the above
5 –1 5 –1 statements is/are correct?
(a) (b)
4 2 (a) 1 only (b) 2 only
(c) Both 1 and 2 (d) Neither 1 nor 2
3 –1 3 –1 168. If sin x + cos x = c then sin6 x + cos6 x is equal to
(c) (d)
4 2 [2015-I]
3 – tan 2 A 1 + 6c 2 – 3c 4 1 + 6c 2 - 3c 4
161. If =K (a) (b)
1 – 3 tan 2 A 16 4
where K is a real number, then cosec A(3 sinA – 4 sin 3A) 1 + 6c 2 + 3c 4 1 + 6c 2 + 3c 4
is equal to [2015-I] (c) (d)
16 4
2K 169. Consider the following statements : [2015-I]
(a) 1. There exists no value of x such that
K –1
2K 1 1 p
(b) , where £ K £ 3 = 4 + 2 3, 0 < x <
K –1 3 1 – sin x 2
2
2K 1 2. sin x = 3sin x does not hold good for any real x.
(c) , where K < or K > 3
K –1 3 Which of the above statements is /are correct?
2K (a) 1 only (b) 2 only
(d) (c) Both 1 and 2 (d) Neither 1 nor 2
K +1
170. The complement angle of 80° is [2015-I]
162. If tan A + cot A = 4
5p
then tan4 A + cot4 A is equal to [2015-I] (a) (b) radian
9
(a) 110 (b) 191
(c) 80 (d) 194 p 9
(c) radian (d) radian
1 – sin x 1– sin x cos x 18 5p
163. If p = ,q = ,r = 171. Consider the following statements : [2015-II]
1 + sin x cos x 1 + sin x
then which of the following is/are correct ? [2015-I] 2
1 + tan 2 q
æ 1 - tan q ö
1. p = q = r 1. =ç ÷ is true for all
2. p2 = qr 1 + cot q è 1 - cot q ø
2

Select the correct answer using the code given below. p p


(a) 1 only (b) 2 only 0<q< ,q ¹ .
(c) Both 1 and 2 (d) Neither 1 nor 2 2 4
Trigonometric Ratios & Identities 181
A-

181. The value of [2015-II]


1
2. cot q - is true for q = 45° only..
tan q æ pö æ pö æ pö
32 cot 2 ç ÷ - 8sec 2 ç ÷ + 8cos3 ç ÷ is equal to
Which of the above statements is/are correct ? è 4ø è 3ø è 6ø
(a) 1 only (b) 2 only
(a) 3 (b) 2 3
(c) Both 1 and 2 (d) Neither 1 nor 2
172. If x = a cosq and y =b cotq, then (ax–1 – by–1)(ax–1 + by–1) (c) 3 (d) 3 3
is equal to [2015-II]
2 2
(a) 0 (b) 1 æ sin35° ö æ cos55°ö
182. ç –
è cos55°÷ø çè sin35° ÷ø
(c) tan2 q (d) sin2 q + 2sin 30° is equal to [2016-I]

cos q æ pö (a) –1 (b) 0


173. is equal to çè where q ¹ ÷ø [2015-II] (c) 1 (d) 2
1 - sin q 2
x y x y
tan q - 1 1 + sin q 183. If – tan q = 1 and tan q + = 1, then the value of
(a) (b) a b a b
tan q + 1 cos q
tan q + 1 1 + cos q x2 y2
(c) (d) + is [2016-I]
tan q - 1 sin q a 2 b2
174. If tan (x + 40)° tan (x + 20)° tan (3x)° tan 70 – x)° tan (50 – x)° = 1, (a) 2 sec2 q (b) sec2 q
then the value of x is equal to [2015-II] (c) 2 cos2 q (d) 2 cos2 q
(a) 30 (b) 20 184. Consider the following : [2016-I]
(c) 15 (d) 10
175. If q is an acute angle and sinqcosq = 2cos3q – 1.5cosq, 1 – cos q
1. = cosec q – cot q
then what is sin q equal to? [2015-II] 1 + cos q

5 -1 1- 5 1 + cos q
(a) (b) 2. = cosec q + cot q
4 4 1 - cos q
Which of the above is/are identity identities?
5 +1 5 +1
(c) (d) - (a) 1 only (b) 2 only
4 4 (c) Both 1 and 2 (d) Neither 1 nor 2
176. Consider the following statements : [2015-II] 185. If p = cot q + tan q and q = sec q – cos q, then
1. sin66° is less than cos66°
2 2
2. sin26° is less than cos26° ( p2 p) 3 (q 2 p ) 3 is equal to [2016-I]
Which of the above statements is/are correct ? (a) 0 (b) 1
(a) 1 only (b) 2 only (c) 2 (d) 3
(c) Both 1 and 2 (d) Neither 1 nor 2
2a + 3b cos 2 q – 3cos q + 2
177. If a and b are positive, then the relation sinq = is
186. If =1 [2016-I]
3b sin 2 q
[2015-II]
(a) not possible (b) possible only if a = b where 0 <q< p, then which of the following statements is/
2
(c) possible if a > b (d) possible if a < b
are correct?
178. If tanq + secq = 2, then tanq is equal to [2015-II]
1. There are two values of q satisfying the above
3 5 equation.
(a) (b)
4 4 2. q = 60° is satisfied by the above equation.
Select the correct answer using the code given below.
3 5
(c) (d) (a) 1 only (b) 2 only
2 2 (c) Both 1 and 2 (d) Neither 1 nor 2
179. An equilateral triangle BOC is drawn inside a square ABCD. 187. Which of the following is correct in respect of the equation
If angle AOD= 2q, what is tanq equal to ? [2015-II] 3–tan 2q = a(1 – 3tan2q)? (Given that a is a real number.)
(a) 2- 3 (b) 1 + 2 [2016-I]

(c)
4- 3 (d) 2 + 3 é1 ù é 1ù
(a) a Îê ,3ú (b) a Îê –¥, ú [3, ¥]
180. The minimum value of cos2 x+ cos2y – cos2z is [2015-II] ë3 û ë 3û
(a) –1 (b) 0 é 1ù
(c) 1 (d) 2 (c) a Îê –¥, ú [3, ¥] (d) None of the above
ë 3û
EBD_7367
182
A- Trigonometric Ratios & Identities

196. Consider the following : [2016-II]


4 p
188. If tan q + cot q = , where 0<q< , then sinq + cosq is 1. sin 1° > sin 1c
3 2 2. cos 1° < cos 1c
equal to [2016-I] 3. tan 1° > tan 1c.
3 –1 Which of the above are not correct? [2016-II]
(a) 1 (b) (a) 1 and 2 only
2
(b) 2 and 3 only
3 +1 (c) 1 and 3 only
(c) (d) 2 (d) 1, 2 and 3
2
1
7 197. If tan2 x + = 2 and 0° < x < 90°, then what is the
189. If sin q + cosq = , then what is sin q – cos q equal to ? tan 2 x
2
[2016-II] value of x ? [2016-II]
(a) 15° (b) 30°
1 (c) 45° (d) 60°
(a) 0 (b)
2 198. Consider the following : [2016-II]
(c) 1 (d) 2 cos 75° sin12° cos18°
190. If sinx + sin x = 1, then what is the value of cos8x + 2cos6x
2 1. + - =1
sin15° cos 78° sin 72°
+ cos4x ? [2016-II]
(a) 0 (b) 1 cos 35° sin11°
2. - + cos 28° cosec 62° = l
(c) 2 (d) 4 sin 55° cos 79°
191. What is the value of _
cosec268° + sec256° – cot234° – tan222° ? [2016-II] sin 80°
3. – sin 59° sec 31° = 0
cos10°
1
(a) 0 (b) Which of the above are correct?
2
(a) 1 and 2 only (b) 2 and 3 only
(c) 1 (d) 2
(c) l and 3 only (d) 1, 2 and 3
192. If 2y cos q = x sin q and 2x sec q – y cosecq = 3,
then what is x2 + 4y2 equal to ? [2016-II] 199. What is the value of
(a) 1 (b) 2 tan1° tan2° tan3° tan4° ... tan89° ? [2016-II]
(c) 4 (d) 8 (a) 0 (b) 1
(c) 2 (d) 3
1+ 3 p
193. If sin q + cosq = where 0 < q < , then what is tan 200. What is the minimum value of
2 2
9 tan2q + 4 cot2 q? [2017-I]
q + cotq equal to ? [2016-II] (a) 6 (b) 9
3 1 (c) 12 (d) 13
(a) (b)
4 3 2z tan q
201. If x sin q = y cos q = , then what is 4z2 (x2 + y2)
4 1 - tan 2 q
(c) 3 (d) equal to ?
3
(a) (x2 + y2)3 (b) (x2 – y2)2
p 2
(c) (x – y ) 2 2 (d) (x2 + y2)2
194. If A = sin 2q + cos4 q where 0 £ q < , then which one of the
2 202. If cos q1 + cos q2 + cos q3 = 3, then what is sin q1 + sin q2+
following is correct? [2016-II] sin q3 equal to ? [2017-I]
3 (a) 0 (b) 1
(a) 1 £A£ 2 (b) £ A £1 (c) 2 (d) 3
4
203. What is the value of q which satisfies the equation cos q +
13 3 13 tan q = 1 ? [2017-I]
(c) £A£2 (d) £A£
16 4 16 (a) 0° (b) 30°
(c) 45° (d) 60°
cot A + cosec A–1
195. What is equal to ? [2016-II] 204. What is the value of [2017-I]
cotA– cosec A + 1
1 + cos A 1 - cos A 1 1
sin x + ?
(a) (b) 1 + cos x 1 - cos x
sin A sin A
1 + sin A 1 - sin A (a) 2 (b) 2 2
(c) (d)
cos A cos A (c) 2 tan x (d) 0
Trigonometric Ratios & Identities 183
A-

cos 4 A - sin 4 A sin1°


205. What is 2 2 equal to ? [2017-I] 212. The value of where 1c represents 1 radian is
cos A - sin A sin1c
(a) cos2 A – sin2 A (b) cos A – sin A [2017-II]
(c) 1 (d) 2 (a) Equal to 1
(b) Less than 1
206. If 7 sin2 x + 3 cos2 x = 4, 0 < x< 90°, then what is the value of
tan x? [2017-I] (c) Greater than 1 but less than 2
(d) Greater than 2
(a) 2 (b) 1
213. What is sin4q – cos4q equal to for any real number q?
1 [2018-I]
3
(c) (d) (a) 1 (b) 1 – 2 sin2q
2 3
(c) 2 cos2q + 1 (d) 1 – 2 cos2q
207. If a triangle has sides 5, 13 and 12 units and q is the acute 214. What is cot 1° cot 23° cot 45° cot 67° cot 89° equal to?
angle of the triangle, then what is the value of (sin q + cos [2018-I]
q)? [2017-II] (a) 0 (b) 1
5 7
(a) (b) 1 1
13 13 (c) (d)
2 3
12 17 215. Consider the following statements : [2018-I]
(c) (d)
13 13 1. (sec2q – 1) (1 – cosec2q) = 1
p 2. sinq (1 + cos q)–1 + (1 + cos q) (sinq)–1 = 2 cosecq
208. If 0 < x < , then (sin x + cosec x) is [2017-II] Which of the above is/are correct?
2
(a) 1 only (b) 2 only
(a) > 2 (b) < 2
(c) Both 1 and 2 (d) Neither 1 nor 2
(c) ³ 2 (d) £ 2 216. If sec x cosec x = 2, then what is tan n x + cotn x equal to?
m2 - n 2 p [2018-I]
209. If sin q = 2 2 and
0 < q < , then what is the value (a) 2 (b) 2n + 1
m +n 2
of cos q? [2017-II] (c) 2n (d) 2n – 1
217. If cos x + cos x = 1, then what is sin2 x + sin4 x equal to?
2
2mn 2mn
(a) (b) [2018-I]
m2 + n 2 m2 - n 2 (a) 1 (b) 1.5
(c) 2 (d) 3
m2 + n 2 m2 - n 2
(c) (d) 218. If sin A + cos A = p and sin3 A + cos3 A = q, then which one
2mn 2mn of the following is correct? [2018-I]
sin 45° - sin 30° sec 45° - tan 45° (a) p3 – 3p + q = 0 (b) q3 – 3q + 2p = 0
210. If A = and B = cosec 45° + cot 45° , (c) p3 – 3p + 2q = 0 (d) p3 + 3p + 2q = 0
cos 45° + cos 60°
then which one of the following is correct? [2017-II]
sec 2 q - tan q
(a) A = B (b) A > B > 0 219. If x = , then which one of the following is
(c) A < B (d) B < A < 0 sec 2 q + tan q
211. Consider the following statements : [2017-II] correct? [2018-I]
1. If 45° < q < 60°, then sec2 q + cosec2 q = a2 for 1 é1 ù
(a) <x<3 (b) x Ï ê , 3ú
some real number a > 1. 3 ë3 û
1 + cos q 1 1
2. If 0° < q < 45°, then = x 2 for some real (c) -3 < x < - (d) £x£3
1 - cos q 3 3
number x > 2. 220. ABC is a right angled triangle with base BC and height AB.
cos q sin q The hypotenuse AC is four times the length of the
3. If 0° < q < 45°, then + ³ 2. perpendicular drawn to it from the opposite vertex. What is
1 - tan q 1 - cot q tan C equal to? [2018-I]
What is the number of true statements?
(a) 2- 3 (b) 3 -1
(a) Zero (b) One
(c) Two (d) Three (c) 2+ 3 (d) 3 +1
EBD_7367
A- 184 Trigonometric Ratios & Identities

HINTS & SOLUTIONS


1. (d) (a) sin q = 2 is not possible, since sin q £ 1. 1
(b) sin q + cos q = 2 is not possible, since \ cosec q =
sin q
- 2 £ sin q + cos q £ 2.
1 3
(c) sin q + cos q = 0 = =
Þ sin q = –cos q Þ tan q = –1 5 5
3
3p 3p
Þ tan q = tan Þ q=
4 4 sin x 1 - cos x
So, q does not lie in 0º £ q £ 90º. 4. (c) =
1 + cos x sin x
Thus, option (c) is not correct.
(d) sin q – cos q = 1 Þ sin2 x = (1 – cos x) (1 + cos x)
Squaring both sides, Þ (1 – cos2 x) = (1 – cos2 x)
sin2 q + cos2 q – 2 sin q cos q = 1 which is possible for all values of x except multiples
\ 1 – sin 2q = 1 Þ sin 2q = 0 = sin 0º of 180º.
For x = 180º, sin x = 0 and 1 + cos x = 0.
np 5. (a) (3 sin q + 4 cos q) = 5
Þ q= , nÎN
2 Now, squaring both sides,
p 9 sin2 q + 16 cos2 q + 24 sin q cos q = 25
q = 0, Þ 9(1 – cos2 q) + 16(1 – sin 2 q) + 24 sin q cos q = 25
2
Þ 9 cos2 q + 16 sin2 q – 24 sin q cos q = 0
Thus, option (d) is correct.
2. (d) sin q + cosec q = 2 Þ (3 cos q – 4 sin q)2 = 0
= 3 cos q – 4 sin q = 0
1
Þ sin q + =2 13
sin q 6. (c) sec q =
5
Þ sin2 q – 2 sin q + 1 = 0
Þ (sin q – 1)2 = 0 169
Þ sec2 q =
Þ sin q = 1 25
Þ sin q = sin 90°
169
Þ q = 90° Þ 1 + tan2 q =
\ sin4 q + cos4 q = sin4 90° + cos4 90° 25
=1+0=1 169
Aliternate Method Þ tan2 q = –1
25
Given that
sinq + cosecq = 2 144
Þ tan2 q =
Now, put – q = 90° 25
1+1=2
12
Similary put q = –90° Þ tan q = ... (i)
sin4q + cos4q 5
= sin490 + cos4 90°
sin q
=1+0=1 2 -3
3. (c) Given, 2 sec2 q + sec q – 6 = 0 2 sin q - 3 cos q cos q
\ =
Þ 2 sec2 q + 4 sec q – 3 sec q – 6 = 0 4 sin q - 9 cos q sin q
4 -9
Þ 2 sec q(sec q + 2) – 3(sec q + 2) = 0 cos q
Þ (2 sec q – 3) (sec q + 2) = 0
Puting the value of tanq from equation (i)
3
Þ sec q = (Q sec q ¹ –2, sec q > 0) æ 12 ö
2 2ç ÷ -3
2 sin q - 3cos q 2 tan q - 3 è5ø
= = =
2 4 sin q - 9 cos q 4 tan q - 9
Þ cos q = æ 12 ö
3 4ç ÷ -9
è5ø
4 5
Þ sin q = 1 - cos 2 q = 1 - = 24 - 15 9
9 3 = = =3
48 - 45 3
Trigonometric Ratios & Identities 185
A-

7. (b) We know that, Þ a2 (1 – sin2 q) + b2 (1 – cos2 q) – 2ab sin q cos q = c2


0 £ sin2 x £ 1 Þ a2 + b2 – c2 = a2 sin2 q + b2 cos2 q + 2ab sin q cos q
Þ 0 £ sin10 x £ 1 Þ (a sin q + b cos q)2 = a2 + b2 – c2
Þ 0£p£1 (Q p = sin10 x)
p pö p p p Þ a sin q + b cos q = ± a 2 + b2 - c2
æ
8. (c) ç cos 2 + sin 2 ÷ + 4 cos2 - sec + 5 tan 2
è 8 8ø 8 3 3 12. (d) Given, 2x2 cos 60º – 4 cot2 45º – 2 tan 60º = q
2 1
æ 1 ö Þ 2x2 × – 4(1)2 – 2 × 3 =0
( 3)
2
=1+4× ç –2+ 5 2
è 2 ÷ø
(Q sin2 q + cos2 q = 1) Þ x2 – 4 – 2 3 = 0
= 1 + 2 – 2 + 15 = 16
Þ x2 = 4 + 2 3
cos q cos q
9. (d) I. + Þ x2 = 3 + 1 + 2 3
1 - sin q 1 + sin q

( 3)
2
+ (1) + 2 3 × 1
2
cos q [1 + sin q + 1 - sin q] Þ x2 =
=
(1 - sin q) (1 + sin q)
( )
2
2 cos q 2 2 cos q Þ x2 = 3 +1
= = =
1 - sin q cos q cos q
2 2
Þ x= 3 +1
cos q cos q 13. (d) We know:
II. +
1 + cosec q cosec q - 1
1
sin 30º =
cos q [cosec q - 1 + 1 + cosec q] 2
=
cosec2 q - 1 Value of sin increases 0º to 90º
\ sin 31º > sin 30º and sin 32º > sin 30º
2 cos cosec q 2 cot q 2
= = = 1 1
2
cot q 2
cot q cot q Þ sin 31° > and sin 32° >
2 2
Thus, neither I nor II independent of q.
On adding both sides, we get
p sin 31º + sin 32º
10. (b) Given, cosec q =
q
1 1
Þ + Þ sin 31º + sin 32º > 1
2 2 2 2
b= p -q
14. (c) We know that, tan q is increasing in 0º to 90º and
C tan 45º = 1.
\ tan 50º > 1.
So, A is true but R is false.
p 15. (c) We know that, sin q is increasing in 0º to 90º.
q
1
Q sin 30º =
2
q
1
A 2 2 B \ sin 32º >
p –q 2

In DABC, cosec2 30° sec 2 45º


16. (a) Given, x = tan2 60° – tan2 30°
q 8 cos 2 45° sin 2 60º
tan q =
p - q2
2

( 2)
2
x ´ (2) ´
2 2
æ 1 ö
( 3)
2 2 2
Þ p -q . tan q = q Þ = -ç
æ 1 ö
2
æ 3ö
2 è 3 ÷ø
11. (c) Given, a cos q – b sin q = c 8´ ç ´ç
è 2ø÷ ÷
On squaring both sides, we get è 2 ø
a2 cos2 q + b2 sin2 q – 2ab cos q sin q = c2
EBD_7367
186
A- Trigonometric Ratios & Identities

x´4´2´4 1 3 3
Þ =3–
1 3 4 = 4 = 3
8´ ´3 =
2 9 25 5
1+
16 16
8x 8
Þ = Þx=1
3 3 1 1 1 4
and cos x = = = =
3 P 1 + tan 2 x æ3ö
2 16 + 9 5
17. (d) Since, tan q = = 1+ ç ÷
4 B è4ø 16

\ H = P 2 + B2 = 9 + 16 = 25 = 5
3 4 12
Let the length of hypotenuse = x cm \ sin x cos x = ´ =
5 5 25
C
1
21. (a) Since, sin x < , 0º < x < 30º
2

5 1
3 and cos x < , 60º < x < 90º
2
then, sin x = cos x only for x = 45º in first quadrant.
Hence, option (a) is correct.
q
22. (b) (sin4 x – cos4 x + 1) cosec2 x
A 4 B
= {(sin2 x – cos2 x) (sin2 x + cos2 x) + 1} cosec2 x
2 3 2 ´ 5 10 [(Q a2 – b2 = (a + b) (a – b)]
\ sin q = = Þx= = cm = (sin2 x – cos2 x + 1) cosec2 x
x 5 3 3
18. (c) sin x – cos x = 0 = (sin2 x + sin2 x) cosec2 x
Þ sin x = cos x (Q 1 – cos2 x = sin2 x)
p 1
Þ x = 45º = = 2 sin2 x . =2
4 sin 2 x
p
Now, x = 45º = 23. (b) cos x cosec y - cos x sin y
4
(Q x + y = 90º, given)
p p 4
4
\ sin x + cos x = sin 4
+ cos 4 = cos x × cosec (90º - x ) - cos x × sin (90º - x )
4 4
4 4
æ 1 ö æ 1 ö 1 1 1 = cos x × sec x - cos 2 x
=ç +ç = + =
è 2 ÷ø è 2 ÷ø 4 4 2
= 1 - cos 2 x = sin 2 x = sin x
tan x tan x
19. (b) Given that, - 24. (a) Both A and R are individually true and R is correct
1 + sec x 1 - sec x
explanation of A.
tan x(1 - sec x - 1 - sec x) 25. (c) A. It is true.
= R. We know that, cos q > sin q, 0º < q < 45º and
1 - sec2 x
sin q > cos q,
2 45º < q < 90º.
-2 tan x sec x cos x Therefore, A is true but R is false.
= = = 2 cosec x
- tan 2 x sin x
2 1 cos 2 q 1 sin 2 q
cos x 26. (b) cot q - = - =- = -1
sin 2 q sin 2 q sin 2 q sin 2 q
3 27. (b) Given equation can be rewritten as
20. (c) Given, tan x = , 0º < x < 90º
4 cos2 (3x – 9º) = 1 – sin 2 60º
tan x Þ cos2 (3x – 9º) = cos2 60º
\ sin x = , 0° < x < 90° Þ 3x – 9º = 60º
1 + tan 2 x Þ 3x = 69º Þ x = 23º
Trigonometric Ratios & Identities A- 187

5 1 1
28. (c) Given, cos A = 31. (c) Given, sin (x – y) = and cos (x + y) =
13 2 2
Þ sin (x – y) = sin 30º
C and cos (x + y) = cos 60º
Þ x – y = 30º and x + y = 60º
\ x = 45º and y = 15º
13
12 32. (b) 1 + tan q = 2
Þ tan q = 2–1

A 5 B 1 2 +1
\ cot q – 1 = –1= –1= 2
2 -1 2 -1
12 5 33. (d) Given, sin(x + 54º) = cos x
-
sin A - cot A 13 12 Þ sin(x + 54º) = sin(90º – x) (Q 0º < x < 90º)
\ =
2 tan A 12 Þ x + 54º = 90º – x

5 Þ 2x = 36º Þ x = 18º
34. (c) Given time =
144 - 65 395
= = æ 45 ö
12 3744 3 : 45 pm = ç 3 + ÷ h
13 ´ 12 ´ 2 ´ è 60 ø
5
29. (c) Since, sin x = cos y æ 3ö 15
As Ðx and Ðy are acute angles, then
= ç3+ ÷h = h
è 4ø 4
p We know that
x=y= The hour hand revolve 360° in 12 hour.
4
So, 12h hour hand made 360°
p
\ x+y = 360°
2 \ 1h hour hand made
12
æ p p p ö
çQ 0 < x < 2 and 0 < y < 2 Þ 0 < x + y < 12 ÷ \
15
h hour hand made =
360 15
´
è ø
4 12 4
m2 - n 2 450°
30. (c) sin q = 2 2
= = 112.5°
m +n 4
4
A 35. (a) BC = 15 cm and sin B =
5
2
n A
2

+
m –n

2
m
2

q 25 cm
C B
20 cm

In DABC, BC = (AC)2 - (AB)2


B C
=
4 4 2 2
m + n + 2m n - m + n - 2m n ( 4 4 2 2
) AC 4
sin B = =
AB 5
= 4 m2 n 2 = 2mn
then BC = 3m
But, BC = 15 (given)
m2 - n 2
\ tan q = then AC = 4 × 5 = 20
2mn AB = 5 × 5 = 25.
Hence, the value of AB is 25 cm.
EBD_7367
188
A- Trigonometric Ratios & Identities

36. (d) Given that, 42. (a) Given, tan 2 y cosec2 x – 1 = tan2 y
x cos 60º + y cos 0º = 3 Þ tan2 y cosec2 x – tan2 y = 1
Þ tan2 y (cosec2 x – 1) = 1
x
Þ +y=3 Þ tan2 y . cot2 x = 1
2 Þ cot2 x = cot2 y
Þ x + 2y = 6 ... (i) Þ x=y
and 4x sin 30º – y cot 45º = 2 \ =x–y=0
1 cos x cos x
Þ 4x × – y.1 = 2 43. (c) Given, + =2
2 1 + cosec x cosec x - 1
Þ 2x – y = 2 ... (ii)
On solving equations (i) and (ii), we get x = y = 2. 2 cos x cosec x
Þ =2
37. (b) In 60 min h hand gains = 5 min cosec2 x - 1
5 cos x cosec x
In 1 min h hand gains = min =1
60 Þ
cot 2 x
5 5 Þ tan x = 1
In 10 min h hand gains = ´ 10 = min
60 6 p
There is 15 min gap between hours and minutes Þ x=
4
5
hands but in 10 min h hand gains min more. 3 1
6 44. (c) sin x : sin y = 3 :1 = :
2 2
5 95 = sin 60º : sin 30º
So, the actual gap = 15 + = min
6 6 \ x : y = 60 : 30
In 1 min, there are 6º, Þ x : y = 2 : 1.
95 cos x sin x
In min, there are 45. (a) Given, = n, =m ... (i)
6 cos y sin y

95 95 p 19 p æ sin 2 x cos 2 x ö 2
× 6° = ×6× = radian
6 6 180 36 Now, (m – n ) sin y = ç 2 -
2 2 2
2 ÷
sin y
è sin y cos y ø
38. (a) log(tan 1º) + log(tan 2º) + ... + log(tan 89º)
= log(tan 1º tan 2º ... tan 45º ... tan 88º tan 89º)
(1 - cos 2 x) cos 2 y - cos 2 x (1 - cos 2 y)
[Q tan 89º = tan (90º – 1º) = cot 1º] =
= log[(tan 1º cot 1º) (tan 2º cot 2º) ... tan 45º] cos 2 y
= log(1º . 1º ... 1º) = 0
cos 2 y - cos 2 x
2 2 = = 1 - n2 [from equation (i)]
cos x + sin x cos 2 y
39. (c) I. L.H.S. = cosec x + sec x =
2 2
sin 2 x cos2 x
p
= cosec2 x sec2 x = R.H.S. 46. (d) For 0 £ x £ , cos2 x and sin2 x lies between 0 to 1.
2
1 + sin 2 x Hence, atleast one of sin 2 x, cos2 x is less than 1.
II. L.H.S. = sec2 x + tan 2 x = ¹ R.H.S. 47. (b) Given, p = tan2 x + cot2 x
cos 2 x
= (tan x + cot x)2 – 2
III. L.H.S. = cosec2 x + tan 2 x
= cot2 x + 1 + tan2 x = cot2 x + sec2 x = R.H.S. æ sin 2 x + cos2 x ö
2
æ 2 ö
2
40. (b) Given, cos x + cos2 x = 1 =ç ÷ - 2 = çè sin 2x ÷ø - 2
Þ cos = sin2 x è sin x cos x ø
On squaring both sides, we get
4
cos2 x = sin4 x = -2
sin2 x + sin4 x = 1 sin 2 2x
41. (c): Now, (sin x – cos x)2 = (sin2 x + cos2 x) – 2 sin x cos x Since, the maximum value of sin 2x is 1.

æ1ö æ 1 ö 4
\ pmin = -2= 2
=1- 2 ç ÷
è2ø çèQ sin x cos x = , given ÷ø 1
2
\ p³2
=0
Hence, p ³ 2.
Trigonometric Ratios & Identities A-189

Alternate Method
8
1 52. (c) cot q =
p = tan2 x + cot2 x = tan2 x + 15
tan 2 x
AC = 82 + 152
\ A.M. ³ G.M.
= 64 + 225 = 17
1
1 æ 1 ö2 C
Q tan 2 x + ³ 2 ç tan 2 x × ÷
2
tan x è tan 2 x ø

2 1 17
Þ tan x + 2
³2 ÞP³2 15
tan x
5 sin 75º sin 77º + 2 cos 13º cos 15º 7 sin 81º q
48. (b) -
cos 15º sin 77º cos 9º A 8 B

5 cos 15º sin 77º + 2 sin 77º cos 15º 7 cos 9º 8


= - Þ cos q = (ÐB = 90º)
cos 15º sin 77º cos 9º 17

7 cos 15 × sin 77 7 cos 9 8


= - =7–7=0 1-
cos 15 × sin 77 cos 9 1 - cos q 17 = 9 3
\ = =
1 + cos q 8 25 5
49. (d) sin x + sin y = a and cos x + cos y = b 1+
Þ sin2 x + sin2 y + 2 sin x sin y = a2 ... (i) 17
2 2 2
and cos x + cos y + 2 cos x cos y = b ... (ii) 53. (c) I. R.H.S. = cos2 q (1 + tan q) (1 – tan q)
Adding equations (i) and (ii), = cos2 q (1 – tan2 q)
Þ (sin2 x + cos2 x) + (sin 2 y + cos2 y)
æ cos2 q - sin 2 q ö
+ 2(sin x sin y + cos x cos y) = cos 2 q ç ÷
= a2 + b2 è cos2 q ø

a 2 + b2 - 2
Þ (sin x sin y + cos x cos y) = cos2 q - sin 2 q
2 = = L.H.S.
cos2 q + sin 2 q
50. (a) cosec4 a – cot4 a = 17 (Given)
2 2 2 2
Þ (cosec a – cot a) (cosec a + cot a) = 17
1 + sin q (1 + sin q) 2
II. L.H.S. = =
æ 1 + cos 2 a ö 1 - sin q 1 - sin 2 q
Þ 1× ç 2 ÷ = 17
è sin a ø 2
æ 1 + sin q ö
Þ 2 – sin2 a = 17 sin2 a =ç = (sec q + tan q)2
è cos q ÷ø
1
Þ 18 sin2 a = 2 Þ sin2 a = Hence, both statements are correct.
9
cos q cos q
1 54. (c) Given, - =2
\ sin a = (since, a lie in first quadrant) 1 - sin q 1 + sin q
3
1 cos q + sin q cos q - cos q + cos q sin q
51. (c) x + = 2 cos a Þ =2
x 1 - sin 2 q
Squaring both sides, then we get Þ 2 sin q cos q = 2 cos2 q
Þ cos2 q (tan q – 1) = 0
1
x2 + + 2 = 4 cos2 a p
x2 Þ tan q = 1 and cos q ¹ 0 Þ q =
4
1 55. (b) Let angles in circular meausres are A and B, then
Þ x2 + = 2(2 cos2 a – 1)
2
x pA pB
degree measures will be and .
= 2(2 cos2 a – sin2 a – cos2 a) 180º 180º
= 2 cos2 a – 2 sin2 a According to question,
A+B=1 ... (i)
EBD_7367
190
A- Trigonometric Ratios & Identities

59. (b) cosec2 q – 2 + sin2 q = (sin q – cosec q)2


pA pB
and - =1 ... (ii) Hence, it is always non-negative.
180º 180º
On solving equations (i) and (ii), we get 2xy
60. (c) Given, cot q =
x - y2
2
90 æ p ö æ 1 90 ö
A= çè + 1÷ Þ A = çè + ÷ø
p 180 ø 2 p C
From equation (i),
1 90 æ 1 90 ö
+ + B =1 Þ B = ç - ÷
2 p è2 p ø x2 – y2
56. (b) As we know, sin x is increasing from 0 to 90º.
\ sin y > sin x. q
57. (b) sin3 60º cot 30º – 2 sec2 45º + 3 cos 60º A 2xy B
tan2 45º – tan2 60º
In DABC,
3 AC2 = (x2 – y2)2 + (2xy)2
æ 3ö 2 æ1ö 2
=ç ÷ ( 3) - 2( 2) + 3 çè 2 ÷ø (1) - ( 3) Þ AC2 = (x2 + y2)2 Þ AC = x2 + y2
è 2 ø
AB 2xy
\ cos q = =
9 3 -35 AC x 2 + y 2
= -4 + -3=
8 2 8 61. (a) Here, (sin q + cosec q) = 2.5
p æ 1 ö 5
58. (c) Given, tan q = Þ ç sin q + =
q è sin q ÷ø 2
Þ 2 sin2 q – 5 sin q + 2 = 0
C
Þ 2 sin2 q – 4 sin q – sin q + 2 = 0
Þ 2 sin q (sin q – 2) – 1(sin q – 2) = 0
Þ (2 sin q – 1) (sin q – 2) = 0
p2 + q 2 p 1
Þ sin q = (Q sin q ¹ 2)
2
\ q = 30º
62. (c) For 0º < q < f < 90º, 0 < sin2 q, cos2 f < 1
q
\ sin2 q + cos2 f < 2.
A q B
Arc 4p p
63. (b) Angle subtended = = =
Radius 8 2
p2 + q 2 p2 + q 2 64. (d) We know that, the value of cos q is decreasing from
Þ sec q = and cosec q =
q p 0 to 90º.
\ cos 1º > cos 89º
p sec q - q cos ecq Þ p>q
\
p sec q + q cosec q Also, cos 1º is close to 1 and cos 89º is close to 0.
Hence, option (d) is correct.
65. (a) It is true, for 0º < q < 90º, there exist only one q such
æ p2 + q2 ö æ p2 + q 2 ö that sin q = a.
pç ÷ - qç ÷
çè q ÷ø çè p ÷ø p
= 66. (d) Given, 7 cos2 q + 3 sin2 q = 4 and 0 < q <
æ p2 + q2 ö æ p2 + q 2 ö 2
pç ÷ + qç ÷ C
çè q ÷ø çè p ÷ø

p q
-
q p p2 - q2 2 3
= =
p q p2 + q2
+
q p
q
A 1 B
Trigonometric Ratios & Identities A-191

Þ 7(1 – sin2 q) + 3(sin2 q) = 4 71. (d) In DABC,


Þ 7 – 4 sin2 q = 4
Þ 4 sin2 q = 3 A
3
Þ sin q = ±
2
p v w
For 0 < q < ,
2

3
sin q =
2 C u B
3
\ tan q = = 3 BC u
1 tan A = =
67. (b) [(1 – sin2 q) sec2 q + tan2 q] (cos2 q + 1) AC v
(Q sin2 q + cos2 q = 1) v
= [cos2 q . sec2 q + tan2 q] (cos2 q + 1) and tan B =
u
= (1 + tan2 q) (cos2 q + 1) (Q sec2 q – tan2 q = 1)
2 2 Also, u2 + v2 = w2 (by Pythagoras theorem) ... (i)
= sec q (cos q + 1)
= sec2 q . cos2 q + sec2 q u v u 2 + v2 w 2
= 1 + sec2 q > 1 + 1 > 2 \ tan A + tan B = + = =
v u uv uv
(Q sec2 q > 1 for q < q < 90º)
[from equation (i)]
é pù 72. (b) Given,
68. (d) We know the interval q Î ê0, ú , sec 2 q is increasing
ë 2û C
from 1 to ¥.
\ p ³ 1.
69. (a) In DBAC,
1k
A

60º
A 3k B
5 cm
In DABC,

90º k
30° tan B =
B C 3k
By Pythogaros theorem,
AB2 + AC2 = BC2
AB 1 5
cos 60º = Þ =
( )
2
AC 2 AC Þ 3k + (1k)2 = BC2
\ AC = 10 cm
Þ BC2 = 4k2
70. (a) Given that, cos q + 3 sin q = 2 Þ BC = 2k
73. (b) sin2 15º + sin 2 20º + sin 2 25º + ... + sin2 75º
1 3 = sin2 (90º – 75º) + sin2 (90º –70°) + ... + sin2 (90° – 15°)
Þ cos q + sin q = 1
2 2 = cos2 75º + cos2 70º + ... + cos2 15º
Þ sin 30º cos q + cos 30º sin q = 1 74. (a) Given that,
Þ sin (30º + q) = sin 90º x -1
30° + q = 90° sin a =
2x
\ q = 60º
In DABC, using Pythagoras theorem,
AC2 = AB2 + BC2
EBD_7367
192
A- Trigonometric Ratios & Identities

C 80. (c) cot 15º cot 20º cot 70º cot 75º
= tan (90º – 15º) tan (90º – 20º) cot 70º cot 75º

1 1
2x x -1 = tan 75º tan 70º × =1
tan 70º tan 75º
81. (b) sin 3q = cos (q – 2º)
a
Þ sin 3q = sin [90º – (q – 2º)]
A B
Þ 3q = 90º – q + 2º
Þ 2x = AB2 + (x – 1)
92º
Þ AB2 = x + 1 Þ 4q = 92º Þ q = = 23º
4
Þ AB = x + 1
sin 6 q - cos6 q (sin 2 q)3 - (cos 2 q)3
BC x -1 82. (b) =
\ tan a = = sin 2 q - cos 2 q sin 2 q - cos 2 q
AB x +1
75. (a) We know that, if value of cos q increases, then the
value of q decreases. (sin 2 q - cos 2 q) (sin 4 q + cos 4 q + sin 2 q cos 2 q)
=
sin 2 q - cos 2 q
1
Q cos q ³ = sin4 q + cos4 q + 2 sin2 q cos2 q – sin2 q cos2 q
2
= (sin2 q + cos2 q)2 – sin2 q cos2 q = 1 – sin2 q cos2 q
p p
\ cos q ³ cos Þq £
3 3 sin 2 q p
83. (d) I. tan 2 q – sin2 q = – sin2 q, q ¹ (2n + 1)
76. (b) We know, cos 90º = 0 2
cos q 2
\ cos 1º cos 2º cos 3º ..... cos 90º = 0.
77. (b) Given, sin q + cos q = 1
sin 2 q (1 - cos2 q)
Squaring both sides, = , q ¹ (2n + 1) p
cos2 q 2
(sin2 q + cos2 q) + 2 sin q cos q = 1
Þ 1 + 2 sin q cos q = 1 Þ sin q cos q = 0.
sin 2 q p
= sin 2 q, q ¹ (2n + 1)
1 + sin q (1 + sin q) (1 + sin q) 2
cos q 2
78. (b) =
1 - sin q (1 - sin q) (1 + sin q)
p
= tan2 q sin2 q, q ¹ (2n + 1)
2
(1 + sin q)2 (1 + sin q)2 1 + sin q
= = = II. (cosec q – sin q) (sec q – cos q) (tan q + cot q)
1 - sin q2 2
cos q cos q
æ 1 öæ 1 ö æ 1 ö
1 sin q =ç - sin q ÷ ç - cos q ÷ çè tan q + tan q ÷ø
= + = sec q + tan q è sin q ø è cos q ø
cos q cos q
é pù
a 2 + b 2 - c2 êq ¹ np (2n + 1) 2 ú
79. (b) Q cos q = By cosine rule ë û
2ab
cos 2 q sin 2 q sec2 q p
62 + 22 - c 2 40 - c 2 = , q ¹ np, (2n + 1)
= = sin q cos q tan q 2
2´6´ 2 24
For acute angle, 1 cos q p
= sin q cos q × , q ¹ n p, (2n + 1)
40 - c 2 cos q sin q
2 2
cos q > 0 Þ > 0 Þ c2 < 40
24 =1
Þ 0 < c < 2 10 (since, c cannot be negative) ... (i) Since, to become an identity, both statements must be
satisfied for every value of q.
Also, b + c > a
Therefore, neither I nor II are the identities.
c>6–2Þc>4
From equations (i) and (ii),

(
c Î 4, 2 10 )
Trigonometric Ratios & Identities 193
A-

1 - cos B 1 1
84. (c) tan A =
sin B = 1
´ (sin a sin b ) 2 = 1
(sin a sin b ) 2
1 - cos B 87. (d) We know that in a cyclic quadrilateral sum of opposite

2 tan A sin B angle is 180º.
\ =
1 - tan 2 A æ 1 - cos B ö
2
\ A + C = 180º ... (i)
1- ç
è sin B ÷ø and B + D = 180º ... (ii)
\ cos A + cos B + cos C + cos D
= cos A + cos B + cos (180º – A) + cos (180º – B)
2 (1 - cos B) sin B
From equations (i) and (ii),
(sin )
= 2
B - 1 - cos 2 B + 2 cos B = cos A + cos B – cos A – cos B = 0
88. (b) We know that,
2(1 - cos B) sin B p radian = 180º
=
-2 cos 2 B + 2 cos B Þ 1 radian =
180º 180º
= ´ 7º
p 22
2 sin B (1 - cos B) sin B
= = = tan B 630º 3º 3 ´ 60
2 cosB (1 - cos B) cos B = = 57 = 57º + min
11 11 11
85. (a) In DOAB,
4
= 57º + 16¢ + min
r 11
A 40º B
4
R = 57º + 16¢ + × 60 s = 57º + 16¢ + 21.8¢¢
O 11
= 57º 16¢ 21.8¢¢ = 57º 16¢ 22¢¢
89. (a) Given that, a + b = 90º ... (i)
According to question,

AB r 2
cos 40º = Þ cos 40º = b= a
OB R 3
Þ r = R cos 40º 2 2
So, the radius of the circle of latitude 40º S is \ b= a = (90º - b ) [from equation (i)]
3 3
R cos 40º.
86. (b) Given, a + b = 90º ... (i) 2
Þ b = 60º – b Þ b = 36º
1 3
-
æ sin a cos a ö 2
\ cosec a × cosec b ç + ÷ sin q cos q
è sin b cos b ø 90. (b) Given that, + =2
cos q sin q
1 \ sin2 q + cos2 q = 2 sin q cos q
-
1 æ sin a cos b + cos a sin b ö 2 Þ sin 2q = 1 = sin 90º
= ç ÷ Þ 2q = 90º Þ q = 45º
1
è sin b cos b ø
(sin a sin b ) 2
p p
91. (c) Given, A = and B =
1 6 3
-
1 ì sin (a + b ) ü 2
= í ý p p
1
î sin b cos b þ I. L.H.S. = sin A + sin B = sin + sin
(sin a sin b ) 2 6 3

1 3 1+ 3
-
1 = + =
1 ì sin 90º ü 2 2 2 2
= í ý
1
î cos (90º - a ) sin b þ p cos p
(sin a sin b ) 2 R.H.S. = cos A + cos B = cos +
6 3
[from equation (i)]
3 1 3 +1
= + =
2 2 2
EBD_7367
194
A- Trigonometric Ratios & Identities

Þ sin A + sin B = cos A + cos B 95. (c) Given that, sin2 x + cos2 x – 1 = 0
Þ sin2 x + cos2 x = 1
p p
II. L.H.S. = tan A + tan B = tan + tan which is an identity of trigonometric ratio and always
6 3
true for every real value of x.
1 4 Therefore, the equation have an infinite solution.
= + 3=
3 3 96. (a) I. We know that,

p p Arc
+ cot Radius = (given, arc length is constant)
R.H.S. = cot A + cot B = cot Angle
6 3
1 4 1
= 3+ = Radius µ
3 3 Angle
Þ tan A + tan B = cot A + cot B So, angular measure in radian decreases, if the
Both statements are true. radius on the arc increases.
Alternate Method: p
p p p II. 1800º ´ = 10 p
A+B= + = 180º
6 3 2 Hence, only Statement I is correct.
97. (b) I. Given that, sin x + cos x = 2
æp ö æp ö
I. sin A + sin B = sin ç - B ÷ + sin ç - A ÷ Þ (sin x + cos x)2 = 4
è2 ø è2 ø Þ (sin2 x + cos2 x) + 2 sin x cos x = 4
= cos B + cos A = cos A + cos B Þ 1 + sin 2x = 4
Þ sin 2x = 3
æp ö æp ö
II. tan A + tan B = tan ç - B ÷ + tan ç - A ÷ Þ sin 2x ¹ 3
è 2 ø è 2 ø Hence, there is no value of x in the first quadrant
= cot B + cot A = cot A + cot B that satisfies
Hence, both statements are true. sin x + cos x = 2
92. (a) Q In 24 h, Earth rotate about its own axis = 360º II. sin x – cos x = 0
360º p p
In 1 h Earth rotate about its own axis = = 15º Þ tan x = 1 = tan Þ x=
24 4 4
In 4 h Earth rotate about its own axis = 15º × 4 = 60º Also, there is only one value of x in the first
Since, in 60 min Earth rotate about its own axis = 15º quadrant that satisfies sin x – cos x = 0.
In 12 min Earth rotate about its own axis
5 Base
15º ´ 12 98. (c) Given that, cos x = =
= = 3º 13 Hypotenuse
60
\ In 4 h 12 min Earth rotate about its own axis P= h 2 - b2 = 132 - 52
= 60º + 3º = 63º
93. (a) We know that, sin2 q + cos2 q = 1 = 169 - 25 = 144 = 12
I. sin2 1º + cos2 1º = 1. It is true.
p b
II. sec2 33º – cot2 57º = cosec2 37º – tan2 53º \ tan x – cot x = -
Now, sec2 (90º – 57º) = cosec2 57º b p
and cot2 57º = cot2 (90º – 33º) = tan2 33º 12 5 144 - 25 119
\ sec2 33º – cot2 57º = cosec2 57º – tan2 33º = - = =
5 12 60 60
Thus, Statement II is incorrect.
94. (c) Here, p = a sin x + b cos x and q = a cos x – b sin x cot 30º + 1
99. (c) I. = 2 (cos 30º + 1)
On squaring both sides, cot 30º - 1
Þ p2 = a2 sin2 x + b2 cos2 x + 2ab sin x cos x ... (i)
q = a cos x – b sin x 3 +1 æ 3 ö
Þ = 2ç + 1÷
On squaring both sides, 3 -1 ç 2 ÷
è ø
and q2 = a2 cos2 x + b2 sin2 x – 2ab sin x cos x... (ii)
Now, add equation (i) and equation (ii), we get
3 +1 3 +1 æ 3 + 2ö
\ p2 + q2 = a2 (sin2 x + cos2 x) + b2 (cos2 x + sin2 x) Þ ´ = 2ç
3 -1 3 +1 ç 2 ÷÷
= a2 + b2 è ø
Trigonometric Ratios & Identities 195
A-

104. (d) q lies is in first quadrant and tan q = 3


3 +1+ 2 3 On squaring both sides,
Þ = 3+2
3 -1 \ tan2 q = 9
Add 1 both sides,
4+2 3
Þ = 3 +2 Þ 1 + tan2 q = 10
2
Þ sec2 q = 10 Þ sec q = 10
2(2 + 3)
Þ = 3 +2 1
2 Þ cos q = ... (i)
10
Þ 3+2= 3+2
1 9
Hence, it is true. Q sin2 q = 1 – cos2 q = 1 – =
II. 2 sin 45º cos 45º – tan 45º cot 45º = 0 10 10

æ 1 1 ö 1 3
Þ 2´ ç ´ ÷ - 1 ´ 1 = 0 or 2 ´ - 1´ 1 = 0 Þ sin q = ... (ii)
è 2 2 ø 2 10
Þ 1–1=0 3 1 4
Hence, both Statements I and II are true. Now, sin q + cos q = + =
10 10 10
100. (c) Given that, 3 sin x + 5 cos x = 5
Now, squaring both sides, (q lies in first quadrant)
9 sin2 x + 25 cos2 x + 30 sin x cos x = 25 105. (d) If 0º < q < 90º, then all the trigonometric ratios can be
Þ 9(1 – cos2 x) + 25(1 – sin 2 x) + 30 sin x cos x = 25 obtained when any one of the six ratios is given.
Þ 9 + 25 – {9 cos2 x + 25 sin2 x – 30 sin x cos x} = 25 Since, we use any of the following identity to get any
Þ 9 = (3 cos x – 5 sin x)2 trigonometric ratios.
Þ 3 cos x – 5 sin x = 3 sin2 q + cos2 q = 1, 1 + tan2 q = sec2 q
and 1 + cot2 q = cosec2 q
101. (b) Given that, tan q =
3
and (0º < q < 90º) (acute) 106. (d) sin A . cos A . tan A + cos A . sin A . cot A
4
sin A cos A
2 2
Q 1 + tan q = sec q = sin A . cos A . + cos A . sin A .
cos A sin A
2
æ3ö 9 25 = sin2 A + cos2 A = 1 (Q sin2 q + cos2 q = 1)
2
Þ sec q = 1 + ç ÷ = 1 + =
4
è ø 16 16 = cosec2 A – cot2 A (Q 1 + cot2 q = cosec2 q)

5 sin q 1 + cos q
107. (a) Let f(q) = +
Þ sec q = (since, q is acute) 1 + cos q sin q
4
4 q q q
\ cos q = 2 sin × cos 1 + 2 cos 2 - 1
5 2 2 + 2
=
2q q q
2 1 + 2 cos - 1 2 sin × cos
æ4ö 16 9 2 2 2
2 2
Q sin q = 1 – cos q = 1 – ç ÷ = 1 – =
5
è ø 25 25
q q
3 sin cos
\ sin q = (since q is acute) 2 + 2
5 = q q
cos sin
102. (c) Given, sec (90 – q)º sin q sec 45º 2 2
= cosec qº sin qº sec 45º
æ 2q qö
=
1
sin q
× sin q × ( 2) = 2 2 çè
sin
2
+ cos2 ÷
2ø 2
= × q q
= = 2 cosec q
2 sin × cos sin q
103. (c): Given that, p° = qc 2 2
c
æ p ö
Þ çp × ÷ =q
c
(Q 180º = pc) 3
è 180 ø 108. (b) sin q . cos q = ... (i)
4
\ (pp)c = (q180)c
\ sin4 q + cos4 q = (sin2 q + cos2 q)2 – 2sin2 q . cos2 q
\ pp = 180q
= (1)2 – 2(sin q . cos q)2
EBD_7367
196
A- Trigonometric Ratios & Identities

After we know that sec2q – tan2q = 1


2
æ 3ö 3 3 5 Similary, sec2 D – tan2 D is always equal to 1.
= 1 – 2 çç ÷÷ = 1 – 2 . =1– =
è 4 ø 16 8 8
sin A
- sin A
109. (b) In 1 min = 60 s, distance travelled by the wheel tan A - sin A cos A
116. (c) =
= 12 × Its circumference sin 3 A sin3 A
= 12 × 2pr
Multiply in Numerator and Denominator by (1 + cos A)
12 ´ 2pr 2
\ In 1 s distance travelled by the wheel = = pr (1 - cos A ) (1 + cos A )
60 5 = ´
cos A × sin A 2 (1 + cos A )
2
pr
Ram 2 (1 - cos2 A)
Q Angle = = 5 = p
Radius r 5 =
cos A × sin 2 A (1 + cos A)
Which is the required angle.
110. (b) Given that, cos A + cos2 A = 1 sin 2 A
Þ cos A = 1 – cos2 A = sin2 A =
cos A × sin 2 A (1 + cos A)
Now, 2(sin 2 A + sin4 A) = 2(sin2 A + cos2 A)
= 2 . (1) (Q sin2 q + cos2 q = 1) 1 1 sec A
=2 = × =
cos A 1 + cos A 1 + cos A
111. (c) (1 – tan A)2 + (1 + tan A)2
+ (1 – cot A)2 + (1 + cot A)2 117. (b) Let f(q) = sin q + cos q
= 1 + tan A – 2 tan A + 1 + tan2 A + 2 tan A + 1
2 Maximum and minimum value of a cos q + b sin q is
+ cot2 A – 2 cot A + 1 + cot2 A + 2 cot A
= 4 + 2 (tan2 A + cot2 A) - a 2 + b2 £ a cos q + b sin q £ a 2 + b2
= (2 + 2tan2 A) + (2 + 2cot2 A) = 2sec2A + 2cosec2 A \ - 1 + 1 £ cos q + sin q £ 1 + 1
æ 1 1 ö æ sin A + cos A ö
2 2
Þ - 2 £ cos q + sin q £ 2
= 2ç 2
+ 2 ÷
= 2ç 2 2 ÷
è cos A sin A ø è sin A × cos A ø Þ –1.414 £ cos q + sin q £ 1.414
\ f(q) = (sin q + cos q) Î [–1.414, 1.414]
2 × (1)
= = 2 sec2 A . cosec2 A 1
sin A × cos2 A
2
and let g(q) = tan q + cot q = tan q +
tan q
112. (a) cos(A + B) = cos(30º + 60º) = cos 90º = 0
113. (b) cosec(C – D + B) = cosec(120º – 150º + 60º) (Q AM ³ GM)
= cosec(180º – 150º) = cosec 30º = 2 1 1
114. (d) If ABCD is a cyclic quadrilateral, then sum of opposite tan q +
tan q æ 1 ö2
angles is 180º. Þ ³ ç tan q × ÷
2 è tan q ø
30º + 120º = 150º ¹ 180º and 60º + 150º = 210º ¹ 180º
So, Statement I is not correct. Þ (tan q + cot q) ³ 2
Statement II : So, (tan q + cot q) is always greater than 1.
sin(B – A) = cos(D – C)
Hence, Statement 1 is false and Statement II is true.
Þ sin(60º – 30º) = cos(150º – 120º)
118. (c) Given that :
1 3
Þ sin 30º = cos 30º Þ ¹ 3
2 2 sin A = (A is acute, i.e. 0 £ A < 90º)
5
So, Statement II is also not correct.
115. (c) Required angles of a quadrilateral ABCD are 30º, 60º, Then, cos A = 1 - sin 2 A
120º and 150º, respectively.
sec 2 D – tan2 D = sec2 (150º) – tan2 (150º)
2
= sec2 (90º + 60º) – tan2 (90º + 60º) æ3ö 9
= 1- ç ÷ = 1-
= cosec2 60º – cot2 60º è5ø 25
2 2
æ 2 ö æ 1 ö 4 1 3
=ç ÷ -ç ÷ = - = =1 16 4
è 3 ø è 3 ø 3 3 3 = =
25 5
Trigonometric Ratios & Identities A-197

Now, squaring both sides, we get


sin A 1 1 + sin A
\ tan A + sec A = + = 25 sin2 q + 144 cos2 q + 120 sin q cos q = 169
cos A cos A cos A
Þ 25(1 – cos2 q) + 144(1 – sin 2 q)
+ 120 sin q cos q = 169
3 8
1+ Þ 25 – 25 cos2 q + 144 – 144 sin 2 q
5 = 5 = 8
= 4 4 =2 + 120 sin q cos q = 169
4
2 2
5 5 Þ 25 cos q + 144 sin q – 120 sin q cos q = 169 – 169
Þ (5 cos q – 12 sin q)2 = 0
x2 - y2 \ 5 cos q – 12 sin q = 0
119. (b) Here, given that: sin q =
x 2 + y2 4 sin q - cos q
122. (c)
2
Q cos q = 1 – sin q
2 4 sin q + 9 cos q

2 On dividing both numerator and denominator by


æ x 2 - y2 ö cos q, we get
=1– ç 2 ÷
ç x + y2 ÷
è ø
4 sin q cos q
-
cos q cos q
(x ) - (x ) =
2 2
2
+ y2 2
- y2 4 sin q 9 cos q
+
= cos q cos q
(x )
2
2
+ y2
Now, put the value of 4 tan q

2x 2 × 2y2 4 tan q - 1 3 - 1 2 1
= = = =
4 tan q + 9 3 + 9 12 6
(x 2
+y 2 2
) 123. (c) Given, sin q – cos q = 0
sin q = cos q, then
2
4x 2 y 2 æ 2xy ö q = 45º
= =ç 2 ÷
(x )
2 ç x + y2 ÷ \ sin4 q + cos4 q = (sin 45º)4 + (cos 45º)4
2
+ y2 è ø
4 4
æ 1 ö æ 1 ö
=ç ÷ +ç ÷
2xy è 2ø è 2ø
\ cos q =
x + y2
2
1 1 1 +1 2 1
120. (d) Given that: = + = = =
4 4 4 4 2
1 + 2 sin q cos q 124. (a) Only Statement I is correct as tan q increases faster
a2 =
1 - 2 sin q cos q than sin q as q increases while Statement II is wrong
as the value of sin q + cos q is not always greater

Þ a =2
(sin q + cos q ) + 2sin q × cos q
2 2 than 1. It may also be equal to 1.

(sin q + cos q ) - 2 sin q × cos q


2 2
125. (a)
(sin q + cos q) (tan q + cot q)
sec q + cosec q

(sin q + cos q ) 2 a sin q + cos q


Þ a2 = Þ = æ sin q cos q ö
(sin q - cos q ) 2 1 sin q - cos q (sin q + cos q) ç +
è cos q sin q ÷ø
=
(applying componendo dividendo formula) 1 1
+
a + 1 (sin q + cos q ) + (sin q - cos q ) cos q sin q
Þ =
a - 1 (sin q + cos q ) - (sin q - cos q )
æ sin 2 q + cos 2 q ö
a + 1 2 sin q (sin q + cos q) ç ÷
Þ = = tan q è sin q cos q ø
a - 1 2 cos q =
sin q + cos q
121. (c) Q 5 sin q + 12 cos q = 13 sin q cos q
EBD_7367
198
A- Trigonometric Ratios & Identities

[Q sin2 q + cos2 q = 1]
cos q + 1 - sin q
=
æ 1 ö cos q + 1 - sin 2 q - sin q cos q
(sin q + cos q ) ç ÷
è sin q cos q ø cos q
=
sin q + cos q (Q 1 – sin2 q = cos2 q)
sin q cos q
cos q + 1 - sin q
=
sin q + cos q cos q + cos 2 q - sin q cos q
sin q cos q cos q
= =1
sin q + cos q
sin q cos q cos q + 1 - sin q
= =1
cos (cos q + 1 - sin q)
cos q
cos 2 (45º + q) + cos 2 (45º - q)
126. (c)
tan (60º + q) tan (30º - q) 129. (a) sin q + cos q = 3
Squaring both sides, we get
cos (90º + 2q ) + 1 cos (90º - 2q ) + 1
+
( 3)
2
2 2 (sin q + cos q)2 =
=
tan (60º + q) × tan éë90º - (60º + q)ùû Þ sin2 q + cos2 q + 2sin q cos q = 3
(Q cos 2q = 2 cos2 q – 1) Þ 1 + 2 sin q cos q = 3
3 -1 2
cos (90º + 2q) + cos (90º - 2q) Þ sin q cos q = = =1 ... (i)
+1 2 2
= 2
tan (60º + q) cot (60º + q) sin q cos q
Now, tan q + cot q = +
cos q sin q
- sin 2q + sin 2q
+1
2 sin 2 q + cos 2 q 1
= =1 = =
1 sin q cos q sin q cos q
127. (b) sin6 q + cos6 q + 3sin2 q cos2 q From equation (i),
= (sin2 q)3 + (cos2 q)3 + 3sin2 q cos2 q (sin2 q +
1
cos2 q) tan q + cot q = = 1
1
[Q (a + b)3 = a3 + b3 + 3ab(a + b)]
= (sin q + cos q)3
2 2
(Q sin 2 q + cos2 q = 1) 130. (b) tan q + sec q = m ... (i)
3 We know that :
= (1) = 1
sec2 q – tan2 q = 1
(1 + sec q - tan q) cos q Þ (sec q + tan q) (sec q – tan q) = 1
128. (a)
(1 + sec q + tan q) (1 - sin q) 1
Þ sec q – tan q = ... (ii)
m
æ 1 sin q ö Now, adding equations (i) and (ii), we get
ç1 + - ÷ cos q
cos q cos q ø
= è 1 m2 + 1
æ 1 sin q ö 2 sec q = m + =
ç1 + + ÷ (1 - sin q ) m m
è cos q cos qø
m2 + 1
Þ sec q =
(cos q + 1 - sin q 2m
= (cos q + 1 + sin q)(1 - sin q) 131. (a) cosec(75º + q) – sec(15º – q)
cos q = cosec(75º + q) – sec[90º – (75º + q)]
= cosec(75º + q) – cosec(75º + q)
cos q + 1 - sin q =0
=
cos q + 1 + sin q - sin q cos q - sin q - sin 2 q 132. (c) In DABC, if ÐC is 90º, then
cos q ÐA + ÐB = 180º – 90º = 90º
then, cos(A + B) + sin (A + B)
Trigonometric Ratios & Identities A-199

= cos 90º + sin 90º 137. (c) Q 2 cot q = 3


=0+1=1 3
Þ cot q =
3 2
133. (c) Given sin a =
2 2 cos q - sin q 2 cot q - 1
\ =
2 cos q + sin q 2 cot q + 1
æ 3ö
Þ a = 60º çQ sin 60º = 2 ÷ 3
è ø 2´ -1
2 3-1 2 1
= = = =
3 3 3+1 4 2
Now, cos b = 2 ´ +1
2 2
138. (d) sin q + cos6 q = (sin2 q)3 + (cos2 q)3
6
æ 3ö = (sin2 q + cos2 q) (sin4 q + cos4 q – sin2 q cos2 q)
Þ b = 30º çQ cos 30º = 2 ÷
è ø = (sin2 q + cos2 q)2 – 2sin2 q cos2 q – sin2 q cos2 q)
and tan g = 1 1 3 1
Þ g = 45º (Q tan 45° = 1) = (1 – 3sin2 q cos2 q) = 1 – 3 × =1– =
4 4 4
\ a + b + g = 60º + 30º + 45º = 135º
134. (a) Since, value of cos q decreases, from 0º to 90º and at 139. (d) Given that
45º, it is equal to the value of sin q. sec q + tan q = 2 ... (i)
Similarly, value of sin q increases from 0° to 90º and We know that
at 45º, it is equal to the value of cos q. sec2 q – tan2 q = 1
For 0º < q < 45º, cos q > sin q (sec q + tan q) (sec q – tan q) = 1
So, value of cos 25º – sin 25º is always positive but
less than 1. 1
135. (b) In DABC, Þ sec q – tan q = ... (ii)
2
4 Now, adding equations (i) and (ii), we get
cos A = , i.e, AB = 4 and AC = 5
5
1
2 sec q = +2
AB 4 2
sin C = =
AC 5
5
A \ sec q =
4
140. (b) cosec(75º + q) – sec(15º – q)
– tan(55º + q) + cot (35º – q)
4 5
Þ cosec(75º + q) – cosec[90º – (75º + q)]
– tan (55º + q) + tan (90º – (35º – q)]
Þ cosec(75º + q) – cosec(75º + q)
B C
– tan (55º + q) + tan (55º + q)] = 0
136. (c) Given, a and b are complementary angles.
141. (c) sin q + 2 cos q = 1
\ a = 90 – b
On squaring both sides, we get
= cos a cosec b - cos a sin b (sin q + 2 cos q)2 = 1
Þ sin2 q + 4cos2 q + 4sin q cos q = 1
cos a Þ (1 – cos2 q) + 4(1 – sin2 q) + 4sin q cos q = 1
= - cos a sin b
sin b Þ –(cos2 q + 4sin2 q) + 4sin q cos q = 1 – 5
Þ cos2 q + 4sin2 q – 4sin q cos q = 4
cos a Þ (2sin q – cos q)2 = 4 Þ 2 sin q – cos q = 2
= - cos a cos (90 - b) 142. (a) cos x + sec x = 2 ... (i)
cos (90 - b)
On squaring both sides, we get
cos2 x + sec2 x + 2 = 4
cos a
= - cos a × cos a Þ cos2 x + sec2 x = 2 ... (ii)
cos a On cubing equation (i), we get
cos3 x + sec3 x + 3(cos x + sec x) = 8
= 1 - cos 2 a = sin 2 a = sin a Þ cos3 x + sec3 x + (3 × 2) = 8
Þ cos3 x + sec3 x = 2 ... (iii)
EBD_7367
200
A- Trigonometric Ratios & Identities

Similarly, when we multiply n in power both sides,


p
cosn x + secn x = 2 148. (a) Given, 0 < q < , then 1 - 2sin q cos q
4
Alternate Method
of we put x = 90°, : = sin 2 q + cos 2 q - 2sin q + cos q
then cos 90° + sec 90° = 1 + 1 = 2
[Q sin2 q cos2 q = 1]
Similary cosechx + sechx
= cosech 90° + sech 90° = 2
(cos q - sin q)2 (cos q – sin q) is always + ve
=1+1=2
143. (d) sin 25º sin 35º sec 65º sec 55º p
[Q 0 < q < , cos q > sin q, so we take (cos q –
1 1 4
= sin 25° . sin 35° . × sin q)2]
cos 65º cos 55º
= cos q – sin q
1 1 149. (c) Take q = 45°
= sin 25° . sin 35° . × tan q + cot q = tan45° + cot45° = 1 + 1 = 2
cos (90 - 25º ) cos (90 - 35º )
Now, sin q + cos q
1 1 = sin45° + cos45°
= sin 25° . sin 35° . × =1
sin 25º sin 35º 1 1 2
= + = = 2
144. (b) tan 8q = cot 2q Þ tan 8q = tan(90 – 2q) 2 2 2
Þ 8q = 90 – 2q Þ q = 9º
sec x
\ tan 5q Þ tan 45º = 1 150. (a)
cot x + tan x
145. (a) Given, sin(A + B) = 1
Þ A + B = sin–1 1 Þ (A + B) = 90º 1
\ B = 90º – A Þ A = 90º – B cos x
cos(A – B) = cos A cos B + sin A sin B Þ cos x sin x
= cos(90 – B) cos B + sin(90 – B) sin B +
sin x cos x
= sin B cos B + cos B sin B
= 2 sin B cos B = sin 2B 1
12 Þ cos x
146. (d) Clock will make right angle at (5n + 15) × min past
11 cos 2 x + sin 2 x
n. sin x cos x
Here n = 3,
1
12 Þ (cos x ) ´ cos x ´ sin x
\ (5 × 3 + 15) × min past 3
11
Þ sin x
12
= 30 × min past 3 sin x - cos x + 1
11 151. (b)
sin x + cos x - 1
8 8
= 32 min past 3, i.e. 3 h 32 min. (sin x - cos x) + 1 (sin x + cos x ) + 1
11 11 = (sin x + cos x ) - 1 ´ (sin x + cos x ) + 1
147. (d) Let AB be the height, AC be the string and the angle
made by string with the post be q. (sin x - cos x + 1) (sin x + cos x + 1)
A =
(sin x + cos x ) 2 - 1
q sin2 x + sin x cos x + sin x - cos x sin x - cos2 x - cos x + sin x + cos x +1

h 2h sin2 x + cos2 x + 2sin x cos x -1

sin 2 x + 2sin x - cos2 x + 1


=
C 1 + 2sin x cos x - 1
B
AB h 1 p sin 2 x + 2sin x - (1 - sin 2 x ) x + 1
From figure cos q = = = = cos =
AC 2h 2 3 2sin x cos x

p sin 2 x + 2sin x - 1 + sin 2 x + 1


q= =
3 2sin x cos x
Trigonometric Ratios & Identities A- 201

158. (b) Let sin x + cos x = p ....(i)


2sin 2 x + 2sin x 2 sin x (sin x + 1)
= = sin3x + cos3x = q ....(ii)
2sin cos x 2sin cos x
On cubing Eq. (i) both sides
sin x + 1 sin3x + cos3x + 3 sin x cos x (sin x + cos x) = p3
=
cos x Put sin3x + cos3x = q from equation (ii)
152. (b) (sin x – cos2x) (1 – sin2x cos2x)
2
Þ q + 3 sin x cos x(p) = p3....(iii)
= (sin2x – cos2x) [(sin2x + cos2x)2 – sin2x cos2x] On squaring Eq. (i) both sides, we get
[Q sin2x + cos2x = 1]
sin2x + cos2x + 2 sin x cos x = p2
(sin x – cos x) [(sin x + cos4x + 2 sin2x cos2x)
2 2 4

– sin2x cos2x] p 2 –1
= (sin2x – cos2x) (sin4 x + cos4 x + sin2x cos2x) Þ sin x cos x = [Q sin2 x + cos2x =1]
2
= sin6x + sin2x cos4x + sin4 x cos2 x – cos2 x sin4x
From Eq. (iii),
– cos6x – sin2x cos4x
= sin6 x – cos6x 3( p 2 –1)
153. (d) (sin x. cos y + cos x . sin y) sin x . cos y – cos x . q+ = p3
2
sin y)
= sin (x + y). sin(x – y) Þ 2q + 3p3 – 3p = 2p3 Þ p3 –3p = –2q
= sin2x – sin2y 159. (c) tan (A + B) = 3
éQ sin A - sin B = ù
2 2
tan(A + B) = tan 60°
ê ú
êësin( A + B) sin( A - B) úû A + B = 60° ....(i)
154. (b) (1 + cot x – cosec x) (1 + tan x + sec x) Now, tan A = 1
tan A = tan 45°
æ 1 ö
= (1 + cot x - cosec x) ç1 + + sec x ÷ A = 45°
è cot x ø
Now putting the value of A in eqn (i)
(1 + cot x - cosec x) (1 + cot x + cosec x) B = 60 ° – 45° = 15°
=
cot x
1
2
(1 + cot x ) – (cosec x) 2 tan (A – B) = tan (45° – 15°) = tan 30° =
= 3
cot x 160. (*) CosA = tanB
2 2 2 Squaring on both sides
1 + cot x + 2 cot x - cosec x
=
cot x cos2A = tan2B

1 + 2 cot x - ( cosec 2 x - cot 2 x ) sin 2 B 1 - cos 2 B


= Þ tan 2 B = =
cot x cos 2 B cos 2 B

1 + 2 cot x - 1 1 - cos 2 B
= =2 \ cos 2 A =
cot x cos 2 B
155. (d) (cosec x – sin x) (sec x – cos x) (tan x + cot x)
1 - tan 2 C
æ 1 öæ 1 ö æ sin x cos x ö cos 2 A =
=ç - sin x ÷ ç - cos x ÷ ç + ÷ tan 2 C
è sin x ø è cos x ø è cos x sin x ø
(Q cos B = tan C )
(1 – sin 2 x)(1 – cos 2 x)(sin 2 x + cos2 x)
=
sin x cos x.sin x cos x sin 2C
Þ cos2A tan2C = 1– tan2C = 1 –
cos 2 x sin 2 x ´ 1 cos 2C
= =1
sin 2 x cos 2 x
sin 2 C cos 2 C – sin 2 C
156. (d) We know that, sin 1° > sin 1 and cos 1 < cos 1° Þ cos 2 A =
cos 2C cos2C
Hence, neither Statement 1 nor 2 is correct.
157. (a) Here put q = 90° Þ cos2A (1 – cos2C) = 2 cos2C –1
sin90° + cosec90° Þ cos2 A (1 – tan2A) = 2tan2A– 1
=1+1=2 2 sin 2 A
Now, sin9x + cosec9x = (sin90)9 + (cosec 90)9 cos2A – sin2A = –1
cos2 A
=1+1=2
EBD_7367
202
A- Trigonometric Ratios & Identities

162. (d) tan A + cot A = 4


2 sin 2 A – cos 2 A
Þ 1 – 2sin A = 2 Þ Squaring both sides
cos2 A tan2A + cot2A + 2 = 16
Þ cos2A (1 – 2 sin2A) = 2 sin2A – cos2A tan2A + cot2A = 14
Þ cos2A (1 – 2 sin 2A) = 2 sin2A – 1 + sin2A Again, squaring both sides
Þ (1 – sin2A) (1 – 2sin 2A) = 3sin2A – 1 tan4A + cot4A + 2 = 196
Þ 1 – 2 sin2 A – sin2A + 2 sin4A = 3 sin2A – 1 tan4A + cot4A = 194
Þ 1 – 3 sin2 A + 2 sin4 A = 3 sin2 A – 1 163. (c) Statement 1
Þ 2 sin4A – 6 sin2 A + 2 = 0 1 - sin n (1– sin x) (1 – sin x)
Þ sin4 A – 3 sin2 A + 1 = 0 1= =
1 + sin x (1– sin x) (1 + sin x)
This is quadratic equation in sin2A
(sin2A)2 – 3 (sin2A) + 1 = 0 1– sin x 1 – sin x 1 - sin x
= = =
2
1 – sin x cos x2 cos x
( -3 ) - 4 (1)(1)
2

sin A =
2
P=q
2
cos x cos x(1 – sin x) cos x(1 – sin x)
r= = =
3± 5 1 + sin x (1 + sin x)(1 – sin x ) 1– sin 2 x
=
2
cos x (1 – sin x) 1– sin x
= =
3+ 5 2 cos x
sin 2 A = not possible because sin2A 1 cos x
2
P=q=r
3- 5 Now, Statement 2 P2 = qr
So sin 2 A = .
2 1 – sin x cos x 1 – sin x
= . = = P2
So none of the options are correct. cos x 1 + sin x 1 + sin x

3 – tan 2 A So, Both are correct.


161. (c) = K 164. (c) Statement 1
1 – 3 tan 2 A
3 – tan2A = K – 3K tan2A cos A sin A
+
3K tan2A – tan2A = K– 3 1 – tan A 1 – cot A
tan2A (3K – 1) = K – 3 cos A.cos A sin A.sin A
Þ +
K –3 cos A – sin A sin A – cos A
tan2A = ...(i)
3K – 1
cos2 A – sin 2 A
1 = = cos A + sin A.
Subject to the condition K > 3 or K < . (cos A – sin A)
3
cosecA (3 sinA – 4sin 3A) = 3 – 4 sin2A Statement 2
3K – 1 (1 – sin A – cos A)2 = 2 (1 – sin A) (1 + cos A)
cot2A =
K –3 LHS = (1 – sin A – cos A)2
K – 3 + 3K – 1 4 K – 4 = 1 + sin2A + cos2A – 2 sin A + 2 sin A cosA – 2 cos A
cosec2A = = = 2 – 2 sinA 2 + cos A + 2 sin A cos A
K –3 K –3
Þ 2 {(1 – sin A) + cos A (1 – sin A)}
K –3
sin 2 A = = 2(1– sin A) (1 + cos A)
4( K –1)
So both (1) and (2) are correct.
4( K – 3)
3 – 4sin2A = 3 – 165. (c) A
4( K –1)

3K – 3 – K + 3 2K 5
= = 3
K –1 K –1

1 90°
where K > 3 or K < .
3 B 4 C
Trigonometric Ratios & Identities 203
A-

AB 3
= 3c - c3 2
Given sin 3 x + cos3 x = ...(iii)
BC 4 2
sin A + sin B + sin C On squaring both sides.

4 3 4 + 3 + 5 12 (3c – c3 )2
= +1+ = = Þ sin6x + cos6x + 2 sin3x cos3x =
5 5 5 5 4
166. (b) cosec2 67° + sec257° – cot233° – tan223° 3
ïì (c –1) ïü 9c + c – 6c
2 2 6 4
= (cosec267° – tan2 23°) + (sec2 57° – cot2 33°) 6
Þ sin x + cos x + 2 í 6
ý =
= (sec2 23° – tan2 23°) + (cosec2 33° – cot2 33°) ïî 2 ïþ 4
(Q cosec (90 – q) = secq) Þ sin6x + cos6x
=1+1
9c 2 + c6 – 6c 4 – c 6 + 1 + 3c 2 (c 2 –1
= 2. =
4
167. (b) Statement (1)
sin4x – 2 sin2x– 1 = 0 1 + 6c 2 – 3c 4
sin6x + cos6x . =
Let sin2x = t 4
Þ t2 – 2t –1 = 0 169. (b) Statement 1
1
2±2 2 =4+ 2 3
Þ t = Þ t = 1± 2 1 – sin x
2
Þ 1 = 4 + 2 3 – 4 sin x – 2 3 sin x
Þ sin2 x = 1 ± 2 i.e., 1– 2 or 1 + 2
sin2x cannot be –ve. 3+ 2 3
Þ sin x =
and sin x lies between –1 and 1 4+2 3
So, for 0 < x < p/2 there is no value that satisfies the Þ sin x = .866 < 1
equation. 0 < sin x < 1, Therefore, value of x exists between
\ (1) is not true
p
Statement (2) sin (1.5) > cos (1.5) 0 to
2
180° Statement 2
1.5 radian is 1.5 × > 90° in 2nd quadrant.
p 2
sin x = 3sin x
sin (1.5) is +ve but cos(1.5) falls in second where it For example x = 45,
is –ve. sin 45° = 3 sin2 45
So, it is always < sin (1.5). 2
So, only (2) is correct. 1 æ 1 ö 1 æ1ö
Þ = 3ç ÷ ; = 3ç ÷
168. (b) sin x + cos x = c ....(i) 2 è 2ø 2 è2ø
Squaring both sides. So this does not hold good for any values.
Þ sin2 x + cos2x + 2sin x cos x = c2 So only statement 2 is true.
170. (d) Complementary angle of 80° = 90° – 80° = 10°
c 2 –1
Þ sin x cos = ...(ii) p p
2 10° can be written as = 10 ´ Rad = rad.
180 18
Now, cubing eqn (i) both sides
Þ sin3 x + cos3x + 3sin x cos x (sin x + cos x) = c3 1 + tan 2 q sec 2 q
171. (a) (LHS) =
1 + cot 2 q cos ec 2 q
(c 2 –1)
3 3
Þ sin x + cos x + 3. ´ c = c3
2 2
æ sec q ÷ö2 æç sin q ö÷
ç
= çç ÷ = çç ÷
÷
3 2 è cos ecq ÷ø è cos q ø÷
Þ sin3x + cos3x = c3 – (c –1) c = tan2q
2
2 2
3 R.H.S. æ 1 – tan q ö = æ 1 – tan q ö
3c + 3c çè ÷ ç 1 ÷
Þ sin3x + cos3x = c3 – 1 – cot q ø
2 çè 1 – ÷
tan q ø
EBD_7367
204
A- Trigonometric Ratios & Identities

2 cosq
é æ 1 – tan q ö ù 176. (b)
ê tan q çè tan q – 1÷ø ú = (–tan q)2
ë û sinq
Þ tan2 q
LHS = RHS.
3p
p
\ Statement 1 is true when q ¹ . 2 q
9
p p p
2p x
Statement 2
4 2
(cot q) (tan q) = 1 for all values of q. Except when q = 0,
90°, 180°..... .
\ Statement 2 is not true.
Option (d) is correct.
172. (b) x = a cos q, y = b cotq From the graph it is clear that
Þ (ax–1 – by–1) (ax–1 + by–1)
p p
æ a b öæ a b ö sinq > cosq, when q£
4 2
Þ ç - ÷ç + ÷
è x y øè x y ø
p
æ a b öæ a b ö and sinq < cosq, when 0 £ 0 £
- + 4
Þ ç ÷ç ÷
è a cosq bcot q øè a cosq bcot q ø So, option (b) is correct
Þ (secq – tanq) (secq + tanq)
2a + 3b
Þ sec2q – tan 2q 177. (a) sin q =
Þ1 3b
So, option (b) is correct.
2a
cos q (1 + sin q) Þ sin q = +1
´ 3b
173. (b)
(1- sin q) (1 + sin q)
æ 2a ö
cos q (1 + sin q) as a and b is positive so ç1 + ÷ will be always greater
è 3b ø
Þ
(1- sin2 q) than 1 that is not possible for sinq.
option (a) is correct.
cos q (1 + sin q) 178. (a) tan q + sec q = 2 -----------(i)
Þ As we know
cos 2 q
Þ sec2q–tan2q = 1
(1 + sin q ) Þ (secq – tanq) (secq + tanq) = 1
Þ
cos q
1
So, option (b) is correct. Þ secq – tanq = -----------(ii)
174. (c) tan(x + 40)° tan (x + 20)° tan 3x° tan(70 – x)° tan(50 – x)° = 1 2
Þ tan (x + 40) tan (x + 20) tan 3x cot [90 –(70 – x)] equation (i) - eq (ii)-
cot [90 – (50 – x] = 1
Þ tan (x + 40) tan (x + 20) tan 3x cot (x + 20) cot(x + 40) = 1 1 3 3
2 tan q = 2 – 2 tan q = Þ tan q =
Þ tan 3x = tan 45° 2 2 4
Þ 3x = 45° So, option (a) is correct.
Þ x = 15° A D
So, option (c) is correct. 179. (d)
175. (a) sinq cosq = 2cos3q – 1.5 cosq 2q q
sinq cosq = [2 cos2q – 1.5] cosq
sinq = 2 (1 – sin2q) – 1.5 O
2 sin2q + sinq – 0.5 = 0 60°
2
-1 ± (1) + 4´ 2´ 0.5
sinq =
4
60° 60°
-1 ± 5
4 B C
as –1 £ sinq £ 1 In DOCD
CD = CO
-1 ± 5 ÐODC = ÐDOC = q
sin q =
4 q + q + 30° = 180°
So, option (a) is correct. q = 75°
Trigonometric Ratios & Identities 205
A-

tan 45° + tan 30° Multiplying (2) by tanq and add in (1) we get
tan 75° = tan (45° + 30º) =
1 - tan 45° tan 30° x y
- tan q = 1
1 a b
1+
3 = 3 + 1´ ( 3 + 1) x y
= 1 3 -1 ( 3 + 1) tan 2 q + tan q = tan q
1 -1. a b
( )
3 x 2
1 + tan q = 1 + tan q
a
2
( 3 +1) 4+2 3
= = x 1 + tan q
2 2 Þ =
a 1 + tan 2 q
= 2+ 3
So, option (d) is correct. y x
Þ = 1 - tan q
180. (a) 0 £ cos2 x £ 1 --------------(i) b a
0 £ cos y £ 1
2 --------------(ii)
0 £ cos2 z £ 1 (1 + tan q )
= 1- tan q
0 ³ –cos2 z ³ –1 1 + tan 2 q
–1 £ –cos2 z £ 0 --------------(iii)
y 1 - tan q
Adding eq (i), (ii) and (iii)- =
0 + 0 – 1 £ cos2 x + cos2 y – cos2 z £ 1 + 1 + 0 b 1 + tan 2 q
–1 £ cos2 x + cos2 y – cos2 z £ 2
so minimum value is –1. x2 y2 (1 + tan q )2 (1 - tan q )2
+ = +
Þ a2 b2
(1 + tan 2 q) (1 + tan 2 q)
So, option (a) is correct. 2 2

æpö æpö æ pö
181. (d) 32 cot 2 çç ÷÷÷ - 8sec2 ççç ÷÷÷ + 8 cos3 ççç ÷÷÷
è4ø è 3ø è6ø 1 + tan 2 q + 2 tan q + 1 + tan 2 q - 2 tan q

(1 + tan2 q)
= 2
æ 3 ö÷2
Þ 322 (1)2 – 8 (2)2 + 8 çç ÷
çè 2 ø÷

3 3
(
2 1 + tan 2 q )
Þ 32 – 32 + 8 × =
(1 + tan 2 q )
2
8
Þ 3 3
2 2
So, option (d) is correct. = = = 2cos 2 q
1 + tan q sec 2 q
2 2
æ sin 35° ö æ cos 55° ö \ Option (d) is correct.
182. (c) çè ÷ø - çè ÷ + 2sin 30° ...(1)
cos55° sin 35° ø
1 - cos q
We know that 184. (c) Statement (1) = cosec q - cot q
1 + cos q
æp ö
sin ç - q ÷ = cos q
è 2 ø 2sin 2 q / 2 1 cos q
Þ = -
Þ sin(90 – 55°) = cos55° 2cos q / 2 2 sin q sin q
Þ sin35° = cos55°
q
So from (1) we get 2sin 2
2 1 - cos q 2
tan q / 2 = =
2
æ sin 35° ö æ cos55° ö 1
2 Þ sin q q q
2sin cos
ç ÷ -ç ÷ + 2´ 2 2
è sin 35° ø è cos55° ø 2
= (1)2 – (1)2 + 1 q q
Þ tan = tan
\ Option (c) is correct. 2 2
x y Hence, (1) is identity
183. (d) - tan q = 1 ...(1)
a b
1 + cos q
(2) Statement = cos ecq + cot q
x y 1 - cos q
tan q + = 1 ...(2)
a b
EBD_7367
206
A- Trigonometric Ratios & Identities

p
q Þ q = 60° As 0 < q <
2 cos2 2
2 = 1 + cos q = 1 + cos q
q sin q sin q sin q There is only one value of q satisfying the above
2sin 2 equation.
2
Statement (1) is not correct.
q Again Put q = 60° in L.H.S. of (1)
2 cos2
2 cos 2 60° - 3cos 60° + 2
= q q
2sin cos sin 2 60°
2 2
1 3
q q - +2
2 1- 6 + 8
cot = cot 2 =
4 2 =
2 2 3 3
4
q q
Þ cot = cot
2 2 3
= = 1 = RHS
Hence, (2) is also an identity 3
\ Option (c) is correct. Statement (2) is correct.
185. (b) p = cotq + tanq q = secq – cosq \ Option (b) is correct.
187. (d) 3 – tan2 q = a (1 – 3 tan 2 q)
cos q sin q 1
= + = - cos q
sin q cos q cos q 3 - tan 2 q
Þ a=
1 - 3tan 2 q
1 sin 2 q
Þ p= q=
sin q cos q cos q 4 - 4 tan 2 q
Þ a= [By componendo and dividendo]
2 2 2 - 2 tan 2 q
\ ( p2q ) - ( q 2 p )
3 3
æ1- tan2 q ö
= 2ç ÷
2 2 è1+ tan2 q ø
æ 1 sin 2 q ö 3 æ sin 4 q 1 ö3
=ç 2 ´ ÷ -ç ´ ÷ Þ a = 2cos 2q
ç sin q.cos 2 q cos q ÷ ç cos2 q sin q cos q ÷
è ø è ø Now –1 £ cos 2q £ 1 Þ –2 £ 2 cos 2q £ 2
4
2 2
188. (c) tan q + cot q =
æ 1 æ sin q
ö3 3 ö3 3
=ç -ç ÷
3 ÷ ç cos3 q ÷
è cos q ø è ø sin q cos q 4
Þ + =
cos q sin q 3
1 sin 2 q
= - 1 4
cos 2 q cos2 q Þ =
sin q cos q 3
1 - sin 2 q cos 2 q
= = =1 3
cos 2 q cos 2 q Þ sin q cos q =
4
\ Option (b) is correct.
3
cos 2 q - 3cos q + 2 Þ 2sin q cos q =
186. (b) Given =1 ...(1) 2
sin 2 q Þ sin 2q = sin 60° Þ 2q = 60° Þ q = 30°
( 2 - cos q )(1 - cos q ) Þ sinq + cosq
Þ =1 = sin30° + cos30°
1 - cos2 q
1 3
2 - cos q = +
Þ =1 2 2
1 + cos q
Þ 2 – cosq = 1 + cosq 3 +1
=
2
1 \ Option (c) is correct.
Þ cos q = = cos 60°
2
Trigonometric Ratios & Identities 207
A-

7 13 13
189. (b) sin q + cos q = Maximum value is either 1 or since 1 >
2 16 16
When q = 0º or 90°
7
sin2q + cos2q + 2 sinq cosq = 3
4 \ £ A £1
4
7-4 3
2 sinq cosq = = cot A + cosec A – 1
4 4 195. (a)
cotA – cosec A + 1
7 æ3ö
(sin q + cosq)2 – 4 sin q cosq = - 2ç ÷
4 è4ø cot A + cosecA – (cosec 2A–cot 2A)
Þ
1 cotA – cosec A + 1
(sin q – cosq)2 = cot A + cosecA [1 - cosec A + cot A ]
4 Þ
1 cot A - cosec A + 1
sin q – cosq = Þ cot A + cosec A
2
190. (b) sinx + sin2 x = 1 cos A 1 1 + cos A
sinx = cos2 x Q 1 – sin2 x = cos2x Þ + Þ
sin A sin A sin A
now
196. (c) 1 and 3 only
cos8 x + 2 cos6 x + cos4x
sin1c = sin 57° (approx)
Þ sin 4x + 2 sin3x + sin2 x
cos 1c = cos 57°
Þ (sin2 x + sinx)2 = 1
191. (d) cosec268° + sec256° – cot234° – tan2 22° 22c
sec222° – tan2 22° + cosec2 34° – cot2 34° tan 1c = tan 57° \ 180° =
1+1=2 7
192. (c) 2y cosq = x sin q cos 0° = 1, cos 1° = 0.99
2y cosec q = x sec q cos 57° = 0.54
2y cosec q – x sec q = 0 ... (i) 1
y cosec q + 2x sec q = 3 ... (ii) 197. (c) tan 2 x + =2
Multiplying eq (ii) with 2 and adding both eq. tan 2 x
3x sec q = 6 æ 2 1 ö
x sec q = 2 ç tan x + - 2÷ = 0
2
x = 2 cos q è tan x ø
using (tan x – cot x)2 = 0
2y cos q = x sinq tan x = cot x
2y cos q = 2 cosq sinq when x = 45°
y = sinq
Putting value of x and y in x2 + 4y2 cos 75° sin12° cos18°
198. (d) 1. + - =1
(2 cosq)2 + 4 sin2q sin15° cos 78° sin 72°
4 cos2q + 4 sin2q = 4 cos 75 ° sin 12 ° cos18 °
+ - =1
1+ 3 1 3 cos 75 ° sin 12 ° cos18 °
193. (d) sinq + cos q = = + i.e. q = 30°
2 2 2 2–1=1
Statement is correct
1 4
than tan30 + cot 30 = + 3= cos 35° sin11°
3 3 2. - + cos 28° cosec 62° = 1
194. (b) A = sin2 q + cos4 q sin 55° cos 79°
= 1 – cos2 q + cos4 q cos 35° sin11°
2 2 - + cos28° sec 28° = 1
æ1ö 2 æ1ö æ1ö cos 35° sin11°
cos4q – 2 ç ÷ cos q + 1 - ç ÷ + ç ÷ 1–1+1=1
2
è ø è2ø è2ø
2nd statement is also true.
2
æ 2 1ö 3 sin 80°
ç cos q - ÷ + 3. – sin59° sec 31° = 0
è 2ø 4 cos10°
2
æ 2 1ö sin 80°
When ç cos q - ÷ = 0 – sin 59° cosec 59° = 0
è 2ø sin 80°
1–1=0
3 3rd statement is correct.
value is minimum that is
4 199. (b) tan1° tan2° tan3° tan4° ....... tan 89°
3 tan1° tan2° ..... tan 45° ...... tan89°
Now from equation b and d are with minimum value , =1
4
EBD_7367
208
A- Trigonometric Ratios & Identities
200. (c) 9 tan 2 q + 4 cot2q 4
Þ (3 tanq)2 + (2 cot q)2 – 2 (3 tan q) (2) cot q + 4 sin 2x =
2 (3 tan q)2 cot q 3
Þ (3 tan q – 2 cotq)2 + 12 1
since (3 tan q – 2 cot q)2 ³ 0 sin x =
Minimum value is 12 3

201. (b) x sin q = y cos q =


2z + tan q
1 - tan 2 q
Let q = 30°
13units
x y 3 z 2 tan q
= = =KQ = tan 2 q = tan 60°
2 2 3 1 - tan 2 q 12units
2K 207. (d)
x = 2K, y = , z= 3 k
3
putting value of x, y and z in 4z2 (x2 + y2) q
5units
æ K ö é
2 2ù
ê 2 æ 2K ö ú Since we know that 5, 13 and 12 forms a Pythagorean
4ç ÷ (2K) + ç ÷
è 3 ø êë è 3 ø úû triplet, the side with 13 units is the longest side and the
angle between the other two sides is 90°.
4K 2 é 2 4K 2 ù P 12 B 5
Þ ê 4K + ú Therefore, sin q = = and cos q = =
3 êë 3 úû H 13 H 13
12 5 12 + 5 17
64K 2 Thus, sin q + cos q =+ = =
Þ 13 13 13 13
9
From option putting value of x, y and z p
208. (c) Since we are given that 0 < x < , then
we get (x2 – y2)2 2
2
æ 2 (2K)
2ö é 4K 2 ù p
= çç (2K) - ÷ = ê 4K 2 - ú sin 0 < x < sin Þ 0 < sin x < 1 ...(1)
è (3) ÷ø êë 3 ûú 2
Similarly,
2 p
æ 8K 2 ö 64K 2 cosec0 > cosec x > cosec Þ ¥ > cosec x > 1 ...(2)
=ç ÷ = 2
ç 3 ÷ 9
è ø Adding (1) and (2), we get
202. (a) When q1 = q2 = q3 = 0 then cosq1 + cosq2 + cosq3 = 3 ¥ > sin x + cosec x > 2
then sin q2 + sin q2 + sinq3 = 0 Thus, we can say that the value of the sum of the two
203. (a) From given option q = 0° Satisfies the equation trigonometric functions would be ³ 2.
cos0° + tan0° = 1 209. (a) Suppose we have right angled triangle with sides a, b
1+0=1 and c where c is the longest side.
1 1
204. (a) sin x +
1 + cos x 1 - cos x c
1 + cos x + 1 - cos x a
Þ sin x
1 - cos2 x
sin x q
Þ 2 Þ 2
sin x b
4 4 a b
cos A - sin A Now, we can see that sin q = and cosq = . Here,
205. (c) c c
cos 2 A - sin 2 A we can see that in both the denominators we have the
same hypotenuse which means from all the given op-
(cos 2 A)2 - (sin 2 A) 2
Þ tions, only option (a) has the same hypotenuse as given
cos 2 A - sin 2 A in the question i.e. m2 + n2.
Thus, option (a) is the correct answer.
(cos 2 A + sin 2 A)(cos 2 A - sin 2 A)
=1 210. (a) 1 1 2- 2
cos2 A - sin 2 A -
sin 45° - sin 30° 2
206. (d) 7sin2x + 3 cos2 x = 4 A= = 2 = 2 2
7 sin 2 x + 3 – 3 sin 2x = 4 cos 45° + cos 60° 1 1 2+ 2
+
2 2 2 2
Trigonometric Ratios & Identities A- 209

2 - 2 2 - 2 (2 - 2) 2 4 + 2 - 4 2
212. (a) sin1° 0.0174 0.0174
= ´ = = c
= = = 0.0206 < 1
2+ 2 2 - 2 4-2 2 sin1 æ 180 ö 0.8415
sin ç
è p ÷ø
6-4 2 4
213. (d) sin q – cos q4
= = 3- 2 2
2 = (sin 2 q) 2 – (cos 2 q) 2 = (sin 2 q + cos 2 q)
(sin2q – cos2q)
sec 45° - tan 45° 2 -1 2 -1 1(sin2q – cos2q) = 1 – cos2q – cos2q = 1 – 2 cos2q
B= = ´
cosec 45° + cot 45° 2 +1 2 -1 214. (b) cot 1° cot 23° cot 45° cot 67° cot 89°
= cot 1° × cot 23° × cot 45° × cot (90 – 23)° ×
( 2 - 1) 2 cot (90 – 1)°
= = 2 +1- 2 2 = 3 - 2 2
2 -1 = cot 1° × cot 23° × cot 45° × tan 23° × tan 1°
Hence, A = B cot 1° ×tan 1° × cot 23° × tan23° × cat 45°
211. (c) In the first statement, we are given that 45° < q < 60°. =1×1×1=1
Therefore, if we consider q < 60°, then let us suppose q 215. (b) Checking statement 1
= 45°, so (sec2q – 1) (1 – cosec2q) = 1
L.H.S
sec 2 q + cosec 2 q = sec 2 45° + cosec 2 45° (sec2q – 1) × (–1) (cosec2q – 1)
= 2 + 2 = 4 = a2 Þ a = 2 > 1 and if we consider q > 45°, tan2q × (–1) × cot2q = –1
then let us suppose q = 60°, so Here L.H.S ¹ R.H.S
Hence, statement 1 is incorrect
sec 2 q + cosec 2 q = sec 2 60° + cosec 2 60° checkin statement 2.
4 16 Sinq (1 + cosq)–1 + (1 + cosq) (sin q)–1 = 2 cosecq
= 4+ = = a 2 Þ a = 2.31 > 1 L.H.S
3 3
Hence, statement 1 is true. 1 + cos q sin q + (1 + cos q )
2 2
sin q
In the second statement, we are given that 0° < q < 45°. + =
Therefore, if we consider q < 45°, then let us suppose q 1 + cos q sin q (1 + cos q) sin q
= 0°, so
sin 2 q + 1 + cos 2 q + 2 cos q
1 + cos q 1 + cos 0° =
= = ¥ = x2 Þ x = ¥ > 2 (1 + cos q) sin q
1 - cos q 1 - cos 0°
and if we consider q > 0°, then let us suppose q = 45°, 1 + 1 + 2 cos q 2 (1 + cos q ) 2
= = =
so sin q (1 + cos q ) sin q (1 + cos q ) sin q
1 + cos q 1 + cos 45° =2cosecθ=R.H.S
= Hence, only statement 2 is correct.
1 - cos q 1 - cos 45°
216. (a) sec x cosec x = 2
2 +1 2 +1 ( 2 + 1) 2 This value is possible is x = 45°
= ´ = tannx + cotnx = (tan x)n + (cot x)n
2 -1 2 +1 2 -1
= (tan 45°)n + (cot 45°)n = (1)n + (1)n = 2
2 +1+ 2 2 217. (a) cos x + cos2x = 1
= = 3 + 2 2 = x 2 Þ x = 2.414 > 2 cos x = 1 – cos2x = sin2x
1
sin 2x + sin4x = sin2x + (sin2x)2
Hence, statement 2 is true. = sin2x + (cos x)2 = sin 2x + cos2x = 1
In the third statement, we are given that 0° < q < 45°. 218. (c) sin A + cos A = p sin3A + cos3A = q
Therefore, if we consider q < 45°, then let us suppose q sin A + cos A = (sin A + cos A) (sin 2A + cos2A – sin
3 3
= 0°, so A cos A)
cos q sin q cos 0° sin 0° q = p (1 – sin A cos A)
+ = +
1 - tan q 1 - cot q 1 - tan 0° 1 - cot 0° q
1 – sin A cos A =
1 0 p
= + = 1+ 0 = 1
1 - 0 -¥ q p-q
which is not ³ 2 and if we consider q > 0°, then let us sin A cos A = 1 – =
p p
suppose q = 45°, so (sin A + cos A)3 = p3
cos q sin q cos 45° sin 45° sin3A + cos3A + 3 sin A cos A (sin A + cos A) = p3
+ = +
1 - tan q 1 - cot q 1 - tan 45° 1 - cot 45° p-q
q+3× ´ p = p3
1 1 p
2 p3 = q + 3p – 3q
= + 2 =¥³2. p3 + 2q – 3p = 0
1 -1 1 -1
219. (d)
Hence, statement 3 is false. 220. (a)
Hence, only two statements are correct.
EBD_7367
210
A- Height and Distance

C HA P T E R
HEIGHT AND DISTANCE
18
1. Two houses are collinear with the base of a tower and are at (c) 46 m (d) 50 m
distance 3 m and 12 m from the base of the tower. The 9. A radio transmitter antenna of height 100 m stands at the
angles of elevation from these two houses of the top of the top of a tall building. At a point on the ground, the angle of
tower are complementary. What is the height of the tower? elevation of bottom of the antenna is 45º and that of top of
[2007-I] antenna is 60º. What is the height of the building?
(a) 4 m (b) 6 m [2009-I]
(c) 7.5 m (d) 36 m (a) 100 m (b) 50 m
2. The angle of elevation from the bank of a river of the top of
a tree standing on the opposite bank is 60º. The angle of (c) 50( 3 1) m (d) 50( 3 1) m
elevation becomes 30º when observed from a point 40 m 10. The angle of elevation of the top of an unfinished pillar at
backwards in a direction perpendicular to the length of the a point 150 m from its base is 30º. If the angle of elevation at
river. What is the width of the river? [2007-I] the same point is to be 45º, then the pillar has to be raised to
(a) 10 m (b) 20 m a height of how many metres? [2009-I]
(c) 30 m (d) 40 m
(a) 59.4 m (b) 61.4 m
3. Person standing at the end of the shadow of a pole measures
(c) 62.4 m (d) 63.4 m
1 11. A ladder 25 m long is leaning against a wall which is
that the length of the shadow is times the length of the
3 perpendicular to the level ground. The bottom of the
pole. At what angle of elevation will the man see the Sun? ladder is 7 m from the base of the wall. If the top of the
[2007-II] ladder slips down 4 m, how much will the bottom of the
(a) 60º (b) 30º ladder slip? [2009-I]
(c) 45º (d) 15º (a) 7 m (b) 8 m
4. The angle of depression of vertex of a regular hexagon (c) 10 m (d) 15 m
lying in a horizontal plane, from the top of tower of height 12. The length of the shadow of a person s cm tall when the
75 m located at the centre of the regular hexagon is 60º. angle of elevation of the Sun is is p cm. It is q cm when
What is the length of each side of the hexagon? [2007-II] the angle of elevation of the Sun is . Which one of the
(a) 50 3 m (b) 75 m following is correct when = 3 ? [2009-II]
(c) 25 3 m (d) 25 m tan tan 3
(a) p – q = s
5. A vertical stick 10 cm long casts a shadow 8 cm long. At tan 3 tan
the same time a tower casts a shadow 30 m long. What
is the height of the tower? [2008-I] tan 3 tan
(a) 37.5 m (b) 36 m (b) p – q = s
3 tan 3 tan
(c) 32.5 m (d) 32 m
6. A round balloon of unit radius subtends an angle of 90º at tan 3 tan
(c) p – q = s
the eye of an observer standing at a point, say A. What is tan 3 tan
the distance of the centre of the balloon from the point A?
[2008-II] tan 2
(d) p – q = s
(a) 1/ 2 (b) 2 tan 3 tan
(c) 2 (d) 1/2 13. A man is watching from the top of a tower a boat speeding
7. The angle of elevation and angle of depression both are away from the tower. The boat makes an angle of depression
measured with [2008-II] of 45º with the man’s eye when at a distance of 60 m from
(a) the vertical only the bottom of tower. After 5 s, the angle of depression
(b) the horizontal line only becomes 30º. What is the approximate speed of the boat
(c) both horizontal and vertical assuming that it is running in still water? [2010-I]
(d) None of the above (a) 31.5 km/h (b) 36.5 km/h
8. A ladder 34 m long is placed in a lane so as to reach window (c) 38.5 km/h (d) 40.5 km/h
30 m high and on turning the ladder to the other side of the 14. Suppose the angle of elevation of the top of a tree at a
lane and keeping is foot at the same place, reaches a point point E due East of the tree is 60º and that at a point F due
16 m height. What is the breadth of the lane? [2008-II] West of the tree is 30º. If the distance between the points E
(a) 18 m (b) 40 m and F is 160 ft, then what is the height of the tree?
Height and Distance A-211
[2010-I] 23. The length of shadow of a tree is 16 m when the angle of
elevation of the Sun is 60º. What is the height of the tree?
(a) 40 3 ft (b) 60 ft
[2011-II]
40 (a) 8 m (b) 16 m
(c) ft (d) 23 ft
3 16
15. A ladder of 17 ft length reaches a window which is 15 ft (c) 16 3 m (d) m
3
above the ground on one side of the street. Keeping its
24. From a lighthouse the angles of depression of two ships
foot at the same point the ladder is turned to the other
on opposite sides of the lighthouse are observed to 30º
side of the street and now it reaches a window 8 ft high.
and 45º. If the height of lighthouse is h, what is the distance
What is the width of the street? [2010-I]
between the ships? [2011-II]
(a) 23 ft (b) 15 ft
(c) 25 ft (d) 30 ft (a) ( 3 1)h (b) ( 3 1)h
16. The angle of elevation of the top of a tower from the bottom
of a building is twice that from its top. What is the height 1
(c) 3h (d) 1 h
of the building, if the height of the tower is 75 m and the 3
angle to elevation of the top of the tower from the bottom 25. From the top of a cliff 200 m high, the angles of depression
of the building is 60º? [2010-II] of the top and bottom of a tower are observed to be 30º and
(a) 25 m (b) 37.5 m 45º, respectively. What is the height of the tower?
(c) 50 m (d) 60 m [2012-I]
17. The shadow of a tower is 15 m when the Sun’s altitude is
30º. What is the length of the shadow when the Sun’s (a) 400 m (b) 400 3 m
altitude is 60º? [2010-II] (c) 400 / 3 m (d) None of these
(a) 3 m (b) 4 m 26. The angle of elevation of the tip of a tower from a point on
(c) 5 m (d) 6 m the ground is 45º. Moving 21 m directly towards the base
18. Two poles of heights 6 m and 11 m stand vertically of the tower, the angle of elevation changes to 60º. What is
upright on a plane ground. If the distance between their the height of the tower, to the nearest meter? [2012-I]
feet is 12 m, what is the distance between their tops? (a) 48 m (b) 49 m
[2010-II] (c) 50 m (d) 51 m
(a) 11 m (b) 12 m 27. What is the angle of elevation of the Sum when the shadow
(c) 13 m (d) 14 m
19. The angle of elevation of the top of a tower at a point on of a pole is 3 times the length of the pole? [2012-I]
level ground is 45º. When moved 20 m towards the tower, (a) 30º (b) 45º
the angle of elevation becomes 60º. What is the height of (c) 60º (d) None of these
the tower? [2011-I] 28. On walking 120 m towards a chimney in a horizontal line
through its base the angle of elevation of tip of the chimney
(a) 10( 3 1) m (b) 10( 3 1) m changes from 30º to 45º. The height of the chimney is.
(c) 10(3 3) m (d) 10(3 3) m [2012-II]
20. A telegraph post gets broken at a point against a storm (a) 120 m (b) 60( 3 1) m
and its top touches the ground at a distance 20 m from the
base of the post making an angle 30º with the ground. (c) 60( 3 1) m (d) None of these
What is the height of the post? [2011-I] 29. A ladder 20 m long is placed against a wall, so that the foot
of the ladder is 10 m from the wall. The angle of inclination
40 of the ladder to the horizontal will be [2012-II]
(a) m (b) 20 3 m
3 (a) 30º (b) 45º
(c) 60º (d) 75º
(c) 40 3 m (d) 30 m
30. The angles of elevation of the top of a tower from two
21. The angles of elevation of the top of a tower from two points which are at distances of 10 m and 5 m from the base
points situated at distances 36 m and 64 m from its base of the tower and in the same straight line with it are
and in the same straight line with it are complementary. complementary. The height of the tower is. [2012-II]
What is the height of the tower? [2011-II] (a) 5 m (b) 15 m
(a) 50 m (b) 48 m
(c) 25 m (d) 24 m (c) 50 m (d) 75 m
22. The angle of elevation of the top of an incomplete vertical 31. The angles of elevation of the top of an inaccessible tower
pillar at a horizontal distance of 100 m from its base is 45º. from two points on the same straight line from the base of
If the angle of elevation of the top of complete pillar at the the tower are 30º and 60º, respectively. If the points are
same point is to be 60º, then the height of the incomplete separated at a distance of 100 m, then the height of the
pillar is to be increased by [2011-II] tower is close to [2012-II]
(a) 86.6 m (b) 84.6 m
(a) 50 2 m (b) 100 m
(c) 82.6 m (d) 80.6 m
(c) 100( 3 1) m (d) 100( 3 1) m 32. Two poles of heights 6m and 11m stand on a plane ground.
If the distance between their feet is 12 m, what is the
EBD_7367
212
A- Height and Distance

distance between their tops? [2012-II] 41. The angle of elevation of the top of a tower 30 m high from
(a) 13 m (b) 17 m the foot of another tower in the same plane is 60º and the
(c) 18 m (d) 23 m angle of elevation of the top of the second tower from the
DIRECTIONS (Q. Nos. 33-36) : Read the following information foot of the first tower is 30º. The distance between the two
carefully to answer the questions that follow. towers in m times the height of the shorter tower. What is
As seen from the top and bottom of a building of height h m, the m equal to? [2014-I]
3 3 h (a) 2 (b) 3
angles of elevation of the top of a tower of height m
2 1 1
(c) (d)
are and , respectively. 2 3
33. If = 30º, then what is the value of tan ? [2013-I] 42. From a certain point on a straight road, a person observe
(a) 1/2 (b) 1/3 a tower in the West direction at a distance of 200 m. He
(c) 1/4 (d) None of these walks some distance along the road and finds that the
34. If = 30º, then what is the value of tan ? [2013-I] same tower is 300 m South of him. What is the shortest
(a) 1 (b) 1/2
distance of the tower from the road ? [2014-II]
(c) 1/3 (d) None of these
35. If = 30º and h = 30 m, then what is the distance between 300 500
(a) m (b) m
the base of the building and the base of the tower? 13 13
[2013-I]
600 900
(a) 15 15 3 m (b) 30 15 3 m (c) m (d) m
13 13
(c) 45 15 3 m (d) None of these
43. The angles of elevation of the top of a tower from two
36. If = 30º and if is the angle of depression of the foot of points P and Q at distances m2 and n2 respectively, from
the tower as seen from the top of the building, then what is
the base and in the same straight line with it are
the value of tan ? [2013-I]
complementary. The height of the tower is [2015-I]
3 3 3 3 (a) (mn)1/2 (b) mn1/2
(a) (b) (c) m1/2n (d) mn
3 3 3 3 44. The angle of elevation of a cloud from a point 200 m above
a lake is 30° and the angle of depression of its reflection
2 3 in the lake is 60°. The height of the cloud is [2015-I]
(c) (d) None of these
3 3 (a) 200 m (b) 300 m
37. A spherical balloon of radius r subtends angle 60º at the (c) 400 m (d) 600 m
eye of an observer. If the angle of elevation of its centre is 45. From the top of a tower, the angles of depression of two
60º and h is the height of the centre of the balloon, then objects P and Q (situated on the ground on the same side of
which one of the following is correct? [2013-II] the tower) separated at a distance of 100(3 – 3)m are 45°
(a) h = r (b) h 2r and 60° respectively. The height of the tower is [2015-I]
(c) h (d) h = 2r (a) 200 m (b) 250 m
3r (c) 300 m (d) None of the above
38. What is the angle of elevation of the Sun, when the shadow 46. An aeroplane flying at a height of 3000 m passes vertically
x above another aeroplane at an instant when the angles of
of a pole of height x m is m? [2013-II] elevation of the two planes from some point on the ground
3
are 60° and 45° respectively. Then the vertical distance
(a) 30º (b) 45º between the two planes is [2015-II]
(c) 60º (d) 75º
39. The heights of two trees are x and y, where x > y. The (a) 1000( 3 1)m (b) 1000 3m
tops of the trees are at a distance z apart. If x is the
(c) 1000(3 3)m (d) 3000 3m
shortest distance between the trees, then what is s2 equal
to? [2013-II] 47. A pole is standing erect on the ground which is horizontal.
(a) x2 + y2 – z2 – 2xy The tip of the pole is tied tight with a rope of length 12 m
(b) x2 + y2 – z2 to a point on the ground. If the rope is making 30° with the
(c) x2 + y2 + z2 – 2xy horizontal, then the height of the pole is [2015-II]
(d) z2 – x2 – y2 + 2xy
40. The shadow of a tower standing on a level plane is found (a) 2 3m (b) 3 2m
to be 50 m longer when the Sun’s elevation is 30º, then (c) 3 m (d) 3m
when it is 60º. What is the height of the tower? [2014-I]
48. Two observers are stationed due north of a tower (of height
(a) 25m (b) 25 3 m x metre) at a distance y metre from each other. The angles of
elevation of the tower observed by them are 30° and 45°
25 respectively. Then x/y is equal to [2016-I]
(c) m (d) 30 m
3
Height and Distance A-213

2 1 3 1 tan tan tan tan


(a) (b) (a) (b)
2 2 tan tan tan tan

3 1 cot cot cot cot


(c) (d) 1 (c) (d)
2 cot cot cot cot
49. A man from the top of a 100 m high tower sees a car moving 56. If angle A of triangle ABC is 30° and the circum-radius of
towards the tower at an angle of depression 30°. After some the triangle is 10 cm, then what is the length of side BC?
time, the angle of depression becomes 60°. What is the [2017-II]
distance travelled by the car during this time ? [2016-II] (a) 5 cm (b) 10 cm
(c) 5 3 cm (d) 10 3 cm
200 3
(a) 100 3m (b) m 57. Let AB represent a building of height h metre with A being
3 its top, B being its bottom. Let A 'B' represent a tower of
100 3 height (h + x) metre (x > 0) with A’ being its top and B' being
(c) m (d) 200 3m its bottom. Let BB' = d metre. Let the angle of elevation of
3 A’ as seen from A be 45°.
50. Two men on either side of a tower 75 m high observe the Consider the following statements :
angle of elevation of the top of the tower to be 30° and 60°. Statement I : h + x > d
What is the distance between the two men ? [2016-II] Statement II : The angle of depression of B as seen from A’
(a) 100 3m (b) 75 3m is less than 45°.
Which one of the following is correct in respect of the above
100 3 statements? [2017-II]
(c) m (d) 60 3m (a) Both Statement I and Statement II are true and Statement
3
II is the correct explanation of Statement I
51. If the length of the shadow of a tower is equal to its height,
(b) Both Statement I and Statement II ar e tr ue
then what is the Sun's altitude at that time ? [2016-II]
but Statement II is not the correct explanation of
(a) 15° (b) 30°
Statement I
(c) 45° (d) 60°
(c) Statement I is true but Statement II is false
52. A pole stands vertically inside a triangular park ABC. If the
(d) Statement I is false but Statement II is true
angle of elevation of the top of the pole from each corner of
58. A man, standing at a point X on the bank XY of a river that
the park is same, then in the triangle ABC, the foot of the cannot be crossed, observes a tower to be N ° E on the
pole is at the [2016-II] opposite parallel bank. He then walks 200 m along the bank
(a) Centroid (b) Circumference to the point Y towards East, and finds the tower to be N °
(c) Incentre (d) Orthocentre W. From these observations, the breadth of the river will be
(Given that tan ° = 2 and tan ° = 0.5) [2017-II]
53. An aeroplane flying at a height of 300 m above the ground (a) 60 m (b) 70 m
passes vertically above another plane at an instant when (c) 80 m (d) 90 m
the angles of elevation of the two planes from the same 59. Each side of a square subtends an angle of 60° at the tip of
point on the ground are 60° and 45° respectively. What is a tower of height h metres standing at the centre of the
the height of the lower plane from the ground ? [2017-I] square. If l is the length of each side of the square, then
100 what is h2 equal to? [2018-I]
(a) 100 3 m (b) m
3 l2
(a) 2l2 (b)
(c) 50 3m (d) 150( 3 1)m 2
54. From the top of a building 90 m high, the angles of depression 3l 2 2l 2
of the top and the bottom of a tree are 30° and 45° (c) (d)
2 3
respectively. What is the height of the tree ? [2017-I]
60. From a height of h units, a man observes the angle of
(a) 30 3m (b) 90 – 30 3m elevation as and angle of depression as of the top and
(c) 90 + 30 3m (d) 60 + 30 3m the bottom respectively of a tower of height H
(> 4h). To what further height should he climb so that the
55. From an aeroplane vertically over a straight horizontal road, values of angle of elevation and angle of depression get
the angles of depression of two consecutive kilometre- interchanged for the top and bottom of the tower?
stones on the opposite sides of the aeroplane are observed [2018-I]
to be and . The height of the aeroplane above the road is (a) H – h units (b) H – 2h units
[2017-I] (c) H – 3h units (d) H – 4h units
EBD_7367
A- 214 Height and Distance

HINTS & SOLUTIONS


1. (b) Let the height of the tower be h m and
h
CBD = then DAC = 90º – 3. (a) Let the height of pole be h then its shadow = and
(Because both angles are complementary) 3
be the angle of elevation.
D In ABC
C

h
h

90º –
A B 3m C A B
12 m h/ 3
In BCD, h
CD h tan = tan
tan = tan h/ 3
BC 3
Now, in ACD = 3 tan 60º
= 60º
CD h 4. (c) Let OP be the height of the tower
tan (90º – ) = cot
AC 12 x = Distance between O and F.
h P
tan =
12 60º
h tan = 12 h = 75 m
put the value of tan E D
h
h× = 12 60º O
3 F C
x
h2 = 36 h=6
Then, height of tower = 6 m. 60º
2. (b) Let the height of the tower be h m and BC = x m A B
A
75 75
In FOP, tan 60º = x 25 3 m
x 3
In regular hexagon OEF, OED ...etc, are equilateral
h triangles.
OF = AF = AB = BC = CD = DE = EF = 25 3 m =
Side of hexagon
30º 60º
D 40 m C x B Length of hexagon = 25 3 m
In ABC 5. (a)
h A P
tan 60º = h 3x
x
Now in ADB,
h
tan 30º = 10 cm x
40 x
we put the volue of h = 3x
40 x C B R Q
8 cm 30 m
h=
3 AB = Stick length PQ = Tower height
40 x BC = Shadow length QR = Shadow length
3x 2x = 40 x = 20m. Let height of a tower be x m
3
Height and Distance A-215
By given condition, In ABC,
Height of stick Height of tower h
tan 45º = x h ...(i)
Length of shadow of stick = Length of shadow of tower x
Now, in ABD,
10 x
= 100 h
8 30 tan 60º =
x
10 30
x= = 37.5 m 100 h
8 3= [from Eq. (i)]
Hence, length of the tower be 37.5 m. h
6. (b) Let O = Centre of the balloon 3 1 h 100
3h 100 h
OB = OC = Radii of the balloon
Eyes 100 100 ( 3 1)
A h h
3 1 3 1 3 1
C 90º
45º
= 50( 3 1) m
1
10. (d) Let BC = x m height of unfinished pillar
O and CD = h m = Raised height of pillar
1 D

B h

OB 1 1 C
In OBA, sin 45º = OA 2
OA 2 OA
45º x
7. (b) The angle of elevation and angle of depression are A 30º B
measured with the horizontal line only. 150 m
D In ABC,
8. (c) E x 150
tan 30º = x
34 150 3
m

30 m

m
34

and in ABD,
16 m

h x h x
tan 45º = 1
150 150
A B C
150
In BCD, 150 = h + [from Eq. (i)]
3
BC = (34)2 (30)2
150( 3 1) (1.732 1)
= 1156 900 = = h 150
256 = 16 m 3 1.732
and In BAE, AB = 2 2
(34) (16) 150 0.732
=
= 1156 256 = 30 m 1.732
Breadth of the lane = AB + BC = 63.39 = 63.4 m (approx)
= 30 + 16 = 46 m 11. (b) A
9. (c) Let BC be a building of height h m and CD = 100 m be
a height of antenna.
x = Distance between A and B
m

D
25

D
100 m

C
E B
C 7m
60º h
In ABC,
A 45º B AC2 = AB2 + BC2
x
EBD_7367
216
A- Height and Distance

(25)2 = (AB)2 + (7)2 60 + x = 60 3


(AB)2 = 625 – 49 AB = 24 m
In EBD, x = 60( 3 1) 60(1.73 1)
ED2 = (EB)2 + (BD)2 = 60 × 0.73 = 43.8 m
(25)2 = (EB)2 + (20)2
625 – 400 = (EB)2 EB = 15 m 43.8 18 788.4
Speed of boat =
EC = 15 – 7 = 8 m 5 5 25
12. (c) In BCA, = 31.5 km/h
s s 14. (a) Let AC = h = Height of a tower
tan = p ...(i) x = Distance between A and F
p tan
AE = 160 – x
In CAF,
B
h
tan 30º =
x

1 h
= x 3h ...(i)
s 3 x

C N

q h W E
A D C
p
30º 60º
F x A (160 – x) E
In BCD, S
160 ft
s s
tan = q ( = 3 , given) ...(ii) And in CAE,
q tan 3
On subtracting Eq. (ii) from Eq. (i), we get h
tan 60º =
160 x
s s
p–q=
tan tan 3 3(160 x) = h

tan 3 tan 3(160 3h) = h [from Eq. (i)]


= s
tan 3 tan
160 3 3h = h
13. (a) Let AB = x m
CD = x m 4h = 160 3
h = 40 3 ft
B CD = x m
30º 45º 15. (a) E D

x 17 ft 17 ft
8 ft 15 ft

A C
45º 30º B
A 60 m C x D
In ABE,
In ACB, BE2 = AE2 + AB2 AB2 = 172 – 82
AB AB or AB2 = 289 – 64 = 225 AB = 225,
tan 45º = 1
AC 60 AB = 15 ft
AB = 60 m In BCD,
Now, in ADB, BD2 = BC2 + CD2 BC2 = 172 – 152
60 1 60 = 289 – 225 = 64
tan 30º = BC = 8 ft
60 x 3 60 x
Width of the street = AB + BC = 15 + 8 = 23 ft
Height and Distance 217
A-

16. (c) Suppose height of building (AB) = h m Since, AE is distance between top point of AB and CE.
BC = x m
E
E
5 cm

11 cm
A D
A 30º
D75 m

6m
h

60º B C
B x C 12 m

In ADE, In ADE,
AE2 = AD2 + ED2 = 122 + 52
ED 1 75 h = 144 + 25 = 169
tan 30º =
AD 3 x AE = 13 m
Hence, the distance between their tops = 13 m
x = 75 3 h 3 ...(i) 19. (d) Let the height of the tower be h m and BD = xm.
Now, in BCE,
A
CE
tan 60º =
BC
75 h
3 = x 3 75 [from Eq. (ii)]
x
45º 60º B
(75 3 h 3) 3 = 75 75 × 3 – 3h = 75 C x
20 m D
3h = 75 × 3 – 75
In ACB,
75 2
h= h
3 tan 45º = h 20 x
20 x
h = 50 m Now, in ABD,
17. (c) In ACD,
h h
CD 1 h 15 tan 60º = x
tan 30º = h x 3
AC 3 15 3
Put the value of h = 20 + x
D h h
h = 20 h 20
3 3
1 3 1
h 1 = 20 h 20
h 3 3

20 3 3 1
h=
30º 60º 3 1 3 1
A B x C
15 m 20 3( 3 1)
= 10(3 3) m
Now, in BCD, 2
20. (b) Let the height of the post be h m.
CD h
tan 60º = 3
BC x
B
15 (h – x) m
put the value of h =
3
x
15
3 = x 5 m.
3x 30º
A C
18. (c) AD = BC = 12 m
and ED= 11 – 6 = 5 m 20 m
EBD_7367
218
A- Height and Distance

In ABC, 22. (c) Here, AB = 100 m


AC = 20 m BC = x m
AB = x m PC = h m
AB x P
tan 30º =
AC 20 h
1 x 20 C
= x m
3 20 3
20 x
cos 30º =
h x
A 45º 60º B
3 20 40 100 m
= h x
2 h x 3 In ABC,
40 40 20 60 x
h= x tan 45º = x = 100 m
3 3 3 3 100
Now in APB,
60 3 60 3 x h
h= 20 3 m tan 60º = x + h = 100 3
3 3 3 100
Thus, the height of the post is 20 3 m. h = 100 3 x 100 3 100
21. (b) Let the height of the tower be h.
h = 100( 3 1) m
D 23. (c) Let the height of the tree is h m.
C

h
h

60º
A B
A B 36 m C 16 m
64 m In ABC,
In BCD, h
tan 60º = h 16 3 m
h 16
tan = 24. (a) In PBT,
36
In ACD, h
tan 45º = 1
h y
tan =
64 y=h ...(i)
Since, angles of elevation are complementary. P
So, DAC and DBC are complementary angle. 30º 45º

= DAC
2 h
h
tan =
2 64 30º 45º
A x T y B
h 1 h
cot = Now, in PTA
64 tan 64
h
h tan 30º = x 3h ...(ii)
Now, put the value tan = x
36 Required distance = x + 4
h = 3h h
h = 64 h 64 36
36 = h( 3 1) m
h = 48 m. (The height of the tower)
Height and Distance 219
A-

25. (d) A h
Put the value of x =
30º 3

200–h
45º h
h = 21
3
30º
B C
x 1

200 m
h 1 = 21
3

h 21 3 ( 3 1)
h=
( 3 1) ( 3 1)

45º 21 3( 3 1)
D x E = = 49.68 50 m
2
27. (a) Let angle of elevation be .
200 h 1 500 h
In ACB, tan 30º = AB
x 3 x tan =
BC
x = (200 h) 3 and in ADE, A
200
tan 45º =
x
200 C B
1= x = 200 m AB 1
x
BC = 3 AB
From Eq. (i) BC 3
200 = (200 – h) 3 1
tan =
3 1 3
h = 200 m = 30º
3
28. (c) Let the height of the Chemney be h
26. (c) Let the height of the tower be h
AB
A In ABD tan 45º =
BD
h
tan 45º = 1 h x
x
h A (Chimney)

h
45º 60º
D C B 30º 45º
21 m x C D B
xm
In ABC,
Now, in ABC
AB
tan C = h 1
BC tan 30º =
120 x 3
h
tan 60º = Now put the value of h = x
x
h 1
h =
3 = 120 h 3
x
3h = 120 + h
h
x= 3h h = 120 h( 3 1) 120
3
In ABD, 120 3 1 120( 3 1)
h=
h 3 1 3 1 2
tan 45º = 1
21 x Height of the chimney (h) = 60( 3 1) m
h = 21 + x
EBD_7367
220
A- Height and Distance

29. (c) Let the inclination of the ladder angle be . Now, in ABD,
h 1
A tan 30º =
100 x 3

r)
de
ad
3h = 100 + x

(L
m
h
20 Now put the value of x =
3
C 10 m B
Now, in ABC 1
3 h= 100
BC 10 1 3
cos =
AC 20 2
cos = cos 60º 2
h = 100 h = 50 3
= 60º 3
A h = 50 × 1.732 = 86.6 m
30. (c)
Thus, the height of the tower (h) = 86.6m.
32. (a) Given that, AB = 6m and EC = 11m

h E

5m
90º– A
B 5m C 12 m D 11 m
D
6m
10 m
Here, given angles are comlementary. B 12 m C
ADB =
then, ACB = 90º – BC = 12 m
AB = h BC = AD = 12 m
BD = 5m and ED = EC – CD = EC – AB ( AB = CD)
BC = 10m = 11 – 6 = 5m
In ABD, In AED, Using Pythagoras theorem
(AE)2 = (AD)2 + (ED)2
h = (12)2 + (5)2 = 144 + 25
tan = ...(i)
5 = 169 = (13)2
In ABC, AE = 13m
h Them the distance between their tops is 13 m.
tan (90º – ) = Solutions (Q. Nos. 33-36) :
10
h A
cot = ...(ii)
10
From equation (i) and (ii),
h h h2 B C
tan cot = 1 3+ 3 hm
5 10 50 2
h= 50 hm
Therefore, height of the tower is 50 m.
31. (a) Let the height of inaccessible tower be h. D E
In ACD
33. (b) Given that, = 30º
h h
tan 60º = 3 x AE 1
x 3 In AED, tan = tan 30º =
DE 3
A
3 3
DE = 3 AE 3
2
h
h (Tower)

30º 60º 3
B C D BC = DE = (1 3)h ( BC = DE) ...(i)
100 m xm 2
Height and Distance 221
A-

Now, in ACB,
3
AC DE = (1 3)h ...(i)
tan = 2
BC In BDE,
BC tan = AE – CE = AE – BD ( BD = CE) BD h
3 3 3 3 2 tan =
BC tan = h h h DE DE
2 2
h
tan = [from Eq. (i)]
3 1 3 3
(1 3)h tan = h [from Eq. (i)] (1 3)h
2 2 2
1 2 ( 3 1) 2( 3 1)
tan = =
3 3 ( 3 1)( 3 1) 3.2
34. (a) Given that, = 30º
AC 1 ( 3 1) 3 (3 3)
=
In ACB, tan = tan 30º = 3 3 3 3
BC 3
BC = 3 AC = 3 (AE – CE) 37. (c)
= 3 (AE – BD) ( BD = CE) O
3 3 3
= 3 1 h (1 3) h ...(ii)
2 2 C
Now, in AED, h
60º
AE AE
tan = tan = ( DE = BC) 60º
DE BC A B
In ABO,
3+ 3 3(1 3)
h h OB OB
2 2 sin 60º = AO = ...(i)
= AO sin 60º
3 3(1 3) Now, in ACO
(1 3)h h
2 2
60º OC OC
tan = 1 sin = AO = ...(ii)
35. (c) Given, = 30º and h = 30 m 2 AO sin30º
In ACB, Comparing equation (i) and equation (ii)
AC 1 OB OC h r
tan = tan 30º = =
BC 3 sin 60º sin 30º 3 1
2 2
BC
= (AE – CE) = (AE – BD) ( BD = CE) h = 3r
3
38. (c) Let angle of elevation be .
3 3
BC = 3 1 h
2

(1 3) x
BC = 3 30 ( 3 3) 15
2
DE = BC = (45 15 3) m ( DE = BC)
x/ 3
36. (a) Given that, = 30º
In ADE, Height of the pole = xm

3+ 3 and length of shadow = x / 3 m


h
AE 2 x 3x
tan = tan 30º = tan = 3
DE DE x x
1 3 3
3 h
1 2 tan = 3 tan 60º
= = 60º
3 DE
EBD_7367
222
A- Height and Distance

A 42. (c) A
39. (d)
x–y z
1 D
300
C m
B s
x
y C
B
200 m
Let a person be at point C observe a tower in west
D s E direction at B. After walking some distance along
road he observed same tower in south direction. Let
Here, BC is the shortest distance angle C be =
In ABC, AB2+ BC2 = AC2 (Use Pythagoras theorem) In ABC
z2 = (x – y)2 + s2 300 3
z2 = x2 + y2 – 2xy + s2 tan = 1
200 2
s2 = z2 – x2 – y2 + 2xy
h (3)2 (2) 2
40. (b) Let the height of tower be h and BC = xm.
9 4 13
h h
In BCD, tan 60º = 3
x x 3
sin
13
h= x 3 ...(i)
In BDC
D P BD
sin
h 200
h BD = 200 sin
3 600
200 m
30º 60º 13 13
A B C
50 m x 43. (d) A
h
In ACD, tan 30º =
50 x
h
1 x 3
=
3 50 x (90 – x)° x°
50 + x = 3x B m 2 P Q
x = 25 m n 2
Now put the value of x in equation (i)
Let h be the height of tower
h = 25 3 m In ABQ
41. (b) Let the height of shorter tower be h then distance
between two tower = hm. AB h
tan x = ...(i)
BQ n2
C
In ABP
D AB h
tan (90 – x) =
30 m BP m2
h
1 h h
cot x ; tan x 2
60º 30º m m2
A B
mh Now, from eqn (i),
1 h
h 1 1
In ABD, tan 30º = h m2
mh 3 m 2
n
m= 3 h2 = m2n2
h = mn
Height and Distance 223
A-

C (cloud) 100 3 3 3 3 1
44. (c) h
h
3 1 3 1
P 30° M
60° 100 3 3 3 3 3 3
200 m 200 m 3 1
Surface of lake 100 2 3 3
h + 200 2
h = 100 × 3 = 300 m

46. (c) P1
C (Reflection)

h
tan 30° = PM = 3h
PM 3000
P2 m
h 400 h 400
tan 60° = PM =
PM 3
h 400 45° 60°
3h O C
= 3h – h = 400
3 CP1 = 3000 m
2h = 400
So, height of the cloud = 200 + 200 = 400m CP1 3000
OC = m
45. (c) Let AB is tower whose height is h. tan 60 3
Distance between objects P and Q are 100(3 – 3)m 3000
and BP is x m. CP2 = OC = m
3
A
60° 45° 3000
P1P2 = CP1 – CP2 = 3000
3
h
= (3000 – 100 3 ) m
= 1000 (3- 3 ) m
60° 45° So, option (c) is correct.
B P Q 47. (d) In PAB
xm 100 (3– 3 )m

In ABP A
AB h
tan 60
x x
h 12 m
x ...(i)
3
In ABQ
AB h 30°
tan 45° = P B
BQ 100(3 – 3) x
h = 100 (3 – 3 ) + x AB
sin 30° =
Now, from eqn (i), we put value of x AP
h 1 AB
h 100 3 3
3 2 12
h 12 2 3
h 100 3 3 AB = =
3 2 2
3 1 AB = 3m
h 100 3 3 So, option (d) is correct.
3
EBD_7367
224
A- Height and Distance

48. (c) Let two observers A and B are stationed at a distances AB


y from each other. = tan 30°
DB
D 100 1
; DB = 100 3
DB 3
in ABC
AB
x = tan 60º
BC
100 100
3 ; BC =
45° BC 3
30°
B C 100
A DC = 100 3
x 3
y t
300 100 200 200 3
Let CD be a tower of height x metre. =
3 3 3
Also DAC = 45° and DBC = 30°
50. (a) in ADC
x x C
In ACD, tan 45 and In DBC, tan 30
t y t

x
1 ...(1)
t
75°
1 x
...(2)
3 y t
From (1) and (2) we get 130° 60°
t 1 A D B
y t 3 DC
= tan 30°
AC
3t y t
75 1
3 1t y ; AC = 75 3
AC 3
in BDC
1
t y DC
3 1 = tan 60°
BD
From (1) we get
x= t 75 75 75 3
3 ; BD =
BD 3 3
1 3 1
x y AB = 75 3 25 3 100 3
3 1 3 1
51. (c) A
3 1 3 1
3 1 2
Option (c) is correct.
49. (b) In ABD x

B x C
When object height is equals to its shadow than sun
is at 45° altitude
100 AB
= tan
BC
AB
30 60 tan = =1
AB
D C B = 45°
Height and Distance 225
A-

52. (a) Centroid A


53. (a) in ACD
AC
= tan 60°
DC
h
A 55. (b)
Plan A
B x C
1 km
h
B Plan B tan = ...(i)
300 x
h
tan = ...(ii)
1 x
60°
h tan
45° tan =
D C tan h
tan . tan
Ground h=
tan tan
300 300 56. (b)
3 ; DC = =
DC 3 100 3
in BCD
BC
= tan 45°
DC
300
BC = 100 3
3
54. (b) AB in ABC
A
90 m

Since A = 30° and we know that the angle subtended


by an arc at the center of a circle is double the angle
30°
D E subtended by it at any point on the remaining part of
the circle, therefore in the center, O = 2 × 30° = 60°
Tree Building
45° Also, since OBC is an isosceles triangle so, its base
angles will be equal i.e. B and C are equal.
C B
Let these angles be x.
AB Therefore, by angle sum property of a triangle,
= tan 45°
BC O B C 180 60 x x 180
AB 60 2 x 180 2 x 120 x 60
= 1; AB = 90 Thus, we say that OBC is an equilateral triangle and
90
in AED hence, BC is also equal to 10 cm.

AE
= tan30° 57. (c)
90
AE 1 90
; AE =
90 3 3
90
BC = 90 –
3
BC = 90 – 30 3 = DC (Tree)
EBD_7367
226
A- Height and Distance

In AA’C, we have ...(1)


d 3 h x
A'C x x If > 45°, let us suppose = 30°.
tan 45 1
AC AC AC
So, in A’BB’, we have
AC x AC BB ' x d
Adding h on both sides, we get h + x = h + d h+ A' B ' h x 1 h x
tan 30 d
x>d BB ' d 3 d
Hence, statement 1 is true.
3(h x) …(2)
Now, in A’BB’, we have
From equation (1), we can conclude that either LHS
A'B' h x h x
tan 45 1 d h x = RHS or LHS > RHS but from equation (2), we con-
BB ' d d clude that LHS < RHS. Hence, we cannot have surety
But, by statement 1, this is not possible. Therefore, that angle of depression of B as seen from A’ is less
45°. than 45° which states that statement 2 is false.
Now, we can say that either < 45° or > 45°. If > 45°,
let us suppose = 60°. 58. (c)
So, in A’BB’, we have 59. (b)
A'B' h x h x 60. (c)
tan 60 3
BB ' d d
Area and Perimeter A-227

C HA P T E R
AREA AND PERIMETER
19
1. Consider a circle C of radius 6 cm with centre at O. What is
the difference in the area of the circle C and the area of the æ3ö 2 æ7ö 2
(a) ç ÷x (b) ç ÷ x
sector of C subtending an angle of 80º at O? [2007-I] è4ø è8ø
(a) 26p cm2 (b) 16p cm2
(c) 28p cm2 (d) 30p cm2 æ 11 ö 2 æ 15 ö 2
(c) ç ÷x (d) ç ÷ x
2. A rectangle carboard is 18 cm × 10 cm. From the four corners 12
è ø è 16 ø
of the rectangle, quarter circles of radius 4 cm are cut. What 10. If x and y are respectively the areas of a square and a rhombus
is the perimeter (approximate) of the remaining portion? of sides of same length, then what is x : y?
[2007-II] [2008-II]
(a) 47.1 cm (b) 49.1 cm
(c) 51.0 cm (d) 53.0 cm (a) 1 : 1 (b) 2: 3
3. A cycle wheel makes 1000 revolutions is moving 440 m.
(c) 4: 3 (d) 3 : 2
What is the diameter of the wheel? [2007-II]
(a) 7 cm (b) 14 cm 11. If the area of a circle, inscribed in an equilateral triangle is
(c) 28 cm (d) 21 cm 4p cm2, then what is the area of the triangle? [2008-II]
4. A circle is inscribed in an equilateral triangle of side a. (a) 12 3 cm2 (b) 9 3 cm2
What is the area of any square inscribed in this circle?
[2007-II] (c) 8 3 cm2 (d) 18 cm2
a2 12. In the given figure, DABC is a right angled triangle, right
a2
(a) (b) angled at A. Semi-circles are drawn on the sides AB, BC
3 4 and AC. Then, the area of shaded portion is equal to which
a2 a2 one of the following? [2008-II]
(c) (d)
6 8 A
5. The perimeter of a square S1 is 12 m more than perimeter of
the square S2. If the area of S1 equals three times, the area of
S2 minus 11, then what is the perimeter of S1? [2008-1]
(a) 24 m (b) 32 m
(c) 36 m (d) 40 m B C
6. From a rectangular sheet of cardboard of size 5 cm × 2 cm, (a) Area of DABC
the greatest possible circle is cut-off. What is the area of (b) 2 times the area of DABC
the remaining part? [2008-I] (c) Area of semi-circle ABC
(a) (25 – p) cm2 (b) (10 – p) cm2 (d) None of the above
(c) (4 – p) cm2 (d) (10 – 2p) cm2
13. In the given figure, ABC is a right angled triangle, right
7. A chord AB of a circle of radius 20 cm makes a right angle at
angled at B. BC = 21 cm and AB = 28 cm. Width AC as
the centre of the circle. What is the area of the minor segment
diameter of a semi-circle and width BC as radius a quarter
in cm2? (take p = 3.14) [2008-I]
(a) 31.4 cm2 (b) 57 cm2 circle are drawn. What is the area of the shaded portion?
(c) 62.8 cm2 (d) 114 cm2 [2008-II]
8. The minute hand of a clock is 14 cm long. How much
distance does the end of the minute hand travel in 15 min? A

æ 22 ö
ç take p = ÷ [2008-I]
è 7 ø
(a) 11 cm (b) 22 cm
(c) 33 cm (d) 44 cm
9. A square of side x is taken. A rectangle is cut out from this
square such that one side of the rectangle is half that of the B C
1 (a) 425 cm2 (b) 425.75 cm2
square and the other is rd the first side of the rectangle.
3 (c) 428 cm2 (d) 428.75 cm2
What is the area of the remaining portion? [2008-I]
EBD_7367
228
A- Area and Perimeter

14. If a man walking at the rate 3 km/h crosses a square field T


diagonally in 1 min, then what is the area of the field?
[2009-I] U
(a) 1000 m 2 (b) 1250 m 2
P S
(c) 2500 m2 (d) 5000 m2 Q O R
15. From a rectangular metal sheet of sides 25 cm and 20 cm, a V
circular sheet as large as possible is cut-off. What is the
(a) 12p cm (b) 14p cm
area of the remaining sheet? [2009-I]
(c) 16p cm (d) 18p cm
(a) 186 cm2 (b) 144 cm2
23. A person rides a bicycle round a circular path of radius 50
(c) 93 cm2 (d) 72 cm2 m. The radius of the wheel of the bicycle is 50 cm. The
16. What is the area of a right angled isosceles triangle whose cycle comes to the starting point for the first time in 1 h.
hypotenuse is 6 2 cm? [2009-I] What is the number of revolutions of the wheel in 15 min?
(a) 12 cm2 (b) 18 cm2 [2009-II]
(c) 24 cm2 (d) 36 cm2 (a) 20 (b) 25
17. If A is the area of a triangle in cm 2, whose sides are 9 cm, 10 (c) 30 (d) 35
cm and 11 cm, then which one of the following is correct? 24. In the given figure, the side of square ABCD is 7 cm. What
is the area of the shaded portion, formed by the arcs BD of
[2009-I]
the circles with centre at C and A? [2010-I]
(a) A < 40 cm2 (b) 40 cm2 < A < 45 cm2
(c) 45 cm2 < A < 50 cm2 (d) A > 50 cm2 A D
18. The difference between the area of a square and that of an
equilateral triangle on the same base is 1/4 cm2. What is the
length of side of triangle? [2009-II]

(4 - 3) (4 + 3)
1/2 1/2
(a) cm (b) cm

(4 - 3) (4 + 3)
-1/2 -1/2
(c) cm (d) cm B C
19. A horse is tied to a pole fixed at one corner of a 50 m × 50 m (a) 7 cm 2 (b) 28 cm2
square field of grass by means of a 20 m long rope. What is (c) 14 cm 2 (d) 21 cm2
the area of that part of the field which the horse can graze? 25. What is the maximum area of a rectangle, the perimeter of
[2009-II] which is 18 cm? [2010-I]
(a) 1256 m2 (b) 942 m2 (a) 20.25 cm2 (b) 20.00 cm2
(c) 628 m2 (d) 314 m2 (c) 19.75 cm2 (d) 19.60 cm2
20. A wire is in the form of a circle of radius 42 cm. If it is bent 26. Three circular laminas of the same radius are cut out from a
into a square, then what is the side of the square? larger circular lamina. When the radius of each lamina cut
out is the largest possible, then what is the ratio
[2009-II]
(approximate) of the area of the residual piece of the original
(a) 66 cm (b) 42 cm
lamina to its original total area? [2010-I]
(c) 36 cm (d) 33 cm
(a) 0.30 (b) 0.35
21. Seven semi-circular areas are revoved from the rectangle
(c) 0.40 (d) 0.45
ABCD as shown in the figure above, in which AB = 2 cm 27. In the figure given below, AB is a line of length 2a, with M
and AD = 0.5 cm. The radius of each semi-circle, r, s, t, u and as mid-point. Semi-circles are drawn on one side with AM,
v is half of that of semi-circle p or q. What is the area of the MB and AB as diameters. A circle with centre O and radius
remaining portion? r is drawn such that this circle touches all the three semi-
A B circles. What is the value of r? [2010-II]
r s t
p q O
u v
D C
(a) (128 – 13p)/128 cm (b) (125 – 13p)/125 cm2
2
A M B
(c) (128 – 15p)/128 cm 2 (d) None of these
22. PQRS is a diameter of a circle of radius 6cm as shown in the 2a a
(a) (b)
figure above. The lengths PQ, QR and RS are equal semi 3 2
circles are drawn on PQ and QS as diameters.What is the a a
peremeters of the shaded region ? [2009-II] (c) (d)
3 4
Area and Perimeter A-229
28. A circle and a square have the same perimeter. Which one (a) 18 cm (b) 14 cm
of the following is correct? [2010-II] (c) 12 cm (d) 8 cm
(a) The area of the circle is equal to that of square 40. What is the area of an equilateral triangle having altitude
(b) The area of the circle is larger than that of square equal to 2 3 cm? [2011-II]
(c) The area of the circle is less than that of square
(d) No conclusion can be drawn (a) 3 sq cm (b) 2 3 sq cm
29. What is the radius of the circle inscribed in a triangle having
(c) 3 3 sq cm (d) 4 3 sq cm
side lengths 35 cm, 44 cm and 75 cm? [2010-II]
(a) 3 cm (b) 4 cm 41. If a lawn 30 m long and 16 m wide is surrounded by a path
(c) 5 cm (d) 6 cm 2 m wide, then what is the area of the path? [2011-II]
30. A rectangular area of 6 sq m is to be painted on a 3m × 4 m (a) 200 m2 (b) 280 m2
board leaving a border of uniform width on all sides. What (c) 300 m2 (d) 320 m2
should be the width of the border? [2010-II] 42. If a circle circumscribes a rectangle with side 16 cm and 12
(a) 0.25 m (b) 0.5 m cm, then what is the area of the circle? [2011-II]
(c) 1 m (d) 3 m (a) 48 p sq cm (b) 50 p sq cm
31. A wheel of a bicycle has inner diameter 50 cm and thickness (c) 100 p sq cm (d) 200 p sq cm
10 cm. What is the speed of the bicycle, if it makes 10 43. The lengths of two sides of a right angled triangle which
revolutions in 5 s? [2010-II] contain the right angle are a and b, respectively. Three
(a) 5.5 m/s (b) 4.4 m/s squares are drawn on the three sides of the triangle on the
(c) 3.3 m/s (d) 2.2 m/s outer side. What is the total area of the triangle and the
32. If a wire of length 36 cm is bent in the form of a semi-circle, three squares? [2011-II]
then what is the radius of the semi-circle? [2010-II] (a) 2 (a2 + b2) + ab (b) 2 (a2 + b2) + 2.5 ab
(a) 9 cm (b) 8 cm (c) 2 (a2 + b2) + 0.5ab (d) 2.5 (a2 + b2)
(c) 7 cm (d) 6 cm 44. A wall is of the form of a trapezium with height 4 m and
33. Three congruent circles each of radius 4 cm touch one parallel sides being 3 m and 5 m. What is the cost of painting
another. What is the area (in cm2) of the portion included the wall, if the rate of painting is Rs. 25 per sq m? [2011-II]
between them? [2011-I] (a) ` 240 (b) ` 400
(c) ` 480 (d) ` 800
(a) 8 p (b) 16 3 - 8p
45. A grassy field has the shape of an equilateral triangle of
(c) 16 3 - 4 p (d) 16 3 - 2 p side 6 m. A horse is tied to one of its vertices with a rope of
34. The two diagonals of a rhombus of lengths 55 cm and 48 length 4.2 m. The percentage of the total area of the field
cm. If p is the perpendicular height of the rhombus, then which is available for grazing is best approximated by
which one of the following is correct? [2011-I] [2011-II]
(a) 36 cm < p < 37 cm (b) 35 cm < p < 36 cm (a) 50% (b) 55%
(c) 34 cm < p < 35 cm (d) 33 cm < p < 34 cm (c) 59% (d) 62%
35. The perimeter of a triangular field is 240 m. If two of its 46. The areas of two circles are in the ratio 1 : 2. If the two
sides are 78 m and 50 m, then what is the length of the circles are bent in the form of squares, then what is the
perpendicular on the side of length 50 m from the opposite ratio of their areas? [2011-II]
vertex? [2011-I] (a) 1 : 2 (b) 1 : 3
(a) 43 m (b) 52.2 m (c) 1 : 2 (d) 1 : 4
(c) 67.2 m (d) 70 m 47. If the area of a rectangle whose length is 5 units more than
36. A piece of wire 78 cm long is bent in the form of an isosceles twice its width is 75 sq units, then what is the width?
triangle. If the ratio of one of the equal sides to the base is [2011-II]
5 : 3, then what is the length of the base? [2011-I] (a) 3 units (b) 5 units
(a) 16 cm (b) 18 cm (c) 7 units (d) 10 units
(c) 20 cm (d) 30 cm 48. If the diameter of a circle circumscribing a square is
37. The length of a minute hand of a wall clock is 9 cm. What is
the area swept (in cm2) by the minute hand in 20 min? (take 15 2 cm, then what is the length of the side of the square?
p = 3.14) [2011-I] [2011-II]
(a) 88.78 (b) 84.78 (a) 15 cm (b) 12 cm
(c) 67.74 (d) 57.78 (c) 10 cm (d) 7.5 cm
38. If the outer and inner diameters of a stone parapet around 49. In the DABC, the base BC is trisected at D and E. The line
a well are 112 cm and 70 cm respectively. Then, what is the through D, parallel to AB, meets AC at F and the line through
area of the parapet? [2011-II] E parallel to AC meets AB at G. If EG and DF intersect at H,
(a) 264 sq cm (b) 3003 sq cm then what is the ratio of the sum of the area of parallelogram
(c) 6006 sq cm (d) 24024 sq cm AGHF and the area of the DDHE to the area of the DABC?
39. If the area of a DABC is equal to area of square of side [2012-I]
length 6 cm, then what is the length of the altitude of AB, (a) 1/2 (b) 1/3
where AB = 9 cm? [2011-II] (c) 1/4 (d) 1/6
EBD_7367
230
A- Area and Perimeter

50. If the area of a circle is equal to the area of a square with 61. In the figure given below, the area of rectangle ABCD is
side 2 p units, then what is the diameter of the circle? 100 sq cm. O is any point on AB and CD = 20 cm. Then, the
area of DCOD is [2013-I]
[2012-I]
(a) 1 unit (b) 2 units A O B
(c) 4 units (d) 8 units
51. A square, a circle and an equilateral triangle have same
perimeter. [2012-I]
Consider the following statements
I. The area of square is greater than the area of the triangle.
II. The area of circle is less than the area of triangle. D C
Which of the above statement is/are correct? (a) 40 sq cm (b) 45 sq cm
(a) Only I (b) Only II (c) 50 sq cm (d) 80 sq cm
(c) Both I and II (d) Neither I nor II 62. If an isosceles right angled triangle has area 1sq unit, then
52. If the area of a rectangle whose length is 5 more than twice what is its perimeter? [2013-I]
its width is 75 sq units. What is the perimeter of the (a) 3 units (b) 2 2 + 1 units
rectangle? [2012-I]
(a) 40 units (b) 30 units (c) ( 2 + 1) units (d) 2( 2 + 1) units
(c) 24 units (d) 20 units 63. A circular water fountain 6.6 m in diameter is surrounded
53. If the altitude of an equilateral triangle is 3 cm, then out side by a path of width 1.5 m. The area of this path (in
what is its perimeter? [2012-I] sq m) is [2013-I]
(a) 13.62 p (b) 13.15 p
(a) 3 cm (b) 3 3 cm (c) 12.15 p (d) None of these
(c) 6 cm (d) 6 3 cm 64. The area of a rectangular field is 4500 sq m. If its length and
54. The area of a rectangle, whose one side is a and other side breadth are in the ratio 9 : 5, then its perimeter is [2013-I]
is 2a2. What is the area of a square having one of the (a) 90 m (b) 150 m
diagonals of the rectangle as side? [2012-I] (c) 280 m (d) 360 m
(a) 2a2 (b) 3a2 65. The area of a square inscribed in a circle of radius 8 cm is
(c) 4a2 (d) 5a2 [2013-I]
55. Consider the following statements [2012-II] (a) 32 sq cm (b) 64 sq cm
I. Area of a segment of a circle is less than area of its (c) 128 sq cm (d) 256 sq cm
corresponding sector. 66. The short and long hands of a clock are 4 cm and 6 cm
II. Distance travelled by a circular wheel of diameter 2d long, respectively. Then, the ratio of distances travelled
cm in one revolution is greater than 6d cm. by tips of short hand in 2 days and long hand in 3 days is
Which of the above statements is/are correct? [2013-I]
(a) Only I (b) Only II (a) 4 : 9 (b) 2 : 9
(c) Both I and II (d) Neither I nor II (c) 2 : 3 (d) 1 : 27
56. The perimeter of a rectangle is 82 m and its area is 400 sq m. 67. The arc AB of the circle with centre at O and radius 10 cm
What is the breadth of the rectangle? [2012-II] has length 16 cm. What is the area of the sector bounded
(a) 18 m (b) 16 m by the radii OA, OB and the arc AB? [2013-I]
(c) 14 m (d) 12 m (a) 40p sq cm (b) 40 sq cm
57. The area enclosed between the circumferences of two (c) 80 sq cm (d) 20p sq cm
concentric circles is 16 p sq cm and their radii are in the 68. The minute hand of a watch is 2.5 cm long. The distance its
ratio 5 : 3. What is the area of the outer circle? [2012-II] extreme end transverses in 40 min is [2013-I]
(a) 9p sq cm (b) 16p sq cm (a) 10p/3 cm (b) 3p/10 cm
(c) 25p sq cm (d) 36p sq cm (c) 10/3 cm (d) 10 cm
58. If the circumference of a circle is equal to the perimeter of 69. If the area of a regular hexagon is 96 3 sq cm, then its
square, then which one of the following is correct? perimeter is [2013-I]
[2012-II] (a) 36 cm (b) 48 cm
(a) Area of circle = Area of square (c) 54 cm (d) 64 cm
(b) Area of circle ³ Area of square 70. What is the area of a circle whose area is equal to that of a
(c) Area of circle > Area of square triangle with sides 7 cm, 24 cm and 25 cm? [2013-II]
(d) Area of circle < Area of square (a) 80 cm2 (b) 84 cm2
59. If the circumferences of two circles are in the ratio 2 : 3, (c) 88 cm2 (d) 90 cm2
then what is the ratio of their areas? [2012-II] 71. If the area of an equilateral triangle is x and its perimeter is
(a) 2 : 3 (b) 4 : 9 y, then which one of the following is correct? [2013-II]
(c) 1 : 3 (d) 8 : 27 (a) y4 = 432x2 (b) y4 = 216x2
60. If the area of a circle inscribed in an equilateral triangle is 154 (c) y2 = 432x2 (d) None of these
sq cm, then what is the perimeter of the triangle? [2012-II] 72. A rectangular field is 22 m long and 10 m wide. Two
(a) 21 cm (b) 42 3 cm hemispherical pitholes of radius 2 m are dug from two places
and the mud is spread over the remaining part of the field.
(c) 21 3 cm (d) 42 cm The rise in the level of the field is. [2013-II]
Area and Perimeter A-231

8 13 (a) 48 cm2 (b) 32 cm2


(a) m (b) m (c) 0 (d) None of these
93 93
85. Consider an equilateral triangle of a side of unit length. A
16 23 new equilateral triangle is formed by joining the mid-points
(c) m (d) m
93 93 of one, then a third equilateral triangle is formed by joining
73. The area of an isosceles DABC with AB = AC and altitude the mid-points of second. The process is continued. The
AD = 3 cm is 12 sq cm. What is its perimeter? [2013-II] perimeter of all triangles, thus formed is [2014-I]
(a) 18 cm (b) 16 m (a) 2 units (b) 3 units
(c) 14 cm (d) 12 cm (c) 6 units (d) Infinity
74. A hospital room is to accommodate 56 patients. It should 86. What is the area of the larger segment of a circle formed by
be done in such a way that every patient gets 2.2 m2 of a chord of length 5 cm subtending an angle of 90º at the
floor and 8.8 m3 of space. If the length of the room is 14 m, centre? [2014-I]
then breadth and the height of the room are respectively
[2013-II] 25 æ p ö 2 25 æ p ö 2
(a) 8.8 m, 4 m (b) 8.4 m, 4.2 m
(a) ç + 1÷ cm (b) ç - 1÷ cm
4 è2 ø 4 è2 ø
(c) 8 m, 4 m (d) 7.8 m, 4.2 m
75. How many 200 mm lengths can be cut from 10 m of ribbon? 25 æ 3p ö 2
[2013-II]
(c) ç + 1÷ cm (d) None of these
4 è 2 ø
(a) 50 (b) 40 87. A rectangle of maximum area of drawn inside a circle of
(c) 30 (d) 20 diameter 5 cm. What is the maximum area of such a
76. What is the area between a square of side 10 cm and two rectangle? [2014-I]
inverted semi-circular, cross-sections each of radius 5 cm
(a) 25 cm2 (b) 12.5 cm2
inscribed in the square? [2013-II]
(a) 17.5 cm2 (b) 18.5 cm2 (c) 12 cm2 (d) None of these
(c) 20.5 cm2 (d) 21.5 cm2 88. If AB and CD are two diameters of a circle of radius r and
77. The perimeter of a rectangle having area equal to 144 cm 2 they are mutually perpendicular, then what is the ratio of
and sides in the ratio 4 : 9 is [2013-II] the area of the circle to the area of the DACD? [2014-I]
(a) 52 cm (b) 56 cm p
(c) 60 cm (d) 64 cm (a) (b) p
2
78. One side of a parallelogram is 8.06 cm and its perpendicular p
distance from opposite side is 2.08 cm. What is the (c) (d) 2p
approximate area of the parallelogram? [2013-II] 4
(a) 12.56 cm2 (b) 14.56 cm2 89. The area of a rectangle lies between 40 cm2 and 45 cm2. If one
(c) 16.76 cm2 (d) 22.56 cm2 of the sides is 5 cm, then its diagonal lies between [2014-I]
79. If the diagonals of a rhombus are 4.8 cm and 1.4 cm, then (a) 8 cm and 10 cm (b) 9 cm and 11 cm
what is the perimeter of the rhombus? [2013-II] (c) 10 cm and 12 cm (d) 11 cm and 13 cm
(a) 5 cm (b) 10 cm 90. The areas of the three adjacent faces of a cuboidal box are x,
(c) 12 cm (d) 20 cm 4x and 9x sq unit. What is the volume of the box ? [2014-II]
80. How many circular plates of diameter d be taken out of a (a) 6x2 cu unit (b) 6x3/2 cu unit
3/2
square plate of side 2d with minimum loss of material? (c) 3x cu unit (d) 2x3/2 cu unit
[2014-I] 91. The sides of a triangular field are 41 m, 40 m and 9 m. The
(a) 8 (b) 6 number of rose beds that can be prepared in the field if each
(c) 4 (d) 2 rose bed, on an average, needs 900 square cm space, is
81. What is the total area of three equilateral triangles inscribed [2015-I]
in a semi-circle of radius 2 cm? [2014-I] (a) 2000 (b) 1800
3 3 cm2 (c) 900 (d) 800
(a) 12 cm2 (b) 92. The ratio of the outer and inner perimeters of a circular path
4 is 23 : 22. If the path is 5 m wide, the diameter of the inner
9 3 circle is [2015-I]
(c) cm2 (d) 3 3 cm2
4 (a) 55 m (b) 110m
82. The area of sector of a circle of radius 36 cm is 72p cm2. The (c) 220 m (d) 230 m
length of the corresponding arc of the sector is [2014-I] 93. Four equal-sized maximum circular plates are cut off from a
(a) p cm (b) 2 p cm square paper sheet of area 784 square cm. The circumference
(c) 3 p cm (d) 4 p cm of each plate is [2015-I]
83. A square is inscribed in a circle of diameter 2a and another
(a) 11 cm (b) 22 cm
square is circumscribing circle. The difference between the
areas of outer and inner squares is [2014-I] (c) 33 cm (d) 44 cm
(a) a2 (b) 2a2 94. A square and an equilateral triangle have equal perimeter. If
(c) 3a2 (d) 4a2 the diagonal of the square is 12 2 cm, then the area of the
84. ABC is a triangle right angled at A. AB = 6 cm and AC = 8 triangle is [2015-II]
cm. Semi-circles drawn (outside the triangle) on AB, AC 2 2
and BC as diameters which enclose areas x, z square units, (a) 24 2 cm (b) 24 3 cm
respectively. What is x + y – z equal to? [2014-I] (c) 48 3 cm
2 (d) 64 3 cm
2
EBD_7367
232
A- Area and Perimeter

95. A boy is cycling such that the wheels of the cycle are making
140 revolutions per minute. If the radius of the wheel is 30 2 3
(a) (b)
cm, the speed of the cycle is [2015-II] 3 2
(a) 15.5 km/hour (b) 15.84 km/hour 3 2
(c) 16 km/hour (d) 16.36 km/hour (c) (d)
96. There are 437 fruit plants in an orchard planted in rows. The 2 3
distance between any two adjacent rows is 2 m and the 105. A circle and a square have the same perimeter. Which one
distance between any two adjacent plants is 2m. Each row of the following is correct ? [2016-II]
has the same number of plants. There is 1 m clearance on all (a) Their areas are equal
sides of the orchard. What is the cost of fencing the area at (b) The area of the circle is larger
the rate of `100 per metre [2015-II] p
(a) ` 15,600 (c) The area of the square is times area of circle
2
(b) `16,800 (d) The area of the square is p times area of circle
(c) ` 18,200 106. What is the area of a triangle with sides of length 12 cm,
(d) More information is required 13 .cm and 5 cm ? [2016-II]
97. The circumference of a circle is 100 cm. The side of the (a) 30 cm2 (b) 35 cm2
square inscribed in the circle is [2015-II] (c) 40 cm2 (d) 42 cm2
100 107. If the perimeter of a rectangle is 10 cm and the area is 4 cm2,
(a) 50 2cm (b) cm then its length is [2017-I]
p (a) 6 cm (b) 5 cm
(c) 4.5 cm (d) 4 cm
50 2 100 2 108. The areas of two circular fields are in the ratio 16 : 49. If the
(c) cm (d) cm
p p radius of the bigger field is 14 m, then what is the radius of
98. The diameter of a wheel that makes 452 revolutions to move the smaller field ? [2017-I]
2 km and 26 decametre is equal to [2015-II] (a) 4 m (b) 8 m
(c) 9 m (d) 10 m
9 13
(a) 1 m (b) 1 m 109. The area of a regular hexagon of side 'a' is equal to
22 22 [2017-I]
5 7 2 2 3 3 2
(c) 2 m (d) 2 m (a) a square units (b) a square units
11 11 3 2
99. A square is inscribed in a right triangle with legs x and y and
has common right angle with the triangle. The perimeter of 1 2 3 2
(c) a square units (d) a square units
the square is given by [2015-II] 3 2
110. An isosceles triangle is drawn outside on one of the sides
2xy 4xy
(a) (b) of a square as base in such a way that the perimeter of the
x+y x+y
7
complete figure is times the perimeter of the original
2xy 4xy 6
(c) 2 2 (d) square. What is the ratio of area of the triangle to the area of
x +y x 2 + y2 the original square? [2017-II]
100. The area of a trapezium is 336 cm2. If its parallel sides are in (a) 1 : 1 (b) 2 : 3
the ratio 5 : 7 and the perpendicular distance between them (c) 1 : 2 (d) 1 : 3
is 14 cm, then the smaller of the parallel sides is [2015-II] 111. What is the area of the triangle whose sides are 51 cm, 37
(a) 20 cm (b) 22 cm cm and 20 cm? [2017-II]
(c) 24 cm (d) 26 cm (a) 300 square cm (b) 305 square cm
101. The number of rounds that a wheel of diameter 7/11 metre (c) 306 square cm (d) 307 square cm
will make in traversing 4 km will be [2016-I] 112. If the length of a side of a square is increased by 8 cm, its
(a) 500 (b) 1000 area increases by 120 square cm. What is the length of a
side of the square? [2018-I]
(c) 1700 (d) 2000 (a) 2.5 cm (b) 3.5 cm
102. The base of an isosceles triangle is 300 unit and each fo tis (c) 4.5 cm (d) 5.5 cm
equal sides is 170 units. Then the area of the triangle is 113. A rectangular pathway having width 4.5 m and length 10 m
[2016-I] will have to be tiled using square tiles of side
(a) 9600 square units (b) 10000 square units 50 cm. Each packet of such tiles contains 20 pieces and
(c) 12000 square units (d) None of the above costs `100. What will be the total cost of tiles for the
103. Four equal discs are placed such that each one touches pathway? [2018-I]
two others. If the area of empty space enclosed by them is (a) `1,200 (b) `1,100
150/847 square centimetre, then the radius of each disc is (c) `1,000 (d) `900
equal to [2016-I] 114. The product of the lengths of the diagonals of a square is
(a) 7/6 cm (b) 5/6 cm 50 square units. What is the length of a side of the square?
(c) 1/2 cm (d) 5/11 cm [2018-I]
104. If a square of side x and an equilateral triangle of side y are (a) 5 2 units (b) 5 units
inscribed in a circle, then what is the ratio of x to y ?[2016-II] (c) 10 units (d) 2 5 units
Area and Perimeter 233
A-

HINTS & SOLUTIONS


1. (c) 1
Þ OD = AD
3
a
O = ...(i)
2 3
80° Now, OD is radius of circle. Therefore diagonal of square
=2×r
a a
= =
2 3 3
Radius of circle, r = 6 cm Let side of a square = y
\ Area of circle = pr2 = p × 62 = 36p cm2 2
and Area of sector subtending an angle of 80º at O æ a ö
\ ç ÷ = y2 + y2
pr 2 q p´ 62 ´ 80º è 3ø
= = = 8p cm2
360º 360º a2
\ Required difference = 36p – 8p = 28p cm2 Þ = 2y2
3
2. (b) 4 cm 4 cm a2
4cm

10 cm
4cm

y2 = = Area of square.
6
2 cm 10 cm 2 cm 5. (b) Suppose side of S1 = a
Side of S2 = b
4cm

Perimeter of S1 = 4a
4cm

18 cm
4cm 4cm Perimeter of S2 = 4b
Area of S1 = a2
Remaining perimeter
Area of S2 = b2
æ 2pr ö According to Question,
=ç ÷ 4 + 10 + 2 + 10 + 2 = 2 ´ 3.14 ´ 4 + 24
è 4 ø 4a = 4b + 12
= 25.12 + 24 = 49.12 cm Þa=b+3
= 49.1 cm (approx) and a2 = 3(b2) – 11 Þ (b + 3)2 = 3(b2) – 11
440 Þ b2 + 9 + 6b = 3b2 – 11
3. (b) Distance travel in 1 revolution = m Þ 2b2– 6b – 20 = 0
1000
Þ b2– 3b – 10 = 0
44000 Þ b2– 5b + 2b – 10 = 0
and circumference = p × d = cm
1000 Þ b(b – 5) + 2(b – 5) = 0
44000 ´ 7 Þ b = 5, b ¹ –2
\ d= = 14 cm \ Perimeter of S1 = 4 × 5 + 12
1000 ´ 22
= 32 M
4. (c) C 6. (b) From a rectangular sheet of cardboard of size 5 × 2
cm2, a cirle of radius 1 cm, can be cut-off.

R a
D C
a S
O
2 cm

30º Q
A D B A 5 cm B
a
Area of rectangular sheet = 5 × 2 = 10 cm2
In DAOD, and area of circle = p(1)2 = p cm2
OD 1 \ Required area = Area of sheet – Area of circle
tan 30º = = = (10 – p) cm2
AD 3
EBD_7367
234
A- Area and Perimeter

1 10. (a) As we know that, if the length of square and rhombus


7. (d) Area of DAOB = ´ OA ´ OB are same, then the area should be same
2 \ x:y=1:1
11. (a) Since, area of circle = 4p cm2 (given)
Þ pr2 = 4p Þ r = 2 cm
C
O
m

20
c 90º
20

cm
A B
O
C
30º 2 cm
A 30º B
1 D
= ´ 20 ´ 20 = 200 cm2
2
and area of sector OACBO OD
In DOAD, tan 30º = Þ AD = 2 3 cm
AD
pr 2 q 3.14 ´ 20 ´ 20 ´ 90º
= = Now, AB = 2 AD = 4 3 cm
360º 360º
\ Area of equilateral DABC
3.14 ´ 400
= = 314 cm2 3 3
4 = (AB) 2 = (4 3) 2
\ Area of minor segment 4 4
= Area of sector OACBO = 12 3 cm2
– Area of DAOB 12. (a) In DABC,
= 314 – 200 = 114 cm2
8. (b) Q Angle made in 60 min by minute hand of a clock A
= 360º D E
and angle made in 15 min by minute hand of a clock
x y
360º
= ´ 15º = 90º B C
60º
\ Required distance BC = x2 + y 2
2 p(14)90º 22 14 ´ 2
= = ´ = 22 cm 1 1
360º 7 4 \ Area of DABC = ´ x ´ y = xy
2 2
x
9. (c) Let the length of rectangle = p( x 2 + y 2 )
2 Area of semi-circle BACB =
4
\ Area of shaded portion = Semi-circle ABDA
x
A B + Area of semi-circle AECA
– (Area of semi-circle BACB – Area of DABC)
px 2 py 2 æ x2 + y 2 ö
= + - pç ÷ + Area of DABC
4 4 è 4 ø
x/6 F = Area of DABC

D x/2 E C 13. (d) In DABC,


AC2 = 282 + 212 = 784 + 441 = 1225
Here, ABCD is a square and DEFG is a rectangle
Where length = x/2 Þ AC = 35 cm
breadth = x/6
A
x x x2
Therefore, Area of rectangle = ´ = D E
2 6 12
and area of square = x2
28 cm

Hence, area of remaining portion


x 2 11x 2
= x2 - =
12 12 B C
21 cm
Area and Perimeter 235
A-

Area of shaded portion = Area of semi-circle ACE 18. (c) Let the side of an square be a cm.
+ Area of DABC – Area of quadrant circle BCD By given condition,
pr 2 1 p 1
= + ´ BC ´ BA - ´ r12 Area of square – Area of an equilateral triangle =
2 2 4 4
22 1 35 1 22 3 2 1 æ 3ö 1
= ´ ´ ´ ´ 21 ´ 28 - ´ 21 ´ 21 Þ a2 - a = Þ a çç1 -
2
÷=
7 2 2 2 7´4 4 4 è 4 ÷ø 4
5 ´11 ´ 35 1
= + (21 ´ 28 - 33 ´ 21) 1
a 2 (4 - 3) = 1 Þ a =
2
4 2 Þ
4- 3
1925 1
= + (-105) = 481.25 - 52.50 = 428.75 cm2 \ a = (4 - 3) cm -1/ 2
4 2
14. (b) Since, the distance covered by a man diagonally is 19. (d) Suppose a pole is fixed at a point C.
\ Area of field in which the horse can graze
3 ´ 100 = Area of curve CFE
d= ´ 1 = 50 m
60
50 m
1 2 1 D F 20 m C
\ Area of field = d = ´ (50)2 = 1250 m2
2 2

20 m
15. (a) Here width of sheet is 20 cm, which is the maximum
diameter of the circular sheet.

50 m
50 m E

O
A B
\ Remaining area of sheet = Area of rectangle sheet
1 3.14 ´ 20 ´ 20
– Area of circular sheet = (pr 2 ) = = 314 m2
= 25 × 20 – p(10)2 = 500 – 314 = 186 cm2 4 4
16. (b) Let the other sides of a right isosceles triangle be a
cm. 20. (a) Circumference of circle = 2p × 42
In DABC, 22
= 2´ ´ 42 = 264 cm
C 7
Perimeter of square = 4x Þ 264 = 4x
6 2 cm x = 66 cm
a
pr 2
21. (a) Area of 2 bigger semi-cirles = 2 ´
A B 2
a
2
æ 0.5 ö 1 0.25p
a +a = 6 2
2 2 = 2p ç ÷ ´ = cm2
è 2 ø 2 4
Þ a 2 = 6 2 and area of 5 smaller semi-circles
Þ a = 6 cm 2
5pr 2 1 æ 0.5 ö
1 2 = = 5´p´ ´ç ÷
\ Area of DABC = ´a 2 2 è 4 ø
2
5p 0.25 1.25p
1 = ´ = cm2
= ´ 6 ´ 6 = 18 cm2 2 16 32
2
Area of rectangle ABCD = 2 × 0.5 = 1 cm2
17. (b) Given A = 9 cm, b = 10 cm and c = 11 cm
0.25p 1.25p
9 + 10 + 11 Area of remaining portion = 1 - -
Q s= = 15 cm 4 32
2
p 5p
\ A= s( s - a)(s - b)( s - c) = 1- -
16 128
= 15(15 - 9)(15 - 10)(15 - 11)
128 - 8p - 5p
= 15 ´ 6 ´ 5 ´ 4 = 30 2 =
128
= 42.3 cm2
128 - 13p
= cm2
128
EBD_7367
236
A- Area and Perimeter

22. (a) Given, OS = 6 cm


26. (b) F
\ PQ = QR = RS = 4 cm
\ Perimeter of shaded region
A 2r B
= Perimeter of semi-circle PTS
O 2r
+ Perimeter of semi-circle QUS
+ Perimeter of semi-circle PVQ
C
= p(6) + p(4) + p(2) = 12p cm
23. (b) Q Circumference of circular path = 2p × 50 m
= 10000p cm Let O be the centre of a larger circular lamina. Three
and circumference of wheel = 2p × 50 = 100p cm circular lamina of same radius are cut out from a larger
lamina. Here ABC is equilateral triangle. Radius of larger
\ Distance covered in 60 min = 10000p cm
lamina = OA + OF.
10000 Let r be radius of smaller circle.
Distance covered in 1 min = p
60 2r
OA = , OF = r
Distance covered in 15 min 9
10000 2r
= p ´ 15 = 2500 p cm Radius of larger lamina = +r
60 3
2500 p r (2 + 3)
\ Number of revolutions = = 25 =
100p 3
Area of 3 laminas = 3pr2
24. (b) A D
2
æ r (2 + 3) ö
E Area of larger lamina = p ç ÷÷
ç 3
è ø

pr 2 (7 + 5)
Area of larger lamina =
F 3
Area of Resideal part of larger lamina
B C
pr 2 (7 + 4 5)
The above figure is symmetrical about BD = - 3pr 2
3
Area of shaded part
= 2 × Area of BEDB
=
(
pr 2 7 + 4 5 - 9 )
= 2 × (Area of BC DEB – Area of DBCD) 3
æ pr 12
ö pr 2 (4 3 - 2)
= 2ç - ´ BC ´ CD ÷ =
è 4 2 ø 3

=
æ 22

1 ö
7´ 7 - ´ 7´7÷ (4 5 -2 ) pr 2
è 7 ´ 4 2 ø Required Ratio =
3
28 (7 + 4 5) 2
= 2´ = 28 cm2 pr
2 3
25. (a) Let sides of a rectangle be l and b.
4 3-2 7-4 5
Then, 2(l + b) = 18 Þ l + b = 9 = ´
\ Area of rectangle = l × b 7+4 5 7-4 5
For maximum, area of rectangle, l = b = 28 5 - 14 - 16 ´ 3 + 8 5
\ 2l = 9 Þ l = 4.5 = 36 3 - 62
\ Maximum area of rectangle = l × b = (4.5)2 = 36 × 1.732 – 62
= 20.25 cm2 = 0.352 = 0.35
Area and Perimeter A-237

30. (b) Width of the border = x m


27. (c)
D C
O
H xm G
D r r E
a/2 a/2 3m xm x
a/2 a/2 a/2 a/2
A P M R B E F
2a xm
A B
a 4m
Here, AP = PM = MR = RB =
2
Given, area of EFGH = 6 m2
It is clear from figure that, Þ (4 – 2x) (3 – 2x) = 6
a Þ 12 – 8x – 6x + 4x2 = 6
OP = OR = r + Þ 4x2 – 14x + 12 = 6
2
2 2 Þ 4x2 – 14x + 6 = 0
In DOPR, OP2 = PM + OM Þ 2x2 – 7x + 3 = 0
æ aö æaö
2 2 Þ 2x2 – 6x – x + 3 = 0
ç r + ÷ = ç ÷ + (a - r )
2
Þ Þ 2x(x – 3) – 1 (x – 3) = 0
è 2ø è2ø Þ (x – 3) (2x – 1) = 0
a2 a2 1
Þ r2 + + ra = + a 2 + r 2 - 2ar \ x = 3,
4 4 2
Þ ra = a2 – 2ar 1
Þ a2 – 2ar – ar = 0 \ x= = 0.5 m (Q x ¹ 3)
2
Þ a2 – 3ar = 0 31. (b) Q Inner radius, r1 = 25 cm
Þ a2 = 3ar
Þ a = 3r 0
+1
=r
1
a r2
\r = r1 = 25
3
28. (b) Let r be the radius of circle and a be the side of square.
By given condition,
2pr = 4a
External radius, r2 = 25 + 10 = 35 cm
pr Distance covered in 1 revolution
Þ a=
2 22
2 = 2p × 35 = 70 × = 220 cm
æ pr ö p 2 r 2 9.86 r 2 7
\ Area of square = ç ÷ = = = 2.46r 2
è 2 ø 4 4 Distance covered in 10 revolutions = 2200 cm
and area of circle = pr 2 = 3.14r 2 Covered distance 2200
\ Speed of bicycle = = cm/s
Hence, area of the circle is larger than that of square. Time 5
29. (d) Let side of triangle a, b, c are 35 cm, 44 cm and 75 22
respectively. = m/s = 4.4 m/s
5
a + b + c 35 + 44 + 75 32. (c) Length of wire = 36 cm
\ s= = = 77
2 2 \ Perimeter of semi-circle = pr + 2r
Area of triangle æ 22 ö
Þ 36 = r ç + 2 ÷
D = s ( s - a)( s - b)( s - c) = 77 ´ 42 ´ 33 ´ 2 è 7 ø
= 7 ´11´ 2 ´ 3 ´ 7 ´ 3 ´11´ 2 36 ´ 7
Þ r= = 7 cm
= 7 × 11 × 2 × 3 = 462 cm2 36
\ Radius of semi-circle = 7 cm
D 462 33. (b) Here, each circle has radius 4 cm.
radius of incircle = = = 6 cm
s 77 Then sides of triangle CA=AB = BC = 4 + 4 = 8 cm.
EBD_7367
238
A- Area and Perimeter

C A

m
c = 50 m

78
b=
h

B B C
a = 112 m

and also, D = s( s - a)(s - b)( s - c)


As, all sides of triangle are equal then triangle formed
a equilateral triangle and in which each angle is 60º. \ = 120(120 - 112)(120 - 78)(120 - 50)
Area of shaded part or the area of portion included = 120 ´ 8 ´ 42 ´ 70 = 1680 m2
between circles
Q Area of triangle
= Area of triangle – Area of 3 sectors of circle.
1
3 2 3 ´ pr 2 = × Base × Height
= a - ´ 60 2
4 360
1
Þ 1680 = ´ 50 ´ h
3 2 60 2
= (8) - 3 ´ p(4)2
4 360 2 ´ 1680
\ h= = 67.2m
50
= (16 3 - 8p) cm2
36. (b) Let the sides of isosceles triangle be 5x, 5x and 3x cm
34. (a) Area of rhombus respectively
1 1 According to question,
= ´ d1 ´ d 2 = ´ 55 ´ 48 = 1320 cm2 Perimeter of isosceles triangle = Length of wire
2 2
5x + 5x + 3x = 78 Þ 13x = 78
A B Þ x = 6 cm
\ Length of base = 3 × 6 = 18 cm
O 37. (b) The angle made by the minute hand in 20 min = 120º
p
\ The area swept by the minute hand in 20 min
q 120º
D C = ´ pr 2 = × 3.14 × 9 × 9 = 84.78 cm2
E 360º 360º
\ Area of rhombus = Base × Height = DC × AE
38. (c) Outer diameter = 112 cm and inner diameter = 70 cm
Þ DC × AE = 1320 Þ p × OD2 + OC2 = 1320
5 2
æ 55 ö æ 48 ö
Þ p ´ ç ÷ + ç ÷ = 1320
è 2 ø è 2 ø 70 cm112cm
æ 1 ö
çQ OD = 2 BD ÷
ç ÷
ç and DC = 1 AC ÷
ç ÷
è 2 ø
Area of parapet
5329 1 1
Þ p´ = 1320 a= p(1122 - 702 ) = (12544 - 4900)p
4 4 4

1320 1 22 1
= 36.16 = ´ 7644 ´ = ´ 24024 = 6006 cm2
Þ p= 4 7
36.5 4
39. (d) Let the length of altitude AB = h
36 cm < p < 37 cm
According to question
35. (c) Given, 2s = 240 Þ s = 120 and c = 50m, b = 78 m, a = Area of DABC = Area of square
112m
1
1 \ × Base × Altitude = (Side length)2
\ Area of triangle = × Base × Height 2
2
Area and Perimeter 239
A-

1
Þ ´ 9 ´ h = 36
2 D C
36 ´ 2
Þ h=
9 O 12 cm
\ h = 8 cm
A B
3 2 16 cm
40. (a) Area of equilateral triangle = a
4
Using Pythagoras theorem
A In ABC, AC2 = AB2 + BC2
AC2 = (16)2 + (12)2 = 256 + 144 = 400
AC
AC = 20 cm, AO =
a 2
a
\ AO = 10 cm (radius)
h Area of circumcircle
= pr 2 = p ´ (10) 2 = 100 p cm2
B C
a 43. (c) In DABC, Use Pythagorias theorem,
BC = AB2 + AC2 = a 2 + b 2
1 3 2
Þ ´a´h = a
2 4
3 2
Þ h= a
2 C
3 2 a2 + b2
Þ 2 3 = a
2 b
Þ a2 = 2 × 2 = 4
a = 2 cm B
A a
Area of equilateral triangle
3 2
= a
4
\ Required total area
3 2 3 2
= (a) = (2)
( ) + 12 ab
2
4 4 = a 2 + b2 + a 2 + b2
= 3 cm2
41. (a) \ Required area of the path, = 2( a 2 + b 2 ) + 0.5ab
EF = 30 + 4 = 34 m, GF = 16 + 4 = 20 m
1
44. (b) Q Area of trapezium = (3 + 5) ´ 4 = 16 m2
H G 2
2m \ Total cost of painting ` 25 per sq m = 16 × 25
D C = ` 400
45. (c) Suppose a horse is tied at vertex A. Then, area available
2m 2m grazing field is ADE.
16 m
A
A B
m

2 m 30 m 60º
4.2

E F D E
\ Area of path = Area of EFGH – Area of ABCD
= 34 × 20 – 30 × 16 = 680 – 480 = 200 m2
42. (c) Diagonal of rectangle passes through its. centre So, 60º 60º
B C
radius of circle 6m
AC Now, area of curve
(r) =
2 pr 2 q 22 ´ (4.2) 2 ´ 60º
ADE = = = 9.24 m2
360º 7 ´ 360º
EBD_7367
240
A- Area and Perimeter

3 Þ (15 2)2 = 2x2 Þ 2x2 = 225 × 2


and area of equilateral D ABC = (side)2
4 Þ x2 = 225 Þ x = 15 cm
So, length of the side of the square = 15 cm.
3 49. (b) Here, DABC forms an equilateral triangle.
= ´ (6)2 = 15.57
4 A
\ Required percentage
x/3 x/3
9.24
= ´ 100 = 59.34% = 59% (approx)
15.57 x/3
46. (a) According to question G F

A1 pr1
2
1 x/3 x/3 x/3
= = x/3
A 2 pr2 2
2

2
æ r1 ö 1 x/3
Þ ç ÷ = ...(i) x/3 x/3 x/3
r
è 2ø 2
As, Circles are bent in the form of square then their
perimeter will become equal. B D E x/3 C
x/3 x/3
2pr1 = 4a1
pr1
Þ a1 =
2
where, AGFH form a rhombus and DHDE is also an
pr2 equilateral triangle.
Similarly, a2 =
2 \ Area of rhombus
2
= (Area of DAGF + Area of DGFH)
æ pr1 ö 2 2 2
A1 a 2 ç 2 ÷ 2 3æxö 3æxö 3æ xö
= 1 è ø = r1 = 1 = ç ÷ + ç ÷ = 2´ ç ÷
\ = 2 2 [from Eq. (i)] 4 è3ø 4 è3ø 4 è3ø
A2 a 2 æ pr2 ö r22 2
ç ÷ 2
è 2 ø 3æxö
Now, area of DHDE = ç ÷
47. (b) Let width of the rectangle = x unit, length of the 4 è 3ø
rectangle = (2x + 5) unit
\ Area of rectangle = (2x + 5) × x 3 2
and area of DABC = x
Þ 75 = 2x2 + 5x 4
Þ 2x2 + 5x – 75 = 0 By given condition,
Þ 2x2 + 15x – 10x – 75 = 0
Þ x(2x + 15) – 5(2x + 15) = 0 Area of rhombus AGHF + Area of D HDE
Þ (x – 5) (2x + 15) = 0 Area of D ABC
Þ x = 5 units
2 2
15 3æxö 3æ xö
2´ ç ÷ + ç ÷
x= - 4 è3ø 4 è3ø
2
x cannot be negative = 3 2
x
So, width of the rectangle = 5 units 4
48. (b) Let the sides of a square = x cm.
In DABC, AC2 = AB2 + BC2 3æxö
2

3´ ç ÷
4 è3ø 3 1
= =
D C = 3 2 9 3
x
4
O 50. (c) Area of the circle = Area of the square = (Side)2
m

x
2c

pr2 = (2 p ) 2 Þ pr 2 = 4p
15

4p
Þ r2 = =4
A x B p
\ r = 4 = 2 units
Use Pythagoras theorem \ Diameter of circle (d) = 2 × r = 2 × 2 = 4 units
AC2 = x2 + x2
Area and Perimeter 241
A-

51. (c) Let the radius of circle is r and the side of a square is
a, then by given condition. 3
Þ 3 = a
2
pr
2pr = 4a Þ a = a = 2 cm
2 Perimeter of triangle = 3 × a = 3 × 2 = 6 cm.
\ Area of square 54. (d) Given that,
2
æ pr ö pr 2 2 2
9.86 r
= ç ÷ = = = 2.46r 2 E
è ø2 4 4
and area of circle = pr2 = 3.14r2
and let the side of equilateral triangle is x.
Then, by given condition, F
2pr
3x = 2pr Þ x = 2a
3 D C
\ Area of equilateral triangle
3 2 3 4 p2 r 2 a
= x = ´
4 4 9
A B
p2
= r = 1.89r
2 2
Area of rectangle = 2a2 = l × b
3 3 Þ l × b = 2a2 = l × a Þ l = 2a
Hence, Area of circle > Area of square Now, in DACD,
> Area of equilateral triangle AC2 = AD2 + CD2 = a2 + 4a2 = 5a2
52. (a) Let the width of the rectangle = x unit
\ AC = 5a unit
Therefore, Length = (2 x + 5) unit
According to the question, Hence, area of square
Area = x (2x + 5) Þ 75 = 2x2 + 5x = (a 5)2
Þ 2x2 + 5x – 75 = 0 = 5a2 sq units
Þ 2x2 + 15x – 10x – 75 = 0 55. (c) I. We know that, Area of segment (PRQP)
Þ x (2x + 15) – 5(2x + 15) = 0 = Area of sector (OPRQO) – Area of DOPQ
Þ (2x + 15) (x – 5) = 0
pr 2 q 1 2
-15 = - r sin q
Þ x = 5 and 360 2
2
Since, width cannot be negative S
\ width = 5 units and length = 2x + 5
= 2 × 5 + 5 = 15 units
\ Perimeter of the rectangle = 2 (15 + 5) = 40 units O
3 2 q
53. (c) Area of equiateral triangle = a
4
P Q
A R
So, the area of a segment of a circle is always less than
area of its corresponding sector.
a II. Distance travelled by a circular wheel of diameter
a
(2d )
h 2 d cm in one revolution = 2p = 2 × 3.14 × d
2
= 6.28 d
B C which is greater than 6d cm.
a
Therefore, statement I and II both are correct.
According to Question 56. (b) Perimeter of a rectangle = 82 m
\ 2(Length + Breadth) = 82 m
1 3 Þ Length + Breadth = 41 m
Þ ´ a ´ h = ´a2
2 4 Þ l + b = 41 m ...(i)
Also, its area = 400 m2
3 Þ l × b = 400 ...(ii)
Þ h= a éQ h = 3 ù
2 ë û Now, (l – b)2 = (l + b)2 – 4lb
= (41)2 – 4 (400)
EBD_7367
242
A- Area and Perimeter

= 1681 – 1600 = 81 60. (b)


\ l–b=9 ...(iii) A
From Eqs. (i) and (iii)
2l = 50 Þ l = 25 m and b = 16 m
\ Required breadth (b) = 16 m a
O
57. (c) Given that, ratio of their radii = 5 : 3
i.e. r1 : r2 = 5 : 3 30
30
B P C
A
r2 Here OP is radius of circle and given triangle is
equilateral.
B
O r1 a
\ BP =
2
In DBOP,
r1 5 OP
Þ = ...(i) tan 30º =
r2 3 BP
Let r1 = 5x and r2 = 3x OP = BP tan 30
Also, given that, area enclosed between the a 1 a
circumferences of two concentric circles = 16p cm2 = ´ = = radius.
2 3 2 3
\ p( r12 - r22 ) = 16p Now, Area of circle = pr2
Þ (5x)2 – (3x)2 = 16 2
Þ 25x2 – 9x2 = 16 æ a ö
154 = p´ ç ÷
Þ 16x2 = 16 è2 3ø
Þ x2 = 1 a2
Þ x=1 Þ 154 = p
12
\ r1 = 5 and r2 = 3
22 a 2
\ Area of the outer circle = pr12 = p(5) 2 = 25 p cm2 Þ 154 = ´
7 12
58. (c) Let the radius of a circle is r and a be the length of the
side of a square. a= 7 ´ 7 ´12
Given, circumference of a circle = Perimeter of a
= 7 × 2 3 = 14 3
square
Perimeter of triangle = 3
Þ 2pr = 4a
= 3 ´ 14 3 = 42 3
p
Þ a= r = 1.57r 61. (c) Given that, CD = 20 cm and area of rectangle ABCD
2
= 100 m2
Now, area of the circle (Ac) = pr2 = 3.14r2
and are of the square (As) = a2 = 2.4649r2 A P B
\ Area of circle > Area of square
59. (b) Let the radii of two circles are r1 and r2, respectively.
Circumference of Ist circle 2 D Q C
Given, =
Circumference of IInd circle 3
Þ AD × CD = 100 cm2 Þ AD × 20 = 100
2pr1 2 r 2
Þ = Þ 1 = æ In the given rectangle, ö
2pr2 3 r2 3 ç ÷
AB = CD = 20 cm and ÷
\ AD = 5 cm ç
2 ç AD = BC = PQ = 5 cm ÷
æ r1 ö 4 è ø
Þ ç ÷ = ...(i)
r
è 2ø 9 1 1
\ Area of DPDC = ´ PQ ´ CD = ´ 5 ´ 20
2 2 2
Area of Ist circle pr ær ö
2
4 = 5 × 10 = 50 cm2
\ = 12 = ç 1 ÷ =
Area of IInd circle pr2 è r2 ø 9
Area and Perimeter 243
A-

62. (d) Let AB = AC = a = 2 (Length + Breadth)


\ BC2 = AB2 + AC2 (by Pythagoras theorem) = 2 (90 + 50) = 2 × 140 = 280 m
65. (c) Given, radius of a circle = 8 cm
A and diameter of circle = 16 cm

A a D
a a
a a
q q
B D C
B a C
In DABC, = a2 + a2 = 2a2 Þ BC = a 2
90º + q + q = 180º \ Length of diagonal of a square = Diameter of a
(since, sum of all interior angles of any triangle is 180º) circle
Þ 2q = 90º Þ a 2 = 16
\ q = 45º
Now, in DABD, \ a = 8 2 cm
\ Area of square ABCD
AD a
sin 45º = Þ AD = = a2 = (8 2) 2
a 2
= 64 × 2 = 128 sq cm
1 66. (d) Given that, length of hour hand = 4 cm
\ Area of DABC = ´ AD ´ BC
2 and length of minute hand = 6 cm
\ Complete Rotation of hour-hard in a day = 2 × 360º
1 a
= ´a 2´ = 1 sq unit (given) = 720°
2 2 \ Rotation of hour-hand in 2 days = 2 × 720º
a2 p
Þ =1 = 1440 × rad
2 180
\ a= Similarly,
2
Rotation of Minute hand in a day = 24 × 360º
\ Perimeter of DABC = 2a + a 2 = 2 2 + 2 ´ 2 \ Rotation of minute hand in 3 days
= 2(1 + 2) units p
= 72 × 360º × rad
63. (c) Area of path = Area of (fountain + path) – Area of fountain 180
\ Distance travelled by hour hand
m
1.5 p
= 4 × 1440º × = 32 p
180º
m and distance travelled by minute hand
3.3
r= p
m = 6 × 72º × 360º × = 6 × 144p cm
1.5 180
\ Required ratio = 32p : 6 × 144p = 1 : 27
= p (3.3 + 1.5)2 – p (3.3)2 = [(4.8)2 – (3.3)2] p
q
= (23.04 – 10.89) p = 12.15 p m2 67. (c) Arc of length = 2pr ´
64. (c) Let length and breadth of a rectangular field are 9x 360º
and 5x.
A D
A
O
5x q 16
10
B 9x C B
\ Area of a rectangular field = 4500 m2
Þ 9x × 5x = 4500 q
Þ 16 = 2pr ´
Þ x2 = 100 = (10)2 360º
\ x = 10
So, the length and breadth of a rectangular field are 90 q 16
Þ =
m and 50 m. 360º 2 pr
\ Perimeter of rectangular field Now, area of sector OAB
EBD_7367
244
A- Area and Perimeter

q 3æ yö
2

= pr 2 ´ Þ ç ÷ =x
360º 4 è3ø
16
= pr 2 ´= 8r = 8 ´ 10 = 80 sq cm 3 y2
2pr Þ ´ =x
4 9
68. (a) Q Angle made in 60 min by minute hand of a clock
= 360º 36
and Angle made in 40 min by minute hand of a clock y2 = x
3
360º
= ´ 40 = 240º 36
60º y2 = x
3
2p(2.5) ´ 240º 10p
\ Required distance = = cm
360º 3 36
y2 = 3 x = 12 3 x
69. (b) Area of regular polygon 3
na 2 180º Squaring both sides
= cot y4 = 144 × 3 x2
4 n
here polygon is hexagon so n = 6 \ y4 = 432 x2
72. (c) Volume of mud dug out in two hemispherical pitholes
6 ´ a2 180º
Now, cot = 96 3
4 6 22 m

6a 2
Þ ´ cot 30º = 96 3
4 2m 2m 10 m
2
6a
Þ ´ 3 = 96 3
4
a2 = 64 2 3 2 22 3 2 ´ 2 ´ 22 ´ 8 704 3
= 2 ´ pr = 2 ´ ´ ´ 2 = = m
Qa = 8 3 3 7 21 21
Perimeter of a regular hexagon Area on which the mud is spread over
= 6 × side = 6 × 8 = 48 cm = Area of field – Area of pitholes
70. (b) Semi-perimeter of triangle
22 2
a+b+c 7 + 24 + 25 56 = l × b – 2 × p r2 = 22 × 10 – 2 × ´2
= = = = 28 cm 7
2 2 2
Area of circle = Area of triangle 176 1540 - 176 1364 2
= 220 - = = m
= s( s - a)(s - b)( s - c) 7 7 7
Now, let the rise in level by h m, then
= 28(28 - 7)(28 - 24)(28 - 25)
Area of remaining field × h = Volume of mud dugged out
= 28 ´ 21´ 4 ´ 3 = 7056 = 84 cm2
1364 704
71. (a) Let side of equilateral triangle = a Þ ´h=
7 21
A
704 ´ 7 16
\ h= = m
1364 ´ 21 93
a 1
73. (a) Area of the DABC = ´b´ h
2
A
B C

Perimeter = 3a = y a a
3 cm
y (h)
a= a
3
B C
3 2 D
b/2 b/2
Area of equilateral trangle = a b
4
Area and Perimeter 245
A-

1 78. (c) Area of parallelogram = Base × Height


Þ 12 = ´b´3 = 8.06 × 2.08 = 16.76 cm2
2
A B
12 ´ 2 79. (b)
\ b= = 8 cm O
3
b 8
Here, BD = CD = = = 4 cm D C
2 2
In right angled DABD, by Pythagoras theorem, BD 4.8 AC 1.4
Here, OD = = = 2.4; OC = = = 0.7
AB = BD + AD
2 2 2 2 2 2
Since, in rhombus diagonal bisect at 90°. Then, in DODC.
Þ a = 42 + 32 = 16 + 9 = 25 = 5 cm
OD2 + OC2 = CD2
Now, perimeter of an isosceles triangle
= 2a + b = 2 × 5 + 8 = 10 + 8 = 18 cm Þ CD = OD 2 + OC 2
74. (a) Let the breadth and height of room be b and h m,
respectively. = (2.4)2 + (0.7) 2
Then, according to the question, 5
Þ l × b = n × Area occupied by one patient CD = 6.25 Þ CD = 2.5 = 2
Þ 14 × b = 56 × 22
56 ´ 2.2 5
Þ b= = 8.8 m Þ Perimeter of rhombus = 4(side) = 4 ´ = 10 cm
14 2
Now, total volume of the room is equal to total patients 80. (c) From the figure it is clear that, 4 circular plates of
multiplied by volume occupied by each patient. diameter can be made of a Square plate of side 2d
Then, 14 × 8.8 × h = 8.8 × 56 with minimum loss of material.
8.8 ´ 56 D C
\ h= =4m
14 ´ 8.8
d
75. (a) As, 1 m = 1000 mm
\ 10 m = 10000 mm O O
Number of 200 mm lengths that can be out from 10 m
of ribbon
10000
= = 50 O O
200
76. (a) Area between square and semi-circles A B
= Area of square – 2x Area of semi-circle 2d
Let ABCD be square
5 cm Diameter of circle = d
d
10 cm Radius of circle =
2
5 cm Here from figure it is clear that side of the square is
equal to diameter of two circle
10 cm Side of square = d + d = 2d
= AB = BC = CD = DA
22 (5) 2 Therefore, perimeter of square = no. of circular plates
= (10)2 - 2x ´ = 100 - 78.5 = 21.5 cm2
7 2 × sum of diameter two circular plates
77. (a) Let l = 4x and b = 9x Þ (2d + 2d + 2d + 2d) no. of circular plates × 2d
\ Area of rectangle = l × b
8d
144 = 4x × 9x no. of circular plates = =4
2d
144
Þ x2 = Þ x2 = 4 81. (d) Here, DAOB, DBOC and DCOD are equilateral triangles.
36 \ Side = 2 cm
\ x=2 Total area of three equilateral triangles
l = 4 × 2 = 8 cm and b = 9 × 2 = 18 cm
\ Perimeter of rectangle 3
= 2(l + b) = 2(8 + 18) = 3´ (Side)2
4
= 2 × 26 = 52 cm
EBD_7367
246
A- Area and Perimeter

B C BC2 = AB2 + AC2 = 36 + 64 = 100 cm


2
p(3) 2 9 p
2 Now, area of semi-circle x = = cm2
2 2 2
2 2
\ BC = 10 cm.
A 2 O 2 D 16 p
Area of semi-circle y = cm2
2
3
= 3´ ´ 4 = 3 3 cm2 25p
4 Area of semi-circle z = cm2
2
82. (d) Area of sector = 72 p cm2
æ 9p 16p ö 25p
Now, value of x + y – z = ç + ÷- =0
è 2 2 ø 2
cm 85. (c) Sides of equelateral triangle are follows:
36
q 3 3 3
3, , , ... so on
2 4 8
These sequence formed a GP serves.
So sum of GP for Infinite terms.
a
pr 2 q S=
Þ = 72 p 1- r
360º
1
72 ´ 360 Here a = 3, r =
\ q= = 20º 2
36 ´ 36
3
pr q p ´ 36 ´ 20 S=
Length of arc = = = 4p cm 1 = 6 units
180º 180 1-
2
83. (b) S R 86. (c) In DAOB, AO = OB = r
D C

2a O
A B 90º
P Q 5
A B
For inscribed square.
C Minor
Diameter of circle = Diagonal of square segment
using Pythagoras theorem,
Þ AB2 + BC2 = AC2 Using Pythagoras theorem,
Þ 2AB2 = (2a)2 [\ AB = BC] AB2 = OA2 + OB2 Þ (5)2 = r2 + r2
2AB2 = 4a2 25
\ AB2 = 2a2 \ r2 = cm
2
Q AB = 2a Now, area of sector
Area of square ABCD = ( 2a )2 = 2a2 q 90º 25 25p
For circumscribed circle, AOB = ´ p r2 = ´ p´ = cm2
360º 360º 2 8
Diameter of circle = Side of square = 2a
Now, area of minor segment
Area of square PQ RS = (2a)2 = 4a2
Difference between area of outer square and inner square = Area of sector – Area of triangle
= 4a2 – 2a2 = 2a2 25p r 2 25p 25 æ 25p - 50 ö
= - = - =
84. (c) In DABC, by Pythagoras theorem, 8 2 8 4 çè 8 ÷
ø
C Area of major segment = Area of circle – Area of minor
z segment
10 æ 25p - 50 ö 25p (25p - 50)
y 8 = pr 2 - ç ÷= -
è 8 ø 2 8
A 6 B 100 p - 25p + 50 75p + 50
x = =
8 8
Area and Perimeter 247
A-

25 25 æ 3p ö
= (3p + 2) = ç + 1÷ cm2
8 4 è 2 ø
87. (c) ABCD be the rectangle inscribed in the circle of h
diameter 5 cm.

A B b
l
Areas of three adjacent faces are x, 4x and 9x sq unit
D C respectively. So lb = x
[Q area of rectangular face = length × breadth]
Similarly, bh = 4x and lh = 9x
\ Diameter = Diagonal of rectangle
Now, (lb) . (bh) . (lh) = (x) . (4x) . (9x)
Now, let x and y be the lengths and breadths of
rectangle, respectively. Þ (lbh)2 = 36x3
Now in DABD, Þ lbh = 36x3
AB2 + AD2 = (5)2 Þ x2 + y2 = 25 \ lbh = 6x3/2
Since, they form Pythagoras triplet,
Therefore, volume of cuboidal box = lbh
\ x = 4 and y = 3
= 6x3/2 cu unit
So, area of rectangle = 3 × 4 = 12 cm2
91. (a) Area of triangular field
Area of circle
88. (b) Required ratio = = s ( s – a)( s – b)( s – c )
Area of DACD
a + b + c 41 + 40 + 9
C s= = = 45m
2 2
r
Area = 45 ´ (45 – 41) ´ (45 – 40) ´ (45 – 9)
A r r B
O = 45 ´ 4 ´ 5 ´ 36
r
= 180 m2 = 1800000 cm2
D 1800000
Number of rose bed = = 2000.
pr 2 900
= =p
1 r2 23
´ 2r ´ r =
2 92. (c)
r1 22
89. (b) Here, Area of Rectangle lies between 40cm2 and 45cm2
Given that one side = 5cm. r2 – r1 23 – 22
Þ =
Area of Rectangle = 5 × second side r1 22
Now, If Area = 40cm2
then, 40 = 3 × second side r2 – r1 1
Þ =
Q second side = 8cm. r1 22
Again, If Area = 45cm2
5 1
45 = 3 × second side Þ =
r1 22
Q Second side = 9cm.
It means that second side varies between 8 cm to 9 cm. r1 = 110 m
Let diagonal = d Diameter of inner circle = 110 × 2 m = 220 m

Þ 82 + 52 < d < 52 + 92 93. (d) A B

Þ 64 + 25 < d < 25 + 81
Þ 87 < d < 106
Þ 81 < 89 < d < 106 < 121
Þ 81 < d < 121
Þ 9cm < d < 11cm.
90. (b) Let length, breadth and height of a cuboidal box be l, b
and h, respectively D C
EBD_7367
248
A- Area and Perimeter

Let ABCD is square 100


Side of square (a) = 784 = 28 cm OC = OD =
2p
28 In DOCD
Diameter of single circle = = 14 cm ÐDOC = 90°
2
OC2 + OD2 = CD2
14 Þ CD2 = 2OC2
Radius of single circle = = 7 cm
2 2
æ100 ö÷
Circumference of each plate = 2pr Þ CD2 = 2 × ççç ÷
è 2p ø÷
22
= 2´ ´ 7 = 44 cm
7 100
Þ CD = 2 ´
94. (d) Diagonal of square = 12 2cm 2p
50 2
12 2 Þ CD = cm
Side of square = = 12cm 2p
2
So, option (c) is correct.
Perimeter of square = 4 × 12 = 48 cm 98. (b) Let the diameter be D.
Perimeter of equilateral triangle = 48 cm According to question-
48 452 × p.D = 2 km + 26 decameter
Side of the equilateral triangle = = 16cm 452 × p.D = (2000 + 260) metre
3
452 × p.D = 2260
3 2 2260 7
Area of triangle ´(16) ´
4 D=
452 22
3 35 13
= ´ 256 = =1 m
4 22 22
= 64 3cm2 So, option (b) is correct.
So, option (d) is correct. 99. (b) B
95. (b) Distance covered in one revolution
= 2p × 30 = 60p cm
Distance covered in one minute by 140 revolutions
= 140 × 60 p cm F E
Distance covered in one hour a
4
= 140 × 60 p × 60 cm = 15.82560 cm = 15.825 km
So, option (b) is correct. a
96. (b) 437 = 19 (23)
There are 19 rows with 23 trees in each row.
The distance between any two adjacent plants is 2m A
and the distance between any two adjacent rows is 2m. D C
\ The dimensions of the orchard are x
Length = [ 1 + 22 × 2 + 1] = 46m Let the side of square be ‘a’.
Breadth = [ 1 + 18 × 2 + 1] = 38m AD = ED = EF = FA = a
Perimeter = 2(46 + 38) = 168 m CD = x – a
Cost of fencing = 100 + 168 = ``16800 ED = y – a
as D CED and D CBA are similar
97. (c)
D C
CD ED
=
CA BA

O x -a a
Þ =
x y
Þ xy – ay = ax
A B Þ ax + ay = xy
xy
Þ a=
100 ( x + y)
Radius of circle =
2p
Area and Perimeter 249
A-

4xy 150
Perimeter of square 4a = x + y 103. (d) Given Area of shaded region = cm 2
( ) 847
So, option (b) is correct. We have to find radius of each disc.
100. (a) D 5k C P
A B
14 cm
Q R
A B
7k
Area of trapezium C D
1 S
= (AB + CD) × height
2 Since these discs are equal then their radius are also
1 equal.
336 = (5k + 7k) × 14 Þ AC = AB = BD = DC = Diameter of disc.
2
12k = 48 Let r be radius of disc.
k=4 Area of square ABCD = (AB)2 = (2r)2 = 4r2
Smaller parallel side = 5k Area of shaded region = Area of square – 4 (Area of
= 5 × 4 = 20 cm sector APQ)
101. (d) Total distance travelled = 4 km = 4000 m q
Distance travelled in one round. Area of sector APQ = ´ pr 2
360
= Circumference of the wheel
[\ ÐA = ÐB = ÐC = ÐD = 90°]
22 7
= 2pr = 2 ´ ´ = 2m 150 90 22 2
7 11 ´ 2 = 4r 2 - 4 ´ ´ ´ (r )
847 360 7
4000
\ Number of rounds in 4 km =
2 150 22 6r 2 150
= 4r 2 - r 2 Þ =
= 2000 rounds 847 7 7 847
\ Option (d) is correct.
102. (c) Let ABC be an isosceles triangle. 25 5
r2 = Þ r = cm
121 11
A \ Option (d) is correct.
104. (a) Let Radius of circle be r
170 Case I (Square)
170
side = x
x2 + x2 = (2r)2

B D C A B
300

150 r

1
Area = ´ AD ´ BC
2
1 D M C
= ´ (170 )2 - (150 )2 ´ 300
2
2x2 = 4 r 2
1 x = 2r
= ´ 28900 - 22500 ´ 300
2
Case II (equilateral triangle)
(... DADC is a right angled triangle then by pythagoras
theorem, we find AD) y cos30° = h
= 150 ´ 6400 = 150 × 80 = 12000 units. 3 éif h = 3ù
y´ =h êr = 2 ú
\ Option (c) is correct. 2 ë û
EBD_7367
250
A- Area and Perimeter

2 3 A
r= ´ y 109. (b)
3 2
F B
y o
r=
3 a

y 3 E C
=
2r 2
D
2 3r
y = = 3r let ABCDEF be a regular hexagon which can be
2 devided into 6 equilateral triangle
x 2r 2 3 2
= = as D = a
y 3r 3 4
105. (b) Let perimeter of square = 4x
then 2pr = 4x 3 2 3 3 2
as 6 × D = 6 × a = a
4 2
2x
r=
p A B
a
area of square = x2
a
2
æ 2x ö
area of circle = p ç ÷ F C
è p ø O
a
28 2
= p4x = x
22 E D
a
28 2
thus x is > x2
22 O
110. (d)
106. (a) Q triangle with side 12cm, 13cm and 5 cm is a right
triangle x x
1 1 h
Area = b × h = × 12 × 5 = 30
2 2 A B
107. (d) Perimeter P = 10 cm a
Area A = 4 cm2
2 (l + B) = 10 a a
B=5–l
l × B = 4 cm2
C D
l (5 – l) = 4 a
5l – l2 = 4
l2 – 5l + 4 = 0 Let the side of each side of the square be a and the
l2 – l – 4l + 4 = 0 other two sides of the triangle be x.
(l – 1) (l – 4) = 0 7
l = 4, l ¹ 1 Perimeter of the complete figure = perimeter of the
6
B=5–4=1 original square (given)
108. (b) R1 = 14 R2 = x
7
Area C1 P(14)2 3a + 2 x = (4a) Þ 9a + 6 x = 14a Þ 6 x = 5a ...(1)
= 6
Area C2 Px 2 (by given and Pythagoras theorem) Height of the tri-
angle,
49 (14)2
= 2 2 2
16 x æ aö a2 æ 5a ö a2
h= x2 - ç ÷ = x2 - = ç ÷ -
è 2ø 4 è 6ø 4
14 ´ 14 ´ 16
x2 = = 82
49
25a 2 a 2 25a 2 - 9a 2
R2 = 8 = - =
36 4 36
Area and Perimeter 251
A-

Þ a2 + 16a + 64 = a2 + 120
16a 2 4a 2a Þ 16a = 120 – 64
= = =
36 6 3
56
Now, ratio of the area of triangle to area of the original Þ 16a = 56 \ a= = 3.5 cm
16
1 2a
´a´ 2 113. (d) Area of the pathway = 10 × 4.5 = 45 m2
square = 2 3 = a ´ 1 = 1 = 1: 3
a2 3 a2 3 50 ´ 50
Area of each tile = = 0.25 m 2
111. (c) Calculating area of triangle using heron’s formula, we 100 ´ 100
get
45
Semi perimeter, Required number of tiles = = 180
0.25
a + b + c 51 + 37 + 20 108
s= = = = 54 100
2 2 2 Cost of 1 tile = =5
20
Area of triangle = s( s - a)(s - b)( s - c)
Total cost = 180 × 5 = 900
= 54(54 - 51)(54 - 37)(54 - 20) 1
114. (b) Area of square = ´ d1 ´ d 2 = a 2
= 54 ´ 3 ´ 17 ´ 34 2
According to the question
= 3.3.3.2.3.17.17.2 = 3.3.2.17 = 306 sq. cm
112. (b) Let length of side of square be a cm 1
a2 = ´ 50
after increase side of square = (a + 8) cm 2
According to the question a2 = 25 \ a = 5 units
(a + 8)2 = a2 + 120
EBD_7367
252
A- Volume And Surface Area

C HA P T E R
VOLUME AND SURFACE AREA
20
1. If the diameter of a wire is decreased by 10%, by how much 9. What is the number of spherical balls of 2.5 mm diameter
per cent (approximately) will the length be increased to that can be obtained by melting a semi-circular disc of 8 cm
keep the volume constant? [2007-II] diameter and 2 cm thickness? [2008-I]
(a) 5% (b) 17% (a) 6144 (b) 3072
(c) 20% (d) 23% (c) 1536 (d) 768
2. A cone is inscribed in a hemisphere such that their bases 10. From a wooden cylindrical block, whose diameter is equal
are common. If C is the volume of the cone and H that of the to its height, a sphere of maximum possible volume is
hemisphere, then what is the value of C : H? [2007-II] carved out. What is the ratio of the volume of the utilised
(a) 1 : 2 (b) 2 : 3 wood to that of the wasted wood? [2008-I]
(c) 3 : 4 (d) 4 : 5 (a) 2 : 1 (b) 1 : 2
3. S1, S2 and S3 are three rectangular sheets of identical areas (c) 2 : 3 (d) 3 : 2
with their lengths in the ratio 1 : 2 : 3. If each is converted 11. A conical flask of base radius r and height h is full of milk.
into a right circular cylinder open at both ends by joining The milk is now poured into a cylinderical flask of radius
its shorter parallel sides, then what is the ratio of the volumes 2r. What is the height to which the milk will rise in the
of the three cylinders, S1, S2 and S3, respectively, so formed? flask? [2008-I]
[2007-II]
(a) 1 : 1 : 1 (b) 4 : 2 : 3 h h
(a) (b)
(c) 1 : 2 : 3 (d) 6 : 3 : 2 3 6
4. The base diameter of a right circular cylinder is 3 cm. There
h h
is a section making an angle of 30º with the cross-section. (c) (d)
What is its area? [2007-II] 9 12
12. From a solid cylinder whose height is 4 cm and radius 3 cm
9p 3 3p a conical cavity of height 4 cm and base radius 3 cm is
(a) sq cm (b) sq cm
4 2 hollowed out. What is the volume of the remaining solid?
[2008-I]
9p 9 3p (a) 9p cu cm (b) 15p cu cm
(c) sq cm (d) sq cm
8 8 (c) 21p cu cm (d) 24p cu cm
5. 27 drops of water form a big drop of water. If the radius of 13. The diameters of two right circular cones are equal. If their
each smaller drop is 0.2 cm, then what is the radius of the slant heights are in the ratio 3 : 2, then what is the ratio of
biggeer drop? [2007-II] their curved surface areas?
(a) 0.4 cm (b) 0.6 cm [2008-I]
(c) 0.8 cm (d) 1.0 cm (a) 9 : 4 (b)
6. The total surface area of a cone, whose stant height is 3: 2
equal to the radius R of its base, is S. If A is the area of a (c) 3 : 2 (d) 2 : 3
circle of radius 2R, then which one of the following is 14. In which one of the following pairs does the first number
correct? represent the perimeter of one face of a cube and the second
[2007-II] number represent the volume of the cube? [2008-I]
(a) A = S (b) A = 2S (a) (16, 32) (b) (20, 125)
(c) A = S/2 (d) A = 4S (c) (9, 27) (d) (10, 100)
7. An iron block is in the form of a cylinder of 1.5 m diameter 15. Three cubes of metal whose edges are 6 cm, 8 cm and 10
and 3.5 m length. The block is to be rolled into the form of cm, respectively are melted and a single cube is formed.
a bar, having a square section of side 5 cm. What will be What is the length of the edge of the newly formed cube?
the length of the bar? [2007-II] [2008-I]
(a) 2375 m (b) 2475 m (a) 10 cm (b) 12 cm
(c) 2575 m (d) 2600 m (c) 16 cm (d) None of these
8. If the circumference of the inner edge of a hemispherical 16. Consider the following
The length of a side of a cube is 1 cm. Which of the following
132
bowl is cm, then what is its capacity? [2008-I] can be the distance between any two vertices?
7
I. 1 cm II. 2 cm
(a) 12p cu cm (b) 18p cu cm
(c) 24p cu cm (d) 36p cu cm III. 3 cm
Volume And Surface Area A-253
Select the correct answer using the codes given below. 24. Assertion (A): The curved surface area of a right circular
[2008-I] con e of base radius r and height h is given by

( )
(a) Only I (b) Only II
(c) Only III (d) I, II and III pr h2 + r 2 .
17. A sphere is cut into two equal halves and both the halves Reason (R): The right circular cone of base radius r and
are painted from all the sides. The radius of the sphere is r height h, when cut opened along the slant height forms a
unit and the rate of painting is ` 8 per sq unit. What is the
total cost of painting the two halves of the sphere in rupees? rectangle of length pr and breadth h 2 + r 2 .
[2008-II] 25. Assertion (A): The volume of a cuboid is the product of
(a) 6pr2 (b) 32pr2 the lengths of its coterminous edges.
(c) 48pr2 (d) Insufficient data to answer Reason (R): The surface area of a cuboid is twice the sum
18. If the number of square centimetres on the surface area of of the products of lengths of its coterminous edges taken
a sphere is three times the number of cubic centimetres in two at a time. [2008-II]
its volume, then what is its diameter? [2008-II] 26 The radius and height of a right circular cone are in the
(a) 1 cm (b) 2 cm ratio 3 : 4 and its volume is 96p cm3. What is its lateral
(c) 3 cm (d) 6 cm surface area? [2008-II]
19. The ratio of the surface areas of two hemispheres is 4 : 1. (a) 24p cm2 (b) 36p cm2
What is the ratio of their volumes? [2008-II] (c) 48p cm2 (d) 60p cm2
(a) 8 : 1 (b) 4 : 1 27. From a solid cube of edge 3 m, a largest solid sphere is
(c) 3 : 1 (d) 2 : 1 curved out. What is the volume of solid left? [2008-II]
20. A container is in the form of a right circular cylinder (a) (27 – 2.25p) cu m (b) (27 – 4.5p) cu m
surmounted by a hemisphere of the same radius 15 cm as (c) 2.25p cu m (d) 4.5p cu m
the cylinder. If the volume of the container is 32400p cm3, 28. The dimensions of a rectangular box are in the ratio 2 : 3 : 4
then the height h of the container satisfies which one of and the difference between the cost of covering it with a
the following? [2008-II] sheet of some metal at ` 8 and ` 10 per sq m is ` 234. What
(a) 135 cm < h < 150 cm are the length, breadth and height of the box, respectively?
(b) 140 cm < h < 147 cm [2008-II]
(c) 145 cm < h < 148 cm (a) 2 m, 3 m, 4 m (b) 3 m, 4.5 m, 6 m
(d) 139 cm < h < 145 cm (c) 4 m, 6 m, 8 m (d) 5 m, 7.5 m, 10 m
21. A hollow cylindrical iron pipe of length 1.4 m has base 29. A solid metallic cube of edge 4 cm is melted and recast into
radius 2.5 cm and thickness of the metal is 1 cm. What is solid cubes of edge 1 cm. If x is the surface area of the
the volume of the iron used in the pipe? [2008-II] melted cube and y is the total surface area of all the cubes
(a) 2640 cu cm (b) 2604 cu cm recast, then what is x : y? [2008-II]
(a) 2 : 1 (b) 1 : 2
(c) 2460 cu cm (d) None of these
(c) 1 : 4 (d) 4 : 1
22. A right circular cone is cut by a plane parallel to its base in 30. What is the total surface area of a one-side open cubical
such a way that the slant heights of the original and the box of outer side of length 5 cm and thickness 0.5 cm?
smaller cone thus obtained are in the ratio 2 : 1. If V1 and V2 [2008-II]
are respectively the volumes of the original cone and of (a) 125 sq cm (b) 214 sq cm
the new cone, then what is V1 : V2? [2008-II] (c) 180 sq cm (d) None of these
(a) 2 : 1 (b) 3 : 1 31. A field is 125 m long and 15 m wide. A tank 10m × 7.5m × 6m
(c) 4 : 1 (d) 8 : 1 was dug in it and the Earth thus dug out was spread equally
23. If C1 is a right circular cone with base radius r1 cm and on the remaining field. The level of the field thus raised is
height h1 cm and C2 is a right circular cylinder with base equal to which one of the following? [2008-II]
radius r2 cm and height h2 cm and if r1 : r2 = 1 : n (where, n (a) 15 cm (b) 20 cm
is a positive integer) and their volumes are equal, then (c) 25 cm (d) 30 cm
which one of the following is correct? [2008-II] 32. A lead pencil is in the shape of a cylinder. The pencil is 21
(a) h1 = 3nh2 (b) h1 = 3n2h2 cm long with radius 0.4 cm and its lead is of radius 0.1 cm.
(c) h1 = 3 h2 (d) h1 = n2 h2 What is the volume of wood in the pencil? [2009-I]
Directions: The following two questions consists of two (a) 9 cu cm (b) 9.4 cu cm
statements, one labelled as the ‘Assertion (A)’ and the other as (c) 9.9 cu cm (d) 10.1 cu cm
‘Reason (R)’. You are to examine these two statements carefully 33. A cylindrical vessel of base radius 14 cm is filled with water
to some height. If a rectangular solid of dimensions
and select the answers to these items using the codes given
22 cm × 7 cm × 5 cm is immersed in it, what is the rise in
below. [2008-II] water level ? [2009-I]
Codes: (a) 0.5 cm (b) 1.0 cm
(a) Both A and R are individually true and R is the correct (c) 1.25 cm (d) 1.5 cm
explanation A 34. A cylindrical rod of length h is melted and cast into a cone
(b) Both A and R are individually true but R is not the of base radius twice that of the cylinder. What is the height
correct explanation of A of the cone? [2009-I]
(c) A is true but R is false (a) 3 h/4 (b) 4h/3
(d) A is false but R is true (c) 2h (d) h/2
EBD_7367
254
A- Volume And Surface Area
35. A roller of diameter 70 cm and length 2 m is rolling on the 47. The paint in a certain container is sufficient to paint an
ground. What is the area covered by the roller in 50 area equal to 5.875 m2. How many bricks of dimensions 12.5
revolutions? [2009-I] cm × 10 cm × 7.5 cm can be painted out of this container?
(a) 180 sq m (b) 200 sq m [2009-II]
(c) 220 sq m (d) 240 sq m (a) 225 (b) 180
36. The diameter of the Moon is approximately one-fourth of
that of the Earth. What is the (approximate) ratio of the (c) 150 (d) 100
volume of the Moon to that of Earth? [2009-I] 48. The inner and outer radii of a 7 m long hollow iron right
(a) 1/16 (b) 1/32 circular cylindrical pipe are 2 cm and 4 cm, respectively. If
(c) 1/48 (d) 1/64 1000 cm3 of iron weight 5 kg, then what is the weight of the
37. Three cubes each of side 5 cm are joined end to end. What pipe? [2010-I]
is the surface area of the resulting cuboid? [2009-I] (a) 264 kg (b) 132 kg
(a) 300 sq cm (b) 350 sq cm (c) 396 kg (d) None of these
(c) 375 sq cm (d) 400 sq cm 49. A sphere and a cube have same surface area. What is the
38. A solid cone of height 8 cm and base radius 6 cm is melted ratio of the square of volume of the sphere to the square of
and recast into identical cones, each of height 2 cm and
volume of the cube? [2010-I]
radius 1 cm. What is the number of cones formed?
[2009-II] (a) p : 6 (b) 1 : 1
(a) 36 (b) 72 (c) 6 : p (d) 3 : p
(c) 144 (d) 180 50. A hemisphere is made of a sheet of a metal 1 cm thick. If the
39. From a cylindrical log whose height is equal to its diameter, outer radius is 5 cm. What is the weight of the hemisphere
the greatest possible sphere has been taken out. What is (1 cm3 of the metal weight 9 g)? [2010-I]
the fraction of the original log which is cut away? (a) 54p g (b) 366p g
[2009-II] (c) 122p g (d) 108p g
(a) 1/2 (b) 1/3 51. A hemispherical bowl of internal radius 20 cm contains
(c) 1/4 (d) 2/3
40. If a sphere of radius 10 cm is intersected by a plane at a sauce. The sauce is to be filled in conical shaped bottles of
distance 8 cm from its centre, what is the radius of the radius 5 cm and height 8 cm. What is the number of bottles
curve of intersection of the plane and the sphere? required? [2010-I]
[2009-II] (a) 100 (b) 90
(a) 8 cm (b) 6 cm (c) 80 (d) 75
(c) 5 cm (d) 4 cm 52. Smaller lead shots are to be prepared by using the material
41. A cylindrical can of internal diameter 24 cm contains water. of a spherical lead shot of radius 1 cm. Some possibilities
A solid sphere of radius 6 cm is completely immersed in are listed in the statements given below [2010-I]
water in the cylinder. The water level increases by
I. The material is just sufficient to prepare 8 shots each
[2009-II]
(a) 0.25 cm (b) 0.5 cm of radius 0.5 cm.
(c) 2 cm (d) 3 cm II. A shot of radius 0.75 cm and a second shot of radius
42. A bucket which is in the form of a frustum of a cone, has 0.8 cm can be prepared from the available matereial.
radii 3 and 5 unit and vertical height 6 unit. How much Which of the above statements is/are correct?
water can the bucket hold? [2009-II] (a) Only I (b) Only II
(a) 33p cu unit (b) 45p cu unit (c) Both I and II (d) Neither I nor II
(c) 48p cu unit (d) None of these 53. A semicircular thin sheet of a metal of diameter 28 cm is
43. How many litres of water (approximately) can a bent and an open conical cup is made. What is the capacity
hemispherical container of radius 21 cm hold? [2009-II] of the cup? [2010-I]
(a) 19.4 L (b) 38.8 L
(c) 194 L (d) 388 L 1000
(a) 3 cu cm (b) 300 3 cu cm
44. How many litres of water flow out of a pipe having an area 3
of cross-section of 5 cm2 in one minute, if the speed of
700 1078
water in the pipe is 30 cm/s? [2009-II] (c) 3 cu cm (d) 3 cu cm
(a) 90 L (b) 15 L 3 3
(c) 9 L (d) 1.5 L 54. The volume of a cone is equal to that of a sphere. If the
45. A right DABC with sides 5 cm, 12 cm and 13 cm is revolved diameter of base of cone is equal to the diameter of the
about the side 12 cm. What is the volume of the solid so sphere, then what is the ratio of height of cone to the
obtained? [2009-II] diameter of the sphere? [2010-I]
(a) 50p cu cm (b) 100p cu cm (a) 2 : 1 (b) 1 : 2
(c) 125p cu cm (d) 150p cu cm (c) 3 : 1 (d) 4 : 1
46. The ratio of volumes of two cones is 4 : 5 and the ratio of 55. A cylinder having base of circumference 60 cm is rolling
the radii of their bases is 2 : 3. What is the ratio of their without sliding at a rate of 5 rounds per second. How much
vertical heights? [2009-II] distance will the cylinder roll in 5 s ? [2010-I]
(a) 5 : 6 (b) 6 : 5 (a) 15 m (b) 1.5 m
(c) 9 : 5 (d) 5 : 9 (c) 30 m (d) 3 m
Volume And Surface Area A-255
56. What is volume of the frustum of a cone with height h and (a) 9 m (b) 18 m
radii r1, r2? [2010-I] (c) 27 m (d) 30 m
65. The volume of a sphere is 8 times that of another sphere.
1 1
(a) ph ( r12 + r22 ) (b) ph ( r12 + r22 + r1r2 ) What is the ratio of their surface areas? [2010-II]
3 3 (a) 8 : 1 (b) 4 : 1
1 1 (c) 2 : 1 (d) 4 : 3
(c) ph ( r12 + r22 - r1r2 ) (d) ph ( r12 - r22 ) 66. What is the maximum length of rod that can be placed
3 3 inside a box having the shape of a cuboid of langth 30 cm,
57. A rectangular tank whose length and breadth are 2.5 m and breadth 24 cm and height 18 cm? [2010-II]
1.5 m, respectively is half full of water. If 750 L more water
is poured into the tank, then what is the height through (a) 30 cm (b) 30 2 cm
which water level further goes up? [2010-I] (c) 24 cm (d) 18 5 cm
(a) 20 cm (b) 18 cm
(c) 15 cm (d) 200 cm DIRECTIONS (Qs. 67-69) : Read the following information
58. The length, breadth and height of a r ectangular carefully to answer the questions that follow.
parallelopiped are in ratio 6 : 3 : 1. If the surface area of a Let C be a right circular cone. It is given that the two ends of a
cube is equal to the surface area of this parallelopiped, frustum of C are of radii 3 cm and 6 cm and the height of the
then what is the ratio of the volume of the cube to the frustum is 9 cm. [2010-II]
volume of the parallelopiped? [2010-I] 67. What is the total surface area of the given frustum?
(a) 1 : 1 (b) 5 : 4
(c) 7 : 5 (d) 3 : 2
(a) 9p(2 10 + 5) sq cm (b) 9p(3 10 + 5) sq cm
59. In order to fix an electric pole along a roadside, a pit with (c) 9p(3 10 + 4) sq cm (d) 27p( 10 + 1) sq cm
dimensions 50 cm × 50 cm is dug with the help of a spade.
The pit is prepared by removing Earth by 250 strokes of 68. What is the height of the cone?
spade. If one stroke of spade removes 500 cm3 of Earth, (a) 9 cm (b) 12 cm
then what is the depth of the pit? [2010-II] (c) 13.5 cm (d) 18 cm
(a) 2 m (b) 1 m 69. What is the slant height of the given frustum?
(c) 0.75 m (d) 0.5 m (a) 3 10 cm (b) 6 10 cm
60. If three cubic biscuits having edges 0.3 m, 0.4 m and 0.5 m (c) 12 cm (d) 15 cm
respectively are melted and formed into a single cubic
70. The diagonal of a cube is 4 3 cm. What is its volume?
biscuit, then what is the total surface area of the cubic
biscuit? [2010-II] [2011-I]
(a) 1.08 sq m (b) 1.56 sq m (a) 16 cu cm (b) 32 cu cm
(c) 1.84 sq m (d) 2.16 sq m (c) 64 cu cm (d) 192 cu cm
61. Half of a large cylindrical tank open at the top is filled with 71. A cylindrical rod of iron whose radius is one-fourth of its
water and identical heavy spherical balls are to be dropped height is melted and cast into spherical balls of the same
into the tank without spilling water out. If the radius and radius as that of the cylinder. What is the number of
the height of the tank are equal and each is four times the spherical balls? [2011-I]
radius of a ball, then what is the maximum number of balls (a) 2 (b) 3
that can be dropped? [2010-II] (c) 4 (d) 5
(a) 12 (b) 24 72. The outer and inner diameters of a circular pipe are 6 cm
(c) 36 (d) 48 and 4 cm, respectively. If its length is 10 cm, then what is
62. The diagonals of the three faces of a cuboid are x, y and z, the total surface area in sq cm? [2011-I]
respectively. What is the volume of the cuboid? [2010-II] (a) 35 p (b) 110 p
(c) 150 p (d) None of these
x yz 73. A cistern 6 m long and 4 m wide contains water to a depth
(a) of 1.25 m. What is the area of wetted surface? [2011-I]
2 2
(a) 40 sq m (b) 45 sq m
( y 2 + z 2 )( z 2 + x 2 )( x 2 + y 2 ) (c) 49 sq m (d) 73 sq m
(b) 74. A toy is in the form of a cone mounted on a hemisphere
2 2
such that the diameter of the base of the cone is equal to
( y 2 + z 2 - x 2 )( z 2 + x 2 - y 2 )( x 2 + y 2 - z 2 ) that of the hemisphere. If the diameter of the base of the
(c) cone is 6 cm and its height is 4 cm, what is the surface area
2 2 of the toy in sq cm? (take p = 3.14) [2011-I]
(d) None of the above (a) 93.62 (b) 103.62
63. A figure is formed by revolving a rectangular sheet of (c) 113.62 (d) 115.50
dimensions 7 cm × 4 cm about its length. What is the volume 75. What is the volume (in cu cm) of a spherical shell with 8 cm
of the figure, thus formed? [2010-II] and 10 cm as its internal and external diameters
(a) 352 cu cm (b) 296 cu cm respectively? [2011-I]
(c) 176 cu cm (d) 616 cu cm 61p 122p
64. A solid cylinder of height 9 m has its curved surface area (a) (b)
equal to one-third of the total surface area. What is the 3 3
radius of the base? [2010-II] 244 p 250 p
(c) (d)
3 3
EBD_7367
256
A- Volume And Surface Area
76. The curved surface of a cylinder is 1000 sq cm. A wire of 88. 10 cylindrical pillars of a building have to be painted. The
diameter 5 mm is wound around it, so as to cover it diameter of each pillar is 70 cm and the height is 4 m. What is
completely. What is the length of the wire used? [2011-I] the cost of painting at the rate of ` 5 per sq m? [2011-II]
(a) 22 m (b) 20 m (a) ` 400 (b) ` 440
(c) 18 m (d) None of these (c) ` 480 (d) ` 500
77. The material of a solid cone is converted into the shape of 89. What is the number of wax balls, each of radius 1 cm, that
a solid cylinder of equal radius. If the height of the cylinder can be molded out of a sphere of radius 8 cm? [2011-II]
is 5 cm, what is the height of the cone? [2011-I] (a) 256 (b) 512
(a) 15 cm (b) 20 cm (c) 768 (d) 1024
(c) 25 cm (d) 30 cm 90. What will be the cost to plaster the inner surface of a well
78. The surface area of a sphere is 616 sq cm. If its radius is 14 m deep and 4 m in diameter at the rate of ` 25 per sq m ?
changed so that the area gets reduced by 75%, then the [2012-I]
radius becomes [2011-II] (a) ` 4000 (b) ` 4200
(a) 1.6 cm (b) 2.3 cm (c) ` 4400 (d) ` 5400
(c) 2.5 cm (d) 3.5 cm 91. What is the length of the uniform wire of diameter 0.4 cm
79. A cylindrical vessel of height 10 cm has base radius 60 cm. that can be drawn from a solid sphere of radius 9 cm?
If d is the diameter of a spherical vessel of equal volume, [2012-I]
then what is d equal to? [2011-II] (a) 243 m (b) 240 m
(a) 30 cm (b) 60 cm (c) 60.75 cm (d) 60 m
(c) 90 cm (d) 120 cm 92. The total surface area of a cube is 150 sq cm. What is its
80. A sphere is inscribed in a cubical box such that the sphere volume? [2012-I]
is tangent to all six faces of the box. What is the ratio of the
(a) 64 cu cm (b) 81 cu cm
volume of the cubical box to the volume of sphere?
(c) 125 cu cm (d) 160 cu cm
[2011-II]
93. If the volume of a cube is 729 cu cm, then what is the length
(a) 6p (b) 36 p
of its diagonal? [2012-I]
(c) 4p/3 (d) 6/p
(a) 9 2 cm (b) 9 3 cm
81. If the diameter of a sphere is doubled, then how does its
(c) 18 cm (d) 18 3 cm
surface area change? [2011-II]
94. The curved surface area of a right circular cone of radius
(a) It increases two times
14 cm is 440 sq cm. What is the slant height of the cone?
(b) It increases three times
[2012-I]
(c) It increases four times
(a) 10 cm (b) 11 cm
(d) It increases eight times
(c) 12 cm (d) 13 cm
82. From a solid cylinder of height 4 cm and radius 3 cm, a 95. A cardboard sheet in the form of a circular sector of radius
conical cavity of height 4 cm and of base radius 3 cm is
30 cm and central angle 144º is folded to make a cone.
hollowed out. What is the total surface area of the remaining
What is the radius of the cone? [2012-I]
solid? [2011-II]
(a) 12 cm (b) 18 cm
(a) 15 p sq cm (b) 22 p sq cm
(c) 21 cm (d) None of these
(c) 33 p sq cm (d) 48 p sq cm
96. A large solid metallic cylinder whose radius and height are
83. A hollow sphere of internal and external diameters 4 cm
equal to each other is to be melted and 48 identical solid
and 8 cm respectively is melted into a cone of base diameter
balls are to be recast from the liquid metal, so formed. What
8 cm. What is the height of the cone? [2011-II]
is the ratio of the radius of a ball to the radius of the
(a) 12 cm (b) 14 cm
cylinder? [2012-I]
(c) 17 cm (d) 18 cm (a) 1 : 16 (b) 1 : 12
84. The radii of two cylinders are in the ratio 2 : 3 and their (c) 1 : 8 (d) 1 : 4
curved surface areas are in the ratio 5 : 3. What is the ratio 97. What are the dimensions (length, breadth and height,
of their volumes? [2011-II] respectively) of a cuboid with volume 720 cu cm, surface
(a) 20 : 27 (b) 10 : 9 area 484 sq cm and the area of the base 72 sq cm?
(c) 9 : 10 (d) 27 : 20 [2012-I]
85. A cylindrical tank 7 m in diameter, contains water to a depth (a) 9, 8 and 10 cm (b) 12, 6 and 10 cm
of 4 m. What is the total area of wetted surface? [2011-II] (c) 18, 4 and 10 cm (d) 30, 2 and 12 cm
(a) 110.5 sq m (b) 126.5 sq m 98. If the surface area of a sphere is 616 sq cm, then what is its
(c) 131.5 sq m (d) 136.5 sq m volume? [2012-I]
86. If S is the total surface area of a cube and V is its volume (a) 4312/3 cu cm (b) 4102/3 cu cm
then which one of the following is correct? [2011-II] (c) 1257 cu cm (d) 1023 cu cm
(a) V3 = 216 S2 (b) S3 = 216 V2
DIRECTIONS (Qs. 99-100) : The areas of the ends of a frustum of
(c) S3 = 6 V2 (d) S2 = 36 V3 a cone are P and Q, where P < Q and H is its thickness. [2012-I]
87. The radii of the circular ends of a bucket of height 40 cm
are of lengths 35 cm and 14 cm. What is the volume of the 99. What is the volume of the frustum?
bucket? [2011-II] (a) 3H(P + Q + PQ ) (b) H(P + Q + PQ )
(a) 60060 cu cm (b) 70040 cu cm
(c) 80080 cu cm (d) 80160 cu cm (c) H(P + Q + PQ )/3 (d) 2H(P + Q + PQ )/3
Volume And Surface Area A-257
100. What is the difference in radii of the ends of the frustum? 112. The ratio of surface area to diameter of a sphere whose
volume is 36 p cu cm is [2013-I]
Q- P Q- P (a) 3p (b) 6p
(a) (b)
p p (c) 6 (d) None of these
113. A cylindrical tube open at both ends is made of metal. The
(c) Q- P (d) None of these internal diameter of the tube is 6 cm and length of the tube
101. 10 circular plates each of thickness 3 cm, each are placed is 10 cm. If the thickness of the metal used is 1 cm, then the
one above the other and a hemisphere of radius 6 cm is outer curved surface area of the tube is [2013-I]
placed on the top just to cover the cylindrical solid. What (a) 140p sq cm (b) 146.5p sq cm
is the volume of the solid so formed? [2012-II] (c) 70p sq cm (d) None of these
(a) 264 p cu cm (b) 252p cu cm 114. The volume of a right circular cone of height 3 cm and slant
(c) 236 p cu cm (d) None of these height 5 cm is [2013-I]
102. Let the largest possible right circular cone and largest (a) 49.3 cu cm (b) 50.3 cu cm
possible sphere be fitted into two cubes of same length. If (c) 52 cu cm (d) 53 cu cm
C and S denote the volume of cone and volume of sphere, 115. A bucket is of a height 25 cm. Its top and bottom radii are
respectively. Then, which one of the following is correct? 20 cm and 10 cm, respectively. Its capacity (in L) is
[2012-II] [2013-I]
(a) C = 2S (b) S = 2C (a) 17.5p/3 (b) 17.5p
(c) C = S (d) C = 3S (c) 20 p (d) 25p
103. A right circular metal cone (solid) is 8 cm high and the 116. The height of a cylinder is 15 cm. The lateral surface area is
radius is 2 cm. It is melted and recast into a sphere. What is 660 sq cm. Its volume is [2013-I]
the radius of the sphere? [2012-II] (a) 1155 cu cm (b) 1215 cu cm
(a) 2 cm (b) 3 cm (c) 1230 cu cm (d) 2310 cu cm
(c) 4 cm (d) 5 cm 117. From a solid wooden right circular cylinder, a right circular
104. The volume of a cube is numerically equal to sum of its cone whose radius and height are same as the radius and
edges. What is the total surface area in square units? height of the cylinder, respectively is carved out. What is
[2012-II] the ratio of the volume of the utilised wood to that of the
(a) 12 (b) 36 wasted wood? [2013-I]
(c) 72 (d) 144 (a) 1 : 2 (b) 2 : 1
105. The diameter of base of a right circular cone is 7 cm and (c) 2 : 3 (d) 1 : 3
slant height is 10 cm, then what is its lateral surface area? 118. A solid spherical ball of iron of radius 4 cm is melted to
[2012-II] form spheres of radius 2 cm each. The number of spheres,
(a) 110 sq cm (b) 100 sq cm so formed is [2013-I]
(c) 70 sq cm (d) 49 sq cm (a) 8 (b) 9
106. What is the height of a solid cylinder of radius 5 cm and (c) 10 (d) 16
total surface area is 660 sq cm? [2012-II] 119. If the heights and the areas of the base of a right circular
(a) 10 cm (b) 12 cm cone and a pyramid with square base are the same, then
(c) 15 cm (d) 16 cm they have [2013-I]
107. If the ratio of the diameters of two spheres is 3 : 5, then (a) same volume and same surface area
what is the ratio of their surface areas? [2012-II] (b) same surface area but different volumes
(a) 9 : 25 (b) 9 : 10 (c) same volume but different surface areas
(c) 3 : 5 (d) 27 : 125 (d) different volumes and different surface areas
108. What is the volume of the largest sphere that can be carved 120. The diameter of the Moon is approximately one-fourth of
out from a cube of edge 3 cm? [2012-II] the diameter of the Earth. What is the ratio (approximate)
(a) 9p cu cm (b) 6p cu cm of their volumes? [2013-II]
(c) 4.5p cu cm (d) 3p cu cm (a) 1 : 16 (b) 1 : 64
109. What is the quantity of cloth required to roll up to form a (c) 1 : 4 (d) 1 : 128
right circular tent whose base is of radius 12 m and height 121. If the total surface area of a cube is 6 sq units, then what is
5 m? [2013-I] the volume of the cube? [2013-II]
(a) 40p sq m (b) 60p sq m (a) 1 cu unit (b) 2 cu units
(c) 78p sq m (d) 156p sq m (c) 4 cu units (d) 6 cu units
110. The length, breadth and height of a box are respectively 122. Let A be a pyramid on a square base and B be a cube. If a,
14 m, 12 m and 13 m. The length of the greatest rod that can b and c denote the number of edges, number of faces and
be put in it is [2013-I] number of corners, respectively. Then, the result a = b + c
(a) 22.31 m (b) 22.56 m is true for [2013-II]
(c) 20 m (d) 19.5 m (a) Only A (b) Only B
111. The volume of the material of a hemispherical shell with (c) Both A and B (d) Neither A nor B
outer and inner radii 9 cm and 7 cm, respectively is 123. What is the volume of a cone having a base of radius 10 cm
approximately [2013-I] and height 21 cm? [2013-II]
(a) 808 cu cm (b) 800 cu cm (a) 2200 cm3 (b) 3000 cm3
(c) 816 cu cm (d) 824 cu cm (c) 5600 cm3 (d) 6600 cm3
EBD_7367
258
A- Volume And Surface Area
124. A conical cap has base diameter 24 cm and height 16 cm. (a) 4 : 1 (b) 8 : 1
What is the cost of painting the surface of the cap at the (c) 2 : 1 (d) 6 : 1
rate of 70 paise per sq cm? [2013-II] 134. If 64 identical small spheres are made out of big sphere of
(a) ` 520 (b) ` 524 diameter 8 cm, then what is surface area of each small
(c) ` 528 (d) ` 532 sphere? [2014-I]
125. What is the whole surface area of a cone of base radius 7 (a) p cm2 (b) 2p cm2
cm and height 24 cm? [2013-II] (c) 4p cm2 (d) 8p cm2
(a) 654 sq cm (b) 704 sq cm 135. The dimensions of a field are 15 m by 12 m. A pit 8 m long,
(c) 724 sq cm (d) 964 sq cm 2.5 m wide and 2 m deep is dug in one corner of the field and
126. A tent is in the form of a right circular cylinder surmounted the earth removed is evenly spread over the remaining area
by a cone. The diameter of the cylinder is 24 m. The height of the field. The level of the field is raised by [2014-I]
of the cylindrical portion is 11 m, while the vertex of the (a) 15 cm (b) 20 cm
cone is 16 m above the ground. What is the area of the 200
curved surface for conical portion? [2013-II] (c) 25 cm (d) cm
(a) 3434/9 sq m (b) 3431/8 sq m 9
(c) 3432/7 sq m (d) 3234/7 sq m 136.. What is the diameter of the largest circle lying on the surface
127. If x is the curved surface area and y is the volume of a right of a sphere of surface area 616 sq cm? [2014-I]
circular cylinder, then which one of the following is correct? (a) 14 cm (b) 10.5 cm
[2013-II] (c) 7 cm (d) 3.5 cm
(a) Only the ratio of the height to radius of the cylinder is 137. The volume of a hollow cube is 216x3. What surface area of
independent of x the largest sphere which be enclosed in it? [2014-I]
(b) Only the ratio of height to radius of the cylinder is (a) 18p x2 (b) 27p x2
independent of y (c) 36p x2 (d) 72p x2
(c) Either (a) or (b) (d) Neither (a) nor (b) 138. A cylinder circumscribes a sphere. What is the ratio of
128. A cylinder is surmounted by a cone at one end, a hemisphere volume of the sphere to that of the cylinder ?[2014-II]
at the other end. The common radius is 3.5 cm, the height (a) 2 : 3 (b) 1 : 2
of the cylinder is 6.5 cm and the total height of the structure (c) 3 : 4 (d) 3 : 2
is 12.8 cm. The volume V of the structure lies between 139. Consider the following statements :
[2014-I] 1. The volume of the cone generated when the triangle
(a) 370 cm3 and 380 cm3 (b) 380 cm3 and 390 cm3 is made to revolve about its longer leg is same as the
(c) 390 cm3 and 400 cm3 (d) None of these volume of the cone generated when the triangle is
129. The diameter of the base of a cone is 6 cm and its altitude made to revolve about its shorter leg.
is 4 cm. What is the approximate curved surface area of the 2. The sum of the volume of the cone generated when
cone? [2014-I] the triangle is made to revolve about its longer leg
(a) 45 cm2 (b) 47 cm2 and the volume of the cone generated when the
(c) 49 cm2 (d) 51 cm2 triangle is made to revolve about its shorter leg is
130. A drainage tile is a cylindrical shell 21 cm long. The inside equal to the volume of the double cone generated
and outside diameters are 4.5 cm and 5.1 cm, respectively.
when the triangle is made to revolve about its
What is the volume of the clay required for the tile?
hypotenuse.
[2014-I]
Which of the above statements is/are correct ?
(a) 6.96p cm3 (b) 6.76p cm3
[2014-II]
(c) 5.76p cm3 (d) None of these
131. For a plot of land of 100 m × 80 m, the length to be raised by (a) Only 1 (b) Only 2
spreading the earth from stack of a rectangular base 10 m × 8 m (c) Both 1 and 2 (d) Neither 1 nor 2
and vertical section being a trapezium of height 2 m. The 140. If the side of a cube is increased by 100%, then by what
top of the stack is 8 m × 5 m. How many centimeters can the percentage is the surface area of the cube increased ?
level raised? [2014-I] [2014-II]
(a) 3 cm (b) 2.5 m (a) 150% (b) 200%
(c) 2 cm (d) 1.5 cm (c) 300% (d) 400%
132 A cube has each edge 2 cm and a cuboid is 1 cm long, 2 cm 141. Consider the following statements in respect of four
wide and 3 cm high. The paint in a certain container is spheres A,B, C and D having respective radii 6, 8, 10 and
sufficient to paint an area equal to 54 cm2. 12 cm.
Which one of the following is correct? [2014-I] 1. The surface area of sphere C is equal to the sum of
(a) Both cube and cuboid can be painted surface areas of sphere A and B .
(b) Only cube can be painted 2. The volume of sphere D is equal to the sum of
(c) Only cuboid can be painted volumes of sphere A, B and C.
(d) Neither cube nor cuboid can be painted Which of the above statements is / are correct ?
133. A cone of radius r cm and height h cm is divided into two [2014-II]
parts by drawing a plane through the middle point of its (a) Only 1 (b) Only 2
height and parallel to the base. What is the ratio of the (c) Both 1 and 2 (d) Neither 1 nor 2
volume of the original cone to the volume of the smaller cone? 142. The diameter of a metallic sphere is 6 cm. The sphere is
[2014-I] melted and drawn into a wire of uniform circular cross-
Volume And Surface Area A- 259
section. If the length of the wire is 36m, then what is its (a) remains unaltered (b) decreases by 25%
radius equal to ? [2014-II] (c) increases by 25% (d) increases by 50%
(a) 0.1 cm (b) 0.01 cm 154. A pipe with square cross–section is supplying water to a
(c) 0.0001 cm (d) 1.0 cm cistern which was initially empty. The area of cross–section
143. A cylindrical vessel of radius 4 cm contains water. A solid is 4 cm2 and the nozzle velocity of water is 40 m/s. The
sphere of radius 3 cm is lowered into the water until it is dimensions of the cistern are 10 m × 8 m × 6 m. Then the
completely immersed. The water level in the vessel will cistern will be full in [2015-II]
rise by [2015-I] (a) 9.5 hours (b) 9 hours
(a) 1.5 cm (b) 2 cm (c) 8 hours 20 minutes (d) 8 hours
(c) 2.25 cm (d) 4.5 cm 155. A hollow cylindrical drum has internal diameter of 30cm
144. If the radius of a sphere is increased by 10%, then the and a height of 1 m. What is the maximum number of
volume will be increased by [2015-I] cylindrical boxes of diameter 10 cm and height 10 cm each
(a) 33.1% (b) 30% that can be packed in the drum ? [2015-II]
(c) 50% (d) 10% (a) 60 (b) 70
145. A sphere and a cube have same surface area. The ratio (c) 80 (d) 90
of square of their volumes is [2015-I] 156. Consider the following statements : [2015-II]
(a) 6 : p (b) 5 : p
(c) 3 : 5 (d) 1 : 1 1. If the height of a cylinder is doubled, the area of the
146. The radius of a sphere is equal to the radius of the base curved surface is doubled.
of a right circular cone, and the volume of the sphere is 2. If the radius, of a hemispherical solid is doubled, its
double the volume of the cone. The ratio of the height of total surface area becomes fourfold.
the cone to the radius of its base is [2015-I] Which of the above statements is/are correct ?
(a) 2 : 1 (b) 1 : 2 (a) 1 only (b) 2 only
(c) 2 : 3 (d) 3 : 2 (c) Both 1 and 2 (d) Neither 1 nor 2
147. Water flows through a cylindrical pipe of internal 157. A large water tank has the shape of a cube. If 128 m3 of
diameter 7 cm at the rate of 5 m/s. The time, in minutes, water is pumped out, the water level goes down by 2 m.
the pipe would take to fill an empty rectangular tank 4m
Then the maximum capacity of the tank is [2015-II]
× 3m × 2.31m is [2015-I]
(a) 28 (b) 24 (a) 512 m3 (b) 480 m3
(c) 20 (d) 12 (c) 324 m3 (d) 256 m3
148. The total outer surface area of a right circular cone of 158. From the solid gold in the form of a cube of side length 1 cm,
height 24 cm with a hemisphere of radius 7 cm upon its 1
base is [2015-I] spherical solid balls each having the surface area cm2
p3
(a) 327p square cm (b) 307p square cm are to be made. Assuming that there is no loss of the material
(c) 293p square cm (d) 273p square cm
in the process of making the balls, the maximum number of
149. A rectangular block of wood having dimensions 3m × 2m
balls made will be [2015-II]
× l.75m has to be painted on all its faces. The layer of
paint must be 0.1 mm thick. Paint comes in cubical boxes (a) 3 (b) 4
having their edges equal to 10 cm. The minimum number (c) 6 (d) 9
of boxes of paint to be purchased is [2015-I] 159. Water is filled in a container in such a manner that its volume
(a) 5 (b) 4 doubles every 5 minutes. If it takes 30 minutes for the
(c) 3 (d) 2 container to be full, in how much time will it be one–fourth
150. The diagonals of three faces of a cuboid are 13, 281 full? [2015-II]
and 20 linear units. Then the total surface area of the (a) 7.5 minutes (b) 15 minutes
cuboid is [2015-I] (c) 20 minutes (d) 17.5 minutes
(a) 650 square units (b) 658 square units 160. 30 metallic cylinders of same size are melted and cast in the
(c) 664 square units (d) 672 square units form of cones having the same radius and height as those
151. A rectangular paper of 44 cm long and 6 cm wide is rolled of the cylinders. [2015-II]
to form a cylinder of height equal to width of the paper. Consider the following statements :
The radius of the base of the cylinder so rolled is
Statement I : A maximum of 90 cones will be obtained.
[2015-I]
(a) 3.5 cm (b) 5 cm Statement II : The curved surface of the cylinder can be
(c) 7 cm (d) 14 cm flattened in the shape of a rectangle but the curved surface
152. If three metallic spheres of radii 6 cm, 8 cm and 10 cm are of the cone when flattened has the shape of triangle.
melted to form a single sphere, then the diameter of the Which one of the following is correct in respect of the
new sphere will be [2015-I] above?
(a) 12 cm (b) 24 cm (a) Both Statement I and Statement II are correct and
(c) 30 cm (d) 36 cm Statement II is the correct explanation of Statement I
153. If the height of a right circular cone is increased by 200% (b) Both Statement I and Statement II are correct and
and the radius of the base is reduced by 50%, then the Statement II is not the correct explanation of Statement I
volume of the cone [2015-I] (c) Statement I is correct but Statement II is not correct
(d) Statement I is not correct but Statement II is correct
EBD_7367
A-260 Volume And Surface Area

161. A water tank, open at the top, is hemispherical at the bottom 171. A building is in the form of a cylinder surmounted by a
and cylindrical above it. The radius is 12m and the capacity hemispherical dome on the diameter of the cylinder. The
is 3312p m3. The ratio of the surface areas of the spherical height of the building is three times the radius of the base
and cylindrical portions is [2015-II] 1 3
(a) 3 : 5 (b) 4 : 5 of the cylinder. The building contains 67 m of air. What
(c) 1 : 1 (d) 6 : 5 21
162. The areas of three mutually perpendicular faces of a cuboid is the height of the building ? [2016-II]
are x, y, z. If V is the volume, then xyz is equal to [2015-II] (a) 6 m (b) 4 m
(a) V (b) V2 (c) 3 m (d) 2 m
(c) 2V (d) 2V2 172. The radius of the base and the height of a solid right circular
163. Let V be the volume of an inverted cone with vertex at cylinder are in the ratio 2:3 and its volume is 1617 cm3.
origin and the axis of the cone is along positive y–axis. The What is the total surface area of the cylinder ? [2016-II]
cone is filled with water up to half of its height. The volume (a) 462 cm2 (b) 616 cm2
of water is [2015-II] (c) 770 cm2 (d) 786 cm2
173. The cost of painting a spherical vessel of diameter 14 cm is
v v `8008. What is the cost of painting per square centimetre ?
(a) (b)
8 6 [2016-II]
v v (a) `8 (b) `9
(c) (c) (c) `13 (d) `14
3 2 174. A drinking glass of height 24 cm is in the shape of frustum
164. If the surface area of a cube is 13254 cm2, then the length of of a cone and ' diameters of its bottom and top circular ends
its diagonal is [2015-II] are 4 cm and 18 cm respectively. If we take capacity of the
(a) 44 2 cm (b) 44 3 cm glass as px cm3, then what is the value of x ? [2016-II]
(a) 824 (b) 1236
(c) 47 2 (d) 47 3 cm
(c) 1628 (d) 2472
165. How many spherical bullets each of 4 cm in diameter can be 175. Rain water from a roof 22m × 20m drains into a cylindrical
made out of a cube of lead whose edge is 44 cm ? vessel having diameter of base 2 m and height 3.5 m. If the
[2015-II] vessel is just full, what is the rainfall? [2016-II]
(a) 2541 (b) 2551 (a) 3.5 cm (b) 3 cm
(c) 2.5 cm (d) 2 cm
(c) 2561 (d) 2571 176. The height of a cone is 60 cm. A small cone is cut off at the
166. A river 2.5 m deep and 45 m wide is flowing at the speed of
1
3.6 km/hour. The amount of water that runs into the sea per top by a plane parallel to the base and its volume is the
minute is [2015-II] 64
volume of original cone. What is the height from the base at
(a) 6650 m3 (b) 6750 m3 which the section is made ? [2016-II]
(c) 6850 m3 (d) 6950 m3 (a) 15 cm (b) 20 cm
167. The area of four walls of a room is 120m2. The length of the (c) 30 cm (d) 45 cm
room is twice its breadth. If the height of the room is 4 m, 177. What is the volume of a sphere of radius 3 cm? [2016-II]
what is area of the floor ? [2016-II] (a) 36p cm3 (b) 18 p cm3
(c) 9 p cm 3 (d) 6 p cm3
(a) 40 m2 (b) 50 m2
(c) 60 m2 (d) 80 m2 DIRECTIONS (Qs. 178-180): A tent of a circus is made of canvas
168. The ratio of the curved surface area to the total surface area and is in the form of right circular cylinder and right circular
of a right circular cylinder is 1 : 2. If the total surface area is cone above it. The height and diameter of the cylindrical part of
616 cm2, what is the volume of the cylinder ? [2016-II] the tent are 5 m and 126 m respectively. The total height of the
(a) 539 cm3 (b) 616 cm3 tent is 21 m.
(c) 1078 cm3 (d) 1232 cm3 [2016-II]
169. A cubic metre of copper weighing 9000 kg is rolled into a 178. What is the slant height of the cone ?
square bar 9 m long. An exact cube is cut off from the bar; (a) 60 m (b) 65 m
How much does the cube weigh ? [2016-II] (c) 68 m (d) 70 m
179. What is the curved surface area of the cylinder ?
1000
(a) 1000 kg (b) kg (a) 1980 m2 (b) 2010 m2
3 (c) 2100 m 2 (d) 2240 m2
500 180. How many square metres of canvas are used ?
(c) 300 kg (d) kg (a) 14450 (b) 14480
3
170. Into a conical tent of radius 8.4m and vertical height 3.5 m, (c) 14580 (d) 14850
how many full bags of wheat can be emptied, if space 181. If the perimeter of a circle is equal to that of a square, then what
required for the wheat in each bag is 1.96 m3 ? . [2016-II] is the ratio of area of circle to that of square ? [2016-II]
(a) 264 (b) 201 (a) 22 : 7 (b) 14 : 11
(c) 132 (d) 105 (c) 7 : 22 (d) 11 : 14
Volume And Surface Area A-261
182. If the radius of a right circular cone is increased by p% the base of the cylinder is 50 m, the height of the cylinder is
without increasing its height, then what is the percentage 10 m and the total height of the tent is 15 m, then what is the
increase in the volume of the cone ? [2017-I] capacity of the tent in cubic metres? [2017-II]
(a) p2 (b) 2P2 87500p
2 (a) 37500 p (b)
p æ p ö 3
(c) (d) p ç 2 + ÷
100 è 100 ø 26500 p
183. If the surface area of a sphere is reduced to one-ninth of the (c) (d) 25000 p
3
area, its radius reduces to [2017-I] 192. Two rectangular sheets of sizes 2p × 4p and p × 5p are
(a) One-fourth (b) One-third available. A hollow right circular cylinder can be formed by
(c) One-fifth (d) One-ninth joining a pair of parallel sides of any sheet. What is the
184. Ice-cream, completely filled in a cylinder of diameter 35 cm maximum possible volume of the circular cylinder that can
and height 32 cm, is to be served by completely filling be formed this way? [2017-II]
identical disposable cones of diameter 4 cm and height 7 (a) 4p2 (b) 8p2
cm. The maximum number of persons that can be served in (c) 1.25p2 (d) 6.25p2
this way is [2017-I] 193. The radius and slant height of a right circular cone are 5 cm
(a) 950 (b) 1000 and 13 cm respectively. What is the volume of the cone?
(c) 1050 (d) 1100 [2018-I]
185. If the HCF of polynomials [2017-II] (a) 100p cm3 (b) 50p cm3
f(x) = (x – 1) (x2 + 3x + a) and (c) 65p cm3 (d) 169p cm3
g(x) = (x + 2) (x2 + 2x + b) is (x2 + x – 2), 194. If the ratio of the radius of the base of a right circular cone
then what are the values of a and b respectively? to its slant height is 1 : 3, what is the ratio of the total
(a) 2, 2 (b) 2, –3 surface area to the curved surface area? [2018-I]
(c) –1, –3 (d) –2, –1 (a) 5 : 3 (b) 3 : 1
186. A cylinder of height 2x is circumscribed by a sphere of (c) 4 : 1 (d) 4 : 3
radius 2x such that the circular ends of the cylinder are two 195. A right circular cone is sliced into a smaller cone and a
small circles on the sphere. What is the ratio of the curved frustum of a cone by a plane perpendicular to its axis. The
surface area of the cylinder to the surface area of the sphere? volume of the smaller cone and the frustum of the cone are
[2017-II] in the ratio 64 : 61. Then their curved surface areas are in the
ratio [2018-I]
(a) 3:4 (b) 3 :3 (a) 4 : 1 (b) 16 : 9
(c) 3:2 (d) 3 :1 (c) 64 : 61 (d) 81 : 64
187. A cylindrical vessel 60 cm in diameter is partially filled with 196. In a room whose floor is a square of side 10 m, an equilateral
water. A sphere 30 cm in diameter is gently dropped into triangular table of side 2 m is placed. Four book-shelves of
size 4m × 1m × 9m are also placed in the room. If half of the
the vessel and is completely immersed. To what further
rest of the area in the room is to be carpeted at the rate of
height will the water in the cylinder rise? [2017-II] `100 per square metre, what is the cost of carpeting
(a) 20 cm (b) 15 cm (approximately)? [2018-I]
(c) 10 cm (d) 5 cm (a) ` 7,600 (b) ` 5,635
p (c) ` 4,113 (d) ` 3,200
188. The vertical angle of a right circular cone is and the slant 197. A region of area A bounded by a circle C is divided
2
into n regions, each of area A/n, by drawing circles of
height is 2 r cm . What is the volume of the cone in cubic radii r1, r2, r3, .... rn–1 such that r1 < r2 < r3 < .... rn–1 concentric
cm? [2017-II] rm +1
(a) pr3 (b) 9pr3 with the circle C. If pm = where
rm
pr 3 m = 1, 2, 3, .... (n – 2), then which one of the following is
(c) (d) 3pr3
3 correct? [2018-I]
189. The radii of the frustum of a right circular cone are in the (a) p increases as m increases
ratio 2 : 1. What is the ratio of the volume of the frustum of (b) p decreases as m increases
the cone to that of the whole cone? [2017-II] (c) p remains constant as m increases
(a) 1 : 8 (b) 1 : 4 (d) p increases for some values of m as m increases and
(c) 3 : 4 (d) 7 : 8 then decreases thereafter
190. From a solid cylinder whose height is 8 cm and of base 198. What is the volume of a cone of maximum volume cut out
radius 6 cm, a conical cavity of height 8 cm and of base from a cube of edge 2a such that their bases are on the same
radius 6 cm is formed by hollowing out. What is the inner plane? [2018-I]
surface area of the cavity. [2017-II]
(a) 6p square cm (b) 8p square cm pa 3
(a) pa3 (b)
(c) 10p square cm (d) 60p square cm 3
191. A tent has been constructed which is in the form of a right
circular cylinder surmounted by a right circular cone whose 2 pa 3 3pa 3
(c) (d)
axis coincides with the axis of the cylinder. If the radius of 3 4
EBD_7367
A- 262 Volume And Surface Area
199. There are as many square centimetres in the surface area of
a sphere as there are cubic centimetres in its volume. What pr 2 p 2 + 1 pr 2 4p 2 + 9
(c) (d)
is the radius of the sphere? [2018-I] 3 3
(a) 4 cm (b) 3 cm
203. A wire is in the form of a circle of radius 98 cm. A square is
(c) 2 cm (d) 1 cm
formed out of the wire. What is the length of a side of the
200. The curved surface area of a right circular cone is
square? (Use p = 22/7) [2018-I]
1.76 m2 and its base diameter is 140 cm. What is the height
of the cone? [2018-I] (a) 146 cm (b) 152 cm
(c) 154 cm (d) 156 cm
(a) 10 cm (b) 10 2 cm
204. What is the area of the largest circular disc cut from a square
(c) 20 2 cm (d) 10 15 cm
2
201. A cube of maximum volume (each corner touching the surface of side units? [2018-I]
from inside) is cut from a sphere. What is the ratio of the p
volume of the cube to that of the sphere? (a) p square units (b) 1 square unit
[2018-I] 2
(c) p square units (d) 2 square units
(a) 3 : 4p (b) 3 : 2p 205. The surface area of closed cylindrical box is
(c) 2 : 3p (d) 4 : 3p 352 square cm. If its height is 10 cm, then what is its diameter?
202. If the ratio of the circumference of the base of a right circular 22
cone of radius r to its height is 3 : 1, then what is the area of (Use p = ) [2018-I]
7
the curved surface of the cone? [2018-I] (a) 4 cm (b) 8 cm
2 2 (c) 9.12 cm (d) 19.26 cm
2pr 4p + 9
(a) 3pr2 (b)
3

HINTS & SOLUTIONS


1. (d) Volume of wire = pr2h 1 3 2 3
r ´ 90 9r \ C:H= pR : pR = 1: 2
New radius of the wire = = 3 3
100 10 3. (d) Let l1 : l2 : l3 = 1 : 2 : 3
Let new length of the wire be L.
x x x
æ 9r ö
2
81 2 \ b1 : b2 : b3 = : : = 6:3: 2
\ Volume of new wire = p ç ÷ ´ L = pr L 1 2 3
è 10 ø 100
According to question, \ Ratio of volumes = pr12 h : pr22 h2 : pr32 h3
81 2 100 9 9
pr2h = pr L Þ L = h = 3 ´ 1: ´ 2 :1´ 3 = 9 : : 3
2
100 81 4 2
100 19
Increase in length = h-h = h 3
81 81 = 3 : :1 = 6 : 3 : 2
2
19h / 81h 4. (b) In DABC,
Per cent increase = ´100% = 23.46%
h
= 23% (approx) 3/ 2
cos 30º =
1 l
2. (a) Volume of cone, C = p R2H
3 3/ 2
Þ l= = 3 cm
3/2
R

O R
1 3
= pR (Q H = R) C
3
l
2 3
Volume of hemisphere, H = pR
3 A 30º D
3/2 B
Volume And Surface Area A-263
\ Area of cone ACD = prl 4 3
\ Volume of utilised wood = pr
3 3
= p´ ´ 3
2 4 3
and volume of wasted wood = 2pr - pr
3

3 3p 3
= sq cm
2 6pr 3 - 4pr 3 2pr 3
5. (b) According to question, = =
3 3
27 × Volume of smaller drops = Volume of bigger drop
4 3 2 3
4 4 \ Required ratio = pr : pr = 2 :1
\ 27 ´ pr 3 = pR
3
3 3
3 3
11. (d) Radius of conical flask = r
Þ 27 × (0.2)3 = R3 Height of conical flask = h
Þ (3 × 0.2)3 = R3
R = 0.6 cm x
6. (b) Given, l = R
\ Total surface area of cone, h
S = pr (r + l) = p R(R + R) = 2pR2
\ Area of circle, A = p(2R)2 = 4pR2
Þ A = 2S r
7. (b) According to question, 2r
Volume of cylinder = Volume of bar
Þ pr2h = base area of block × length 1 2
\ Volume of conical flask = pr h
2
5 5 3
22 æ 1.5 ö
Þ ´ç ÷ ´ 3.5 = 100 ´ 100 ´ L Radius of cylindirical flask = 2r
7 è 2 ø
Height of cylindirical flask = x
22 2.25 100 100 \ Volume of cylindirical flask = p(2r)2 x
Þ ´ ´ 3.5 ´ ´ =L = p 4r2x
7 4 5 5
\ L = 2475 m According to question,
8. (b) Let r be the radius of hemispherical bowl. 1 2
pr h = 4pr2x
132 3
\ 2pr =
7 1.pr 2 h h
x= =
132 7 3 ´ 4pr 2 12
Þ r= ´ = 3 cm
7 2 ´ 22 h
\ Capacity of hemispherical bowl \ Height of the cylinderical flask is .
12
= Volume of hemispherical bowl
12. (d) Volume of solid cylinder = p(3)24 = 36p cm3
2 and volume of conical cavity
= p(3)3 = 18p cu cm
3
1
9. (d) Volume of semi-circular disc = p(3) 2 (4) = 12p cm3
3
1
= p´ 4 ´ 4 ´ 2 cu cm (Q r = 4 cm and h = 2 cm) \ Volume of remaining solid
2 = 36p – 12p = 24p cm3
and volume of spherical ball 13. (c) Let the radii and slant height of two right circular cones
4 æ 2.5 ö
3
are r1, l1 and r2, l2, respectively.
= pç ÷ cu cm \ Ratio of their curved surface areas
3 è 10 ø
pr1l1 l1
1 = = (Q r1 = r2)
p´ 4 ´ 4 ´ 2 pr2 l2 l2
2 = 768
\ Number of balls = 3
4 æ 2.5 ö 3
pç ÷ = = 3: 2
3 è 10 ø 2
10. (a) Let r be the radius of cylindrical block, then height 14. (b) Let perimeter 4a = 20
will be 2r. Þ a=5
Volume of block = p(r2) (2r) = 2pr3 Volume of cube = a3 = 125
A sphere of maximum possible volume is carved out Hence, pairs (20, 125), represents the perimeter of one
whose radius will be r. face of a cube and volume of cube.
4 3 15. (b) Volume of the new cube = Sum of all the three cubes.
\ Volume of sphere = pr = 63 + 83 + 103 = 216 + 512 + 1000 = 1728 cm3
3
EBD_7367
264
A- Volume And Surface Area
let Length of the edge of new cube = a cm
(a) \ Volume of pipe, V = p ( r1 - r2 ) ´ h
2 2
a3 = 1728 cm3 21.
a = 3 1728 = 12 cm. 22 22
= [(3.5)2 - (2.5) 2 ] ´ 140 = (12.25 - 6.25) ´ 140
16. (d) The distance between vertices B and C is 1 cm. 7 7
= 22 × 6 × 20 = 2640 cu cm
G
F 22. (d) Here similar triangle
B
C DABC ~ In DADE
A
E D
l
O A
2l r
D E
The distance between A and B is 12 + 12 = 2 cm
The distance between diagonal B and D is
12 + 12 + 12 = 3 cm C
B
17. (c) Total surface area = 3pr2 + 3pr2 = 6pr2 2r
\ Painting cost of two
halves = 6 pr2 × 8 1
= 48pr2 p ´ 4r 2 ´ (2l )2 - (2 r ) 2
V1 3 8
18. (b) According to question \ = = =8:1
V2 1 1
Surface area of sphere = 3 (Volume of sphere) p´ r 2 ´ l 2 - r 2
3
4 3 23. (b) Let r1 = 1k and r2 = nk
Þ 4pr2 = 3 ´ pr Þ r = 1
3 According to question,
\ Diameter = 2r = 2 cm V1 = V2
19. (a) According to question 1 2
\ pr1 h1 = pr22 h2 Þ 1 p k 2 ´ h1 = pn 2 k 2 h2
3pr12 4 r 2 3 3
2 =
Þ 1 =
3pr2 1 r2 1 Þ h1 = 3n 2 h2
2 3 24. (c) A: it is a true statement.
pr1 R. It is a false statement.
3 8
\ Required ratio = = or 8 : 1 Because the length may be p r but the breadth is
2 3 1
pr2
3 always less than h 2 + r 2 of formed rectangle.
20. (a) Let height of the cylinder be h. 25. (b) Both statements are true but (R) is not a correct
explanation of (A).
15 cm 26. (d) Let radius and height of a cone be r and h
r 3
15 cm Q =
h 4
1 2
Volume of cone = pr h
H 3
1 4r
\ 96p = p´ r2 ´
3 3
96 ´ 3 ´ 3
According to question, Þ r3 = = 216 Þ r = 6 cm
4
Volume of hemisphere + Volume of cylinder and h = 8 cm
= Volume of container \ Lateral surface area
2 3
Þ pr + pr 2 h = 32400 p = pr r 2 + h2
3
2 = p´ 6 36 + 64
Þ p ´ 3375 + p ´ 225 h = 32400p = 60p sq cm
3 27. (b) The maximum diameter of a sphere in a cube = 3 m.
Þ 2p × 1125 + p × 225 h = 32400p \ Radius of the sphere = 1.5 m
Þ 10 + h = 144
Þ h = 134 4
\ Volume of sphere, V1 = p(1.5)3 = 4.5p cu m
\ Height of container (h) = 15 + 134 = 149 3
Volume And Surface Area A-265
\ Volume of cube, V2 = (3)3 = 27 m3 34. (a) R = 2r (given)
\ Volume of solid left = V2 – V1 = (27 – 4.5p) m3 According to question
28. (b) Suppose ratios of dimensions of a rectangular box be Volume of cylinder = Volume of cone
2x, 3x and 4x.
1
According to question, \ pr2 h = p R2H
3
= (10 - 8) ´ 2 [ 2 x ´ 3x + 3x ´ 4 x + 4 x ´ 2 x ] = 234
Þ 2 × 2 × 26x2 = 234 1
Þ r2 h = (2r ) 2 H
3
\ x2 = 234 = 2.25 \ x = 1.5
104 3h
\ H=
Thus, the dimensions are, 4
2x = 2 × 1.5 = 3.0 35. (c) Radius of roller = 0.35 m.
3x = 3 × 1.5 = 4.5 The area covered in one revolution curved surface
4x = 4 × 1.5 = 6.0 area of roller
29. (c) Volume of solid cube = (4)3 = 64 cm3 22
Volume of recast cube = (1)3 = 1 cm3 = 2´ ´ 0.35 ´ 2
\ Total surface area of cube : Total surface area of 7
recast cube = 4.4 sq m
= x:y \ Total area covered in 50 revolutions
Þ x : y = 6(4)2 : 6(1)2 × 64 = 1 : 4 = 4.4 × 50 = 220 sq m
30. (d) Since, the outer edges of a cubical box is 5 cm. 36. (d) Let x be the diameter of moon.
\ Surface area of the outer cubical box Volume of Moon
= 5 (edge)2 Required Ratio =
Volume of Earth
= 5 (5)2 = 125 sq cm
Surface area of the inner cubical box 4 æxö
3

= 5 × (4)2 pç ÷
3 è8ø 1
Þ 80sq cm = 3
=
\ Total surface area = 125 + 80 =205 4 æxö 64
pç ÷
= 205 sq cm 3 è 2ø
31. (c) Area of tank, BCDE = 10 × 7.5 = 75 m2 37. (b) When we join three cubes to form a cuboid. The length
G F of the cuboid becomes 15 cm and height, width remains
the 5 cm each.
E D \ Surface area of cuboid = 2 (lb + bh + hl)
7.5 m = 2 (15 × 5 + 5 × 5 + 15 × 5)
A C = 2 (75 + 25 + 75)
B 10 m = 350 sq cm
Area of remaining field ABEDFGA 38. (c) Volume of bigger cone
= 125 × 15 – 75 = 1800 sq m 1
Volume of Earth dug = 10 ×7.5 × 6 = 450 cu m = p(6)2 ´ 8 = 96 p cm3
By given condition, 1800 × h = 450 3

1 1 1
Volume of smaller cone = p (1) 2 ´ 2
Þ h= m = ´ 100 cm = 25 cm 3
4 4
32. (c) Volume of wood = Volume of lead pencil – Volume of lead 2p 3
= cm
= p(0.4) 2 21 - p(0.1) 2 ´ 21 3
22 96 p
= 21´ (0.16 - 0.01) Number of cones = 2p = 144
7
= 66 (0.15) = 9.9 cu cm 3
33. (c) Volume of soild = l × b × h 39. (d) Diameter of cylindrical log = d
= 22 × 7 × 5 = 770 cu cm Then, height of cylindrical log = d
Let the water rise in height be h. Diameter of greatest possible sphere = d
Q Volume of water rise in vessel = Volume of solid d
Þ pr2h = 770 Radius of sphere =
2
22 Volume of cylindrical log = pr2 h
Þ ´ 14 ´ 14 ´ h = 770
7 2
æd ö pd 3
= pç ÷ d =
770 ´ 7 5 è2ø 4
Þ h= = = 1.25 cm
22 ´14 ´14 4
EBD_7367
266
A- Volume And Surface Area

3
4 3 4 æd ö pd 3 pd 3 4 1 2
Volume of sphere pr = p ç ÷ = = ´ 45. (b) Volume of cone = pr h
3 3 è2ø 6 4 6 3
2 A
= volume of cylinder
3
40. (b) In DOAB, OA2 + AB2 = OB2
13 cm
12cm
O
10 cm
8 cm 5 cm
C
2cm A B B

AB2 = 102 – 82 = 36 1
= p ´ 52 ´ 12 = 100 p cu cm
AB = 6 cm 3
41. (c) Let water level increase by x cm. 46. (c) According to question
\ Volume of cylindrical can = p (12)2 × x = 144p x Volume of first cone 4
=
4 Volume of second cone 5
Volume of sphere = p(6)3 = 288p cu cm
3
1
According to question p(2)2 h1 4
3
144px = 288p \ =
1 5
x = 2 cm p(3) 2 h2
3
42. (d) Volume of frustum of cone
4h1 4 h1 9
5=R Þ 9h2
= Þ =
5 h2 5
47. (d) The dimensions of a bricks are
l = 12.5 cm
b = 10 cm
6 h = 7.5 cm
\ Area of brick = 2 (lb + bh + hl)
3=r = 2 (12.5 × 10 + 10 × 7.5 + 12.5 × 7.5)
= 2 (125 + 75 + 93.75)
ph 2 = 2 × 293.75
= [R + r 2 + Rr ]
3 = 587.50 sq cm
Area to be painted by the available paint = 5.875 sq m
p´6 2 2 = 5.875 × 104 sq cm
= [5 + 3 + 5 ´ 3]
3 = 58750 sq cm
= p × 2 × 49 = 98p cu unit 58750
No. of bricks can be painted = = 100 bricks.
2 3 587.50
43. (a) Volume of hemisphere = pr 48. (b) Volume of hollow cylindrical pipe
3
22
= p ( r2 - r1 ) ´ h = {(4) 2 - (2) 2 } ´ 700
2 2
2 22
= ´ ´ 21´ 21´ 21 7
3 7
22
= 44 ´ 441 = 19404 cu cm = ´ 12 ´ 700 = 26400 cu cm
7
19404
= L = 19.4 L 5
1000 = 26400 ´ kg (Q 1000 cu cm = 5 kg)
1000
44. (c) Height of water in a second = 30 cm = 132 kg
Height of water in 60 s = 30 × 60 49. (c) According to question,
h = 1800 cm Surface area of sphere = Surface area of cube
Area of cross section, pr2 = 5 sq cm 2

Volume of water flow in one minute = pr2 h ærö 3


Þ 4pr2 = 6a2 Þ ç ÷ =
= 5 × 1800 = 9000 cu cm èaø 2p

9000 (Volume of sphere) 2


= L = 9L Required Ratio =
1000 (Volume of cube) 2
Volume And Surface Area A-267

2 53. (d) Let r of sheet = 14 cm


æ4 3ö it bent and form a conical cup of length l = 14 cm
ç pr ÷ 2 3
è3 ø = 16 p2 éæ r ö ù According to question
= êç ÷ ú Circumference of base of the cone = Circumferecne of
(a3 )2 9 ëêè a ø ûú semi-circle
3
16 2 æ 3 ö 16 2 27 6 Þ 2pR = p r
= p ç ÷ = p ´ 3 = Þ 2R = r Þ 2R = 14
9 è 2p ø 9 8p p Þ R = 7 cm
2 Q l2 = R2 + h2
50. (b) Volume of hemisphere = p(53 - 43 ) Þ (14)2 = (7)2 + h2
3 Þ h2 = 196 – 49 = 147
Þ h= 7 3
\ Capacity of cup
1 1 22
= p R2h = ´ ´ 7 ´ 7 ´ 7 3
3 3 7
1078
2 = 3 cu cm
= p(125 - 64) 3
3 54. (a) Let the radius of cone and sphere be r,
2 Height = h
= p ´ 61 cu cm diameter = 2r
3
According to question
2 Volume of cone = Volume of sphere
= p ´ 61 ´ 9 g ( Given, 1cu cm = 9 g)
3 1 2 4
= 366p g Þ pr h = pr 3
3 3
51. (c) Volume of hemispherical bowl
h 4
2 Þ =
= p (20)3 r 1
3
h 2 h 2
16000 Þ = = =
= p cu cm 2r 1 d 1
3
55. (a) In one round, distance covered by cylinder
Volume of conical shape bottle
= 60 cm
1 200p In one second, distance covered by cylinder
= p (5) 2 8 = cu cm
3 3 = 60 × 5 = 300 cm
\ Required number of bottles In five seconds, distance covered by cylinder
= 300 × 5 = 1500 cm = 15 m
16000 p / 3 56. (b) Volume of frustum of a cone
= = 80
200 p / 3 ph 2
52. (a) Volume of spherical lead shot =
3
( r1 + r22 + r1r2 )
4 4 57. (a) Increase in the height of water level
= p (1)3 = p cu cm
3 3 0.75
= m = 0.2 m = 20 cm
4 4 2.5 ´1.5
I. Volume of shots = p(0.5)3 ´ 8 = p cu cm
3 3 58. (d) Let Length, breadth and height are 6x, 3x and x
respectively.
4 4
II. Volume of both shots = p(0.75)3 + p(0.8)3 Let the side of a cube be a.
3 3 According to question,
Surface area of a cube = Surface area of rectangular
4 éæ 3 ö æ 4 ö ù 4 é 27 64 ù
3 3

= p êç ÷ + ç ÷ ú = p ê + parallelopiped
3 êëè 4 ø è 5 ø úû 3 ë 64 125 ûú Þ 6(a)2 = 2 (6x × 3x + 3x × x + x × 6x)
Þ 6a2 = 2 (18x2 + 3x2 + 6x2)
4 é 3375 + 4096 ù 4 æ 7471 ö Þ 6a2 = 54x2 Þ a2 = 9x2
= p ú = pç ÷
3 êë 8000 û 3 è 8000 ø \ a = 3x
Volume of cube
4 Required ratio =
= p(0.93) cu cm Volume of rectangular parallelopiped
3
Thus, only Statement I is true. a3 (3x )3 3
= = =
6 x ´ 3x ´ x 18 x 3 2
EBD_7367
268
A- Volume And Surface Area
59. (d) Let h be the depth of the pit. 64. (b) Let radius and height of cylinder be r and h respectively.
\ Volume of Earth dug = 500 × 250 cm3 = 125000 cu cm According to question
Volume of pit = 50 × 50 × h 2 pr
2 pr × h = (h + r )
125000 3
\ h= = 50 cm = 0.5 m
50 ´ 50
1
60. (d) Given a1 = 0.3m, a2 = 0.4 m and a3 = 0.5 m Þ 9= (9 + r )
\ Side of new cubic biscuit 3
Þ 27 = 9 + r
= 3
a13 + a23 + a33 \ r = 18 m
65. (b) Let r1 and r2 be the radii of these spheres.
= 3
(0.3) 3 + (0.4)3 + (0.5)3 4 3
v1 pr1
= 3
0.027 + 0.064 + 0.125 3 8
Given Ratio = =
v2 4 3 1
= 3 0.216 = 0.6 m pr2
Thus, the total surface area of cubic biscuit 3
= 6(0.6)2 r1 2
= 2.16 sq m Þ =
r2 1
61. (b) Let the radius of ball = r
\ Radius of base of cylinder = 4r \ Ratio of their surface areas
Height of cylinder = 4r 4pr12 æ 2 ö
2

= = ç ÷ = 4 :1
4 3 4pr22 è 1 ø
\ Volume of spherical ball = pr
3 66. (b) Given, l = 30 cm, b = 24 cm and h = 18 cm
and volume of water = p(4r)2 (2r) Here, Maximum length of the rod can be placed in the
= 32 pr3 cuboid is equal to diagonal of box
Volume of remaining portion of cylinder = 32 pr3
= 302 + 242 + 182 = 900 + 576 + 324 = 1800
Let number of spherical balls = n
4 3 = 30 2 cm
\ 32pr3 = n ´ pr (Q. Nos. 67-69)
3
Þ n = 8 × 3 = 24 M
62. (c) Let l, b and h be the sides of cuboid.
l2 + b2 = x2 ...(i) r1
b2 + h2 = y2 ...(ii) x
P
h2 + l2 = z2 ...(iii) A
On adding equation (i), (ii) and (iii)
2 (l2 + b2 + h2) = x2 + y2 + z2 3cm
9 cm

h r2
1 2
Þ l2 + b2 + h2 = (x + y2 + z 2 ) ...(iv) B
2 C
O 6 cm
On solving Eqs. (i), (ii), (iii) and (iv);
Here, r1 = 3 cm, r2 = 6 cm and h = 9 cm
y 2 + z 2 - x2 z 2 + x2 - y 2 67. (b) Total surface area of the frustum
h= ,l =
2 2 = p [(R + r) l + r2 + R2]
x2 + y2 - z 2 where, l = h 2 + (R - r ) 2
and b =
2
= p [(6 + 3) 81 + 9 + 9 + 36]
Volume of cuboid = l b h
( y 2 + z 2 - x 2 )( z 2 + x 2 - y 2 )( x 2 + y 2 - z 2 ) = p[9 90 + 45] = 9p[3 10 + 5] sq cm
= 68. (d) By using properties at similar triangle in DMPA and
2´ 2´ 2
DMOC,
1
= ( y 2 + z 2 - x 2 )( z 2 + x 2 - y 2 )( x 2 + y 2 - z 2 ) MP PA
2 2 =
MO OC
63. (a) Sheet is revolved about its length
\ h = 7 cm and r = 4 cm x 3
Þ = Þ x =9
\ Then, Volume of the figure 9+ x 6
22 Height of the cone= MO = x + 9
= pr 2 h = ´ 4 ´ 4 ´ 7 = 352 cu cm
7 = (9 + 9) cm = 18 cm
Volume And Surface Area A-269
69. (a) In DABC, 77. (a) According to question,
AC2 = AB2 + BC2 Volume of cone = Volume of cylinder
Þ AC2 = 92 + (6 – 3)2 = 81 + 9 = 90 1 2
\ pr h = pr 2 ´ 5
Þ AC = 3 10 cm 3
Þ h = 5 × 3 = 15 cm
Diagonal 4 3 78. (d) According to question,
70. (c) Side of cube = =
= 4 cm
3 3 Surface area of sphere = 25% of 616
\ Volume of cube = (Side)3 = 43 = 64 cu cm 4p r2 = 154
71. (b) Let the radius of cylindrical rod = r and height = 4r 154 æ7ö
2

\ Required number of spherical balls Þ r2 = = ç ÷ Þ r = 3.5 cm


22 2
´4 è ø
Volume of cylindrical rod pr 2 (4r ) 7
= = =3 79. (b) According to question
Volume of spherical balls 4 3
pr Volume of cylindrical vessel = Volume of sphere
3
72. (b) Given, R = 3 cm, r = 2 cm, h = 10 cm 4
\ pr2h = p R3
Total surface area = 2p Rh + 2p rh + 2p (R2–r2) 3
= 60p + 40p + 10p 3
4æd ö
= 110p sq cm Þ (60)2 × 10 = ç ÷
73. (c) Given l = 6 m, b = 4 m and h = 1.25 m 3è 2 ø
Area of wetted surface 4 d3
= 2 (l × h + b × h) + 6 × 4 Þ (60)2 × 10 = ´
3 8
= 2 (7.5 + 5) + 24
Þ d 3 = (60)2 × (60)
= 25 + 24 = 49 sq m Þ d = 60 cm
74. (b) For conical part, 80. (d) Let side of a cube = a.
6 a
r = = 3 cm, h = 4 cm, Radius of sphere is unit.
2 2
l = h 2 + r 2 = 5 cm Volume of cube (a )3 6
Required Ratio = = =
Surface area of conical part = prl Volume of sphere 4 p æ a ö3 p
= 3.14 × 3 × 5 = 47.1 cm2 ç ÷
3 è2ø
6 81. (c) Surface area of sphere, S1 = 4pr2
For hemispherical part, r = = 3 cm
2 If radius is 2r, then surface area of sphere,
Surface area of hemispherical part = 2 pr2 S2 = 4p (2r)2 = 16pr2
= 2 × 3.14 × 3 × 3 = 56.52 sq cm \ S2 = 4S1
\ Surface area of toy = 47.1 + 56.52 = 103.62 sq cm Hence, it increases four times.
75. (c) Volume of a spherical shell 82. (d) Total surface area = Curved surface area of cylinder +
4 Curved surface area of cone + Top surface area of
= p(R 3 - r 3 ) cylinder
3 = 2 prh + prl + pr2
4
= p(2 ´ 3 ´ 4 + 3 3 + 4 + 3 ) (Q l = r 2 + h 2 )
2 2 2
= p(53 - 43 )
3 = p (24 + 15 + 9) = 48p sq cm
4 8
= p ´ 61 = 4 cm and Internal Radius(r)
3 83. (b) External Radius(R) =
2
244 p 4
= = = 2cm
3 2
76. (b) Curved surface of a cylinder = 1000 cm2 \ Volume of hollow sphere
\ 2 prh = 1000
Length of wire used in a round = Perimeter of cylinder’s base 4
= p (R 3 - r 3 )
= 2 pr 3
Height of cylinder h 4
Number of rounds = = = p(43 - 23 )
Diameter of wire 0.5 3
h 2prh 1000 4p
\ Required length of wire = 2pr ´ Þ = = ´ 56
0.5 0.5 0.5 3
= 2000 cm or 20 m Let h = Height of the cone
EBD_7367
270
A- Volume And Surface Area
According to question \ Total cost of painting at the rate of ` 5 per sq m.
Volume of cone = Volume of hollow sphere = 88 × 5 = ` 440
1 2 4 4 4
\ pr1 h = p ´ 56 89. (b) Q Volume of wax balls = p (1) 3 = p cm3
3 3 3 3
Þ (4)2 h = 4 × 56 4
Volume of sphere = p (8)3
4 ´ 56 3
Þ h=
16 \ Required number of balls
\ h = 14 cm
4
84. (b) Let height h1, radius r1, area S1 and volume V1 of first p(8)3
cylinder. 3 = 512
= 4
Similarly, for second cylinder, height h2, radius r2, area p
S2 and volume V2 3
According to question 90. (c) Curved surface area of the well = 2 prh
r1 2 22
= ...(i) = 2´ ´ 2 ´14 = 176 m2
r2 3 7
\ Expense of getting per square metre plastered
S1 5 2p r1 h1 5 = ` 25
\ = Þ =
S2 3 2pr2 h2 3 \ Expense of 176 m2 = 176 × 25
= ` 4400
h1 2
Þ ´ = 5 Þ h1 = 5 ...(ii) 91. (a) Radius of sphere (r) = 9 cm = 0.09 m
h2 3 3 h2 2 Diameter of wire (d) = 0.4 cm
2 Þ R = 0.2 cm = 0.002 m
V1 pr12 h1 2
æ r1 ö æ h1 ö æ 2 ö æ 5 ö 10 According to question,
\ = 2 = ç ÷ ç ÷ ç ÷ ç ÷=
=
V2 pr2 h2 è r2 ø è h2 ø è 3 ø è 2 ø 9 Volume of sphere = Volume of wire
85. (b) Total area of wetted surface = Curved surface area of 4 3
cylinder + Area of base of cylinder Þ pr = p R2 h
3
= 2prh + pr 2 = p[2 ´ 3.5 ´ 4 + (3.5) 2 ]
4 r3
22 22 \ h= ´
= (28 + 12.25) = ´ 40.25 3 R2
7 7
4 0.09 ´ 0.09 ´ 0.09
= 126.5 sq m = ´ = 81´ 3
86. (b) Let side of a cube be a unit. 3 0.002 ´ 0.002
\ Volume of cube, V = a3 = 243 m
and total surface area of cube, S = 6 (a)2 92. (c) Total surface area of cube = 6 × (Side)2
V = a3 \ 150 = 6 × (Side)2
Squaring on both sides 150
V2 = a6 Þ V3 = (a2)3 Þ Side2 = = 25
6
3
æSö \ Side = 25 = 5 cm
V2 = ç ÷ Þ 216 V 2 = S3
è6ø \ Volume of cube = (Side)3
87. (c) Given, R = 35 cm, r = 14 cm and h = 40 cm = 5 × 5 × 5 = 125 cm3
93. (b) Volume of cube = (Side)3
ph 2 \ 729 = a3
\ Volume of the bucket = (R + r 2 + Rr )
3 Þ a = 9 cm
22 ´ 40 \ Diagonal of cube = Side ´ 3 = 9 ´ 3 = 9 3 cm
= (352 + 14 2 + 35 ´ 14) 94. (a) Curved surface area of right circular cone = p r l
7´3
880 22
(1225 + 196 + 490) \ 440 = ´ 14 ´ l
= 7
21
880 440 ´ 7
= ´1911 = 80080 cu cm Þ l= = 10 cm
21 22 ´ 14
70 æ q ö
88. (b) Given that r = cm = 0.35 cm. h = 4 m 95. (a) length of the are = 2 pr ç ÷
2 è 360º ø
\ Surface area of cylinder Radius of arc(r) = 30 am
144
= 10 (2prh) = 10 æç 2 ´ ´ 0.35 ´ 4 ö÷ = 88 m
22
Length of the arc = 2pr = 2p × 30 × = 24 p
è 7 ø 360
Volume And Surface Area A-271
Let the radius of the cone = R \ 2pR = 24p
pH ïì Q P PQ ïü
30 ´144 = í + + ý
Þ R= = 12 cm 3 îï p p p þï
360
96. (d) Volume of cylinder = pr12 h H
= (P + Q + PQ )
3
4 3
Volume of ball = pr2
3 P
100. (a) Area of first end P = pr2 Þ r =
Number of balls = 48 p
Volume of cylinder pr12 h Q
\ Number of balls = = 4 Area of second end Q = pR2 Þ R =
Volume of balls p r23 p
3
r
p r12 h pr13
Þ = 48 Þ = 48 (Q r1 = h)
4 3 4 3
p r2 pr2
3 3
3 3 H
3 æ r1 ö æ r1 ö 48 ´ 4
Þ ç ÷ = 48 Þ ç ÷ =
4 è r2 ø è r2 ø 3

r1 r 1
Þ = (16 ´ 4)1/3 Þ 2 = R
r2 r1 4
Thus, the ratio of radius of ball to cylinder is 1 : 4. According to question P < Q
97. (a) Volume of the cuboid = 720 cm3 \ Difference in radii of the ends of the frustum
Volume of the cuboid =R–r
Height of the cuboid =
Base area of the cuboid Q P Q- P
= - =
lbh 720 p p p
= = = 10 cm
lb 72 101. (d) Thickness of 10 circular plates are 3 cm cach.
Surface area of cuboid = 2(lb + bh + hl) \ Height of the cylindrical (h) = 3 × 10 = 30 cm
484 = 2 (72 + 10 (l + b)) Radius of the hemisphere (r) = 6 cm
484 – 144 = 20(l + b) \ Volume of the solid
340
= (l + b) Þ 17 = l + b 6 cm
20 6 cm
By checking options, we get option (a) is the correct
dimensions of the cuboid.
98. (a) Curved surface area of the sphere = 4pr2
Þ 616 = 4pr2
616 154 ´ 7 30 cm
Þ pr2 = = 154 Þ r 2 = = 49
4 22
\ r= 49 = 7 cm
4 3
\ Volume of the sphere = pr
3 = Volume of cylinder + Volume of hemisphere.
4 22 4312 3 2 3
= ´ ´7´7´7 = cm . = pr h + pr
2
3 7 3 3
99. (c) Volume of frustum
é 2 2 ù
pH 2 = p ê (6) ´ 30 + ´ 6 ´ 6 ´ 6ú
= (R + r 2 + Rr ) ë 3 û
3
= p[36 × 30 + 2 × 72]
p ìïæ Q ö æ P ö Q P üï = p[1080 + 144] = 1224 pcm3.
2 2

H íç ÷ + ç ÷ + ý Therefore, the volume of the solid = 1224 p cm3.


= 3 ç p÷ ç p÷ p pï
ïè
î ø è ø þ 102. (b) Let the side of cube = a
According to question
EBD_7367
272
A- Volume And Surface Area

a Þ 2prh + 2pr2 = 660


Height = a, radius = Þ 2pr (h + r) = 660
2
330 330 7
a Þ (h + 5) = = ´
r= ,h=a 5p 5 22
2
66 ´ 7
Þ h= - 5 = 21 - 5 = 16 cm.
22
107. (a) Given that, let the diameters of two spheres are d1 and
d2, respectively.
\ d1 : d2 = 3:5
‘C’ ‘S’ \ Ratio of their surface areas
a 4pr12 (2r1 )2 d12
Radius of sphere (R) = = = =
2 4pr22 (2r2 )2 d 22
\ Volume of cone (C)
2 2
1 2 1 æaö
2
pa 3 æ d1 ö æ 3 ö 9
= pr h = p ç ÷ a = ...(i) = ç ÷ =ç ÷ = = 9 : 25
3 3 è 2ø 12 d
è 2ø è ø 5 25

4 4 æaö
3 108. (c) The diameter of largest sphere that is carved out from
Volume of sphere (S) = p R = p ç ÷
3
cube is equal to side of the cube.
3 3 è2ø
Diameter of a sphere = Side of the cube = 3 cm
pa 3 3
= ...(ii) \ Radius = cm
6 2
On solving eqs. (i) and (ii),
S = 2C
103. (a) Given that, the height and radius of a right circular
metal cone (solid) are 8 cm and 2 cm, respectively.
i.e., h = 8 cm and r = 2 cm
Let the radius of the sphere is R.
1 2 4
Then, by condition, pr h = p R 3
3 3 \ Volume of the largest sphere
Þ 4 × 8 = 4R3 3
Þ R3 = (2)3 4
p (radius)3 = p æç ö÷
4 3
=
\ R= 2 3 3 è2ø
\ Radius of the sphere = 2 cm
104. (c) Let the edge of a square x. Then, its volume = x3 and 4 27 9
= p = p = 4.5p cm3
sum of its edges = 12x 3 8 2
According to question
109. (d) Radius of circular cone (r) = 12 m
x3 = 12x Þ x(x2 – 12) = 0
Þ x2 = 12 (Q x ¹ 0) Height of a circular cone (h) = 5 m
\ Total surface area = 6x2 = 6(12) = 72 sq units \ Slant height (l) = r 2 + h2 = 144 + 25 = 169
105. (a) Diameter of a cone = 7 cm
= 13 m
7 \ Required quantity of cloth to roll up to form a
\ Radius of cone = cm
2 circulartent = pr l = p (12) (13) = 156 p sq m
110. (b) Given that,
Length of a box (l) = 14 m
l = 10 cm
Breadth of a box (b) = 12 m
Height of a box (h) = 13 m
d = 7 cm The greatest rod in a box is equal to diagonal of a box
Slant height of a right circular cone (l) = 10 cm = l 2 + b 2 + h 2 = (14) 2 + (12) 2 + (13) 2
\ Lateral surface area of a cone = p r l
22 7 = 196 + 144 + 169
= ´ ´ 10 = 11´ 10 = 110 cm2
7 2 = 509 = 22.56 m
106. (d) Radius (r) = 5 cm 111. (a) Given that,
Total surface area = 660 cm2 Outer radius of hemispherical shell (R) = 9 cm
Volume And Surface Area A-273

r = 7 cm Slant height of cone (l) = 5 cm


9 cm = R
l = r 2 + h2 = 5
Þ r2 + h2 = 25 Þ r2 = 25 – 9 = 16
\ r = 4 cm
1 2
\ Volume of cone = pr h
3
and inner radius of hemispherical shell (r) = 7 cm
1 22 352
\ Volume of a hemispherical shell = ´ ´ 16 ´ 3 = = 50.3 cu cm
3 7 7
2 2 22 115. (a) Given that,
= p(R 3 - r 3 ) = ´ ´ (729 - 343)
3 3 7 Height of bucket = 25 cm
Radii of top (R) = 20 cm
2 22 16984 radii of bottom (r) = 10 cm
= ´ ´ 386 = = 808.76
3 7 21
» 808 cu cm (approx) 20cm
112. (b) Let sphere radius = r
Volume of sphere = 36 p
25cm
4 3
Þ p r = 36 p
3
Þ r3 = 27 = (3)3
\ r = 3 cm 10cm
\ Diameter of sphere \ Capacity of bucket
= 2r = 2 (3) = 6 cm p
= h(R2 + r2 + Rr)
Surface area of sphere = 4pr 2 = 4 p(3) 2 = 36p sq cm 3
Surface area of sphere 36p p
Ratio = = = 6p = × 25 (400 + 100 + 200) cm3
Diameter of sphere 6 3
113. (d) Internal diameter of the tube = 6 cm p
= × 25 × 700 cm3
3
6 3 cm
p 175 ´ 100 17.5p
= × = L
3 1000 3
116. (d) Given that, h = 15 cm
10 cm
and lateral surface = 660 cm2
Let radius of cylinder = r
Lateral surface area of cylinder = 2 prh
Þ 2 prh = 660
4cm 22
Þ prh = 330 Þ ´ r ´ 15 = 330
7
\ Internal radius (r) = 3 cm
Height of the tube (h) = 10 cm 22
Þ ´ r = 22
Thickness of the metal = 1 cm 7
\ Outer radius (R) = Thickness of the metal + \ r = 7 cm
Internal radius = 1 + 3 = 4 cm \ Volume of cylinder
\ Outer curved surface area 22
= pr2h = ´ 49 ´ 15 = 22 × 7 × 15 = 2310 cu cm.
= 2prh + 2 p (R - r )
2 2 7
117. (a) Let height and radius of cylinder is h and r respectively.
= 2p(3)(10) + 2p(16 - 9) Volume of cylinder = pr2h
= 60 p + 14 p = 74p sq cm
114. (b) Given that, height of cone (h) = 3 cm

3cm 5 cm h

r r
EBD_7367
274
A- Volume And Surface Area

1 2
Volume of circular cone = pr h
3
l
Volume of utilised wood 16cm
\ Required ratio =
Volume of wasted wood
Volume of right circular cone 12cm
= (Volume of right circular cylinder 24 cm
–Volume of right circular cone) 22
Curved surface area = prl = ´ 12 ´ 20
1 2 1 2 7
pr h pr h
3 3 1 22
= Þ = = 1: 2 Cost of painting = ´ 12 ´ 20 ´ 0.70 = ` 528
1 2 2
pr 2 h - pr 2 h pr h 2 7
3 3
118. (a) Number of spheres 125. (b) Slant height, l = h2 + r 2
Volume of a solid sphere with radius 4 cm
=
Volume of a solid sphere with radius 2 cm
4 l
p(4)3 h
3 (2)6
= = = (2)3 = 8
4 (2)3
p(2) 3

3
O r
1
119. (c) Volume of cone and pyramid = × Base area × Height
3
Since, volume of cone and pyramid are same but their = (24) 2 + (7) 2 = 576 + 49 = 625 = 25
surface area are not same because of their slant height. Total surface area = prl + pr2
120. (b) The diameter of Moon is approximately one-fourth of 22 22
the diameter of Earth. = ´ 7 ´ 25 + ´ 7 ´ 7 = 704 sq cm.
Let radius of Moon = r, 7 7
then radius of Earth = 4r 126. (c) Slant height of the cone = 52 + 122
Required ratio =
= 25 + 144 = 169 = 13 m
4 3
pr
Volume of Moon 3 r3 1
= = = = 1: 64
Volume of Earth 4 64 r 3
64
p(4r ) 3
5m
3
121. (a) Total surface area of a cube = 6a2
Þ 6 = 6a2 Þ a2 = 1 12 m
\ a = 1 unit 16 m
Volume of the cube = a3 = 13 = 1 cu unit
122. (d) Cube figure Pyramid figure 11 m

24 m
Edges, a = 12 Edges, a = 8
Faces, b = 6 Faces, b = 5 Curved surface area for conical portion = prl
Corner, c = 8 Corner, c = 5 22 3432
Therefore, the result a = b + c is neither true for cube = ´ 12 ´ 13 = sq m
7 7
nor for the pyramid.
127. (d) According to question,
1 2 1 22 Curved surface area of cylinder = 2 p rh = x
123. (a) Volume of cone = pr h = ´ ´ 10 ´ 10 ´ 21 Volume of cylinder = pr2 h = y
3 3 7
= 2200 cm3 2p rh x
Þ =
p r2h y
124. (c) l = h2 + r 2 = 162 + 122 = 256 + 144
2y
= 400 = 20 cm Þ r=
x
Volume And Surface Area A-275
Now, Curved surface area of cylinder 132. (a) Surface area of cube = 6 (Side)2
Þ 2prh = x = 6(2)2 = 24 cm2
Surface area of cuboid
x x2 = 2(lb + bh + lh)
\ h= =
2 pr 4py = 2(2 + 6 + 3) = 22 cm2
Total surface area of both cube and cuboid
= 24 + 22 = 46 cm2
Give area to point is 54 cm2
\ Required ratio = But total surface area which is need to be painted is
46 cm2. So both, cube and cuboid painted.
So, Neither (A) nor (B) is correct. 133. (b) Let the cone is divided into two parts by a line l.
128. (a) Let common radius = r cm A
Height of cylinder = h1
Height of cone = h2 h/2
\ Volume of the complete structure l
C D h
1 2 2 r/2
= pr h2 + pr 2 h1 + pr 3
3 3
O r
2 æ h2 2 ö B
= pr ç + h1 + r ÷ Now triangle ACD and AOB are similar.
è 3 3 ø
(According to proportionality theorem)
2 æ 2.8 2 ö r h
= p(3.5) ç + 6.5 + ´ 3.5 ÷
CD = , since AC =
è 3 3 ø 2 2
= p´ 3.5 ´ 3.5 ´ 9.76 = 375.86 cm3
Volume of original cone
Volume (V) of the structure lies between 370 cm 3 and Required ratio =
380 cm3. Volume of smaller cone
6 1 2
129. (b) Radius of the cone = = 3 cm pr h
2 3 8
= 2
=
Height of the cone = 4 cm 1 ær ö æhö 1
pç ÷ ç ÷
3 è2ø è2ø
\ Required ratio = 8 : 1
4 134. (c) Volume of each small sphere
Volume of bigger sphere
=
Number of small spheres
3
4
l = r 2 + h 2 = 32 + 42 = 9 + 16 = 5 cm p(4)3
3 4 p´ 4 ´ 4 ´ 4 4
Curved surface area = prl = = ´ = p cm3
64 3 64 3
\ Curved surface area = p × 3 × 5 = 15p » 47 sq cm
130. (d) Volume of clay required Let radius of small sphere = R
4 4
éæ 5.1 ö2 æ 4.5 ö2 ù \ p R 3 = p Þ R = 1 cm
= p êç ÷ -ç ÷ ú ´ 21 3 3
êëè 2 ø è 2 ø úû Surface area of small sphere = 4p R2 = 4p cm2
= p [(2.55)2 – (2.25)2] × 21 135. (c)
= p (0.3 × 4.8) × 21 = 30.24 p cm3
131. (d) The stack is in the form having vertical cross section
of trapezium.
12 m 8m
\ Volume of Earth in the stack = Area of cross
section of trapezium × Height
2.5 m
1
\ Volume = ´ (10 + 5) ´ 2 ´ 8 = 15 ´ 8 m2 15m
2
According to the question, Volume of pit = l b h = 8 × 2.5 × 2 = 40 m3.
Volume of Earth to be spread = (Area of field) × Level Let the label of the earth spread over remaining area = h.
raised Volume of the earth spread = Volume of a pit
15 ´ 8 15 Þ [(12 × 15) – (8 × 2.5)] × h = 40
\ Level raised = = m = 1.5 cm 40
100 ´ 80 1000 40 1
\ h= = = m = 25 cm
180 - 20 160 4
EBD_7367
276
A- Volume And Surface Area
136. (a) Surface area of sphere = 616 cm2 From equations (i) and (ii), it is clear that volume of
4pr2 = 616 both cones are not same. So, statement 1 is not correct.
616 ´ 7 2. The triangle revolve about hypotenus, then we get
Þ r = 2 double cones ABD and BCD.
4 ´ 22 B
Þ r2 = 7 × 7
\ r = 7 cm
\ Diameter of largest circle which lying
on sphere = 2 × r = 14 cm
137. (c) Volume of the cube = 216x3
A C
(Side)3 = 216x3 Þ Side = 6x E
Largest sphere which is enclosed in cube the diameter
of sphere is equal to side of the cube.
\ Diameter of sphere = 6x
2
æ 6x ö D
Surface area of the sphere = 4pr2 = 4p ç ÷ = 36px 2 Q DBEA ~ DBAC
è 2 ø
138. (a) Let radius of the sphere be r. BE AB BE 3
\ = Þ =
Cylinder circumscribes a sphere it means BC AC 4 5
that radius of the base of cylinder = r = sphere radius. BE = 2.4 cm
and height of cylinder = 2r = Diameter of sphere Radius of the base of cone, BE = 2.4 cm
In right angled DBEA,
By Pythagoras theorem,
AE = ( AB )2 - ( BE ) 2 = 9 - (2.4) 2 = 1.8 cm
h
Height of cone ABD = AE = 1.8 cm
\ Height of cone BCD = AC – AE = 5 – 1.8 = 3.2 cm
1 2 1
Now, volume of cone ABD = pr h = p (2.4)2 × 1.8
3 3
4 = 3.456p cm3
Volume of sphere = pr3 1
3
volume of cone BCD = p(2.4)2 × 3.2 = 6.144p cm3
Volume fo cylinder = pr2h = pr2 (2r) = 2pr3 3
4 3 Volume of double cone = 3.456p + 6.144p
pr
4 2 = 9.6p cm3 ... (iii)
3 = = =2:3 From equations (i) and (ii), we get
\ Required ratio =
2pr 3 3 ´ 2 3 Volume of both cones = 12p + 16p = 28p cm3 ...(iv)
139. (d) DABC is right angled triangle. From equations (iii) and (iv), we get
AB = 3cm, BC = 4cm and AC = 5cm Volume of double cone Volume of both cones
1. When the triangle revolves about its longer leg, BC= So, Statement 2 is also not correct.
4 cm.
xy
A 140. (c) x + y +
100
100 ´100
= 100 + 100 +
5 100
3 = 200 + 100 = 300%
141. (c) \ Surface area of sphere A = 4pr2 = 4p(6)2 = 144p cm2
Surface area of sphere B = 4p(8)2 = 256p cm2
C Surface area of sphere C = 4p(10)2 = 400p cm2
B 4 and Surface area of sphere D = 4p(12)2 = 576 p cm2
Sum of surface area of spheres A and B
1 2 1
Volume of cone pr h = p(3)2 × 4 = 144p + 256p = 400p cm2
3 3 = Surface area of sphere, C
= 12p cm3 .... (i) Hence, Statements 1 is correct.
Now triangle revolve about its shorter leg,
AB = 3 cm 2. Q Volume of sphere D = 4 pr 3 = 4 p(12)3
3 3
1 2 1
Volume of cone = pr h = p(4)2 × 3 = 2304p cm3
3 3
= 16p cm3 4 3
.... (ii) Volume of sphere A = p(6)3 = 288p cm
3
Volume And Surface Area 277
A-

4 2048 4 3
Volume of sphere B = p(8)3 = = p cm3 pr
3 3
Ratio of their volume = 3
4 4000 a3
and Volume of sphere C = p(10)3 = = p cm3
3 3 3
4 ærö 4p 6 6 6
According to question = pç ÷ = . =
sum of volumes of sphere A, B and C 3 èaø 3 4p 4p p
2048p 4000 ö 3 6
= æç 288p + + p ÷ cm Square of their volume ratio = =6:p
è 3 3 ø p
146. (a) Radius of sphere = Radius of right circular cone
864 + 2048 + 4000 6912
= p cm3 = p cm3 Now, Volume of sphere = 2 × Volume of cone
3 3 4 3 1
= 2304p cm3 = Volume of sphere D Þ pr = 2 ´ pr 2 h
Hence, Statement 2 is also correct. 3 3
142. (a) Diameter of a sphere, d = 6 cm Þ 2r = h
d 6 h 2
\ Radius of a sphere, r = = = 3 cm = = 2:1
2 2 r 1
Let the radius of wire be R cm. 147. (b) Area of cross-section of cylindrical pipe
Also, given the length of wire, H = 36 m = 3600 cm 2
æ7ö 49p 2
According to question = p´ ç ÷ = cm
Volume of sphere = Volume of wire è2ø 4
4 3 Volume of water flow per minute
Þ pr = pR 2 H
3 49p
= × 5 × 100 × 60 cm3
4 4
Þ ´ (3)3 = R 2 ´ 3600 Volume of the tank = 3 × 4 × 231 × 10000 cm3
3
Now, time taken to fill up tank
4 ´ 32 (6)2 3 ´ 4 ´ 231´10000
Þ R2 = = = = 24 min.
3600 (60)2 49 22
´ ´ 5 ´100 ´ 60
4 7
6 1
\ R= = = 0.1 cm 148. (d) Surface area of cone = prl
60 10
4 3 l = r 2 + h2
143. (c) Volume of sphere = pr
3 = (7)2 + (24)2
Here,
Volume of Sphere = Volume displaced in cylinder = 49 + 576 = 625 = 25 cm
4 Surface area of cone = prl
Þ prS 3 = prC 2 (h – h ') = p × 7 × 25
3
= 175p
4 Total surface area with hemisphere
Þ p´ 27 = p × 16 (h – h')
3 = 175p + 2pr2
9 = 175p + 2 × p × 49
h –h' = = 2.25cm
4 = 273p cm2
xy + yz + zx xyz
144. (a) x + y + z + + 5m
100 (100)2
149. (c) 1.7
Increase in volume of sphere
10 ´ 10 + 100 + 100 10 ´ 10 ´ 10
= 10 + 10 + 10 + +
100 (100)2
= 30 + 3 + .1 = 33.1% 2m
145. (a) According to question
4pr 2 = 6 a 2
3m
r2 6
= Surface Area of rectangular blocks
a 2 4 p = 2 (3 × 2 + 2 × 1.75 + 3 × 1.75) = 29.5 m2
EBD_7367
278
A- Volume And Surface Area
Paint required for 0.1 mm thickness 4 3 4
1 Þ pr = p(1728)
= 29.5× = 0.00295m3 3 3
10,000 r3 = 1728
Volume of cubical boxes \ r = 12
10 10 10 1 153. (b) Volulme of cone
= ´ ´ = cm3
100 100 100 1000 æ x = 200 ö
0.00295 xy + yz + zx xyz ç y = -50 ÷
=x+y+z+ + ç ÷
So boxes required = = 2.95 » 3 100 (100)2 çè z = -50 ÷ø
0.001
150. (c) Diagonals of the three faces are 13, 281 and 20
{200 ´ (–50)} + {200 ´ (–50)} + 50 ´ 50
Let the sides of cuboid be l, b and h respectively = 200 –50–50+
l2 + b2 = (13)2 = 169 ... (i) 100
2 2 2
b + h = ( 281 ) = 281 ... (ii)
+
{200 ´ ( -50 ) ´ ( -50 )}
h2 + l2 = (20)2 = 400 ... (iii) 100 ´100
On subtracting equation (ii) from equation (iii) = 100 + (–100 – 100 + 25) + 50
l2 – b2 = 400 – 281 = 119... (iv) = 100 – 100 – 100 + 25 + 50
Now add equations (iv) and (i) = – 25%
2l2 = 119 + 169 Decrease by 25%
154. (c) Area of cross-section of nozzle = 4 cm 2
288 Velocity of water = 40 m/s = 4000 cm/sec
l2 = = 144 Þ l = 12
2 Water coming from nozzle in 1 sec
Put l = 12 in equation (i) = 4 × 4000 = 16000 cm3
b2 = 169 – 144 Dimension of cistern = 10 × 8 × 6 m 3
b2 = 25, b = 5 = 480 × 106 cm3
Now, put value of b in equation (ii)
b2 + h2 = 281 480´106
Time taken to fill the cistern = sec
25 + h2 = 281 16000
h2 = 256
480 ´106 1
Q h = 16 = ´ hours
Total surface area of cuboid 16000 3600
= 2(lb + bh + hl) 4800
= hours
= 2(12 × 5 + 5 × 16 + 16 × 12) 16 ´ 36
= 2(60 + 80 + 192)
= 664 square units 25
= hours
3
151. (c) A 44 cm D = 8 hours 20 minutes.
So, option (c) is correct.
155. (d) Number of cylindrical boxes that can be packed in the
6 cm drum
2
p´(15) ´100
B C = 2 = 90
p´(5) ´10
Radius of rolled cylinder
Þ 2pr = 44 So, option (d) is correct.
156. (c) Curved surface area of cylinder = 2prh
22
Þ 2´ ´ r = 44 Þ r = 7 cm S = 2prh
7 If height is doubled-
152. (a) Volume of new sphere S¢ = 2pr (2h)
V = V1 + V2 + V3 S¢ = 4prh
4 3
Formula for volume of sphere is pr S¢ = 2S
3
Total surface area of hemisphere = 3pR2
4 3 4 3 4 3 If radius is doubled-
Þ V = pr1 + pr2 + pr3
3 3 3 total Surface Area = 3p(2R)2
= 12pR2 = 4 × 3pR2
4 = 4 × (Initial Surface Area)
= p (216 + 512 + 1000)
3 So option (c) is correct.
Volume And Surface Area 279
A-

157. (a) Let side of cubical water tank be ‘x’ meter. 2 3


Capacity of tank = x3 Þ 3312p = pr2h + pr
According to question- 3
Þ x3 – 128 = (x – 2).x2 é 2 2 3ù
Þ x3 – 128 = x3 – 2x2 Þ 3312p = p êê(12) h + (12) úú
ë 3 û
Þ 2x2 = 128
Þ x2 = 64 2
Þ 3312 = 144 [h + ×12]
Þ x = 8 metre 3
Capacity of tank (8)3 = 512 m3 Þ h + 8 = 23
So, option (a) is correct. Þ h = 15 metre
158. (c) Volume of solid cubical gold = (1)3 = 1 cm3
Let radius of spherical solid balls be r. Surface area of hemisphere 2pr 2
=
Surface area of cylinder 2prh
1 1
1
p3 p3 r 12 4
4pr2 = p 3 = r2 = = r= = = =
4p 2 h 15 5
So, option (b) is correct.
1 162. (b) Let sides of cuboid be a, b, c
No of balls =
æ 1 ö‚3 x = a2
çç - ‚ y = b2
4 çç p 3 ‚‚‚ z = c2
pç ‚
2 çè 2 ø‚ volume of cuboid V = a. b. c
V2 = a2 b2 c2
3 8
= 4p ´ - 1 = 6 V2 = x . y. z
p So, option (b) is correct
So, option (c) is correct 163. (a) y
159. (c) At sometimes, the contoiner was full. (0, y)
At 25 minutes, it was holy full. (x, y)
At 20 minutes, it was the fourth full. A B
So, option (c) is correct.
C (0, y)
160. (c) Let the radius and height of each cylinder be ‘r’ and D 2
‘h’ respectively. water
Volume of 30 metallic cylinders
= 30 × p2h
Let the no. of cones casted be ‘N’
1 2 O (0, 0) x
N× pr h = 30 pr2h
3
N = 90
The curved surface of cylinder in reactangle and
1 2
curved surface of cone is semi-circle when they are V= px y -------------(i)
flattened. 3
So, option (c) is correct. D COD and D AOB are similar
CO CD
161. (b) A D =
AO AB
y
h 2 = CD
y x
O x
B C CD =
2
1 ( )2 y 1 æ x ö‚2 y
12 cm Volume of water = p CD . = p.çç ‚ .
3 2 3 çè 2 ø‚ 2
P 1 é1 2 ù 1
= ê px y ú = v
Total capacity 8 êë 3 úû 8
= volume of cylinder + volume of hemisphere So, option (a) is correct.
EBD_7367
280
A- Volume And Surface Area
164. (c) Let length of the cube be a 170 (c) R = 8.4m, h = 3.5 Space required for 1 bag of wheat =
6a2 = 13254 1.96 m3
a2 = 2209 Let number of bag = n
a = 47 1 2
n (1.96) = pr h
Length of diagonal = 47 2 cm 3
165. (a) No. of bullets 1 22 8.4 ´ 8.4
n= ´ ´ 3.5 = 132
44´ 44 ´ 44 3 7 1.96
= 4 ´ 22 ´ æç 4 ö÷ ´ æç 4 ö÷ ´ æç 4 ö÷ 171. (a) Radius of hemisphere = r
3 7 è 2ø è 2ø è 2ø
11´11´11´ 21´8
= = 2541 r
22 ´ 4
So, option (a) is correct.
166. (b) Water running per hour in the river
= 2.5 × 45 × 3600 m3. 3r
Water running per minutes in the river
2.5´ 45´3600 3
= m
60
= 2.5 × 45 × 60 radius of cylinder = r
= 6750 m3 height of cylinder = 2r
So, option (a) is correct Total volume = Vhemisphere + Vcylinder
167. (b) area of 4 wall = 2 (l + b) h 2 3 1
= p r + p (r) 2 (2r) = 67
l = 2b b=b h=4 3 21
2 (2b + b) × 4 = 120
3b = 15 3 é1 ù 1408
= 2pr ê + 1ú =
b= 5 ë3 û 21
l = 2 × 5 = 10
area = 5 × 10 = 50 m2 2 ´ 22 4 3 1408
´ r =
7 3 21
168. (c) C.S.A cylinder : T.S.A. cylinder = 1 : 2 1408
r3 =
2 prh 1 h 2 ´ 22 ´ 4
= =
2pr(r + h) 2 r + h r 3 = 8 = 23
r=2
Þ 2h = h + r
height of building = 3r = 6
h=r
172. (c) Let radius and height of cylinder be 2x and 3x
Þ Gives T.S.A = 616 cm2
Respectively
2pr [r + r] = 61.6 cm2 pr2 h = 1617
4p r2 = 616 p (2x)2 3x = 1617
616 ´ 7 p12x3 = 1617
r2 = = 49
22 ´ 4 3
1617 ´ 7 æ 7 ö
r=7 x3 = =ç ÷
h=7 22 ´12 è 2 ø
22 7 21
Volume pr2h = ´ 7 ´ 7 ´ 7 = 1078 cm3 x= ;r =7 h=
7 2 2
169. (b) Let side of square be a Total surface area = 2pr (r + h)
Volume = 1 2 ´ 22 é 21 ù
a2 × 9 = 1 = ´ 7 ê7 + ú
7 ë 2û
1
a= é 35 ù
3 Þ 44 ê ú Þ 22 × 35 Þ 770 cm2
ë2û
1
Volume of cube = a3 = m3 14
27 173. (c) Radius of sphere = cm = 7 cm
2
1 1000
Required weight = 9000 × = kg 22
27 3 Total surface area = 4pr2 = 4 ´ ´7´7
7
Volume And Surface Area 281
A-

Total cost of painting = `8008 180. (d) Canvas used = CSAcylinder + CSAcone
8008 = 2prh + prl
Cost per square cm = = ` 13 per cm2 22
4 ´ 22 ´ 7
= 1980 + × 63 × 65 = 14850 m2
174. (a) Frustum 7
4 18 181. (b) Let side of square = x
r= ,R= , h = 24 perimeter = 4x
2 2 Circumference of circle = 4x
p é2 2 2pr = 4x
Volume = h r1 + r2 + r,r2 ù
3 ë û 4x 2x
r= =
22 2p p
px = ´ 24 [(2)2 + (9)2 + 2 × 9] = 824 cm3
7´3 2
175. (c) Volume of roof = volume of cylindrical vessel æ 2x ö
Area Circle xç ÷
p
22m × 20m × xm = pr2h = è 2ø
Area Square x
22
22 × 20 × x = ´ 1 ´1 ´ 3.5
7 p 4x 2 4 ´ 7 14
= = =
22 ´ 3.5 .5 2
p x 22 11
x= = = .025 m = 2.5 cm
722 ´ 20 20 1 2
182. (d) Volume of cone when R = r H = h = pr h
176. (d) 3
Volume of cone when R = r + p%r H = n
2
60 1 æ pr ö
V= pçr + ÷ h
3 è 100 ø
2
1 é pr ù 1
increase = ph ê r + ú - pr 2 h
3 ë 100 û 3

1 é 2 æ pr ö 2rpr 2 ù
2
Volume of cone be V1 and V2 p ê +
= 3 h r ç ÷ + -r ú
êë è 100 ø 100 úû
V1 64 r2h é r hù
= = , ê = ú \similar triangles
V2 1 2
R H ë R H û 1 r2p é P ù
= ph ê R+
h1 æ 43 ö 4 3 (100) ë 100 úû
=3ç ÷ =
h2 ç 1 ÷ 1
è ø 1 é P ù
phr 2 p ê R +
3 ë 100 úû
1 ´ 100
= ´ 60 cm = 15 cm %= 1 2
4 pr p100
3
height of small cone = 15 cm
height from base to where cut = 60 – 15 = 45 cm é P ù
= P êR +
4 3 ë 100 úû
177. (a) Volume of sphere where R is 3 cm = pr
3 or
4 P´P æ P ö
= p3 ´ 3 ´ 3 = 36p cm3 P+P+ = Pç2 + ÷
3 100 è 100 ø
126 183. (b) When r = r Surface area = 9
178. (b) Radius cone = = 63m
2 When r = x Surface area = 1
height of cone = 21 – 5 = 16 m
9 4 pr 2
slant height = =
632 + 162 = 3969 + 256 1 4 px 2
= 4225 = 65 m 2
179. (a) Radius of cylinder = 63 m r2 æ r ö
x2 = =ç ÷
height = 5 m 9 è3ø
22 1 rd
CSA = 2prh = 2 × × 63 × 5 x=
7
3
CSA = 1980 m2
EBD_7367
282
A- Volume And Surface Area
184. (c) Cylinder Cone Surface area of the sphere
35 4 = 4pr 2 = 4.p.4 x 2 = 16px 2
Radius = / 2cm Radius = cm
2 2 Ratio of CSA of the cylinder and surface area of the
height = 32 cm height = 7 cm
4 3px 2 3
1 2 sphere = = = 3:4
volume = p(rs)2h Volume = pr h 16px 2
4
3
Volume of cylinder = volume of n cone 187. (d) Since the sphere is dropped in the cylindrical vessel
partially filled with water and is completely immersed,
2 therefore the volumes of 3d-shapes will be equal.
æ 35 ö np
p ç ÷ 32 = (2) 2 7 Let r be the radius of cylinder and R be the radius of
è ø2 3
the sphere.
n = 1050
185. (b) Since HCF of two polynomials is x2 + x – 2, therefore Thus, volume of cylinder = volume of sphere
splitting this polynomial by middle term, we get 4 3 4
pr 2 h = pR Þ 30 ´ 30 ´ h = ´ 15 ´ 15 ´ 15
2 2
x + x - 2 = x + 2 x - x - 2 = x( x + 2) - ( x + 2) 3 3
= (x – 1)(x + 2) 4 ´ 15 ´ 15 ´ 15
Þh = Þ h = 5 cm
Being the HCF of the given polynomials, we conclude 3 ´ 30 ´ 30
that (x – 1)(x + 2) is a factor of f (x) and g (x). 188. (c) Let the radius of the base of the cone be r cm.
Therefore, dividing f (x) by the HCF and dividing g (x) Slant height = 2r cm (given)
by the HCF will give the values of a and b.
Height of the cone =
( x - 1)( x 2 + 3 x + a ) ( x 2 + 3 x + a )
Now, = l 2 - r 2 = ( 2r )2 - r 2 = 2r 2 - r 2 = r 2 = r
( x - 1)( x + 2) ( x + 2)
1 2 1 pr 3
( x + 2)( x 2 + 2 x + b ) ( x 2 + 2 x + b ) Volume of the cone = pr h = pr 2 .r =
and = 3 3 3
( x - 1)( x + 2) ( x - 1)
Since (x + 2) is a factor of (x2 + 3x + a), therefore, x = – 189. (d) A
2will satisfy this polynomial. Thus,

x 2 + 3x + a = 0 Þ (-2)2 + 3. - 2 + a = 0
Þ 4 - 6 + a = 0 Þ -2 + a = 0 Þ a = 2 h
Also, since (x – 1) is a factor of x2 + 2x + b, therefore, x r
= 1 will satisfy this polynomial. Thus, B' O C'
2
x + 2 x + b = 0 Þ 1 + 2 + b = 0 Þ b + 3 = 0 Þ b = -3
x
Hence, a = 2, b = – 3

R
B O C
Let the radii of frustum of a cone be r1 and r2.
2x R 2
We are given that = ...(1)
O 2x r 1
x 2x Now, let ÐAC ' O ' = ÐACO = q
186. (a)
h
A B Therefore, in DAC¢O¢, tan q = = h ...(2) (by (1))
r2
h+ x h+ x
In DACO, tan q = = ...(3)
In DOAB, by Pythagoras theorem, r1 2
Radius of cylinder = h+ x
(2) and (3) Þ h = Þ h + x = 2h Þ x = h .
2 2 2 2 2 2 2
OB - OA = (2 x) - x = 4 x - x
H 2
= 3x 2 = 3x Therefore, H = 2h Þ =
h 1
CSA of the cylinder Now, volume of frustum of the cone
2
= 2prh = 2.p. 3x.2 x = 4 3px
Volume And Surface Area 283
A-

ph 2 Curved surface area of cone = prl = p × x × 3x = 3x2p


= ( R + Rr + r 2 ) Required ratio = 4x2p : 3x2p = 4 : 3
3
195. (b) Let volume of smaller cone be 64 unit and volume of
1 2 frustum be 61 unit
and volume of the cone = pr h
3 Total volume of bigger cone = 64 + 61 = 125 units
Here smaller cone is cut from bigger cone then
ph 2
( R + Rr + r 2 ) respective ratio of their radius, hight will be equal
3 h( R 2 + Rr + r 2 )
Ratio = = \ Respective ratio of area of bigger cone to that of smaller
1 2 R2H
pR H cone
3
( ) ( )
2 2
2 2 2 2 = 3 125 : 3 64 = 25 :16
h(4r + r + 2r ) 7r 7
= = 2 = = 7:8
2
4r .2h 8r 8 Respective ratio of area of smaller cone to that of
190. (d) Since the conical cavity in the cylinder is hollowed frustum
out, therefore inner surface area of the cavity is curved = 16 : 25 – 16 = 16 : 9
surface area of the cone. 196. (c) Side of the room = 10m
Area of the room = (10)2 = 100m2
2 2
Therefore, CSA of the cone = prl = p.6. 8 + 6 Side of table = 2m
= p.6. 64 + 36 = 100 = p.6.10 = 60p 3
´ ( 2) = 3 = 1.73m 2
2
2 Area of table =
191. (b) Volume of cylinder = pr h = p.50.50.10 4
= 25000p cu. m Area of 4 Book-shelves = 4 × (4 × 1) = 16m2
1 2 1 Area of remaining room = 100 – 1.73 – 16 = 82.27 m2
Volume of cone = pr h = p.50.50.(15 - 10)
3 3 82.27
Required cost = ´ 100 = 4113
1 12500 p 2
= p.50.50.5 = cu. m 197. (b)
3 3
198. (c) edge of cube = 2a
12500 p 87500p 3
Total volume = 25000p + = cm 2a
3 3 radius of cone = =a
192. (b) The maximum possible volume of the circular cylinder 2
that can be formed from a rectangular sheet will have hight of cone = 2a
the largest length and breadth i.e. we will consider the 1 2 1 2
rectangular sheet with length 4p and breadth 2p. Volume of cone = pr h = ´ p ´ a ´ 2a
3 3
The length of the rectangular sheet = circumference of
the cylinder 2a 3 p 2pa 3
4p = 2 pr Þ r = 2 = =
3 3
2 2
Volume of the cylinder = pr h = p.4.2p = 8p 199. (b) Let radius of the sphere be r
193. (a) According to the question
4 3
4pr 2 = pr
3
13 r3
r2 =
3
\ r = 3 cm
5
140 70
r = 5 cm lk= 13 cm 200. (d) Radius of cone = = 70 cm = = 0.7 m
2 100
h = l 2 - r 2 = 132 - 5 2 = 144 = 12 Curved surface are = pr l = 1.76 m2
1 22
Hence, volume of cone = ´ p ´ r2 ´ h ´ 0.7 ´ l = 1.76
3 7
1 1.76 ´ 7
´ p ´ ( 5) ´ 12 = 100p
=
2 l= = 0.8 m = 80 cm
3 22 ´ 0.7
194. (d) Let radius of cone be x and slant height of cone be 3x height of cone = l 2 - r 2 = 802 - 702
Total surface area of cone = pr(r + l)
= p × x (x + 3x) = 4x2p = 1500 = 10 15 cm
EBD_7367
284
A- Volume And Surface Area
201. (c) According to the question 22
203. (c) length of wire = 2pr = ´ 2 ´ 98 = 616
2r 7
3 a = 2r \ a=
3 Perimeter of square = 616
3 616
æ 2r ö 4 3 Side of square = = 154
Required ratio = ç ÷ : pr 4
è 3ø 3
2
8r3
4 204. (b) Side of square =
= : pr3 = 2 : 3 p p
3 3 3
2 1 1
radius of circular disc = ´ =
p 2 p
202. (d) According to the question
2pr : h = 3 : 1 æ 1 ö
2
2
area of disc = pr = p ´ ç = 1 unit 2
2 pr è p ÷ø
h=
3 205. (b) According to the question
2 2pr (r + h) = 352
æ 2 pr ö 2 4p2 r 2 + r 2
l= ç ÷ +r = 22
è 3 ø 9 2´ ´ r ( r + 10) = 352
7

= r 2 ( 4p + 9 ) = r
2
4p 2 + 9
r2 + 10r = 56
r2 + 10r – 56 = 0
9 3 r2 + 14r – 4r – 56 = 0
r(r + 4) – 4 (r + 14) = 0
r 4p 2 + 9 (r – 4) (r + 14) = 0
Curved surface area = prl = p ´ r ´
3 \ r = 4, – 14
Hence radius of circle = 4
pr 2 4p 2 + 9 diameter = 2 × 4 = 8
=
3
Lines & Angles 285
A-

C HA P T E R
LINES & ANGLES
21
1. What is the angle which equals one-fifth of its supplement? In the figure given above, what is the value of x?
[2007-I] [2008-I]
(a) 15º (b) 30º (a) 30º (b) 40º
(c) 36º (d) 75º (c) 45º (d) 60º
2. Consider the following statements
If two parallel lines are intersected by a transversal, then D E
I. each pair of corresponding angles are equal. 7. A B 140°
II. each pair of alternate are unequal. 115°
Which of the statements given above is/are correct?
[2007-I]
(a) Only I (b) Only II C
(c) Both I and II (d) Neither I nor II If given that AB || DE, ÐABC = 115º, ÐCDE = 140º.
3.
C D Then, what is the value of ÐBCD? [2008-I]
(a) 45º (b) 55º
(c) 65º (d) 75º
130°
A B 8. A D
x
60° 30°

G
E F
In the figure given above, AC || BD and AE || BF.
x
What is Ðx? 85° E
(a) 130º (b) 110º B
C
(c) 70º (d) 50º [2007-II]
If AD || BE, ÐDCE = 85º and ÐBDC = 30º, then what is
4. A a c B
the value of x? [2008-I]
C (a) 30º (b) 35º
b
x (c) 45º (d) 55º
9. LM is a straight line and O is a point on LM. Line ON is
D drawn not coinciding with OL or OM. If ÐMON is
What is the value of x in the figure given above? one-third of ÐLON, then what is ÐMON equal to?
[2007-II] [2008-I]
(a) b – a – c (b) b – a + c (a) 45º (b) 60º
(c) b + a – c (d) p – (a + b + c) (c) 75º (d) 80º
5. The angle between the legs of a compass is 60º and each 10. What is the least number of straight lines for a bounded
leg is 10 cm long. How far apart are the points on which plane figure? [2008-I]
the legs of the compass rest? [2008-I] (a) 1 (b) 2
(a) 5 cm (b) 10 cm (c) 3 (d) 4
(c) 11. Consider the following statements
5 3 cm (d) 10 3 cm
Two lines intersected by a transversal are parallel, if
B C I. the pairs of cooresponding angles are equal.
6. D E
2x II. the interior angles on the same side of the transversal
120°
are supplementary.
Which of the statements given above is/are correct?
[2008-I]
x
(a) Only I (b) Only II
A (c) Both I and II (d) Neither I nor II
EBD_7367
286
A- Lines & Angles

12. Assertion (A) Two distinct lines cannot have more than (a) 110º (b) 120º
one point in common. (c) 125º (d) 130º
Reason (R) Any number of lines can be drawn through 18. A b B
one point. [2008-II]
(a) A and R are correct and R is correct explanation of
a
A
(b) A and R are correct but R is not correct explanation c
of A L M
(c) A is correct but R is wrong In the figure given above, AB is parallel to LM, What is
(d) A is wrong but R is correct the angle a equal to? [2010-I]
(a) p + b + c (b) 2p – b + c
13. (c) 2p – b – c (d) 2p + b – c
R P 19. Three lines intersect each other in pairs. What is the
number of angles so formed? [2010-I]
T
A B (a) 3 (b) 6
(c) 9 (d) 12
20. D E
x
C D 80°
125°
U V C
30°
y A B
Q S

In the given figure, If AB || CD, ÐPTB = 55º and ÐDVS z M


F
= 45º, then what is the sum of the measures of ÐCUQ and Three straight lines x, y, and z are parallel and the angles
ÐRTP? [2008-II] are as shown in the figure above. What is ÐAFB equal
(a) 180º (b) 135º to? [2010-I]
(c) 110º (d) 100º (a) 20º (b) 15º
14. Let AB and AC be two rays intersecting at A. If D, E be (c) 30º (d) 10º
the points lying on AB, AC respectively and P be the
point such that P divides the line DE such that PD : PE A X B
21.
= AD : AE. Then, what is the locus of the point P? 125°
[2009-I]
O
(a) The angle bisector of angle A
(b) The angle trisector of angle A
(c) The perpendicular bisector of angle A 35°
(d) None of the above C Y D
15. The length of a line segment AB is 2 unit. It is divided In the figure given above, AB is parallel to CD. What is
into two parts at the point C such that AC2 = AB × CB. the value ÐXOY? [2010-II]
What is the length of CB? [2009-II] (a) 80º (b) 90º
(a) 3 + 5 units (b) 3 – 5 units (c) 95º (d) 100º
22. The line segments AB and CD intersect at O, OF is the
(c) 2 – 5 units (d) 3 units internal bisector of obtuse ÐBOC and OE is the internal
16. If a point P moves such that its distance from two given bisector of acute ÐAOC. If ÐBOC = 130º, what is the
points A and B are equal. Then, what is the locus of the measure of ÐFOE? [2010-II]
point P? [2009-II] (a) 90º (b) 110º
(a) A straight line which is the right bisector of AB (c) 115º (d) 120º
(b) A circle with centre at B
23. P
(c) A straight line passing through A and B. Q
55°
(d) A straight line passing through either A or B
F
17. A B L
M
H 30° 155°
R 25° S
In the figure given above, PQ is parallel to RS, What is
60° 50° the angle between the lines PQ and LM? [2010-II]
C D
E (a) 175º (b) 177º
In the figure given above, AB is parallel to CD and BE is (c) 179º (d) 180º
parallel to FH. What is ÐFHE equal to? [2009-II]
Lines & Angles A-287
24. In a DABC, side AB is extended beyond B, side BC (a) 4 (b) 6
beyond C and side CA beyond A, What is the sum of the (c) 8 (d) 9
three exterior angles? [2011-I] 31. The ratio of two complementary angle is 1 : 5. What is the
(a) 270º (b) 305º difference between the two angles? [2012-I]
(c) 360º (d) 540º (a) 60º (b) 90º
25. Let A and B be two points. What is the locus of the point (c) 120º
P such that ÐAPB = 90º? [2011-II] (d) Cannot be determined with the given data
(a) The line AB itself 32. In the quadrilateral ABCD shown below ÐDAB = ÐDCX
(b) The point P itself = 120º. If ÐABC = 105º, what is the value of ÐADC ?
(c) The circumference of the circle with AB as diameter [2012-I]
(d) The line perpendicular to AB and bisecting AB
X
26. C D D C
x

B
A y
E A B
In the figure given above, AB is parallel to CD. If ÐDCE (a) 45º (b) 60º
= x and ÐABE = y, then what is ÐCEB equal to? (c) 75º (d) 95º
[2011-II] 33. In the figure given below AO = CD, where O is the centre
(a) y – x (b) (x + y)/2 of the circle. What is the value of ÐAPB ? [2012-I]
(c) x + y – (p/2) (d) x + y – p
27. A B
A O B
98°
F 144° C D

C D P
E (a) 60º (b) 50º
In the figure given above, AB is parallel to CD. If ÐBAF (c) 45º (d) 30º
= 98º and ÐAFC = 144º, then what is ÐECD equal to? 1 1 1
[2011-II] 34. In a DABC, ÐA + ÐC + ÐB = 80º , then what is the
(a) 62º (b) 64º 2 3 2
(c) 82º (d) 84º value of ÐC? [2012-II]
(a) 35º (b) 40º
L (c) 60º (d) 70º
28. P 47° Q
55° 35. Consider the following statements
67° N I. The locus of points which are equidistant from two
parallel lines is a line parallel to both of them and drawn
mid way between them.
R S II. The perpendicular distances of any point on this locus
M line from two original parallel lines are equal. Further, no
In the figure given above, PQ is parallel to RS. What is point outside this locus line has this property.
ÐNMS equal to? [2011-II] Which of the above statements is/are correct? [2012-II]
(a) 20º (b) 23º (a) Only I (b) Only II
(c) 27º (d) 47º (c) Both I and II (d) Neither I nor II
29. Consider the following statements 36. The angles xº, aº, cº and (p – b)º are indicated in the figure
If two straight lines intersect, then given below
I. Vertically opposite angles are equal.
II. Vertically opposite angles are supplementary. E
III. adjacent angles are complementary. D x°
Which of the statements given above is/are correct?
[2012-I]
(a) Only III (b) Only I (p – b)°
(c) I and III (d) II and III c° a°
A
30. Two transversals S and T cut a set of distinct parallel B C
lines. S cuts the parallel lines in points A, B, C, D, and Which one of the following is correct? [2012-II]
T cuts the parallel lines in points E, F, G and H, (a) x = a + c – b (b) x = b – a – c
respectively. If AB = 4, CD = 3 and EF = 12, then what (c) x = a + b + c (d) x = a – b + c
is the length of GH? [2012-I]
EBD_7367
288
A- Lines & Angles

37. In the figure given below, AB is parallel to CD. (b) not equal but supplementary
ÐABC = 65º, ÐCDE = 15º and AB = AE, [2012-II] (c) equal but not supplementary
A F (d) either equal or supplementary
42. Let OA, OB, OC and OD are rays in the anticlockwise direction
such that ÐAOB = ÐCOD = 100°, ÐBOC = 82° and ÐAOD =
78°. Consider the following statements : [2015-I]
1. AOC and BOD are lines.
B C 2. ÐBOC and ÐAOD are supplementary.
E
Which of the above statements is /are correct?
(a) 1 only (b) 2 only
(c) Both 1 and 2 (d) Neither 1 nor 2
43. At 8 : 30, the hour hand and the minute hand of a clock form
D
an angle of [2015-I]
What is the value of ÐAEF? (a) 80° (b) 75°
(a) 30º (b) 35º (c) 70° (d) 60°
(c) 40º (d) 45º
38. In the figure given below, ABC is a triangle. BC is parallel 44. A clock is started at noon. By 10 minutes past 5, through
to AE. If BC = AC, then what is the value of ÐCAE? what angle, the hour hand moves ? [2015-II]
[2012-II] (a) 160° (b) 145°
B C (c) 150° (d) 155°
65° 45. Two poles are placed at P and Q on either side of a road
such that the line joining P and Q is perpendicular to the
D 95° length of the road. A person moves x metre away from P
parallel to the road and places another pole at R. Then the
person moves further x metre in the same direction and
E
A turns and moves a distance y metre away from the road
(a) 20º (b) 30º perpendicularly, where he finds himself, Q and R on the
(c) 40º (d) 50º same line. The distance between P and Q (i.e., the width of
39. In the figure given below, EC is parallel to AB, ÐECD the road in metre is) [2016-I]
= 70º and ÐBDO = 20º. [2012-II]
x
E A (a) x (b)
2
(c) y (d) 2y
46. There are five lines in a plane, no two of which are parallel.
The maximum number of points in which they can intersect
C D is [2016-I]
O
(a) 4 (b) 6
B (c) 10 (d) None of the above
47. If a transversal intersects four parallel straight lines, then
What is the value of ÐOBD ? the number of distinct values of the angles formed will be
(a) 20º (b) 30º [2016-I]
(c) 40º (d) 50º (a) 2 (b) 4
40. In the given figure below LOM is a straight line. (c) 8 (d) 16
[2012-II] 48. p
s
80° z 40
Q P °
100° x y q
In the figure given above, P and q are parallel lines. What
50° are the values of the angles x, y and z? [2016-I]
(x + 20)° (x – 10)° (a) x = 80°, y = 40°, z = 100°
L M (b) x = 80°, y = 50°, z = 105°
O
(c) x = 70°, y = 40°, z = 110°
What is the value of xº ? (d) x = 60°, y = 20°, z = 120°
(a) 45º (b) 60º 49. If D is the number of degrees and R is the number of radians
(c) 70º (d) 80º in an angle q, then which one of the following is correct ?
41. If the arms of one angle are respectively parallel to the arms [2017-I]
of another angle, then the two angles are [2013-I] (a) pD = 180R (b) pD = 90R
(a) neither equal nor supplementary (c) pR = 180D (d) pR = 90D
Lines & Angles A-289

50. In the figure given below, PQ is parallel to RS and PR is 54. What angle does the hour hand of a clock describe in 10
parallel to QS. If ÐLPR = 35° and Ð UST = 70°, then what is minutes of time? [2018-I]
ÐMPQ equal to ? [2017-I] (a) 1° (b) 5°
M (c) 6° (d) 10°
55. The length of a line segment AB is 2 cm. It is divided into
two parts at a point C such that AC2 = AB × CB. What is the
length of CB? [2018-I]
L (a) 3 5 cm (b) 3 - 5 cm
P
(c) 5 3 cm (d) 5 - 1 cm
R Q 56. The locus of a point equidistant from two intersecting lines
is [2018-I]
(a) A straight line (b) A circle
S U (c) A pair of straight lines (d) None of the above
T DIRECTIONS (Qs. 57-58): In a triangle ABC, a, b and c are the
lengths of the sides and p, q and r are the lengths of its medians.
(a) 55° (b) 70°
(c) 75° (d) 80° [2018-I]
51. a, b, c, b are non-zero integers such that (ab) divides (cd). If 57. Which one of the following is correct?
a and c are coprime, then which one of the following is (a) 2 (p + q + r) = (a + b + c)
correct? [2017-II] (b) 2 (p + q + r) > 3 (a + b + c)
(a) a is a factor of c (b) a is a factor of b (c) 2 (p + q + r) < 3 (a + b + c)
(c) a is a factor of d (d) d is a factor of a (d) 11 (p + q + r) > 10 (a + b + c)
52. If q measured in radians is the angle between the hour hand 58. In the figure given below, p, q, r are parallel lines; l and m
and the minute hand of a clock when the time is are two transversals.
4 : 36 pm, then which one of the following is correct?
[2017-II] l m
3p 4p 2p 3p
(a) <q< (b) <q<
5 5 5 5
A D p
p 2p 7p 8p
(c) £q£ (d) £q£
5 5 15 15 B E
53. Consider the following statements in respect of three q
straight lines A, B and C on a plane : [2017-II]
1. If A and C are parallel and B and C are parallel; C F
r
then A and B are parallel.
2. If A is perpendicular to C and B is perpendicular
to C; then A and B are parallel.
3. If the acute angle between A and C is equal to the Consider the following :
acute angle between B and C; then A and B are 1. AB : AC = DE : DF
parallel. 2. AB × EF = BC × DE
Which of the above statements are correct? Which of the above is/are correct?
(a) 1, 2 and 3 (b) 1 and 2 only (a) 1 only (b) 2 only
(c) 1 and 3 only (d) 2 and 3 only (c) Both 1 and 2 (d) Neither 1 nor 2
EBD_7367
A- 290 Lines & Angles

HINTS & SOLUTIONS


1. (b) Let the angle be x, then its supplement angle is x+a+c=b
(180º – x). \ x=b–a–c
By given condition, 5. (b) In DAOB,
1 AB 1 AB
x= (180º – x ) Þ 5x = 180º – x sin 30º = Þ =
5 OA 2 10
Þ AB = 5 cm
180º
Þ x= = 30º Now, AC = 2 AB = 2 × 5 = 10 cm
6
O
2. (a) If two parallel lines are intersected by a transversal,
then each pair of corresponding angles and of alternate
angles are equal.

30°
Therefore, statement I is correct.

cm
60°

10
3. (b) Since, AC || BD

10

cm
C D
A C
B
130° 6. (d) \ ÐABC = 180º – ÐDBA
B C
A B D E
x
2x 120°
60°

G
E F x
\ ÐDBA = 180º – 130º = 50º
Since, DBG is straight line, A
= 180º – 2x
\ ÐDBA + ÐABF + ÐFBG = 180º
and ÐACB = 180º – ÐACE = 180º – 120º = 60º
Þ 50º + ÐABF + 60º = 180º
We know that,
Þ ÐABF = 70º
ÐABC + ÐACB +ÐBAC =180º
Since, AE || BF
Þ 180º – 2x + 60º + x = 180º
\ x = 180º – ÐABF = 180º – 70º = 110º
Þ 240º – 180º = x
4. (a) Construction:
\ x = 60º
A 7. (d) Draw a line GF through C parallel to AB and DE.
Now.
a a D E
B
2 2 A 140°
115°
C
G F
x b b c C
D 2 2 B \ ÐBCG = 180º – ÐABC
= 180º –115º = 65º (Interior angles
on the same side of
Y the transversal)
Draw a bisector AY of ÐA and ÐC and ÐDCF = 180º – ÐCDE
In DADC, = 180º – 140º = 40º
a b (Interior angles on the same side of the transversal)
x+ = (i) Now, ÐBCG + ÐBCD + ÐDCF = 180º
2 2
In ACB, Þ 65º + ÐBCD + 40° = 180°
\ ÐBCD = 180º – 105º = 75º
a b 8. (b) AD || BE
c+ = ...(ii)
2 2 \ ÐADC = ÐDCE (alternate angles)
on adding eq. (i) and (ii) Þ ÐADB + 30º = 85º
a a b b Þ ÐADB = 55º
x+ + +c = + and ÐBAD = 90º (given)
2 2 2 2
Lines & Angles A- 291
Now, in DABD, A
ÐABD + ÐADB + ÐBAD = 180º
Þ x + 55º + 90º = 180º 12
Þ x = 180º – 145º = 35º
9. (a) Given that,
N
D P E
B C
L M DDAP and DAPE are similar. So, Ð1 = Ð2
O AP is bisector of ÐA.
1 Hence, the locus of P is the bisector of angle A.
ÐMON = ÐLON
3 15. (b) Given, AC2 = AB × CB
Let ÐLON = x Þ x2 = 2 × (2 – x)
x Þ x2 = 4 – 2x
Then, ÐMON = A B
3 x C (2 – x)
We know that, ÐLON + ÐMON = 180º 2
(linear pair) Þ x2 + 2x – 4 = 0
x 180º ´3 –2 ± 4 + 16
Þ x+ = 180 º Þ x = = 135 º Þ x=
3 4 2 ´1
x 135º Þ x = –1± 5
= = 45º
Thus, ÐMON = 3 3
Now, BC = 2 – (–1 ± 5 )
10. (c) The least number of straight lines for a bounded plane
figure is 3. = 3 – 5 (neglect 3 + 5 Þ 3 + 5 > 2)
11. (c) Hence, both Statements I and II are correct. 16. (a) The locus of P is a straight line which is the right
12. (b) A. It is true that two distinct lines intersect only one bisector of AB.
point. 17. (a) ÐHEB = 180º – 60º – 50º = 70°
R. It is true that, from a one point we can draw any number
of lines but R is not a correct explanation of A. F
A B
13. (b) AB || CD ù
H
ÐPTB = 55° úú (Given)
ÐDVS = 45° úû
ÐPTB = ÐATU = 55° (Vertically opposite angles) 60° 50°
C D
Similarly, E
ÐDVS = ÐBTV = 45° (Corresponding angles) Since, HF || BE and HE is transversal line
\ ÐRTP = 180° – (45° + 55°) = 80° \ ÐFHE + ÐHEB = 180º (interior angle)
ÐPTB = ÐTUV = 55° (Corresponding angle) Þ ÐFHE + 70º = 180º
and ÐCUQ = ÐTUV = 55° Þ ÐFHE = 110º.
(Vertically opposite angles) 18. (c) Let we draw a line parallel to AB which is EF.
\ Sum of angles = ÐRTP + ÐCUQ
= 80° + 55° = 135° P
A b B
R P
80° E F
55° O a
A T B c
55° L M
80° N
55° 45° \ ÐEOP = ÐOPB (alternate angle)
C 55° U V Þ ÐEOP = b
45° D and ÐEON = ÐONM (alternate angle)
Þ ÐEON = c
Q S \ ÐPON = b + c
Q ÐPON + a = 2p Þ a = 2p – (b + c)
PD AD AP 19. (d) We know that, when two lines intersect each other it
14. (a) Since, = =
PE AE AP makes 4 angles.
The total number of pairs = 3
EBD_7367
292
A- Lines & Angles

Hence, ÐEOF = ÐEOC + ÐFOC = 25º + 65º = 90º


23. (d) Since, PQ || RS
P
Q
55°

L
M
30° 155°
R 25° S
\ Total number of angles = 3 × 4 = 12
20. (b) ÐCDE = 180º – 125º = 55º \ ÐPQR = ÐQRS (alternate angle)
and ÐSRL + ÐRLM = 180º Þ RS || LM ...(ii)
D E
x From Eqs.(i) and (ii),
125° 80° PQ || LM
C
Therefore, angle between the lines PQ and LM is 180º
30°
y A B
24. (c)
C
z F M
3
In DDCE, ÐCED = 180º – 55º – 80º = 45º
and ÐABF = 30º (vertically opposite angle)
Also, ÐABF = ÐBFM = 30º(alternate angle)
and ÐDEF = ÐEFM (alternate angle)
ÐEFM = 45º Þ ÐEFB + ÐBFM = 45º
Þ ÐEFB = 45º – 30º Þ ÐAFB = 15º
21. (b) Draw a line EF such that EF || AB || CD. 1 2
A
A X B B
125°
E O F Sum of the three exterior angles
= (Ð1 + Ð2) + (Ð2 + Ð3) + (Ð3 + Ð1)
35° = 2 (Ð1 + Ð2 + Ð3)
C Y D (Sum of the interior angles are 180º)
Now, AB || EF = 2 × 180º = 360º
\ ÐAXO + ÐXOE = 180º Sum of exterior angles of any polygon = 360º
(Interior angle as the same side of line) Alternate Method
Þ ÐXOE = 180º – 125º = 55º Sum of the Exterior angles
Also, EF || CD = (180º – ÐA) + (180º – ÐB) + (180º – ÐC)
Þ ÐEOY = ÐOYD = 35º (linear pair) = 540º – (ÐA + ÐB + ÐC)
Hence, ÐXOY = ÐXOE + ÐEOY = 540º – 180º (Sum of angles of a triangle is 180º)
= 55º + 35º = 90º = 360º
22. (a) Given, ÐBOC = 130º 25. (c) Hence, the locus of a point is the circumference of the
\ ÐBOC + ÐAOC = 180º (linear pair) circle with AB as diameter.
A D P

O 90°
E
A B
C B
F
Þ 130º + ÐAOC = 180º Þ ÐAOC = 50º 26. (a) Here, we produced AB line to M.
Now ÐBOC = 130º Þ ÐBOF + ÐFOC = 130º C D
Þ ÐFOC + ÐFOC = 130º x
(Q OF is bisector of ÐBOC )
Þ ÐFOC = 65º now ÐAOC = 50º
Þ ÐAOE + ÐEOC = 50º B M
Þ ÐEOC + ÐEOC = 50º A y
(Q OE is bisector of ÐAOC) E
Since, AM is parallel to CD.
Þ ÐEOC = 25º
Lines & Angles 293
A-

ÐDCM = ÐBMC = x (alternate angle) AB EF 4 12


Also, ABM is a straight line. = Þ =
CD GH 3 x
ÐEBM = p – y
\ x=3×3=9
Now in DBEM.
ÐB + ÐM +ÐE= p a 1
31. (a) Given that, = Þ a = k and b = 5k (say)
Þ p – y + x + ÐE= p b 5
Þ ÐE = y – x or complementary angles,
a = 90º – b Þ k = 90º – 5k
A B Þ k = 15º
27. (a)
\ a = 15º and b = 75º
98° \ Difference between angles = 75º – 15º = 60º
F N 32. (c) X
120°
D C
C 60°
D
E
120° 105°
parallel to AB and CD. A B
\ ÐAFN = 180º – 98º = 82º Given, ÐABC = 105º
Þ ÐCFN = 144º – 82º = 62º ÐDAB = 120º
\ ÐFCD = 180º – 62º = 118º ÐDCX = 120º
Þ ÐECD = 180º – 118º = 62º Þ ÐDCB = 180º – 120º = 60º
28. (a) ÐPLM = ÐLMS (alternate angle) Angles of a quadrilateral is equal to 360º
= 55º \ ÐADC = 360º – (120º + 105º + 60º)
Let draw EF line which is parallel to PQ and bisects by LN = 360º – 285º = 75º
Then, ÐQLN = ÐLNE = 47º 33. (a)
Q ÐENL + ÐMNE = 67º
L O
P 55° 47° Q A B
x y
60° 60°
E N F 60° 60°
C D

R M S

Then, ÐLNE = 47º


Þ 47° + ÐMNE = 67º
ÐMNE = 67º – 47º
ÐMNE = 20º
Similarly, EF || RS, then ÐENM = ÐNMS = 20º P
29. (b) AB and CD are two lines. AO = CD
A D Þ OC = OD = CD
(Q AO = OC = OD = radii)
So, DCOD is equilateral,
Ðx + Ðy = 180 – 60
and Ðx = Ðy
\ Ð2x = 120°
C B Ðx = 60°
We know that when two lines intersect each other
then opposite vertically angles are equal ÐAOC is also equilateral
30. (d) From figure. ÐDCP = 180º – 120º = 60º
and ÐCDP = 60º
S T
\ ÐAPB = 360º – (60º + 120º + 120º) = 60º
A E 34. (c) We know
4 B 12 A
F
C G
3 x
D H

By proportionality law, B C
EBD_7367
294
A- Lines & Angles

ÐA + ÐB + ÐC = 180º BC and FD intersect each other at E, then


ÐA + ÐB = 180º – ÐC ...(i) ÐBEF = ÐDEC = 100º
ÐA ÐB ÐC ÐAEF = 100º – 65º = 35º
+ + = 80º (Given) (Because ÐBEF = ÐBEA + ÐAEF)
2 2 3 38. (d) Given that, BC || AE
æ ÐA + ÐB ö ÐC ÐCBA + ÐEAB = 180º
ç ÷+ = 80º Þ ÐEAB =180º – 65º = 115º
è 2 ø 3 Q BC = AC
from equation (i) Hence, DABC is an isosceles triangle
180º – ÐC ÐC B C
Þ + = 80º 65°
2 3
180º ´3 – 3ÐC + 2ÐC
Þ = 80º D 95°
6
Þ ÐC = (540º – 480º) x°
Þ ÐC = 60º E
35. (c) Statements I and II are both true, because the locus of A
points which are equidistant from two parallel lines is Þ ÐCBA = ÐCAB = 65º
a line parallel to both of them and draw mid way Now, ÐEAB = ÐEAC + ÐCAB
between them. Þ 115º = x + 65º Þ x = 50°
Also, it is true that the perpendicular distances of any 39. (d) Given that, EC || AB
point on this locus line from two original parallel lines ÐECO + ÐAOC = 180º
are equal. Further, no point outside this locus line has E A
this property.
A C X¢
X

B¢ D¢ 70° 110°
Z Z¢
B D D
C O 20°

Y Y¢ B
A¢ C¢ Þ ÐAOC = 180º – 70º = 110º
36. (c) Q ÐPCT + ÐPCB = p \ ÐBOD = ÐAOC = 110º (alternate angle)
ÐPCB = p – (p – bº) = bº Now, in DOBD
S ÐBOD + ÐODB + ÐDBO = 180º
E \ 110º + 20º + xº = 180º Þ xº = 50º
D x° 40. (b) From the given figure. We know that sum of all angles is
180º because of straight lines.
P ÐLOQ + ÐQOP + ÐPOM = 180º
(p – b)° \ (xº + 20º) + 50º + (xº – 10º) = 180º
c° a° Þ 2xº + 60º = 180º Þ 2xº = 120º
A T
B C xº = 60º
In DBPC, 41. (b) l1 and l2 are two parallel lines and Ð1 and Ð2 are interior
ÐPCB + ÐBPC + ÐPBC = p angles on the same side of the transversal.
Þ ÐPBC = p – ÐPCB – ÐBPC
= p – bº – aº
Q ÐABE + ÐEBC = p (linear pair)
Þ ÐABE = p – ÐPBC = p – (p – bº – aº) l1
= aº + bº (Q ÐPBC = ÐEBC) 1
Now,
Sum of two interior angles = Exterior angle
\ in DABE, ÐEAB + ÐABE = ÐBES Þ cº + bº + aº = xº
\ xº = aº + bº + cº 2 l2
37. (b) AB = AE (Given)
\ ÐABC = 65º = ÐAEB
AB || CD, then Ð1 ¹ Ð2
ÐABE = ÐDCE = 65º (Alternate angles) Ð1 + Ð2 = 180º.
In DDCE, Therefore, these are supplementary angles or
ÐD + ÐC + ÐE = 180º consecutive interior angles.
ÐE = 180º – 65º – 15º = 100º
Lines & Angles A- 295

46. (d)
anticlockwise
42. (d) A B 47. (a) Let l1, l2, l3 and l4 be your straight lines and t be a
transversal.
100°
78° O 82° 2 1
l1
100° 4
3
5
6
D C l2
Statement 1 8
7
AOC and BOD are not lines because ÐAOD = 78°,
So, it is not correct. l3
Statement 2
ÐBOC and ÐAOD are 82° and 78° respectively, so it is
not supplementry angle.
So, neither statement 1 nor 2 are correct. l4
30 1 17
43. (b) Total hour = 8 : 30 = 8 + = 8+ = h
60 2 2
12 h angle made by hour hand = 360°
t
360 Ð1 = Ð3
1 h angle made by hour hand =
12 Ð2 = Ð4
17 360 17 [alternate angles]
h angle made by hour hand = ´ Similarly angles formed by
2 12 2
= 15 × 17 = 255° l2, l3 and l4 are also equal in same way as l1
Now, 60 min angle made by min hand = 360° corresponding angles
360° Ð1 = Ð5 & Ð5 = Ð7
1 min angle made by min hand = Ð6 = Ð2 & Ð6 = Ð8 and so on
60
30 min angle made by min hand = 6 × 30° = 180° Only two distinct angles are formed
Angle between hour and min hand \ Option (a) is correct.
= 255° – 180°
E A
= 75° 48. (d) p
80° q x/3
44. (d) Hour hand moves by 30° in 1 hour. z 40°
In one hour movement of hour hand = 30°
æ 10 ö æ 10 ö
In çççè5 + ø÷÷÷ hour movement is = 30 çççè5 + ø÷÷÷ y
60 60 100° x
q
æ 1ö D B C
30 = çççè5 + ø÷÷÷ Now ABDE is a quadrilateral
6
AB works as a transversal on parallel lines.
31
= 30 × = 155° x
6 Þ Ðx = 40° + (alternative angles)
So, option (d) is correct. 3
45. (c) In the figure, x
Þ x- = 40
D PQR @D STR 3
(By ASA) 40 ´ 3
Þ x=
\PQ = TS = y metre 2
Þ x = 60°
y
T S x
Þ = 20° = Ðy (alternate angles)
3
x Þ x = 60°
y = 20°
R Since ABDE is a quadrilateral.
Þ Ðz + Ðx = 180°
x Þ Ðz = 180 – Ðx
= 180° – 60°
= 120°
B Q
\ option (d) is correct.
EBD_7367
296
A- Lines & Angles

49. (d) pC = 180° 53. (b) A B


p°R = 180D
pR = 180D
50. (c)
M
C

L P Q A

B
U
C
R S The above two figures shows that (b) is the only cor-
rect answer.
54. (b) Angle described by hour hand of a clock in 12 hrs =
360°
PQ || RS
Angle described by hour hand of clock in 1 hr
PR|| QS
\ PQRS is a || gm 360
= = 30°
ÐLPR = 35° and ÐUST = 70° 12
ÐUST = ÐRSQ (Vertically opposite) Angle described by hour hand of clock in 1 min
ÐRSQ = ÐRPQ
(opposite angle of 11 gm) 30 1°
= =
ÐLPR + ÐRPQ + ÐMPQ = 180° 60 2
35° + 70° + ÐMPQ = 180° Angle described by hour hand of clock in 10 min
ÐMPQ = 180 – 105
1
ÐMPQ = 75° ´ 10 = 5°
51. (c) Since we are given that a and c are co-prime i.e. HCF of 2
a and c is 1, therefore we can say that a definitely 55. (b)
divides d exactly. So, a is a factor of d. A C B
52. (b) To calculate the angle between the hour hand and the
minute hand of a clock when the time is 4 : 36 pm, we 2
AB = 2 cm
can say that the angle will be approximately equal to
the angle made from 4 : 20 pm to 4 : 36 pm. AC2 = AB × CB
(2 – CB)2 = 2CB
Therefore, we need to calculate the angle made by the
hands of a clock in 16 minutes. CB2 – 4CB + 4 – 2CB = 0
CB2 – 6CB + 4 = 0
In 60 minutes, the angle made by the hands of a clock
is 360°. So, the angle made by the hands of a clock in
-b ± b2 - 4ac
360 Hence, from , value of CB = 3 - 5
16 minutes will be ´ 16 = 96° . 2a
60 56. (c)
Thus, the angle lies between 72° to 108° i.e. 57. (c)
2p 3p 58. (c)
<q< .
5 5
Triangles and its Properties A-297

C HA P T E R
TRIANGLES AND
22 ITS PROPERTIES
1. In D ABC and DDEF, it is given that AB = 5 cm, BC = 4 cm 6. A
and CA = 4. 2 cm, DE = 10 cm, EF = 8 cm and FD = 8.4 cm. If
AL is perpendicular to BC and DM is perpendicular to EF,
then what is the ratio of AL to DM? [2007-I]
D E
1 1
(a) (b)
2 3

1 B C
(c) (d) 1 In the figure given above. BC is parallel to DE and DE : BC
4 = 3 : 5. What is the ratio of area of the D ABC to that of
2. A D ABC is permitted to move around when its vertex A is DDEA? [2007-II]
fixed. What is the locus of the circumcentre? (a) 3 : 1 (b) 5 : 3
[2007-II] (c) 9 : 2 (d) 25 : 9
(a) A straight line (b) A circle 7. The vertical angle of an isosceles triangle is 15° more than
(c) A point (d) A curve other than a circle each of its base angles. What is the vertical angle?
3. Assertion (A) ABC is a triangle and AD is its angular bisector. [2008-I]
If AB = 6 cm, BC = 7 cm, AC = 8 cm, then BD = 3 cm and CD (a) 35° (b) 55°
= 4 cm. (c) 65° (d) 70°
8. Assertion (A) If two triangles are congruent, then their
Reason (R) The angular bisector AD of a triangle cuts the
corresponding angles are equal.
base BC in the ratio AB : AC. [2007-II]
Reason (R) Two congruent triangles have same area.
(a) Both A and R individually true and R is the correct [2008-I]
explanation of A (a) Both A and R individually true and R is the correct
(b) Both A and R are individually true but R is not the explanation of A
correct explanation of A (b) Both A and R are individually true but R is not the
(c) A is true but R is false correct explanation of A
(d) A is false but R is true (c) A is true but R is false
4. BAC is triangle with ÐA = 90°. From A, a perpendicular AD is (d) A is false but R is true
drawn on BC. Which one of the following is correct? A
9.
[2007-II]
(a) Only D ABC ~ D DAC
(b) Only D DAC ~ D DBA D E
(c) Only DABC ~ DDBA ~ D DAC
(d) Only DABC ~ DDAB
where ~ stands for the notation of similarity. B C
5. In the given figure, QR is parallel to AB and DR is parallel to In the figure shown above, DE is parallel to BC and the ratio
QB. What is the number of distinct pairs of similar triangles? of the areas of D ADE and trapezium BDEC is 4 : 5. What is
[2007-II] DE : BC ? [2008-I]
(a) 1 (b) 2 (a) 1 : 2 (b) 2 : 3
(c) 3 (d) 4 (c) 4 : 5 (d) None of these
10. Consider the following statements
P A triangle can be constructed if its
I. two sides and the included angles are given.
D II. three angles are given.
Q R III. two angles and the included side are given.
Which of the statements given above are correct?
[2008-I]
A B
EBD_7367
298
A- Triangles and its Properties

(a) I and II (b) I and III 17. A


(c) II and III (d) All of these
11. AB, EF and CD are parallel lines. If EG = 5 cm GC = 10 cm,
AB = 15 cm and DC = 18 cm, then what is the value of AC?
[2008-I]
(a) 20 cm (b) 24 cm
(c) 25 cm (d) 28 cm
12. B 30° 20°
Q B D C
In the given figure, ÐABD = 90°, ÐBDA = 30° and ÐBCA =
20°. What is ÐCAD? [2008-II]
(a) 10° (b) 20°
A P C (c) 30° (d) 15°
In the given triangle, AB is parallel to PQ. AP = c, PC = b, 18. If the angles of a triangle are 90°, 60° and 30°, then what is
PQ = a, AB = x. What is the value of x? [2008-I] the ratio of the sides opposite to these angles?
[2008-II]
ab bc
(a) a+ (b) a+ (a) 3 : 2 :1 (b) 1: 2 : 2
c a
(c) 2 : 3 :1 (d) 3 : 2 : 1
ca ac 19. The sides of a triangle are 50 m, 40 m and 30 m. What is the
(c) b+ (d) a+
b b length of the altitude of the vertex opposite to the side 50 m
13. Assertion (A) Triangles on the same base and between the long? [2008-II]
same parallel lines are equal in area. (a) 22 m (b) 24 m
Reason (R) The distance between two parallel lines is same (c) 25 m (d) 26 m
everywhere. [2008-II] 20. D
C
(a) Both A and R individually true and R is the correct 90°
explanation of A 90°
(b) Both A and R are individually true but R is not the x y
correct explanation of A
(c) A is true but R is false 30° 45°
A B
(d) A is false but R is true In the figure given above. What is the value of (2x – y)?
[2008-II]
14. The hypotenuse of a right triangle is 3 10 unit. If the
(a) 10° (b) 15°
smaller side is tripled and the longer side is doubled, new (c) 20° (d) 25°
hypotenuse becomes 9 5 unit. What are the lengths of 21. If ABC is a triangle, right angled at B, M and N are mid-
the smaller and longer sides of the right triangle, points of AB and BC respectively, then what is 4 (AN2 +
respectively? [2008-II] CM2) equal to? [2008-II]
(a) 5 and 9 units (b) 5 and 6 units (a) 3 AC2 (b) 4 AC2
(c) 3 and 9 units (d) 3 and 6 units (c) 5 AC2 (d) 6 AC2
22. If A is the area of a right angled triangle and b is one of the
S
sides containing the right angle, then what is the length of
15. the altitude on the hypotenuse? [2008-II]
2 Ab 2 A2 b
P Q R (a) (b)
b4 + 4 A2 b 4 + 4 A2
In the figure given above, PQ = QS and QR = RS. If ÐSRQ
= 100°, then how many degrees is ÐQPS? 2 Ab 2 2 A2b2
[2008-II] (c) (d)
b 4 + 4 A2 b 4 + A2
(a) 40° (b) 30°
23. D ABC is right angled at A. AB = 3 units, AC = 4 units and
(c) 20° (d) 15°
AD is perpendicular to BC. What is the area of the DADB?
16. ABC is a right angled triangle, right angled at C and p is the
[2008-II]
length of the perpendicular from C on AB. If a, b and c are
the sides of the triangle, then which one of the following is 9 54
(a) sq units (b) sq units
correct? [2008-II] 25 25
(a) (a2 + b2) p2 = a2b2 (b) a2 + b2 = a2 b2 p2
72 96
(c) p2 = a2 + b2 (d) p2 = a2 – b2 (c) sq units (d) sq units
25 25
Triangles and its Properties A-299

24. A DIRECTIONS (Q. Nos. 30-32) : The following three questions


consists of two statements, one labelled as the ‘Assertion (A)’
and the other as ‘Reason (R)’. You are to examine these two
C statements carefully and select the answers to these items using
x P the codes given below :
y (a) Both A and R are individually true and R is the correct
z explanation of A.
B D (b) Both A and R are individually true but R is not the correct
Q explanation of a.
p (c) A is true but R is false
In the figure given above. ÐABD = ÐPQD = ÐCDQ = . (d) A is false but R is true.
2 30. Assertion (A) If two triangles have same perimeter, then
If AB = x, PQ = z and CD = y, then which one of the following they are congruent.
is correct? [2009-I] Reason (R) If under a given correspondence, the three sides
1 1 1 1 1 1 of one triangle are equal to the three sides of the other
(a) + = (b) + = triangle, then the two triangle, are congruent. [2009-I]
x y z x z y 31. ABC is a triangle. Let D, E denote the mid-points of BC and
1 1 1 1 1 2 CA, respectively. Let AD and BE intersect at G. Let O be a
(c) + = (d) + = point on AD such AO : OD = 2 : 7. Where, AD is a median.
z y x x y z
[2009-I]
25. DPQR is right angled at Q, PR = 5 cm and QR = 4 cm. If the
lengths of sides of another D ABC are 3 cm, 4 cm and 5 cm, (2GD )
Assertion (A) AO =
then which one of the following is correct? 3
[2009-I]
(2 AG )
(a) Area of D PQR is double that of D ABC Reason (R) OD =
(b) Area of D ABC is double that of D PQR 3
32. ABC is a triangle. AD, BE and CF are altitudes of D ABC.
ÐQ
(c) ÐB = Assertion (A)
2 (AB2 + BC2 + CA2) > (AD2 + BE2 + CF2)
(d) Both triangles are congruent Reason (R)
26. If C1 and C2 and r1 and r2 are respectively the centroids (AE2 – AF2) + (BF2 – BD2) + (CD2 – CE2) = 0 [2009-I]
and radii of incircles of two congruent triangles, then which 33. ABC is a triangle right angled at C. If p is the length of the
one of the following is correct? [2009-I] perpendicular from C to AB and a, b and c are the sides,
(a) C1 and C2 are the same points and r1 = r2 then which one of the following is correct?
(b) C1 and C2 are not necessarily the same point and r1 = [2009-II]
r2 (a) pa = bc (b) pb = ca
(c) C1 and C2 are same point and r1 is not necessarily (c) pc = ab (d) p2 = ab
equal to r2 34. In the DABC, AB = 2 cm, BC = 3 cm and AC = 4 cm. D is the
(d) C1 and C2 are not necessarily the same point and r1 is middle point of AC. If a square is constructed on the side
not necessarily equal to r2 BD, what is the area of the square? [2009-II]
27. C (a) 4.5 cm2 (b) 2.5 cm2
(c) 6.35 cm2 (d) None of these
35. ABC is a triangle. X is a point outside the DABC such that
CD = CX, where D is the point of intersection of BC and AX
Q and ÐBAX = ÐXAC. [2009-II]
R
Which one of the following is correct?
S (a) DABD and DACX are similar
(b) ÐABD < ÐDXC
A B (c) AC = CX
P
In the figure given above, P is a point on AB and PQ is (d) ÐADB > ÐDCX
parallel to AC. What is the number of pairs of distinct similar 36. Consider the following statements
triangles in the figure. [2009-I] I. Congruent triangles are similar.
(a) 1 (b) 2 II. Similar triangles are congruent.
(c) 3 (d) 4 III. If the hypotenuse and a side of one right triangle are
28. If the medians of two equilateral triangles are in the ratio 3 : equal to the hypotenuse and a side of another right
2, then what is the ratio of their sides? [2009-I] triangle respectively, then the two right triangles are
(a) 1 : 1 (b) 2 : 3 congruent.
Which of the statement given above is/are correct?
(c) 3 : 2 (d) 3: 2 [2009-II]
29. The centroid and the orthocentre are coincident for which (a) Only I (b) Only I
one of the following triangles? [2009-I] (c) Both II and III (d) Both I and III
(a) Scalene triangle 37. ABC is a triangle and the perpendicular drawn from A meets
(b) Isosceles triangle BC in D. If AD2 = BC.DC, then
(c) Equilateral triangle Which one of the following is correct? [2009-II]
(d) Right angled triangle
EBD_7367
300
A- Triangles and its Properties
(a) ABC must be an obtuse angled triangle Which of the statements given above is/are correct?
(b) ABC must be an acute angled triangle [2010-I]
(c) Either ÐB ³ 45º or ÐC ³ 45º (a) Only I (b) Only II
(d) BC2 = AB2 + AC2 (c) Both I and II (d) Neither I nor II
38. The bisectors of the angles ABC and BCA of a D ABC meet 45. Consider the following in respect of the given figure
in a point O. What is the angle at O facing the side BC? C
[2009-II]
æ Aö æ Aö
(a) 90º – ç ÷ (b) 90º + ç ÷
2
è ø è2ø E D
(c) 90º – A (d) 90º + A
39. What is the number of points in the plane of a DABC which
are at equal distance from the vertices of the triangle? A B
[2009-II] I. DDAC ~ DEBC II. CA/CB = CD/CE
(a) 0 (b) 1 III. AD/BE = CD/CE
(c) 2 (d) 3 Which of the above are correct? [2010-I]
40. A (a) I, II and III (b) I and II
(c) I and III (d) II and III
a a 1
46. The median BD of the DABC meets AC at D. If BD = AC,
x 2
30° 60° 30° then which one of the following is correct? [2010-I]
(a) ÐACB = 1 right angle
B D C (b) ÐBAC = 1 right angle
In the above figure, what is x equal to? [2010-I] (c) ÐABC = 1 right angle
a a (d) None of the above
(a) (b) 47. The three sides of a triangle are 10, 100 and x. Which one of
3 2
the following is correct? [2010-I]
a a (a) 10 < x < 100 (b) 90 < x < 110
(c) (d) (c) 90 £ x £ 100 (d) 90 £ x < 110
3 2
41. Statement I Let LMN be a triangle. Let P, Q be the mid- 48. In the figure given below, what is the sum of the angles
points of the sides LM and LN, respectively. If PQ2 = MP2 formed around A, B, C except the angles of the DABC?
+ NQ2, then LMN is a right angled triangle at L. [2010-II]
Statement II If in a DABC, AB2 > BC2 + CA2, then ÐACB A
is obtuse. [2010-I]
Which of the following is correct of the above statements?
(a) Both Statements I and II are individually true and
Statements II is the correct explanation of Statement I
(b) Both Statements I and II are individually true but
Statement II is not the correct explanation on
Statement I
(c) Statement I is true and statement II is false
(d) Statement I is false and statement II is true B C
42. ABC is a triangle. The internal bisector of ÐABC and the (a) 360º (b) 720º
external bisector of ÐACB meet at D. (c) 900º (d) 1000º
Which one of the following is correct? [2010-I] 49. In the given figure, ABC is an equilateral triangle of side
1 length 30 cm. XY is parallel to BC, XP is parallel to AC and
(a) ÐBDC = ÐBAC (b) ÐBDC = Ð BAC YQ is parallel to AB. If (XY + XP + YQ) is 40 cm, then what is
2
PQ equal to? [2010-II]
1
(c) ÐBDC = ÐDBC (d) ÐBDC = Ð ABC A
2
43. The following sets of conditions relate to two triangle ABC
and DEF. Which set of conditions does not guarantee the
congruence of DABC and DDEF? [2010-I]
(a) a = d, b = e, c = f
(b) ÐB = ÐE, ÐC = ÐF, a = d X Y
(c) a = f, b = e, ÐA = ÐD B C
(d) c = f, b = e, ÐB = ÐE P Q
44. Consider the following statements in respect of any triangle (a) 5 cm (b) 12 cm
I. The three medians of a triangle divide it into six triangles (c) 15 cm (d) None of these
of equal area. 50. Consider the following statements
II. The perimeter of a triangle is greater than the sum of the I. If two triangles are equiangular, then they are similar.
lengths of its three medians. II. If two triangles have equal area, then they are similar.
Triangles and its Properties A-301
Which of the statements given above is/are correct? Which one of the following is correct? [2011-I]
[2010-II] (a) AC2 = AD2 – 3CD2 (b) AC2 = AD2 – 2CD2
(a) Only I (b) Only II (c) AC2 = AD2 – 4CD2 (d) AC2 = AD2 – 5CD2
(c) Both I and II (d) Neither I nor II 59. PQR is a triangle right angled at Q. If X and Y are the mid-
51. If two corresponding sides of two similar triangles are in points of the sides PQ and OR respectively then which one
the ratio 9 : 4, then what is the ratio of their areas? of the following is not correct? [2011-I]
[2010-II] (a) RX2 + PY2 = 5XY2
(a) 9 : 4 (b) 3 : 2 (b) RX2 + PY2 = XY2 + PR2
(c) 81 : 16 (d) 27 : 8 (c) 4(RX2 + PY2) = 5PR2
52. In a triangle, if sum of two angles is equal to the third angle (d) RX2 + PY2 = 3(PQ2 + QR2)
(considering the interior angles only), then the triangle is 60. The centroid of a DABC is 8 cm from the vertex A. What is
[2010-II] the length of the median of the trianlgle through A?
(a) right angled (b) acute angled [2011-I]
(c) equilateral (d) obtuse angled (a) 20 cm (b) 16 cm
53. In the given figure, L is any point on the bisector of the (c) 12 cm (d) 10 cm
acute angle ABC and the line ML is parallel to BC. Which 61. DABC is an isosceles triangle such that AB = BC = 8 cm and
one of the following is correct? [2010-II] ÐABC = 90º. What is the length of the perpendicular drawn
A from B on AC? [2011-II]
(a) 4 cm (b) 4 2 cm
(c) 2 2 cm (d) 2 cm
M L 62. Q

B C L
(a) The D BML is equilateral
(b) The D BML is isosceles but right angled
(c) The D BML is isosceles but not right angled
(d) The D BML is not isosceles
54. The side BC of the DABC is extended to D. If ÐACD = 120º R M P
2 In the figure given above, LM is parallel to QR. If LM divides
and ÐABC = ÐCAB, then what is ÐBAC? [2010-II] the DPQR such that area of trapezium LMRQ is two times
3
(a) 60º (b) 45º PL
the area of DPLM, then what is equal to? [2011-II]
(c) 30º (d) 72º PO
55. Let D, E be the points on sides AB and AC respectively of 1 1
a DABC such that DE is parallel to BC. Let AD = 2 cm, DB = (a) (b)
1 cm, AE = 3 cm and area of DADE = 3 cm2. What is EC equal 2 3
to? [2011-I] 1 1
(a) 1.5 cm (b) 1.6 cm (c) (d)
2 3
(c) 1.8 cm (d) 2.1 cm 63. The point of concurrence of the altitudes of a triangle is
56. In DABC, a line PQ is drawn parallel to BC, points P, Q called [2011-II]
being on AB and AC, respectively. If AB = 3 AP, then what (a) circumcentre (b) orthocentre
is the ratio of the area of DAPQ to the area of DABC? (c) incentre (d) centroid
[2011-I] 64.
(a) 1 : 3 (b) 1 : 5 A
(c) 1 : 7 (d) 1 : 9
57. Consider the following statements
Statement I Let PQR be a triangle in which PQ = 3 cm, QR
= 4 cm and RP = 5 cm. If D is a point in the plane of the
DPQR such that D is either outside it or inside it, then DP +
DQ + DR > 6 cm 38°
Statement II PQR is a right–angled triangle.
Which one of the following is correct in respect of the B C D
above two statements? [2011-I] In the figure given, ÐB = 38º, AC = BC and AD = CD. What
(a) Both Statements I and II are individually true and is ÐD equal to? [2011-II]
Statements II is the correct explanation of Statement I (a) 26º (b) 28º
(b) Both Statements I and II are individually true but (c) 38º (d) 52º
Statement II is not the correct explanation on 65. Statement I Let the side DE of a DDEF be divided at S. so
Statement I DS 1
(c) Statement I is true and statement II is false that = . If a line through S parallel to EF meets DF
DE 2
(d) Statement I is false and statement II is true
58. ABC is a triangle right angled at B and D is a point on BC at T, then the area of DDEF is twice the area of the DDST.
produced (BD > BC), such that BD = 2DC. Statement II The areas of the similar triangles are
proportional to the squares on the corresponding sides.
EBD_7367
302
A- Triangles and its Properties
Which one of the following is correct in respect of the above 73. Let ABC be an equilateral triangle. If the side BC is produced
statements? [2012-I] to the point D so that BC = 2 CD, then AD2 is equal to
(a) Both Statements I and II are true and Statement II is [2013-I]
the correct explanation of Statement I (a) 3CD2 (b) 4CD2
(b) Both Statements I and II are true but Statement II is (c) 5CD2 (d) 7CD2
not the correct explanation of Statement I 74. ABC is a triangle, where BC = 2AB, ÐC = 30º and ÐA = 90º.
(c) Statement I is true but Statement II is false The magnitude of the side AC is [2013-I]
(d) Statement II is true but statement I is false 2 BC 3BC
66. The base of a triangular wall is 7 times its height. If the cost (a) (b)
of painting the wall at ` 350 per 100 sq m is ` 1225, then 3 4
what is the base length? [2012-I] BC 3BC
(a) 50 m (b) 70 m (3) ` (4)
(c) 75 m (d) 100 m 3 2
67. Which one among the following is not correct? [2012-I] 75. The bisectors BI and CI of the ÐB and ÐC of a DABC meet
(a) Two congruent triangles are necessarily similar in I. What is ÐBIC equal to? [2013-I]
(b) All equiangular triangles are similar A A
(c) Two isosceles right triangles are similar (a) 90º - (b) 90º +
4 4
(d) All isosceles triangles are similar
68. Consider the following statement in respect of an equilateral A A
DABC. (c) 90º – (d) 90º +
2 2
I. There is a point P inside the DABC such that each of its 76. In a DABC, ÐBCA = 90º and CD is perpendicular to AB. If
sides subtends an angle of 120º at P. AD = 4 cm and BD = 9 cm, then the value of DC will be
II. There is a point P inside the D ABC such that the D PBC [2013-I]
is obtuse angled and A is the orthocentre of DPBC. (a)
Which of the above statements is/are correct? [2012-I] 18 cm (b) 20 cm
(a) Only I (b) Only II (c) 65 cm (d) 6 cm
(c) Both I and II (d) Neither I nor II 77.
69. The medians of DABC intersect at G. Which one of the P
following is correct? [2012-I]
(a) Five times the area of DAGB is equal to four times the L
area of DABC
(b) Four times the area of DAGB is equal to three times the
area of DABC
(c) Three times the area of DAGB is equal to the area of Q R
DABC In the figure given above, ÐPQR = 90º and QL is a median,
(d) None of the above PQ = 5 cm and QR = 12 cm. Then, QL is equal to [2013-I]
70. In the figure given below ÐABC = ÐAED = 90º. (a) 5 cm (b) 5.5 cm
Consider the following statements [2012-I] (c) 6 cm (d) 6.5 cm
I. ABC and ADE are similar triangles. 78. In a right angled DABC, ÐC = 90º and CD is perpendicular
II. The four points B, C, E and D may lie on a circle. 1
Which of the above statements is/are correct? to AB. If AB × CD = CA × CB, then is equal to
CD 2
A
[2013-I]
1 1 1 1
E (a) 2
- 2 (b) 2
-
AB CA AB CB 2
1 1 1 1
(c) - (d) - , if CA > CB
D BC 2
CA 2
BC 2
CA2
79. Each side of the equilateral triangle is 6 cm. Its altitude is
C B [2013-I]
(a) Only I (b) Only II (a) 6 3 cm (b) 3 3 cm
(c) Both I and II (d) Neither I nor II
71. The side BC of a DABC is produced to D, bisectors of the (c) 2 3 cm (d) 3 cm
ÐABC and ÐACD meet at P. If ÐBPC = xº and ÐBAC = yº, 80. In a DABC, ÐBCA = 60º and
then which one of the following option is correct? AB2 = BC2 + CA2 + X, What is the value of X? [2013-I]
[2013-I] (a) (BC) (CA) (b) – (BC) (CA)
(a) xº = yº (b) xº + yº = 90º (c) (AB) (BC) (d) Zero
(c) xº + yº = 180º (d) 2xº = yº 81. In DABC, XY is drawn parallel to BC, cutting sides at X and
72. ABC is a right angled triangle such that AB = a – b, BC = a Y, where AB = 4.8 cm, BC = 7.2 cm and BX = 2 cm. What is the
and CA = a + b . D is a point on BC such that BD = AB. The length of XY? [2013-I]
ratio of BD : DC for any value of a and b is given by (a) 4 cm (b) 4.1 cm
[2013-I] (c) 4.2 cm (d) 4.3 cm
(a) 3 : 2 (b) 4 : 3
(c) 5 : 4 (d) 3 : 1
Triangles and its Properties A-303
82. Let ABC be a triangle with AB = 3 cm and AC = 5 cm. If AD lengths 1, 2 and 3 units, what is the perimeter of the DABC?
is a median drawn from the vertex A to the side BC, then [2013-II]
which one of the following is correct? [2013-I] (a) 18 units (b) 24 units
(a) AD is always greater than 4 cm but less than 5 cm (c) 48 units (d) Cannot be determined
(b) AD is always greater than 5 cm 90. PQR is an equilateral triangle. O is the point of intersection
(c) AD is always less than 4 cm of altitudes PL, QM and RN. If OP = 8 cm, then what is the
(d) None of the above perimeter of the DPQR? [2013-II]
83. In the figure given below, YZ is parallel to MN, XY is parallel (a) 8 3 cm (b) 12 3 cm
to LM and XZ is parallel to LN. Then MY is [2013-I]
L (c) 16 3 cm (d) 24 3 cm
91. In a DABC, ÐB = 90º and ÐC = 2 ÐA, then what is AB2 equal
to? [2013-II]
(a) 2BC2 (b) 3BC2
Z Y
(c) 4BC2 (d) 5BC2
92. The side AC of a DABC is produced to D such that BC =
CD. If DACB is 70°, then what is DADB equal to?
M N [2013-II]
X (a) 35º (b) 45º
(a) The median of DLMN
(b) the angular bisector of ÐLMN (c) 70º (d) 110º
(c) perpendicular to LN 93. Consider the following statements
(d) perpendicular bisector of LN I. If the diagonals of a par allelogr am ABCD are
84. The sum of the perpendiculars drawn from an interior point perpendicular, then ABCD may be a rhombus.
of an equilateral triangle is 20 cm. What is the length of side II. If the diagonals of a quadrilateral ABCD are equal and
of the triangle? [2013-I] perpendicular, then ABCD is a square.
Which of the statements given above is/are correct?
40 [2013-II]
(a) cm (b) 40 3 cm
3 (a) Only I (b) Only II
(c) Both I and II (d) Neither I nor II
20 94. ABC is triangle right angled at A and a perpendicular AD is
(c) 20 3 cm (d) cm drawn on the hypotenuse BC. What is BC.AD equal to?
3
85. The lengths of three line segments (in cm) are given in each [2013-II]
of the four cases. Which one of the following cases is not (a) AB.AC (b) AB.AD
suitable to be the three sides of a triangle? [2013-I] (c) CA.CD (d) AD.DB
(a) 2, 3, 4 (b) 2, 3, 5 95. ABC and XYZ are two similar triangles with ÐC = ÐZ, whose
(c) 2, 4, 5 (d) 3, 4, 5 areas are respectively 32 cm2 and 60.5 cm2. If XY = 7.7 cm,
86. Consider the following statements then what is AB equal to? [2013-II]
I. If G is the centroid of DABC, then (a) 5.6 cm (b) 5.8 cm
GA = GB = GC. (c) 6.0 cm (d) 6.2 cm
II. If H is the orthocentre of DABC, then 96. The diameter of circle with centre at C is 50 cm. CP is a
HA = HB = HC. radial segment of the circle. AB is a chord perpendicular to
Which of the statements given above is/are correct? CP and passes through P. CP produced intersects the circle
[2013-II] at D. If DP = 18 cm then what is the length of AB?
(a) Only I (b) Only II [2013-II]
(c) Both I and II (d) Neither I nor II (a) 24 cm (b) 32 cm
87. If the bisectors BI and CI of the angles B and C of a DABC (c) 40 cm (d) 48 cm
meet at the point I, then what is ÐBIC equal to? 97. Consider the following statements
[2013-II] I. The perpendicular bisector of a chord of a circle does
not pass through the centre of the circle.
A II. The angle in a semi-circle is a right angle.
(a) 2A (b) 90º +
2 Which of the statements given above is/are correct?
A [2013-II]
(c) 90º – (d) 90º + A (a) Only I (b) Only II
2 (c) Both I and II (d) Neither I nor II
88. E is the mid-point of the median AD of a DABC, If BE 98. The three sides of a triangle are 15, 25, x units.
produced meets the side AC at F, then CF is equal to Which one of the following is correct? [2014-I]
[2013-II] (a) 10 < x < 40 (b) 10 £ x £ 40
AC 2 AC (c) 10 £ x < 40 (d) 10 < x £ 40
(a) (b) 99. Which one of the following is a Pythagorean triple in which
3 3
one side differs from the hypotenuse by two units?
AC [2014-I]
(c) (d) None of these (a) (2n + 1, 4n, 2n2 + 2n) (b) (2n, 4n, n2 + 1)
2
89. A DDEF is formed by joining the mid-points of the sides of (c) (2n2, 2n, 2n + 1) (d) (2n, n2 – 1, n2 + 1)
DABC. Similarly, a DPQR is formed by joining the mid-points Where, n is a positive real number.
of the sides of the DDEF. If the sides of the DPQR are of
EBD_7367
A-304 Triangles and its Properties
100. The sides of a right angled triangle are equal to three Which of the above statements is/are correct?
consecutive numbers expressed in centimeters. What can (a) Only 1 (b) Only 2
be the area of such a triangle? [2014-I] (c) Both 1 and 2 (d) Neither 1 nor 2
(a) 6 cm2 (b) 8 cm2 109. In a DABC, if ÐB = 2ÐC = 2ÐA. Then, what is the ratio of
(c) 10 cm2 (d) 12 cm2 AC to AB ? [2014-II]
101. If triangles ABC and DEF are similar such that 2AB = DE (a) (b)
2 :1 3 :1
and BC = 8 cm, then what is EF equal to? [2014-I]
(a) 16 cm (b) 12 cm (c) 1 : 1 (d) 1: 2
(c) 10 cm (d) 8 cm 110. For a triangle, the radius of the circumcircle is double the
102. The sides of a triangle are in geometric progression with radius of the inscribed circle, then which one of the
common ratio r < 1. If the triangle is a right angled triangle, the following is correct ? [2014-II]
square of common ratio is given by [2014-I] (a) The triangle is a right-angled
(b) The triangle is an isosceles
5 +1 5 –1
(a) (b) (c) The triangle is an equilateral
2 2 (d) None of the above
3 +1 3 –1 111. Consider the following statements in respect of an equilateral
(c) (d) triangle : [2014-II]
2 2 1. The altitudes are congruent.
103. In a DABC, AD is perpendicular of BC and BE is
perpendicular to AC. Which of the following is correct? 2. The three medians are congruent.
[2014-I] 3. The centroid bisects the altitude.
(a) CE × CB = CA × CD Which of the above statements are correct ?
(b) CE × CA = CD × CB (a) 1 and 2 (b) 2 and 3
(c) AD × BD = AE × BE (c) 1 and 3 (d) 1,2 and 3
(d) AB × AC = AD × BE 112. Consider the following : [2014-II]
104. Let ABC is triangle right angled at B. If AB = 6 cm and BC = ABC and DEF are triangles in a plane such that AB is parallel
8 cm, then what is the length of the circumradius of the to DE, BC is parallel to EF and CA is parallel to FD.
DABC? [2014-I] Statement I If Ð ABC is a right angle, then Ð DEF is also a
(a) 10 cm (b) 7 cm right angle.
(c) 6 cm (d) 5 cm Statement II Triangles of the type ABC and DEF are always
105. If AD is the internal angular bisector of DABC with AB = 3 congruent.
cm and AC = 1 cm then what is BD : BC equal to? Which one of the following is correct in respect of the above
[2014-I] statements ?
(a) 1 : 3 (b) 1: 4 (a) Statements I and II are correct and Statement II is the
(c) 2 : 3 (d) 3 : 4 correct explanation of Statement I
106. AB is a straight line, C and D are points the same side of AB (b) Statements I and II are correct and Statement II is not
such that AC is perpendicular to AB and DB is perpendicular the correct explanation of Statement I
(c) Statement I is correct and Statement II is incorrect
AE BE (d) Statement I is incorrect and Statement II is correct
to AB. Let AD and BC meet at E. What is + equal
AD BC 113. Let the incircle to a DABC touch BC, AC and AB respectively
to? [2014-I] at the points X, Y and Z. [2014-II]
(a) 2 (b) 1.5 Statement I If AB > BC, then AB + AZ < BC + XC
(c) 1 (d) None of these Statement II AZ = AY
107. In a D ABC, AD is the median through A and E is the mid– Which one of the following is correct in respect of the above
point of AD and BE produced meets AC at F. Then, AF is statements ?
equal to [2014-II] (a) Statement I and II are correct and Statements II is the
(a) AC/ 5 (b) AC/ 4 correct explanation of Statement I
(c) AC/ 3 (d) AC/ 2 (b) Statement I and II are correct and Statement II is not
A the correct explanation of Statement I
(c) Statement I is correct and Statement II is incorrect
(d) Statement I is incorrect and Statement II is correct
F 114. Let ABC be a triangle in which ÐACB = 60° and AC = x <
E BC. Let the circle with centre at C and radius x meet BC at D.
Let CF be the perpendicular drawn from C meeting AD at F.
l Statement I DACD is isosceles but not equilateral.
G x
B C
Statement II DF = [2014-II]
D 2
108. Three straight lines are drawn through the three vertices of Which one of the following is correct in respect of the above
a DABC, the line through each vertex being parallel to the statements ?
opposite side. The DDEF is bounded by these parallel (a) Statements I and II are correct and Statement II is the
lines. [2014-II] correct explanation of Statement I
Consider the following statements in respect of theDDEF. (b) Statementes I and II are correct and Statement II is not
1. Each side of DDEF is double the side of DABC to the correct explanation of Statement I
which it is parallel. (c) Statement I is correct and Statement II is incorrect
2. Area of DDEF is four times the area of DABC. (d) Statemente I is incorrect and Statement II is correct
Triangles and its Properties A-305
115. If every side of an equilateral triangle is doubled, then the (a) Both Statement I and Statement II are true and
area of new triangle becomes k times the area of the old
Statement II are true and Statement II is the correct
one. What is k equal to? [2014-II]
explanation of Statement I.
(a) 3 (b) 2 (b) Both Statement I and Statement II are true but Statement
(c) 4 (d) 8 II is not the correct explanation of Statement I
116. The angles of a triangle are in the ratio 4 : 1 : 1. Then the (c) Statement I is true but Statement II is false
ratio of the largest side to the perimeter is [2015-I] (d) Statement I is false but Statement II is true
2 1 123. Suppose chords AB and CD of a circle intersect at a point P
(a) (b) inside the circle. Two right-angled triangles A'P'B' and C'Q'D'
3 2+ 3
are formed as shown in the figures below such that A'P' =
3 2 AP, B'P' = BP, C'Q' = CP, D'Q' = DP and Ð A'P'B' = 90° =
(c) (d)
2+ 3 1+ 3 Ð C'Q'D': [2016-I]
117. Let a, b, c be the sides of a right triangle, where c is the
hypotenuse. The radius of the circle which touches the A'
sides of the triangle is [2015-I]
(a) (a + b–c)/2 (b) (a + b + c)/2 C'
(c) (a + 2b + 2c)/2 (d) (2a + 2b–c)/2
118. The area of the largest triangle that can be inscribed in a
semicircle of radius r is [2015-I]
(a) r2 (b) 2r2
(c) 3r2 (d) 4r2
119. Consider the following statements :
1. Let D be a point on the side BC of a triangle ABC. If P' B' Q' D'
area of triangle ABD = area of triangle ACD, then for all
points O on’ AD, area of triangle ABO = area of triangle Which of the following statements are not correct?
ACO. 1. A'P'B' and C'Q'D' are similar triangles, but need not be
2. If G is the point of concurrence of the medians of a congruent.
triangle ABC, then area of triangle ABG = area of 2. A'P'B' and C'Q'D' are congruent triangles.
triangle BCG = area of triangle ACG. 3. A'P'B' and C'Q'D' are triangles of same area.
Which of the above statements is /are correct? [2015-I] 4. A'P'B' and C'Q'D' are triangles of same perimeter.
(a) 1 only (b) 2 only Select the correct answer using the code given below.
(c) Both 1 and 2 (d) Neither 1 nor 2 (a) 2 and 3 only (b) 1 and 3 only
120. The point O is equidistant from the three sides of a triangle (c) 1, 2 and 4 only (d) 1, 2, 3 and 4
ABC. Consider the following statements : [2015-II] 124. Suppose ABC is a triangle with AB of unit length D and E
1. ÐOAC + ÐOCB + ÐOBA = 90° are the points lying on AB and AC respectively such that
2. ÐBOC = 2 ÐBAC BC and DE are parallel. If the area of triangle ABC is twice
the area of triangle ADE, then the length of AD is [2016-I]
3. The perpendiculars drawn from any point on OA to
AB and AC are always equal 1 1
(a) unit (b) unit
Which of the above statements are correct ? 2 3
(a) 1 and 2 only (b) 2 and 3 only 1 1
(c) unit (d) unit
(c) 1 and 3 only (d) 1, 2 and 3 2 3
121. How many right angled triangles can be formed by joining
125. Let the triangles ABC and DEF be such that Ð ABC
the vertices of a cuboid ? [2015-II]
(a) 24 (b) 28 = Ð DEF, Ð ACB = Ð DFB and Ð BAC = Ð EDF. Let L
be the midpoint of BC and M be the midpoint of EF. Consider
(c) 32 (d) None of the above
the following statements: [2016-I]
122. Let ABC and A'B'C' be two triangles in which AB > AB', Statement I.
BC > B'C' and CA > C'A'. Let D, E and F be the mid. Points Triangles ABL and DEM are similar.
of the sides BC, CA and AB respectively. Let D', E' and F' Statement II.
be the midpoints of the sides B'C', C'A' and A'B' respectively. Triangle ALC is congruent to triangle DMF even in AC ¹
Consider the following statement: [2016-I] DF
Statement I. Which one of the following is correct in respect of the above
AD > A'D', BE > B'E' and CF > C'F' are always true. statements?
Statement II. (a) Both Statement I and Statement II are true and
Statement II is the correct explanation of Statement I.
AB 2 + BC 2 + CA2 A¢ B¢ 2 + B ¢C ¢ 2 + C ¢A¢ 2
= (b) Both Statement I and Statement II are true but Statement
AD 2 + BE 2 + CF 2 A¢ D ¢ 2 + B ¢E ¢ 2 + C ¢F ¢ 2 II is not the correct explanation of Statement I
Which one of the following is correct in respect of the above (c) Statement I is true but Statement II is false
statements? (d) Statement I is false but Statement II is true
EBD_7367
306
A- Triangles and its Properties

126. ABC and DEF are similar triangles. If the ratio of side AB to A
132.
side DE is ( 2 + 1) : 3, then the ratio of area of triangle
ABC to that of triangle DEF is [2016-I]
(a) (3 – 2 2) : 3 (b) (9 – 6 2) : 2
P
(c) 1: (9 – 6 2) (d) (3 + 2 2) : 3 K

p
127. In a triangle ABC if A – B = , then C + 2B is equal to B Q C
2
[2016-I] ABC is a triangle right angled at C as shown in the figure
above. Which one of the following is correct? [2016-I]
2p 3p (a) AQ2 + AB2 = BP2 + PQ2
(a) (b) (b) AQ2 + PQ2 = AB2 + BP2
3 4
(c) AQ2 + BP2 = AB2 + PQ2
p (d) AQ2 + AP2 = BK2 + KQ2
(c) p (d)
2 133.
128. Let ABC be a triangle in which AB = AC. Let L be the locus C 96°
of points X inside or on the triangle such that BX = CX.
Which of the following statements are correct? [2016-I]
1. L is a straight line passing through A and in-centre of
triangle ABC is on L.
2. L is a straight line passing through A and orthocentre
of triangle ABC is a point on L.
3. L is a straight line passing through A and centroid of A D B
triangle ABC is a point on L.
In the figure given above, AD = CD = BC. What is the value
Select the correct answer using the code given below:
(a) 1 and 2 only (b) 2 and 3 only of Ð CDB [2016-I]
(c) 1 and 3 only (d) 1, 2 and 3 (a) 32°
129. In a triangle PQR, point X is on PQ and point Y is on PR (b) 64°
(c) 78°
such that XP = 1 5 units, XQ = 6 units, PY = 2 units and Y R
(d) Cannot be determined due to insufficient data
= 8 units. Which of the following are correct? [2016-I]
134. ABC is an equilateral triangle and X,Y and Z are the points
1. QR = 5XY on BC, CA and AB respectively such that BX = CY = AZ.
2. QR is parallel to XY. Which of the following is/are correct? [2016-I]
3. Triangle PYX is similar to triangle PRQ. 1. XYZ is an equilateral triangle.
Select the correct answer using the code given below. 2. Triangle XYZ is similar to triangle ABC.
(a) 1 and 2 only (b) 2 and 3 only Select the correct answer using the code given below.
(c) 1 and 3 only (d) 1, 2 and 3 (a) 1 only (b) 2 only
130. A person travels 7 km north and then turns right and travels (c) Both 1 and 2 (d) Neither 1 nor 2
3 km and further turns right and travels 13 km. What is the 135. Let ABC be a right angled triangle with BC = 5 cm and AC =
shortest distance of the present position of the person from 12 cm. Let D be a point on the hypotenuse AB such that
his starting point? [2016-I] ÐBCD = 30°. What is length of CD ? [2016-II]
(a) 6 km (b) 3 5 km 60 17
(a) cm (b) cm
13 2
(c) 7 km (d) 4 5 km
131. ABC is a triangle in which D is the midpoint of BC and E is 120 120
(c) cm (d) cm
the midpoint of AD. Which of the following statements is/ 5 + 12 2 5 + 12 3
are correct? [2016-I] 136. In an equilateral triangle another equilateral triangle is drawn
1. The area of triangle ABC is equal to four times the area inside joining the mid–points of the sides of given
of triangle BED. equilateral triangle and the process is continued up to 7
2. The area of triangle ADC is twice the area of triangle times. What is the ratio of area of fourth triangle to that of
BED. seventh triangle ?
Select the correct answer using the code given below. [2016-II]
(a) 1 only (b) 2 only (a) 256 : l (b) 128 : l
(c) Both 1 and 2 (d) Neither 1 nor 2 (c) 64 : l (d) 16 : 1
Triangles and its Properties 307
A-

(a) ÐPQT (b) ÐLRM


137. The sides of a triangle are given by a 2 + b 2 , c2 + a 2 (c) Ð RML (d) ÐQPT
and (b + c) where a, b, c are positive. What is the area of the 144. In the figure given below, ABC is a triangle with AB = BC
triangle equal to ? [2016-II] and D is an interior point of the triangle ABC such that
a 2 + b2 + c 2 ÐDAC = Ð DCA. [2017-I]
(a) B
2
a 2 b 2 + b 2 c2 + c 2 a 2
(b)
2
a(b + c) D
(c)
2
A C
3(a 2 b2 + b2 c2 + c2a 2 )
(d) Consider the following statements :
2 1. Triangle ADC is an isosceles triangle.
138. What is area of largest triangle inscribed in a semi circle of 2. D is the centroid of the triangle ABC.
radius r units ? [2016-II]
(a) r2 square units 3. Triangle ABD is congruent to the triangle CBD.
(b) 2r2 square units Which of the above statements are correct ?
(c) 3r2 square units (a) l and 2 only
(d) 4r2 square units (b) 2 and 3 Only
139. Which one of the following triples does not represent the (c) 1 and 3 only
sides of a triangle ? [2017-I] (d) 1, 2 and 3
(a) (3, 4, 5) (b) (4, 7, 10) 145. In the figure given below, M is the mid-point of AB and
(c) (3, 6, 8) (d) (2, 3, 6) ÐDAB = ÐCBA and ÐAMC = ÐBMD. Then the triangle
140. The angles of a triangle are in the ratio 2 : 4 : 3. The smallest
angle of the triangle is. [2017-I] ADM is congruent to the triangle BCM by [2017-I]
(a) 20° (b) 40°
(c) 50° (d) 60° C D
141. ABC is a triangle and D is a point on the side BC. If BC = 12
cm, BD = 9 cm and ÐADC = ÐBAC, then the length of AC
is equal to [2017-I]
(a) 5 cm (b) 6 cm
(c) 8 cm (d) 9 cm
142. In the figure given below, Ð A = 80° and ÐABC = 60°. BD
and CD bisect angles B and C respectively. What are the A B
values of x and y respectively ? [2017-I] M
A
(a) SAS rule (b) SSS rule
(c) ASA rule (d) AAA rule
146. ABCD is a square. X is the mid-point of AB and Y is the
D mid-point of BC. [2017-I]
y° x° Consider the following statements :
x° 1. Triangles ADX and BAY are congruent.
C 2. Ð DXA = Ð AYB.
B
(a) 10 and 130 (b) 10 and 125 3. DX is inclined at an angle 60° with AY.
(c) 20 and 130 (d) 20 and 125 4. DX is not perpendicular to AY.
143. In the figure given below, PQR is a non-isosceles right- Which of the above statements are correct ?
angled triangle, right angled at Q. If LM and QT are parallel (a) 2, 3 and 4 only (b) 1, 2 and 4 only
and QT = PT, then what is ÐRLM equal to ? [2017-I] (c) 1, 3 and 4 only (d) 1 and 2 only
P 147. In a trian gle ABC, AD is perpendicular on BC.
If ÐBAC = 90°, AB = c, BC = a, CA = b and AD = p, then
which one of the following is correct? [2017-II]
(a) p = abc (b) p2 = bc
T bc ab
(c) p= (d) p =
a c
M 148. In an equilateral triangle ABC, BD is drawn perpendicular to
AC. What is BD2 equal to? [2017-II]
R (a) AD2 (b) 2AD2
Q L (c) 3AD2 (d) 4AD2
EBD_7367
308
A- Triangles and its Properties

149. If PL, QM and RN are the altitudes of triangle PQR whose 1. The orthocentre of a triangle always lies inside the
orthocentre is O, then Q is the orthocentre of the triangle triangle.
[2017-II] 2. The centroid of a triangle always lies inside the triangle.
(a) OPQ (b) OQR 3. The orthocentre of a right angled triangle lies on the
(c) PLR (d) OPR triangle.
150. In triangle ABC, ÐC = 90° and CD is the perpendicular from 4. The centroid of a right angled triangle lies on the
C to AB. triangle.
If (CD)–2 = (BC)–2 + (CA)–2, then which one of the following Which of the above statements are correct?
is correct? [2017-II] (a) 1 and 2 (b) 1 and 4
(a) BC . CD = AB . CA (c) 2 and 3 (d) 2 and 4
(b) AB . BC = CD . CA 157. Consider the following statements : [2018-I]
(c) CA2 + CB2 = 2 (AD2 + CD2) Two triangles are said to be congruent, if
(d) AB . CD = BC . CA 1. Three angles of one triangle are equal to the
151. In a triangle ABC, the medians AD and BE intersect corresponding three angles of the other triangle.
at G. A line DF is drawn parallel to BE such that F is 2. Three sides of one triangle are equal to the
on AC. If AC = 9 cm, then what is CF equal to? [2017-II] corresponding three sides of the other triangle.
(a) 2.25 cm (b) 3 cm 3. Two sides and the included angle of one triangle are
(c) 4.5 cm (d) 6 cm equal to the corresponding two sides and the included
152. In a triangle PQR, X is a point on PR and Y is a point angle of the other triangle.
on QR such that PR = 10 cm, RX = 4 cm, YR = 2 cm, 4. Two angles and the included side of one triangle are
QR = 5 cm. Which one of the following is correct? equal to the corresponding two angles and the included
[2017-II] side of the other triangle.
(a) XY is parallel to PQ (b) PQ = 2XY Which of the above statements are correct?
(c) PX = QY (d) PQ = 3XY (a) 1, 2 and 3 (b) 1, 3 and 4
153. One-fifth of the area of a triangle ABC is cut off by a line DE (c) 1, 2 and 4 (d) 2, 3 and 4
drawn parallel to BC such that D is on AB and E is on AC. 158. A square and an equilateral triangle have the same perimeter.
If BC = 10 cm, then what is DE equal to? [2017-II] If the diagonal of the square is 6 2 cm, then what is the
(a) 5 cm (b) 2 5 cm area of the triangle? [2018-I]
(c) 3 5 cm (d) 4 5 cm (a) 12 2 cm 2 (b) 12 3 cm 2
154. Consider the following statements : [2017-II]
(c) 16 2 cm 2 (d) 16 3 cm 2
1. The point of intersection of the perpendicular
bisectors of the side of a triangle may lie outside 159. In the equilateral triangle ABC given below, AD = DB and
the triangle. AE = EC. If l is the length of a side of the triangle, then what
2. The point of intersection of the perpendiculars is the area of the shaded region? [2018-I]
drawn from the vertices to the opposite sides of a
A
triangle may lie on two sides.
Which of the above statements is/are correct?
(a) 1 only (b) 2 only
(c) Both 1 and 2 (d) Neither 1 not 2 D E
155. ABC is a triangle right angled at C with BC = a and
AC = b. If p is the length of the perpendicular from C on AB,
then which one of the following is correct? [2018-I]
(a) a2 b2 = p2 (a2 + b2) B C
(b) a2 b2 = p2 (b2 – a2)
(c) 2a2 b2 = p2 (a2 + b2)
3 3 l2 3 l2
(d) a2 b2 = 2p2 (a2 + b2) (a) (b)
156. Consider the following statements : [2018-I] 16 16

3 3 l2 3 l2
(c) (d)
32 32
Triangles and its Properties A-309

HINTS & SOLUTIONS


1. (a) Given that, AB = 5 cm, BC = 4 cm, CA = 4.2 cm and DE = 6. (d) DE : BE =3:5
10 cm, EF = 8 cm and FD = 8.4 cm 2
Area of DABC æ BC ö 25
A D \ = ç ÷ = or 25 : 9
Area of DDAE è DE ø 9
7. (b): Let each base angle of isosceles triangle be x.
4.2

A
m

8.4
m

c
5c

cm

10

cm
B C E F (x + 15)°
L M
4 cm 8 cm
AB 5 1
Now, = = x x
DE 10 2 B C
\ Vertical angle of an isosceles triangle = x + 15°
BC 4 1 We know that,
= =
EF 8 2 ÐA + ÐB + C = 180°
CA 4.2 1 Þ x + 15° + x + x = 180°
and = = Þ 3x = 165° Þ x = 55°
FD 8.4 2 8. (a) Assertion (A) is true, of two triangles are congruent then
AB BC CA their corresponding angles and side are same.
\ = = Reason (R) is alsoe true, because two congruent triangle
DE EF FD
are same area.
\ D ABC ~ D DEF
So A, R are inddividually true R is correct explanation of A.
We know that,
9. (b) Given, Area (D ADE) : Area (trape BDEC) = 4 : 5
AB AL area (D ADE) = 4x
=
DE DM and area of trapezium BDEC = 5x
Area of D ABC = 4x + 5x = 9x
AL 1
Þ = Now, D ADE and D ABC
DM 2
Alternate Method area DADE DE 2
area DABC
=
Every sides of second is double of first one. BC 2
AL 1
Hence, = . 4x DE 2 DE 2
DM 2 Þ = 2 Þ BC = 3
9x BC
2. (b) When D ABC is moved around fixed ponit A then its locus
is circle. \ DE : BC = 2 : 3
3. (a) BD : CD = 3 : 4 and AB : AC = 6 : 8 or 3 : 4 in D ABC, AD 10.(d) A triangle can be constructed by given all three
is a angular bisector. statements.
Both A and R are true and R is the correct explanation of A D
A.
4. (c) According to triangle property.
E
B G
D
B F C
90° 11.(c) : Q AB || EF || CD
A C
D ABC is a right angled triangle at A. D
AD ^ BC, then D ABC ~ D ADC ~ D ADB are similar is A
each other. 5 cm
\ Option (c) is correct.
18 cm
15 cm

5. (c) Also QR is parallel to AB. E


G
\ D PQR ~ D QPB 10
9 cm cm
DR is also parallel to QB.
D PQB ~ D QDR B F C
Again, DR|| QB and QR|| AB
\ DDQR ~ D AQB In D EFG and D CDG, Here triangle EFG and D CDG
similar
EBD_7367
A-310 Triangles and its Properties

EG EF 5 EF 16. (a) DACB is right angled at C.


= Þ = c2 = a2 + b2
GC CD 10 18
Þ EF = 9 cm Ak
Also, DABC and D EFC are similar
In D ABC and D EFC, Dc
b
EC EF 15 9
= Þ = p
AC AB AC 15
C a B
15 ´15
Þ AC = = 25 cm 1
9
Area of DACB = ´ AB ´ DC
12.(d) In D ABC and D PQC, 2
B 1 1
Q ´a´b = ´ c´ p
2 2
x a æ 2 2ö
ab = p ç a + b ÷ (\ C2 = a2 + b2)
è ø
A c C Squaring on both sides we get
P b a2b2 = p2(a2 + b2)
Alternate Method : DACB and D CDB are similar
PC PQ AC DC b P
\ = \ = Þ = Þ ab = cp
AC AB AB BC c a
b a æ 2 2ö
Þ
c+a
=
x Þ ab = p ç a + b ÷
è ø
a (c + b ) ac Squaring on both sides we get,
\ x = = +a Þ a2b2 = p2(a2 + b2)
b b
13. (a) 17. (a) In D ADC, A
E F
C
D

A B
30° 20°
A. By the properties of triangle, it is true. B C
D
R. It is also true that the distance between two parallel lines
is same everywhere. ÐADC = 180° – 30° = 150°
Hence, A and R are true and R is the correct explanation of A. \ ÐDAC = 180° – (150° + 20°) = 10°
14. (c) Suppose the smaller and larger sides of a right triangle 18. (c) Ratio of sides, A
be x and y, respectively. According to angles order.
By given condition, AC : AB : BC
30°
x2 + y2 = (3 10)2 3 1
=1: :
Þ x2 + y2 = 90 ... (i) 2 2
60°
and 9x2 + 4y2 = 405 ... (ii) = 2: 3 :1 B C
On solving Eqs. (i) and (ii), we get 19.(b) Let a = 30 m, b = 40 m and c = 50 m
x = 3 units and y = 9 units
15. (c) D QRS is an isosceles triangle. C
S

m
30

40
m

100°
P Q R
\ ÐQSR = ÐSQR = 40° A D B
50 m
Þ ÐPQS = Ð180° – ÐRQS = Ð140°
Again, DÐPQS is an isosceles triangle. Now, 2s = 30 + 40 + 50
Þ s = 60 m
180 - 140 40
\ ÐPQS= ÐQSR = = = 20° \ D = 60(60 - 30)(60 - 40)(60 - 50)
2 2
Triangles and its Properties 311
A-

= 60 ´ 30 ´ 20 ´10 = 600 4 A2
AC = + b2
1 600 ´ 2 b2
Þ ´ AB ´ CD = 600 Þ CD = = 24 m
2 50 Again in D ABC
20.(b) In D ABD, 1
D A = ´ AC ´ BD
C 2
90° 2A 2A
90° BD = =
2 2
4A b 4 A2 + b 4
+
x y b2 1 b2
30° 45° 2 Ab
A B =
ÐDAB + ÐABD + ÐBDA = 180° 4 A2 + b 4
Þ (30° + x) + 45° + 90° = 180° Þ x = 15° 23.(b) By Pythagoras theorem,
In D ACB,
ÐCAB + ÐABC + ÐBCA = 180° C
Þ 30° + (45° + y) + 90° = 180° Þ y = 15°
\ 2x – y = 2(15°) – 15° = 15°
21. (c) In right angled D ABN, A
4 D

B C A B
N 3
By Pythagoras theorem, CB2 = AC2 + AB2 = 16 + 9 = 25
Þ CB = 5
BC 2
AN2 = AB2 + BN2 = AB2 + 1
4 Area of D ABC = ´ 3´ 4 = 6
2
æ BC ö
çQ BN = ÷ ... (i) 1
è 2 ø Again in D ABC, Area = ´ BC ´ AD
In D CBM, 2
AB 2 5 12
CM2 = BC2 + BM2 = BC2 + 6 = AD Þ AD =
4 2 5
In right angled D ADB,
æ AB ö AB2 = AD2 + BD2
çQ BM = ÷ ... (ii)
è 2 ø 2
From Eqs. (i) and (ii) æ 12 ö 2
Þ (3) = ç ÷ + BD
2

AB 2 BC 2 è ø5
2 2 2
AN + CM = AB + + BC2 +
4 4 144 9
Þ BD = 9- =
2 2 25 5
5( AB + BC )
=
4 1
Þ 4(AN2 + CM2) = 5AC2 \ Area of D ABD = ´ BD ´ AD
2
1 1 9 12 54
22. (a) Area of D ABC, A= ´ b ´ AB ´ ´
2 = = sq units
2 5 5 25
2A 24. (a) Since, ÐABD= ÐPQD = 90°
AB = ... (i) So, D ABD ~ D PQD (right angled triangle)
b
A x BD
Then, = QD (by Thales theorem) ...(i)
z
D Since, ÐCDB = ÐPQB = 90°
So, D BCD ~ D BPQ
z BQ
C \ = (by Thales theorem)
B b y BD
By Phthagoras theorem, AC2 = AB2 + BC2
EBD_7367
A-312 Triangles and its Properties

z BD - QD z QD AO
Þ = Þ = 1- 31. (c) =2:7
y BD y BD OD
(where, AD is a median)
z z A
Þ y
= 1- [from Eq. (i)]
x
z z 1 1 1 O
Þ + =1Þ x + y = E
x y z
25.(d) In right angle D PQR G
R
B C
D
2 7
OA = AD, OD = AD
5 cm 9 9
4 cm
We know that, centroid makes a ratio 2 : 1 on the median
2 1
So, AG = AD, GD = AD ... (i)
3 3
Q P
2
QP2 = (5)2 – (4)2 = 9 Þ QP = 3cm A. OA = AD
In second D ABC whose sides are 3 cm, 4 cm and 5 cm, 9
So, the sides of both triangle are same, hence they are æ 1 ö 1 2GD
congruent. OA = ç 2. AD ÷ = [from Eq. (i)]
26. (a) When both triangles are congruent then both shape and è 3 ø3 3
size are equal. 7 æ 2 ö 1 7 AG
So, C1 and C2 are the same points, and r1 = r2 R. OD = AD = ç 7. AD ÷ =
27.(b) PQ is parallel to AC. 9 è 3 ø 3 ´ 2 6
\ DPQB ~ D ABC Hence, A is true but R is false.
32.(b) We know that, in a right angled triangle, hypotenuse is a
C largest side.
In D ABD, AB2 > AD2 ... (i)
In D BEC, BC2 > BE2 ... (ii)
In D ACF, AC2 > CF2 ... (iii)
Q A
R
S

F E
A B
P O
In D PQR and D ASC,
ÐASC = ÐRPQ (alternate angle) B C
ÐR = ÐS = 90° Þ ÐA = ÐQ (rest angle) D
\ D PQR ~ D ASC On adding Eqs. (i), (ii) and (iii), we get
Two distinct similar triangles are exist. (AB2 + BC2 + AC2) > (AD2 + BE2 + CF2)
3 R. Now,
28. (c) Median of an equilateral triangle = a (AE2 – AF2) + (BF2 – BD2) + (CD2 – CE2)
2 = [(OA2 – OE2) – (OA2 – OF2)] + [(OB2 – OF2) – (OB2 –
3 OD2)] + [(OC2 – OD2) – (OC2 – OE2)] = 0
a1
2 3 Hence, both A and R are individually true but R is not
According to question, = correct explanation of A.
3 2
a2
2 1
33. (c) Area of D ABC = ab
a1 3 2
\ = or 3 : 2 A
a2 2
29. (c) In an equilateral triangle, the centroid and the othocentre
are coincident.
30.(d) (A) If two triangles have same perimeter, then it is not
necessary that they have same area. So, they need not b c
be congruent.
(R) This condition is true, because two triangles are p
congruent by (SSS) property.

C a B
Triangles and its Properties A-313

1 AD DC
Again area of DABC = cp Þ =
2 BD AD
1 1 Hence, DABC must be right angled triangle.
\ ab = pc Þ pc = ab So, BC2 = AB2 + AC2
2 2 38.(b) Q ÐA + ÐB + ÐC = 180º
34.(b) In DABC, According to triangle property
A A

D
c = 2 cm

C
B a = 3 cm
O
b 2 + c 2 - a 2 4 2 + 2 2 – 32
cos A = =
2bc 2´ 4´ 2
B/2 C/2
16 + 4 – 9 11
Þ cos A = = ...(i) B
B/2 C/2
C
16 16
In DABD, Let BD = x, AB = c = 2 cm, AD = b = 2 cm B C A
Þ Ð + Ð = 90º -Ð
2 2 2
b2 + c 2 - a 2 11 22 + 22 – x 2 B C
\ cos A = Þ = In D BOC, Ð + Ð + ÐO = 180º
2bc 16 2´ 2´2 2 2
5 A A
Þ 11 = 16 – 2x2 Þ x2 = Þ x 2 = 2.5 Þ 90º -Ð + ÐO=180º Þ ÐO = 90º +
2 2 2
\ Area of square = (BD)2 = x2 = 2.5 cm2 39.(b) Number of points is one, because circumcentre is the only
35. (a) In D DCX, point in the plane of a triangle, which is equidistant from
CD = CX (given) the vertices of the triangle.
Ð3 = Ð4 OA= OB = OC = r
(opposite angles of same sides are equal.) A
But Ð3 = Ð5
A

1 2 O
r
B C

5
B C 40. (c)
D 3 A
90°
°

4 a a
30

x

X 30° 12 60° 30°
So, Ð4 = Ð5 ...(i)
In DABD and DACX, B D C
Ð1 = Ð2 (given) In DADC, ÐADB = 180° – ÐADC = 180º – 60º = 120º
Ð4 = Ð5 [from Eq. (i)] ÐDAC = 180º – 60º – 30º = 90º
ÐABD = ÐACX (rest angle) Again, in right angled DDAC,
Þ DABD~ DACX AC a a
36.(d) It is true that congruent triangles are similar but converse tan 60º = Þ 3= Þx=
is not true. Also, Statement III is true. DA x 3
37.(d) Here, AD2 = BD.DC 41.(b) Given, PQ2 = MP2 + NQ2
B N

D Q A B

L M C
P
A C
EBD_7367
A-314 Triangles and its Properties

Hence, both statements are individually true but statements II \ 10 + 100 > x ...(i)
is not the correct explanation of statement I. x > 100 – 10
Hence, option (b) is correct. and x > 90 ...(ii)
42.(b) We know that angle subtended by the bisect or two Difference of two sides of a triangle is always less than
internal angles, is the twice of the third angle and angle third side.
subtended the bisectors of one internal and one external From Eqs. (i) and (ii), we get,
angle, is the third angle. 90 < x < 110
According to this property 48. (c) ÐA = 360º – Ext ÐA. ....(i)
1 ÐB = 360 – Ext ÐB .... (ii)
ÐBDC = ÐBAC. ÐC = 360 – Ext ÐC .... (iii)
2
43.(d) Let, the set C = f, b = e, ÐB = ÐE. A
A D

c b f e
.
B a C E d F
Does not guarantee the congruence of D ABC and D DEF. B C
(According to congruence property of triangle)
44. (c) Similarly,
I. It is true that the three medians of a triangle divide it into ÐA + ÐB + ÐC = 180º
six triangle of equal area. From Eqs. (i), (ii), (iii) and (iv),
II. It is also true that, the perimeter of a triangle is greater 360º – Ext ÐA + 360º – Ext ÐB + 360º – Ext ÐC = 180º
than the sum of its three medians. Þ Ext Ð A + Ext ÐB + Ext ÐC
45. (a) In D CAD and D CEB, = 1080º – 180º
C = 900º
49.(d) Since, XP || AC, YQ || AB
ÐXBP = ÐYQC and ÐXPB = ÐYCQ

E D A

A B
ÐC = ÐC (common) X Y
ÐCEB = ÐADC (each 90º)
ÐCAD = ÐCBE (rest angle) B C
P Q
\ DCAD ~ CEB
Since, the sides will be in same proportion, So, DXBP and DYCQ are equilateral triangles.
Now, XY || BC
CA CD
= AX XY
CB CE \ = Þ AX = XY (Q AB = BC = 30 cm)
AB BC
AD CD Also XY + XP + YQ = 40
and =
BE CE AX + XB + YQ = 40 (Q XY = AX, XP = XB)
Hence, all three statements are correct. Þ AB + YQ = 40
46. (c) Here, we see that Þ YQ = 40 – 30 = 10 cm
C \ YQ = XP = 10 cm Þ BP = CQ = 10 cm
PQ = 30 – BP – CQ = 30 – 10 – 10 = 10 cm
50. (a) We know that, if two triangles are equiangular, then they
are similar (refer similarity conditions).
D Statement II is not true.
51. (c) Given, ratio of sides = 9 : 4
By properties of two similar triangle,
Area of first triangle (9)2 81
= =
B A Area of second triangle (4)2 16 or 81 : 16
CD = BD = DA 52. (a) In a triangle, if sum of two angles is equal to the third
This is possible only when ABC is right angled triangle. angle, then triangle is right angled.
47.(b) We know that, the sum of two sides is always greater than 53. (c) Since, BL is bisector of ÐABC.
third side.
Triangles and its Properties 315
A-

A In DABC and DAPQ, ÐA = ÐA and PQ || BC


DAPQ and DABC are similar triangles,
2
Area of DAPQ AP2 æ 1 ö 1
\ = = ç ÷ = or 1 : 9
M L Area of DABC AB 2
è 3ø 9
57. (a) Given, PQ = 3 cm, QR = 4 cm and RP = 5 cm
x Here, RP2 = PQ2 + QR2
x
B C So, PQR is a right angle triangle.
ÐMBL = ÐLBC = x (say) Both Statements I and II are individually true and
Also, ML || BC Statement II is the correct explanation of Statement I.
Ð LBC = Ð MLB = x Þ Ð MLB = Ð MBL Hence, option (a) is correct.
DBLM is an isosceles triangle and Ð BML need not to 58. (a) Given, BD = 2DC
be 90º.
A
DBML is isosceles but not right angled.
54.(d) Given, ÐACD = 120º
A

120°
B
B C D
Þ ÐCAB + ÐABC = 120º \ BC + CD = 2DCÞ BC = CD ...(i)
(since, exterior angle is equal to sum of two interior In DABC,
opposite angles) AC2 = AB2 + BC2 ...(ii)
2 5 In DABD,
Þ ÐCAB + ÐCAB = 120º Þ ÐCAB = 120º AD2 = AB2 + BD2 ...(iii)
3 3
Subtracting Eq. (ii) from Eq (iii), we get
120º ´3
Þ ÐCAB = = 72º AD2 – AC2 = BD2 – BC2 = (BD – BC) (BD + BC)
5 = CD (2CD + CD) = 3CD2
55. (a) In D ADE and D ABC AC = AD2 – 3CD2
2

A 59.(d) In D PQY

2 cm 3 cm P

D E
1 cm
X
B C
ÐA = ÐA
DE || BC
DADE ~ DABC Q Y R
AD AE
= According to Pythagoras theorem, PY2 = PQ2 + QY2
AB AC
2
AD AE æ QR ö æ QR ö
Þ = Þ PY2 =PQ2 + ç ÷ ç\ QY = YR = ÷ ...(i)
BD EC è 2 ø è 2 ø
2 3 3 and in D XQR,
= Þ EC = = 1.5 cm RX2 = QX2 + QR2
1 EC 2
A 2
æ PQ ö 2æ PQ ö
÷ + QR çQ PX = XQ =
56.(d) Þ RX2 = ç ...(ii)
÷
è 2 ø è 2 ø
P Q On adding Eqs. (i) and (ii), we get

5PQ 2 5QR 2
PY 2 + RX 2 = + Þ
B C .... (Given) 4 4
AB = 3AP 4(PY2 + RX2) = 5(PR2)
option (d) is not correct.
EBD_7367
A-316 Triangles and its Properties

60. (c) Let G centroid of a triangle, then 63. (b) The point of intersection of the altitudes of a triangle is
called orthocentre.
A 64. (b) Given, AC = BC
A

38° 76°
G

38°
B C
D B C D
In DABC, ÐABC = ÐCAB
In DABC, ÐABC = ÐCAB = 38° (\ AC = BC)
Q Then, AD is a median of DABC. ÐACB = 180º – (ÐABC + ÐCAB)
AG : GD = 2 : 1 = 180º – (38º + 38º) = 180º – 76º = 104º
Þ 8 : GD = 2 : 1 In DACD, ÐACD = 180º – 104º = 76º
and ÐACD = ÐCAD = 76º (Q CD = AD)
8 \ ÐADC = 180º – (ÐACD + ÐCAD)
Þ = 4 cm
GD = = 180º – (76º + 76º) = 28º
2
65. (a) D
\ AD = 8 + 4 = 12 cm
61. (b) By using Pythagoras theorem in DABC.
C
S T

P
8 cm
E F
DS 1
Given that, =
8 cmB A DE 2
2 2 2 2 DE = 2 × DS
AC =AB + BC = AC = 64 + 64
\ AC2 = 8 2 When two triangles DDTS and DDEF are similar then their
ratio of area is equal to square of corresponding sides.
ABC is isosceles right angle triangle, then
2
D DST æ 1 ö 1
AC Þ =ç ÷ = Þ D DEF = 2 D DST
AP = PC = PB = =4 2 D DEF è 2 ø 2
2
62. (b) In the given figure. So, both Statements I and II are true and Statement II is
the correct explanation of Statement I.
ar MRQL = 2 ar DPLM 66. (b) Let the height of the triangle be x, then, BC = 7x
Let area of D PLM be x, then
\ the area of trapezium = 2x 1 7
\ Area of the D ABC = ´ 7 x ´ x = x2
\ ar DPQR = 2x + x = 3x 2 2
Here it is clear from the given figure that D PQR ~ DPLM A
Q

L x
B C
D 7x
Cost of painting the wall at ` 350 per 100 sq m = ` 1225
\ Cost of painting 100 m2 = ` 350
R M P
350
ar D PQR 3x Cost of painting 1 m2 = `
= 100
\ ar D PLM x
7 2 350 7 2
2 \ Cost of painting x = ´ x
PL 1 PL 1 2 100 2
= =
PQ 2 3 \ PQ 3
Triangles and its Properties 317
A-

350 7 2 y
´ ´ x = 1225 (Given) \ yº = 2xº or x =
100 2 2
Alternate Method:—
2 1225 ´100 ´ 2 35 ´10 ´ 2 In DABC, yº + ÐABC + ÐACB = 180º ... (i)
Þ x = =
350 ´ 7 7 In DBPC, ÐBPC + ÐPBC + ÐPCB = 180º
= 5 × 10 × 2 = 100 = 10 m ÐABC æ ÐACD ö
Base = 7x = 70 m xº + +ç + ÐACB ÷ = 180º ... (ii)
2 è 2 ø
67. (d) All isosceles triangles are similar.
Comparing equation (i) and (ii)
68. (a) Statement–I

A xº =
2
72. (d) In DABC
Using Pythagoras theorem
P (a + b)2 = (a – b)2 + a2
A
30° 1 2 0° 30°
B C a+b
In equilateral triangle ABC, P is in incentre and AP = BP a–b
= CP and each side of an equilateral triangle make 120º
angle at P.
B (a–b) D b C
Statement–II a
2 2 2 2 2
From the figure it is clear that A is not an orthocentre of Þ a + b + 2ab = a + b – 2ab + a
the triangle PBC. Þ 4ab = a2 Þ 4b = a
Therefore option (a) is correct BD a – b 4b – b 3b 3
69. (c) Suppose DABC is an equilateral triangle. So, = = = =
DC b b b 1
A median divides an equilateral triangle into the three 73. (d) Let AB = BC = AC = x Because of equilateral triangle.
equal area of triangles.
x
(DABC ) Now, BM = MC =
DAGB = ar = ar BGC = ar DAGC 2
3
A
1
\ ar DAGB = DABC .
3
A
70. (a)
E B M C D
90° Then AM is the median.
x
D then, CD = (given BC = 2CD)
90° 2
Again, in DAMC,
C B
From fig. It is clear that ÐABC = ÐAED = 90º x 2 3x 2
So statement I is correct. AM2 = AC2 – MC2 = x2 – =
4 4
Statement II.
in DAMD,
Clear from fig that B, C, E and D do not lie on a circle.
71. (d) A P 3x 2 7x2
AD2 = AM2 + MD2 = + x2 =
x° 4 4
y° æ x2 ö 2 æ xö
= çç 4 ÷÷ = 7CD
7 çQ CD = ÷
è ø è 2 ø
74.(d) Given that, ÐA = 90º and ÐC = 30º
Then ÐB = 60º
C D
Given ÐBAC = yº B
and ÐBPC = xº
According to triangle property, the angle subtended by 60°
the bisector of an internal angle and an external angle is
half of the third angle.
90° 30°
A C
EBD_7367
A-318 Triangles and its Properties

In DABC, By Pythagoras theorem, From eq. (i) and (ii)


BC2 = AC2 + AB2 Þ (2AB)2 = AC2 + AB2
1 1 1
ÞAC2 = 4AB2 – AB2 = 3AB2 Þ AC = 3.AB 2
= 2

CD BC CA2
3 79. (b) Given that, DABC is an equilateral triangle.
= .(2 AB) \ AB = BC = CA = 6 cm
2
AD = Altitude
3 A
\ AC = .BC [Given DABC = 2AB]
2
75. (d) If the bisectors of angles B and C of a triangle meet at
A
point I, then BIC is equal to 90º +
2
76. (d) DADC ~ DBDC
A B D C
4
D 3 2
9 Area of equilateral triangle = a
4
C B
1 3 2
CD AD Þ ´b´h = a (here a2 b)
\ = 2 4
BD CD
CD2 = AD × BD 1 3
Þ ´ 6´ h = ´ 6 ´ 6 = h = 3 3 cm
CD2 = 9 × 4 2 4
CD = 36 = 6 cm 80. (b) According to cosine rule.

P AC 2 + BC 2 – AB 2 1
77. (d) cos 60º = =
2. AC.BC 2
13 cm A
5 cm L
90°
Q 12 cm R
PQ = 5 cm
QR = 12 cm 60°
QL is a median B C
PR Þ AC2 + BC2 – AB2 = AC.BC
\ PL= LR = ...(1) Þ AB2 = AC2 + BC2 – AC.BC
2
In DPQR, Þ Required value of X = – (BC)(CA)
PQ2 + QR2 = PR2 (Pythagoras theorem) 81. (c) Given
52 + 122 = PR2 AB = 4.8 cm
25 + 144 = PR2 BC = 7.2 and BX = 2 cm
According to theorem, PR = 13 cm A
If L is the mid point of the hypotenuse PR of a right angled
triangle PQR, then
1 1 X Y
OL = PR = ´13 = 6.5 cm
2 2
78. (c) A
B C
D \ AX = AB – BX = 4.8 – 2 = 2.8 cm
DAXY » DABC
90° 90°
XY AX AX 2.8
\ = Þ XY = .BC = ´ 7.2
C B BC AB AB 4.8
In DABC \ XY = 4.2 cm
CD ^ AB 82. (c) According to theorem:– the sum of any two sides of a
and AB × CD = CA × CB .... (i) triangle is greater than twice the median drawn to the third
In DCDB side.
BC2 = BD2 + CD2 ... (ii)
CD2 = BC2 – BD2
Triangles and its Properties 319
A-

A 88.(b) We draw a line segment parallel to BF.


A
5 cm
3 cm F
E G
B D C
(AB + AC) > 2 AD
(3 + 5) > 2AD B D C
AD < 4 In DADG,
Thus, AD is always less than 4 cm. BF or EF || DG and AE = ED (since, E is mid-point of AD)
83. (a) Given that, YZ || MN and XZ || LN \ AF = GC ... (i)
Similarly, in DBCF,
\ XNYZ is a parallelogram.
Þ ZX = YN ...(i) DG || BF and BD = DC
Also, ZX ||YN and XY || ZL FG = GC ... (ii)
Hence, XYLZ is a parallelogram. From Eqs. (i) and (ii),
\ XZ = LY ...(ii) 2
Now, From Eqs. (i) and (ii), CF = AC
\ YN = LY 3
So, MY is a median of DLMN. 89.(b) Perimeter of DPQR = 1 + 2 + 3 = 6 units
3 2 A
84. (a) Area of equilateral triangle = a
4
1 3 2 3 P
a´h = D E
\ a \h= a
2 4 2
Q R
2
or a =
3h B F C
We know, the length of a side of an equilateral triangle = Now, in DDEF,
2
(sum of the perpendiculars drawn from an interior DQ 1 PQ
3 = =
DF 2 FE
points) So, 2 PQ = FE
2 40 Similarly, DF = 2 PR and DE = 2QR
´ 20 = cm
3 3 \ perimeter of DDEF = 2 × 6 = 12 units
85. (b) We know that, in any triangle the sum of two sides is Similarly, perimeter of DABC = 2 × Perimeter of DDEF
always greater than its third side and the difference of = 2 × 12
two sides is always less than its third side. = 24 units
Only option (b) is not satisfy the above conditions 90. (d) PQR is an equilateral.
(i) 2 + 3 >| 5 (ii) | 5 – 2| <| 3 Then, PL is also the median of DPQR. Similarly, RN and
QM are also the median and O is the centroid.
A
86. (d) PO 2
So, =
OL 1
PO 8
G OL = = = 4 cm
2 2
3a
Now, attitude of DPQR=
2
B C
3a
GA = GB = GC is true. If DABC is an equilateral triangle PO + OL =
and here it is not given that ABC is an equilateral triangle. 2
Statement II, If H is orthocantre, then HA = HB = HC is 3a
equal then DABD is an equilateral triangle so II is also 8+ 4 =
not correct. 2
87. (b) If the bisectors of angles B and C of a triangle meet at 12 ´ 2 24
a= = cm
A 3 3
point I, then ÐBIC is equal to 90º +
2 \ perimeter of DPQR = 3a
EBD_7367
A-320 Triangles and its Properties
perpendicular then it is square.
3 ´ 24
= = 24 3 cm So it is also true.
3 94. (a) In case of a right an gled triangle, if we draw a
91. (b) Given, In DABC perpendicular from the vertex containing right angle to
the hypotenuse, we get three triangles, two smaller and
A one original and these three triangles are similar triangles.
C
30°
90°
90°
90°
A B
90° 60° So, DABC ~ DABD ~ DADC
B C BC . AD = AB . AC
95. (a) We know that when two triangles are similar then ratio of
ÐC = 2ÐA
their areas is equal to square of corresponding sides.
ÐB = 90º
ÐA + 90º + 2ÐA = 180º A X
Ð3A = 180º – 90º = 90º
90º
ÐA = = 30º
3
\ ÐC = 2 × 30º = 60º
B C Y Z
BC
sin 30º =
AC area of DABC AB 2 32 AB 2
= Þ =
area of DXYZ XY 2 60.5 (7.7)2
1 BC 1
= Þ BC = AC 32 ´ 59.29
2 AC 2 = AB 2 Þ 31.36 = AB 2
Þ
60.5
AB 3 AB
Now, sin 60º = Þ = \ AB = 31.36 = 5.6 cm
AC 2 AC
3 96. (d)
AB = AC ... (ii)
2
Squaring on both sides.
C
3 2 cm
2
AB = AC , Now putting the value of AC. 25
4
A P 18 cm B
3
AB = × 4 BC2 Þ AB2 = 3BC2
2
4 D
92. (a) ÐACB + ÐBCD = 180º (linear pair) In DACP, CP = CD – PD = 25 – 18 = 7
ÐBCD = 180º – 70º = 110º By Pythagoras theorem
B AC2 = CP2 + AP2

AP = AC 2 – CP 2 = ( 25)2 – (7)2
70° = 625 – 49 = 576 = 24 cm
A D
C Similarly,
PB = 24 cm
In DBCD,BC = CD Þ ÐCBD = ÐCDB ...(i) \ AB = AP + PB
(angles opposite to equal side) = 24 + 24 = 48 cm
Now, ÐBCD + ÐCBD + ÐCDB = 180º 97. (b) The perpendicular bisector of the chord of a circle always
2ÐCDB = 180º – ÐBCD = 180º – 110º = 70º pass through the centre. So, Statement I is wrong
70º C
\ ÐCDB = ÐADB = = 35º
2
90°
93. (c) Statement-1
If the diagonal of a parallelogram ABCD are perpendicular A B
then ABCD may Rectangle or Rhombus. O
So it is true. The angle in a semi-circle is a right angle. So, Statement
Statement-II II is correct.
If the diagonal of quadrilateral ABCD are equal and
Triangles and its Properties 321
A-

a2 + a2r2 = a2r4
A
98. (a) 1 + r2 = r4 or r4 – r2 – 1 = 0
-1 ± 1 - 4(-1)
\ r2 =
2
15 x
-1 ± 5
r2 =
2
B C
25 -1 + 5
r=
AB = 15 2
BC = 25
AC = x, then -1 - 5
r¹ (Because Radius is not negative)
We know that the sum of two sides of a triangle is always 2
greater than third side.
Þ AB + BC > x 5 -1
So, common ratio = .
Þ 15 + 25 > x Þ 40 > x ... (i) 2
Also, the differences of two sides is always less than third 103. (c) Area of DABD and DABE are same because both are on
side. the same base.
BC – AB < AC 1
25 – 15 < x Area of DABD = ´ BD ´ AD ... (i)
10 < x ... (ii) 2
From eq. (i) and (ii) 1
10 < x < 40 Area of DABE = ´ AE ´ BE ... (ii)
2
99. (d) According to Pythagorean triplet.
The sum of square of base and perpendicular equal to C
square of hypotenuse.
By hit and trial method:—
(2n)2 + (n2 – 1)2 = (n2 + 1)2 D
E
4n2 + n4 + 1 – 2n2 = n4 + 2n2 + 1
n4 + 2n2 + 1 = n4 + 2n2 + 1
LHS = RHS
100. (a) In right angle triangle, the sides of triangle are three A B
consecutive number then AB = 4 cm, BC = 3 cm, AC = 5 Now comparing both equations (i) and (ii)
cm
1 1
A ´ BD ´ AD = ´ AE ´ BE Þ BD ´ AD = AE ´ BE
2 2
104. (d) DABC is right angled at B. So that circum
radius lies on its hypotenuse
5 cm AC
4 cm Here OA is circumradius OA =
2

A
B 3 cm C
1 6
\ Area of triangle, ´ 3 ´ 4 = 6 cm 2 O
2 cm
A D

B 8 cm C
101. (a)
B C E F By Pythagoras theorem,
Q Given that DABC ~ DDEF AC2 = AB2 + BC2
AC2 = (6)2 + (8)2
AB BC 1 8 Þ AC2 = 36 + 64 \ AC = 10
\ = Þ =
DE EF 2 EF
10
EF = 16 cm \ Radius of circumcircle = = 5 cm = OA = OC
102. (b) The sides of a triangle in geometric progression are a, ar, 2
ar2
Triangle is right angled. Therefore, we use Pythagoras
theorem.
(a)2 + (ar)2 = (ar2)2
EBD_7367
322
A- Triangles and its Properties

105. (d) In DABC, BF || DG


AD is the internal angular bisector of angle A. given that AD is median so thad DB the mid point of BC.
A G will be The mid point of CF
CG = GF ...(ii)
From equations (i) and (ii), we get
AF = FG = CG ...(iii)
From figure, AC = AF = FG + CG
= AF + AF + AF + 3AF
1
B D C Þ AF = AC
3
BD AB AC DC 108. (c)
= Þ =
DC AC AB BD 1. On drawing the three straight lines through the three
vertices of DABC, we get the following figure.
DC + BD AC + AB Here, AB||DF, BC || DE and AC || EF.
Þ =
BD AB Clearly, A, B and C are the mid–points of DE, EF and
BC 3 + 1 4 DF respectively.
Þ = =
BD 3 3 1
By mid–point theorem, BC = DE or DE = 2BC
BD 3 2
= Similarly, DF = 2AB and EF = 2 AC. Hence, Statement
BC 4
1 is correct.
BD : BC = 3 : 4
106. (d) If AB is a straight line and C and D are points such that E
AC ^ AB and BD ^ AB.
D
C A B

E
A B D F
C
\ AC || BD
ABCD forms trapezium. 1
Now, by property of trapezium diagonals intersect each 2. Also, area of DABC = area ofDDEF or area of D
4
other in the ratio of lengths of parallel side. DEF = 4 area of DABC. Hence, Statement 2 is also
AE BE AE BE coreect.
= Þ =
ED CE AD – AE BC – BE 109. (a) We know that, sum of angles of a triangle = 180°
Þ ÐA + ÐB + ÐC = 180°
BC – BE A D – AE BC AD Þ ÐA + 2ÐA + ÐA = 180°
= Þ –1 = –1
BE AE BE AE Þ 4ÐA = 180°
BC AD 180°
\ = Þ ÐA = = 45°
BE AE 4
AE BE ÐB = 90° and ÐC = 45° Givan that 2ÐC + 2ÐA = ÐB]
\ = DABC is a right angled triangle,
AD BC ÐB = 90°, ÐC = 45° and ÐA = 45°
AE BE A
But the value of or cannot be determined
AD BC
A E BE
Therefore, we cannot find the value of + .
AD BC
107. (c) In DABC, we draw a line l || BF which intersect AC at G.
In DADG and DAEF;
given that EA is the mid point of AD and DL || EF. 90°
C
So, concept of similar triangle. B
F is also mid point of AG. Pythagoras theorem,
AF = FG (i) AB2 + BC2 = AC2
DADG and DAEF are similar. Þ AB2 + AB2 = AC2 [Q AB = BC]
Again Þ 2AB2 = AC2
DFBC and DDCl
Triangles and its Properties A-323

A
AC 2 2
Þ =
AB 2 1

AC 2 Z Y
Þ =
AB 1
O
\ AC : AB = 2 :1
B C
A X
110. (c) So, AZ = AY [by CPCT]
Similarly, CX = CY and BX = BZ
Now, AB > BC
2r O \ AZ + ZB > BX + XC
1q AZ > XC [Q BX = ZB]
B C If AB > BC, then AB + AZ > BC + XC
q D
So, Statement I is incorrect and Statement II is correct.
114. (d) In DABC, ÐACB = 60° and AC = x < BC
Here, OD is the radius of incircle while OB is the radius A
of outer circle.
F x
Given that
Radius of outer circle = 2 × radius of incircle.
x C
Þ OB = 2 × OD B D
OB OD 1 The circle with centre at C and radius x meet BC at D.
Þ =2Þ =
OD OB 2 \ CD = x = Radius
Again, in DACD, AC = CD = x
1
Þ sin Ð BDO = = sin 30° 120°
2 \ ÐCAD = ÐCDA = = 60°
2
ÐDBO = 30°, ÐABC = 2 × ÐDBO = 2 × 30° = 60°
DACD is an equilateral triangle.
So, it is equilateral triangle.
Now, AC is a chord of circle, then perpendicular from
111. (a) The altitude and medians of an equilateral triangle are C on chord AD bisect the chord.
congruent but centroid divide the altitude in
2 : 1. So, Statements 1 and 2 are correct. AD x
\ DF = AF = =
112. (c) In DABC and DDEF, 2 2
AB || DE, BC || EF and CA || FD Statement I is incorrect and Statement II is correct.
If Ð ABC is right angle, then Ð DEF is also a right 115. (c) Let the sides of an old triangle be a, then area of an old
angle. 3 2
equilateral triangle, Aold = a
A 4
Again, let the sides of a new triangle be 2a, then are of
a new equilateral triangle ,
3 3
F
E Anew = (2a) 2 = ´ 4a 2
4 4
According to question, Anew = KAold
B D C 3 3 2
Þ ´ 4a2 = k ´ a
Both triangles are similar but not congruent. 4 4
Statement I is correct and Statement I is correct and \ k=4
Statement II is incorrect. 116. (c) C
113. (d) In DAOZ and DAOY,
AO = OA [common] 30° a
ÐOAZ = ÐOAY [Since, OA bisect ÐA] b
and ÐAZO = ÐAYO [each 90°] 120° 30°
\ D AZO @ DAYO A c B
EBD_7367
324
A- Triangles and its Properties

By Sine rule O is point


on anywhere on AD
a b c A
= =
sin A sin B sin C
a b c
Þ = = O
sin120° sin 30° sin 30°
b sin 30° c sin 30°
= and = B
a sin120° a sin120° D C
So area of triangle
a 1 DABO = DAOC,
=
a + b + c 1+ b + c So statement 1 is true.
a a Statement 2
G is the point of concurrence of the medians
1 then
= A
sin 30° sin 30°
1+ +
sin120° sin120°
1
= F E
1/ 2 1/ 2 G
1+ +
3/2 3/2

1 3
= =
1 1 2 + 3 B D C
1+ +
3 3 area of D ABG = area of D BCG = Area of DACG
Both are true.
117. (a) B 120. (a) If O is equidistant from all the sides, it will be circum-
(a– centre of the triangle so-
r)
c A
a–r
a (b– x x
r)
r
r

C r b–r A y O z
b y z
B C
c=b–r+a–r
c = a + b – 2r 1. ÐAOC = 2ÐABC; ÐBOC = 2ÐBAC; ÐAOB = 2ÐACB
2r = a + b – c and ÐOAC = ÐOCA
a +b – c In DAOC–
\ r=
2 ÐAOC + ÐOAC + ÐOCA = 180°
118. (a) B ÐAOC + ÐOAC + ÐOAC = 180°
2ÐOAC = 180 – 2 ÐABC
90
ÐOAC = 90 – ÐABC ------------(i)
Similarly-
A O C ÐOCB = 90 – ÐBAC ------------(ii)
2r
Let ABC is triangle, which have maximum area, while ÐOBA = 90 – ÐACB ------------(iii)
AC is 2r Adding eq. (i), (ii) and (iii)-
But OB = OC = r ÐOAC + Ð OCB + Ð OBA
By Pythagoras theorem = 90 + 90 + 90 – [ÐABC + ÐBAC + ÐACB]
OB2 + OC2 = BC2
= 270° – 180°
BC = 2r = AC = 90°
1 2. ÐBOC = 2x + y + z = x + (x + y+ z) = 90 + ÐA
Area of triangle = ´ 2r ´ 2 r = r2
2 It is not necessarily equal to 2ÐA
119. (c) Statement 1 3. This is a property of the angle bisector.
AD divides D ABC in equal area of two parts. Then So, option (a) is correct.
Triangles and its Properties 325
A-

121. (a) D C According to condition of circle if two chords


intersects at P then
AP × PB = CP × CD

AP PD
=
CP PB
...(1)
A B Given two right angled triangleA¢P¢B¢ and C¢Q¢D¢ then
On single face of cube no. of right angled
Triangles formed = 4 (i.e., DABD, DABC, DABD, A¢
DACD)
Total faces of a cube = 6
So, no. of right angle triangles = 4 × 6 = 24
So, option (a) is correct.
122. (a) Since AB > A¢B¢, BC > B¢C¢ and CA > C¢A¢ B¢

Also D, E, F are mid points of BC, CA and AB.
Also D¢E¢F¢ are mid points of B¢C¢, C¢A¢ and A¢B¢ C¢

1 1 1
Now AD = BC , BE = CA , CF = AB
2 2 2
...(1)

1 1 1 Q¢
Also A¢D¢ = B¢C¢ , B¢E¢ = C¢A¢ , C¢F¢ = A¢B¢
2 2 2
A¢P¢ = AP, B¢P¢ = BP, C¢Q¢ = CP, D¢Q¢ = DP
...(2)
Þ If AB > A¢B¢. BC > B¢C¢ and CA > C¢A¢ From (1) we get
Þ A¢D¢ > AD, BE > B¢E¢ and CF > C¢F¢ C¢A¢ AP PD A ¢P¢ D¢Q¢
= Þ =
from (1) & (2) CP PB C¢Q¢ B¢P¢
Statement II
Þ DA¢P¢B¢ : DC¢Q¢D¢
AB2 + BC 2 + CA 2 A ¢B¢2 + B¢C ¢2 + C¢A¢2
= Q their corresponding ratios are equal.
AD2 + BF2 + CF2 A ¢D ¢2 + B¢E ¢2 + C¢F¢2 Also we know that in similar triangles.
2 2
Area of DA¢P¢B¢ æ A¢P¢ ö æ AP ö
( 2CF )2 + ( 2AD )2 + ( 2BE )2 =ç ÷ = ç ÷
Þ Area of DC ¢Q¢D¢ è C¢Q¢ ø è CP ø
AD2 + BE 2 + CF2
1
( 2C¢F¢ )2 + ( 2A ¢D¢ )2 + ( 2B¢F¢ )2 ´ A ¢P ¢ ´ P¢B¢ 2 2
2 æ AP ö AP ´ BP æ AP ö
= =ç ÷ Þ =ç ÷
A ¢D¢2 + B¢E¢2 + C¢F¢2 Þ 1 CP ø CP ´ DP è CP ø
´ C¢Q¢ ´ Q¢D¢ è
Þ 4 = 4 Þ Statement II is true. 2
\ Option (a) is correct. We know that
123. (b) Given two chords AB and CD intersects at a point P AP PD
inside the circle. =
CP DB
2
A æ AP ö æ BP ö æ AP ö
Þ ç ÷=ç ÷=ç ÷
è CP ø è DP ø è CP ø
D 2
æ AP ö æ AP ö æ AP ö
Þ ç ÷=ç ÷=ç ÷
è CP ø è CP ø è CP ø
P
2 2
æ AP ö æ AP ö
Þ ç ÷ =ç ÷
B è CP ø è CP ø
C Þ Area of DA¢P¢B¢ = Area of DC¢Q¢D¢
EBD_7367
326
A- Triangles and its Properties

These triangles are not congruent because none of


125. (b) A
the criterion are satisfied by these triangles perimeter
of triangle. Sum of all three sides. But in these
triangles, all the sides are not equal, then their perimeter
is not equal
Þ Statement (1) and (3) is correct.
124. (c) We have BE || AC (Given)
So ÐADE = ÐB and ÐAED = ÐC B L C
(corresponding angles) D
A

D E

E M F
Here ABC and DEF be two triangles such that
ÐA = ÐD, ÐB = ÐE and ÐC = ÐF
B C (Given)
Also ÐL = ÐM = 90°
Therefore DABC : DADE by A similarity criterion. Þ ÐALB = ÐALC = ÐDME = ÐDMF = 45°
Also given area of DABC = 2 area of DADE (... M and L are mid points of EF and BC respectively)
...(1) Þ DABC : DDEF by DAA similarity
We know that the ratio of the areas of two similar Also DABC @ DDEF by DAA Similarity
triangles is equal to the square of the ratio of their
corresponding sides. In DABL and DDEM
ÐALB = ÐDME = 45°
ar ( ABC )
æ AB ö
2 ÐA = ÐD and ÐB = ÐE (Given)
Þ =ç ÷
ar ( ADE ) è AD ø Þ DABÐ ~ DDEM by AAA similarity criterion.
\ Statement I is true.
...(2) In DALC and DDMF
From (1) we get Given AC ¹ DF
But ÐALC = ÐDMF = 45°
ar ( ABC ) 2 Also ÐA = ÐD and ÐC = ÐF (Given)
=
ar ( ADE ) 1 Þ DALC : DDMF by AAA similarity..
...(3) Þ DALC @ DDMF by AAA similarity..
Therefore from (2) and (3) \ Statement II is true.
But II is not the correct explanation of Statement-I
2
æ AB ö 2 . .
. these are different triangles.
ç ÷ =
è AD ø 1 \ Option (b) is correct.
126. (d) A
AB 2
Þ =
AD 1
1
Þ = 2unit
AD C
(...AB = 1 unit) B
D
1
AD = units
2
\ Option (c) is correct.
E F
Here DABC : DDEF
Triangles and its Properties A-327

Then ratio of the areas of two similar triangles is equal 129. (d) In DPYX and DPRQ
to the square of the ratio of their corresponding sides. P
2
area of DABC æ AB ö
Þ =ç ÷
area of DDEF è DE ø 1.5 2
2 x y
æ 2 +1 ö
= çç ÷ 6 8
è 3 ÷ø
Q R
2 +1+ 2 2 3 + 2 2
= =
3 3 PX PY
=
= 3+2 2 :3 PQ PR
127. (d) In a triangle DABC
1.5 2
ÐA + ÐB + ÐC = p Þ =
...(1) 7.5 10
p 1 1
Given A - B = Þ =
2 5 5
p Now corresponding ratios of two triangles are equal.
Þ A = B+ Also ÐP = ÐP (Common)
2
Þ DPXY : DPQR by SAS similarity
Putting this value of A in (1), we get
p PX PY XY
B+ + B+C = p Þ = =
2 PQ PR QR
p p 1 1 XY
C + 2B = p - = Þ = =
2 2 5 5 QR
\ Option (d) is correct. Þ QR = 5XY
128. (d) Locus of the point X is L. Also QR || XY (By B.P.T)
A \ Option (d) is correct.
130. (b) Let the position of person = A
We have to find out the distance between A and E.

B 3 km
C
x

B D C 7 km
Here L is line segment AD.
Now, D A X B @ D A X C (By SSS)
13 km
\ ÐBAX =ÐCAX (Corr. Angles) A D
Þ AX and hence AD is the bisector
of ÐBAC.
Hence incentre of the DABC lies on
AD i.e., L (statemetn 1 correct)
Since AB = AC fand AD is the bisector of DABC
E
BD AB
\ = = 1 Þ BD = DC AB = 7 km then CD = 7 km
DC AC
Now DXBD @ DXCD BC = 3 m then AD = 3 km
(By SSS) DE = CE – CD = 13 – 7 = 6 km
\ ÐXDB = ÐXDC = 90° Draw AD ^ CE
Hence AD is the perpendicular bisector of BC. In right angled triangle AED, we get
Therefore, orthocentre of the DABC lies on AD i.e, L. (AE)2 = AD2 = DE2
= (3)2 + (6)2
(Statement 2 correct) AE = 9 + 36
Since D is the mid-point of BC, therefore AD is the
mediam. = 45 = 5 ´ 9
Hence centroid of the DABC lies ons AD i.e., L. AE = 3 5km
\ Option (b) is correct.
(Statement 3 correct)
EBD_7367
328
A- Triangles and its Properties
131. (c) ABC is a triangle in which D and E are the mid points Also In DBPC, we get
of BC and AD respectively.
A BP2 = BC2 + CP2 ...(2)
In DAQC, we get
AQ2 = AC2 = CQ2 ...(3)
In DPQC, we get
PQ2 = PC2 + QC2 ...(4)
Adding (2) and (3) we get
E BP2 + AQ2 = BC2 + CP2 + AC2 + CQ2
= (BC2 +AC2) + (CP2 + CQ2)
Using (1) and (4) we get
BP2 + AQ2 = AB2 + PQ2
\ Option (c) is correct.
B D C 133. (b) C 96°
AD BC
Þ AE = ED = and BD = DC = x
2 2
Area of triangle ABC
Statement (1)
Area of triangle BED
180° – 2x 2x
1 1 x 2x
´ BC ´ AD ´ BC ´ AD
2 = 2 A D B
= 1 1 1 1
´ ED ´ BD ´ AD ´ BC Let ÐCAD = ÐACD = x
2 2 2 2 Since AD = CD = BC
1 Q AD = CD
= =4
1 Þ ÐA = ÐC
4 Also CD = BC
Area of DABC = 4 Area of DBED Þ ÐD = ÐB
(1) is true. In DACD
1 ÐA + ÐC + ÐD = 180°
´ AD ´ DC Þ ÐD = 180° – 2x
Area of DADC 2
=
Statement (2) Area of DBED 1 ÐCDB = 180° – ÐCDA
´ BD ´ ED = 180º – (180° – 2x)
2
= 2x
AD ´ DC
= Þ ÐCDB = ÐCBD = 2x Þ ÐDCB = 180° – (2x + 2x)
1
DC ´ AD = 180° – 4x
2 At point c, we have
æ 1 ö x + (180° – 4x) + 96° = 180°
çQ BD = DC and ED = AD ÷ Þ 180° – 3x + 96° = 180°
è 2 ø
=2 Þ – 3x = – 96°
Area of DADC = 2 Area of DBED Þ x = 32°
2 is true. Hence ÐDBC = 2x = 2 × 32 = 64°
\ Option (c) is correct. \ Option b is correct.
132. (c) Since ABC is a right angled triangle QtC 134. (c) Let ABC be an equilateral triangle and x, y, z are points
A on BC, CA and AB.
A

Z Y

P B C
K X
Also given
BX = CY = AZ
Since ÐA = ÐB = ÐC = 60° (Equilatual triangle)
B Q C Þ If BX = CY
In DABC, we have by pythagorus theorem, Þ ÐX = ÐY
AB2 = AC2 + BC2 ...(1) BX = AZ
Triangles and its Properties A-329

Þ ÐX = ÐZ Direction ® make 2 more trangle


Also If ÐY = Az total equitateral D
Þ ÐY = ÐZ =q
Þ ÐX = ÐY = ÐZ = 60° 3 3
DXYZ is an equilateral triangle. Area = (12 q) 2 : (16)2 = 64 : 1
Consider triangle DABC and DXYZ 4 4
Since ÐX = ÐY = ÐZ = 60° 137. (c) Let a = b = c then the sides an 2a, 2a and 2a by
and ÐA = ÐB = ÐC = 60° heroni formula
Þ DABC : DXYZ by AA similarity criterion
\ Option (c) is correct. 2 2a + 2a
S= = 2a + a
135. (d) B 2
as D =

D
13
( 2 a+ a )( 2 a+ a- 2 a )( 2 a+ a- 2 a )( 2 a + a - 2a )
5
30°
= ( 2a + a )( )
2a - a a 2

= (2a 2 - a 2 )a 2 = a 2
C 12 A Now by putting a = b = c in option
in D BCD (c) satisfies the area a2
sin30° sin B a(b+ c) a(2 a)
= ie = = a2
BD CD 2 2
1 sin B
138. (a) Largest triangle in semicircle is
Þ = C
2BD CD
1 12 12
Þ = Q sin B = 90º 2r
2BD 13CD 13 r
24
CD = BD ...(i) A r B
13 O
in D ADC when height of triangle h = r
1 1
sin 60° sin A 5 then area = b ´ h = 2r ´ r = r2
= Q sin A = 2 2
AD CD 13
139. (d) Remembering the fact that sum of any two small sides
5 2 of triangle is larger than its third side
CD = ´ AD
13 3 Option D is not a triangle
2 + 3 = 5 is not greater than 6
10 140. (b) let the angles be = 2x, 4x and 3x
Þ CD = (13 - BD) ...(ii)
13 3 2x + 4x + 3x = 180°
equating both eq (i) and (ii) 9x = 180°
x = 20°
10 24
(13 - BD) = BD Smallest angle = 2x
13 3 13 = 2 × 20 = 40°
65 141. (b) in DABC and DDAC
BD = A
5 + 12 3
24 65 120
CD = =
13 5 + 12 3 5 + 12 3
136. (c)

B 9 D 3 C
12
ÐBAC = ÐADC
ÐACB = ÐDCA \ common angle
\ DABC ~ DDAC
BC AC
=
AC DC
12 × 3 = AC2
AC = 6
EBD_7367
330
A- Triangles and its Properties

142. (c) 1. in DADC and DBAY


ÐA = ÐB = 90°
A
AX = BY half of side of square
AD = AB
DABY @ DDAX By SAS
D 2. ÐDXA = ÐAYB CPCT
y 3. DX is not included at 60° with AY
x
x 4. DX is not is perpendicular to AY
B C 147. (c)
148. (c) B
ÐA = 80, ÐABC = 60°
ÐACB = 180 – (80 + 60) = 40°
40
x= = 20
2
60
ÐDBC = = 30
2 A C
Ðy = 180 – (30 + 20) = 130 D
= x = 20, y = 130 Since DABC is an equilateral triangle and BD is a per-
143. (b) pendicular, therefore AD = DC.
P In DBCD, by Pythagoras theorem, we have,
BC 2 = BD 2 + CD 2 Þ BD 2 = BC 2 - CD 2
Þ BD 2 = AC – CD (Œ BC = AC)
2 2
T
M
BD 2 = ( AD + DC ) 2 - CD 2 Þ BD 2

L = AD 2 + DC 2 +2ADCD - CD 2
Q R
Assuming right angled triangle be in a circle BD 2 = AD 2 + 2DC 2 Þ BD 2 = AD 2 + 2 AD 2
where PR is diameter = 3AD 2 (ΠCD = AD)
PT = QT = TR (Radii of circle)
QT = TR 149. (d) P
ÐTQR = ÐTRQ
ÐTQR = ÐRLM Corresponding angles
ÐRLM = ÐLRM i.e. (ÐTRQ) N M
144. (c) (1) Q ÐDAC = ÐDCA \ DA = DC hence DADC is an Q
isosceles triangle
(2) it cannot be said weather is the centroid
3. AB = CB
AD = DC Q R
BD = BD L
We can see that Q is outside triangle OPR and DOPR
DABD @ CBD is obtuse-angled triangle.
hence statement 1 and 3 are correct. Since orthocenter of an obtuse-angled triangle is
145. (c) ÐAMC = ÐBMD always outside the triangle, therefore Q is the ortho-
ÐCMD = ÐCMD center of DOPR.
ÐAMC + ÐCMD = ÐBMD + ÐCMD
{Adding both equation} 150. (d) B
ÐAMD = ÐBMC
ÐDAM = ÐCBM D
AM = BM
By ASA
DADM @ BCM C A

146. (d) A X B (CD) -2 = ( BC )-2 + (CA) -2


1 1 1
Þ 2
= 2
+
CD BC CA2
1 CA2 + BC 2 1 AB 2
Y Þ = Þ =
CD 2 BC 2 .CA2 CD 2 BC 2 .CA2
BC 2 .CA2
CD 2 = Þ CD 2 . AB 2 = BC 2 .CA2
D C AB 2
Þ CD. AB = BC.CA
Triangles and its Properties A-331

151. (a) A ar.( DABC ) ( DE )2 ( DE )2


Þ = =
5ar.( DABC ) (10)2 100

1 ( DE )2 100
E Þ = Þ DE = =2 5
5 100 5
G 154. (a) Since the point of intersection of the perpendicular
F bisectors of th e sides of a triangle is called
circumcentre and the circumcentre for an obtuse tri-
angle lie outside the triangle, therefore this statement
B C
D is true.
Also, since the point of intersection of the perpen-
Since BE is the median of AC, therefore AE = EC. diculars drawn from the vertices to the opposite sides
Since AC = 9 cm, therefore AE = EC = 3 cm. of a triangle is called orthocentre and orthocentre can-
This means FC < EC = 3 cm. therefore, FC = 2.25 cm not lie on two sides, therefore this statement is false.
P Consider the given table:
152. (a)
Subjects/ Mathematics Physics Statistics Chemistry
6 girls and
10 boys
X
3
4 Number 240 – 150 300 – 180 250 ´ 340
5
of girls = 90 = 120 = 68 × 3
3 2 = 204
Q 5 Y R Number 150 60% of 300 320 – 250 136
of boys = 180 = 70
RY RX 2 4 2
Since = Þ = = therefore, by converse Total 20% of 1200
1200
320 1200 – (240
YQ XP 3 6 3 4
of BPT theorem, XY 11 PQ. = 240 = 300 + 300 + 320)
A = 340
153. (b)
155. (a)
156. (c)
D E
157. (d)
158. (d) diagonal of square = 6 2
6 2
side of square = =6
B C 2
10 3 × side of triangle = 6 × 4
According to the question and by BPT, side of triangle = 8
AD AE 3 2 3 2
= and ÐA is common. area of triangle = a = ´ 8 = 16 3 cm 2
AB AC 4 4
Therefore, DADE i ABC (by SAS similarity)
3 2
Since we know that the ratio of areas of two similar 159. (a) Area of DABC = l
triangles is equal to the square of its proportional sides, 4
from mid point theoram
ar.( DADE ) ( DE )2
therefore, = ...(1) 2
3 æ lö 3 2
ar.( DABC ) ( BC ) 2 Area of DADE = ´ç ÷ = l
Also, we are given that 4 è 2 ø 16
1 3 2 3 2
ar .( DADE ) = ar.( DABC ) Area of shaded region = l - l
5 4 16
Thus, (1)
3 3 l2
=
16
EBD_7367
332
A- Quadrilateral and Polygon

C HA P T E R
QUADRILATERAL
23 AND POLYGON
1. An obtuse angle made by a side of a parallelogram PQRS ABCD is a rectangle of dimensions 8 units and 6 units.
with other pair of parallel sides is 150°. If the perpendicular AEFC is a rectangle drawn in such a way that diagonal
distance between these parallel sides (PQ and SR) is 20 AC of the first rectangle is one side and side opposite to
cm, what is the length of the side RQ? [2007-I] it is touching the first rectangle at D as shown in the
(a) 40 cm (b) 50 cm figure given above. What is the ratio of the area of
(c) 60 cm (d) 70 cm rectangle ABCD to that of AEFC? [2008-II]
2. Assertion (A) If the side of a rhombus is 10 cm. Its (a) 2 (b) 3/2
diagonals should have values 16 cm and 12 cm. (c) 1 (d) 8/9
Reason (R) The diagonals of a rhombus cut at right 8. ABCD is a square. The diagonals AC and BD meet at O.
angles. [2007-II] Let K, L be the points on AB such that AO = AK and BO
(a) A and R are correct and R is correct explanation of A = BL. If = LOK, then what is the value of tan ?
(b) A and R are correct but R is not correct explanation [2008-II]
of A (a) 1/ 3 (b)
(c) A is correct but R is wrong 3
(c) 1 (d) 1/2
(d) A is wrong but R is correct
9. Two sides of a parallelogram are 10 cm and 15 cm. If the
3. An equilateral triangle and a regular hexagon are inscribed
altitude corresponding to the side of length 15 cm is 5 cm,
in a given circle. If a and b are the lengths of their sides
then what is the altitude to the side of length 10 cm?
respectively, then which one of the following is correct.
[2009-I]
[2007-II]
(a) 5 cm (b) 7.5 cm
(a) a2 = 2b2 (b) b2 = 3a3
(c) 10 cm (d) 15 cm
(c) b2 = 2a2 (d) a2 = 3b2
10. Which one of the following figures has only one line of
4. In a cricket match, the first 5 bastmen of a team scored
symmetry? [2009-I]
runs : 30, 40, 50, 30 and 40. If these data represent a four
(a) Rhombus (b) Rectangle
sided figure with 50 as its one of the diagonals, then what
(c) Isosceles trapezium (d) Parallelogram
does second diagonal represent? [2007-II]
(a) 30 runs (b) 40 runs 11. A B
(c) 50 runs (d) 70 runs O
5. The incircle of a quadrilateral of perimeter 2p has radius r.
What is the area of the quadrilateral? [2007-II] D C
(a) p(r + 1) (b) 2 pr
(c) pr (d) None of these
N
6. A D
y In the figure given above, M is the mid-point of the side
CD of the parallelogram ABCD. What is ON : OB ?
x [2009-I]
E F (a) 3 : 2 (b) 2 : 1
(c) 3 : 1 (d) 5 : 2
12. ABC is a triangle in which AB = AC. Let BC be produced
z to D. From a point E on the line AC let EF be a straight
B C line such that EF is parallel to AB. Consider the
ABCD is a trapezium in which EF is parallel to BC. quadrilateral ECDF thus formed. If ABC = 65° and
x = 120° and x = 50°, then what is y? [2007-II] EFD = 80°, then what is D equal to? [2009-II]
(a) 50° (b) 60° (a) 43° (b) 41°
(c) 70° (d) 80° (c) 37° (d) 35°
7. A 8 13. D C
B
6 O
E
D C

A X B
F
Quadrilateral and Polygon A-333
In the figure given above, ABCD is a square in which AO E. Further, AO : OC = BO : OD, AB = 30 cm, CD = 40 cm and
= AX What is XOB ? [2009-II] the area of the quadrilateral ABCD is 1050 sq cm. [2010-II]
(a) 22.5° (b) 25° 21. What is BE equal to?
(c) 30° (d) 45° (a) 30 cm (b) 30 2 cm
14. The quadrilateral formed by joining the mid-points of the
sides AB, BC, CD and DA of a quadrilateral ABCD is (c) 30 3 cm (d) None of these
[2009-II] 22. What is the area of the ADC equal to?
(a) a trapezium but not a parallelogram (a) 300 cm2 (b) 450 cm2
2
(b) a quadrilateral but not a trapezium (c) 600 cm (d) None of these
(c) a parallelogram only 23. What is AEB equal to?
(d) a rhombus (a) 30° (b) 45°
15. ABCD is a square, P, Q, R and S are points on the sides (c) 60° (d) None of these
AB, BC, CD and DA respectively such that AP = BQ = 24. In the given figure, ABCD is a quadrilateral with AB
CR = DS. What is SPQ equal to? [2010-I] parallel to DC and AD parallel to BC, ADC is a right angle.
(a) 30° (b) 45° If the perimeter of the ABE is 6 units. What is the area
(c) 60° (d) 90° of the quadrilateral ? [2010-II]
16. The middle points of the parallel sides AB and CD of a A B
parallelogram ABCD are P and Q, respectively. If AQ and 60° 60°
CP divide the diagonal BD into three parts BX, XY and
YD, then which one of the following is correct?
[2010-I]
(a) BX XY YD (b) BX YD XY
(c) BX XY YD (d) XY 2BX D E C
17. A parallelogram and a rectangle stand on the same base
and on the same side of the base with the same height. (a) 2 3 sq units (b) 4 sq units
If I1, I2 be the perimeters of the parallelogram and the (c) 3 sq units (d) 4 3 sq units
rectangle respectively, then which one of the following is 25. In the figure given below, ABCD is a parallelogram. P is
correct? [2010-I] a point in BC such that PB : PC = 1 : 2. DP produced
(a) I1 < I2 (b) I1 = I2 meets AB produced at Q. If the area of the BPQ is 20
(c) I1 > I2 but I1 2I2 (d) I1 = 2l2 sq units, what is the area of the DCP? [2010-II]
18. Two similar parallelograms have corresponding sides in
the ratio 1 : k. What is the ratio of their areas? [2010-I] A B Q
(a) 1 : 3k2 (b) 1 : 4k2
2
(c) 1 : k (d) 1 : 2k2 P
19. Consider the following statements in respect of a
quadrilateral.
I. The line segments joining the mid-points of the two D C
pairs of opposite sides bisect each other at the point (a) 20 sq units (b) 30 sq units
of intersection. (c) 40 sq units (d) None of these
II. The area of the quadrilateral formed by joining the mid- 26. The sides of a parallelogram are 12 cm and 8 cm long and
points of the four adjacent sides is half of the total area one of the diagonals is 10 cm long. If d is the length of
of the quadrilateral. other diagonal, then which one of the following is
Which of the statements given above is/are correct? correct? [2012-I]
[2010-I] (a) d < 8 cm (b) 8 cm < d < 10 cm
(a) Only I (b) Only II (c) 10 cm < d < 12 cm (d) d > 12 cm
(c) Both I and II (d) Neither I nor II 27. ABCD is a rhombus with diagonals AC and BD. Then,
20. Let WXYZ be a square. If P, Q and R be the mid-points which one among the following is correct? [2012-I]
of WX, XY and ZW, respectively and K, L be the mid- (a) AC and BD bisect each other but not necessarily
point of PQ and PR, respectively. Then, what is the value perpendicular to each other
area of PKL (b) AC and BD are perpendicular to each other but not
of ? [2010-I] necessarily bisect each other
area of square WXYZ (c) AC and BD bisect each other and perpendicular to
1 1 each other
(a) (b) (d) AC and BD neither bisect each other nor
32 16 perpendicular to each other.
1 1 28. Let ABCD be a parallelogram. Let m and n be positive integers
(c) (d) such that n < m < 2n. Let AC = 2 mn, BD = m2 – n2 and AB
8 64
= (m2 + n2)/2.
Directions (Q. Nos. 21-23) Read the following information Statement I AC > BD
carefully to answer the questions that follow. Statement II ABCD is rhombus
Let ABCD be a quadrilateral. Let the diagonals AC and BD meet Which one of the following is correct in respect of the
at O. Let the perpendicular drawn from A to CD, meet CD at above statements? [2012-I]
EBD_7367
334
A- Quadrilateral and Polygon

(a) Both Statements I and II are true and Statement II is P Q


the correct explanation of Statement I
C
(b) Both Statement I and II are true but Statement II is
not the correct explanation of Statement I D B
(c) Statement I is true but Statement II is false
A
(d) Statement II is true but Statement I is false
S R
29. ABCD is a rectangle. Let E be a point on AB and F be
(a) square (b) rhombus
a point on CD, such that DE is parallel to BF. If AE = 3
(c) rectangle (d) None of these
cm and if the area of BFC = 6 sq cm. 38. In the figure given above, ABCD is a trapezium. EF is
Consider the following statements parallel to AD and BG. y is equal to [2013-I]
I. Area of rectangle ABCD can be of the form pq2 sq cm, A D
where p and q are distinct primes. x y
II. Area of the figure EBFD is of the form r2 sq cm, where
r is rational but not an integer. 120°
Which of the above statements is/are correct? [2012-I] E z F
(a) Only I (b) Only II
(c) Both I and II (d) Neither I nor II 90° 50°
30. Let LMNP be a parallelogram and NR be perpendicular to B C
LP. If the area of the parallelogram is six times the area of (a) 30° (b) 45°
RNP and RP = 6 cm what is LR equal to? [2012-I] (c) 60° (d) 65°
(a) 15 cm (b) 12 cm 39. A quadrilateral ABCD is inscribed in a circle. If AB is
(c) 9 cm (d) 8 cm parallel to CD and AC = BD, then the quadrilateral must
31. If the diagonals of a quadrilateral are equal and bisect each be a [2013-II]
other at right angles, then the quadrilateral is a [2012-II] (a) parallelogram (b) rhombus
(a) rectangle (b) square (c) trapezium (d) None of these
(c) rhombus (d) trapezium 40. ABCD is a quadrilateral such that BC = BA and CD > AD.
Which one of the following is correct? [2013-II]
32. If two parallel lines are cut by two distinct transversals,
(a) BAD = BCD (b) BAD < BCD
then the quadrilateral formed by the four lines is always
(c) BAD > BCD (d) None of these
a [2012-II]
41. Two light rods AB = a + b, CD = a – b symmetrically lying
(a) square (b) parallelogram
on a horizontal AB. There are kept intact by two strings
(c) rhombus (d) trapezium
AC and BD. The perpendicular distance between rods is
33. ABCD is a parallelogram. If the bisectors of the A and a. The length of AC is given by [2014-I]
C meet the diagonal BD at point P and Q respectively, (a) a (b) b
then which one of the following is correct? [2012-II]
(a) PCQA is a straight line (c) a2 b2 (d) a2 b2
(b) APQ is similar to PCQ 42. If PQRS be a rectangle such PQ = 3 QR. Then, what is
(c) AP = CP
PRS equal to? [2014-I]
(d) AP = AQ
(a) 60° (b) 45°
34. Let X be any point within a square ABCD. On AX a
(c) 30° (d)
square AXYZ is described such that D is within it. Which 43. In a trapezium, the two non-parallel sides are equal in
one of the following is correct ? [2012-II] length, each being of 5 cm. The parallel sides are at a
(a) AX = DZ (b) ADZ = BAX distance of 3 cm apart. If the smaller side of the parallel
(c) AD = DZ (d) BX = DZ sides is of length 2 cm, then the sum of the diagonals of
35. The locus of a point in rhombus ABCD which is the trapezium is [2014-I]
equidistant from A and C is [2012-II]
(a) 10 5 cm (b) 6 5 cm
(a) a fixed point on diagonal BD
(b) diagonal BD (c) 5 5 cm (d) 3 5 cm
(c) diagonal AC 44. The area of a rectangle lies between 40 cm2 and 45 cm2.
(d) None of the above If one of the sides is 5 cm, then its diagonal lies between
36. ABCD is a trapezium with parallel sides AB = 2 cm and DC = [2014-I]
3 cm. E and F are the mid-points of the non-parallel sides. The (a) 8 cm and 10 cm (b) 9 cm and 11 cm
ratio of area of ABFE to area of EFCD is [2013-I] (c) 10 cm and 12 cm (d) 11 cm and 13 cm
(a) 9 : 10 (b) 8 : 9 45. Let ABCD be a parallelogram. Let P, Q, R and S be the
(c) 9 : 11 (d) 11 : 9 mid-points of sides AB, BC, CD and, DA, respectively.
37. In the figure given below, PQRS is a parallelogram. If AP, Consider the following statements.
AQ, CR and CS are bisectors of P, Q, R and S I. Area of triangle APS Area of triangle DSR, if BD AC.
respectively, then ABCD is a [2013-I]
Quadrilateral and Polygon A-335

II. Area of triangle ABC = 4 [Area of triangle BPQ]. (a) 0 (b) 4


Select the correct answer using the codes given below. (c) 8 (d) None of these
[2014-I] Directions (Q. Nos. 55-56) Read the following information
(a) Only I (b) Only II carefully and answer the given questions that follow.
(c) Both I and II (d) Neither I nor II ABCD is a trapezium, in which AB is parallel to CD. Let M
46. Consider the following statements be the mid-point of BC. [2014-II]
I. Let ABCD be a parallelogram which is not a rectangle. 55. Consider the following statements :
then, 2(AB2 + BC2) AC2 + BD2 1. ‘Area of ADM + Area of DCM’ is equal to three-
II. If ABCD is a rhombus with AB = 4cm, then AC2 + BD2 fourth of the area of trapezium ABCD, if AB = CD.
= n3 for some positive integer n. 2. ‘Area of DCM + Area of ABM’ is always greater
Which of the above statements is/are correct? than half of the area of trapezium ABCD.
[2014-I] Which of the above statements is/are correct?
(a) Only I (b) Only II (a) Only 1 (b) Only 2
(c) Both I and II (d) Neither I nor II (c) Both 1 and 2 (d) Neither 1 nor 2
47. ABCD is a parallelogram, E is a point on BC such that BE 56. Consider the following statements :
: EC = m : n. If AE and DB intersect in F, then what is 1. ‘Area of ADM – Area of ABM’ is always equal to
the ratio of the area of FEB to the area of AFD? area of DCM, if AB = CD.
[2014-I] 2. Half of area of ABM is equal to one-eight of area
(a) m/n (b) (m/n)2 of trapezium ABCD, if AB = CD.
(c) (n/ m2) (d) [m/(m + n)]2 Which of the above statements is /are correct?
48. Let ABCD be a parallelogram. Let X and Y be the mid– (a) Only 1 (b) Only 2
points of the sides BC and AD, respectively. Let M and (c) Both 1 and 2 (d) Neither 1 nor 2
N be the mid–points of the sides AB and CD, respectively. 57. ABCD is a parallelogram. P and R are the mid-points of
Consider the following statements: [2014-II] DC and BC, respectively. The line PR intersects the
1. The straight line MX cannot be parallel to YN. diagonal AC at Q. The distance CQ will be [2014-II]
2. The straight lines AC, BD, XY and MN meet at a (a) AC/4 (b) BD/3
point. (c) BD/4 (d) AC/3
Which of the above statements is/are correct ? 58. Bisectors of two adjacent angles A and B of a quadrilateral
(a) Only 1 (b) Only 2 ABCD intersect each other at a point P. Which one of the
(c) Both 1 and 2 (d) Neither 1 nor 2 following is correct ? [2014-II]
49. What is the maximum distance between two points of a (a) 2 APB = C + D
cube of side 2 cm ? [2014-II] (b) APB = C + D
(c) APB =180° – C + B)
(a) 3 cm (b) 2 3 cm (d) APB =180° – C + D)
59. If each interior angle of a regular polygon is 135°, then the
(c) 4 3 cm (d) 2 2 cm number of diagonals of the polygon is equal to [2015-I]
Directions (Q. Nos. 50–52) Read the following information (a) 54 (b) 48
carefully and answer the given questions that follow. (c) 20 (d) 18
A piece of land is in the form of a parallelogram and the 60. AD is the diameter of a circle and AB is a chord. If AD = 34
perimeter of the land is 86m. The length of one side exeeds the cm, AB = 30 cm, the distance of AB from the centre of the
other by 13 m and one of the diagonals is 41m. [2014-II] circle is [2015-I]
50. What is the area of the parallelogram ? (a) 17 cm (b) 15 cm
(a) 63 m2 (b) 96 m2 (c) 13 cm (d) 8 cm
2
(c) 126 m (d) 252 m2 61. If a star figure is formed by elongating the sides of a
51. What is the shorter height of the parallelogram ? regular pentagon, then the measure of each angle at the
angular points of the star figure is [2015-I]
(a) 9.0 m (b) 7.5 m (a) 36° (b) 35°
(c) 5.5 m (d) 4.5 m (c) 32° (d) 30°
52. Consider the following statements : 62. The area of a rhombus with side 13cm and one diagonal
1. The difference between the diagonals of the 10 cm will be [2015-I]
parallelogram is more than 20 m. (a) 140 square cm (b) 130 square cm
2. The difference between the heights of the (c) 120 square cm (d) 110 square cm
parallelogram is more than 10 m. 63. The diagonals of a trapezium are at right angles, and the
Which of the above statements is / are correct ? slant sides, if produced, form an equilateral triangle with
(a) Only 1 (b) Only 2 the greater of the two parallel sides. If the area of the
(c) Both 1 and 2 (d) Neither 1 nor 2 trapezium is 16 square cm, then the distance between the
53. What is the number of pairs of perpendicular planes in a parallel sides is [2015-I]
cuboid ? [2014-II] (a) 2 cm
(a) 4 (b) 8 (b) 4 cm
(c) 12 (d) None of these (c) 8 cm
54. How many equilateral triangles can be formed by joining (d) Cannot be determined due to insufficient data
any three vertices of a cube ? [2014-II]
EBD_7367
336
A- Quadrilateral and Polygon

64. ABCD is a parallelogram with AB and AD as adjacent sides. 71. If each interior angle of a regular polygon is 140°, then the
If A = 60° and AB = 2AD, then the diagonal BD will be number of vertices of the polygon is equal to [2016-I]
equal to [2015-II] (a) 10 (b) 9
(a) (b) (c) 8 (d) 7
2AD 3AD 72. A square is inscribed in a right-angled triangle with legs p
(c) 2AD (d) 3AD and q, and has a common right angle with the triangle. The
65. If X is any point within a square ABCD and on AX a square diagonal of the square is given by [2016-I]
AXYZ is described, which of the following is/are correct?
pq pq
[2015-II] (a) (b)
1. BX = DZ or BZ = DX p 2q 2p q
2. ABX = ADZ or ADX = ABZ 2 pq 2 pq
Select the correct answer using the code given below: (c) (d)
p q p q
(a) 1 only (b) 2 only
(c) Both 1 and 2 (d) Neither 1 nor 2 73. Consider the following statements: [2016-I]
66. ABCD is a square. If the sides AB and CD are increased by 1. If n 3 and m 3 are distinct positive integers, then
30%, sides BC and AD are increased by 20%, then the area the sum of the exterior angles of a regular polygon of
of the resulting rectangle exceeds the area of the square by m sides is different from the sum of the exterior angles
[2015-II] of a regular polygon of n sides.
(a) 50% (b) 52% 2. Let m, n be integers such that m > n 3. Then the sum
(c) 54% (d) 56% of the interior angles of a regular polygon of m sides is
67. Three rectangles R1, R2 and R3 have the same area. Their greater than the sum of the interior angles of a regular
lengths x1, x2 and x3 respectively are such that x1 < x2 < x3.
If V1, V2 and V3 are the volumes of the cylinders formed polygon of n sides, and their sum is (m n) .
2
from the rectangles R1, R2 and R3 respectively by joining Which of the above statements is/are correct?
the parallel sides along the breadth, then [2015-II] (a) 1 only (b) 2 only
which one of the following is correct? (c) Both 1 and 2 (d) Neither 1 nor 2
(a) v3 v2 v1 (b) v1 v3 v2 74. Consider the following statements: [2016-I]
(c) v1 v2 v3 (d) v3 v1 v2 1. There exists a regular polygon whose exterior angle is
70°.
A B Q 2. Let n 5. Then the exterior angle of any regular
68. polygon of n sides is acute.
P Which of the above statements is/are correct?
(a) 1 only (b) 2 only
(c) Both 1 and 2 (d) Neither 1 nor 2
75. If a quadrilateral has an inscribed circle, then the sum of a
pair of opposite sides equals [2016-II]
(a) Half the sum of the diagonals
D C (b) Sum of the other pair of opposite sides
In the above figure, ABCD is a parallelogram. P is a point on (c) Sum of two adjacent sides
BC such that PB : PC = 1 : 2. DP and AB when both produced (d) None of the above
meet at Q. If area of triangle BPQ is 20 square unit, the area 76. Let ABCD be a rectangle. Let P, Q, R, S be the mid-points of
of triangle DCP is [2015-II] sides AB, BC, CD, DA respectively. Then the quadrilateral
PQRS is a [2017-I]
(a) 20 square unit (b) 30 square unit (a) Square
(c) 40 square unit (d) None of the above (b) Rectangle, but need not be a square
69. A circle of radius r is inscribed in a regular polygon with n (c) Rhombus, but need not be a square
sides (the circle touches all sides of the polygon). If the (d) Parallelogram, but need not be a rhombus
perimeter of the polygon is p, then the area of the polygon 77. ABCDEF is a regular polygon. Two poles at C and D are
is [2015-II] standing vertically and subtend angles of elevation 30° and
(a) (p + n) r (b) (2p – n) r 60° at A respectively. What is the ratio of the height of the
pole at C to that of the pole at D ? [2017-I]
pr
(c) (d) None of the above (a) 1: 1 (b) 1 : 2 3
2
70. A rhombus is formed by joining midpoints of the sides of a (c) 2 3 : 1 (d) 2 : 3
rectangle in the suitable order. If the area of the rhombus is 78. In a trapezium ABCD, AB is parallel to CD and the diagonals
2 square units, then the area of the rectangle is [2016-I] intersect each other at O. What is the ratio of OA to OC
(a) 2 2 square units (b) 4 square units equal to ? [2017-I]
(a) Ratio of OB to OD (b) Ratio of BC to CD.
(c) 4 2 square units (d) 8 square units (c) Ratio of AD to AB (d) Ratio of AC to BD
Quadrilateral and Polygon A- 337
79. ABCD is a rectangle. The diagonals AC and BD intersect at 82. If a point O in the interior of a rectangle ABCD is joined
O If AB = 32 cm and AD = 24 cm, then what is OD equal to ? with each of the vertices A, B, C and D, then
[2017-I] OB2 + OD2 will be equal to [2017-II]
(a) 22 cm (b) 20 cm (a) 2OC2 + OA2 (b) OC2 – OA2
(c) 18 cm (d) 16 cm (c) OC2 + OA2 (d) OC2 + 2OA2
80. In the figure given below, AC is parallel to ED andAB =DE 83. The diagonals of a rhombus are of length 20 cm and
=5 cm and BC = 7 cm. What is the area ABDE : area BDE : 48 cm. What is the length of a side of the rhombus?
area BCD equal to ? [2017-I] [2017-II]
5cm (a) 13 cm (b) 26 cm
E D
(c) 36 cm (d) 39 cm
84. A closed polygon has six sides and one of its angles is 30°
greater than each of the other five equal angles. What is the
value of one of the equal angles? [2017-II]
(a) 55° (b) 115°
A 5cm B 7cm C (c) 150° (d) 175°
(a) 10 : 5 : 7 (b) 8 : 4 : 7 85. Given that the angles of a polygon are all equal and each
(c) 2 : 1 : 2 (d) 8 : 4 : 5 angle is a right angle. [2018-1]
81. In the figure given below, PQRS is a parallelogram. PA bisects Statement-1 : The polygon has exactly four sides.
angle P and SA bisects angle S. What is angle PAS equal to ? Statement-2 : The sum of the angles of a polygon having n
[2017-I] sides is (3n – 8) right angles.
Which one of the following is correct in respect of the above
statements?
(a) Both Statement-1 and Statement-2 are true and Statement-
2 is the correct explanation of Statement-1
(b) Both Statement-1 and Statement-2 are true
but Statement-2 is not the correct explanation of
(a) 60° (b) 75° Statement-1
(c) 90° (d) 100° (c) Statement-1 is true but Statement-2 is false
(d) Statement-1 is false but Statement-2 is true

HINTS & SOLUTIONS


1. (a) Given that, SPQ = 150° and PM = 20 cm
In parallelogram PQRS, D C
RSP + SPQ = 180° (interior angles) 6 cm m
RSP = 180° – 150° = 30° 8c
RSP = = 30° M

S M R 8 cm 90° 6c
m

20 cm
A 10 cm B
150° Hence, both A and R are true and R is the correct
P Q explanation of A.
In PSM, 3
3. (d) We know altitude of equilateral ABC is a.
PM 2
sin = sin 30° = C
SP
1 20
SP = 40 cm
2 SP
RQ = SP = 40 cm.
2. (a) H2 = P2+ B2 O
AB2 + BC2 + CD2 + AD2 = AC2 + BD2
(10)2 + (10)2 + (10)2 + (10)2 = (16)2 + (12)2 A B
400 = 400 60°
D F
EBD_7367
338
A- Quadrilateral and Polygon

3 2 a 24
Length of OC= a = radius AE =
2 3 3 5

b Area of rectangle ABCD 8 6


Also, DF= b DE = Area of rectangle AEFC
= =1
2 24
10
5
DE b/2
In ODE, cos 60°= 8. (c) Let sides of the square be a.
OD a/ 3 D C
1 3b
= a= 3b
2 2a
a2 = 3b2 O
4. (c) Here we see (50)2 = (30)2 + (40)2 a/ 2
2500 = 900 + 1600 2500 = 2500
It means given scores are the sides of a rectangle.
So, other diagonal should be 50 runs. 2
5. (c) We know that, if r be the radius of incircle and 2p be A L M K B
the perimeter of a quadrilateral, then a/ 2
Area of quadrilateral = pr
6. (b) ABCD is a trapezium. a/ 2
AD||BC and EF||BC (given) a
Hence, EF||AD Then, AC = a 2 and AO = OC =
2
x + y = 180°(interior angles)
y= 180° – 120° = 60° a
7. (c) Let ED = x Here, AM =
2
and area of rectangle ABCD
= AB × BC = 8 × 6 = 48 units a a a
LM = – and OM =
8 2 2 2
A B
a a 2 1
10 2 2 2
E 6 6 In OML, tan = = 2 1
2 a 1
x 8 2 2
C
D
10 2 tan
– x 2( 2 1)
2
tan =
F 1 tan 2 1 (2 1 2 2)
2
Now, AC = 82 62 = 10
In AED, AE + ED2 = AD2
2 2( 2 1) 2( 2 1)
=
AE2 = AD2 – x2 = 36 – x2 ...(i) 1 3 2 2 2 2 2
and in CFD, tan = 1
CF2 + DF2 = CD2 9. (b) Area of parallelogram = Base × Height
CF2 = (8)2 – (10 – x)2 ... (ii) = 15 × 5 = 75 sq cm
From eqs. (i) and (ii), we get
36 – x2 = 64 – (10 – x)2 ( AE = FC) C
36 – x2 = 64 – (100 + x2 – 20x)
(because AECE is rectangle) D
m

18
10 c

20x = 72 x=
5 5 cm
N
2
18
From eq. (i) AE2 = 36
5 A B
M
15 cm
324 900 324
AE2 = 36 = Area of parallelogram = Base × Height = 10 × DN
25 25
10 × DN = 75
576
AE2 = 75
25 DN = = 7.5 cm
10
Quadrilateral and Polygon 339
A-

10. (c) Isosceles trapezium has only line of symmetry. 13. (a) Let XOB =
D C

11. (b) In DMN and BMC,


DM = MC (mid-point) (given)
A X B
A B In OXB,
XOB + OBX + OXB = 180°
10 7 + 45° + OXB = 180°
O 6
5 9 OXB = 180° – 45° –
D 2 4 C OXB = 135° –
3 1 M Here, OXA + OXB = 180°
OXA + 135° – = 180°
OXA = 45° +
8
In OAX,
N AO = AX
OXA = AOX = 45° +
1 = 2 (vertically opposite angle) Since, AOX+ XOB = 90°
3 = 4 + 9(alternate interior angle) 45 + + = 90°
Since, BC||AD and intersects by CD. 2 = 45°
DMN BMC = 22.5°
DN = BC = AD (ASA) 14. (c) The quadrilateral formed by joining the mid-points of
the sides is a parallelogram.
AN 2 15. (d) In APS and PBQ,
So, AN = 2 BC ... (i)
BC 1
D C
In OAN an OBC, R
5 = 6 (vertically opposite angle)
7 = 8 (alternate interior angle) S
9 = 10 (rest angle)
OAN ~ OBC Q
So, the sides will be in same ratio
AN ON
= A B
BC OB P
AS = PB (given)
2 ON
= [from eq. (i)] AP = BQ (given)
1 OB and A = B = 90°
12. (d) Here, B = C = 65° (since, ABCD is square)
Here, GF||AB, which is intersects. Therefore, APS BQP are congruent.
SP = PQ
A F SPA = BQP and ASP = BPQ
SPQ = 90° (by RHS rule)
80° 16. (c) Since ABCD is a parallelogram and P, Q are the mid-
points of AB, CD respectively.
E AP = QC and AP || QC
APCQ is a parallelogram.
Now in DXC, Q is the mid point of CD and QY || CX,
65° 1 65° therefore Y will be the mid point of DX.
B G C D Q C
D
Now, BG 1 = B = 65°
(corresponding angles) Y
In FGD,
1+ F+ D = 180° X
65° + 80° + D = 180° D = 35°
A P B
EBD_7367
340
A- Quadrilateral and Polygon

DY = YX ...(i)
Z Y
Similarly
YX = XB ...(ii)
From equations (i) and (ii)
DY = YX = XB R Q
17. (c) If a parallelogram and a rectangle stand on the same
base and on the same side of the base with the same L K
height, then perimeter of parallelogram is greater than
perimeter of rectangle. W X
I1 > I2 P
18. (c) Let the sides of a parallelograms are x, y and xk, yk. ar( PRQ) RP 2
C E T S =
ar( PLK ) LP 2
(by properties of similar triangle)
y yk ar( PRQ) (2 LP ) 2
ar( PLK )
=
LP 2
A B D QP R ar(PRQ) = 4 ar(PLK)
x xk 1
Since, sides of two parallelogram are in 1 : k. ar(WXYZ) = 4 ar(PLK) [from eq. (i)]
4
AC BC BC y 1
ABC ~ PQT = = 1 ar( PLK ) 1
PT QT QT yk k ar(WXYZ) = ar(PLK) =
16 ar(WXYZ ) 16
Let BC = z and QT = zk
Ratio of areas of two similar parallelograms Explanations (Q. Nos. 21-23) :
Given, AO : OC = BO : OD
x z 1 and AB = 30 cm and CD = 40 cm
= = 2
xk zk k
19. (c) D R C D
40
cm
E

S Q
C

O
A P B

AP = PB A B
30 cm
CQ = QB
RC = DR
OA AB 3
SD = AS AOB ~ ACOD =
According to above ABCD is a parallelogram. OC CD 4
Then, the diagonals PR and SQ bisect each other. OAB = OCD and OBA = ODC
It means DC || AB. So, it is a trapezium.
1
Now, ar (RSQ) = ar (SQCD) ... (i) 1
2 Area of quadrilateral ABCD = ( AB CD) AE
2
1
and ar (PQS) =
ar(ABQS) ... (ii) 1
2 1050 = (30 40) AE
From addition of both (i) and (ii), we get 2
1 AE = 30 cm
ar (PQRS) =
ar(ABCD) Also, BAE = 90°
2 21. (b) In right EAB,
Thus, both statements are correct.
EB = AE 2 AB 2 EB = 302 302 = 30 2 cm
1
20. (b) ar(PRQ) = ar(WXQR)
2 1 1
22. (c) Area of ADC = CD AE 40 30 = 600 cm2
1 1 1 2 2
= ar(WXYZ ) = ar(WXYZ ) ... (i) 23. (b) Also BAE = 90°, AE = AB = 30 cm
2 2 4
AEB = ABE = 45°
Quadrilateral and Polygon 341
A-

24. (a) AB||DC and AD||BC ... (Given) 28. (b) In parallelogram ABCD.
A R D
A B
60° 60°
S Q

B P C

60° m2 n2
AC = 2 mn, BD = m2 – n2 and AB =
2
D C We know that,
E
(AC2 + BD2) = 2 (AB2 + BC2)
In ABE, (m + n + m4 + n4 – 2m2n2)
2 2
EAB = ABE = 60°
AEB = 60° 1 2
= 2 (m n2 )2 BC 2
ABE is an equilateral triangle. 4
Now, AB = BE = EA perimeter of ABE = 6
1 2
AB + BE + EA = 6 (m2 + n2)2 = (m n2 )2 2 BC 2
AB = 2 units 2
AB = BE = EA = 2 1 2
and in ADE, AE2 = AD2 + ED2 2BC2 = (m n2 )2
2
4 = AD2 + 1
(since, E is mid-point of CD) (m 2 n 2 )2
BC2 =
AD = 3 units 4
2
Therefore, area of quadrilateral ABCD = AB × AD m n2
BC =
= 2 × 3 = 2 3 sq units. 2
25. (d) We know that, ratio of the areas of two similar triangles Therefore, ABCD is a rhombus.
are equal to the ratios of squares of their corresponding Let AC > BD 2mn > m2 – n2
2 2
sides. (m + n) > 2m
which is always true for every positive integers m and
ar( BPQ ) PB 2 n, where n < m < 2n.
ar( DPC ) PC 2 29. (a) Statement I
Let side BC = x cm.
20 1 4
= A D
ar( DPC ) 4
ar ( DPC) = 80 sq units 3 5
26. (d) In parallelogram, d2 + d22 = 2 (l2 + b2) F
E 5
D C 3
d1
B C
b=12cm 4
m Given Area of BFC = 6 cm2
10c
d 1= 1
A I=8cm B 3 x =6 x = 4 cm
2 2 2
d + (10) = 2 (64 + 144)
In BFC, BF2 = x2 + 9 = 16 + 9 BF2 = 25
d2 = 2 × 208 – 100
BF = 5
d2 = 416 – 100 = 316 d= 316 area of rectangle ABCD, pq2 = p (2)2 cm2
d = 17.76 cm Statement II
d > 12 which is of the form pq2.
27. (c) ABCD is a rhombus. While the area of EBFD cannot be the form of r2 cm2.
AB = BC = CD = DA
30. (b) N M
A D

B C P 6 cm R L
and diagonals bisect each other at right angles
Area of parallelogram = 6 × Area of NPR
EBD_7367
342
A- Quadrilateral and Polygon

1 ZAX = 90° ZAD + XAD = 90°


NR × PL = 6 × NR PR ZAD = 90° – (90° – ) =
2 i.e., BAX = ZAD
PL = 3PR (here, PL = PR + RL) ABX ADZ
PR + RL = 3PR BX = DZ (by CPCT)
RL = 2PL = 2 × 6 = 12 cm 35. (b) Diagonals of a rhombus are perpendicular bisector of
31. (b) We know that in a square diagonals are equal and each other. Hence each point on the diagonal BD is
bisect each other at 90°. equidistant from vertices A and C.
Hence, the required quadrilateral is a square. Therefore required locus is the diagonal BD.
32. (d) If two parallel lines are cut by two distinct transversals, 36. (c) Join AC.
the quadrillateral formed by the four lines is always a In ACD, EG||DC and E and G are mid-points of AD
‘Trapezium’. and AC, respectively.
Case I If two distinct transversals (are not parallel),
then always (Trapezium) A B
h
A D
E F
G
h
D C
B C
1 3
Case II If two distinct transversals are parallel, then EG = DC
2 2
always (Trapezium + Parallelogram) Similarly, in ABC
A D 1
GF = AB = 1
2
3 5
EF = EG + GF = 1
B C 2 2

33. (b) Since, line segment 1


Area of trapezium = (Sum of parallel side × Height)
AP and CQ bisects the A and C, respectively. 2
Then, AP||CQ Area of ABFE
Now in APQ and CQP Now, the ratio = Area of
AP || QC EFCD
D C 1 5
P 2 h
2 2 9
= =
1 5 11
3 h
Q 2 2
A B 37. (c) In parallelogram PQRS, AP, AQ, CR and CS are
APQ = PQC (alternate angle) bisector of P, Q, R and S.
PQ = PQ (common) In RBQ B = 90° (Because, Q + R = 180° and
Also, PC || AQ Q R 180
CPQ = PQA (alternate angle) =
2 2 2
APQ ~ CQP (by ASA)
In PDS, D = 90° (Because P+ S = 180° and
Thus, APQ is similar to PCQ.
34. (d) In ABX and ADZ. P S 180
AB = AD (side of a square ABCD) = = 90°
2 2 2
and AX = AZ (side of square AXYZ) ABCD is a rectangle.
Let BAX = 38. (c) From figure.
XAD = 90° –
A D
A B x° y°

X 120°
E Q z° F
Z
C
D
90° 50°
Y B C
Also, AXYZ is a square, BC || EF || AD
Quadrilateral and Polygon 343
A-

x° = z° = 50° (corresponding interior angle Now in PQR,


+ z° = 180° (linear pair)
RQ
= 180° – 50° = 130° tan QPR =
In quadrilateral PQ
AQFD, x° + y° + 120° + = 360°
QR 1
50° + y° + 120° + 130° = 360° tan QPR = =
y = 360° – 300° = 60° 3QR 3
39. (c) The quadrilateral must be a trapezium because a QPR = 30°
quadrilateral where only one pair of opposite sides are PRS = 30° [from eq. (i)]
parallel (in this case AB || CD) is a trapezium. 43. (b) In BCF,
40. (c) Constraction : In quadrilateral ABCD, form A to C.
Now, in ABC D C
2
A B
5 3 3 5

A 4 E F B
D C 2 4
AB = BC ...(Given) Pythagoras theorem,
BAC = BCA (5)2 = (3)2 + (BF)2 BF = 4cm
(angles opposite to equal side) AB = 2 + 4 + 4 = 10 cm
In ADC, Now in ACF, AC2 = CF2 + FA2 AC2 = 32 + 62
CD > AD
DAC > DCA AC = 45 cm
(since in a triangle, angle opposite to greater side is
Similarly, BD = 45 cm
bigger than the angle opposite to smaller side)
On adding eqs. (i) and (ii), we get Sum of diagonal = 2 × 45 = 2 × 3 5
BAD > BCD
41. (d) Since, they are symmetrically lying on horizontal plane. = 6 5 cm
44. (b) Here, Area of Rectangle lies between 40cm2 and
A 45cm2
x Given that one sides = 5cm.
E C Area of Rectangle = 5 × second sides
Now, If Area = 40cm2
a–b a–b then, 40 = 5 × second sides
second sides = 8cm.
F Again, If Area = 45cm2
D 45 = 5 × second sides.
x
Second sides = 9cm.
B
it means that second sides varies.
AC = BD between 8 cm to 9 cm.
AE = BF = x Let diagonal = d
Now, AB = (a – b) + 2x
i.e., a+b = a – b + 2x 2b = 2x 82 52 d 52 92
x =b 64 25 d 25 81
Now in ACE,
2 2 2
x +a = AC 87 d 106
2 2 2 81 89 d 106 121
AC = b + a AC = b2 a2
42. (c) In rectangle PQRS, 81 d 121
S R 9cm < d < 11cm.
45. (b) Area of APS = Area of DSR
R
D C

S Q
P Q
3 QR
PQ || RS A B
P
RPQ = PRS AS = SD and AP = DR
(Alternate interior angles) ... (i) ar ( ABC) = 4ar ( BPQ)
EBD_7367
344
A- Quadrilateral and Polygon

46. (b) I. ABCD is a parallelogram, then So, Statement 1 is not correct.


AC2 + BD2 = 2(AB2 + BC2) Clearly, straight lines AC, BD, XY and MN meet at
D C a point, So Statements 2 is correct.
49. (b) Side of cube = 2 cm
Maximum distance between two points of a cube
O = Length of diagonal
= 3 × side = 2 3 cm
A B
So it is not true. 50. (d) Perimeter of parallelogram land = 86 m and diagonal
II. ABCD is a rhombus and diagonals AC and BD = 41m
bisect each other. Suppose one side of parallelogram be x m
AO = OC other side = (x + 13) m
and OB = OD Perimeter = 2(x + x +13) = 86
In AOB, AB2 = AO2 + OB2 86
2 2
2x + 13 = = 43 2x = 43 – 13 = 30
AC BD 2
2
(4) = 30
2 2
x= = 15
AC2 + BD2 = 64 2
= (4)3 i.e., n3 one side of parallelogram = 15 m
So only II is true. other side = 15 + 13 = 28 m
47. (d) In AFD and BFE,
A 28 m B
AFD = BFE
(vertically opposite angles)
D C
15 m
nx 41 m 15 m
F E N
mx

A B D C
and ADF = FBE (alternate angles) M 28 m
AFD ~ EFB (By AA)
area of ABD = s( s a)(s b)( s c)
ar( EFB ) EB 2
So, = = 42(42 15)(42 28)(42 41)
ar( AFD) AD 2
2 15 41 28 84
(mx )2 m2 m s 42
= 2 = 2 = 2 2
(mx nx ) (m n) ( m n)
48. (b) From Statement 1. Given, ABCD is a parallelogram. = 42 27 14 1 = 126 m2
X and Y are mid–points of BC and AD, respectively. Required area of parallelogram = 2 × Area of ABD
M and N are the mid–points of AB and CD, = 2 × 126 = 252 m2
respectively. 51. (a) From the above figure, shorter height of parallelogrm
= AM
D N From question 30
C
Area of parallelogram = Base Height = 252m2
Y 252
X 28 × AM = 252 AM = =9m
28
Therefore, shorter height of the parallelogram is 9 m.
A B 52. (a)
M 1. Let second diagonal be x.
From statement 2. Here join point A and C. Now, x2 + 412 = 2(152 + 282)
In ABC, M and X are mid–points of AB and BC. x2 = 337 = 18.36 (approx)
1 Difference between the diagonals
MX || AC and MX = AC ...(i) = 41 – 18.36 = 22.64
2 Which is more than 20
In ADC, Y and N are mid–points of AD and CD. Therefore, Statement 1 is correct.
1 2. Second height of parallelogram ABCD = CN
YN || AC and YN = AC ...(ii) Base × Height = Area
2
From equations (i) and (ii), we get MX || YN 252
From statement 2 × CN = 252 CN = = 16.8 m
15
Quadrilateral and Polygon 345
A-

Difference between the heights = 16.8 – 9 = 7.8 1 1


which is not more than 10. = Area of ABCD + Area of ABCD
Statement 2 is not correct. 2 4
53. (c) In a cuboid, 4 perpendicular face pairs in bottom 3
surface, 4 perpendicular face pairs in top surface and = Area of ABCD
4
4 perpendicular face pairs in vertical surface. Therefore, Statement 1 is correct.
Total perpendicular pairs are 12. 2. ABCD is a trapezium.
54. (c) In a cube, there are six faces.
Let the sides of a cube be a. A B
Diagonal of face = a 2 a 2 a 2
Hence, there is no equilateral triangle will be formed
in faces. M

Z
D C
C (0, 0, a) Area of trapezium = Area of DCM + Area of ABM
F (a, a, a) + Area of AMD
E
(0,0, a) A (a, 0, 0) Area of DCM + Area of ABM
X = Area of trapezium ABCD – Area of AMD
O (0, 0, 0) If AD = BC, then using eq. (i), we get
B (0, a, 0) D (a, a, 0) Area of DCM + Area of ABM
Y
1
In ABC, Area of trapezium ABCD
2
AB a2 a2 a 2, BC a2 a2 a 2 If AD = BD, then it is true otherwise are of DCM
and area of ABM is greater than half of the area of
and AC a2 a2 a 2 trapezium ABCD.
ABC, is an equilateral triangle. Therefore Statement 2 is also correct.
Similarly, In ABE, ODG, ODE, CEB, CEA, 56. (a) 1. If AB = CD, then ABCD is a parallelogram
FGO and FGD
1
Eight equilateral triangles are possible. Area of ADM Area of ABCD
55. (c) 1. Given, ABCD is a trapezium. 2
If AB = CD, then it becomes a parallelogram. A B
M is the mid-point of BC.
1
Area of AMD = area of ABCD ...(i)
2 M
A B

D C
M [since, both are in same base and between same
parallels]
Area of ADM – Area of ABM
D C 1
Area of ABCD
Because, both are between same parallels and in 2
same base. 1
By according to question – Area of ABM = Area of DMC
4
1 Statement 1 is correct.
Area of ABM + Area of DCM area of ABCD
2 1
Area of ABM = Area of DCM 2. Area of ABM + area of DCM = Area of ABCD
2
1
2 Area of DCM = Area of ABCD 1
2 2 Area of ABM = Area of ABCD
2
1 [ area ABM = area of DCM]
Area of DCM = Area of ABCD ...(ii)
4 1
On adding equations (i) and (ii), we get Area of ABM = Area of ABCD
4
Area of AMD + Area of DCM
Statement 2 is not correct.
EBD_7367
346
A- Quadrilateral and Polygon

57. (a) Given, ABCD is a parallelogram. Join AC and BD 60. (d) Let AD is diameter of circle of centre O. Find OP = q
which intersect each other at O.
A B

17 cm O
A D
O R
15 17 cm
P
Q
D C
P B
AD = 34 cm, AO = OD = 17 cm
1
OC AC 30
2 AB = 30 cm, AP = = 15 cm
In CBD, P and R are mid-points of DC and BC. 2
PR || BD or PQ || DO and RQ || BO OP = (17)2 – (15)2 = 64 = 8cm.
Again in OCD, PQ || OD
So, Q is mid-point of OC. 61. (a) A
1 1 1 1 72° 72°
CQ OC AC AC G F
2 2 2 4 B E
58. (a) A quadrilateral ABCD, AP and BP are bisectors of 108°
H J
A and B, respectively.
Pentagon I
D D
C
C
Sum of Interior angles
P
= (number of sides –2) × 180°
= (5 – 2) × 180° = 540°
540
A Interior angle of regular pentagon = 108
B 5

1 1
APB = 180° – A B
2 2
72°
We know that sum of all angles of a quadrilateral
= 360° 108°
A + B + C + D = 360°
1 1 1 1 360
A B C D
2 2 2 2 2
1 1 1 1
C D 180 – A B Now suppliment angle = 180° – 108° = 72°
2 2 2 2 Now, we can find angle at the top point of the star
1 by adding the two equal base angles.
( C D) APB [from eq. (i)] = 180° – (72° + 72°)
2 = 180° – 144° = 36°
C + D = 2 APB So each point of the stars = 36°
n 2 180
59. (c) Sum of angle of regular polygon
n 62. (c) A B
135 n = 180n – 360
45 n = 360
360 O
n 8
45
5cm
Number of diagonals = 8C2 – 8 90°
8 7
– 8 = 20
2 D C
13cm
Quadrilateral and Polygon A-347

OC = 169 – 25 = 144 = 12 cm 64. (b) D


C
AC = 2OC = 24 cm
1
Area of Rhombus = d1 d 2 a
2
1
= 10 24 = 120 cm2 60°
2 A B
2a
E Using cosine law
63. (b)
AD 2 + AB2 - BD2
D C cos A =
y G 2.AD.AB

AD2 + (2AD) - BD 2
2
O cos 60° =
x x 2.AD.2AD
1 AD + 4AD2 - BD2
2
A F B =
EAB is equilateral
2 4AD 2
EDC is also equilateral 2AD2 = 5AD2 – BD2
BD2 = 5AD2 – 2AD2
Area of trapezium ABCD BD = 3AD2
1 1 BD = 3 AD
DB OA DB OC
2 2 So, option (b) is correct.
1 65. (c)
DB AC 66. (d) D C
2
Let AO = OB = x and DO = OC = y
1
Area (ABCD) (x y)2 16 (given)
2
x+y= 4 2 ... (i)
AOB is a right angled isosceles triangle. A a B
Let initially the side of square be ‘a’.
So, AB x2 x2 2x Area of square = a2
Area of reactangle = 1.3 a × 1.2 a
Similarly, DC = 2y Percentage increase = 1.56 a2 in Area
Now, FG = EF – EG (1.56a 2 - a 2 )
FG = AB sin 60° – DC sin 60° = ´100
a2
3 6 = (1.56 – 1) 100 = 56%
( AB – DC ) ( x – y ) ... (ii)
2 2 So, option (d) is correct.
Area of trapezium 67. (c) The dimensions of the 3 rectangles are x1y1; x2y2;
= Area EAB – Area EDC x3y3. x1 < x2 < x3 and x1y1 = x2y2 = x3y3
3 ( y1 > y2 > y3). By joining the parallel sides along the
( AB 2 – DC 2 ) breadth to form a cylender, the length becames the
4 circumference of the base (i.e.x = 2 r) and the breadth
3 becomes the height. The quantities are fabulated below:
( x 2) 2 – ( y 2)2
4 R1 R2 R3
3 Length x1 x2 x3
Area (x + y) (x – y)
2 Breadth y1 y2 y3
Height of y y2 y3
3 cylinder 1
Now, (x y )( x – y) 16
2 Base x1 x2 x3
32 8 radus 2 2 2
x–y= x– y x y 4 2 Volume x1y1 x2y2 x3y3
3( x y) 6 4 4 4
6 6 8 As x1y1 = x2 y2 = x3 y3
Height = ( x – y) = 4 cm and x1, x2< x3 of follows v1 < v2 < v3.
2 2 6
EBD_7367
348
A- Quadrilateral and Polygon

68. (d) No. of vertices or sides = 9


A B Q Option (b) is correct.
72. (c) Let length of the side of the square = x units
P Length of the diagonal = 2 x units
ADE EFC (By AAA) A
AD DE
D C EF FC
BPQ and CPD are similar then,
q x x
2
area of DBPQ æç BP ö÷ x p x
= çç ÷÷
area of DCPD è PC ø
q–x
x = pq–px – qx + x2
2
q
20 æ 1 ö2 pq
= ççç ÷÷÷ x=
area of DCPD è 2 ø p q D x E
Area of CPD = 4 × 20 = 80 square unit 2 pq x
So, option (d) is correct. BE = = Length 2x x
69. (c) The n-sided polygon can be dinded into ‘n’ triangle p q
with O, the Centre of the circule as one veotex for each of the diagonal x p–x
triange. The altitude of each triangle is r. Let the sides of B F C
p
the polygon be ‘a1’, a2 -----an. (Given a1 = a2 = ------an) 73. (a) Statement (1) Since m 3 and n 3
nr pr Both of them can take equal values or different values
The area of polygon is
2 2 Let m = 4 and n = 3
a1r a2r a n r pr 360 360
Area of polygon = Sum of Exterior angles =
2 2 2 2 No. of sides m
So, option (c) is correct.
70. (b) Let PQRS be a rectangle and ABCD be a rhombus which 360
= 90
is formed by joining the mid points of a rectangle. 4
P A Q 360 360
Sum of Exterior angles = = 120°
n 3
It takes different values at different values of m and n
B D Statement 1 is true.
Statement (2) take m = 4 and n = 3
Sum of interior angles = (m – 2) × 180
R C S
= (4 – 2) × 180 = 360°
Given Area of rhombus = 2 unit.
Sum of interior angles = (n – 2) × 180°
1 = (3 – 2) × 180 = 180°
But Area = d1 d2 2
2
d1d2 = 2 × 2 = 4 units m n 4 3 90 = 630°
2
where d1 = diagonal AC
But sum = 360 + 180 = 540
d2 = diagonal BD
Statement 2 is not true.
But AC = Breadth of rectangle
Option (a) is correct.
BD = length of rectangle
74. (b) (1) If Exterior angle = 70°
Area of rectangle = AC × BD
= d1 × d2 360
then no. of sides = 5.14
= 4 units 70
option (b) is correct. which is not possible
71. (b) Since interior angle of the regular polygon = 140° Because side of polygon does not exist in decimals
Hence exterior angle = 180° – 140° = 40° (1) is not true.
Statement (2) Let n 5
360 360
No. of sides = = =9 Let n = 5
Exterior angle 40
Quadrilateral and Polygon 349
A-

360 D
Exterior angle = 72
5
60
An exterior angle of any regular polygon of n sides is 2a
acute. a
(2) is true.
Option (b) is correct.
A C
3a
75. (b) 7cm 9cm
A B
AC
9cm = cot 30°
7cm a

r AC = 3a
AD
= cosec 30°
5cm 8cm a
AD = 2a
C D Now taking ASD, Let 'S' is the vertex of pole.
5cm 8cm
S
BD = 9 + 8 = 17
AC = 7 + 5 = 12
AC + BD = 29
Same AB + CD = 29
So, the sum of a pair of opposite sides equals to sum
of the other pair of opposite sides. 60°
76. (c) P D
A B A 2a
T
DS
S Q = tan 60°
AD

D C DS = 2 3
R
Taking TCA
1 1 30°
PQ = AC; SR = AC C
2 2 A
TC 3a
{Mid point theorem} = tan 30°
AC
1 1
Similarly PS = BD; QR = BD 3a
2 2
TC = =a
BD = AC 3
(Diagonal of rectangle) Ratio
PQ = QR = RS = SP
CT a 1
So PQRS is a Rhombus but need not be a square. DS 2 3a 2 3
77. (b) ABC = 120° 78. (a)
(Angle of regular hexagon)
A B
180 120
BAC = BCA = = 30°
2
DCA = 120 – 30° = 90° O
i.e. DCA is a right
when side DC = a
S D C
D in ADB and COD
T D= B
{Alternate interior}
E C= A
C By A
AOB ~ COD
AO OB
F B =
OC OD
A
EBD_7367
350
A- Quadrilateral and Polygon

79. (b) Draw rectangle ABCD with arbitrary point O within it,
A B
and then draw lines OA, OB, OC, OD. Then draw lines
from point O perpendicular to the sides: OE, OF, OG,
O
OH. Using Pythagorean theorem we have from the
above diagram:

D C OA2 AH 2 OH 2 AH 2 AE 2
AB = 32, AD = 24 OD = ?
DB2 = 322 + 242 OC 2 CG 2 OG 2 EB 2 HD 2
= 82 [42 + 32] OB 2 EO 2 BE 2 AH 2 BE 2
DB2 = 8 [52] = (40)2
DB = 40 OD 2 HD 2 OH 2 HD 2 AE 2
OD = 20
Adding these equalities, we get
80. (a) Let height of triangle be = h
as ABDE = 5 × h OA2 OC 2 AH 2 HD 2 AE 2 EB 2
1
ar BDE = ×5×h OB 2 OD 2 AH 2 HD 2 AE 2 EB 2
2
From these equalities, we get
1
ar BCD = ×7×h OA2 OC 2 OB 2 OD 2
2
E 5 D 83. (b)

A B

h
O
A 5 B 7 C

5h 7h
Ratio = 5 h : :
2 2
= 10 : 5 : 7 D C
81. (c) P Q Since the diagonals of rhombus bisect each other,
therefore AO = OC = 10 cm and BO = OD = 24 cm.
In AOB, by Pythagoras theorem, we get
A AB AO 2 OB 2 AB 102 242 AB
S R
100 576 AB 676 AB 26 cm.
P + S = 180
{Sum of adjacent angles of 11 gm} Hence, the side of rhombus is 26 cm.
P S 84. (b) For finding the sum of the interior angles of
= 90 a polygon is the same, whether the polygon is regular
2 2
or irregular. So, we would use the formula (n – 2) ×
P S 180°, where n is the number of sides in the polygon.
A = 180
2 2 Let one angle be of the polygon be x and other 5 equal
A = 180 – 90 = 90 angles be y.
B F C So, according to the question,
82. (c)
x y 30 (n 2) 180 x 5y
O (6 2) 180 y 30 5 y 180.4 6 y 30
E G
720 6 y 30 6y 690 y 115
85. (c)

A H D
Circle 351
A-

C HA P T E R

CIRCLE
24
1. C is a point on the minor arc AB of a circle with centre In the figure given above, PT = 6 cm and PA = 4 cm. What
O. If ÐAOB = 100°, then what is ÐACB? [2007-I] is the length of AB? [2007-II]
(a) 80° (b) 90° (a) 9 cm (b) 5 cm
(c) 100° (d) 130° (c) 4 cm (d) 6 cm
2. In a DABC, AB = AC. A circle through B touches AC at 6. C
D and intersects AB at P. If D is the mid-point of AC, then
which one of the following is correct? [2007-I]
(a) AB = 2AP (b) AB = 3AP D
(c) AB = 4AP (d) 2 AB = 5AP
35°
3. P 75°
A B
In the figure given above, C and D are points on the semi-
40° circle described on AB as diameter. If ÐABD = 75° and
ÐDAC = 35°, then what is the ÐBDC? [2007-II]
B (a) 130° (b) 110°
C (c) 90° (d) 100°
7.

59°
A Q
D O
100° C
In the given figure, if ÐPAQ = 59°, ÐAPD = 40°, then what
is ÐAQB? [2007-I]
(a) 19° (b) 20° A B
(c) 22° (d) 27°
In the figure given above, the ÐAOC is 100°, where O is
x y z the centre of the circle. What is the ÐABC? [2007-II]
4. In the given figure, if = = , where ÐDCQ = x,
3 4 5 (a) 100° (b) 80°
ÐBPC = y and ÐDQC = z, then what are the values of (c) 120° (d) 130°
x, y and z, respectively? [2007-II] R
8.
A
Q
O
75°
D 120° B
B A
C
P P
Q
In the figure given above, if ÐAOP = 75° and ÐAOB =
(a) 33°, 44° and 55° (b) 36°, 48° and 60° 120°, then what is the value of ÐAQP? [2007-II]
(c) 39°, 52° and 65° (d) 42°, 56° and 70° (a) 45° (b) 37.5°
T 6 cm (c) 30° (d) 22.5°
5. P
9. X
T
4 cm
A

Z Y
B
EBD_7367
352
A- Circle

In the figure given above, O is the centre of the In the figure given above, from a point T, 13 cm away from
circumcircle of the DXYZ. Tangents at X and Y intersect the centre O of a circle of radius 5 cm, tangents PT and
at T. If ÐXTY = 80°, then what is the value of ÐZXY? QT are drawn. What is the length of AB? [2008-I]
[2007-II] 19 20
(a) 20° (b) 40° (a) cm (b) cm
3 3
(c) 60° (d) 80°
40 22
10. P (c) cm (d) cm
13 3
14. With A, B and C as centres, three circles are drawn such
that they touch each other externally. If the sides of the
DABC are 4 cm, 6 cm and 8 cm, then what is the sum of
A B the radii of the circles? [2008-I]
(a) 9 cm (b) 10 cm
(c) 12 cm (d) 14 cm
15. If two circles C1 and C2 have three points in common,
Q then which of the following is correct? [2008-I]
(a) C1 and C2 are concentric
Two circles with centres A and B touch each other (b) C1 anc C2 are the same circle
internally, as shown in the figure given above. Their radii (c) C1 and C2 have different centres
are 5 and 3 units, respectively. Perpendicular bisector of (d) None of the above
AB meets the bigger circle in P and Q. What is the length 16. What is the number of tangents that can be drawn to a
of PQ? [2007-II] circle from a point on the circle? [2008-I]
(a) 2 6 (b) (a) 0 (b) 1
34 (c) 2 (d) 3
(c) 4 6 (d) 6 2 17.
11. A
B
45° O

A B P C D
50°
X Y In the figure given above, AD is a straight line, OP
perpendicular to AD and O is the centre of both circles.
In the figure given above, what is ÐBYX equal to? If OA = 20 cm, OB = 15 cm and OP = 12 cm, then what
[2008-I] is AB equal to ? [2008-I]
(a) 85° (b) 50° (a) 7 cm (b) 8 cm
(c) 45° (d) 90° (c) 10 cm (d) 12 cm
18.
12. P B

O
20° 30°
O
C
A
A B
In the figure shown above, the radius OA is equal to the In the figure given above, O is the centre of the circle.
What is ÐAOC? [2008-II]
chord AB. Then, what is ÐAPB? [2008-I]
(a) 160° (b) 150°
(a) 30° (b) 60°
(c) 120° (d) 100°
(c) 15° (d) 45°
19. F
13. P C
A
55°
E
O T A B
O E
110°
B
Q D
Circle 353
A-

In the figure given above, AB is a diameter of the circle 25. ABCD is concyclic quadrilateral. The tangents at A and
with centre O and EC = ED. What is ÐEFO? [2008-II] C intersect each other at P. If ÐABC = 100°, then what
(a) 15° (b) 20° is ÐAPC equal to? [2009-I]
(c) 25° (d) 30° (a) 10° (b) 20°
(c) 30° (d) 40°
B
20. 26. E
x

A C B
O
D
Q
In the figure given above, A is the centre of the circle and
AB = BC = CD. What is the value of x? [2008-II] Y X
A
1 In the figure given above, YAX is a tangent to the circle
(a) 20° (b) 22 °
2 with centre O. If ÐBAX = 70° and ÐBAQ = 40°, then what
(c) 25° (d) None of these is ÐABQ equal to? [2009-I]
(a) 20° (b) 30°
21. B (c) 35° (d) 40°
27. B
Q
A
O P

R
A C
Q
D C In the figure given above, If AP = 3 cm, PB = 5 cm,
In the figure given above, a circle is inscribed in a AQ = 2 cm and QC = x, then what is the value of x?
quadrilateral ABCD. Given that, BC = 38 cm, QB = 27 cm, [2009-I]
DC = 25 cm and AD is perpendicular to DC. What is the (a) 6 cm (b) 8 cm
radius of the circle? [2009-I] (c) 10 cm (d) 12 cm
(a) 11 cm (b) 14 cm 28. D
(c) 15 cm (d) 16 cm
22. A
O

A 40° C
75º C
P B
Q 30º In the figure given above, O is the centre of a circle
B circumscribing a quadrilateral ABCD. If AB = BC and
R ÐBAC = 40°, then what is ÐADC equal to? [2009-I]
In the figure given above, what is ÐCBA? [2009-I] (a) 50° (b) 60°
(a) 30° (b) 45° (c) 70° (d) 80°
(c) 50° (d) 60° 29. C
23. A, B, C and D are four distinct points on a circle whose B
centre is at O.
If ÐOBD – ÐCDB = ÐCBD – ÐODB, then what is ÐA P O
equal to? [2009-I]
(a) 45° (b) 60°
(c) 120° (d) 135° T
24. PQ is a common chord of two circles. APB is a secant line In the figure given above, PT is a tangent to a circle of
joining points A and B on the two circles. Two tangents radius 6 cm. If P is at a distance of 10 cm from the centre
AC and BC are drawn. If ÐACB = 45°, then what is ÐAQB O and PB = 5 cm, then what is the length of the chord
equal to? [2009-I] BC? [2009-II]
(a) 75° (b) 90° (a) 7.8 cm (b) 8 cm
(c) 120° (d) 135° (c) 8.4 cm (d) 9 cm
EBD_7367
354
A- Circle

30. Q 36. In the given figure, AB is a diameter of a circle and CD


is perpendicular to AB, if AB = 10 cm and AE = 2 cm, then
what is the length of ED? [2010-I]
P R C
x
O z M
A B
E O
y
U V
T D
In the figure given above, O is the centre of the circle. (a) 5 cm (b) 4 cm
The line UTV is a tangent to the circle at T, ÐVTR = 52°
and DPTR is an isosceles triangle such that TP = TR. (c) 10 cm (d) 20 cm
What is Ðx + Ðy + Ðz equal to? [2009-II] 37. ABCD is a quadrilateral, the sides of which touch a circle.
(a) 175° (b) 208° Which one of the following is correct? [2010-II]
(c) 218° (d) 250° (a) AB + AD = CB + CD
31. (b) AB : CD = AD : BC
(c) AB + CD = AD + BC
(d) AB : AD = CB : CD
O 38. The diameter of two circles are 18 cm and 8 cm. The
C distance between their centres is 13 cm. What is the
number of common tangents? [2010-II]
(a) 1 (b) 2
A B (c) 3 (d) None of these
In the figure given above, ÐAOB = 46°, AC and OB
39. Let PAB be a secant to a circle intersecting at points A
intersect each other at right angles. What is the measure and B and PC is a tangent. Which one of the following
of ÐOBC (where, O is the centre of the circle)? is correct? [2010-II]
[2009-II] (a) The area of rectangle with PA, PB as sides is equal
(a) 44° (b) 46° to the area of square with PC as sides
(c) 67° (d) 78.5° (b) The area of rectangle with PA, PC as sides is equal
32. B D to the area of square with PB as sides
(c) The area of rectangular with PC, PB as sides is
C equal to the area of square with PA as side
(d) The perimeter of rectangle with PA, PB as sides is
O 106°
equal to the perimeter of square with PC as side
40. In the figure given below, if ÐBAD = 60°, ÐADC = 105°,
A then what is ÐDPC equal to? [2010-II]
In the figure given above. If O is the centre of the circle P
ÐAOD = 106°, then what is ÐBCD equal to? [2009-II]
(a) 53° (b) 43°
(c) 40° (d) 37° D C
33. What is the number of circles passing through a given
pair of points? [2010-I]
(a) One
(b) Two A B
(c) More than two, but finite
(d) Infinitely many (a) 40° (b) 45°
34. Two circles touch each other externally at P. Two secants (c) 50° (d) 60°
APB and CPD are drawn through P to meet the circle at 41. In the figure given below, PQ is a diameter of the circle
A, C and B, D respectively. Then, which one of the whose centre is at O. If ÐROS = 44° and OR is a bisector
following is correct? [2010-I] of ÐPRQ, then what is the value of ÐRTS? [2010-II]
(a) AC is perpendicular to BD
(b) AC intersects BD P R T
(c) AC is parallel to BD
(d) None of the above
35. What is the locus of centres of circles which touch a O S
given line at a given point? [2010-I]
(a) A line perpendicular to the given line, passing
through the given point Q
(b) A line parallel to the given line
(c) A circle tangent to the given line at the given point (a) 46° (b) 64°
(d) A closed curve other than a circle (c) 69° (d) None of these
Circle A-355
42. In the figure given below. O is the centre of the circle. AC length of the transverse common tangent of these circles?
and BD intersect at P. If ÐAOB =100° and ÐDAP = 30°, [2011-I]
then what is ÐAPB? [2010-II] (a) 8 cm (b) 7 cm
(c) 6 cm (d) None of these
E 48. ABC is an equilateral triangle inscribed in a circle with AB
= 5 cm. Let the bisector of the angle A meet BC in X and
the circle in Y. What is the value of AX . AY?
[2011-I]
D C (a) 16 cm2 (b) 20 cm2
P (c) 25 cm2 (d) 30 cm2
49. Two unequal circle are touching each other externally at
O P, APB and CPD are two secants cutting the circles at A,
B, C and D. Which one of the following is correct?
A B [2011-I]
(a) ACBD is parallelogram(b) ACBD is a trapezium
(c) ACBD is a rhombus (d) None of the above
50. Let C be a circle. A point P moves such that the tangents
(a) 77° (b) 80° from P to C include an angle of 60°. What is the locus
(c) 85° (d) 90° of P? [2011-I]
43. In the given figure, A and B are end points of diameter (a) Straight line
of a circle with centres at P and C is a point on the (b) A circle concentric with C
circumference of the circle such that ÐABC = 35°, then (c) A circle touching C
what is ÐPCA? [2010-II] (d) A circle intersecting C at two points
51. A bicycle is running straight towards North. What is the
A locus of the centre of the front wheel of the bicycle
C whose diameter is d? [2011-I]
(a) A line parallel to the path of the wheel of the bicycle
P at a height d cm
(b) A line parallel to the path of the wheel of the bicycle
at a height d/2 cm
(c) A circle of radius d/2 cm
B (d) A circle of radius d cm
(a) 25° (b) 30° 52. What is the length of the perpendicular drawn from the
(c) 35° (d) 55° centre of circle of radius r on the chord of length 3 r ?
44. What is the number of circles passing through all the [2011-II]
vertices of a given triangle? [2011-I]
(a) One (b) Two (a) r (b) 2r
(c) Three (d) Infinite (c) r/2 (d) r/4
45. Consider the following statements 53. Consider the following statements
I. Let P be a point on a straight line L. Let Q, R and S I. The opposite angles of a cyclic quadrilateral are
be the points on the same plane containing the line supplementary.
L such that PQ, PR and PS are perpendicular to L. II. Angle subtended by an arc at the centre is double the
Then, there exists no triangle with vertices Q, R and angle subtended by it at any point on the remaining
S. part of the circle.
II. Let C be a circle passing through three distinct points Which one of the following is correct in respect of the
D, E and F such that the tangent at D to the circle above statements? [2011-II]
C is parallel to EF. Then, DEF is an isosceles triangle. (a) Statement I Þ Statement II
Which of the statement (s) given above is/are correct? (b) Statement II Þ Statement I
[2011-I] (c) Statement I Û Statement II
(a) Only I (b) Only II (d) Neither Statement I Þ Statement II
(c) Both I and II (d) Neither I and II nor Statement II Þ Statement I
46. Two circles touch each other internally. Their radii are 4 54. B
cm and 6 cm. What is the length of the longest chord of
the outer circle which is outside the inner circle?
[2011-I]
(a) 4 2 cm (b) 4 3 cm O
(c) 6 3 cm (d) 8 2 cm
A M C
47. The distance between the centres of two circles having
radii 4.5 cm and 3.5 cm respectively is 10 cm. What is the
EBD_7367
356
A- Circle

In the figure given above, O is the centre of the circle, If I. PX . PY = PZ . PT


OA = 3 cm, AC = 3 cm and OM is perpendicular to AC, II. PXZ and PTY are similar triangles.
then what is ÐABC equal to? [2011-II] Which of the statements given above is/are correct?
(a) 60° (b) 45° [2013-II]
(c) 30° (d) None of these (a) Only I (b) Only II
55. AC is the diameter of the circumcircle of the cyclic (c) Both I and II (d) Neither I nor II
quadrilateral ABCD. If ÐBDC = 42°, then what is ÐACB 63. The diameter of a circle with centre at C is 50 cm. CP is
equal to? [2011-II] a radial segment of the circle. AB is a chord perpendicular
(a) 42° (b) 45° to CP and passes through P. CP produced intersects the
(c) 48° (d) 58° circle at D. If DP = 18 cm, then what is the length of AB?
56. What is the number of circles which pass through three [2013-II]
given points not in a straight line? [2011-II] (a) 24 cm (b) 32 cm
(a) 0 (b) 1 (c) 40 cm (d) 48 cm
(c) 2 (d) 3 64. A regular hexagon is inscribed in a circle of radius 5 cm.
57. Let AB be a line segment such that a point P on the locus If x is the area inside the circle but outside the regular
satisfies AB2 – AP2 – BP2 = 0. What is the locus? hexagon, then which one of the following is correct?
[2012-I] [2013-II]
(a) The straight line AB itself (a) 13 cm2 < x< 15 cm2 (b) 15 cm2 < x< 17 cm2
(b) The perpendicular bisector of AB (c) 17 cm2 < x< 19 cm2 (d) 19 cm2 < x< 21 cm2
(c) A straight line parallel to AB 65. Consider the following statements
(d) A circle
I. The perpendicular bisector of a chord of a circle does
58. A circular ring with centre O is kept in the vertical
not pass through the centre of the circle.
position by two weightless thin strings TP and TQ
attached to the ring at P and Q. The line OT meets the II. The angle in a semi-circle is a right angle.
ring at E whereas a tangential string at E meets TP and Which of the statements given above is/are correct?
TQ at A and B, respectively. If the radius of the ring is 5 (a) Only I (b) Only II
cm and OT = 13 cm, then what is the length of AB? (c) Both I and II (d) Neither I nor II
[2012-I] 66. ABC is an equilateral triangle inscribed in a circle. D is
(a) 10/3 cm (b) 20/3 cm any point on the arc BC. What is ÐADB equal to?
(c) 10 cm (d) 40/3 cm [2013-II]
59. The locus of the mid-points of all equal chords in a (a) 90° (b) 60°
circle is [2012-I] (c) 45° (d) None of the above
(a) The circumference of the circle concentric with the 67. A circle of radius 10 cm has an equilateral triangle
given circle and having radius equal to the length of inscribed in it. The length of the perpendicular drawn
the chords. from the centre to any side of the triangle is [2014-I]
(b) The circumference of the circle concentric with the
given circle and having radius equal to the distance (a) 2.5 3 cm (b) 5 3 cm
of the chords from the centre.
(c) 10 3 cm (d) None of these
(c) The circumference of the circle concentric with the
given circle and having radius equal to half of the 68. AB and CD are two chords of a circle meeting externally
radius of the given circle. at P. Then, which of the following is/are correct?
(d) The circumference of the circle concentric with the I. PA × PD = PC × PB
given circle and having radius equal to half of the II. DPAC and DPDB are similar.
distance of the chords from the centre. Select the correct answer using the codes given below.
60. Consider a circle with centre at O and radius r. Points A [2014-I]
and B lie on its circumference and a point M lies outside (a) Only I (b) Only II
of it such that M, A and O lie on the same straight line. (c) Both I and II (d) Neither I nor II
Then, the ratio of MA to MB is [2013-I] 69. In a DABC, AB = BC = CA. The ratio of the radius of the
(a) equal to 1 (b) equal to r circumcircle to that of the incircle is [2014-I]
(c) greater than 1 (d) less than 1 (a) 2 : 1 (b) 3 : 1
61. Consider the following statements (c) 3 : 2 (d) None of these
I. The tangent of a circle is a line that meets the circle 70. If the chord of an arc of a circle is of length x, the height
in one and only one point. of the arc is y and the radius of the circle is z.
II. The tangent of a circle at the end point of the diameter Then, which one of the following is correct ? [2014-II]
is perpendicular to the diameter. (a) y(2z – y) = x2 (b) y(2z – y) = 4x2
2
Which of the above statements is/are correct? (c) 2y(2z – y) = x (d) 4y(2z – y) = x2
[2013-I] 71. A railroad curve is to be laid on a circle. What radius
(a) Only I (b) Only II (approximate) should be used, if the track is to change
(c) Both I and II (d) Neither I nor II direction by 25° in a distance of 120 m? [2014-II]
62. Consider the following statements in respect of two (a) 300 m (b) 280 m
chords XY and ZT of a circle intersecting at P. (c) 275 m (d) 264 m
Circle A-357
72. If the radius of a circle is increased by 6%, then its area 80.
will increase by [2014-II] A C
(a) 6% (b) 9%
(c) 12.36% (d) 16.64% E P
73. Two circles, each of radius r, with centres P and Q, are
such that each circle passes through the centre of the
B D
other circle. Then the area common to the circles is less
than one-third of the sum of the areas of the two circles From an external point P tangents PA and PB are drawn to
by [2015-I] the circle as shown in the above figure. CD is the tangent to
the circle at E. If AP = 16 cm, then the perimeter of the
3r 2 3r 2 triangle PCD is equal to [2015-II]
(a) (b)
4 3 (a) 24 cm (b) 28cm
(c) 30 cm (d) 32 cm
3r 2
(c) (d) 3r 2 81. Chord CD intersects the diameter AB of a circle at right
2 angle at a point P in the ratio 1 : 2. If diameter of circle is D,
74. Three equal circles each of diameter d are drawn on a then CD is equal to [2015-II]
plane in such a way that each circle touches the other
two circles. A big circle is drawn in such a manner that 2d 2d
(a) (b)
it touches each of the small circles internally. The area of 3 3
the big circle is [2015-I]
2 2d 2 3d
(a) pd 2 (b) pd 2 (2 – 3)2 (c) (d)
3 3
pd 2 ( 3 + 1)2 pd 2 ( 3 + 2)2 82. In a circle of radius 2 units, a diameter AB intersects a chord
(c) (d) of length 2 units perpendicularly at P. If AP > BP, then AP
2 12 is equal to [2016-I]
75. If the angle between the radii of a circle is 130°, then the angle
between the tangents at the ends of the radii is [2015-I]
(a) 90° (b) 70°
(a) ( 2 + 5) units (b) ( 2 + 3) units
76.
(c) 50° (d) 40°
Out of two concentric circles, the diameter of the outer
(c) ( 2 + 2 ) units (d) 3 units
circle is 26 cm and the chord MN of length 24cm is 83. A truck moves along a circular path and describes 100 m
tangent to the inner circle. The radius of the inner circle when it has traced out 36° at the centre. The radius of the
is [2015-I] circle is equal to [2016-I]
(a) 5 cm (b) 6 cm 100 250
(a) m (b) m
(c) 8 cm (d) 10 cm p p
77. The two adjacent sides of a cyclic quadrilateral are 2 cm and
5 cm and the angle between them is 60°. If the third side is 3 500 600
(c) m (d) m
cm, then the fourth side is of length [2015-II] p p
(a) 2 cm (b) 3 cm 84. A tangent is drawn from an external point D to a circle of
(c) 4 cm (d) 5 cm radius 3 units at P such that DP = 4 units. If O is the centre
78. From a rectangular sheet of sides 18 cm and 14 cm, a of the circle, the the sine of the angle ODP is [2016-I]
semicircular portion with smaller side as diameter is taken (a) 4/5 (b) 3/4
out. Then the area of the remaining sheet will be [2015-II] (c) 3/5 (d) 1/2
(a) 98 cm2 (b) 100 cm2 85. Consider a circle with centre at O and radius 7 cm. Let OR be
(c) 108 cm2 (d) 175 cm2 a chord of length 2 cm and let P be the midpoint of QR. Let
CD be another chord of the circle passing through P such
79. that ÐCPQ is acute. If M is the midpoint of CD and MP =
r o
24 cm, then which of the following statements are correct?
1. If CP = 135°
A M B 2. If CP = m cm and PD = n cm, then m and n are the roots
of the quadratic equation x2 – 10 x + 1 = 0
AB is a line segment of length 2a, with M as mid–point. 3. The ratio of the area of triangle OPR to the area of
Semicircles are drawn on one side with AM, MB and AB as triangle OMP is 1: 2 2.
diameter as shown in the above figure. A circle with centre Select the correct answer using the code given below.
O and radius r is drawn such that this circle touches all the (a) 1 and 2 only (b) 2 and 2 only
three semicircles. The value of r is [2015-II] (c) 1 and 3 only (d) 1, 2 and 3
86. Consider a circle with centre at C Let OP, OQ denote
2a a
(a) (b) respectively the tangents to the circle drawn from a point O
3 2 outside the circle. Let R be a point on OP and S be a point
on OQ such that OR × SQ = OS × RP. Which of the following
a a statement is/are correct? [2016-I]
(c) (d)
3 4
EBD_7367
358
A- Circle
1. If X is the circle with centre at O and radius OR, and Y 93. If two tangents inclined at an angle 60° are drawn to a circle
is the circle with centre at O and radius OS, then X = Y. of radius 3 cm, then what is the length of each tangent ?
2. Ð POC + Ð QCO = 90° [2016-II]
Select the correct answer using the code given below. (a) 3 3cm (b) 3cm
(a) 1 only (b) 2 only
(c) Both 1 and 2 (d) Neither 1 nor 2 (c) 6 cm (d) 2 2cm
87. A circular path is made from two concentric circular rings in 94. The diameter of the front wheel of an engine is 2x cm and
such a way that the smaller ring when allowed to roll over that of rear wheel is 2y cm. To cover the same distance,
the circumference of the bigger ring, it takes three full what is the number of times the rear wheel revolves when
revolutions. If the area of the pathway is equal to n times the front wheel revolves n times ? [2016-II]
the area of the smaller ring, then n is equal to [2016-I] n ny
(a) 4 (b) 6 (a) xy (b)
(c) 8 (d) 10 x
88. nx xy
(c) y
(d)
C n
95. Three circles each of radius 3.5 cm touch one another. The
B area subtended between them is [2017-I]
(a) 6( 3p - 2) square units
A (b) 6(2p - 3p) square units
49
A circle of 3 m radius is divided into three areas by (c) (2 3 - p) square units
semicircles of radii 1m and 2m as shown in the figure above. 8
The ratio of the three areas A, B and C will be [2016-I] 49
(d) ( 3 - p) square units
8
(a) 2 : 3 : 2 (b) 1 : 1 : 1 96. Two parallel chords of a circle whose diameter is 13 cm are
(c) 4 : 3 : 4 (d) 1 : 2 : 1 respectively 5 cm and 12 cm in length. If both the chords are
89. on the same side of the diameter, then the distance between
these chords is [2017-I]
(a) 5.5 cm (b) 5 cm
C (c) 3.5 cm (d) 3 cm
A D 97. A copper wire when bent in the form of a square encloses
an area of 121 cm2. If the same wire is bent in the form of a
circle, it encloses an area equal to [2017-I]
(a) 121 cm2 (b) 144 cm2
AD is the diameter of a circle with area 707 m2 and AB = BC (c) 154 cm2 (d) 168 cm2
= CD as shown in the figure above. All curves inside the 98. The radius of a circle is increased so that it's circumference
circle are semicircles with their diameters on AD. What is increases by 15%. The area of the circle will increase by
the cost of levelling the shaded region at the rate of 63 per [2017-I]
square metre? [2016-I] (a) 31.25% (b) 32.25%
(a) ` 29,700 (b) ` 22,400 (c) 33.25% (d) 34.25%
(c) ` 14,847 (d) None of the above 99. A field is divided into four regions as shown in the given
90. Two circles touch externally and sum of their areas is 130 p figure. What is the area of the field in square metres ?
cm2 and the distance between their centres is 14 cm. What [2017-I]
is the difference in the radii of the circles ? [2016-II]
(a) 5 cm (b) 6 cm 2m 2m
(c) 7 cm (d) 8 cm 1m
2m

91. In a circle of radius 3 units, a diameter AB, intersects a


chord of length 2 units perpendicularly at P. If AP > BP, then 2m
3m
what'is the ratio of AP to BP ? [2016-II]
(a) 3 + 10 : 33 - 10 3m
(b) 3+ 8 :3- 8 3m
(c) 3+ 3 :3- 3
(d) 3 : 3
92. What is the number of rounds that a wheel of diameter
5 3 3
m will make in travelling of 7 km ? [2016-II] (a) 6+ 5 (b) 5 + 3
11 4 2
(a) 3300 (b) 3500 3
(c) 4400 (d) 4900 (c) 9+ 15 (d) 7+2 2
4
Circle A-359
100. In the figure given below, D is the diameter of each circle. 108. The radii of two circles are 4.5 cm and 3.5 cm respectively.
What is the diameter of the shaded circle ? [2017-I] The distance between the centres of the circles is 10 cm.
What is the length of the transverse common tangent?
[2018-1]
(a) 4 cm (b) 5 cm
(c) 6 cm (d) 7 cm
109. The locus of the mid-points of the radii of length 16 cm of a
circle is [2018-1]
(a) A concentric circle of radius 8 cm
(b) A concentric circle of radius 16 cm
(c) The diameter of the circle
(d) A straight line passing through the centre of the circle
110. What is the area of the region bounded externally by a
(a) D( 2 - 1) (b) D( 2 + 1) square of side of length ‘a’ and internally by a circle passing
through the four corners of the square?
(c) D( 2 + 2) (d) D(2 - 2)
[2018-1]
101. A hollow cube is formed by joining six identical squares. A (a) (p – 1) a2 square units
rectangular cello tape of length 4 cm and breadth
0.5 cm is used for joining each pair of edges. What is the (b)
( p - 1) a 2square units
total area of cello tape used? [2017-II] 2
(a) 12 square cm (b) 24 square cm (c) (p – 2) a2 square units
(c) 36 square cm (d) 48 square cm
(d)
( p - 2) a 2square units
102. Two straight lines AB and AC include an angle. A circle is
2
drawn in this angle which touches both these lines. One 111. In the figure given below, XA and XB are two tangents to a
more circle is drawn which touches both these lines as well circle. If ÐAXB = 50° and AC is parallel to XB, then what is
as the previous circle. If the area of the bigger circle is 9 ÐACB equal to? [2018-1]
times the area of the smaller circle, then what must be the A
angle A? [2017-II]
(a) 45° (b) 60°
(c) 75° (d) 90° C 50° X
103. AB and CD are parallel chords of a circle 3 cm apart.
If AB = 4 cm, CD = 10 cm, then what is the radius of the
circle? [2017-II] B
(a) 7 cm (b) 19 cm (a) 70° (b) 65°
(c) 60° (d) 55°
(c) 29 cm (d) 14 cm 112. In the figure given below, SPT is a tangent to the circle at P
104. The diagonals of a cyclic quadrilateral ABCD intersect at P and O is the centre of the circle. If ÐQPT = a, then what is
and the area of the triangle APB is 24 square cm. If AB = 8 ÐPOQ equal to? [2018-1]
cm and CD = 5 cm, then what is the area of the triangle CPD?
[2017-II]
(a) 24 square cm (b) 15 square cm O
(c) 12.5 square cm (d) 9.375 square cm
105. The distance between the centres of two circles having Q
radii 9 cm and 4 cm is 13 cm. What is the length of the direct a
common tangent of these circles? [2017-II] S P T
(a) 12 cm (b) 11 cm (a) a (b) 2a
(c) 10 cm (d) 9.5 cm (c) 90° – a (d) 180° – 2a
106. An arc of a circle subtends an angle p at the centre. 113. In the figure given below, two equal chords cut at point P.
If the length of the arc is 22 cm, then what is the radius of If AB = CD = 10 cm, OC = 13 cm (O is the centre of the circle)
the circle? [2017-II] and PB = 3 cm, then what is the length of OP?
[2018-1]
22
(Take p = )
7
(a) 5 cm (b) 7 cm O
(c) 9 cm (d) 11 cm
107. Two equal circular regions of greatest possible area are cut
off from a given circular sheet of area A. What is the A P C
remaining area of the sheet? [2018-1] D B
(a) A/2 (b) A/3 (a) 5 cm (b) 6 cm
(c) 3A/5 (d) 2A/5
(c) 2 29 cm (d) 2 37 cm
EBD_7367
360
A- Circle

HINTS & SOLUTIONS


1. (d) Given, ÐAOB = 100° ÐADC + ÐABC = 180° (cyclic quadrilateral)
ÐABC = 180° – 81° = 99°
Now in DABQ
ÐABQ + ÐBAQ + Ð AQB = 180°
\ ÐAQB = 180° – (99° + 59°)
= 180° – 158° = 22°
x y z
O 4. (b) Given = = = a (say)
3 4 5
100 \ x = 3a, y = 4a and z = 5a

A B A

C
\ Reflex ÐAOB = 360° – ÐAOB D
B
= 360° – 100° = 260°
4a C 3a
5a
Reflex Ð AOB 260° P
\ ÐACB = = Q
2 2
= 130° Since, ÐDCQ = ÐBCP = 3a
2. (c) By using theorem, (vertically opposite angle)
In DDCQ, ÐCDQ = 180° – (3a + 5a)=180° – 8a by
A properation of cyclic quadrilateral,
P ÐQDC= ÐCBA = 180° – 8a Þ ÐPBC = 8a
In DPBC,
D ÐP + ÐB + ÐC = 180°
180°
\ 4a + 8a + 3a = 180° Þ a = Þ a = 12°
15
\ x = 36° , y = 48°, z = 60°
B C 5. (b) By using theorem,
2
(PT)2 = PA × PB
æ AC ö 1 2
AB × AP = AD2 = ç ÷ = ( AC ) 36
è 2 ø 4 \ (6)2 = 4 × (4 + AB) Þ = 4 + AB
4
1 \ AB = 5 cm
Þ AB ×AP = ( AB )2 (Q AC = AB given) 6. (a) Since, DADB is a right angled triangle at D.
4
Þ AB = 4AP C
3. (c) Given ÐPAQ = 59°
and ÐAPD = 40°
In DADP D

P 35°
75°
A B
40° \ ÐDAB = 180° – (90° + 75°)
Þ ÐDAB = 15°
B Also, ABCD is cyclic quadrilateral.
C \ ÐCAB + ÐBDC = 180°
Þ ÐBDC = 180° – (35° + 15°) = 130°
7. (d) Reflex ÐAOC = 360° – 100° = 260°
59° 1
A Q ÐABC = Reflex ÐAOC
D 2
1
ÐADP = 180° – 59° – 40° = 81° ÐABC = × 260° = 130°
2
Circle A- 361

1 75° \ ÐAOB = 2 ÐAPB


8. (b) ÐAQP = ´Ð AOP = = 37.5°
2 2 60°
Þ ÐAPB =
= 30°
9. (d) Since, XT = YT 2
13. (b) OT = 13 cm, OE = 5 cm, = OP (radius)
X 90° ET = 13 – 5 = 8 cm
T
50° 80°
90° P
A

m
O

5c
50°
90° E q
O T
Z Y 5 cm
M

B
\ ÐTXY = ÐTYX = 50° Q
Also, OX is perpendicular to XT.
\ ÐOXT = 90° In DOPT, 132 – 52 = PT2 Þ PT = 12
\ ÐOXY = 90° – 50° = 40° Let ÐATE = q
Also, OM is perpendicular to ZY.
OP 5
In DXMY, In DOPT, tan q = = ... (i)
ÐMXY + ÐXYM + ÐXMY = 180° PT 12
\ 40° + ÐXYM + 90° = 180° AE AE
Þ ÐXYM = 50° Now in DATE, tan q = = [from Eq. (i)]
Also, by property of circle, ET 8
ÐTXY = ÐXZY = 50° 5 10
In DXYZ, Þ AE = 8 tanq = 8× =
ÐX + ÐY + ÐZ = 180° 12 3
\ ÐX = 180° – 50° – 50° = 80° 10 20
\ AB = 2AE = 2 ´ = cm
10. (c) P 3 3
14. (a) r1 + r2 = 4
r2 + r3 = 6
r3 + r1 = 8
o
A B M

A
Q
r2 r2
AM = 5 cm
BM = 3 cm
AB = 5 – 3 = 2 cm r1 r3
PQ Bisect AB
\ OA = OB = 1 cm
B C
In DAOP r1 r3
\ OP = AP 2 - OA2 = 52 - 12
= 25 - 1 = 24 = 2 6
\ PQ = 2 × OP = 2 × 2 6 = 4 6 cm (r1 + r2) + (r2 + r3) + (r3 + r1)
11. (a) We know that, the triangle of same segment of a circle =4+6+8
makes an equal angles. 2(r1 + r2 + r3) = 18
\ ÐXBY = ÐXAY = 45° 18
In DBXY, ÐBXY + ÐXBY + ÐBYX = 180° r1 + r2 + r3 = =9
Þ 50° + 45° + ÐBYX = 180° (Q ÐBXY = 50°) 2
Þ ÐBYX = 180° – 95° = 85° \ r1 + r2 + r3 = 9
12. (a) OA = OB = AB (given) 15. (b) Two circles have three points in common only when
So, DAOB is an equilateral triangle. C1 and C2 are same circle.
Þ ÐAOB = 60° 16. (b) From a point on the circle only one tangent can be
We know that, the angle subtended at the centre of drawn to a circle.
circle is twice the angle subtended at circumference of
the circle.
EBD_7367
362
A- Circle

17. (a) Given OA = 20 cm Here, OE is a segment, which makes a DOFE and


OB = 15 cm and OP = 12 cm DOCE.
Therefore, ÐOCE = ÐEFO = 20°
20. (d) Given that AB = BC = CD, also since AB is the radius
then AB = AC = AD = radius, so we have that: AB =
BC = CD = AC = AD, so basically we have two
O equilaterial triangles ABC and ACD with common base
c m
20 of AC (ABC and ACD are mirror images of each other).
cm
12 cm Line segment BD cuts the angle ABC in half and since
A B C D
15
P all angles in equileteral traiangle equal to 60 degrees
then x = 60/2 = 30 degrees.
B
AP = AO 2 - OP 2
x
= 202 - 122
= 400 - 144 A C
= 256 = 16 cm
BP = 152 - 122 D
= 225 - 144 = 81 21. (b) Given BC = 38 cm
= 9 cm QB = 27 cm
\ AB = AP – BP = 16 – 9 DC = 25 cm
= 7 cm AD ^ DC
18. (d)
B B

c m
27
20° 30°
O 30° Q
20° A
C

38 cm
A
O
T
R
Join OB. OA = OB = OC
Then,
ÐOAB = ÐOBA = 20°
ÐOCB = ÐOBC = 30° D P C
ÐABC = 50° 25 m
1 We know that tangents are always equal, when they
We know that ÐABC = Ð AOC
2 drawn to the circle from a point outside the circle.
\ ÐAOC = 2 ÐABC = 2 × 50° = 100° \ BQ = BR = 27 cm
19. (b) The Given, EC = ED RC = BC – BR = 38 – 27 = 11 cm
RC = PC = 11 cm
F
DC = 25 cm
C DP = DC – PC = 25 – 11 = 14 cm
20° 55° DP = OT = OP
\ Radius of the circle = 14 cm
A B 22. (d) The sum of opposite angles in cyclic quadrilateral is
O E
110° always 180°.
\ ÐACQ + ÐAPQ = 180°
D 75° + ÐAPQ = 180°
\ ÐAPQ = 180° – 75° = 105°
Þ ÐEDC = ÐECD = 35°
ÐACQ + ÐQCR = 180° ( Q Straight line)
Since, ÐOCD = 55°
75° + ÐQCR = 180°
Then, ÐOCE = 20°
ÐQCR = 180° – 75° = 105°
By using then theorem that triangle on the same
ÐCQR = 180° – 105° – 30° = 45°
segment of a circle makes as equal angles.
Since, ÐAPQ + ÐBPQ = 180° (Straight line)
Circle A-363
\ 105° + ÐBPQ = 180° 25. (b) We know that, the sum of opposite angles of a cyclic
ÐBPQ = 75° quadrilateral is always 180°.
In DBPQ ÐB + ÐP + ÐQ = 180° A
ÐB + 75° + 45° = 180°
Þ ÐB = 60° \ ÐCBA = 60°
23. (b) Given ÐOBD + ÐODB = ÐCBD + ÐCDB

100°
B D
P

C
\ ÐB + ÐD = 180°
Þ 100 + ÐD = 180°
q q Þ ÐD = 80°
B q2 \ ÐACP = ÐPAC = 80°
q1
(by theorem of alternate interior segment)
In DPAC,
ÐP + ÐPAC + ÐPCA = 180°
Let ÐOBD = ÐODB = q Þ ÐP + 80° + 80° = 180°
and ÐDBC = q1, ÐBDC = q2 Þ ÐP = 180° – 160° = 20°
\ q+q = q1 + q2 ... (i) 26. (b) Given , ÐBAX = 70° and ÐBAQ = 40°
Þ 2q = q1 + q2
E
\ ÐBOD = 180° – 2q
360° - (180° - 2q)
Þ ÐBCD = B
2
(by properties of circle)
O
Þ 180° – (q1 + q2) = 90° + q
Þ 180° – 2q = 90° + q 40°
Þ 90° = 3q Q
70°
Þ q = 30°
Y A X
\ ÐBOD = 120°
ÐBAD = 60° ÐQAX = 70° – 40° = 30°
24. (d) The tangents drawn from an outer point on a circle are \ ÐEAX = 90°
always equal = ÐCBA. Þ ÐEAB = 90° – 70° = 20°
Therefore, ÐCAB = ÐCBA Since, AQBE is a cyclic quadrilateral.
\ ÐEAQ + ÐEBQ = 180°
C Þ ÐEBQ = 180° – 60° = 120°
But ÐEBA = 90°
45° \ ÐABQ = 120° – 90° = 30°
27. (c) By using theorem, we have
B
x m
A x P
B 5c
P
m
3c
A x C
Q 2 cm Q
AB × AP= AC × AQ Þ 8 × 3 = (2 + x) × 2
\ 45° + x + x = 180°
Þ 2x = 180° – 45° 8´ 3
Þ = 2 + x Þ x = 10 cm
2
1° 28. (d) Since, AB = CB
Þ x = 67
2 D
ÐAQP = Ðx = ÐBQP
1° 80°
= 67
2 O
(alternate interior segments properties)
Þ ÐAQB = ÐAQP + ÐBQP A C
40° 40°
1° 1° 100°
= 67 + 67 = 135°
2 2 B
EBD_7367
364
A- Circle

\ ÐCAB = ÐACB = 40° In D MCB,


Þ ÐABC = 180° – 2 (40°) = 100° ÐC + ÐB + ÐM = 180°
We know that, in cyclic quadrilateral, the sum of Þ 23° + ÐB + 90° = 180°
opposite angles is 180°. \ ÐB = 67°
\ ÐB + ÐD = 180° 32. (d) ÐBOD = 180° – 106° = 74°
Þ ÐD = 180° – 100° = 80°
29. (a) Given, PO = 10 cm, radius OT = 6 cm B D
and PB = 5 cm
In DOTP,
C
C 106°
B O

P O
A
Since, ÐBOD is an angle made by arc BD on centre.
T
Here, ÐBCD is an angle made by arc BD on
(OP)2 = (PT)2 + (OT)2 circumference.
Þ (10)2 = (PT)2 + 62
Þ PT = 8 cm 1
\ ÐBCD = ´Ð BOD
From properties of circle, 2
(PT)2 = PB × PC
1
Þ 82 = 5 × (BC + PB) =´ 74° = 37°
Þ 64 = 5 (BC + 5) Þ 5BC = 39 2
\ BC = 7.8 cm 33. (d) Infinite number of circles passing through a given pair
30. (c) x = ÐVTR = 52° of points.
x + z = 180° 34. (d) It is clear from the figure that none of the option is correct.
(since, PTMR is a cyclic quadrilateral)
Þ 52° + z = 180° B
Þ z = 128° D

Q P
C
P R A
x 1
O z M 35. (a) A line perpendicular to the given line, passing through
the given point is the required locus.
36. (b) In DOED,
y ° (OD)2 = (DE)2 + (EO)2
52
U V C
T
2 cm
In DPTR, PT = TR (given)
3 cm

x = Ð1 = 52°
O
ÐPTU = Ð1 = 52° Þ ÐQTU = y + 52° A B
E 5 cm
Þ 90° = y + 52° Þ y = 38°
\ x + y + z = 52° + 38° + 128° = 218°
31. (c) Since, angle subtend on the circumference is half of
the angle subtend on centre. D
1 Þ (5)2 = (DE)2 + (3)2
\ ÐACB = Ð AOB Þ (DE)2 = 25 – 9 = 16
2
\ DE = 4 cm
1 37. (c) When two tangents drawn from an external point to a
= ´ 46° = 23°
2 circle, the length of the tangent are equal.
Q
D C

O
P R
46° M C
90° 90°
A B A S B
Circle 365
A-

AS = AP ... (i) Also, OP = OR


BS = BR ... (ii) \ ÐOPR = 45°
CQ = CR ... (iii)
DP = DQ ... (iv) R T
Adding (i), (ii), (iii) and (iv), we get. P 45°
AS + BS + CQ + DQ = AP + BR + DP + CR

45°
= AB + CD = AD + BC
44°
\ Option (c) is correct.
38. (c) Here, r1 = 9 cm and r2 = 4 cm O M S
r1 + r2 = 9 + 4 = 13 cm
and r1 – r2 = 9 – 4 = 5cm
Q
Also, d = 13 cm
Here, d = r1 + r2 In DORS,
= 13 cm
Hence, two circles touch each other externally, so three 180° - 44°
OR = OS Þ ÐORS = ÐOSR = = 68°
total no of common tangents are three. 2
39. (a) If a secant to a circle intersect circle at points A and \ ÐMRS = 68° – 45° = 23°
B PC is a tangent to circle, then Þ ÐPRS = 90° + 23° = 113°
By properties of cyclic quadrilateral.
B ÐPRS + ÐPQS = 180°
Þ ÐPQS = 180°– 113° = 67°
In DPTQ,
ÐQPT + ÐPQT + ÐPTQ = 180°
A Þ ÐPTQ = 180° – 45° – 67° = 68°
1
42. (b) Since, ÐADB = ÐAOB = 50°
2
P C E
PC2 = PA × PB
which is equivalent to area of rectangle with PA and PB
as sides is equal to the area of square with PC as side.
40. (b) Given, ÐBAD = 60°, ÐADC = 105° D C
in cycle quadrilateral ABCD, P
P 30° O
A 100° B

D C
4 3
In DDPA, ÐDAP + ÐADP + ÐDPA = 180°
Þ 30° + 50° + ÐDPA= 180° Þ ÐDPA = 100°
Also, DPB be a straight line.
\ ÐDPA + ÐAPB = 180°
1 2 Þ ÐAPB = 180° – 100° = 80°
A B 43. (d) PC = PB (radii of circle)
A
Ð1 + Ð3 = 180°
Þ Ð3 = 180° – 60° = 120°
C
Now,ÐBCD + ÐDCP = 180° (straight line)
Þ ÐDCP = 180° – 120° = 60°
and, ÐADC + ÐCDP = 180° (straight line) P
Þ 105° + ÐCDP = 180° Þ ÐCDP = 75°
35°

Now in DCPD,
ÐDCP + ÐCDP + ÐDPC = 180°
Þ 60° + 75° + ÐDPC = 180° B
\ ÐDPC = 180° – 135° = 45° Þ ÐPBC = ÐPCB
41. (d) Since, OR is a bisector of ÐPRQ. (angles opposite to equal sides)
\ ÐPRO = ÐORQ = 45° Þ ÐPCB = 35°
and ÐACB = 90° (angle in semi-circle)
EBD_7367
366
A- Circle

Þ ÐPCA + ÐPCB = 90° 47. (c)


Þ ÐPCA = 90° – 35° = 55°
44. (a) Only one circle passing through all the vertices of a P
given triangle. 10 cm
45. (c) I. It is clear from the figure that points Q, S and R in 3.5 O¢
a straight line. r2 r1
O
4.5 cm
Q
S Q
L
P Length of common tangent,
R
It is true
= d 2 - (r1 + r2 )2
II. Since, PQ is parealled to EF.
\ ÐPDE = ÐDEF (alternate angle)
= 102 - (4.5 + 3.5)2
= 100 - 64 = 36
D
P Q = 6 cm
48. (c) In DABC,
A

5c
5c

m
E F
X
C B
5 5
Also, ÐPDE = ÐEFD
2 2
(angle in the alternate segments of chord ED)
\ ÐDEF = ÐDFE Y
Therefore, DDEF is an isosceles triangle.
5 5
BX = cm, CX = cm
46. (d) 2 2

3 5 3
and AX =´5 = cm
2 2
AY and BC are the chord of circle.
\ AX × XY = BX × XC

5 3 5 5
Þ . XY = ×
2 2 2
Let O is centre of big circle, and O' is centre of smaller 5
circle. Both are touch internally each other. `Þ XY =
OA = 6 cm O'A = 4cm 2 3
Here PR is longest chord of big circle
æ5 3 5 ö 5 3
PR \ AX × AY = ç 2 + ÷ ´ 2 = 25 cm
2
P.S = è 2 3ø
2
OS = AS – OA 49. (d)
= 8 – 6 = 2 cm.
B
In DPSO
(PS)2 + (OS)2 = (OP)2
Þ (PS)2 + (2)2 = (6)2 D
PS = 36 - 4 = 32 = 4 2
Now, P
PR
PS =
2
PR = 2 × PS C
A
= 2 ×4 2 = 8 2 It is clear from the figure that ACBD is a quadrilateral.
Circle A-367
50. (b) The locus of P is a circle concentric with C.
P

60°

C
Hence, both statements are individually true but
neither statements implies of each other.
54. (c) OA = 3 cm and AC = 3 cm ... (Given)
51. (b) 3
AM = cm
2
B
O

d/2
O
The locus of the centre of the front wheel of the q
bicycle is a line parallel to the path of the wheel of the A M C
bicycle at a height d/2 cm.
52. (c) Given, AC = 3 r
In DOAM,
3 AM 3/ 2
AB = r sin q= =
2 OA 3
1
Þ sin q= Þ q = 30°
2
\ ÐAOC= 2q = 60°
O 1
ÐABC= Ð AOC (By property of circle)
r 2
60°
A C = = 30°
B 2
55. (c) Q ÐADC = 90° (angle in semi-circle is a right angle)
In DOAB,
(OB)2 = (OA)2 – (AB)2 D C
2
2 æ 3 ö
= r - çç r÷
÷
è 2 ø O
B
2 3 2 r2
= r - r =
4 4 A
r ÐADB = ÐADC – ÐBDC = 90° – 42° = 48°
Þ OB =
2 ÐADB = ÐACB = 48°
53. (d) I. It is true that opposite angles of a cyclic quadrilateral 56. (b) The total number of circles passes through three
are supplementary. non-collinear points is one.
57. (d) Given,
AB2 – AP2– BP2 = 0
P

A B
II. It is also true that the angle subtend by an arc at the
centre is double the angle subtended by it at any point
on the remaining part of the circle.

AB2 = AP2 + BP2


EBD_7367
368
A- Circle

Angle in a semi circle is a right angle.


Thus, this is a equation of a circle.
Hence, locus of point P is a circle. A O
M L
58. (b) In DOQT
N
OT2 = OQ2 + TQ2 B
Þ (13)2 = (5)2 + (TQ)2
LM × AM = BM × NM
T MA MN
Þ = <1
MB LM
61. (c) By definition of tangent.
12 – x
A tangent to a circle is a straight line that touches the
circle at a single point. Also, tangent at the end points
13 cm of a diameter of a circle is perpendicular to the
x B diameter.
A E So, both statements are correct.
x P
5m A B
P Q
O 5m O

62. (c) When two chords of a circle are intersect internally,


2
Þ TQ = 169 – 25 = 144 Þ TQ = 12 cm then they are divided in a proportion.
Then, in DTEB, Y Z
TB2 = EB2 + TE2 P
\ (EB = BQ) (Beacuse they are tangent) T X
O
Þ (12 – x)2 = BQ2 + TE2
Þ 144 + x2 – 24x = x2 + (8)2
Þ 144 + x2 – 24x = x2 + 64 Þ 24x = 80 i.e., PX . PY = PZ . PT
20 10 In DPXZ and DPTY,
Þ x= = cm ÐZPX = ÐYPT (vertically opposite angles)
6 3
ÐPZX = ÐPYT (angle in same segment)
10 20 ÐPXZ = ÐPTY (angles in same segment)
\ AB = 2EB = 2x = 2 ´ = cm
3 3 \ DPXZ ~ DPTY
59. (b) Hence, the both statements are correct.
63. (d) CD = 25 cm d = 50 cm
P E Q d 50
PD = 18 cm r = = = 25 cm
2 2

H F

C
m
R G S 2 5c
7 cm
A P 18 cm B
Here PQ = QS = SR = RP
Since, E, F, G and H are the mid-point of the chords.
D
The locus obtained is the circumference of the circle \ CP = CD – PD
concentric with the given circle and having radius
= 25 – 18 = 7 cm
equal to the distance of the chords from the centre.
60. (d) Since, secants ÐA and BN are intersecting at an AP = (25)2 - (7)2 = 625 - 49
exterior point M, then
= 576 = 24 cm
\ AB = 2AP = 24 × 2 = 48 cm
Circle A-369
64. (a) OB = OA = radius 68. (d) AB and CD are chords when produced meet externally
at P.
æ 360° ö
Also, ÐAOB = 60° ç = 60° ÷ B
è 6 ø A
and ÐOAB = ÐOBA = 60°
P
A
D C
F B
60° m
\ AP × BP = CP × DP
5c
O
Now, as AC || BD and DPAC is not similar to DPDB
E C
69. (a) In DABC
D
A
So, DAOB is an equilateral triangle.
Then, AB = 5 cm
So, Area, x = Area of circle – Area of hexagon O

3 3(a)2 B C
= pr 2 - D
2

22 3 3 AB = BC = CA ... (given)
=´ (5)2 - ´ (5)2 \ DABC is an equilateral triangle.
7 2
(Q r = a = 5) Side
Now radius of incircle =
= 78.57 – 64.95 = 13.62 cm2 2 3
65. (b) The perpendicular bisector of the chord of a circle
always pass through the centre. So, statement I is AB
=
wrong. 2 3
The angle in a semi-circle is a right angle. So,
Side AB
Statement II is correct. Radius of circumcircle = =
66. (b) 3 3

A AB / 3 2
\ Required Ratio = =
AB / 2 3 1
=2:1
70. (d) Let O be the centre of circle and AB be the chord of
an arc.
Given that
B C Length of chord AB = x, Radius of circle OA = z
D and height of an arc MN = y
ÐADB= ÐACB = 60° (angles in the same segment are equal)
2
67. (d) Circumradius = ´ Height
3 O
Z
A M B
H

x
10 ´ 3 AM = MB = Q OM ^ AB
\ Height = = 15 cm 2
2
and OM = ON – MN = z – y
So, length of perpendicular drawn from center = 15 –
10 = 5 cm, [Q ON = z (radius) and MN = y]
In DOMA, OA2 = OM2 + AM2
[by Pythagoras theorem]
EBD_7367
370
A- Circle

2
2 æ xö
2 74. (d)
Þ z = (z – y) + ç ÷
è2ø E
x 2 2r
Þ z2 = z2 + y2 – 2yz + B A
4
O
2
x
Þ 2yz – y2 =
4
Þ 4 (2yz – y2) = x2 C
Þ 4y (2z – y) = x2
2 pr OE is the radius of big circle (R)
71. (c) From figure, arc length PQ = ´q
360°
2r
OE = OB + BE Þ OB =
3

2r 2r + 3r r (2 + 3)
O \ OE = +r = =
3 3 3
r 25° Q Area of big circle = pR2
Q P 2 2
ïì r (2 + 3) ïü 2 æ (2 + 3) ö æ dö
= pí ý = pr ç ÷ ; çr = 2 ÷
120 m ïî 3 ïþ è 3 ø è ø

2 ´ 3.14 ´ r ´ 25°
( )
2 2
\ 120 =
360° pd 2 (2 + 3)2 pd 2 + 3
= =
4´3 12
120 ´ 360 43200
Þ r= = = 275.15 m (c) 50° (d) 40°
2 ´ 3.14 ´ 25 157
75. (c) C
r = 275 m (approx)
A
xy
72. (c) We know that x + y +
100
Here x = 6% and y = 6%
130°
6´6 B
=6+6+
100 O
= 12 + .36 = 12.36%
A
73. (c)
60° OA and OB are radii of circle, AC and BC are
tangents
60° 60° Q Now, ÐAOB + ÐACB = 180°
P
60° 60° ÐACB = 180° – 130° = 50°
60° 76. (a)

B
Area of common circle.
< 2 × Area of equilateral triangle APQ O
3 2 3 2
< 2´ r < r
4 2
M P N
Circle A- 371
MN = 24 cm, MP = 12 cm Remaining Area
26 = Area of ABCD – Area of APB
Radius of outer circle = = 13 cm
2 2
1 æç14 ö÷
= 18 × 14 – pçç ÷÷
OP = (OM )2 – ( MP )2 4 è2ø

= 1 22 14 14
169 –144 = 25 = 5 cm = 252 – ´ ´ ´ = 252 – 77 = 175 cm2
2 7 2 2
C So, option (d) is correct.
D
120° a
79. (c) AB = 2a i.e., AL = LM = MN = NB =
2 2
A 60° Let OC = r
77. (a) 3
a
In DOML, OM= a – r and OL = OP + PL = r +
5 2
B C
In DABD-

22 + 52 - BD 2
cos 60° =
2.2.5 A L M N B

1 4 + 25 - BD 2 æaö
2
2 æ aö
2
= \ ç ÷ + (a - r) = ç r + ÷
2 20 è2ø è 2ø
BD2 = 29 – 10
a2 a2
BD = 19 Þ + a 2 - 2ar + r 2 = r 2 + + ar
4 4
In DBCD-
2 a
32 + CD 2 - BD 2 Þ a = 3ar Þ r =
cos120° = 3
2.3.CD So, option is correct.
1 9 + CD2 - ^ 9 80. (d) A
- =
2 6 CD C
CD2 – 10 = –3 CD
CD2 + 3 CD = x
O P
Let CD = x E
x2 + 3x – 10 = 0
(x + 5) (x – 2) = 0
D
x ¹ –5
B
x= 2
CD = 2 cm PA = PC + CA
So, option (a) is correct. Þ PC + CA = 16
Þ PC + CE = 16 -------------(i)
78. (d) 14 cm Again,
A B
Þ PB = PD + DB
Þ PD + DB = 16
18cm
Þ PD + ED = 16
PD + ED = 16 -------------(ii)
P eq (i) + eq (ii) -
PC + CE + PD + ED = 16 + 16
PC + PD + CE + ED = 32
PC + PD + CD = 32
C Perimeter of D PCD = 32 cm
D
EBD_7367
372
A- Circle

So, option (d) is correct. AP = AO + OP


81. (c) Let OA = r and OP = x = 2 + 3 (where OA is the radius of the circle)
\ BP = r + x and PA = r – x
\ Option (b) is correct.
r +x 2 83. (c) Let radius = r
= Þ 3x = r
r- x 1 Length of an arc = 100 m
C æ q ö
But length of arc = 2pr ç ÷
è 360 ø
36
P Þ 100 = 2p ´ r ´
A B 360
O
100 ´ 360 500
Þ r= = m
2 p ´ 36 p
D \ Option (c) is correct.
OP2 + PC2 = OC2 (Q CP ^ AB) 84. (c) Given D is an external point DP is tangent to circle.
Then DP = 4
r2
+ PC 2 = r 2 P
9
3
8r2 q
PC2 = D
9 O

2 2 4 2 2 2
CP = r and CD = r= d (where d = 2r)
3 3 3 OP = 3 (radius of circle)
So, option (c) is correct. OP 3
82. (b) Let us consider a circle of radius 2 units. Þ tan q = =
DP 4
Diameter = AB = 2 × 2 = 4 units
Þ OD = OP2 + DP 2 = (3)2 + ( 4 )2
A
OD = 9 + 16 = 5
2 3
Þ sin q =
5
O \ Option(c) is correct.
85. (c) CP ´ PD = QP ´ PR
2
Þ m ´n = 1´1
P Þ m´n=1 ... (i)
Q R
Option (a) sol. ?
B D
Q R
2
1 P 1
QR be a chord of circle
then QR = 2 units
Let O be the centre of the circle M
Given AP > BP
In right angled triangle POR O
By applying pythagorus theorem, we get

OP = ( RO )2 - ( PR )2
C
2 2
OP = ( 2) - (1) = 4 -1 = 3
Now x2 – 10 x + 1 = 0
OP = 3
Circle A-373
87. (c) Let two cocentric circular rings with centre O and
10 ± 100 - 4
Þ x=
2
Þ x = 5± 4 3
r2
\ m ´ n = (5 + 4 3)(5 - 4 3)

Þ m ´ n = 25 – 16 ´ 3 r1
O
Þ m ´ n = – 23 ...(ii)
From eq. (i) and (ii), we can say that statement 2 is not
correct.

86. (c) P
R Radius of large ring = r2
Radius of smaller ring = r 1
t
C O Area of circular both
= Area of larger ring – Area of smaller ring ...(1)
S Given circumference of larger ring = 3 × circumference
Q of smaller ring
Þ 2pr2 = 3 × 2pr1
Given a circle with centre c.
Þ r2 = 3r1
OP and OQ are tangents to the circle from Q + O point
O outside the circle. Also given
Given OR × SQ = OS × RP Area of circular both = n (area of smaller ring) ...(2)
Comparing (1) & (2) we have
OR OS Area of larger ring – Area of smaller ring
Þ =
RP SQ = n(Area of smaller ring)
Þ RS || PQ (By Basic proportionality theorem)
(1) Also CP = CQ = radius of the circle.
2 2
Þ pr2 - pl1 = n pr1
2
( )
A perpendicular drawn from P to Q,
Draw circle X and Y with centre O and radius OR and Þ pr22 ( n + 1) pr12
DS respectively.
Since RS || PQ
Þ ( n + 1) r12 = r22
Here O is the center of circle X and Y both Radius OR
and OS lies in the same circle.
Þ ( n + 1) r12 = ( 3r1 )2
Þ OR = OS Þ Area of Circle X = Area of circle Y (Q r2 = 3l1 )
Þ X= Y Þ n+1=9
Statement (1) is true. Þ n=8
(2) Also we know that if two tangents are drawn to \ Option (c) is correct.
the circle then ÐPOC = ÐQOC and ÐPCO = ÐQCO
88. (d)
Also we know that CP = CQ = radius
89. (c) Given AD be the diameter of circle.
So DPtc and DQtc are similar by AA similarly.
This circle consist of three semi-circles.
i.e., ÐP = ÐQ = 45° ...(1)
and Ðt = Ðt = 90°
Also ÐPCO = ÐQOC (Alternate angles)
ÐPOC = ÐQCO (Alternate angles)
from (1) if ÐP = ÐQ = 45°
Þ ÐQCO = ÐPLO = 45° A D
B C
Þ ÐPOC + ÐQCO = 45° + 45° = 90°
Statement (2) is true.
\ Option (c) is correct.
EBD_7367
374
A- Circle
So Area of circle = Area of all 3 semi circles OP2 = OC2 – CP2
...(i)
OP2 = 32 – 12 = 8
AD is the diameter of all circles.
Let AD = d OP = 8
Then radius = d/2 AP = 3 + 8 , BP = 3 – 8
Given are of circle = 707 m 2 AP 3 + 8
=
From (i) we get BP 3 - 8
2 2 2
p ædö p ædö p ædö 5
707 = ç ÷ + ç ÷ + ç ÷ 92. (d) Radius of circle = m
2è2ø 2è2ø 2è2ø 22
2 distance covered = 7 km or 7000 m
3p ædö
707 = ç ÷ 22 5
2 è2ø distance covered in 1 revolution = 2Pr = 2 × ´
7 22
2
ædö 707 ´ 2
Þ ç ÷ = 10
è2ø 3p = m
7
2
p ædö 7000
Area of shaded region = ç ÷ Number of revolutions = ´ 7 = 4900
2è2ø 10
p 707 ´ 2 707 2 93. (a) in D OAB ÐA = 30°, OB = 3cm
= ´ = m
2 3p 3 OB
Lost of levelling the shaded region = 63 rs/m2 = tan30°
AB
707
Lost = 63 ´ = 21 × 707 = 14847 Rs. B
3
? 3
90. (d)
A 60 O
q
14 0¢
3
C
Let radius of both circles D
be R and r respectively
PR2 + Pr2 = 130P
R2 + r2 = 130 and R + r = 14 given
3
(R + r) = 14
Squaring Both side 30
R2 + r2 + 2rR = 196 A ? B
2rR = 196 – 130 = 66
(R + r)2 – 4rR = 196 – 132 3 1
=
(R – r)2 = 64 AB 3
R–r =8
AB = 3 3
91. (b) in D OCP OC = 3 cm, CP = 1 cm
94. (c) Radius of both the wheel is x cm and y cm
A
Distance covered by front wheel in n rotation be 2pxn
Than by rear wheel in rT
revolution will be = T2 py
O 2pxn = 2py T
3
2pnx nx
T= =
C 2py y
P 2 D
B
Circle 375
A-

44 ´ 7
95. (c) r= =7
3.5 3.5 22 ´ 2
Area = p(7)2
22
= ´ 7 ´ 7 = 154 cm2
7
3.5 98. (b) When radius = r
C = 2pr
Shaded region area area = pr2
= area D – area 3 sector 115
2p ( r + x) = 2pr
3 2 60 100
= (7) - 3 ´ p(3.5) 2 When Radius = x
4 360
C = 2 p (r + x)
3 22 3.5 ´ 3.5 49 Area = p (r + x)2
= 49 - ´ = (2 3 -p)
4 7 2 8
115
r+x= r
100
New area
96. (c) A
2
æ 115 ö
p (r + x)2 = p ç r÷
è 100 ø
P 2
B C æ 115 ö 2
increase in area = p ç ÷ - pr
è 100 ø
D Q E
é115 2 ù
in D APB pr 2 ê 2
- 1ú
ëê100 úû
´ 100
13 12 % increase =
AB = cm, BP = = 6 cm pr 2
2 2
AP = ? (115 + 100)(115 - 100)
=
AP2 = AB2 – BP2 100
2 2 2 32.25 %
æ 13 ö æ 12 ö æ5ö
AP2 = ç ÷ - ç ÷ = ç ÷ or
è2ø è2ø è2ø
5 % increase in C = % increase in R
AP =
2 15 ´15
in D AQD increase in area = 15 + 15 +
100
AQ2 = AD2 – DQ2 = 32.25%
2 2 2 99. (c)
æ 13 ö æ 5 ö æ 12 ö
= ç ÷ -ç ÷ = ç ÷ 2
2
è ø è ø 2 è 2ø
3 2
12
AQ = =6 1
2
4
5 7
PQ = 6 - Þ = 3.5 2+3+ 4 9
2 2 S= = as r = 2 × 1 = 2m2
97. (c) area of square = 121 cm2 2 2

side = 121 = 11 9æ9 öæ9 öæ9 ö


ç - 2÷ ç - 3÷ ç - 4÷
perimeter = 4 × 11 = 44 curcumference of circle 2è2 ø è2 øè2 ø
Area =
2pr = 44 9 é 5 ù æ 3ö æ 1 ö
ç ÷ç ÷
2 êë 2 úû è 2 ø è 2 ø
EBD_7367
376
A- Circle
Then the angle made by direct common tangents when
3
= 15 m2 two circles of radius a and b touch externally is given
4
a - bö
by q = 2sin -1 æç .
2 2 è a + b ÷ø
1 We are given that area of the bigger circle = 9 area of
3 3 the smaller circle
Þ pa 2 = 9pb2 Þ a 2 = 9b2 Þ a = 3b
Let us consider ÐBAC = q
Thus,
æ a - bö æ 3b - b ö æ 2b ö
1 1 q = 2sin -1 ç = 2sin -1 ç = 2sin -1 ç ÷
Area = ´ 2 ´ 3 = 3 m2 Area = [1 + 3] ´ 2 = 4 è a + b ÷ø è 3b + b ÷ø è 4b ø
2 2
æ 1ö
3 = 2sin -1 ç ÷ = 2sin -1 (sin 30°) = 2 ´ 30° = 60°
Total area = 2 + 3 + 4 + 5 è 2ø
4
103. (c)
3
9+ 5 A O B
4
100. (a) D is diameter of each circle
3
\ Side of square = D P
C x D
A B
X

D C
Given that AB = 4 cm and CD = 10 cm, let the radius of
the circle be r cm.
Since the perpendicular from the center of a circle to a
Diagonal of square = D2 + D 2 = D 2 chord bisects the chord, therefore AO = OB = 2 cm
and CP = PD = 5 cm.
Diameter of shaded circle = D 2 - D = D( 2 - 1)
In DAOX, by Pythagoras theorem, we have
Diameter = 2 D - D = D( 2 - 1) 22 + (3 + x)2 = r 2 Þ 4 + 9 + 6 x + x 2 = r 2 ...(1)
101. (b) We are given that a rectangular cello tape of length 4 Similarly, in DCPX, we have
cm and breadth 0.5 cm is used for joining each pair of
edges. Therefore, area of the cello tape used for each 52 + x 2 = r 2 Þ 25 + x 2 = r 2 ...(2)
face of the cube is a2. Thus, total area of the cello tape Equation (1) and (2) gives
used is 6a2. Now, we have 6a2 = 6(4 × 0.4)(4 × 0.5) =
6.2.2 = 24 sq. cm. 4 + 9 + 6 x + x 2 = 25 + x 2 Þ 13 + 6 x = 25 Þ 6 x
102. (b) = 12 Þ x = 2
B Now, equation (2) gives
25 + 4 = r 2 Þ r 2 = 29 Þ r = 29 cm
104. (d) D 5 C

b a
q
A
P

C
Let the radius of the bigger circle be a and radius of
the smaller circle be b. A 8 B
Circle 377
A-

This is in radians. To convert in degrees, we will


Since ABCD is a cyclic quadrilateral and a trapezium, 180
therefore AB » CD. multiply .
p
Now, in APB and CPD, we have
q p
ÐCDP = ÐABP ´ 2pr = 22 Þ ´ 2pr = 22
360 360
ÐDCP = ÐPAB
Therefore, by AA similarity criteria, DAPB i DCPD 180 p
Þ ´ ´ 2pr = 22 Þ r = 7 cm.
Now, the ratio of areas of similar triangles is equal to p 360
ratio of the squares of one of its proportional sides,
therefore, 107. (a)
ar .( DAPB ) AB 2 24 82
= Þ = p
ar .( DCPD ) CD 2 ar .( DCPD ) 52

24 64
Þ = R
ar.( DCPD) 25
24 ´ 25 75 p
Þ ar .( DCPD) = = = 9.375 sq. cm
64 8
105. (a) We are given that r1 = 9 cm, r2 = 4 cm and the distance Let radius of a smaller circular region be r area of a
between the centers of two circles is 13 cm. smaller region = pr2
Therefore, the length of the direct common tangent of radius of bigger circle = 2r
Area of bigger circle = p × (2r)2 = 4pr2
these circles is given by d 2 - (r1 - r2 )2 where d is
Total area of two smaller circles = 2 × pr2 = 2 pr2
the distance between the centers of the two circles.
Area of remaining part = 4pr2 – 2pr2 = 2pr2
d 2 - (r1 - r2 ) 2 = 132 - (9 - 4) 2 = 169 - 25
2pr 2 A
Required value = 2
´A =
= 144 = 12 cm 4pr 2

106. (b) 108. (c) Tangent = 102 - (4.5 + 3.5)2 = 6 cm


109. (a)
110. (d)
o
ÐA ´ B 50
p 111. (b) ÐACB = 90 - = 90 - = 65
2 2
112. (b)
22 B
A 113. (d) OP = AB2 + PB 2 + OC × PB
Length of the arc
q p
= ´ 2pr = 22 Þ ´ 2pr = 22
360 360
EBD_7367
378
A- Statistics

C HA P T E R

STATISTICS
25
1. What is the geometrical mean of the variate which takes II. Both the middle classes have a total frequency of 30.
values 210, 201, 102, 20, 12, 10, 2, 1 and 0? [2007-I] III. The frequency distribution does not have a mode.
(a) 3 (b) 2 Which of the above statements are correct? [2007-II]
(c) 1 (d) 0 (a) I and II (b) I and III
2. What type of classification is needed to enumerate the (c) II and III (d) I, II and III
female population of India? [2007-I] 9. Square diagrams are drawn to represent the following
(a) Geographical (b) Chronological data:
(c) Qualitative (d) Quantitative
3. If the mean of the variable X is M, what is the mean of Country Pakistan India Myanmar China
X–4 Labour 36 81 25 100
variable Y = ? [2007-I] Production
5
(in ` )
M–4
(a) (b) M Using the scale 1 cm2 = ` 25, what is the length of the
5
(c) M – 5 (d) 5M – 4 representative square for India? [2007-II]
(a) 1.8 cm (b) 1.2 cm
1 (c) 1 cm (d) 2 cm
4. If mean of y and is M, then what is the mean of
y 10. An average Indian family allocates its monthly income
under different heads as follows:
1
y3 and ? [2007-II]
y3 Items Percentage Share
Food 40
M M2 – 3
(a) (b) M 3 House Re nt 15
3 Saving x
(c) M 3 – 3 (d) M(4M 2 – 3)
5. The following observation have been arranged in Transport 12
ascending order: [2007-II] Miscellaneous 23
29, 32, 48, 52, x, x + 3, 71, 75, 80, 92
If the median of data is 61.5, then what is the value of x? A pie diagram of this data is to be drawn. What is the
(a) 54 (b) 60 value of x, if the angle which the sector representing
(c) 62 (d) 56 saving makes at the centre is 36°? [2008-I]
6. For less than ogive, the cumulative frequencies are (a) 13 (b) 11
plotted against which of the following? [2007-II] (c) 10 (d) 8
(a) Upper limits of class intervals 11. From a series of 50 observations, an observation with
(b) Lower limits of the class intervals value 45 is dropped but the mean remains the same. What
(c) Mid-points of the class intervals was the mean of 50 observations? [2008-I]
(d) Either (b) or (c) (a) 50 (b) 49
7. For the following frequency distribution: (c) 45 (d) 40

Class 12. The yield of paddy per plot of one acre were obtained
0-5 5-10 10-15 15-20 20-25 25-30 from a number of plots from two different districts in a
interval
state and are summarized in the following table:
Frequency 10 15 30 80 40 20 Yield of paddy District A District B
If m is the value of mode, then which one of the following per plot Number Number
is correct? [2007-II] in quintals of plots of plots
(a) 5 < m < 10 (b) 10 < m < 15
(c) 15 < m < 20 (d) 20 < m < 25 38.0 41.0 25 14
8. The cumulative frequency curve of a frequency distribution 41.0 44.0 36 29
with 6 classes and total frequency 60 is a straight line. 44.0 47.0 59 35
Consider the following statements:
I. The first and the last classes have a frequency of 47.0 50.0 30 54
10 each. 50.0 53.0 25 41
Statistics A- 379
Which of the following statements is correct? [2008-I] (c) Pie diagram
(a) The mode for district A is higher than the mode for (d) Since population density is a ratio, it cannot be
district B represented by any diagram
(b) The mode for district B is higher than the mode for 21. Which one of the following represents statistical data?
district A [2009-I]
(c) Both the distributions are symmetric (a) The names of all owners of shops located in a shopping
(d) Both the distributions have the same mean complex
13. If every number of a finite set is increased by any (b) A list giving the names of all states of India
number k, the measure of central tendency should also (c) A list of all European countries and their respective
increase by k. Which one of the following measures of central capital cities
tendency does not have this property? [2008-I]
(d) The volume of a rainfall in certain geographical area,
(a) Arithmetic mean
recorded every month for 24 consecutive months
(b) Median
(c) Mid-range, i.e. the arithmetic mean of the largest and 22. The arithmetic mean of a set of 10 numbers is 20. If each
smallest numbers number is first multiplied by 2 and then increased by 5, then
(d) Geometric mean what is the mean of new numbers? [2009-I]
14. If the median of the distribution (arranged in ascending (a) 20 (b) 25
order) 1, 3, 5, 7, 9, x, 15, 17 is 8, what is the value of x? (c) 40 (d) 45
[2008-I] 23. Consider the following types of data:
(a) 11 (b) 13 I. Marks of students who appeared for a test of
(c) 9 < x < 15 (d) 9 x 15 100 marks.
15. In Statistics, a suitable graph for representing the II. Collar sizes of 200 shirts sold in a week.
partitioning of total into subparts is [2008-I] III. Monthly incomes of 250 employees of a factory.
(a) An ogive (b) A pictograph For which of the above data, mode is a suitable measure of
(c) A histogram (d) A pie chart central tendency? [2009-I]
16. Suppose X is some statistical variable with mean . (a) Both I and II (b) Only II
Let x1, x2, ... xn be its deviations from mean with the respective (c) Both I and III (d) I, II and III
frequencies f1, f2, ..., fn. What is the value of the sum x1 f1 + 24. The mean of 25 observations is 36. The mean of first
x2 f2 + ... + xn fn? [2008-II] 13 observations is 32 and that of last 13 observations is 39.
(a) 0 (b) 1 What is the value of 13th observation? [2009-I]
(c) (d) + 1 (a) 20 (b) 23
17. Sets A, B and C contain 5 numbers each. The medians of the (c) 32 (d) 40
numbers in these sets are 3, 8 and 11, respectively. What is 25. Data on percentage distribution of area of land in acres
the median of the combined 15 numbers of the three sets? owned by households in two districts of a particular state
[2008-II] are as follows:
(a) 8 (b) 7
Land Holding District A District B
22
(c) (d) Cannot be determined 0.01 0.99 5.62 13.53
3
18. Which one of the following statements is not correct with 1.0 2.49 18.35 21.84
reference to a histogram? [2008-II] 2.5 7.49 47.12 39.32
(a) Frequency curve is obtained by joining the mid-points
of the top of the adjacent rectangles with smooth curves 7.5 12.49 19.34 12.15
(b) Histogram is drawn for continuous data 12.5 19.99 7.21 7.43
(c) The height of the bar is proportional to the frequency 20.0 29.99 2.36 5.73
of that class What is the appropriate diagram to represent the
(d) Mode of the distribution can be obtained from the above data? [2009-I]
histogram (a) Pie diagram (b) Histogram
19. X 0 1 2 3 4 (c) Bar chart (d) None of the above
Frequency 4 f 9 g 4 26. The following table shows the percentage of male and
female coffee drinkers and non-coffee drinkers in two
The table above gives the frequency distribution of a towns A and B.
discrete variable X with two missing frequencies. If the
total frequency is 25 and the arithmetic mean of X is 2, Attributes Town A Town B
then what is the value of the missing frequency f? Male Female Male Female
[2008-II] Coffee drinkers 40% 5% 25% 15%
(a) 4 (b) 5
(c) 6 (d) 7 Non-coffee drinkers 20% 35% 30% 30%
20. Assume that population densities of 5 major states of If the total population of the towns A and B are 10000 and
India are given. Which one of the following diagrams is 20000 respectively, then what is the total number of
suitable to represent the data? [2008-II] female coffee drinkers in both towns? [2009-II]
(a) Single bar diagram (a) 8000 (b) 6000
(b) Percentage bar diagram (c) 3500 (d) 2500
EBD_7367
380
A- Statistics
27. The median of three positive integers, two of which are 34. What is the mean of the ages of minor children?
equal, is 5. What is the least possible value of the arithmetic [2010-I]
means of these integers? [2009-II] (a) 3 years (b) 4 years
(a) 2 (c) 5 years (d) 6 years
(b) 3 35. What is the median of the ages of minor children?
(c) 4 [2010-I]
(d) No such least possible value exists (a) 3 years (b) 5 years
28. Which one of the following can be obtained from a (c) 7 years (d) Cannot be determined
histogram? [2009-II] 36. A new frequency distribution is constructed by doubling
(a) Mean (b) Median each frequency of the original distribution keeping the other
(c) Mode (d) None of these entries intact. The following measures are computed for
29. Examples of data are given below: both the tables:
I. Information on households collected by an investigator I. Arithmetic mean
by door to door visits. II. Median
II. Data on the percentage of literates, sexwise, for the III. Harmonic mean
different districts of a state collected from records of Which of the following statements with reference to above
the census of India. is correct? [2010-I]
III. General information about families, collected by
(a) Corresponding values of I and II only are equal in
telephonic interviews.
both the distributions
Which one of the following in respect of the above is
correct? [2009-II] (b) Corresponding values of I and III only are equal in
(a) I and II are primary data both the distributions
(b) I and III are primary data (c) Corresponding values of II and III only are equal in
(c) II and III are primary data both the distributions
(d) I, II and III are primary data (d) Corresponding values of I, II and III are equal in both
30. What is the geometric means of the observations the distributions
125, 729, 1331? [2009-II] 37. Consider the following data:
(a) 495 (b) 1485
(c) 2221 (d) None of these Year Birth rate Death rate
31. Consider the following pairs of numbers: [2009-II] 1911 21 48.1 38.5
I. (8, 12) II. (9, 11)
III. (6, 24) 1921 31 46.4 36.3
Which pairs of number have the same harmonic means? 1931 41 45.2 31.2
(a) I and II (b) II and III 1941 51 39.9 27.4
(c) I and III (d) I, II and III
32. The marks of the students of a class who appeared for a test 1951 61 41.7 22.8
in English are represented in the following 1961 71 41.1 35.9
frequency table.
1971 81 37.1 14.8
Class interval Frequency
For which period is the natural growth rate minimum?
1 10 9
[2010-I]
11 20 22 (a) 1911-21 (b) 1921-31
21 30 — (c) 1951-61 (d) 1961-71
38. With the help of histogram one can prepare [2010-I]
31 40 20 (a) frequency polygon (b) frequency curve
41 50 12 (c) frequency distribution (d) All of the above
51 60 8 39. The table below gives the number of members of a club
classified by sex and nativity:
100 total frequency
What is/ are the modal class(es)? [2010-I] Nativity
(a) Only 10.5 – 20.5 Locals Migrants Total
(b) Only 20.5 – 30.5 Sex
(c) 10.5 – 20.5 and 20.5 – 30.5
(d) There is no modal class Male 85 45 130
33. What is the weighted mean of first 10 natural numbers whose Female 35 35 70
weights are equal to the corresponding number? [2010-I]
(a) 7 (b) 5.5 Total 120 80 200
(c) 5 (d) 4.5
The above data are represented by a pie diagram. What is
DIRECTIONS (Q. Nos. 34-35): Read the following information
the sectorial angle of the area representing male-migrant
carefully to answer the questions that follow.
category? [2010-I]
The average age of 6 persons living in a house is 23.5 years.
(a) 45° (b) 22.5°
Three of them are majors and their average age is 42 years.
(c) 81° (d) 67.5°
The difference in ages of the three minor children is same.
Statistics 381
A-

DIRECTIONS (Q. Nos. 40-42): Read the following information Consider the following:
carefully to answer the questions that follow. I. M(xi + yi ) = M(xi ) + M(yi)
The arithmetic mean, geometric mean and median of 6 positive II. G(xi yi ) = G(xi) G(yi)
7 Which of the above is/ are correct? [2010-II]
numbers a, a, b, b, c, c, where a < b < c are , 2, 2, respectively.. (a) Only I (b) Only II
3 (c) Both I and II (d) Neither I nor II
40. What is the sum of the squares of all the six numbers? 49. The arithmetic mean of 100 numbers was computed
[2010-I] as 89.05. It was later found that two numbers 92 and 83 have
(a) 40 (b) 42 been misread as 192 and 33, respectively. What is the correct
(c) 45 (d) 48 arithmetic mean of the numbers? [2010-II]
41. What is the value of c? [2010-I] (a) 88.55 (b) 87.55
(a) 1 (b) 2 (c) 89.55 (d) Cannot be determined
(c) 3 (d) 4 DIRECTIONS (Q. Nos. 50-51): The item-wise expenditure of a
42. What is the mode? [2010-I] Non-Government Organisation for the year 2008-09 is given below:
(a) 1 (b) 2
(c) 1, 2 and 4 (d) None of these Expenditure
Item
43. Consider the following data: (in Rs. lakh)
x 1 2 3 4 5 Salary of employees 6
f 3 5 9 – 2 Social welfare activities 7
Office contingency 3
If the arithmetic mean of the above distribution is 2.96, then
what is the missing frequency? [2010-I] Vehicle maintenance 4
(a) 4 (b) 6 Rent and hire charges 2.5
(c) 7 (d) 8
Miscellaneous exp enses 1.5
44. For a set of positive numbers, consider the following
statements: The above data are represented by a pie diagram.
I. If each number is reduced by 2, then the geometric 50. What is the central angle of the largest component?
mean of the set may not always exist. [2011-I]
II. If each number is increased by 2, then the geometric (a) 120° (b) 105°
mean of the set is increased by 2. (c) 90° (d) 85°
Which of the above statements is/ are correct? [2010-II] 51. What is the difference between central the angles of the
(a) Only I (b) Only II largest and the smallest component? [2011-I]
(c) Both I and II (d) Neither I nor II (a) 90° (b) 85°
45. Consider the following frequency distribution: (c) 82.5° (d) 77.5°
DIRECTIONS (Q. Nos. 52-53): The following table gives the
Class 0 10 0 20 0 30 0 40 0 50 frequency distribution of life length in hours of 100 electric
Frequency 3 8 14 14 20 bulbs having median life 20 h.
Life of bulbs
What is the above frequency distribution known as? Number of bulbs
[2010-II] (in hours)
(a) Cumulative distribution in more than type 8 13 7
(b) Cumulative distribution in less than type 13 18 x
(c) Continuous frequency distribution
(d) None of the above 18 23 40
46. Consider the following statements in respect of histogram. 23 28 y
I. Histogram is an equivalent graphical representation of
28 33 10
the frequency distribution.
II. Histogram is suitable for continuous random variables, 33 38 2
where the total frequency of an interval is evenly 52. What is the missing frequency ‘x’? [2011-I]
distributed over the interval. (a) 31 (b) 27
Which of the statements given above is/ are correct? (c) 24 (d) 14
[2010-II] 53. What is the missing frequency ‘y’? [2011-I]
(a) 27 (b) 24
(a) Only I (b) Only II (c) 14 (d) 11
(c) Both I and II (d) Neither I nor II 54. Consider the following statements in respect of a histogram:
47. What is the median of the values 11, 7, 6, 9, 12, 15 and 19? I. The histogram consists of vertical rectangular bars with
[2010-II] a common base such that there is no gap between
(a) 9 (b) 11 consecutive bars.
(c) 12 (d) 15 II. The height of the rectangle is determined by the
48. Let (x1, y1), (x2, y2), ..., (xn, yn) are n pairs of positive numbers. frequency of the class it represents.
The arithmetic mean and geometric mean of any set of Which of the statements given above is/are correct?
positive numbers (c 1 , c 2 , ..., c n) ar e denoted by [2011-I]
M(ci ) and G(ci ), respectively. (a) Only I (b) Only II
(c) Both I and II (d) Neither I nor II
EBD_7367
382
A- Statistics

55. The arithmetic mean of 10 numbers was computed as 7.6. It (a) 36 (b) 44
was later discovered that a number 8 was wrongly read as 3 (c) 45 (d) 52
during the computation. What should be the correct mean? 63. Consider the following statements in respect of the set
[2011-I] S = {1, 2, 3, ..., n}.
(a) 7.1 (b) 7.6 n 1
(c) 8.1 (d) 8.6 I. is the median of the numbers in S.
56. Which one among the following statements is correct? 2
[2012-I] II. n is the mode of the numbers in S.
(a) Simple bar diagrams are those diagrams which show Which of the above statements is/are correct? [2012-I]
two characteristics of the data (a) Only I (b) Only II
(b) In pie diagrams all the items are converted into angles (c) Both I and II (d) Neither I nor II
(c) A bar diagram is one in which data are shown in terms 64. The arithmetic mean and geometric mean of two numbers
of bars are 14 and 12, respectively. What is the harmonic mean of
(d) Bar diagrams present data through length and breadth the numbers? [2012-I]
57. Two following characteristics relate to the persons (a) 10 (b) 18
participating in athletic events: 32 72
I. height of the person. (c) (d)
3 7
II. colour of the eye of the person.
65. Consider the following table:
III. number of times a person correctly hits a target in ten
attempts. x 0 1 2 3 4
Which of the following in respect of the above is correct? y 100 90 70 40 0
[2012-I]
(a) I is a continuous variable, II is not a variable and III is How are x and y related? [2012-II]
a discrete variable (a) y = 100 – 10x (b) y = 100 – 5x2
(b) I is a continuous variable, II and III are discrete variables (c) y = 20 – x – x2 (d) y = 100 – 5x – 5x2
66. In histogram, the width of the bars is proportional to
(c) I and III are discrete variables and II is not a variable
[2012-II]
(d) I, II and III all are discrete variables (a) Frequency (b) Number of classes
58. Consider the following statements: (c) Class interval (d) None of these
I. The data collected by the investigator to be used by 67. Which one of the following relations for the numbers
himself are called primary data. 10, 7, 8, 5, 6, 8, 5, 8 and 6 is correct? [2012-II]
II. The data obtained from government agencies are called (a) Mean = Median (b) Mean = Mode
secondary data. (c) Mean > Median (d) Mean > Mode
Which of the above statements is/are correct? [2012-I] 68. The mean of 100 values is 45. If 15 is added to each of the
(a) Only I (b) Only II first forty values and 5 is subtracted from each of the
(c) Both I and II (d) Neither I nor II remaining sixty values, the new mean becomes [2012-II]
59. Which one among the following statements is not correct? (a) 45 (b) 48
[2012-I] (c) 51 (d) 55
(a) For size of readymade garments, mode is the best 69. Which of the following is/are correctly matched?
suitable measure I. Weight of a person : Continuous variable.
(b) For average rate of increase when the rate of population II. Educational qualification of the person : Attribute
growth is given, geometric mean is best suitable Select the correct answer using the codes given below.
(c) For average rate of speed when different distances are [2012-II]
covered by different rates of speed, harmonic mean is (a) Only I (b) Only II
best suitable (c) Both I and II (d) Neither I nor II
(d) For average level of intelligence of students in a class, 70. Consider the following distribution:
arithmetic mean is the best suitable Value of the variable 1 2 3 4 5
60. Let the observations at hand be arranged in increasing order.
Which one of the following measures will not be affected Frequency 3 f 6 5 3
when the smallest and the largest observations are removed? For what value of f, is the arithmetic mean of the above
[2012-I] distribution 3.1? [2012-II]
(a) Mean (b) Median (a) 2 (b) 3
(c) Mode (d) Standard deviation (c) 4 (d) 5
61. If the population figures are given for each state of India, 71. The average of u, v, w, x, y and z is 10. What is the average
then the data can be classified as [2012-I] of u + 10, v + 20, w + 30, x + 40, y + 50 and z + 60?[2013-II]
(a) qualitative (b) quantitative (a) 30 (b) 35
(c) chronological (d) geographical (c) 40 (d) 45
62. The frequency distribution of a variate x is as given below: 72. If m is the mean of p, q, r, s, t, u and v, then what is
(p – m) + (q – m) + (r – m) + (s – m) + (t – m) +
x 2 3 4 5 6
(u – m) + (v – m) equal to? [2013-II]
Frequency 2 7 14 22 30 (a) 0 (b) s
What is the cumulative frequency corresponding to variate p v
x = 5? [2012-I] (c) (d) None of these
2
Statistics A-383
DIRECTIONS (Q. Nos. 73-74): Read the following information (c) A frequency polygon is obtained by connecting the
carefully and answer the questions given below. corner points of the class intervals in a histogram
The median of the following distribution is 14.4 and the total (d) None of the above
frequency is 20. 83. Consider the following:
I. The arithmetic mean of two unequal positive numbers
Class Interval 0 6 6 12 12 18 18 24 24 30
is always greater than their geometric mean.
Frequency 4 x 5 y 1 II. The geometric mean of two unequal positive numbers
73. What is x equal to? [2013-II] is always greater than their harmonic mean.
(a) 4 (b) 5 Which of the above statements is/are correct? [2014-I]
(c) 6 (d) 7 (a) Only I (b) Only II
74. What is the relation between x and y? [2013-II] (c) Both I and II (d) Neither I nor II
(a) 2x = 3y (b) 3x = 2y 84. Consider the following statements in respect of a discrete
(c) x = y (d) 2x = y set of numbers.
75. There are 45 male and 15 female employees in an office. If I. The arithmetic mean uses all the data is always uniquely
the mean salary of the 60 employees is ` 4800 and the mean defined.
salary of the male employees is ` 5000, then the mean salary II. The median uses only one or two numbers from the
of the female employees is [2013-II] data and may not be unique.
(a) ` 4200 (b) ` 4500 Which of the above statements is/are correct? [2014-I]
(c) ` 5600 (d) ` 6000 (a) Only I (b) Only II
76. The mean of 7 observations is 7. If each observation is (c) Both I and II (d) Neither I nor II
increased by 2, then the new mean is: [2013-II] 85. The geometric mean of (x1, x2, x3, ..., xn) is x and the geometric
(a) 12 (b) 10 mean of (y1, y2, y3, ..., yn) is y. Which of the following is/are
(c) 9 (d) 8 correct?
77. Which of the following are the examples of discrete I. The geometric mean of (x1 y1, x2 y2, x3 y3, ..., xn yn) is
variables? XY.
I. Number of errors per page in a book. x1 x 2 x3 xn X
II. Height of individuals measured in centimetre. II. The geometric mean of y , y , y , ..., y is .
III. Waiting time to failure of electric bulbs. 1 2 3 n Y
IV. Number of leaves on branches of a tree. Select the correct answer using the code given below.
Select the correct answer using the codes given below. [2014-I]
[2013-II] (a) Only I (b) Only II
(a) Only I (b) I and IV (c) Both I and II (d) Neither I nor II
(c) III and IV (d) II and IV 86. The following table gives ‘less than’ type frequency
DIRECTIONS (Q. Nos. 78-80): Read the following information distribution of income per day.
carefully to answer the questions that follow. Income (in ` ) less than Number of persons
In a frequency distribution having class intervals 0-10, 10-20, 20-
1500 100
30 and 30-40 the respective frequencies are x, x + 8, x – 2 and x – 4
and the arithmetic mean of the distribution is 17.8. 1250 80
78. The value of x is [2013-II] 1000 70
(a) 3 (b) 6
(c) 8 (d) 12 750 55
79. The median lies in which one of the following class intervals? 500 32
[2013-II] 250 12
(a) 0-10 (b) 10-20
What is the modal class? [2014-I]
(c) 20-30 (d) 30-40
(a) 250-500 (b) 500-750
80. The modal class is: [2013-II] (c) 750-1000 (d) None of these
(a) 0-10 (b) 10-20
87. Which of the following items of information is a good
(c) 20-30 (d) 30-40
example of statistical data? [2014-I]
81. Consider the following statements:
(a) A table of logarithms of numbers
I. A frequency distribution condenses the data and
(b) A list of names of 120 students of a class
reveals its important features.
(c) A list of annual incomes of the members of a club
II. A frequency distribution is an equivalent representation (d) Holiday list of the offices of Government of India in the
of original data.
year 2013
Which of the above statements is/are correct? [2013-II]
88. Consider the following in respect of variate which takes
(a) Only I (b) Only II
values 2, 2, 2, 2, 7, 7, 7 and 7.
(c) Both I and II (d) Neither I nor II
I. The median is equal to mean.
82. Which one of the following statements is correct?
II. The mode is both 2 and 7.
[2013-II] Which of the above statements is/are correct? [2014-I]
(a) A frequency polygon is obtained by connecting the
(a) Only I
corner points of the rectangles in a histogram
(b) Only II
(b) A frequency polygon is obtained by connecting the
(c) Both I and II
mid-points of the tops of the rectangles in a histrogram
(d) Neither I nor II
EBD_7367
384
A- Statistics

89. Consider the following statements pertaining to a frequency 95. If xi ’s are the mid–points of the class intervals of grouped
polygon of a frequency distribution of a continuous variable data, fi’s are the corresponding frequencies and x is the
having seven class intervals of equal width.
mean, then what is f i ( xi x ) equal to?
I. The original frequency distribution can be
reconstructed from the frequency polygon. [2014-II]
(a) 0 (b) –1
II. The frequency polygon touches the X-axis in its
(c) 1 (d) 2
extreme right and extreme left. 96. Ten observations 6, 14, 15, 17, x + 1, 2x – 13, 30, 32, 34 and 43
Which of the above statements is/are correct? [2014-I] are written in ascending order. The median of the data is 24.
(a) Only I (b) Only II What is the value of x? [2014-II]
(c) Both I and II (d) Neither I nor II (a) 15 (b) 18
90. The mean of the following distribution is 18. (c) 20 (d) 24
97. If A, G and H are the arithmetic, geometric and harmonic
Class interval Frequency
means between a and b respectively, then which one of the
11 13 3 following relations is correct? [2015-I]
13 15 6 (a) G is the geometric mean between A and H
(b) A is the arithmetic mean between G and H
15 17 9 (c) H is the harmonic mean between A and G
17 19 13 (d) None of the above
19 21 f 98. The geometric mean of three positive numbers a, b, c is 3
and the geometric mean of another three positive numbers
21 23 5 d, e, f, is 4. Also, at least three elelments in the set {a, b, c, d,
23 25 4 e, f} are distinct. Which one of the following inequalities
What is the value of f ? [2014-I] gives the best information about M, the arithmetic mean of
(a) 8 (b) 9 the six numbers ? [2015-I]
(c) 10 (d) 11 (a) M 2 3
91. The class which has maximum frequency is known as (b) M > 3.5
[2014-II] (c) M > 3.5
(a) median class (b) mean class (d) It is not possible to set any precise lower limit for M
(c) modal class (d) None of these 99. There are five parties A, B, C, D and E in an election. Out of
92. Consider the following statements related to cumulative total 100000 votes cast, 36000 were cast to party A, 24000 to
frequency polygon of a frequency distribution, the party B, 18000 to party C, 7000 to party D and rest to party
frequencies being cumulated from the lower end of the range E. What angle will be allocated for party E in the pie chart?
[2015-I]
: [2014-II] (a) 15° (b) 54°
1. The cumulative frequency polygon gives an equivalent (c) 60° (d) 72°
representation of frequency distribution table. For the next four (4) items that follow:
2. The cumulative frequency polygon is a closed polygon Consider the following frequency distribution :
with one horizontal and one vertical side.
The other sides have non–negative slope. Class Frequency
Which of the above statements is / are correct ? 0-10 4
(a) Only 1 (b) Only 2 10-20 5
(c) Both 1 and 2 (d) Neither 1 nor 2 20-30 7
93. Consider the following data : [2014-II] 30-40 10
1. Number of complaints lodged due to road accidents in
40-50 12
a state within a year for 5 consecutive years.
2. Budgetary allocation of the total available funds to 50-60 8
the various items of expenditure. 60-70 4
Which of the above data is / are suitable for repsentation 100. What is the mean of the distribution? [2015-I]
of a pie diagram ? (a) 37.2 (b) 38.1
(a) Only 1 (b) Only 2 (c) 39.2 (d) 40.1
(c) Both 1 and 2 (d) Neither 1 nor 2 101. What is the median class? [2015-I]
94. When we take class intervals on the X–axis and (a) 20–30 (b) 30–40
(c) 40–50 (d) 50–60
corresponding frequencies on the Y-axis and draw
102. What is the median of the distribution? [2015-I]
rectangles with the areas proprotional to the frequencies of (a) 37 (b) 38
the respective class intervals, the graph so obtained is called (c) 39 (d) 40
[2014-II] 103. What is the mode of the distribution? [2015-I]
(a) bar diagram (b) frequency curve (a) 38.33 (b) 40.66
(c) ogive (d) None of the above (c) 42.66 (d) 43.33
Statistics A-385

104. If a variable takes discrete values a + 4, a – 3.5, a – 2.5, a – The average score of class X and Y is 79
3, a–2, a + 0.5, a + 5 and a – 0.5 where a > 0, then the median and average score of class Y and Z is 81.
of the data set is [2015-II] What is the average score of X, Y and Z ?
(a) a – 2.5 (b) a – 1.25 (a) 81.5 (b) 80.5
(c) a – 1.5 (d) a – 0.75 (c) 79.0 (d) 78.0
105. If each of n numbers xi = i (i = 1, 2, 3,..... n) is replaced by 113. For x > 0, if a variable takes discrete values x + 4, x – 3.5,
(i + 1) xi, then the new mean is [2015-II] x – 2.5, x – 3, x – 2, x + 0.5, x – 0.5, x + 5, then what is the
n 3 n(n 1) median ? [2016-II]
(a) (b) (a) x – 1.25 (b) x – 0.5
2 2
(c) x + 0.5 (d) x + 1.25
(n 1)(n 2) (n 1)(n 2) 114. The median of a set of 9 distinct observations is 20.5. If
(c) (d)
3n 3 each of the largest 4 observations of the set is increased by
106. The weighted arithmetic mean of first 10 natural numbers 2, then the median of the new set [2016-II]
whose weights are equal to the corresponding numbers is (a) is increased by 2
equal to [2015-II] (b) is decreased by 2
(a) 7 (b) 14 (c) is two times the original median
(c) 35 (d) 38.5 (d) remains the same as that of original set
107. The election result in which six parties contested was 115. Number of credit cards held by an individual can be treated
depicted by a pic chart. Party A had an angle 135° on this
as [2016-II]
pic chart. If it secured 21960 votes, how many valid votes in
(a) Qualitative data
total were cast? [2016-I]
(a) 51240 (b) 58560 (b) Discrete data
(c) 78320 (d) 87840 (c) Categorical data
108. The mean and median of 5 observations are 9 and 8 (d) None of the above
respectively. If 1 is substracted from each observation, then 116. A pie chart depicts the classification of total funds of an
the new mean and the new median will respectively be organization according to different sources of funds. A
[2016-I] particular sector of pie chart for corporate tax has 108° angle
(a) 8 and 7 at' the centre. What is the percentage of income from
(b) 9 and 7 corporate tax to total funds ? [2016-II]
(c) 8 and 9 (a) 20% (b) 25%
(d) Cannot be determined due to insufficient data (c) 30% (d) 35%
109. The age distribution of 40 children is as follows: 117. Consider the following statements : [2016-II]
Age (in years) 5–6 6–7 7–8 8–9 9–10 10–11 1. The classes of type 15–19, 20–24, 25–29 etc. are
No. of children 4 7 9 12 6 2 exclusive classes.
Consider the following statements in respect of the above 2. The classes of type 15–20, 20–25, 25–30 etc. are
frequency distribution: [2016-I] inclusive classes.
1. The median of the age distribution is 7 years. Which of the above statements is/are correct?
2. 70% of the children are in the age group 6–9 years. (a) 1 only (b) 2 only
3. The modal age of the children is 8 years. (c) Both 1 and 2 (d) Neither 1 nor 2
Which of the above statements are correct? [2016-I] 118. Suppose the class interval 10–15 has frequency 30, then what
(a) 1 and 2 only (b) 2 and 3 only is the frequency density of this class interval? [2016-II]
(c) 1 and 3 only (d) 1, 2 and 3 (a) 2 (b) 3
110. Suppose x1 = 8 for 0 < 10, where > 1. [2016-I] (c) 5 (d) 6
Which one of the following is correct?
119. Consider the following distribution : [2017-I]
(a) AM – Median (b) GM – Median
(c) GM – Median (d) AM – Median Class Frequency
0 – 20 17
1 20 – 40 28
111. Suppose x1 for i = 1, 2, 3, ......11. [2016-I]
1 40 – 60 32
Which one of the following is not correct? 60 – 80 f
(a) AM > 1/6 (b) GM > 1/6 80 – 100 19
(c) HM > 1/6 (d) Median – HM If the mean of the above distribution is 50, what is the value
112. The average score of class X is 83. [2016-II] of f ?
The average score of class Y is 76. (a) 24 (b) 34
The average score of class Z is 85. (c) 56 (d) 96
EBD_7367
386
A- Statistics

120. In a pie diagram, there are four slices with angles 150°, 90°, 128. What is the ratio of the total number of boys to the total
60° and 60°. A new pie diagram is formed by deleting one of number of girls? [2017-II]
the slices having angle 60° in the given pie diagram. In the (a) 67 : 83 (b) 17 : 26
new pie diagram [2017-I] (c) 27 : 19 (d) 189 : 179
(a) The largest slice has angle 150° 129. Frequency density of a class is computed by the ratio
[2017-II]
(b) The smallest slice has angle 70°
(a) Class frequency to the class width
(c) The largest slice has angle 180°
(b) Class frequency to total frequency
(d) The smallest slice has angle 90° (c) Class frequency to total number of classes
121. In an asymmetrical distribution, if the mean and median of (d) Cumulative frequency up to that class to total
the distribution are 270 and 220 respectively, then the mode frequency
of the data is [2017-I] 130. A small company pays each of its 5 category ‘C’ workers `
(a) 120 (b) 220 20,000, each of its 3 category ‘B’ workers ` 25,000 and a
(c) 280 (d) 370 category ‘A’ worker ` 65,000. The number of workers earning
122. An individual purchases three qualities of pencils. The less than the mean salary is [2017-II]
relevant data is given below : [2017-I] (a) 8 (b) 5
(c) 4 (d) 3
Quality Price per Money spent
131. The pie diagrams on the monthly expenditure of two families
Pencil (in `) (in `)
A and B are drawn with radii of two circles taken in the ratio
A 1.00 50 16 : 9 to compare their expenditures.
B 1.50 x Which one of the following is the appropriate data used for
C 2.00 20 the above mentioned pie diagrams? [2017-II]
It is known that the average price per pencil is ` 1.25. What (a) ` 16,000 and ` 9,000 (b) ` 8,000 and ` 4,500
is the value of x ? (c) ` 25,600 and ` 8,100 (d) ` 4,000 and ` 3,000
(a) `10 (b) ` 30 132. Consider the following statements :
(c) ` 40 (d) ` 60 Statement I : The value of a random variable having the
123. Consider the following frequency distribution : highest frequency is mode.
x Frequency Cumulative frequency Statement II : Mode is unique.
1 8 8 Which one of the following is correct in respect of the above
2 10 18 statements? [2017-II]
3 f1 29 (a) Both Statement I and Statement II are true and
4 f2 45 Statement II is the correct explanation of Statement I
(b) Bothe Statement I and Statement II are true
What are the values of f1 and f2 respectively ?
but Statement II is not the correct explanation of
(a) 10 and 17 (b) 17 and 10
(c) 11 and 16 (d) 16 and 11 Statement I
124. What is the total number of boys studying Statistics and (c) Statement I is true but Statement II is false
Physics? [2017-II] (d) Statement I is false but Statement II is true
(a) 180 (b) 240 133. Which one of the following is not correct?
(c) 250 (d) 310 The proportion of various items in a pie diagram is
125. The number of girls studying Statistics is what percent proportional to the [2017-II]
(approximate) of the total number of students studying (a) Areas of slices
Chemistry? [2017-II] (b) Angles of slices
(a) 58.8 (b) 73.5 (c) Lengths of the curved arcs of the slices
(c) 78.7 (d) 80.6 (d) Perimeters of the slices
126. In which subjects is the difference between the number of 134. The geometric mean of x and y is 6 and the geometric mean
boys and girls equal? [2017-II] of x, y and z is also 6. Then the value of z is [2017-II]
(a) Mathematics and Chemistry (a) 12 (b) 6
(b) Statistics and Chemistry
(c) Mathematics and Physics (c) 6 (d) 3 6
(d) Mathematics and Statistics 135. The total number of live births in a specific locality during
127. What is the difference between the number of boys studying different months of a specific year was obtained from the
Mathematics and the number of girls studying Physics? office of the Birth Registrar. This set of data may be called
[2017-II] [2017-II]
(a) 20 (b) 30 (a) Primary data (b) Secondary data
(c) 60 (d) 80 (c) Recorded data (d) Countable data
Statistics A-387

136. The heights (in cm) of 5 students are 150, 165, 161, 144 and 139. The arithmetic mean of two numbers is 10 and their geometric
155. What are the values of mean and median mean is 8. What are the two numbers?
(in cm) respectively? [2017-II] [2018-I]
(a) 165 and 161 (b) 155 and 155 (a) 15, 5 (b) 12, 8
(c) 160 and 155 (d) 155 and 161 (c) 16, 4 (d) 18, 2
137. In the following table of inverse variation, what are the
values of A, B and C respectively? [2018-I] 140. The arithmetic mean of 11 observations is 11. The arithmetic
mean of the first 6 observations is 10.5 and the arithmetic
M 15 6 2 C
mean of the last 6 observations is 11.5. What is the sixth
N 4 A B 60 observation? [2018-I]
(a) 10, –30, –1 (b) 10, –1, 30 (a) 10.0 (b) 10.5
(c) –30, 10, –1 (d) –1, –30, 10 (c) 11.0 (d) 11.5
138. A Pie Chart is drawn for the following data : [2018-I] Consider the following for the next two (02) questions.
In a triangle ABC, a, b and c are the lengths of the sides and p, q
Sector Percentage and r are the lengths of its medians.
Agriculture and Rural Development 12.9 141. Which one of the following is correct? [2018-I]
Irrigation 12.5 (a) (a + b + c) < (p + q + r)
Energy 27.2 (b) 3 (a + b + c) < 4 (p + q + r)
Industry and Minerals 15.4 (c) 2 (a + b + c) > 3 (p + q + r)
Transport and Communication 15.9 (d) 3 (a + b + c) > 4 (p + q + r)
Social Services 16.1
What is the angle (approximately) subtended by the Social
Services Sector at the centre of the circle?
[2018-I]
(a) 45° (b) 46°
(c) 58° (d) 98°
EBD_7367
388
A- Statistics

HINTS & SOLUTIONS


1. (d) The variables are: 210, 201, 102, 20, 12, 10, 2, 1 and 0. 9. (a) ` 25 = 1 cm2
Geometric mean of variates 1
= 9 210 201 102 20 12 10 2 1 0 `1= cm2
25
= 90 =0 81
` 81 = cm2 = Area of square
2. (d) Quantitative is needed to enumerate the female 25
population of India.
81 9
3. (a) Since, the mean of the variable X is M, then mean of Side of square = = 1.8 cm
25 5
X–4 M–4 10. (c) We know that:
the variable Y = is .
5 5
Value of item
1 Central angle = 360
Sum of values of items
4. (d) Mean of y and = M
y x
1 x
y 36° = 360
y 1 40 15 x 12 23
=M y+ = 2M ... (i)
2 y
36 x
1
Now, mean of y3 and is 360 90 x
y3 90 + x = 10x
1
3 9x = 90
1 1
y3 3 y –3 y x = 10
y y y 11. (c) Let the observation mean = x
2 2 Sum of 50 observations = 50x
1 According to question,
y3 3
y3 2M – 6M 50x – 45
=x
2 2 49
2
50x – 45 = 49x
2M 2M –3 x = 45
= = M(4M2 – 3) 12. (b) For District A:
2 Maximum frequency = 59
5. (b) n = 10, which is even ...[Given]
Modal class = 44-47
n n l = 44, f1 = 59, f0 = 36, f2 = 30, h = 3
Value of th 1 th term
2 2 f1 – f0
Median (M) = Mode = l + ×h
2 2f1 – f0 – f2
5th term 6th term 59 – 36
61.5 = = 44 + ×3
2 2 59 – 36 – 30
x x 3 2x 3 23 3
61.5 = = 44 + = 44 + 1.33 = 45.33
2 2 52
2x + 3 = 123 For District B:
2x = 123 – 3 = 120 Maximum frequency = 54
x = 60 Modal class = 47-50
6. (a) For an ogive, the cumulative frequencies are plotted l = 47, f1 = 54, f0 = 35, f2 = 41, h = 3
as a upper limit of class intervals.
54 – 35
7. (c): Here, maximum frequency is 80, hence mode will be Mode = 47 + ×3
between 15-20. 2 54 – 35 – 41
8. (a) Since, the frequency in a straight line, so we take all 19 3
classes have equal frequency, i.e. 10 = 47 + = 47 + 1.78 = 48.78
I. It is true that first and last class have 10 32
frequency. Mode of District B > mode of District A.
II. Both the middle classes have 10 + 10 = 20. 13. (d) If every number of a finite set is increased by any
III. Since, all have equal frequency, so we cannot number k, the measure of central tendency should
determined the mode. also increase by k. Geometric mean does not have
this property.
Statistics A-389
14. (d) Distributions are 1, 3, 5, 7, 9, x, 15, 17. According to question,
Total number of terms = 8 [even]
2a1 5 2a 2 5 2a 3 5 ... 2a10 5
Median
= 10
8 8
Value of th term + Value of 1 th term 2a1 2a 2 2a 3 ... 2a10 5 10
2 2
= =
2 10

Value of 4 th term + Value of 5th term 2 a1 a2 a3 ... a10 50


= =
2 10 10
= 2 × 20 + 5 = 45
7 9 16 23. (c) Only I and III, mode is a suitable measure of central
= =8
2 2 tendency.
Now, the distribution is arranged in ascending order. 24. (b) Total number of observations = 25
So, 9 x 15. Sum of 25 observations = 25 × 36 = 900
15. (d) In statistics, a suitable graph for representing the Sum of first 13 observations = 13 × 32 = 416
partitioning of total into sub parts is a pie chart. Sum of last 13 observations = 13 × 39 = 507
16. (a) According to property of arithmetic mean zero, 13th observation = 416 + 507 – 900
x1 f1 + x2 f2 + ... + xn fn = 0 = 923 – 900 = 23
17. (d) Median cannot be determined from the given data. 25. (c) Because there is a gap between two adjacent bars, so
18. (c) The height of the bar is not proportional to the both the districts can be represented by bar chart.
frequency of the class. 26. (c) Total number of female coffee drinkers
= 5% of 10000 + 15% of 20000 = 500 + 3000 = 3500
19. (a) X f fx 27. (c) Let three positive integers whose median is 5 are
0 4 0 1, 5, 6
1 f f 1 5 6 12
A.M. = =4
2 9 18 3 3
28. (c) Mode can be obtained from a histogram.
3 g 3g 29. (b) Statements I and III are primary data.
4 4 16 3 3
30. (a) G.M. = 3 125 729 1331 5 93 113
fx = f + 3g + 34 = 5 × 9 × 11 = 495
N = 25, X = 2 2 8 12 2 8 12
f + g + 17 = 25 31. (c) I. H.M. =
f+g=8 ... (i) 8 12 20
fx 2ab
X harmonic mean (HM)of a and b
N a b
f
3g 34
2= 48
25 = = 9.6
5
25 × 2 = f + 3g + 34
f + 3g = 16 ... (ii) 2 9 11 2 9 11
From equation (i) and (ii), II. H.M. = = 9.9
9 11 20
f 3g 16 2 6
24 2 6 24 48
f g 8 III. H.M. = = 9.6
– – – 6 24 30 5
2g 8 Thus, Ist and IIIrd pairs have same harmonic means.
32. (b) Let the frequency of class interval 21 – 30 be f.
8 9 + 22 + f + 20 + 12 + 8 = 100
g 4
2 71 + f = 100
f = 4, g = 4. f = 100 – 71 = 29
20. (c) To determine the population of 5 major states of Highest frequency = 29
India, the best suitable data is pie diagram. Modal class = Class corresponding to 29
21. (d) The volume of rainfall in certain geographical area, = 20.5 – 30.5
recorded every month for 24 consecutive months. 33. (a) Weighted mean
22. (d) Given that: w1 x1 w 2 x 2 ... w10 x10
Let a1, a2, a3, a4, a5, a6, a7, a8, a9 and a10 be 10 =
w1 w 2 ... w10
numbers.
a1 a2 a 3 ... a10 1 1 2 2 ... 10 10 12 22 ... 102
Then, = 20 xn
10 1 2 3 ... 10 1 2 ... 10
EBD_7367
390
A- Statistics

385 b b
= =7 2= =b
55 2
By using the formula, a a b b c c
Arithmetic mean =
n n 1 2n 1 n n 1 6
n2 and n
6 2 7 a b c
3 3
10 10 1 20 1
a+b+c=7
6 10 11 21 a+c=7–2=5 ... (i)
= =7
10 1 6 55 1
10
2 Geometric mean = a 2 b2 c 2 6

Solution (Q. Nos. 34-35):


1
Total age of six persons = 23.5 × 6 = 141 years
Total age of three major persons = 42 × 3 = 126 years 2 = abc 3
Total age of three minor children = 141 – 126 = 15 years abc = 8
The difference in ages of the three minor children is same. 8
Therefore, we take ages may be: ac = =4 ... (ii)
5, 5, 5; 3, 5, 7; 2, 5, 8 and 1, 5, 9 2
In all the cases, median will be 5 years. 4
c=
15 a
34. (c) Mean age of minor children = = 5 years. From equation (i),
3
35. (b) Median age of minor children = 5 years. 4
36. (d) If we double each value of original frequency a+ =5
a
distribution, then mean, median and harmonic mean
remain same. Hence, option (d) is correct. a2 4
Since, in case observation since arithmetic mean, =5
a
median and harmonic mean is dependent of change a2 – 5a + 4 = 0
of origin but if we multiply the frequency of same a2 – 4a – a + 4 = 0
quantity, then, these are independent. a(a – 4) – 1(a – 4) = 0
37. (d) (a – 4) (a – 1) = 0
Growth Rate if a = 1 then c = 4
Year Birth rate Death rate
= B.R. – D.R. a = 4 then c = 1
1911 21 48.1 38.5 9.6 a = 1, c = 4 and b = 2
1921 31 46.4 36.3 10.1 40. (b) Required sum = 2(a)2 + 2(b)2 + 2(c)2
= 2(1)2 + 2(2)2 + 2(4)2
1931 41 45.2 31.2 14.0 = 2 + 8 + 32 = 42
1941 51 39.9 27.4 12.5 41. (d) The value of c is 4.
1951 61 41.7 22.8 18.9 42. (d) Mode = 3 (Median) – 2 (Mean)
1961 71 41.1 35.9 5.2 7 18 – 14 4
= 3(2) – 2
1971 81 37.1 14.8 22.3 3 3 3
It is clear from the above table that minimum growth 43. (b) x f xf
rate is 5.2 in the year 1961-1971.
38. (d) With the help of histogram, we can prepare frequency 1 3 3
polygon, frequency curve and frequency distribution. 2 5 10
39. (c) Male migrants category in pie diagram
3 9 27
45
= × 360° = 81° 4 f1 4f1
200
Solutions (Q. Nos. 40-42): 5 2 10
a<b<c Total 19 f1 50 4f1
Total numbers = 6
Increasing order a, a, b, b, c, c xi fi
6 6 Mean =
th term 1 th term fi
2 2
Median = 50 4f1
2 2.96 = [given]
19 f1
3rd term 4th term
= 56.24 + 2.96 f1 = 50 + 4f1
2
Statistics 391
A-

6.24 = 1.04 f1 5.5


f1 = 6 Required angle = × 360° = 82.5°
44. (a) I. Let the numbers be 1, 3, 5. If each number is 24
reduced by 2, then new number will be –1, 1, 3. Solutions (Q. Nos. 52-53):
Number of total bulbs = 100
G = 3 –1 1 3 –3 does not exist 7 + x + 40 + y + 10 + 2 = 100
II. If we increased each number by 2 then the x + y = 41 ... (i)
geometric mean of set is not increased by 2. Life of bulbs Cumulative
45. (b) Cumulative Frequency: The number of values less Number of bulbs
(in hours) Frequency
than the upper class boundary for the currect class.
So, given class is less than type class. 8 13 7 7
46. (c) We know that histogram is an equivalent graphical 13 18 x 7 x
representation of the frequency distribution and is
suitable for continuous random variables, where, the 18 23 40 47 x
total frequency of an interval is evenly distributed 23 28 y 47 x y
over the interval. 28 33 10 57 x y
Hence, both the given statements are correct.
47. (b) Arrange in ascending order: 33 38 2 59 x y
6, 7, 9, 11, 12, 15, 19 N 100
Now, n = 7 which is odd
The median life is 20 h, so median interval will be
7 1 (18-23).
Median = Value of th term
2 N
= Value of 4th term = 11 Here, l = 18, = 50
2
48. (c) Given (x1, y1), (x2, y2), ..., (xn, yn) are n pairs of c = 7 + x, f = 40, h = 5
positive numbers.
I. M(xn + yn) N
–C
2
y x1 y1 x2 y2 ... xn yn Median = l + ×h
= f
n
50 – 7 – x
x1 x2 ... xn y1 y2 ... yn 20 = 18 + ×5
= 40
n 50 – 7 – x
= M(xn ) + M(yn ) 2=
8
M(xn + yn ) = M(xn ) + M(yn ) 16 = 50 – 7 – x
1 x = 43 – 16
II. G(xn yn ) = x1 y1 x 2 y 2 x 3 y3 ... x n y n n x = 27
52. (d) Missing frequency ‘x’ is 27
1 1 Put the value of x in equation (i), we get
= x1 x 2 ... x n n y1 y2 ... yn n y = 41 – 27 = 14
= G(xn ) . G(yn ) 53. (c) Missing frequency ‘y’ is 14.
G(xn, yn ) = G(xn ) . G(yn ) 54. (c) Statement I :
Thus, both statements are true. A graph which displays the data by using vertical
49. (a) Arithmetic Mean of 100 numbers = 89.05 bars of various heights in rectangular shapes to
So, sum of the 100 numbers = 89.05 × 100 = 8905 represent frequencies. Such that there is no gap
Now, between consecutive bars and also the height of the
Two numbers 92 and 83 has been read as 192 and 33. rectangle.
Comparatively increased = (192 + 33) – (92 + 83) Statement II :
= 225 – 175 = 50 The height of the rectangle is determined by the
Sum is decreased by 50. frequency of the class it represents.
So, sum of 100 numbers = 8905 – 50 = 8855 So, both the statements are correct.
Again,
n x – Sum of wrong observations
8855 Sum of correct observations
Average of 100 numbers = = 88.55 55. (c) Correct A.M. =
100 n
50. (b) Total expenditure = 6 + 7 + 3 + 4 + 2.5 + 1.5
= ` 24 lakh 10 7.6 – 3 8 81
= = 8.1
360 10 10
Central angle of largest component = ×7 56. (a) Simple Bar Diagram represents only one variable. For
24
examples: Sales, Production, Population, Figures, etc.
= 105°
For various years may be shown by simple bar
51. (c) Difference between the expenditure of the largest and
charts. Since these are of the same width and vary
smallest component = 7 – 1.5 = 5.5
only in heights (or lengths). It becomes very easy for
EBD_7367
392
A- Statistics

readers to study the relationship. Simple bar diagrams (12)2 = 14 × H


are very popular in practice. A bar can be either
12 12 72
vertical or horizontal; vertical bars are more popular. H=
57. (a) 14 7
58. (c) Primary Data: 65. (d) Given data,
(i) Primary data are always original as it is collected
x 0 1 2 3 4
by the investigator.
(ii) Suitability of the primary data will be positive y 100 90 70 40 0
because it has been systematically collected. Here is only relation which satisfy the above data, i.e.
(iii) Primary data are expensive and time consuming. y = 100 – 5x – 5x2
(iv) Extra precautions are not required. (i) x = 0, y = 100 – 0 – 0 = 100
(v) Primary data are in the shape of raw material. (ii) x = 1, y = 100 – 5 – 5 = 100 – 10 = 90
(vi) Possibility of personal prejudice. (iii) x = 2, y = 100 – 10 – 20 = 100 – 30 = 70
Secondary Data: (iv) x = 3, y = 100 – 15 – 45 = 100 – 60 = 40
(i) Secondary data lacks originality. The investigator (v) x = 4, y = 100 – 20 – 80 = 100 – 100 = 0
makes use of the data collected by other 66. (c) A diagram consisting of rectangles, whose area is
agencies. proportional to the frequency of a variance and
(ii) Secondary data may or may not suit the objects whose width is equal to the class interval.
of enquiry. 67. (a) Given numbers are 10, 7, 8, 5, 6, 8, 5, 8 and 6
(iii) Secondary data are relatively cheaper. Arrange in ascending order
(iv) It is used with great care and caution. 5, 5, 6, 6, 7, 8, 8, 8, 10
(v) Secondary data are usually in the shape of Total term, n = 9 (odd)
readymade products. Now,
(vi) Possibility of lesser degree of personal prejudice.
So, both I and II are correct. 5 5 6 6 7 8 8 8 10
(i) Mean =
59. (d) Since, intelligence of students is an attribute, arithmetic 9
mean is not suitable method.
63
60. (b) In an increasing order arrangements of observations, = =7
the median will not be affected when the smallest and 9
the largest observations are removed. n 1 9 1
61. (d) Geographical Data it is simply a collection of (ii) Median = th term = th term
2 2
information that can describes objects and things
with relation to space. So, population figure of a state = 5th term = 7
classified into geographical data. (iii) Mode = 8 because of higher frequency term
Mean = Median
62. (c) x 2 3 4 5 6 68. (b) Given that, mean of 100 values is 45
f 2 7 14 22 30 100
Sum of 100 values, i.e. x = 45 × 100 = 4500
cf 2 9 23 45 75
i 1
Cumulative frequency corresponding to variate x = 5 According to condition,
is 45.
40 100
63. (d) S = {1, 2, 3, ..., n}
Statement I xi 15 xi – 5
i 1 i 41
n 1
Median : It is the middle term. So, , here it is 40 100
2
= xi 15 40 xi – 5 60
not define n is even or odd.
i 1 i 41
n 1
Hence, we cannot say that is median. It is not 40 100 100
2
= xi xi + 600 – 300 = xi + 300
correct.
i 1 i 41 i 1
Mode : It is the value that appears most of ten in a
set of data. = 4500 + 300 = 4800 [from equation (i)]
S = {1, 2, 3, ..., n} 4800
Here, all elements in S have same frequency. New mean = = 48
So, both the statements are not correct. 100
64. (d) Arithmetic mean, A = 14 69. (a) Continuous Variable :
Geometric mean, G = 12 It is available whose value is obtained by measuring.
We know that, Examples:
G2 = AH (i) Height of students in class.
(ii) Weight of students in class.
Statistics 393
A-

(iii) Weight of a person. According to question,


Here, weight of a person is continuous variable while Here, 10 + x + y = 20
educational qualification of the person is Discrete x + y = 20 – 10
Variables. x + y = 10 ... (i)
70. (b) Given the arithmetic mean of the above distribution
n
= 3.1 – cf
2
73. (a) Median = l + ×h
x f fx f
1 3 3
2 f 2f 20
– 4 x
2
3 6 18 14.4 = 12 + ×6
5
4 5 20
5 3 15
10 – 4 – x
N 17 f fx 56 2f 14.4 = 12 + ×6
5
6–x
fx 14.4 – 12 = ×6
Arithmetic Mean = 5
N
36 – 6x
56 2f 2.4 =
5
3.1 = 17 f
12 = 36 – 6x
52.7 + 3.1f = 56 + 2f 6x = 24
1.1f = 3.3 x=4
74. (b) Now, putting the value of x in equation (i), then,
3.3 4 + y = 10
f= =3
1.1 y=6
71. (d) Given average of 6 letters = 10 x 4 2
u v w x y z y 6 3
= 10
6 3x = 2y
According to the question, 75. (a) Given that,
z 60 u 10 v 20 w 30 x 40 y 50 Number of male employees (M) = 45
Number of female employees (F) = 15
6
u v w x y z 60 10 20 30 40 50 Mean salary of male employee x M = ` 5000
= 6 Total number of employees = (M + F) = 45 + 15 = 60
u v w x y z 210 210 Mean salary of employees x MF = ` 4800
= = 10 + = 45
6 6 6 Let mean salary of female employee is x F
p q
s t u v r By formula,
72. (a) =m
7 M xM F xF
x MF
p + q + r + s + t + u + v = 7m M F
(p – m) + (q – m) + (r – m) + (s – m) + (t – m)
+ (u – m) + (v – m) 45
5000 15 x F
4800 =
= (p + q + r + s + t + u + v) – 7m 60
= 7m – 7m = 0 4800 × 60 – 45 × 5000 = 15 × x F
Solutions (Q. Nos. 73-74):
x F = 4800 × 4 – 3 × 5000
Class interval Frequency Cumulative frequency = 300(16 × 4 – 50) = 300 × 14 = 4200.
76. (c) Given that, mean of 7 observations = 7
0 6 4 4
7 7
6 12 x 4 x 1
xi 7 xi 49 ... (i)
12 18 5 9 x 7 i 1 i 1
18 24 y 9 x y According to question,
24 30 1 10 x y Each observation is increased by 2. Then the new
mean,
20
EBD_7367
394
A- Statistics

histogram. It gives the idea about the shape of the


1 7 1 7 data distribution.
= xi 2 xi 2 7
7i 1 7 i 1 The two end-points of a frequency polygon always
lie on the x-axis.
1 1 83. (c) The decreasing order of mean are:
= (49 + 14) = × 63 = 9 Arithmetic mean > Geometric mean > Harmonic mean
7 7
77. (b) Discrete Variable: 84. (a) Statement I:
It is a variable whose value is obtained by counting. The Arithmetic Mean is obtained by sum of all the
Examples: elements of the data set and dividing by the number
(i) Number of students present. of elements and it is always uniquely defined.
(ii) Number of red marbles in a jar. Statement II:
The median is the middle element when the data set
(iii) Students’ grade level.
is arranged in order of magnitude.
Continuous Variable:
Mean, Median and mid-range always exist and are
It is a variable whose value is obtained by measuring.
unique.
Examples:
85. (c) Geometric mean of (x1, x2, x3, ..., xn)
(i) Height of students in class.
1
(ii) Weight of students in class.
(iii) Time it takes to get to school. = x1 x 2 xn n = X
(iv) Distance travelled between class. Geometric mean of (y1, y2, y3, ..., yn)
So, statement I and IV are examples of discrete 1
variables. = y1 y 2 yn n = Y
Solutions (Q. Nos. 78-80): Geometric mean of (x1 y1, x2 y2, ..., xn yn)
1 1
78. (d) C.I. x midvalues (xi) f fxi = x1 x 2 xn n y1 y 2 yn n
0 10 5 x 5x
1
10 20 15 x 8 15x 120 = x1 y1 x 2 y 2 x n y n n = XY
20 30 25 x–2 25x – 50
30 40 35 x–4 35x – 140 x1 x 2 xn
Geometric mean of , ,
N 4x 2 fxi 80x – 70 y1 y2 yn
1
Arithmetic Mean = 17.8
x1 x 2 xn n
fxi 80x – 70 = 1
17.8 =
N 4x 2 y1 y 2 yn n

17.8(4x + 2) = 80x – 70 1
71.2x + 35.6 = 80x – 70 x1 x 2 xn n X
8.8x = 105.6 =
y1 y 2 yn Y
x = 12

79. (b) 86. (b) Income Class Number


C.I. f C.F. Frequency
less than interval of persons
0 10 x 12 12
1500 1250 1500 100 20
10 20 x 8 20 32
1250 1000 1250 80 10
20 30 x 2 10 42
1000 750 1000 70 15
30 40 x 4 8 50
750 500 750 55 23
N 50
500 250 500 32 20
N 50 250 0 250 12 12
= 25
2 2
Hence, median class is 10-20. f1 – f 0
Mode = l + ×h
80. (b) New, modal class is 10-20, because it has maximum 2f1 – f0 – f 2
frequency, i.e. 20.
81. (c) A frequency distribution is a summary of the data set 20 – 12
in which the interval of possible values is divided = 250 + × 250
40 – 12 – 23
into sub-intervals known as class.
82. (b) In a frequency polygon, a line graph is drawn by 8
joining all the mid-points of the top of the bars of a = 250 + × 250
5
Statistics 395
A-

= 250 + 400 = 650 the same purpose as histograms. It is formed by


So, the modal class is 500-750. joining the mid-points of histogram.
87. (c) Statistical data: Statement II :
In statistics and quantitative research methodology, Frequency polygon touch the x-axis in its extreme left
a data sample is a set of data collected and or and extreme right of graph. See graph below:
selected from a different sources and good example
Histogram / Frequency Polygon
of statistical data – A list of annual incomes of the
members of a club. 40 —
7 4 2 4 35 —
88. (c) I. Mean of all observations = = 4.5
8 30 —
For median, first we arrange in ascending order
= 2, 2, 2, 2, 7, 7, 7, 7 25 —

Frequency
4th 5th 2 7 20 —
Median = = 4.5
2 2
II. Mode is both 2 and 7, since frequency of 15 —
occurrence is same, i.e. maximum frequency. 10 —
89. (c) Statement I :
Frequency polygons are a graphical device for 5—
understanding the shapes of distribution. They serve
0—
94 89 84 79 74 69 64 59 54 49 44 39 34
Value

90. (a) Mid-values Frequency x – 18


Classes d = x i – 18 ui = i fi u i
xi fi 2
11 13 12 3 –6 –3 –9
13 15 14 6 –4 –2 –12
15 17 16 9 –2 –1 –9
17 19 18 13 0 0 0
19 21 20 f 2 1 f
21 23 22 5 4 2 10
23 25 24 4 6 3 12
40 f f –8

fi u i f –8
Mean (x) = A + × h = 18 + ×2 95. (a) fi ( xi x ) 0 because sum of product of deviations
f 40 f
and frequencies from mean value will be 0.
Given mean = 18 96. (c) Observations can by arranged in ascending order.
f –8 6, 14, 15, 17, x + 1, 2x – 13, 30, 32, 34 and 43.
18 = 18 + ×2 Here, n = 10 [even]
40 f
Median
2f – 16 = 0 2f = 16
n n
16 Value of th term + Value of 1 th term
f= =8 2 2
2 =
2
f=8
91. (c) The modal class means that the class which has 10 10
maximum frequency. Value of th term + Value of 1 th term
2 2
92. (a) Here, Statement 1 is correct but Statement 2 is not =
correct. 2
93. (c) Both Statements 1 and 2 are suitable for represention Value of 5th term + Value of 6th term
of a pie diagram. =
2
94. (d) In bar diagram, the frequency is shown by the
height of the bar whereas in histogram the frequency x 1 2 x 13 3x 12
=
is shown by the area of the bar. So obtained graph 2 2
is histogram. But given, median = 24
EBD_7367
396
A- Statistics

3x 12 fiUi
= 24 100. (a) Mean A h
2 N
11
3x – 12 = 24 × 2 = 48 35 10= 37.2
50
3x = 48 + 12 101. (b) Median Class = 30 – 40.
3x = 60 l =30, F = 16, f =10, h = 10
x = 20
N
Hence, the value of x is 20. –F
102. (c) Median = l 2 h
97. (a) A > G > H between a and b
f
So, G is the Geometric mean between A and H.
98. (b) G.M. of a,b,c = 3 25 – 16
30 10 = 30 + 9 = 39.
1 10
abc 3 3 abc 27
f – f1
Also, a, b, c are in Geometric progression 103. (d) Mode = l h
So a = 1, b = 3, c = 9 2 f – f1 – f 2
Geometric mean of d,e,f = 4 Here, the maximum frequency is 12 their class is
1
40 – 50, then l = 40 f = 12, f1 = 10, f2 = 8
def 3 4 12 – 10
mode = 40 10
def = 64 2 12 – 10 – 8
Also, d, e, f are in Geometric progression 2
So, d = 2, e = 4, f = 8 40 10 = 40 + 3.33 = 43.33
6
Set = {1, 3, 9, 2, 4, 8} 104. (b) Arranging the data in ascending order.
Arithmetic mean (a – 3.5), (a – 3), (a – 2.5), (a – 2), (a – 0.5), (a + 0.5),
1 3 9 2 4 8 (a + 4), (a + 5).
6 Total terms = 8
4th term + 5th term
27 9 Medium =
= 4.5 > 3.5 2
6 2
So option (b) is correct a - 2 + a - 0.5
=
99. (b) Votes cast in favour of E 2
= 100000 – (36000 + 24000 + 18000 + 7000) 2a - 2.5
= 15,000 =
2
Angle allocated for party E in
= a – 1.25
360 So, option (b) is correct.
Pie chart = 15000 = 54°
100000 105. (d) (i + 1)xi = (i + 1) xi where i = 1, 2, 3, -------n
Solutions (Q. Nos. 100-103): n
Ui Cumulative å i (i +1) = 1.2 + 2.3 + 3.4 + 4.5 + -------meters
Class Mid F
Frequency
dI= xi –35 xI – 35 fI UI i=1
Values i 10
Frequency
0 – 10 5 4 –30 –3 –12 4 n

10–20 15 5 –20 –2 –10 9 = å Tn


n =1
20–30 25 7 –10 –1 –7 16
30–40 35 10 0 0 0 26 = å n (n +1)
40–50 45 12 10 1 12 38
50–60 55 8 20 2 16 46
= ån2 +ån
60–70 65 4 30 3 12 50 (n + 1) n ( 2n + 1) n (n + 1)
= +
FU 11 6 2
i i
N fi 50 1 é (n +1) n (2n + 1) n (n +1)ù
Mean = ê + ú
N 50 n ëê 6 2 ûú
25
2 2 (n + 1) é 2n + 1 ù
= ê + 1ú
2 êë 3 úû
Statistics A- 397

(n + 1) (2n + 4) But median = p


= z= p
2 3
New median = 8 – 1 = 7
(n + 1)(n + 2) Mean = 8
= Median = 7
3
Option (a) is correct.
So, option (d) is correct.
106. (a) Weighted arithmetic mean N 40
109. (c) 20
2 2
1´1 + 2 ´ 2 + 3´3 + 4 ´ 4 +----+10 ´10
=
1 + 2 + 3 +----10 Age No. of Cumulative
(in years) children frequency
(1)2 + ( 2)2 + (3)2 +----+ (10)2
= 5–6 4 4
1 + 2 + 3 +----10
6–7 7 11
(10)(10 + 1)(2 ´10 + 1) / 6 7–8 9 20
= (10´11) / 2 8 9 12 32
10 ´11´ 21´ 2 9–10 6 38
=
6 ´10 ´11 10–11 2 40
=7 Total 40
So, option (a) is correct.
107. (b) In a pie chart Median class i.e, (7 – 8)

Frequency N
Central angle 360 fpc
2 h
Total frequency Median =
fm
Here central angle = 135°
Frequency i.e., secured votes = 21960 20 11
Find valid votes in total. =7 1
9
21960 =7+ 1=8
135 360 Median age = 8 years
total valid votes
Hence, statement-1 is not correct.
21960 Total number of childrens in the age group 6 – 9 years
total valid votes = 360 = 7 + 9 + 12 = 28 percentage of children in the age
135
group 6 – 9 years.
= 58560
Option (b) is correct. 28
= 100 70
108. (a) Let 5 observations be x, y, z, p and q. 40
x y z p q
Hence statement-2 is correct.
Mean Mode class is (8 – 9)
5
But mean = 9 fm fmp
Mode = h
x + y + z + p + q = 45 2fm fmp fms
Also 1 is subtracted from each observation, then
x 1 y 1 z 1 p 1 q 1 12 9
Mean =8 1
5 2 12 9 6

x y z p q 5 45 5 1
= =8 8
5 5 3
=8 (Nearest integer)
New mean = 8 Hence model age = 8 years
Therefore statement-3 is correct.
Since n = 5 is odd
110. (c)
n 1 5 1
Median 3 1
2 2 111. (c) xi i = 1, 2, 3.... 11
i
i.e., 3rd observation. Then x1, x2, x3 ....... x11
z is median
EBD_7367
398
A- Statistics

1 1 1 1 4z = 5y
= 1, , , ,....
2 3 4 11 y z
...(ii)
n 1 4 5
Median = Here n is odd
2 From (i) and (ii)

11 1 x y z
Median = 6th observation
2 3 4 5

1 Let x, y and z be 3t, 4t and 5t


Median = then
6
83x 76y 85z 83(3t) 76(4t) 85(5t)
A.M. between four numbers
x y z 3t 4t 5t
1 1 1
1, , , = 81.50
2 3 4
113. (a) After an anging data in ascending order
1 1 1 12 6 4 3 x – 3.5, x – 3, x – 2.5. x –2 x – 0.5, x + 0.5
1
2 3 4 12 25 x + 4, x + 5
A.M 0.6
4 4 36
Then number of term is even
1
If A.M. of four numbers 7 n th n th
6 term 1 term
2 2
1 then average =
If A.M. of Eleven numbers 7 2
6
1 x 2 x 0.5
x 1.25
1 2 1 1 2
G.M. of two numbers = 1
2 2 1.414 114. (d) let data be
a, b, c, d, 20.5, e, F, G, H
1 after increasing largest 4 no. by 2
G.M. of 11 Numbers also 7
6 a, b, c, d, 20.5, e + 2, F + 2, G + 2, H + 2
H.M. of Eleven numbers Median remains the same
115. (c) Categorical data
11
1 2 3 4 5 6 7 8 9 10 11 108
116. (c) Required percentage = 100 = 30%
360
11 1
= Median
66 6 117. (c) Both the statements are correct.
Option (c) is correct. 30
112. (a) Total score class x = 83x 118. (d) Density of CI =
15 10
class y = 76y 119. (a)
class z = 85 z
di
83x 76y LI X1 di A x1 u1 F1 F1u i
20
Given = x y = 79
0 20 10 40 2 17 34
83x + 76y = 79x + 79y 20 40 30 20 1 28 28
4x = 3y 40 60 50 0 0 32 0
60 80 70 20 1 F F
x y 80 100 90 40 2 19 38
...(1)
3 4 F1 96 F 24 F

76y 85z A = 50
and = 81
y z
Statistics 399
A-

Fi u i 25
A h x =5
X = Fi 150
x = 30
24 f 20 123. (c)
50 = 50 + X F CF
96 F
1 8 8
F = 24
2 10 18
120. (c) Four slices 150°, 90°, 60°, 60°
3 F1 29
when 60° is deleted
4 F2 45
Remaining 150°, 90°, 60°
Total = 300° F1 = 29 – 18 = 11
While making Pie chart where 300° is taken as 100% F2 = 45 – 29 = 16
150
100 = 50%
300 124. (c) because 180 + 70 = 250
125. (b) According to the question,
90
100 = 30%
300 250 100
250 x % of 340 x 73.5%
340
60
100 = 20%
300 126. (c) Difference in the number of boys and girls studying
Mathematics = 150 – 90 = 60
50% of 360° will be 180°
largest slice as angle – 180° Difference in the number of boys and girls studying
Physics = 180 – 120 = 60
121. (a) Mode = 3 median – 2 mean
127. (b) Difference in the number of boys studying Mathemat-
mode = 3 (220) – 2 (270)
ics and Physics = 180 – 150 = 30
= 660 – 540 = 120
128. (a) Total number of boys = 150 + 180 + 70 + 136 = 536
50 Total number of girls = 90 + 120 + 250 + 204 = 664
122. (b) Number of Type A pencil = = 50
1
Ratio of number of boys to number of girls
x
Number of Type B pencil = 536 67
1.50 =
664 83
20 129. (a)
Number of Type C pencil = = 10
2
20000 5 25000 3 65000
Total money spent 130. (a) Mean salary =
9
Average = total no.of pencil = 1.25
100000 75000 65000
x 50 20 9
= 50 10 x = 1.25
240000
1.50 26666.6
9
x The number of workers earning salaries less than the
= 70 + x = 1.25 60 mean salary are category ‘B’ and ‘C’ workers
1.50
i.e. 5 + 3 = 8.
1.25
70 + x = 75.00 + x 131. (a) 16000 : 9000 = 16 : 9
1.50
132. (c) as mode can be more than one.
125
x x =5 133. (d) as perimeter of the complete pie diagram is equal to the
150 circumference which gives the length of the arcs of
each slice.
EBD_7367
400
A- Statistics

Arranging in ascending order, we get


134. (c) We know that geometric mean of two numbers = xy ,
144, 150, 155, 161, 165
where x and y are two numbers.
Therefore, median = 155
This means, xy 6 (given)
137. (a)
xy 6 xy 36
15 2.5 C
Now, we have geometric mean of three numbers = M 15 6 3 2
3
6 2 2 1
xyz , where x, y and z are three numbers.
A B 30 2
3
N 4
This means, xyz = 6 (given) 4 2.5 10 10 3 30 60
3 xyz 6 3
36 z 6 36 z 63 216 Hence, values of A, B and C are 10, –30 and –1
respectively.
216
z =6
36 16.1
138. (c) Required angle 360 57.96 58
100
135. (b) as secondary data is the data collected from sources
other than the user itself. 139. (c)
136. (b) Mean = sum of all the observations ÷ total number of 140. (c) Sum of 11 observations = 11 × 11 = 121
observations Sum of first 6 observations = 10.5 × 6 = 63
150 165 161 144 155 775 Sum of last 6 observations = 11.5 × 6 = 69
= 155
5 5 Value of 6th observations = 69 + 63 – 121 = 132 – 121 =
11
Median = the middle value after arranging in ascend-
ing or descending order in case of odd number of ob- 141. (b)
servations
Section-B : English

C HA P T E R
SYNONYMS
26
DIRECTIONS (Qs. 1-20) : Each question below consists of a 15. FLAGRANT
word in capital letters followed by four words or group of words. (a) Scandalous bad (b) Widely scattered
Select the word or group of words that is most similar in meaning (c) Particularly (d) Poisonous
to the words in capital letters. [2007 - II] 16. TRANSGRESS
(a) To convert
1. IMPETUS (b) To insult
(a) Courage (b) Impatience (c) To violate
(c) Arrogance (d) Driving energy (d) To hand over possession to another
2. PHILANDERER 17. FACADE
(a) Time waster (b) Spendthrift (a) Front of a building (b) Haughtiness
(c) Male flirt (d) Wanderer (c) Complexity (d) Hypocrisy
3. PALPABLE 18. PALL
(a) Trembling (b) Weak (a) To anger (b) To frighten
(c) Obvious (d) Foolish (c) To become boring (d) To darken
4. REPAST 19. SPLEEN
(a) Rest (b) Fond memories (a) Surface (b) Lustre
(c) Peacefulness (d) Meal (c) Bad temper (d) Smoothness
5. ABEYANCE 20. SWATHE
(a) Temporary suspension (a) To soothe (b) To wrap
(b) Abstinence (c) To sweep (d) To wash
(c) Hatred
(d) Memory DIRECTIONS (Qs. 21-27) : Each question in this section consists
6. TENOR of a word in capital letters followed by four words or group of
(a) Kindness (b) Effort words. Select the word or group of words that is most similar in
(c) Duration (d) Purport meaning to the words in capital letters. [2007 - I]
7. CRAVEN 21. COUNTERVAILING
(a) Carved (b) Cowardly (a) Argumentative (b) Compensating
(c) Out-dated (d) Obscure (c) Crying (d) Complaining
8. QUIBBLE 22. TO DEBILITATE
(a) To deny (b) To tremble (a) To outlaw (b) To support
(c) To evade truth (d) To examine (c) To weaken (d) To honour
9. PRIVATION 23. FERRET
(a) Dominance (b) Deprivation (a) Steal (b) Search
(c) Secrecy (d) Imprisonment (c) Forget (d) Fight
10. GLEAN 24. PECUNIARY
(a) To groom (b) To gather bit by bit (a) Pertaining to money
(c) To discover (d) To polish (b) Pertaining to politics
11. TACTILE (c) Pertaining to Geography
(a) Considerate (d) Pertaining to History
(b) Strong 25. REPRISAL
(c) Sharp (a) Loneliness (b) Retaliation
(d) Pertaining to the organs of touch (c) Remuneration (d) Pleasure
12. USURP 26. PROPRIETY
(a) To climb upon (a) Rightness (b) Brand name
(b) To yield (c) Cultivate land (d) Market share
(c) To seize power or position illegally 27. CAVIL
(d) To demand unlawfully high interest on a loan (a) To play (b) To complain
13. ENSCONCE (c) To prepare (d) To cut a joke
(a) To promote (b) To surround
(c) To settle comfortably (d) To honour DIRECTIONS (Qs. 28-44): Each of the following question consists
14. CHAUVINISM of a word in capital letters, followed by four words or group of
(a) Blind patriotism (b) Defeatism words. Select the word or group of words that is most similar in
(c) Political trickery (d) Obstinacy meaning to the word in capital letters. [2008-I]
EBD_7367
2
B- Synonyms

28. SLOTHFUL 45. Infertile : Desert


(a) Fraught with risk (b) Lazy (a) Diligent : Damp (b) Verdant : Jungle
(c) Revengeful (d) Lenient (c) Tree : Cut (d) Skin : Wrinkle
29. PREPOSTEROUS 46. Squander : Money
(a) Predefined (b) Obvious (a) Foresee : Predict (b) Achieve : Victory
(c) Absurd (d) Arduous (c) Dally : Time (d) Trespass : Land
30. BREATHTAKING 47. Altruism : Philanthropist
(a) Tiring (b) Costly (a) Publication : Writer (b) Soldier : War
(c) Spectacular (d) Trick (c) Self-denial : Puritan (d) Artist : Sculptor
31. INSIDIOUS 48. Skate : Rink
(a) Substantial (b) Unstable (a) Build : Column (b) Capital : Punishment
(c) Instantaneous (d) Cunning (c) Feathers : Bird (d) Play : Stadium
32. COMMISERATION
49. Sympathy : Merciless
(a) Sympathy (b) Simplification
(a) Pathetic : Pity (b) Reticence : Frank
(c) Slip of tongue (d) Arrogance
(c) Predictability : Routine (d) Noise : Ear
33. CONGENITAL
50. Society : Human
(a) Compulsory (b) Dating from birth
(c) Official (d) Pertaining to commerce (a) Spectators : Game (b) Jungle : Animal
34. REPRISAL (c) Kinship : Family (d) Rivals : Team
(a) Surprise (b) Award 51. Ship : Disembark
(c) Revision (d) Retaliation (a) Discharge : Board (b) Fail : Arrest
35. CELERITY (c) Train : Board (d) Horse : Dismount
(a) Grace (b) Fame 52. Exercise : Strength
(c) Slipperiness (d) Speed (a) Games : Contest (b) Defeat : Gain
36. QUAGMIRE (c) Business : Resource (d) Practice : Skill
(a) Rugged cliff (b) Lacking in morals 53. Education : Illiterate
(c) Quizzical (d) Whirlpool (a) Rude : Well-mannered (b) Experience : Naive
37. FRUGAL (c) Money : Miserly (d) Expel : Admit
(a) Quiet (b) Economical 54. Light : Opaque
(c) Shy (d) Generous (a) Air : Hermetic (b) Document : Author
38. BANAL (c) Sand : Desert (d) Wood : Tree
(a) Hostile (b) Commonplace DIRECTIONS (Qs. 55-72) : Each of the following questions
(c) Forbidden (d) Genial consists of a word in capital letters, followed by four words or
39. RETICENCE group or words. Select the word or group of words that is most
(a) Modesty (b) Caution
similar in meaning to the word in capital letters. [2008-II]
(c) Cognizance (d) Tendency to keep silent
40. PREMONITION 55. INNOCUOUS
(a) Sadness (b) Strength (a) Harmless (b) Benevolent
(c) Warning (d) Advice (c) Gentle (d) Non-infectious
41. EFFRONTERY 56. HANDY
(a) Affirmation (b) Commentary (a) Intact (b) Accessible
(c) Rudeness (d) Compliance (c) Supportive (d) Handful
42. PHLEGMATIC 57. LEGACY
(a) Thin (b) Even-tempered (a) Point (b) Principle
(c) Worldly (d) Huge (c) Inheritance (d) Teaching
43. SEMANTIC 58. DISSEMINATE
(a) Mild-mannered (a) Develop (b) Popular
(b) Pertaining to financial matters (c) Impart (d) Spread
(c) Pertaining to meaning of words 59. EXHORTED
(d) Tenable (a) Urged (b) Forced
44. PEDAGOGICAL (c) Persuaded (d) Demanded
(a) Related to evolutionary process 60. AFFLICTION
(b) Pertaining to teaching methods
(a) Problem (b) Hardship
(c) Pertaining to dictatorship
(c) Plight (d) Distress
(d) Pertaining to social customs
61. PANDEMONIUM
DIRECTIONS (Qs. 45-54): In each of the following question, a (a) Violence
related pair of words is followed by four pairs of words. Select (b) Quarrel
the pair that best expresses a relationship similar to that (c) Wild and noisy disorder
expressed in the original pair. [2008-I] (d) Verbal exchange
Synonyms 3
B-

62. DISPARATE 80. FELICITY


(a) Different (b) Contrary (a) Peace (b) Comfort
(c) Incongruous (d) Unequal (c) Faithfulness (d) Great happiness
63. DECADENT 81. UNRUFFLED
(a) Deceased (a) Simple (b) Indifferent
(b) Disingenuous (c) Sluggish (d) Calm
(c) Deteriorating 82. FELINE
(d) Belonging to the last decade (a) Feminine (b) Cat-like
64. FACTITIOUS (c) Ferocious (d) Ugly-and clumsy
(a) Amorous (b) Causing dissension 83. TENUOUS
(c) Controversial (d) Artificial (a) Being tenacious (b) Not substantial
65. STARTLED (c) Gross (d) Popular
(a) Jumped (b) Shocked 84. EXQUISITE
(c) Surprised (d) Offended (a) Highly refined (b) Too costly
66. INDIGENOUS (c) Extemporaneous (d) Exotic
(a) State-of-the-art (b) Indigent 85. WRESTED
(c) Native (d) Foreign (a) Took by force (b) Took away easily
67. GUSTO (c) Lost narrowly (d) Won easily
(a) Guts (b) Enthusiasm 86. EXPOSTULATED
(c) Enjoyment (d) Hope (a) Requested
68. VERACITY (b) Quarrelled ferociously
(a) Forcefulness (b) Simplicity (c) Remonstrated
(c) Truthfulness (d) Humour (d) Appealed with good reason
69. SUSTENANCE 87. PLACATE
(a) Courage (b) Nourishment (a) Implore (b) Flatter
(c) Health (d) Wisdom (c) Pacify (d) Compensate
70. RAPPORT 88. PERPETUAL
(a) Love and affection (b) Full support (a) Contagious (b) Continuous
(c) Harmonious accord (d) Sportsmanship (c) Perplexing (d) Perennial
71. INTRUDE 89. DAUNTLESS
(a) Fault (b) Disagree (a) Praiseworthy
(c) Interrupt (d) Leave (b) Effortless and smooth
72. IMPERATIVE (c) Fearless and determined
(a) Negligible (b) Indifferent (d) Unceasing
(c) Significant (d) Urgent 90. SEETHING
DIRECTIONS (Qs. 73-95) : Each of the following question (a) Dissatisfied (b) Be agitated
consists of a word in capital letters followed by four words or (c) Noisy (d) Soothing
group of words. Select the word or group of words that is most 91. STINGY
similar in meaning to the word in capital letters. [2009-I] (a) Violent (b) Miserly
(c) Cautious (d) Quarrelsome
73. GENIAL 92. EXULTATION
(a) Generous (b) Wonderful (a) Extinction (b) Anxiety
(c) Liberal (d) Friendly and cheerful (c) Jubilation (d) Expectation
74. REVAMP 93. DOUGHTY
(a) Retreat (b) Reconstruct (a) Dastardly (b) Deceptive
(c) Retrial (d) Retrace (c) Flabby (d) Valiant
75. RESCIND 94. OBVIATE
(a) Reunite (b) Repeal (a) Delaying the solution of a problem
(c) Reserve (d) Reproach (b) To remove a difficulty
76. RESPLENDENT (c) Make obstruction
(a) Wonderful (b) Dazzling (d) Supercede
(c) Beautiful (d) Respectful 95. SACRILEGE
77. CORPULENT (a) Offering sacrifice (b) Privilege
(a) Fleshy (b) Thin (c) Blasphemy (d) Being sacred
(c) Stout (d) Bony
78. ACRIMONY DIRECTIONS (Qs 96-100) : You are given four alternative
(a) Disagreement (b) Despondency meanings to each of the following idioms/phrases. Choose the
(c) Bitterness (d) Difference most appropriate one. [2009-I]
79. CANTANKEROUS 96. BLOW BY BLOW
(a) Bad tempered (b) Full of contempt (a) Eruptions in quick succession from a great volcano.
(c) Very sick (d) Very envious (b) Continuously raining with thunders.
EBD_7367
4
B- Synonyms

(c) Describe an event as it occurred in every detail 114. FURTIVELY


(d) A rapid decline of business leading to its closure (a) Quietly (b) Stealthily
97. BY HOOK OR BY CROOK (c) Delicately (d) Haphazardly
(a) In a completely insane manner 115. DISPARITY
(b) In whatever way one can (a) Enmity (b) Inequality
(c) Diversity (d) Segregation
(c) Being very evil and destructive 116. RECTIFY
(d) To be very tenacious (a) Undo (b) Record
98. CHOCK-A-BLOCK (c) Direct (d) Correct
(a) Filled to capacity. 117. ABSURD
(b) Not being able to fulfil one’s desires. (a) Feeble (b) Childish
(c) Blocking somebody’s path to progress. (c) Devious (d) Foolish
(d) Suffocation caused due to living in a congested area. 118. LURID
99. A FEATHER IN ONE’S CAP (a) Exaggerated and intentional
(a) Accumulating more money or property (b) Extravagant and silly
(b) Taking more burden on oneself (c) Unreasonable and malicious
(d) Shocking and violent
(c) Being felicitated for one’s artistic merits
119. STEADFAST
(d) Something achieved that constitutes a victory (a) Fixed (b) Stubborn
100. FOR GOOD (c) Tactful (d) Tolerant
(a) For a good cause (b) Temporarily 120. KINDLE
(c) Permanently (d) Seriously (a) Show kindness (b) Incite
DIRECTIONS (Qs. 101-120): Each of the following question (c) Helpful (d) Vindicate
consists of a word in capital letters, followed by four words or DIRECTIONS (Qs. 121-140) : Each of the following question
group of words. Select the word or group of words that is most below consists of a word in capital letters, followed by four
similar in meaning to the word in capital letters. [2009-II] words or groups of words. Select the word or group of words
101. MAGNIFICENT that is most similar in meaning to the word in capital letters.
(a) Magnanimous (b) Graceful [2010-I]
(c) Magical (d) Splendid 121. INDICTMENT
102. CORROBORATE (a) Arraignment (b) Entrapment
(a) Reward (b) Confirm (c) Indoctrination (d) Inducement
(c) Correct (d) Contradict 122. ABSTEMIOUS
103. ELUCIDATE
(a) Describe (b) Annotate (a) Resistant (b) Temperate
(c) Explain (d) Complicate (c) Superstitions (d) Careful
104. APPREHENSION 123. PROBITY
(a) Alertness (b) Anticipation (a) Courtesy (b) Promptness
(c) Anxiety (d) Doubt (c) Efficiency (d) Uprightness
105. DIFFIDENT 124. DAINTY
(a) Bold (b) Disobedient (a) Carefree (b) Feminine and happy
(c) Hesitate (d) Improper (c) Rich and famous (d) Small and graceful
106. DIGRESS 125. GARISH
(a) Deviate (b) Come back (a) Beautifully decorated (b) Boldly arranged
(c) Remove (d) Slow down
(c) Unpleasantly gaudy (d) Carefully prepared
107. CONTENDING
(a) Competing (b) Antagonizing 126. INNOCUOUS
(c) Fighting (d) Warning (a) Abominable (b) Harmless
108. ACCOLADE (c) Harmful (d) Useful
(a) Wish (b) Congratulate 127. ABJURE
(c) Comment (d) Award (a) Relinquish (b) Recant
109. SALUBRIOUS (c) Retract (d) Renounce
(a) Convenient (b) Warm 128. EQUANIMITY
(c) Healthy (d) Chilly (a) Calm (b) Indifference
110. RESTIVE (c) Silence (d) Satisfaction
(a) Impatient (b) Inactive 129. PONDEROUS
(c) Lonely (d) Peaceful
111. PREPOSTEROUS (a) Reasonable (b) Thoughtful
(a) Incredible (b) Outrageous (c) Persuasive (d) Laboured
(c) Astonishing (d) Exorbitant 130. GNARLED
112. PERSUASIVE (a) Weak and bent (b) Wrinkled and dark
(a) Passionate (b) Impressive (c) Rough and twisted (d) Swollen and hard
(c) Influential (d) Convincing 131. INURED
113. ALLEVIATE (a) Being indifferent (b) Exposed
(a) Aggravate (b) Amend (c) Being sensitive (d) Accustomed
(c) Maintain (d) Mitigate
Synonyms B-5
132. DISDAINFUL 150. REVELATION
(a) Discriminatory (b) Discursive (a) Anticipation (b) Imagination
(c) Dispassionate (d) Dismissive (c) Revel in (d) Disclosure
133. INNUENDOES 151. OSTRACISE
(a) Indications (b) Inspiring quotes (a) Censure (b) Sentence
(c) Witty remarks (d) Insinuations (c) Banish (d) Berate
134. APPRAISAL 152. APPALLED
(a) Estimation (b) Enlightenment (a) Shocked (b) Saddened
(c) Appropriation (d) Application (c) Scared (d) Alarmed
135. INDOMITABLE 153. IMBECILITY
(a) Extraordinary (b) Uncontrollable (a) Stupidity (b) Rusticity
(c) Dominating (d) Unyielding (c) Verbosity (d) Incoherence
136. CONTRIVED 154. COLLUSION
(a) Devised (b) Defaced (a) A conflict (b) A secret agreement
(c) Defected (d) Deferred (c) A consultation (d) A misunderstanding
137. DEFT 155. HESITANT
(a) Dangerous and swift (b) Defiant (a) Antagonistic (b) Grumbling
(c) Skilful and quick (d) Slow and steady (c) Contradict (d) Undecided
138. RIFE 156. DECEPTIVE
(a) Restive (b) Shake (a) Disargreeable (b) Misleading
(c) Troublesome (d) Widespread (c) Mistake (d) Debatable
139. GHASTLY 157. PALPABLE
(a) Painful (b) Exciting (a) Obvious (b) Immense
(c) Dreadful (d) Unforgettable (c) Sufficient (d) Hidden
140. BROACHED 158. LAID-BACK
(a) Admonished (b) Advised (a) Lie in wait (b) Sorry state
(c) Discussed (d) Raised (c) Lame (d) Easy-going
DIRECTIONS (Qs. 141-160) : Each question in this section 159. SAUNTERING
consists of a word in capital letters followed by four words or (a) Jogging (b) Brisk walking
groups of words. Select the word or groups of words that is most (c) Travelling (d) Strolling
similar in meaning to the word in capital letters. [2010-II] 160. POMPOUS
(a) Grandiose (b) Polished
141. CRASS (c) Modest (d) Skilled
(a) Casual (b) Coarse
(c) Stupid (d) Sterile DIRECTIONS (Qs. 161-182) : Each question in this section
142. ACCOMPLISH consists of a word or group of words in capital letters, followed
(a) Amass (b) Acquire by four words or group of words. Select the word or group of
(c) Adhere (d) Achieve words that is most similar in meaning to the word or group of
143. CURSORY words in capital letters. [2011-I]
(a) Little (b) Quick 161. FLYING COLOURS
(c) Eager (d) Tender (a) Enviable (b) Notable success
144. ENVISAGED (c) Being pleasant (d) Highly ambitious
(a) Ensured (b) Idealized 162. SANCTUM
(c) Contemplated (d) Created (a) Environment (b) The holy part
145. FICKLE (c) Corridor (d) Room for pilgrims
(a) Awkward (b) Incosistent 163. FRAIL
(c) Silly (d) Ugly (a) Unhealthy (b) Weak
146. RESENTMENT (c) Ugly (d) Tender
(a) Annoyance (b) Anger 164. HOBNOB
(c) Disagreement (d) Dismissal (a) Nagging (b) Friendly chat
147. OBDURATE (c) Quarrel (d) Pestering manner
(a) Angry (b) Calm 165. DILLY DALLYING
(c) Obsessed (d) Adamant (a) Quarrelling
148. OBSCURE (b) Arguing with each other
(a) Unknown (b) Neglectful (c) Waiting for orders
(c) Occasional (d) Old (d) Wasting time
149. RELUCTANT 166. HUSH UP
(a) Averse (b) Forego (a) Blow up (b) To keep quiet
(c) Redundant (d) Amenable (c) To remove (d) Wind up
EBD_7367
6
B- Synonyms

167. GRIEVANCES 185. CONDEMN


(a) Complaints (b) Problems (a) Censure (b) Despair
(c) Obstacles (d) Hardships (c) Kill (d) Hit
168. AFFINITY 186. DEFECT
(a) Admiration (b) Adoration (a) Loss (b) Harm
(c) Respect (d) Intimacy (c) Shortcoming (d) Delicate
169. IGNOMINY 187. JEALOUS
(a) Condemnation (b) Disgrace (a) Envious (b) Unhappy
(c) Failure (d) Criticism (c) Regretful (d) Remorse
170. ALLEVIATE 188. WEARY
(a) Eradicate (b) Remove (a) Careless (b) Shivering
(c) Understand (d) Lessen (c) Troubled (d) Weak
171. IMPERCEPTIBLY 189. PLAUSIBLE
(a) Extremely (b) Invisibly (a) Pleasing to the taste (b) Seemingly true
(c) Inconceivably (d) Intensely (c) Manageable (d) Cannot be proved
172. DOLEFUL 190. CREDITABLE
(a) Sobre (b) Depressed (a) Able to lend money (b) Bringing praise
(c) Dreary (d) Mournful (c) Able to repay a loan (d) Fit to believed
173. MODULATE 191. SALIENT
(a) Set (b) Tune (a) Most important (b) Salt-like taste
(c) Balance (d) Adjust (c) Pleasing (d) Satisfactory
174. PRESUMPTUOUS 192. FRAGILE
(a) Bold (b) Egotistical (a) Soft (b) Smooth
(c) Arrogant (d) Audacious (c) Flexible (d) Easily broken
175. RHYTHM 193. POIGNANT
(a) Flow (b) Measure (a) Bitter (b) Deep
(c) Tempo (d) Regular pattern (c) Very painful (d) Sharp
176. PRECIPITATE 194. OUTLOOK
(a) Hasten (b) Advance (a) Looking out (b) Prospect
(c) Arrive (d) Instigate (c) Outward appearance (d) Scrutiny
177. OUTLAST 195. FORTITUDE
(a) To come to an end (b) To fight till the end (a) Fortune (b) Fortification
(c) To make a last attempt (d) To endure longer than (c) Bravery (d) Breakthrough
178. INTERREGNUM 196. RETICENT
(a) Pause (b) Interference (a) Sensitive (b) Secretive
(c) Intrusion (d) Prevention (c) Not feeling well (d) Not saying much
179. DEMOLISH 197. RECIPROCAL
(a) Abolish (b) Overthrow (a) Formal (b) Favourable
(c) Destroy (d) Overturn (c) Mutual (d) Sustainable
180. INITIATE 198. APPRISE
(a) Enlighten (b) Begin (a) Inform (b) Estimate
(c) Guide (d) Lead (c) Admire (d) Require
181. STROLL 199. CONSENSUS
(a) Exercise (b) Jogging (a) Awareness (b) General agreement
(c) A ride (d) A walk (c) Careful observation (d) Praiseworthy
182. LANGUID 200. UNCEREMONIOUS
(a) Absent minded (b) Negligent (a) Impolite (b) Informal
(c) Sluggish (d) Thoughtless (c) Incomplete (d) Irregular
DIRECTIONS (Qs. 183-202) : Each question in this section 201. TREASON
consists of a word in capital letters followed by four words or (a) Absurdity (b) Disloyalty
groups of words. Select the word or group of words that is most (c) Dishonesty (d) Deception
similar in meaning to the word in capital letters. [2011-II] 202. AMIABLE
(a) Popular (b) Generous
183. HOSTILITY (c) Friendly (d) Reasonable
(a) Illness (b) Enmity
(c) Disturbance (d) Derogatory DIRECTIONS (Qs. 203-219) : In this section, select the word or
184. INDIFFERENT group of words that is most similar in meaning to the underlined
(a) Dissimilar (b) Various word or group of words in the given sentence. [2012-I]
(c) Interference (d) Unconcerned 203. I never believed that it was the authentic signature of the
Prime Minister.
Synonyms B- 7
(a) hand-written (b) genuine DIRECTIONS (Qs. 220-234) : Each question in this section
(c) proper (d) authoritative consists of a word or a group of words in capital letters followed
204. Valiant Vicky used to boast of his bravery to his beloved by four words or group of words: Select the word or group of
wife. words that is most similar in meaning to the word or words in
(a) cry (b) abuse capital letters. [2012-II]
(c) hate (d) brag
205. I can no longer be satisfied with fictitious good conduct. 220. PROHIBIT
(a) based on facts (a) Forbid (b) Forfeit
(b) based on imagination (c) Forsake (d) Forbear
(c) based on works of fiction 221. DUBIOUS
(d) based on feudal ties (a) Dismal (b) Doubtful
206. Mass murder is very often a result of communal frenzy. (c) Derogatory (d) Devilish
(a) patricide (b) fratricide 222. ECCENTRIC
(c) regicide (d) genocide (a) Destructive (b) Deceitful
207. The two executioners approached the tree with a red-mark (c) Conformist (d) Unconventional
on its side. 223. CONTRADICT
(a) executive engineers (a) Request politely (b) Deny emphatically
(b) explorers (c) Talk abusively (d) Contempt
(c) experimenters 224. GIVE UP
(d) those who inflict capital punishment (a) Abandon (b) Present
208. The maiden speech of the young member of the Lok Sabha (c) Lift (d) Scatter
was very much appreciated by the people. 225. SPECIFIC
(a) farewell speech (b) short speech (a) Proper (b) Uncommon
(c) first speech (d) speech about women (c) Noteworthy (d) Precise
209. Please do not interfere with my work. 226. ERADICATE
(a) meddle (b) help (a) Put up (b) Remove
(c) object (d) copy (c) Soften (d) Suppress
210. This job is very tedious. 227. PROBABLE
(a) tiresome (b) dull (a) Certain (b) Indefinite
(c) interesting (d) exciting (c) Likely (d) Temporary
211. She takes after her mother. 228. ADHERE
(a) follows (b) comes after (a) Act reluctantly (b) Give allegiance
(c) resembles (d) imitates (c) Waver frequently (d) React promptly
212. He is very intelligent, but ill-favoured by nature. 229. INFER
(a) unlucky (b) weak in health (a) Conclude (b) Calculate
(c) short-tempered (d) ugly (c) Debate (d) Suggest
213. People thronged to pay homage to the departed leader. 230. HESITATE
(a) humility (b) tribute (a) Afraid to talk
(c) obedience (d) allegiance (b) Unwilling to act
214. He kept his eyes peeled and his ears pricked for some (c) Unwilling to speak
important clue. (d) Unable to decide quickly
(a) hint (b) inkling 231. EXPLICIT
(c) intimation (d) signal (a) Immediate (b) Plainly stated
215. The spectators looked at the batsman in amazement when (c) Marked (d) Revealing the secret
he hit sixer after sixer. 232. SUFFICIENT
(a) shock (b) wonder (a) Adequate (b) Complete
(c) surprise (d) suspicion (c) Full (d) Frugal
216. Very few of our batsmen have any real consistency. 233. REPRESS
(a) constancy (b) competence (a) Express (b) Impress
(c) permanence (d) uniformity (c) Curb (d) Confuse
217. He has an electrifying presence. 234. VINDICTIVE
(a) attractive (b) fearsome (a) Cruel (b) Harsh
(c) exciting (d) disturbing (c) Revengeful (d) Rude
218. For better health we must refrain from smoking. DIRECTIONS (Qs. 235-249) : In each of these questions, a
(a) dissuade (b) desist word or a phrase has been underlined followed by words or
(c) prevent (d) curb phrases listed (a), (b), (c) and (d). Choose the most appropriate
219. The article was so well-written that it merited careful study. word/phrase closest in meaning to the underlined word or phrase
(a) deserved (b) encouraged and mark your Answer Sheet accordingly. [2013-I]
(c) prompted (d) supported
EBD_7367
8
B- Synonyms

235. He disdained to notice the insult. (b) Thorough and rigorous


(a) was very proud (b) refused (c) Taking risk
(c) was unhappy (d) was too reluctant (d) Painful and sorrowful
236. He gave an evocative rendering of his recent poems. 251. SUBSTANTIAL
(a) thrilling (b) enchanting (a) Delicate (b) Graceful
(c) emotional (d) incantatory (c) Sensational (d) Significant
237. Expeditious steps should be taken to complete the work in 252. INDOLENT
two weeks. (a) Cautious (b) Lazy
(a) unmerciful and punitive (b) strict and forceful (c) Unintelligent (d) Stubborn
(c) disciplinary (d) prompt and efficient 253. VEHEMENTLY
238. Why did you make that, flippant remark ? (a) Devoutly (b) Serenely
(a) highly critical (c) Hysterically (d) Forcefully
(b) not showing deserved respect 254. OBLIVIOUS
(c) casual (a) Narrow-minded (b) Daring
(d) indifferent (c) Stubborn (d) Unaware
239. That is not an occasion to make an impromptu speech. 255. FEIGNED
(a) without preparation (b) thoughtless (a) Pretended (b) Played
(c) improper (d) long and boring (c) Developed (d) Failed
240. The weavers have to do monotonous work. 256. PENURY
(a) autonomous (b) irksome (a) Poverty (b) Petty
(c) exhausting (d) repetitive (c) Phony (d) Pathetic
241. There is not a single word that is redundant in the report. 257. AFFECTATION
(a) unimportant (b) not needed (a) Adoration (b) Artificiality
(c) bombastic (d) flowery (c) Appreciation (d) Proficiency
242. The country’s economy is beginning to look up now. 258. STIFLE
(a) look clear (b) go down (a) Starve (b) Stumble
(c) remain static (d) improve (c) Smother (d) Stagger
243. The old man shows no signs of infirmity even though he is 259. BESEECH
eighty years old. (a) Request (b) Overwhelm
(a) lack of fitness (b) feebleness (c) Bless (d) Promise
(c) fickleness (d) indolence 260. STALEMATE
244. It needs an expert to decipher the secret message sent to (a) Degeneration (b) Deadlock
the Army Officers during war time. (c) Exhaustion (d) Settlement
(a) swindle (b) decode 261. IMPROMPTU
(c) make up (d) defy (a) Important (b) Impressive
245. Bana in Sanskrit, Lyly in English and Prasad in Hindi are (c) Inspiring (d) Offhand
celebrated for their florid style. 262. LUCRATIVE
(a) ornate (b) rich (a) Profitable (b) Important
(c) refined (d) rambling (c) Challenging (d) Worthwhile
246. My brother’s zest for the new project was infectious. 263. FORTHRIGHT
(a) greed (b) enthusiasm (a) Honest and outspoken (b) Serious and stubborn
(c) attraction (d) fascination (c) Strong and brave (d) Willing and determined
247. The eyewitness testimony was incontrovertible. 264. REPRIMAND
(a) disputable (b) debatable (a) Praise (b) Reminder
(c) unacceptable (d) unquestionable (c) Rebuke (d) Reward
248. His forthright behaviour shows that he is honest but he 265. PUNCTILIOUS
seems rude to some people. (a) Serious (b) Careful
(a) courteous (b) straightforward (c) Punctual (d) Hardworking
(c) tactful (d) correct 266. DISCREET
249. The minister’s speech was comprehensive and was greatly (a) Mature (b) Intelligent
appreciated. (c) Clever (d) Prudent
(a) praiseworthy (b) full of fine words 267. MOMENTOUS
(c) covering all aspects (d) understandable (a) Time-tested (b) Momentary
DIRECTIONS (Qs. 250-269) : Each of the following questions (c) Very important (d) Hectic
consists of a word in capital letters, followed by four words or 268. PORTEND
group of words. Select the word or group of words that is most (a) Profess (b) Portray
similar in meaning to the word in capital letters. [2013-II] (c) Think (d) Foreshadow
269. ODIUM
250. PAINSTAKING (a) Illness (b) Hatred
(a) Feeling panic (c) Oddity (d) Devious
Synonyms B-9

DIRECTIONS (Qs. 270-279) : In this section, you find a number 281. SEQUEL
of sentences, parts of which are underlined. For each underlined (a) ending (b) beginning
part, four words/ phrases are listed below. Choose the word/ (c) continuation (d) similarity
phrase nearest in meaning to the underlined part and blacken 282. PERPETUAL
the corresponding space on the Answer Sheet. (a) constant (b) real
[2014-I] (c) mistaken (d) painful
283. ASSENT
270. She is a woman of sterling qualities.
(a) despatch (b) climb
(a) interesting (b) genuine
(c) flavour (d) agreement
(c) irritating (d) exciting
284. DEFIANCE
271. Although the boys in his class were naughty, he never
(a) attack (b) disobedience
resorted to corporal punishment.
(a) harsh (b) physical (c) protection (d) shyness
(c) unjust (d) general 285. HOSTILITY
272. He wanted to mitigate his burdens. (a) kindness (b) enmity
(a) lessen (b) increase (c) entertainment (d) illness
(c) postpone (d) leave 286. INDIFFERENT
273. She adjusted quite well with her husband's idiosyncrasies. (a) similar (b) various
(a) peculiar habits (b) bad habits (c) unconcerned (d) Shy
(c) weaknesses (d) stupid manners 287. CONDEMN
274. The Deputy Inspector General made a perfunctory inspection (a) censure (b) approve
of the police station. (c) qualify (d) despair
(a) thorough and complete 288. CONFIDENT
(b) superficial (a) full (b) friendly
(c) done as a routine but without interest (c) sure (d) secret
(d) intensive 289. DEFECT
275. The decision to drop the atom bomb on Hiroshima was a (a) truth (b) deception
grave one. (c) shortcoming (d) loss
(a) serious (b) momentous , 290. JEALOUS
(c) instinctive (d) impulsive (a) envious (b) happy
276. A scientist generally carries out his investigations) (c) prisoner (d) enthusiastic
empirically. 291. TRIP
(a) intuitively (a) journey (b) plant
(b) verbally (c) design (d) press
(c) through written communication
DIRECTIONS (Qs. 292-299): Each question in this section
(d) by observation and experiment
consists of a word in capital letters followed by four words as
277. He is employed in an ordnance factory.
(a), (b), (c) and (d). Select the word which is most nearly the
(a) orthodox
same in meaning as the original word and mark the correct
(b) arms and ammunition
response as (a), (b), (c) or (d) as the case may be, in your Answer
(c) electrical and electronic
Sheet. [2015-I]
(d) ordinary and common
278. He is a sycophant who tries to win over politicians. 292. DEPLORE
(a) a psychologist (a) lose heart (b) entreat
(b) an opportunist (c) regret (d) malign
(c) an unscrupulous man 293. MOTIVATION
(d) a flatterer (a) inducement (b) emotion
279. I cannot believe in the veracity of his statement. (c) ambition (d) incitement
(a) truth (b) usefulness 294. RESIDUE
(c) sincerity (d) falsity (a) remainder (b) nothing
DIRECTIONS (Qs. 280-291) : Each question in this section (c) recede (d) little
consists of a word in capital letters followed by four words or 295. PERPETUAL
phrases as (a), (b), (c) and (d). Select the word or phrase which (a) perfect (b) confused
is most nearly the same in meaning as the original word and (c) never ending (d) seasonal
mark the correct response as (a), (b), (c) or (d) as the case may 296. ASSENT
be, in your Answer Sheet. [2014-II] (a) climb (b) confirm
(c) answer (d) agree
280. ASPIRE
297. DEFIANCE
(a) breathe (b) stairs
(c) hope for (d) thorn (a) insult (b) denial
(c) degradation (d) resistance
EBD_7367
10
B- Synonyms

298. EMANCIPATE 311. Devotees believe that God dwells in their heart.
(a) liberate (b) release (a) lives (b) insists
(c) acquit (d) unchain (c) travels (d) enters
299. HOSTILITY 312. Not everyone can respond to a difficult question quickly.
(a) hospitality (b) jealousy (a) discuss (b) argue
(c) enmity (d) envy (c) answer (d) deny
DIRECTIONS (Qs. 300-304): For the expression which has been 313. The sage did not want to be bothered with mundane
underlined in each of the following sentences, choose the concerns.
response (a), (b), (c) or (d) which most nearly expresses its (a) worldly (b) meaningless
meaning. [2015-I] (c) trivial (d) superfluous
300. Moralists are usually persons who abstain from alcoholic 314. Mountaineering in bad weather is dangerous.
drinks. (a) threatening (b) Shaky
(a) teetotallers (b) ascetics (c) perilous (d) Slippery
(c) pedants (d) celibates 315. Stellar groupings tend to. be unlimited.
301. The chairman is quick to find fault and is hard to please. (a) lengthy (b) heavenly
(a) frivolous (b) facetious (c) huge (d) infinite
(c) fastidious (d) ferocious 316. One who rules with unlimited power is called a dictator.
302. Scriptural injunctions should not be opposed or treated (a) anarchist (b) autocrat
with contempt. (c) egoist (d) sychophant
(a) flouted (b) flounced 317. He was not ready with his annual accounts.
(c) floundered (d) flaunted (a) yearly (b) important
303. A fortress on a commanding height for defense of a city is (c) monthly (d) permanent
called 318. Society cannot depend upon a fanatic for guidance.
(a) citadel (b) metropolis (a) optimist (b) martyr
(c) megapolis (d) headquarters (c) bigot (d) anarchist
304. Nostalgia is
319. A busy person cannot waste his time on trivial issues.
(a) anxiety about future
(a) unimportant (b) rude
(b) feeling of insecurity
(c) crude (d) tribal
(c) longing for a period in the past
(d) an allergy to certain foods DIRECTIONS (Qs. 320-328) : Each item in this section consists
of a word in capital letters followed by four words or groups of
DIRECTIONS (Qs. 305-319) : Each item below consists of a words. Select the word or groups of words that is most similar in
sentence with an underlined word followed by four words or meaning to the word in capital letters. [2016-I]
group of words. Select the word or group of words that is most
320. AMBIGUOUS
similar in meaning to the underlined word. [2015-II] (a) contrasting (b) connivance
305. All these items have been marked down. (c) vague (d) wilful
(a) reserved (b) packed up 321. ELUCIDATE
(c) reduced in price (b) entered (a) clarify (b) calculate
306. How you have the effrontery to ask for another loan? (c) summarise (d) update
(a) right (b) impudence 322. MONOTONOUS
(c) heart (d) courage (a) dreary (b) dreadful
307. There was a devastating attack on his work. (c) single-minded (d) monologue
(a) terrible (b) casual 323. KINDLE
(c) unethical (b) motivated (a) make fun of (b) excite
(c) very kind (d) kind-hearted
308. The committee conducted an exhaustive inquiry.
324. PALATIAL
(a) time-consuming (b) complicated
(a) very clean (b) very special
(c) renewed (d) thorough
(c) sense of taste (d) magnificent
309. He is just laying up a lot of trouble for himself.
325. TACTFUL
(a) clearing (b) accumulating (a) diplomatic (b) indifferent
(c) accepting (d) removing (c) intelligent (d) deceitful
310. He fought the demon with all his might. 326. VORACIOUS
(a) heaviness (b) strength (a) very bad (b) insatiable
(c) density (d) popularity (c) stingy (d) malicious
Synonyms 11
B-

327. STRICTURE (c) One who is employed in food serving


(a) strictness (b) stinging (d) One engaged in selling
(c) discipline (d) censure 333. They will not admit children under fourteen.
328. OBEISANCE (a) avow (b) receive
(a) homage (b) pilgrimage (c) accept (d) concede
(c) subjugation (d) obligation 334. The jewels have been stolen from her bedroom.
DIRECTIONS (Qs 329-338): Each item in this section consists (a) embezzled (b) asserted
of a sentence with an underlined word/words followed by four (c) yielded (d) abdicated
words. Select the option that is nearest in meaning to the 335. The soldier showed an exemplary courage.
underlined word/words and mark your response on your Answer (a) flawed (b) faulty
sheet accordingly [2018-I] (c) ideal (d) boisterous
336. They served fruits after the dinner.
329. A truly respectable old man is a ripe person. (a) assisted (b) obliged
(a) senior (b) mature (c) waited (d) offered
(c) perfect (d) seasoned
337. The committee should recommend his name to the
330. The soldiers repulsed the enemy.
government.
(a) defeated (b) destroyed
(a) praise (b) advise
(c) rejected (d) repelled
(c) counsel (d) suggest
331. She deftly masked her feelings.
338. Can medicines save us from death?
(a) hid (b) flaunted (a) hide (b) rescue
(c) oblique (d) obscured (c) protect (d) liberate
332. Vendors must have licence.
(a) One who drives a car
(b) One who works in a hospital
EBD_7367
B- 12 Synonyms

HINTS & SOLUTIONS


1. (d) ‘Impetus’ means ‘something that encourages a process 39. (d) The word Reticence means the state of being reticent,
or activity.’ or reserved, especially with regard to speaking freely;
2. (c) ‘Philanderer’ means ‘a man who has sexual relations restraint; silent.
with different women.’ 40. (c) The word Premonition means a strong feeling that
3. (c) ‘Palpable’ means ‘easily noticed’. something is about to happen, especially something
4. (d) ‘Repast’ means ‘meal’. unpleasant.
5. (a) ‘Abeyance’ means ‘not being used for a period of time’. 41. (c) The word Effrontery means shameless or impudent
6. (d) ‘Tenor’ means ‘the general character or meaning of boldness; rudeness; barefaced; audacity
something’. 42. (b) The word Phlegmatic means not easily excited to action
7. (b) ‘Craven’ means ‘lacking courage’. or display of emotion; apathetic; sluggish.
8. (c) ‘Quibble’ means ‘to argue or complain about an 43. (c) The word Semantic means of, relating to, or arising
unimportant detail’. from the different meanings of words or other symbols.
9. (b) ‘Privation’ means ‘lack of basic things that people 44. (b) The word Pedagogical means the art or science of
need’. teaching; education; instructional methods.
10. (b) ‘Glean’ means ‘to gather after a harvest often with 45. (b) Infertile describes the characteristic of a desert,
difficulty’. similarly, Verdant describes the characteristic of a jungle
11. (d) ‘Tactile’ means ‘connecting with your sense of touch’. i.e. wilderness full of plant and animal life.
12. (c) ‘Usurp’ means to seize power or position of somebody 46. (c) Squander means to spend thoughtlessly and the word
else without right. dally means to waste time; delay.
13. (c) ‘Ensconce’ means ‘to make yourself comfortable and 47. (c) The word altruism is the synonym of philanthropy
safe’. which means the principle or practice of unselfish
14. (a) ‘Chauvinism’ means an aggressive and unreasonable concern for or devotion to the welfare of others and
belief or patriotism’. the word self denial means the sacrifice of one's own
15. (a) ‘Flagrant’ means ‘shocking is an unacceptable way’. desires; unselfishness.
16. (c) ‘Transgress’ means ‘to go beyond the limit that is 48. (d) Just the way skates are used in the rink for skating
acceptable.’ similarly a game is played in a stadium.
17. (a) ‘Facade’ means ‘front of a building.’ 49. (b) Merciless is the antonym of sympathy whereas the
18. (c) ‘Pall’ means ‘to become less interesting’ word reticence means reserve or restraint that is the
19. (c) ‘Spleen’ means ‘anger’. antonym of frank.
20. (b) ‘Swathe’ means ‘to wrap or cover’. 50. (b) Just as humans all together make a society similarly all
21. (b) ‘Countervailing’ means ‘compensating.’ animals live together in a jungle.
22. (c) ‘To debilitate’ means ‘to weaken.’ 51. (d) When you disembark (from a ship, train or plain)
23. (b) ‘Ferret’ means ‘to search’ similarly dismount is to get off or alight from a horse.
52. (d) Just the way exercise strengthens the body; practice
24. (a) ‘Pecuniary’ means ‘pertaining to money’
improves the skill.
25. (b) ‘Reprisal’ means ‘an aggressive behaviour in retaliation’. 53. (b) Just the way an illiterate person is limited to education;
26. (a) ‘Propriety’ means ‘correctness or rightness’. a naïve person lacks experience.
27. (b) ‘Cavil’ means ‘to complain about unecessary things’. 54. (a) Like an opaque object does not let light pass through;
28. (b) The word Slothful means sluggard; indolent; lazy. hermetic means being made air tight.
29. (c) The word Preposterous means completely contrary to 55. (a) The word Innocuous means not harmful or injurious;
nature, reason, or common sense; absurd; senseless; harmless.
utterly foolish. 56. (b) The word Handy means within easy reach;
30. (c) The word Breathtaking means thrillingly beautiful, conveniently available; accessible.
remarkable, spectacular, astonishing, exciting. 57. (c) The word Legacy means a gift of property, especially
31. (d) The word insidious means stealthily treacherous, personal property, as money, by will; a bequest or
cunning or deceitful. inherited.
32. (a) The word Commiseration means to feel or express 58. (d) The word Disseminate means to scatter or spread
sorrow or sympathy for; empathize with; pity. widely.
33. (b) The word Congenital means of or relating to a 59. (a) The word Exhorted means to urge, advise, or caution
condition present at birth. earnestly.
34. (d) The word Reprisal means (in warfare) retaliation 60. (d) The word Affliction means a state of pain, distress, or
against an enemy, for injuries received, by the infliction grief; misery.
of equal or greater injuries. 61. (c) The word Pandemonium means wild uproar or
35. (d) The word Celerity means swiftness or speed. unrestrained disorder.
36. (d) The word Quagmire means a soft boggy area of land 62. (a) The word Disparate means distinct in kind; essentially
that gives way underfoot; whirlpool. different; dissimilar.
37. (b) The word Frugal means economical in use or 63. (b) The word Decadent means characterized by or
expenditure; prudently saving or sparing; not wasteful. reflecting a state of moral or cultural decline and the
38. (b) The word Banal means so lacking in originality as to word Disingenuous means lacking in frankness,
be obvious and boring. candour, or sincerity; falsely or hypocritically.
Synonyms B-13
64. (d) The word Factitious means not spontaneous or 92. (c) Exultation is a feeling of triumphant elation, jubilation
natural; artificial; contrived. and rejoicing. Jubilation is the correct synonym.
65. (c) The word Startled means to disturb or agitate suddenly 93. (d) The word Doughty means brave and persistent. Valiant
as by surprise or alarm. is the correct synonym which means possessing or
66. (c) The word Indigenous means originating in and showing courage or determination.
characteristic of a particular region or country; native. 94. (b) The word Obviate means to remove a need or difficulty.
67. (b) The word Gusto means hearty or keen enjoyment; The correct synonym is ‘to remove a difficulty’.
enthusiasm. 95. (c) Sacrilege means violation or misuse of what is regarded
68. (c) The word Veracity means habitual observance of truth as sacred. Blasphemy means the action or offence of
in speech or statement; truthfulness. speaking sacrilegiously about God or sacred things;
69. (b) The word Sustenance means 'means of sustaining life; profane talk.
nourishment; livelihood.' 96. (c) The phrase 'Blow by Blow' means (of a description of
70. (c) The word Rapport means relation; connection, an event) giving all the details in the order in which
especially harmonious or sympathetic relation. they occurred.
71. (c) The word Intrude means to trespass or interrupt. 97. (b) The phrase 'By Hook or by Crook' means by any
72. (d) The word Imperative means absolutely necessary or possible way.
required; unavoidable. 98. (a) The phrase 'chock-a-block' means crammed full of
73. (d) The word Genial means extremely nice and happy; thus people or things.
the synonym of the word is ----friendly and cheerful. 99. (d) The phrase 'A Feather in One's Cap' means an
74. (b) The word Revamp means to renovate or redo; thus achievement to be proud of.
the correct synonym is Reconstruct. 100. (c) The phrase 'for good' means forever; definitely.
75. (b) The word Rescind means to revoke, cancel, or repeal a 101. (d) The word magnificent means extremely beautiful,
law, order, or agreement. So the word----- Repeal is the elaborate, or impressive and splendid is the correct
right synonym. synonym that means magnificent; very impressive.
76. (b) The word Resplendent means something bright, 102. (b) The word Corroborate means confirm or give support
radiant or dazzling therefore-Dazzling is the right to a statement, theory, or finding. Thus, confirm is the
synonym. correct synonym.
77. (a) The word Corpulent means something fat and chubby; 103. (c) The word Elucidate means to make lucid or clear; throw
thus, the correct synonym is Fleshy. light upon; explain. The synonym Explain means to
78. (c) The word Acrimony means nasty behaviour or make (an idea or situation) clear to someone by
bitterness. Therefore, bitterness is the right synonym. describing it in more detail or revealing relevant facts.
79. (a) The word Cantankerous means ill natured; bad 104. (c) The word Apprehension means anticipation of
tempered is the right synonym. adversity or misfortune; suspicion or fear of future
80. (d) The word Felicity means intense happiness. The trouble or evil. Whereas the word Anxiety means
correct synonym is-Great happiness. distress or uneasiness of mind caused by fear of danger
81. (d) The word Unruffled means not disordered or or misfortune is the correct synonym.
disarranged or of a person or their manner not agitated 105. (c) Diffident means modest or shy because of a lack of
or disturbed; calm. Thus, calm is the right synonym. self-confidence. Hesitant is the correct synonym that
82. (b) The word Feline means--- resembling or suggestive of means tentative, unsure, or slow in acting or speaking.
a cat. Therefore -Cat-like is the correct synonym. 106. (a) The word Digress means to deviate or wander away
83. (b) The word Tenuous means, very slender or fine; from the main topic or purpose while speaking or
insubstantial. Therefore - Not substantial is the correct writing. The synonym is Deviate which means depart
synonym. from usual or accepted standards.
84. (a) The word Exquisite means highly sensitive or 107. (a) The word Contending means to strive in rivalry;
discriminating. So the correct synonym is----Highly compete. The synonym competing means strive to
refined. gain or win something by defeating or establishing
85. (a) The word Wrested means, forcibly pull something from superiority over others.
a person's grasp. So the synonym of the word is ---- 108. (d) The word Accolade means any award, honour, or
Took by Force laudatory notice. The correct synonym is award that
86. (c) The word Expostulated express strong disapproval or means strive to gain or win something by defeating or
disagreement. The correct synonym is -Remonstrated establishing superiority over others.
that means ---make a forcefully reproachful protest. 109. (c) The word Salubrious means favourable to or
87. (c) The word placate means to make (someone) less angry promoting health; healthful. The correct synonym is
or hostile. The synonym is pacify that means quell the Healthy.
anger, agitation, or excitement. 110. (a) The word Restive means unable to remain still, silent,
88. (b) The word Perpetual means never ending or changing. or submissive, especially because of boredom or
Continuous which means without interruption is the dissatisfaction. Impatient is the correct synonym that
correct synonym. means showing a tendency to be quickly irritated or
89. (c) The word Dauntless means showing fearlessness and provoked.
determination. Fearless is the correct synonym which 111. (a) The word Preposterous means something contrary to
means showing lack of fear. reason or common sense; whereas the word Incredible
90. (b) The word seething means full of intense but which means something impossible or difficult to
unexpected anger. Be agitated is the correct synonym believe is the correct synonym.
91. (b) The word stingy means somebody averse to spending; 112. (c) The word Persuasive means being good at persuading
miser, mean or ungenerous. or influencing someone to do or believe something
Miserly is the correct synonym having all the above through reasoning or the use of temptation. Thus,
mentioned qualities. Influential is the correct synonym.
EBD_7367
14
B- Synonyms
113. (d) Alleviate means make suffering, deficiency, or a 136. (a) The word Contrived means something obviously
problem less severe. The synonym is Mitigate which planned or forced, thus the answer is Devised.
means make something bad less severe, serious, or 137. (c) The word Deft means skillful and clever, thus the
painful. answer is skilful and quick.
114. (b) Furtively means in a way that attempts to avoid notice 138. (d) The word Rife means in widespread existence, activity
or attention; secretively. The correct synonym is or use; hence the answer is Widespread.
Stealthily which means marked by or acting with quiet, 139. (c) The word Ghastly means something shockingly
caution, and secrecy intended to avoid notice. frightful or horrible and the word Dreadful which
115. (b) The word Disparity means lack of similarity or equality; means something that causes great fear is the answer.
inequality; difference. The synonym is Inequality 140. (c) The word Broached means raise (a difficult subject)
which means the condition of being unequal. for discussion; hence the answer is Discussed.
116. (d) The word Rectify means to make, put, or set right; 141. (b) The word Crass means without refinement, delicacy,
remedy; correct. The right synonym is Correct which or sensitivity; thus its synonym is Coarse.
means free from error; in accordance with fact or truth. 142. (d) The word accomplish means to bring to its goal or
117. (d) The word Absurd means utterly or obviously conclusion, to complete; hence the synonym is
senseless, illogical, or untrue; contrary to all reason or Achieve.
common sense; laughably foolish or false. Foolish is 143. (b) The word Cursory means going rapidly over
the right synonym which means lacking good sense something, without noticing details; hence the
or judgement; unwise. synonym Quick is the correct answer.
118. (d) Lurid means presented in vividly shocking or 144. (c) The word Envisaged means to contemplate; visualize;
sensational terms. Shocking and violent is the right hence the synonym contemplate is the answer.
synonym. 145. (b) The word Fickle means likely to change, especially
119. (b) The word Steadfast means unwavering; stubborn. due to caprice, irresolution, or instability; thus the
120. (b) Kindle means to arouse or inspire an emotion or feeling synonym Inconsistent is the answer.
and the synonym is Incite which means urge or 146. (b) The word Resentment means the feeling of hate or
persuade someone to act. anger; thus the synonym Anger is the answer.
121. (a) The word indictment means a formal accusation 147. (d) The word Obdurate means stubborn or adamant; hence
initiating a criminal case, presented by a grand jury. the answer is adamant.
Also, Arraignment is a formal reading of a criminal 148. (a) The word Obscure means something not easily
charging document in the presence of the defendant understood; hence the synonym Unknown is the
to inform the defendant of the charges against them. answer.
122. (b) The word abstemious means sparing or moderate in 149. (a) The word reluctant means unwilling or disinclined and
eating and drinking; temperate in diet and so does the the word averse which means having strong feeling of
word temperate, which implies, moderate or self- opposition is the answer.
restrained. 150. (d) The word Revelation means the act of revealing or
123. (d) The word Probity means integrity and uprightness disclosing; hence the synonym Disclosure is the
124. (d) The word Dainty means something of a delicate answer.
beauty, thus the answer is small and graceful. 151. (c) The word Ostracise means to exclude by general
125. (c) The word Garish means crudely or tastelessly colorful consent from society, friendship and the word Banish
or showy thus the answer is Unpleasantly gaudy. which means to expel from or relegate to a country or
126. (b) The word Innocuous means not harmful or injurious, place by authoritative decree is the answer.
thus the answer is harmless. 152. (d) The word Appalled means to fill or overcome with
127. (b) The word Abjure means to renounce, repudiate or horror, consternation, or fear; thus the synonym
retract and so does the word recant which means to Alarmed.
withdraw or disavow. 153. (a) The word Imbecility means stupidity or silliness; thus
128. (a) The word Equanimity means mental or emotional the answer is (a).
stability or composure, thus the answer is calm 154. (b) The word Collusion means a secret agreement,
129. (d) The word Ponderous means dull and laborious thus especially for fraudulent or treacherous purposes;
the answer is laboured. conspiracy.
130. (c) The word Gnarled means something rough and 155. (d) The word Hesitant means being undecided or
twisted, hence the answer is (c). doubtful.
131. (d) The word Inured means accustoned, hence the answer 156. (b) The word Deceptive means giving an appearance or
(d). impression different from the true one; misleading.
132. (d) The word Disdainful means showing contempt or lack 157. (a) The word palpable means able to be touched or felt;
of respect, thus the answer is dismissive. hence the synonym Obvious is the answer.
133. (d) The word Innuendoes is the plural form of the word 158. (d) The word laid-back means relaxed or easy-going.
Innuendo which means an indirect intimation about a 159. (d) The word Sauntering means to walk in a slow relaxed
person or thing and the word Insinuations which manner; strolling.
means an indirect or covert suggestion or hint is the 160. (a) The word Pompous means affectedly grand, solemn,
answer. or self-important and the word Grandiose which means
134. (a) The word Appraisal means the act of estimating or extravagantly or pretentiously imposing in appearance
judging the nature or value of something or someone, or style is the answer.
hence the answer should be Estimation. 161. (b) Flying Colours: Victory, Triumph, Good times,
135. (d) The word Indomitable means something that cannot fortunate
be overcome or unconquerable.The word unyielding (a) Enviable: Desired, Blessed
means something that does not give away easily under (b) Notable success: Outstanding victory
pressure, hence it is the answer.
Synonyms B-15
(c) Being pleasant: delightful, enchanting 172. (d) Doleful: pitiful
(d) Highly ambitious: deeply determined (a) Sober: calm
Most Similar word is Notable success. (b) Depressed: discouraged
162. (b) Sanctum: holy place (c) Dreary: Lonely, cheerless
(a) Environment: Surrounding, atmosphere (d) Mournful: grieving, sad, depressed
(b) Holy Part: Religious, hallowed Most similar word is Mournful.
(c) Corridor: hallway 173. (d) Modulate: Adjust, regulate, restrain, fine tune
(d) Room for Pilgrims: wayfarer, pioneer (a) Set: decided, agreed
Most similar word is the holy part. (b) Tune: melody, harmony
163. (b) Frail: Breakable, weak (c) Balance: equilibrium, equal
(a) Unhealthy: Sick (d) Adjust: Regulate, fine-tune, fix
(b) Weak: Not strong Most similar word is Adjust.
(c) Ugly: Unattractive, awful 174. (a) Presumptuous: Bold, self-confident, arrogant, over-
(d) Tender: delicate, soft confident
Most similar word is weak. (a) Bold: brave, courageous
164. (b) Hobnob: Hangout with, socialize (b) Egotistical: self-cantered, smug
(a) Nagging: Painful, sensitive, irritated (c) Arrogant: having exaggerated self-opinion
(b) Friendly Chat: Sociable talk (d) Audacious: reckless, daring
(c) Quarrel: argument, disagreement Most similar word is bold.
(d) Pestering manner: annoyance manner 175. (d) Rhythm: pattern, regularity
Most similar word is friendly chat. (a) Flow: discharge, outflow, run, sequence
165. (d) Dilly-dallying: waste time (b) Measure: portion, part, quota
(a) Quarrelling: Fight, battle (c) Tempo: beat, velocity
(b) Arguing with each other: verbally fight with each (d) Regular pattern: common
other Most similar word is regular pattern.
(c) Waiting for others: stand by 176. (a) Precipitate: hurry, speed
(d) Wasting time: Idle away time (a) Hasten: hurry, rush, race, run
Most similar word is Wasting time. (b) Advance: forward, leading
166. (b) Hush up: Keep secret, conceal (c) Arrive: come to destination, reach
(a) Blow up: Inflate, enlarge, expand (d) Instigate: provoke, raise
(b) To keep quite : Silent Most similar word is Hasten.
(c) To remove: clear away, to eliminate 177. (d) Outlast: endure beyond another
(d) Wind up: come to an end, to finish, tie up loose (a) To come to an end: stop, break off
end, liquidate (b) To fight till the end: everlasting, endlessly
Most similar word is to keep quite. (c) To make a last attempt: try, make an effort
167. (a) Grievances: injustice, unhappiness (d) To endure longer than: Long lasting
(a) Complaints: statement of disagreement, Most similar word is to endure longer than.
grievance. 178. (a) Interregnum: break, gap, interval
(b) Problems: Difficulty, bad situation (a) Pause: wait, halt, interval
(c) Obstacles: barrier, difficulty (b) Interference: conflict, blocking
(d) Hardships: sorrow, personal burden (c) Intrusion: meddling, interruption
Most similar word is Complaints. (d) Prevention: stop, blockage
168. (d) Affinity: closeness, affection Most similar word is Pause.
(a) Admiration: Great respect 179. (c) Demolish: destroy, consume, crush
(b) Adoration: Intense love (a) Abolish: cancel, dissolve
(c) Respect: Admiration given by others. (b) Overthrow: defeat, conquer
(d) Intimacy: Affection, affinity (c) Destroy: demolish, crush
Most similar word is Intimacy. (d) Overturn: reverse, flip over
169. (b) Ignominy: Offensive behaviour Most similar word is Destroy.
(a) Condemnation: Blaming, Accusation 180. (b) Initiate: start, introduce, begin, commence, inaugurate
(b) Disgrace: State of shame, bad reputation. (a) Enlighten: inform, make aware
(b) Begin: commence, start, create
(c) Failure: Lack of success, break down
(d) Criticism: critique, interpretation. (c) Guide: advisor, counsellor
Most similar word is Disgrace. (d) Lead: supremacy, start, top
Most similar word is Begin.
170. (d) Alleviate: Lessen, relieve 181. (d) Stroll: Lazy walk, airing
(a) Eradicate: Destroy, Remove (a) Exercise: workout, activity
(b) Remove: take off, clear away, eliminate
(b) Jogging: activate, running
(c) Understand: figure out, distinguish (c) A ride: journey, ride
(d) Lessen: Lower, reduce (d) A walk: airing, brief travel on foot
Most similar word is Lessen. Most similar word is a walk.
171. (b) Imperceptibly: Hard to sense, faint 182. (c) Languid: listless, dull, energy less
(a) Extremely: Intensely (a) Absent mind: distracted, inattentive, lost
(b) Invisibly: Out of sight (b) Negligent: Careless, indifferent
(c) Inconceivably: Beyond reason, unbelievable. (c) Sluggish: Dull, slow moving, inactive
(d) Intensely: deeply, strongly (d) Thoughtless: reckless, inconsiderate
Most similar word is invisibly Most similar word is Sluggish.
EBD_7367
16
B- Synonyms
183. (b) Hostility means hostile behaviour; unfriendliness or 204. (d) Boast means talk with excessive pride and self-
opposition. Thus the correct synonym is Enmity. satisfaction about one's achievements, possessions,
184. (d) Indifferent means having no particular interest or or abilities. Brag means to say something in a boastful
sympathy; unconcerned. manner. Cry, abuse and hate are its antonyms.
185. (a) Condemn means to express complete disapproval of; 205. (b) Fictitious means not real or true; imaginary or
censure. The correct synonym is Censure that means fabricated. Fiction means something that is invented
to express severe disapproval of someone or or untrue. Feudal means absurdly outdated or old-
something, especially in a formal statement. fashioned.
186. (c) Defect means a shortcoming, imperfection, or lack and 206. (d) Genocide is the systematic destruction of all or a
the similar word is shortcoming that means a fault or significant part of a racial, ethnic, religious or national
failure to meet a certain standard, typically in a person's group. Patricide is the killing of one's father. Fratricide
character, a plan, or a system. means the killing of one's brother or sister. Regicide
187. (a) Jealous means a feeling or showing an envious means the action of killing a king.
resentment of someone or their achievements, 207. (d) Executioner means hangman or the one who inflicts
possessions, or perceived advantages. The right capital punishment i.e. death penalty. Explorer isa
synonym is Envious that means a feeling of person who explores a new or unfamiliar area.
discontented or resentful longing aroused by someone 208. (c) Maiden means being or involving the first attempt or
else's possessions, qualities, or luck. act of its kind. First speech is the maiden speech.
188. (d) Weary means feeling or showing extreme tiredness, 209. (a) Interfere means prevent (a process or activity) from
especially as a result of excessive exertion. The correct continuing or being carried out properly. Meddle means
synonym is Weak that means lacking the power to interfere in something that is not one's concern.
perform physically demanding tasks; having little 210. (a) Tedious means too long, slow, or dull. Tiresome means
physical strength or energy. exhausting.
189. (b) Plausible means an argument or statement seeming 211. (c) Take after means to resemble a close, older relative.
reasonable or probable. Seemingly true is the right Imitates means pretend to be. Comes after means start
synonym. to happen. Follows mean take the place of.
190. (b) Creditable means of a performance, effort, or action 212. (d) Ill favoured means ugly, objectionable.
deserving public acknowledgement and praise but not 213. (b) Homage means special honour or respect shown
necessarily outstanding or successful. Bringing praise publicly. Tribute means praise. Humility means
is the correct synonym. humbleness. Obedience means submissiveness.
191. (a) Salient means most noticeable or important therefore, Allegiance means loyalty.
most important is the right synonym. 214. (a) Clue means hint. Inkling means indication. Intimation
192. (d) Fragile means an object which can be easily broken or also means indication. Signal means sign.
damaged. Easily broken is the correct synonym. 215. (b) Amazement means state of surprise. Shock means a
193. (c) Poignant means evoking a keen sense of sadness or sudden surprise (but it is usually used in a negative
regret. Very painful is the correct synonym. sentence). Wonder means amazement. Surprise means
194. (b) Outlook means a place from which a view is possible; something amazing. Suspicion means doubt.
a vantage point. Prospect is the correct synonym that 216. (d) Consistency means consistent or uniform behaviour.
means an extensive view of landscape. Constancy means fixedness. Competence means
195. (c) Fortitude means courage in pain or adversity therefore capabilities. Permanence means stability and fixedness.
Bravery is the correct synonym. 217. (c) Electrifying means energized and exciting. Attractive
196. (d) Reticent means not revealing one's thoughts or means appealing. Fearsome means frightening.
feelings readily and most similar word is not saying Disturbing means alarming.
much. 218. (b) Refrain means stop oneself from doing something.
197. (c) Reciprocal means expressing mutual action or Desist means stop doing something. 'Dissuade' means
relationship. The most similar word is mutual which to discourage. Prevent means to stop something from
means having the same specified relationship to each happening. Curb means a check or restraint on
other. something.
198. (a) Apprise means inform or tell someone. The most similar 219. (a) Merited means deserve or be worthy of. Encourage
word is Inform. means to persuade. Prompt means to encourage.
199. (b) Consensus means a general agreement. So option (b) Support means assistance.
is correct. 220. (a) Prohibit means formally forbid (something) by law, rule,
200. (a) Unceremonious means having or showing a lack of or other authority. Thus forbid is the most suitable
courtesy; rough or abrupt. The right synonym is synonym of prohibit. Forfeit means penalty, forsake
Impolite that means not having or showing good means abandon and forbear means to restrain.
manners; rude. 221. (b) Dubious means hesitating or doubtful. Dismal means
201. (b) Treason means the action of betraying someone or gloomy; Derogatory means disrespectful attitude and
something. Disloyalty is the right synonym that means devilish means evil.
The quality of not being loyal to a person, country, or 222. (d) Eccentric means unconventional and slightly strange.
organization; unfaithfulness. Destructive means causing great and irreparable
202. (c) Amiable means having or displaying a friendly and damage; deceitful means dishonest; and conformist
pleasant manner. Friendly is the right synonym which means a person who conforms to accepted behaviour
means a person on good or affectionate terms. or established practices.
203. (b) Authentic means of undisputed origin and not a copy; 223. (b) Contradict means deny the truth of (a statement) by
genuine. Authoritative means reliable; proper means asserting the opposite. Talking abusively and request
denoting something that is truly what it is said or politely are incorrect. Contempt means disrespect.
regarded to be.
Synonyms B-17
224. (a) Give-up means stop making an effort. Abandon means options are not exact in delivering the meaning of the
to leave; present means existing; lift means to raise word.
and elevate; scatter means to disperse. 240. (d) Monotonous means dull and repetitive. Again from
225. (d) Specific means clearly defined. Its synonym is precise the given options, we can easily see that option d has
meaning marked by exactness and accuracy. Proper the same word repetitive which brings us to the
means genuine; uncommon means unusual; and conclusion that option (d) is the answer.
noteworthy means worth paying attention to; 241. (b) Redundant means unneeded or unnecessary. If we look
interesting or significant. at the given options, we see that option (b) has an
226. (b) Eradicate means to eliminate. Its synonym is remove option not needed which is very close in meaning to
that means to get rid of. Soften means to become soft the main word. Thus, option (b) is the answer.
and suppress means forcibly put an end to something. 242. (d) Look up means to make better. The option a (look clear)
227. (c) Probable means likely. Certain means sure; indefinite is incomplete and vague. 'Go down' is opposite of it.
means not clearly expressed or defined; and temporary Remain static means to be in the same place. Only
means lasting for only a limited period of time. option (d) "improve" looks convincing as a synonym.
228. (b) Adhere means to be devoted in support or allegiance. 243. (b) Infirmity means ill-health. Infirmity is related to physical
Act reluctantly means to hesitate; waver frequently ill-heath not emotional weakness. Lack of firmness
means to flicker continuously. React promptly means shows the lack of will power. Feebleness is related to
to react quickly. physical weakness. Fickleness is again related to
229. (a) Infer means to deduce or conclude (something) from mental turbulence. Indolence means laziness. Thus,
evidence and reasoning rather than from explicit out of all , option (b) is the best answer.
statements. Hence its synonym is conclude. Calculate 244. (b) Decipher means to make sense of or interpret. Swindle
means to compute; debate means to discuss and means to cheat. Make up means to cover up. Defy
suggest means to propose. means to challenge or confront. But option b (decode)
230. (d) Hesitate means pause in indecision before saying or means to interpret as well like the main word.
doing something. Unwilling to act and speak is 245. (a) Florid means ornate or elaborate. Option (a) ornate is
incorrect because it means reluctant to act and speak closest in meaning to the actual word. Rich can have
respectively. many meanings so it is not specific. Refined means
231. (b) Explicit means stated clearly and in detail, leaving no sophisticated or superior. Rambling means confused
room for confusion or doubt. Thus its synonym is or incoherent. Out of all the words, option a looks
plainly stated. Immediate means instant; marked means best.
noticeable and revealing the secret means to divulge 246. (b) Zest mean gusto or passion or enthusiasm. Option (a)
the secret. greed is negative in appeal although zest is a positive
232. (a) Sufficient means enough; adequate. Complete means word. Enthusiasm looks the most convincing answer.
absolute; full means filled up; and frugal means thrifty. Attraction means appeal. Fascination means attraction
233. (c) Repress means to suppress. Curb means restrain. as well. Thus, over all, we see that option (b) looks as
Express means to convey (a thought or feeling) in the best answer.
words or by gestures and conduct. Impress means 247. (d) Disputable as we see will be complete opposite of the
make (someone) feel admiration and respect. And word. Debatable means discussion worthy or doubtful.
confuse means make (someone) bewildered or Unacceptable again has no relation with the word.
perplexed. Option (d) "unquestionable" looks as the most
234. (c) Vindictive means having or showing a strong or convincing answer as it has the same meaning.
unreasoning desire for revenge. Thus its synonym is 248. (b) Forthright means direct and frank. Courteous means
revengeful. Cruel means brutal; harsh means polite and well-mannered. Straightforward means frank
unpleasantly rough; and rude means offensively and simple. Tactful means polite and thoughtful.
impolite. Correct means socially. According to the given
235. (b) Disdained means ignored refused or rejected. Thus, explanations, option (b) straightforward looks as the
the answer is refused. correct answer.
236. (b) Evocative means suggestive or reminiscent. The 249. (c) Comprehensive means complete. Praiseworthy means
sentence suggests that the poet is trying to suggest commendable or admirable. Full of fine words also
something memorable about his poems. According to means the same as praiseworthy. Covering all aspects
the feel of the sentence, enchanting means capturing means complete. Understandable means which can be
interest as if by a spell. Thus, option c (emotional) is easily interpreted. Thus, from the given options,
the answer. covering all aspects looks the most suitable answer.
250. (b) Painstaking means thorough and careful. Option (a),
237. (d) Expeditious means quick, prompt and speedy. If we
see the options, we notice that only option d has the feeling panic means feeling nervous. Option (b),
thorough and rigorous matches in meaning with the
word prompt which is close to the word expeditious.
original word. Taking risk means taking trouble. Option
All other words do not fit into the context. Thus, option
(d) painful and sorrow means gloomy and aching.
d (prompt and efficient) is the answer. 251. (d) The word substantial means large and extensive.
238. (b) The word flippant means not showing a serious or Option (a) delicate means fragile. Graceful means
respectful attitude; frivolous and factitious. Highly elegant. Sensational means amazing. Significant means
Critical means disapproving. "Not showing respect" important. Out of the four options, the most suitable
is closest to the meaning. The word "casual" does not option is (d), significant.
elaborate the meaning. Indifferent means not caring. 252. (b) Indolent means lazy, cautious means careful. Lazy
239. (a) Impromptu means unprepared or unrehearsed. We can means lethargic. Unintelligent means dull. Stubborn
easily notice from the given options that option a means obstinate. Thus, we see indolent and lazy are
"without preparation" is the most apt synonym. Other identical. Option (b) is correct.
EBD_7367
18
B- Synonyms

253. (d) Vehemently means passionately. Devoutly means 264. (c) Reprimand means rebuke. Praise means admire.
religiously. Serenely means peacefully. Hysterically Reminder means prompt. Rebuke means scold. Reward
means wildly. Forcefully means vehemently. Thus, we means prize. Out of the following options, option (c)
see that for vehemently, the correct synonym is option rebuke is the correct synonym.
(d) forcefully. 265. (b) Punctilious means scrupulous or careful. Serious
254. (d) Oblivious means unaware, narrow-minded means small- means grave. Careful means cautious. Punctual means
minded, daring means bold. Stubborn means
on time. Hardworking means diligent. Out of all the
obstinate. Unaware is the last option which is similar
options, the correct answer is option (b) careful.
in meaning to oblivious.
266. (d) Discreet means careful. Option a mature means
255. (a) Feigned means artificial. Pretended means fake. Played
grown- up. Intelligent means clever. Clever means
means amused. Developed means urbanized, failed
bright. Prudent means careful. From the given options,
means unsuccessful. Thus, by looking at the options,
we can say that option a pretended is the correct we see that discreet and prudent are identical.
answer. 267. (c) Momentous means important. Option a time tested
256. (a) Penury means poverty. Poverty is same as penury. means checked. Momentary means brief. Hectic means
Petty means little, phony means fake. Pathetic means frantic. Momentous is the synonym of very important.
wretched. By seeing the options, it is clear that option Thus option (c) is the answer.
(a) is the correct answer. 268. (d) Portend means foretell. Profess means admit. Portray
257. (b) Affectation means showing off. Adoration means love. means depict. Think means imagine. Foreshadow
Appreciation means approval. Proficiency means skill. means foretell. Out of these options, option (d) is the
Artificiality means dishonesty. According to the given correct answer.
options, artificiality is the correct synonym of 269. (b) Odium means abhorrence. Illness means sickness.
affectation. Hatred means detestation. Oddity means peculiarity.
258. (c) Stifle means smother. Starve means to go hungry. Devious means tricky. Odium means hatred so (b) is
Smother means to stifle or throttle. Stagger means to
the answer.
lurch. The options tell that smother is the correct
270. (b) Sterling when used as an adjective means genuine and
synonym of the word stifle.
reliable. Its synonym is genuine meaning real.
259. (a) Beseech means to request. Request means to appeal.
Interesting means appealing. Irritating means annoying.
Overwhelm means overpower. Bless means approve.
And exciting means exhilarating.
Promise means assure. From the given options, it is
clear that request is the correct synonym for beseech. 271. (b) Corporal means bodily, physical. Its synonym is
260. (b) Stalemate means deadlock. Degeneration means physical. Harsh means rough. Unjust means unfair.
deterioration. Exhaustion means tiredness. Settlement General means common.
means resolution. Deadlock means standstill. Thus, 272. (a) Mitigate means to diminish the effect of something. Its
from the given options, it is clear that option (b), synonym is lessen. Postpone means to delay.
‘deadlock’ is the answer. 273. (a) Idiosyncrasies means a mode of behaviour or way of
261. (d) Impromptu means unprepared. Important means thought peculiar to an individual.
significant. Impressive means inspiring. Inspiring 274. (c) Perfunctory means an action carried out without real
means rousing. Offhand means without preparation. interest, feeling, or effort. Superficial means without
Out of the options, offhand is the synonym of depth.
impromptu. Thus option (d) is the correct answer. 275. (a) Grave means serious and gloomy. Momentous means
262. (a) Lucrative means profitable. Profitable means lucrative. important. Instinctive means reflex. Impulsive means
Important means significant. Challenging means
tending to act without thought.
demanding. Worthwhile means valuable. Option (a) is
276. (d) Empirical means based on, concerned with,or verifiable
the answer.
by observation or experience rather than theory or pure
263. (a) Forthright means straightforward. Option (a) is honest
logic. Intuitively means seemingly.Verbally means
and outspoken. Option (b) is Serious and stubborn.
orally.
Option (c) is strong and brave. Option (d) is willing
and determined. Out of these options option (a) honest 277. (b) Ordnance means mounted guns; artillery. Its synonym
and outspoken is the synonym for forthright. is arms and ammunition. Orthodox means traditional.
Synonyms B-19

278. (d) Sycophant means a person who acts obsequiously 290. (a) Jealous means feeling or showing an envious resentment
towards someone important in order to gain advantage. of someone or their achievements, possessions, or
Its synonym is flatterer meaning a person who lavishes perceived advantages. Its synonym is envious.
praise, often insincerely. Psychologist means person Enthusiastic means excited. Happy means joyful.
who treats mental. Opportunist means one who takes Prisoner means person jailed for crime.
advantage of any opportunity to achieve an end, often 291. (a) Trip means excursion. Its synonym is journey. Plant
with no regard for principles or consequences. means a place where an industrial or manufacturing
Unscrupulous means immoral. process takes place. Design means sketch. Press means
to push.
279. (a) Veracity means truth, conformity to facts; accuracy. Its
292. (c) Deplore means feel or express strong condemnation
synonym is truth. Falsity means dishonesty. Sincerity
of. Its synonym is regret meaning feel sad, repentant,
means honesty.
or disappointed over something that one has done or
280. (c) Aspire means to hope for. Breathe means to respire.
failed to do. Losing heart means being discouraged.
Stair means stairway. Thorn means spike.
Entreat means to urge and malign means hurtful
281. (c) Sequel means to follow up. Its synonym is continuation 293. (c) Motivation means a reason or reasons for acting or
means the action of carrying something on over time or behaving in a particular way. Its synonym is ambition
the state of being carried on. Similarity means very much meaning a strong desire to do or achieve something.
alike. Ending means the end of something and beginning Inducement means incentive. Emotion means mental
means the start of something. state. Incitement means motive.
282. (a) Perpetual means continual, lasting. Its synonym is 294. (a) Residue means left over part. Its synonym is remainder
constant meaning unchanging. Real means actual. meaning left over. Recede means to reduce. Nothing
Mistaken means wrong. Painful means physically or means empty. Little means very less.
mentally agonizing. 295. (c) Perpetual means lasting, its synonym is never-ending
283. (d) Assent means the expression of approval or agreement. meaning continual. Seasonal means periodic. Confused
Its synonym is agreement. Despatch means to transact. means disoriented. Perfect means superlative.
Flavour means taste. Climb means to move up. 296. (d) Assent means agreement. Its synonym is to agree.
284. (b) Defiance means open resistance; bold disobedience. Climb means to ascend/move up. Confirm means to
Its synonym is disobedience. Attack means physical validate. Answer means reaction.
assault. Protection means care. Shyness means 297. (d) Which defiance means open resistance. Denial means
bashfulness. rejection. Insult means disgrace. Degradation means
285. (b) Hostility means meanness, antagonism. Its synonym depravity.
298. (a) Emancipate means the fact or process of being set free
is enmity meaning animosity, hatred. Entertainment
from legal, social, or political restrictions; liberation. Its
means pleasure. Illness means disease. Kindness
synonym is liberate meaning give freedom. Acquit
means generosity.
means free (someone) from a criminal charge by a
286. (a) Indifferent means showing a lack of interest its synonym
verdict of not guilty. Unchain means to set free.
is inconcerned’. Various means many, shy means quiet,
299. (c) Hostility means unfriendliness or opposition. Its
self-conscious. Unconcerned means carefree.
synonym is enmity means hatred.Hospitality means
287. (a) Condemn means to blame, criticize. Its synonym is neighbourliness. Envy and jealousy means grudge.
censure meaning severe criticism. Approve means to 300. (a) Teetotaller is a person who abstains totally from
accept. Qualify means to certify. Despair means intoxicating drink. Ascetics mean a person who follows
depression. an ascetic life. Pedants mean a person who is
288. (c) Confident means feeling or showing confidence in excessively concerned with minor details and rules or
oneself or one's abilities or qualities. Its synonym is with displaying academic learning. Celibates mean a
sure meaning completely confident that one is right. person who abstains from marriage and sexual relations.
Friendly means kind. Full means filled up. Secret means 301. (c) The person who is hard to please is called fastidious.
confidential. Frivolous means silly. Factitious means kidding.
289. (c) Defect means blemish, imperfection. Its synonym is Ferocious means savagely fierce, cruel, or violent.
short coming meaning weak point. Truth means reality. 302. (a) Flouts mean openly disregard. Flounce means
Deception means misleading. Loss means something intermittently move. Flounder means struggle mentally.
misplaced or lost. Flaunt means to show off.
EBD_7367
B-20 Synonyms

303. (a) Citadel means a fortress, typically one on high ground 322. (a) 'monotonous' means lacking in variety and interest.
above a city. Metropolis means the capital or chief city 'dreary' means something uninteresting or depressingly
of a country or region. Megapolis means a very large dull. Therefore, 'dreary' is the synonym of
city. Headquarters mean the premises occupied by a 'monotonous'.
military commander and the commander's staff. 323. (b) 'kindle' means to call forth or excite the emotions,
304. (c) Nostalgia means a sentimental longing or wistful feelings and responses in someone. Therefore, 'excite'
affection for a period in the past. is the correct synonym of 'kindle'.
305. (c) 'marked down' means a reduction in price. 324. (d) 'palatial' means something suitable for a palace or
306. (b) 'Effrontery' means insolent behaviour here it refers to something magnificent. Therefore, 'magnificent' is
the person who is asking for another loan in the given correct synonym of 'palatial'.
sentence. In the given options 'Impudence'is closest
325. (a) 'tactful' means showing skill and sensitivity in dealing
synonym to 'effrontery' as it means shamelessness or
with others or with tough issues. 'diplomatic' means
shameless behaviour.
managing critical issues between two parties or
307. (a) 'devastating' means something causing shock or
countries. Therefore, 'diplomatic' is the correct
distress. And in the given options 'terrible' is the
synonym of 'tactful'.
correct synonym as it also means something which
326. (b) 'voracious' means devouring great quantities of food.
causes shocking or horrible results.
'insatiable' means someone or something impossible
308. (d) 'thorough' is the correct synonym of exhaustive as it
to satisfy in relation to appetite. Therefore, 'insatiable'
means a research done in regards to every details.
is the correct synonym of 'voracious'.
309. (b) 'laying up' means to store something therefore
'accumulating' is the closest synonym of it. 327. (d) 'stricture' means a restriction on a person or activity.
310. (b) In the given context 'might' means physical strength, 'censure' means to express severe disapproval of
therefore 'strength' is the correct synonym. someone or something. Therefore, 'censure' is the
311. (a) In the given context 'dwells' means that God resides appropriate synonym of 'stricture'.
in the heart of its devotees, therefore 'lives' is the 328. (a) 'Obeisance' means the fact of obeying or respecting
appropriate synonym of 'dwells'. someone. 'homage' means expressing special honour
312. (c) In the context of question, answer can be used instead or respect to someone. Therefore, 'homage' is the most
of respond similar synonym of 'Obeisance'.
313. (a) 'Mundane' in the given context means relating to
329. (b) Ripe means experienced, mature and enlightened etc.
ordinary life or characteristic of the world, therefore
thus, option (b) i.e. mature is the correct answer.
'worldly' is the correct synonym as it also means
something pertaining to ordinary life. 330. (d) Repulse means to fight back, push back and repel etc.
314. (c) In the given context 'perilous' is the correct synonym thus, option (d) i.e. repelled is the correct answer.
of 'dangerous' as it means full of danger. 331. (a) Masked means concealed, disguised and hid etc. thus,
315. (d) In the given context stellar groupings refers to the option (a) i.e. hid is the correct answer.
cluster of stars which are 'unlimited'. Therefore, 'infinite' 332. (d) Vendors are those who offer something for sale thus,
is correct synonym of 'unlimited'. Both mean limitless. option (d) i.e. one engaged in selling is the correct
316. (b) "Autocrat" means a ruler who has absolute power. answer.
Therefore, it is the synonym of "dictator". 333. (c) Admit means to allow entry, accept and grant etc. thus,
317. (a) 'yearly' is the correct synonym for "annual" as both option (c) i.e. accept is the correct answer.
mean once a year. 334. (a) Stolen means embezzled, filched and robbed etc. thus,
318. (c) 'Bigot' means a person who is intolerant towards those option (a) i.e. embezzled is the correct answer.
holding different opinions. Therefore it is the
335. (c) Exemplary means commendable, ideal and admirable
synonym of 'fanatic' means a person filled with
etc. thus, option (c) i.e. ideal is the correct answer.
excessive zeal for a particular group or cause.
319. (a) 'Trivial' in the given sentence means something of 336. (d) Serve means to give, provide or distribute etc. thus,
little importance. Therefore 'unimportant' is the correct option (d) i.e. offered is the correct answer.
synonym. 337. (d) Recommend means to endorse, advocate, suggest and
320. (c) propose etc. thus, option (d) i.e. suggest is the correct
321. (a) 'Elucidate' means make clear and Comprehensible answer.
clarify means the same as Elucidate. Therefore, 'Clarify' 338. (b) Save means to defend, rescue, prevent etc. thus, option
is the most similar synonym of 'Elucidate' . (b) i.e. rescue is the correct answer.
C HA P T E R
ANTONYMS
27
DIRECTIONS (Qs. 1-6) : Each question below consists of a word DIRECTIONS (Qs. 14-29): Each of the following questions
in capital letters followed by four words or group of words. Select consists of a word in capital letters, followed by four words or
the word or group of words that is most opposite in meaning to groups of words. Select the word or group of words that is farthest
the words in capital letters. in meaning to the word in capital letters. [2008-I]
1. PROCRASTINATE [2007 - I] 14. PUERILE
(a) To be prompt (b) To adjudicate (a) Fresh (b) Suspenseful
(c) To teach (d) To help others (c) Matured thinking (d) Easily attainable
2. PROCLIVITY [2007 - I] 15. IGNOMINY
(a) Speed (b) Weakness (a) Entrance (b) Activity
(c) Disgust (d) Disinclination (c) Eligibility (d) Honour
3. OUTLANDISH [2007 - I]
16. PENURIOUS
(a) Modern (b) Moderate
(a) Timid (b) Without any perforation
(c) Disrespectful (d) Coward
(c) Affluent (d) Inescapable
4. SUPPLICATE [2007 - I]
(a) Short of supply (b) To be arrogant 17. SCURRILOUS
(c) To be tolerant (d) To fall on lean days (a) Scandalous (b) Admiration
5. TERSE [2007 - I] (c) Primary (d) Opposition
(a) Detailed (b) Nasty 18. ENNUI
(c) Advanced (d) Impure (a) Poverty (b) Shortage
6. CELERITY [2007 - I] (c) Excitement (d) Fearlessness
(a) Lack of wisdom (b) Silence 19. SLAPDASH
(c) Victory (d) Sluggishness (a) Careful (b) Violent
(c) Pervasive (d) Scarce
DIRECTIONS (Qs. 7-13) : Each question below consists of a 20. CHIMERICAL
word in capital letters followed by four words or group of words.
(a) Complement (b) Feasible
Select the word or group of words that is most opposite in meaning
(c) Hypercritical (d) Spurious
to the words in capital letters.
21. THWART
7. VOLITION [2007 - II] (a) Encourage (b) Suffer
(a) Pretext (b) Unwillingness (c) Agonize (d) Pacify
(c) Compliance (c) Interlude 22. VENERATION
8. COPIOUS [2007 - II] (a) Embezzlement (b) Awe
(a) Imitable (b) Unprecedented (c) Disrespect (d) Far-Sighted
(c) Shaky (d) Shortage 23. INVECTIVE
9. ARDUOUS [2007 - II] (a) Adjective
(a) Facile (b) Influential (b) Polite language
(c) Deplorable (d) Tenuous (c) Pertaining to legal matters
10. DEFERENTIAL [2007 - II]
(d) Brief
(a) Discount (b) Disrespectful
24. COPIOUS
(c) Preconception (d) Acute
(a) Shortage (b) Imitation
11. FELICITOUS [2007 - II]
(a) Unfriendly (b) Uneasy (c) Warm (d) Delightful
(c) Unheard of (d) Inappropriate 25. ACERBIC
12. ABSOLVE [2007 - II] (a) The accused (b) Polite
(a) To remember someone fondly (c) Unpleasant (d) Improbable
(b) To imitate someone 26. ASCETIC
(c) To pretend (a) Ardent (b) Indulging in pleasures
(d) To declare someone guilty (c) Careless (d) Patriotic
13. PAROCHIAL [2007 - II] 27. UBIQUITOUS
(a) Open-minded (b) Temporary (a) Honest (b) Rarely found anywhere
(c) Irrelevant (d) Disinclination (c) Out of fashion (d) Scrupulous
EBD_7367
22
B- Antonyms

28. PARSIMONIOUS 46. HERETICAL


(a) Partisan (b) Permissible (a) Pious (b) Orthodox
(c) Revealing (d) Extravagant (c) Superstitious (d) Outdated
29. EPHEMERAL 47. OBSCURE
(a) Mysterious (b) Impolite (a) Filthy (b) Unknown
(c) Long-lasting (d) Exciting (c) Untidy (d) Well known
48. PLIABLE
DIRECTIONS (Qs. 30-48) : Each of the following questions
(a) Wilful (b) Stubborn
consists of a word in capital letters, followed by four words or
(c) Firm (d) Opinionated
group of words. Select the word or group of words that is farthest
in meaning to the word in capital letters. [2008-II] DIRECTIONS (Qs. 49-69): Each of the following questions
30. INSUPERABLE consists of a word or a group of words in capital letters followed
(a) Predictable (b) Surmountable by four words or groups of words. Select the word or group of
(c) Countable (d) Unendurable worded that is farthest in meaning to the word in capital letters.
31. EXONERATED [2009-I]
(a) Implicated (b) Criticised 49. REPLENISH
(c) Condemned (d) Convicted (a) Deplete (b) Increase
32. NADIR (c) Enlarge (d) Unprotect
(a) Zenith (b) Climax 50. RELENTLESS
(c) Peak (d) The lowest point (a) Ruthless (b) Restless
33. EVASIVE (c) Be mercifull (d) Be harsh
(a) Unreliable (b) Tricky 51. RESILIENCE
(c) Straightforward (d) Elusive (a) Silence (b) Buoyancy
34. SCRUPULOUS (c) Rigidity (d) Emptiness
(a) Lax (b) Shameful 52. QUERULOUS
(c) Awful (d) Pitiful (a) Confident (b) Cheerful
35. AMENABLE (c) Firm (d) Quarrelsome
(a) Obedient (b) Diplomatic 53. ACQUITTED
(c) Stubborn (d) Answerable (a) Neglected (b) Discharged
36. ENIGMATIC (c) Arrested (d) Convicted
(a) Lethargic (b) Unambiguous 54. AT THE ELEVENTH HOUR
(c) Wrangling (d) Obscure (a) At night (b) At noon
37. ABHORRENCE (c) Late (d) Early
(a) Admiration (b) Abomination 55. GRUDGING
(c) Repulsion (d) Acceptance (a) Wholehearted (b) Reluctant
38. DILAPIDATED (c) Convincing (d) Secretive
(a) Costly (b) Well-furnished 56. DIVULGE
(c) Broken-down (d) Renovated (a) Reveal (b) Hide
39. LOQUACIOUS (c) Tell (d) Inform
(a) Flamboyant (b) Silent 57. GARRULOUS
(c) Dispirited (d) Dumb (a) Reticent (b) Soft-spoken
40. LOFTY (c) Peaceful (d) Kind
(a) Mean (b) Glorious 58. REJUVENATED
(c) Envious (d) Devilish (a) Reaffirmed (b) Reincarnated
41. OSTENTATION (c) Exhausted (d) Devastated
(a) Miserliness (b) Simplicity 59. THWARTED
(c) Purity (d) Innocence (a) Foiled (b) Opposed
42. TEMPORAL (c) Supported (d) Tightened
(a) Serious (b) Political 60. ENORMOUS
(c) Eternal (d) Divine (a) Soft (b) Average
43. VERDANT (c) Tiny (d) Weak
(a) Dry (b) Green 61. PRONE TO
(c) Beautiful (d) Water-logged (a) Prior to (b) Preceding
44. INFERNAL (c) Immune to (d) Vulnerable to
(a) Paradisiac (b) Majestic 62. OBSEQUIOUS
(c) Peaceful (d) Graceful (a) Offensive (b) Dignified
45. IMPUDENT (c) Irritating (d) Subservient
(a) Brave (b) Gentle 63. FIGHT SHY OF
(c) Polite (d) Boisterous (a) Welcome (b) Avoid
(c) Quarrel with (d) Feel shy of
Antonyms B-23
64. BY FITS AND STARTS 81. REPELLENT
(a) Regularly (a) Troublesome (b) Attractive
(b) When in a fit (c) Tiring (d) Hostile
(c) From time to time 82. DEFUSE
(d) Without steady application (a) Control (b) Understand
65. PAUCITY (c) Aggravate (d) Decelerate
(a) Plenty (b) Pressure 83. BAFFLING
(c) Pause (d) Retention (a) Simple (b) Puzzling
66. DIFFIDENCE (c) Difficult (d) Worrying
(a) Confusion (b) Confidence 84. AMIABLE
(c) Contentment (d) Dissatisfaction (a) Unkind (b) Inhospitable
67. DELETERIOUS (c) Unapproachable (d) Unfriendly
(a) Dilatory (b) Harmless 85. INGENIOUS
(c) Being delicate (d) Salubrious (a) Uneducated (b) Uninventive
68. EXASPERATED (c) Unintelligent (d) Untrained
(a) Encouraged (b) Impressed 86. PLAUSIBLE
(c) Diverted (d) Delighted (a) Unacceptable (b) Incredible
69. ON THE SLY (c) Unlikely (d) Untrue
(a) Openly (b) Secretly 87. GENEROUS
(c) Casually (d) Actively (a) Cruel (b) Stupid
(c) Ignorant (d) Mean
DIRECTIONS (Qs. 70-89) : Each of the following questions
88. SLUGGISH
consists of a word in capital letters, followed by four words.
(a) Aggressive (b) Optimistic
Select the word that is farthest in meaning to the word in capital
(c) Vigorous (d) Budding
letters. [2009-II]
89. RUTHLESS
70. ESTEEM (a) Kind (b) Soft
(a) Power (b) Guess (c) Forgetful (d) Tender
(c) Contempt (d) Estimate DIRECTIONS (Qs. 90-109) : Each of the following questions
71. SPURIOUS consists of a word in capital letters, followed by four words or
(a) Sumptuous (b) Relevant groups of words. Select the words or group of worded that is
(c) Sporadic (d) Genuine farthest in meaning to the word in capital letters. [2010-I]
72. CONVICTED
90. DESPONDENT
(a) Charged (b) Relieved
(a) Pleased (b) Satisfied
(c) Dismissed (d) Acquitted
(c) Infuriated (d) Elated
73. INTIMIDATING 91. REPUDIATE
(a) Authoritative (b) Casual (a) Prefer (b) Unite
(c) Non-serious (d) Friendly (c) Explain (d) Accept
74. FRUGAL 92. OFFICIOUS
(a) Stingy (b) Extravagant (a) Restrained (b) Unofficial
(c) Timid (d) Frightening (c) Indifferent (d) Detached
75. ZENITH 93. DENIGRATE
(a) Under (d) Nadir (a) Belittle (b) Believe
(c) Root (d) Base (c) Doubt (d) Praise
76. LUCRATIVE 94. CONVOLUTED
(a) Advantageous (b) Economical (a) Simple (b) Complicated
(c) Unprofitable (d) Gainful (c) Difficult (d) Majestic
77. UNPRECEDENTED 95. DILIGENT
(a) Exceptional (b) Consistent (a) Cautious (b) Careless
(c) Abnormal (d) Usual (c) Dishonest (d) Delightful
78. PENURY 96. COMPLACENT
(a) Wealth (b) Abundance (a) Agitated (b) Frightened
(d) Prosperity (d) Surplus (c) Degenerate (d) Dissatisfied
79. PRUDENT 97. REITERATE
(a) Absurdity (b) Desperation (a) Withdraw (b) Rectify
(c) Detriment (d) Recklessness (c) Affirm (d) Acknowledge
80. OBSCURE 98. SUPERCILIOUS
(a) Clear (d) Dogged (a) Considerate (b) Respectful
(c) Decent (d) Sensible (c) Thoughtful (d) Differential
EBD_7367
24
B- Antonyms

99. BOISTEROUS 117. DISMAL


(a) Friendly (b) Sincere (a) Bright (b) Indifferent
(c) Humble (d) Restrained (c) Fast (d) Energetic
100. LUCIDITY 118. SANGUINE TEMPER
(a) Confused (b) Dull (a) Despairing nature (b) Peaceful temperament
(c) Simple (d) Verbose (c) Rude behaviour (d) Selfish nature
101. ONEROUS 119. MEAGRE
(a) Egregious (b) Effusive (a) Extravagant (b) Abundant
(c) Easy (d) Efficient (c) Prosperous (d) Surplus
102. CATASTROPHE 120. IMPERIL
(a) Beneficial (b) Blessing (a) Safeguard (b) Construct
(c) Soothing (d) Disastrous (c) Create (d) Brighten
103. GRATUITOUS 121. OSTENSIBLY
(a) Charitable (b) Grand appearance (a) Elaborately (b) Really
(c) Warranted (d) Being grateful (c) Vaguely (d) Sensibly
104. VERACITY 122. CONSOLIDATED
(a) Purity (b) Being just (a) Disjointed (b) Broken
(c) Falsity (d) Immorality (c) Weakend (d) Lost
105. GRIT 123. CHRONIC
(a) Anxiety (b) Cowardice (a) Pathetic (b) Characteristic
(c) Impatience (d) Perseverance (c) Temporary (d) Mild
106. SPORADIC 124. DILIGENT
(a) Spreading easily (b) Inciting (a) Forgetful (b) Imprudent
(c) Regular (d) Uncommon (c) Careless (d) Confused
107. MALADROIT 125. CALLOUS
(a) Authoritative (b) Skilful (a) Considerate (b) Indifferent
(c) Maladjusted (d) Malevolent (c) Indulgent (d) Generous
108. ROTUND 126. VITUPERATIVE
(a) Feeble (b) Healthy (a) Joyous (b) Congratulatory
(c) Slim (d) Weak (c) Critical (d) Virtuous
109. PUNCTILIOUS 127. AWKWARD
(a) Foolish (b) Uncommunicative (a) Awful (b) Satisfactory
(c) Careless (d) Inexperienced (c) Graceful (d) Easy
128. SMUG
DIRECTIONS (Qs. 110 - 133) : Each question in this section
(a) Satisfied (b) Dissatisfied
consists of a word in capital letters followed by four words or
(c) Serious (d) Delighted
groups of words. Select the word or groups of words that is
129. OPPRESSIVE
most nearly opposite in meaning to the word in capital letters.
(a) Gentle (b) Kindly
[2010-II]
(c) Smooth (d) Orderly
110. PROBITY 130. OBSCURE
(a) Dishonesty (b) Timidity (a) Inconspicuous (b) Veiled
(c) Treachery (d) Insincerity (c) Clear (d) Distinct
111. CONTEST 131. CLINCH
(a) Accept (b) Defeat (a) Lose (b) Clasp
(c) Lose (d) Run for (c) Deal (d) Seal
112. SPORADIC 132. ENERVATE
(a) Surviving (b) Sweeping (a) Decelerate (b) Strengthen
(c) Restrained (d) Persistent (c) Push forward (d) Aggravate
113. IMPLICATION 133. TO PUT UP WITH
(a) Consideration (b) Exoneration (a) To stay together (b) To talk politely
(c) Conclusion (d) Interpretation (c) To dislike (d) To move along with
114. APPOSITE
DIRECTIONS (Qs. 134 - 153 ) : Each question in this section
(a) Intemperate (b) Inappropriate
consists of a word or group of words in capital letters followed
(c) Indecent (d) Incriminatory
by four words or groups of words. Select the word or group of
115. GREGARIOUS
words that is most nearly opposite in meaning to the word in
(a) Indecent (b) Unsociable
capital letter. [2011-I]
(c) Above reproach (d) Unlovable
116. CHIVALRY 134. DIMINISH
(a) Dishonesty (b) Discourtesy (a) Enkindle (b) Increase
(c) Disobedience (d) Disaffection (c) Produce (d) Arouse
Antonyms B-25

135. HAMPERED DIRECTIONS (Qs. 154 - 172): Each question this section consists
(a) Facilitated (b) Prompted of a word in capital letters followed by four words or groups of
(c) Relieved (d) Instigated words. Select the word or group of words that is most nearly
136. EQUANIMITY opposite in meaning to the word in capital letters. [2011-II]
(a) Sorrow (b) Discontentment
(c) Agitation (d) Silence 154. DENY
137. SPURIOUS (a) Accept (b) Proceed
(a) Fresh (b) Modern (c) Hold (d) Promote
(c) Genuine (d) Interesting 155. ACQUIT
138. THREW COLD WATER ON (a) Inform (b) Release
(a) Supported (b) Defeated (c) Abuse (d) Condemn
(c) Amended (d) Modified 156. FERTILE
139. AGGRAVATED (a) Dry (b) Useless
(a) Increased (b) Mitigated (c) Barren (d) Arid
(c) Aggregated (d) Magnified 157. AFFLUENCE
140. DEFICIT (a) Not being fluent (b) Poverty
(a) Surplus (b) Sufficiency (c) Difficulty (d) Unhealthy
(c) Luxury (d) Explicit 158. CONFIDENT
141. LENGTHEN (a) Vague (b) Cowardly
(a) Protract (b) Brighten (c) Secretive (d) Unsure
(c) Abandon (d) Shorten 159. GLOOMY
142. CRUELTY (a) Cheerful (b) Forgetful
(a) Love (b) Efficiency (c) Faithful (d) Harmful
(c) Heroism (d) Kindness 160. FRUGAL
143. FREQUENTLY (a) Excessive (b) Extravagant
(a) Habitually (b) Commonly (c) Rich (d) Generous
(c) Usually (d) Rarely 161. OBVIOUS
144. HOSTILE (a) Obscure (b) Obsolete
(a) Dogmatic (b) Easy going (c) Indifferent (d) Difficult
(c) Steady (d) Friendly 162. FORBID
145. PUBLIC HONOUR (a) Defy (b) Dislike
(a) Accusation (b) Punishment (c) Permit (d) Understand
(c) Ignominy (d) Criticism 163. HARMONIOUS
146. SUBTLE (a) Selfish (b) Aggressive
(a) Distant (b) Gross (c) Mechanical (d) Discordant
(c) Plain (d) Higher 164. INCONSPICUOUS
147. TAME (a) Open-hearted (b) Open-minded
(a) Wild (b) Clean (c) Prominent (d) Smooth
(c) Dangerous (d) Active 165. FACTUAL
148. DECLINED (a) Imaginary (b) Idealistic
(a) Inclined (b) Liked (c) Unrealistic (d) Verbal
(c) Agreed (d) Prepared 166. ABANDON
149. ILLAT EASE (a) Assert (b) Retain
(a) Easy to deal with (b) Comfortable (c) Produce (d) Twist
(c) Chronically ill (d) Strong 167. ASSENT
150. COMPLEX (a) Breakdown (b) Misunderstand
(a) Ordinary (b) Simple (c) Dispatch (d) Disagreement
(c) Common (d) Compound 168. FICKLE
151. ABOLISHED (a) Constant (b) Convenient
(a) Continued (b) Established (c) Questionable (d) Faithful
(c) Encouraged (d) Revived 169. DISCREET
152. OFFENCE
(a) Worthy (b) Wishful thinking
(a) Preference (b) Vengeance
(c) Honest (d) Careless in behaviour
(c) Defence (d) Negligence
170. ARTICULATE
153. IRREMEDIABLE
(a) Unable to understand
(a) That which can be avoided
(b) Unable to express oneself
(b) Incurable
(c) Unable to agree
(c) Profitable
(d) Unable to live
(d) That which can be corrected
EBD_7367
26
B- Antonyms

171. TRIVIAL (a) displayed (b) concealed


(a) Temporary (b) Delicate (c) suppressed (d) marked
(c) Important (d) Arbitrary 188. The fallen trees blocked our passage to freedom from the
172. UNJUST wood.
(a) Serious (b) Self-centred (a) facilitated (b) started
(c) Fair-minded (d) Considerable (c) checked (d) promoted
189. He fell foul of me when I asked him to apologise to his
DIRECTIONS (Qs. 173-192): In this section, select the word or
teacher.
group of words that is most opposite in meaning to the underlined
(a) quarrelled with me
word or group of words in the given sentence. [2012-I]
(b) attacked me
173. Poisonous gases emitted from factories contaminate the air (c) made friends with me
we breathe in. (d) showed appreciation for me
(a) sanctify (b) invigorate 190. The outcome of his mission was more rewarding than he
(c) taint (d) purify had expected.
174. Reckless driving causes accidents. (a) lucrative (b) disappointing
(a) careful (b) slow (c) thrilling (d) nail-biting
(c) good (d) correct 191. He wanted to inaugurate the project right on schedule.
175. He often went to the theatre. (a) terminate (b) inculcate
(a) seldom (b) rarely (c) facilitate (d) ameliorate
(c) sometimes (d) occasionally 192. They decided to embellish the dress with imported ribbons.
176. He is frugal in his spending. (a) dishonour (b) demolish
(a) economical (b) extravagant (c) spoil (d) suffice
(c) miserly (d) greedy
177. The students expected an eminent scientist to inaugurate DIRECTIONS (Qs. 193-206): Each question in this section
the programme. consists of a word in capital letters followed by four words or
(a) illustrious (b) notorious group of words. Select the word or group of words that is most
(c) intelligent (d) unknown opposite in meaning to the word in capital letters. [2012-II]
178. Some of their customs are barbarous. 193. PLENTIFUL
(a) civilized (b) modern (a) Handful (b) Rare
(c) polite (d) praiseworthy (c) Small (d) Scanty
179. They are going to embark upon a mountaineering expedition. 194. URBANE
(a) launch (b) analyse (a) Elegant (b) Slow
(c) break off (d) conclude (c) Crude (d) Foolish
180. There has been a gradual falling off in the quality of articles 195. FAMOUS
manufactured locally. (a) Notorious (b) Ignorant
(a) shrinkage (b) erosion (c) Completely unknown (d) Worthless
(c) improvement (d) descent 196. SPURIOUS
181. He was deeply depressed over the news. (a) Genuine (b) Authentic
(a) satisfied (b) elated (c) Real (d) Artificial
(c) impressed (d) affected 197. SUCCEED
182. Though he had lost the battle, he decided not to yield to (a) Achieve (b) Fail
the enemy. (c) Move fast (d) Lose
(a) submit to (b) persuade 198. PROUD
(c) resist (d) seek terms with (a) Humble (b) Kind
183. Because of the failure of the monsoon, there was paucity of (c) Gentle (d) Decent
foodgrains. 199. LEISURELY
(a) overflow (b) inflow (a) Idly (b) Lazily
(c) plenty (d) glut (c) Foolishly (d) Hurriedly
184. The evidence against the accused is conclusive. 200. BARREN
(a) powerful (b) indecisive (a) Wet (b) Rich
(c) exclusive (d) partial (c) Fertile (d) Exception
185. It was a baseless rumour that triggered riots and arson at an 201. CONCEAL
unprecedented scale throughout the country. (a) Reveal (b) Show off
(a) choked (b) tapered off (c) Describe (d) Explain
(c) diluted (d) ignited 202. ACCEPTANCE
186. Their careers followed a parallel path. (a) Demote (b) Throw in
(a) divergent (b) difficult (c) Rejection (d) Turn in
(c) similar (d) dissimilar 203. GRACEFUL
187. The old man manifested his greed at the sight of a huge (a) Awkward (b) Ignorant
amount of money. (c) Slow (d) Di sloyal
Antonyms 27
B-

204. ATTRACT 220. PERTINENT


(a) Severe (b) Repel (a) Eloquent (b) Distant
(c) Dispel (d) Diminish (c) Relevant (d) Irrelevant
205. PROVOKE 221. COLOSSAL
(a) Attract (b) Convince (a) Fragile (b) Small
(c) Gratify (d) Pacify (c) Colourful (d) Impressive
206. VAGUE 222. INDISPENSABLE
(a) Clear (b) Pleasant (a) Tolerable (b) Superfluous
(c) Profound (d) Sufficient (c) Expensive (d) Hostile
DIRECTIONS (Qs. 207-218) : In each of these questions, a 223. VINDICTIVE
word has been underlined followed by words listed (a), (b), (c) (a) Forgiving (b) Humane
and (d). Choose the appropriate word most opposite in meaning (c) Polite (d) Liberal
to the underlined word and mark your Answer Sheet 224. FRIVOLOUS
accordingly. [2013-I] (a) Amusing (b) Serious
207. His partners felt that it was a viable business proposition. (c) Confusing (d) Teasing
(a) enviable (b) unenviable 225. TRANSIENT
(c) inviolable (d) impracticable (a) Lasting (b) Moving
208. The usual adulation of officers must end. (c) Persistent (d) Abiding
(a) back-biting (b) condemnation 226. APPARENT
(c) flsttery (d) praise (a) Real (b) Significant
209. There is an obscure cave on the other side of the hill. (c) Unimportant (d) Vague
(a) well-known (b) infamous 227. SLACKEN
(c) notorious (d) admired (a) Weaken (b) Intensify
210. Her impetuous behaviour was attributed to her upbringing. (c) Quicken (d) Provoke
(a) rash (b) poised 228. ZEAL
(c) sluggish (d) aggressive (a) Intolerance (b) Inefficiency
211. All these measures will augment employment. (c) Semle (d) Apathy
(a) diminish (b) circumscribe
229. DEFILE
(c) restrain (d) constrain
(a) Purify (b) Pollute
212. The two friends were distinct in everything; dress, manners,
(c) Disturb (d) Glorify
hair-style and food-habits.
(a) opposite (b) different 230. FORTUITOUS
(c) uniform (d) similar (a) Unfortunate (b) Accidental
213. The man at the gate had a forbidding appearance. (c) Planned (d) Ludierous
(a) handsome (b) lenient 231. AVERSION
(c) filthy (d) mild (a) Promotion (b) Attraction
214. This is a trivial matter. (c) Hatred (d) Passion
(a) important (b) small 232. IMPERIOUS
(c) easy (d) difficult (a) Characterless (b) Impermanent
215. Everybody called it a lavish party. (c) Imperfect (d) Submissive
(a) big (b) wasteful 233. MARVELLOUS
(c) frugal (d) expensive (a) Awful (b) Mechanical
216. Popular fiction in a sense has enriched world literature. (c) Meaningless (d) Unsentimental
(a) devalued (b) undervalued 234. VEXATION
(c) depreciated (d) impoverished (a) Comfort (b) Slyness
217. Her manner has always been hostile to him. (c) Fright (d) Nervousness
(a) sincere (b) friendly 235. DOLEFUL
(c) fair (d) good (a) Aggressive (b) Cheerful
218. The speaker was irritated with his puerile questions. (c) Tired (d) Involved
(a) impertinent (b) serious 236. SENILITY
(c) inane (d) irrelevant (a) Virility (b) Laziness
DIRECTIONS (Qs. 219-238): Each of the following questions (c) Maturity (d) Exhaustion
consists of a word in capital letters, followed by four words or 237. UNGAINLY
group of words. Select the word that is farthest in meaning to (a) Quick (b) Short
the word in capital letters. [2013-II] (c) Awkward (d) Graceful
219. LAUDATORY 238. SPORADIC
(a) Laughable (b) Derogatory (a) Rare (b) Frequent
(c) Abusive (d) Detriment (c) Sharp (d) Coordinated
EBD_7367
28
B- Antonyms

DIRECTIONS (Qs. 239-248): : In this section, each question 254. CREATE


consists of a word or a phrase which is underlined in the sentence (a) destroy (b) envy
given. It is followed by four words or phrases. Select the word or (c) satisfy (d) begin
phrase which is closest to the opposite in meaning of the 255. FORBID
underlined word or phrase. [2014-I] (a) defy (b) dislike
(c) permit (d) understand
239. Kapil's bowling yesterday proved very costly. 256. MASTER
(a) economical (b) frugal (a) compamon (b) follower
(c) thrifty (d) expensive (c) slave (d) boss
240. I cannot see much likeness between the two boys.
(a) enmity (b) hatred DIRECTIONS (Qs. 257-263): Each question in this section
(c) difference (d) dislike consists of a word in capital letters followed by four words or
241. I am still dubious about that plan. phrases as (a), (b), (c) and (d). Select the word or phrase which is
(a) certain (b) doubtful nearly opposite to the meaning of the original word and mark the
(c) docile (d) faithful correct response as (a), (b), (c) or (d) as the case may be, in your
242. The wise say that life is meant not merely to accumulate Answer sheet. [2015-I]
wealth but for self-realization. 257. FORBID
(a) amass (b) produce (a) forgive (b) allow
(c) scatter (d) gather (c) refuse (d) deprive
243. He will never turn down your request. 258. AMBIGUOUS
(a) turn up (b) turn over (a) definite (b) constant
(c) reject (d) accept (c) shapeless (d) determined
244. Real happiness does not lie in material possessions alone. 259. COUNTERFEIT
(a) physical (b) essential (a) destructive (b) genuine
(c) spiritual (d) manual (c) affirm (d) harmonize
245. I was upset by his hostile attitude. 260. FUSION
(a) friendly (b) positive (a) melting (b) fixture
(c) negative (d) inimical (c) amendment (d) separation
246. Ashoka was a magnanimous king. 261. ESCALATE
(a) Small (b) petty (a) bring down (b) isolate
(c) kind (d) majestic (c) slope down (d) reject
247. Mala is always defiant in her behaviour. 262. TERMINATE
(a) obedient (b) rebellious (a) imitate (b) interrupt
(c) meek (d) friendly (c) initiate (d) examine
248. I find his views repugnant. 263. GRAVITATE
(a) amiable (b) repulsive (a) meditate (b) become serious
(c) amoral (d) apolitical (c) deteriorate (d) retreat
DIRECTIONS (Qs. 249-256): Each question in this section DIRECTIONS (Qs. 264-273): Each below item in this section
consists of a word in capital letters followed by four words as consists of a sentence with an underlined word followed by four
(a), (b), (c) and (d). Select the word which is nearly opposite to words or group of words. Select the word or group of words that
the meaning of the original word and mark the correct response is most nearly opposite in meaning to the underlined word.
as (a), (b), (c) or (d) as the case may be, in your Answer Sheet. [2015-II]
[2014-II]
264. The officer exaggerated the damage caused by the rowdies.
249. FRESH (a) underwrote (b) condemned
(a) laden (b) soft (c) ignored (d) underestimated
(c) sour (d) stale
265. The speaker was unable to pacify the crowd.
250. DENY
(a) excite (b) antagonize
(a) accept (b) accuse
(c) threaten (d) challenge
(c) curse (d) except
266. His officer was a very strict person.
251. CHEERFUL
(a) sad (b) happy (a) pleasant (b) open hearted
(c) expensive (d) carelesse (c) lenient (d) indifferent
252. AFFLUENCE 267. Servitude is not helpful for mental growth.
(a) continuance (b) poverty (a) Disservice (b) Retirement
(c) diffidence (d) insurance (c) Freedom (d) Termination
253. TIMID 268. His attitude to poor people is deplorable.
(a) bold (b) bashful (a) commendable (b) miserable
(c) nervous (d) soft (c) equitable (d) desirable
Antonyms B-29
269. The guest made derogatory remarks about the food he was (a) reprieves (b) absolves
served. (c) indicts (d) summons
(a) interesting (b) complimentary 281. In those days many monarchs enjoyed vast ecclesiastical
(c) unneccessary (d) cheerful powers.
270. He has an aversion to milk. (a) permanent (b) temporal
(a) dear (b) loving (c) contemporary (d) constitutional
(c) liking (d) pet
271. The paucity of good teachers is the chief reason for the DIRECTIONS (Qs 282-291): Each item in this section consists
present condition of these schools. of a sentence with an underlined word/words followed by four
(a) presence (b) surplus words. Select the option that is opposite in meaning to the
(c) appointment (d) retention underlined word/words and mark your response on your Answer
272. Don't you think his account of things was monotonous ? Sheet accordingly. [2018-I]
(a) agreeable (b) acceptable 282. It was a mystery as to where the young girl had acquired
(c) varied (d) indecent such a naive belief.
273. Spurious drugs can prove to be fatal. (a) credulous (b) childlike
(a) Virtuous (b) Inferior (c) wise (d) innocent
(c) Genuine (d) Contemptuous 283. It’s the only treatment suitable for cancer.
(a) insufficient (b) impertinent
DIRECTIONS (Qs. 274-281) : Each item in this section consists (c) befitting (d) congenial
of a sentence with an underlined word followed by four words. 284. Some of the criticisms which they had to put up were very
Select the word that is opposite in meaning to the underlined unfair.
word and mark your response in your Answer Sheet accordingly. (a) scold (b) scorn
[2017-II] (c) appreciation (d) censure
274. Whether the rewards are in commensurate with the efforts 285. I would beg of all friends not to rush to Birla house nor try
or not, a society will always have workaholics and the shirk to dissuade me or be anxious about me.
work groups. (a) certain (b) composed
(a) disproportionate (b) equal to (c) careless (d) heedless
(c) matched (d) unparalleled 286. It could not have been expected that, with such a bent of
275. Wars leave behind a large number of emaciated soldiers in mind of the people, there should have been much activity
the camps of both the victorious and the vanquished. for the cultivation of the physical sciences in this part of
(a) hefty (b) thin the world.
(c) disillusioned (d) determined (a) dull (b) dormant
(c) indolence (d) idle
276. There was a mammoth gathering to listen to the leader. 287. Indian culture has been, from time immemorial, of a peculiar
(a) negligible (b) tiny cast and mould.
(c) poor (d) large (a) common (b) customary
277. The audience thoroughly enjoyed the hilarious drama. (c) natural (d) familiar
(a) amusing (b) delightful 288. The princess charming was the centre of attraction today.
(c) serious (d) momentous (a) enchanting (b) hypnotic
278. The writer’s erudition in science is revealed in every page (c) repulsive (d) fascinating
of the book. 289. Macbeth is a/an abominable figure.
(a) unenlightened (b) ignorance (a) abhorrent (b) repugnant
(c) intelligence (d) hollowness (c) reputable (d) attractive
279. The seminar which Ravi organised proved to be momentous 290. Terrorists profess fanatical ideology.
event. (a) bigoted (b) militant
(a) trivial (b) futile (c) moderate (d) fervid
(c) vain (d) useless 291. Rakesh is vulnerable to political pressure.
280. The question is not whether the court vindicates him with (a) weak (b) unguarded
regard to his involvement in the case, but how he feels (c) exposed (d) resilient
about it.
EBD_7367
B- 30 Antonyms

HINTS & SOLUTIONS


1. (a) ‘Procrastinate’ means to delay or linger in a decision. 30. (b) The word Insuperable means incapable of being
‘Prompt’ means done without delay. passed over, overcome, or surmounted.
2. (d) ‘Proclivity’ means a natural tendency (or inclination) 31. (d) The word Exonerated means to clear, as of an
to do something. accusation; free from guilt or blame; exculpate.
3. (a) ‘Outlandish’ means odd or strange. 32. (a) The word Nadir means an extreme state of adversity;
4. (b) ‘Supplicate’ means to be humble or requestful. the lowest point of anything; zenith is the opposite.
5. (a) ‘Terse’ means brief and straighforward. 33. (c) The word evasive means deliberately vague or
6. (d) ‘Celerity’ means speed and ‘Sluggishness’ means ambiguous; tending to avoid commitment or self-
dullness. revelation, especially by responding only indirectly.
7. (b) 'Volition' means power to choose something freely. 34. (a) The word Scrupulous means punctiliously or minutely
8. (d) 'Copious' means in large amounts. careful, precise, or exact; whereas the word Lax means
9. (a) 'Arduous' means involving a lot of effort and energy. not strict or severe; careless or negligent.
10. (b) 'Deferential' means behaviour that shows respect. 35. (c) The word Amenable means capable of or agreeable to
11. (d) ‘Felicitous’ means very suitable or giving a good result. being tested, tried, analyzed, etc.
12. (d) ‘Absolve’ means to state officially that someone is 36. (b) The word Enigmatic means difficult to interpret or
not guilty. understand; ambiguous; mysterious.
13. (a) ‘Parochial’ means connected with small issues. 37. (a) The word Abhorrence means a feeling of revulsion;
14. (c) The word Puerile means of or relating to a child or to disgusted loathing.
childhood. 38. (d) The word Dilapidated means (of a building or object)
15. (d) The word Ignominy means disgrace; dishonour; public in a state of disrepair or ruin as a result of age or neglect.
contempt. 39. (b) The word Loquacious means tending to talk a great
16. (c) The word Penurious means extremely poor; destitute. deal; talkative.
17. (b) The word Scurrilous means grossly or obscenely 40. (a) The word Lofty means arrogantly or condescendingly
abusive. superior in manner.
18. (c) The word Ennui means a feeling of utter weariness 41. (b) The word Ostentation means pretentious or
and discontent resulting from satiety or lack of interest; conspicuous show, as of wealth or importance; display
boredom. intended to impress others.
19. (a) The word Slapdash means hasty and careless; 42. (c) The word Temporal means of or relating to or limited
offhand. by time; material or worldly.
20. (b) The word Chimerical means wildly fanciful; highly 43. (a) The word Verdant means green or blooming.
unrealistic. 44. (a) The word Infernal means hellish; fiendish; diabolical
21. (a) The word Thwart means to frustrate or baffle. whereas the word paradisiac means of, like, or befitting
22. (c) The word Veneration means the feeling of a person paradise.
who venerates; a feeling of awe, respect, etc.; 45. (c) The word Impudent means shameless or immodest.
reverence. 46. (b) The word Heretical means holding an opinion at odds
23. (b) The word Invective means an insulting or abusive word with what is generally accepted.
or expression. 47. (d) The word Obscure means not discovered or known
24. (a) The word Copious means large in quantity or number; about; uncertain.
abundant; plentiful. 48. (b) The word Pliable means being easily influenced.
25. (b) The word Acerbic means harsh or severe, as of temper 49. (a) The word Replenish means to fill or stock; hence the
or expression. correct antonym is Deplete that means diminish in
26. (b) The word Ascetic means a person who leads an number or quantity.
austerely simple life, especially one who abstains from 50. (c) The word Relentless means being cruel or merciless;
the normal pleasures of life or denies himself or herself hence, the antonym Be merciful is correct.
material satisfaction. 51. (c) The word Resilience means flexibility or elasticity; thus
27. (b) The word Ubiquitous means existing or being the correct antonym is Rigidity that means unable to
everywhere, especially at the same time; omnipresent. bend or be forced out of shape; not flexible.
28. (d) The word Parsimonious means very unwilling to spend 52. (b) The word Querulous means complaining in a rather
money or use resources. petulant or whining manner. Cheerful is the right word
29. (c) The word Ephemeral means lasting a very short time; with farthest meaning.
short-lived; transitory. 53. (d) The word Acquitted means free (someone) from a
criminal charge by a verdict of not guilty. Convicted
Antonyms B-31

which means a person found guilty of a criminal offence 73. (d) The word Intimidating means to induce fear; thus the
and serving a sentence of imprisonment is the right correct antonym is friendly.
antonym. 74. (b) The word Frugal means economical in use or
54. (d) The word "At the eleventh hour" means the last expenditure; thus, the antonym is extravagant that
possible moment for doing something; hence the right means lacking restraint in spending money or using
antonym is Early. resources.
55. (a ) The word Grudging means to give unwillingly; thus 75. (d) The word Zenith means a highest point or state; thus
the antonym wholehearted is correct which means the correct antonym is base.
showing or characterized by complete sincerity and 76. (c) The word Lucrative means something profitable or
commitment. remunerable. Unprofitable is the right antonym.
56. (b) The word Divulge means to admit or disclose; hence 77. (d) The word Unprecedented means without previous
the antonym Hide which means to keep secret is the instance; never before known or experienced; unusual.
correct antomym. The correct antonym is usual which means not
57. (a) The word Garrulous means talkative and the word habitually or commonly occurring or done.
Reticent means secretive or quiet. 78. (a) The word Penury mean s extreme poverty or
58. (c) The word rejuvenated means reinvigorate or revitalize; destitution. Wealth is the correct antonym.
thus, the right antonym is Exhausted which means very 79. (d) The word Prudent means caution with regard to
tired. practical matters; whereas the word recklessness
59. (c) The word Thwarted means prevent (someone) from means without caution, carelessness.
accomplishing something; thus the antonym Supported 80. (a) The word Obscure means not clear or plain; ambiguous;
which means give approval, comfort, or encouragement uncertain. Clear is the correct antonym.
to is the correct antomym. 81. (b) The word Repellent means causing disgust or distaste.
60. (c) Attractive is the correct antonym.
61. (c) The word Prone to means likely or liable to suffer from 82. (c) The word Defuse means to make less dangerous, tense,
something; whereas the word immune to means or embarrassing; whereas the word aggravate means
resistant to a particular infection or toxin. to make worse or more severe.
62. (b) The word Obsequious means obedient or attentive to 83. (a) The word Baffling means to confuse; bewilder; perplex.
an excessive or servile degree. Dignified is the proper The correct antonym is Simple.
farthest word. 84. (d) The word Amiable means friendly or sociable.
63. (a) The word 'fight shy of' means be unwilling to undertake Unfriendly is the correct antonym.
or become involved with; thus the antonym welcome 85. (b) The word Ingenious means being characterized by
is correct. cleverness or originality of invention or construction.
64. (a) The word 'By Fits and Starts' means with irregular Uninventive means not showing creativity or original
bursts of activity; thus the antonym is regularly. thought.
65. (a) The word Paucity means the presence of something in 86. (c) The word Plausible means having an appearance of
only small or insufficient quantities or amounts; thus truth or reason; seemingly worthy of approval or
the correct antonym is Plenty. acceptance; likely. Unlikely is the right antonym.
66. (b) The word Diffidence means modesty or shyness 87. (d) The word Generous means liberal in giving or sharing.
resulting from a lack of self-confidence; hence, the The right antonym is mean that means limited and
correct antonym is Confidence which means the feeling lacking generosity.
or belief that one can have faith in or rely on someone 88. (c) The word Sluggish means lacking in energy or vigour;
or something. lazy. Vigorous is the correct antonym.
67. (b) The word Deleterious means causing harm or damage; 89. (a) The word Ruthless means having or showing no pity
thus, harmless is the correct antonym. or compassion for others. Kind is is the correct
68. (d) The word Exasperated means to irritate intensely or antonym.
infuriate; hence the right antonym is delighted. 90. (d) The word Despondent means to be depressed by loss
69. (a) The word 'On the sly' means in a secretive fashion; of hope or confidence, whereas the word Elated means
thus, the correct antonym is openly. very happy or proud.
70. (c) The word Esteem means to consider as of a certain 91. (d) The word Repudiate means to reject as having no
value or of a certain type; regard; whereas the word authority or binding force, thus the answer is Accept
contempt means the state of being despised; which means to receive something with favour or
dishonour; disgrace. approval.
71. (d) The word Spurious means not genuine, authentic, or 92. (c) The word Officious means objectionably aggressive
true. in offering one's unrequested and unwanted services,
72. (d) The word Convicted means to prove or declare guilty help or advice; whereas the word indifferent means
of an offense; whereas the word Acquitted which showing no concern in attitude or action.
means to declare not guilty; free of any charge is the 93. (d) The word Denigrate means to speak damagingly of;
correct antomym. criticize in a derogatory manner, sully, defame; hence
the antonym Praise is the correct answer.
EBD_7367
32
B- Antonyms

94. (a) The word Convoluted means something complicated whereas the word Exoneration means the release of
or intricately involved; hence the correct antonym is someone from a duty or obligation.
Simple. 114. (b) The word Apposite means something suitable; hence
95. (b) The word Diligent means having or showing care and the antonym Inappropriate is the correct answer.
conscientiousness in one's work or duties; thus the 115. (b) The word Gregarious means fond of the company of
antonym, Careless is the correct answer. others; sociable; hence the correct antonym is
96. (d) The word Complacent means pleased, especially with Unsociable.
oneself or one's merits, advantages orsituation; hence 116. (b) The word Chivalry means the medieval knightly
the antonym Dissatisfied is the correct answer. system with its religious, moral, and social code; hence
97. (a) To reiterate something is to say or do something again, the antonym Discourtesy is the correct answer.
or many times; hence the correct antonym is Withdraw, 117. (a) The word Dismal means causing a mood of gloom or
which means to draw back, away or aside. depression; hence the correct antonym is bright.
98. (b) The word Supercilious means behaving or looking as 118. (a) The word Sanguine means cheerfully optimistic,
though one thinks one is superior to others; thus the hopeful, or confident and the word Temper means a
antonym, Respectful is the correct answer. particular state of mind or feelings.
99. (d) The word Boisterous means loud or noisy and lively Whereas the word Despairing means hopelessness.
or unrestrained; hence the correct antonym is Hence the answer is (a).
Restrained. 119. (b) The word Meagre means (of something provided or
100. (a) The word Lucidity means clearly expressed; easily available) lacking in quantity or quality; hence the
understood; hence the correct antonym is Confused. antonym Abundant is the correct answer.
101. (c) The word Onerous means involving a great deal of 120. (a) The word Imperial means put at risk of being harmed,
effort, trouble or difficulty; thus the correct antonym injured, or destroyed; hence the correct antonym is
is Easy. Safeguard.
102. (b) Catastrophe is an event causing great and usually 121. (c) The word Ostensibly means something apparent,
sudden damage or suffering; a disaster; hence the evident, or conspicuous; hence the correct antonym
antonym Blessing is the correct answer. is Vaguely.
103. (c) The word Gratuitous means done without good 122. (a) The word Consolidated means something brought
reason; uncalled for or Unwarranted; hence the together into a single whole; hence the antonym
antonym Warranted which means to justify or disjointed is the correct answer.
necessitate is the answer. 123. (c) The word Chronic means something continuing a long
104. (c) The word Veracity means habitual observance of truth time or recurring frequently; hence the antonym
in speech or statement; truthfulness, thus the antonym Temporary is the correct answer.
Falsity is the correct answer. 124. (c) The word Diligent means having or showing care and
105. (b) The word Grit means firmness of character; indomitable conscientiousness in one's work or duties; hence the
spirit; pluck, whereas Cowardice which means lack of antonym Careless is the correct answer.
courage, is the answer. 125. (a) The word Callous means insensitive; indifferent;
106. (c) The word Sporadic means appearing or happening at hence the antonym considerate is the correct answer.
irregular intervals in time; occasional, hence the 126. (b) The word Vituperative means bitter and abusive; hence
antonym, Regular is the correct answer. the antonym Congratulatoryis the correct answer.
107. (b) The word Maladroit means lacking in adroitness or 127. (c) The word Awkward means lacking grace or ease in
unskillful; hence the correct antonym is Skillful. movement; hence the correct antonym is Graceful.
108. (c) The word Rotund means plump or fat; thus the 128. (b) The word Smug means having or showing an excessive
antonym, Slim is the correct answer. pride, self-satisfied; hence the correct antonym is
109. (c) Punctilious means being strict or exact in the Dissatisfied.
observance of the formalities or amenities of conduct 129. (a) The word Oppressive means burdensome, unjustly
or actions, thus the antonym, Careless is the correct harsh; hence the antonym Gentle in the answer.
answer. 130. (c) The word Obscure means not clear or plain,
110. (a) The word Probity means integrity and uprightness; ambiguous, vague, or uncertain, thus the antonym
honesty; hence the correct antonym is Dishonesty. Clear is the correct answer.
111. (a) The word Contest means a race, conflict, or other 131. (a) The word Clinch means to hold in tight grasp; hence
competition between rivals; hence the correct antonym the antonym lose is the correct answer.
is Accept. 132. (b) The word Enervate means to deprive of force or
112. (d) The word Sporadic means appearing or happening at strength; hence the antonym Strengthen is the correct
irregular intervals in time; whereas the word Persistent answer.
means something constantly repeated. 133. (c) 'To put up with' means 'to tolerate or endure'; thus the
113. (b) The word Implication means something implied or antonym 'to dislike' is the correct answer.
suggested as naturally to be inferred or understood
Antonyms 33
B-

134. (b) Diminish: To lessen, to decrease 142. (d) Cruelty: Brutality, Harshness, Torture
(a) Enkindle: To set afire/ light, arouse, to make (a) Love: Affection.
luminous (b) Efficiency: Effectiveness, ability, capability
(b) Increase: Growth, Gain, increment (c) Heroism: bravery, boldness, courage
(c) Produce: To generate (d) Kindness: Mildness, Compassion, Generosity
(d) Arouse: Awake, Awaken Most nearly opposite is Kindness.
Most nearly opposite is Increase. 143. (d) Frequently: Repeatedly
135. (a) Hampered: To create difficulty for the work or the (a) Habitually: Usually
activity of. (b) Commonly: Usually, Frequently
(a) Facilitate: to free from obstruction or difficulty. (c) Usually: occasionally, generally
(b) Prompted: to be the cause of (d) Rarely: Not often, barely, hardly
(c) Relieved: Free, discharged Most nearly opposite is rarely.
(d) Instigated: Induce, stimulate 144. (d) Hostile: Unsympathetic, Bitter, contrary, hateful
Most nearly opposite is facilitated. (a) Dogmatic: Dictatorial, Arrogant, stubborn
136. (c) Equanimity: Evenness emotion or temper. (b) Easy going: Smooth, progressive, adaptable
(a) Sorrow: Feeling unhappiness (c) Steady: stable, fixed, regular
(b) Discontentment: Bitterness, unhappiness, (d) Friendly: helpful, affectionate, companionable
uneasiness. Most nearly opposite is Friendly.
(c) Agitation: A state of anxiety and nervousness. 145. (c) Public Honour: Dignity, fame, tribute, prestige,
(d) Silence: quieten reputation, esteem
Most nearly opposite is Agitation (a) Accusation: Blame, charge, crimination, allegation
137. (c) Spurious: Fake, false, inauthentic (b) Punishment: penalty
(a) Fresh: refreshing, new (c) Ignominy: offensive behaviour
(b) Modern: innovative, advanced (d) Criticism: judgement
(c) Genuine: Authentic, true, liberal Most nearly opposite is Ignominy.
(d) Interesting: fascinating, amusing, diverting 146. (b) Subtle: Nice, quite, delicate, elusive
Most nearly opposite is Genuine. (a) Distant: faraway, remote
138. (a) Threw cold water on: Deject, make despondent, (b) Gross: Broad, general
exhaust (c) Plain: apparent, distinct, evident, manifest
(a) Supported: promoted, Hold, sustain, to help (d) Higher: lofty, towering
(b) Defeated: overcome, unsuccessful Most nearly opposite is Gross.
(c) Amended: Change, alter, modify, better 147. (a) Tame: Domestic, compliant, trained
(d) Modified: Adapted, restricted, altered (a) Wild: uncivilized, undomesticated
Most nearly opposite is supported. (b) Clean: unsoiled, uncluttered
139. (b) Aggravated: Annoy, Angry, Upset, Enraged, Irritated, (c) Dangerous: Hazardous, troubling
change (d) Active: Dynamic, restless
(a) Increased: Raised, elevated, heightened Most nearly opposite is Wild.
(b) Mitigated: Check, diminish, lighten, calm, blunt, 148. (c) Declined: Refuse, reject, deny
quite, moderate (a) Inclined: Willing, likely, apt
(c) Aggregated: Mix, collect, combine. To (b) Liked: admired, cherished
accumulate. (c) Agreed: Acknowledge, admit, allow
(d) Magnified: Enlarge, intensify, hike, increase, (d) Prepared: Able, inclined, fit.
enhance Most nearly opposite is Agreed.
Most nearly opposite is mitigated. 149. (b) Ill at ease: Awkward, embarrassed, and uneasy
140. (a) Deficit: Shortage of something needed, required, (a) Easy to deal with: Uncomplicated, Not difficult
shortfall, insufficiency (b) Comfortable: relax, untroubled
(a) Surplus: Excess, spare, extra (c) Chronically ill: confirmed, habitual
(b) Sufficiency: Enough, plenty (d) Strong: forceful, powerful
(c) Luxury: Indulgence, comfort Most nearly opposite is Comfortable.
(d) Explicit: Specific, unambiguous, definite 150. (b) Complex: Complicated, dense, difficult
Most nearly opposite is surplus. (a) Ordinary: Common, regular, general
141. (d) Lengthen: Extend, make longer, augment (b) Simple: Clear, Understandable, easy
(a) Protract: Extend, stretch, prolong, and keep going. (c) Common: general, ordinary
(b) Brighten: to make shine or glow, enliven, light up (d) Compound: Combination, mixture
(c) Abandon: freedom, recklessness, unrestraint, Most nearly opposite is Simple.
disregard 151. (a) Abolished: Cancel, Dissolve, Put to an end
(d) Shorten: diminish, decrease, compress, reduce (a) Continued: carry on, maintain
Most nearly opposite is Shorten. (b) Established: Begin, create
EBD_7367
34
B- Antonyms

(c) Encourage: Buck up, embolden 167. (d) Assent means the expression of approval or agreement.
(d) Revived: refreshed, recovered, returned The most nearly antonym is Disagreement which
Most nearly opposite is Established. means lack of consensus or approval.
152. (c) Offence: violation, crime, Sin 168. (a) Fickle means changing frequently, especially as regards
(a) Preference: first choice, desire, one's loyalties or affections. The correct antonym is
(b) Vengeance: revenge, retaliation Constant that means remaining the same over a period
(c) Defence: Arms, armament, protection system of time.
(d) Negligence: Carelessness, neglect, oversight. 169. (d) Discreet means careful and prudent in one's speech or
Most nearly opposite is defence. actions, especially in order to keep something
153. (d) Irremediable: Hopeless, incurable, lost, irreversible confidential or to avoid embarrassment. The correct
(a) That which can be avoided: antonym is Careless in behaviour.
(b) Incurable: Hopeless, Impossible 170. (b) Articulate means having or showing the ability to
(c) Profitable: Advantageous, Effective speak fluently and coherently. The correct antonym is
(d) That which can be corrected: Modify, remediable Unable to express oneself.
Most nearly opposite is that which can be corrected. 171. (c) Trivial means of little value or importance and
154. (a) Deny is a state where one refuses to admit the truth or Important is the correct antonym.
existence of. Accept is the correct antonym that means 172. (c) Unjust means not based on or behaving according to
to give an affirmative answer to an offer or proposal; what is morally right and fair thus, Fair-minded is the
say yes to. correct antonym.
155. (d) Acquit means to free someone from a criminal charge 173. (d) Contaminate means to adulterate. Its antonym is Purify
by a verdict of not guilty. Condemn is the correct meaning make clean. Sanctify means free from sin, and
antonym that means sentence someone to a particular taint means to contaminate.
punishment, especially death. 174. (a) Reckless means carefree. Its antonym is careful. Slow,
156. (c) Fertile is of soil or land producing or capable of good and correct are not appropriate.
producing abundant vegetation or crops. Barren is 175. (b) Often means frequently. Its antonym is rarely which
the correct antonym that means land too poor to means infrequently. Occasionally, sometimes and
produce much or any vegetation. seldom are not appropriate.
157. (b) Affluence means the state of having a great deal of 176. (b) Frugal, economical and stingy all means the same i.e.
money; wealth. Poverty is the correct antonym that economical. Miserly and greedy means the same i.e.
means state of being extremely poor. greedy. Its antonym is extravagant meaning indulgent,
158. (d) Confident means feeling or showing certainty about wasteful.
something. The correct antonym is Unsure that means 177. (d) Eminent means very important and famous. Its antonym
not feeling, showing, or done with confidence and is unknown. Notorious means famous or well known,
certainty. typically for some bad quality or deed. Intelligent
159. (a) Gloomy means causing or feeling depression or means sharp wit.
despondency. Cheerful is the correct antonym that 178. (a) Barbarous means extremely brutal. Its antonym is
means happy and optimistic. civilized meaning cultured. Praiseworthy means
160. (b) Frugal means to be economical as regards money or deserving congratulations. Polite means civilized.
food. The right antonym is Extravagant that means Modern means up-to-date.
lacking restraint in spending money or using resources. 179. (d) Embark upon means to undertake. Launch means start
161. (a) Obvious means easily perceived or understood; clear, an activity. Analyze means examine. Break off means
self-evident, or apparent. The right antonym is Obscure become severed. Conclude means to finish.
that means not clearly expressed or easily understood. 180. (c) Falling off means a noticeable deterioration in
162. (c) Forbid means refuse to allow something. The antonym performance or quality. Its antonym is improvement.
is Permit that means allow for; admit of. Shrinkage means decrease. Erosion means
163. (d) Harmonious means tuneful; not discordant. The correct deterioration and descent means lowering down.
antonym is Discordant that means harsh and jarring 181. (b) Depressed means .discouraged and unhappy. Its
because of a lack of harmony. antonym is elated meaning very happy. Satisfied means
164. (c) Inconspicuous means not clearly visible or attracting content. Impressed means fascinated and affected
attention. The correct antonym is Prominent that means deeply moved.
means, catch the attention; noticeable. 182. (c) Yield to means to give up something to someone.
165. (a) Factual means concerned with what is actually the Submit to and seek terms with mean to give upon
case. The correct antonym is Imaginary which means something. Its antonym is resist which means oppose.
existing only in the imagination. Persuade means convince.
166. (b) Abandon means give up completely. The correct 183. (c) Paucity means lack, scarcity. Its antonym is plenty
antonym is Retain that means continue to have meaning much, abundance. Inflow means flow. Outflow
something keep possession of. means discharge. Glut means over abundance.
Antonyms B-35

184. (b) Conclusive means definite, final. Its antonym is 200. (c) Barren means unable to support growth. Its opposite
indecisive meaning uncertain, indefinite. Powerful is fertile. Wet means damp, and rich means opulent.
means strong. Exclusive means restricted. Partial 201. (a) Conceal means to hide. Its antonym should be reveal
means incomplete. meaning to disclose. Show off means to flaunt;
185. (b) Triggered means cause to happen. Its antonym is describe and explain means to communicate.
tapered off which means to gradually stop doing 202. (c) Acceptance means agreement, to take in. Its antonym
something. Choked means to block. Diluted means is rejection. Demote means to downgrade; throw in
make something weaker in force. Ignite means to trigger. means to contribute; turn in means to retire.
186. (a) Parallel means aligned, side by side in same direction. 203. (a) Graceful means charming. Awkward means clumsy.
Divergent means develop in different directions. Ignorant means unaware. Slow means unhurried,
Difficult means tough. Similar means same. Dissimilar disloyal means unfaithful.
means different. 204. (b) Attract means to draw attention. Its antonym should
187. (b) Manifest means to exhibit. Its antonym is conceal be repel meaning to push away.
meaning to hide. Display means to show. Marked means 205. (d) Provoke means make angry. Its antonym is pacify,
apparent and suppress means to restrain. meaning making peace. Attract means to draw
188. (a) Blocked means obstructed. Its antonym is facilitated attention; convince means gaining the confidence of;
meaningassist the progress of. Promote means to gratify means satisfy.
advance. Started means to initiate. And checked means 206. (a) Vague means of uncertain, indefinite, or unclear
to inspect. character or meaning. Its antonym should be clear.
189. (c) Fell foul of something means to get into a situation Pleasant means enjoyable; Profound means heartfelt;
where one is opposed to someone or something. Its and sufficient means enough.
antonym is making friends with. To quarrel or attack 207. (d) Viable means feasible or practical. Enviable means
means to get into fight.To show appreciation means to desirable. Unenviable means undesirable. Inviolable
praise someone. means unbreakable. Impracticable means non-feasible.
190. (b) Rewarding means pleasing and fulfilling. Lucrative Thus, we conclude that, for viable, the antonym would
means productive. Its antonym is disappointing which be impracticable.
means unsatisfactory. Thrilling means exciting. Nail 208. (a) Adulation means praise. Therefore option (c) and (d)
biting means getting tensed. can be excluded. Condemnation means to blame
191. (a) Inaugurate means to begin and its antonym is publicly. However option a back- biting is the perfect
'terminate' meaning to finish, cease. Inculcate means antonym for adulation as it means to speak bad about.
to infuse information. Facilitate means to assist. And 209. (a) Obscure means not famous or acclaimed; unclear or
ameliorate means to make better. vague. Infamous means notorious or villainous.
192. (c) Embellish means to make beautiful, decorate. Its Notorious is the same as infamous. Admired means
antonym is spoil meaning to ruin. Demolish means to respected or accepted. Well known means famous.
destroy. Dishonour means a state of shame. Suffice Option a well- known seems the best antonym for the
means adequate. word obscure.
193. (d) Plentiful means abundant. Its antonym should be 210. (b) Impetuous means impulsive or rash. Rash is similar to
scanty. Scanty means insufficient. Handful means a the word itself. Poised means balanced. Sluggish
small quantity. Rare means limited. And small means means lethargic, slow. Aggressive means violent,
little. hostile. Going through the meanings, we can conclude
194. (c) Urbane means civilized and polished. Crude means that option (b) poised should be the correct antonym.
unpolished. Foolish means idiotic, slow means 211. (a) Augment means supplement or enlarge. Diminish
unhurried and elegant means beautiful. means reduce or lessen. Circumscribe means limit or
195. (c) Famous means very well known. Its antonym is restrict. Restrain means hold down. Constrain means
completely unknown. Notorious means known for a to restrain. Since augment means to enlarge then to
trait, ignorant means unaware; reduce should be the correct antonym. Therefore
196. (a) Spurious means fake or false. Its antonym should be option (a) diminish should be the answer.
genuine meaning real. Though authentic and real are 212. (d) Distinct means separate, different. Opposite cannot
also correct because they also mean real. Artificial be antonym for distinct as it means almost the same.
means fake. Different is also similar in meaning to distinct. Uniform
197. (b) Succeed means attain good outcome. Its antonym means consistent, standardized. Similar means the
should be fail. Lose means to be deprived of. same. Distinct is very much opposite to similar. Thus
198. (a) Proud means pleasing. Humble means shy, meek. Kind option (d) is the answer.
means generous. Gentle means mild. Decent means 213. (d) Forbidding means threatening or frightening.
respectable. Handsome means good-looking. Lenient means
199. (d) Leisurely means casual, unhurried and lazy. Idly means compassionate. Filthy means dirty. Mild means gentle
indolently. Foolishly means idiotic. or kind. Mild should be the correct antonym for
forbidding.
EBD_7367
36
B- Antonyms

214. (a) Trivial means unimportant. Important means go faster. Provoke means incite. Out of the following,
significant. Small means little or minute. Easy means option (b), intensify is the correct antonym for slacken.
simple. Difficult means hard. From the above meanings, 228. (d) Zeal means passion. Intolerance means small-
it is clear that option (a) important is the answer. mindedness. Inefficiency means incompetence. Senile
215. (c) Lavish means plentiful, abundant. Wasteful means means having weakness of old age. Apathy means
lavish, extravagant. Big means huge. Frugal means lack of concern. The correct answer is (d) apathy.
prudent, economical. Expensive means luxurious or 229. (a) Defile means pollute or corrupt. Purify means cleanse.
classy. Option (c) frugal is the antonym for lavish. Disturb means bother. Glorify means worship. Option
216. (d) Enriched means augment or supplement. Devalued (a) will be the correct antonym for defile.
means diminished. Depreciated means devalued or 230. (c) Fortuitous means accidental. Unfortunate means
decreased. Impoverished means deprived or ruined. unlucky. Accidental is the synonym of fortuitous.
Seeing the meanings, it can be said that impoverished Planned means scheduled or premeditated. Ludicrous
is the antonym for enriched. means ridiculous. Option (c) planned is the correct
217. (b) Hostile means aggressive or unfriendly. Sincere means answer.
genuine or honest. Friendly means polite. Fair means 231. (b) Aversion means dislike. Promotion means
pale or light. Good means of high-quality. Since hostile endorsement. Attraction means magnetism. Hatred
means unfriendly, the antonym should be friendly. means extreme dislike. Passion means fervour. The
218. (b) Puerile means childish. Impertinent means correct antonym would be (b), attraction.
disrespectful or impolite. Serious means solemn. Inane 232. (d) Imperious means domineering. Characterless means
means silly or absurd. Irrelevant means inappropriate. soulless. Impermanent means temporary. Imperfect
Since puerile means childish, the correct antonym means flawed. Submissive means obedient. The correct
should be serious. answer is option (d) submissive.
219. (b) Laudatory means admiring. Laughable means pathetic. 233. (a) Marvellous means wonderful. Awful means dreadful.
Derogatory means disparaging. Abusive means rude. Mechanical means automatic. Meaningless means
Detriment means loss. Option (b) derogatory is the empty. Unsentimental means hard bitten. The correct
correct answer. antonym is awful.
220. (d) Pertinent means relevant. Eloquent means expressive. 234. (a) Vexation means displeasure. Comfort means soothe.
Distant means far away. Relevant means important. Slyness means cunning. Fright means fear.
Irrelevant means unimportant. Out of these options, Nervousness means anxiety. By looking at the options,
option (d) is the correct antonym. we can say that option (a), comfort is the correct answer.
221. (b) Colossal means huge and massive. Fragile means 235. (b) Doleful means unhappy. Aggressive means violent.
easily broken. Small means little. Colourful means Cheerful means happy. Tired means weary. Involved
vibrant. Impressive means inspiring. From the means occupied. Thus, we see that correct antonym
following options, the correct antonym would be for doleful should be option (b), cheerful.
option (b), small. 236. (a) Senility means weakness of old age. Virility means
222. (b) Indispensable means essential. Tolerable means power of youth. Laziness means lethargy. Maturity
bearable. Superfluous means extra or surplus. means adulthood. Exhaustion means tiredness. The
Expensive means costly. Hostile means unfriendly. Out correct answer is virility.
of the following options, the correct antonym would 237. (d) Ungainly means clumsy. Quick means short. Awkward
be (b), superfluous. means uncomfortable. Graceful means elegant. Option
223. (a) Vindictive means spiteful. Forgiving means merciful. (d), graceful is the correct antonym.
Humane means caring. Polite means courteous. Liberal 238. (b) Sporadic means irregular. Rare means uncommon.
means open-minded. Out of the above options , the Frequent means recurrent. Sharp means pointed.
correct antonym would be option (a) forgiving. Coordinated means synchronized. The correct
224. (b) Frivolous means playful, amusing means funny. antonym is (b), frequent.
Serious means grave. Confusing means puzzling. 239. (a) Costly means expensive. Its opposite is economical.
Teasing means banter. Of the given options, option Frugal means sparing or economical as regards money
(b) is the correct antonym, i.e. serious. or food. Thrifty means using money and other resources
225. (a) Transient means temporary. Lasting means permanent. carefully and not wastefully.
Moving means touching. Persistent means constant. 240. (c) Likeness means something that corresponds. Its
Abiding means enduring. Out of all the options the opposite is difference. Enmity means hatred.
best answer would be (a). 241. (a) Dubious means doubtful or uncertain. Docile means
226. (d) Apparent means evident. Real means genuine. compliant.
Significant means important. Unimportant means 242. (c) Accumulate means to gather or amass something.
insignificant. Vague means unclear. The correct answer Scatter means to disperse. Amass means to gather.
should be option (d), vague. 243. (d), Turn down means to reject. Its opposite is accept. Turn
227. (b) Slacken means loosen. Weaken means deteriorate. up means be found, especially by chance, after being
Intensify means deepen or strengthen. Quicken means lost. Turn over means start or continue to run properly.
Antonyms B-37

244. (c) Material possession means property or belongings that 261. (c) Escalate means to increase rapidly. Its antonym is slope
are tangible. Its opposite should be spiritual. Manual down. Isolate means to set apart. Reject means to refuse.
means done by hand. Bring down means to reduce.
245. (a) Hostile means showing or feeling opposition or dislike; 262. (c) Terminate means to stop. Its antonym is to initiate
unfriendly. Its opposite is friendly. Inimical means meaning to start. Imitate means to ape. Interrupt means
unfriendly. to interfere. Examine means to test.
246. (b) Magnanimous means giving and kind. Petty means 263. (d) Gravitate means be drawn toward. Its antonym is retreat
small, insignificant and trivial. Majestic means meaning to depart. Meditate means to contemplate.
impressive. Deteriorate means to degrade.
247. (a) Defiant means disobedient and disregardful. Its 264. (d)
opposite is obedient. Meek means shy. Rebellious 265. (b)
means disobedient. 266. (c) 'Strict' means demanding that rules concerning
248. (a) Repugnant means extremely distasteful, unacceptable. behaviour are obeyed, whereas ' lenient' means being
Whereas amiable is its complete opposite meaning more merciful or tolerant. Therefore, ' Lenient' is the
having or displaying a friendly and pleasant manner. most appropriate antonym of 'strict'.
Amoral means lacking a moral sense. Repulsive means 267. (c) 'Servitude' means the state of being a slave to someone
arousing intense distaste or disgust. Apolitical means more powerful whereas 'Freedom' means the state of
not interested or involved in politics. not being imprisoned or enslaved.
249. (d) Fresh means newly produced. Its antonym means stale
268 (a) 'Deplorable' means deserving strong condemnation
meaning decayed. Laden means loaded with. Soft
whereas 'Commendable' means deserving praise,
means cushioned. Sour means bad-tasting.
Therefore, 'commendable' is the right antonym of
250. (a) Deny means to refuse. Its antonym is to accept meaning
'Deplorable'.
to obtain. Except means apart from. Accuse means to
269. (b) 'derogatory' means showing disrespectful attitude
place blame for wrongdoing. Curse means bane.
251. (a) Cheerful and happy means the same. Its antonym is whereas 'complimentary' means praising or
sad. Expensive means costly. Careless means casual. appreciating someones efforts. Therefore,
252. (b) Affluence means wealth. Its antonym is poverty. 'complimentary' is the most appropriate antonym of
Continuance means duration. Diffidence means 'derogatory.'
shyness. Insurance means protection. 270. (c) 'aversion' means a strong dislike whereas ' liking'
253. (a) Timid means shy. Its antonym is bold meaning daring. means having a fondness or taste for something.
Bashful means shy. Nervous means anxious. Soft means Therefore, 'liking' is the antonym of 'aversion.'
comfortable. 271. (b) ' Paucity' means presence of something in insufficient
254. (a) Create means to develop. Its antonym is destroy quantity whereas 'Surplus' means an excess of supply.
meaning demolish. Envy means to be jealous. Satisfy Therefore Surplus is the antonym of 'paucity.'
means to be content. Begin means to start. 272. (c) 'monotonous' means lacking in variety and interest
255. (c) Forbid means to ban. Its antonym is to permit meaning whereas 'varied' means showing variation or variety.
to allow. Understand means to comprehend. Defy Therefore, 'varied' is the antonym of 'monotonous'.
means to openly resist. Dislike means not likeable. 273. (c) 'Spurious means false or fake whereas 'genuine' means
256. (c) Master means a man who has people working for him, authentic. Therefore, 'genuine' is the antonym of
especially servants or slaves. Its antonym is slave 'spurious.'
meaning servant. Companion means associate. 274. (a) The opposite of commensurate is ‘disproportionate’,
Follower is a person who supports and admires a hence (a) is the correct option.
particular person or set of ideas. Boss is a person who 275. (a) The opposite of ‘emaciated’ is ‘hefty’ cause the
is in charge of a worker or organization. synonyms of emaciated are lean, thin etc. Hence, the
257. (b) Forbid means to ban. Its antonym is allow meaning correct answer is (a)
grant. Forgive means to grant pardon. Deprive means 276. (b) The opposite of mamooth is ‘tiny’ as it means ‘very
keep or take away something wanted. Refuse means to small’ and ‘mamooth’ means ‘huge’. Hence, the correct
deny. option is (b).
258. (a) Ambiguous means open to more th an one 277. (c) Hilarious means ‘funny or extremely amusing’ and its
interpretation; not having one obvious meaning. Its antonym is ‘serious’. thus i.e., (c) is the correct option.
antonym is definite meaning exact, clear. Constant 278. (b) Erudition means ‘the quality of having or showing
means consistent. Determined means persistent. great knowledge or learning ‘thus it’s opposite will be
Shapeless means formless. (b) ignorance.
259. (b) Counterfeit means fake. Its antonym is genuine meaning 279. (a) Momentous means ‘of great importance or
real. Affirm means declare the truth of something. significance’, it’s opposite is ‘trivial’ meaning ‘of little
Destructive means injurious. Harmonise means adjust. value or importance’. Thus option (a) is the correct
260. (d) Fusion means melding. Its antonym is separation. answer.
Fixture means attachment. Amendment means
correction. Melting means to softening.
EBD_7367
38
B- Antonyms

280. (c) ‘Indicts’ is the opposite of vindicates cause it means 287. (a) Peculiar means different to what is normal or expected
‘to charge with a fault or offence’. Thus the answer is thus, 'common' will be the correct antonym for it.
(c) Customary means usual or traditional, natural means
281. (b) The meaning of ‘temporal’ is ‘related to worldly as existing in or derived from nature and familiar means
opposed to spiritual affairs or secular’ thus it is the well known.
antonym of ‘ecclesiastical’ which means belonging to 288. (c) Charming means very pleasant or attractive.
or connected with church i.e., a religious place. Hence Enchanting means delightfully charming or attractive,
answer is (b). hypnotic means mesmeric, repulsive means arousing
282. (c) Naive means childlike, innocent and simple etc. The intense distaste or disgust and fascinating means
correct antonym of naive will be wise. Rest of the extremely interesting. Among all these words, only
options are its synonyms. repulsive appears to be the correct antonym of
283. (a) Suitable means appropriate, sufficient and acceptable 'charming'.
etc. The correct antonym of suitable will be insufficient. 289. (c) Abominable means awful. Both 'reputable' and
Impertinent means not showing proper respect, 'attractive' are the antonyms of abominable however,
befitting means appropriate to the occasion and in the context of the sentence, reputable suits the most
congenial means like-minded or compatible. to be the antonym of abominable. Abhorrent means
284. (c) Among the given options, the correct antonym of hateful or detestable and repugnant means extremely
criticism is appreciation. distasteful.
285. (b) Anxious means worried or tensed. The correct antonym 290. (c) Fanatical means extreme or zealous. Moderate which
of anxious will be composed which means calm and means average in amount is the correct antonym of
poised etc. Certain means sure or confident, careless fanatical. Bigoted means prejudiced, militant means
means without sufficient attention and heedless means aggressive or violent and fervid means intensely
careless. enthusiastic or passionate, especially to an excessive
286. (b) Activity means the condition in which things are degree.
happening or being done. The correct antonym of 291. (d) Vulnerable means exposed to the possibility of being
activity will be dormant which means inactive or latent. attacked or harmed, either physically or emotionally.
Dull means lacking interest or excitement, indolence Resilient which means able to withstand or recover
means laziness and idle is the synonym of indolent. quickly from difficult conditions is the correct antonym
of vulnerable. Rest of the options are the synonyms
of Vulnerable.
C HA P T E R
SPOTTING ERRORS
28
DIRECTIONS (Qs. 1-15) : 11. The main determination (a) / of economic success is (b) /
(i) In this Section, a number of sentences are given. The our ability to control inflation. (c) / No error. (d)
sentences are underlined in three separate parts and each 12. Because of (a) / extenuating circumstances (b) / the court
one is labelled (a), (b)and (c). Read each sentence to find acquitted him from the crime. (c) / No error. (d)
out whether there is an error in any underlined part. No 13. In consideration for (a) / the bereaved family’s feelings (b) /
sentence has more than one error. When you find an error the papers did not print the story. (c) / No error. (d)
in any one of the underlined parts (a), (b) or (c), indicate 14. No sooner had (a) / he arrived then (b) / he was asked to
your response on the separate Answer Sheet at the leave again. (c) / No error. / (d)
appropriate space. You may feel that there is no error in a 15. I haven’t been (a) / to New York before and (b) / neither my
sentence. In that case, letter (d) will signify a ‘No error’ sister. (c) / No error./(d)
response. [2008-I] DIRECTIONS (Qs. 16-33) :
(ii) You are to indicate only one response for each item in your (i) In this Section, a number of sentences are given. The
Answer Sheet. (If you indicate more. than one response, sentences are underlined in three separate parts and each
your answer will be considered wrong.) Errors may be in one is labelled (a), (b) and (c). Read each sentence to find
grammar, word usage or idioms. There may be a word out whether there is an error in any underlined part. No
missing or there may be a word which should be removed. sentence has more than one error. When you find an error in
(iii) You are not required to correct the error. You are required any one of the underlined parts (a), (b) or (c), indicate your
only to indicate your response on the Answer Sheet. response on the separate Answer Sheet at the appropriate
Examples : ‘P’ and ‘Q’ have been solved for you. space. You may feel that there is no error in a sentence. In
P. The young child / (a) singed / (b) a very sweet song. / (c) No that case letter (d) will signify a ‘No error’ response.
error. / (d) (ii) You are to indicate only one response for each item in your
Q. We worked / (a) very hard/ (b) throughout the season. / (c) Answer Sheet. (If you indicate more than one response,
No error. / (d). your answer will be considered wrong : Errors may be in
Explanation : grammar word usage or idioms. They may be a word
In item P, the word ‘singed’ is wrong. The letter under this missing or there may be a word which should be removed.
part is (b) ; so (b) is the correct answer. Similarly, for item Q, (iii) You are not required to correct the error. You are required
(d) is the correct answer, as the sentence does not contain only to indicate your response on the Answer Sheet.
any error. Examples ‘P’ and ‘Q’ have been solved for you. [2008-II]
1. Old age and infirmity (a) / had began to (b) / catch up with P. The young child singed a very sweet song. No error.
him. (c) / No error. (d) (a) (b) (c) (d)
2. Its colour makes (a) / the moth undistinguished (b) / from Q. We worked very hard throughout the season. No error.
the branch it rests on. (c) / No error. (d) (a) (b) (c) (d)
3. With only a week (a) / to go for the election (b) / things are Explanation :
really hotting up. (c) / No error. (d) In item P, the word ‘singed’ is wrong. The letter under this
4. Romanticism of melancholy (a) / in art and literature are the part is (b), so (b) is the correct answer. Similarly, for item Q
reasons (b) / for insensitivity of those suffering from (d) is the correct answer, the sentence does not contain any
depression. (c) / No error. (d) error.
5. Taking life as it comes, (a) / I am someone who thrives one 16. The watch is a valuable present my uncle and it costed him
challenges and believe that my purpose in life (b) / is to give (a) (b)
one hundred percent to very opportunity that comes my more than two thousand rupees. No error.
way. (c) / No error. (d) (c) (d)
6. The police were baffled (a) / and Sherlock Holmes was (b) / 17. The question is often raised
called in to investigate. (c) / No error. (d) (a)
7. The boat came (a) / abreast at us (b) / and signalled us to that whether it is desirable to send Indian students abroad
stop. (c) / No error. (d) (b)
8. The Chairman’s comments (a) / on future policy introduced since they rarely return. No error.
(b) / a jarring note for the proceedings. (c) / No error. (d) (c) (d)
9. The collapse of the business (a) / was a moral blow to the 18. The work of an uneducated farmer is far important than
(b) / business tycoon and his family. (c) / No error. (d) (a) (b)
10. The taxi (a) / came at 8 o’clock (b) / in the next morning. (c) that of a professor, No error.
/ No error. (d) (c) (d)
EBD_7367
40
B- Spotting Errors

19. Every man is conditioned by the age in which he lives, DIRECTIONS (Qs. 34-49) : [2009-I]
(a) (i) In this Section, a number of sentences are given. The
and if he were to return to another age sentences are underlined in three separate parts and each
(b) one is labelled (a), (b) and (c). Read each sentence to find
he would not be happy No error. out whether there is an error in any underlined part. No
(c) (d) sentence has more than one error. When you find an error
20. Language is a skill activity in any one of the underlined parts (a), (b) or (c), indicate
(a) your response on the separate Answer Sheet at the
by which fine distinctions on meaning can be made appropriate space. You may feel that there is no error in a
(b) sentence. In that case letter (d) will dignify a ‘No error’
for a better understanding of behaviour. No error. response.
(c) (d) (ii) You are to indicate only one response for each item in
21. If you don’t start behaving properly, I will be forced to your Answer Sheet. (If you indicate more than one response,
(a) (b) your answer will be considered wrong.) Errors may be in
tell to your supervisor. No error. grammar, word usage or idioms. There may be considered
(c) (d) wrong.) Errors may be in grammar, word usage or idioms.
22. The reason why the plane crashed There may be a word missing or there may be a word which
(a) should be removed.
as soon as it took off (iii) You are not required to correct the error. You are required
(b) only to indicate your response on the Answer Sheet.
is the failure of one of the engines. No error. Examples ‘P’ and ‘Q’ have been solved for you.
(c) (d) P. The young child singed a very sweet song. No error.
23. He went in the room opened a box (a) (b) (c) (d)
(a) (b) Q. We worked very hard throughout the season. No error.
and took out a gun. No error. (a) (b) (c) (d)
(c) (d)
24. Thousands of people far and near Explanation :
(a) In item P, the word ‘singed’ is wrong. The letter under this
started gathering outside the auditorium part is (b); so (b) is the correct answer. Similarly, for item Q,
(b) (d) is the correct answer, as the sentence does not contain
from early morning to pay their tributes to the departed leader any error.
(c) 34. I went to his house but couldn’t see him
No error. (a) (b)
(d) because he went out before I arrived. No error.
25. Nobody in their sense would have acted so. No error. (c) (d)
(a) (b) (c) (d) 35. When I shall see him. I will tell him that
26. I was baffled with the instructions he gave me. No error. (a) (b)
(a) (b) (c) (d) What he has done is wrong. No error.
27. I always like to have about four toasts for breakfast. No error. (c) (d)
(a) (b) (c) (d)
28. She could not believe that it had all happened to her. No error. 36. Literature remains the interest of a minority
(a) (b) (c) (d) (a)
29. People shouldn’t drop litter on pavements, and the majority has chosen to ignore those aspects of
(a) (b) language
should they? No error. (b)
(c) (d) which, at school they were told to value highly. No error.
30. He had no objection to forward my application (c) (d)
(a) (b) 37. I look forward to meet you in future. No error.
to the higher authority. No error (a) (b) (c) (d)
(c) (d) 38. If I was the king, I would change the face of my country.
31. When the plane landed he found that (a) (b) (c)
(a) (b) No error.
one of the wings is damaged by a shell. No error. (d)
(c) (d) 39. He admits that he is not following the instructions. No error.
32. My friend has got an appointment in a television company (a) (b) (c) (d)
(a) (b) 40. Life on board ship was not as I expected it to be. No error.
some three months ago. No error. (a) (b) (c) (d)
(c) (d)
33. During the earthquake people listened a strange noise 41. He did not pass the examination inspite his best efforts.
(a) (b) (a) (b) (c)
and rushed out of homes. No error. No error.
(c) (d) (d)
Spotting Errors B-41
42. I tried to read your letter, but it was no badly written October. No error.
(a) (b) (d)
that I had to leave the attempt. No error. 51. I have not seen him since twenty years and so I cannot
(c) (d) say
43. I prefer my job to yours. No error. (a) (b)
(a) (b) (c) (d) with certainty whether he is alive or dead. No error.
44. If there a guarantee then we are prepared (c) (d)
(a) (b) 52. He walked five miles which are really a great distance for a
to place to bulk order for your product. No error. (a) (b)
(c) (d) man like him who is not only old but also ill. No error.
45. I meet him once a blue moon (c) (d)
(a) (b) 53. The student requested the teacher to explain him the
so I do not know much about his activities. No error. (a) (b)
(c) (d) theory of relativity with some examples which he could
46. My daughter-in-laws who are in Kolkata (c)
(a) (b) understand easily. No error.
have come to visit us. No error. (d)
(c) (d) 54. When he did not find his cook in the kitchen he asked his
47. He asked me what my name is and where I came from. (a) (b)
(a) (b) (c) wife where had he gone. No error.
No error. (c) (d)
(d) 55. We are proud to announce that every one on our team has
48. There has always been some form of education (a) (b)
(a) (b) earned a good name. No error.
but there has not always been schools. No error. (c) (d)
(c) (d) 56. Either my colleague or a peon are coming home with the
49. At the annual function of the school (a) (b)
(a) (b) material today. No error.
the principal advised to the students to be ideal citizens. (c) (d)
(c) 57. Never I asked my Englishman how much he earned.
No error. (a) (b) (c)
(d) No error.
DIRECTIONS (Qs. 50-67) : [2009-II] (d)
(i) In this Section, a number of sentences are given. The 58. The Vice-Chancellor of our university urged to
sentences are underlined in three separate parts and each (a)
one is labelled (a), (b) and (c). Read each sentence to find the agitating students to shun violence
out whether there is an error in any underlined part. No (b)
sentence has more than one error. When you find an error in and maintain peace on the campus. No error.
any one of the underlined parts (a), (b) or (c), indicate your (c) (d)
response on the separate Answer Sheet at the appropriate 59. When her son got a job she was besides herself with joy.
space. You may feel that there is no error in a sentence. In (a) (b) (c)
that case letter (d) will signify a ‘No error’ response. No error.
(ii) You are to indicate only one response for each item in (d)
your Answer Sheet. (If you indicate more than one response, 60. Here is the man whom I think committed the crime.
your answer will be considered wrong). Errors may be in (a) (b) (c)
grammar, word usage or idioms. There may be a word No error.
missing or there may be a word which should be removed. (d)
(iii) You are not required to correct the error. You are required 61. We have studied the two specimens carefully
(a)
only to indicate your response on the Answer Sheet.
X differs to Y in only one respect. No error.
Examples ‘P’ and ‘Q’ have been solved for you. (b) (c) (d)
P. The young child signed a very sweet song. No error 62. Bread and butter is all we want. No error.
(a) (b) (c) (d) (a) (b) (c) (d)
Q. We worked very hard throughout the season. No error 63. What are their reasons to say it. No error.
(a) (b) (c) (d) (a) (b) (c) (d)
Explanation : 64. He took his younger sister with himself. No error.
In item P, the word ‘singed’ is wrong. The letter under this (a) (b) (c) (d)
part is (b); so (b) is the correct answer. Similarly, for item Q, 65. Can you cite any precedent in support of the case?
(d) is the correct answer, as the sentence does not contain (a) (b) (c)
any error. No error.
50. Neither of them are going to attend the party on 10th (d)
(a) (b) (c)
EBD_7367
42
B- Spotting Errors

66. We must work very hard now 74. Put you in my position and you would realise
(a) (b) (a) (b)
to making up for the lost time. No error. the problems faced in my profession. No error.
(c) (d) (c) (d)
67. Right from his childhood he used to prefer sports than 75. Your Association
(a) (b) (c) is doing good work and we would like to help it
studies. No error. (a) (b)
(d) in meaningful way. No error
(c) (d)
DIRECTIONS (Qs. 68-86): [2010-I] 76. He asked me if I know where the principal lived.
(i) In this Section, a number of sentences are given. The (a) (b) (c)
sentences are underlined in three separate parts and each No error.
one is labelled (a), (b) and (c). Read each sentence to find (d)
out whether there is an error in any underlined part. No 77. Fish and chips is my favourite dish for lunch. No error
sentence has more than one error. When you find an error in (a) (b) (c) (d)
any one of the underlined parts (a), (b) or (c), indicate your 78. One of the peculiarities
response on the separate Answer Sheet at the appropriate (a)
space. You may feel that there is no error in a sentence. In which distinguishes the present age
that case, letter (d) will signify a ‘No error’ response. (b)
(ii) You are to indicate only one response for each item in is the multiplication of books. No error.
your Answer Sheet. (If you indicate more than one response, (c) (d)
your answer will be considered wrong). Errors may be in 79. If you had just hinted at your difficulty
grammar, word usage or idioms. There may be considered (a) (b)
wrong). There may be a word missing or there may be a I would most certainly help you. No error.
word which should be removed. (c) (d)
(iii) You are not required to correct the error. You are required 80. This T. V. serial is going on for 3 years. No error
only to indicate your response on the Answer Sheet. (a) (b) (c) (d)
81. It is my pleasure to congratulate you for your success
Examples ‘P’ and ‘Q’ have been solved for you. (a) (b)
P. The young child singed a very sweet song. No error. in the Civil Services Examination. No error
(a) (b) (c) (d) (c) (d)
Q. We worked very hard throughout the season. No error. 82. Despite of repeated warnings,
(a) (b) (c) (d) (a)
Explanation : he touched a live electric wire
In item P, the word ‘singed’ is wrong. The letter under this (b)
part is (b); so (b) is the correct answer. Similarly, for item Q, and was electrocuted. No error
(d) is the correct answer, as the sentence does not contain (c) (d)
any error. 83. He says that he has renounced the world
68. There will be no more supplies unless all arrears of payment (a)
(a) (b) and that he has nothing
were cleared by next Monday. No error (b)
(c) (d) that he can call as his own. No error
69. After her latest experience of eve-testing she seems worried (c) (d)
(a) (b) 84. Drydus’ prose, which is meant to be popular
as to how she would reach her office everyday. No error (a)
(c) (d) loses nothing of its value
70. He is every bit as guilty as I No error (b)
(a) (b) (c) (d) by being compared with his contemporaries. No error
71. A woman opened the door (c) (d)
(a) 85. In the early years of the renaissance of Bharatanatyam,
and standing at a distance she pushed the plate hereditary dancers have their
(b) (a)
containing the food to him No error own set of accompanists
(c) (d) (b)
72. We were greatly worried that the train might be late who lived with the dancers and travelled with them
(a) (b) (c)
but it arrived exactly in time. No error No error.
(c) (d) (d)
86. At this turn of conversation Vikram blurted out
73. He is one of those few post-colonial writer who believes
(a)
that he was not knowing me
that this talk about colonialism has gone too far (b)
(b) when I used to live in the same town as he. No error.
and has turned in to a cliché. No error. (c) (d)
(c) (d)
Spotting Errors B-43

DIRECTIONS (Qs. 87-108) : Each question in this section has which has promised their full support. No error.
a sentence with three underlined parts labelled (a), (b) and (c) (d)
(c). Read each sentence to find out whether there is my error in 98. He collected his bags, said good-bye to us
any underlined part and indicate your answer in the Answer (a) (b)
Sheet against the corresponding letter i.e., (a) or (b) or (c). If and left for home immediately. No error.
you find no error, your answer should be indicated as (d). (c) (d)
[2010-II] 99. All the players agreed to divide
(a)
87. Young school students now-a days the cash prize between themselves
(a) (b)
are subjected to intense pressure from peers and parents without any argument. No error.
alike (c) (d)
(b) 100. He is very ill; I m afraid he is going to die. No error.
to fetch high marks in public examinations. No error. (a) (b) (c) (d)
(c) (d) 101. The tourist did not know the local language,
88. The candidate’s performance was not upto mark (a)
(a) (b) but he used signs to make people understand
in the interview. No error. (b)
(c) (d) that he wanted to reach to Darjeeling quickly. No error.
89. After a successful tour of Europe
(c) (d)
(a) 102. The Vice Chancellor consulted the students as well as the
my old parents returned back to India on New Year’s day.
teachers
(b) (c)
(a)
No error.
on last Monday and decided to reopen the university on
(d)
(b) (c)
90. The commission set up to submit a report
friday next No error.
(a)
(d)
about the reasons for the fall in educational standards
103. Everybody was trying to shake hand with the Minister.
(b)
(a) (b) (c)
could not complete its work even after two years. No error.
No error.
(c) (d)
91. Though death is a daily fact, it is wonder that people (d)
104. A friend of his received him at the station. No error.
should
(a) (b) (c) (d)
(a) (b)
behave to be immortal. No error. 105. Mohan was your best friend a month ago
(c) (d)
92. It would be more better if you could paint the gate green. but you now seem to
(a) (b) (c) (b)
No error. have broken with him altogether. No error.
(d) (c) (d)
93. Being his sole companion, I was the one to who 106. It is a great loss indeed but how far he is to be blamed for it
(a) (b) (a) (b)
he naturally looked for help. No error. I am not quite sure. No error.
(c) (d) (c) (d)
94. He came to report that the work went very slowly 107. The “Akbar Nama” is among the major historical texts
(a) (a) (b)
because the X-ray machine was not working very good in the Indian past. No error.
(b) (c) (d)
that morning. No error. 108. Whether he is witting about a taxi driver
(c) (d) (a)
95. The principal objected to them wearing short skirts and an interesting sketch of a woman,
(a) (b) (b)
at the function. No error. he is always at his ease. No error.
(c) (d) (c) (d)
96. I spent nearly four and half years at Harrow DIRECTIONS (Qs. 109-133) : Each questions in this section
(a) (b) has a sentence with three underlined parts labelled (a), (b),
of which three were in the Army class. No error. and (c). Read each sentence to find out whether there is any
(c) (d) error in any underlined part and indicate your answer in the
97. Considerable encouragement for the scheme Answer Sheet against the corresponding letter i.e., (a) or (b)
(a) or (c). If you find no error, your answer should be indicated as
has been received from the Bengal Chamber of Commerce (d). [2011-I]
(b)
EBD_7367
B-44 Spotting Errors

109. Everybody, it must be admitted, has their ups and downs. 124. As a dramatist. Shaw is superior than
(a) (b) (c) (a) (b)
No error. any other twentieth century writer. No error.
(d) (c) (d)
110. When the thief broke into their house, they raised a hue 125. Molly speaks French well; isn’t it? No error.
(a) (b) (a) (b) (c) (d)
and cry and the thief caught immediately by the people 126. Mutton is more hard to digest than vegetables. No error.
(c) (a) (b) (c) (d)
No error. 127. I have found that he is neither willing or capable.
(d) (a) (b) (c)
111. I have tried to meet him several times; he isn’t never at No error.
(a) (b) (c) (d)
home. No error. 128. For times immemorial sea shells have been used by man
(d) (a) (b)
112. This house is mine. No error. in many ways. No error.
(a) (b) (c) (d) (c) (d)
113. He is proficient in Hindi and can speak English. 129. He will not listen what you say. No error.
(a) (b) (a) (b) (c) (d)
but he does not know to read and write English. No error. 130. I have done my best; the whole thing is now
(c) (d) (a) (b)
114. Every woman in the world fervently hopes that their child in the hands of the Gods. No error.
(a) (b) (c) (d)
will be a normal and healthy baby. No error. 131. Oh for God sake leave me alone and go away from here.
(c) (d) (a) (b) (c)
115. Neither of them send their papers
No error.
(a) (b)
(d)
in time for the last seminar. No error.
132. This is a strange world
(c) (d)
(a)
116. There is not many traffic along the street where I live.
where each one pursues their own golden bubble
(a) (b) (c)
No error. (b)
and laughs at others for doing the same. No error.
(d)
117. The front page story was about a schoolgirl, and had hurt (c) (d)
(a) (b) 133. Each of the boys were to blame for the accident. No error.
herself, while saving a child in an accident. No error. (a) (b) (c) (d)
(c) (d) DIRECTIONS (Qs. 134-148) : Each question in this section has
118. He took leave of four days. No error. a sentence with three underlined parts labelled (a), (b) and (c).
(a) (b) (c) (d) Read each sentence to find out whether there is any error in any
119. The police arrived and discovered a large number of underlined part and indicate your answer in the Answer Sheet
hoarded against the corresponding letter i.e., (a) or (b) or (c). If you find no
(a) (b) error, your answer should be indicated as (d). [2011-II]
sugar in his shop. No error.
134. The composition contained even no less
(c) (d)
120. Raju doesn’t come to our house because our dog barks at (a) (b)
him than twenty mistakes. No error
(a) (c) (d)
and licks him although I have often told him not to afraid of it 135. He told us that he has not read the book. No error
(b) (c) (a) (b) (c) (d)
No error. 136. The minister announced compensation for
(d) (a) (b)
121. Running across the playground, my pen fell in the mud; the victims from the accident. No error
(a) (c) (d)
fortunately, I noticed it. and picked it up. No error. 137. I should have preferred to go by myself. No error
(b) (c) (d) (a) (b) (c) (d)
122. The last thing that the fond mother gave her only son 138. There is no place in this compartment. No error
(a) (b) (a) (b) (c) (d)
was his blessing. No error. 139. The young man had no manner. No error
(c) (d) (a) (b) (c) (d)
123. To his innovative ideas and practices in farming 140. There are many beautiful furnitures in the room.
(a) (a) (b) (c)
he was given the Krishi Pandit Award last year. No error. No error
(b) (c) (d) (d)
Spotting Errors B-45
141. The policeman prevented us from entering into the hall. who doubts the truth of his statement. No error.
(a) (b) (c) (c) (d)
No error 155. I like this book because the writer has explained
(d) (a)
142. The world comprises good and bad people. No error the reasons of his failure truly. No error.
(a) (b) (c) (d) (b) (c) (d)
143. Would you please order for tea and biscuits for all of us? 156. She is very weak in the subject and does not understand
(a) (b) (c) (a) (b)
No error things though the teacher explains her repeatedly.
(d) (c)
144. I have paid my bill for electricity only a week ago. No error.
(a) (b) (c) (d)
No error 157. The speaker from the Fifth Avenue, who was a rich banker's
(d) (a) (b)
145. He has just been appointed wife was simple and compassionate. No error.
(a) (c) (d)
ambassador to an important country 158. There was no any piece of paper in my pocket
(b) (a) (b)
for a five-year term. No error as I had expected. No error.
(c) (d) (c) (d)
146. This is the old man whom I said had helped me. 159. Neither the teacher or the student is keen on joining the
(a) (b) (c) (a) (b) (c)
No error dance. No error.
(d) (d)
147. The dog pushed the door open and stole the meat. 160. My neighbour Deepak is a person that will help anyone
(a) (b) (c) (a) (b) (c)
No error No error.
(d) (d)
148. The truck driver accused the lady for walking 161. I'll ask that man which of the roads are the one we want.
(a) (b) (a) (b) (c)
in the middle of the road. No error. No error.
(c) (d) (d)
DIRECTIONS (Qs. 149-168) : Each question in this section has 162. Now we have banks and people deposit there money their,
a sentence with three underlined parts labelled (a), (b) and (c). (a) (b)
Read each sentence to find out whether there is any error in any and draw it out by cheques. No error.
underlined part and indicate your answer in the Answer Sheet (c) (d)
against the corresponding letter i.e. (a) or (b) or (c). If you find 163. Apart government agencies, a number of private organisations
no error, your answer should be indicated as (d). [2012-I] (a) (b)
149. These are the ideas and ideals which have shaped too have been making use of satellites. No error.
(a) (b) (c) (d)
our economic thought in the past, No error. 164. What sort of a drug this is that no one seems to be able to
(c) (d) (a) (b)
150. India's problems are not similar with those of other countries predict its long-term effects with any certainty ?
(a) (b) (c)
in several ways. No error. No error.
(c) (d) (d)
151. He had lost a ring in the sand and I helped him search for it, 165. You will lose your dog if you did not tie it up. No error.
(a) (b) (a) (b) (c) (d)
but it was like a look for a needle in a haystack. No error. 166. In view of the fact that almost all varieties of rural games
and sports are fast gaining national importance it is desired
(c) (d)
152. The Ganges and it's tributaries constitute (a) (b)
that the rules of such games are strictly adhered.
(a) (b)
(c)
one of the largest river-systems in the world. No error.
No error.
(c) (d)
(d)
153. The sudden change of place effected her health.
167. More than one workmen was killed. No error.
(a) (b) (c)
(a) (b) (c) (d)
No error.
168. The parties disagreed on the two first clauses
(d)
(a) (b)
154. There are a number of people of every class and nationality
in the agreement. No error.
(a) (b)
(c) (d)
EBD_7367
46
B- Spotting Errors

DIRECTIONS (Qs. 169-186) : Each question in this section has 182. He is going everyday for a morning walk
a sentence with three underlined parts labelled (a), (b) and (c). (a) (b)
Read each sentence to find out whether there is any error in any with his friends and neighbours. No error.
underlined part and indicate your answer in the Answer Sheet (c) (d)
against the corresponding letter i.e., (a) or (b) or (c). If you find 183. Her relatives could not explain to us
no error, your answer should be indicated as (d). [2012-II] (a)
why did not she come for the wedding
169. The scientist was seemed to be excited (b)
(a) (b) as she was expected. No error.
over the result of his experiment. No error.
(c) (d)
(c) (d)
184. He was prevented to accept the assignment
170. The student could not answer the teacher
(a) (a)
when he was asked to explain because he was a government employee
(b) (b)
why he was so late that day. No error. and as such barred from accepting such assignments.
(c) (d) (c)
171. John could not come to school as he was ill from cold. No error.
(a) (b) (c) (d)
No error. 185. If you repeat this mistake, I will inform to your father
(d) (a) (b)
172. Though she has aptitude in Mathematics and do not blame me then. No error.
(a) (c) (d)
I won't allow her to take it up as a subject of study for the 186. Lieutenant Anand was short and muscular
Master's degree (a)
(b) with shoulders that bulged impressively
because I know the labour involved will tell upon her health. (b)
(c) against his smart uniform. No error.
No error. (c) (d)
(d)
173. I am not familiar with all the important places in this town, DIRECTIONS (Qs. 187-201) : Each question in this section
(a) (b) has a sentence with three underlined parts labelled (a), (b)
although I have been living here since two years. No error. and (c). Read each sentence to find out whether there is any
(c) (d) error in any underlined part and indicate your answer in the
174. If I would be a millionaire, I would not have wasted my time Answer Sheet against the corresponding letter i.e., (a) or (b)
(a) (b) or (c). If you find no error, your answer should be indicated as
waiting for a bus. No error. (d). [2013-I]
(c) (d)
187. I should do the same if I were in your place. No error.
175. Until you begin to make a better use of your time,
(a) (a) (b) (c) (d)
I shall not stop finding fault in you. 188. He has been suffering with fever for the last six weeks.
(b) (c) (a) (b) (c)
No error. No error.
(d) (d)
176. Neither of the two boys is sensible 189. The examination begins from Monday next weak.
(a) (b) (a) (b) (c)
enough to do this job. No error. No error.
(c) (d) (d)
177. They left their luggages at the railway station. No error. 190. My father says that one should always be sincere to his duties.
(a) (b) (c) (d) (a) (b) (c)
178. You will get all the informations No error.
(a) (b) (d)
if you read this booklet carefully. No error. 191. There has been a number of railway accidents
(c) (d) (a) (b)
179. She sang very well, isn't it ? No error. during the last month. No error.
(a) (b) (c) (d) (c) (d)
180. He is working in a bank in New Delhi 192. In spite of all efforts to eradicate malaria it still prevalent
(a) (b) (a) (b)
for the past several months. No error. in many parts of India. No error.
(c) (d)
(c) (d)
181. There is no question of my failing in the examination.
193. It is only three days ago that he has arrived.
(a) (b) (c)
No error. (a) (b) (c)
(d) No error.
(d)
Spotting Errors B-47
194. He has lost all what I gave him. No error. 205. He has been going to the office for a year now
(a) (b) (c) (d) (a) (b)
195. I have no news from him for a long time. No error. and he even can't understand its working.
(a) (b) (c) (d) (c)
196. Mahatma Gandhi’s entire life No error.
(a) (d)
was one unrelenting experiment on truth. No error 206. He boasts of having visited Europe many times
(b) (c) (d) (a)
197. As the thieves ran out of the bank but he can neither speak English
(a) (b)
they got into the getaway car nor he can speak French. No error.
(b) (c) (d)
which was waiting with its engine running. 207. Whenever possible, one should avail the opportunity
(a)
(c)
that come one's way if one wants to achieve success in life
No error
(b) (c)
(d) No error.
198. He denied that he had not stolen my purse. (d)
(a) 208. When my friends came to visit us at the railway station
though I was quite sure that he had. No error (a) (b)
(b) (c) (d) they left some of their luggages. No error.
199. The media of films has been accepted by all (c) (d)
(a) 209. As an officer he not only was competent but also honest.
as the most powerful force (a) (b) (c)
(b) No error.
that influences the younger generation. (d)
(c) 210. If you will come tomorrow we can go to the market
No error. (a) (b)
(d) and do our own shopping together. No error.
200. The French Embassy employs him regularly. (c) (d)
(a) (b) 211. If we exercise regularly we will be more healthier.
as he knows to speak French (a) (b) (c)
(c) No error.
No error (d)
(d) 212. News travel very fast today
201. How is it that neither your friend Mahesh (a) (b)
(a) due to advancement in technology. No error.
nor his brother Ramesh (c) (d)
(b) 213. The Chairman made it clear at the meeting
have protested against this injustice. No error. (a)
(c) (d) that he will not step down from his position as chairman
(b) (c)
DIRECTIONS (Qs. 202-220) : Each question in this section has No error.
a sentence with three underlined parts labelled (a), (b) and (c). (d)
Read each sentence to find out whether there is any error in any 214. We had lot difficulty in finding the way here. No error.
underlined part and indicate your answer in the Answer Sheet (a) (b) (c) (d)
against the corresponding letter i.e., (a) or (b) or (c). If you find 215. Just as he was driving along the road,
no error, your answer should be indicated as (d). [2013-II] (a)
202. Lack of winter rains have delayed the sowing of a bus pulled up and the driver asked him
(a) (b) (b)
wheat crop in this area. No error. if he has seen a briefcase on the road. No error.
(c) (d) (c) (d)
203. The teacher let the boy off with a warning 216. Experience has taught me
(a) (b) (a)
though he was convinced with his guilt. No error. not to ignore any man, high or low,
(c) (d) (b)
204. Our first trip was the most interesting one. not to ignore anything great or small. No error.
(a) (c) (d)
but our second one was even more interesting. 217. I have spent most of my money.
(a) (b)
(b) (c)
so I can travel only by bus. No error.
No error.
(c) (d)
(d)
EBD_7367
48
B- Spotting Errors

218. When he asked me as to why 228. Now it can be easily said


(a) (a)
I had not finished my work in time. I felt confused. that the population of this city is greater
(b) (c) (b)
No error. than any other city in India. No error
(d) (c) (d)
219. The Foreign Minister said 229. It is difficult to explain why did Raj gopalachari resigned
(a) (a) (b)
there was no use to criticize the policy of non-alignment from the Congress in 1940. No error
(b) (c) (d)
which had stood the test of time. No error. 230. The boss reminded them of the old saying
(c) (d) (a)
220. The train should arrive at 7-30 in the morning that honesty was the best policy,
(a) (b) (b)
but it was almost an hour late. No error. and told them that they had better be honest in their work.
(c) (d) (c)
DIRECTIONS (Qs. 221-240) : Each question in this section has No error
a sentence with three underlined parts labelled (a), (b) and (c). (d)
Read each sentence to find out whether there is any error in any 231. "Gulliver's Travels" are
underlined part and indicate your answer in the Answer Sheet (a)
against the corresponding letter i.e. (a) or (b) or (c). If you find the most fascinating adventure story
no error, your answer should be indicated as. (d). [2014-I] (b)
221. He asked her that whether she knew that I have ever read. No error
(a) (b) (c) (d)
232. The teenager reassured his father at the station
what had happened last week No error
(a)
(c) (d)
"Don't worry, dad'
222. Until you do not go to the station to receive him
(b)
(a) (b)
I will pull on very nicely at the hostel." No error
I can hardly feel at ease. No error
(c) (d)
(c) (d)
233. The way he's behaving, he'll soon spill the beans,
223. I did not know where they were going nor could I
(a) (b)
understand
I'm afraid. No error
(a) (b) (c) (d)
why had they left so soon. No error 234. Most of the developing countries find it
(c) (d) (a)
224. The distinguished visitor said that he had great pleasure to difficult to cope up with the problems
be with us for some time (b)
(a) created by the sudden impact of technological progress.
and that the pleasure was all the greater (c)
(b) No error
because his visit afforded him an opportunity (d)
(c) 235. People blamed him for being a coward person.
to study the working of an institution of such eminence as (a) (b) (c)
ours. No error No error
(d) (d)
225. Please convey my best wishes back to your parents. 236. We swam up to the drowning man, caught hold of his clothes
(a) (b) (c) (a)
No error before he could go down again
(d) (b)
226. The call of the seas have always found an echo in me. and pulled him out, safe to the shore. No error
(a) (b) (c) (c) (d)
No error 237. Meena was so tired that she could not hardly
(d) (a) (b)
227. Hardly I had left home for Bombay. talk to the guests for a few minutes. No error
(a) (c) (d)
when my son who is settled in Calcutta arrived 238. If I was knowing why he was absent,
(b) (a) (b)
without any prior information. No error I would have informed you. No error
(c) (d) (c) (d)
Spotting Errors B-49
239. He goes to office by foot. No error 253. I wanted to see whether they
(a) (b) (c) (d) (a) (b)
240. The hundred-rupees notes had actually read the notes. No error
(a) (c) (d)
that he gave them for the goods bought from them 254. They made him treasurer because they considered
looked genuine but later they reliably learnt that the (a) (b)
(b) him to be honest and efficient No error
notes were all counterfeit. No error (c) (d)
(c) (d) 255. Having finished the paper early he had came out of the hall
DIRECTIONS (Qs. 241-265) : In this section, you are required (a) (b)
to spot errors in sentences. Each sentence is divided into three almost an hour before the bell rang. No error
parts. Read each sentence to find out whether there is an error (c) (d)
in any of the parts. No sentence has more than one error. Some 256. The young man had no manners No error
of the sentences do not have any error. When you find an error in (a) (b) (c) (d)
a sentence, the letter indicated under that part of the sentence is 257. No news is good news No error
the answer and therefore the same may be marked on the
(a) (b) (c) (d)
separate Answer Sheet. If there is no error in any part, your
258. The work involved is almost impossible
answer should be indicated as (d). [2014-II]
(a) (b)
241. He went to England to work as a doctor to cope with No error
(a) (c) (d)
but returned 259. There is no seats in this compartment No error
(b)
as he could not endure the weather there. No error (a) (b) (c) (d)
(c) (d) 260. Shakespeare is greater than any poet No error
242. She inquired whether anyone seen her baby. No error (a) (b) (c) (d)
(a) (b) (c) (d) 261. I should have preferred to go by myself No error
243. When I went outdoor I found frost everywhere. (a) (b) (c) (d)
(a) (b) (c) 262. The minister announced compensation for
No error (a) (b)
(d) the victims from the accident No error
244. These are his conclusion remarks. No error (c) (d)
(a) (b) (c) (d) 263. The Australian team losed the match
245. The shopkeeper offered either to exchange (a) (b)
(a)
yesterday No error
the goods or refund the money. No error
(b) (c) (d) (c) (d)
246. Churchill was one of the greatest war leaders. No error 264. He told us that he has not read the book
(a) (b) (c) (d) (a) (b) (c)
247. We should keep such people No error
(a) (b) (d)
at an arm's length. No error 265. The composition contained even no less
(c) (d) (a) (b)
248. He did not know as much as he claimed he knew. than twenty mistakes No error
(a) (b) (c)
(c) (d)
No error
(d) DIRECTIONS (Qs. 266-285): Each question in this section has
249. That was very dangerous : you might a sentence with three underlined parts labelled (a), (b) and (c).
(a) (b) Read each sentence to find out whether there is any error in any
have been killed. No error
(c) (d) underlined part and indicate your response in the Answer Sheet
250. My friend is going to a movie against the corresponding letter i.e., (a) or (b) or (c). If you find
(a) (b) (c) no error, your response should be indicated as (d). [2015-I]
every week. No error
(d) 266. The reason for his failure is because he did not work hard.
251. They sit at the window and watch the traffic (a) (b) (c)
(a) (b) (c) No error.
No error (d)
(d) 267. Food and water is necessary for life. No error.
252. I started early for the station lest 1 (a) (b) (c) (d)
(a) (b) 268. India is larger than any democracies in the world No error.
should miss the train No error (a) (b) (c) (d)
(c) (d)
EBD_7367
50
B- Spotting Errors

269. The Judge heard the arguments of the lawyers and found DIRECTIONS (Qs. 286-300): Each below item has a sentence
(a) (b) with three underlined parts labelled (a), (b) and (c). Read each
that the boy was innocent. No error. sentence to find out whether there is any error in any underlined
(c) (d) part and indicate your response in the Answer Sheet against
270. I have lived in Delhi from 1965. No error. the corresponding letter i.e., (a) or (b) or (c). If you find no
(a) (b) (c) (d) error, your response should be indicated as (d). 2015-II
271. All scientists agree that there should be
(a) (b) Of all those involved with the accident
a total ban on nuclear explosions. No error. 286.
(c) (d) (a) (b)
272. Such books which you read are not worth reading. none was seriously injured No error
(a) (b) (c)
No error. (c) (d)
(d) Radar equipments that is to be used
273. Tagore was one of the greatest poet 287.
(a) (b) (a) (b)
that ever lived. No error. for ships must be installed carefully. No error
(c) (d)
274. You may please apply for an advance of salary (c) (d)
(a) (b) New types of electrical circuits has been developed
to cover costs of transport. No error. 288.
(a) (b)
(c) (d)
275. The taxi that will take the family to Haridwar by our engineers. No error
(a)
(c) (d)
had to be ready at six the next morning. No error.
(b) (c) (d) Recently I visited Kashmir and found the sceneries
276. Employees are expected to adhere the rules 289.
(a) (b)
(a) (b)
laid down by the management. No error. to be marvellous. No error
(c) (d)
(c) (d)
277. The owner of the horse greedily ask
(a) (b) It is of primary importance in swimming to learn
too high a price. No error. 290.
(a) (b)
(c) (d)
278. I convinced him to see the play. No error. to breathe properly. No error
(a) (b) (c) (d)
(c) (d)
279. Some man are born great. No error.
(a) (b) (c) (d) When the party was over, he looked around for the girl
280. We must sympathise for others in their troubles. 291.
(a) (b)
(a) (b) (c)
No error. who had come with him. No error
(d) (c) (d)
281. My detailed statement is respectively
(a) (b) After we were driving for miles on the winding road
submitted. No error. 292.
(a) (b)
(c) (d)
282. I am waiting for my friend since morning. I was suddenly sick. No error
(a) (b) (c) (c) (d)
No error.
(d) The forecast was for fair and warm weather
283. He is representing my constituency 293.
(a) (b)
(a) (b)
for the last five years. No error. and the day dawned dark and chill. No error
(c) (d)
(c) (d)
284. If he hears of your conduct he is to be unhappy.
(a) (b) (c) To write, to speak or to act seems very easy.
No error. 294.
(d) (a) (b) (c)
285. No sooner he appeared on the stage than the people
(a) (b) No error
began to cheer loudly. No error. (d)
(c) (d)
Spotting Errors B- 51

I have not had tea since two days. No error 304. I hoped that the train will arrive on time,
295. (a) (b)
(a) (b) (c) (d)

Beside his mother he has two aunts but it did not No error
296. (c) (d)
(a) (b)

who stay with him. No error Their all belongings were lost
305.
(c) (d) (a) (b)
in the fire No error
This photograph appears to be the best of the two.
297. (c) (d)
(a) (b) (c)
He was in the temper and refused
No error 306.
(a) (b)
(d)
to discuss the matter again. No error.
Either the operator or the foreman are (c) (d)
298.
(a) (b)
The decorations in your house. are similar
to blame for the accident. No error 307.
(a) (b)
(c) (d)
to his house. No error.
The article offers good advice to (c) (d)
299.
(a) (b)
308. Despite of the increase in air fares.
whomever must accept it. No error (a)
(c) (d)
most people still prefer to travel by plane No error
She dislikes you being most organized (b) (c) (d)
300.
(a) (b)
He told the boys that if they worked hard.
309.
than she is. No error (a) (b)
(c) (d)
they will surely pass. No error.
DIRECTIONS (Qs. 301-315) : Each question in this section has (c) (d)
a sentence with three underlined parts labelled (a), (b) and
(c). Read each sentence to find out whether there is any error in I shall write to you when I shall reach Chennai.
any underlined part and indicate your response in the Answer 310.
(a) (b) (c)
Sheet against the corresponding letter i.e., (a) or (b) or (c). If
you find no error, your response should be indicated as (d). No error.
[2016-I] (d)
This hardly won freedom should not be lost Neither of these two documents support your claim
301. 311.
(a) (b) (a) (b)
so soon No error on the property. No error.
(c) (d) (c) (d)
I tried to meet the person whom you said He is school teacher, but all his sons are doctors.
302. 312.
(a) (b) (a) (b) (c)
was looking for me. No error No error.
(c) (d) (d)
We looked after the thief. but he was nowhere His grandfather
303. 313.
(a) (b) (a)
to be found. No error had told him to smoke was a bad habit. No error.
(c) (d) (b) (c) (d)
EBD_7367
52
B- Spotting Errors

My book which I gave it to you yesterday to attend the class the previous day
314.
(a) (b)
because of heavy rains. No error.
is very interesting No error (c) (d)
(c) (d) 329. Coleridge as well as Wordsworth were of the opinion that
(a)
I am entirely agreeing with you, but I regret the opposite of poetry is not prose but science.
315.
(a) (b) (b) (c)
No error.
I can't help you. No error.
(d)
(c) (d) 330. He was courted arrest in order to protest against
DIRECTIONS (Qs. 316-335): Each question in this section has (a) (b)
a sentence with three underlined parts labelled (a), (b) and (c). corruption among the government servants. No error.
Read each sentence to find out whether there is any error in any (c) (d)
underlined part and indicate your response in the Answer Sheet 331. Mr. Joshi was, however, sure that the idea
against the corresponding letter i.e., (a) or (b) or (c). If you find (a)
no error, your response should be indicated as (d). [2016-II] would never work in practice. No error.
(b) (c) (d)
316. I am senior than him by two years. No error. 332. As I was leaving for Delhi, he asked me whether I could
(a) (b) (c) (d) (a) (b)
317. When I finished writing the letter, buy a tape recorder for him. No error.
(a) (c) (d)
I could not help admiring myself
333. The boy's parents pleaded with the Principal
(b)
(a) (b)
to have achieved the impossible. No error.
that they were too poor to pay his tuition fee.
(c) (d)
(c)
318. I am glad that you are here. No error.
No error
(a) (b) (c) (d)
(d)
319. He will be cured from his fever. No error.
(a) (b) (c) (d) 334. For young Donald, peace in Vietnam
320. Though he is a gifted comedian, (a) (b)
(a) was almost terrible as war. No error.
he prefers spend his spare time watching horror movies. (c) (d)
(b) (c) 335. That Brutus, who was his trusted friend
No error. (a)
(d) had attacked on him
321. The writer of this poetry is Wordsworth. No error. (b)
(a) (b) (c) (d) caused heartbreak to Julius Caesar. No error.
322. The jug is made out of china clay No error. (c) (d)
(a) (b) (c) (d) DIRECTIONS (Qs. 336-361): Each question in this section has
323. Sita with all her sisters were here. No error. a sentence with three underlined parts labelled (a), (b) and (c).
(a) (b) (c) (d) Read each sentence to find out whether there is any error in any
324. As you know that the ignorant are easily duped. underlined part and indicate your response in the Answer Sheet
(a) (b) (c) against the corresponding letter i.e., (a) or (b) or (c). If you find
No error. no error, your response should be indicated as (d). 2017-1
(d)
325. Pay attention to what I am saying. No error. 336. I waited for her return with growing unease. No error.
(a) (b) (c) (d) (a) (b) (c) (d)
326. One of the assistant was Alfred. No error. 337. I do not understand why
(a) (b) (c) (d) (a)
327. Though George is a honourable man his activities in spite of my best efforts to please him
(a) (b) (b)
my boss is so angry at me. No error.
arouse suspicion. No error. (c) (d)
(c) (d) 338. Being a rainy day, we did not feel like going out or doing
328. She told her teacher that she could not be able (a) (b)
(a) (b) anything
Spotting Errors B-53

except playing chess in our room. No error. 352. The memoranda are on the table. No error.
(c) (d) (a) (b) (c) (d)
339. It is not difficult to believe that a man 353. Nandita asked me if I was working hardly these days.
(a) (a) (b) (c)
who has lived in this city for a long time No error.
(b) (d)
he will never feel at home anywhere else in the world. 354. He couldn't find an answer. No error.
(c) (a) (b) (c) (d)
No error. 355. Whom you think will be dismissed first ? No error.
340. Although we reached his house on time he was left (a) (b) (c) (d)
(a) (b) 356. Of the many problems that confront the leaders of the world
for the airport. No error. (a)
(c) (d) none are of grave consequence
341. If a thing is worth doing at all it is worth done well. (b)
(a) (b) (c) than the problem of saving the normal human race from
No error. extinction.
(d) (c)
342. All the boys returned back home well in time for lunch. No error.
(a) (b) (c) (d)
No error. 357. All of them speak good English. No error.
(d) (a) (b) (c) (d)
343. Tell me the name of 358. The peon has not sweeped the floor today. No error.
(a) (a) (b) (c) (d)
a country where every citizen is law-abiding 359. Her parents has not permitted her to marry Sunil. No error.
(b) (a) (b) (c) (d)
and no trouble is there. No error. 360. Emperor Ashoka have conquered Kalinga
(c) (d) (a) (b)
344. Considering about these facts before he embraced Buddhism. No error.
(a) (c) (d)
the principal has offered him a seat. No error. 361. Every student should be asked
(b) (c) (d) (a) (b)
345. His friends feel that he will be suspended to give their ideas on the subject. No error.
(a) (b) (c) (d)
unless he does not report for duty immediately.
DIRECTIONS (Qs. 362-386): Each item in this section has a
(c)
sentence with three underlined parts labelled (a), (b) and (c).
No error.
Read each sentence to find out whether there is any error in any
(d)
underlined part and indicate your response in the Answer Sheet
346. One of the most interesting feature of travel in Himachal
against the corresponding letter i.e., (a) or (b) or (c). If you find
Pradesh is
no error, your response should be indicated as (d). [2017-II]
(a)
the large number of travellers' lodges
It is identification with the audience
(b) 362.
(a )
provided by the State Government. No error.
that makes one come home from the play so
(c) (d)
347. It was raining cats and dogs. No error. much more
(a) (b) (c) (d) ( b)
348. I prefer this book than that one. No error.
(a) (b) (c) (d) satisfied than one ever is after merely passive
349. Their belongings were lost in the fire. No error. enjoyment of the show.
(a) (b) (c) (d)
350. Tell him to take another photograph of the group. No
( c)
error. No error.
(a) (b) (c) (d)
351. I courteously asked him where was he going
(d)
(a) (b) CV Raman was one of the greatest sons of India
but he did not reply. No error. 363.
( a)
(c) (d)
EBD_7367
54
B- Spotting Errors

who has earned everlasting fame in the evening. No error.


( b) ( c) (d)
for scientific researches. No error. Each student
373.
( c) (d) (a )
This box is heavy than the other one. No error. from amongst the hundred students in the class
364.
(a ) ( b) ( c) (d) ( b)
writer does not have the freedom want to watch this movie. No error.
365. The
( a) ( c) (d)
Although there is virtually no production in India,
to choose his own themes, 374.
( b) ( a)
the Encyclopaedia Britannica estimate
society thrusts them on him. No error.
( b)
( c) (d)
that India has perhaps the largest accumulated stocks
No one knows as to why he did it,
366. of silver in the world.
(a ) ( b) ( c)
or who was behind his doing it. No error. No error.
( c) (d) (d)
How long you are in this profession ? No error. We have to reach there at ten
367.
(a) ( b) ( c) (d) 375.
(a ) ( b)
I know that ignorance is not bliss, will you please walk little faster. No error.
368. ( c) (d)
(a ) (b)
yet I am ignorant in many things. No error. It is almost difficult, in case impossible,
376.
( c) (d) (a ) ( b)
You are just sixteen years old, isn ' t it ? to keep awake late after dinner. No error.
369.
( a) ( b) ( c) ( c) (d)
What most students need, above all else
No error. 377.
(d) (a)
is practice in writing
The old widower, living in remittances from his sons,
370. ( b)
(a ) ( b)
and particularly in writing things that matter to them.
could not make both ends meet. No error. ( c)
( c) (d)
No error.
371.
Debate about biotechnology and genetic engineering (d)
(a ) She was out of the mind,
378.
is under way around the world, ( a)
( b) when she made that plan to go abroad
and India is fully engrossed with the discussion. (b)
( c) without taking into consideration her present
No error. family position.
(d) ( c)
I like to listen the song of the nightingale No error.
372.
(a) ( b) (d)
Spotting Errors B-55

Are you through with that newspaper ? No error. The best way in which you can open the bottle is
379. 387.
(a) (b) ( c) (d) (a ) ( b)
My college is besides the lake. No error. by putting it into hot water first. No error.
380.
(a ) ( b) ( c) (d) ( c) (d)
The Department of Fine Arts has been criticised Somebody who I enjoy reading is Tagore.
381. 388.
(a ) (a ) ( b) ( c)
for not having much required courses
No error.
( b) (d)
scheduled for this semester. No error.
Electricity companies are working throughout
( c) (d) 389.
(a ) ( b)
If you have thought about the alternatives,
382.
(a ) days and nights to repair the damage. No error.
( c) (d)
you would not have chosen
( b) 390.
The students ' test results were pleasant No error.
such a difficult topic for the term paper. No error. (a ) ( b) ( c) (d)
( c) (d) Two thirds of the book were rubbish. No error.
391.
The duties of the secretary are ( a) (b) ( c) (d)
383.
(a ) You will be answerable for the court with
392.
to take the minutes, mailing the correspondence, (a ) ( b)
( b)
any lies you have told. No error.
and calling the members before meeting. No error.
( c) (d)
( c) (d)
She felt terribly anxious for have to sing
If I was you, I would not go to film 393.
384.
( a) (b) (a ) ( b)
in my mother's absence. No error. in front of such a large audience. No error.
( c) (d) ( c) (d)
385.
I don ' t agree with smacking children
Those of us who have a family history of heart disease 394.
(a ) ( b)
( a)
if they do something wrong. No error.
should make a yearly appointment
( b) ( c) (d)
with their doctors. No error. The fruit can be made to jam. No error.
395.
( c) (d) (a) ( b) ( c) (d)
The old furnitures was disposed of
386. I asked him what he has done. No error.
(a) ( b) 396.
(a ) ( b) ( c) (d)
and the new ones were placed. No error.
There have been a tornado watch
( c) (d) 397.
(a )
DIRECTIONS (Qs. 387-406): Each item in this section has a
sentence with three underlined parts labelled (a), (b) and (c). issued for Texas country until eleven O 'clock tonight.
Read each sentence to find out whether there is any error in any ( b) ( c)
underlined part and indicate your response on the Answer Sheet
against the corresponding letter i.e., (a) or (b) or (c). If you find No error.
no error, your response should be indicated as (d). [2018-1] (d)
EBD_7367
B- 56 Spotting Errors

398. Although the Re d Cross accepts blood from the 402. Gandhi ji always regretted for the fact
donors (a )
(a ) that people gave him adulation while what
the nurses will not leave you give blood, he wanted
( b) ( b)
if you have just had cold. No error. was acceptance of his way of life. No error.
( c) (d) ( c) (d)
399. A prism is used to refract white light The party was ousted in power after twelve years.
403.
(a ) (a ) (b) ( c)
so it spreads out in a continuous spectrum. No error.
( b) ( c) (d)
No error. the prevention of Terrorism act.
404. He was held in
(d) (a) ( b) ( c)
400. Because of the movement of a glacier, the form No error.
(a ) (b) (d)
of the Great Lakes was very slow. No error. He has great fascination for each and every thing
405.
( c) (d) (a ) ( b)
401. The new model costs twice more than that are connected with drama. No error.
(a ) ( b) ( c) (d)
last year's model. No error. It's no secret that the President wants to
406.
( c) (d) ( a) ( b)
have a second term of office. No error.
( c) (d)

HINTS & SOLUTIONS


13. (a) Replace 'for' with 'of'.
1. (b) Replace 'began' with 'begun'. 14. (b) Replace 'then' with 'than'.
2. (b) Explanation:It should be 'undistinguishable'. 15. (c) Replace 'neither my sister' with 'neither has my sister.'
3. (d) No error 16. (b) It should be 'cost' instead of 'costed'.
4. (b) Romanticism of melancholy in art and literature is the 17. (b) Delete 'that'.
reason …… is the correct sentence. 18. (b) It should be 'far more'.
5. (b) Replace 'someone' with 'the one.' 19. (a) Replace 'by' with 'to'.
6. (d) No error 20. (b) It should be 'distinction'.
7. (b) 'Abreast' is used to say that two or more people are 21. (c) Delete 'to'.
next to each other and moving in the same direction: 22. (c) Replace 'crashed with 'had crashed'.
The motorcyclist came abreast of her car and shouted 23. (a) Replace 'in' with 'inside'.
abuse at her. Here, replace 'at' with 'of'. 24. (a) Replace 'far and near' with 'from far and near.'
8. (c) Replace 'for' with 'to'. 25. (b) Replace 'sense' with 'senses'.
9. (d) No error 26. (b) Replace 'with' with 'by'.
10. (c) Replace 'in the next morning' with 'next morning'. 27. (b) The adverb 'always' demonstrates a habit here, so,
11. (a) Replace 'determination' with 'determinant'. about is redundant.
12. (c) Replace 'from' with 'of'.
Spotting Errors B-57
28. (d) No error 87. (d) No error
29. (d) No error 88. (b) Add 'the'; "was not up to the mark".
30. (b) It should be 'in forwarding'. 89. (b) Delete 'back'.
31. (c) Replace 'is' with 'was'. 90. (a) Replace 'to submit' with 'for submitting'.
32. (a) Delete 'has'. 91. (c) Replace 'behave to be' with 'behave like being'.
33. (b) Replace 'listened' with 'heard'. 92. (a) Replace 'more' with 'much'.
34. (c) Replace 'went out' with 'had gone' 93. (b) Replace 'who' with 'whom'.
35. (a) Use of shall is not required. 94. (b) Replace 'very good' with 'very well'.
36. (c) Replace 'highly' with 'high' 95. (b) Replace 'them' with 'their'.
37. (b) ' to meet' should be replaced with ‘to meeting’ 96. (a) four-and-a-half years.
38. (a) 'I was' should be replaced with 'I were' 97. (a) A considerable encouragement for the scheme ……..
39. (d) No error is the right usage.
40. (b) Use 'so' after was not. 98. (d) No error.
41. (a) Replace 'did' with 'could' 99. (b) Replace 'between' with 'among'.
42. (d) No error 100. (d) No error.
43. (d) No error 101. (c) Delete 'to' after reach
44. (a) Add 'is' "if there is a guarantee" 102. (c) Replace 'Friday next' with 'next Friday'.
45. (b) Add 'in' "Once in a blue moon". 103. (b) Replace 'hand' with 'hands'.
46. (a) My daughters-in-law is the correct usage. 104. (d) No error
47. (b) It should be 'What is my name' 105. (c) Replace 'broken' with 'broken up'.
48. (c) ‘has’ should be replaced with ‘have’ 106. (b) Replace 'to be blamed' with 'to blame'.
49. (c) It should be 'principal advised the students' 107. (b) Replace 'among the major historical texts' with 'one of
50. (b) Replace 'are' with 'is'. the major historical texts.'
51. (b) The use of 'and' is not required. 108. (b) Replace 'and' with 'or'.
52. (a) Replace 'are' with 'is'. 109. (c) Instead of 'their' it should be 'his'.
53. (c) Add 'not' after could. 110. (c) Correct sentence: When the thief broke into their
54. (c) Where he had gone is the correct form. house, they raised a hue and cry and the thief was
55. (b) Replace 'on' with 'in'. caught immediately.
56. (b) Replace 'are' with 'is'. 111. (c) Correct sentence: I have tried to meet him several times,
57. (a) It should be "I have never asked". he has never been at home.
58. (c) Replace 'on' with 'in'. 112. (d) No error
59. (c) Replace 'besides' with 'beside'. 113. (c) Correct Sentence: He is proficient in Hindi and can
60. (b) Replace ‘whom’ with ‘who’ speak English, But he does not know how to read and
61. (b) differs from. write English.
62. (d) No error. 114. (b) Correct Sentence: Every woman in this world fervently
63. (c) saying this is the correct usage. hopes that her child will be a normal and healthy baby.
64. (c) With him is the correct usage. 115. (b) Correct sentence: Neither of them sent their papers in
65. (d) No error. time for the last seminar.
66. (c) Replace 'making' with 'make'. 116. (a) Correct sentence: There is not much traffic along the
67. (c) Replace 'than' with 'to'. street where I live.
68. (c) It should be ‘are’ instead of ‘were’ 117. (b) Correct Sentence: The front page story was about a
69. (d) No error school girl, who had hurt herself while saving a child
70. (c) Instead of I, it should be 'I am'. in an accident.
71. (d) No error 118. (b) Correct sentence: He took leave for four days.
72. (c) It should be on time. 119. (b) Correct sentence: The police arrived and discovered a
73. (a) It should be: writers who believe large quantity of hoarded sugars in his shop.
74. (a) It should be yourself 120. (c) Correct sentence: Raju doesn't come to our house
75. (c) In a meaningful way is the correct usage. because our dog barks at him and licks him, although
76. (b) It should be knew. I have often told him not to be afraid of it.
77. (b) are my favourite is the correct usage. 121. (a) Running across the playground, I dropped my pen in
78. (d) No error the mud …… (dangling participle)
79. (c) would have most certainly helped you. 122. (c) Correct sentence: The last thing that the fond mother
80. (b) has been going on is the correct usage. gave her only son was her blessing.
81. (b) on your success is the correct usage. 123. (a) Correct Sentence: For his innovative ideas and
82. (a) Delete 'of' practices in farming, he was given the Krishi Pandit
83. (b) remove 'that he' Award last year
84. (c) Instead of ‘with his contemporaries’, it should be ‘with 124. (b) Correct sentence: As a dramatist, Shaw is superior to
those of his contemporaries’. any other twentieth century writer.
85. (a) hereditary dancers h ad their own set of 125. (c) Molly speaks French well, doesn't she?
accompanist….. 126. (a) Correct sentence: Mutton is harder to digest than
86. (b) Replace 'he was not knowing' with 'he did not know vegetables.
me'
EBD_7367
58
B- Spotting Errors

127. (c) Correct sentence: I have found that he is neither willing 157. (b) The speaker from the Fifth Avenue, a rich banker's
nor capable. wife was simple and compassionate.
Explanation: Neither is always followed by nor instead 158. (a) There was no piece of paper in my pocket
of or. 159. (b) Neither the teacher nor the student is keen on joining
128. (a) Correct Sentence: Since time immemorial, sea shells the dance.
have been used by man in many ways. 160. (c) My neighbour Deepak is a person who will help
129. (b) Correct Sentence: He will not listen to what you say. anyone.
130. (c) Correct Sentence: I have done my best; the whole thing 161. (c) I will ask that man which of the roads is the one we
is now in the hands of God. want.
131. (a) Correct sentence: Oh for God's sake! leave me alone 162. (b) Now we have banks and people deposit their money
and go away from here. there and draw it out by cheques.
132. (b) Correct Sentence: This is a strange world where each 163. (a) Apart from government agencies, a number of private
one pursues his own golden bubble and laughs at organisations too have been making use of satellites.
others for doing the same. 164. (c) What sort of a drug this is, that no one seems to be
133. (b) Correct Sentence: Each of the boys was to blame for able to predict its long term effects with certainity?
the accident. 165. (c) You will lose your dog if you don't tie it up.
Each is often followed by a prepositional phrase ending 166. (c) ............adhered to.
in a plural word (Each of the cars), thus confusing the 167. (b) More than one workman was killed.
verb choice. Each is always singular and requires a 168. (b) The parties disagreed on the first two clauses in the
singular verb. agreement.
134. (b) Correct Sentence: The composition contained no fewer 169. (a) It should have been ' The scientist seemed to be excited
than twenty mistakes. over the result of his experiment'.
135. (c) Correct Sentence: He told us that he had not read the 170. (d) No error.
book. 171. (c) John could not come to school as he was ill with cold.
136. (c) Correct Sentence: The minister announced 172. (c) Though she has aptitude in Mathematics, I won't allow
compensation for the victims of the accident. her to take it up as a subject of study for the Master's
137. (d) No error degree because I know the labour involved will have a
138. (b) Corr ect sentence: There is no space in the telling upon her health.
compartment. 173. (c) I am not familiar with all the important places in this
139. (c) Correct sentence: The young man had no manners. town, although I have been living here for two years.
140. (b) Correct Sentence: There are many beautiful furniture 174. (a) If I were a millionaire, I would not have wasted my
in the room. time waiting for a bus.
141. (b) Correct Sentence: The policeman prevented us from 175. (c) Until you begin to make a better use of your time, I
entering the hall shall not stop finding fault with you.
142. (c) Correct Sentence: The world comprises of good and 176. (d) No error
bad people. 177. (b) They left their luggage at the railway station. ( Some
143. (b) Correct Sentence: Would you please order tea and nouns only have singular form and no plural form. Ex
biscuits for all of us? luggage.)
144. (a) Correct Sentence: I paid my bill for electricity only a 178. (b) You will get all the information if you read this booklet
week ago. carefully.
145. (d) No error 179. (c) She sang very well, didn't she?
146. (b) Correct Sentence: This is the old man who I said had 180. (a) ‘has been’ should replace "is"
helped me. 181. (b) There is no question of me failing in the examination.
147. (d) No error 182. (a) He goes everyday for a morning walk with his friends
148. (b) Correct Sentence: The truck driver accused the lady and neighbours.
of walking in the middle of the road. 183. (b) Her relatives could not explain to us why she didn't
149. (b) These are the ideas and ideals that have shaped our come for the wedding as she was expected.
economic thought in the past. 184. (c) barred should be replaced with was barred.
150. (a) India's problems are not similar to those of other 185. (b) If you repeat this mistake, I will inform your father and
countries in several ways. do not blame me then.
151. (c) He had lost a ring in the sand and I helped him search 186. (c) Lieutenant Anand was short and muscular with
for it but it was like looking for a needle in the haystack. shoulders that bulged impressively through his smart
152. (b) Replace ‘it’s’ with ‘its’ uniform.
153. (c) The sudden change of place affected her health. 187. (a) The sentence here proposes a situation that has not
154. (c) There are number of people of every class and actually happened. The error is in part (a) as 'should'
nationality who doubt the truth of his statement. must be replaced by would to give the sense of what
155. (a) I like this book because the writer explained the reasons that person would do in the same situation. Hence,
of his failure truly. option (a).
156. (d) No error 188. (b) When we use the word suffer, the only preposition to
be used along with the object is 'from' and not 'with' as
Spotting Errors B-59
used incorrectly in part (b). Hence, option B has an 204. (a) When we look at this sentence, the very first option
error and the correct form is 'suffer' from fever. has a clear error in it. When we already have used the
189. (a) When we talk about something beginning and most interesting regarding the first trip, we cannot use
continuing for a long time in the form of several events, even more in the second part of sentence as the most
it generally implies it is in plural form. Here, the signifies superlative degree of adjective. Thus, ‘most’
examination is not just one but several beginning from should be removed.
Monday. Hence, part (a) has an error with the correct 205. (c) In this sentence if we look at the segment (c), we find
form being 'the examinations begin'. an error. He even can't understand its working is a
190. (c) When we use the word sincere, the correct usage is wrong way of writing. We never use even before
sincere 'in' something and not 'to'. Part (c) has an error cannot. We always use even after cannot. Other
as the flow gets disrupted by following to after sincere. options have no mistakes.
191. (a) Replace 'has been' with 'have been'. 206. (c) In this sentence we see that we have formed the
192. (b) Add ‘is’ after ‘it’ sentence using neither. Neither is supplemented with
193. (c) The first part of sentence indicates the action nor in the second part. Now since we have already
happening in the past, that is, 3 days ago. However, said that neither he can speak English, it is not
usage of has in part (c) not only is unnecessary but is necessary to use he can again as it is understood that
also inconsistent with the tense of the event. So, 'that he is not capable of speaking both English and French.
he arrived' is sufficient to convey the meaning. 207. (a) The idiom 'avail oneself is followed by ‘of’.
194. (b) The usage of what in part (b) is highly inappropriate. 208. (c) Some of their “luggages” is incorrectly used.
The writer knows what he gave and hence, what “Luggages” word does not exist. The singular and
should be replaced by 'that'. 'that' works as a plural of luggage is luggage itself.
demonstrative pronoun here and is best suited. 209. (b) In this sentence option (b) is incorrect. "He not only
195. (a) 'have' should be replaced with 'have had'. was" should be restructured as 'he was not only.'
196. (b) Part (b) has an unnecessary usage of the word 'one' 210. (a) The phrase is framed incorrectly as in an uncertain
when it can be more appropriately and simply put situation or when using if, ‘will’ is not used. In case of
across with the usage of word 'an'. The sentence when uncertainty, generally, shall is used. Thus ‘will’ should
formed like 'was an unrelenting experiment' is be replaced with ‘shall’.
grammatically correct and makes sense. 211. (c) In this sentence, option c is incorrect. “more healthier”
197. (d) No error is wrong form of usage. Healthier is already a
198. (a) The first part of the sentence clearly is incorrect as it comparative degree of adjective and therefore, there
contains two negative words 'denied' as well as 'not', is no need to use “more” before healthier.
each nullifying the other. Thus, this changes the 212. (a) News is a singular noun so the verb used with it will
meaning of the sentence and conveys that the person have “s” beside it. Thus the correct form should be
has accepted stealing while the last part of sentence ‘news travels’ and not ‘news travel’.
contradicts it. Therefore, not is to be omitted in option 213. (b) In this sentence, option b has an error. When the
(a). principal clause is in the past, the subordinate clause
199. (a) The first part contains an error as the word media is in should also be in the past. Hence, 'would' should
plural form while the only thing we are talking about is replace 'will'.
films which is only a 'medium'. Thus, the word to be 214. (b) While using lot, we should also use “a” before it. Also,
used should have been 'medium'. lots can be used without using “a”. Here, in this
200. (c) Part (a) and (b) have no error but part (c) misses an sentence lot has been used without using “a” before
important word that would allow it to make sense. The it. Therefore, option (b) has error.
part should have been knows 'how' to speak French. 215. (c) Replace ‘has’ with ‘had’.
The abruptness caused by the flow of words initially 216. (c) Replace ‘great’ with ‘big’.
is removed by the word 'how'. 217. (d) No error
201. (c) Even though the sentence mentions two people 218. (a) There is an error in option (a) "he asked me as to why".
Mahesh and Ramesh, it does not mean the sentence "As to" is incorrect form of usage as it is generally not
has to be in plural form. When we use neither...nor, used in grammar. "He asked me why" would be a
each subject is taken only one at a time, with the verb correct form.
only in singular form. Thus, part (c) using 'have' should 219. (b) The (b) part should start with 'that.'
be replaced by the word 'has'. Hence, option (c) has 220. (a) Use ‘should have arrived’ instead of ‘should arrive’.
the error. 221. (a) In the sentence, ‘that’ is unnecessarily used.
202. (b) This sentence has error in section b as “lack of winter 222. (a) ‘Until you go to the station to receive him I can hardly
rains” is in singular form. Therefore here the lack of feel at ease’ is the correct sentence.
winter rains should be used with the word ‘has’. Other 223. (c) ‘I did not know where they were going nor could I
segments do not have any error in the sentence. understand why they had left so soon’ is the correct
203. (c) In this question, part c “convinced with his guilt” has sentence.
an error in it. Generally we never use “with” with 224. (c) ‘The distinguished visitor said that see had great
convinced. “Convinced about” or ‘convinced of’ are pleasure to be with us for some time and that the
generally used prepositions with convinced. pleasure was all the greater because his visit had
EBD_7367
60
B- Spotting Errors

afforded him an opportunity to study the working of 253. (d) No error


an institution of such eminence as ours’ is the correct 254. (c) They made him treasurer because they considered him
sentence. honest and efficient is the correct sentence.
225. (c) ‘Please convey my best wishes to your parents’ is the 255. (b) Having finished the paper he had come out of the hall
correct sentence. almost an hour before the bell rang.
226. (b) ‘The call of the seas has always found an echo in me’ is 256. (d) No error
the correct sentence. 257. (d) No error
227. (a) Hardly had I left home for Bombay when my son who is 258. (b) The work involved is impossible to cope with is the
settled in Calcutta arrived without any prior information correct sentence.
is the correct sentence. 259. (b) There is no seat in this compartment is the correct
228. (b) Now it can be easily said that the population of this sentence.
city is greater than that of any other city in India is the 260. (c) Use ‘other’ before ‘poet’.
correct sentence. 261. (c) ‘Going by myself’ is the correct usage.
229. (b) It is difficult to explain why Rajgopalachari resigned 262. (c) ‘The minister announced compensation for the victims
from the Congress in 1940 is the correct sentence. of the accident’ is the correct sentence.
230. (b) The boss reminded them of the old saying that honesty 263. (b) ‘The Australian team lost the match yesterday’ is the
is the best policy, and told them that they had better be correct sentence.
honest in their work is the correct sentence. 264. (c) ‘he had not read the book’ is the correct usage.
231. (a) Since the sentence talks about a single story, 'is' will be 265. (b) The composition contained not fewer than twenty
used instead of 'are'. mistakes is the correct sentence.
232. (c) The teenager reassured his father at the station ‘Don't 266. (b) The reason for his failure is that he did not work hard is
worry dad, I will pull through very nicely at the hostel the correct sentence.
is the correct sentence. 267. (b) Food and water are necessary for life.
233. (d) No error 268. (b) India is larger than any other democracy in the world is
234. (b) Most of the developing countries find it difficult to the correct sentence.
cope with the problems created by the sudden impact 269. (d) No error
of technological progress is the correct sentence. 270. (c) ‘I have lived in Delhi since 1965’ is the correct sentence.
235. (c) Coward means a person who lacks courage in facing 271. (d) No error
danger, difficulty, opposition etc. We only write coward 272. (a) ‘The books which you read are not worth reading’ is
and do not add person/man/woman to it. the correct sentence.
236. (c) We swam up to the drowning man, caught hold of his 273. (b) ‘Tagore was one of the greatest poets that ever lived’
clothes before he could go down again and pulled him is the correct sentence.
out, safely to the shore is the correct sentence. 274. (b) ‘You may please apply for an advance on salary to
237. (b) ‘not hardly’ is wrong and instead only ‘hardly’ should cover the costs of transport’.
be used. 275. (b) ‘The taxi that will take the family to Haridwar had to be
238. (a) If I had known why he was absent, I would have readied at six the next morning’ is the correct sentence.
informed you is the correct sentence. 276. (b) ‘Employees are expected to adhere to the rules laid
239. (c) He goes to office on foot. "By foot' is not idiomatic and down by the management’ is the correct sentence.
should not be used. 277. (b) ‘The owner of the horse greedily asked too high a price’
240. (a) Hundred rupees notes is incorrect and instead, is the correct sentence.
hundred- rupee notes should be used. 278. (d) No error
241. (d) No error 279. (a) ‘Some men are born great’ is the correct sentence.
242. (c) She inquired whether anyone had seen her baby is the 280. (b) ‘We must sympathize with others in their troubles’ is
correct sentence. the correct sentence..
243. (b) When I went outdoors, I found frost everywhere is the 281. (b) ‘My detailed statement is respectfully submitted’ is the
correct sentence. correct sentence..
244. (c) These are his concluding remarks is the correct 282. (a) ‘I have been waiting for my friend since morning’ is the
sentence. correct sentence..
245. (a) The shopkeeper offered to either exchange the goods 283. (a) ‘He has been representing my constituency for the past
or refund the money is the correct sentence. five years’ is the correct sentence.
246. (d) No error 284. (c) ‘If he hears of your conduct, he will be unhappy’ is the
247. (c) We should keep such people at arm's length is the correct sentence.
correct sentence. 285. (a) ‘No sooner had he appeared on the stage, than the
248. (c) He did not know as much as he claimed to know is the people began to cheer loudly’ is the correct sentence.
correct sentence. 286. (b) Replace ‘with’ with ‘in’.
249. (d) No error. 287. (a) Replace ‘equipments’ with ‘equipment’.
250. (b) My friend goes to a movie every week is the correct 288. (b) Replace ‘has’ with ‘have’
sentence. 289. (b) Replace ‘sceneries’ with ‘scenery’.
251. (b) They sit by the window and watch the traffic is the 290. (d) No error.
correct sentence. 291. (d) No error.
252. (d) I started early for the station lest I should miss the
292. (c) We had driven' will be the correct tone.
train.
Spotting Errors B-61

293. (c) Replace 'and' with (c) 327. (a) 'a' should be replaced with 'an' because the word
294. (d) No error 'honourable' begins with a vowel sound.
295. (b) Replace ‘since’ with ‘for’. 328. (b) 'Could' expresses only ability to do an act, but not the
296. (a) Replace ‘beside’ with ‘besides’. performance of an act. Therefore, 'was not able' should
297. (c) Replace ‘best’ with ‘better’. be used instead of the underlined part for ability as
298. (b) Replace ‘are’ with ‘is’. well as action in the past.
299. (c) Replace ‘whomever’ with ‘whoever’. 329. (a) 'were' should be replaced with 'was' to make the
300. (b) Replace ‘most’ with ‘more’. sentence grammatically correct.
301. (a) Replace 'hardly' with 'hard'. 330. (d) The sentence is grammatically correct, hence, it does
302. (b) Replace 'whom' with 'who'. not require any correction.
303. (a) Replace 'after' with 'for'. 331. (d) The sentence is grammatically correct, hence, it does
304. (b) Replace 'will' with 'would'. not require any correction.
305. (a) Replace 'their all belongings' with 'All their belongings'. 332. (d) The sentence is grammatically correct, hence, it does
306. (a) Delete 'the'. not require any correction.
307. (c) Replace 'to his house' with 'to the decorations in his 333. (d) The sentence is grammatically correct, hence, it does
house'. not require any correction.
308. (a) Delete 'of'. 334. (c) In the sentence, 'peace' has been compared to 'war'.
309. (c) Replace 'will' with 'would'. Thus, 'as' should be used before 'terrible' to make the
310. (c) Delete 'shall'. sentence contextually correct.
311. (b) Replace 'support' with 'supports'. 335. (b) The verb 'attack' is not followed by any preposition
and should be used directly. Thus, 'on' should be
312. (a) Replace 'He is school teacher' with 'He is a school
teacher'. omitted to make the sentence correct.
313. (b) Replace 'had told him to smoke' with 'had told him that 336. (b) In underlined part ‘b’ there is incorrect use of ‘for her
smoking'. return’. The correct usage is ‘for her to return’ So, the
314. (a) Replace 'which' with 'that'. correct sentence is - I waited for her to return with
315. (a) Replace 'I am entirely agreeing with you' with 'I entirely growing unease.
agree with you'. 337. (c) There is in incorrect use of preposition in the
316. (b) The word 'senior' usually takes the preposition 'to'. underlined part (c). In place of ‘angry at’, ‘angry with’
Hence, 'then' should be replaced with 'to' to make the should be used and the correct sentence should be- I
sentence correct. do not understand why in spite of my best efforts to
317. (a) 'When' should be replaced with 'after' because the please him my boss is so angry with me.
second part of the sentence took place after the first 338. (a) There is ‘tense’ error in the underlined part (a) of the
task was over. sentence. As other parts are in past tense so, part (a)
318. (d) The sentence is grammatically correct, hence, it does should also be in past tense. ‘Being a rainy day’
not require any correction. should be replaced with ‘As it was a rainy day’.
319. (b) 'cured from' should be replaced with 'cured of' to make 339. (c) There is unnecessary use of pronoun ‘he’ in the
the sentence correct as 'cured' is followed by the underlined part (c). It should be omitted to make the
preposition of. sentence correct.
320. (b) The verb 'spend' should be replaced with' spending' 340. (b) There is subject Verb disagreement in the underlined
to make the sentence grammatically correct. part (b) of the sentence. ‘He was left’ is incorrect usage,
321. (b) The word 'poetry' should be replaced with 'poem' to it should be replaced with ‘he had left’ to make the
make the sentence correct. Poetry is the process of sentence correct.
creating a literary piece using metaphor, symbols and 341. (c) There is incorrect use of proverb in underlined part
ambiguity, while a poem is the end result of this (c) of the sentence. The correct proverb is- If a thing
process. is worth doing at all, it is worth doing well. So, ‘done
322. (c) 'Out' is inappropriately used in the sentence. Thus, it well’ in part (c) should be replaced with ‘doing well’ to
should be omitted to make the sentence correct. make the sentence correct.
323. (c) The words joined to a singular subject by 'with' are 342. (b) Returned and back are not used together because
parenthetical. Therefore, the verb 'were' should be ‘return’ itself means ‘come or go back’, So, ‘back’
replaced with 'was' to make the sentence grammatically should be removed from part (b) to make the sentence
correct. grammatically correct.
324. (d) The sentence is grammatically correct, hence, it does 343. (c) There is structure error in the underlined part (c). In
not require any correction. part (c), structure should be same as in part (b). So,
325. (d) The sentence is grammatically correct, hence, it does ‘and no trouble is there’ should be replaced with ‘and
not require any correction. there is no trouble’ to make the sentence correct.
326. (a) The word 'assistant' should be used in the plural form 344. (a) There is tense error in the underlined part (a). As the
because the phrase 'one of the' agrees to a plural noun. sentence is in part tense, part (a) should also be in
EBD_7367
62
B- Spotting Errors

past tense. So, ‘considering about these facts’ should something particular but here it is used in general
be replaced with ‘Having considered’ to make the sense, about of any writer thus ‘A’.
sentence grammatically correct. 366. (d) No error.
345. (c) There is incorrect use of conjunction ‘unless’ in the 367. (b) Replace ‘you are’ of part (b) with ‘have you been’.
underlined part (c). It should be replaced with ‘if’ to 368. (c) Replace ‘in’ with ‘of’ in part (c).
make the sentence correct and meaningful. 369. (c) Replace ‘isn’t it?’ with ‘aren’t you?’ in part (c).
346. (a) There is number error in the underlined part (a) of the 370. (b) Replace ‘in’ with ‘on’ in part (b)
sentence. ‘One of the most interesting feature’ should 371. (c) Replace ‘with’ with ‘in’.
be replaced with ‘one of the most interesting features’ 372. (b) Add ‘to’ before the song to make the sentence
to make the sentence correct. grammatically correct.
347. (d) There is no error in the sentence. 373. (c) Replace ‘want’ with ‘wants’ as the subject is singular
348. (c) There is incorrect use of preposition ‘than’ in the noun.
underlined part (c). ‘Than that one’ should be replaced 374. (b) Replace ‘estimate’ with ‘estimates’ as the subject is
with ‘to that one’ to make the sentence correct. singular noun.
349. (d) There is no error in the sentence. 375. (c) Add ‘a’ before little in part (c).
350. (d) There is no error in the sentence. 376. (b) Add ‘but’ before ‘in’ and ‘no’ before ‘case’ to make
351. (b) There is structure error in the underlined part (b) of the sentence grammatically/contextually correct. So,
the sentence. The sentence is a declarative sentence, (b) should be ‘but in no case impossible’.
but in part (b) helping verb ‘was’ is placed before the 377. (d) No error.
subject ‘he’ which is an error. It should be replaced 378. (a) Replace ‘the’ with ‘her’, in part (a)
with ‘where he was going’. 379. (d) No error
352. (d) There is no error in the sentence. 380. (b) Replace ‘besides’ with ‘beside’.
353. (c) There is incorrect use of adverb ‘hardly’ in the 381. (b) Replace ‘much’ with ‘the most’.
underlined part (c). It should be replaced with ‘hard’ 382. (a) Replace ‘have’ with ‘had’.
to make the sentence meaningful and grammatically 383. (b) Replace ‘to take’ with ‘taking’.
correct. 384. (a) Replace ‘was’ with ‘were’.
354. (d) There is no error is the sentence. 385. (c) Replace ‘their’ with our, as the subject is ‘us’.
355. (a) There is incorrect structure in the underlined part (a). 386. (a) Replace ‘furnitures’ with ‘furniture’ as the plural form
‘Whom’ should be replaced with ‘who’ to make the of furniture is also the same- ‘furniture’.
sentence correct. 387. (a) Replace 'in which you can' with 'to'.
356. (b) There is subject verb(–) agreement error in the 388. (d) No error
underlined part (b) of the sentence. As ‘none’ is
389. (b) Omit 'throughout'.
singular, it should be followed with ‘is’ not ‘are’ to
390. (d) No error
make the sentence correct.
357. (d) There is no error in the sentence. 391. (b) Replace 'were' with 'was'
358. (b) There is incorrect use of verb form in the underlined 392. (b) Replace the preposition 'for' with 'to'
part (b). ‘Sweeped’ should be replaced with ‘swept’ 393. (b) Replace 'for have to' with 'about having to'.
to make the sentence correct. 394. (d) No error
359. (b) There is subject verb agreement error in part (b). Her 395. (c) Replace 'to' with 'into'.
parents (plural noun) should be followed by ‘have’ 396. (c) Replace 'has' with 'had'.
not ‘has’ to make the sentence correct. 397. (a) Replace 'have' with 'has'.
360. (b) There is subject-verb agreement error in the underlined 398. (b) Replace 'leave' with 'let' to make the sentence
part (b) of the sentence ‘have’ should be replaced contextually correct.
with ‘had’ to make the sentence grammatically correct. 399. (b) Add 'that' before 'it'.
361. (c) The subject of the sentence is ‘every student’ which
400. (b) Replace 'form' with 'formation'.
is singular, so, ‘their’ in part(c) should be replaced
401. (b) Replace 'twice more than' with 'twice as much as'.
with ‘his’ to make the sentence grammatically correct.
402. (a) Omit 'the’.
362. (c) Replace the adverb ‘mersely’ with adjective ‘mere’. 403. (b) Replace 'in' with 'from'.
363. (b) Omit ‘has’ from part (b) 404. (b) Replace 'in' with 'under'.
364. (b) Replace ‘heavy’ with ‘heavier’ in part (b).
405. (c) Replace 'are' with 'is'
365. (a) Replace ‘The’ with ‘A’ in part (a). ‘The’ is a definite
406. (d) No error
article and is used when we talk of someone or
C HA P T E R
ORDERING OF WORDS
29 IN A SENTENCE
DIRECTIONS (Qs. 1-17) : In the following questions, some parts R : particulary in the last two decades or so the mouse
of the sentence have been jumbled up. You are required to S : engineering era of modern biology
rearrange these parts which are labelled P, Q, R and S to produce Which one of the following is the correct sequence ?
the correct sentence. Choose the proper sequence and mark in (a) QRSP (b) SPQR
your Answer Sheet accordingly. Example ‘Z’ has been solved for (c) QPSR (d) SRQP
you. [2008-I] 6. It was not
P : conditions with fuel prices climbing to dizzy heights
Z. It is well-known that
P : the effect Q : is very bad Q : but resentment arising out of harsh economic
R : on children S : of cinema R : which led to the recent protests in Myanmar
Which one of the following is the correct sequence ? S : an inspiration for democracy
(a) PSRQ (b) SPQR Which one of the following is the correct sequence ?
(c) SRPQ (d) QSRP (a) Q S R P (b) S Q P R
Explanation : (c) Q S P R (d) S Q R P
The proper way of writing the sentence is “It is well-known 7. You have been writing to me often about
that the effect of cinema on children is very bad.” This is P : getting a first prize in sports etc.
indicated by the sequence P S R Q and so (a) is the correct Q : but my daughter
answer. R : distinguishing in a fancy dress show, or
1. Climate S : such achievements make me apprehensive of your
P : cultures and political structures educational progress
Q : in that it affects the entire Which one of the following is the correct sequence ?
R : change is a truly unifying phenomenon (a) QPRS (b) RSQP
S : world irrespective of national borders (c) QSRP (d) RPQS
Which one of the following is the correct sequence ? 8. The Secretary announced that
(a) RSQP (b) P Q S R
P : to find an answer to these questions
(c) RQSP (d) PSQR
2. This may Q : in an attempt
P : after decades of relative neglect and even reversal R : a national workshop on technical training had been
Q : when land reforms were finally put back on organised
R : go down as the year S : by the Confederation of Engineering Industry
S : the nation’s policy agenda Which one of the following is the correct sequence ?
Which one of the following is the correct sequence ? (a) QPSR (b) PQRS
(a) SQRP (b) RPSQ (c) QPRS (d) PQSR
(c) SPRQ (d) RQSP 9. Technology transfer
3. There is, however, P : from one country to another
P : to clear the backlog in respect of education and poverty Q : either through a government policy
alleviation programmes R : or via private channels of communications
Q : a strong perception that reservation S : implies the transfer of technical knowledge
R : unless it is followed by sincere and intensive efforts Which one of the following is the correct sequence ?
S : alone cannot transform the lives of oppressed people (a) S P R Q (b) P S Q R
Which one of the following is the correct sequence ? (c) S P Q R (d) P S R Q
(a) QSRP (b) SQRP 10. Physical training
(c) SQPR (d) QSPR P : the character of
4. More than ever before, Q : who are going to be responsible citizens
P : good governance and sound public policies for the
R : can shape well
benefit of its people
Q : to provide leadership and to deliver S : young boys and girls
R : the continued prosperity of Asia depends Which one of the following is the correct sequence ?
S : on the ability of its politicians and policy makers (a) R P S Q (b) Q S P R
Which one of the following is the correct sequence? (c) R S P Q (d) Q P S R
(a) RPQS (b) QSRP 11. His favourite subject
(c) RSQP (d) QPRS P : the secrets of life and of nature
5. In the post-genetic Q : because Science reveals
P : of choice for the entire spectrum of research R : happens to be Science
Q : has acquired the status of the experimental animal or S : which men did not know in the past
the animal model system
EBD_7367
64
B- Ordering of Words in a Sentence

Which one of the following is the correct sequence ? Which one of the following is the correct sequence ?
(a) P S R Q (b) R Q P S (a) P S R Q (b) S P Q R
(c) P Q R S (d) R S P Q (c) S R P Q (d) Q S R P
12. From my plane flying over Namibia Explanation :
P : stretching 320 kilometres away The proper way of writing the sentence is “It is well-known
Q : a shore of pink and silver sand that the effect of cinema on children is very bad”. This is
R : I looked down at a haunting sight indicated by the sequence P S R Q so (a) is the correct answer.
S : edged by a line of smoking surf 18. Please
Which one of the following is the correct sequence ? equally share the prize money that all are satisfied
(P) (Q) (R)
(a) Q R P S (b) R Q S P
among your friends to make sure
(c) Q R S P (d) R Q Q S (S)
13. I was Which one of the following is the correct sequence ?
P : to see a mongoose (a) Q R S P (b) P Q S R
Q : just in time (c) Q P R S (d) Q P S R
R : with an egg in its mouth 19. You can learn
S : scurrying across the grass and can be quite difficult as well as the difficult ones
Which one of the following is the correct sequence ? (P) (Q)
(a) S R Q P (b) Q P S R the easy things although the easy ones are not so easy
(c) S P Q R (d) Q R S P (R) (S)
14. How strange that a refugee Which one of the following is the correct sequence ?
P : should fall in love (a) S P R Q (b) P Q R S
Q : when he had got to America (c) Q S P R (d) R Q S P
R : with a girl less than half his age 20. We met
S : who had by the skin of his teeth escaped death in this evening on our way home Mr and Mrs Gupta
Germany (P) (Q) (R)
Which one of the following is the correct sequence ? and asked them if they would be free
(a) R Q P S (b) S P R Q (S)
Which one of the following is the correct sequence ?
(c) R P Q S (d) S Q P R
(a) Q R P S (b) P Q R S
15. I think it should be necessary for eggs (c) R Q S P (d) R Q P S
P : by the poulterer 21. after his arrival immediately
Q : as soon as they are laid (P) (Q)
R : to be stamped with the date he began to quarrel with his wife
S : by the hen (R) (S)
Which one of the following is the correct sequence ? Which one of the following is the correct sequence ?
(a) Q S P R (b) S Q P R (a) Q R S P (b) P Q R S
(c) S Q R P (d) Q S R P (c) Q P R S (d) R Q S P
16. Reuter reports that soldiers 22. in the middle of he stopped his speech to take a glass of water
P : fought each other in the streets of the capital today (P) (Q) (R) (S)
Q : crack down on demonstrators Which one of the following is the correct sequence ?
R : of armies loyal to rival political factions (a) Q R S P (b) Q R P S
S : following the week-end military (c) Q P R S (d) P Q R S
Which one of the following is the correct sequence ? 23. the man made a complaint at the police station
(a) R P S Q (b) Q S P R (P) (Q) (R)
(c) R S P Q (d) Q P S R whose cycle was stolen
(S)
17. The old lady
Which one of the following is the correct sequence ?
P : wanted to sell her precious table (a) P Q R S (b) P R S Q
Q : having the capacity to pay a huge amount as its price (c) P S Q R (d) R P Q S
R : to someone 24. the clerk was not present in the office
S : with gold-coated legs and glossy surface (P) (Q) (R)
Which one of the following is the correct sequence ? whom I wanted to meet
(a) QSRP (b) PRSQ (S)
(c) QRSP (d) PSRQ Which one of the following is the correct sequence ?
DIRECTIONS (Qs. 18-37) : In the following questions, some (a) P S Q R (b) P Q R S
(c) P R S Q (d) R P Q S
parts of the sentence have been jumbled up. You are required to
25. Nobody in the house
re-arrange these parts which are labelled P, Q R and S to produce was kept where the key
the correct sentence. Choose the proper sequence and mark in (P) (Q)
your Answer Sheet accordingly. [2008-II] seemed to know to the main door
Example ‘Z’ has been solved for you. (R) (S)
Z. It is well-known that Which one of the following is the correct sequence ?
the effect is very bad on children of cinema (a) R Q P S (b) Q P R S
(P) (Q) (R) (S) (c) S Q P R (d) R Q S P
Ordering of Words in a Sentence 65
B-

26. A tortoise Which one of the following is the correct sequence ?


fly in the air two geese everyday watched (a) P Q R S (b) S Q R P
(P) (Q) (R) (S) (c) S R Q P (d) P R Q S
Which one of the following is the correct sequence ? 34. Some men
(a) S Q P R (b) R S Q P were rarely successful but they tried bribing their guards
(c) P S Q R (d) R P S Q (P) (Q) (R) (S)
27. If in their daily lives Which one of the following is the correct sequence ?
the people of our country and never deceive one another
(a) P Q R S (b) R Q P S
(P) (Q)
(c) R S Q P (d) P Q S R
always speak the truth then out prestige will be great
35. She introduced me
(R) (S)
at the party I had met to a man the previous night
Which one of the following is the correct sequence ?
(a) P Q R S (b) R Q P S (P) (Q) (R) (S)
(c) S P Q R (d) P R Q S Which one of the following is the correct sequence ?
28. The clothes (a) R P Q S (b) R Q P S
were sent by mistake to my father (c) S P R Q (d) P Q R S
(P) (Q) 36. Of all things that distinguish man
who was very much surprised that I ordered is his brain the most important
(R) (S) (P) (Q)
Which one of the following is the correct sequence ? of the animal kingdom from-the rest
(a) S Q P R (b) P Q S R (R) (S)
(c) P Q R S (d) S P Q R Which one of the following is the correct sequence ?
29. The President (a) S R Q P (b) Q P S R
a brief vacation at Shimla is returning from (c) S R P Q (d) R S Q P
(P) (Q) 37. they forget between defeat and victory,
in order to attend a press conference (P) (Q)
(R) that the conditions of welfare have so altered
with his wife and children (R)
(S) that there is not much difference today
Which one of the following is the correct sequence ? (S)
(a) Q S R P (b) Q P R S Which one of the following is the correct sequence ?
(c) S Q P R (d) R S Q P (a) S R P Q (b) P Q S R
30. This letter (c) R S Q P (d) P R S Q
is addressed to Mr. Shah which is written in Gujarati
(P) (Q) DIRECTIONS (Qs. 38-57): In the following questions, some
who is my father’s best friend parts of the sentence have been jumbled up. You are required to
(R) (S) re-arrange these parts which are labelled P, Q, R and S to
Which one of the following is the correct sequence ? produce the correct sentence. Choose the proper sequence and
(a) P Q R S (b) Q P R S mark in your Answer Sheet accordingly.
(c) Q R S P (d) R S P Q Example ‘Z’ has been solved for you. [2009-I]
31. Whether the plan Z. It is well-known that
depends on how it suggested will succeed or fail the effect is very bad on children of cinema
(P) (Q) (P) (Q) (R) (S)
interested in its progress will be received by those The correct sequence should be
(R) (S) (a) P S R Q (b) S P Q R
Which one of the following is the correct sequence ? (c) S R P Q (d) Q S R P
(a) S R P Q (b) Q P S R Explanation :
(c) P Q R S (d) R S Q P The proper way of writing the sentence is “It is well-known
32. the nun who was the Principal called an assembly that the effect of cinema on children is very bad”. This is
(P) (Q) indicated by the sequence P S R Q and so (a) is the correct
of my grand - daughter’s school answer.
(R) 38. He found the house
to announce the result of a fund - raising drive and knocked at without any difficulty the door twice
(S) (P) (Q) (R) (S)
Which one of the following is the correct sequence ? The correct sequence should be
(a) P Q R S (b) P R Q S (a) P Q R S (b) P R S Q
(c) P Q S R (d) P S R Q (c) Q P S R (d) Q P R S
33. Self-taught experts 39. A good garden,
expected to go through which the escapers should display round about a huge building or palace
(P) (Q) (R) (P) (Q)
prepared maps of the country laid out on a magnificent scale fresh beauties every month
(S) (R) (S)
EBD_7367
66
B- Ordering of Words in a Sentence

The correct sequence should be 47. If everything


(a) P Q R S (b) P S Q R will be a great success the closing function we are sure
(c) R Q P S (d) R P Q S (P) (Q) (R)
40. When a spider has a meal. goes on well
its next meal, until it has (S)
(P) (Q) The correct sequence should be
to last for many months it eats enough (a) P Q R S (b) S R Q P
(R) (S) (c) P R Q S (d) S Q P R
The correct sequence should be 48. With an idea to reach the deprived child
(a) Q P S R (b) S R Q P (P)
(c) S Q P R (d) Q P R S by the United Nations the year 1979 has been declared as
41. Mother tongue is (Q) (R)
for the development as natural of man’s mind the International Year of the Child
(P) (Q) (R) (S)
as mother’s milk The correct sequence should be
(S) (a) R S P Q (b) R S Q P
The correct sequence should be (c) P R S Q (d) P Q R S
(a) S R P Q (b) P Q S R 49. The gardener
(c) S R Q P (d) Q S P R with a little brown moustache a short fellow
42. The year that has just ended has proved to be disastrous (P) (Q)
(P) (Q) (R) and sharp little brown eyes
for my uncle’s family (R)
(S) tiptoed into the room
The correct sequence should be (S)
(a) P R S Q (b) P Q R S The correct sequence should be
(c) P S R Q (d) S P R Q (a) P Q R S (b) Q P R S
43. When I look back on my life (c) Q P S R (d) Q S R P
I find it hard to believe which has been eventful 50. If you have something interesting
(P) (Q) express it clearly to write about simply and
despite what cynics say that it is an illusion (P) (Q) (R)
(R) (S) with the human touch
The correct sequence should be (S)
(a) P S Q R (b) P Q S R The correct sequence should be
(c) Q R S P (d) Q P S R (a) P Q S R (b) S P R Q
44. Guards often use (c) R S Q P (d) Q P R S
to search for cavities metal probing rods which they push 51. One cloudless morning
(P) (Q) (R) towards the valley the pilots flew off together
through the ground (P) (Q)
(S) of the farmer’s house in close formation
The correct sequence should be (R) (S)
(a) S Q R P (b) Q R S P The correct sequence should be
(c) S R Q P (d) Q S P R (a) Q S P R (b) S Q P R
45. I realized (c) P Q S R (d) R Q P S
more than ever and how we lived and worked 52. When he had ridden
(P) (Q) to the end of his mad little journey
how cut off we were from our people (P)
(R) in front of his rocking horse he climbed down and stood
and agitated in a little world apart from them (Q) (R)
(S) staring fixedly into its lowered face
The correct sequence should be (S)
(a) Q S P R (b) S Q R P The correct sequence should be
(c) P R Q S (d) R S Q P (a) Q R P S (b) S Q P R
46. Medical practice (c) P R Q S (d) P S R Q
in recent years has changed so radically 53. The meeting
if Mr Rai would have been over on time
(P) (Q)
you only get specialists in a variety of fields (P) (Q)
with his long financial report
(R)
(R)
that you can no longer find a good general practitioner
did not hold us up
(S)
(S)
The correct sequence should be
The correct sequence should be
(a) P S Q R (b) Q P S R
(a) Q R P S (b) S P R Q
(c) S P R Q (d) P Q R S
(c) Q P S R (d) P R S Q
Ordering of Words in a Sentence B-67
54. The long gruelling hours The correct sequence should be:
had finally paid off on drawing after drawing, (a) R – Q – P – S (b) P – R – Q – S
(P) (Q) (c) R – P – S – Q (d) R – P – Q – S
working tirelessly painting after painting 60. I came finally and saw an article to the editorial page
(R) (S) (P) (Q) (R)
The correct sequence should be dealing with Pandit Nehru
(a) P R Q S (b) R Q S P (S)
(c) Q S R P (d) R P Q S The correct sequence should be:
55. For us (a) Q – R – P – S (b) P – R – Q – S
before this Monday to complete this work (c) S – Q – R – P (d) P – R – S – Q
(P) (Q) 61. Although many institutions these can have have tried to
and it should not be delayed (P) (Q)
(R) organise remedial programmes for the weaker students,
is very important (R)
(S) only a peripheral impact
The correct sequence should be (S)
(a) P Q R S (b) Q P S R The correct sequence should be:
(c) R S P Q (d) Q S P R (a) P – Q – R – S (b) Q – R – P – S
56. When the (c) S – R – P – Q (d) P – R – Q – S
the party realized that elections were over in the assembly 62. Our society to-day that people have lost their frankness
(P) (Q) (R) (P)
is so fragmented and the art of conversation seems to be
it had lost its majority
(Q) (R)
(S) disappearing fast
The correct sequence should be (S)
(a) P S Q R (b) Q P S R The correct sequence should be:
(c) Q R P S (d) P Q R S (a) Q – P – R – S (b) P – Q – S – R
57. If you must arm yourself so before starting a war (c) Q – S – P – R (d) R – S – P – Q
(P) (Q) 63. Even today many superstitious exist by a great many
that the enemy think twice you want peace (P) (Q)
(R) (S) people and are believed in among the most civilized
The correct sequence should be (R) (S)
(a) S P R Q (b) P R Q S nations.
(c) R Q S P (d) P R S Q The correct sequence should be:
(a) S – P – R – Q (b) P – S – Q – R
DIRECTIONS (Qs. 58-75) : In the following questions, some
(c) S – Q – P – R (d) Q – P – R – S
parts of the sentence have been jumbled up. You are required to 64. The eyes of seeing persons to the routine of their
re-arrange these parts which are labelled P, Q, R and S to (P)
produce the correct sentence. Choose the proper sequence and surroundings and they actually see soon become
mark in your Answer Sheet accordingly. [2009-II] (Q) (R)
Example ‘Z’ has been solved for you. accustomed only the startling and spectacular
(S)
Z. It is well-known that The correct sequence should be:
the effect is very bad on children of cinema
(P) (Q) (R) (S) (a) S – P – R – Q (b) P – S – Q – R
The correct sequence should be: (c) R – P – Q – S (d) Q – P – R – S
(a) P – S – R – Q (b) S – P – Q – R 65. Though he was a man when there was no choice who held
(c) S – R – P – Q (d) Q – S – R – P (P) (Q)
Explanation : all life scared, he did not hesitate to kill and loved all forms
The proper way of writing the sentence is “It is well-known (R) (S)
that the effect of cinema on children is very bad”. This is of life
indicated by the sequence P – S – R – Q and so (a) is the The correct sequence should be:
correct answer. (a) R – P – Q – S (b) P – S – Q – R
58. no conclusive evidence the enquiry committee found to (c) Q – S – R – P (d) S – Q – P – R
(P) (Q) 66. For a moment that I am terribly old since I was a child
the airplane of a thermal shock. (P) (Q)
(R) (S) and that it is a very long time ago I forget.
The correct sequence should be: (R) (S)
(a) R – Q – P – S (b) Q – P – S – R The correct sequence should be:
(c) R – P – S – Q (d) R – P – Q – S (a) Q – P – R – S (b) P – S – R – Q
(c) R – P – Q – S (d) S – P – R – Q
59. For thirty years with slave-like docility of her tribe 67. from behind a curtain he held it in such a manner bringing
(P) (Q) (P) (Q) (R)
his wife had submitted to his persecution that is the badge the light that it fell slandwise on her face.
(R) (S) (S)
EBD_7367
68
B- Ordering of Words in a Sentence
The correct sequence should be: The correct sequence should be:
(a) S – Q – R – P (b) R – P – Q – S (a) R – Q – P – S (b) P – S – Q – R
(c) S – P – Q – R (d) S – R – P – Q (c) P – R – Q – S (d) P – S – R – Q
68. The man had been sportsman and received many prizes
(P) (Q) DIRECTIONS (Qs. 76-89) : In the following questions, some
throughout his educational career and medals in parts of the sentence have been jumbled up. You are required to
(R) (S) re-arrange these parts which are labelled P, Q, R and S to produce
competitions. the correct sentence. Choose the proper sequence and mark in
The correct sequence should be: your Answer Sheet accordingly. [2010-I]
(a) P – Q – R – S (b) Q – P – R – S
(c) S – P – Q – R (d) P – R – Q – S Example ‘Z’ has been solved for you.
69. He was passing when he heard by a hut the cries of a Z. It is well-known that
(P) (Q) (R) the effect is very bad on children of cinema
child and went in. (P) (Q) (R) (S)
(S) The correct sequence should be :
The correct sequence should be: (a) P S R Q (b) S P Q R
(a) P – Q – R – S (b) Q – P – R – S (c) S R P Q (d) Q S R P
(c) P – R – Q – S (d) S – P – Q – R Explanation :
70. Only if we worked hard the teacher warned us that we The proper way of writing the sentence is “It is well-known
(P) (Q) that the effect of cinema on children is very bad”. This is
would pass the test for at least eight hours a day. indicated by the sequence P S R Q and so (a) is the correct
(R) (S) answer.
The correct sequence should be: 76. The teacher had to be specially careful
(a) Q – R – P – S (b) Q – P – R – S (P)
(c) P – S – R – Q (d) S – Q – P – R because he enjoyed the confidence
71. There is among the great powers no agreement to ban (Q)
about how he faced up to this problem
(P) (Q)
(R)
nuclear weapons on a treaty.
of all the boys
(R) (S)
(S)
The correct sequence should be:
The correct sequence should be :
(a) R – S – Q – P (b) S – R – P – Q
(a) P R Q S (b) Q P S R
(c) Q – P – S – R (d) R – P – Q – S (c) S P R Q (d) P S R Q
72. According to one theory have descended from all land 77. Movies made in
(P) (Q) all around the globe Hollywood in America by people
animals organisms of the sea including man (P) (Q) (R)
(R) (S) are seen at the same time
The correct sequence should be: (S)
(a) S – P – Q – R (b) Q – S – P – R The correct sequence should be :
(c) S – Q – P – R (d) R – P – Q – S (a) Q S R P (b) Q R P S
73. The judge when he was presented in court sentenced the (c) P S R Q (d) Q P S R
(P) (Q) 78. The foundations of the prosperity of a state
prisoner to life imprisonment by the police on a charge of (P)
(R) (S) primary health and education but also
murder (Q)
The correct sequence should be: involves the creation of job opportunities
(a) P – R – S – Q (b) S – R – P – Q (R)
(c) Q – P – R – S (d) Q – R – S – P does not merely rest on
74. the problems of working wives are different they have to (S)
(P) (Q) The correct sequence should be :
look after, from those of housewives because their family as (a) P S Q R (b) P Q R S
(R) (S) (c) P R Q S (d) P S R Q
well as their jobs 79. I am sure and will be happy sooner or later a day will come
The correct sequence should be: (P) (Q) (R)
(a) S – P – R – Q (b) P – S – Q – R when all will be equal
(c) S – Q – P – R (d) Q – P – R – S (S)
75. as the marketing and distribution of drugs to eradicate the The correct sequence should be :
(P) (Q) (a) Q P R S (b) Q S R P
menace of drug addiction it has become increasingly (c) R Q S P (d) R S Q P
(R) 80. To do his/her work properly
problematic generates huge illegal profits. (P)
(S) it should be the pride and honour
(Q)
Ordering of Words in a Sentence B-69
without anybody forcing him/her 86. They knew him
(R) to leave work early that day when he permitted them
of every citizen in India (P) (Q)
(S) to be a hard taskmaster and were surprised
The correct sequence should be : (R) (S)
(a) Q S R P (b) P R Q S The correct sequence should be :
(c) Q S P R (d) P Q R S (a) P Q R S (b) R S Q P
81. The person who can state (c) P S Q R (d) P Q R S
correct than the person who cannot is more likely to be 87. He was known
(P) (Q) and therefore his arrest surprised everyone who knew him
his antagonist’s point of view (P) (Q)
(R) on charges of corruption to be an honest and kind man
to the satisfaction of the antagonist (R) (S)
(S) The correct sequence should be :
The correct sequence should be : (a) S P R Q (b) R Q P S
(a) R S Q P (b) R Q P S (c) P Q R S (d) Q R P S
(c) P Q R S (d) S Q R P 88. If suddenly you throw a brick at me
82. The time has come : and not a result of deliberate thought
for future generations to come (P)
(P) and my hand goes up to protect myself instinctive action
that the ideal of peace is a distant ideal (Q) (R)
(Q) it is an automatic
or one which can be postponed (S)
(R) The correct sequence should be :
when man must no longer think (a) P Q R S (b) R Q S P
(S) (c) Q S R P (d) S P Q R
The correct sequence should be : 89. Or just one of you I will not have breathed in vain today
(a) P Q R S (b) S Q R P (P) (Q)
(c) Q R S P (d) R S P Q if I have made all of you
83. I had been staying with (R)
at his cottage among the Yokshire fells a friend of mine repent of this career and seek a decent work
(P) (Q) (S)
a delightfully lazy fellow The correct sequence should be
(R) (a) Q R S P (b) R P S Q
some ten miles away from the railway station (c) R S Q P (d) Q P R S
(S) DIRECTIONS (Qs. 90-104) : Each of the following questions in
The correct sequence should be :
this section consists of a sentence the parts of which have been
(a) P Q R S (b) Q R P S jumbled. These parts have been labelled P, Q, R, and S. Given
(c) Q R S P (d) R Q P S
below each sentence are four sequences namely (a), (b), (c) and
84. All the evil in this world is brought about by persons (d). You are required to re-arrange the jumbled parts of the
when they ought to be up but do not know
sentence and select the correct sequence. [2010-II]
(P) (Q)
nor what they ought to be doing 90. It is foolish of
(R) of those who passes them to believe that will result in victory
who are always up and doing (P) (Q) (R)
(S) the use of nuclear weapons
The correct sequence should be : (S)
(a) P Q S R (b) Q P R S The correct sequence should be :
(c) S Q P R (d) P Q R S (a) R S P Q (b) Q S R P
85. If all the countries (c) P R Q S (d) S Q P R
of mankind and agree to obey 91. A distressing fact is that
(P) social accountability are dominated only by greed
work together for the common good (P) (Q)
(Q) many people today and there is hardly any
with each other and there will be no more war (R) (S)
(R) The correct sequence should be :
the laws, then they will never fight (a) S R P Q (b) Q S R P
(S) (c) P R Q S (d) R Q S P
The correct sequence should be : 92. I once had
(a) P Q R S (b) Q S P R every morning a client who swore for the past four years
(c) Q P S R (d) R Q P S (P) (Q) (R)
EBD_7367
70
B- Ordering of Words in a Sentence

she had a headache 99. The teacher warned that


(S) he would not let go home those students
The correct sequence should be : (P) (Q) (R)
(a) P R S Q (b) Q S P R who do not finish the class work
(c) R P Q S (d) S Q R P (S)
93. People know The correct sequence should be :
not only of the smokers themselves, that smoking tobacco (a) P Q R S (b) P R Q S
(P) (Q) (c) P R S Q (d) R S P Q
but also of their companions is injurious to the health 100. Towards the end of the eighteenth century, quite a number
(R) (S) of economists
The correct sequence should be : in the near future at the possibility of
(a) P S Q R (b) R P S Q (P) (Q)
(c) Q P R S (d) Q S P R were seriously perturbed the world facing starvation
94. He had (R) (S)
finished his lunch hardly at the door The correct sequence should be :
(P) (Q) (R) (a) P R Q S (b) R Q S P
when someone knocked (c) Q S P R (d) R P Q S
(S) 101. The best way of understanding our own civilization
The correct sequence should be : is to examine an ordinary man in the life of
(a) Q P R S (b) P Q R S (P) (Q) (R)
(c) Q P S R (d) R P Q S an ordinary day
95. Mr. Saxena was a profound scholar who (S)
was held in high esteem by all those The correct sequence should be :
(P) (a) P Q R S (b) R Q P S
who read his books and visited him regularly (c) P S R Q (d) R S P Q
(Q) 102. What greater thing is there
till his untimely death for two human souls to feel
(R) (P)
though not popular with the general public to rest on each other in all sorrow
(S) (Q)
The correct sequence should be : that they are joined for life
(a) P Q R S (b) R P Q S (R)
(c) S R Q P (d) S P Q R to strengthen each other in all labour
96. The Government wants that (S)
by the veterinary surgeons by the butchers The correct sequence should be :
(P) (Q) (a) S Q R P (b) R P Q S
all the goats slaughtered (c) Q R S P (d) P R S Q
(R) 103. Fame
must be medically examined by showing off to the best advantage
(S) (P) (Q)
The correct sequence should be : one’s ability and virtue is earned
(a) R P S Q (b) Q S R P (R) (S)
(c) R Q S P (d) P R S Q The correct sequence should be :
97. The general line about television (a) P Q R S (b) S P R Q
is that it is very exciting, but also potentially very dangerous (c) P R S Q (d) P Q S R
(P) (Q) 104. When he was a child
immensely powerful that I took myself passed his happiest hours
(R) (S) (P)
The correct sequence should be : the boy who was to become Britain’s Haden
(a) P Q R S (b) S P R Q (Q)
(c) P R Q S (d) R P Q S staring out of his apartment window living in New York
98. The second test of good government is that (R) (S)
to every man and woman and act only with their consent The correct sequence should be :
(P) (Q) (a) Q S P R (b) P R Q S
it should give a lot of freedom (c) S Q P R (d) R S Q P
(R) DIRECTIONS (Qs. 105-129) : In the following questions, some
and should treat their personalities with respect and parts of the sentence have been jumbled up. You are required to
sympathy rearrange these parts which are labelled P, Q, R and S to
(S) produce, the correct sentence. Choose the proper sequence
The correct sequence should be : and mark in your Answer Sheet accordingly. [2011-I]
(a) Q S P R (b) S R Q P
(c) R P S Q (d) P Q R S
Ordering of Words in a Sentence 71
B-

105. If you buy the economy pack 113. We have to provide both people and ecosystems
the manufacturers, wanting to promote the sales P Q
P the ability with the water they need.
which is quite cheap have devised a number of schemes R S
Q R The correct sequence should be :
and to ensure clearance of stocks. (a) R P Q S (b) Q R P S
S (c) S R P Q (d) R S P Q
The correct sequence should be : 114. People from doing something themselves
(a) Q P S R (b) P Q R S P
(c) R S Q P (d) Q R S P or reading about it usually learn more
106. Happiness does not consist Q R
in rank or position, it is a state of mind than by watching someone else.
P Q S
who are contended available to those. The correct sequence should be :
R S (a) R S Q P (b) R P S Q
The correct sequence should be : (c) P R Q S (d) P R S Q
(a) P Q R S (b) Q P R S 115. The prevention of disease facing the authorities
(c) P Q S R (d) R P S Q P
107. A person another’s view point was by far after the earthquake the most urgent problem.
P Q R S
limited in mind and culture who cannot understand The correct sequence should be :
Q R (a) Q S P R (b) R P Q S
is to that extent. (c) Q R P S (d) S P R Q
S 116. They did not grow well although
The correct sequence should be : P Q
(a) Q R P S (b) R P S Q he watered the plants regularly and put manure in them.
(c) P S Q R (d) S R Q P R S
108. The whole valley with date-palms was pleasantly The correct sequence should be :
P Q (a) P Q R S (b) Q R S P
and was well planted green with crops. (c) R S Q P (d) Q P R S
R S 117. On the propagation of the idea of family planning
The correct sequence should be : P Q
(a) Q S R P (b) R S Q P the governments of many countries
(c) P Q S R (d) S R P Q R
109. The extent of social progress have been spending a lot of money.
P S
in various nation-building activities The correct sequence should be :
Q (a) P Q R S (b) P S Q R
is an important indicator of women’s participation. (c) S P Q R (d) R S P Q
R S 118. I sat down this morning to write an article but found
The correct sequence should be : P Q R
(a) P R S Q (b) S Q R P I could make no progress.
(c) R S Q P (d) Q P R S S
110. We intend of shift with our family to Bombay which The correct sequence should be :
P Q R (a) S P R Q (b) Q R P S
consists of three persons. (c) Q P R S (d) R S Q P
S 119. If it had not been where would I have been today
The correct sequence should be : P Q
(a) P Q R S (b) P R Q S for your timely help five years ago?
(c) Q R P S (d) R Q P S R S
111. The wings are so small useless for flying of some birds The correct sequence should be :
P Q R (a) P Q R S (b) Q P R S
that they are. (c) Q R S P (d) P R Q S
S 120. It was almost midnight sitting at the table,
The correct sequence should be : P
(a) R P S Q (b) Q S P R when I arrived home, but I found them both waiting for
(c) P R S Q (d) S R P Q me.
112. The person at night next door sings loudly who lives. Q R S
P Q R S The correct sequence should be :
The correct sequence should be : (a) Q R P S (b) R Q P S
(a) P R Q S (b) Q S R P (c) Q P S R (d) R P Q S
(c) S Q R P (d) P Q S R
EBD_7367
72
B- Ordering of Words in a Sentence

121. Do or die was the call that Gandhiji gave The correct sequence should be :
P Q (a) P S Q R (b) S Q P R
when he asked the British to quit India (c) S Q R P (d) R P S Q
R 129. Shyam took a photograph from the window
to all freedom fighters. P Q R
S which was overexposed.
The correct sequence should be : S
(a) Q P R S (b) P R Q S The correct sequence should be :
(c) R S P Q (d) P Q S R (a) S Q R P (b) R P S Q
122. The scientist refused to talk to the press who discovered (c) P R Q S (d) R Q S P
P Q
DIRECTIONS (Qs. 130-145) : Each of the following questions
that everyone is talking about the ancient cure.
in this section consists of a sentence parts of which have been
R S
jumbled. There parts have been labelled P, Q, R and S. Given
The correct sequence should be :
below each sentence are four sequences namely (a), (b), (c)
(a) Q R P S (b) R S P Q
and (d). You are required to rearrange the jumbled parts of the
(c) Q S R P (d) P R S Q
sentence and select the correct sequence. [2011-II]
123. His grandmother on all auspicious days was so pious
P Q 130. Now that the office is closed
that she would visit the temple without fail. have gone the typists all
R S P Q R S
The correct sequence should be : The correct sequence should be :
(a) Q R P S (b) R Q S P (a) S Q R P (b) R Q P S
(c) P Q R S (d) S R Q P (c) S R Q P (d) Q R P S
124. There are parents who consider a needless luxury 131. He gave
P Q to the child a doll was broken whose head
in our country the education of their daughters. P Q R S
R S The correct sequence should be :
The correct sequence should be : (a) R Q P S (b) P S Q R
(a) Q R S P (b) P S R Q (c) R S Q P (d) P Q S R
(c) R P S Q (d) S R Q P 132. My father
125. Based firmly on current lexical and learning theory when he was at school last year John’s fees paid
not only to present and to explain words P Q R S
P The correct sequence should be :
“English Vocabulary in Use” aims. (a) Q R S P (b) S R P Q
Q (c) R P S Q (d) P Q S R
and to help them work out the rules themselves 133. The Prime Minister
R little chance of winning his party to have clearly suspects
but also to show students how to use them. P Q R S
S the next election
The correct sequence should be : The correct sequence should be :
(a) Q P R S (b) Q P S R (a) R S P Q (b) Q P R S
(c) R Q P S (d) P S Q R (c) S R P Q (d) P Q R S
126. The budget with high expectations by all types of 134. All the students
taxpayers affirmed positively that
P Q P
of tax relief is being awaited. responded eagerly to the question and
R S Q
The correct sequence should be : the political affiliation of student unions was undersirable
(a) P R Q S (b) Q P R S R
(c) R P S Q (d) S P R Q interviewed on television
127. I walked on until I reached my house as fast as I could S
P Q R The correct sequence should be :
the road which led to. (a) P Q R S (b) S R P Q
S (c) R P Q S (d) S Q P R
The correct sequence should be 135. Although the motion
(a) P Q S R (b) R P S Q until it had been considerably amended
(c) R S P Q (d) S R P Q P
128. A very old man with a wrinkled face and a long beard from the House received general support
P Q R
a pretty girl of sixteen married. it was not carried
Q R S S
Ordering of Words in a Sentence 73
B-

The correct sequence should be : 145. At last


(a) P Q R S (b) S Q P R had come she had been the moment waiting for
(c) R Q S P (d) Q R S P P Q R S
136. The doctor told The correct sequence should be
the patient to give an injection to the nurse after four (a) R Q S P (b) Q S P R
hours (c) S Q R P (d) Q R P S
P Q R S
DIRECTIONS (Qs. 146-169) : Each of the following questions
The correct sequence should be
in this section consists of a sentence, the parts of which have
(a) R Q P S (b) R P S Q
been jumbled. These parts have been labelled P, Q, R and S.
(c) Q P R S (d) P S R Q
Given below each sentence are four sequences namely (a), (b),
137. The judge
(c) and (d). You are required to re-arrange the jumbled parts of
lied to the court why he had asked the accused
the sentence and select the correct sequence. [2012-I]
P Q R
in spite of his oath 146. Coming late,
S lest he should disturb the class John tiptoed into the room
The correct sequence should be P Q R
(a) P S R Q (b) Q P S R which was going on seriously
(c) P R Q S (d) R Q P S S
138. His father said The correct sequence should be
and stop on trivial things wasting your time now get up (a) PQRS (b) RSPQ
P Q R S (c) QSPR (d) RPQS
The correct sequence should be 147. If people who commit crimes they will go on
(a) S P R Q (b) P S R Q P Q
(c) Q R S P (d) R Q S P committing crimes are not punished
139. The stranger’s movements R S
and the police him arrested aroused suspicion The correct sequence should be
P Q R S (a) PQRS (b) PSQR
The correct sequence should be (c) RSPQ (d) QSPR
(a) S Q P R (b) R Q P S 148. to cross the broken bridge not knowing how
(c) S P R Q (d) R P Q S P Q
140. The future to wade through the river they decided
is gloomy outlook animal of this noble R S
P Q R S The correct sequence should be
The correct sequence should be (a) PQRS (b) RSQP
(a) P Q S R (b) Q P S R (c) SRPQ (d) QPSR
(c) R P S Q (d) Q S R P 149. to switch off the television please feel free
141. The food P Q R
at the party served was delicious but not wholesome if you do not like this programme
P Q R S S
The correct sequence should be The correct sequence should be
(a) P R S Q (b) Q P R S (a) PQRS (b) QPRS
(c) R S P Q (d) Q R S P (c) SRPQ (d) RSQP
142. In one of the Asian countries 150 The rising prices out of gear have thrown
as well as women wear salwars men P Q
P Q R S important industries many of our
The correct sequence should be R S
(a) P S R Q (b) P Q R S The correct sequence should be
(c) S P Q R (d) R Q P S (a) SPQR (b) RSPQ
143. You won’t believe me, (c) QPSR (d) QPRS
to see him I went but whenever he was out 151. lying dead on the roadside his uncle saw an old man
P Q R S P Q
The correct sequence should be returning home in the evening
(a) P Q R S (b) Q R S P R
(c) S R P Q (d) R Q P S with his faithful dog standing beside him
144. As the car S
stood up to greet him came near the door the waiters The correct sequence should be
P Q R S (a) PQRS (b) QPSR
The correct sequence should be (c) RQPS (d) QSPR
(a) Q P S R (b) S R P Q 152. He was a scholar who was adored by all those
(c) R S P Q (d) P R S Q P
EBD_7367
74
B- Ordering of Words in a Sentence

who came close to him to the last day of his life 159. Although the train was delayed by half an hour,
Q R he could not reach the station
though not widely known P
S to catch the train in time bound for Delhi
The correct sequence should be Q R S
(a) PSRQ (b) RPQS The correct sequence should be
(c) SRPQ (d) SPQR (a) PQRS (b) PRQS
153. a couple and their teenage daughter (c) QRPS (d) PSQR
P 160. When the rain stopped, we set out to stay the night
in their one room house in North Delhi P Q
Q where we had planned for the next town
reportedly committed suicide on Tuesday night R S
R The correct sequence should be
driven by abject poverty (a) PQRS (b) QPRS
S (c) PRQS (d) PSRQ
The correct sequence should be 161. Dr. Ravi has been appointed Reader in the University
(a) SQPR (b) PSRQ P
(c) PRSQ (d) SPRQ in the recent selections
154. One should remember that lying at the side of the road Q
P in a Private Degree College who was working' as a Lecturer
if we found our neighbour R S
Q The correct sequence should be
unable to move because of a broken leg (a) PQRS (b) SRPQ
R (c) QPSR (d) SQRP
we should show our neighbourliness 162. The Principal has issued a notice
S will have to vacate the hostel
The correct sequence should be P
(a) SQPR (b) PQRS that those junior doctors
(c) QRPS (d) SQRP Q
155. that he sought my advice to such an extent if they fail to join duty by Monday next
P Q R
even in private domestic matters I won his confidence who are participating in the strike
R S S
The correct sequence should be The correct sequence should be
(a) QRSP (b) QPRS (a) QSPR (b) RQSP
(c) RQSP (d) SQPR (c) PRQS (d) QRSP
156. It is believed that the tiger Mr. Saxena 163. as I stood on the mountain I watched the early sun
P Q P Q
being severely injured in the first encounter burn off the mist that covered the forest
R R
was too terrified to make a second attempt at shooting on the fifth morning
S S
The correct sequence should be The correct sequence should be
(a) RPSQ (b) PRSQ (a) SQRP (b) SPQR
(c) RQSP (d) PSQR (c) PSQR (d) QSPR
157. I felt like a Cold War spy for a moment 164. The TV Newsreader announced that ten people
P Q P
to whom a critical secret had just been revealed when a mad dog bit them sustained injuries
R S Q R S
The correct sequence should be The correct sequence should be
(a) PQRS (b) PSQR (a) PQRS (b) QPRS
(c) PRSQ (d) QPRS (c) RPQS (d) PSQR
158. The dacoit many heinous crimes had committed 165. we were now moving slowly but steadily and had
P Q p
who carried a reward of fifty thousand rupees no need to stop our breath in order to regain
R Q R S
on his head The correct sequence should be
S (a) PQSR (b) PQRS
The correct sequence should be (c) SPQR (d) QPRS
(a) PQRS (b) QPSR 166. I have read who by some mysterious laws of her nature
(c) RQPS (d) RSQP P
Ordering of Words in a Sentence B-75
in the form of a foul and poisonous snake The correct sequence should be
Q (a) Q S P R (b) Q P S R
the story of a fairy, (c) Q R S P (d) P Q R S
R 172. It must not be imagined
was condemned to appear in certain seasons that a walking tour, is merely a better or worse way
S P Q
The correct sequence should be as some would say, of seeing the country
(a) QRSP (b) PQRS R S
(c) RQPS (d) RPSQ The correct sequence should be
167. About half of the blindness of vitamin A shortage (a) R P Q S (b) P R Q S
P Q (c) P Q S R (d) P Q R S
is attributed to children in India 173. It was reported that the table belonging to a lady
R S P Q
The correct sequence is should be with three carved legs was sold the next day by auction
(a) QRPS (b) RPSQ R S
(c) PSRQ (d) SPQR The correct sequence should be
168. What a wonderful thing it is the sun is the source (a) P R Q S (b) Q P R S
P (c) P Q S R (d) P Q R S
of all power in some way or the other. 174. Everything was after plain sailing
Q R P Q R
to think that we engaged a guide
S S
The correct sequence should be The correct sequence should be
(a) SRPQ (b) SRQP (a) S P Q R (b) P Q R S
(c) SQRP (d) PQSR (c) S R Q P (d) P R Q S
169. it rained and thundered and the child 175. The boy in the Nehru Centenary Debate
P P
who was collecting clothes on the line is the eldest son of my friend
Q Q
ran upto his mother to save them from getting wet who has won the gold medal held in the Town hall today
R S R S
The correct sequence should be The correct sequence should be
(a) PQRS (b) PRQS (a) P Q R S (b) Q R S P
(c) PSRQ (d) PQSR (c) R P S Q (d) S P Q R
176. Of paint two coats give the doors
DIRECTIONS (Qs. 170-180) : Each of the following questions
P Q R S
in this section consists of a sentence, the parts of which have
The correct sequence should be
been jumbled. These parts-have been labelled P, Q, R and S.
(a) R S Q P (b) P S Q R
Given below each sentence are four sequences namely (a), (b),
(c) S P Q R (d) R S P Q
(c) and (d). You are required to rearrange the jumbled parts of
177. Many of the old sailors say it is unlucky to sail on a Friday
the sentence and select the correct sequence. [2012-II]
P Q R
170. Most of Hitchcock's films were critically acclaimed on both who are generally superstitious
sides of the Atlantic S
P The correct sequence should be
earning him both fame and fortune (a) P R S Q (b) P S Q R
Q (c) S R Q P (d) P Q R S
and made good money at the box office, 178. It in the news bulletin did not feature that this matter
R P Q R
in no small measure was surprising
S S
The correct sequence should be The correct sequence should be
(a) P R Q S (b) P R S Q (a) P S Q R (b) R Q P S
(c) Q P R S (d) Q S P R (c) Q S P R (d) S R Q P
171. One monsoon evening, the farmer returned from the fields 179. Don't stop him for an hour if you he will run
P P Q R S
when the sky was overcast with threatening clouds The correct sequence should be
Q (a) Q P S R (b) P Q S R
and found a group of children playing on the road (c) S Q R P (d) S P R Q
R 180. In the evenings she went out once a week shopping
a little earlier than usual P Q R S
S
EBD_7367
76
B- Ordering of Words in a Sentence

The correct sequence should be 187. The natives of Caribbean


(a) P Q R S (b) R Q S P regarded the papaya because of its ability as a magic tree
(c) Q P R S (d) S P R Q (P) (Q) (R)
DIRECTIONS (Qs. 181-193) : Each of the following questions to keep them healthy
in this section consists of a sentence, the parts of which have (S)
been jumbled. These parts have been labelled P, Q, R and S. The proper sequence should be :
Given below each sentence are four sequences namely (a), (b), (a) P R S Q (b) P R Q S
(c) and (d). You are required to re-arrange the jumbled parts of (c) R P Q S (d) R P S Q
the sentence and select the correct proper sequence and mark 188. The Indian woman wants
in your Answer sheet accordingly. [2013-I] in a male dominated society as an equal partner
181. It was true that the pet dog would never sleep anywhere (P) (Q)
(P) (Q) and it is not too much to demand her rightful place
we once had except on sofa (R) (S)
(R) (S) The proper sequence should be :
The proper sequence should be : (a) S R P Q (b) R S Q P
(a) P Q R S (b) S P Q R (c) S Q P R (d) Q P S R
(c) R P Q S (d) P R Q S 189. This is a letter
182. Recently containing memorable letters of Churchill a book by a young lady who was lately wounded in a duel
(P) (Q) (P) (Q)
has been published by a reputed publisher written passionately
(R) (S)
(R)
The proper sequence should be :
wherein she laments the misfortune of a gentleman
(a) Q R P S (b) Q P R S
(c) P Q R S (d) R P Q S (S)
183. As the situation has changed, The proper sequence should be :
since we last discussed this matter (a) S R P Q (b) R S Q P
(P) (c) R P S Q (d) Q P S R
it was best to contact you it appeeared to me 190. It is
(Q) (R) for a man when he accompanies a lady an accepted custom
without losing time (P) (Q) (R)
(S) to open the door
The proper sequence should be : (S)
(a) P Q R S (b) P R S Q The proper sequence should be :
(c) P R Q S (d) S P R Q (a) P S Q R (b) R P S Q
184. After the awarding speeches (c) P S R Q (d) R P Q S
the prizes given and had been delivered
191. We
(P) (Q) (R)
agreed with the manner in which you said it
I got up to address the gathering
(S) (P) (Q)
The proper sequence should be : but was objected to what you said
(a) S R Q P (b) S P Q R (R) (S)
(c) R S Q P (d) R Q P S The proper sequence should be :
185. The management having agreed (a) P S Q R (b) S P Q R
(P) (c) S P R Q (d) P S R Q
the workers called off the strike to increase their wages 192. It is a wonder that migratory birds without ever getting lost
(Q) (R) (P) (Q)
and returned to work always manage to find their home
(S) (R)
The proper sequence should be : flying thousands of kilometres
(a) P Q R S (b) P S R Q (S)
(c) P R Q S (d) S P Q R
The proper sequence should be :
186. All religions are
(a) P Q R S (b) S P Q R
to advance the cause of peace in a holy partnership
(P) (Q) (c) S Q R P (d) P S Q R
justice and freedom bound together 193. He asked me
(R) (S) would telephone him as soon as I if I reach home
The proper sequence should be : (P) (Q) (R) (S)
(a) P Q R S (b) P R Q S The proper sequence should be :
(c) S P Q R (d) S Q P R (a) P S Q R (b) Q P R S
(c) R P Q S (d) S R Q P
Ordering of Words in a Sentence 77
B-

DIRECTIONS (Qs. 194-209) : In the following questions, some 202. He said that two years before and that he could produce
parts of the sentence have been jumbled up. You are required to P Q
re-arrange these parts which are labelled P, Q, R and S to produce he had passed his examination his certificate.
the correct sentence. Choose the proper sequence and mark in R S
your Answer Sheet accordingly. [2013-II] The correct sequence should be
(a) S P Q R (b) Q S R P
194. The fire before any serious damage was done (c) R P Q S (d) P R S Q
P 203. However noble their ends appear
by volunteers was controlled in the godown. since they make use of violence are bound to end up
Q R S P Q
The correct sequence should be all their acts with untold misery for all.
(a) S R Q P (b) R S P Q R S
(c) R Q P S (d) Q R S P The correct sequence should be
195. Some educationists should not be exposed to believe that (a) P Q R S (b) R Q S P
P Q (c) P R Q S (d) R Q P S
young children too much television viewing. 204. When the artist had started singing a popular song
R S P Q
The correct sequence should be in chorus everyone in the audience joined.
(a) R S P Q (b) Q P R S R S
(c) Q R P S (d) P Q R S The correct sequence should be
196. Like energy crisis is one of the most talked about (a) R P Q S (b) P Q R S
P Q (c) P Q S R (d) R Q P S
population explosion topics of the world these days. 205. We know that he passed but that he set for the examination
R S P Q R
The correct sequence should be we have not heard.
(a) S R Q P (b) R Q P S S
(c) S P Q R (d) P R Q S The correct sequence should be
197. The dog with its customary fondness before the master (a) P Q R S (b) R Q S P
P Q R (c) Q S R P (d) S P Q R
wagged his tail. 206. When he knew he sat down under a tree
S P
The correct sequence should be about what to do next and thought for a long time
(a) P Q R S (b) P S R Q Q R
(c) P R S Q (d) Q R S P that there was no more hope.
198. When the driver in front of me slammed the brakes on S
P Q The correct sequence should be
I was driving in rush hour without warning. (a) S R Q P (b) S P R Q
R S (c) Q P S R (d) R S P Q
The correct sequence should be 207. He wanted to reach his place of work
(a) P Q S R (b) R S Q P P
(c) R P Q S (d) P Q R S who used to travel 20 miles to buy a scooter for his son.
199. When John saw coming head on towards him Q R S
P Q The correct sequence should be
a speeding truck he ran for life. (a) P Q R S (b) Q R S P
R S (c) Q S R P (d) R S Q P
The correct sequence should be 208. They want out in the morning by a bus
(a) R P Q S (b) R S P Q P
(c) P R S Q (d) P Q R S to spend the day at Bhimli, a famous picnic spot,
200. They decided for their friends that afternoon Q R
P Q some 25 kms away
to buy some presents to go shopping. S
R S The correct sequence should be
The correct sequence should be (a) P Q R S (b) Q R S P
(a) Q R S P (b) S Q R P (c) R S P Q (d) S R Q P
(c) Q R P S (d) P Q S R 209. Just as the goodness of movies
201. This is the book about that I told you. like the things which they represent,
P Q R S P
The correct sequence should be does not consist in being
(a) P R S Q (b) P S Q R Q
(c) S P Q R (d) P Q R S so the goodness of music does not consist
R
EBD_7367
78
B- Ordering of Words in a Sentence

in its being like the noises we know The proper sequence should be :
S (a) SPQR (b) PQRS
The correct sequence should be (c) RSPQ (d) RSQP
(a) Q P R S (b) P Q R S 215. The Prime Minister declared that those states
(c) R P S Q (d) S P R Q P
will get all help and aid
DIRECTIONS (Qs. 210-220) : In each of the following questions,
Q
there is a sentence of which some parts have been jumbled up.
where family planning is effected very efficiently
You are required to rearrange these parts which are labeled P,
R S
Q, R and S, to produce the correct sentence. Choose the correct
The proper sequence should be :
sequence and mark in your answer sheet accordingly. [2014-I]
(a) PRSQ (b) PQRS
210. There must be countries now in which peasants (c) RSPQ (d) QPSR
can spend several years in universities 216. Hardly had my brother descended from the plane
P when the people waved and cheered
so that P Q
Q who had come to receive him from the lounge
a lot of young persons R S
R The proper sequence should be :
are going without substantial meals (a) PRQS (b) PQRS
S (c) SPQR (d) PRSQ
The proper sequence should be : 217. My friend when he was going to his office
(a) SRQP (b) PQRS P
(c) SQRP (d) QPSR met with an accident on his scooter
211. Athens it was also Q R
P due to rash driving
the first democracy in the world was not only S
Q R The proper sequence should be :
an almost perfect democracy (a) PQRS (b) PRQS
S (c) SRQP (d) QSRP
The proper sequence should be : 218. The boy said I am not going to the school
(a) RSPQ (b) PQRS P
(c) RQPS (d) QPSR with my friends in the class room
212. The practice of talking performance-boosting drugs among Q
athletes where my teacher scolds me when I want to play
but checking it is not going to be easy R S
P The proper sequence should be :
is generally conceded to be unfair (a) PQRS (b) PSQR
Q (c) SQPR (d) PRSQ
of the detection technology 219. Mohan, the son of my friend, gave me a set of pens
R P
for the user is generally one jump ahead which is very precious while working in Japan
S Q R
The proper sequence should be : who died in an accident
(a) RSPQ (b) QPSR S
(c) QPRS (d) PQRS The proper sequence should be :
213. All the religions are to advance the cause of peace (a) PQRS (b) SRPQ
P (c) RSPQ (d) SPQR
in a holy partnership justice and freedom 220. The clerk on the desk left the money
Q R P Q
bound together in the safe which he should have locked up
S R S
The proper sequence should be : The proper'sequence should be :
(a) PRQS (b) PQRS (a) PQRS (b) RSPQ
(c) SQPR (d) SPQR (c) QPRS (d) QPSR
214. Seventy-two people reports PTI DIRECTIONS (Qs. 221-231) : In each of the questions there is a
P sentence of which some parts have been jumbled up. You are
were affected by food poisoning'
required to rearrange these parts which are labelled P, Q, R and
Q
S, to produce the correct sentence. Choose the correct sequence
including several women and children
R and mark in your Answer Sheet accordingly. [2014-II]
of the central part of the city
S
Ordering of Words in a Sentence B-79
221. The soldiers decided to hold out in the fort was killed 230. For an hour because it had to wait due to dense fog
P Q R P Q R
till the last man among them the plane couldn't take off
S S
(a) QPSR (b) PQSR (a) PQRS (b) RQPS
(c) QRSP (d) SRPQ (c) QPRS (d) SPQR
222. Her friend when she was very ill last year on Sheila 231. She complained that that it took a month to clean it
P Q R P Q
could not attend the house was so dirty
S R S
(a) PQRS (b) SRPQ (a) PQRS (b) SPQR
(c) RSPQ (d) RPQS (c) PSQR (d) RSPQ
223. The poems and stories have been taken for this book
DIRECTIONS (Qs. 232-238): In each of the questions in this
P Q
section there is a sentence of which some parts have been jumbled
from a variety of sources that have been selected
up. You are required to rearrange these parts which are labelled
R S
P, Q, R and S, to produce the correct sentence. Choose the correct
(a) SQPR (b) RPQS
sequence and mark in your Answer Sheet accordingly.[2015-I]
(c) QPSR (d) PQRS
224. The belief that the moon has great influence 232. When the car passed by he threw a stone,
P P
still exists with great force over the weather raising dust in the road with all his might
Q R Q R
among many people and hit the man who was driving it
S S
(a) PRQS (b) QPRS The proper sequence should be :
(c) PQSR (d) QSPR (a) P Q R S (b) Q P R S
225. There is an old saying in our country (c) R P Q S (d) R P S Q
that soldiers 233. A moment comes
P when we step out from the old to the new
not only cover themselves with glory on the earth P
Q which comes but rarely in history
who die for their motherland but attain heaven Q
R S and when the soul of a nation
(a) PSQR (b) SPRQ R
(c) QPSR (d) PRQS long suppressed, finds utterance
226. Boys are on their parents invariably dependent S
P Q The proper sequence should be :
until they can earn money to support themselves (a) Q P R S (b) P R S Q
R S (c) Q R S P (d) R S Q P
(a) RSQP (b) QRSP 234. She had a blind belief that inside the bag
(c) RPQS (d) QPRS two or three other children there were perhaps
227. The principal said that those students P Q
P which the big man carried like herself
would not be permitted to enter who do not produce R S
Q R The proper sequence should be :
identity cards (a) Q R P S (b) Q P S R
S (c) R Q S P (d) R Q P S
(a) RSPQ (b) RPSQ 235. He approached the teacher at school to know
(c) PRSQ (d) SRPQ P Q
228. The doctor remarked that lying in bed if students are in his studies how his son was getting on
P Q R S
fond of reading it will be bad for the eyes The proper sequence should be :
R S (a) P Q R S (b) P Q S R
(a) PQRS (b) QRSP (c) Q S R P (d) Q S P R
(c) SQRP (d) QRPS 236. I was so angry that if I had met him in the street
229. We saw while playing with the boys that Ram fell down P
P Q R and not waited to ask him
and hurt his leg Q
S why he had written me an insulting letter
(a) RSPQ (b) PQRS R
(c) RPQS (d) RPSQ I would have knocked him down
S
EBD_7367
80
B- Ordering of Words in a Sentence

The proper sequence should be : The proper sequence should be


(a) P S Q R (b) P Q R S (a) P S Q R (b) Q P S R
(c) S Q R P (d) R S Q P (c) R Q P S (d) P R S Q
237. On the contrary, about family planning and its benefits
quickly he gave orders to catch the thief
P 243.
to all citizens the Government wants P Q R
Q R to his men
to provide information and education
S. S
The proper sequence should be: The proper sequence should be
(a) R Q S P (b) S P Q R (a) S P R Q (b) Q S R P
(c) R S P Q (d) Q P R S (c) P S R Q (d) R S P Q
238. In spite of the poor and hungry people to give a definition if I were I would begin like this
P 244.
P Q R S
made by medical sciences the extraordinary progress
Q R The proper sequence should be
(a) Q P R S (b) P Q R S
often die
(c) S R Q P (d) R S P Q
S
The proper sequence should be : deserve all honour in society in doing their job well
(a) R P S Q (b) R Q P S 245.
P Q
(c) P Q S R (d) R S P Q
men of conscience who take pride What ever its nature
DIRECTIONS (Qs. 239-253) : Each of the following items in
this section consists of a sentence the parts of which have been R S
jumbled. These parts have been labelled P, Q, R and S. Given The proper sequence should he
below each sentence are four sequences namely (a), (b), (c) and (a) R Q S P (b) Q R P S
(d). You are required to rearrange the jumbled parts of the (c) P R S Q (d) S P Q R
sentence and mark your response accordingly. [2016-I] 246. while some live
to eat and drink
P Q
239. that it would affect the investigation process
in luxury
P many do not have enough
S
they refused of these raids saying R
Thc proper sequence should be
Q R (a) P S R Q (b) P R S Q
(c) S P Q R (d) R Q S P
to divulge the venues
S that no matter
247. I believed then
The proper sequence should be P
(a) P Q R S (b) S R P Q one should always find some time for exercise
(c) Q S R P (d) R P Q S
Q
240. that he already has buying things that rich man
and I believe even now the amount of work one has
P Q R
R S
goes on The proper sequence should be
S (a) P R Q S (b) P S R Q
The proper sequence should be (c) R P Q S (d) R P S Q
(a) P S Q R (b) R S Q P
I wonder whenever I decide to go to the cinema
(c) S Q R P (d) R P Q S 248.
P Q
241. the police commissioner rushed the crowd
with my scooter why I always have trouble
P Q
R S
to control the police force The proper sequence should be
R S (a) Q S P R (b) Q R S P
The proper sequence should be (c) P S R Q (d) P R S Q
(a) Q R S P (b) P Q R S by means or snares
(c) PSRQ (d) R S P Q 249. The bird-catcher
P
to attend his friend's wedding
242. my brother knew all the birds of the forest by the hundred
P Q
Q R
is going to Chennai tomorrow
and was accustomed to capturing the winged creatures
R S
S
Ordering of Words in a Sentence 81
B-

The proper sequence should be that the destructive nature of weapons


(a) Q P S R (b) Q S R P Q
(c) P Q S R (d) P R Q S to give up war has not been fulfilled
R S
250. Man is a biological being The correct sequence should be
P (a) P S Q R (b) P Q R S
his physical and material needs confined to (c) Q P R S (d) P R Q S
Q 256. The leader of the opposition,
R
not merely in the manner he had planned to convince them,
S P
The proper sequence should be on realising that he had failed to convince the assembly,
(a) R S P Q (b) S R Q P Q
(c) R P S Q (d) S P R Q who had a reputation for speech making,
R
251. A gang of robbers while they were fast asleep was very much disappointed
P S
The correct sequence should be
entered the village and stole the property of the villagers (a) P S R Q (b) Q S R P
Q R (c) R S Q P (d) S P R Q
257. We can think of often confused in the puble mind
at night
P
S which suggest the need for two factors
The proper sequence should be Q R
(a) Q S R P (b) S Q P R an international language
(c) S P Q R (d) Q P S R S
The correct sequence should be
252. The opposition members the ruling of the speaker (a) P R Q S (b) R P Q S
P
(c) R Q S P (d) S Q R P
to protest against of the parliament walked out 258. The man is generally the one who can work very hard
Q R S P Q
The proper sequence should be when he must work
(a) S P Q R (b) Q R P S R
(c) R S P Q (d) S R Q P who can play most heartily when he has the chance of
playing
253. When a boy saved her by a speeding car S
P Q The correct sequence should be
(a) QRSP (b) PSQR
at the risk of his life (c) S P Q R (d) QRPS
R 259. Dear Lodger, I agree, the roof is leaking; but
a little girl was about to be run over
there would be no need while it is raining
S P Q
The proper sequence should be and when the Sun shines, I can't get it repaired
(a) S P R Q (b) R S Q P R S
(c) S Q P R (d) Q P S R The correct sequence should be
DIRECTIONS (Qs. 254-264): Each of the following items in (a) Q R S P (b) S Q R P
this section consists of a sentence the parts of which have been (c) R S P Q (d) P R Q S
jumbled. These parts have been labelled P, Q, R and S. Given 260. There is no reason that life may exist
below each sentence are four sequences namely (a), (b), (c) and P
(d). You are required to rearrange the jumbled parts of the in great profusion why we should not be willing to think
sentence and mark your response accordingly. [2016-II] Q R
in other worlds
254. Georgian loyalists and rebel forces after a week of fighting
S
P
The correct sequence should be
agreed to a ceasefire today in which 51 people were killed
(a) R S P Q (b) P Q S R
Q R S
(c) R P Q S (d) R Q S P
The correct sequence should be
261. The Regent released Voltaire having discovered that
(a) P Q R S (b) Q R S P
(c) Q P S R (d) Q R P S P Q
255. The ultimate hope will force the nations he had imprisoned an innocent man
R
P
EBD_7367
82
B- Ordering of Words in a Sentence

and gave him a pension but also because those Who recovered
S R
The correct sequence should be were permanently disfigured
(a) P Q R S (b) P R S Q S
(c) Q R S P (d) Q R P S The correct sequence should be
262. which is sold for its horn (a) PQRS (b) QRSP
P Q
the Rhinoceros is hunted by poachers at high prices (c) PRSQ (d) QPRS
R S 268. a person bitten by a rabid dog
The correct sequence should be P
(a) S Q P R (b) Q R S P would be seized by violent symptoms
(c) R Q P S (d) Q P S R Q
263. as environmental tools for military purposes alter an incubation period of a month or two
P Q R
are finding various new uses and die an agonizing death
R S
the world's fastest computers initially conceived The correct sequence should be
S (a) PRSQ (b) SRQP
The correct sequence should be (c) PRQS (d) PQRS
(a) S Q R P (b) P Q R S 269. The traveller, sat down to rest by the roadside being weary,
(c) R P S Q (d) Q P S R P Q R S
264. is a painful feeling The correct sequence should be
P (a) PQRS (b) RSPQ
without this indirect appeal to our self love, (c) SPQR (d) RPQS
Q 270. the house was away with its liveliest member gloomy
the sense of inferiority in others. P Q R S
R The correct sequence should be
and not an exalting one (a) PQRS (b) PSRQ
S (c) RSPQ (d) QRPS
The correct sequence should be 271. he was a tiny man with a sprightly walk tall
(a) P R S Q (b) R S P Q P Q R
(c) SRQP (d) R Q S P barely five feet
DIRECTIONS (Qs. 265-284): In each of the items in this section, S
there is a sentence of which some parts have been jumbled up. The correct sequence should be
You are required to rearrange these parts which are labelled P, (a) RSQP (b) SRQP
Q, R and S to produce the correct sentence. Choose the proper (c) QSPR (d) PSRQ
sequence and mark in your Answer Sheet accordingly. 272. rules and regulations he cheerfully ignored its demands
[2017-1] P Q
a life of unaccustomed to
265. he almost planned the entire strategy of operation R S
P Q R The correct sequence should be
single-handed (a) PQRS (b) QRPS
S (c) SRPQ (d) QSRP
The correct sequence should be 273. The man for a moment, looked at me and thus emptied my
(a) RSPQ (b) PRQS P Q R
(c) SQRP (d) QPSR pockets turned me upside down
266. it is a fact that rice is one of the most prolific S
P The correct sequence should be
than almost any other crop (a) PQRS (b) QPRS
Q (c) QPSR (d) PQSR
yielding a greater return per acre of food crops 274. The lady today is visiting us who composed this poem
R S P Q R S
The correct sequence should be The correct sequence should be
(a) SRQP (b) RPSQ (a) RQPS (b) PRQS
(c) QRSP (d) PSRQ (c) QRPS (d) SQRP
275. Animals are able to measure the passing of the seasons
267. up to the end of the eighteenth century, not only because it P Q
was often fatal, changes in day-length by recognizing
P R S
smallpox was a particularly dreaded disease, The correct sequence should be
Q (a) PQSR (b) SQPR
(c) PRSQ (d) SRPQ
Ordering of Words in a Sentence B-83
276. Many people inaudible to others in the ears or head is built around the automobile
P Q S
suffer persistent noises with hearing problems The correct sequence should be
R S (a) RPQS (b) QRSP
The correct sequence should be (c) RSPQ (d) SPQR
(a) RQSP (b) PSQR 284. with an idea to reach the deprived child
(c) SRQP (d) RSQP P
277. Researchers found that allergic reactions are brought about by the United Nations Organization
P Q
in the bloodstream in the 1940s by the liberation of small the year 1979 has been declared as
Q R R
quantities of a substance called histamine the International Year of the Child
S S
The correct sequence should be The correct sequence Should be
(a) RPSQ (b) QRPS (a) PQRS (b) PSQR
(c) RQSP (d) PSQR (c) PRSQ (d) RSPQ
278. Attempts at transplanting other organs, but that which most
caught the public fancy DIRECTIONS (Qs. 285-304) : Each of the following items in
P this section consists of a sentence the parts of which have been
was the heart transplant such as the lungs or the liver, jumbled. These parts have been labelled P, Q, R and S. Given
Q R below each sentence are four sequences namely (a), (b), (c)
have been made and (d). You are required to re-arrange the jumbled parts of the
S sentence and mark your response accordingly. [2017-II]
The correct sequence should be 285. My unmarried aunt
(a) SRQP (b) RSPQ
(c) RPSQ (d) QRPS is creating a lot of problems for us who stays with us
279. sports cars appeal to some motorists only P Q
P Q R
with noisy exhausts because of her interfering
S nature in our personal lives
The correct sequence should be R
(a) RQSP (b) PSQR
(c) RSPQ (d) PQSR and there is a misunderstanding
280. Friendship has its place in truth and justice among family members
P Q S
but it cannot override life,
The correct sequence should be
R S
(a) Q S R P (b) Q P S R
The correct sequence should be
(c) P Q S R (d) S P Q R
(a) PQRS (b) RSPQ
286. No criminal proceeding
(c) PSRQ (d) RQPS
281. the workaholic often becomes a prisoner of success in any Court during his term of office
P Q P
inadvertently reaping greater and greater rewards
R S whatsoever shall be initiated
The correct sequence should be Q
(a) SPRQ (b) SPQR
or continued against the President or a Governor
(c) RPQS (d) PQRS
282. the school masters and the professors tend to believe R S
P The correct sequence should be
that innate intelligence is a quality (a) Q R S P (b) P Q R S
Q (c) Q P S R (d) S Q P R
but that you can't do much about it except measure it with a severe tooth-ache
R 287. The dentist
P
which varies very greatly from one individual to another
S when he was brought to hospital
The correct sequence should be Q
(a) PSQR (b) PRSQ
(c) PQSR (d) QSRP extracted Manish 's tooth and relieved his pain
283. People do not realize that modern society R S
P Q The correct sequence should be
who object to driver education (a) P Q R S (b) Q P R S
R (c) R S P Q (d) R Q P S
EBD_7367
84
B- Ordering of Words in a Sentence

has been selected as the best heroine and makes them take pride in their work;
288. The actress
P Q Q
who is the daughter of a famous male singer of high quality products
R R
of the year also enhances employee satisfaction, because
S S
The correct sequence should be The correct sequence should be
(a) R P Q S (b) Q P S R (a) P Q R S (b) R Q P S
(c) R S P Q (d) Q S R P (c) P S R Q (d) R S P Q
everyday kept pestering a pretty girl where modern Malay executives
289. Last summer 294. But, Kuala Lumpur
P Q P
One persistent admirer with phone calls but will never miss Friday prayers
R S Q
The correct sequence should be might have a cellular phone in hand,
(a) Q R S P (b) R S Q P
(c) R Q S P (d) P Q R S R
to make wishes come true is a city firmly rooted in tradition
290. In many cultures S
P
The correct sequence should be
that blue has the power people believe (a) R Q S P (b) S P R Q
Q R (c) R P S Q (d) S Q R P
295. Perhaps the most significant factor
and be successful in life
is a failure of planning and
S
The correct sequence should be P
(a) P S R Q (b) R S P Q in the growth of all metropolitan crimes,
(c) R Q P S (d) S Q P R
Q
through the playground
291. From a picnic table governance in the urban sprawl
P
R
while we unpacked a basket
including crimes against the elderly,
Q
S
we watched them laugh and leap The correct sequence should be
R (a) Q S P R (b) P R Q S
(c) Q R P S (d) P S Q R
bulging with sandwiches and cookies
S a small college he 'd rather go to
296. He said that
The correct sequence should be P Q
(a) R S Q P (b) Q P R S not studying at all instead of
(c) R P Q S (d) Q S R P
R S
where he had served, The correct sequence should be
292. My father retired at the age of 68,
P (a) Q S P R (b) P R Q S
(c) Q P R S (d) Q P S R
in South Carolina as Pastor for 12 years,
Q R since there are few
297. Teaching a child
P
from a Baptist Church
is becoming difficult and expensive
S
The correct sequence should be Q
(a) P R S Q (b) S Q P R open ponds around how to swim
(c) S P Q R (d) Q P R S
293. The completion R S
The correct sequence should be
it enables employees to feel a sense of accomplishment (a) S Q P R (b) Q R S P
P (c) S R Q P (d) P R Q S
Ordering of Words in a Sentence B- 85

able to find out what had caused a university is essentially a community of


298. The doctor
P Q students and teachers
the food poisoning had not been S
R S The correct sequence should be
The correct sequence should be (a) S R Q P (b) S P Q R
(a) S P R Q (b) P R Q S (c) P Q R S (d) S Q P R
(c) P R S Q (d) S P Q R DIRECTIONS (Qs 305-314) : Each of the following items in
was suspended being corrupt this section consists of a sentence the parts of which have been
299. The officer jumbled. These parts have been labelled P, Q, R and S. Given
P Q below each sentence are four sequences namely (a), (b), (c) and
from service before his dismissal (d). You are required to re-arranged the jumbled parts of the
R S
sentence and mark your response accordingly [2018-1]
The correct sequence should be
(a) Q P S R (b) Q P R S the British manufacturers popularity of Indian textiles
305.
(c) R S Q P (d) R S P Q P Q
her old coat to a beggar were jealous of the from the very beginning
300. She gave
P Q R S
The correct sequence should be
the one with the brown fur on it shivering with cold (a) P Q R S (b) S P Q R
R S (c) S P R Q (d) Q R S P
The correct sequence should be
(a) P R Q S (b) S Q P R dress fashions changed and light cotton of the English
306.
(c) P Q R S (d) R P Q S P Q
at the ground said that the injured textiles began to replace
301. The medical teams
P R
by the surging crowds, the coarse woollens
Q S
(a) The correct sequence should be
included women and children who were trampled
R S (a) P R S Q (b) R S P Q
The correct sequence should be (c) Q P R S (d) S P R Q
(a) P R Q S (b) P Q R S
put pressure on their government
(c) Q P R S (d) P S Q R 307.
P
at the hurdles on his way who has his eyes
302. He Indian goods in England the British manufacturers
P Q
Q R
does not look fixed on the goal
to restrict and prohibit
R S
The correct sequence should be S
(a) P Q R S (b) S R P Q The correct sequence should be
(c) Q S R P (d) R Q P S (a) P Q R S (b) S P R Q
(c) R P S Q (d) Q R S P
but we know
303. We do not know when however still held their own in foreign markets
P 308.
P Q
the exact date of his death for certain
Q R in spite of these laws Indian silk and cotton textiles
R S
Shakespeare was born The correct sequence should be
S (a) Q P R S (b) S P R Q
The correct sequence should be (c) S R P Q (d) R P S Q
(a) S P R Q (b) P Q R S
and it let to rapid economic development
(c) P S Q R (d) S R Q P 309.
P
The purpose is to advance knowledge
304. the Industrial Revolution transformed the British
P
Q R
the two have to work together and disseminate it
Q R
EBD_7367
B- 86 Ordering of Words in a Sentence

society in a fundamental manner modern Japan after 1868


S R
The correct sequence should be
Asian country could develop itself without
(a) S P R Q (b) Q P R S
(c) Q R S P (d) S R P Q Western control
Muhammad Iqbal S
310. The correct sequence should be
P
(a) P Q R S (b) S R Q P
the philosophical and religious outlook of (c) P R Q S (d) Q R P S
people through his poetry and the current Hindu emphasis
313.
Q P
profoundly influenced and urged the people to imbibe the spirit of
R free - thinking
one of the greatest poets of modern India Q
S on rituals, ceremonies and superstitions
The correct sequence should be
R
(a) Q R S P (b) S R Q P
(c) S R P Q (d) S P R Q Vivekananda condemned the caste system
to accept any of the important disillusionment S
311. The correct sequence should be
P Q
(a) P Q R S (b) S P R Q
demands of the nationalists produced (c) S P Q R (d) R P S Q
R who was hardly six months old Charles
314. Mr. John
the failure of the British government P Q
S as his son adopted
The correct sequence should be
R S
(a) S P R Q (b) P Q R S
(c) S R Q P (d) Q R P S The correct sequence should be
(a) S Q R P (b) P S Q R
showed that a backward the rise of (c) R S P Q (d) P R S Q
312.
P Q

HINTS & SOLUTIONS


1. (c) The proper sequence should be "Climate change is a 5. (d) The proper sequence should be "In the post-genetic
truly unifying phenomenon in that it affects the entire engineering era of modern biology particularly in the
world irrespective of national borders cultures and last two decades or so the mouse has acquired the
political structures". status of the experimental animal or the animal model
2. (d) The proper sequence should be "This may go down system of choice for the entire spectrum of research."
as the year when land reforms were finally put back on 6. (b) The proper sequence should be "It was not an
the nation's policy agenda after decades of relative inspiration for democracy but resentment arising out
neglect and even reversal." of harsh economic conditions with fuel prices climbing
3. (a) The proper sequence is "There is however, a strong to dizzy heights which led to the recent protests in
perception that reservation alone cannot transform the Myanmar."
lives of oppressed people unless it is followed by 7. (d) The correct sequence should be "You have been
sincere and intensive efforts to clear the backlog in writing to me often about distinguishing in a fancy
respect of education and poverty alleviation dress show, or getting a first prize in sports etc. but my
programmes." daughter such achievements make me apprehensive
4. (c) The proper sequence should be "More than ever of your educational progress. "
before the continued prosperity of Asia depends on 8. (c) The proper sequence should be "The Secretary
the ability of its politicians and policy makers to provide announced that in an attempt to find an answer to
leadership and to deliver good governance and sound these questions a national workshop on technical
public policies for the benefit of its people." training had been organized by the Confederation of
Engineering Industry."
Ordering of Words in a Sentence B-87
9. (b) The proper sequence should be "Technology transfer 23. (c) As we can see that P is the beginning of the sentence
from one country to another implies the transfer of but there are three options starting with P. Now, neither
technical knowledge either through a government S nor Q makes a sensible ending. Thus, the correct
policy or via private channels of communications." answer is option (c) and the right sequence is "The
10. (a) The proper sequence should be "Physical training can man whose cycle was stolen made a complaint at the
shape well the character of young boys and girls who police station."
are going to be responsible citizens." 24. (a) As we can see that P is the beginning of the sentence
11. (b) The proper sequence should be "His favourite subject but there are three options starting with P. Now, neither
happens to be Science because Science reveals the Q nor S makes a sensible ending. Thus, the right answer
secrets of life and of nature which men did not know in is (a) and the correct sequence is "The clerk whom I
the past." wanted to meet was not present in the office."
12. (b) The proper sequence should be "From my plane flying 25. (d) As we can see that R is the beginning of the sentence
over Namibia I looked down at a haunting sight a shore but there are two options starting with R. Now, S does
of pink and silver sand edged by a line of smoking surf not make a sensible ending, thus the right option is (d)
stretching 320 kilometres away." and the correct sequence "Nobody in the house
13. (b) The proper sequence should be "I was just in time to seemed to know where the key to the main door was
see a mongoose scurrying across the grass with an kept."
egg in its mouth."
26. (a) As we can see that S is the beginning of the sentence
14. (b) The proper sequence should be "How strange that a and there is only one option starting with it. So the
refugee who had by the skin of his teeth escaped death correct sequence is "A tortoise watched two geese fly
in Germany should fall in love with a girl less than half in the air everyday."
his age when he had got to America." 27. (d) As we can see that P is the beginning of the sentence
15. (d) The proper sequence should be "I think it should be but there are two options staring with it. Now, Q
necessary for eggs as soon as they are laid by the hen followed by R does not make much sense 'and never
to be stamped with the date by the poulterer." deceive one another always speak the truth'. Thus the
16. (a) The proper sequence should be "Reuter reports that right answer is option (d) and the correct sequence is
soldiers of armies loyal to rival political factions fought "If in their daily lives the people of our country always
each other in the streets of the capital today following speak the truth and never deceive one another, then
the week-end military crackdown on demonstrators." our prestige will be great."
17. (d) The proper sequence should be "The old lady wanted 28. (d) As we can see that S is the beginning of the sentence
to sell her precious table with gold-coated legs and but there are two options starting with it. Now, R does
glossy surface to someone having the capacity to pay not make a sensible ending. Thus the right answer is
a huge amount as its price." option (d) and the correct sequence is "The clothes
18. (d) As we can see that Q is the beginning of the sentence that I ordered were sent by mistake to my father who
but there are three options starting with it. Now, P was very much surprised."
does not make a sensible ending and R followed by S 29. (c) As we can see that S is the beginning of the sentence
does not make any sense 'that all are satisfied among and there is only one option starting with it. So the
your friends to make sure'. Thus, the right answer is correct sequence is "The President with his wife and
option (d) and the sequence becomes "Please share children is returning from a brief vacation at Shimla in
the prize money equally among your friends to make order to attend a Press conference."
sure that all are satisfied." 30. (b) As we can see that Q is the beginning of the sentence
19. (d) As we can see that R is the beginning of the sentence but there are two options starting with Q. Now, Q
and there is only one option starting with it. Thus, the followed by R and S does not make much sense 'which
sequence becomes "You can learn the easy things as is written in Gujarati who is my father's best friend'.
well as the difficult ones although the easy ones are Thus the right answer is option (b) and the correct
not so easy and can be quite difficult." sequence is "This letter which is written in Gujrati is
20. (c) As we can see that R is the beginning of the sentence addressed to Mr Shah who is my father's best friend."
but there are two options starting with it. Now, S does 31. (b) As we can see that Q is the beginning of the sentence
not make a sensible ending. Thus, the right answer is and there is only one option starting with Q. So the
option (c) and the sequence becomes "We met Mr correct sequence is "Whether the plan suggested will
and Mrs Gupta on our way home and asked them if succeed or fail depends on how it will be received by
they would be free this evening". those interested in its progress."
21. (c) As we can see that Q is the beginning of the sentence 32. (b) As we can see that P is the beginning of the sentence
but there are two options starting with Q. Now, P does but all the options are starting with P. Also, R and Q do
not make a sensible ending. Thus, the right answer is not make a sensible ending. So, we have to decide
(c) and the correct sequence "immediately after his between option (a) and (b). Now, Q followed by R
arrival he began to quarrel with his wife". does not make much sense 'called an assembly of my
22. (c) As we can see that Q is the beginning of the sentence grand - daughter's school'. Thus the right answer is
but there are three options starting with it. Now, P option (b) and the correct sequence is "the nun who
does not make a sensible ending and P followed by S was the Principal of my granddaughter's school called
does not make any sense 'in the middle of to take a an assembly to announce the result of a fund-raising
glass of water'. Thus, the right answer is option (c) drive."
and the correct sequence is "he stopped in the middle 33. (b) As we can see that S is the beginning of the sentence
of his speech to take a glass of water." but there are two options starting with S. Now, R
EBD_7367
88
B- Ordering of Words in a Sentence

followed by Q does not make much sense 'the escapers and the sequence is "When I look back on my life
through which'. Thus, the right answer is option (b) which has been eventful I find it hard to believe that is
and the correct sequence is "Self-taught experts an illusion despite what cynics say." The correct
prepared maps of the country through which the sequence is QPSR.
escapers expected to go". 44. (b) As we can see that Q is the beginning of the sentence
34. (c) As we can see that R is the beginning of the sentence but there are two options starting with it. Now, R does
but there are two options starting with R. Now, S does not make a sensible ending. Thus the correct answer
not make a sensible ending. Thus, the right answer is is (b), "Guards often use metal probing rods which
option (c) and the correct sequence is "Some men tried they push through the ground to search for cavities".
The correct sequence is QRSP.
bribing their guards but they were rarely successful."
45. (c) As we can see that P is the beginning of the sentence
35. (b) As we can see that R is the beginning of the sentence and there is one option starting with it. Thus, the
but there are two options starting with R. Now, P answer I realized more than ever how cut off we were
followed by Q does not make much sense 'at the party from people and how we lived and worked and agitated
I had met'. Thus, the right answer is option (b) and the in a little world apart from them." The correct sequence
correct sequence is "She introduced me to a man I had is PRQS.
met at the party the previous night." 46. (b) As we can see that Q is the beginning of the sentence
36. (a) As we can see that S is the beginning of the sentence and there is only one option starting with it. Thus the
but there are two options starting with it. Now, P answer is (b) and the sequence is "Medical practice
followed by Q does not make much sense 'is his brain has changed so radically in recent years that you can
the most important'. Thus, the right answer is (a) and no longer find a good general practitioner you only
the correct sequence is "Of all things that distinguish get specialists in a variety of fields." The correct
man from the rest of the animal kingdom the most sequence is QPSR.
important is his brain." 47. (b) As we can see that S is the beginning of the sentence
37. (d) As we can see that P is the beginning of the sentence but there are two options starting with it. Now Q
but there are two options starting with P. Now, R does followed by S does not make much sense 'goes on
not make a sensible ending. Thus, the right answer is well will be a great success'. Thus the sequence
option (d) and the correct sequence is "they forget becomes "If everything goes on well we are sure the
that the conditions of welfare have so altered that there closing function will be a great success." The correct
is not much difference today between defeat and sequence is SRQP.
victory." 48. (b) As we can see that R is the beginning of the sentence
38. (d) As we can see that Q is the beginning of the sentence but there are two options starting with it. Now Q
but there are two options starting with Q. Now, S after followed by P does not make much sense, 'With an
R does not make much sense 'twice the door'. Thus, idea to reach the deprived child' by the United Nations'.
the correct answer is (d) and the sequence is "He Thus the correct answer is (b) and the sequence "The
found the house without any difficulty and knocked year 1979 has been declared as the International Year
at the door twice." The correct sequence is QPRS of the Child by the United Nations with an idea to
39. (c) As we can see that R is the beginning of the sentence reach the deprived child." The right sequence is RSQP.
but there are two options starting with R. Now, Q 49. (b) As we can see that Q is the beginning but there are
followed by P does not make much sense, 'should three options starting with it and S makes the most
display round about a huge building or palace'. Thus sensible ending. Thus the correct answer is (b) and
the correct answer is (c) and the sequence becomes the correct sequence is "The gardener a short fellow
"A good garden laid out on a magnificent scale round with a little brown moustache and sharp little brown
about a huge building or palace should display fresh eyes tiptoed into the room". The right structure is
beauties every month". The correct sequence is RQPS. QPRS.
40. (b) As we can see that S is the beginning of the sentence 50. (d) As we can see that Q is the beginning of the sentence
but there are two options starting with it. Now, R and there is only one option starting with it. So, the
followed by P does not make much sense. Thus, the answer is (d) and the proper sequence becomes "If
answer is (b), with the sequence "When a spider has a you have something interesting to write about express
meal, it eats enough to last for many months until it it clearly simply and with a human touch". The right
has its next meal." The correct sequence is SRQP. sequence is QPRS.
41. (d) As we can see that Q is the beginning of the sentence 51. (a) As we can see that Q is the beginning of the sentence
and there is only one option starting with Q. Thus the and there is only one option starting with it. So, the
answer is (d). QSPR and the sequence, "Mother tongue answer is (a) and the proper sequence becomes "One
is as natural as mother's milk for the development of cloudless morning the pilots flew off together in a close
man's mind." The right sequence is QSPR. formation towards the valley of the farmer's house."
42. (b) As we can see that P is the beginning of the sentence The correct sequence is QSPR.
but there are three options starting with P. Also, S 52. (c) As we can see that P is the beginning of the sentence
makes the most sensible ending. Thus the correct but there are two options starting with it. Now, P
answer is (b), "The year that has just ended has proved followed by S does not make much sense 'to the end
to be disastrous for my uncle's family." The correct of his mad little journey staring fixedly into its lowered
sequence is PQRS. face'. Thus the correct answer is (c) and the proper
43. (d) As we can see that Q is the beginning of the sentence sequence is "When he had ridden to the end of his
but there are two options starting with it. Now, S mad little journey he climbed down and stood in front
followed by P does not make sense, 'that it is an illusion of his rocking horse staring fixedly into its lowered
I find it hard to believe'. So the correct answer is (d) face". Thus the sequence is PRQS.
Ordering of Words in a Sentence B-89
53. (c) As we can see that Q is the beginning of the sentence to-day is so fragmented that people have lost their
but there are two options starting with Q. Now Q frankness and the art of conversation seems to be
followed R does not make much sense 'would have disappearing fast."
been over on time with his long financial report'. Thus 63. (a) As we can see that S is the beginning of the sentence
the answer is (c) and the proper sequence is "The but there are two options starting with it. Now, Q
meeting would have been over on time if Mr Rai did followed by S does not make much sense 'among the
not hold us up with his long financial report". The most civilized nations by a great many people'. Thus,
right sequence is QPSR. the answer is (a) and the sequence is "Even today
54. (b) As we can see that R is the beginning of the sentence among the most civilized nations many superstitions
but there are two options starting with it. Now, P exist and are believed in by a great many people."
followed by Q does not make much sense, 'had finally 64. (c) As we can see that R is the beginning of the sentence
paid off on drawing after drawing'. Thus the correct and there is only one option starting with R. Thus, the
answer is (b) and the proper sequence is "The long correct sequence is "The eyes of seeing people soon
grueling hours working tirelessly on drawing after become accustomed to the routine of their
drawing painting after painting had finally paid off". surroundings and they actually see only the startling
The right sequence is RQSP. and spectacular."
55. (b) As we can see that Q is the beginning of the sentence 65. (c) As we can see that Q is the beginning of the sentence
but there are two options starting with it. Now, P and there is only one option starting with it. So the
followed by S does not make much sense 'is very proper sequence becomes "Though he was a man who
important before this Monday'. Thus, the answer is held all life sacred and loved all forms of life he did not
(b) and the correct sequence is "For us to complete hesitate to kill when there was no choice."
this work before Monday is very important and it 66. (d) As we can see that s is the beginning of the sentence
should not be delayed". The right sequence is QPSR. and there is only option starting with it. So the proper
56. (b) As we can see that Q is the beginning of the sentence sequence becomes "For a moment I forget that I am
but there are two options starting with Q. Now, R terribly old and that it is a very long time ago since I
followed by P does not make much sense 'in the was a child."
Assembly the party realized that'. Thus the correct 67. (b) As we can see that r is the beginning of the sentence
answer is (b) and the proper sequence is "When the and there is only option starting with it. Thus, the
elections were over the party realized that it had lost proper sequence becomes "Bringing the light from
its majority in the assembly". The right sequence is behind a curtain he held it in such a manner that it fell
QPSR. slantwise on her face".
57. (a) As we can see that S makes the most sensible 68. (d) As we can see that P is the beginning of the sentence
beginning and there is only one option starting with but there are two options starting with it. Now, S
it. So the answer is (a) and the proper sequence is "If followed by R does not make much sense 'throughout
you want peace you must arm yourself so that the his educational career and medals in competitions'.
enemy may think twice before starting a war". The Thus the answer is (d) and the sequence "The man
right sequence is SPRQ. had been a sportsman throughout his educational
58. (b) As we can see that Q is the beginning of the sentence career and received many prizes and medals in
and there is only one option starting with Q. Thus, the competitions."
proper sequence is "The enquiry committee found no 69. (b) As we can see that Q is the beginning of the sentence
conclusive evidence of a thermal shock to the airplane." and there is only one option starting with it. So the
59. (c) As we can see that R is the beginning of the sentence sequence becomes "He was passing by a hut when he
but there are two options starting with it. Now, Q heard the cries of a child and went in."
followed by P does not make much sense 'with slave 70. (a) As we can see that Q is the beginning of the sentence
like docility of her tribe'. Thus, the answer is (c) and but there are two options starting with it. Now, R
the proper sequence is "For thirty years his wife had followed by P does not make sense 'Only if we worked
submitted to his persecution with slave-like docility hard that we would pass the test' does not make much
of her tribe that is the badge." sense. So the answer is (a) and the sequence is "The
60. (b) As we can see that P is the beginning of the sentence teacher warned us that we would pass the test only if
but there are two options starting with P. Now, s we worked hard for at least eight hours a day."
followed by R does not make much sense 'to the 71. (c) As we can see that Q is the beginning of the sentence
editorial page dealing with Pandit Nehru'. Thus the and there is only one option starting with it. Thus the
answer is (b) and the sequence "I came finally to the correct sequence is "There is no agreement among
editorial page and saw an article dealing with Pandit the great powers on a treaty to ban nuclear weapons."
Nehru." 72. (b) As we can see that Q is the beginning of the sentence
61. (b) As we can see that Q is the beginning of the sentence and there is only option starting with it. Thus, the
and there is only option starting with Q. So, the correct sequence is "According to one theory all land
sequence becomes "Although many institutions have animals including man have descended from organisms
tried to organise remedial programmes for the weaker of the sea."
students these can have only peripheral effect." 73. (c) As we can see that Q is the beginning of the sentence
62. (a) As we can see that Q is the beginning of the sentence but there are two options starting with it. Now, R
but there are two options starting with Q. Now, s followed by Q does not make sense 'sentenced the
followed by Q does not make much sense 'is so prisoner to life imprisonment by the police'. Thus, the
fragmented seems to be disappearing fast'. Thus, the answer is (c) and the sequence is "The judge sentenced
answer is (a) and the proper sequence is "Our society
EBD_7367
90
B- Ordering of Words in a Sentence
the prisoner to life imprisonment when he was don't make much sense 'correct that person who cannot
presented in court by the police on a charge of murder." to the satisfaction of the antagonist'.
74. (b) As we can see that P is the beginning of the sentence So the correct answer is option (a) with the proper
but there are three options starting with it. Now, Q sequencing "The person who can state his antagonist's
does not make a sensible ending. Thus the correct point of view to the satisfaction of the antagonist is
answer is (b) and the proper sequence "The problems more likely to be correct than the person who cannot"
of working wives are different from those of So the correct sequence is RSQP that is answer (a)
housewives because they have to look after their 82. (b) As we can see that S is the beginning of the sentence,
family as well as their jobs." so we just have one option, i.e. (b). So the proper way
75. (d) As we can see that P is the beginning of the sentence of writing the sentence is "The time has come when
but there are three options starting with it. Also, Q man must no longer think that the ideal of peace is a
makes the most sensible ending. Thus, the correct distant ideal or one which can be postponed for future
answer is (d) and the sequence "As the marketing, generation to come"
and distribution of drugs generates huge illegal profits So the correct sequence is SQRP which is answer (b)
it has become increasingly problematic to eradicate 83. (b) As we can see that Q is the beginning of the sentence,
the menace of drug addiction." we have two options (b) and (c). Now, S followed by P
76. (a) As we can see that P is the starting of sentence, so we doesn't make much sense, 'some ten miles away from
can eliminate option (b) and (c). This narrows down
the railway station at his cottage among the Yorkshire
our possibilities to option (a) and option (d).
fells'. So we can eliminate option (c).
Now we can see in option (d) the sequencing of R and
Q i.e.,'about how he faced up to this problem because Thus, the proper way of writing the sentence is "I had
he enjoyed the confidence' doesn't make sense. been staying with a friend of mine a delightfully lazy
Therefore the answer is (a) as the complete sequence fellow at his cottage among the Yorkshire fells some
becomes "The teacher had to be specially careful about ten miles away from the railway station"
how he faced up to this problem because he enjoyed So the correct sequence is QRPS that is answer (b)
the confidence of all the boys". 84. (c) As we can see that S is the beginning of the sentence,
So the correct sequence is PRQS that is answer (a) so we just have one option (c).
77. (a) As we can see that Q is beginning of the sentence, so The proper way of writing the sentence is "All the evil
we have three options, (a), (b) and (d). Now, 'Hollywood in this world is brought about by persons who are
in America by people' doesn't make sense, so we rule always up and doing but do not know when they ought
out option (b). Also, 'Hollywood in America all around to be up nor what they ought to be doing"
the globe' doesn't not make sense. So the correct sequence should be SQPR that is answer (c)
So, the correct option is (a) and the sequencing is 85. (c) As we can see that Q is the beginning of the sentence,
"Movie made in Hollywood in America are seen at the so we have two options (b) and (c). Now, S followed
same time by the people all around the globe" by P does not make much sense, 'the laws then they
So the correct sequence is QSRP that is answer (a) will never fight of mankind and agree to obey'. So we
78. (a) As we can see that P is the beginning of the sentence, can eliminate option (b).
but all the options start from P only. Now, Q ends with The proper way of writing the sentence is "work
'but also' and S ends with 'does not merely rest on' together for the common good of mankind and agree
which gives the hint of sentence being continued. So to obey the laws, then they will never fight with each
we can eliminate three options i.e. (b), (c) and (d). other and there will be no more war"
Hence, the correct answer if (a) with the proper So the correct sequence should be QPSR that is answer (c)
sequencing "The foundations of prosperity of a state 86. (b) As we can see that P as the beginning of the sentence
does not merely rest on primary health and education does not make sense thus eliminating the three options
but also involves the creation of job opportunities" (a), (c) and (d).
So the correct sequence is PSQR that is answer (a) The proper way of writing the sentence is "They knew
79. (c) As we can see R is the beginning of the sentence, so him to be a hard taskmaster and were surprised when
we narrow down to 2 options i.e. (c) and (d). Now, Q he permitted them to leave work early that day"
and P together doesn't make sense, 'sooner or later So the correct sequence should be RSPQ that is answer (b)
and will be happy'. So the right answer is option (c),
87. (a) As we can see that S makes most sense as beginning
with the proper sequencing "I am sure a day will come
sooner or later when all will be equal and will be happy. of the sentence. So the only option is (a).
So the correct sequence is RQSP that is answer (c) Thus, the proper way of writing the sentence is "He
80. (c) As we can see that Q is the beginning of the sentence, was known to be honest and kind man and therefore
so we have two options (a) and (c). Now, R followed his arrest on charges of corruption surprised everyone
by P does not make much sense, 'to do his work without who knew him"
anybody forcing him/her'. So the correct sequence should be SPRQ that is answer (a)
Thus the right answer is option (c), with the 88. (c) As we can see that Q as the beginning of the sentence
sequencing, "it should be the pride and honour of makes most sense, thus the only option is (c).
every citizen in India without anybody forcing him/ The proper way of writing the sentence is "if suddenly
her to do his work" you throw a brick at me and my hand goes up to protect
So the correct sequence is QSPR that is answer (c) myself it is an automatic instinctive action and not a
81. (a) Now we can see that R is beginning of the sentence, result of deliberate thought"
so we have two options (a) and (b). but P and Q together So the correct sequence should be QSRP that is answer (c)
Ordering of Words in a Sentence B-91
89. (b) As we can see that R is the beginning of the sentence, 105. (a) If you buy the economy pack which is quite cheap,
so we have two options, (b) and (c). Also, P as the last the manufacturers, wanting to promote the sales and
option does not make sense, with the sentence ending to ensure clearance of stocks have devised a number
like 'or just one of you'. Thus eliminating option (c). of schemes.
The proper way of writing the sentence is "If I have 106. (c) Happiness does not consist in rank or position, it is a
made all of you just one of you repent of this career state of mind available to those who are contended.
and seek a decent work I will not have breathed in vain 107. (b) A person who cannot understand another's view point
today"
is to that extent limited in mind and culture.
So the correct sequence should be RPSQ that is answer (b)
90. (b) As we can see that Q makes the most sensible 108. (a) The whole valley was pleasantly green with crops and
beginning of the sentence and there is only option was well planted with date-palms.
starting with it. 109. (b) The extent of women's participation in various nation-
91. (d) The correct sequence is SPQR. building activities is an important indicator of social
92. (b) As we can see that Q makes the most sensible progress.
beginning of the sentence and there is only option 110. (b) We intend to shift to Bombay with our family which
starting with it. consists of three persons.
93. (d) As we can see that Q makes the most sensible 111. (a) The wings of some birds are so small that they are
beginning of the sentence and there are two options useless for flying.
starting with Q. Now P followed by R does not make 112. (c) The person who lives next door sings loudly at night.
much sense 'not only of the smokers themselves, but 113. (a) We have the ability to provide both people and
also of their companion'. Hence the answer is (d). ecosystems with the water they need.
94. (c) As we can see that Q makes the most sensible 114. (b) People usually learn more from doing something
beginning of the sentence and there are two options themselves than by watching someone else or reading
starting with Q. Now R followed by S does not make
about it.
much sense 'at the door when someone knocked'.
Hence the answer is (c). 115. (a) The prevention of disease was by far the most urgent
95. (d) The correct sequence is SPOR. problem facing the authorities after the earthquake.
96. (c) As we can see that R makes the most sensible 116. (b) Although he watered the plants regularly and put
beginning of the sentence and there are two options manure in them, they did not grow well.
starting with R. Now S followed by Q does not make 117. (d) The governments of many countries have been
sense ‘must be medically examined by the butchers’. spending a lot of money on the propagation of the
Hence the answer is (c). idea of family planning.
97. (c) As we can see that P makes the most sensible 118. (c) I sat down to write an article this morning but found I
beginning of the sentence and there are two options could make no progress.
starting with P. Now Q followed by R does not make 119. (b) Where would I have been today if it had not been for
much sense. Hence the answer is (c). your timely help five years ago?
98. (c) As we can see that R makes the most sensible 120. (a) It was almost midnight when I arrived home but I found
beginning of the sentence and there is only option them both sitting at the table waiting for me.
starting with it. 121. (d) Do or die was the call that Gandiji gave to all freedom
99. (b) As we can see that P makes the most sensible fighters when he asked the British to quit India.
beginning of the sentence. But there are three options
starting with P. Now Q in the end does not make sense, 122. (c) The scientist who discovered the ancient cure that
hence we can eliminate option (c). everyone is talking about refused to talk to the press.
Now between option (a) and (b); we can see that Q 123. (a) His grandmother was so pious that she would visit
followed by R does not make much sense 'go home the temple on all auspicious days without fail.
those students'. Hence the answer is option (b). 124. (c) There are parents in our country who consider the
100. (b) As we can see that R makes the most sensible education of their daughters a needless luxury.
beginning of the sentence and there are two options 125. (b) Based firmly on current lexical and learning theory
starting with R. Now R followed by P does not make "English vocabulary in use" aims not only to present
much sense 'were seriously perturbed in the near and to explain words but also to show students how
future'. Hence the answer is (b). to use them and to help them work out the rules
101. (c) As we can see that P makes the most sensible themselves
beginning of the sentence and there are two options 126. (d) The budget is being awaited with high expectations of
starting with P. Now, R followed by S does not make tax relief by all types of taxpayers.
much sense, 'in the life of an ordinary day. Hence the 127. (b) I walked on as fast as I could until I reached the road
answer is option (c). which lead to my house.
102. (d) As we can see that P makes the most sensible
beginning of the sentence and there is only option 128. (a) A very old man with a wrinkled face and a long beard
starting with it. married a pretty girl of sixteen.
103. (b) As we can see that S makes the most sensible 129 (c) Shyam took from the window a photograph which was
beginning of the sentence and there is only option overexposed.
starting with it. 130. (a) The correct order is ---- Now that the office is closed
104. (a) As we can see that Q makes the most sensible all the typists have gone.
beginning of the sentence and there is only option 131. (d) The correct sequence should be---- He gave to the
starting with it. child a doll whose head was broken.
EBD_7367
92
B- Ordering of Words in a Sentence

132. (b) The correct sequence should be---- My father paid 175. (c) The correct sequence is RPSQ.
John's fees when he was at school last year. 176. (a) The correct sequence is RSQP.
133. (c) The correct sequence should be--- The prime minister 177. (b) The correct sequence is PSQR.
clearly suspects his party to have little chance of 178. (d) The correct sequence is SRQP.
winning in the next election. 179. (c) The correct sequence is SQRP.
134. (d) The correct sequence should be---All the students 180. (b) The correct sequence is RQSP.
interviewed on television responded eagerly to the 181. (d) The correct option is (d). 'It was true that' should be
question and affirmed positively that the political followed by a subject ' the pet dog'. Using the phrase
affiliation of the student unions was undesirable. 'we once had' right after, requires the next phrase to be
135. (c) The correct sequence should be----Although the 'a' pet dog, not given in the question or else the phrase
motion received general support from the house it was 'who would never sleep' to follow. With PR as the first
not carried until it had been considerably amended. two parts, this reduces to only option (d).
136. (a) The correct sequence should be----The doctor told 182. (b) The major dilemma can be if recently should be
the nurse to give an injection to the patient after four followed by part P or Q. However, starting the sentence
hours. with 'containing.....' breaks the flow of the sentence
137. (d) The correct sequence should be----The judge asked and makes it sound absurd. Giving the subject 'a book'
the accused why he had lied to the court in spite of his would give the flow smoothness and right sequence.
oath. Hence, option (b).
138. (a) The correct sequence should be---- His father said, 183. (c) The sentence begins like 'as the situation has changed',
now get up and stop wasting your time on trivial things. it's important to convey how or since when has the
139. (c) The correct sequence should be---- The stranger's situation has changed. This is explained well through
movements aroused suspicion and the police arrested part P describing the last time they discussed the matter.
him. Further, the adverbial phrase 'without losing time'
140. (d) The correct sequence should be---- The future outlook describes the action of contacting hence must follow
of this noble animal is gloomy. phrase Q and not precede it. Hence, option (c).
141. (b) The correct sequence should be---- The food served 184. (d) When we describe the noun speeches, we must do it
at the party was delicious but not wholesome. with 'delivering'. This helps us quickly find the first
142. (c) The correct sequence should be---- In one of the Asian clue. There are two actions in the sentence, the speech
countries men as well as women wear salwars. delivery being followed by prize giving. Thus, this
143. (d) The correct sequence should be---- You won't believe sets the first 3 parts in sequence RQP. Thus, option
me, but whenever I went to see him, he was out. (d).
144. (c) The correct sequence should be---- As the car came 185. (c) The biggest clue for this sentence is not in much logic
near the door the waiters stood up to greet him. but simple grammar rules. The last word of part P ends
145. (a) The correct sequence should be-- At last the moment with agreed which is (out of all options especially)
she had been waiting for had come. always followed by the word 'to' given only in the part
146. (d) The correct sequence is RPQS. R. Thus, PR has to be together confirming option (c)
147. (b) The correct sequence is PSQR. to be the answer. Also, the sequence sounds most
148. (d) The correct sequence is QPSR. perfect fit.
149. (c) The correct sequence is SRPQ. 186. (d) The phrase 'bound together' requires a description of
150. (c) The correct sequence is QPSR. how are the religions bound. This is explained through
151. (c) The correct sequence isRQPS. part Q 'in a holy partnership'. Also, concepts like peace
152. (d) The correct sequence is SPQR. are to be followed by justice and freedom mandating
153. (d) The correct sequence is SPRQ. PR to be together.
154. (a) The correct sequence is SQPR. 187. (b) The phrase natives of Caribbean must be followed by
155. (d) The correct sequence is SQPR. a verb making part P the only choice. Also, 'regarding
156. (c) The correct sequence is RQSP. as' shall answer question 'as what'. This is answered
157. (d) The correct sequence is QPRS. by the phrase 'magic tree'. This is further explained by
158. (d) The correct sequence is RSQP. the reason 'because of its ability to keep them healthy'.
159. (b) The correct sequence is PRQS. 188. (c) What the woman wants can easily be found in part S
160. (d) The correct sequence is PSRQ. 'her rightful position'. The description of this position
161. (b) The correct sequence is SRPQ. is definitely in 'an equal partner'. Thus, SQ go together.
162. (a) The correct sequence is QSPR. This is to be followed by where she wants the position
163. (b) The correct sequence is SPQR. of being equal explained in part P.
164. (d) The correct sequence is PSQR. 189. (c) This sentence must have P before S as the young lady
165. (c) The correct sequence is SPQR. needs to be introduced before the use of pronoun
166. (d) The correct sequence is RPSQ. 'she'. This reduces our options to only C and D. Further,
167. (a) The correct sequence is QRPS. the first part this letter most conveniently can be said
168. (a) The correct sequence is SRPQ. to be followed by 'written by' and not a part like
169. (b) The correct sequence is PRQS. 'wherein...'.
170. (a) The correct sequence is PRQS. 190. (c) 'It is' should be followed by the subject an accepted
171. (b) The correct sequence is QPSR custom which is further defined in the consequent
172. (a) The correct sequence is RPQS. parts. With 2 options remaining, if part Q precedes
173. (a) The correct sequence is PRQS. part S it will break the flow of the sentence and make it
174. (d) The correct sequence is PRQS. sound absurd.
Ordering of Words in a Sentence B-93
191. (d) The normal sequence PQRS might seem grammatically how? This is given in part S 'with untold misery for all'.
fine if we read it once. But rereading will help us find Thus, as Q must be followed by only S, option (b)
that the sense of the sentence is wrong or inappropriate. seems to be answer.
The content of what he said is, actually, acceptable 204. (c) The first phrase ends with 'started' that must be
but the manner was objected. necessarily followed by a verb. This is only plausible
192. (d) The correct sequence is PSQR. in part Q 'singing....'. Also, since one artist cannot
193. (c) 'he asked me if I' is a very standard form of sentence as sing in chorus, it is the audience 'joining in chorus
one asking the other must be further described as to (implying together) ' mandating S to precede R. Hence,
what was asked. If I must follow either 'would or could'. option (c).
This is available in option P. 205. (b) The first phrase must be followed by either part P or R
194. (a) The fire needs to be described first as where or when. as 'we know' must be followed by a conjunction 'that'.
This is answered by last part S 'in the godown'. Further, But since, it is common sense that they can't know he
was controlled by volunteers seems to be the most passed unless they know he sat for examination, this
appropriate and grammatically correct pairing to follow. makes it clear that part R should precede part P. As
This gets us to SRQ. Hence, option (a). only one option has R as the beginner of the sentence,
195. (c) The word educationists must be followed by either option (b) is the correct answer.
Part P or Q. However, it is common sense that the 206. (b) This question too applies the same logic as 'he knew'
educationists need not be saved from exposure rather must be followed by 'that' which is part S. This leaves
their belief is what is being discussed. Thus, the us with options A and B. Part R 'and thought......' must
remaining pairing that 'young children should not be be only after a part that already speaks of an action
exposed' makes grammatical as well as logical sense. which is part P 'sat under the tree'. Thus, P precedes R.
Hence, option (c). Also the adverbial phrase in Q must be placed in the
196. (d) The sentence should begin with part P or R being the end. Hence, option (b).
subjects of sentence. On further reading, it is quite 207. (d) 'He wanted' must be followed by a phrase that starts
obvious that the phrase end 'talked about' must be with 'to' under normal grammar rules. This is available
followed by the word 'topics' given in part S. Hence, both in part P as well as part R. However, using P
QS must go together. We are left with option (d) only. would leave R as a misfit not finding a place anywhere
197. (b) The subject of the sentence 'the dog' shall start it which else in the sentence. Thus, 'he wanted to buy a scooter'
then must be followed by a verb that is 'wagged his is the beginning. Further, 'for his son' fits perfectly
tail'. 'With customary fondness' is an adverbial phrase after he talked about buying a scooter. R is followed
which is most appropriate when placed at the end of by S. Q precedes P to explain for what he travelled 20
sentence. Hence, option (b). miles; thus, RSQP or option (d).
198. (c) It seems quite obvious that the sentence starts with a 208. (b) The correct sequence is QRSP.
subject 'I was driving....' and also because pronoun 209. (a) 'Just as the goodness of movies' must be followed by a
'me' in part P can only be used after the use of I. R verb or an auxiliary verb. 'does not consist in being' is
precedes P. The driver's action need to be described in an obvious choice. This has to be further explained in
the next part, that is, part Q. This leads to RPQ. Hence, not being "like" something making part P the next part.
option (c). This is a comparative sentence and the second part
199. (a) 'When john saw' needs to be followed by an object must also follow the same pattern. Hence, option (a).
which is a truck. This has to be followed by the 210. (c) The correct sequence is SQRP.
describing what happened after seeing the truck, 211. (c) The correct sequence is RQPS.
bringing part P as the next part. Hence, option (a). 212. (b) The correct sequence is QPSR.
200. (b) The first part can either be R or S. However, if we 213. (c) The correct sequence is SQPR.
choose 'buying presents', the other part 'to go 214. (d) The correct sequence is RSQP.
shopping' will be unfit. So, part S begins the sentence 215. (a) The correct sequence is PRSQ.
with describing the time as the next part Q. Also 'to 216. (a) The correct sequence is PRQS.
buy presents' is naturally followed by 'for their friends'. 217. (b) The correct sequence is PRQS.
Hence, option (b). 218. (d) The correct sequence is PRSQ.
201. (a) Demonstrative pronouns like 'this is the book' mostly 219. (b) The correct sequence is SRPQ.
begin the sentence. To further describe it, use of that 220. (d) The correct sequence is QPSR.
is next. About is an adverb to be used at the end of 221. (a) The correct sequence isQPSR.
sentence. PRSQ or option (a). 222. (b) The correct sequence is SRPQ.
202. (c) Since passing of the exam happened in the past (use 223. (a) The correct sequence is SQPR.
of had), it can be conveniently followed by the phrase 224. (a) The correct sequence is PRQS.
two years ago. The word produce is most 225. (d) The correct sequence is PRQS.
appropriately used along with certificate, hence Q 226. (d) The correct sequence is QPRS.
precedes S. RP followed by QS. Option (c). 227. (c) The correct sequence is PRSQ.
203. (b) When we use words like 'however' it is mostly to bring 228. (c) The correct sequence is SQRP.
contradiction. Thus, use of ends must be along with a 229. (a) The correct sequence is RSPQ.
word of opposite meaning. This can be clearly seen in 230. (d) The correct sequence is SPQR.
part R due to use of 'acts'. This leaves us with only 231. (d) The correct sequence is RSPQ.
two options B and D as part R begins the sentence in 232. (b) The correct sequence is QPRS.
only these two options. Also, part Q 'are bound to 233. (b) The correct sequence is PRSQ.
end up' needs to be followed by an explanation of 234. (d) The correct sequence is RQPS.
EBD_7367
94
B- Ordering of Words in a Sentence
235. (c) The correct sequence is QSRP. sequence of sentence is " The bird catcher knew all
236. (a) The correct sequence is PSQR. the birds of the forest and was accustomed to
237. (c) The correct sequence is RSPQ. capturing the winged creatures by the hundred by
238. (b) The correct sequence is RQPS. means of snares". Therefore the correct option is (b).
239. (c) We can see that Q is the most suitable beginning of 250. (a) The correct sequence is SPRQ.
the sentence and the other options do not start with it. 251. (a) The sentence starts with "A gang of robbers" and if it
Therefore the correct sequence will be, " They refused is followed by Q and the sentence ends with P, then
to divulge the venues of these raids saying that it the sentence makes complete sense. Thus the correct
would affect the investigation process. option is (a). And the correct sequence of sentence is
240. (b) We can see that R is the beginning of the sentence, "A gang of robbers entered the village at night and
but there are two options starting with it. Now, S stole the property of the villagers while they were fast
followed by Q and P as the ending makes perfect sense. asleep".
Therefore, the correct sequence is "That rich man goes 252. (d) The sentence starts with "The opposition members"
on buying things that he already has". and if it is followed by S and has P in the end then the
241. (c) As we can see that P is the beginning of the sentence sentence makes complete sense and forms the proper
but two options are starting with it. Now, S followed sequence. The correct option in that case is (d). The
by R and Q as the ending make perfect sense. Therefore, proper sentence will be "The opposition members
the correct sequence is "The police commissioner walked out of the parliament to protest against the
rushed the police force to control the crowd". ruling of the speaker".
242. (d) We can see that P is the beginning of the sentence but 253. (c) As the sentence begins with "When in that case S is
two sentence are starting with it. Now, R followed by the most sensible part to follow it but there are two
S and Q as the ending makes a sensible sentence. options starting with S. Now, if S is followed by Q and
Therefore, the correct sequence is "My brother is the sentence ends with R then the sentence make
going to Chennai tommorow to attend his friend's complete sense. In that case option (c) is correct.
wedding". Therefore, the proper sequence will be "When a little
243. (b) As we can seen Q is the correct start to the sentence girl was about to be run over by a speeding car a boy
and no other option starts with it. Therefore, the correct saved her at the risk of his life".
sequence is "He gave orders to his men to catch the 254. (d) The correct sequence is QRPS
thief quickly". 255. (c) The correct sequence is QPRS.
256. (c) The correct sequence is RSQP.
244. (a) As we can see that Q is most appropriate beginning of
257. (c) The correct sequence is RQSP.
the sentence and no other option starts with it.
258. (c) The correct sequence is SPQR.
Therefore, the proper sequence is "If I were to give a
259. (b) The correct sequence is SQRP.
definition I would begin like this".
260. (c) The correct sequence is RPQS.
245. (a) As we can see that R is the right beginning for the 261. (d) The correct sequence is QRPS.
sentence and there are no option starting with it. 262. (c) The correct sequence is RQPS.
Therefore, the proper sequence is "Men of conscience 263. (a) The correct sequence is SQRP.
who take pride in doing their job well whatever its 264. (b) The correct sequence is RSPQ.
nature deserve all honour in society". 265. (b) The sentence should begin with 'P' followed by
246. (a) We can see that P is the beginning of the sentence fragment 'R' as it contains the erb and object of the
and there are two options starting with it. Now, P sentence. Fragment 'Q' should precede 'S' since 'Q' is
followed by S makes more sense than P followed by R an adverb of manner. So, sequence PRQS makes a
which is "While some live many do not have enough". coherent sentence.
Therefore option (a) is the correct answer and 266. (d) The sentence cannot begin with fragments 'Q', 'R' or 'S'
sequence becomes " While some live in luxury many as they do not make a sense. 'P' should be first sentence
do not have enough to eat and drink". followed by 'S', as they together complete the
247. (d) R is the correct part of the sentence to follow the given expression. Similarly, the first word of fragment 'Q' -
beginning and there are two options starting with R. than' makes it the last part of the sentence. So,
Now, R followed by P and ending with Q makes the sequence PSRQ is the right answer.
correct sequence. Therefore, the sequence of sentence 267. (d) Fragment 'Q' should be first part of the sentence as it
will be "I believe then and I believe even now that now contains a subject. It should be followed by fragments
matter the amount of work one has one should always 'P' and 'R' respectively as presence of 'not only' in 'P' is
find some time for exercise". complemented by 'but also' in 'R'. Fragment 'S' should
248. (c) As we can see that P is the apt beginning of the be concluding part. So, sequence QPRS makes a
sentence. Now, if P is followed by S and ends with Q it meaningful sentence.
makes perfect sequence. Therefore, the correct 268. (d) Fragments 'P' and 'Q' together make a sense. They
sequence of the sentence is " I wonder why I always should be followed by fragments 'R' an d 'S'
have trouble with my scooter whenever I decide to go respectively as these further extend the sentence by
to the cinema". explaing the outcome of a rabid dog bite. So, PQRS is
the right sequence.
249. (b) As we can see that Q is the correct part to follow the
269. (c) Fragments 'P', 'Q' and 'R' are in proper sequence and
given beginning of the sentence. If Q is followed by S
make a meaningful sentence. Fragment 'S' talk about
then the sentence form proper sequence. The correct
Ordering of Words in a Sentence B-95
'the traveller' so, it should be placed before the word fragment 'P'. Fragment 'Q' should follow 'P' as it talks
or immediately after it. So, SPQR is the right sequence. about drawback of being an workoholic. Fragment 'R'
270. (b) Fragments 'P' and 'S' make a sense together. Next, should be last in the sequence as it is an adverb.
fragments 'R' and 'Q' together explain why the house Therefore, SPQR makes a coherent and meaningful
was gloomy. So, PSRQ is the correct sequence. sentence.
271. (d) Fragment 'P' should be first part of the sentence as it 282. (c) Fragments 'P' and 'Q' should be in sequence. P contains
contains subject of the sentence. Fragments 'S', 'R' subject 'the school masters and the professors' and
and 'Q' should follow fragment 'P' as they describe verb phrase 'tend to believe' - Fragment 'Q' contains
how tall the person is and how does he walk? Therefore, the object 'innate intelligence'. Next, fragment 'S' further
sequence PSRQ makes a meaningful sentence. describes 'innate intelligence' mentioned in fragment
272. (c) Fragments S, R, P in same order make a sense. They Q. Fragment 'R' is last in the sequence. So, sequence
should be followed by fragment Q as it contains 'its' PQSR is the correct sequence.
which refers to expressions contained in R and P - a 283. (a) Fragment 'R' describes 'People' mentioned in the
life of rules and regulations. begining of sentence. So it should be first in the
273. (c) "Looked at me for a moment" is a meaningful sequence. Next, fragments 'P', 'Q' and 'S' should follow
expression. So fragments 'Q' and 'P' should be together fragment 'R' to make the sentence coherent and
in the same order. These should be followed by 'S' and meaningful. Therefore. RPQS is the correct sequence.
'R' respectively as fragment 'S' contains and action 284. (c) Fragment 'P' should be first in the sequence as it gives
and 'R' an outcome of that action. Therefore, QPSR is the reason why a particular year has been given a
the correct sequence. special status by an international body. Fragment 'R'
274. (d) Fragment 'S' should be first in this sequence as it tells is next in the sequence as it contains the year '1979'.
something about the lady. Next, fragments 'Q', 'R', and Fragment S should follow fragment R as it contains
'P' in the same order make a coherent and meaningful special status given to year 1979 as the International
sentence. So, SQRP is the correct answer. year of the child. Fragment 'Q' is last in the sequence.
275. (a) Fragments 'P' and 'Q' are in sequence and make a sense. Therefore, sequence PRSQ is the correct answer.
But fragment 'S' should precede 'R' as together they 285. (b) My unmarried aunt who stays with us is creating a lot
explain how animals measure passing of the seasons. of problems for us and there is a misunder-standing
So, the correct sequence is PQSR. among family members because of her interfering
276. (c) Fragment 'S' should be first in the sequence as 'Many nature in our personal lives. The correct answer is
people with hearing problems' may have some QPSR.
suffering. Next, fragment 'R' and 'Q' should follow 286. (a) The correct sequence should be QRSP. No criminal
fragment 'S' as together they make sense. Fragment 'P' proceeding whatsoever shall be initiated or continued
should be last in the sequence as it is an adjective against the president or a Governor in any court during
phrase describing 'Persistent noises' mentioned in his term of office.
fragment 'R'. So, SRQP is the right sequence. 287. (d) The correct sequence should be RQPS.
277. (a) Fragment 'R' should either be first part of the sentence 288. (a) The correct sequence should be RPQS.
or last part because it indicates the time when 289. (c) The correct sequence should be RQSP.
something happened. Fragment 'P' should be next part 290. (c) The correct sequence should be RQPS.
of the sequence as it talks about a discovery. Next, 291. (c) The correct sequence should be RPQS.
fragments 'S' and 'Q' should follow fragment 'P' as they 292. (b) The correct sequence should be SQPR.
explain how and where of concept mentioned in 293. (d) The correct sequence should be RSPQ.
fragment 'P' So, RPSQ is the correct sequence. 294. (b) The correct sequence should be SPRQ.
278. (b) In the beginning, the sentence talks about 295. (a) The correct sequence should be QSPR.
transplantation of organs. So, fragment 'R' should be 296. (d) The correct sequence should be QPSR.
first in this sequence as it contains examples of organs. 297. (a) The correct sequence should be SQPR.
Fragment 'S' should follow 'R' to make the sentence 298. (d) The correct sequence should be SPQR.
meaningful Next, fragments 'P' and 'S' should come 299. (b) The correct sequence should be QPRS.
together to express a contradictory idea. Therefore, 300. (a) The correct sequence should be PRQS.
RSPQ is the correct sequence. 301. (d) The correct sequence should be PSQR.
302. (c) The correct sequence should be QSRP.
279. (b) Fragment 'P' should be first in the sequence as it
303. (a) The correct sequence should be SPRQ.
contains a subject. Fragment 'S' should follow 'P'
304. (d) The correct sequence should be SQPR.
because it further describes sports cars mentioned in
'S'. Fragments 'Q' and 'R' should follow P and S in same 305. (c) The correct sequence is SPRQ
order to make the sentence meaningful. So, PSQR is 306. (a) The correct sequence is PRSQ
the right answer. 307. (c) The correct sequence is RPSQ
280. (c) Fragment 'P' and 'S' together complete the expression 308. (d) The correct sequence is RPSQ
'Friendship has its place in life'. Next, fragments 'R' and 309. (c) The correct sequence is QRSP
'Q' together extend the sentence and make entire 310. (d) The correct sequence is SPRQ
sentence meaningful. So, PSRQ is the correct 311. (a) The correct sequence is SPRQ
sequence. 312. (d) The correct sequence is QRPS
281. (b) Fragment 'S' should be first in the sequence as it 313. (b) The correct sequence is SPRQ
describes the workoholic' mentioned in the next 314. (a) The correct sequence is SQRP
EBD_7367
B- 96 Ordering of Sentence

C HA P T E R

30 ORDERING OF SENTENCE

DIRECTIONS (Qs. 1-6) : In the following questions, each passage Q : This is evident from a comparison between prices in
consists of six sentences. The first and the sixth sentences are December 1998 and December 1999.
given in the beginning as S1 and S6. The middle four sentences in R : This indicates the inability of inflationary tendencies
each have been removed and jumbled up. These are labelled as to move upwards in 1999.
P, Q, R and S. You are required to find out the proper sequence of S : This is a unique phenomenon.
the four sentences and mark accordingly on the answer sheet. The proper sequence should be: [2007 - II]
(a) PSQR (b) QRPS
Example ‘X’ has been solved for you.
(c) PRQS (d) QSPR
X. S1 : There was a boy named jack.
4. S1 : There were two poignant farewell last week to two great
S6 : At last she turned him out of the house.
sports personalities.
P : So the mother asked him to find work.
S6 : This showed how a great player should be recognised
Q : They were very poor.
by sports authorities.
R : He lived with his mother.
P : the great West Indian fast bowler Malcolm Marshall
S : But jack refused to work.
passed away prematurely.
Which one of the following is the correct sequence ?
Q : this was Steffi Grafs farewell in New York.
(a) R Q P S (b) P Q R S
R : the other was a player who dominated her sport like no
(c) Q P R S (d) R P S Q
one else had done before.
Explanation :
S : this is an age when most sportspersons enjoy the
The correct sequence in this example is R Q P S which is
rewards and recognition of their efforts over the years.
marked by (a). Therefore, (a) is the correct answer.
The proper sequence should be : [2007 - II]
1. S1 : I first came to Poland in the autumn of 1986 (a) QSRP (b) PRSQ
S6 : was I still interested?
(c) QRSP (d) PSRQ
P : I had been keeping an eye on the notice board of the
5. S1 : The North-East of the Caliph's dominions comprised a
English Department for a prospective job number of Turkish tribes.
Q : It was my last month as a student at the University of S6 : In 1071, the Byzantine army was utterly smashed in the
York Battle of Melasgird.
R : It was at the University of Lo'dz, which I had never P : In the 10th century, these Turks grew strong and
heard of vigorous.
S : one day I spotted a little note concerning a lectureship Q : they had been converted to Islam but they held their
The proper sequence should be : [2007 - II] faith much more fiercely than the Arabs and the Persians
(a) SRQP (b) QPSR to the South.
(c) SPQR (d) QRSP R : they conquered Armenia and struck at the remnants of
2. S1 : It is perhaps coincidental that the final Booker Prize of the Byzantine power.
the 20th century should go to South African writer S : In the next century a group of Tbrkish tribes came down
S6 : the formal structures of a modern democracy have beem to Mesopotamia and made the Caliph their nominal
put in place. ruler.
P : In political terms, there has been, if not a revolution, The proper sequence should be : [2007 - II]
but a partial overturning of the old order (a) QPSR (b) PQSR
Q : apartheid, the dominant reality of South Africa's 20th (c) QPRS (d) PQRS
century, has been dismantled. 6. S1 : When there is a disaster its impact depends on how
R : It is not so much that South Africa is poised to enter close you are to the people and the place where it
the new millennium weighed down by its grim happened
oppressed past. S6 : If I were to make one request to God it would be that I
S : but as you read "Disgrace", JM Coetzee's eighth novel, shouldn't die trapped under water.
you become aware of a certain bleak appropriateness P : the train accident that happened in Quilon some days
The proper sequence should be : [2007 - II] ago is the kind of nightmare I have often imagined
(a) SPRQ (b) QRPS Q : It gave me restless sleep for several nights afterwards
(c) SRPQ (d) QPRS R : though I lived in Delhi, I was quite familiar to Quilon
3. S1 : For the first time in the last 30 years, the retail prices of S : It is where I spent the first seventeen years of my life
essential goods have shown a declining trend. The proper sequence should be : [2007 - II]
S6 : but they seem to have made an about-turn. (a) SRPQ (b) RSQP
P : The retail prices of essential articles have always been (c) RSPQ (d) SRQP
going up.
Ordering of Sentence B- 97

DIRECTIONS (Qs. 7-21): In the following questions, each Which one of the following is the correct sequence ?
passage consists of six sentences. The first sentence (S1) and the (a) S R P Q (b) P Q S R
final sentence (S6) are given in the beginning. The middle four (c) P R S Q (d) S Q P R
sentences in each have been removed and jumbled up. These are 11. Sl : Materially advertisements do us no good.
labelled P, Q R and S. You are required to find out the proper S6 : They have but one requirement that their intrusion
sequence of the four sentences and mark accordingly on the should be conspicuous.
Answer Sheet. [2008-I] P : The advertisements tread closely on their heels and
destroy its effect.
7. S1 : Did anything happen ? Q : Spiritually they are one of the worst avoidable evils.
S6 : He did not download or tamper with any of the code to R : Our buildings are covered with prints and pictures
which he gained access. that distract and wear us.
P : But the intruder did little more than peek and poke S : Architects might give their designs dignity or the
around. beauty of pattern.
Q : Not much, according to Microsoft. Which one of the following is the correct sequence?
R : He may have been some of its “Source code” the (a) R Q P S (b) Q R P S
secret recipe for its software. (c) R Q S P (d) Q R S P
S : Yes, somebody invaded the firm’s corporate network. 12. S1 : Both Rattan and his son Moti were idlers and did not
Which one of the following is the correct sequence ? like to do any work.
(a) R P Q S (b) Q S R P S6 : In fact, they prided themselves on their inactivity and
(c) R S Q P (d) Q P R S idleness.
8. Sl : Another significant change in the past five years have P : The result was that their idleness increased all the
been the global resurgence of ethnic and religious more.
aspirations. Q : His wife had introduced order and industry in the
S6 : This phenomenon is evident not only in Africa and house.
Asia but also in Europe, notably in Italy and Spain. R : Rattan’s wife had died long ago, Mod had married in
P : No less than 15 new States have emerged in that region the preceding year.
since 1990. S : She would work herself to death and earn the daily
Q : This manifests itself in many different ways. feed for both of them.
R : Another manifestation is the growth of ethnicity or Which one of the following is the correct sequence ?
regionalism as catalyst for new political parties. (a) S P R Q (b) S Q R P
S : One is the ‘new’ nationalism that is thriving in the (c) R Q S P (d) R P S Q
wake of the break up of the Soviet Empire. Which one 13. S1 : A hundred metres further along the trail Mahesh and
Rohini stopped short.
of the following is the correct sequence ?
S6 : But now mother bear-perhaps the same grizzly bear
(a) Q S P R (b) P R Q S
could be just over the ridge obscured by the bushes.
(c) P S Q R (d) Q R P S
P : They had shouted and waved and watched through
9. Sl : Eighteen years ago, Germany became whole and free binoculars as the mother bear reared up and roared at
again after forty years of cruel partition. them.
S6 : On October 3, 1990, reunification was ceremoniously Q : Two bear cubs were playing in the creek gully about
enacted. 20 metres to their right.
P : But on November 9, 1989, the Berlin Wall was R : They had enjoyed the roaring of the mother bear as a
miraculously breached. distance of a kilometre and a half separated them.
Q : Within ten months after the Wall came down, Germany S : The day before, they had seen a mother bear and two
had regained its national unity. cubs.
R : Not many contemporaries had expected this to happen, Which one of the following is the correct sequence ?
or to happen within the span of their lifetime. (a) R P S Q (b) Q S P R
S : The Cold War was suddenly over, the Iron Curtain (c) R S P Q (d) Q P S R
collapsed, communism melted away. 14. Sl : In an ordinary power station we burn fuel to get heat.
Which one of the following is the correct sequence ? S6 : However, instead of getting heat by burning fuel, we
(a) P R S Q (b) R P Q S get it from the nuclear reactor.
(c) R P S Q (d) P R Q S P : In a nuclear power station we burn water into steam
10. S1 : There are a number of bad habits which poor readers and then use the steam in the same way.
adopt. Q : It is from the generator that we get electricity.
S6 : Young children and very poor readers often point with R : The steam is then made to turn a turbine and through
a finger at each word in turn. the turbine a generator.
P : Of course, there must be vigorous mental activity. S : This heat turns water into steam.
Q : But extra body movements, such as pointing with the Which one of the following is the correct sequence ?
fingers or moving the lips, do not help reading. (a) P R Q S (b) S Q R P
R : In efficient reading, the muscles of the eye should (c) S R Q P (d) P Q R S
make the only external movement. 15. Sl : One morning, a few days before Rahman, the
S : Most of these involve using extra body movement in Cabuliwallah, was due to return to his country, Tagore
the reading process. was working in his study.
EBD_7367
B- 98 Ordering of Sentence

S6 : They had quarrelled, and Rahman had struck the man P : His fellow clerks in the department approached him
with his knife. with proposals to launch some joint projects.
P : There were blood stains on his cloth, and one of the Q : But he shied away from both his friends and relatives.
policemen carried a knife stained with blood. R : His relatives advised him to leave service and start his
Q : Suddenly he heard shouting in the street, and he saw own business.
Rahman being led away between two policemen, S : He listened to all patiently.
followed by a crowd of curious boys. Which one of the following is the correct sequence ?
R : He learned that a certain neighbour had owed the (a) PQ SR (b) S R P Q
Cabuliwallah some money, but had denied it. (c) P R S Q (d) SQRR
S : He hurried out and stopped them to inquire what it all 20. Sl : What is freedom ?
meant. S6 : Some people feel that we actually suffer from what
Which one of the following is the correct sequence ? may be called choice fatigue.
(a) R S P Q (b) Q P P R P : There is so much choice that we find it difficult to
(c) R P S Q (d) Q S P R choose.
16. Sl : An experiment was conducted in England to study Q : Fortunately, we are now living in a world full of choice.
what happens to the bodies and minds of people R : Without the possibility of choice and the exercise of
travelling at high speeds. choice, we are not human beings but only inanimate
S6 : Psychological tests showed that their ability to make objects.
decisions decreased quickly. S : Freedom is the right to choose.
P : Then they were flown to America. Which one of the following is the correct sequence ?
Q : Travelling also had an effect on the minds of these (a) P Q R S (b) S R Q P
people. (c) P R Q S (d) S Q R P
R : They were first kept under observation in London. 21. S1 : I did not know Nehru at all intimately.
S : It was found that as they travelled from one time zone S6 : It is more correct to use the word captivation than
to another, their blood pressure moved away from the impression.
normal. P : But his personality made an immediate impression at
Which one of the following is the correct sequence ? my very first meeting with him.
(a) R Q S P (b) S P R Q Q : Nor was the effect he made just an impression.
(c) R P S Q (d) S Q R P R : This impression did not change over the years.
17. Sl : All the fossil fuel that we use today came from green S : In fact, I had not even met him many times.
plants. Which one of the following is the correct sequence ?
S6 : The income that Dr. Calvin is talking about is the sun’s
(a) S Q R P (b) R P S Q
energy which living green plants capture and store up
(c) S P R Q (d) R Q S P
every day.
P : “And now we are burning it all up in just 100 or 200 DIRECTIONS (Qs. 22-36) : In the following questions, each
years!” says Dr. Calvin. passage consists of six sentences. The first sentence (Sl) and the
Q : We have nearly used up all our savings. final sentence (S6) are given in the beginning. The middle four
R : It took hundreds of millions of years for those plants sentences in each have been removed and jumbled up. These are
to change into coal, oil, and gas. labelled P, Q, R and S. You are required to find out the proper
S : “It is time for us now to begin living on our income”, sequence of the four sentences and mark accordingly on the
says Calvin. Answer Sheet. Example ‘X’ has been solved for you. [2008-II]
Which one of the following is the correct sequence ?
X. S1 : There was a boy named Jack.
(a) Q S R P (b) R P Q S
S6 : At last she turned him out of the house.
(c) Q P R S (d) R Q Q P
18. S1 : The ‘touch-me-not’ plant folds up its leaves when P : So the mother asked him to find work.
touched. Q : They were very poor.
S6 : The folding up of the leaves is controlled by the R : He lived with his mother.
pulvinus. S : But Jack refused to work.
P : How is the plant able to do this? Which one of the following is the correct sequence ?
Q : At the lower end of each leaf is a tiny swelling, called (a) R Q P S (b) P Q R S
the pulvinus. (c) Q P R S (d) R P S Q
R : The pulvinus acts as the ‘brain’ or control centre of Explanation :
the leaf. The correct sequence in this example is R Q P S which is
S : It is only in recent years that a possible answer has marked by (a). Therefore, (a)is the correct answer.
been found. 22. Sl : The life of early men had advantages and disadvantages.
Which one of the following is the correct sequence ? S6 : On the whole, there was friendship and amity within
(a) R S Q P (b) P Q S R
the tribes.
(c) R Q S P (d) P S Q R
19. Sl : A lower division clerk who has been working in CPWD P : Then, they had enough physical exercise which made
for the past 15 years, won Rs.1 crore in the recently them healthy and active.
launched programme ‘Kaun Banega Crorepati’. Q : They could roam for months without fear of meeting a
S6 : Because the publicity that he received was really stranger.
disturbing him. R : One of the advantages was that they were not
overcrowded.
Ordering of Sentence B- 99
S : They lived in small tribes where everybody knew 27. S1 : I had not seen my father for several years.
everybody else. S6 : His words sank deep into my heart.
Which one of the following is the correct sequence ? P : I met him late one evening in his flat.
(a) Q S R P (b) R Q P S Q : I wrote him a note suggesting a very early meeting.
(c) S P R Q (d) P R Q S R : He listened to my story in silence.
23. S1 : My journey was to last for thirty-six hours. S : When he spoke, his voice was soft but without warmth.
S6 : In the steel trunk under the seat there was a bag Which one of the following is the correct sequence ?
containing two hundred rupees that did not belong to (a) Q S R P (b) P Q R S
me. (c) Q P R S (d) Q P S R
P : Every mile of the country through which the train was 28. S1 : There is no reason for the terror which the sight of a
running, was interesting. snake causes in most people.
Q : Yet I was not happy. S6 : Being aggressive by nature, they can attack human
R : I had the carriage for myself. beings for no reason at all, taking a fisherman or
S : The train would stop for breakfast, lunch and dinner. swimmer by surprise in the water, where the man is
Which one of the following is the correct sequence ? somewhat helpless.
(a) R S P Q (b) S Q P R P : Of the poisonous snakes, only those found in the sea
(c) P Q S R (d) R P Q S are always dangerous.
24. Sl : There was a check-post for passing vehicles at a Q : They are only too anxious to avoid human beings.
village called Gobindapur, a short distance from where R : Many more people are killed, much more frequently
the road to Dhanbad branched off from the Grand by motor-cycles and cigarettes than by snakes.
Trunk Road. S : The majority of snakes are harmless.
S6 : He asked me more than once if I was sure that the man Which one of the following is the correct sequence ?
had noted the number down. (a) S Q R P (b) R S Q P
P : When I came sufficiently near, he moved back and the (c) R P Q S (d) P Q R S
barrier was lifted. 29. Sl : One day I went into the water off the coast of Africa.
Q : I slowed down and found a man coming forward with S6 : I hurled at him the rubber fins.
P : I sighted a shark at short distance from me.
pencil and book in hand.
Q : He launched towards me as hard and swift as a missile.
R : I told uncle that the man had taken down the number
R : I was floating at a shallow depth, without making a
of the car, adding that it was a routine practice.
movement.
S : As I approached it, I saw the barrier coming down
S : Every muscle of my body tensed.
slowly. Which one of the following is the correct sequence ?
Which one of the following is the correct sequence ? (a) P S Q R (b) S Q R P
(a) R Q P S (b) Q R P S (c) R P S Q (d) P Q S R
(c) S Q P R (d) R S P Q 30. S1 : The clerk read the statement loudly and clearly.
25. S1 : The ancestors of whales, it is said, lived on land, for S6 : They returned in five minutes.
they still have slight traces of hind-legs. P : The judge brought down the gavel sharply and roared.
S6 : He has flippers on his sides to keep him balanced and Q : He told the jury to return a lawful verdict.
layers of fat or oil under the skin which furnish heat R : We, the jury. Find the defendant not guilty, provided
and make the huge body light and buoyant. he returns the mule.
P : But ages ago, whales changed their homemoving from S : There is not such verdict in the law; the defendant is
the land to the sea. either guilty or not guilty.
Q : He is shaped like a submarine boat, with a tail turned Which one of the following is the correct sequence ?
into a power paddle. (a) R P S Q (b) R S P Q
R : The whale is suited to live in water. (c) Q R P S (d) P S R Q
S : They are warm-blooded animals, and feed their babies 31. S1 : Some people prefer country life to city life.
as land mammals do. S6 : For these reasons more and more people are leaving
Which one of the following is the correct sequence ? the country to live in the city.
(a) P R S Q (b) S P R Q P : Life in the country is quieter, cleaner and less hectic.
(c) R Q P S (d) Q P R S Q : The city also offers more privacy since neighbours
26. Sl : Gopal worked as a labourer at the building site. are too busy to be interfering in the affairs of others.
S6 : He anscrewed the lid and found a valuable collection R : However, the city offers more excitement, a wide variety
of old silver in it. of activities and a chance to meet more people.
P : But Gopal made a bid and he got the box. S : Country people five longer and generally seem to be
Q : There was no key to it and it seemed useless but Gopal healthier and happier.
took it home. Which one of the following is the correct sequence
R : Once while returning from his work Gopal stopped at (a) P S R Q (b) S R Q P
an auction sale. (c) R Q P S (d) Q P S R
S : No one seemed to want a rough old box when it was 32. Sl : When my car broke down, I took it to the only mechanic
put up for sale. available in our town.
Which one of the following is the correct sequence ? S6 : They pushed the car down one street and up another
(a) S P R Q (b) S R P Q and soon we had gone through most of the, streets in
(c) R S P Q (d) R P Q S the town but the car wouldn’t start.
EBD_7367
100
B- Ordering of Sentence

P : But it just refused to start. the proper sequence of the four sentences and mark accordingly
Q : I went there at the appointed time to collect it. on the Answer Sheet. [2009-I]
R : So I sat at the wheel and the mechanic and his helper Example ‘X’ has been solved for you.
started to push it.
S : He said it required some minor repairs and asked me to X. S1 : There was a boy named Jack.
collect it in the evening. S6 : At last she turned him out of the house.
Which one of the following is the correct sequence ? P : So the mother asked him to find work.
(a) S Q P R (b) Q S R P Q : They were very poor.
(c) R P Q S (d) P Q R S R : He lived with his mother.
33. S1 : Dinner had been served-his daughter laid out the S : But Jack refused to work.
plates. The proper sequence should be
S6 : Then, silently she left the table to retire for the night- (a) R Q P S (b) P Q R S
it was as if she had never been there. (c) Q P R S (d) R P S Q
P : She was just a child, only 14-too young, too simple to Explanation:
know to understand. The correct sequence in this example R Q P S which is marked
Q : He sat down groundly, not saying a word to her. by (a) Therefore, (a) is the correct answer.
R : She had already had her meal and was standing by his 37. S1 : Having visited the Taj Mahal many tourists think that
side, not quite knowing what to do. Agra has little else to offer.
S : Bread and cheese-a simple subsistence at the end of a S6 : There are few other buildings to match the delicacy of
not-so-simple life. this tomb.
Which one of the following is the correct sequence ? P : One of these is surely the tomb of Itimad-ud-daulah.
(a) Q P S R (b) R P S Q Q : The design of the whole tomb was given by his
(c) Q R P S (d) S Q P R daughter Nur Jahan.
34. Sl : It was a bitterly cold night and even at the far end of R : After seeing the Taj one could profitably visit half a
the bus the wind cut like a knife. dozen other Mughal buildings.
S6 : I saw trouble brewing. S : This tomb has the delicacy of a baroque jewel case.
P : The conductor came in and took the fares. The proper sequence should be
Q : The younger of the two women was dressed in sealskin (a) R S Q P (b) Q S R P
and carried one of those little Pekinese dogs that (c) S P R Q (d) R P S Q
women like to carry in their laps. 38. S1 : For years the old chair stood in one of the empty
R : The bus stopped, and two women and a man got in antics.
together and filled the vacant places. S6 : I saw my parents madly in love again.
S : Then his eyes tested with cold malice on the beady- P : So when I saw it for the last time, it stood there.
eyed little dog. Q : When my mother died, I wanted to sell it but could
Which one of the following is the correct sequence ? not.
(a) R Q P S (b) R S Q P R : It was there for many years after my father died.
(c) R P Q S (d) P S R Q S : I peeped in the past.
35. Sl : The officer rose to his feet, trembling. The proper sequence should be
S6 : A half-hour later he returned to camp. (a) P Q R S (b) S R Q P
P : He failed to find him there. (c) R P Q S (d) R Q P S
Q : Pulling himself together, he ran rapidly away from the 39. S1 : Illness may start with almost any sign, but some of
cliff to a point a half-mile from its foot. them are much commoner than others.
R : He was disappointed. S6 : As soon as this happens he must become alert about
S : He expected to find the horseman somewhere there. the signs.
Which one of the following is the correct sequence ? P : It is important to note these signs.
(a) R P Q S (b) Q P S R Q : Often the first sign of something wrong is that the
(c) Q S P R (d) S Q P R patient just does not feel fit.
36. Sl : It was Saturday. R : They may help a doctor to decide what is wrong.
S6 : The children had already bought roasted gram and S : He usually relies on these signs for the diagnosis of
peanuts to get into a picnic mood. the illness.
P : A taxi carried us all to the zoo. The proper sequence should be
Q : They wanted to be taken out and we decided to take (a) P R S Q (b) R S Q P
them to the local zoo. (c) P Q S R (d) Q R S P
R : My sister’s two young children were at our house. 40. S1 : Newton was perhaps the greatest scientist that ever
S : We bought the tickets and entered the zoo. lived.
Which one of the following is the correct sequence ? S6 : Newton went home and worked quietly by himself for
(a) R Q S P (b) R P Q S about 18 months.
(c) R Q P S (d) P R Q S P : But when he was only 22, a terrible plague epidemic
swept over England.
DIRECTIONS (Qs. 37-51) : In the following questions, each Q : He was the son of a Lincolnshire farmer, and was born
passage consists of six sentences. The first sentence (S1) and in 1642.
the final sentence (S6) are given in the beginning. The middle R : Therefore the universities were closed.
four sentences in each have been removed and jumbled up. S : He went to Cambridge to study mathematics when he
These are labelled P, Q, R and S. You are required to find out was 19.
Ordering of Sentence B-101
The proper sequence should be all of whom were trying to carry on important
(a) S P R Q (b) Q P S R industries.
(c) S Q P R (d) Q S P R Q : He worked hard in his laboratory with test tubes and
41. S1 : Poverty is a God’s curse. all kinds of experiments.
S6 : Is not poverty a God’s boon ! R : He was working to help people who were suffering in
P : These persons get themselves enrolled as poor persons some special way from disease.
and get all the benefits of poverty. S : He not only made some exciting discoveries about
Q : It is not necessary for a person to be actually poor for germs but he was able to use his discoveries in very
getting enrolled because it can be easily managed. practical ways.
R : It may be true for a few but to many it is just its The proper sequence should be
opposite. (a) S Q R P (b) P Q R S
S : Such persons consider it to be a source of enjoying (c) Q R S P (d) R S Q P
life without earning enjoyment. 46. S1 : Tom Walker and his wife were always at loggerheads.
The proper sequence should be S6 : Tom silently thanked God for this relief.
(a) R Q P S (b) Q R S P P : She never tired of reproaching him on this score.
(c) R S P Q (d) S R Q P Q : Tom was incurably lazy and talkative
42. S1 : Belur is 35 km. from Hassan. R : One day Mrs. Walker caught a deadly cold, and shortly
S6 : They depict young women – musicians and dancers – afterwards died.
in various poses. S : This was a constant source of irritation to his wife.
P : Seen from afar, the star-shaped temple, characteristically
The proper sequence should be
Hoysala, is not very impressive.
(a) P R Q S (b) Q S P R
Q : It stands in a courtyard surrounded by a rectangular
wall. (c) S Q R P (d) P S Q R
R : But closer it is dazzling and marvellous; and the entire 47. S1 : So we went on in the quiet, and the twilight depened
exterior is decorated with sculptures, the loveliest being into night.
the panels right and left of the main door. S6 : Then as the darkness grew deeper, she put her arms
S : Chennakesava Temple is dedicated to Vishnu. round my neck, and, closing her eyes tightly pressed
The proper sequence should be her face against my shoulder.
(a) Q R S P (b) S R P Q P : The ground grew dim and the trees black.
(c) S P Q R (d) S Q P R Q : The clear blue of the distance faded, and one star after
43. S1 : When his business failed, he began to look for a job in another came out.
an office. R : Neena’s fears and her fatigue grew upon her.
S6 : Unable to bear misfortunes any further, he started S : I took her in my arms and talked to her and caressed
toying with the idea of ending his life of burdens and her.
strains. The proper sequence should be
P : To overcome depression he took to drinking and (a) P R Q S (b) Q P R S
became addicted to it. (c) Q P S R (d) R S P Q
Q : He soon realized that nothing was more difficult than 48. S1 : Kennedy kicked moodily at the leg of the chair which
to find a job. he was holding.
R : Without job he failed to meet the daily requirements of S6 : It was a depressing beginning.
his family which made him highly depressed. P : Now his chief desire seemed to be to score off the
S : Quarrel ensued invariably between husband and wife human race in general, his best friend included.
when he returned home in a state of drunkenness. Q : If he had asked Fenn to help him in a tight place, then
The proper sequence should be he knew he could have relied on him.
(a) P Q R S (b) S R Q P R : Last term he and Fenn had been as close friends as
(c) Q S R P (d) Q R P S you could wish to see.
44. S1 : Unlike many modern thinkers, Tagore had no blueprint S : The feeling that his whole world had fallen about his
for the world’s salvation. ears was increasing with every hour he spend at Kay’s
S6 : As a poet he will always delight, as a singer he will The proper sequence should be
always enchant, as a teacher he will always enlighten. (a) P R S Q (b) R S P Q
P : His thought will therefore never be out of date. (c) S R Q P (d) R Q P S
Q : He merely emphasised certain basic truths which men 49. S1 : I sat at the table and ate.
may ignore only at their peril. S6 : I was just the normal Ramaswamy, husband of
R : He believed in no particular ‘ism’. Madeleine.
S : He was what Gandhiji rightly termed the Great Sentinel. P : My breathing became suddenly difficult.
The proper sequence should be Q : I concentrated on my food and I was convinced I had
(a) S R P Q (b) P R Q S to eat.
(c) R S P Q (d) R Q P S
R : I stopped, however, any exhibition of the extraordinary.
45. S1 : Louis Pasteur had a very busy and interesting life.
S : But lungs have temperament.
S6 : He was always very proud of being able to help his
The proper sequence should be
country in this way.
(a) P R S Q (b) Q S P R
P : Among the people whom Pasteur was able to help
(c) Q S R P (d) S P R Q
were brewers, breeders of silk worms, and cow keepers,
EBD_7367
102
B- Ordering of Sentence

50. S1 : Todd borrowed this dollar last year on the 8th of April.. Q : Each one knows more and more about less and less.
S6 : And I said, ‘certainly’. R : We should not only be specialists but also have a
P : He needed a dollar to pay his taxi and I lent it to him. sense of the meaning of life and of social responsibility.
Q: He merely said, ‘Let me have a dollar, will you’! S : We concentrate on some narrow field and forget the
R : It happened quite simply and naturally. largest context in which we can see the meaning of our
S : I hardly realised it till it was all over. own specialization.
The propber sequence should be The proper sequence should be
(a) PRSQ (b) R S P Q (a) P Q S R (b) Q P R S
(c) QRPS (d) P Q R S (c) S P Q R (d) Q S P R
51. S1 : People have always been fascinated by dreams. 54. S1 : He went to his friends, but none of them was ready to
S6 : But a small number, perhaps less than 5% have them help him now.
regularly. S6 : Now the youngman understood that he had made a
P : In fact, only recently have there been serious studies mistake.
to find out how many of us actually have nightmares. P : He had to look after the farmer’s pigs.
Q : Now that is changing. Q : And when he was hungry he ate the food which was
R : But the study of nightmares has been curiously given to the pigs.
neglected. R : He went away from the city and wandered round the
S : While results so far are inconclusive, it seems fair to countryside looking for food to eat.
say that at least half the population has occasional S : In the end he went to the house of a farmer and became
nightmares. his servant.
The propber sequence should be The proper sequence should be
(a) S R P Q (b) R Q P S (a) P S R Q (b) R S P Q
(c) P Q R S (d) S Q R P (c) S Q P R (d) Q P S R
55. S1 : Man, when first came, must have been surrounded by
DIRECTIONS (Qs. 52-70): In the following questions, each great animals and he must have lived in fear of them.
passage consists of six sentences. The first sentence (S1) and S6 : Gradually, however, man raised himself and became
the final sentence (S6) are given in the beginning. The middle more and more powerful till he became stronger than
four sentences in each have been removed and jumbled up. any animal.
These are labelled P, Q, R and S. You are required to find out P : He makes the animals do what he likes.
the proper sequence of the four sentences and mark accordingly Q : But in those days he was not the master but a poor
on the Answer Sheet: [2009-II] hunted creature himself trying to keep away from the
Example ‘X’ has been solved for you. great animals.
X. S1 : There was a boy named Jack. R : Today man is the master of the world.
S6 : At last she turned him out of the house. S : Same he tames, same he eats and same he shoots for
P : So the mother asked him to find work. pleasure.
Q : They were very poor. The proper sequence should be
R : He lived with his mother. (a) R Q P S (b) R S P Q
S : But Jack refused to work. (c) R P S Q (d) P Q S R
The proper sequence should be 56. S1 : Convention has a necessary part to play in the life of
(a) R Q P S (b) P Q R S everyone.
(c) Q P R S (d) R P S Q S6 : Their purpose, however, is always the same.
Explanation : P : It is necessary and useful for this reason.
The correct sequence in this example is R Q P S which is Q : Of course, there is nothing absolute about conventions.
marked by (a). Therefore, (a) is the correct answer. R : They vary from country to country, from age to age.
52. S1 : Long long ago there lived a king who was crude and S : In all social affairs it prescribes more or less generally
very much like a savage. accepted rules of behaviour.
S6 : Or at least he tried to. The proper sequence should be
P : He was a man of great fancies and even greater (a) S Q R P (b) P Q S R
enthusiasm. (c) R P S Q (d) S P Q R
Q : Because he had so much authority as a king, he was 57. S1 : No one can seriously pretend to remain unaffected by
able to force some of these fancies into reality. advertisement.
R : He had none of the grace and polish of his neighbours. S6 : Though they seem so varied all these advertisements
S : He had learned some manners from his Latin have one thing in common: they make strong appeals
neighbours, but mostly he was barbaric, loud and gruff. to our emotions.
The proper sequence should be P : Even in the sanctity of our living rooms advertisers
(a) R S P Q (b) S Q P R pounce upon their helpless victims as they tune into
(c) R P Q S (d) P R Q S their favourite radio or television programmes.
53. S1 : Our age is the age of the specialist. Q : It is impossible to turn a blind eye to the solicitous
S6 : We have to reckon with the spirit of science, overtures to buy this or that article that fill our streets,
understand its limitations and an outlook which is newspapers and magazines.
consistent with its findings. R : Most often they bank upon our sense of fear for not
P : Modern specialization had led to the fragmentation of doing or possessing this or that for our youth, beauty,
knowledge. health and security.
Ordering of Sentence B-103
S : No matter how hard we resist, clever little tunes and P : The village people now seldom live under the overall
catch-phrases seep into our subconscious mind and command of any single powerful headman.
stay there. Q : Local political leaders mostly control and command
The proper sequence should be them.
(a) P Q S R (b) Q R P S R : Party politics had entered into the peaceful villages
(c) R P S Q (d) S Q P R forcing the villagers to live polarized political lives.
58. S1 : A great part of Arabia is desert. S : One dangerous consequence of this is that they are
S6 : Such place is called an oasis. often dragged into clashes even with their own
P : The sand is so hot that you cannot walk over it with relations of different political beliefs.
your bare feet in the day time. The proper sequence should be:
Q : These springs are few and far apart, but wherever there (a) P R S Q (b) R P Q S
is one, green grass very soon covers the ground all (c) Q P R S (d) P S R Q
around it. 63. S1 : People think that poverty is a great curse and wealth
R : Here there is nothing but sand and rock. is a source of happiness in life.
S : Here and there is the desert are springs of water that S6 : All great men were born poor.
come from deep down under the ground. P : A life of poverty gives more genuine satisfaction than a life
The proper sequence should be passed in affluence which encourages dependence on
(a) P S R Q (b) R S Q P others.
(c) R P S Q (d) P Q S R Q : This is untrue.
59. S1 : Many things about Konarak seem shrouded in mystery. R : Poor people are free from the evils which surround the
S6 : King Narasimha probably had it built as much as a rich.
memorial to himself as he did in honour of the Sun- S : They have sympathy for one another and are more
God. self -reliant.
P : It is, of course, a religious shrine. The proper sequence should be:
Q : But here there also seems to be a great emphasis on (a) P S Q R (b) Q P R S
purely human grandeur. (c) S R Q P (d) R S P Q
R : Why was it built? 64. S1 : Before man settled down, his life was not governed by
S : Long before the temple was built, sometime in the 13th the seasons.
century, Konarak was one of the five holiest places in S6 : However, we do not know when man first began to
Orissa. divide the day into twenty-four hours, and the hour
The proper sequence should be: into sixty minutes.
(a) R P Q S (b) R P S Q P : So man began to measure time, and first he looked for
(c) S R P Q (d) S P Q R convenient units of measurement, one of which was
60. S1 : Most of you probably did not see Gandhiji at close the day.
quarters. Q : This was done by making a mark for each day on a tree
S6 : The result was that the poor man had to try to be trunk.
good. R : Once he settled down to the agricultural way of life, he
P : One of these qualities was that he managed to draw had to be able to calculate the change of seasons on
out the good in another person. which agriculture depended.
Q : But he somehow spotted the good and laid emphasis S : In order to calculate the passing of time, it was
on that good. necessary for man to record the number of days in
R : He had amazing qualities. each season.
S : The other person may have had plenty of evil in him. The proper sequence should be:
The proper sequence should be: (a) P Q R S (b) R P S Q
(a) Q P S R (b) R P S Q (c) Q P S R (d) R S Q P
(c) P R Q S (d) Q S P R 65. S1 : Imitation is not civilization.
61. S1 : Everybody thinks that this is the age of reason. S6 : When a man has begun to be ashamed of his ancestors,
S6 : Obviously we must reexamine the view that this is the the end has come.
age of reason. P : As ass in a lion’s skin never makes a lion.
P : Reason takes a backseat at such times. Q : It is verily the sign of awful degradation in a man.
Q : The ordinary events of life seem to support this view. R : Cowardly imitation never makes for progress.
R : We must therefore avoid the development of such S : When a man has begun to hate himself; then the last
situations. blow has come.
S : But the behaviour of people in crises makes us doubt The proper sequence should be:
this. (a) P Q R S (b) S P R Q
The proper sequence should be (c) P R Q S (d) Q S R P
(a) R Q P S (b) S R Q P 66. S1 : Democracy, more than any other form of government,
(c) P S R Q (d) Q S R P needs good citizenship.
62. S1 : Our villages are not now as serene and peaceful as S6 : That is not the instinct of the natural man; yet somehow
they had been twenty five years ago. that habit has to be acquired.
S6 : This often vitiates the traditional tranquility of the rural P : But it has to be freedom of service self-chosen and
way of living. sometimes of sacrifice self-imposed.
EBD_7367
104
B- Ordering of Sentence

Q : Under absolutism or dictatorship, men are forced into Q : And then, over 28% of these hold a Ph.D — the highest
line. percentage among all ethnic groups involved in the
R : Freedom is the essence of democracy: the more software industry.
complete the democracy, the more complete the R : Some dollars might be flowing back but the best of
freedom. brains are surely being lost to the nation.
S : But in a democracy things are not so simple. S : What catches the attention at the very outset is the
The proper sequence should be: number — over a million Indians work in the American
(a) S Q P R (b) R S Q P software industry.
(c) P R Q S (d) Q S R P The proper sequence should be:
67. S1 : In 1668 an important discovery was made. (a) P Q S R (b) S Q P R
S6 : That was the beginning of the end of the theory of (c) S Q R P (d) S P Q R
spontaneous generation of higher plants and animals. DIRECTIONS (Qs. 71-90) : In the following questions, each
P : He put some rotting meat and fish in open jars and passage consists of six sentences. The first sentence (S1) and
watched them. the final sentence (S6) are given in the beginning. The middle
Q : When he covered the jars with muslin, he found that four sentences in each have been removed and jumbled up.
flies came and laid their eggs on the muslin, but no These are labelled P, Q, R and S. You are required to find out
maggots developed on the meat. the proper sequence of the four sentences and mark accordingly
R : In time, he noticed that flies came and laid their eggs in on the Answer Sheet. [2010-I]
the meat and the maggots hatched from the eggs.
S : An Italian physician named Redi decided to test the Example ‘X’ has been solved for you.
idea that worms were generated spontaneously in X. S1 : There was a boy named Jack.
rotting meat. S6 : At last she turned him out of the house.
The proper sequence should be: P : So the mother asked him to find work.
(a) P Q S R (b) S P R Q Q : They were very poor.
(c) S R P Q (d) R P Q S R : He lived with his mother.
68. S1 : There is no sound more familiar in Calcutta than the S : But Jack refused to work.
clanking of its tramcars. The proper sequence should be :
S6 : Their progress is regarded with the mixture of dread (a) R Q P S (b) P Q R S
and embarrassment reserved for the approach of a (c) Q P R S (d) R P S Q
cantankerous old aunt. Explanation :
P : For a growing body of opinion in the city, trams are a The correct sequence in this example R Q P S which is marked
symbol of urban blight. by (a) Therefore, (a) is the correct answer.
Q : It is also an irritant. 71. S1 : Unity in diversity is a unique feature of India.
R : All day and late into the night the trundling roll of a S6 : Have you read this book? If not, you should do so
tram has become a symbol, a portent. now.
S : Slow, overcrowded and unwieldy, they are thought of P : They should, therefore, first try to know the mind and
as obstructionist and outmoded. spirit of India–the Indian-ethos, so to say.
The proper sequence should be: Q : Nehru has referred to this unity very feelingly in ‘The
(a) R Q P S (b) R Q S P Discovery of India’.
(c) Q S P R (d) P S R Q R : Those who do not see this underlying unity in
69. S1 : In England, yesterday, villagers were amazed to see a apparent diversity fail to understand this country
number of youths floating over a field. properly..
S6 : The others, however, returned safely to earth by S : Then, they will discern strands of common heritage
puncturing their umbrellas before they had risen very running all through the differences of languages,
far. dress, food, method of worship etc.
P : One boy let go his umbrella and landed on a cow from The proper sequence should be :
a height of about ten feet. (a) Q R S P (b) P S R Q
Q : Luckily, there were not as many casualties as there (c) S Q P R (d) R P S Q
might have been. 72. S1 : There was a legend among out people that the island
R : Another ended up in a pond. had once been covered with tall trees.
S : The youths, it appeared, had inflated their fathers’ S6 : Tumai angrily went down, down to another world; so
umbrellas with hydrogen and as a result were borne people die today because he did.
skywards. P : This was a long time ago, at the beginning of the world
The proper sequence should be:
when Tumai and Mukat ruled.
(a) Q S P R (b) P Q S R
Q : Tumai wished people to die.
(c) S Q P R (d) P Q R S
R : The two gods quarrelled about many things.
70. S1 : A recent survey reveals that Indian Engineers play an
S : Mukat did not.
immensely significant role in the American software
The proper sequence should be :
industry.
(a) P R Q S (b) P S R Q
S6 : It is high time the leaders addressed themselves to
(c) S P R Q (d) S Q P R
this issue seriously.
73. S1 : To a foreign learner, English pronunciation presents
P : Not only this, the Indian Americans own almost 25%
of the small information technology firms in the U.S. the greatest difficulty.
Ordering of Sentence B-105
S6 : But a knowledge of Phonetics will help a great deal in R : That is why we have universal education as a target in
learning correct English pronunciation. our Constitution.
P : Words are spelt in one way and pronounced in another. S : For this, certain minimum cultural and economic
Q : For instance ‘u’ has different pronunciations in ‘but’, conditions must be provided.
‘put’, ‘build’ and ‘bury’. The proper sequence should be :
R : The English language is notoriously un-phonetic. (a) Q S R P (b) P R S Q
S : The same letters give different sounds in different (c) R Q P S (d) S Q R P
words. 78. S1 : The advancement of science is not a secret or a
The proper sequence should be mysterious process.
(a) P R S Q (b) Q S P R S6 : What is he to think but to marvel at the skill of science,
(c) S P Q R (d) R P S Q and to fear its power ?
74. S1 : There was something about the smile of Mr. Acton, P : No one tells the layman about the years of experiment
when he come over to Sharma’s table, which betokened and failure.
disaster. Q : If it sometimes seems so, that is just because the day-
S6 : Specially, since Mr. Acton was not known to smile too to-day work of science is so unspectacular.
much, being a morose, old Sahib, hard-working and R : How is he to know what has not been done, or to
conscientious. guess the labour of what has ?
P : The fact that Mr. Acton should come over to his table S : You hear nothing from the research worker for years,
at all, fawn upon him and say what he had said was, of and then, suddenly, there is the result in the headlines;
course, most flattering. penicillin or the jet engine or nuclear fusion.
Q : For, very rarely did the head of the firm condescend to The proper sequence should be :
move down the corridor, where the Indian staff of (a) Q S P R (b) P Q R S
Henry King and Co., worked. (c) Q P S R (d) S Q P R
R : But that smile on Mr. Acton’s face! 79. S1 : I am the manager of travel agency in the city.
S : But as the Sahib had only said, “Mr. Sharma, I have S6: I spend those twenty five minutes doing crosswords.
brought something for you specially from London, P : I usually catch the 8 O’clock train for my journey to
you must come into my office on Monday and take the office..
it”......,. Sharma could not surmise the real meaning of Q : I live fifteen kilometres out of the city where I have a
the General Manager’s remark. small room.
The proper sequence should be : R : I go to work on the electric train everyday.
(a) S P Q R (b) P R Q S S : The train takes about twenty five minutes to get to the
(c) S Q R P (d) Q R S P city.
75. S1 : Arun suddenly found himself in the streets. The proper sequence should be :
S6 : When he applied for the refund of his security, there (a) P R S Q (b) R Q P S
was hardly anyone at the other end to receive his (c) Q S P R (d) S P R Q
application. 80. S1 : Until the first atomic energy bombs fell on Hiroshima
P : There was a little money in the bank and he had some and Nagasaki, the atom and its behaviour had been
stock on hand. remote from everyday affairs.
Q : The prices were going down, and he could hardly S6 : So the idea grew up that the product of long years at
realize a few hundred rupees. atomic research had led only to a fresh weapon of
R : At first he could hardly understand the full significance destruction more powerful than any known before.
of this collapse. P : There was something queer and incomprehensible
S : But the stock moved out slowly. about them.
The proper sequence should be : Q : And secondly, that something new and devastating
(a) P S Q R (b) S Q R P had been added to the list of man’s victories over
(c) R P S Q (d) P Q S R nature.
76. S1 : Science has given us powers fit for the gods. R : But it, in the first place, made us realize that the atom
S6 : And we should remember that they are very stern and its ways were no longer something apart from
masters. everyday life.
P : For example, we do not know how to manage our S : The atomic bombing of Japan did not resolve that
machines. incomprehensibility.
Q : Yet we use them like small children. The proper sequence should be :
R : But in practice, they have become man’s masters. (a) P S R Q (b) P Q S R
S : Machines were made to be man’s servants. (c) R S Q P (d) S Q P R
The proper sequence should be : 81. S1 : Jagdish Chandra Bose was born on 30 November, 1853
(a) R Q P S (b) P Q R S at Parikhal, now in Bangladesh.
(c) Q P S R (d) Q P R S
S6 : He himself founded some technical and industrial
77. S1 : It follows that we should enable all individuals to live
schools.
a full, free, rich life.
P : But he also knew that without technical education
S6 : This does not mean the regimentation of the individual.
India could not become a great nation.
P : We talk often of a socialistic pattern of society.
Q : His father Babu Bhagwandas was a remarkable man.
Q : We must help to bring up the buried treasure in each
R : He had a high regard for India’s ancient culture.
individual without breaking any of it.
EBD_7367
106
B- Ordering of Sentence

S : Through he was an important government official, yet Q : Tribes from Madhya Pradesh and Orissa were also
he did not give up his independence of thought. invited to participate in this darbar.
The proper sequence should be : R : We could achieve emotional integrity amongst tribals
(a) Q P R S (b) R S Q P which resulted in this success.
(c) Q S R P (d) R P S Q S : For centuries they were celebrating it on different days
82. S1 : A gentleman trying to get a fly out of the milk or a and in different ways.
piece of cork out of his glass of wine often imagines The proper sequence should be :
himself to be irritated. (a) S R P Q (b) P R S Q
S6 : But I pointed out to him that this sense of wrong was (c) Q S P R (d) R S Q P
really subjective and relative; it rested entirely upon 86. S1 : Happiness, after all, is an inner state of mind.
the assumption that the drawer could, should, and S6 : My point is that it is not wealth but coordination of
would come out easily. one’s thought and action which removes inner conflicts.
P : Again, I have known some people of very modern P : Some of the most miserable persons I have come across
views driven by their distress to the use of theological in my life are rich.
terms to which they attached no doctrinal significance, Q : It is little dependent on outside environment.
merely because a drawer was jammed tight and they R : Happiness has very little to do, for instance, with
could not pull it out. whether you are rich or not rich.
Q : Everyday his drawer was jammed, and everyday in S : It is true that poverty makes one miserable in a very
consequence it was something else that rhymes to it. acute way.
R : A friend of mine was particularly afflicted in this way. The proper sequence should be :
S : Let him think for a moment of the patience of anglers (a) S P Q R (b) Q P S R
sitting by dark pools, and let his soul be immediately (c) R S P Q (d) Q R S P
irradiated with gratification and repose. 87. S1 : It was in 264 B. C. that the great struggle between
The proper sequence should be : Rome and Carthage, the Punic Wars began.
(a) S P R Q (b) P S R Q S6 : But the Romans, with extraordinary energy, set
(c) Q P R S (d) S P Q R themselves to outbuild the Carthagians.
83. S1 : Even in his earliest days, man had government. P : It gradually developed into a struggle for the
S6 : As the number of men multiplied, hunting bands grew possession of Sicily.
larger, divided and formed independent groups.. Q : The advantage of the sea was at first with the
P : When he grew old and dull, another leader took his Carthagians.
place. R : The First Punic War began in that year about the
Q : As he stepped outside, he joined with other men to pirates of Messina.
form a hunting tribe that learned to work together. S : They had great fighting ships of what was hitherto an
R : Its simplest form was the family, where man had unheard-of size.
authority over his wife and children. The proper sequence should be :
S : Probably the hunter with the right combination of (a) R Q S P (b) R P Q S
strength and cleverness became the leader of the tribe. (c) P R S Q (d) Q S P R
The proper sequence should be : 88. S1 : Over-eating is one of the most wasteful practices
(a) Q R P S (b) R Q S P among those who can afford it.
(c) S R P Q (d) R P Q S S6 : The evening meal should be light and should be taken
84. S1 : The oil found in natural state is called crude oil. three or four hours before going to bed.
S6 : Last of all, the lubricating oils of various grades are P : It is largely wasted.
produced. Q : A heavy meal at night before retiring is the fashion
P : It is used as fuel in heaters and lamps. with many.
Q : It is treated in refineries, the most common form of R : While sleeping, this food is converted into excess fat
treatment is heating. and thus makes a person fat and ungainly.
R : Gas that comes off the oil later is condensed into S : Three to five hours are needed to digest the food.
paraffin. The proper sequence should be :
S : When crude oil is heated, the first vapours to rise are (a) P Q S R (b) Q P S R
cooled and become the finest petrol. (c) Q S R P (d) S R Q P
The proper sequence should be : 89. S1 : I passed all the other courses that I took at my
(a) Q S P R (b) S P Q R university.
(c) S Q R P (d) Q S R P S6 : This used to enrage my instructor..
85. S1 : As a first step we have made the tribals celebrate the P : I never once saw a cell through a microscope.
Itu Kula festival on the same day. Q : This was because all botany students had to spend
S6 : Village-wise environmental status reports were several hours a week looking through microscopes at
plant cells.
prepared to help people in assessing the remaining
R : But I could never pass botany.
natural resources such as drinking water, extent of
S : I could never see through a microscope
grazing land, and number of fruit-bearing trees, bird,
The proper sequence should be :
animals etc.
(a) R Q S P (b) Q P S R
P : For the first time in the history of this region, during
(c) R S P Q (d) P Q S R
this festival, an Adivasa darbar was conducted.
Ordering of Sentence B-107
90. S1 : It is very warm and sticky today. The proper sequence should be
S6 : A good rain would cool things off a little. (a) S R P Q (b) Q R S P
P : That is a good idea. (c) P R Q S (d) S Q R P
Q: I wonder what the weather is going to be like tomorrow. 94. S1 : In ancient India the city of Ujjain was quite famous.
R : Let’s listen to the weather report for tomorrow on the S6 : So one can see what a great love all who care for India
radio. must feel for this ancient city.
S : The paper here says its going to be fair and sunny. P : Here lived at one time the poet Kalidas.
The proper sequence should be : Q : He was a famous learned astronomer.
(a) Q S R P (b) R Q S P R : And here also came and worked Raja Jai Singh of
(c) Q R P S (d) R S P Q Jaipur.
DIRECTIONS (Qs. 91-106) : In the following questions, each S : It was always renowned as a seat of learning.
passage consists of six sentences. The first and sixth sentence The proper sequence should be :
are given in the beginning as S 1 and S 6. The middle four (a) P S R Q (b) S P R Q
sentences in each have been removed and jumbled up. These (c) Q S R P (d) S R P Q
are labelled P, Q, R and S. You are required to find out the 95. S1 : One of the odd-looking birds of the ocean is the
proper sequence of the four sentences and mark accordingly penguin.
on the Answer Sheet. [2010-II] S6 : With the aid of flippers they can dive into the water.
P : Penguins cannot fly.
An example has been solved for you. Q : It is found in the cold regions of the Antarctic circle.
X. S1 : There was a boy named Jack. R : Their wings have been changed into swimming
S6 : At last she turned him out of the house. flippers.
P : So the mother asked him to find the work. S : They use them like oars.
Q : They were very poor. The proper sequence should be :
R : He lived with his mother. (a) P Q R S (b) Q P R S
S : But Jack refused to work. (c) R S P Q (d) Q R S P
The proper sequence should be 96. S1 : Clothes should fit well.
(a) R Q P S (b) P Q R S S6 : They absorb sweat from the body and pick up dust
(c) Q P R S (d) R P S Q from the surroundings.
Explanation : P : Clothing that is too slack chafes the skin and is
The proper sequence in this example is R Q P S which is uncomfortable.
marked (a) Therefore, (a) is the correct answer. Q : Tight garments can constrict the organs of the body,
91. S1 : When he joined the college, he was an adolescent and especially those of the abdomen.
immature. R : Clothes should be kept clean.
S6 : But his sagacity helped him get a good job. S : If they are too tight they can constrict the blood
P : He learnt a lot from his experiences at the college. vessels in the skin.
Q : He had to get suitably employed. The proper sequence should be :
R : Four years of study in the college changed him (a) P Q S R (b) Q R P S
completely. (c) S R Q P (d) S Q P R
S : When he left it he was ready to face the problems of 97. S1 : Get hold of the catalogues of the colleges in the United
life. States.
The proper sequence should be : S6 : They all aim at the first rate.
(a) P Q R S (b) S R Q P P : I think there is a common feature and that every course
(c) R P S Q (d) Q S R P given has a similar aim.
92. S1 : There is still another important characteristic of living Q : Is there any common feature in these courses ?
things. R : You will find courses in innumerable subjects.
S6 : And abrupt changes occur too, which are called S : Is there any aim which all of them have ?
mutations. The proper sequence should be :
P : One generation is not a perfect copy of the preceding (a) R Q S P (b) P Q R S
generation. (c) R Q P S (d) P R S Q
Q : Plants and animals are not exactly like their parents. 98. S1 : If we dump sewage into a stream, on a small scale, the
R : That attribute is the capacity to evolve. stream dissolves it and purifies it.
S : There is a continual realignment of inherited characteristics. S6 : For this overwhelming kind of pollution we need to
The proper sequence should be : coin a new term which we call super-pollution.
(a) P S Q R (b) R Q P S P : It can no longer deal even with the small quantity of
(c) Q S R P (d) R S P Q sewage which it once accepted without difficulty.
93. S1 : Your resources, like money are limited. Q : Ten miles downstream the water is pure again.
S6 : For that matter the need for a leader arises only in R : The system has broken down.
relation to other people. S : But if we dump large quantities of sewage, we end by
P : You must invest them wisely. killing the purifying bacteria and them the stream has
Q : Everything cannot be directly attended to by yourself. lost its power to purify.
R : Sooner or later you should learn to delegate your The proper sequence should be :
authority to others. (a) Q S P R (b) R S Q P
S : You have to depend on others. (c) P R Q S (d) P Q R S
EBD_7367
108
B- Ordering of Sentence

99. S1 : She was born into a poor family of labourers and died Q : His father established the first flax mill in South Island.
because poverty could not let her live. R : He was brought up in a real frontier atmosphere and it
S6 : So, while the world was celebrating International entered into the nature of the man.
Women’s Day, Rita’s mother, Mrs. Veena, spent the S : His mother was the first woman school teacher in New
day trying to get her daughter justice, which eluded Zealand.
her in life. The proper sequence should be :
P : She died not because she was ill, but because she had (a) P R Q S (b) Q S R P
been allegedly beaten by her husband and in-laws for (c) Q R S P (d) Q S P R
not bringing in sufficient dowry. 104. S1 : Isasc possessed a wonderful faculty of acquiring
Q : At the young age of twenty, Rita breathed her last at J. knowledge by the simplest means.
P. Hospital on February 20. S6 : Thus, even in his boyish sports, he was continually
R : Ironically, however, no one paid need to Rita’s woes searching out the secrets of philosophy.
when she used to cry for help. P : Yet nothing could be more simple.
S : The police have not taken any action either till date. Q : You will never guess how the boy could compel that
unseen wonder, the wind to tell him the measure of its
The proper sequence should be :
strength.
(a) Q P R S (b) S R P Q
R : For instance, what methods do you suppose he took
(c) P R S Q (d) R P Q S to find out the strength of the wind ?
100. S1 : We were so evenly matched that for a time the end S : He jumped against the wind and by the length of the
was difficult to tell. jump he could calculate the force of the wind.
S6 : Then his comrade’s knife, thrown at me, struck him on The proper sequence should be :
the back and, piercing it, quite finished him. (a) P Q R S (b) Q R S P
P : Already I was counting him a dead man and myself (c) R Q P S (d) P S Q R
victorious. 105. S1 : Life is a gift of God to man.
Q : I had only to wear him out to have him at my mercy. S6 : Oh Lord! What fools we mortals are ?
R : Presently, however, there came a change. P : What an invaluable gift ?
S : My opponent’s wild living made him incapable of Q : Every morning He fills 24 hours in wallets of each of
coping with, a prolonged bout and his strength seemed us and never asks questions.
to start ebbing away. R : The other is time.
The proper sequence should be : S : Irrespective of these gifts, we grumble.
(a) S R Q P (b) Q R S P The proper sequence should be :
(c) R Q S P (d) R S Q P (a) P Q R S (b) P R Q S
101. S1 : William Cowper was born in 1731 at the rectory of (c) Q R P S (d) R S P Q
Berkhamstead. 106. S1 : The common man has a vote in Parliament.
S6 : He left Westminster in 1748. S6 : For that, his sole resource is his native wit and will.
P : From both sides (mother and father) he was well- P : If he likes to make use of the machinery of a democracy,
connected. he can have questions asked in the house.
Q : He was the son of country rector. Q : But there is no machinery by which he can control the
R : Cowper was sent to a boarding house at the nearby organs which mould opinion.
village where the bullying brought on a nervous R : In the last resort he can destroy one government and
inflammation of the eyes. make another.
S : He then moved to Westminster School where he was S : He has a parliamentary representative whom he can
badger and heckle.
reasonably happy.
The proper sequence should be
The proper sequence should be : (a) Q P S R (b) S Q P R
(a) P Q R S (b) P R Q S (c) P R Q S (d) S P R Q
(c) R S Q P (d) Q P R S
102. S1 : John Young, the astronaut, piloted the 75-ton space- DIRECTIONS (Qs. 107-114) : In the following questions, each
shuttle, Columbia. passage consists of six sentences. The first sentence (S1) and
S6 : And it moved smoothly to a stop as the crew rushed (S6). The middle four sentences in each have been removed and
to greet it. jumbled up. These are labelled P, Q, R and S. You are required
P : The Columbia, however, landed safely at the Edwards to find out the proper sequence of the four sentences and mark
Air Force base in the Mojave desert. accordingly on the Answer Sheet. [2011-I]
Q : The space-shuttle circled the Earth 36 times. 107. S1 : Einstein was very simple in his ways of life and
R : It was dangerous because a re-usable craft was being indifferent to his astounding fame.
used now. S6 : So they went back to the Queen and informed her that
S : This flight was fraught with dangers. he had not come by the train.
The proper sequence should be : P : They could never imaging that this shabby man would
(a) P Q S R (b) S Q R P be Einstein himself.
(c) P R S Q (d) Q S R P Q : Once the queen of Belgium invited him to Brussels.
103. S1 : Rutherford was the son of a Scot’s emigrant to New R : The officials also expected to see somebody who
Zealand. would appear to be rich and aristocratic before them.
S6 : Life was hard, but it was adventurous. S : When he got down from the train at Brussels, he could
P : They had 12 children, of whom Rutherford was the not think that there were actually many gorgeously
fourth. dressed officials to receive him at the station.
Ordering of Sentence B-109
The proper sequence should be S : Even in the finest of world’s cities thousands of people
(a) P R S Q (b) R Q P S live in dreadful surroundings.
(c) Q S R P (d) S P R Q The proper sequence should be
108. S1 : Jagdish was tired after the long walk through the thick (a) Q R S P (b) S R P Q
jungle. (c) Q P S R (d) P Q R S
S6 : Here, to his satisfaction, he found that there were 112. S1 : Mental disability, whether mental retardation or any
hardly any mosquitoes. other developmental disability, is not the same as
P : As night fell he came to a swampy place near a lake, mental illness.
where he decided to camp. S6 : And the earlier the better.
Q : At last, in despair, he sprang into the branches of a P : A person with mental disability can certainly be helped
nearby tree and climbed to the top. to improve, but mainly in childhood.
R : But the place was so full of mosquitoes that he found Q : But mental disability has no cure.
it impossible to sleep. R : Mental illness has an identifiable cause and may be
S : He spread a blanket on the ground and stretched cured.
himself out on it. S : Not the best doctors nor all the medicines in the world
The proper sequence should be can cure mental disability.
(a) P S R Q (b) P R S Q The proper sequence should be
(c) P S Q R (d) P Q R S (a) P S Q R (b) Q P R S
109. S1 : What kind of India are we working for, and what kind (c) R Q S P (d) S R P Q
of world ? 113. S1 : A devastating earthquake had hit Mexico city.
S6 : Some small part of that dream has come true, but not in S6 : The condition of 1000 others was reported unsafe.
the manner I had imagined, and so much still remains. P : Fifty more were later judged dangerously close to
P : Surely not, if there has been any truth in us and in our falling.
Q : The quake’s force was measured at 7.8 on the Richter
professions.
scale.
Q : Here in this city of Allahabad my boyhood and youth
R : In four chaotic minutes, an estimated 250 buildings
were spent in dreaming dreams and seeing visions of
collapsed in downtown Mexico city.
India’s future.
S : It was the world’s most severe quake since the quake
R : Was there any real substance in those dreams, or were in Chile last March.
they merely the fancies of a fevered brain ? The proper sequence should be
S : Are hatred and violence and fear and communalism (a) S R P Q (b) Q S R P
and narrow provincialism to mould our future ? (c) P Q S R (d) R P Q S
The proper sequence should be 114. S1 : Undoubtedly, science has done wonders.
(a) P Q R S (b) Q R S P S6 : Thus, science, which was once considered a
(c) S P Q R (d) S Q R P destructive power only in war, must be recognized as
110. S1 : Pasteur began his fruitful scientific investigations when one also in its apparently constructive activities during
he was Professor of Chemistry at Strasburg. peace.
S6 : The pasteurization process which he prescribed for P : But in giving such an answer, our attention must not
wine and beer is used now to safeguard milk too. be taken up entirely by the danger from nuclear
P : He also found how germs causing changes in materials weapons and chemical warfare.
could be controled by heat. Q : The honest answer has to be, “Not always”.
Q : He made a thorough study of the wine industry in R : But has it stopped with wonders which are beneficial
France. to mankind ?
R : He thus saved the wine industry from ruin by his work. S : There is far greater real danger from the damage to the
S : After careful and extensive investigation he produced environment arising from the so-called peaceful uses
the germ theory of fermentation. of science.
The proper sequence should be The proper sequence should be
(a) P Q R S (b) Q P S R (a) R Q P S (b) S P Q R
(c) S Q P R (d) Q S P R (c) S R P Q (d) P Q R S
111. S1 : In democratic countries men are equal before law, and
DIRECTIONS (Qs. 115-122) : In the following question, each
have a voice in deciding how and by whom they shall
passage consists of six sentences. The first and the sixth sentence
be governed.
are given in the beginning as S 1 and S 6. The middle four
S6 : And they live like this not for fun, but because they
sentences in each have been removed and jumbled up. These
are too poor to afford another room.
are labelled P, Q, R and S. You are required to find out the
P : While some few people live in luxury, many have not
proper sequence of the four sentences and mark accordingly
even enough to drink and wear.
on the Answer Sheet. [2011-II]
Q : But with respect to the sharing of money — which
means the sharing of food and clothing, and houses 115. S1 : I will not be able to see you tomorrow.
and books and so on the system is still very unfair. S6 : At any rate I’ll see you later in the week.
R : There are many families of five or six persons who live P : There is only a simple laboratory test be done.
in a single room; they sleep and dress and wash and Q: Once I know what the diagnosis is, I can contact the
eat their meals; in this same room they are born; and in doctor.
this same they die. R : However, if you like, you can phone me.
S : We will then follow his instructions.
EBD_7367
110
B- Ordering of Sentence

The correct sequence should be : 121. S1 : The lion used to be very widely distributed in Africa
(a) S P R Q (b) R P Q S and Asia.
(c) S R P Q (d) R Q S P S6 : No hunting is permitted in such reserved areas.
116. S1 : James Watt used the power of steam to drive machines. P : There are special forest zones set aside for wildlife in
S6 : The jet engine is relatively more recent. various countries.
P : With petrol engines people were able to build motor Q : Indiscriminate killing by hunters has been the cause
cars and aeroplanes. of this drastic fall in their numbers.
Q : Then, many years later, the petrol engine was invented. R : Today they are a relatively rare species.
R : These provided quicker means of traveling. S : If the species survives at all, it will be only in national
S : His invention was used later by other clever men to parks.
give us the railway engine. The correct sequence should be :
The correct sequence should be : (a) Q S P R (b) R S P Q
(a) S Q P R (b) P Q R S (c) S R P Q (d) R Q S P
(c) P S R Q (d) Q S R P 122. S1 : Though the Finance Minister claims that prices have
117. S1 : Stalin sent General Zhukov to assume command in become stable, facts do not support his stand.
Leningard. S6 : This proves that the government is not properly
S6 : The battle for Leningrad was the fiercest ever fought. informed.
P : True, the city was prepared for street fighting. P : What is worse, in some places it is not available even
Q : At that time no one knew whether the city could be at the exorbitant price.
saved. Q : Now it is above 10 rupees.
R : No one knew the answer. R : Sugar is a glaring example.
S : But would the city’s defences hold? S : A month back its open-market price was 7 rupees per
The correct sequence should be kg.
(a) R S Q P (b) Q S P R The correct sequence should be :
(c) Q P S R (d) R Q P S (a) P R Q S (b) S R Q P
118. S1 : Bacteria in the mouth can cause bad odour and painful (c) P S R Q (d) R S Q P
tooth decay. DIRECTIONS (Qs. 123-128) : In the following questions, each
S6 : Toothpaste advertisements are truthful when they passage consists of six sentences. The first and the sixth sentence
advise us to brush regularly. are given in the beginning at S1 and S6 . The middle four
P : Children’s eating habits leave them prone to tooth sentences in each have been removed and jumbled up. These are
decay. labelled P, Q, R and S. You are requested to find out the proper
Q : They grow in food particles left between teeth. sequence of the four sentence and mark accordingly on the
R : They often eat sweets and other items between meals. Answer Sheet. [2012-I]
S : Therefore, regular brushing after every meal eaten at 123. S1 : There is no doubt that democracy is the best of the
home is essential. systems of government available to us.
The correct sequence should be : S6: It is the feature that puts democracy in a class by
(a) P Q S R (b) Q P R S itself among political systems.
(c) R S P Q (d) Q R P S P : For another, even an individual can, through appeal
119. S1 : The five-year plans are meant for the nation’s to the judiciary, prevent the government from doing
economic development. any injustice.
S6 : The most important is, certainly, failure on the family- Q : This means that, in a way, the people can exercise
planning front. some control over the rulers even during their period
P : What are the reasons for it? of rule.
Q : One way of measuring a nation’s development is by R : This right of the individual to secure justice even
finding out how much progress there has been in the against the powerful, government is even more
fight against poverty.. important than the right to vote.
R : The reasons are many. S : For one thing, it permits, if necessary, a periodical
S : In India, every five-year plan ends with more people change of those who govern the country.
coming under the poverty line. The proper sequence should be
The correct sequence should be : (a) RPQS (b) SQPR
(a) R P Q S (b) Q S P R (c) PQRS (d) SRPQ
(c) P R S Q (d) S Q R P 124. S1 : The authorities decided to streamline the procedure
120. S1 : Man’s growth from barbarism to civilization is for admitting students to the new course.
supposed to be the theme of history. S6 : The selection was made from this list in the order of
S6 : In many ways, therefore, man has not made very great priority.
progress. P : Those who performed badly at the interview were
P : Strong countries attack and oppress weaker ones. eliminated.
Q : But it is difficult to believe this ideal has been reached. Q : The candidates were first required to take a written
R : There is ample evidence of barbarism even today. test.
S : Individuals too exploit their fellows. R : A list of successful candidates at the written test was
The correct sequence should be : prepared in the ascending order of total marks.
(a) Q R P S (b) R Q S P S : The top fifty among those who qualified were called
(c) Q R S P (d) R P S Q for an interview.
Ordering of Sentence B-111
The proper sequence should be DIRECTIONS (Qs. 129-138) : In the following questions, each
(a) QRSP (b) PRSQ passage consists of six sentences. The first and sixth sentence are
(c) SRQP (d) RSQP given in the beginning as S1 and S6. The middle four sentences in
125. S1 : I was only 12 years of age then. each have been jumbled up. These are labelled P, Q, R and S. You
S6 : I needed real money. are requested to find out the proper sequence of the four sentences
P : But this was not enough even to buy comics or toys. and mark accordingly on the Answer Sheet. [2012-II]
Q : I earned a couple of dollars by doing chores around
129. S1 : A boy tried to discover how wasps find their way
the house.
home.
R : I could never buy an airgun or a bike in this way. S6: These wasps flew straight home.
S : I found a pressing need for money of my own. P: He carried them two miles away and let them go.
The proper sequence should be Q: First, he marked each of a number of wasps with a
(a) SQPR (b) PSQR white spot.
(c) SRPQ (d) QRPS R: He then put them into a paper box.
126. S1 : You ask me what I intend to do after graduation. S: To do this he carried out a test on them.
S6 : That would be a great relief to my parents. The proper sequence should be
P : My ambition is to become an engineer. (a) P S Q R (b) S R Q P
Q : I can complete this course in two years. (c) S Q R P (d) Q P S R
R : But I am not 'sure whether I shall get enough marks to 130. S1 : Rivers have played a great role in our lives.
qualify for admission to an engineering college. S6: That was how Sindhu was pronounced by some
S : If I don't take up engineering, I would like to do an foreigners.
M.Sc. in Physics. P: It is famous as the Indus valley civilization.
The proper sequence should be Q: Communication and trade depended on the rivers for
(a) PRSQ (b) SQPR centuries.
(c) QSRP (d) RPQS R : Who does not know that the very term India originated
127. S1 : In 1974, Mark and Delia ventured into the Kalahari from Indus ?
desert to study the ecology of the region. S : One of the earliest civilizations of the world flourished
S6 : She was moving towards them from about five metres on the river Sindhu.
away, her head swaying from side to side. The proper sequence should be
(a) S R Q P (b) R S P Q
P : He slowly lifted his head and surveyed the scene.
(c) Q S P R (d) P R S Q
Q : A soft groan startled Mark.
131. S1: The next day Hiralal woke early.
R : They had driven north the evening before, trying to S6: He could find nothing inside them except two letters.
locate the roars of a lion. P : Then he began to take out the bags containing the
S : His breath caught -- it was a huge lioness, silver coins and the currency notes.
The proper sequence should be Q : Three of the bags were empty.
(a) RQPS (b) RPQS R : After washing his face and hands he went back to his
(c) RQSP (d) QPSR bedroom.
128. S1 : In almost all developing countries, estimates indicate S : Suddenly his heart seemed to stop beating.
that every six seconds, one child dies and another The proper sequence should be
becomes disabled as a consequence of diseases that (a) Q S P R (b) R P S Q
can be prevented with immunization. (c) P Q R S (d) S P Q R
S6 : After each round of immunization during the IIP, all 132. S1: My friend had a dog.
computerized data were brought up-to-date. S6: He took it to the veterinary hospital.
P : At the start of the programme, local Anganwadi P: He was very much upset.
centres for pre-school children operating as a part of Q: Two days later it returned home.
the National Integrated Child Development Services R: Its right ear had been torn off.
Scheme were involved in conducting house-to-house S: One day he could not find the dog anywhere.
surveys. The proper sequence should be
Q : The experiment has been launched with the objective (a) R P S Q (b) P S Q R
of introducing new and efficient strategies for large- (c) S P Q R (d) S Q R P
scale immunization programmes which, if successful, 133. S1: A water tap is a device for turning on and off a flow of water.
S6: The washer is made of a flexible material such as
could be used as models for similar endeavours in the
rubber or plastic.
future.
P: The metal parts of a water tap are usually made of
R : Two areas were included in the IIP : Trilokpuri and brass because brass resists corrosion.
Khanjawala. Q: The other part is a washer which is fixed to the bottom
S: Between December 1983 and March 1984, a large-scale of the rod.
experiment – the Intensive Immunization Programme R: One is a rod with a handle on the top.
(IIP) – was conducted in Delhi. S: It has two important parts.
The proper sequence should be The proper sequence should be
(a) QSPR (b) RPQS (a) P Q R S (b) S R Q P
(c) PQRS (d) SQRP (c) P Q S R (d) P R S Q
EBD_7367
112
B- Ordering of Sentence

134. S1: Now these two types of courage, physical and moral, Q: Going home one evening, he found a poor woman
are very distinct. lying in the street and took her upon his back and
S6: Moral courage is a higher and rarer virtue than carried her to his house.
physical courage. R: Soon she was restored to health.
P: I have known many men who had marked physical S: Instead of harshly scolding her, he had her taken care
courage, but lacked moral courage. of with all tenderness.
Q: On the other hand, I've seen men who undoubtedly The proper sequence should be
possessed moral courage but were very cautious (a) P Q R S (b) S R Q P
about taking physical risks. (c) P R Q S (d) Q P S R
R: But I've never met a man with moral courage, who DIRECTIONS (Qs. 139-147) : In the following questions, each
wouldn't, when it was really necessary, face bodily passage consists of six sentences. The first and sixth sentence
danger. are given in the beginning as S 1 and S 6. The middle four
S: Some of them were in high places but they failed to be sentences in each have been jumbled up. These are labelled P,
great in themselves because they lacked it. Q, R and S. You are required to find out the proper sequence of
The proper sequence should be the four sentences and mark your answer accordingly on the
(a) P Q R S (b) P S Q R Answer Sheet. [2013-I]
(c) P S R Q (d) Q S R P
135. S1: Forests have an influence on the climate of a region. 139. S1 : Ronald Ross was born in Almora, in the Himalayas in 1857.
S6: Thus we see that forests in a region often make the S6 : Manson directed him to an effective study of the
climate cooler. disease and with his help. Rose solved the mystery in
P: Much of the rain that falls beneath the trees dissolves three years.
plant food in the soil. P : He began to feel that he ought to try to do something
Q: This gives to the air over the forests a coolness which about it.
is felt by balloonists and aviators three thousand feet Q : He was educated in England and returned to India as
above the earth. an officer in the Indian Medical Service.
R: The plants absorb all the food and much of the water, R : He started to study malaria and during a vacation to
but the rest of the water is breathed out through the England, met Patrick Manson and studied tropical
leaves into the air. diseases under him.
S: This is taken up by the roots of plants and carried S : His medical conscience was stirred by the appllying
upward to the leaves. disease and misery with which he was surrounded in
The proper sequence should be the course of his work.
(a) Q S R P (b) P S R Q The proper sequence should be :
(c) R S P Q (d) S Q P R (a) S R P Q (b) Q S P R
136. S1: Albert Edward did very well. (c) P Q R S (d) R P S Q
S6: He then appointed a manager to take care of it. 140. S1 : Science has turned the world into one unit.
P: He started looking for a place to open the new shop. S6 : Practically every part of the world has friendly or hostile
Q: He started his shop with great enthusiasm. relationship with every other part.
R: He did so well that he began to think of opening P : Now a days such pleasing illusions are impossible to have.
another shop. Q : Since that time they have been coming closer to each
S: One fine morning he found it on a nearby street. other.
The proper sequence should be R : Before the 16th century, America and the Far East were
(a) P Q R S (b) R S P Q almost unrelated to Europe.
(c) R P S Q (d) P S R Q S : Augustua in Rome and Han Emperor in China
137. S1: Why do birds migrate in spite of the heavy loss of life simultaneously imagined themselves masters of the
on the way ? world.
S6: The migration of birds thus is a fascinating study, The proper sequence should be :
indeed. (a) P Q R S (b) R S Q P
P: But birds also migrate during summer. (c) S R P Q (d) R Q S P
Q: Primarily they migrate during the summer. 141. S1 : Plants need carbon for building the tissue of their
R: Also they cannot endure the summer heat. bodies.
S: And the main reason now is not hunger but availability S6 : Thus through a complex process called photosynthesis,
of the nesting sites. plants receive their requirements from the soil and Sun.
The proper sequence should be P : The breaking up of carbon dioxide into its components
(a) Q P S R (b) P Q R S requires energy, which they derive from the Sun.
(c) R S P Q (d) Q R P S Q : Plants’ other needs of nutrients are derived from the
138. S1: Though hard to please and easily offended, Johnson soil and water through their roots.
had a most humane and benevolent heart. R : They derive this carbon from the carbon dioxide in the air.
S6: He got her a job and put her into a virtuous way of S : They break up the carbon dioxide, absorb the carbon
living. and discharge oxygen into the air for animals to
P: There he discovered she was one of those wretched breathe.
persons who had fallen into the lowest state of vice, The proper sequence should be :
poverty and disease. (a) Q S P R (b) R S P Q
(c) S Q R P (d) P R Q S
Ordering of Sentence B-113
142. S1 : I searched for my friend all day. S : You learn to write by writing.
S6 : When I woke up the Sun was already above the horizon. The proper sequence should be
P : Although I was weary and hungry, I was not (a) S R P Q (b) R P S Q
dicouraged. (c) S P R Q (d) Q P S R
Q : I erupt in and lay on the ground with my bag for a 147. S1 : Human ways of life have steadily changed.
pillow. S6 : During the last few years change has been even more
R : When, midnight came I felt that I could not walk much rapid than usual.
further. P : From that time to this, civilization has always been
S : At last I came to a place where the pavement was changing.
raised and had a hollow underneath. Q : About ten thousand years ago, man lived entirely by
The proper sequence should be : hunting.
(a) P R S Q (b) P S Q R R : Ancient Egypt - Greece - The Roman Empire - the Dark
(c) R P S Q (d) S R Q P Ages and the Middle Ages - The Renaissance - the
143. S1 : While on a fishing trip last summer, I watched an elderly age of modern science and of modern nations - one
man fishing off the edge of a clock has succeeded the other; and history has never stood
S6 : Cheerfully, the old man replied, “Small frying pan”. still.
P : “Why didn’t you keep the other big ones” I asked? S : A settled civilized life began only when agriculture
Q : He caught an enormous trout, but apparently not was discovered.
satisfied with its size, he threw it back into the water. The proper sequence should be
R : He finally caught a small pike, threw it into his pall, (a) Q S R P (b) Q S P R
and smiling happily prepared to leave. (c) R S Q P (d) S P R Q
S : Amazed, I watched him repeat this performance. DIRECTIONS (Qs. 148-165) items which follow : In the following
The proper sequence should be questions, each passage consists of six sentences. The first
(a) P S Q R (b) Q S P R sentence(S1) and the final sentence(S6) are given the beginning.
(c) P Q S R (d) Q P S R The middle four sentences in each have been removed and
144. S1 : Mr. Johnsen looked at his watch. jumbled up. These are labelled P, Q, R and S. You are required to
S6 : He always says to his friends at the office: ‘It is nice to
find out the proper sequence of the four sentences and mark
have breakfast in the morning, but it is nicer to lie in
accordingly on the Answer Sheet. [2013-II]
bed’.
P : He was late as usual, so he did not have time for 148. S1 : Her sister recognized her at once.
breakfast. S6 : As for the young prince, he found her more lovely and
Q : Then he washed and dressed. lovable than ever, and insisted upon marrying her
R : He ran all the way to the station and he arrived there immediately.
just in time for the train. P : She embraced them, telling them she forgave them with
S : It was half past seven and he got out of bed quickly. all her heart.
The proper sequence should be Q : Then she departed with the herald to the King's palace.
(a) P S Q R (b) S Q P R R : They were not in the least surprised, for everybody
(c) S R Q P (d) R S Q P believed in fairies, and everybody longed to have a
145. S1 : The essence of democracy is the active participation fairy godmother.
of the people in government affairs. S : She told her whole story to His Majesty and the royal
S6 : By and large it is the actual practice of our way of life. family.
P : When the people are active watchmen and participants, The proper sequence should be
we have that fertile soil in which democracy flourishes. (a) R S Q P (b) Q P S R
Q : Our democracy is founded upon a faith in the overall (c) P Q S R (d) P S Q R
judgement of the people as a whole. 149. S1 : No daily paper has ever found its way into this village.
R : When the people do not participate, the spirit of S6 : They carry this with them to the trading centres in the
democracy dies. plains and cities.
S : When the people are honestly and clearly informed, P : These travellers come from distant places.
their commonsense can be relied upon to carry the Q : On their return journey they have news from the hills.
nation safely through any crisis. R : The only news the inhabitants get is from travellers.
The proper sequence should be S : On their way into the hills they bring news from distant
(a) R P S Q (b) P R S Q plains and cities of India.
(c) S P Q R (d) P S R Q The proper sequence should be
146. S1 : Always remember that regular and frequent practice is (a) P Q S R (b) R P S Q
essential if you are to learn to write well. (c) Q S P R (d) R Q P S
S6 : If you keep your eyes and ears open, you will find 150. S1 : A stag was drinking water at a pool.
plenty of things to write about. S6 : He was thus caught by the hunters.
P : Even, with the most famous writers, inspiration is rare. P : He was admiring his shadow in the water.
Q : Writing is ninetly nine per cent hard work and one per Q : All of a sudden some hunters came to the pool.
cent inspiration, so the sooner you get into the habit R : From what he saw, he liked his horns, but he was rather
of writing, the better. unhappy about his legs.
R : It is no good waiting until you have an inspiration S : His legs helped him in getting away from the hunters,
before you write. but his horns were caught in a bush.
EBD_7367
114
B- Ordering of Sentence

The proper sequence should be R : Moreover, there was a storm almost every evening,
(a) Q P R S (b) P Q R S when everything had to be dragged hurriedly to the
(c) P R Q S (d) R P Q S shelter of the verandah.
151. S1 : The wise men laughed at Galileo for presuming to differ S : Working in the sun was very tiring.
from the great Aristotle. The proper sequence should be
S6 : Both struck the ground together, as he had asserted (a) Q R P S (b) Q P S R
that they would. (c) R P S Q (d) P R S Q
P : He them climbed to the top, carrying with him a ten- 156. S1 : In 1739, Nadir Shah, the mighty king of Iran, invaded
pound shot and a one-pound shot. India.
Q : So one morning he took some students and teachers S6 : Another trophy he took with him was the Peacock
to the base of the famous Leaning Tower. Throne built by Shahjahan.
R : Balancing them on the edge of the tower, he let them P : After a stay of two months, Nadir Shah went back to
fall together. Iran.
S : But Galileo said he could prove his statement. Q : He defeated the Mughal army in the battle at Karmal.
The proper sequence should be R : He took with him immense wealth as well as the Koh-
(a) S Q P R (b) Q P S R i-noor diamond.
(c) P R S Q (d) R S Q P S : This was followed by the cruel massacre and reckless
152. S1 : The earth was initially very hot and without an plunder of Delhi.
atmosphere. The proper sequence should be
S6 : But it contained a lot of other gases that are poisonous (a) Q S P R (b) R Q P S
to us. (c) P R Q S (d) S P R Q
P : The atmosphere came from the emission of gasoes 157. S1 : It was a very pleasant walk long the banks of the river.
from the rocks. S6 : It charmed me beyond measure.
Q : Because it contained no oxygen. P : The buffalo had huge horns.
R : In the course of time it cooled and acquired an Q : A kind of unknown calm and peace seemed to slide
atmosphere. into my soul.
S : This early atmosphere was not one in which we could R : What delighted me most was the sight of a little boy
have survived. sitting on the last buffalo in the herd.
The proper sequence should be S : I saw a herd of buffaloes swimming across the river.
(a) R P S Q (b) P S R Q The proper sequence should be
(c) S P Q R (d) Q R P S (a) S R P Q (b) Q S R P
153. S1 : When Madhu opened the living room, an extraordinary (c) P Q R S (d) R P Q S
sight met her eyes. 158. S1 : A reporter is someone who gathers and writes news.
S6 : Hurrying upstairs, she went to her dressing table but S6 : The reporter and the editor are both called newsmen.
to her relief found that the man had taken only an P : An editor is someone who prepares all the news for
imitation diamond necklace. printing in the newspaper.
P : She soon returned home in a police car with two Q : A reporter cannot do a good job unless he has a good
policemen. understanding of the requirements of the editor of his
Q : A strange man was fast asleep is an armchair. newspaper.
R : Madhu left the house immediately and went to the R : The editor decides how important each news story is
police station. and where it should be placed.
S : But it was now too late, the man had disappeared. S : Many editors are former reporters.
The proper sequence should be The proper sequence should be
(a) R P Q S (b) Q R P S (a) Q P R S (b) Q R S P
(c) P Q R S (d) S Q R P (c) P R Q S (d) R S P Q
154. S1 : When Galileo was young, people believed that the 159. S1 : Of men's earliest inventions we know very little.
earth was the centre of the Universe. S6 : Once man had fire, he was the master of all lower
S6 : But time has proved that Galileo's view was right. creatures.
P : But Galileo began to argue that it was not so. P : Man used stick and stone long before he dared to
Q : This belief was supported by the State and the Church. meddle with fire.
R : He said that the Earth and other planets moved round Q : The first was to use a stone to crack a nut.
the sun. R : The next was the use of a stick to strike an enemy.
S : He was imprisoned for voicing this unorthodox view. S : It was only a step further that he made a rude weapon
The proper sequence should be by fastening a stone to the end of stick.
(a) P Q R S (b) Q P R S The proper sequence should be
(c) Q P S R (d) P S R Q (a) P Q R S (b) Q R S P
155. S1 : Until the hospital could be built there was no surgery (c) S P R Q (d) S R Q P
available for the patients.
160. S1 : The detective walked into the dark room alone.
S6 : After a short time an emergency hospital was created S6 : He carefully picked up the gun making sure not to
from a rough fowl-house.
smudge the fingerprints
P : But there were disadvantages. P : It was later that he found the gun lying under chair.
Q : Schweitzer therefore started his medical treatment in Q : First he felt for the switch and turned on the light.
an open space outside his house.
Ordering of Sentence B-115
R : The furniture was smashed and the curtains torn. DIRECTIONS (Qs. 166-175) : In the following questions each
S : At once he saw the disorder and confusion in the passage consists of six sentences. The first and sixth sentence
room. are given in the beginning as S1 and S6 . The middle four
The proper sequence should be sentences in each have been jumbled up. These are labelled P,
(a) S Q P R (b) P Q R S
Q, R and S. You are required to find out the proper sequence of
(c) Q S R P (d) R P S Q
the four sentences and indicate your answer accordingly on the
161. S1 : Last night I was very tired and dropped off to sleep.
S6 : A goat was chewing up my rose plants. Answer Sheet. [2014-I]
P : I thought I heard somebody move outside. 166. S1 : The lions used to be widely distributed in Africa and
Q : All of a sudden, a slight noise disturbed my sleep. Asia.
R : I got out of bed quietly and peeped out of the window. S6 : No hunting is permitted in such reserved areas.
S : I was frightened. P : There are special forest zones set aside for wildlife in
The proper sequence should be various countries.
(a) Q P R S (b) Q S P R Q : Indiscriminate killing by hunters has been the cause
(c) S P Q R (d) R S P Q of this drastic fall in their numbers.
162. S1 : The parents of Bose wanted him to become an I.C.S. R : Today they are a relatively rare species.
Officer. S : If the species survives at all, it will be only in national
S6 : His resignation showed that his country's freedom was parks.
more important to him than his personal ambitions. The proper sequence should be :
P : But his heart was not for government service. (a) RSQP (b) SQRP
Q : He studied zealously and got through the I.C.S. (c) RQSP (d) SRPQ
examination in the fourth rank. 167. S1 : The woman who lives a normal life is able to check the
R : Soon he resigned from the I.C.S. to the bewilderment swelling conceit and egotism of her menfolk simply
of all. because her outlook is so different.
S : Very much against his will, he was sent to England. S6 : And both ranges of interest make her what only fools
The proper sequence should be
deny her to be, namely, essentially practical; her eye is
(a) Q R P S (b) R P S Q
steadily fixed on the concrete thing, and she mistrusts
(c) S Q P R (d) P S Q R
163. S1 : The train was running fast and the next station was that chasing of the wild goose which is one of the
nearly an hour ahead. chief pastimes and delights of man.
S6 : She bought four cups of ice cream and all of them P : She is primarily concerned with little ordinary things,
became busy with eating. the minutiae of talk and behaviour for example, on the
P : The children were pestering their aunt with one hand, and with very big ones, the colossal
embarrassing questions. elementary facts of life, such as birth, mating and death
Q : The occupants of the first carriage among others were on the other.
a tall girl, a little girl and a little boy. Q : The first are personal and particular; whereas the
R : Luckily an ice cream vendor came to rescue the aunt. second, those enormous facts about lifewhich women
S : Their aunt was occupying a corner seat. are never allowed to lose sight of, are, of course,
The proper sequence should be universal, meaning just as much in the Fiji Islands as
(a) P Q R S (b) Q S P R they do here.
(c) R P Q S (d) S R Q P R : Her interests are at once narrower and wider than those
164. S1 : One winter afternoon she had been buying something of men.
in a little antique shop in Curzon Street. S : It is more personal and yet more impersonal.
S6 : He clasped his hands, he was so gratified he could The proper sequence should be :
scarcely speak. (a) P Q S R (b) P R S Q
P : Therefore she visited this shop once again. (c) S P Q R (d) S R P Q
Q : It was a shop she always liked to visit. 168. S1 : What soda-water is composed of you may see for
R : He beamed whenever she came in. yourself if you watch your glass as it stands on the
S : And the man who kept it was very fond of serving her. table after you have slaked your first thirst.
The proper sequence should be S6 : 'Carbonic acid' is the old name for it, but it is more
(a) P S Q R (b) Q P S R correct to name it, when it is out of the water, 'Carbon
(c) P R Q S (d) Q S P R dioxide'.
165. S1 : My friend Todd owes me a dollar. P : The liquid is plain water, as you will find out if you are
S6 : I see that I shall never get it back. too slow about drinking.
P : Whenever I meet him, he does not show any indication Q : You will see that it is separating into two different
that he owes me a dollar. things, aliquid and a gas.
Q : My dollar has gone out of his mind. R : The gas is so heavy that you can fairly drink it from
R : Thus he meets me in the same frank friendly way as the glass, and it has, as you know, a tingle-tangle taste.
always. S : The other is a heavy, sour and invisible gas that slips
S : He has owed it to me for twelve months, and I fear up through the water in little bubbles and collects in
there is little prospect of his ever returning it. the empty half of the glass.
The proper sequence should be The proper sequence should be :
(a) R S P Q (b) S Q P R (a) QRSP (b) PRQS
(c) S P R Q (d) R Q S P (c) QPSR (d) RSPQ
EBD_7367
116
B- Ordering of Sentence

169. S1 : What Martin Luther King, the peaceful warrior and S : When the film is run through the film projector in the
his followers suffered, it is very difficult to describe. camera twenty-four pictures a second appear on the
S6 : For they had taken an oath to 'refrain from the violence screen.
of the fist, tongue or heart'. The proper sequence should be :
P : The police used fire hoses and ferocious dogs to rout (a) PSRQ (b) PSQR
them. (c) SRQP (d) RSQP
Q : The law courts sent them to solitary confinement where 173. S1 : Great quantities of animal oil come from whales.
not a ray of the sun entered. S6 : A few other creatures also yield oil.
R : They were abused and stoned by the mob, slapped P : It produces a great quantity of oil which can be made
and kicked by the police. into food for human consumption.
S : They suffered and tolerated all this without ever lifting Q : These enormous creatures of the sea are the largest
a hand in self defence. remaining animals in the world.
The proper sequence should be : R : When the whale is killed, the blubber is stripped off
(a) SRPQ (b) RPQS and boiled down.
(c) PRSQ (d) QRSP S : To protect the whales from the cold of the Arctic seas
170. S1 : There are, I think, several factors that contribute to nature has provided it with a thick covering of fat called
wisdom. blubber
S6 : You have not time to consider the effect which your The proper sequence should be :
discoveries or inventions may have outside the field (a) PSRQ (b) QSRP
of medicine. (c) PRQS (d) RPQS
P : This has become more difficult than it used to be owing 174. S1 : The distance between theatre and reality has stretched
to the extent and complexity of the specialized so far that when play, it is a cause for rejoicing.
knowledge required of various kinds of technicians. S6 : But the question is, have we forgotten his legacy in
modern India ?
Q : Of these I should put first a sense of proportion : the
P : It searches our collective psyche like an unrelenting
capacity to take account of all the important factors in
laser beam.
a problem and to attach to each its due weight.
Q : Most importantly, the play questions whether religion
R : The work is difficult and is likely to absorb the whole
and politics can fuse together in modern India.
of your intellectual energy. R : Gandhiji had both the spiritual and political dimensions
S : Suppose, for example, that you are engaged in research that we so lack today.
in scientific medicine. S : Prasanna's Gandhiji' staged recently by the National
The proper sequence should be : School of Drama is one such play.
(a) QPSR (b) QRPS The proper sequence should be :
(c) QSPR (d) QSRP (a) SRPQ (b) RSPQ
171. S1 : There were no finger prints anywhere. (c) SPQR (d) RQPS
S6 : These conclusions made the detectives think that it 175. S1 : We who live in the present day world are proud to call
was a fake theft. ourselves civilized.
P : First of all it was impossible even for a child to enter S6 : In fact science has added to our worries.
through the hole in the roof. P : But let us search our hearts and ask ourselves, 'Has
Q : When the investigators tried to reconstruct the crime, science solved our problem?'
they came up against facts. Q : Is it because we live and dress better than our
R : Moreover, when the detectives tried to push a silver forefathers ?
vase, it was found to be double the size of the hole. R :Frankly speaking, the answer is 'No'.
S : Again, the size of the hole was examined by the experts S : of course, we have the advantages of the inventions of
who said that nothing had been passed through it. science which our ancestors had never known.
The proper sequence should be : The proper sequence should be :
(a) PQRS (b) QPRS (a) PQRS (b) QSPR
(c) SQRP (d) QRSP (c) PRSQ (d) SPRQ
172. S1 : If you want to film a scene in slow motion you run the
DIRECTIONS (Qs. 176-183): In this section each questions
camera twice as fast as usual, which sounds ridiculous
consists of six sentences of a passage. The first and sixth sentences
but isn't.
are given in' the beginning as S1 and S6. The middle four
S6 : On the screen, everything appears at half the speed at
sentences in each have been jumbled up and labelled P, Q, R
which the camera recorded it when it was filmed.
and S. You are required to find the proper sequence of the four
P : If you are filming in slow motion, however, the camera sentences and mark your response accordingly on the Answer
runs at twice the normal speed, yet, in spite of this, the Sheet. [2014-II]
projector which shows the film will be run at the normal
speed, and this means that the projector will show the 176. S1 : There have been many stories of porpoises saving
film at half the speed at which it was photographed. human lives.
Q : This is because the camera which took the pictures S6 : Marine scientists point out that the porpoise's spirit
and the projector which shows them run at the same of play is responsible for such incidents.
speed. P : 'When I got to my feet no one was near, but in the
R : When a film camera is running at normal speed, it takes water about 18 feet out a porpoise was leaping about.'
twenty-four pictures a second. Q : One woman was wading waist deep off the Florida
coast when an undertow pulled her down.
Ordering of Sentence B-117
R : 'I felt something give me a terriffic shove up on to the S6 : As a result, the political scene witnesses endless dog
beach', she says. fights for power which threaten the very existence of
S : 'A man standing nearby said that the porpoise had democracy.
shoved me ashore.' P : These qualities are very rare and cannot be had for the
The proper sequence should be wishing.
(a) PRSQ (b) QPRS Q : For the right for power must, if it is to be useful, be
(c) QRPS (d) QPSR accompanied by the ability to exercise it with
177. S1 : Rome, the greatest city of the ancient world, did not competence, wisdom, foresight and broadmindedness.
achieve its glory all of a sudden. R : Yet all those who have the right to vote believe that
S6 : Achievement of great moment cannot be accomplished they have them and try by hook or crook to capture
without patient perseverance and a considerable power.
interval of time. S : The former has much to commend it but one cannot be
P : The same is true of every great achievement. so sure about the latter.
Q : We should carry on our work with patience and The proper sequence should be
perseverance. (a) PQRS (b) SQPR
R : It took several years to build Rome and bring it to the
(c) PRQS (d) RQPS
state of pomp and splendor.
182. S1 : There are several tribes in East Africa.
S : When we wish to do a great thing, we cannot expect
success in a moment. S6 : All the other tribes were afraid of them because of
The proper sequence should be their skill in war.
(a) RPSQ (b) PQSR P : The Masais were famous fighters.
(c) QSPR (d) SRPQ Q : They used to raid the neighbouring tribes and carry
178. S1 : The bus stopped. away their cattle.
S6 : Then his eyes rested with cold malice on the dog. R : They lived on the wide plains in Southern Kenya and
P : The conductor came in and took the fares. Northern Tanzania.
Q : A woman and a man got in together. S : But the most famous among them is the Masai tribe.
R : The young woman was carrying a pet dog. The proper sequence should be
S : They took their seats. (a) SPRQ (b) PRQS
The proper sequence should be (c) RQSP (d) QRPS
(a) PQRS (b) QSRP 183. S1 : I had my eye especially on the long jump.
(c) QPSR (d) QSPR S6 : He turned out to be a German named Luz Long.
179. S1 : James Watt used the power of steam to drive machines. P : Everyone expected me to win that Olympic event hands
S6 : The jet engine is relatively more recent. down.
P : With petrol engines people were able to build motor Q : I was in for a surprise.
cars and aeroplanes. R : When the time came for the long jump trials, I was
Q : Then many years later, the petrol engine was invented. startled to see a tall boy hitting the pit at almost 26 feet
R : These provided quicker means of traveling. on his practice leaps.
S : His invention was used later by other clever men to S : A year before I had set the world record of 26 feet 3
give us the railway engine. inches.
The proper sequence should be The proper sequence should be
(a) SQPR (b) PQRS (a) PQRS (b) PSQR
(c) PSRQ (d) QSRP (c) PRSQ (d) SRPQ
180. S1 : A man handed a pair of trousers to the departmental
store clerk and said, “I'd like these altered, please.” DIRECTIONS: (Qs. 184-193): In this section, each questions
S6 : Triumphantly he put the trousers and the receipts on consists of six sentences of a passage. The first and the sixth
the counter and said, “I'd like to have these altered, sentences are given in the beginning as S1 and S6. The middle
please.” four sentences in each have been jumbled up and labelled P, Q,
P : He said that free alteration is not possible without a R and S. You are required to find the proper sequence of the four
receipt. sentences and mark your response accordingly on the Answer
Q : The man said, “Okay, l'd like to return the trousers.” Sheet. [2015-I]
The clerk took them back and returned the money.
184. Sl : While teaching in the class-room, our teacher suddenly
R : The man pushed the money and said, "Now I want to
fainted.
buy them." The clerk put the trousers in a bag, issued
S6 : The headmaster at once sanctioned his leave.
a receipt and handed him both.
S : The clerk asked for the sales receipt but after searching P : The headmaster soon joined us and spoke to them in
his pockets the man replied that he had lost it. a soft voice.
The proper sequence should be Q : He was told that the patient needed complete rest for
(a) QRPS (b) SPQR a month.
(c) PSRQ (d) PSQR R : He was at once taken to the hospital.
181. S1 : It is generally assumed by the admirers of democracy S : The doctors examined him with serious faces.
that the right to vote also confers a right for power The proper sequence should be :
which threaten the very existence of democracy. (a) P Q R S (b) S P Q R
(c) Q P S R (d) R S P Q
EBD_7367
118
B- Ordering of Sentence

185. S1 : The colonial powers had a very simple technique to 190. S1 : When a lamb is born its mother may die.
rule the world. S6 : If a means of overcoming this natural tendency is
S6 : Partition was the culmination. found, the lives of millions of lambs can be saved.
P : They lumped tribes and people together, played one P : Thus there will nearly always be both motherless lambs
against the other. and sheep without lambs.
Q : India's provinces were more elaborately designed to Q : However a sheep which has lost its own lamb will not
play the game of divide and rule. feed or look after a motherless lamb.
R : Africa was divided, believe it or not, on the basis of R : At the same time some new born lambs are too weak to
the lines of longitude and latitude. live.
S : They also purchased the loyalties of those locals who S : This happens in large flocks where many sheep give
were needed as supports for the colonial presence. birth to lambs at the same time.
The proper sequence should be : The proper sequence should be :
(a) P R S Q (b) P S R Q (a) P Q S R (b) R P Q S
(c) S P Q R (d) R P S Q (c) S R Q P (d) S R P Q
186. S1 : The bank opened at 10.00 a.m. 191. S1 : People very seldom have everything they want.
S6 : The safe was empty. S6 : Our decisions indicate our scale of preferences and
P : The peon opened the safe and returned the keys to therefore our priorities.
the manager. P : Usually we have to decide carefully how to spend our
Q : The manager and the peon went to the safe in the income.
vault. Q : They may all seem important, but their true importance
R : The manager and the peon looked into the safe. can be measured by deciding which we are prepared
S : They were shocked at what they saw there. to live without.
The proper sequence should be: R : When we exercise our choice, we do so according to
(a) Q R P S (b) Q P R S our personal scale of preferences.
(c) S Q R P (d) Q R S P S : In this scale of preferences essential commodities come
187. S1 : The crowd swelled round the thief. first, then the kind of luxuries which help us to be
S6 : They were followed by the crowd which left the thief comfortable, and finally those non-essentials which
alone. give us personal pleasure.
P : Suddenly he whipped out a knife from under his shirt. The proper sequence should be :
Q : The thief stood quiet, his head hung in shame. (a) P S Q R (b) P R S Q
R : The two young men holding him were scared by the (c) Q P S R (d) R P Q S
sight of the shining knife. 192. S1 : On 5th October 1818, when young Lincoln was
S : They took to their heels. approaching his tenth year, his mother Nancy died of
The proper sequence should be : fever.
(a) Q P R S (b) S Q P R S6 : His total education at school comprised only about a
(c) S P Q R (d) R Q S P year during which he, however, managed to master
188. S1 : The old man wanted to cross the road. reading, writing, spelling and some arithmetic.
S6 : Holding him by hand the driver helped him to cross P : She was illiterate, but she brought with her several
the road. books, among which were Pilgrim's Progress, Sindbad
P : The driver got off and came to him. the Sailor, Robinson Crusoe and Aesop's Fables.
Q : He was fed up and was about to return. Q : Lincoln always acknowledged this moral and
R : Then a car stopped in front of him. intellectual debt to his step mother.
S : He waited for a long time. R : The following year, his father married Sarah Bush
The proper sequence should be : Johnson, a widow with three children.
(a) S Q R P (b) S P R Q S : These books provided Lincoln with a mass of
(c) Q R S P (d) P S R Q knowledge.
189. S1 : The first thing you have to do is to speak with a strong The proper sequence should be:
foreign accent and speak broken English. (a) R P Q S (b) P S R Q
S6 : Half a dozen people will immediately overwhelm you (c) R P S Q (d) P S Q R
with directions. 193. S1 : Crude oil obtained from the field is taken to a refinery
P : He will be interested in you because you are a foreigner for treatment.
and he will be pleased that he could figure out what S6 : Lubricating oils of various grades are obtained last of
you said. all.
Q : He will not expect you to be polite and use elaborate P : The gas that comes off later is condensed into parafin.
grammatical phrases. Q : This allows substances with different boiling points
R : Then every English person to whom you speak will at to be separated.
once know that you are a foreigner and try to R : The first vapours to rise when cooled provide the finest
understand you and be ready to help you. petrol.
S : If you shout, "Please! Charing Cross! Which way?" S : The commonest form of treatment is heating.
you will have no difficulty. The proper sequence should be :
The proper sequence should be : (a) S Q R P (b) R S P Q
(a) S R Q P (b) S R P Q (c) S R P Q (d) R P Q S
(c) R Q P S (d) R S P Q
Ordering of Sentence 119
B-

DIRECTIONS (Qs.194-203) : In the following items each P : I have heard of a father who, having offered to take
passage consists of six sentences. The first and the sixth sentence the baby out in a perambulator, was tempted by the
are given in the beginning as S1 and S6. The middle four sunny morning to pause on his journey and slip
sentences in each have been removed and jumbled up. These are into a public house for a glass of beer.
labelled P, Q, R and S. You are required to find out the proper Q : A little later, his wife had to do some shopping
sequence of the four sentences and mark your response which took her past the public house where to her
horror, she discovered her sleeping baby.
accordingly on the Answer Sheet. [2016-I]
R : Leaving the perambulator outside, he disappeared
194. S1 : Once upon a time there was a king who had a into the drink shop.
wonderfully nice garden. S : Indignant at her husband's behaviour, she decided
S6 : In the trees lived a nightingale that sang so sweetly to teach him a lesson.
that all who passed by stood still and listened. The proper sequence should be
P : In the middle of the garden there was a lovely forest (a) P Q R S (b) P R Q S
with tall trees and deep lakes. (c) P S Q R (d) P Q S R
Q : In this garden were to be seen the most wonderful 198. S1 : Human ways of life have steadily changed.
flowers with silver bells tied to them. S6 : Even if we try to do nothing. We cannot prevent
R : The garden was so large that even the gardener change.
himself did not know where it began and where it P : Ancient Egypt - Greece - the Roman Empire - the
ended. Dark Ages and the Middle Ages the Renaissance -
S : These bells always sounded so that nobody should the age of modern science and of modern nations
pass by without noticing the flowers. one has succeeded the other, the history has never
These proper sequence should be stood still.
(a) Q P R S (b) S P Q R Q : About ten thousand years ago. man lived entirely
(c) Q S R P (d) Q P S R by hunting.
195. S1 : One of the first things the learning of a new R : A settled civilized life only began when agriculture
language teaches you is that language comes from was discovered
the region of the unconscious S : From that time to this, civilization has alwavs been
S6 : The test of how much you know is : how much can changing
you say without having to think how you are going The proper sequence should he
to say it ? (a) Q R S P (b) Q P S R
P : What is often meant by "thinking in a language" is (c) Q S R P (d) P R S Q
really the ability to use it without thinking about it. 199. S1 : In our youth we are apt to think that applause and
Q : We grown-up people have to filter it through our publicity constitute success.
minds - a much more laborious process. S6 : So let us be initiated into the mysteries of maturity
R : That is why children learn a new language so and be taught how to resist and spurn the lure of
effortlessly : it comes straight from their instincts. hollow shows.
S : But we cannot say that we know a language, or P : The man who values the applause more than his
know' what we have studied of it, until we can use own effort has not outgrown his youth.
it instinctively. Q : It is our achievement or work which wins lasting
The Proper sequence should be rewards.
(a) S Q R P (b) R P S Q R : But these are only the trappings, the ephemeral
(c) P Q S R (d) R Q S P illusions.
196. S1 : For seventeen years she led a sheltered life in the S : One should concentrate on one is work knowing
convent. that applause will come unsought.
S6 : Two years later she left the Loreto Convent where The proper sequence should be
she had spent many happy and useful years. (a) S R Q P (b) P S R Q
P : Her heart went out to the people living there. (b) Q P S R (d) R Q P S
Q : In 1946 she asked for permission to work in the 200. S1 : My office sent an urgent email asking me to return.
slums. S6 : It was evening before I could sit and write to my
R : Then one day, while she was returning from an parents that I would he joining them soon
errand. she saw the slums of Calcutta. P : I immediately replied requesting a few days of grace
S : She felt she had found her second vocation, her as I had to book the return passage, pack and attend
real calling sundry matters before winding up my establishment
The proper sequence should be here.
(a) P R S Q (b) R P S Q Q : On the way. I went to the laundry and made sure I
(c) R P Q S (d) Q R P S would get my clothes in time.
197. S1 : Good memory is so common that we regard a man R : Then I rushed to the bank, collected all my money
who does not possess it as eccentric. and made reservations for my return journey.
S6 : She wheeled away the perambulator. picturing to S : From the shop next to it. I bought a couple of trunks
herself his terror when he would come out and find to dump my books and other odd articles so that I
the baby gone could send them away in advance.
EBD_7367
120
B- Ordering of Sentence

The proper sequence should be S6 : In fact, most of the wars in the beginning of humanity's
(a) P Q R S (b) P R Q S history were fought for the gain of territory.
(c) P R S Q (d) P S R Q P : The more land they had, the more they were satisfied.
201. S1 : Wordsworth knew the behaviour of owls in the night Q : The nature and the quality of the land they possessed
better than most of us know the ways of black birds were of great importance to them.
in day time. R : All ancient communities worked hard to produce food.
S6 : His great poetry owes much to the night. S : There was also a great desire among them to posses
P : Out of school there were no restrictions on the as much land as possible.
hours he kept. The proper sequence should be
Q : No poet ever had happier school days.
(a) R Q S P (b) Q R S P
R : He would skate by the light of the stars, snare
(c) S Q R P (d) P R Q S
woodcocks at dead of night, watch the sunrise after
a long rumble. 205. S1 : There were shots as I ran.
S : Throughout life he was an inveterate walker by S6 : Staying submerged was only too easy with so much
night. clothing and my army boots.
The proper sequence should be P : The water was icy, but I stayed until I thought my
(a) Q P R S (b) P S Q R lungs would burst.
(c) Q R P S (d) S Q P R Q : I tripped at the edge and went in with a splash.
202. S1 : Science has already conferred an immense boon R : The minute I came up I took a breath and plunged
on mankind by the growth of medicine. down again.
S6 : The general death rate in 1948 (10.8) was the lowest S : I ducked down, pushed between two men and ran for
ever recorded up to that date. the river.
P : It has continued ever since and is still continuing. The proper sequence should be
Q : In the eighteenth century people expected most of (a) SQPR (b) PRQS
their children to die before they were grown up. (c) SQRP (d) QSPR
R : In 1920 the infant mortality rate in England and 206. S1 : Why do the English travel ?
Wales was 80 per thousand; in 1948 it was 34 per S6 : For here, in cosmopolitan England, one is always
thousand. exposed to the danger of meeting all sorts of peculiar
S : Improvement began at the start of the nineteenth aliens.
Century, chielly owing to vaccination.
P : Besides, they are taught that travel broadens the mind.
The Proper sequence should be
Q : They do so mainly because their neighbour does this
(a) R P Q S (b) Q S P R
(c) S Q R P (d) P Q S R and they have caught the bug from him.
203. SI : The young traveller gazed out into the dismal R : Although they have now discovered the sad truth
country with a face of mingled repulsion and interest that whatever travel may do to the mind, it certainly
S6 : He quickly restored it to his secret pocket. broadens other parts of the body.
P : At intervals he drew from his pocket a bulky letter S : But, and perhaps mainly, they travel to avoid foreigners.
to which he referred, and on the margins of which The proper sequence should be
he scribbled some notes (a) RSQP (b) PRSQ
Q : It was a navy revolver of the largest size. (c) SQPR (d) QPRS
R : From the back of his waist he produced something 207. S1 : Jumbo, the famous 3.3 metre elephant was born in
which one would hardly have expected to find in Africa over a hundred years ago.
the possession of so mild- mannered a man. S6 : Before his death in September 1882, he had been seen
S : As he turned it slantwise to the light, the glint upon by over 20 million Americans.
the rims of the copper shells within the drum P : After disembarkation in New York, he was take in a
showed that it was fully loaded. procession to the place where he was to be kept.
The proper sequence should be Q : Another admirer was the famous American showman
(a) P Q R S (b) R P Q S Barnum who bought it for a huge sum in February
(c) Q P R S (d) P R Q S 1882.
DIRECTIONS (Qs. 204-217): In the following items, each R : Transported from his native land to London Zoo, he
passage consists of six sentences. The first and the sixth sentence became a favourite of Queen Victoria.
are given in the beginning as SI and S6. The middle four S : He made his transatlantic voyage aboard SS Assyrian
sentences in each have been and jumbled up. These are labelled Monarch.
P, Q, R and S. You are required to find the proper sequence of the The proper sequence should be
four sentences and mark your response accordingly on the (a) PQRS (b) SRQP
Answer Sheet. [2016-II] (c) PSRQ (d) RQSP
204. S1 : History shows that the growth of civilization depends 208. S1 : The bureaucrat and the social worker are men of totally
upon the gifts of nature, particularly the wealth yielded different orientations and styles of functioning.
by the soil. S6 : The world will be a better place to live in if they learn
a little from each other.
Ordering of Sentence B-121

P: The other is considered to be a man ever on the move. S6 : We now know that Gamma rays are a particularly fierce
Q: He is portrayed as a man fond of rules above all other form of X-rays.
things. P : His association with Rutherford lasted only two years,
R : The one is regarded as given sedentary habits, doing but that was long enough to change the whole face to
a lot of paper work. physics.
S : Driven by an urge to help others he is impatient with Q : He was just 23.
red tape and unnecessary delays. R : They found that thorium changed into a new element,
The proper sequence should be thorium X, and in the process gave off what was
(a) PQRS (b) RQPS apparently a gas and at the same time a third type of
(c) SRQP (d) QPSR ray, which they named after the Greek letter 'Gamma'.
209. S1: Universities are peculiar institutions. S : When he teamed up with Rutherford, they investigated
S6 : It is the most important institution in the complex thorium which, as Marie Curie had shown, was
process of knowledge creation and distribution. radioactive.
P: Traditionally elite institutions, the modern university The proper sequence should be
has provided social mobility to previously (a) PQSR (b) QPSR
disfranchised groups. (c) QPRS (d) QSRP
Q : The contemporary university stands at the centre of 213. S1 : The boy felt his way up the creaking stairs through
its society. thick darkness.
R : They have common historical roots yet are deeply S6 : He was just telling himself he was safe when the door
embedded in their societies. was flung open and the gaunt old man grabbed his
S : Established in the medieval period to transmit shoulder.
established knowledge and provide training for a few P : All he had to do was just get past the central door on
key professions, universities have become a primary the landing.
creator of new knowledge through basic research. Q : He stopped as the great clock below whined for a few
The proper sequence should be seconds and gave out a single, solemn stroke.
(a) PSQR (b) SQRP R : His eyes were raised to the faint moonlight that shone
(c) SPRQ (d) RPSQ above the landing.
210. S1 : At the age of eighteen Gandhi went to college, but S : He hesitated as the sound died down and then crept
remained for only part of the year. on, thinking that if they could sleep through that,
S6 : This was against his religion, and most of his relatives they would sleep through any noise he could make.
were against his going. The proper sequence should be
P : Soon after this, he was advised to go to England to (a) PQSR (b) RQSP
study to be a lawyer. (c) PRQS (d) RQPS
Q : Studies did not interest him and he did not do well. 214. S1 : Don't you think that the housefly is a nuisance?
R : It was difficult for him to leave India and go to a foreign S6 : When you see a fly rubbing its legs together, it is just
land where he would have to eat and drink with cleaning itself, and scraping off some of the material
foreigners. that has gathered there.
S : This would not be easy. P : For ages that's what man considered the fly to be–
The proper sequence should be just a nuisance.
(a) PSRQ (b) SQPR Q : It was discovered that these flies carried disease germs
(c) PRQS (d) QPSR that cause the death of millions of people every year.
211. S1 : Helen graduated in 1904 with special honours in R : But now we knew that the innocentlooking housefly
English. is one of man's worst enemies.
S6 : Her dress was torn and roses were snatched from her S : It makes an irritating buzzing sound, it annoys you
hat. when it crawls on your skin; and so on.
P : She was twenty-four years old. The proper sequence should be
Q : She was invited to the St. Louis Exposition in 1904 to (a) RQPS (b) PQSR
awaken worldwide interest in the education of the (c) QPRS (d) SPRQ
deaf- blind. 215. S1 : Iguanodon was one of the first dinosaurs to be
R : But on Helen Keller Day the crowds get out of hand. scientifically described.
S : Requests were already flowing in for appearance and S6 : This quadrupedal dinosaur lived about 70 million years
for magazine articles. ago and its fossils are found in many parts of England,
The proper sequence should be Europe, Asia and North Africa.
(a) QPSR (b) SPRQ P : These teeth formed a dental battery in which replacing
(c) PSQR (d) SQRP teeth were constantly growing.
212. S1 : One of the many young scientists who chose to throw Q : The fore limbs were slightly larger than other members
in their lot with Rutherford was an Oxford physical of its group known as Ornithischia.
chemist, Frederick Soddy.
EBD_7367
122
B- Ordering of Sentence

R: It had numerous teeth in the sides of the jaws, arranged S : The King told her to hold out the net to the sunbeams
in rows. while he sang a magic song.
S : It was about 40 feet in length. The proper sequence should be
The proper sequence should be (a) PSQR (b) RPSQ
(a) QSRP (b) SRPQ (c) QRSP (d) PQSR
(c) RQPS (d) PQSR 219. S1 : Ramu was in a great hurry.
216. S1 : A single device can heat fluids without requiring an S6 : Fortunately, it wasn't a very serious one.
electrical element. P : As a result of this carelessness, he met with an accident.
S6 : Because there is no electrical element, there is no risk Q : The examination was to begin in five minutes time.
of fire, so the system is suitable for dangerous R : He just rushed across the road.
environments. S : So he forgot to look to the left or right as he always did.
P : A heat exchanger transfers heat from the central cavity The proper sequence should be
to an air heater, a water heater or an industrial (a) R Q S P (b) S R P Q
processor. (c) R S P Q (d) Q S R P
Q : It has a container like the crank case of a car's engine, 220. S1 : Chandran was terribly angry.
which contains the fluid to be heated. S6 : Saro ran screaming.
R : Compression and friction at the nozzles heat the fluid P : His pen was in her hand and Chandran rushed at her.
so that the temperature in the central cavity rises Q : Just then his sister, Saro, appeared at the door-step.
steadily. R : His pen was missing from the place where he had left it.
S : A motor and pulley inside the container turn a rotor S : He thought it was lost.
wheel, which in turn forces the fluid again and again The proper sequence should be
through narrow nozzles into a central cavity. (a) PRSQ (b) RSQP
The proper sequence should be (c) PSQR (d) SQPR
(a) RPSQ (b) SPQR 221. S1 : One fine morning, I heard a sound of fire engines with
(c) QSRP (d) SRPQ loud alarm bells.
217. S1 : There is no doubt that the rules governing imports, S6 : It was a terrible scene, as huge flames of fire Were
manufacture and the use of pesticides need to be coming out of the house.
further tightened up. P : I thought, perhaps a house was on fire.
S6 : At the same time, better pollution control measures Q : They were regulating the crowd.
are needed to check the discharge of poisonous gases R : The policemen were standing around a big house.
and chemicals by a host of other industries which are S : I immediately rushed out to see what was happening.
equally responsible for poisoning our world. The proper sequence should be
P : But a more practical and feasible approach is required (a) PQRS (b) RSQP
to tackle this problem. (c) SRPQ (d) PSRQ
Q : No one can deny the importance of pesticides in our 222. S1 : The sun-dried stalks of rice had caught fire and burst
agriculture. into flames.
R : The proposed amendment is an attempt at doing this, S6 : Even those who were too feeble to keep up with the
but in the process it seems to have gone overboard is first rush were on their feet, eager to join the fire-
most cases. fighters. .
S : Even today, nearly a fourth of our crop is lost due to P : Then came most of the older people, and mothers
pests and weeds. with babies at their backs.
DIRECTIONS (QS. 218-237): In this section each item consists Q : And all the active women and girls followed them to
of six sentences of a passage. The first and the sixth sentences assist them in fighting the fire.
are given in the beginning as S1 and S6. The middle four R : All the young men and boys were soon on the spot.
sentences in each have been jumbled up and labelled P, Q, R S : Staring wildly at the blazing rice, the people of the
and S. You are required to find the proper sequence of the four village ran to extinguish the fire.
sentences and mark your response accordingly on the Answer The proper sequence should be
Sheet. [2017-I] (a) PSQR (b) QPRS
(c) SRPQ (d) SRQP
218. S1 : According to the legend it was during a famine in China 223. S1 : People's fear of snakes seems to be based on a series
many thousands of years ago that rice first came into of misconceptions.
the lives of the Asians. S6 : Yet people will insist that they cannot touch a snake
S6 : And the little girl gathered a harvest of rice. because of its sliminess.
P : The golden sunbeams dropped through the meshes P : The most common one is the conviction that all these
and transformed themselves into golden grains. creatures are poisonous.
Q : A little girl went fishing. Q : Another very popular idea is that these reptiles are
R : But instead of catching fish she netted the King of slimy to touch.
Frogs.
Ordering of Sentence B-123

R: Snakes are really dry and cold; not slimy like a wet P: In his experiment, he used two banana plants.
cake of soap but rather like a crocodile-skin handbag Q: The music was in fact a high-pitched humming sound.
to the touch. R: He gave both plants the same light, heat and water.
S : In fact, the non-poisonous ones outnumber the S: But for about an hour a day, one of the plants 'listened'
poisonous ones by about ten to one. to some music.
The proper sequence should be The proper sequence should be
(a) RQPS (b) PRQS (a) QSRP (b) PQRS
(c) PSQR (d) RQSP (c) PRSQ (d) QRPS
224. S1 : The first great discovery that man probably made was 228. S1 : Some students may feel that fast readers do not
that of fire. understand as much as slow readers.
S6 : The animals were not clever enough to learn anything S6 : This statement, however, needs to be made with
from this. caution.
P : Fires sometimes occur by themselves in the forests P : Some slow readers will have good comprehension and
perhaps by rubbing together of flints or something others poor.
else. Q : A quick glance at the scores will show that fast readers
Q : We light a fire now by a match. sometimes have very good comprehension and
R : In olden times fires were made by striking two flints sometimes poor.
against each other till a spark came for setting fire to a R : This fallacy can easily be disproved when you give
dry thing. the first reading test in a class.
S : But matches are quite recent things. S : In short, there is little relation between reading speed
The proper sequence should be and comprehension.
(a) PRSQ (b) QSPR The proper sequence should be
(c) PQSR (d) RPQS (a) RQPS (b) SQRP
(c) PQRS (d) QSRP
225. S1 : A common disease of the eye is conjunctivitis or 'pink
229. S1 : There isn't a cricketer worth his salt who does not
eye', which often occurs in school-children.
aspire for captaincy.
S6 : Students with this condition should be sent to the
S6 : It is against this background that any emotions
doctor.
surrounding the captaincy should be viewed.
P : This disease spreads rapidly among school-children.
P : Even parents are proud when their sons become the
Q : A white discharge also appears in the corners of the
captain.
eyes.
Q : At the lower levels, it is the best player who gets to
R : One or both eyeballs turn quite red and have a feeling
lead the team.
of irritation. R : In cricket, the greatest honour any player can get is
S : This discharge forms a crust which can often be seen the captaincy.
in the morning after the eyes have been shut all night. S : Right from their school days, boys dream of leading
The proper sequence should be the team.
(a) PSQR (b) RSQP The proper sequence should be
(c) RQSP (d) QSRP (a) PQRS (b) SRQP
226. S1 : My office sent a message with a terrific urgency (c) PQSR (d) SQPR
asking me to return. 230. S1 : Systematic and ceaseless efforts are being made to
S6 : It was the evening before I could sit and write to my tap scientifically the abundant solar energy available
parents that I would join them soon. in the country.
P: I immediately sent a message requesting a few days S6 : Installation of solar thermal systems and devices has
of grace as I had to book the return ticket and attend helped to save or generate energy to the extent of 350
sundry matters before winding up my establishment kWh per annum.
here. P : These include cooking, water heating, water
Q : On the way, I went to the laundry and made sure I desalination, space heating, etc.
would get my clothes in time. Q : A large number of applications in the area have already
R : Then I rushed to the bank, collected all my money become commercial.
and made reservations for my return journey. R : Efforts are also afoot to develop economically viable
S : From the shop next to it, I bought a strong box to solar collectors for high temperature applications.
dump my books and other odd articles so that I could S : A simple and common mode in solar energy utilization
send them away in advance. is solar thermal conversion.
The proper sequence should be The proper sequence should be
(a) QRSP (b) RQPS (a) QSRP (b) QPRS
(c) SPQR (d) PRQS (c) SQPR (d) RQSP
227. S1 : Harsten's theory was that plants definitely react to 231. S1 : The houses in the Indus Valley were built of baked
music. bricks.
S6 : He found that this plant grew faster and 70 percent S6 : They led outside into covered sewers which ran down
taller than the other plant. the side of the streets.
EBD_7367
124
B- Ordering of Sentence

P: This staircase sometimes continued upwards on to Q: After I complete the assessment of examination
the roof. papers, I plan to go and stay at our house in Lonavla
Q : Access to the upstairs rooms was by a narrow stone for at least a week.
staircase at the back of the house. R : Next week the terminal examination begins.
R : The drains were built in the walls. S : There is a lot of repair work that needs to be carried
S : The houses had bathrooms and water closets, rubbish out in the house.
chutes and excellent drainage systems. The proper sequence should be
The proper sequence should be (a) SQRP (b) QSPR
(a) SPQR (b) PSQR (c) RPSQ (d) RPQS
(c) QRPS (d) QPSR 236. S1 : A proposal to remove from circulation 5 paise coins
232. S1 : I remember, some years ago, the library of a famous has been given up by the Centre on advice from the
divine and literary critic, who had died, being sold. Reserve Bank of India.
S6 : Yet, he was a holy man and preached admirable S6 : It is, however, proposed to reduce the costs of minting
sermons. these coins by changing their metallic content.
P : Multitudes of the books had the marks of libraries all P : The Government had been thinking of removing from
over the country. circulation even the 10 paise coin.
Q : It was a splendid library of rare books, chiefly Q : The cost of minting a 5 paise coin is said to be 7 paise
concerned with seventeenth century writers. while the cost of minting a 10 paise coin is 10.5 paise.
R : Evidently, he was very possessive about the books R : Moreover their removal would cause tremendous
he borrowed. hardship to some people.
S : He had borrowed them and never found a convenient S : The RBI had opposed this saying that they figure
opportunity of returning them. largely in public transactions.
The proper sequence should be The proper sequence should be
(a) RPQS (b) QPSR (a) SRQP (b) QPSR
(c) SPQR (d) PSRQ (c) PSRQ (d) PQSR
233. S1 : Of course, it is silly to try to overcome fears that keep 237. S1 : It is common knowledge that people go after different
us from destroying ourselves. objects in the world to get happiness.
S 6 : The only fears you need to avoid are silly fears which S6 : He is conscious of the fact that happiness is within
prevent you from doing what you should do. and not without.
P : This is sensible. P : The wise man with a properly attuned mind is happy
Q : You wait until it is out of the way before crossing. with them, in spite of them and without them too.
R : You need some fears to keep you from doing foolish Q : Can a condemned prisoner, awaiting execution on the
things. morrow, relish food, however delicious ?
S : You are afraid of an automobile coming rapidly down R : But a little reflection will prove that in reality these
the street you wish to cross. sense-objects, by themselves, can never make a
The proper sequence should be person happy.
(a) P R S Q (b) R S Q P S : It is a folly to equate objects with happiness.
(c) R P S Q (d) P Q R S The proper sequence should be
234. S1 : Nobody likes staying at home on a public holiday - (a) RQSP (b) SQRP
especially if the weather is fine. (c) SPQR (d) RSQP
S6 : It was very peaceful in the cool grass - until we heard DIRECTIONS (Qs. 238-257) : In this section each item consists
bells ringing at the top of the hill. of six sentences of a passage. The first and sixth sentences are
P : We had brought plenty of food with us and we got it given in the beginning as S1 and S6. The middle four sentences
out of the car. in each have been jumbled up and labelled P, Q, R and S. You
Q : The only difficulty was that millions of other people are required to find the proper sequence of the four sentences
had the same idea. and mark your response accordingly on the Answer Sheet.
R : Now everything was ready so we sat down near a [2017-II]
path at the foot of a hill.
S : We moved out of the city slowly behind a long line of 238. S1 : Egypt lies in the nort-east corner of Africa.
cars, but at last we came to a quiet country road and S6 : The whole country depends on the water of the
after some time, stopped at a lonely farm. Nile.
The proper sequence should be P : Most of it is desert or semi-desert.
(a) PSQR (b) QSPR Q : It has very little rainfall.
(c) PQRS (d) SPQR R : It is four times as big as Great Britain in size.
235. S1 : The teaching work for the term is over. S : Only a twenty-fifth of the total area is cultivable.
S6 : For this, weekend trips do not suffice, and a longer The correct sequence should be
stay is necessary. (a) P Q R S (b) S R P Q
P : That will end on October 13 and the Diwali vacation (c) R P S Q (d) Q P R S
will begin from October 14. 239. S1 : In mechanical efforts, you improve by perpetual
practice.
Ordering of Sentence 125
B-

S6 : There is neither excuse not temptation for the 243. S1 : Science first began to become important after
latter. A.D. 1500.
P : He cannot go on shooting wide or falling short, S6 : Men read them, became inquisitive again, and
and still fancy that he is making progress. began to want to find things out.
Q : This is so because the object to be attained is a P : As a result of this, books came to be circulated.
matter of actual experiment in which you either Q : During the Middle Ages the coming of Science
succeed or fail. was hindered by the Church.
R : He must either correct his aim, or persevere in his R : In the middle of the fifteenth century, however,
error with his eyes open. the Turks captured the city of Constantinople and
S : If a man aims at a mark with bow and arrow, he the Greek books were scattered far and wide.
must either hit it or aim it. S : It considered free inquiry into the nature of things
The correct sequence should be to be wicked.
(a) P S Q R (b) R P S Q The correct sequence should be
(c) S Q R P (d) Q S P R (a) P Q S R (b) Q S R P
240. S1 : Isaac’s mother married again. (c) S R P Q (d) R P Q S
S6 : He had a set of little tools and saw of various sizes 244. S1 : Phatik was a mischievous boy of fourteen.
made by himself. S6 : Ultimately he distinguished himself as a scholar.
P : But he was known to be very clever at making P : It was then that Phatik’s uncle offered to take the
things. boy to Kolkata.
Q : She sent him to school. Q : She was much worried about his education.
R : Isaac was left to th e care of his good old R : His mother found it difficult to bring him up.
grandmother. S : Away from his home Phatik became sober and
S : In his early years Isaac did not appear to be a very industrious.
bright student. The correct sequence should be
The correct sequence should be (a) P Q R S (b) S R Q P
(a) R Q S P (b) Q R S P (c) R S P Q (d) R Q P S
(c) S Q R P (d) R P Q S 245. S1 : Whenever I met Baba Amte I was reminded of an
241. S1 : The examination system must be regarded as the anecdote my grandmother used to tell me.
chief wrecker of young nerves. S6 : He forgot that he had made it.
S6 : If I become a Vice-Chancellor, my first act would P : He once made an idol of God.
be to abolish all examinations in my university. Q : As the idol was nearing completion, the sculptor
P : It makes me jump out of the bed, all in a sweat. was becoming more and more withdrawn into
Q : It does this by building up a tension, for a part of himself.
the year, all through one’s youth. R : And the moment it was complete, he threw away
R : And after four decades, the same nervousness his chisel and hammer and bowed to the idol of
sometimes recurs to me in nightmares. God he had just created.
S : I remember the desperate nervousness that used S : There was a great sculptor.
to grip me from January to April every year. The correct sequence should be
The correct sequence should be (a) P Q R S (b) S R P Q
(a) R P Q S (b) R S P Q (c) S P Q R (d) Q P R S
(c) R Q P S (d) Q S R P 246. S1 : Ross sent an account of his work, together with
242. S1 : History is a subject that is so little valued today slide and specimens to Manson.
that it is almost impossible to win world fame as a S6 : Ross was elected a fellow of the Royal Society in
historian; yet that is exactly what Toynbee was 1901.
able to. P : They produced a profound sensation.
S6 : Among the civilizations that he studied was that Q : In July 1898, Manson described Ross’s results to
of India. the British Medical Association.
P : We usually think of history as a chronological R : The President of the Royal Society came to
account of the development of various states and Manson’s house and inspected Ross’s materials
empires under ruler. and said that ‘it was of remarkable interest and
Q : Toynbee’s view of history was different. value’.
R : He tries to find the pattern behind the birth, growth S : When Manson had finished, the whole audience
and decay of civilization. rose and cheered.
S : Though he used the recorded history of mankind, The correct sequence should be
but he was interested not merely in the chronology (a) R S P Q (b) P S R Q
of single states or group but in the rise and fall of (c) Q P S R (d) S P Q R
whole civilizations. 247. S1 : Civilization dawned when early man learnt how to
The correct sequence should be produce heat and energy by burning wood.
(a) P S Q R (b) Q S P R S6 : When they have been used, they cannot be
(c) S Q R P (d) P Q S R replaced.
EBD_7367
126
B- Ordering of Sentence

P : Then steam was used to produce electricity. R : Then, one must learn that error or selfishness will
Q : In this century great use has been made of oil and disgrace and endanger the rest.
natural gas and the use of atomic reactors also S : A sense of equanimity brings order and method
has provided another source of energy. into the life of the people.
R : Much later, the first industrial revolution was The correct sequence should be
based on the production of steam by burning coal. (a) Q P R S (b) R Q S P
S : But none of these fuels is renewable. (c) P Q S R (d) P R S Q
The correct sequence should be 252. S1 : Mr. and Mrs. Robert went home late last night.
(a) P R Q S (b) R Q S P S6 : Mr. Robert rushed to the police station immediately.
(c) R P Q S (d) R P S Q P : Somebody had broken open the lock.
248. S1 : Ghost is a subject which baffles everyone everywhere Q : To their dismay they found all their things missing.
throughout the world. R : They got into the house with a lot of fear.
S6 : Yet it is a subject which has held people spellbound S : When they reached home they found the front
and the belief in them continues to flourish. door open.
P : But human beings have always been curious to The correct sequence should be
know more about them. (a) R S P Q (b) S P R Q
Q : Needless to say, such attempts have proved to be (c) Q S R P (d) R Q P S
useless. 253. S1 : The miseries of the world cannot be cured by
R : There have been attempts even to photograph physical help only.
these creatures of darkness. S6 : Then alone will misery ease in the world.
S : Even after the advancement of science, the reality P : Let men have light, let them be strong an d
of ghosts remains a mystery till this day. educated.
The correct sequence should be Q : No amount of physical help will remove them
completely.
(a) Q R P S (b) S Q P R
R : Until man’s nature changes, his physical needs
(c) S P R Q (d) S Q R P
will always rise, and miseries will always be felt.
249. S1 : There have been two schools of thought which
S : The only solution is to make mankind enlightened.
deal with the errors of learners.
The correct sequence should be
S6 : Both views are popular today but the second is
(a) Q P R S (b) R Q S P
gaining ground fast.
(c) S P Q R (d) P Q R S
P : The philosophy of the second school is that errors
254. S1 : Aristotle worked under limitations.
are natural and they will occur in any learning.
S6 : The age was not a period of experiment.
Q : So errors, they say, is a sign of faulty teaching P : Physical events were mostly attributed to the
methods. intervention of God.
R : The first school maintains that if teaching methods Q : There had been little industrial invention in Greece,
are perfect, errors will never occur. perhaps because slave labour was cheap and
S : They argue that we should concentrate on how to plentiful.
deal with errors, instead of on method of teaching. R : The only equipment he had for his study was a
The correct sequence should be ruler and compass and some crude instruments.
(a) Q S R P (b) P S Q R S : The facts on which modern theories of science
(c) Q P S R (d) R Q P S have been based had not been discovered.
250. S1 : Down the stairway of the Holiday Inn hotel, I enter The correct sequence should be
the conference hall. (a) R P Q S (b) R S P Q
S6 : Some are leaning against the sidewall. (c) Q S R P (d) S Q R P
P : I take a seat in the back row as more chairs are 255. S1 : The bus sped along the road.
quietly slipped in for late comers. S6 : The dog wailed for a long time.
Q : The hall is already packed with delegates. P : But the bus could stop only after covering a few
R : Still quite a few people are left standing. yards.
S : Most of the delegates are executives of Indian or Q : It injured the dog in the leg.
Indo-US companies. R : The driver applied the brake.
The correct sequence should be S : Suddenly a stray dog ran on to the middle of the
(a) S Q R P (b) P R Q S road.
(c) S R Q P (d) Q S P R The correct sequence should be
251. S1 : A sportsman is noted for his sense of discipline. (a) S P R Q (b) S R P Q
S6 : Once discipline is accrued in the play field, it can (c) R P Q S (d) P R S Q
be applied and practised in other spheres of life. 256. S1 : The status of women in our country is, on the
P : The first lesson in discipline is to win without pride whole, far from high.
and to lose without bitterness. S6 : Education can lift women out of the depths of
Q : One is no longer swayed by the sudden gusts of misery and ignorance into which they have sunk.
passion. P : But the plight of women in villages is still miserable.
Ordering of Sentence 127
B-

Q : The educated women in cities enjoy equality with The correct sequence should be
the men folk. (a) R P Q S (b) P R S Q
R : The movement for the freedom and right of women (c) Q S P R (d) S P R Q
has certainly been steadily gaining momentum. 260. S1 : Tomorrow it will be a year since we lost our great
S : Their education has been thoroughly neglected. leader.
The correct sequence should be S6 : Though he is no more with us, the qualities he
(a) R P Q S (b) R Q P S possessed and the ideals he cherished remain with
(c) S Q P R (d) S P Q R us.
257. S1 : Hiuen-tsang became a Buddhist monk at the age P : To these he added a feminine sensitiveness to
of twelve and soon discovered that the Buddhist atmosphere.
texts available in China were insufficient. Q : He was involved in the major events of his time.
S6 : But he was on a quest and returned after a while to R : He participated in them all while maintaining the
his motherland with a rich collection of texts, highest standards of public conduct.
documents and relics. S : He was incomparably the greatest figure in our
P : Wherever he went, he was asked by the local rulers history – a man of dynamic force, intellectual power
and monks to stay in the place. and profound vision.
Q : He entered India through Kashmir, where he spent The correct sequence should be
some time in Srinagar. (a) P S R Q (b) R Q P S
R : He therefore decided to go on a pilgrimage to India (c) R P Q S (d) S P Q R
to collect further material. 261. S1 : It would be possible to adduce many examples
S : From India, he attempted to go to Ceylon, but gave showing what could be done with the limited
up the attempt. means at our ancestor’s disposal in the way of
The correct sequence should be making life comfortable.
(a) P Q S R (b) R S P Q S6 : I hope, in this essay, to make that connection
(c) Q S R P (d) R Q S P manifest.
DIRECTIONS (Qs 258-267) : In this section each item consists P : What have comfort and cleanliness to do with
of six sentences of a passage. The first and sixth sentences are politics, morals, and religion?
given in the beginning as S1 and S6. The middle four sentences Q : But look more closely and you will discover that
in each have been jumbled up and labelled P, Q, R and S. You there exists the closest connection between the
are required to find the proper sequence of the four sentences recent growth of comfort and the recent history of
and mark your response accordingly on the Answer Sheet. ideas.
[2018-I] R : They show that if they lived in filth and discomfort,
it was because filth and discomfort fitted in with
258. S1 : First and foremost, there are order and safety in their principles, political, moral and religious.
our civilization. S : At a first glance one would say that there could be
S6 : Nobody may come and break into my house and no causal connection between arm chairs and
steal my goods. democracies, sofas and the family system, hot
P : Thus in disputes between man and man, right has baths and religious orthodoxy.
taken the place of might. The correct sequence should be
Q : If today I have a quarrel with another man, I do not (a) P R Q S (b) R P S Q
get beaten merely because I am physically weaker. (c) Q S R P (d) Q S P R
R : I go to law, and the law will decide fairly between 262. S1 : To most people, the term technology conjures up
the two of us. images of mills or machines.
S : Moreover, the law protects me from robbery and S6 : It includes ways to make chemical reactions occur,
violence. ways to breed fish, plant forests or teach history.
The correct sequence should be P : The classic symbol of techn ology is still the
(a) R Q P S (b) S R Q P assembly line created by Henry Ford half a century
(c) Q R P S (d) P R S Q ago.
259. S1 : In democratic countries, men are equal before the Q : The invention of the horse collar in the Middle
law. Ages led to changes in agricultural methods and
S6 : And they live like this not for fun, but because was as such a technological advance.
they are too poor to afford another room. R : Moreover, technology includes techniques, as
P : While some few people live in luxury, many have well as the machines that may or may not be
not enough to eat, drink and wear. necessary to apply them.
Q : There are many families of five or six persons who S : This symbol, however, has always been
live in a single room. inadequate, for technology has always been more
R : But the sharing-out of money — which means the than factories and machines.
sharing-out of food and clothing and houses – is The correct sequence should be
still very unfair. (a) S P R Q (b) P S Q R
S : In this room they sleep and dress and wash and (c) R S P Q (d) Q S R P
eat, and in this same room they die.
EBD_7367
B- 128 Ordering of Sentence

263. S1 : I was the secretary of the Philosophical Society of Q : All his usual formalities of perfidy were observed
the Patna College. with scrupulous technique.
S6 : I have been to Kolkata many times since, but never R : No one could have expected that Hitler would
has it been more pleasant than that first visit. do it.
P : It was my first visit to the city and its impression S : A non-aggression treaty had been solemnly
on my mind was indelible. signed and was in force between the two countries.
Q : In that capacity, I once led a trip to Kolkata. The correct sequence should be
R : I felt I had landed in the midst of beautiful dream (a) R Q S P (b) R S Q P
world of a fairy land. (c) P S Q R (d) Q P S R
S : I saw the roads, the trams, the skyscrapers and 266. S1 : Roderick Usher has always been a quiet person
the magnificent shops at the Chowranghee lane. who talked little of himself.
The correct sequence should be S6 : In the part of the country where he lived, the
(a) Q P S R (b) P S Q R “House of Usher” had come to mean both the
(c) S R P Q (d) S Q R P family and its ancestral mansion.
264. S1 : Union finance ministry announced a series of P : Many of his ancestors had been famous for their
concessions to trade and industry last month. artistic and musical abilities.
S6 : Man ufacturers feel that prices of certain Q : Others were known for their exceptional generosity
components may not be brought down because and charity.
of the imposition of a 30 per cent duty where there R : Yet I did know that his family was an old one.
was none earlier. S : So I did not know too much about him.
P : Together, these will result in a loss of revenue of The correct sequence should be
Rs. 100 crore to the exchequer. (a) P Q R S (b) S R Q P
Q : Earlier, these were attracting customs duty varying (c) S P R Q (d) S R P Q
from zero to 100 per cent. 267. S1 : Mass production has increased the tendency to
R : The chunk of the relief of Rs. 60 crore has gone to view things as useful rather than delightful.
the electronics industry. S6 : Indeed a lowering of quality usually results when
S : Raw materials and piece parts now carry customs mass production is substituted for more primitive
duty of 30 per cent and 40 per cent and valorem methods.
respectively. P : These various things share nothing with the
The correct sequence should be buttons except money value.
(a) R S Q P (b) P R S Q Q : All the rest you wish to exchange for food, shelter,
(c) S Q P R (d) Q P R S and many other things.
265. S1 : At four O’clock this morning, Hitler attacked and R : Suppose you are a manufacturer of buttons :
invaded Russia. however excellent your buttons may be, you do
S6 : Under its cloak of false confidence, the German not want more than a few for your own use.
armies drew up in immense strength along a line S : And it is not even the money value of the buttons
which stretches from the White Sea to the Black that is important to you : what is important is profit
Sea. which may be increased by lowering their quality.
P : No complaint had been made by Germany of its The correct sequence should be
non-fulfilment. (a) P Q R S (b) R Q P S
(c) S P Q R (d) Q R P S

HINTS & SOLUTIONS


1. (b) The proper sequence is QPSR. tamper with any of the code to which he gained
2. (c) The proper sequence is SRPQ. access."
3. (b) The proper sequence is QRPS. 8. (a) So, the proper sequence should be "Another
4. (d) The proper sequence is PSRQ. significant change in the past five years have been
5. (a) The proper sequence is QPSR. the global resurgence of ethnic and religious
6. (c) The proper sequence is RSPQ. aspirations. This manifests itself in many different
7. (d) So, the proper sequence should be "Did anything ways. One is the 'new' nationalism that is thriving in
happen? Not much, according to Microsoft. But the the wake of the break-up of the Soviet Empire. No less
intruder did little more than peek and poke around. He than 15 new States have emerged in that region since
may have been some of its "Source code" the secret 1990. Another manifestation is the growth of ethnicity
recipe for its software. Yes, somebody invaded the or regionalism as catalyst for new political parties. This
firm's corporate network. He did not download or phenomenon is evident not only in Africa and Asia
but also in Europe, notably in Italy and Spain."
Ordering of Sentence B-129
9. (c) So, the proper sequence should be "Eighteen years study. Suddenly he heard shouting in the street, and
ago, Germany became whole and free again after forty he saw Rahman being led away between two
years of cruel partition. Not many contemporaries had policemen, followed by a crowd of curious boys. He
expected this to happen, or to happen within the span hurried out and stopped them to inquire what it all
of their lifetime. But on November 9, 1989, the Berlin meant. He learned that a certain neighbour had owed
Wall was miraculously breached. The Cold War was the Cabuliwallah some money, but had denied it. They
suddenly over, the Iron Curtain collapsed, communism had quarrelled, and Rahman had struck the man with
melted away. Within ten months after the Wall came his knife."
down, Germany had regained its national unity. On 16. (c) So, the proper sequence should be "An experiment
October 3, 1990, reunification was ceremoniously was conducted in England to study what happens to
enacted." the bodies and minds of people travelling at high
10. (d) So, the proper sequence should be "There are a number speeds. They were first kept under observation in
of bad habits which poor readers adopt. Most of these London. Then they were flown to America. It was found
involve using extra body movement in the reading that as they travelled from one time zone to another,
process. But extra body movements, such as pointing their blood pressure moved away from the normal.
with the fingers or moving the lips, do not help reading. Travelling also had an effect on the minds of these
Of course, there must be vigorous mental activity. In people. Psychological tests showed that their ability
efficient reading, the muscles of the eye should make to make decisions decreased quickly."
the only external movement. Young children and very 17. (b) So, the proper sequence should be "All the fossil fuel
poor readers often point with a finger at each word in that we use today came from green plants. It took
turn." hundreds of millions of years for those plants to
11. (d) So, the proper sequence should be "Materially change into coal, oil, and gas. "And now we are
advertisements do us no good. Spiritually they are burning it all up in just 100 or 200 years!" says Dr.
one of the worst avoidable evils. Our buildings are Calvin. We have nearly used up all our savings. "It is
covered with prints and pictures that distract and wear time for us now to begin living on our income", says
us. Architects might give their designs dignity or the Calvin. The income that Dr. Calvin is talking about is
beauty of pattern. The advertisements tread closely the sun's energy which living green plants capture
on their heels and destroy its effect. They have but and store up every day."
one requirement that their intrusion should be 18. (d) So, the proper sequence should be "The 'touch-me-
conspicuous." not' plant folds up its leaves when touched. How is
12. (c) So, the proper sequence should be "Both Rattan and the plant able to do this? It is only in recent years that
his son Moti were idlers and did not like to do any a possible answer has been found. At the lower end of
work. Rattan's wife had died long ago, Moti had each leaf is a tiny swelling, called the pulvinus. The
married in the preceding year. His wife had introduced pulvinus acts as the 'brain' or control centre of the
order and industry in the house. She would work leaf. The folding up of the leaves is controlled by the
herself to death and earn the daily feed for both of pulvinus."
19. (c) So, the proper sequence should be "A lower division
them. The result was that their idleness increased all
clerk who has been working in CPWD for the past 15
the more. In fact, they prided themselves on their
years, won Rs.1 crore in the recently launched
inactivity and idleness."
programme 'Kaun Banega Crorepati'. His fellow clerks
13. (b) So, the proper sequence should be "A hundred metres in the department approached him with proposals to
further along the trail Mahesh and Rohini stopped launch some joint projects. His relatives advised him
short. Two bear cubs were playing in the creek gully to leave service and start his own business. He listened
about 20 metres to their right. The day before, they to all patiently. But he shied away from both his friends
had seen a mother bear and two cubs. They had and relatives. Because the publicity that he received
shouted and waved and watched through binoculars was really disturbing him."
as the mother bear reared up and roared at them. They 20. (b) SRQP
had enjoyed the roaring of the mother bear as a So, the proper sequence should be "What is freedom?
distance of a kilometre and a half separated them. But Freedom is the right to choose. Without the possibility
now mother bear-perhaps the same grizzly bear could of choice and the exercise of choice, we are not human
be just over the ridge obscured by the bushes." beings but only inanimate objects. Fortunately, we are
14. (c) So, the proper sequence should be "In an ordinary now living in a world full of choice. There is so much
power station we burn fuel to get heat. This heat turns choice that we find it difficult to choose. Some people
water into steam. The steam is then made to turn a feel that we actually suffer from what may be called
turbine and through the turbine a generator. It is from choice fatigue."
the generator that we get electricity. In a nuclear power 21. (c) So, the proper sequence should be "I did not know
station we burn water into steam and then use the Nehru at all intimately. In fact, I had not even met him
steam in the same way. However, instead of getting many times. But his personality made an immediate
heat by burning fuel, we get it from the nuclear reactor." impression at my very first meeting with him. This
15. (b) So, the proper sequence should be "One morning, a impression did not change over the years. Nor was
few days before Rahman, the Cabuliwallah, was due the effect he made just an impression. It is more correct
to return to his country, Tagore was working in his to use the word captivation than impression."
EBD_7367
130
B- Ordering of Sentence
22. (b) The proper sequence should be "The life of early men can attack human beings for no reason at all, taking a
had advantages and disadvantages. One of the fisherman or swimmer by surprise in the water, where
advantages was that they were not overcrowded. They the man is somewhat helpless."
could roam for months without fear of meeting a 29. (c) The proper sequence should be "One day I went into
stranger. Then, they had enough physical exercise the water off the coast of Africa. I was floating at a
which made them healthy and active. They lived in shallow depth, without making a movement. I sighted
small tribes where everybody knew everybody else. a shark at short distance from me. Every muscle of my
On the whole, there was friendship and amity within body tensed. He launched towards me as hard and
the tribes." swift as a missile. I hurled at him the rubber fins."
23. (c) The proper sequence should be "My journey was to 30. (a) The proper sequence should be "The clerk read the
last for thirty-six hours. Every mile of the country statement loudly and clearly. We, the jury find the
through which the train was running, was interesting. defendant not guilty, provided he returns the mule.
Yet, I was not happy. The train would stop for breakfast, The judge brought down the gavel sharply and roared.
lunch and dinner. I had the carriage for myself. In the There is not such verdict in the law; the defendant is
steel trunk under the seat there was a bag containing either guilty or not guilty. He told the jury to return a
two hundred rupees that did not belong to me." lawful verdict. They returned in five minutes."
24. (c) The proper sequence should be "There was a check- 31. (a) The proper sequence should be "Some people prefer
post for passing vehicles at a village called Gobindapur, country life to city life. Life in the country is quieter,
a short distance from where the road to Dhanbad cleaner and less hectic. Country people live longer
branched off from the Grand Trunk Road. As I and generally seem to be healthier and happier.
approached it, I saw the barrier coming down slowly. I However, the city offers more excitement, a wide variety
slowed down and found a man coming forward with of activities and a chance to meet more people. The
pencil and book in hand. When I came sufficiently city also offers more privacy since neighbours are too
near, he moved back and the barrier was lifted. I told busy to be interfering in the affairs of others. For these
uncle that the man had taken down the number of the reasons more and more people are leaving the country
car, adding that it was a routine practice. He asked me to live in the city."
more than once if I was sure that the man had noted 32. (a) The proper sequence should be "When my car broke
the number down." down, I took it to the only mechanic available in our
25. (b) The proper sequence should be "The ancestors of town. He said it required some minor repairs and asked
whales, it is said, lived on land, for they still have me to collect it in the evening. I went there at the
slight traces of hind-legs. They are warm-blooded appointed time to collect it. But it just refused to start.
animals, and feed their babies as land mammals do. So I sat at the wheel and the mechanic and his helper
But ages ago, whales changed their home moving from started to push it. They pushed the car down one
the land to the sea. The whale is suited to live in water. street and up another and soon we had gone through
He is shaped like a submarine boat, with a tail turned most of the streets in the town but the car wouldn't
into a power paddle. He has flippers on his sides to start."
keep him balanced and layers of fat or oil under the 33. (d) The proper sequence should be "Dinner had been
skin which furnish heat and make the huge body light served - his daughter laid out the plates. Bread and
and buoyant." cheese - a simple subsistence at the end of a not-so-
26. (c) The proper sequence should be "Gopal worked as a simple life. He sat down gruffly, not saying a word to
labourer at the building site. Once while returning from her. She was just a child, only 14 - too young, too
his work Gopal stopped at an auction sale. No one simple to know to understand. She had already had
seemed to want a rough old box when it was put up for her meal and was standing by his side, not quite
sale. But Gopal made a bid and he got the box. There knowing what to do. Then, silently she left the table to
was no key to it and it seemed useless but Gopal took retire for the night - it was as if she had never been
it home. He unscrewed the lid and found a valuable there."
collection of old silver in it." 34. (c) The proper sequence should be "It was a bitterly cold
27. (c) The proper sequence should be "I had not seen my
night and even at the far end of the bus the wind cut
father for several years. I wrote him a note suggesting
like a knife. The bus stopped, and two women and a
a very early meeting. I met him late one evening in his
man got in together and filled the vacant places. The
flat. He listened to my story in silence. When he spoke,
his voice was soft but without warmth. His words sank conductor came in and took the fares. The younger of
deep into my heart." the two women was dressed in sealskin and carried
28. (b) The proper sequence should be "There is no reason one of those little Pekinese dogs that women like to
for the terror which the sight of a snake causes in most carry in their laps. Then his eyes tested with cold malice
people. Many more people are killed, much more on the beady-eyed little dog. I saw trouble brewing."
frequently by motor-cycles and cigarettes than by 35. (c) The proper sequence should be "The officer rose to
snakes. The majority of snakes are harmless. They are his feet, trembling. Pulling himself together, he ran
only too anxious to avoid human beings. Of the rapidly away from the cliff to a point a half-mile from
poisonous snakes, only those found in the sea are its foot. He expected to find the horseman somewhere
always dangerous. Being aggressive by nature, they there. He failed to find him there. He was disappointed.
A half-hour later he returned to camp."
Ordering of Sentence 131
B-

36. (c) The proper sequence should be "It was Saturday. My 56. (d) So, the proper sequence should be "Convention has
sister's two young children were at our house. They a necessary part to play in the life of everyone. In all
wanted to be taken out and we decided to take them to social affairs it prescribes more or less generally
the local zoo. A taxi carried us all to the zoo. We bought accepted rules of behaviour. It is necessary and useful
the tickets and entered the zoo. The children had for this reason. Of course, there is nothing absolute
already bought roasted gram and peanuts to get into a about conventions. They vary from country to country,
picnic mood." from age to age. Their purpose, however, is always the
37. (d) The correct sequence is RPSQ, marked by (d) same."
38. (a) The correct sequence is PQRS, marked by (a) 57. (b) The proper sequence should be "No one can seriously
39. (a) The correct sequence is PRSQ pretend to remain unaffected by advertisement. It is
40. (d) The correct sequence is QSPR impossible to turn a blind eye to the solicitous overtures
41. (c) The correct sequence is RSPQ to buy this or that article that fill our streets, newspapers
42. (b) The correct option is SRPQ and magazines. Most often they bank upon our sense
43. (d) The correct sequence is QRPS of fear for not doing or possessing this or that for our
44. (a) The correct sequence is SRPQ youth, beauty, health and security. Even in the sanctity
45. (d) The correct sequence is RSQP of our living rooms advertisers pounce upon their
46. (b) The correct sequence is QSPR helpless victims as they tune in to their favourite radio
47. (b) The correct sequence is QPSR or television programmes. No matter how hard we
48. (d) The correct sequence is RQPS resist, clever little tunes and catch-phrases seep into
49. (a) The proper sequence should be PRSQ our subconscious mind and stay there. Though they
50. (d) The correct option is PQRS seem so varied all these advertisements have one thing
51. (b) The correct sequence is RQPS in common: they make strong appeals to our emotions."
52. (a) The proper sequence should be "Long long ago there 58. (c) So, the proper sequence should be "A great part of
lived a king who was crude and very much like a Arabia is desert. Here there is nothing but sand and
savage. He had none of the grace and polish of his rock. The sand is so hot that you cannot walk over it
neighbours. He had learned some manners from his with your bare feet in the day time. Here and there in
Latin neighbours, but mostly he was barbaric, loud the desert are springs of water that come from deep
and ruff. He was a man of great fancies and even down under the ground. These springs are few and far
greater enthusiasm. Because he had so much authority apart, but wherever there is one, green grass very soon
as a king, he was able to force some of these fancies covers the ground all around it. Such place is called an
into reality. Or at least he tried to." oasis."
53. (a) The proper sequence should be "Our age is the age of 59. (d) So, the proper sequence should be "Many things
the specialist. Modern specialization had led to the about Konark seem shrouded in mystery. Long before
fragmentation of knowledge. Each one knows more the temple was built, sometime in the 13th century,
and more about less and less. We concentrate on some Konark was one of the five holiest places in Orissa. It
narrow field and forget the larger context in which we is, of course, a religious shrine. But here there also
can see the meaning of our own specialization. We seems to be a great emphasis on purely human
should not only be specialists but also have a sense grandeur. Why was it built? King Narasimha probably
of the meaning of life and of social responsibility. We had it built as much as a memorial to himself as he did
have to reckon with the spirit of science, understand in honour of the Sun-God."
its limitations and an outlook which is consistent with 60. (b) So, the proper sequence should be "Most of you
its findings." probably did not see Gandhiji at close quarters. He
54. (b) So, the proper sequence should be "He went to his had amazing qualities. One of these qualities was that
friends, but none of them was ready to help him now. he managed to draw out the good in another person.
He went away from the city and wandered round the The other person may have had plenty of evil in him.
countryside looking for food to eat. In the end he went But he somehow spotted the good and laid emphasis
to the house of a farmer and became his servant. He on that good. The result was that the poor man had to
had to look after the farmer's pigs. And when he was try to be good."
hungry he ate the food which was given to the pigs. 61. (d) So, the proper sequence should be "Everybody thinks
Now the Youngman understood that he had made a that this is the Age of Reason. The ordinary events of
mistake." life seem to support this view. But the behaviour of
55. (c) The proper sequence should be "Man, when first came, people in crises makes us doubt this. We must
must have been surrounded by great animals and he therefore avoid the development of such situation.
must have lived in fear of them. Today man is the Reason takes a backseat at such times. Obviously we
master of the world. He makes the animals do what he must reexamine the view that this is the age of Reason."
likes. Same he tames, same he eats and same he shoots 62. (b) So, the proper sequence should be "Our villages are
for pleasure. But in those days he was not the master not now as serene and peaceful as they had been
but a poor hunted creature himself trying to keep away twenty five years ago. Party politics had entered into
from the great animals. Gradually, however, man raised the peaceful villages forcing the villagers to live
himself and became more and more powerful till he polarized political lives. The village people now seldom
became stronger than any animal." live under the overall command of any single powerful
EBD_7367
132
B- Ordering of Sentence

headman. Local political leaders mostly control and of urban blight. Their progress is regarded with the
command them. One dangerous consequence of this mixture of dread and embarrassment reserved for the
is that they are often dragged into clashes even with approach of a cantankerous old aunt."
their own relations of different political belief. This 69. (a) So, the proper sequence should be "In England,
often vitiates the traditional tranquility of the rural way yesterday, villagers were amazed to see a number of
of living." youths floating over a field. Luckily, there were not as
63. (b) So, the proper sequence should be "People think that many casualties as there might have been. The youths,
poverty is a great curse and wealth is a source of it appeared, had inflated their fathers' umbrellas with
happiness in life. This is untrue. A life of poverty gives hydrogen and as a result were borne skywards. One
more genuine satisfaction than a life passed in affluence boy let go his umbrella and landed on a cow from a
which encourages dependence on others. Poor people height of about ten feet. Another ended up in a pond.
are free from the evils which surround the rich. They The others, however, returned safely to earth by
puncturing their umbrellas before they had risen very
have sympathy for one another and are more self-
far."
reliant. All great men were born poor."
70. (b) So, the proper sequence should be "A recent survey
64. (b) So, the proper sequence should be "Before man settled reveals that Indian Engineers play an immensely
down, his life was not governed by the seasons. Once significant role in the American software industry.
he settled down to the agricultural way of life, he had What catches the attention at the very outset is the
to be able to calculate the change of seasons on which number-over a million Indians work in the American
agriculture depended. So man began to measure time, software industry. And then, 28% of these hold a Ph.
and first he looked for convenient unit of measurement, one D the highest percentage among all ethnic groups
of which was the day. In order to calculate the passing involved in the software industry. Not only this, the
of time, it was necessary for man to record the number Indian Americans own almost 25% of the small
of days in each season. This was done by making a information technology firms in the US. Some dollars
mark for each day on a tree trunk. However, we do not might be flowing back but the best of brains are surely
know when man first began to divide the day into being lost to the nation. It is high time the leaders
twenty-four hours, and the hour into sixty minutes." addressed themselves to this issue seriously."
65. (c) So, the proper sequence should be "Imitation is not 71. (a) QRSP 72. (a) PRQS 73. (b) QSPR
civilization. As ass in a lion's skin never makes a lion. 74. (b) PRQS 75. (d) PQSR 76. (c) QPSR
Cowardly imitation never makes for progress. It is verily 77. (d) SQRP 78. (d) SQPR 79. (a) PRSQ
the sign of awful degradation in a man. When a man 80. (c) RSQP 81. (a) QPRS 82. (b) PSRQ
has begun to hate himself, then the last blow has come. 83. (a) QRPS 84. (c) SQRP 85. (c) QSPR
When a man has begun to be ashamed of his ancestors, 86. (c) RSPQ 87. (a) RQSP 88. (d) SRQP
the end has come." 89. (a) RQSP 90. (d) RSPQ 91. (c) RPSQ
66. (d) So, the proper sequence should be "Democracy, more 92. (c) QSRP 93. (c) PRQS 94. (b) SPRQ
than any other form of government, needs good 95. (b) QPRS 96. (d) SQPR 97. (c) RQPS
citizenship. Under absolutism or dictatorship, men are 98. (a) QSPR 99. (a) QPRS 100. (d) RSQP
forced into line. But in a democracy things are not so 101. (d) QPRS 102. (d) QSRP 103. (d) QSPR
simple. Freedom is the essence of democracy the more 104. (c) RQPS 105. (b) PRQS 106. (d) SPRQ
complete the democracy, the more complete the 107. (c) QSRP 108. (a) PSRQ 109. (c) SPQR
freedom. But it has to be freedom of service self chosen 110. (d) QSPR 111. (c) QPSR 112. (c) RQSP
and sometimes of sacrifice self-imposed. That is not 113. (b) QSRP 114. (a) RQPS
the instinct of the natural man; yet somehow that habit, 115. (b) The right sequence is, I will not be able to see you
has to be acquired." tomorrow. However, if you like, you can phone me.
67. (b) So, the proper sequence should be "In 1668 an There is only a simple laboratory test to be done. Once
important discovery was made. An Italian physician I know what the diagnosis is, I can contact the doctor.
named Redi decided to test the idea that worms were We will then follow his instructions. At any rate I'll see
generated spontaneously in rotting meat. He put some you later in the week.
rotting meat and fish in open jars and watched them. 116. (a) The correct sequence is – James Watt used the power
In time, he noticed that flies came and laid their eggs in of steam to drive machines. His invention was used
the meat and the maggots hatched from the eggs. later by other clever men to give us the railway engine.
When he covered the jars with muslin, he found that Then, many years later, the petrol engine was invented.
flies came and laid their eggs on the muslin, but not With petrol engines people were able to build motor
maggots developed on the meat. That was the cars and aeroplanes. These provided quicker means
beginning of the end of the theory of spontaneous of traveling. The Jet engine is relatively more recent.
generation of higher plants and animals." 117. (c) The correct sequence is – Stalin sent General Zhukov
68. (b) So, the proper sequence should be "There is not sound to assume command in Leningrad. At that time no one
more familiar in Calcutta than the clanking of its knew whether the city could be saved. True, the city
tramcars. All day and late into the night the trundling was prepared for street fighting. But would the city's
roll of a tram has become a symbol, a portent. It is also defences hold? No one knew the answer. The battle
an irritant. Slow, overcrowded and unwieldy, they are for Leningrad was the fiercest ever fought.
thought of as obstructionist and outmoded. For a 118. (b) The correct sequence is – Bacteria in the mouth can
growing body of opinion in the city, trams are a symbol cause bad odour and painful tooth decay. They grow
Ordering of Sentence B-133
in food practices left between teeth. Children's eating 125. (a) The correct sequence is SQPR. The sentence S1 talks
habits leave them prone to tooth decay. They often about the age of the person talking. The next sentence
eat sweets and other items between meals. Therefore, in continuation should be S because it tells that at age
regular brushing after every meal eaten at home is of 12, he had the urge to earn lots of money. The next
essential. Toothpaste advertisements are truthful when sentence should be Q because it tells that how he
they advise us to brush regularly. earned money by doing small chores. The next
119. (b) The correct sequence is – The five-year plans are sentence should be P as it tells that the money he
meant for the nation's economic development. One earned from such chores were not much and did not
way of measuring a nation's development is by finding even buy him comics or toys. The next sentence R is
out how much progress there has been in the fight in sync with S6 where he talks about real money to
against poverty. In India, every five-year plan ends buy airguns and bike.
with more people coming under the poverty line. What 126. (a) The correct sequence is PRSQ. The sentence S1 talks
are the reasons for it? The reasons are many. The most about what I intends to do after graduation. The next
important is, certainly, failure on the family-planning sentence should be P because here the author answers
front. that his ambition is to become an engineer. The next
120. (a) The correct sequence is – Man's growth from sentence should be R because it talks about his
barbarism to civilization is supposed to be the theme apprehension whether he will get the requisite marks
for admission into four year degree course. The next
of history. But it is difficult to believe this ideal has
sentence should be S where he tells the alternative of
been reached. There is ample evidence of barbarism
choosing a M.Sc course if he does not get admission
even today. Strong countries attack and oppress in engineering. The last sentence should be Q because
weaker ones. Individuals too exploit their fellows. In the author explains that this course will be completed
many ways, therefore, man has not made very great in two years time.
progress. 127. (a) The correct sequence is RQPS. The sentence S1 talks
121. (d) The correct sequence is – The lion used to be very about that Mark and Delia ventured into the desert to
widely distributed in Africa and Asia. Today they are a study the ecology of that region. The next sentence
relatively rare species. Indiscriminate killing by hunters should be R because it tells the directions that the two
has been the cause of this drastic fall in their numbers. have taken to find roar of a lion. The next sentence
If the species survives at all, it will be only in national should be Q because it tells that he heard a soft groan
parks. There are special forest zones set aside for that startled him. After listening the roar, he lifted his
wildlife in various countries. No hunting is permitted head to survey the scene. The next sentence thus
in such reserved areas. should be P. The last sentence S is in sync with S6
122. (d) The correct sequence is – Though the Finance and talks about lioness that was standing in front and
Minister claims that prices have become stable, facts coming towards them.
do not support his stand. Sugar is a glaring example. A 128. (d) The correct sequence is SQRP. The sentence S1 talks
month back its open-market price was 7 rupees per kg. about the survey and how many children can be saved
Now it is above 10 rupees. What is worse, in some from immunizations. The next sentence should be S
places it is not available even at the exorbitant price. because it is in continuation of the sentence and talks
This proves that the government is not properly about the immunization experiment that was conducted
informed. in Delhi. The next sentence should be Q because it
123. (b) the correct sequence is SQPR. The sentence S1 talks discusses about the objective of the experiment. The
about the democracy. The next sentence should be S next sentence should be R because it tells the areas
because it starts to tell about the advantage and 'one' that are covered under IIP. The last sentence P is in
is mentioned in the sentence. The next sentence should sync with S6 and talks about the working of the IIP.
be the explanation of the sentence, thus Q because 129. (c) The correct sequence is SQRP. The sentence S1 talks
the periodical change of government discussed in about how a boy tried to figure out how wasps find
sentence S means that people have the right to change their home. The next sentence should be S because it
talks about what he did to find out. The next sentence
the government as discussed in Q. The next sentence
should be Q because it tells the first step of how he
should be P because it gives another advantage of the
did the experiment. The next sentence should be R
democracy. After that the sentence R should come because it tells the next step that he put them into a
because it is its explanation. box. The last sentence P is in sync with S6 and talks
124. (a) the correct sequence is QRSP. The sentence S1 talks about how he took them two miles away and how these
about the procedure for admitting the students to new wasps came back to their home.
courses. The next sentence should be Q because it 130. (c) The correct sequence is QSPR. The sentence S1 talks
talks about the very first step of the procedure- written that river have played a great roles in our lives. The
test. Q should be followed by R because it is in next sentence should be Q because it talks about how
continuation of Q and talks about the successful rivers have played an important role. The next sentence
candidates of written test. The next sentence should should be S because it tells about how civilization was
be S because it talks about the students that have first to develop along river Sindhu. The next sentence
cleared the exam as discussed in sentence S are called should be P which tells about the name of the
for interview. The last sentence should be P because it civilisation. The next sentence should be R because it
is in sync with sentence S6 and talks about how the describes about how India got its name from Indus
students are eliminated from the interview process. that has been described in P.
EBD_7367
134
B- Ordering of Sentence
131. (b) The correct sequence is RPSQ. The sentence S1 tells the streets and took him home. The next sentences
that Hiralal woke up early the next day. The next should be P and S because they describe that how he
sentence should be R because it describes what he found that the poor woman has fallen into the state of
did after waking up. The next sentence should be P poverty and took care of her without scolding her.
because it describes what he did after he came back to The last sentence should be R because it tells that she
room i.e. took the bags of coins and notes. The next was soon restored to good health. And sentence S6
sentences should be S and Q because it describes completes the phrase that he got her a job.
that his heart beat stop as he realised the three bags 139. (b) The correct sequence is QSPR.
were empty. 140. (b) After the first sentence, the conversation reflects back
132. (c) The correct sequence is SPQR. The sentence S1 says to the 16th century so option R follows. After this
that my friend had a dog. The next sentence should be
sentence Q should follow as it tells how nations started
S because it tells that how one day he could not find
coming closer to each other. Then S should follow as
the dog. The next sentence should be P because it
describes that he was upset because his dog was it contrasts the situation with an example where
missing. The next sentence should be Q and then R as Romans and Hans believed themselves to be a global
it tells that how after two days the dog came back and leader. Last comes P where it is concluded that such
his ear was torn off. S6 completes the paragraph that illusions cannot exist today.
he took him to the veterinary doctor. 141. (b) The first sentence tells about need of carbon dioxide.
133. (b) The correct sequence is SRQP. The sentence S1 tells Next R should follow as it tells where this carbon
what a water tap is. The next sentence should be S dioxide comes from. S should follow next as it tells
because it describes a water tap that it has two parts. what happens with this carbon dioxide after being
The next sentences should be R and Q which describes absorbed. P comes next as it describes the secondary
the two parts of the water tap. It should be followed process associated with the breaking of carbon
by sentence P which describes the parts as described dioxide. After P, Q should follow as it sums up talking
in R. The sentence S6 describes about the second part about source of other nutrients. So the answer is RSPQ.
as described in P. 142. (c) The first sentence says that day came to an end over
134. (b) The correct sequence is PSQR. The sentence S1 talks searching for friend. R should be the next sentence as
about how the two types of courage- moral and it talks about commencement of midnight. The mental
physical are very distinct. And sentence S6 talks about condition and physical hardship is mentioned sentence
moral courage, thus it should be described later. The no. P at night his sleeping spot is talked about in
sentence after S1 should be P and S because it talks sentence S. how he accommodated himself there is
about physical courage. After that Q and R should mentioned sentence Q. thus the answer is RPSQ.
come because it describes moral courage. 143. (d) QPSR should be the correct option.
135. (b) The correct sequence is PSRQ. The sentence S1 tells 144. (b) First S should come as it says person comes out of
that forests have an influence on the weather. The bed after looking at the watch. Then Q should follow
next sentence should be P because it describes how as it hints at his getting dressed. P says that he had no
the process of food absorption starts wherein the trees time for breakfast. Finally comes R as it says that the
dissolves plant food in the soil. Then sentence S person dashes towards his destination.
should come because in continuation it describes how 145. (b) First comes P where importance of peoples' participation in
the roots absorb the food and water. Next sentence R a democracy is emphasized. Then comes R which
is used because it describes what happens next after contradicts the previous sentence. Then follows "S"
roots absorb food and send it the leaves. Last sentence which tells about the benefits of informing people in a
Q is in sync with the sentence S6 as both describes democracy. Then Q sums up the sentence saying about
about how the air is cooler above the forests. the faith of people in a democracy. So the answer is
136. (c) The correct sequence is RPSQ. The sentence S1 tells PRSQ.
that Albert Edward did very well. The next sentence 146. (a) Option (a) SRPQ is the answer. In s the sentence simply
should be R because it describes how well he did and says that we learn to write by writing. Then comes
how he started to open a new shop. The next sentence option R which supports previous sentence by saying
should be P because it tells that he started looking for that we should not wait to write in hope of an
a shop. The next sentences are S and Q because it inspiration. Then comes P which further supports the
describes how he found the shop on a nearby street argument saying that even famous writers don't sit
and opened a new shop. The sentence Q is in sync idle in search of an inspiration. Q gives statistical proof
with S6. for the above sentences.
137. (d) The correct sequence is QRPS. The sentence S1 asks 147. (b) Option (b) QSPR is the answer. Q starts the sentence
the question why do birds migrate despite of heavy with describing hunting about ten thousand years ago.
loss of life. The next sentence should be Q. The next Then follows S which tells about settled life in contrast.
sentences should be R and P because it describes the Then P further supports the theme saying that
reasons for Q. The last sentence should be S because civilizations have been changing since then. In the
it describes another reason for migration. last sentence R examples have been given.
138. (d) The correct sequence is QPSR. The sentence S1 talks 148. (c) Option (c) PQSR is the correct answer. Then comes P
about John and his qualities. The next sentence should i.e. after her sisters recognized her, she embraced them
be Q because it describes how he saw a poor lady on and forgave them. Then, comes Q which says she
Ordering of Sentence B-135
departed to the herald. Then comes S, which says she various such areas are reserved for animals in every
told the whole story to her majesty and the royal family. country. The last statement S6 syncs with P and explains
R follows next which says which talks about the people that no hunting is allowed in such reserved areas.
believing her story about fairies. 167. (d) The correct sequence is SRPQ. The sentence S1 talks
149. (b) For this item, the correct answer would be (b). the very about how a woman who lives normal life has a different
first sentence says that village people don't have access outlook. The next sentence should be S because it
to newspapers. The next would be R, which says they describes her outlook which is more personal and yet
only get news from travellers. Then comes P which more impersonal. The next sentence should be R
tells that these travellers come from distant places. because it describes her interests. The next sentence
Then comes S which says that they move into the hills should be P because it describes about the interests.
with news. Then comes Q which says that in return The last sentence should be Q because it is in sync
they take away news form the hills. Thus, the correct with statement S6 and talks about two interests that are
squence is RPSQ. described in S6.
150. (c) First comes P which says a stag was admiring his 168. (c) The correct sequence is QPSR. The sentence S1 tells
shadow in the pool. Then comes r which says that the that you can see what soda water is composed of by
stag was not happy with his legs. Then comes Q which looking at the glass. The next sentence should be Q
says that suddenly hunters came. Then comes s which because it describes the two distinct things it is made
of- a liquid and a gas. The next sentence should be P
says that he could run with the help of his legs but his
because it describes the liquid. The next sentences
horns got caught in the bush. Thus the answer is (c).
should be S and R, that explains about the gas part. S6
151. (a) When people laughed at Galileo, in S, he says he would
describes the name of the gas.
prove his statement. Then comes Q which says how 169. (b) The correct sequence is RPQS. The sentence S1 talk
he takes people to leaning tower of Pisa. Then comes about how Martin Luther King and his followers
P whereby he himself climbs up the tower. Then comes suffered is difficult to describe. The next sentence
R which says that he let them fall together. Thus the should be R and P because it describes how police
answer is a SQPR. abused them. The next sentence should be Q because
152. (a) The answer is (a) RPSQ first comes R then comes P it tells how court abused them. S should be the last
next follows S and last is Q. sentence because it is in sync with the S6 statement
153. (b) After S1 the next sentence should be Q then comes R that they tolerated and did not revert back because
Then follows P Last comes S. Thus the answer is qrps. they had taken a oath against violence.
154. (b) First comes Q then comes P then follows R and last 170. (a) The correct sequence is QPSR. The sentence S1 tells
comes S. that there are several factors that contribute to wisdom.
155. (b) The answer is (b), QPSR. First comes Q, then comes P. As given in all the options, the first statement is Q. The
P is followed by S and finally comes R. next statement should be P because it explains the
156. (a) The answer is (a), QSPR. First comes Q which is meaning of statement Q. the next statement should be
followed by S, then comes P and lastly comes R. S that is an example and then R should come because it
157. (b) The answer is QSRP. First comes Q then it is followed explains the example. Also sentence S6 is in sync with
by S then comes R and finally comes P. R.
158. (a) The answer is (a) QPRS. First comes Q then comes P 171. (b) The correct sequence is QPRS. The sentence S1 talks
next is R and lastly comes S. about that there were no finger prints anywhere. The
159. (b) The answer is (b) QRSP. First comes Q then comes R next statement should be Q because it is in continuation
then arrives S and last is P. of the S1 and describes that the investigators tried to
160. (c) The answer is (c) QSRP. The first is Q then comes S reconstruct the crime scene. The next statement should
then follows R and finally comes P. be P and R because it concludes that it is impossible
161. (b) The answer is (b) QSPR. The first sentence is Q then even for a child to enter through the hole and the silver
comes S. then comes P and last is R. vase was found to be double its size. The next sentence
162. (c) The answer is (c) SQPR. First comes S then comes Q S is in sync with S6 and explains that size of the hole
then comes P and finally comes R. was again examined and the theft was said to be fake.
163. (b) The answer is (b) QSPR. First comes Q then comes S 172. (d) The correct sequence is RSQP. The sentence S1 tells
then follows P and lastly comes R. how to film slow scenes. It tells how a film camera should
164. (b) The answer is (b) QPSR. First comes Q then comes P role. The next sentences should be R and S because
then comes S and lastly comes R. they are in continuation and describe the way camera
165. (c) The answer is (c) SPRQ. First comes S then comes P works in normal speed. Sentence Q should be the next
then follows R and lastly comes Q. because it describes how it works. The next sentence
166. (c) The correct sequence is RQSP. The sentence S1 talks should be P because it tells how it works in the slow
about that the lions used to be widely distributed in motion.
Asia and Africa. It talks about past tense. The next 173. (b) The correct sequence is QSRP. The sentence S1 tells
sentence after S1 should be R because it contrasts the that great quantities of animal oil come from whales.
statement with the situation today. The next statement The next sentence should be about whales only, thus
should be Q which gives the reason for the drastic fall Q. The next statement should be S and R that are in
of number of lions. S and P should follow the Q sync with each other and talks about the blubber of the
statement because it draws the outcome that if the lions whale. P should precede sentence S6 because it talks
survive, that can be done only in national parks and about the oil yield.
EBD_7367
136
B- Ordering of Sentence

174. (c) The correct sequence is SPQR. The sentence S1 talks follow because it is in continuation of sentence Q
about how the reality gap between theatre and plays regarding petrol engines. The last sentence should be
have become vast and how it's a matter of joy if a R because it describes how invention of motor vehicles
contemporary play is staged. The next statement should have eased the transportation.
be S because it describes about a recently staged play- 180. (b) The correct sequence is SPQR. The sentence S 1
Prassana's Gandhiji. The next sentences should be P describes about that man asks clerk to get his trousers
and Q because it describes the meaning of the play. altered. The next sentence should be S because usually
The last statement should be R because it is sync with when the salesman ask for a fee receipt in case of
S6 and talks about Gandhiji's legacy. exchange and alterations. The next sentence should be
175. (b) The correct sequence is QSPR. The sentence S1 talks P because it is in continuation of the last sentence
about how people in present day are proud to call where the man replies to the salesman that he does not
themselves civilized. The next statement is Q because have the receipt. The next sentence should be Q wherein
it questions the statement S1 that being dressed up because after the salesman denies alteration without
and living properly means civilised. The next statement receipt, the clever man decides to return it to and get
should be S because it highlights that science has given his money back. The last sentence should be R which
us inventions which our forefathers didn't had. The precedes S6 and describes how the clever man returned
next statements should be P and R that are in sync with the trouser and bought it back with.
each other and also S6 and describes how science is 181. (b) The correct sequence is SQPR. The sentence S 1
not a boon but a bane also. describes about right to vote and right for power. The
176. (c) The correct sequence is QRPS. The sentence S1 talks next sentence should be S because it describes about
about there are many stories of porpoises saving human the two rights and tells that right to power does not
lives. The next sentence should be Q because it starts always exists. The next sentence should be Q because
describing a story about the porpoises (fish) saving a it describes about the qualities of right for power. The
life of a woman and how suddenly she felt a pull sentence following Q is P because it is in continuation
downward in the water. The next sentence should be R and also talks about the qualities of right for power.
where the woman tells how something pushed her up. The next sentence should be R which is in sync and
The next sentence should be P describing that when continuation of S6.
the woman came on shore, she saw no body who would 182. (a) The correct sequence is SPRQ. The sentence S 1
have helped her but a porpoise who was jumping and describes about the many tribes present in East Africa.
taking a leap in the water. The last sentence should be The next sentence should be S because it tells that
S because it tells that man told the woman that she was among the many tribes described in S1, the most famous
helped by a porpoise. is Masai tribe. Next sentence should be an introduction
177. (a) The correct sequence is RPSQ. The sentence S1 talks to the Masai tribe, thus it should be P. The next sentence
about that the ancient Rome did not got its glory should be R that is in continuation of describing the
suddenly in a day or two. The next sentence should be Masai tribe. The last sentence should be Q because it
R because it tells that it took several years to build the describes that Masai tribes used to raid people. The
architecture and city of Rome. Now this thing is related last sentence S6 is in sync with Q and gives the reason
to how every great thing takes time. Thus the next why all the tribes were scared of Masai tribe
sentence should be P. the sentence following P should 183. (b) The correct sequence is PSQR. The sentence S 1
be S because it describes how to achieve something describes that the man had his eyes set on winning the
we should not expect miracles suddenly. The last long jump. The next sentence should be in continuation
sentence should be Q because it completes the previous and thus it should be P which tells that everyone also
sentence that without expecting miracles in a day, we expected him to win. The next sentence should be the
should work with patience and perseverance. reason why everyone expects him to win. Thus the
178. (b) The correct sequence is QSRP. The sentence S1 talks next sentence is S. Sentence S6 and R are in sync with
about a bus being stopped. Next sentence should be each other and talks about the German long jumper.
the one in continuation and describing what happened Thus R should precede sentence S6. And the sentence
next. And therefore the next sentence should be Q. The Q should follow sentence S.
sentence following Q should be S because it describes 184. (d) The correct sequence is RSPQ. The sentence S1 tells
further events that both the man and woman took their us that the teacher suddenly fainted in the class. The
next sentence should be R because it talks about the
seats. Now as we can see, the sentence S6 talks about
he was taken to the hospital and is in continuation of
conductor and so does the sentence P. Hence P should
the S1. Next sentence should describe what happened
come before the sentence S6. And sentence R should
in the hospital. Thus the correct option is S. The next
follow sentence S because it talks about the woman
sentence should be P because it describes what
who we have described in the sentence S. happened next i.e. the headmaster came. It should be
179. (a) The correct sequence is SQPR. The sentence S 1 followed by Q because it describes that the headmaster
describes about James Watt and his invention. The was told that the teacher needed rest and he gave him
next sentence should be S because it syncs the leave as described in S6.
invention of James Watt and how other men have 185. (b) The correct sequence is PSRQ. The sentence S1 talks
invented railway engines. The next sentence should about a technique used by the colonial power. The
be Q because it describes about the other related next sentence should what technique, hence the
invention of petrol engine. After that sentence P should sentence P. the next sentence should be S because it
Ordering of Sentence B-137
describes what else they did. The last sentence S6 talks priorities. The last sentence should be Q because it is
about that partition was the culmination which was in in sync with S6.
the case of India. Thus sentence Q should precede S6. 192. (c) The correct sequence is RPSQ. The sentence S1 talks
And Sentence R should follow sentence S. about that Abraham Lincoln mother died. The next
186. (b) The correct sequence is QPRS. The sentence S1 talks sentence should be R because it tells that his father
about that the bank opened at 10 am. All the other married another lady the following year. The next
sentences talk about peon, manager and safe. Of all the statement should be the one describing the lady and
sentences, Q should follow S1 because it talks about her significance in Abraham's life, thus it should be
what happened after the bank was open i.e the manager followed by statement P. Sentence P describes about
and peon went to the safe in the vault. Next sentence the books that his step mother gave him. Thus the next
should be P because in continuation, it describes that step should be S which tells that these books gave
the manager asked the peon to open the safe. What Abraham immense knowledge. The last sentence is Q because
happened next is described in sentence R where the it tells how these Abraham Lincoln acknowledges his
peon and manager look into the open safe. The sentence knowledge to his step mother.
S and S6 are in continuation and thus last sentence 193. (a) The correct sequence is SQRP. The sentence S1 talks
should be S. that crude oil is taken for treatment. The net sentence
187. (a) The correct sequence is QPRS. The sentence S1 talks should be related to the treatment, thus S. the sentence
about how the crowd encircled around the thief. The S talks about heating, hence the next sentence should
next sentence should be Q because it describes that on be Q because it tells that heating helps in differentiation
seeing so many people, his head hung in shame. The of materials according to the boiling points. Now R and
next sentence should be P because it describes that P also describes the process. But in Sentence R, it is
the notorious thief took out a knife from his shirt. Seeing mentioned that 'first vapours' and in sentence P it is
this the two men holding him got scared and ran away. written 'later the gas'. Thus R should come first and
The sequence thus should be R and S. sentence P should precede S6.
188. (a) The correct sequence is SQRP.The sentence S1 talks 194. (c) The proper sequence is QSRP.
about that the old man wanted to cross the road. The 195. (b) The proper sequence is RPSQ.
next sentence should be S because it tells that he waited
196. (b) The proper sequence is RPSQ.
for a long time. The next sentence should be Q because
it describes that after have been waited for long, he 197. (b) The proper sequence is PRQS.
was about to return. The next sentence should be R 198. (a) The proper sequence is QRSP.
which describes the next event that as he decided to 199. (d) The proper sequence is RQPS.
return back, a car came and stopped in front of him. 200. (b) The proper sequence is PRQS.
And a driver came out which helped him in crossing 201. (a) The proper sequence is QPRS.
the road. Thus the sequence should be P and S6. 202. (b) The proper sequence is QSPR.
189. (c) The correct sequence is RQPS. The sentence S1 talks 203. (d) The proper sequence is PRQS.
about that you should speak in strong foreign accent 204. (b) Among all the options, only sentence R appears to
and broken English. The next statement should be the come next to S1. When we look at the options, we see
reason why you should talk like this, thus R. The next that there is only one option (option (a)) which starts
statement should be Q because it describes how a
with sentence R. Hence, we have found the answer by
English person after realising that you are a foreigner
elimination method just by looking at the options.
will not expect you to be polite and use grammatical
205. (a) Sentence S will be the first one because 'shots' in S1 is
phrases. Next sentence should be P because it's in
continuation of the previous sentence. The last directly related to the act of ducking down and running
sentence should be S because it gives us an example fast to a safe place. Now, Q will follow S because only
and S6 gives the aftermath of it. after reaching the river, I jumped into it. P will come
190. (c) The correct sequence is SRQP. The sentence S1 talks next as I came to know that the water was icy only
about that when a lamb is born, its mother may die. The when I jumped into the river and finally, the left sentence
next sentence should be S which explains that when R will come to complete the incident. So, the correct
and why this happens. After S, it should be R because sequence is SQPR, thus, option (a) is the correct
it tells that along with sheep, some lambs that are weak answer.
also die. The next sentence should be Q which describes 206. (d) Sentence Q will be the first sentence to follow S1 as it
the aftermaths. It should be followed by P which tells is the answer to the question asked in S1. Sentence P
the consequence of the same. will come next as it gives another idea why English
191. (b) The correct sequence is PRSQ.The sentence S1 talk travels. R will come next as it talks about mind etc.
about that people does not always get what they which has been mentioned in sentence P. The left
want.The next sentence should be P because in option S will come at the last. So, the correct sequence
continuation of the last sentence which says that you of sentences is QPRS, thus, option (d) is the correct
can't have everything you want and you have to answer.
carefully decide how to spend your income. Priority 207. (d) Among all the sentences, only sentence R is the one
factor comes next which is discussed in sentence R that will come after S1 because S1 talks about the birth
and thus it should come after P. The sentence after R place of the Elephant and sentence R corresponds in
should be S because it talks about how we set our synchronisation by stating 'transported from his native
EBD_7367
138
B- Ordering of Sentence

……… '. Q will come next as it talks about another it will be our answer. Interestingly, we found the answer
admirer apart from Queen Victoria (stated in R). S will just by finding two correct sentences in a sequence.
come after Q as it talks about its Voyoge that started Many times, this method helps to find the answer
from its birth place. P will come at last as it correctly easily where in we don't even bother to find the first
makes connection with S6. Hence, the correct sentence.
sequence is RQSP, thus, option (d) is the correct 214. (d) When we look at the sentences, we see that sentence
answer. S is the continuing part of S1 and should be the first
208. (b) S1 takes about two types of men, therefore, the first statements. Now, when we look at the options, we see
sentence should be the one that talks about the first that only option (d) starts with statements S thereby,
kind of men and we can clearly see that R talks about making it the answer. Hence, option (d) is the correct
the first kind of men i.e. bureaucrat so, it should be the answer and the correct sequence is SPQR.
first sentence. Q should come next to R as it talks 215. (b) Sentence S will come first as it describes the length of
about the characteristic of bureaucrat. Similarly, P will the dinosaur; when we describe the physical feature
come next as it talks about another type of men i.e. of any animal or living being, we usually describe its
social worker. Lastly, S will come at the end as it talks height/length first. When we look at the options, we
about the characteristics of another kind of men. see that only option (b) has sentence S in the
Hence, the correct sequence is RQPS, thereby making beginning, therefore, option (b) is the answer.
option (b) the correct answer. 216. (c) When we read S1, we find that it is talking about a
209. (c) Sentence S properly corresponds to S1 and therefore, single device which is capable of heating fluids without
it should come first. Moreover, it talks about the requiring an electrical elements. When we look at the
history of universities which appears to be a good options, we see that sentence Q directly co-relates
point to start a paragraph. P should come next to it as with S1 as it describes the design of the container, so,
it continues the discussion of S. R should come after P we get the sequence QS. Looking at the options, we
as it directly corresponds to 'modern university' see that only option (c) has QS in the beginning, hence,
discussed in P. and lastly, Q will come at the last as it is it is our answer as rest of the options get eliminated.
the only option left and secondly, it is perfect to precede 217. (c) It is evident that sentence Q should be the first
S6. Hence, the correct sequence is SPRQ, thus, option sentence as all other sentences are not at all in
(c) is the correct answer. coordination with S1. When we look at the options,
210. (d) Q will be the first sentence as it gives the reason why we see that only option (c) sharts with Q, hence, the
Gandhi remained in college for only part of the year. answer is option (c).
When we look at the options, we see that only option 218. (c) ‘Q’ should be first sentence in the sequence as it
(d) starts with Q. hence, option (d) is the answer, logically follows sentence S1. Sentences ‘R’ and ‘S’
thereby, making QPSR the correct sequence. are logical extension of sentence ‘Q’, so, they should
211. (c) Sentence P will come first as it expresses the meaning come together. Sentence ‘P’ should precede the
that Helen was 24 years old when she graduated in concluding sentence S6 to make the paragraph.
English (as stated in S1). No other sentence seems to meaningful. Hence, QRSP is the right sequence.
come after S1 as they do not correspond to S1. When 219. (d) ‘Q’ should be first sentence in this sequence as it
we look at the options, we see that only one option i.e. explains the reason why Ramu was in a hurry. ‘S’
option (c) starts with P. Hence option (c) is the correct should follow ‘Q’ as it explains an action of Ramu and
thereby making PSQR the correct sequence. outcome of that action is detailed in sentences ‘R’
212. (a) All P, Q and S are good options to come after S1 and ‘P’. Sentence ‘P’ should be last in this sequence
therefore, we will consider all these sentences as as it appropriately relates with the concluding
starters and will find the correct sequence by sentence S6. So, QSRP is the correct sequence.
elimination method. After reading S6, it can be 220. (b) ‘R’ should be first in the sequence as it relates with
concluded that the sentence preceding it should also opening sentence S1 and gives a reason behind anger
discuss Gamma Ray. Only sentence R discusses of Chandran. Sentence ‘S’ logically follows sentence
Gamma Ray. So, it should be the last sentence of the ‘R’. Next, sentences ‘Q’ and ‘P’ should be together to
sequence. Now, looking at the option, we see that only extend the idea contained in the paragraph. and P
two options end with R viz. Option (a) and (b); thus, should be last sentence in the sequence which relates
option (c) and (d) get eliminated. Now, when we look to concluding sentence S6. So, RSQP is the right
at option (b) wherein Q precedes P, we see that it is not sequence.
forming a right sequence as P does not correspond to 221. (d) Sentence ‘P’ should be first in the sequence as it is
Q, hence, this option also gets eliminated. Finally, we the first thought of the author after hearing fire alarms
are left with option (a) which should be the answer. as mentioned in S1. Next, sentence ‘S’ should follow
Moreover, PQSR make a proper sequence with correct ‘P’ as it contains first action of the author. Sentences
continuation. Hence, (a) is the answer. ‘R’ and ‘Q’ further extend the idea contained in the
213. (d) In this question, after reading all the sentences, it is paragraph in which ‘Q’ should be last sentence in the
clear that Q and P will go together as they form a correct sequence. So, PSRQ in the right answer.
sequence. Moreover, P will come after Q. New, when 222. (d) Sentence ‘S’ relates with opening sentence as the first
are look at the options, we see that there is only one action of people after watching flames. Other
option where Q is preceded by P i.e. option (d), hence, sentences describe how people of different age group
Ordering of Sentence B-139
arrived on the spot. Sentence ‘R’ logically follows 230. (b) Sentence S1 talks about efforts to tap solar energy.
sentence ‘S’ and ‘Q’ should be the next sentence in Sentence ‘Q’ is an extension of idea contained in S1 ,
the sequence as it talks about assistance provided by so, it should be first sentence in the sequence.
girls and woman. Sentence ‘P’ should be last in this Sentence ‘P’ gives examples of applications of solar
sequence as it relates to the concluding sentence of energy mentioned in sentence ‘Q’. So it should be
the paragraph. So, SRQP is the correct sequence. second sentence in the sequence. Sentences ‘R’ and
223. (c) Sentence S1 talks about misconceptions related to ‘S’ should follow sentence ‘P’ as they talk about other
snakes. Sentence ‘P’ should be first in the sequence possible application of solar energy. It also relates
as it contains one common misconception. Sentence with concluding sentence S6. So, QPRS is the correct
S should follow P as it gives a fact related to sequence of sentence in the paragraph.
misconception mentioned in sentence P. Sentence Q 231. (d) The opening sentence S1 is about structure of houses
should be next sentence in the sequence as it presents in the Indus Valley civilisation. Sentence ‘Q’ further
another misconceptions about snakes. Sentence R elaborates idea contained in sentence S1 by giving
should be last in the sequence as it given a fact related details of the upstairs rooms. So, it should be first
to misconception mentioned in ‘Q’. It also relates to sentence of the sequence. Sentences ‘P’ and ‘S’
concluding sentence S6. should follow sentence ‘Q’ as they provide other
224. (c) The correct sequence is PQSR. structural details of the Indus Valley houses. Sentence
225. (c) Sentence S1 talks about an eye disease. So, the first ‘R’ should be last in this sequence as it provides details
sentences in the sequence should be about its of drainage system and relates with the concluding
symptoms. So, sentence ‘R’ should be first in the sentence S6. So, QPSR is the correct sequence of
sequence as it contains symptoms of conjunctivitis. sentences in the paragraph.
Sentences ‘Q’ and ‘S’ should follow sentence ‘R’ as 232. (b) Sentence ‘Q’ should be first sentence in the sequence
they indicate other symptoms of conjunctivitis. as it gives details of the library mentioned in the
Sentence ‘P’ gives a reason which complements the opening sentence S1 of the paragraph. Sentence ‘P’
concluding sentence S6 of the paragraph. So, RQSP should follow ‘Q’ as it provides additional details of
is the correct sequence. the library. Sentences ‘S’ and ‘R’ should follow
226. (d) Sentence ‘P’ should be first in the sequence as it sentence ‘P’ as they give details of the famous person
contains response of the author after receiving the who owned the library. Sentence ‘R’ should be last in
message from the office. As sentence ‘P’ talks about the sequence as it relates with the concluding
winding up establishment of the author, sentences sentence S6 of the paragraph. So, QPSR is the right
‘R’, ‘Q’ and ‘S’ should logically follow it to make the sequence of sentences in the paragraph.
paragraph meaningful. Therefore, PRQS is the right 233. (c) This paragraph is about some genuine and some silly
sequence. fears. The opening sentence S1 of the paragraph is
227. (c) Sentence ‘P’ should be first in the sequence as it about some genuine fears. So, sentences R and P
logically follows the opening sentence S1 which is should be first and second sentences of the sequence
about Harsten’s theory. Sentences ‘Q’ and ‘S’ should as they give example of a genuine fear and a reason
come next in the sequence as they provide details of behind the fear. Sentences ‘S’ and ‘Q’ should follow
the experiment mentioned in sentence ‘P’. Sentence sentences ‘R’ and ‘P’ as they give another example
‘Q’ should be last in the sequence as it further and reason behind another genuine fear. Therefore
describes type of music mentioned in preceding RPSQ in the correct sequence of the sentences in the
sentence ‘S’. So, PRSQ is the correct sequence of paragraph.
sentences in the paragraph. 234. (b) The opening sentences S1 of the paragraph contains
228. (a) Sentence ‘R’ should be first in the sequence as it is an the idea of going out on a public holiday. Sentence
argument made against first statement S1 of the ‘Q’ should be first in the sequence as it talks about an
paragraph. Sentences ‘Q’ and ‘P’ are supporting issue related with idea contained in sentence S 1 .
arguments in which different viewpoints about fast Sentence ‘S’ should follow ‘Q’ as it indicates the
and slow readers are presented. Sentence ‘S’ should problem of high traffic on public holidays. Sentences
be last in the sequence as it contains a conclusion P and R should be next in the sequence after ‘Q’ and
which relates with preceding sentences of the ‘S’ as they provide details of the place where the author
paragraph. Therefore, RQPS is the correct sequence has stopped to spend the day. Sentence R should be
of sentences in the paragraph. last in the sequence as it relates with idea contained
229. (d) Sentence ‘S’ should be first in the sequence as it in the concluding sentence S6. So, QSPR is the correct
logically follows the idea contained in sentence S1 of sequence of the sentences.
the paragraph. Sentence S1 talks about choice of 235. (d) Sentence ‘R’ should be first in this sequence as it is
captaincy among cricketers and sentence ‘S’ is about logical extension of idea contained in sentence S1.
how dreams of captaincy is nurtured among boys Since teaching work in over, exams are likely to follow
since their school days. Sentences ‘Q’, ‘P’ and ‘R’ which is mentioned in the sentence ‘R’. ‘P’ should
should follow sentence ‘S’ to make the idea contained follow sentence ‘R’ as it gives further details of
in the paragraph coherent and meaningful. So, SQPR examination, its dates and the vacation that follows
is the right sequence of sentences in the paragraph. the exam. Sentences ‘Q’ and ‘S’ should follow
EBD_7367
140
B- Ordering of Sentence

sentences R and P as they provide details of author’s follows next and R as the fourth one which tells the
plan after the examination is over. So, RPSQ is the history of the middle of 15th century. S6 tells the effects
correct sequence of sentences in the paragraph. caused by those on man.
226. (d) Statement S1 is about the proposal to remove 5 Paise 244. (d) RQPS
coin from circulation by the government. Sentence P The passage opens up with S1 telling the nature of a
should be first in the sequence followed by sentence boy. R should be the next one cause it tells the problem
‘Q’ as these two sentences explain government’s his mother faced cause of it. Next Q states her
plans to remove low value coins from circulation and situation, her worries. Then comes P which tells the
the reason behind such a proposal. Sentences ‘S’ and step his uncle took for a solution. S states the effect
‘R’ should follow ‘P’ and ‘Q’ as they present opposing of the uncle’s action. S6 tells the final consequence of
viewpoint of RBI and the reason behind RBI’s view S.
Point. So, PQSR is the correct sequence of the 245. (c) SPQR
sentences in the paragraph. S1 starts with the author’s memories related to meeting
237. (b) Sentence ‘S’ should be first in the sequence as it is an Baba Amte. In next S we are introduced to another
argument against viewpoint mentioned in sentence angle, another character. In next P his action and then
S1. Sentences ‘Q’ and ‘R’ should follow ‘S’ as they in Q the effect on him caused by his work are
give supporting argument and example in favour of discussed. R tells the final effect of his work, his action
sentence ‘S’. Sentence ‘P’ should be last in this and the reaction. S6 states the reason behind doing
sequence as it indicates approach of a wise person that.
which relates to concluding sentence S 6 of the 246. (c) QPSR
paragraph. So, SQRP is the correct sequence of S1 tells an action or work done by Ross. Q states a
sentences in the paragraph. fact related to Ross a and P the effect it caused. S tells
238. (c) The correct sequence should be RPSQ. The first the reaction it had on the audience followed by R the
sentences S1 introduces the location of Egypt. Next President of Royal Society’s action and then the
should be R cause it further add details of the country, compliment he gave.
its size. Next should be P which further throws light 247. (c) RPQS
on its geographical features. S follows next adding S1 states a fact from history. R states what happened
more about it, shifting to the weather, and about the later on then P says what next, followed by Q in which
rainfall which is directly linked with S6. the present time is discussed, followed by S which
239. (d) QSPR is the correct sequence. tells the problem in present time related to fuels.
In the first S1, a statement is given which tells some 248. (c) SPRQ
information Q should follow as it tells the reason S1 is a general statement, an assertion. Next ‘S’ tells
behind it. S should follow next, as it states the results that though things should have changed but is still a
or the possibility of the experiment discussed in Q. P mystery. R tells what attempts are taken to help clear
is next which further discusses those and R is last as the mystery. Last Q tells that all such attempts proved
it states the solution as what is required. Then comes to be in vain.
S6. 249. (d) RQPS
240. (a) RQSP S1 starts with a fact that there are two schools of
S1 tells a fact, R further states the consequence of thought. Then R explains the Ist school. Q elaborates
that act related to Issac. ‘Q’ the next one states the R a bit more. P, the next one discusses the second
action done by the grandmother. In S, Issac is the school of thought. The last S elaborates the thoughts
focus, as his past is discussed. Fourth ‘P’ starts with of the second school. S 6 states second one is
but as it contrasts the thought of ‘S’and tells a becoming more popular.
positive point of Issac. S6 discusses the P. 250. (d) QSPR
241. (d) QSRP The passage starts with S1, in which the writer tells
S1 states something, Q tells what happens cause of an incident in his life. He then describes the event’s
that. Next should be S, the experience of the author, place in Q, S tells who the delegates were in the
his past memory of the effect. Third one is R the conference. P tells the action done by the writer and
present thoughts of the author. Last should be P reason behind it and R the consequence of it. This
which elaborates more the present scenario. S6 tells order makes a meaningful passage.
the future plan of the author, what he will do. 251. (d) PRSQ
242. (d) PQSR S1 tells an assertion about sportsmen, followed by P
S1 introduces us with the present situation of history which describes what discipline means in their term.
as a subject. P should be next as it discusses what R tells that what all is required to be learned by the
our thoughts are about it. Q states that Tonybee’s sportsmen. S tells what effect it causes and why is it
thought were the contrast. S tells his way of studying important. Q further tells the effect, followed by the
and analyzing. R follows next with more information benefits of discipline in life in S6.
related to his style. 252. (b) SPRQ
243. (a) PQSR S1 starts with a night in the life of Mr. and Mrs. Robert.
S1 starts the passage with a fact. P states the result of S tell what happened that night, what can be the reason
it. Q tells the effect on science by another event. S behind is discussed in P and sentences that follow. R
Ordering of Sentence B-141

tells what they did and how they were feeling then. Q find the answer among these two. When we read S2,
tells the reality what they saw. S6 tells what Mr. we see that the sentence preceding it must talk about
Robert’s reaction was, he rushed to police. the subject mentioned i.e. politics, morals and religion;
253. (b) RQSP sentence P talks about all these thus, it will be the last
S1 starts with an opinion or thought. R tells why it sentence. Hence, option (c) i.e. QSRP will be the
happens i.e. the physical needs. The sentences that answer.
follow discusses it more. S tests the solution and in P 262. (b) S1talks about the image of the term 'technology' and
how it should be done. when we look at the sentences, we see that only
254. (b) RSPQ sentence P talks in continuation of S1about the symbol
S1 tells about Aristotle, his times and experiments. R, of technology and appears to be the first sentence of
the first sentence what all he had, then S what was the sequence. All other sentences are not in
not available or lacking. P what was the reason continuation to S1. Now when we look at the options,
considered to be behind that. Q what happened in we see that only one option starts with sentence P.
other part. S6 the fact. Hence, option (b) will be the answer.
255. (b) SRPQ 263. (a) Sentence Q will be the first sentence of the sequence
S1 starts with a bus’s action it’s position, then S, what as it talks about the trip led by the writer in capacity to
happens on the road, then Q the driver’s reaction. the designation he held at Patna College (mentioned
What happened by applying. Q-what was the effect in S1); it is very clear that sentence Q directly
by this series of actions. corresponds to S1 and therefore, it should be the first
256. (b) RQPS sentence. When we look at the options, we see that
S1 starts with the present scenario. R talks about the only one option starts with sentence Q i.e. option (a).
present progress in women’s status. The Next two Hence, it is the answer.
sentences Q and R highlight the contrast between
264. (c) When we read all the sentences carefully we see that
women in rural and urban places. S too adds to it. S6
sentence S talks about the result of the announcement
tells how can education help in women’s condition.
made by the Union Finance Ministry in S1 thus, it
257. (d) RQSP
should be the first sentence as it goes with the flow in
The central character of passage is Hiuen-tsang, S1
correct context. Now, when we look at the options, we
introduces him. R tells what he did to help himself in
see that only one option starts with sentence S i.e.
research. In Q where he spent time the places. S is the
option (c). Hence, it is the answer.
same topic’s expansion. P - What he did to carry out
his work. What was suggested by the learned people. 265. (a) It is clear that sentence R will be the first sentence
after S1 as it goes with the continuation of the fact
258. (c) After reading all the sentences, only statement Q
that at four o'clock, Hitler attacked and invaded Russia
appears to be the one that corresponds to S1 properly.
and no one could have expected him to do so. Sentence
When we look at the options, we see that only option
Q will come next as sentence P and S do not correspond
(c) starts with S thus, option (c) is the answer. We
to the flow of the sentences. When we look at the
don't even need to look for other sentences.
options, we see that only one option starts with RQ
259. (a) In this question, all sentences except sentence R
i.e. option (a). Hence, it is the answer.
appears to be related to each other but not to S1.
Statement R which is in contrast to S1 will be the first 266. (d) Sentence S will be the first sentence as it has a direct
sentence as it directly contradicts S1 with a valid point. connection with S1 i.e. since Roderick Usher has
P,Q, and S will come next to complete the sequence always been a quiet person who talked little of himself
correctly. When we look at the options, it is clear that so, I do not know too much about him. All other
option (a) i.e. RPQS will be the answer. sentences do not correspond directly to S1 as S does.
R will come next as it goes with the flow of the
260. (d) Among all the sentences, sentence S and Q could be
preceding sentences. P will come after R as it talks
the first sentence as they directly corresponds to the
about his ancestors who were famous for their artistic
subject (great leader) mentioned in S1. When we look
at the options, we see that only one option starts with and musical abilities. The left sentence Q will conclude
sentence S and none with sentence Q. Hence, option the sequence. When we look at the option we see that
(d) which starts with sentence S will be the answer. option (d) has the sequence SRPQ, hence, it is the
We have solved this question by elimination method answer.
where we didn't look for other sentences. 267. (b) After reading all the sentences, it is clear that sentence
261. (c) It is very clear that sentence Q will be the first sentence R will be the first sentence of the sequence as rest of
as it directly corresponds to S1 and sentence S will the sentences except R talk about different subject
come next as it correctly goes with the continuation. and do not appear to be the first sentence after S1.
When we look at the options, we see that there are Looking at the options, we see that only one option
two options with QS in the starting so, we will have to i.e. option (b) starts with R hence, it is the answer.
EBD_7367
142
B- Sentence Improvement

C HA P T E R
SENTENCE IMPROVEMENT
31
DIRECTIONS (Qs. 1-17) : Look at the underlined part of each 14. Fifty miles are a long distance to walk.
sentence. Below each sentence, three possible substitutions for (a) is (b) become
the underlined part are given. If one of them i.e (a), (b) or (c) is (c) be (d) No improvement
better than the underlined part, indicate your response on the 15. After the vote was taken the meeting broke.
Answer Sheet against the corresponding letter (a), (b) or (c). If (a) broke down (b) broke up
none of the substitutions improves the sentence, indicate (d) as (c) broke out (d) No improvement
16. The speaker asked the audience to bear upon him for a few
your response on the Answer Sheet. Thus a ‘No improvement’
minutes more.
response will be signified by the letter (d). [2011-II]
(a) on (b) with
1. If I were you, I would do it at once. (c) for (d) No improvement
(a) was (b) am 17. Many people find it difficult to make both hands meet.
(c) would be (d) No improvement (a) both accounts (b) both hand to mouth
2. They set a strong guard, lest anyone could escape. (c) both ends meet (d) No improvement
(a) would (b) might
DIRECTIONS (Qs. 18-31) : Look at the underlined part of each
(c) should (d) No improvement
sentence. Below each sentence, three possible situations for the
3. The matter called up an explanation of his conduct. underlined part are given. If one of them (a), (b) or (c) is better than
(a) out (b) in the underlined part, indicate your response on the Answer Sheet
(c) for (d) No improvement against the corresponding letter (a), (b) or (c). If none of these
4. The accused refused having murdered anybody. substitutions improves the sentence, indicate (d) as your response
(a) disagreed (b) denied on the Answer Sheet. Thus a "No Improvement" response will be
(c) declaimed (d) No improvement signified by the letter (d). [2012-II]
5. We need honest workers, not people of redoubtable integrity.
18. More than one person was killed in the accident.
(a) doubting (b) doubtful
(a) were killed
(c) doubtless (d) No improvement
(b) are killed
6. By the time he arrived, everybody had gone home. (c) have been killed
(a) when he arrived (b) at which he arrived (d) No improvement
(c) by which he arrived (d) No improvement 19. Not a word they spoke to the unfortunate wife about it.
7. There is no alternate, so we must leave now. (a) did they speak
(a) altering (b) alternative (b) they will speak
(c) alternation (d) No improvement (c) they had spoken
8. I cannot listen what she is saying. (d) No improvement
(a) hear what (b) listen for what 20. The poor villagers have waited in bitter cold for more than
(c) listen to that (d) No improvement four hours now.
9. He is still in vigorous health although he is on the right side (a) have been waiting
of sixty. (b) had waited
(a) wrong (b) left (c) has been waiting
(c) negative (d) No improvement (d) No improvement
10. We are sorry to hear regarding your father’s death. 21. If he had time he will call you.
(a) of (b) over (a) would have
(c) for (d) No improvement (b) would have had
11. Babu asked his friend, “Where you went yesterday?” (c) has
(a) “Where did you go yesterday?” (d) No improvement
(b) “Where you had gone yesterday?” 22. All, but her, had made an attempt.
(c) “Where you did go yesterday?” (a) All, but she,
(d) No improvement (b) All but her
(c) All, but her
12. He lay on the grass enjoying the sunshine.
(d) No improvement
(a) laid (b) lied 23. I am used to hard work.
(c) led (d) No improvement (a) work hard
13. Some passengers were flown to Paris on the last trip. (b) work hardly
(a) fled (b) flied (c) hard working
(c) flew (d) No improvement (d) No improvement
Sentence Improvement B-143
24. Twenty kilometres are not a great distance in these days of 38. I am not sure why she is wanting to see him.
fast moving vehicles. (a) she wants (b) does she want
(a) is not a great distance (c) is she wanting (d) No improvement
(b) is no distance 39. Everybody who finished writing can go home.
(c) aren't a great distance (a) had finished (b) have finished
(d) No improvement (c) has finished (d) No improvement
25. They were working as usually. 40. I wish I can sing as well as you do.
(a) usual (b) as usual (a) do (b) could
(c) usually (d) No improvement (c) did (d) No improvement
26. He is unlikely to come to the party. But if he comes I would 41. It has been two years since I have seen him last.
talk to him. (a) when I have seen him (b) since I had seen him
(a) if he would come (c) since I saw him (d) No improvement
(b) if he is to come 42. My neighbour is having two cars.
(c) if he will come (a) is owning (b) has
(d) No improvement (c) is possessing (d) No improvement
27. The passengers have formed queue at the booking-counter 43. His flute recitation was highly appreciated.
much before a train arrives. (a) flute recital (b) flute play
(a) formed (b) have been forming (c) flute singing (d) No improvement
(c) form (d) No improvement 44. She must try at making him understand.
28. You must complete this work up to Sunday. (a) to make him to understand
(a) within Sunday (b) by Sunday (b) to make him understand
(c) until Sunday (d) No improvement (c) to make his understanding
29. If it will rain, the match will be abandoned. (d) No improvement
(a) If it rains 45. Those are your new shoes, aren’t they ?
(b) If it would rain (a) isn’t it ? (b) is it so ?
(c) If it rained (c) are they ? (d) No improvement
(d) No improvement 46. He told to us everything he knew.
30. How long do you think Mr. Kamal knew John ? (a) us everything he knew
(a) will know (b) knows (b) us everything he is knowing
(c) has known (d) No improvement (c) us everything he was knowing
31. I have still to meet a person who is perfectly satisfied with (d) No improvement
his job. 47. Unless you do not work hard, you won’t succeed in life.
(a) am still to meet (b) am yet to meet (a) cannot work hard (b) will not work hard
(c) might still meet (d) No improvement. (c) work hard (d) No improvement
DIRECTIONS (Qs. 32-51) : Look at the underlined part of 48. As I am tired, I cannot be able to climb this hill now.
each sentence. Below each sentence, three possible situations (a) can be able (b) will not be able
for the underlined part are given. If one of them (a), (b) or (c) is (c) will not be possible (d) No improvement
better than the underlined part, indicate your response on the 49. I should not spend money for luxuries.
Answer Sheet against the corresponding letter (a), (b) or (c). If (a) on luxuries (b) in luxuries
none of the substitutions improves the sentence, indicate (d) as (c) through luxuries (d) No improvement
your response on the Answer Sheet. Thus a “No improvement” 50. On being asked by the judge if he had murdered his wife the
response will be signified by the letter (d). [2013-I] accused denied the charge.
32. She told the children not to stop the work. (a) refused (b) rebutted
(a) not stopping (b) don’t stop (c) contradicted (d) No improvement
(c) not stopping of (d) No improvement 51. The new headmaster affected many changes in the school.
33. I am not telling that you should hunt out people to pursue (a) injected (b) effected
your policies. (c) inflicted (d) No improvement
(a) asking (b) saying DIRECTIONS (Qs 52-71): Look at the underlined part of each
(c) speaking (d) No improvement sentence. Below each sentence are given three possible
34. He succeeded by dint of hard work. substitutions for the underlined part. If one of them (a), (b) or (c)
(a) by means of (b) by doing is better than the underlined part, indicate your response on the
(c) by virtue of (d) No improvement Answer Sheet against the corresponding letter (a), (b) or (c). If
35. You have read that book for ages. none of the substitutions improve the sentence, indicate (d) as
(a) have been reading (b) had read your response on the Answer Sheet. Thus a 'No improvement'
(c) will be reading (d) No improvement response will be signified by the letter (d). [2014-I]
36. The only bit of relief for the victims has been the increase in
compensation. 52. There is no rain in our village for the last six months.
(a) were (b) have been (a) has been (b) was
(c) was that they were given (d) No improvement (c) had been (d) No improvement
37. He is resembling his father. 53. The police investigated into the matter.
(a) has been resembling (b) resembles like (a) with the matter (b) at the matter
(c) resembles (d) No improvement (c) the matter (d) No improvement
EBD_7367
144
B- Sentence Improvement

54. Ramachandra Murthy and his family have been in Guyana 71. Never I have seen such breathtaking scenery.
from 1985. (a) Never have I (b) Ever I have
(a) since (b) about (c) I cannot ever (d) No improvement
(c) on (d) No improvement DIRECTIONS (Qs. 72-91) : Look at the underlined part of each
55. I am living in this town since 1980. sentence. Below each sentence are given three possible
(a) was living (b) shall live substitutions for the underlined part. If one of them is better
(c) have been living (d) No improvement than the underlined part, mark accordingly on the Answer Sheet.
56. If I was you I should tell him the truth. If none of the substitutions improve the sentence, mark (d) on
(a) am you (b) were you the Answer Sheet. [2014-II]
(c) had been you (d) No improvement
57. He is better than any boy in the class. 72. I never have and probably never will write good letters.
(a) any boys (b) all the boys (a) I never have written
(c) any other boy (d) No improvement (b) I never have wrote
58. Anil ought not to tell me your secret, but he did. (c) I never have been writing
(a) to be telling (b) tell (d) NO IMPROVEMENT
(c) to have told (d) No improvement 73. I think his feet are bigger than any boy in town
59. If I were him I would have not accepted the offer. (a) his feet are bigger than many boys in town
(a) If I was him (b) If I were he (b) his feet are bigger than no boys in town
(c) If I had he (d) No improvement (c) his feet are bigger than any other boys in town
60. What the nation needs is people of character. (d) NO IMPROVEMENT
(a) are the people of character 74. I haven't hardly studied for this examination.
(b) are people of character (a) Hardly I have studied.
(c) is a people of character (b) I have hardly studied
(d) No improvement (c) Not hardly I have studied
61. We now come to the important question of where this great (d) NO IMPROVEMENT
swarm of galaxies have come from. 75. As you look across the street, lighted windows can be seen.
(a) have come (b) has come from (a) you saw lighted windows
(c) are coming from (d) No improvement (b) lighted windows may be seen
62. Fewer rainfall means less traffic accidents, according to the (c) you can see lighted windows
(d) NO IMPROVEMENT
experts' report on highway safety.
76. Her sister is a nurse and she intends to be one too.
(a) Less rainfall means fewer traffic accidents
(a) this is the profession she intends
(b) Less rainfall means less traffic accidents
(b) her intention is the-same profession
(c) Fewer rainfall means fewer traffic accidents
(c) she intending to be a nurse too
(d) No improvement
(d) NO IMPROVEMENT
63. I never saw you at the party yesterday.
77. He asked for the cup of tea.
(a) have not seen (b) did not see
(a) some cup of tea (b) cup of tea
(c) had never seen (d) No improvement
(c) a cup of tea (d) NO IMPROVEMENT
64. Ajeet is a bigger scholar than his brother.
78. Several people saw the thief snatch her gold chain.
(a) better (b) smaller (a) people have seen (b) people were seeing
(c) superior (d) No improvement (c) people must see (d) NO IMPROVEMENT
65. I did not wait for him because he went out before I arrived. 79. We shall not wait for anyone who will arrive late.
(a) has gone out (b) had gone out (a) who arrives late (b) who arrived late
(c) had been out (d) No improvement (c) who shall arrive (d) NO IMPROVEMENT
66. Whenever I saw him, he has been reading the same novel. 80. We had a hard time in the war.
(a) had been reading (b) read (a) from the war (b) since the war
(c) was reading (d) No improvement (c) during the war (d) NO IMPROVEMENT
67. Since the beginning of the term, we are spending a lot of 81. He aimed a blow on me.
time on poetry. (a) at me (b) to me
(a) spent (b) will spend (c) against me (d) NO IMPROVEMENT
(c) have spent (d) No improvement 82. He waited for her by dinnertime.
68. Your sister cooks well, isn't she ? (a) at dinnertime (b) till dinnertime
(a) isn't it ? (b) doesn't she ? (c) on dinnertime (d) NO IMPROVEMENT
(c) doesn't it ? (d) No improvement 83. He does not have the last idea of it.
69. Dickens' novels, like many writers, are largely autobiographical. (a) little (b) less
(a) like those of many other writers (c) least (d) NO IMPROVEMENT
(b) like so many others 84. Born of poor, illiterate farm workers, Lincoln rose to become
(c) like many other novelists the president of the U.S.A.
(d) No improvement (a) raised to become (b) arose to become
70. She was as preety as, if not prettier than any other girl at the (c) risen to become (d) NO IMPROVEMENT
party. 85. Gopal is two years older than his brother
(a) She was very pretty (b) She was pretty (a) than own brother (b) to his brother
(c) She was the prettiest (d) No improvement (c) by his brother (d) NO IMPROVEMENT
Sentence Improvement B-145
86. The editor regretted that he was inable to make use of the 101. Any English are known for their practical instincts.
article. (a) Some (b) Many
(a) was disabled (b) was unable (c) The (d) No improvement
(c) was enabled (d) NO IMPROVEMENT 102. Fifty miles are a long distance to walk.
87. He walked softly lest he may wake the baby. (a) is (b) become
(a) he would wake (b) he wake (c) be (d) No improvement
(c) he should wake (d) NO IMPROVEMENT 103. Economics today were not what it was a century ago.
88. I look forward to meet you in Delhi. (a) are (b) was
(a) to meeting you (b) to meet with you (c) is (d) No improvement
(c) at meeting you (d) NO IMPROVEMENT 104. Hearing the news of the accident, he broke.
89. I do not know where has he gone. (a) broke down (b) broke up
(a) where had be gone (b) where he has gone (c) broke out (d) No improvement
(c) when has he gone (d) NO IMPROVEMENT 105. The speaker asked the audience to bear upon him for a few
90. The teacher taught the students that the moon goes round minutes more.
the earth. (a) on (b) with
(a) the moon went round the earth (c) for (d) No improvement
(b) the moon is going round the earth 106. Those men as are false to their friends should be avoided.
(c) the moon has gone round the earth
(a) The men (b) Men
(d) NO IMPROVEMENT
(c) Such men (d) No improvement
91. This is the boy that I talked to you about.
107. Many people find it difficult to make both hands meet.
(a) who I talked to you about
(b) whom I talked to you about (a) both accounts (b) both hands to mouth
(c) which I talked to you about (c) both ends meet (d) No improvement
(d) NO IMPROVEMENT 108. Young children are not physically capable to carry these
loads.
DIRECTIONS (92-111): Look at the underlined part of each (a) have to carry these loads
sentence. Below each sentence are given three possible (b) of carrying these loads
substitutions for the underlined part. If one of them is better (c) carry these loads
than the underlined part, mark accordingly on the Answer Sheet. (d) No improvement
If none of the substitutions improve the sentence, mark (d) on 109. I enjoyed the ballet by a troupe of Russian dancers.
your Answer Sheet. [2015-I] (a) troop (b) trouper
92. We need honest workers, not people of redoubtable (c) trooper (d) No improvement
integrity. 110. The secretary threatened to resign his post.
(a) doubting (b) doubtful (a) from his post (b) to his post
(c) doubtless (d) No improvement (c) for his post
93. I expect every player here to be conversant at the rules of game. (d) No improvement
(a) on (b) about 111. By the time he arrived, everybody had gone home.
(c) with (d) No improvement (a) when he arrived
94. There is no alternate, so we must leave now. (b) at which he arrived
(a) altering (b) alternative (c) by which he arrive
(c) alternation (d) No improvement (d) No improvement
95. If I were you, I would do it at once.
(a) was (b) am DIRECTIONS (Qs. 112-136): Look at the underlined part of
(c) would be (d) No improvement each sentence. Below each sentence are given three possible
96. They set a strong guard, lest any one could escape. substitutions for the underlined part. If one of them (a), (b) or
(a) would (b) might (c) is better than the underlined part, indicate your response on
(c) should (d) No improvement the Answer Sheet against the corresponding letter (a), (b) or
97. The matter called up an explanation of his conduct. (c). If none of the substitutions improves the sentence, indicate
(a) out (b) in (d) as your response on the Answer Sheet. Thus a "No
(c) for (d) No improvement improvement" response will be signified by the letter (d).
98. The accused refused having murdered anybody. [2015-II]
(a) disagreed (b) denied
(c) declaimed (d) No improvement 112. It's ten o'clock already. It's high time you went home.
99. We cannot trust a man who plays false and loose with (a) you had gone (b) you were going
others. (c) you had been (d) No Improvement
(a) false or loose (b) fast or loose 113. The students are playing volley-ball since 8 A.M.
(c) fast and loose (d) No improvement (a) were playing (b) have playing
100. He is still in vigorous health although he is on the right side (c) have been playing (d) No Improvement
of sixty. 114. Our plans for the trip fell down because we had no money
(a) wrong (b) left (a) off (b) out
(c) negative (d) No improvement (c) through (d) No Improvement
EBD_7367
146
B- Sentence Improvement

115. None of these groups has reported accurately on the 130. He does not know what the university is.
prevailing situation. (a) an (b) a
(a) have reported (b) was reported (c) one (d) No Improvement
(c) has reporting (d) No Improvement 131. The Prime Minister called on the President.
116. Among the athletes undergoing training Ramesh was easily (a) by (b) in
the better. (c) to (d) No Improvement
(a) best (b) better 132. No sooner did we reach the railway station when it began
(c) the best (d) No improvement to rain.
117. The earth moves round the sun, isn't it ? (a) than (b) and
(a) wasn't it? (b) hasn't it? (c) while (d) No Improvement
(c) doesn't it? (d) No improvement
133. Well-bred children always listen to their parents' advice.
118. Supposing if he is arrested what will he do ?
(a) hear to (b) agree
(a) if he will be arrested (b) he is arrested
(c) obey (d) No Improvement
(c) if he was arrested (d) No Improvement
134. We had not met since then, neither did I wish to meet him
119. My students have been interesting in learning French.
(a) have been interested (b) are being interesting now.
(c) have also interesting (d) No Improvement (a) never did I wish (b) nor did I wish
120. Hardly I had fallen asleep, when the bell rang. (c) did not I wish (d) No Improvement
(a) I was fallen (b) had I fallen 135. Even she had taken a taxi, she would have been late.
(c) I fell asleep (d) No improvement (a) Even if she had (b) Although she had
121. These days, Radha finds it difficult to make both her ends (c) As if she had (d) No Improvement
meet. 136. He felt sure of his success, though he was beginning to get
(a) both ends meet (b) both the ends meet worried.
(c) ends meet (d) No Improvement (a) his succeed (b) his successes
122. My mother always asks us to close the fan when we leave (c) being succeeded (d) No Improvement
the room.
DIRECTIONS (Qs. 137-156) : Look at the underline part of each
(a) on the fan (b) off the fans
sentence. Below each sentence are given three possible
(c) turn off the fan (d) No Improvement
substiutions for the underlined part. If one of them (a), (b) or (c)
123. If she does not get more high salary, she will resign
is better than the underlined part, indicate your response on the
(a) more higher salary (b) high salaries
Answer sheet against the corresponding letter (a), (b) or (c). If
(c) a higher salary (d) No Improvement
none of the substiutions improves the sentence, indicate (d) as
124. The parents should not discriminated from the girl child.
your response on the Answer Sheet. Thus, a “No improvement”
(a) discriminate between (b) discriminate against
response will be signified by the letter (d). [2016-I]
(c) discriminate at (d) No Improvement
125. Shakespeare's play 'Macbeth' is another of his greatest 137. The police accused him for theft.
works. (a) With (b) in
(a) one of his greatest (b) best of his greatest (c) of (d) No improvement
(c) greatest of his (d) No Improvement 138. He wanted that I left immediately.
126. Some schools require children to wear black leather (a) I may leave (b) me to leave
expensive shoes. (c) I leave (d) No improvement
(a) black expensive leather shoes 139. This is to certify that I know Mr. J. Mathews since 1970.
(b) expensive leather black shoes (a) am knowing (b) had known
(c) expensive black leather shoes (c) have known (d) No improvement
(d) No Improvement 140. They took away everything that belonged to him.
127. It is a good thing for him should recognize his faults. (a) that had been belonging (b) that belong
(a) that he to recognise his faults (c) that has been belonging (d) No improvement
(b) him recognizing his faults 141. It was the mother of the girl of whose voice I had recognised.
(c) for him to recognize his faults (a) whose voice (b) the voice of who
(d) No Improvement (c) voice whose (d) No improvement
128. Some boys speak their mother-tongue among one another. 142. The Executive Council is consisted of ten members.
(a) between them (b) among themselves (a) consists of (b) comprises of
(c) with them (d) No Improvement (c) constituted of (d) No improvement
129. Prior than taking any decisions he always consults his lawyer. 143. The maid was laying the table for dinner.
(a) Prior to (b) Previous to (a) setting up (b) lying
(c) Prior as (d) No Improvement (c) sorting out (d) No improvement
Sentence Improvement B-147

144. We have so arranged the matters and one of us is always 157. Suppose if you are selected, will you give us a treat ?
on duty. (a) Supposing if (b) If suppose
(a) that one of us (b) so that one of us (c) If (d) No improvement
(c) such that one of us (d) No improvement 158. I would rather have a noble enemy than a mean friend.
145. Hardly have we got into the forest when it began to rain. (a) would more have (b) would have
(a) Hardly we got (b) We had hardly got (c) will have (d) No improvement
(c) We had got hard (d) No improvement 159. He decided to take the help of a guide lest he may miss the
146. Each time he felt tired he lied down. way.
(a) lies (b) lays (a) he should miss (b) he will miss
(c) lay (d) No improvement (c) he might fail to see (d) No improvement
147. Though it was raining. but I went out. 160. He wanted my permission to taking part in sports.
(a) but yet I (b) I (a) to take part in
(b) for to take part in
(c) however I (d) No improvement
(c) for to taking part in
148. There is no chance of success unless you do not work
(d) No improvement
hard.
161. We are doing this in the interest of the poors.
(a) unless you work (b) until your working
(a) in the interests of the poors
(c) until you do not work (d) No improvement (b) in the interests of the poor
149. She has grown too old to do little work. (c) for the interests of the poor
(a) some (b) any (d) No improvement
(c) a little (d) No improvement 162. He reached his destination at night.
150. No one enjoys to deceive his family (a) destination
(a) deceiving (b) for deceiving (b) at his destination
(e) deceive (d) No improvement (c) on his destination
151. Have you ever saw the flower of a pumpkin plant? (d) No improvement
(a) see (b) seeing 163. One is often pleased with himself.
(c) seen (d) No improvement (a) with one's self (b) with themselves
152. It is an ancient, historical place and it once belongs to the (c) with oneself (d) No improvement
Pandavas. 164. Unless you are not very careful, you will run into debt.
(a) belonged (b) belonging (a) are very (b) will be very
(c) belong (d) No improvement (c) may be (d) No improvement
153. Since we were knowing the correct route, we did not worry 165. I am living in Bombay for the last ten years.
at all (a) had lived (b) have been living
(a) knew (b) have known (c) lived (d) No improvement
(c) know (d) No improvement 166. This scooter is not as efficient as it used to be; instead it is
154. Our country can progress when only people work hard. still a very useful machine.
(a) when people only work hard (a) similarly (b) furthermore
(b) when people work hard only (c) nevertheless (d) No improvement
(c) only when people work hard 167. The teacher as well as his wife were invited.
(d) No improvement (a) was invited (b) were also invited
155. Wake me up when father will come. (c) were being invited (d) No improvement
(a) comes (b) will have come 168. I wish I can help you.
(a) may help (b) could have helped
(b) came (d) No improvement
(c) could help (d) No improvement
156. Do take an umbrella with you lest you do not get wet.
169. My brother is looking forward to meeting his employer
(a) lest you should get wet
tomorrow.
(b) lest you should not get wet
(a) to meet (b) for meeting
(c) lest you might not get wet
(c) that he may meet (d) No improvement
(d) No improvement 170. My father has given his ascent for my long tour.
DIRECTIONS (Qs. 157-181): For the following 25 (twenty five) (a) accent (b) approof
items: Look at the underlined part of each sentence. Below each (c) assent (d) No improvement
sentence are given three possible substitutions for the underlined 171. Heavy work has been thrusted on me.
part. If one of them (a), (b) or (c) is better than the underlined (a) has been thrust (b) has thrusted
part, indicate your response on the Answer Sheet against the (c) has thrust down (d) No improvement
corresponding letter (a), (b) or (c). If none of the substitutions 172. No sooner had he completed his first novel than he fell
improves the sentence, indicate (d) as your response on the seriously ill.
Answer Sheet. Thus a "No improvement" response will be signified (a) he had completed (b) could he completed
by the letter (d). [2016-II] (c) he completed (d) No improvement
EBD_7367
148
B- Sentence Improvement

173. There is many a slip between the cup and lip. (a) widely-ranged (b) wide-ranged
(a) cup and lip (b) cups and lips (c) wide-range (d) No improvement
(c) the cup and the lip (d) No improvement 188. The man disappeared after he was rescuing a boy from
174. We can go out whenever we choose to, isn't it ? drowning.
(a) can't we (b) are we not (a) was rescued (b) has been rescued
(c) don't we (d) No improvement (c) had rescued (d) No improvement
175. He was too conscientious in the discharge of his duties 189. I shall be obliged if you could grant me an interview.
that he could not serve that exploiter for long. (a) give (b) allow
(a) that he would not serve (b) for serving (c) permit (d) No improvement
(c) to serve (d) No improvement 190. By 8.00 in the morning he wrote four letters to his friends.
176. Decide one way or the other; you can't be sitting on the (a) had written (b) had been writing
fence forever. (c) was writing (d) No improvement
(a) be seated on the fence (b) be sitting and fencing 191. But for one witness the accused ought to have been sent to
(c) be dancing on the fence (d) No improvement jail.
177. Raman wants to dispose off his house. (a) would have (b) had
(a) to dispose of (b) the disposal off (c) should have (d) No improvement
(c) the disposal off of (d) No improvement 192. He is now looking about a job.
(a) for (b) after
178. I regret for using objectionable words against a man so
(c) into (d) No improvement
mighty.
193. He did not abide with my decision.
(a) repent for (b) sorry for
(a) to (b) by
(c) regret (d) No improvement
(c) for (d) No improvement
179. Ramesh is working in this factory for the past three months.
194. I was living in Chennai for ten years when I was a child. '
(a) has been working (b) has been worked (a) had lived (b) lived
(c) had worked (d) No improvement (c) had been living (d) No improvement
180. I am waiting for three-quarters of an hour. 195. I didn't feel like going out yesterday, but on account of my
(a) I am waiting since (b) I have waited since son's illness I had to go to the doctor.
(c) I have been waiting for (d) No improvement (a) have (b) might have
181. This book is the more interesting of the three. (c) ought (d) No improvement
(a) the interesting (b) the most interesting 196. Your services are dispensed for.
(c) most interesting (d) No improvement (a) from (b) with
DIRECTIONS (Qs. 182-203): Look at the underlined part of (c) off (d) No improvement
each sentence. Below each sentence are given three possible 197. I didn't go to office because I was ill.
substitutions for the underlined part. If one of them (a), (b) or (a) felt (b) had become
(c) is better than the underlined part, indicate your response on (c) had felt (d) No improvement
the Answer Sheet against the corresponding letter. If none of the 198. The angry neighbours never passed from each other
substitutions improves the sentence, indicate (d) as your without making rude remarks.
response on the Answer Sheet. 2017-I (a) passed on (b) passed against
(c) passed (d) No improvement
182. Go north-east across the mountains till you will reach an 199. The chairman with the other members of the board are
island. touring Europe these days.
(a) reached (b) reach (a) have been on touring (b) is touring
(c) have reached (d) No improvement (c) have toured (d) No improvement
183. It is hard these days to cope with the rising prices. 200. Mahatma Gandhi is called as the Father of the Nation.
(a) cope by (b) cope up with (a) called Father of the Nation
(c) to be coped with (d) No improvement (b) called the Father of the Nation
184. He took a loan of hundred rupees from me. (c) Father of the Nation
(a) debt (b) demand (d) No improvement
(c) advance (d) No improvement 201. Poor Tom laid in the shade of a tree before he could walk
185. From the last five days it has been raining torrentially. further.
(a) Since the last (b) For the last (a) lied (b) lain
(c) Since last (d) No improvement (c) lay (d) No improvement
186. The teacher, along with her three children, were taken to 202. You must accustom yourself with new ideas.
hospital. (a) accustomed with (b) accustom to
(a) was taken (b) had taken (c) accustom yourself to (d) No improvement
(c) had being taken (d) No improvement 203. This telephone number is not existing.
187. The Prime Minister had wide-ranging discussions on the (a) does not exist (b) has not been existing
international situation. (c) has had no existence (d) No improvement
Sentence Improvement B-149

HINTS & SOLUTIONS


1. (d) If I were you, I would do it at once. 25. (b) they are working as usual.
2. (c) They set a strong guard, lest anyone should escape. 26. (d) No improvement.
3. (c) The matter called for an explanation of his conduct. 27. (c) The passengers form queue at the booking counter
Incorrect preposition is used. much before the train arrives.
4. (b) The accused denied having murdered anybody. 28. (b) You must complete this work by Sunday.
When you refuse something it means you do not accept 29. (a) If it rains, the match will be abandoned.
it. To refuse to do something is to say that you won't 30. (c) How long do you think Mr. Kamal has known John?
do that. Where as to deny is something is not true. To 31. (b) I am yet to meet a person who is perfectly satisfied
deny somebody something is to refuse to give it to with his job.
them. 32. (d) The sentence as given in the question is grammatically
5. (b) We need honest workers, not people of doubtful correct and requires no improvement. 'not to stop' is a
integrity. command that fits perfectly in the context of the
Redoubtable (adj.)(Of a person): causing fear and sentence both tense wise and structure wise. Hence,
respect option (d).
Doubtful: Uncertain, undecided and contingent, often 33. (b) While option 'speaking' is extremely inappropriate as
use to admitting of doubt. there is no speech given. 'Saying' fits the sentence
6. (d) By the time he arrived, everybody had gone home. grammatically.
7. (b) There is no alternative, so we must leave now. An 34. (d) Dint means archaic a blow or stroke. While 'by means
alternate is something or someone that serves in the of' does not suit when talking about hard work. 'by
place of another. Whereas alternative is the second doing' sounds abrupt. By dint of makes good sense in
option that does not replace the first. the sentence as here it means due to efforts of. Hence,
8. (a) I cannot hear what she is saying. To hear is to no improvement, option (d).
physically experience the sense of sound. As long as 35. (a) The sentence suggests that the activity of reading
one's ear and brain are capable of processing sound has been going on for a long time and is still continuing.
waves, one can hear. This confirms that the tense of the verb shall be in
To listen is to deliberately apply the ability to hear. present continuous. Only option (a) suits that, have
One who listens is thinking about what is heard, what
been reading is correct.
it means, how to respond, and whether to continue to
36. (c) Option (c) helps to clearly give an understanding of
listen / pay attention.
what has been the exact relief. Also, it provides flow to
9. (a) He is still in vigorous health although he is on the
the sentence and suggests that the relief was the act
wrong side of sixty.
of 'giving' the increase in compensation. Hence, option
10. (a) We are sorry to hear of your father's death.
(c).
11. (a) Babu asked his friend, "Where did you go yesterday?"
37. (c) Resemble already means looks 'like'. Thus, like does
12. (a) He laid on the grass enjoying the sunshine. Wrong
verb tense is used with the subject of this sentence. It not need to follow resemble. Also, it is not a momentary
should be laid in spite of lay. or a temporary act, hence, present continuous shall be
13. (d) Some passengers were flown to Paris on the last trip. not used. For such facts or truths, simple present
14. (a) Fifty miles is a long distance to walk. Fifty miles is 'resembles' should be used. Hence, option (c).
considered a singular unit of distance (made up of a 38. (a) '….. why she wants to see him' is the correct option.
more than one mile. 39. (c) Here, though everybody is plural the word 'who' is
15. (b) After the vote was taken the meeting broke up. singular and the verb 'has' must be in consistency
16. (b) The speaker asked the audience to bear with him for a with 'who'. Also, the sentence is describing an ongoing
few minutes more. act, thus tense should be present. These rules out
17. (c) Many people find it difficult to make both ends meet. 'have' and 'had' respectively. Hence, 'has finished' is
18. (a) More than one person were killed in the accident. most appropriate, option (c).
19. (a) Not a word did they speak to the unfortunate wife 40. (b) This statement provides a hypothetical statement. This
about it. is best conveyed through the word 'could' suggesting
20. (a) The poor villagers have been waiting in bitter cold for the willingness to sing but the incapacity to do the
more than four hours now. same. Hence, option (b).
21. (c) Has is the correct choice. 41. (c) The sentence sounds grammatically incorrect as 'have
22. (b) All but her, had made an attempt. seen him' should not be followed by the adverb 'last'.
23. (a) work hard is correct choice. Use of has/have twice in a sentence is unnecessary.
24. (a) Twenty kilometres is not a great distance in these days Hence, the most appropriate part is '. Since I saw him
of fast moving vehicles. last'. Option (c).
EBD_7367
150
B- Sentence Improvement

42. (b) When stating a fact, we need not use present 67. (c) Since the beginning of the term, we have spent a lot of
continuous 'is having'. Rather, the sentence should time on poetry.
have its verb in the form of simple past. Thus, my 68. (b) Your sister cooks well, doesn't she?
neighbour has two cars is correct. Option (b). 69. (a) Dicken's novels, like those of many other writers are
43. (a) Recital is the act of reciting. Hence (a) his flute recital largely autobiographical.
was highly appreciated. 70. (d) No improvement
44. (b) Try should be followed by 'to make….'. Also, make 71. (a) Never have I seen such breathtaking scenery!
him to understand is wrong usage for the unnecessary 72. (a) I never have written and probably never will write good
use of to twice. Hence, option (b). letters.
45. (d) As are has been used in the first part of the sentence, 73. (c) I think his feet are bigger than any other boy in town.
the negating second part must have a compatible 74. (b) I have hardly studied for this examination.
negative word which is obviously 'aren't. hence, option 75. (c) As you look across the street, you can see lighted
(d) as no improvement is required. windows.
46. (a) A basic grammar rule requires not to follow 'to' after 76. (d) No improvement.
using told as it already implies said to. Rest everything 77. (c) he asked for a cup of tea.
is correct making option (a) the correct choice. 78. (d) No improvement.
47. (b) Unless means if not and it cannot follow a negative 79. (a) We shall not wait for anyone who arrives late.
statement (double negative). Hence, do not work hard 80. (c) We had a hard time during the war.
should be replaced with work hard. Thus option (c) is 81. (a) He aimed a blow at me.
the answer. 82. (a) He waited for her at dinner time.
48. (c) The person in this sentence is projected to be tired 83. (c) He does not have the least idea of it.
and because of that he will not be able to climb the 84. (d) No improvement.
tree. Option (c) "will not be possible" is not apt here 85. (d) No improvement.
as the sentence shows reluctance of the person more 86. (b) The editor regretted that he was unable to make use of
than incapability. the article.
49. (a) When we spend money to buy something we generally 87. (c) He walked softly, lest he should wake the baby.
use preposition "on" rather than "for or in". Through 88. (a) I look forward to meeting you in Delhi.
is used where means is indicated. Thus option (a) is 89. (b) I do not know where he has gone.
the answer. 90. (d) No improvement.
50. (d) In general speaking, we use the word denied with 91. (b) This is the boy whom I talked to you about.
charge. Denial means to outright reject something 92. (b) doubtful is the most appropriate word.
completely without leaving any scope for acceptance. 93. (c) with is the most appropriate word.
So the accused is completely rejecting the accusation. 94. (b) alternate is no word. Alternative is the most appropriate
Refuse means not accepting which is not appropriate word.
here. Rebut means overthrow by argument. 95. (d) The phrase if ‘I were you’ is known as the subjective
51. (b) When a change is brought into existence, it is effected. mood Ex: If I were you I would refuse to marry her.
Affect is related to cause whereas inject means to infuse 96. (c) The conjunction ‘lest’ is followed by ‘should’.
internally. Inflict is generally used with pain or pleasure. 97. (c) ‘for’ is the most appropriate word.
Thus here the word effect suits aptly to the situation. 98. (b) ‘denied’ is the most appropriate word.
52. (a) There has been no rain in our village for the last six 99. (c) ‘fast and loose’ is the most appropriate word. Fast and
months. loose means reckless, irresponsible manner.
53. (c) The police investigated the matter. 100. (a) ‘wrong’ is the most appropriate word.
54. (a) Ramachandra Murthy and his family have been in 101. (c) ‘The’ is the most appropriate word.
Guyana since 1985. 102. (a) ‘is’ is the most appropriate word.
55. (c) I have been living in this town since 1980. 103. (c) ‘is’ is the most appropriate word.
56. (b) If I were you I should tell him the truth. 104. (a) broke down is the most appropriate word.
57. (c) He is better than any other boy in the class. 105. (b) ‘with’ is the most appropriate word.
58. (c) Anil ought not to have told me your secret, but he did. 106. (c) Such men
59. (b) If I were he I would have not accepted the offer. 107. (c) ‘Both ends meet’ is the most appropriate phrase.
60. (d) No improvement 108. (b) ‘of carrying these loads’ is the most appropriate phrase.
61. (b) We now come to the important question of where this 109. (d) No improvement.
great swarm of galaxies has come from. 110. (a) ‘from his post’ is the most appropriate word.
62. (a) Less rainfall means fewer traffic accidents, according 111. (d) no improvement.
to the expert's report on highway safely.
63. (b) I did not see you at the party yesterday. 112. (d) No improvement
64. (a) Ajeet is a better scholar than his brother. 113. (c) “have been playing” here refers to present perfect
65. (b) I did not wait for him because he had gone out before I continuous action of the students playing volley-ball.
arrived. 114. (c) Fell through is the correct phrase use of through with
66. (a) Whenever I saw him, he had been reading the same fell in the given context denotes the failure of the trip.
novel. 115. (d) No improvement
Sentence Improvement B-151
116. (c) ‘the best’ is most appropriate substitution as the 156. (a)
degree of comparison here is superlative. 157. (c) The use of 'suppose' and 'if' together in superfluous
117. (c) “doesn’t it” is the correct substitution because when so, 'suppose' should be omitted to make the sentence
the verb in the main sentence is in simple present, we correct.
form the question tag with do/does. 158. (d) The sentence is correct hence, it does not require any
118. (b) ‘he is arrested’ is the correct substitution because improvement.
‘suppose’ and ‘if’shouldn't be used together. 159. (a) The modal web 'should' is used to express purpose
119. (a) 120. (b) after 'lest'. Therefore, 'he may miss' should be replaced
121. (a) ‘both ends meet’ is the right phrase and here as it with 'he should miss' to make the sentence
denotes Radha’s situation of not being able to earn grammatically correct.
enough money. 160. (a) 'to taking part' should be replaced with' to take part' to
122. (c) ‘turn off the fan’ is the correct substitution as we make the sentence grammatically correct.
generally use ‘ turn off’ for switching off an electrical 161. (b) There is no word such as 'poors' because the plural
appliance. from of 'poor' is also 'poor' and not 'poors'. Secondly,
123. (c) ‘a higher salary’ is most appropriate because more is according to subject-verb agreement, 'interest' should
used in comparison but in the given context there is be substituted with 'interests'. Hence, option (b) is the
no comparison as such. correct answer.
124. (b) 125. (a) 126. (c) 127. (c) 128. (b) 162. (d) The sentence is correct hence, it does not require any
improvement.
129. (a) ‘prior to’ refers to a decision taken before or until.
163. (c) When the subject is pronoun 'one', then the
130. (b) ‘a’ will replace ‘the’ as ‘the’ cannot be used before possessive form' one self' is used. So, option (c) is the
university as used for a specifly one. correct answer.
131. (d) No improvement. 164. (a) The sentence begins with the conjunction 'unless'
132 (a) Replace ‘when’ with ‘than’ because in any case the which is used instead of 'if…….not'. Hence, 'are very'
second event occurs immediately after the first, we should be used instead of 'are not very' to make the
express that using the structure “no sooner .... than”. sentence correct.
133. (d) 134. (b) 135. (a) 136. (d) 165. (b) The sentence shows an action which began in the
137. (c) 'of' is the correct substitution in the given sentence as part and is still continuing. Therefore, Present perfect
'of' is used for belonging to, relating to, or connected continuous tense 'have been living' should be used to
with and in the given sentence the thief is connected make the sentence grammatically correct.
to theft. 166. (c) 'Instead' should be replaced with 'nevertheless' which
138. (b) 139. (c) 140. (d) 141. (a) 142. (a) means in spite of that' to express the meaning of the
143. (d) The sentence doesn't need any improvement as 'laying' sentence correctly.
means to put someone or something in a careful way, 167. (a) The subject of the sentence is 'the teacher' which is
especially so that they are lying flat. singular, therefore, the verb should be used in singular
144. (a) form i.e. 'was invited' and not 'were invited'.
145. (b) 168. (c) Since the sentence denotes a possibility therefore.
146. (a) 'lay' will be the correct replacement as lay is the past 'could help' should be used instead of 'can help' to
form of 'lie' which means the subject is setting itself make the sentence correct.
169. (d) The sentence is correct hence, it does not require any
down for rest or recline.
improvement.
147. (b) 148. (a) 170. (c) The correct word to be used for permission is 'assent'.
149. (b) 'any' will substitute 'little' as in the given sentence 'any' Therefore, 'ascent' should be replaced with 'assent'.
is used as an adverb which shows that the old lady 171. (d) Then sentence is correct hence, it does not require
cannot work at all. Therefore 'any' is used to emphasise any improvement.
the degree. 172. (d) The sentence is correct hence, it does not require any
150. (a) 'deceive' will be replaced by ' deceiving' which is the improvement.
present participle of decieve used for forming 173. (c) The underlined part used in the sentence is a proverb
continuous tenses. which has not been used correctly. The correct proverb
151. (c) 'seen' will replace 'saw' because 'seen' is a past participle is 'the cup and the lip' therefore, option (c) should be
used with auxiliary verbs such as has, have, etc. used instead of the underlined part.
152. (a) 'belonged' will be correct substitution of 'belongs' as 174. (a) The question tag used in the sentence is wrongly used.
the word 'belonged' denotes something which was the The correct tag should be 'can't we'. So option (a) is
property of someone in past whereas 'belongs' is in the correct answer.
the present tense. 175. (d) The sentence is correct hence, it does not require any
153. (a) 'knew' will replace 'were knowing' because the first part improvement.
of the sentence refers to past as since is used in it and 176. (d) The sentence is correct hence, it does not require any
'knew' is the past form of 'know' improvement.
177. (a) The sentence expresses the meaning that Raman wants
154. (c)
to get rid of his house by giving or selling it to someone
155. (a) 'comes' will be the correct substitute of 'will come' as
'comes' denotes a present situation when the child's else. Therefore, the correct preposition to be used with
father will come but 'will come' refers to the event dispose is 'of' and not 'off'. Hence, option (a) is the
happening in future after a long time ahead. correct answer.
EBD_7367
152
B- Sentence Improvement

178. (c) In the given sentence, 'regret for' needs to be grant’ should be replaced with ‘allow’ to make the
substituted with 'regret' as regret should not be sentence correct.
followed by a preposition. Therefore, option (c) is the 190. (a) There is tense error in the underlined part of sentence.
correct answer. As the sentence indicates a completed action by 8 in
179. (a) The sentence denotes an action which began in the the morning, so the verb in the sentence should be in
part and is still continuing. Hence present perfect past perfect tense. Therefore, ‘wrote’ should be
continuous tense should be used. Therefore 'is replaced with ‘had written’ to make the sentence
working' should be substituted with 'has been working' correct.
to make the sentence grammatically correct. 191. (a) ‘ought to have’ should be replaced with ‘would have’
180. (c) The sentence denotes an action which began in the to make the sentence correct.
past and is still continuing. Hence, present perfect 192. (a) There is incorrect use of preposition ‘about’ in the
continuous tense should be used and not present sentence. It should be replaced with ‘for’ to make the
continuous. Therefore, 'I am waiting' should be replaced sentence correct.
with 'I have been waiting for' to make the sentence 193. (b) There is incorrect use of phrase ‘abide with’ in the
grammatically correct. underlined part. ‘With’ should be replaced with ‘by ’
181. (b) 'more' in the underlined part should be replaced with to make the phrase correct.
'most to show the highest degree of the quality i.e. the 194. (c) There is tense error in the underlined part of the
superlative degree of the adjective should be used. sentence. The verb should be in past perfect
Hence, option (b) is the correct choice. continuous tense. So, ‘was living’ should be replaced
182. (b) There is incorrect use of ‘will’ in the underlined part with had been living to make the sentence correct.
of the sentence. It should be omitted to make the 195. (d) The sentence is correct. No improvement is required.
sentence correct. 196. (b) There is incorrect use of phrase ‘dispensed for’ in the
183. (d) The sentence is correct. No improvement is required. sentence. It should be replaced with ‘dispensed with’
184. (d) The sentence is correct. No improvement is required. which is the correct phrase.
197. (d) The sentence is correct. No improvement is required.
185. (b) There is incorrect use of preposition ‘from’ in the
198. (c) There is incorrect use of preposition ‘from’ in the
underlined part of the sentence. It should be replaced
underlined part. It should be omitted to make the
with ‘for’ to make the sentence correct.
sentence correct.
186. (a) There is subject-verb error in the underlined part of
199. (b) There is subject verb error in the underlined part. As
the sentence. Subject ‘the teacher’ of the sentence is
the subject of the sentence is singular, it should take
singular, so, it should take a singular verb. So, ‘were’
a singular verb. So, ‘are touching’ should be replaced
should be replaced with ‘was’ to make the sentence
with ‘is touching’ to make sentence correct.
correct.
200. (b) There is incorrect use of adverb ‘as’ in the underlined
187. (b) ‘Wide ranged’ should be used instead of ‘wide-
part. It should be omitted to make the sentence correct.
ranging’ because adjective form should be used.
201. (c) There is incorrect use of verb form in the underlined
188. (c) There is tense error in the underlined part. As the
part. To make sentence correct, ‘laid’ should be
sentence is in past tense and there is use of ‘after’
replaced with ‘lay’ which is past tense of ‘lie’.
after. The man disappeared which indicates that the
202. (c) There is incorrect use of preposition ‘with’ in the
next part should be in past perfect tense. So, ‘was
underlined part. It should be replaced with ‘to’ to make
rescuing’ should be replaced with ‘had rescued’ to
the sentence correct.
make the sentence correct.
203. (a) There is tense error in the underlined part. The
189. (b) There is incorrect word use in the underlined part.
underlined part in present continuous tense, but it
Since the request has been made for an interview, so, should be in simple present tense. So ‘is not existing’
‘allow’ is the correct word in this context. So ‘could
should be replaced with ‘does not exist’.
Selecting Words B-153

C HA P T E R
SELECTING WORDS
32
DIRECTIONS (Qs. 1-12) : In the following passage, at certain frank and courteous —– 22. (a) discussion./ (b) bargaining./ (c)
points you are given a choice of three words, one of which fits argument.The problem can be solved only —– 23. (a) through/
the meaning of the passage. Choose the best word from each (b) by/ (c) from mutual understanding, and not through insistence
bracket. Mark the letter, viz. (a), (b) or (c), relating to this word on a —– 24. (a) vital, / (b) hard, / (c) rigid observance of a code of
on your Answer Sheet. Examples Y and Z have been solved for —– 25. (a) agreement./ (b) conduct./ (c) rights. If we truly desire
you. [2012-I] a solution to this —– 26. (a) difficult problem,/ (b) doubtful/ (c)
Y. the [(a) boy/(b) horse/(c) dog] was in the school in Simla small problem we should be prepared for —– 27. (a) one/ (b) a/ (c)
Z. (a) She/ (b) It/ (c) He was homesick no compromise. The system cannot be changed overnight. There
Explanation : —– 28. (a) was/ (b) is/ (c) has been a great need for patience in
Out of the list given in item Y, only boy is the correct answer —– 29. (a) dealing/ (b) handling/ (c) tackling with this complex
because usually a boy, and not a horse or a dog, attends matter. Problems are —– 30. (a) immensely/ (b) best/ (c) rarely
school. So (a) is to be marked on the Answer Sheet for item solved through the application of —– 31. (a) brain/ (b) wisdom/
Y. A boy is usually referred to as "he", so for item Z, the (c) theory and certainly not through angry —– 32. (a) enmity./ (b)
letter (c) is the correct answer. Notice that to solve the first hostility./ (c) conflicts. At every step in our life we —– 33. (a)
item Y you have to read the rest of the sentence and then understand,/ (b) compromise,/ (c) discuss, then why should we
see what fits best. take up rigid stand in matters which concern us intimately ?
PASSAGE DIRECTIONS (Qs. 34-39) : Each of the questions in this section
According to a report in yesterday's newspaper —– 1. (a) once/ has a sentence with a blank space and four words given after
(b) a/ (c) new police dog was taken to Raj Bhavan —– 2. (a) at/ (b) the sentence. Select whichever word you consider most
next/ (c) on Monday. This was to trace the —–3. (a) killers/ (b) appropriate for the blank space and indicate your choice on
dogs/ (c) police of the "very important horse" which —– 4. (a) the Answer Sheet. [2013-I]
has/ (b) were/ (c) was reported missing on Sunday. The dog picked 34. An accomplice is a partner in ___________.
—– 5. (a) on/ (b) at/ (c) up the scent on some traces of —– 6. (a) (a) business (b) crime
those/ (b) blood/ (c) report and ran a few yards before losing the (c) construction (d) gambling
—– 7. (a) bet/ (b) track/ (c) game; The police have launched a 35. A person who pretends to be what he is not is called
vigorous —– 8. (a) search/ (b) investigation/(c) campaign into an___________.
the whole affair. They have —– 9. (a) given up/ (b) requisitioned/ (a) imbiber (b) impresario
(c) report the services of a forensic expert, —– 10. (a) a/ (b) an/ (c) (c) imitator (d) imposter
two fingerprint expert and a photographer. —– 11. So (a) There/ 36. His ___________ nature would not let him leave his office
(b) We/ (c) are now fourteen horses at Raj Bhavan —– 12. (a) before 5 p.m.
who / (b) where/ (c) which are kept in a large shed near the gate. (a) honest (b) selfish
DIRECTIONS (Qs. 13-33) : In the following passage, at certain (c) unscrupulous (d) conscientious
points you are given a choice of three words, one of which fits 37. The Committee’s appeal to the people for money __________
the meaning of the passage. Choose the best word from each little response.
bracket. (a) evoked (b) provided
Mark the letter viz., (a), (b) or (c), relating to this word on your (c) provoked (d) prevented
Answer Sheet. [2012-II] 38. Too many skyscrapers ___________ the view along the
PASSAGE beach.
There has been quite some talk in recent times about the status of (a) reveal (b) obstruct
women in our society, some of it sentimental and some of it, angry. (c) make (d) clear
But the status of women —– 13. (a) can/ (b) shall/ (c) should be 39. Though he has several interim plans, his ___________ aim
improved only if we join —– 14. (a) ourselves/ (b) heads/(c) hands is to become a billionaire.
earnestly to achieve the desired —– 15. (a) goal / (b) intention /(c) (a) absolute (b) determined
wish. Obviously, it cannot be done merely through —– 16. (a) (c) only (d) ultimate
enforcing/ (b) enacting/ (c) passing legislation. It requires an DIRECTIONS (Qs. 40-51) : In the following two passages, at
intelligent and sympathetic —– 17. (a) presentation/ (b) certain points you are given a choice of three words in a bracket,
understanding / (c) vision of each others' problems and a —– 18. one of which fits the meaning of the passage. Choose the best
(a) consent/ (b) co-operation/ (c) willingness to give up pointless word from each bracket. Mark the letter, viz., (a), (b) or (c),
claims —– 19. (a) of /(b) about/ (c) over superiority. The delicate relating to this word on your Answer Sheet. [2013-I]
adjustment can be achieved not —– 20. (a) by/ (b) through/ (c) PASSAGE-I
from conflict but by peaceful and dispassionate —– 21. (a) A young man riding a motor-cycle approached a policeman in a
behaviour, / (b) confrontation,/ (c) means, in other words, through market place and sought his assistance in reaching a particular
EBD_7367
B-154 Selecting Words

locality. The policeman gave him some —– 40. (a) instructions/ PASSAGE-II
(b) directions/ (c) advice and the motor-cyclist left. He —– Sometimes the messages are sent with —– 62. (a) many / (b) few/
41. (a) reached / (b) come / (c) went back after some time and (c) no accompanying words and we speak in —– 63. (a) body/ (b)
—– 42. (a) asked/ (b) convinced/ (c) told the policeman that he signal/ (c) foreign language alone. But, what gestures make
could not —– 43. (a) find /(b) hit/ (c) see the place. The policeman —– 64. (a) with body (b) up/ (c) into language? Most of us are
got the —– 44. (a) idea/ (b) inclination/ (c) urge to help him and —– 65. (a) easy/ (b) familiar/ (c) efficient with the common hand
agreed to —– 45. (a) start/ (b) go/ (c) proceed with the motor- gestures. Some —– 66. (a) students (b) people (c) officers cannot
cyclist. On reaching the —– 46. (a) station/ (b) destination / (c) talk without using their hands. —– 67. (a) we/ (b) Arms/ (c) They
spot the motor-cyclist left in a hurry leaving the policeman on reach out as they explain almost —– 68. (a) exacting / (b) shapping/
the road. The policeman was surprised and returned to his spot. (c) changing their words, emphasizing and exaggerating and
A little latter, senior police office reached the place and took the —– 69. (a) teaching / (b) holding/ (c) punctuating with their hands.
policeman to task for dereliction of duty. Other people hardly —– 70. (a) adjust/ (b) use/ (c) wave their
PASSAGE-II hands at all when they —– 71. (a) lecture/ (b) unite/ (c) talk.
Picasso is considered by many as the greatest painter of the DIRECTIONS (Qs. 72-91): In the following passage at certain
modern age. There are stories and legends about him. Once, on points you are given a choice of three words, one of which fits
a beach in Southern France, a little boy, obviously. sent by his the meaning of the passage. Choose the best word. Mark the
parents, approached Picasso —– 47. (a) with/ (b) by/ (c) on a letter, viz. (a), (b) or (c) relating to this word on your Answer
sheet of paper and begged for a small autographed drawing. Sheet. [2014-II]
Picasso —– 48. (a) painted/ (b) thought/ (c) stood for a moment, PASSAGE
then tore up the paper, took —– 49. (a) back/ (b) some/ (c) aside We know that the average depth of the sea is about two and a
colour crayons, drew designs on the boy’s chest —– 50. (a) half miles, but in a few places it is very deep indeed–over six
near/(b) and/ (c) to neck and signed his ‘work’ and sent the miles. The air presses upon our bodies with a weight of about
youngster —– 51. (a) after/ (b) again/(c) back to his parents. fifteen pounds to the square inch at —– 72. (a) sea-water/ (b)
DIRECTIONS (Qs. 52-71) : In the following passages at certain sea-level/ (c) sea-bed We are used to this air pressure —– 73. (a)
points you are given a choice of three words, one of which fits do/ (b) did/ (c) does not notice it. In the sea this —– 74. (a)
the meaning of the passage. Choose the word which best fits the weight/ (b) volume/ (c) pressure is doubled at a depth of thirty-
meaning of the passage and mark the corresponding letter viz five feet, and it —– 75. (a) expands/ (b) decreases/ (c) increases
(a), (b) or (c) on your Answer Sheet. Examples Y and Z have at this rate for greater depths. In the great deeps —– 76. (a) off/
been solved for you. [2014-I] (b) of/ (c) on the Philippine Islands, a man would be squeezed
Y (a) Boy was in the school in Shimla. and utterly crushed by a pressure of —– 77. (a) severe / (b)
(b) Horse several/ (c) sheer tons per square inch. The pressure near the
(c) Dog ocean floor is —– 78. (a) such/ (b) not/ (c) so great that if you
Z (a) She was homesick. were to weigh a piece of wood and —– 79. (a) measure/ (b) follow/
(b) It (c) lower it to a great depth and then pull it —– 80. (a) up/ (b) off/
(c) He (c) down again it would no longer float, —– 81. (a) but/ (b) for/ (c)
Explanation : when it would have become waterlogged. All the tiny wood cells
Out of the list given in item Y, only boy is the correct answer and cavities —– 82. (a) should/ (b) could/ (c) would have burst
because usually a boy, and not a horse or a dog, attends and become filled with water —– 83. (a) know / (b) have known/
school. So (a) is to be marked on the Answer Sheet for item Y. (c) are knowing that animals live at a depth of three miles and
A boy is usually referred to as "he", so for item Z, the letter more and we wonder —– 84. (a) why/ (b) how/ (c) what this can
be. This bodies of animals down —– 85. (a) here/ (b) where/ (c)
(c) is correct answer. Notice that to solve this kind of items
there are almost entirely filled with water, and —– 86. (a) this/ (b)
you have to read the preceding or succeeding sentences of
these/ (c) thus saves them from being crushed. However, many
the given passage.
of —– 87. (a) such/ (b) those/ (c) there animals contain some
PASSAGE-I
gases as well, for —– 88. (a) then/ (b) since/ (c) when they are
One of the most interesting new books published recently is
captured in nets and drawn —– 89. (a) on/ (b) to/ (c) from the
"Spaceship" by prof. E.C. Walker.
surface these gases expand so much that the animal —– 90. (a)
—– 52. (a) is / (b) have been/ (c) will be is like a spaceship, and all immediately/ (b) eventually/ (c) actually explodes. Its body is
the 400 million people (a) over earth are passengers on it. And we
torn to shreds as it —– 91. (a) bursts/ (b) jumps/ (c) lands.
are heading —– 53. (a) over / (b) on (c) upon earth are passengers
on it. And we are heading —– 54. (a) about/ (b) to (c) towards a DIRECTIONS (92-111): In the following passage at certain
disaster. The levels of atmospheric pollution —– 55. (a) increasing points you are given a choice of three words marked (a), (b)
(b) arriving/ (c) coming/ in the cities and industrial areas of the and (c), one of which fits the meaning of the passage. Choose
world could in time change the weather patterns of the earth, the best word out of the three. Mark the letter, viz., (a), (b) or
raising the temperature —– 56. (a) in / (b) of/ (c) for/ the whole (c), relating to this word on your Answer Sheet. Examples K
planet. If this rose a few —– 57. (a) degrees/ (b) steps/ (c) miles and L have been solved for you.
the deserts of the world would expand to double their size. The K : The (a) boy/ (b) horse/ (c) dog was in the school in Simla.
polar ice caps would start melting. If the polar ice caps melted the L : (a) She/ (b) It/ (c) He was homesick.
—– 58. (a) water/ (b) ice/ (c) sea/ level all over would rise Explanation :
—– 59. (a) in/ (b) by/ (c) to in about 60 meters. Prof walker's Out of the list given in item K, only 'boy' is the correct answer
—– 60. (a) idea/ (b) thought/ (c) book is not at all about gloom because usually, a boy, and not a horse or a dog, attends
and doom. He admits that the —– 61. (a) ideas/ (b) solutions/ (c) school. So '(a)' is to be marked on the Answer Sheet for item
changes he describes could take thousands of years.
Selecting Words 155
B-

K. A boy is usually referred to as 'he', so for item L, '(c)' is the instrument of his own 122. (a) experiment, / (b) construction, / (c)
correct answer. Notice that to solve the first you have to read calculation, Galileo observed Jupiter and four of 123. (a) the / (b)
the rest of the sentence and then see what fits best. their/(c) its moons, the phases of Venus and the spots on the
[2015-I] sun. His 124. (a) observing / (b) observations / (c) observed and
PASSAGE calculations confirrmed that Copernicus and Kepler were right.
We all like listening to —– 92. (a) lectures/ (b) theories/(c) stories He saw 125. (a) by / (b) from / (c) with his own eyes and made
and the person who is good —– 93. (a) on / (b) at/(c) by telling
other people 126. (a) to see/ (b) see / (c) seeing too that the earth
stories will always be a —– 94. (a) popular/ (b) good/ (c) necessary
member of any company. The art —– 95. (a) on/ (b) of/ (c) at was not the fixed center of 127. (a) an / (b) a/ (c) the universe as
good story-telling covers much more than —– 96. (a) describing Ptolemy had said. Galileo 128. (a) instead/ (b) also / (c) therefore
/ (b) making / (c) showing up fabulous adventures; it includes made some important discoveries in mechanics. He did not 129.
telling —– 97. (a) about/ (b) by/ (c) for the doings of living people (a) as legend says/ (b) as stories say / (c) as people say drop
or —– 98. (a) insignificant/ (b) dead/ (c) famous men and women cannon balls from the Leaning Tower of Pisa 130. (a) having
of the past, —– 99. (a) in/ (b) about/ (c) through your own travels proved/ (b) proving / (c) to prove that all bodies fall at the 131. (a)
and adventures and —– 100. (a) experiences/ (b) desires/ (c) same/ (b) equal / (c) similar speed, but he did roll balls 132. (a)
worries about the books you have read —– 101. (a) either/ (b) all/ downside/ (b) down over / (c) down a slope to show that the 133.
(c) and the films you have seen. Practising —– 102. (a) an/ (b) a/ (a) track/ (b) distance / (c) path a body falls is proportionate 134.
(c) the art of story telling can be very —– 103. (a) useful (b) (a) to / (b) of/ (c) with the square of the time it takes to fall. Galileo
dangerous/ (c) contagious too/. It will help you to —– 104. (a) also noticed the regular 135. (a) swings / (b) swinging / (c) swaying
listen / (b) remember/ (c) think clearly and logically, to sort out — of the lamps in Pisa Cathedral; 136. (a) these / (b) this / (c) those
– 105. (a) her (b) their / (c) your ideas/to express yourself clearly gave him the idea of the pendulum, a device that enabled him to
and —– 106. (a) timidly,/ (b) effectively/ (c) bluntly to gain and
make the clock a scientific instrument for the first time.
hold the attention —– 107. (a) at/ (b) on/ (c) of others. It will help
you to —– 108. (a) shake / (b) lay (c) hit off shyness and self- DIRECTIONS (Qs. 137-146): Each of the following sentences
consciousness, and give —– 109. (a) she / (b) you/ (c) I that has a blank space and four words are given below it. Select the
feeling of freedom —– 110. (a) then / (b) what/ (c) which is so word you consider most appropriate for the blank space and
important to —– 111. (a) success/ (b) victory/ (c) gain in life.
indicate your choice on the answer sheet. [2015-II]
DIRECTIONS (Qs. 112-136) : In the following passage at certain 137. We must not _______the real and important advances
points you are given a choice of three words marked (a), (b) science has made.
and (c), one of which fits the meaning of the passage. Choose
(a) oppose (b) question
the best word out of the three. Mark the letter, viz., (a), (b) or
(c), relating to this word on your Answer Sheet. Examples K (c) ignore (d) doubt
and L have been solved for you. [2015-II 138. These trousers are too long, _______ the length please.
(a) diminish (b) lessen
K The (a) boy/ (b) horse/ (c) dog was in the school in Simla.
L (a) She/ (b) It/ (c) He was homesick. (c) curtail (d) reduce
139. "What did you think of the film?" "________ I didn't like it
Explanation: Out of the list given in item K, only 'boy' is the
correct answer because usually, a boy, and not a horse or a dog, very much."
attends school. So '(a)' is to be marked on the Answer Sheet for (a) To be honest (b) Being honest
item K. A boy is usually referred to as 'he', so for item L, '(c)' is the (c) To be fair (d) In honesty
correct answer. Notice that to solve the first item K you have to 140. He _________ in this school since 2010.
read the rest of the sentence and then see what fits best. (a) studied (b) was studying
PASSAGE-I (c) has been studying (d) had studied
What 112. (a) can / (b) will / (c) must happen to them after us? 141. "His parents died when he was young." "I think that's why
This most 113. (a) joyous/(b) distressing / (c) distracting question he has _________ problems."
continually torments the parents of these 114. (a) unethical/(b) (a) too many (b) so many
unnatural/ (c) unfortunate children. So they are mainly interested (c) any more (d) much more
in 115. (a) providing / (b) making / (c) giving some kind of
142. _________ if he is willing to fit in with the plans of the
vocational training for them 116. (a) But /(b) Yet /(c) Hence special
group.
schools for such children, spread all 117. (a) above / (b) under /
(c) over the world, lay emphasis 118. (a) However,/ (b) (a) There is no objection to him joining the party
Accordingly/(c) Similarly, on vocational training. They are taught (b) There is no objection on his joining the Party
to make paper bags, 119. (a) dubious/(b) simple / (c) clumsy wall (c) There is no objection to his joining the Part
hangings etc. This, of course is quite 120. (a) agreeable / (b) (d) There was no objection for his joining the party
astute/ (c) additional and admirable. But what about play and 143. Having secured the highest marks in the class, _________,
121. (a) sport?/ (b) game ?/ (c) grounds? _________.
PASSAGE-II (a) the college had offered him a scholarship
Galileo used mathematical calculation as well as observation of (b) he was offered a scholarship by the college
nature and was the first astronomer to use a telescope. With an
EBD_7367
156
B- Selecting Words

(c) a scholarship was offered him by the college 156. Creative people are often ________ with their own
(d) a college scholarship had been offered to him uniqueness.
144. After they _________ lunch, the boys ran outside. (a) obsessed (b) deranged
(c) unbalanced (d) dissatisfied
(a) have eaten (b) had eaten
(c) were eating (d) would cat DIRECTIONS (Qs. 157-176) : In the following passage at certain
145. As he was _________ and had saved enough money, his points you are given a choice of three words marked (a) , (b)
family escaped misery when he died suddenly. and (c), one of which fits the meaning of the passage. Choose
(a) prudent (b) preparatory the best word out of the three. Mark the letter, viz., (a), (b) or
(c), relating to this word on your Answer Sheet. Examples K
(c) persistent (d) providential
and L have been solved for you. [2016-I]
146. Some mysteries remain in desert research : especially relating
to why some regions, once fertile, are now _________. K L
The (a) boy was in the school in Simla, (a) She was home
(a) blossoming (b) cultivable
sick.
(c) barren (d) irrigated (b) horse (b) It
DIRECTIONS (Qs. 147-156) : Each of the following sentences (c) dog (c) He
in this section has a blank space and four words are given after Explanation : Out of the list given in item K, only. ‘boy’ is the
each sentence. Select whichever word you consider most correct answer because usually, a boy, and not a horse or a dog,
appropriate for the blank space and indicate your response on attends school. So ‘(a)’ is to be marked on the Answer Sheet for
the Answer Sheet accordingly. [2016-I] item K. A boy is usually referred to as ‘he’, so for item L, ‘(c)’ is
the correct answer. Notice that to solve the first item K you have
147. The mounting pressure was so over whelming that he to read the rest of the sentence and then see what fits best.
ultimately ________ to her wish. PASSAGE
(a) yielded in (b) gave in
(e) cowed in (d) agreed in I was engaged in many activities and I wanted a proper
148. Authority ________ when it is not supported by the moral reconciliation between.
purity of its user. activity and thought. Thought without 157 (a) wish is undeve-
(a) prevails (b) entails loped thought. Action (b) action / (c) idea without 158 (a) thought
(c) crumbles (d) waits /(b) wish others/(c) idea is folly. Of course we 159 (a) never/ (b)
149. In a developing country like India some industries will have belatedly / (c) sometimes act on some impulse or 160 (a) peaceful
to be brought within public ________ and control, for (b) uncontrollable / (c) indisputable urge If suddenly you throw
other-wise rapid growth of the economy may be impossible. 161 (a) no / (b) an / (c) a brick at me and my162. (a) hand/ (b)
(a) perspective (b) hegemony wrist/ (c) finger goes up in front to 163 (a) stimulate/(b) rescue/
(c) observation (d) ownership (c) protect myself, it is an automatic, 164 (a) uncontrollable/(b)
150. Gandhiji conceived of the idea of channelizing the powerful instinctive/ (c) impulsive action and not a result 165 (a) to/(b) in/
currents of the united mass movement so as to give the (c) of deliberate thought Our living is 166 (a) made / (b)
utmost impetus to the national ________ for independence conditioned / (c) developed by a series of automatic 167 (a)
(a) struggle (b) conflict thoughts/(b) actions /(c) wishes from morning till night. Anything
(c) onslaught (d) march 168 (a) we/(b) I /(c) they do outside that common range of 169 (a)
151. Because of his ________ habits, he could not save much thoughts/ (b) ideas / (c) action however, has to be 170 (a) proceeded
money.
/ (b) preceded / (c) followed by some measure of thinking. 171 (a)
(a) extravagant (b) frugal
Some/ (b) If / (c) The more action and thought are 172 (a)
(c) unsavoury (d) bad
developed / (b) allied / (c) hostile and integrated, the more effective
152. Socrates was ________ of spreading discontent among
young men of Athens and of trying to destroy their faith in 173 (a) they / (b) thoughts / (c) we become and the happier you 174
the old gods. (a) appear. / (b) develop / (c) grow There will then be no 175 (a)
(a) rebuked (b) disparaged reconciliation /(b) conflict / (c) inflict between a wish to do something
(c) accused (d) demonised and 176 (a) inability/ (b) probability / (c) Plausibility to act.
153. The robbers fell ________ amongst themselves over the
sharing of the loot. DIRECTIONS (Qs. 177-206) : In the following passages, at
(a) out (b) through certain points you are given a choice of three words marked
(c) off (d) across (a), (b) and (c), one of which fits the meaning of the passage.
154. A really sophisticated person would never be ________ Choose the best word out of the three. Mark the letter, viz., (a),
enough to think that he is always right. (b) or (c), relating to this word on your Answer Sheet. Examples
(a) reverent (b) naive K and L have been solved for you. [2016-II]
(c) articulate (d) humble K L
155. Speeding and blocking are traffic offences which lead to
The (a) boy was in the school in Simla. (a) She was homesick.
________ accidents
(b) horse (b) It
(a) troublesome (b) final
(c) dog (c) He
(c) great (d) gruesome
Selecting Words 157
B-

Explanation : Out of the list given in item K, only, 'boy' is the 203. (a) liquid/ (b) solid/ (c) soiled
correct answer because usually, a boy, and not a horse or a dog, 204. metal shapes are taken out of the moulds.
attends school. So '(a)' is to be marked on the Answer Sheet for (a) That process is called casting. Metal objects that/ (b)
item K. A boy is usually referred to as 'he', so for item L. '(c)' is the This/ (c) Thus it
correct answer. Notice that to solve the first item K, you have to 205. (a) have been fashion by this process break more easily
read the rest of the sentence and then see what fits best. than those made by hammering. When/ (b) have been
Passage - I fashioned/ (c) are in fashion
Many of us believe that science is something modern, 206. (a) those metals are melted they can be mixed together. The
177. (a) if the truth is that/ (b) though/ (c) unless mixture is called an alloy./ (b) two/ (c) all
178. (a) men has been/ (b) people/ (c) man
DIRECTIONS (Qs. 207-216): Each of the following sentences
179. using science for
in this section has a blank space and four words or group of
(a) the very long time. However, it has/ (b) a/ (c) that
words given after the sentence. Select whichever word or group
180. (a) has a greater effect on human lives in/ (b) have/ (c) had
of words you consider most appropriate for the blank space and
181. the last 25
indicate your response on the Answer Sheet accordingly.
(a) and 30 years than in the hundreds of years/ (b) or / (c)
[2017-II]
either
182. (a) from the invention of the plough. The/ (b) for/ (c)since 207. With the less rapid expansion of the economy, we should
183. (a) marvellous gifts of science have made modern life/ make ________ progress toward stable price levels.
(b) costly/ (c) inexpensive (a) detailed (b) substantial
184. (a) dull and comfortable. But science has/ (b) exciting/ (c) (c) definite (d) infinite
aimless 208. At times he gets very angry, and then no one can
185. (a) at the same time created new problems. One of these _________ him.
which may become/ (b) in/ (c) within (a) prevent (b) humour
186. (a) bad in the years to/ (b) worse/ (c) good (c) mollify (d) satisfy
187. come, is (a) those of 'jetlag'. With the coming of modern 209. Many people today have fallen into utter confusion of values
jets, flying at more than 900 km an hour, the/ (b) this/ (c) that with the result that they cannot ________ the good from
188. world (a) can become very small indeed. Today if you/ the bad.
(b) become/ (c) has become (a) divide (b) differentiate
189. (a) leave New Delhi at 4.00 in the morning,/ (b) will leave/ (c) see (d) alter
(c) would leave 210. If Mohan _________ at 5 a.m., he would not have missed
190. you (a) will eat an early breakfast in the sky/ (b) can/ (c) the train.
must (a) started (b) had started
191. (a) at Kabul, and be in London by about 1.00 p.m./ (b) on/ (c) would start (d) has started
(c) over 211. His property was divided _________ his daughters and
PASSAGE 2 sons.
All of us know the kinds of substances that are known as metals. (a) between (b) among
They are commonly distinguished from other substances by their (c) from (d) with
bright and shiny surfaces. The majority of them are fairly heavy. 212. His persistence in his misdemeanours has lowered him in
192. (a) Since most metals are hard and strong, they/ (b) the _________ of everyone who knows him.
Although/ (c) When (a) eyes (b) estimation
193. (a) can be hammered, pulled and pressed into/ (b) cannot (c) estimate (d) esteem
be/ (c) a 213. The clouds of suspicion will clear ________ soon.
(a) up (b) away
194. various shapes (a) by mean of strong machines. It is more
(c) off (d) by
easy to shape/ (b) by meaning of/ (c) by means of
214. The teachers said that they were no longer prepared to
195. (a) the piece of metal after/ (b) that/ (c) a
________ the ways of the new Headmaster.
196. it has just been (a) lightly heated because heat softens it.
(a) put over with (b) put on with
Very great heat/ (b) warmly/ (c) now
(c) put up with (d) put up to
197. (a) must need to melt a/ (b) been needed/ (c) is needed
215. ________ the construction of new housing units at the
198. metal to its liquid (a) flow. Metal workers always pour the
rate of one every month, there is still a shortage of
liquid metal/ (b) condition./ (c) content.
accommodation.
199. (a) out of a hollow form/ (b) into/ (c) onto
(a) Through (b) Despite
200. called a mould, usually made of (c) By (d) For
(a) specialist prepared moulding sand held in a box of wood 216. Democracy requires the equal right of all to the development
or iron./ (b) specified/ (c) specially of such capacity for good as nature has __________ them
201. (a) Before the metal/ (b) After/ (c) While with.
202. (a) has cooled the moulding boxes are broken open and (a) presented (b) endowed
the/ (b) have cooled/ (c) has been cooled (c) fortified (d) replenished
EBD_7367
158
B- Selecting Words

DIRECTIONS (Qs. 217-226): Each of the following sentences 231. In spite of his hard work, he was made a scapegoat for the
in this section has a blank space with four words or group of failure of the project.
words given. Select whichever word or group of words you (a) freed from any responsibility for
consider most appropriate for the blank space. [2017-II] (b) suspected of causing
(c) blamed without reason for
Whenever I go into a bank, I feel scared. Everybody and (d) was severely punished for
everything that I see there ________ 217. 232. If food supply fails to keep pace with population, civilization
(a) pleases (b) frightens wil collapse.
(c) saddens (d) terrifles
(a) to grow along with
me. As for the manager the sight ________ 218.
(b) to walk side by side
(a) of (b) at
(c) to gain momentum
(c) by (d) on
(d) to move at the same speed as
him simply terrifies me and _________ 219.
(a) shapes (b) makes DIRECTIONS (Qs 233-242): Each of the following sentences
(c) shields (d) asks in this section has a blank space and four words or group of
me want to runaway _________ 220. words given after the sentence. Select the word or group of
(a) as slow as (b) as fast as words you consider most appropriate for the blank space
(c) as steadily as (d) as actively as [2018-1]
I can. As soon as I _________ 221.
(a) open (b) close 233. In the face of the overwhelming mass of evidence against
(c) shut (d) see him, we cannot _________ him of the crime.
the door of the bank I lose my head ________ 222. (a) punish (b) absolve
(a) or (b) and
(c) release (d) ignore
(c) either (d) neither
234. I hope that the rain will _________ for our picnic tomorrow.
when I try to do any _________ 223.
(a) keep off (b) put off
(a) service (b) business
(c) deed (d) act (c) set back (d) stay out
there, I behave like an idiot. I cannot explain 235. After the marathon, some of the competitors felt completely
________ 224. _________.
(a) the reasons (b) the responses (a) cut up (b) done in
(c) the answers (d) the causes (c) done out (d) run out
for this but that is how it ________ 225. 236. Scarcely _________ the teacher entered the class when he
(a) always (b) no time heard the noise.
(c) any time (d) many times (a) did (b) has
has been that is how it is _________ 226. (c) had (d) will have
(a) then (b) now 237. I do not think he will ever _________ the shock of his
(c) later (d) after wife’s death.
(a) get by (b) get off
DIRECTIONS (Qs. 227-232): For the expression which has been
(c) get through (d) get over
underlined in each of the following sentences, choose the
238. It is no use in crying over _________
response (a), (b), (c) or (d) which most nearly expresses its
(a) spoiled milk (b) spirited milk
meaning. [2017-II]
(c) split milk (d) spilt milk
227. The institution decided to give Mrs. Roy’s job to her son 239. You must go to the station now, your brother ________ go
not because she was a social worker, but because she had just yet as his train leaves after three hours.
died in harness. (a) shouldn’t (b) mustn’t
(a) died after retirement (b) died before retirement (c) wouldn’t (d) needn’t
(c) died while riding a horse(d) died for a great cause 240. Every rash driver becomes a _________ killer.
228. Your ambition should be in consonance with your (a) sure (b) reckless
capabilities. (c) potential (d) powerful
(a) parallel to (b) in accordance with 241. The country owes a deep debt of _________ for the
(c) in harmony with (d) in tune with freedom fighters.
229. I could not go up in life for want of proper guidance. (a) patriotism (b) sincerity
(a) for lack of (b) for need of (c) remembrance (d) gratitude
(c) for the desire of (d) for the necessity of 242. The whole lot of young men was very enthusiastic but
230. Though he was arrogant, I could not dispense with his your friend alone was _________.
services.
(a) quarrelsome (b) complaining
(a) terminate (b) align with
(c) a wet blanket (d) sleepy
(c) claim (d) disregard with
Selecting Words 159
B-

DIRECTIONS (Qs. 243-257): Each of the following sentences DIRECTIONS (Qs 258-272): Given below are some idioms/
in this section has a blank space with four words or group of phrases followed by four alternative meanings to each. Choose
words given. Select whichever word or group of words you the response (a), (b), (c) or (d) which is the most appropriate
consider most appropriate for the blank space [2018-1] expression. [2018-1]
One of India’s greatest musicians is M.S. Subbulakshmi, 258. A hot potato.
affectionately known to most people as M.S. Her singing (a) A dish to relish when it is hot
has brought _______ 243. (b) A very important person in a gathering
(a) sorrow (b) joy (c) An issue which is disputed, and catching the attention
(c) boredom (d) pain of people
to millions of people not only _________ 244. (d) A way of thinking what someone is thinking
(a) over (b) on 259. You snooze, you lose.
(c) in (d) with (a) Don’t take it lightly
all parts of India, but in _________ 245. (b) Don’t be over-enthusiastic
(a) strange (b) unknown (c) Don’t hesitate to do it
(c) other (d) familiar (d) Don’t be pessimistic
countries around the world as well. _________ 246. 260. I don’t buy it.
(a) Within (b) On (a) I don’t believe it (b) I have no money
(c) In (d) By (c) I summarize it (d) I don’t need it
261. My two cents.
October 1966 Subbulakshmi was invited to ______ 247.
(a) My money (b) My opinion
(a) dance (b) sing
(c) My decision (d) My explanation
(c) speak (d) enjoy
262. Out of the blue.
in New York, where people of _________ 248. (a) Undoubtedly (b) Unexpectedly
(a) many (b) few (c) Unbelievably (d) Unconcerned
(c) backward (d) all 263. What a small world.
foreign countries listened to her music _________ 249. (a) What a coincidence (b) What a challenging task
(a) attentively. (b) quitely. (c) What a narrow space (d) What a beautiful place
(c) indifferently. (d) boldly. 264. Down the road.
This was one of the greatest _________ 250. (a) In future (b) In the past
(a) awards (b) honours (c) At present (d) No particular time
(c) prizes (d) recognitions 265. Raising eyebrows.
ever given to any musician. For _________ 251. (a) To show surprise (b) Criticize
(a) seconds (b) minutes (c) Support (d) Instruct
(c) hours (d) days 266. Step up the plate.
together M.S. kept that international _________ 252. (a) Take control (b) Take a job
(a) spectator (b) audience (c) Take a responsibility (d) Take an opportunity
(c) viewer (d) businessmen 267. The Holy Grail.
spell-bound with the beauty of her voice and her style of (a) The pious place of worship
singing. (b) An important object or goal
The Second Anglo-Maratha War had shattered the (c) A very important place
________ 253. (d) Someone’s destination of life
(a) power (b) dignity 268. You scratch my back, I’ll scratch yours.
(c) time (d) patience (a) Mutual favour (b) Mutual understanding
of the Maratha chiefs, but not their spirit. The ______ 254. (c) Mutual respect (d) Mutual disliking
(a) disappearance (b) empowerment 269. At the drop of a hat.
(c) loss (d) disappointment (a) Without any hesitation
of their freedom rankled in their hearts. They made a last (b) When attempt fails and it’s time to start all over
(c) To further a loss with mockery
_________ 255.
(d) Judging other’s intentions too much
(a) horrible (b) desperate
270. Ball is in your court.
(c) poor (d) strong
(a) Be happy at the dance/ball room
attempt to regain their independence and old ____ 256.
(b) It’s up to you to make the decision
(a) prestige (b) army (c) A very powerful person
(c) rebellion (d) infantry (d) Not speaking directly about an issue
in 1817. The lead in organizing a united front of the Maratha 271. Best of both worlds.
chiefs was taken by the Peshwa who was smarting under (a) A happy person who is the best with all
the _________ 257. (b) All the advantages
(a) pleasant (b) satisfying (c) To take on a task that is way too big
(c) rigid (d) orthodox (d) Someone whom everybody likes
control exercised by the British Resident.
EBD_7367
B- 160 Selecting Words

272. Costs an arm and a leg. (a) confer (b) present


(a) Severe punishment to someone (c) execute (d) administer
(b) Too much consciousness about one’s body 278. Most of the students have forgotten to bring their
(c) Two difficult alternatives certificates; they will be ________ by the Principal for this
(d) Something very expensive reason.
DIRECTIONS (273-282) : In this section, each of the following (a) reproached (b) abused
sentences has a blank space and four words are given below it. (c) accused (d) reprimanded
Select the word or group of words you consider most appropriate 279. The consequences of economic growth have now
for the blank space and indicate your response on the Answer ________ to the lowest level.
Sheet accordingly. (a) drawn (b) slipped
(c) percolated (d) crept
273. In this University, there is no ________ for awarding
280. Mrs. Ramsay did not know whether Miss Jane knew
scholarships on the basis of merit in examination alone.
________ .
(a) precedent (b) opportunity
(a) swimming (b) to swim
(c) chance (d) possibility
(c) how to swim (d) how to swimming
274. Can you pay ________ all these articles ?
281. He ________ the role of the organization in creating
(a) out (b) of
environmental awareness among the people.
(c) for (d) off
(a) collaborated (b) commanded
275. He may not come, but we'll get ready in case he ________
(c) contrasted (d) commended
(a) may (b) will
282. We should give everyone training in citizenship but we
(c) shall (d) does
276. He is quite ________ you can never count on him in matters have ________ this aspect till now.
of secrecy. (a) denied (b) neglected
(a) unjust (b) unbearable (c) refused (d) disallowed
(c) inefficient (d) unreliable (c) To take on a task that is way too big
277. The Governor will ________ the oath of office to the thirty- (d) Someone whom everybody likes
five new ministers at 9.30 p.m. tomorrow.

HINTS & SOLUTIONS


1. (b) 'a' is the most suitable option because we are not talking 17. (b) Understanding is the most suitable word.
about any particular dog. 18. (c) Willingness is the most suitable word.
2. (c) 'on' is the most suitable option because when we refer 19. (a) Of is the most suitable word.
to days, we use on. Next is inappropriate because the 20. (b) through is the most suitable word.
sentence is in past tense. 21. (c) Means is the most suitable word.
3. (a) dogs are used to track the killers by the police. 22. (a) Discussion is the most suitable word.
4. (c) 'was' is the most suitable option because the sentence 23. (a) Through is the most suitable word.
is in past tense. 24. (c) Rigid is the most suitable word.
5. (c) ‘up’, picked up is the correct phrase. 25. (b) Conduct is the most suitable word.
6. (b) ' blood' is the most suitable option. 'Traces of blood' is 26. (a) Difficult is the most suitable word.
idiomatically used. 27. (b) 'a' is the most suitable word.
7. (b) after running a few miles, the dog lost the track that he 28. (b) 'is' is the most suitable word.
has been tracing from the scent of the blood. 29. (a) Dealing is the most suitable word.
8. (b) murder cases are usually investigated. 30. (b) Best is the most suitable word.
9. (b) the police have requisitioned (asked for) the services 31. (b) Wisdom is the most suitable word.
of the forensic expert 32. (c) Conflicts is the most suitable word.
10. (a) 'a' is used because we are talking about one fingerprint 33. (b) Compromise is the most suitable word.
expert 34. (b) An accomplice is a partner in crime. Thus option (b) is
11. (a) ' There' is the most suitable option. the answer.
12. (c) 'which' is the most suitable option. 35. (d) Imbiber means one who absorbs something.
13. (a) Can is used because there is a probability of happening Impresario means a person who organizes concert and
of a thing in the future. plays. Imitator is the one who copies another person.
14. (c) Hands are joined to complete a work and not ourselves Imposter is the pretender so correct answer is option
or heads. (d).
15. (a) Goals are achieved and not intention or wish. 36. (d) Honest means truthful. Selfish is a person who just
16. (c) According to the sentence, there is no use of passing thinks about himself. Unscrupulous means dishonest.
legislation if we do not enforce and enact on the same. Conscientious means diligent or hardworking. Looking
Selecting Words B-161
at the sentence, it is understood that the person is which something takes place. On is used to indicate
very hardworking. Thus option (d) is the best answer. the position above. Thus option (a) is the answer.
37. (a) Evoke means bring to mind and when a response is 48. (b) It is hinted from this item that Picasso did not paint.
needed. Thus clearly option a is the answer. Provided He in fact thought for a moment and then tore up the
means given with. Provoked means to incite. Prevent paper. For this to take place, Picasso must have thought
means to stop. Thus these meanings do not fit here. of something. Whether he was standing or sitting, is
38. (b) It is evident from the question that too many not depicted in the previous sentences. Thus option
skyscrapers are blocking the sight of the beach. Now, (b) is the correct answer.
reveal means to show. Obstruct means to block or stop 49. (b) Picasso took some colour crayons should be the correct
which perfectly fits here. Make means to build. Clear form of usage. Back will not fit suitably as Picasso
means clean or without any blemish. Thus we see that took crayons for the first time from the boy. Aside is
option (b) is the best answer. also not suitable because the word took comes before
39. (d) When we read this sentence we get to know that the and thereby implying its unsuitability.
person's most desired wish is to become billionaire. 50. (c) In this item we see that Picasso drew designs on the
This does not mean it is his only wish. Absolute means boy's chest to neck. Hence 'to' is the best option as
complete so a wish cannot be complete in itself thus near is vague description; 'and' also does not fit because
option (d) ultimate seems as the most suitable answer. chest to neck is one unit of his work and not separate
40. (b) In this question the motorcyclist wants to reach a place parts.
and thus he seeks a policeman's help. The policeman 51. (c)
gave him some directions. Option (b) should be the 52. (a) 'is' is the most appropriate word.
correct answer. Instructions are given on some 53. (b) 'on' is the most appropriate word.
particular things and advice where general talks take 54. (c) 'towards' is the most appropriate word.
place. The directions are given having particular 55. (a) 'increasing' is the most appropriate word.
protocol e.g. in explaining road map. 56. (b) 'of' is the most appropriate word.
41. (b) The person came back should be the correct form of 57. (a) 'degrees' is the most appropriate word.
sentence as the sentence points out at the person 58. (c) 'sea' is the most appropriate word.
coming back to the place. Reached is incorrect as the 59. (b) 'by' is the most appropriate word.
person has not reached the destination yet. Went is 60. (c) 'book' is the most appropriate word.
also wrong as the person is coming back after leaving. 61. (c) 'changes' is the most appropriate word.
42. (c) When the motorcyclist came back he told the policeman 62. (c) 'no' is the most appropriate word.
that he could not find the place. Told is the correct 63. (a) 'body' is the most appropriate word.
form of usage here because the person is conveying 64. (b) 'up' is the most appropriate word.
the message. He is not asking any question. Convince 65. (b) 'familiar' is the most appropriate word.
is used where a person has to prove his stand. Thus 66. (b) 'people' is the most appropriate word.
option (c) is the correct option. 67. (c) 'they' is the most appropriate word.
43. (a) Since we know from the previous sentences that the 68. (b) 'shaping' is the most appropriate word.
person for searching for a place, the most suitable 69. (c) 'punctuating' is the most appropriate word.
word would be "find". Hit or see does not make sense 70. (b) 'use' is the most appropriate word.
as he is not looking at a place randomly. Thus the 71. (c) 'talk' is the most appropriate word.
correct option is (a) "find". 72. (b) sea level is the most appropriate word.
44. (c) We always use either urge or idea with the word got. 73. (a) do is the most appropriate word.
Inclination is not used with got. But idea does not fit 74. (c) pressure is the most appropriate word.
in the sentence as the policeman wants to help the 75. (c) Increases is the most appropriate word.
person. It is not an idea that he gets. Want or feeling 76. (b) of is the most appropriate word.
like can be easily substituted with urge. Thus option 77. (b) several is the most appropriate word.
(c) urge is the answer. 78. (c) so is the most appropriate word.
45. (b) We see in this item that policeman is ready to help the 79. (c) lower is the most appropriate word.
man and thus he agrees to "go" with him. Go should 80. (a) up is the most appropriate word.
be the correct option as the policeman is going along 81. (b) for is the most appropriate word.
with the person to the desired destination. He is not 82. (c) would is the most appropriate word.
just starting or proceeding from there but he is sure 83. (a) know is the most appropriate word.
where he has to reach. Thus option (b) is the correct 84. (b) how is the most appropriate word.
answer. 85. (c) there is the most appropriate word.
46. (b) We already know from the previous sentences that 86. (a) this is the most appropriate word.
the person is wanting to reach a particular place and 87. (b) those is the most appropriate word.
this can be substituted with the word destination. It 88. (c) when is the most appropriate word.
was not a spot or station he wanted to go to. Thus 89. (b) to is the most appropriate word.
option b destination should be the correct answer. 90. (b) eventually is the most appropriate word.
47. (a) The little boy approached Picasso "with" a drawing 91. (a) burst is the most appropriate word.
paper. We always use with when indicating "having 92. (c) stories is the most appropriate word because in the
something". By is used to indicate means through continuing sentence,' telling stories' is written.
EBD_7367
162
B- Selecting Words

93. (b) at is the most appropriate word. 147. (b) “gave in” means to agree to what someone wants after
94. (a) popular is the most appropriate word. refusing for a period of time. Therefore, the right option
95. (b) of is the most appropriate word. is option (b).
96. (b) ‘making here’ means make up something untrue 148. (c) “Crumbles” means to become weaker in strength or
97. (a) about is the most appropriate word. influence. Therefore, the right option is option (c).
98. (b) ‘dead men’ complements with the living people 149. (c) “Observation” means the action or process of closely
99. (b) about is the most appropriate word. observing or monitoring something or someone.
100. (a) ‘experiences’ is the most appropriate word. Therefore, the correct option is option (c).
101. (c) 'and' is the most appropriate word. 150. (a) “struggle” means making forceful efforts to get free of
102. (c) 'the' is the most appropriate word. restraint or constriction. Therefore, the correct option
103. (a) 'useful' is the most appropriate word. is option (c).
104. (c) 'think' is the most appropriate word. 151. (a) “extravagant” means lacking control in spending
105. (c) 'your' is the most appropriate word. money or using resources. Therefore, the option is
106. (b) effectively is the most appropriate word. option (a).
107. (c) 'of' is the most appropriate word. 152. (c) “accused” means a person charged with or on trial for
108. (a) shake off is the most appropriate word. a crime. Therefore, the correct option is option (c).
109. (b) 'you' is the most appropriate word. 153. (a) In the given case fell ‘out’ means to quarrel or disagree
110. (c) which is the most appropriate word. with someone over something. Therefore, the correct
111. (c) 'success' is the most appropriate word. option is option (a).
112. (b) ‘will’ relates to the future of the special kids in the
154. (b) “naive” means showing a lack of experience, wisdom
passage.
or judgement. Therefore, the correct option is option
113. (b) ‘distressing’ means discomfort or agony of the parents
(b).
in the passage.
155. (d) ‘gruesome’ means extremely unpleasant and showing,
114. (c) ‘unfortunate’ relates to special children who are
usually dealing with death or injury. Therefore, the
different than the normal children.
correct option is option (d).
115. (a) ‘providing’ is most appropriate here as providing in
this case means importing training to the children. 156. (a) “obsessed” means preoccupy or fill the mind of
116. (c) ‘Hence’ is appropriate as it is related to consequence someone to a troubling extent. Therefore, the correct
option is option (a).
of an action.
117. (c) ‘over’ is most correct as it shows extent or reach. 157. (b) 158. (a) 159. (c) 160. (b) 161. (c) 162. (a)
118. (a) ‘However’ is the correct option as it used to introduce 163. (c)
a statement that contrasts with something said 164. (b) “instinctive” means done with conscious thought.
previously. And in this case the children getting only Therefore, option (b) is correct.
trained to make paper bags etc is in contrast with their 165. (c) 166. (b) 167. (b) 168. (a) 169. (c)
future prospects. 170. (b) “preceded” means to come before in time. The option
119. (b) ‘Simple’ is the appropriate option as ‘clumsy’ means (b) is correct.
something difficult to handle where as ‘dubious’ 171. (c) 172. (b) 173. (a) 174. (c) 175. (b) 176. (a)
means something of questionable value. 177. (b) 178. (c) 179. (d) 180. (c) 181. (b) 182. (c)
120. (a) ‘agreeable’ is most appropriate as it means something 183. (a) 184 (b) 185. (a) 186. (b) 187. (c) 188. (c)
which is enjoyable and pleasant. 189. (a) 190. (b) 191. (a) 192. (b) 193. (a) 194. (c)
121. (a) sport is the correct option. 195. (c) 196. (a) 197. (c) 198. (b) 199. (b) 200. (c)
122. (b) ‘Construction’ here refers to building of telescope 201. (b) 202. (c) 203. (b) 204. (b) 205. (b) 206. (a)
by Galileo. 207. (b) ‘Substantial’ is the correct answer, which means of
123. (c) 124. (b) 125. (c) 126. (b) considerable importance, size or worth.
127. (c) ‘the’ is correct option as the definite article is used for 208. (c) ‘Mollify’ is the correct answer which means appease
‘universe’. the anger and anxiety of someone.
128. (c) also is correct option. 209. (b) ‘Differentiate’ is the correct option as in the sentence
129. (b) 130. (c) a comparison of two things is done.
131. (a) ‘same’ here refers to the cannon balls travelling or 210. (b) ‘Had started’ is the correct option, as the sentence is
moving in identical speed. in past tense : If done this ... then this ....
132. (c) 'down' should be used 211. (b) ‘Among’ fits best it is the correct option as here there
133. (b) ‘distance’ here refers to the linear extent covered by can be many sons and daughter and between is used
the cannon balls. for two.
134. (a) ‘to’ makes sense in the sentence. 212. (a) ‘Eyes’ fits best in the sentence.
135. (b) ‘swinging’ here refers to the ‘to and fro’ movement of 213. (a) ‘Clearup’ will be the best option.
the lamps in Pisa Cathedral. 214. (c) ‘Put up with’ is the correct answer.
136. (c) ‘those’ is correct as the lamps denote plurality and 215. (b) ‘Despite’ is the correct answer.
those is pronoun used for denoting more than one 216. (b) ‘Endowed’ is the correct option.
object. 217. (b) frightens
137. (c) 138. (d) 139. (a) 140. (c) 141. (b) 218. (a) of
142. (c) 143. (b) 144. (b) 145. (a) 146. (c) 219. (b) makes
Selecting Words B-163
220. (b) as fast as 246. (c) In
221. (a) open 247. (b) sing
222. (b) and 248. (a) many
223. (b) business 249. (a) attentively
224. (a) the reasons 250. (b) honours
225. (a) always 251. (c) hours
226. (b) now 252. (b) audience
227. (b) Died in harness’s meaning is ‘died before retirements’. 253. (a) power
228. (c) ‘In harmony with’ nearly express the meaning of ‘in 254. (c) loss
consonance with’. 255. (b) desperate
229. (a) The meaning of ‘for want of’ is conveyed by (a) for 256. (a) prestige
lack of. 257. (c) rigid
230. (a) The meaning of ‘dispense with’ is conveyed by (a) 258. (c) The phrase ‘A hot potato’ means a controversial issue
terminate. or situation which is awkward to deal with or a disputed
231. (c) ‘Made a scapegoat’ means ‘blamed without reason’. issue that catches the attention of the people.
232. (d) ‘To keep pace with something’ means ‘to move at the 259. (a) The idiom ‘You snooze, you lose’ is used to warn
same speed’ as the other one. people who aren't paying attention or those who take
233. (b) Absolve which means declare someone free from guilt, things lightly.
obligation, or punishment is the best suited word to 260. (a) The phrase ‘I don't buy it’ means to not be convinced.
fill the blank correctly. 261. (b) The phrase ‘My two cents’ means my opinion.
234. (a) The phrase keep off which means fail to occur (of bad 262. (b) The phrase ‘Out of the blue’ means without warning;
weather) is the best suited phrase to fill the blank unexpectedly.
correctly and contextually. 263. (a) The phrase ‘What a small world’ means what a
235. (b) The phrase done in which means extremely tired is the coincidence.
best suited phrase to fill the blank correctly and 264. (a) The phrase ‘Down the road’ means in the future.
contextually. Cut up means upset or troubles, run out 265. (a) The phrase 'Raising eyebrows' means to feel surprised
means to be used up (of a supply of something) and or disapproving.
the word 'done out' does not exist. 266. (c) The phrase ‘Step up the plate’ means to take
236. (c) The sentence should be in past perfect tense so, 'had' responsibility for doing something, even though it is
should be used to make the sentence grammatically difficult.
correct.
267. (b) ‘The Holy Grail’ means something that is extremely
237. (d) The phrase 'get over' which means to recover from (an
difficult to find or get; a very important object or goal.
ailment or an upsetting or startling experience) is the
268. (a) The phrase ‘You scratch my back, I'll scratch yours’
best suited word to fill the blank appropriately and
means mutual favour.
contextually.
269. (a) The phrase ‘At the drop of a hat’ means without any
238. (d) ‘Crying over spilt milk’ altogether is a phrase which
hesitation.
means getting upset over certain things like spilled
270. (b) The idiom ‘Ball is in your court’ means it is up to you
milk, is not going to fix it. So, option (d) i.e. spilt milk is
to make the decisions.
the correct word to complete the phrase correctly.
271. (b) The idiom ‘Best of all worlds’ means a win-win
239. (d) In this sentence, the use of ‘wouldn't’ would be
absolutely inappropriate. Option (a), (b) and (c) can situation or a situation where one can enjoy all the
advantages.
be used to fill the blank however, the use of ‘shouldn't’
and ‘mustn't’ gives a sense of compulsion which, if 272. (d) The phrase ‘Cost an arm and a leg’ means to be
used here, will be inappropriate in the context of the extremely expensive.
sentence. ‘Needn't’ which doesn't show any compulsion 273. (a) The word ‘precedent’ means an earlier event that is
is the correct word to fill the blank. regarded as an example to be considered in subsequent
240. (c) Potential which means likely, possibly and probable similar circumstances. This word suitably fills the
etc. is the best suited word to fill the blank. Rash blank.
driving many a times result to accidents and deaths 274. (c) To pay ‘for’ something is the correct phrase. So, (c) is
thus, rash drivers are more likely to become killers while the right answer.
driving. 275. (d) The sentence should be in simple present tense. So,
241. (d) Apart from ‘gratitude’, rest of the words are ‘does’ is the most suitable option.
inappropriate in the context of the passage. 276. (d) ‘Unreliable’ is the most suitable word to fill the blank.
242. (c) The phrase ‘wet blanket’ which is used for a person 277. (d) ‘Oath’ is administered. So, (d) is the correct option to
who spoils others’ fun by failing to join in or by fill the blank.
disapproving of their activities is the best suited word 278. (d) ‘Reprimand’ is used to indicate a formal expression of
to fill the blank in the context of the sentence. Rest of disapproval. It is the most suitable word to fill the
the options are inappropriate in the context of the blank.
sentence. 279. (b) ‘Slipped’ is the most suitable word to fill the blank.
243. (b) joy 280. (c) ‘how to swim’ is the correct option to choose.
244. (c) in 281. (d) ‘commended’ is most suitable word to fill the blank.
245. (c) other 282. (b) The word ‘neglected’ suitably fills the blank.
EBD_7367
B-164 Reading Comprehension

C HA P T E R
READING COMPREHENSION
33
DIRECTIONS (Qs. 1-30): In this Section, you have seven short (b) The present state of civilization
passages. After each passage, you will find several questions (c) The past stage of civilization
based on the passage. First, read a passage and then answer the (d) After having provided the basic essentials of life
questions based on it. You are required to select your answers 2. What does the passage suggest about th e use of
based on the contents of the passage and opinion of the author technology?
only. Examples ‘I’ and ‘J’ are solved for you. [2008-II] (a) It creates new and essential needs for mankind
(b) It is opposed to the basic essentials of life
PASSAGE (c) It is complementary to a raised standard of living
In our approach of life, be it pragmatic or otherwise, a basic fact (d) It is responsible for man’s love of comfort and
that confronts us squarely and unmistakable is the desire for peace, happiness
security and happiness. Different forms of life at different levels 3. What does increased use of technology imply?
of existence make up the teeming denizens of this earth of ours, (a) An advanced stage in human civilization
And, no matter whether they belong to the higher groups such as (b) A backward step in human culture
human beings or to the lower groups such as animals, all beings (c) Unnecessary comfort and happiness for mankind
primarily seek peace, comfort and security. Life is as dear to a (d) Man’s zest for more and more work
mute creature as it is to a man. Even the lowliest insect strives for 4. What does the author suggest ?
protection against dangers that threaten its life. Just as each one (a) Man will gradually rise above his present stage in
of us wants to live and not to die, so do all other creatures. civilization
I. The author’s main point is that (b) Man will gradually settle down to the same stage with
(a) different forms of life are found on earth fewer hours of work
(b) different levels of existence are possible in nature (c) Man will gradually raise his standard of living by
(c) peace and security are the chief goals of all living beings working longer hours
(d) even the weakest creature struggles to preserve its life (d) Man will gradually earn a given standard of living with
J. Which one of the following assumptions or steps is essential the help of technology
in developing the author’s position ? PASSAGE – II
(a) All forms of life have a single overriding goal It is said that ideas are explosive and dangerous. To allow them
(b) The will to survive of a creature is identified with a unfettered freedom is, in fact, to invite disorder. But, to this position,
desire for peace there are at least two final answers. It is impossible to draw a line
(c) All beings are divided into higher and lower groups round dangerous ideas and any attempt at their definition involves
(d) A parallel is drawn between happiness and life, and monstrous folly. If views, moreover, which imply disorder are able
pain and death to disturb the foundations of the state, there is something
Explanation : supremely wrong with the governance of the state. For disorder is
I. The idea which represents the author’s main point is ‘peace not a habit of mankind. We cling so eagerly to our accustomed
and security are the chief goals of all living beings’, Which ways that, as even Burke insisted, popular violence is always the
is response (c). So (c) is the correct answer. outcome of a deep popular sense of wrong.
J. The best assumption underlying the passage is ‘The will to 5. What is the central point that the passage emphasizes ?
survive of a creature is identified with a desire for peace’, (a) It is unnecessary to define dangerous ideas
which is response (b). So (b) is the correct answer. (b) Dangerous ideas are born out of the enjoyment of
PASSAGE – I freedom
(c) A well-governed state is unaffected by dangerous ideas
As civilization proceeds in the direction of technology, it passes
(d) Dangerous ideas originate from man’s preoccupation
the points of supplying all the basic essentials of life, food, shelter,
with politics
cloth, and warmth.
6. From a close study of the passage, which one of the following
Then we are faced with a choice between using technology statements emerges most clearly ?
to provide and fulfil needs which have hitherto been regarded as (a) The author is against the exercise of political freedom
unnecessary or, on the other hand, using technology to reduce (b) He is indifferent to dangerous and explosive ideas
the number of hours of work which a man must do in order to earn (c) He welcomes violence as a method to change
a given standard of living. In other words, we either raise our governments
standard of living above that necessary for comfort and happiness (d) He warns that violence is the outcome of popular
or we leave it at this level and work shorter hours. dissatisfaction with the government
I shall take it as axiomatic that mankind has, by that time, 7. The author says, “We cling so eagerly to our accustomed
chosen the latter alternative. Men will be working shorter and ways”. Which one of the following statements may be
shorter hours in their paid employment. considered as the assumption of the author ?
1. “Then we are faced with a choice ...” what does `then’ refer to? (a) We are afraid of social changes
(a) When automation takes over many aspects of human (b) Mankind is averse to any disorder
life
Reading Comprehension B-165
(c) We have developed inertia that makes us incapable of (a) ignore an offence if it is nice
social action (b) forgive people if they bring us nice presents
(d) There is an all round lack of initiative in the society (c) forgive pretty offenders
8. Which of the following statements may most correctly bring (d) not punish each and every offence
out the significance of the opinion of Burke quoted in the 14. Which one of the following is the correct statement ?
passage ? We must
(a) Burke advocated violence against injustice (a) be blind if we want to forgive others
(b) Burke’s opinion coincides with the author’s opinion (b) be blind to the faults of our friends
on explosive and dangerous ideas (c) be indifferent to what others do
(c) Burke hated any popular uprising (d) overlook certain things
(d) Burke had no belief in political liberty 15. Which one of the following is the correct statement ?
PASSAGE – III In ancient times people were
The psychological causes of unhappiness, it is clear, are many (a) ordered to lose their tempers
and various. But all have something in common. The typical (b) permitted to lose their tempers and not forgive their
unhappy man is one who, having been deprived in youth of enemies
some normal satisfaction, some come to value this one kind of (c) told that it was not good to have an unforgiving temper
satisfaction more than any other, and has therefore given to his (d) advised to forgive each and every offence committed
life a one-sided direction, together with a quite undue emphasis by both friends and foes
upon the achievement as opposed to the activities connected 16. What is the underlying tone of the passage?
with it. There is, however, a further development which is very (a) We must be forgiving in general
common in the present day. A man may feel so completely thwarted (b) We must forgive our friends
that he seeks no form of satisfaction, but only distraction and (c) There is no limit whatsoever to our duty to forgive
oblivion. He then becomes a devotee of ‘pleasure’. This is to say, (d) We must always punish the wrong doer
he seeks to make life bearable by becoming less alive. PASSAGE – V
Drunkenness, for example, is temporary suicide - the happiness Once upon a time I went for a week’s holiday in the Continent
that it brings is merely negative, a momentary cessation of with an Indian friend. We both enjoyed ourselves and were sorry
unhappiness. when the week was over, but on parting our behaviour was
9. Who is a typical unhappy man ? absolutely different. He was plunged in despair. He felt that
(a) One who has been deprived of normal satisfaction in because the holiday was overs all happiness was over until the
youth world ended. He could not express his sorrow too much. But in
(b) One who finds life unbearable and attempts suicide me the Englishman came out strong. I could not see what there
(c) One who does not mind momentary unhappiness was to make a fuss about. It wasn’t as if we were parting forever
(d) One who seeks every form of satisfaction or dying. “Buck up”, I said, “do buck up”. He refused to buck up,
10. “One sided direction” refers to the pursuit of which one of and I left him plunged in gloom.
the following ? 17. What is the Continent in the context of the passage ?
(a) Drinking and forgetfulness (a) An island (b) The countryside
(b) The satisfaction one had been deprived of (c) Africa (d) Europe
(c) Activities leading to happiness 18. What does the author mean by ‘buck up’ ?
(d) Every form of psychological satisfaction (a) Buckle yourself up (b) Stand up
11. Which one of the following is the correct statement ? (c) Cheer up (d) Shut up
Drinking helps the unhappy only to 19. Why was the Indian friend plunged in despair ?
(a) forget their dissatisfaction (a) He was hopeless
(b) get sublime happiness (b) He experienced racial discrimination
(c) get the motivational needs fulfilled (c) He would never be so happy again
(d) concentrate harder (d) He had spent lot of money
12. What does “becoming less alive” imply? 20. What does ‘But in me the Englishman came out strong’
(a) Neglect of health imply ?
(b) Decline in moral values (a) He was a strong Englishman
(c) Living in a make believe world (b) He had the typical English character
(d) Leading a sedentary way of living (c) The Englishman went out of him
PASSAGE – IV (d) He started following Indian traditions
What is to, be the limit of forgiveness? It would probably have 21. What is the author’s intention in the passage?
been allowed by many of the ancients that an unforgiving temper (a) To contrast the Indian character with the English
was not to be commended. character
They would have said, we are not to exact a penalty for (b) To show that an Indian is sorrowful
every nice offence, we are to overlook some things, we are to be (c) To ridicule the Indian traditions
blind sometimes. (d) To praise the Englishman
But they would have said at the same time, we must be PASSAGE – VI
careful to keep our self-respect, and to be on a level with the The world is very full of people appallingly- full, it has never been
world. On the whole, they would have said, it is the part of a man so full before, and they are all tumbling over each other. Most of
fully to requite to his friends their benefits and to his enemies these people one doesn’t know and some of them one doesn’t
their injuries. like. Well, that is one to do There are two solutions. One of them
13. Which one of the following is the correct statement ? is the Nazi solution. If you don’t like people, kill them, banish
According to the writer we must them, and segregate them. The other way is much less thrilling,
EBD_7367
166
B- Reading Comprehension
but it is on the whole the way of the democracies, and I prefer it. (a) We love beauty
If you don’t like people, put up with them as well as you can. (b) It is full of green vegetation
Don’t try to love them : you can’t, you’ll only strain yourself. But (c) It will ensure our future existence
try to tolerate them. (d) It will show our command over Nature
22. What does the author mean by ‘appallingly’?
(a) He is making an appeal to the leaders of the masses DIRECTIONS (Qs. 31-55) : In this Section you have FIVE short
(b) In disconcertingly large numbers passages. After each passage, you will find several questions
(c) Very interesting based on the passage. First, read a passage, and then answer
(d) Unpredictably the questions based on it. You are required to select your answers
23. Which one of the following is the correct statement ? based on the contents of the passage and opinion of the author
According to the writer Nazi solution is only. [2008-II]
(a) the easiest solution PASSAGE – I
(b) the readiest solution The New Year is a time for resolutions. Mentally at least, most of
(c) the national solution us could compile formidable lists of do’s and don’ts. The same
(d) the Hitlerian solution old favourites recur year in and year out with monotonous regularity.
24. Which one of the following is the correct statement ? The Past experience has taught us that certain accomplishments are
author thinks that the other solution is much less thrilling beyond attainment. If we remain inveterate smokers, it is only
because it is because we have so often experienced the frustration that results
(a) dull from failure. Most of us fail in our efforts at self improvement
(b) based on tolerance because our schemes are too ambitious and we never have time
(c) not based on love to carry them out. We also make the fundamental error of
(d) lacking in adventure announcing our resolutions to everybody so that we look even
25. Which one of the following is the correct statement ? The more foolish when we slip back into our old bad ways.
author prefers the second solution because 31. The author seems to think that others
(a) he likes it (a) feel happy when we slip back to our old ways
(b) he is not a Nazi (b) do not really want us to improve ourselves
(c) he is essentially being a democrat (c) are ready to tease and laugh at our attempts if we fail
(d) there is no other way (d) might embarrass us by praising our attempts
PASSAGE – VII 32. The author says that most of us fail in our attempts at self-
We should preserve Nature to preserve life and beauty. A beautiful improvement because
landscape, full of green vegetation, will not just attract our (a) we set too high ambitions for ourselves
attention but will fill us with infinite satisfaction. Unfortunately, (b) we do not have the persistence of mind
because of modernization, much of nature is now yielding to (c) our nature is such that we cannot become perfect
towns, roads and industrial areas. In a few places some Natural (d) certain imperfections have become a part and parcel of
reserves are now being carved out to avert the danger of our lives
destroying Nature completely. Man will perish without Nature, 33. The author seems to imply that many are inveterate smokers
so modern man should continue this struggle to save plants, because
which give us oxygen, from extinction. Moreover, Nature is (a) they have not really tried to give up smoking
essential to man’s health. (b) they know from past experience that they can never
26. What does ‘Nature’ in the passage mean ? succeed in their attempt to give up
(a) Countryside covered with plants and trees (c) they want to forget the frustration of not smoking
(b) Physical power that created the world (d) they do not have the will power to stop smoking
(c) Inherent things that determine character 34. The same old favourites recur ... with monotonous
(d) Practical study of plants and animals regularity’ implies that
27. Which one of the following is the correct statement ? (a) we want to be so perfect that we include some items
According to the passage regularly
(a) beauty is only skin-deep (b) we have been so regularly doing certain things that
(b) everything is beautiful in its natural state they have become monotonous
(c) there is beauty in Nature (c) in spite of repeated failures, we still would like to try
(d) Nature is a moray teacher one more time
28. What does the writer suggest ? (d) some favourite actions if repeated often could become
(a) We should not modernize, so that Nature can be monotonous
preserved 35. The phrase ‘formidable lists of do’s and don’ts’ means that
(b) While modernizing we should be careful not to destroy (a) the bad points of our character are formidable
Nature completely (b) the list is so long that it is frightening
(c) All Nature has been destroyed by modern living (c) the things that need to be included is frightening
(d) Carving out Natural reserves will hamper the growth (d) the realisation that we are so imperfect is frightening
of industries PASSAGE – II
29. What does ‘struggle’ in the passage mean ? Deriving your authority from the government, your position would
(a) Man’s struggle to exist in the world secure the respect and consideration of everyone, especially in a
(b) Man’s struggle to save Nature service where official rank carries so much weight. This would
(c) Man’s struggle to catch up with modern trends secure to you every attention and comfort on your way and there,
(d) Man’s struggle to conserve oxygen together with a complete submission to your orders.
30. Why a beautiful landscape ‘will fill us with infinite I know these things are a matter of indifference to you except
satisfaction’ ? so far as they may further the great objects you have in view, but
Reading Comprehension B-167
they are of importance in themselves, and of every importance to (a) facilitate easy understanding
those who have a right to take an interest in your personal position (b) give a plain narration
and comfort. [2008-II] (c) convey breathless excitement
36. The above passage most probably is a part of a (d) imply the inability of the author to write in a better way
(a) speech (b) official communication 47. Which one of these expressions best brings out the effect
(c) written report (d) personal letter of the foot-print on the author?
37. The writer’s attitude towards the person addressed is (a) Seen an apparition
characterised by (b) Stood like one thunder-struck
(a) officiousness (b) flattery (c) Went up the shore
(c) humility (d) arrogance (d) Looked round himself
38. The person addressed is most likely a 48. How does the author convince himself that the foot-print is
(a) social worker (b) government servant a real one ?
(c) commercial agent (d) foreign dignitary (a) By finding the person who made it
39. The writer is asking his reader to accept (b) By being told about it by a witness
(a) great objects (b) a respected position (c) By thinking about it for some time
(c) an official rank (d) a significant assignment (d) By examining it carefully and noticing its details
40. ‘The great objects’ in the passage means 49. Which one of the following words best describes the
(a) Significant items (b) Noble goals emotion evoked by the footprint in the author ?
(c) Precious merchandise (d) Objects of praise (a) Curiosity (b) Indifference
PASSAGE – III (c) Fear (d) Surprise
While I stood drinking in he beauty of this placid scene I became 50. On finding the foot-print on the shore, what did the author
conscious of an alteration. In a moment the sole porter emerged do ?
from his midday nap, operated a signal that clanked noisily into (a) Did not pay much attention to it
position, and then ambled slowly towards me for my return-half- (b) Observed it with curiosity
ticket, whilst I remarked that his red amiable face and easygoing (c) Began to investigate its origin
gait were in perfect harmony with the tranquil surroundings. A (d) Ran away in fear
wisp of smoke on the horizon with a dark snake crawling beneath PASSAGE – V
it announced the approach of the train. As it drew nearer, the He dropped off to sleep. The cigarette slipped out of his mouth
deep silence of the place was gradually displaced by a creaking and burnt a great black hole in his only shirt. The smart of the
of brakes and a hissing of steam. Save for myself, no one entered burn awoke him, and he got up, cursing under his breath, and
the train and no one alighted. The porter with leisurely expertness, fumbled in the dark for a needle in order to sew up the hole.
trundled a couple of milk churns on board, the door was slammed, Otherwise his wife would see it in the morning and would hag
the guard signalled to the driver, and we moved off, leaving the away at him for a couple of hours. But he could not find a needle.
small station once more to its drowsy silence. [2008-II] He fell asleep again. [2008-II]
41. The meaning of drowsy is 51. Which one of the following statements best sums up the
(a) Untidy (b) Sleepy man’s reaction to his problem?
(c) Freezing (d) Drugged (a) The man is extremely upset to find the shirt burnt and
42. The central idea of the passage is frantically tries to repair the damage
(a) Leisure and Peace (b) Hurry and Noise (b) The hole in the shirt and the wife’s anticipated nagging
(c) Activity (d) The Porter are minor problems, the greater one is that the man
43. Who had a midday nap ? cannot find a needle
(a) The author (b) The passenger (c) Neither the shirt hole nor the nagging nor the lack of a
(c) The Station-master (d) The porter needle is of great consequence
44. Who/what does first break the silence of the station ? (d) The man is terrified of his wife and dreads her
(a) The train (b) The porter discovering the burnt shirt
(c) The passenger (d) Milk churns 52. The man wanted to sew the hole because
45. What does the author suggest by the word ‘placid’ ? (a) he wanted to avoid being scolded by his wife
(a) The scene was filled with noise of the train (b) he had nothing else to do
(b) The place was filled with lively humanity (c) he had no other shirt
(c) The place was quite and lonely (d) he wanted to sleep again
(d) The horizon looked smoke laden 53. The man got up to search for a needle because
PASSAGE – IV (a) his wife would be very upset
It happened one day, about noon, going towards my boat, I was (b) the cigarette had damaged his only shirt
exceedingly surprised with the print of a man’s naked foot on the (c) he wanted to mend the shirt
shore, which was very plain to be seen in the sand. I stood like (d) the burn-hole was huge and black
one thunder-struck, or as if I had seen an apparition. I listened, I 54. The man woke up in the dark because
looked round me, I could hear nothing, nor see anything. I went (a) the cigarette had burnt his favourite shirt
up the shore, and down the shore, but it was all one; I could see (b) the cigarette had burnt his only shirt
no other impression but that one. I went to it again to see if there (c) the cigarette had to be lit again
were any more, and to observe if it might not be my fancy; but (d) the cigarette had burnt him
there was no room for that, for there was exactly the very print of 55. The cigarette fell out of the man’s mouth because
a foot toes, heel and every part of a foot. [2008-II] (a) he fell off his stool
46. The passage is full of short simple sentences. Their purpose (b) he was surprised to see that it had made a hole in his shirt
is to (c) he fell asleep while smoking
(d) he fumbled in the dark for a needle
EBD_7367
168
B- Reading Comprehension
DIRECTIONS (Qs. 56-75) : In this section you have six short 63. From the passage we gather that
passages. After each passage, you will find several questions (a) The author is very inquisitive.
based on the passage. First, read a passage, and then answer (b) The author tries to poke his nose in other people’s
the questions based on it. You are required to select your business
answers based on the contents of the passage and opinion of (c) The author is interested in mathematical notations.
the author only. [2009-I] (d) The author wants to talk to fellow passengers in the
bus.
PASSAGE – I PASSAGE – III
Ah! whatever could be said was said. All held him guilty. Even With the inevitable growth of specialization I see the universities
his own mother who claimed to understand him the best. All had facing two great dangers. First, it is very easy to get so involved
betrayed him in his hour of need. Yet, there he was, still with a in the technical details of education that the object of education
sparkling hope and knew that the truth must prevail. In the cold, is lost. And secondly, in an effort to condition a university to the
dark and damp cell he never for a moment lost faith in God and needs of its students and to the needs of the state it may lose its
goodness and was waiting anxiously for an to come, plead angel power to make or mould those students into responsible men,
non guilty for him and free him of his miseries. capable of thinking for themselves and capable of expressing
56. Three of the following statements indicate that he had a the results of their thoughts to others.
sparkling hope. Which statement does not? 64. The author calls growth of specialisation ‘inevitable’.
(a) He had never lost faith in God. Which one of the following statements is likely to be the
(b) He was sure there was goodness. most correct reason for this inevitability ?
(c) He could have evidence in his favour. (a) Universities give grants only to do specialised work
(d) He knew that the truth must prevail. in different disciplines
57. Whatever others said about him, he (b) The professors and researchers in universities are
(a) Betrayed no one. competent only for specialised work
(b) Thought over the problem. (c) Specialization helps economic growth of the nation.
(c) Never lost faith in goodness. (d) In an age of science and technology specialization
(d) Raised his voice against injustice. becomes necessary
65. Which one of the following statements most correctly
58. In the dark dungeon he always waited for
suggests the central theme of the passage ?
(a) His mother.
(a) The aim of education is specialization
(b) The jailer. (b) The aim of education is to mould the youth to work for
(c) The verdict freeing him of his miseries. the state
(d) The angle to come and plead for him. (c) The aim of education is to make the youth capable of
59. The truth must prevail means independent thought and expression
(a) He was true (d) The aim of education is to enable the youth to lead a
(b) Angle will reveal truth comfortable living
(c) Truth always wins in the end 66. Which one of the following statements most correctly
(d) We must plead for the truth suggests the warning implied in the passage ?
PASSAGE – II (a) University education should not be concerned with
Once while travelling by the local bus, I got a seat beside a very technical details.
strange man. He seemed interested in every passenger aboard. (b) Universities should not subordinate themselves to the
He would stare at a person, scribble some odd mathematical interests of the state.
notations on his long notebook and then move on to the next. (c) Universities should be concerned only with the needs
Being quite interested in what he was doing I asked him what all of students.
those notations meant and then came the starling reply. He saw (d) Universities should not go in for any specialization.
a man’s face not as a single unit but as thousands of squares put PASSAGE – IV
together. He was in fact a statistical expert and a budding artist One December night, a family had gathered around their fireside
learning the art of graphics. and piled it high with wood gathered from mountain streams and
60. The man was scribbling down ruins of great trees that had come falling down the mountain
(a) The figures of co-passengers sides. The fire roared and brightened the room with its light. The
(b) The details of thousands of squares put together faces of the father and mother had a quiet gladness’ the children
(c) Some mathematical formulae and calculations laughed; the oldest daughter was the picture of happiness at
(d) Some mathematical signs seventeen’ and the aged grandmother who sat sewing in the
61. The man caught author’s attention because warmest place was the picture of happiness grown old.
(a) He was sitting next to him. 67. The firewood had been
(b) He was staring at every person in the bus. (a) brought (b) stolen
(c) He would stare at every person and then scribble down (c) collected (d) found
some mathematical notation. 68. The parents looked
(d) He was budding artist learning the art of graphics. (a) serene (b) cheerful
62. The author found that man’s reply quite startling because (c) gloomy (d) dull
(a) A statistical expert cannot be a budding scientist 69. The oldest daughter looked
(b) A budding artist cannot be a statistical expert (a) stupid (b) glad
(c) Graphics is still a rare art form and he was learning in (c) intelligent (d) mournful
while travelling in a bus PASSAGE – V
(d) The fact that “a man’s face can be analysed as The unpleasant feeling passed and she glanced guardedly up at
thousands of squares” was a strange concept. him. He was walking unmarked in moonlight, innocent of her
Reading Comprehension B-169
reaction to him. She felt then – this thought had come to her (b) our ideas about birds are derived from the most
before – that there might be more to him than she had imagined. common types.
She felt ashamed she had never thanked him for the help he had (c) owls and ducks are our favourite birds.
given her father. (d) domesticated birds are our source of information about
70. She glanced at him when the bird world.
(a) He walked alone and unnoticed in moonlight 78. The common duck is not a separate specie but a tamed
(b) She was sure that she was not being noticed version of wild variety because it
(c) Her reactions did not have any effect on him (a) whistles like most wild ducks.
(d) The unpleasant feeling passed (b) grunts like the tufted duck.
71. Her unpleasant feeling passed when (c) has the same call as other ducks.
(a) He did not take any notice of her (d) quacks like the mallard.
(b) The moonlight was beautiful PASSAGE – II
(c) She realized her sense of shame “What is sixteen and three multiplied?” asked the teacher. The
(d) She looked carefully at him boy blinked. The teacher persisted, and the boy promptly
72. She was ashamed because answered: “twenty-four”, with, as it seemed to the teacher, a
(a) She was spying on him wicked smile on his lips. The boy evidently was trying to fool
(b) There was more to him than she had imagined him and was going contrary on purpose. He had corrected this
(c) A recurring thought came back to her error repeatedly, and now the boy persisted in saying “twenty-
(d) She had never thanked him for his help to her father four”. How could this fellow be made to obtain fifty in the
PASSAGE – VI classtest and go up by double-promotion to the first form, as his
As I slung my pack onto my shoulders a big mosquito thudded parents fondly hoped? At the mention of “twenty-four” the
against my cheek. There had been a few through the day, but it teacher felt all his blood rushing to his head. He controlled himself,
was early in the season – the ice had gone out just two weeks and asked again: “How Much”? as a last chance. What the boy
before – and I had scarcely noticed them. But now as I would said the same thing obstinately, he felt as if his finger was
down the ridge, the last breeze faded, and they were on me. releasing the trigger: he reached across the table, and delivered
Rising in clouds from the soggy tundra, they pelted against my a wholesome slap on the youngster’s cheek.
face. I reached in my pocket for the repellent, and came up empty. 79. The boy answered the question
73. The traveller could not feel the breeze because (a) with fear and anxiety
(a) The mosquito had bitten him.
(b) with the intention to fail in the class test.
(b) He was at the foot of the ridge. (c) to make some fun in the class.
(c) There was no ice on the mountain.
(d) when the teacher persisted in asking.
(d) There was no breeze on the tundra mountain.
74. The traveller carried with him 80. The teacher felt blood rushing in his head because
(a) Mosquito repellent (a) there was no arrogant smile on the boy’s lips.
(b) A pack of food (b) he thought the boy was hopelessly dull.
(c) A sling (c) he thought the boy made the mistake deliberately.
(d) A back-pack (d) the boy gave a wrong answer to such as simple
75. When he was in the arctic, the time of the year was question.
(a) Middle of winter. (b) Early autumn 81. The teacher controlled his anger because
(c) Early spring (d) Middle of summer (a) he remembered the fond hopes of boys’ parents.
(b) he wanted to give another opportunity to the boy.
DIRECTIONS (Qs. 76-99): In this Section you have Eight short (c) the boy was too young to pick up mathematics fast.
passages. After each passage, you will find several questions (d) he believed that the boy must be taught the lesson
based on the passage. First, read a passage, and then answer again.
the questions based on it. You are required to select your PASSAGE – III
answers based on the contents of the passage and opinion of I was at the shop early. He was standing behind the counter and
the author only. as soon as I saw him, I knew that there would be some
PASSAGE – I unpleasantness. Mr. Higson is never at his best unshaven, in
Popular illusion about birds extend further than the use of the slippers and braces and smoking on the empty stomach. The
word ‘egg-shape’ that would suggest that all eggs are alike. For atmosphere of the little shop was heavy with the bitter odour of
instance, there is the popular idea that owls hoot. Actually, only fresh newspaper print and ink: stacks of crisp newspapers and
very few owls hoot and these include the common brown or magazines lay neatly on the counter, the Higson and the boy
tawny wood owl. The white barn own screeches; the little own were making up the daily mend.
has a wailing cry; the long-earned owl barks; and the short- 82. At the shop Mr. Higson appeared in
eared owl snorts! Another mistaken idea is that all ducks ‘quack’, (a) his joy of smoking.
because the common farmyard duck is a domesticated form of (b) his most slip-shod condition.
the common wild duck or mallard that quacks. Actually most (c) the most unusual condition.
wild ducks call with whistles. (d) in a strange mood.
76. The main purpose of this passage is 83. The overall atmosphere in the shop was
(a) to describe the life of popular birds. (a) pleasant, wholesome and welcome.
(b) to show our incorrect ideas of bird life. (b) heavy with a lot of goods placed there.
(c) to show our perfect knowledge about birds. (c) with the inactivity of the early morning.
(d) to describe the calls of owls and ducks. (d) congenial and businesslike.
77. The impression created by repeating the terms, ‘popular’ PASSAGE – IV
and ‘common’ are A village must have some trade; and this village has always
(a) human beings are closely attached to birds. been full of virility and power. Obscure and happy, its splendid
EBD_7367
170
B- Reading Comprehension
energies had found employment in wrestling a livelihood out of 90. ‘The old comfortable picture of a friendly universe’ was:
the earth, whence had come a certain dignity, and kindliness, (a) a universe governed by religious beliefs
and love for other men. Civilization did not relax these energies, (b) a universe with men like Newton who were deeply
but it had diverted them; and all the special qualities, which religious
might have helped to heal the world, had been destroyed. The (c) a universe investigated by Galileo and Newton
family affection, the affection for the commune, the sane pastoral (d) the century immediately following Newton
virtues – all had perished. No villain had done this thing: it was 91. Religious skepticism arose because:
the work of ladies and gentlemen who were rich and often clever. (a) Galileo and Newton were not religious, being scientists
84. Village life is praised by the author because it (b) Newton discovered the law of gravitation
(a) helps villagers to achieve material prosperity. (c) of the discoveries of Darwin and the geologists of the
(b) makes men complacent. nineteenth century
(c) breeds humane virtues. (d) of the picture of the world that became the basis of
(d) is free from the din and hurry of city life. science after the seventeenth century
85. Civilization mainly destroys PASSAGE – VII
(a) the ability to create employment. Although Louis Braille died when he was only forty-three years
(b) family affection and pastoral virtues. old, he succeeded in devising a system of reading and writing
(c) medical facilities for the rural people. for the blind which is now taught all over the world. Braille lost
(d) agricultural trade. his sight accidentally as a child. Nevertheless, he was able to
PASSAGE – V complete his education at a school for the blind in Paris and
The art of growing old is one which the passage of time has became a teacher. In his day, the few books that were available
forced upon my attention. Psychologically there are two dangers for blind people were printed in big, raised type; the letters used
to be guarded against in old age. One of these is undue absorption were those of the ordinary alphabet. The reading of such books
in the past. It does not do to live in memories, in regrets for the required immense effort. Not only that, writing was almost
good old days. One’s thoughts must be directed to the future. impossible, for a blind person was still restricted to an alphabet
This is not always easy; one’s past is a gradually increasing which was extraordinarily difficult to reproduce on paper. Braille’s
weight. The other thing to be avoided is clinging to youth on the idea was to use raised dots, instead of raised letters. He evolved
hope of sucking vigour from its vitality. a system, which made use of only six dots in all. By various
86. “By the art of growing old”, the author means combinations of these dots, it not only proved possible to
(a) some special skill by applying which one can grow old. represent each letter in the alphabet, but punctuation marks,
(b) that growing old is like creating a work of art. numbers and musical notation as well. Reading and writing for
(c) the acceptance of old age as a fact of life. the blind have thus become enormously simplified. The sensitive
(d) the refusal on one’s part to grow old. fingers of a blind person can travel rapidly over the dots; and
87. The passage deals with the process of growing old. What there is a small machine, something like a typewriter, which
does it describe? enables the blind to write quickly and clearly.
(a) The decay of the senses in old age 92. Louis Braille:
(b) The psychological problems of old men (a) was born blind
(c) The desire in man to grow old (b) lost his sight when he was a child
(d) An old man’s ability to recollect his past. (c) lost his sight accidentally when he was forty-three
88. “It does not do to live in memories”. This statement means years old
that (d) was not blind, but studies at a school for the blind
(a) the past is not contained in memories. 93. Before Braille’s invention, the blind had difficulty in reading
(b) the old men are very forgetful. because:
(c) old men often think of the past. (a) there were only printed books
(d) thinking of the past does no good to old men. (b) there were no schools for the blind
89. It is difficult for old men to think of the future because (c) the few books available used the raised letters of the
(a) they are unable to think. ordinary alphabet
(b) they do not know what the future is like. (d) the books meant for the blind were heavy
(c) the past occupies their minds. 94. Braille’s system uses:
(d) their feeling that the past was far more happier than (a) only six dots
the present grows stronger day by day. (b) numbers and musical notation
PASSAGE – VI (c) ordinary alphabets in big raised type
It was Galileo and Newton – notwithstanding that Newton himself (d) a combination of alphabet and punctuation
was a deeply religious man – who destroyed the old comfortable 95. Braille’s system allows the blind
picture of a friendly universe governed by spiritual values. And (a) to write with ease
this was effected, not by Newton’s discovery of law of gravitation (b) to read easily
nor by any of Galileo’s brilliant investigations, but by the general (c) to read as well as write with ease
picture of the world which these men and others of their time (d) to read easily but to write with great effort
made the basis of the science, not only of their own day, but of PASSAGE – VIII
all succeeding generations down to the present. That is why the The Indian culture of our times is in the making. Many of us are
century immediately following Newton, the eighteenth century, striving to produce a blend of all cultures that seem today to be
was notoriously an age of religious skepticism. Skepticism did in clash with one another. No culture can live, if it attempts to be
not have to wait for the discoveries of Darwin and the geologists exclusive. There is no such thing as pure Aryan culture in
in the nineteenth century. It flooded the world immediately after existence in India today. Whether the Aryans were indigenous
the age of the rise of science. to India or were unwelcome intruders, does not interest me much.
Reading Comprehension B-171
What does interest me is the fact that my remote ancestors 3. Jean was too tired to walk after swimming.
blended with one another with the utmost freedom and we of the 4. Jean’s house was too far away from the shore.
present generation are a result of that blend. Which of the statements given above is/are correct ?
I do not want my house to be walled in, on all sides and my (a) 1 and 2 (b) 2 only
windows to be stuffed. I want the cultures of all lands to be (c) 2, 3 and 4 (d) 1 and 4
blown about my house as freely as possible. But I refuse to be PASSAGE – II
blown off my feet by any. I would have any young men and Punctually at midday he opened his bag and spread out his
women with literary tastes to learn as much of English and other professional equipment, which consisted of a dozen cowrie shells,
world-languages as they like, and then expect them to give the a square piece of cloths with obscure mystic charts on it, a note
benefits of their learning to India and the world alike like a Bose, book, and a bundle of Palmyra writing. His forehead was
a Ray or Tagore. But I would not have a single Indian forget, resplendent with sacred ash and vermilion, and his eyes sparkled
neglect or be ashamed of his mother tongue, or feel that he or with a sharp abnormal gleam which was really an outcome of a
she cannot think or express the best thoughts in his or her own continual searching look for customers, but which his simple
vernacular. Mine is not a religion of the prison house. clients took to be prophetic light and felt comforted. The power
96. The author views Indian Culture as: of his eyes was considerably enhanced by their position placed
(a) pure Aryan culture as they were between the painted forehead and the dark whiskers
(b) a clash of cultures which streamed down his cheeks : even a half-wit’s eyes would
(c) a continual blend of cultures sparkle in such a setting. To crown the effect he wound a saffron-
(d) the culture of remote ancestors. coloured turban around his head. This colour scheme never failed.
97. The author thinks that: People were attracted to him as bees are attracted to cosmos or
(a) The Aryans were indigenous to India. dahila stalks.
(b) The Aryans were unwelcome intruders 102. From the description of this passage one can make out the
(c) the question whether the Aryans were indigenous or person to be a :
not is not of interest (a) Snake Charmer (b) Footpath vendor
(d) the culture that we have inherited is the Aryan culture. (c) Astrologer (d) Priest
98. The author wants: 103. The eyes of person is described as sparkled because :
(a) the cultures of others to be kept out (a) He was sitting under midday sun
(b) the cultures of others to replace our old culture
(b) He was always looking for possible clients
(c) the freedom to blend other cultures with our own
(c) His forehead was bright with ash and vermilion
(d) the preservation of the culture of our ancestors.
(d) He was full of joy
99. The author wants Indians to:
(a) learn only English, as much as they like 104. The person opened his bag
(b) learn English and other world languages (a) to search for something he needed
(b) to indicate the start of his work
(c) learn only the mother tongue or the vernacular
(d) learn English and other world languages in addition (c) to keep his professional equipment
(d) to take out things for display
to the mother tongue.
105. The tone of the description is
DIRECTIONS (Qs. 100-126): In this section you have eight (a) sad (b) neutral
short passages. After each passage, you will find several (c) ironic (d) sympathetic
questions based on the passage. First, read a passage, and PASSAGE – III
then answer the questions based on it. You are required to ‘Punctuality’ said Louis XIV, ‘is the politeness of kings’. It is the
select your answers based on the contents of the passage and mark of a gentleman, and ‘the necessity of men in business’.
opinion of the author only. [2010-I] Washington once took his Secretary to task for being late. The
PASSAGE – I Secretary laid the blame upon his watch. Washington retorted.
The sky was already full of rusting wings. But when Jean stepped “Then, sir, either you must get a new watch or I must get a new
into the still lusterless water, he seemed to be swimming in an in Secretary”. Much of our success in life depends upon our being
determinate darkness until he saw the streaks of red and gold punctual in our undertakings and appointments. Hence the
over the horizon. Then he suddenly swam back to land and necessity of steadily cultivating this virtue in our daily life.
106. Washington wanted
clambered up the winding path to his house. After a great deal of
(a) to get a new watch
panting he reached a little gate, pushed it open and climbed a (b) to get a new secretary
stairway. The house above the world had its huge bay-windows (c) his secretary to get a new watch
through which one could see the horizon from one edge to the (d) his secretary to be punctual
other. Here, no one complained of exhaustion. Every one had his 107. Punctuality is a virtue
joy to conquer, every day. (a) which is appreciated by kings
100. Which of the following is/are indicated by the description (b) which is difficult to cultivates
in the passage ? (c) which is necessary for success in life
1. Time before sunrise 2. Time after sunset (d) which can make daily life pleasant
3. Clouds 4. Birds PASSAGE – IV
Select the correct answer using the code given below : Fortunately it is as yet only through fantasy that we can see
(a) 2 and 3 only (b) 2 only what the destruction of the scholarly and scientific disciplines
(c) 2, 3 and 4 only (d) 1 and 4 only would mean to mankind. From history we can learn what their
101. What do the words “great deal of panting” imply? existence has meant. The sheer power of disciplined thought is
1. Jean was too weak to walk. revealed in practically all the great intellectual and technological
2. Jean’s house was on a hill. advances which the human race has made. The ability of the
EBD_7367
172
B- Reading Comprehension
man of disciplined mind to direct this power effectively upon (d) the behaviour of any male to win a female of the same
problems for which he has not specifically trained is proved by species
examples without number. The real evidence for the value of 115. Which of the phrases best help to bring out the precise
liberal education lies in history and in the biographies of men meaning of ‘consciously strained’?
who have met the valid criteria of greatness. These support (a) Proudly reared and exhibited
overwhelmingly the claim of liberal education that it can equip a (b) Permanently in a state of display
man with fundamental powers of decision and action, applicable (c) Purposefully put in a state of display
not only to boy-girl relationship, to tinkering hobbies, or to (d) Possession of nesting sight
choosing the family dentist, but to all the great and varied 116. The most important ‘trick’ of the jackdaw to win his female
concerns of human life ------------- not least, those that are is
unforeseen. (a) displaying his head and neck
108. Liberal education enables a person to (b) to be like a human being
(a) read with more discernment than others (c) to become embroiled with other birds
(b) apply general principles to resolve issues (d) to possess a place for building a nest
(c) gain prestige 117. The jackdaw fights with other jackdaws because
(d) develop a clearer understanding of history than others (a) he does not respect the senior birds
109. In this passage, the author stresses the importance of (b) he wants to fight with his rivals
(a) education for living (c) he does not like to be challenged
(b) technological advances (d) he wants to show off
(c) increased interest in the study of history 118. Which of the following statements best describes the central
(d) satisfying the desire for security theme of the passage ?
110. In this passage, the expression ‘specifically trained’ refers (a) The courtship behaviour of birds
to (b) The similarities between the courtship behaviour of
(a) characteristically trained (b) particularly trained birds and man
(c) peculiarly trained (d) ostensibly trained (c) The astonishing facts about the jackdaw’s love life
111. According to the author, ‘the great and varied concerns of (d) The scientific study of the jackdaw’s life
human life’ are about PASSAGE – VII
(a) fundamental rights Young seekers after peace know that only trust shown to all the
(b) challenges facing mankind peoples of the earth and not just to a few of them, can lead to the
(c) tinkering hobbies healing of the wounds that tear them apart and so it is essential
(d) liberal education never to humiliate the members of a nation whose leaders have
PASSAGE – V committed inhuman acts. Essential also is boundless concern
Now a days we are amused by professionals. Why listen to your for so many men and women who today, as exiles or immigrants,
friends singing when you can hear the great singers of the world live on foreign soil. If every home was open to somebody of
on the gramophone or the radio? Why read even a detective foreign origin, the racial problem would be partially solved.
story if you can see one at the cinema, and why play football 119. For the reconciliation and unity it is essential.
with players who are not very good when you can go, by train (a) to have no discrimination in the trust shown to the
or car, to see some of the best players in your country playing an peoples of the earth
important match; if you have a television set, just sit comfortably (b) to have young seekers after peace
at home and watch the same without the trouble of going outside ? (c) to have confidence in at least a few of the peoples
112. The primary criticism of the author about his contemporaries (d) to punish the leaders who committed in human sins
is that 120. To make the members of a nation responsible for the in
(a) they are unprofessional and unskilled. human acts of their leaders is
(b) they want to enjoy all the good things of life. (a) unimportant (b) understandable
(c) they waste all their time with amusements. (c) unjust (d) undesirable
(d) they have a lazy and mechanical attitude towards 121. Boundless concern should be shown to
amusements. (a) one’s own country men alone
113. The impression you get about the author is that he is (b) the people belonging to our friendly nations only
(a) a cynic (b) an old timer (c) the foreigners who just visit our country
(c) a reformer (d) a social critic (d) all the exiles living away from their native lands
PASSAGE – VI 122. The theme of the passage is
A male jackdaw’s courtship behaviour is astonishingly human. (a) Rigid nationalism alone can help peaceful co-existence
All his movements are consciously strained and his proudly (b) Trusting all the peoples of the earth may lead to serious
reared head and neck are permanently in a state of self-display. problems
He provokes the other jackdaws continually if the female jackdaw (c) Being cautious of others is a must to live in peace
is looking on and he purposefully becomes embroiled in conflicts (d) Concern for everyone irrespective of the race of
with otherwise deeply respected superiors. Above all, he seeks country
to impress his loved one with the possession of a potential PASSAGE – VIII
nesting site, from which he drives all other jackdaws, irrespective We shall go on to the end; we shall fight in France, we shall fight
of their rank. on the seas and oceans, we shall fight with growing confidence
114. A ‘courtship behaviour’ may best be described as and strength in the air, we shall defend our island whatever the
(a) the behaviour of a jackdaw who shows off his feather. cost may be, we shall fight on the beaches, we shall fight on the
(b) the behaviour of a male bird to attract a female bird. landing grounds, we shall fight in the fields and in the streets,
(c) the behaviour by which a male bird displays its beauty we shall fight in the hills. We shall never surrender, and even if
to the female bird
Reading Comprehension B-173
this island or a large part of it were subjugated and starving, 1907. Mandela toured the Cape, Natal and the Transvaal, visiting
then our empire beyond the seas would carry on the struggle, houses in the townships, explaining the plans, sometimes talking
until the New World steps forth to the rescue and the liberation through the night. His task was to inspire people with confidence
of the Old. in their ability to overcome oppression through a direct non-
123. On the basis of the passage which of the following violent challenge to the government. As always, there were the
statements may be said to be correct ? problems of being black in small towns, no hotels or taxis for
(a) The speaker is encouraging his men for the conquest Africans, nor were there telephone lines in township homes.
of France. This meant walking miles to the location and knocking on a
(b) The speaker is aggressive and maniacal war-monger. likely looking door. Sometimes they were welcomed by an
(c) The speaker is not satisfied with the conquest of the enthusiastic stranger, sometimes rebuffed by the cautious.
island. 130. The purpose of Mandela’s talks was to help people
(d) The speaker is a patriot urging the defence of his (a) court arrests
mother-land. (b) oppose oppression
124. The speaker in the passage wants to go on fighting because. (c) join him in his campaign tours
(a) he is raving lunatic (d) get small town facilities
(b) he is in a state of utter despair 131. It is clear from the passage that the black Africans.
(c) he expects help from other quarters (a) enjoyed small town facilities .
(d) he is the leader of a suicide squad (b) were helped by Gandhi in their freedom struggle.
125. Which of the following pairs of the phrases helps best to (c) had an unfavourable government.
bring out the intention of the speaker ? (d) liked visiting homes in townships
(a) “go on to the end”, “shall never surrender” PASSAGE – III
(b) “growing confidence”, subjugated and starving” One day an army group won a land battle against the enemy. The
(c) “subjugated and starving”; fight on the landing commander feared that the enemy’s powerful air force might
ground” bomb his camp that night in revenge. So he ordered all lights to
(d) “fight in the streets”, “subjugated and starving” be put out at 7.00 PM. At midnight the commander went round
126. The passage consists of repetitive patterns in syntax and inspecting the camp. Seeing a light in a tent, he entered it. His
vocabulary. The effect of this style is that it son, an officer under him, was writing a letter. The son explained
(a) reveals the speaker’s defects in giving a speech that he was writing to his mother about his brave deeds in battle.
(b) produces the impression of bad poetry The commander told his son to add to his letter that by the time
(c) coveys the speaker’s helpless situation his mother received the letter he would have been shot dead for
(d) reinforces the speaker’s basic intention which is to indiscipline.
motivate fellow countrymen to fight for their 132. The commander went round the camp at midnight because
motherland. he
DIRECTIONS (Qs. 127-142) : In this section you have five (a) was too tired from the day’s battle to go to sleep
short passages. Each is followed by questions based on the (b) wished to check if his soldiers had obeyed his order
passage. First, read the passage and answer the questions based (c) was too worried about the next day’s battle
on the passage. You are required to select your answers from (d) wished to check if enemies hand entered his camp
133. The commander entered his son’s tent because he
the given options. [2010-II]
(a) wished to see and talk to his son
PASSAGE – I (b) suspected that enemies had entered his tent
One day we were becalmed among a group of small islands, most (c) wished to send a message to his wife
of which appeared to be uninhabited. As soon as we were in (d) had to punish any soldier who disobeyed his order
want of fresh water, the captain sent the boat ashore to bring off 134. The son was writing a letter because he
a cask or two. But we were mistaken in thinking there were no (a) wanted to write to his mother about his father’s brave
natives, for scarcely had we drawn near to the shore when a deeds in battle
band of savages rushed out of the bush and assembled on the (b) loved his mother so much that he had to write to her
beach, brandishing their clubs and spears in a threatening manner. (c) was eager to tell his mother about his own deeds
127. The captain sent the boat to the shore to (d) did not care for orders since his father was the
(a) look for inhabitants commander
(b) find help PASSAGE – IV
(c) find a place to settle there The heat-wave deepened during the following few days while
(d) fetch some water Jack and I lazed about in the house and yards, wearing ragged
128. The savages brandished their spears in order to shirts and discarded garments, because the more presentable
(a) display their skill ones were being packed by Mother. She was obviously not strong
(b) frighten the crew enough to cycle down to Hemisphere, where Father and Jack
(c) welcome the crew to the island had been one week-end, to see and rent a cottage in Ropley, near
(d) tell the crew to leave Alresford. From this prospective journey Jack had returned with
129. The inhabitants of the islands were half a dozen photographs taken with a plate-camera which he
(a) man-eaters (b) pirates had made for himself, the aperture being a pinhole. This was
(c) cruel people (d) primitive tribes only one of his many ingenious artefacts. I had studied the
PASSAGE – II pictures, which included a church that leaned backwards, in the
Nelson Mandela was appointed national volunteer-in-chief of hope of finding the perpetually teasing certainty which we look
the Defence Campaign; his deputy was Maulvi Cachalia, whose for when about to take some adventurous step into the unknown.
father had been one of the bravest resisters alongside Gandhi in But Ropley remained unreal.
EBD_7367
174
B- Reading Comprehension
135. During the hot summer days the author and Jack based on the contents of the passage and opinions of the author
(a) were taking adventurous steps into the unknown only. [2011-I]
(b) went visiting several churches in Ropley PASSAGE – I
(c) were busy repairing a camera Literature and history are twin sisters, inseparable. In the days
(d) were passing their time in idleness of our own grandfathers, and for many generations before them,
136. They were planning the basis of education was the Greek and Roman classics for the
(a) to move out Ropley educated, and the Bible for all. In the classical authors and in the
(b) a trekking expedition to Alresford Bible, history and literature were closely intervolved, and it is
(c) to do some photography that circumstance which made the old form of education so
(d) to make some artefacts stimulating to the thought and imagination of our ancestors. To
137. The author and Jack were wearing ragged shirts and read the classical authors and to read the Bible was to read at
discarded garments because once the history and the literature of the three greatest races of
(a) they were very poor the ancient world. No doubt the classics and the Bible were read
(b) it was summer in a manner we now consider uncritical but they were read
(c) all their good clothes were already packed according to the best tenets of the time and formed a great
(d) they were lazing about humanistic education. Today the study both of the classics and
138. The plate camera of the Bible has dwindled to small proportions. What has taken
(a) was the only artefact made by Father and Jack
their place? To some extent the vacuum has been filled by a more
(b) was the only artefact Jack had made for himself
correct knowledge of history and a wider range of literature. But
(c) was only one of Jack’s many such artefacts
(d) was borrowed by Jack from his friend for taking half a I fear that the greater part of it has been filled up with rubbish.
dozen photographs. 143. Which of the following statements best reflects the
139. Father and Jack had been to Ropley one weekend underlying tone of the passage ?
(a) to take photographs of the church that leaned (a) Literature and history are mutually exclusive
backwards (b) Literature and history are complementary to each other
(b) because Ropley was unreal (c) The study of literature is meaningless without any
(c) to take some adventurous step into the unknown knowledge of history.
(d) to see and rent a cottage there (d) Literature and history are inseparably linked together
PASSAGE – V in the classics and the Bible
One day a tea contractor, Mr. Sharma was working on an estate 144. The author of the above passage says that in the past the
with his wife and daughter. He noticed a light movement on the basis of education for all people, irrespective of their
edge of the jungle, so he stopped to watch for a moment. To his intellectual calibre, was
astonishment a large tigress appeared and came towards Mr. (a) Greek and Roman classics
Sharma. The tea contractor was a very brave man. He told his (b) The Bible
wife and daughter to run towards a nearby road, while he stepped (c) A correct knowledge of history
to fight the tigress with a knife. (d) A wider range of literature
The tigress sprang at Mr. Sharma and knocked him down, but he 145. The author of the above passage says that the classics and
managed to wound it with his knife. As a result, the tigress and the Bible were read by his ancestors
Mr. Sharma knocked each other unconscious. Luckily for Mr. (a) methodically and with discretion.
Sharma, a friend heard the noise of the fight and came to (b) in a manner that broadened their view of life
investigate. He found Mr. Sharma and carried him to the road. (c) with great emphasis on their literary values
Then he stopped a car and sent the injured man to a hospital, (d) without critical discrimination but in the light of their
where he eventually recovered. humanistic culture
The tigress disappeared for a few days but was later hunted 146. According to the author of the above passage, the old form
down and shot by a Game Ranger. The Game Ranger discovered of education, based on the study of the classics and of the
that the tigress had injured her paw in a wire trap and had been Bible, has
unable to hunt wild animals in its normal manner. (a) succeeded in creating interest in history
140. When did Mr. Sharma’s friend go to see what was wrong ? (b) laid the basis of human civilization
(a) When he saw the tigress attacking his friend (c) had a gradual decline in our time
(b) Just before the tigress was knocked down (d) been rejuvenated in the context of modern education
(c) When the friend was knocked down 147. The author of the above passage fears that the greater part
(d) When he heard something unusual of the vacuum created by lack of interest in the classics and
141. When Mr. Sharma saw the tigress, he was the Bible had been filled up by
(a) surprised (b) afraid (a) a richer sense of history
(c) nervous (d) angry (b) a wider range of literature
142. Mr. Sharma stayed on to fight the tigress for (c) worthless ideas
(a) he wanted to try to protect his wife and daughter (d) a new philosophy of life
(b) he was strong man who loved fighting PASSAGE – II
(c) he couldn’t run as quickly as his wife During his early days as editor of the popular magazine. Saturday
(d) he tigress caught him before he could run Evening Post, George Lorimer did much of the reading of
DIRECTIONS (Qs. 143-162) : In this Section you have four unsolicited stories. This meant endless hours of sitting at the
short passages. After each passage, you will find some questions desk, pouring over big stacks of manuscripts, trying to decide
which were worthy of publication and which were not. Lorimer
based on the passage. First, read a passage and answer the
became an expert at making these decisions.
questions based on it. You are required to select your answers One day he received a huffy letter from a would-be writer who
Reading Comprehension B-175
had a complaint. “Last week you rejected my story,” she wrote. (a) Destruction of the human body
“I am positive you did not read it, because, as a test, I pasted (b) Breaking of the physical environment of man
together pages 14, 15 and 16. The manuscript came back with the (c) Attempt to destroy the man’s character
pages still pasted. There is no question in my mind but that you (d) Inflicting physical and mental agony on man
are a sham and a disgrace to your profession.” 157. The passage consists of two long sentences and a short
Lorimer’s reply was succinct: “Madam, at breakfast when I crack one. The purpose of this style could be to suggest that
open an egg. I don’t have to eat whole egg to know it is bad.” (a) it is impossible for man to attain perfection of character
148. Lorimer did much reading of the stories (b) the attainment of perfect character is the result of a
(a) if they were the solicited ones long process of mental discipline
(b) when they appeared to be bad (c) the whole life process is clumsy
(c) when they were from women writer (d) there is a lot of confusion in our understanding of
(d) when they came unsolicited sense organs, character, etc.
149. Lorimer was a good editor because PASSAGE – IV
(a) his reply to the angry writer was polite One of the most serious problems confronting our country is
(b) he apologized for rejecting the story without reading it that of a fast-growing population. In fact, it is at the root of many
(c) he could find the worth of a story with a little effort other problems. At the moment, thanks to planning, we are able
(d) he was prompt, in writing letters to produce food and cloth sufficient for our people and even in
150. The lady wrote a huffy letter because some excess. But if the population continues to grow at this rate,
(a) her story was rejected it will not be long before the surplus turns into a bare minimum
(b) her story was rejected unread and even a deficit. The position in regard to accomodation is
(c) her story was rejected although it was good even now far from satisfactory in spite of our efforts.
(d) Lorimer was biased in his decision. 158. “It is at the root of many other problems” means that
151. Lorimer’s reply was (a) it is found along with many other problems
(a) irrelevant (b) rude (b) it is caused by many other problems
(c) witty (d) funny (c) it gives rise to many other problems
152. Lorimer read the stories (d) it is buried under many other problems
(a) because he enjoyed reading them 159. The present satisfactory position in regard to food and
(b) in order to publish them cloth is due to
(c) only to find fault with them (a) the fact that the population has been controlled
(d) in order to review them (b) our good luck
PASSAGE – III (c) good rainfall
As the tortoise tucks its feet and head inside the shell and will (d) our economic planning
not come out even though you may break the shell into pieces, 160. If the population of India continues to increase at this rate,
even so the character of the man who has control over his motives the situation in regard to food and cloth
and organs, is unchangeably establishment. He controls his own (a) is likely to remain the same
inner forces, and nothing can draw them out against his will. By (b) is likely to become less satisfactory
this continuous reflex of good thoughts and good impressions (c) is likely to improve
moving over the surface of the mind, the tendency to do good (d) is likely to vary up and down
becomes strong, and in consequence, we are able to control the 161. The situation in respect of accommodation
Indriyas or sense organs. (a) is less than satisfactory
153. The author uses the phrase ‘inner forces’ in this passage. (b) is quite satisfactory
Which of the following would be its most correct meaning (c) is improving rapidly
in the context ? (d) is the result of total neglect
(a) Emotional disturbances in man 162. At present Indians have
(b) Strength of the internal organs (a) more provision for cloth than accommodation
(c) Forces produced by sense organs (b) more provision for accommodation than cloth
(d) Reflection of the intellect (c) abundance of cloth and accommodation
154. Which of the following statements would illustrate the (d) scarcity of cloth and accommodation
metaphor in the passage ?
(a) Man is slow-moving and slow-witted DIRECTIONS( Qs. 163-187): In this section you have six short
(b) A man of character refuses to be influenced by outside passages. Each passage is followed by questions based on the
compulsions against his will passage. First, read the passage and answer the questions based
(c) Man confines himself to a life of isolation on the passage. You are required to select your answers from
(d) Man cannot have a good character or strong will the given options. [2011-II]
155. Which of the following statements may be assumed to reflect PASSAGE – I
the central theme of the passage ? Cozette could have been a pretty child, but she was thin and
(a) Good thoughts lead to the control of the sense organs pale and her eyes were stained with weeping. She was dressed
(b) Control of the sense organs leads to good thoughts in her thin torn cotton dress and she shivered all the time. Here
(c) Character, though established, may be disturbed by and there on her body were blue marks from the beatings that
outside forces her mistress had given her. Her naked legs were red and rough.
(d) No man can achieve success in destroying the inner When she spoke, her voice trembled. Everything about the child,
forces her looks, her behaviour, her speech, her silence, every small
156. Which of the following statements would be most correct gesture she made, showed a terrible fear. She was so afraid that,
in explaining the metaphorical meaning of ‘break the shell even though she was wet through, she dared not go near the fire
into pieces’ ? to warm herself, but sat shivering in a corner of the room.
EBD_7367
176
B- Reading Comprehension
163. Cozette could not be a pretty child because services in which he has concentrated, than he can consume
(a) she was pale and emaciated himself. The surplus he has exchanged with other members of
(b) she was weeping the community, acquiring, in exchange the things he needs that
(c) she was scantily dressed others have produced.
(d) she was trembling with fear 172. Very few people can satisfy their needs today by
164. Cozette’s voice trembled because (a) providing things for themselves
(a) she was feeling cold (b) exchanging goods and services
(b) she was frightened (c) concentrating on what they can do best
(c) there was no fire near her (d) individual specialization
(d) she was wearing a thin, cotton dress 173. Exchange of goods becomes possible only when
165. Cozette’s terrible fear is conveyed most vividly by the (a) there is no specialization
description of (b) the goods are produced in surplus
(a) her constant shivering (c) primitive societies become modern
(b) her tear-stained eyes (d) individuals make things for themselves
(c) her trembling voice 174. Specialization and exchange began when men started
(d) her inability to go near the fire (a) big industries
166. The cause of Cozette’s fear is explained by (b) concentrating on their work
(a) her pale appearance (c) producing things for individual use
(b) her torn dress (d) living in communities
(c) the blue marks on her body 175. Exchange of goods and services becomes necessary
(d) her speech because
PASSAGE – II (a) man is a social animal
Discussions on drug addiction should also be concerned with (b) reciprocity is the law of life
the vast majority of people who are not addicts. Their homes (c) trade and commerce are means of progress
and lives are insecure because our narcotics laws drive such (d) we cannot produce everything we need ourselves
people to crime. The drug addict is almost never dangerous when PASSAGE – IV
he is under the influence of drugs. What makes him dangerous I came home from one vacation to find that my brother Ron had
is the desperate need for money to buy the next dose. Drugs are brought a dog while I was away. A big burly, choleric dog, he
available only in an illegal black market. The costs are always acted as if he thought I wasn’t one of the family. There
stupendous, and this is what drives the addict to steal, rob and was a slight advantage in being one of the family. For he didn’t
even kill. bite the family as often as he bit strangers. Mother used to send
167. According to the author, discussions of drug addiction are a box of candy every Christmas to the people he bit. The list
generally concerned with finally contained forty or more names. Nobody could understand
(a) addicts (b) non-addicts why we didn’t get rid of the dog!
(c) criminals (d) black marketers 176. Which of the following descriptions fits the dog?
168. Addicts take criminal acts because (a) The dog was tiny and delicate
(a) drugs make them lose self-control (b) The dog was sturdy and short-tempered
(b) the habit of robbing and stealing is hard to break (c) The dog was huge and cool
(c) they need large sums of money to buy drugs (d) The dog was small and sweet-tempered
(d) law is powerless against them 177. The dog did not consider the writer as one of the family.
169. The author seems to criticize the narcotics laws for What do you think was the consequence of this?
(a) being too lenient (a) The dog barked at him all the time
(b) being too complicated (b) The dog drove him out of his own house
(c) being ineffective (c) The dog behaved with him in an unfriendly way
(d) driving addicts to crime (d) The dog bit him more than he bit others in the family
170. The word ‘stupendous’’ in the passage means 178. The Christmas list contained more than forty names. What
(a) very high (b) foolish does this suggest?
(c) shocking (d) illegal (a) The writer’s mother had a lot of friends
171. With reference to the passage, consider the following (b) The writer’s family celebrated Christmas well
statements : (c) The writer’s dog had bitten at least forty people
1. Addiction to drugs is a criminal act. (d) The writer’s mother sent them candy boxes
2. Drug addicts cannot be rehabilitated. PASSAGE – V
Which of the statements given above is/are correct? A little girl was learning a history lesson with her governess. All
(a) 1 only (b) 2 only the morning she had been reading it over and hearing it explained
(c) Both 1 and 2 (d) Neither 1 nor 2 by her governess, but no good came of either the reading or the
PASSAGE – III teaching. The governess went over the lesson several times,
Even in the most primitive societies the great majority of people explained the meaning, and for the last time, asked her pupil to
satisfy a large part of their material needs by exchanging goods read it over. After due time had been given, the girl was examined
and services. Very few people indeed can make for themselves as to her knowledge of the lesson; but not a single answer could
everything they need–all their food, their clothes, their housing, she give correctly. The governess lost patience with her, and
their tools. Ever since men started living in communities, they threatened to punish her unless she could state where a certain
have been satisfying their needs by means of specialization and treaty was signed.
exchange; increasingly each individual has concentrated on what 179. According to the passage, the little girl read the lesson and
he can do best, and has produced more of the special goods or heard it explained all the morning because
Reading Comprehension B-177
(a) the girl did not like her governess 187. With reference to the passage, consider the following
(b) the governess could not explain it long enough statements:
(c) the girl could not understand it 1. Many doctors did not believe that Jenner was a doctor.
(d) the girl read the lesson only once 2. There are no outbreaks of smallpox nowadays.
180. With reference to the passage, consider the following Which of the statements given above is/are correct?
statements: (a) 1 only (b) 2 only
1. The governess taught the same lesson several times. (c) Both 1 and 2 (d) Neither 1 nor 2
2. The governess wanted to complete her teaching work
quickly. DIRECTIONS (Qs. 188-208) : In this section there are six short
Which of the statements given above is/are correct? passages. Each passage is followed by items based on the
(a) 1 only (b) 2 only passage. Read each passage and answer the items that follow.
(c) Both 1 and 2 (d) Neither 1 nor 2 [2012-I]
181. After reading the whole passage, which of the following PASSAGE – I
impressions do you think correct about the inability of the Our voyage was very prosperous, but I shall not trouble the
girl to answer questions correctly? reader with a journal of it. The captain called in at one or two parts
(a) The dullness of the girl and sent in his long-boat for provisions and fresh water, but I
(b) The incompetence of the governess never went out of the ship till we came into the Downs, which
(c) The difficulty of the language was on the 3rd day of June, 1706, about nine months after my
(d) The lack of time escape. I offered to leave my goods in security for payment of
182. Which of the following correctly expresses the meaning of my freight, but the captain protested he would not receive one
‘lost patience with her’? farthing. We took kind leave of each other, and I made him promise
(a) The inability of the governess to endure further the that he would come to see me at my house in Redriff. I hired a
girl’s failure to answer. house and a guide for five shillings which I borrowed from the
(b) The governess lost her enthusiasm to teach the girl captain.
(c) The governess felt that the girl cannot be taught the 188. When the writer uses the word “prosperous” to describe
lesson the voyage, he means that
(d) The governess felt that she was not good enough to (a) it made him rich.
teach the girl (b) it made him healthy.
PASSAGE – VI (c) it was very pleasant
Many doctors flatly refused to believe Jenner when he announced (d) it was uneventful.
that he had found a preventive against smallpox. They declared 189. On the voyage, the author
vaccination to be a dangerous practice. But the dread of smallpox (a) left the ship at intervals.
was in everybody’s heart, and people flocked to Jenner to be (b) was not able to leave the ship because it did not stop.
vaccinated. The Latin word for cow is ‘vacca’; it is the root from (c) never left the ship at all.
which the word vaccination was formed. Some of the ‘vacca’ (d) never left the ship till they came into the downs.
used by Jenner were not pure and some harms were done; but 190. In the context of the passage, the word “provisions” means
when supplies of pure vaccine were available, the practice of (a) mainly food.
vaccinating spread all over England and from England to other (b) mainly security.
countries. We hardly hear of outbreaks of smallpox now. (c) money.
183. The passage describes (d) mainly ammunition.
(a) how smallpox may be treated 191. For the payment of the author's freight, the captain
(b) how vaccines were manufactured in England (a) kept his goods as security.
(c) the dangers of vaccination especially for children (b) refused to accept any money.
(d) the gradual acceptance of vaccination as a preventive (c) protested against being paid only a farthing.
against smallpox (d) accepted a sum of money.
184. Vaccination sometimes proved harmful because 192. From the passage, it is clear that the captain's attitude to
(a) vaccination was a dangerous practice the author was
(b) some of the vaccines used were of a poor hygienic (a) one of hostility.
standard (b) one of indifference.
(c) there are physiological difference between cows and (c) one of extreme friendliness and kindness.
human beings (d) one of disgust and irritation.
(d) vaccination is given at a very early age PASSAGE – II
185. People hastened to get themselves vaccinoid because What were the early ideas of men about the sky and the earth?
(a) many doctors supported Jenner’s claims They naturally believed that the earth was motionless, and they
(b) fear of the terrible disease drove them to take the risk also supposed that it was flat. These two ideas do not surprise
of vaccination us. Children now-a-days think the same until they are taught
(c) supplies of pure vaccine had now become available differently. How were men to know that the earth was a ball circling
(d) the practice of vaccinating had spread all over the round the sun? They had no telescope for accurate observation.
world They had not travelled round the world. In fact, many parts of the
186. Vaccination was intended by Jenner to world in those days were unexplored and unknown. They thought
(a) cure people suffering from smallpox they lived on a kind of flat plate, and that the sky with the sun and
(b) delay the death of smallpox victims the moon and the stars, was a kind of inverted bowl turning
(c) build up a defence against smallpox germs round above them. The sun, the moon and the stars were their
(d) prevent cows from spreading the disease lamps for day and night.
EBD_7367
178
B- Reading Comprehension
193. Unless children 'are taught differently they think that Eventually, George returned home after a visit, halfway
(a) the earth is round and moving. expecting everyone in town to be at the railway station to welcome
(b) the sun and the moon are motionless. him.
(c) the sun and the moon are moving. Much to his surprise, George saw that no one, not even his
(d) the sun and the moon are moving round the earth. family, was around to meet him when he descended from the
194. The early ideas of man were wrong because train. He looked very neat in a new suit and carried a bulky suitcase
(a) man did not use the telescope. full of fashionable clothes.
(b) man did not like to travel. After a little while, the station master came from his office
(c) man never had the scientific knowledge. and went over to the young fellow. “Well, hello there,
(d) man was foolish and lazy. George,” he called out cheerily, “Are you going away ”?
195. What was true for the early man ? 200. The station master's question implied that
(a) The earth was round and moving. (a) he offered help to George in climbing the train.
(b) The telescope was accurate. (b) he is known to George very intimately.
(c) Travelling and, exploring were the methods to gain (c) he is making fun of George.
knowledge. (d) George’s absence from the town was not noticed by
(d) The sun was motionless. him.
196. What was the main cause of the early man’s wrong ideas ? 201. George hoped for a big welcome because
(a) Lack of scientific knowledge (a) of his achievement and success.
(b) Seeing and believing (b) he is returning home after a very long time.
(c) Lack of desire to know (c) people loved him.
(d) Lack of desire to observe and explore (d) his community wanted dynamic leaders like him.
PASSAGE – III 202. George's great expectations are an indication of his
The man sat up in the snow for a moment and struggled for (a) humility. (b) optimism.
calmness. Then he pulled on his gloves by means of his teeth, (c) pride. (d) love of his community.
203. George's success was most clearly visible in
and got upon his feet. He glanced down at first in order to assure
(a) the station master's words.
himself that he was really standing up, for the absence of
(b) his clothes.
sensation in his feet left him unrelated to the earth. His erect (c) his being unbearably conceited.
position in itself started to drive the webs of suspicion from the (d) the manner in which he was received by the village.
dog's mind; and when he spoke peremptorily, with the sound of PASSAGE – V
whip-lashes in his voice, the dog rendered its customary The assault on the purity of the environment is the price that we
allegiance and came to him. As it came within reaching distance, pay for many of the benefits of modern technology. For the
the man lost his control. His arms flashed out to the dog and he advantages of automotive transportation we pay a price in smog-
experienced genuine surprise when he discovered that his hands induced diseases; for the powerful effects of new insecticides,
could not clutch, that there was neither bend not feeling in the we pay a price in dwindling wildlife and disturbances in the relation
fingers. He had forgotten for the moment that they were frozen of living things and their surroundings; for nuclear power, we
and that they were freezing more and more. All this happened risk the biological hazards of radiation. By increasing agricultural
quickly and before the animal could get away, he encircled its production with fertilizers, we increase water pollution.
body with his arms. He sat down in the snow and in this fashion The highly developed nations of the world are not only the
held the dog, while it snarled and whined and struggled. immediate beneficiaries of the good that technology can do, they
197. From the passage, which group of words expresses the are also the first victims of the environmental diseases that
effect of snow upon the man's feet ? technology breeds. In the past, the environmental effects which
(a) With the sound of whip-lashes in his voice. accompanied technological progress were restricted to a small
(b) His arms, flashed out to the dog. place and relatively a short time. The new hazards are neither
(c) The absence of sensation in his feet left him unrelated local nor brief. Modern air pollution covers vast areas of
to the earth. continents. Radioactive fallout from nuclear explosions is
(d) The man sat up in the snow for a moment and struggled worldwide. Radioactive pollutants now on the Earth's surface
for calmness. will be found there for generations, and in the case of Carbon-14,
198. The statement that, the man experienced genuine surprise for thousands of years.
when he discovered that his hands could not clutch means 204. The passage emphasizes that modern technology
that (a) is totally avoidable.
(a) the man did not see anything to clutch. (b) has caused serious hazards to life.
(b) the man had nothing to clutch. (c) has greater effect on developed countries.
(c) the man was afraid of the dog. (d) is the source of the miseries of mankind.
(d) there was neither bend nor feeling in the fingers. 205. The harmful effects of modern technology are
199. Which word of group of words shows the exact condition (a) widespread but short lived.
of being ‘frozen’? (b) widespread and long lasting.
(a) Whip-lashes in his voice (c) local and long lasting.
(b) He pulled on his gloves (d) severe but short lived.
(c) His hands could not clutch 206. With reference to the passage, the following assumptions
(d) Lost his control have been made :
PASSAGE – IV 1. The widespread use of insecticides has caused
George was a young man who had gone to the big city from a ecological imbalance.
small rural community and, in a relatively short time, attained 2. Conservation of natural flora and fauna is impossible
prominence in the business world. His sudden rise had gone into in this age of modern technology.
his head, however, and he became unbearably conceited. Which of the assumptions is/are valid ?
Reading Comprehension B-179
(a) 1 only (b) 2 only PASSAGE – II
(c) Both 1 and 2 (d) Neither 1 nor 2 It was a bitterly cold night, and even at the far end of the bus the
PASSAGE – VI east wind that raved along the street cut like a knife. The bus
To what extent, though, are modern farming methods sustainable? stopped, the two women and a man got in together and filled the
There is abundant evidence that a high price has to be paid to vacant places. The younger woman was dressed in sealskin and
sustain the high rates of food production achieved by farmed carried one of those Pekinese dogs that women in sealskin like to
monocultures. For example, they offer ideal conditions for the carry in their laps. The conductor came in and took the fares.
epidemic spread of diseases such as mastitis, brucellosis and Then his eye rested with cold malice on the beady-eyed toy dog.
swine fever among livestock and coccidiosis among poultry. I saw trouble brewing. This was the opportunity for which he had
Farmed animals are normally kept at densities far higher than been waiting, and he intended to make the most of it.
their-species would meet in nature with the result that disease 212. The wind that blew on the night was
transmission rates are magnified. In addition, high rates of (a) mild (b) pleasant
transmission between herds occur as animals are sold from one (c) bitter (d) sharp
farming enterprise to another; and it is easy for the farmers 213. The younger woman was carrying the dog as
themselves, with mud on their-boots and their vehicles, to act as (a) a necessity
vectors of pests and disease. (b) a fashion
207. With reference to the passage, consider the following (c) an expression of provocation
statements : (d) an escort
1. The modern practices of farming are undesirable for 214. Which of the following statements best describes the nature
developing countries. of the conductor ?
2. Monoculture practices should be given up to eliminate (a) He was dutiful
disease transmission in animals. (b) He was a law-abiding person
Which of the above statements is/are correct ? (c) He liked dogs
(a) 1 only (b) 2 only (d) He was unfriendly and malicious
(c) Both 1 and 2 (d) Neither 1 nor 2 215. It was a bitterly cold night, and even at the far end of the
208. What is the essence of this passage ? bus the east wind that raved along the street cut like a
(a) Farming is a very costly affair. knife".
(b) Farmed animals are kept at higher densities in This sentence gives us an idea of
monocultures. (a) a lonely night-bus journey
(c) There is a widespread transmission of animal diseases (b) an unbearable cold night
now-a-days. (c) the wind at the time that was still and cold
(d) Human dependence of monoculture is fragile. (d) the hardship of author's journey
DIRECTIONS (Qs. 209-225) : In this section there are five short PASSAGE – III
passages. Each passage is followed by questions based on the Before an armed robber locked Mary Graves in the sweltering
passage. Read each passage and answer the questions that trunk of her car, she dialled an emergency number on her portable
follow : [2012-II] telephone and slipped it to her three-year-old daughter. Though
confused, the little girl saved the day. She told emergency
PASSAGE – I operators that her mother was locked in the trunk. Although she
“Sit down”, the Principal said, but Mr. Tagde continued to stand, didn't know where she was, she provided some important clues:
gaining courage from his own straight-backed stance, because she could see airplanes and the sky, according to the transcript
he was beginning to feel a little afraid now. of the emergency phone call. The operator called Tampa
The Principal looked unhappy. He disliked being forced to international airport police, who searched the top floor of the
perform this sort of an unpleasant task. airport parking garage where the car was parked. The operator
“I wish you would consider withdrawing this report”, he told the girl to honk, enabling the police to locate the car and free
said. Mrs. Graves.
“I am sorry, Sir, I cannot do that” Mr. Tagde said. He was 216. The clues provided by the little girl suggest that the car
pleased with his unwavering voice and uncompromising words. was parked
“It will be a very damaging report if put on record”. (a) outside but adjacent to the airport
“It is a factual report on very damaging conduct”. (b) by the main street of the city
“You are asking for the boy's expulsion from school. Don't (c) at the airport
you think the punishment is too harsh for a few boyish pranks” ? (d) in a parking garage in the side-lane
209. Mr. Tagde did not sit down because 217. The passage indicates that the girl was
(a) he was angry with the Principal (a) clever and brave
(b) he was in a defiant mood (b) had the maturity of an adult
(c) he did not like the student (c) coy and shy
(d) he was in a hurry (d) worried and excited
210. He would not withdraw the report because 218. The girl helped the police trace the car by
(a) he was arrogant and bitter (a) shouting loudly
(b) it was an accurate report (b) making too much din
(c) he wanted to create problems for the Principal (c) shrieking frightfully
(d) he wanted to show his authority (d) sounding the horn
211. The Principal was unhappy because he 219. Through the passage, the writer suggests the importance
(a) did not like to deal with an arrogant person and utility of
(b) was angry with Mr. Tagde
(c) could not enforce discipline in
(d) did not want to expel the boy
EBD_7367
180
B- Reading Comprehension
(a) education of little children PASSAGE – I
(b) parking garages Galileo desired to use his telescope to make more discoveries in
(c) portable telephones the heavens, but his instrument was too small. He made another
(d) airports and larger telescope which magnified eight times, and then
PASSAGE – IV another which magnified thirty times, and pointed it at the moon.
For days I trudged from one property-dealer to another, from one His heart leaped with joy, for he saw what no human eye had
"to-let" notice to another, with the estimated advance money ever before seen – ranges of mountains, deep hollows, and broad
tucked safely in the inner lining of my handbag, but in vain. At plains! He turned his telescope on the planets, and found they
one place they needed a couple, at another a young man, and at appcared with disks like the moon at a quarter full. He turned it
another they wished to know my employment status. And I realized on the Milky Way, and beheld innumerable tiny stars.
that I was a freak called the single woman and the job status 226. Galileo made several telescopes because
being nothing more than a freelance writer with hardly any (a) he needed all of them to explore the heavens
assignments in hand, only dreams of making it some day. So the (b) he wanted to compare the findings obtained from
dream-house remained far away, gradually turning into a fantasy. different telescopes
220. The author “trudged from one property dealer to another” (c) the earlier ones he made were not powerful enough
means that the author (d) only some of them could magnify the stars
(a) had to walk a lot of distance 227.When Galileo saw what no human eye had ever before seen
he
(b) had to do a brisk walk to save the time
(a) was overjoyed (b) was shocked
(c) visited several property-dealers without any success (c) felt humble (d) was very proud
(d) acquired a good knowledge about the property-dealers PASSAGE – II
of that area My father was passionate about two things: education and
221. According to the passage, the author was socialism. He was himself a born teacher. Indeed, he could never
(a) a novelist restrain himself form teaching, and as a small boy I was frequently
(b) an independent writer embarrassed by his desire to instruct everybody – people in
(c) a publisher of journals railway carriages, for instance – though I realized even then that
(d) an unemployed person it was an innocent desire, quite free from vanity. He was equally
222. Which of the following statements best reflects the ready to receive instruction. Education, to men of his generation
underlying tone of the passage ? and temperament, was something it has largely ceased to be
(a) People always let out their houses to well employed nowadays. It was the great golden gateway to the enchanted
persons only realms of the mind.
(b) People always let out their houses only to couples 228. The author wants us to know that his father
(c) Single jobless women find it difficult to rent a house (a) was a school teacher
(d) Women always dream of a house (b) was an educationist and socialist
PASSAGE – V (c) used to travel a lot
Martin had many little tricks highly entertaining to his son. On an (d) loved teaching
evening, returning from the market, he would buy a paper mask, 229. The author often felt embarrassed by the behaviour of his
the head of a hissing dragon. He would put it on and knock at the father because
door. On opening the door, the boy would be terrified for a moment, (a) he taught badly
but only for a moment, for he would soon remove it and the two (b) he taught even at odd places
would roll with laughter. Tom would, then, go out with the mask (c) he wanted to show off his learning
and knock at the door for his father to open. Martin had to act as (d) he lost self-control while teaching
if he was paralysed with fear. 230. To the generation of the writer’s father, education was
223. Martin played his little tricks because (a) an old fashioned enterprise
(a) he was very much interested in them (b) the result of good teaching
(b) he wanted to terrify his son (c) an exploration of the world of imagination
(c) his son got pleasure from them (d) one aspect of socialism
(d) it was his habit to make tricks 231. From the passage it is clear that the author
224. Which of the following statements is the most appropriate (a) loved and admired his father
description of the mask ? (b) disapproved his father’s love of teaching
(a) It is a mask looking like a dragon with long tail and (c) thought of him as vain
covering the whole body of Martin (d) considered his father’s education inadequate.
(b) It is mask looking like the head of a dragon with its PASSAGE – III
tongue hanging out We started looking on the ground for blood hair, or a drag mark
(c) It is mask looking like an animal with horns, wings and that would lead us to the deer killed by the tiger. We had proceeded
a pair of ferocious eyes emitting fire a hundered yards, examining every foot of the ground and going
(d) It is a mask looking like the head of a king cobra dead slow, when Mothi, just as I turned may head to look at him,
225. The father and son rolled with laughter after the started backwards, screaming as he did so. Then he whipped
(a) father put on the mask round and ran for dear life, beating the air with his hands as if
(b) opening of the door warding off a swarm of bees and continuing to scream as he ran.
(c) son saw the mask The sudden and piercing scream of a human being in a jungle
(d) father removed the mask where a moment before all has been silent is terrifying to hear.
DIRECTIONS (Qs. 226-243) : In this section there are six short Instinctively I knew what had happened. With his eyes fixed on
passages. Each passage is followed by questions based on the the ground, looking for the blood or hair of the kill, Mothi had
passage. Read each passage and answer the questions that failed to see where he was going, and had walked towards the
follow. [2013-I] tiger.
Reading Comprehension B-181
232. Mothi and the narrator were scanning the ground because PASSAGE – VI
(a) they were looking for the tiger The first day out we met our first rhino, two of them, and I had
(b) the forest was full of unpleasant surprises the fright of my life. The pair had got our scent before we spotted
(c) they were trying to discover the tiger’s footprints them, and being bad tempered beasts, they rushed towards
(d) they were looking for marks left by the tiger’s pray where they thought we were. Now it just happened that we were
233. Mothi began to scream when he about fifty yards to one side of where they expected to find us –
(a) was attacked by a swarm of bees which was just as well, for I must say I did not like their look. As
(b) was frightened by the sight of blood they thundered past, we crouched low and left them go. It did
(c) came face to face with the tiger not strike me as a good opportunity for rhino photography.
(d) stumbled on the tiger Anyhow. I was much too frightened to have been able to hold
234. In the context of the passage ‘kill means’ the camera steady.
(a) the act of killing 241. From the above passage it appears that rhinos
(b) an animal killed by the tiger (a) run away they see human beings
(c) a human being killed by the tiger (b) rush to attack when they smell human scent
(d) a wounded tiger (c) hide under the bushes at the sight of human beings
235. Before Mothi screamed, the jungle was (d) stand still if they are not attacked
(a) quiet (b) dark 242. When the author saw a rhino for the first time, he was
(c) noisy (d) terrifying (a) excited (b) frightened
PASSAGE – IV (c) charmed (d) surprised
When Ibbotson returned from Pauri, I told him of the leopard’s 243. The author could not take the photographs of the rhinos
habit of going down the road between Rudraprayag and Golabrai because
on an average once in every five days. convinced him that the (a) he was too far away from rhinos
only hope I now had of shooting the man-eater was by sitting (b) he was not carrying a good camera
over the road for ten nights; for, the leopard would be almost (c) it did not occur to him that he had a chance to do so
certain to use the road at least once during the period. Ibbotson (d) he did not like the look of rhinos
agreed to my plan reluetantly, for I had already sat up many
nights, and he was afraid that another ten nights on end would DIRECTIONS (Qs. 244-270): In this section. there are seven
be too much for me. short passages. After each passage, you will find several
236. Ibbotson was reluctant to agree to the narrator’s plan questions based on the passage, First, read a passage, and then
because he was afraid that answer the questions based on it. You are required to select
(a) the leopard would kill him your answers based on the contents of the passage and opinion
(b) the narrator would become very tired of the author only. [2013-II]
(c) the narrator would kill the leopard PASSAGE – I
(d) the leopard might not come A well-dressed young man entered a big textile shop one evening.
237. The narrator wanted to He was able to draw the attention of the salesmen who thought
(a) shoot the leopard (b) see the leopard him rich and likely to make heavy purchases. He was shown the
(c) capture the leopard (d) frighten the leopard superior varieties of suit lengths and sarees. But after casually
PASSAGE – V examining them, he kept moving to the next section where
Many poor farmers had been compelled to take up indigo readymade goods were being sold and further on to the hosiery
cultivation when the British settlers were given the right to section. By then, the salesmen had begun to doubt his intentions,
purchase and cultivate land in India. Many whites, therefore, and drew the attention of the manager. The manager asked him
either acquired lanel or advanced loans to poor farmers and what exactly he wanted and he replied that he wanted courteous
pressurised them to for sake the farming food grains and other treatment. He explained that he had come to the same shop in
cash crops for indigo cultivation. Indigo export to Europe was casual dress that morning and drawn little attention. His pride
lucrative for the British settlers who held a monopoly of this was hurt and he wanted to assert himself. He had come in good
business. Within a few years, most of the textile lands had dress only to get decent treatment. Not for getting any textiles.
undergone forcible indigo cultivation, resulting in a famine He left without making any purchase.
situation in Bengal. When the farmers declined to cultivate 244. The young man was well dressed because
indigo, they were tortured, jailed and even killed. (a) it was his habit to dress well
238. The poor farmers in Bengal took up indigo cultivation (b) it was his wedding day
because (c) he wanted to meet the manager of the shop
(a) the government encouraged them to do so (d) he wanted to impress the salesmen
(b) it was a money earning crop 245. The salesmen in the shop are described as people who pay
(c) they were forced to do so attention to
(d) this was the only crop that would grow in that region (a) only young men and women
239. British settlers bought land in Bengal in order to (b) pretty women
(a) introduce cultivation of cash crops in India (c) only rich customers
(b) cultivatie indigo (d) regular customers
(c) settle down in India 246. The young man moved away to the hosiery section because
(d) promote export business in Bengal he
240. Indigo export was profitable for the British settlers because (a) was not interested in purchasing anything now
(a) they had no competitors (b) did not like the readymade clothes
(b) the crop yield was good (c) wanted better clothes
(c) they could oppress the farmers (d) was restless
(d) the labour was cheap 247. The manager asked the young man what he wanted because
(a) he would give him exactly what he was looking for
EBD_7367
182
B- Reading Comprehension
(b) the salesman had drawn his attention to the (c) Nationalism as a cause of war
indifferent attitude of the young man (d) Evils of narrow and aggressive nationalism
(c) he thought they could do more business which him 255. From the passage which of the following statements can be
that way assumed to be most likely to be true ?
(d) he thought the visitor was dissatisfied (a) The author believes that nationalism is always a curse
248. The young man left without making purchases because he (b) He believes that it is possible for men to misuse
(a) did not have money religion
(b) could not find any item of his choice (c) He thinks that religion always leads man astray
(c) had come only to make a point about the indifferent (d) He pleads for a mix-up of religion and nationalism
attitude of the salesmen towards casually dressed PASSAGE IV
customers Brown and his men, huddling round a fire, ate the last of the food
(d) decided to come to make the purchases later on that Kassim had brought them that day, Cornelius sat among
PASSAGE – II them, half-asleep. Then one of the crew remembered that some
The prisoner awaited his chance. For three solid years he had tobacco had been left in the boat, and said he would go and fetch
schemed for this opportunity. Now that escape seemed so near at it. He didn't think there was any danger in going to the creek in
hand, those three years lost some of their monotony. But he the dark. He disappeared down the hillside, and a moment later he
would never forget the lashes, the close confinement, low diet was heard climbing into the boat and then climbing out again.
and worse still the mental strain of those black days. Suddenly 256. Consider the following statements :
the warden did what he had hoped. He stopped to unlock the 1. Brown and Cornelius sat round the fire.
lower padlock. With a dull thud he slumped forward with keys in 2. Comelius lay half-asleep at a little distance from the
his hands. Swiftly the prisoner seized his keys, unlocked the cell fire.
and ran into the courtyard. It took him four seconds to reach the 3. All the people sat round the fire.
rope-ladder secretly placed there by his accomplices, five more Which of the statements given above is/are correct ?
to clamber over the wall, and three more to jump into the waiting (a) 1 and 2 (b) 2 only
car to be whisked away to freedom. Even though he was guilty,
(c) 3 only (d) 1 and 3
the prisoner felt he had paid for his crime. For the man he robbed
257. One of them disappeared down the hill implies that
three years ago was still a millionaire.
249. For what crime had the prisoner been punished ? (a) the slope of the hill was slippery
(a) Murder (b) Arson (b) he fell from the edge of the hill
(c) Robbery (d) Kidnapping (c) there was suddenly a sea beside the hill
250. When had the crime been committed ? (d) he walked down the hill
(a) Just before the escape 258. "He didn't think...in the dark". This sentence actually implies
(b) Three years earlier that he
(c) Long ago (a) was bold and adventurous
(d) Dav earlier (b) was addicted to smoking
251. Who slumped forward with a dull thud ? (c) would face some trouble
(a) The millionaire (b) The warden (d) was the only person who knew where in the boat
(c) The prisoner (d) Prisoner's accomplice tobacco was
252. What did the prisoner suffer the most during imprisonment? 259. What does the word "huddling" imply ?
(a) Poor health (a) Moving around (b) Falling into a slumber
(b) Mental strain (c) Being close together (d) Merrymaking
(c) Physical torture PASSAGE V
(d) Absence from his family As soon as I saw the elephant I knew with perfect certainty that
PASSAGE III I ought not to shoot him. It is a serious matter to shoot a working
Nationalism is only a curse when it becomes narrow and fanatical elephant – it is comparable to destroying a huge and costly piece
Like so many other things available to man, say, religion, it can of machinery – and obviously one ought not to do it if it can
easily lead men astray. Nationalism can lead people into thinking possibly be avoided. And at that distance, peacefully eating, the
only of themselves, of their own struggles of their own misery. It elephant looked no more dangerous than a cow.
can also cause a nation to become suspicious and fearful of its 260. The writer was against shooting the elephant because
neighbours to look upon itself as superior, and to become (a) he suspected it to be a wild one and was afraid of it
aggressive and it is when nationalism impels a state to become (b) his heart was full of compassion for animals
expansionist and seek domination over others that it becomes a (c) he was certain that the elephant was innocent
positive curse and harmful internationally. (d) it would amount to avoidable waste of useful property
253. From the passage, which of the following statements most 261. The author compares the elephant to a costly machine
correctly reflects the opinion of the author ? because
(a) Nationalism makes people self centered and self- (a) Ivory is very expensive
concelted (b) it can do as much work as an expensive machine
(b) It helps a nation to become superior to other nations (c) elephants look like big machines
(c) It regulates international relationships (d) elephants and machines have similar prices
(d) It helps a nation to expand its territories and become 262. The elephant looked no more dangerous than a cow because
powerful (a) it was quietly doing its work
254. Which of the following phrases most correctly suggests (b) unlike lions, it is a vegetarian animal
the central theme of the passage ? (c) its tusks resemble the cow's horns
(a) Nationalism and religion (d) cows can be very dangerous sometimes
(b) Nationalism as an inspiration for development
Reading Comprehension B-183
PASSAGE VI DIRECTIONS (Qs. 271-285): Read the following passages and
I was very fond of the old soldier in little town. He had only one answer the items that follow.
leg, having lost the other somewhere in Assam in 1942. He used
to tell me about his adventures. He told me that he had run away PASSAGE-l
from home to join the army. He had experienced his first battle in Those responsible for teaching young people have resorted to a
the Libyan desert. Out of his dozens of war stories, the one I liked variety of means to make their pupils learn. The earliest of these
best was the one of his escape from a Japanese prison-of-war was the threat of punishment. This meant that the pupil who was
camp in Burma. He told me again and again how he walked two slow, careless or inattentive risked either physical chastisement
hundred miles in two weeks. On the way he was bitten on the toe or the loss of some expected privilege. Learning was thus
by a poisonous snake and he had to cut off part of the toe in associated with fear. At a later period, pupils were encouraged to
order to survive. But by the time he got to an Indian camp the learn in the hope of some kind of reward. This often took the form
wound had turned septic and the leg had to be amputated. He is, of marks awarded for work done and sometimes of prizes given at
however, quite contented with his lot. the end of the year to the best scholar. Such a system appealed to
263. The author was very fond of the old soldier because the competitive spirit, but was just as depressing as the older
(a) He had lost one of his legs in war system for the slow pupil.
(b) he used to tell the author about his adventures In the nineteenth century sprang up a new type of teacher,
(c) he was contented with his lot convinced that learning was worthwhile for its own sake and that
(d) he had been to many countries the young pupil's principal stimulus should neither be anxiety to
264. Why did the old soldier repeatedly tell that he walked two avoid a penalty nor ambition to win a reward, but sheer desire to
hundred miles ? learn. Interest, direct or indirect, became the keyword of
(a) He ran away from home to join the army instruction. [2014-I]
(b) He had to cross the Libyan desert 271. The educational system which caused fear in the pupil's
(c) He had to escape from a prison-of-war camp mind was based on :
(d) He was a strong soldier (a) rewards based
265. The story of the old soldier that the author liked most was (b) labour
that about (c) punishment
(a) his running away from home to join the army (d) competition
(b) his first battle in the Libyan desert 272. The system based on rewards satisfied all except :
(c) the loss of his leg in Assam (a) the slow pupil
(d) his escape from a Japanese prison-of-war camp (b) the very intelligent pupil
266. The old soldier, according to the author, was (c) the laborious pupil
(a) unhappy about his life (d) the casual pupil
(b) satisfied with his lot 273. The system which appealed to the competitive spirit in the
(c) angry about his fate pupils was largely based on:
(d) disgusted with his misfortune (a) punishment (b) marks
267. The soldier's leg had to be amputated because (c) chastisement (d) cash prizes
(a) he had walked two hundred miles in two weeks PASSAGE-II
(b) he was wounded in war On a surface which is free from obstacles, such as a
(c) the wounded toe turned septic clear road or a path, only two or three species of snakes can
(d) he was shot in the leg while escaping from the hope to catch up with a human being, even if they are foolish
Japanese camp to try. A snake seems to move very fast but its movements are
PASSAGE VII deceptive. Inspite of the swift, wave-like motions of its body, the
A large number of people had come to attend the meeting to be snake crawls along the ground at no more than the speed of
addressed by the gifted speaker. The organizers had a difficult man's walk. It may, however, have an advantage inside a jungle,
time keeping the assembled people quiet as the meeting did not where the progress of a man is obstructed by thorny bushes. But
commence at the scheduled time. After some time the people lost in such places, the footsteps of a man are usually more than
their patience and began to shout and heckle. The organizers had enough to warn snakes to keep away; Although they have no
great difficulty in assuaging the anger of the crowd when they cars of the usual kind, they can feel slight vibrations of the ground
were forced to cancel the meeting as the speaker had to be through their bodies, and thus get an early warning of danger.
hospitalized due to sudden illness. [2014-I]
268. What was the actual reason for the organizers to have a 274. The snake has an advantage over men inside a jungle,
difficult time ? because there:
(a) a large number of people had come to the meeting (a) it can crawl faster.
(b) the organizers could not make proper arrangements (b) it gets advance warning.
(c) the meeting could not be started in time (c) man's movement is obstructed.
(d) the speaker was ill (d) it is dark inside a jungle.
269. What does the word "assuaging" imply ? 275. What helps the snakes to receive advance warning is their
(a) accepting (b) tolerating sensitivity to:
(c) reducing (d) removing (a) obstacles in the path.
270. Further delay resulted in the people (b) smell of other beings.
(a) leaving the place (c) sounds made by other beings.
(b) fighting with the organizers (d) movements of other beings.
(c) making noise PASSAGE-III
(d) making the speaker ill This rule of always trying to do things as well as one can do them
has an important bearing upon the problem of ambition. No man
EBD_7367
184
B- Reading Comprehension
or woman should be without ambition, which is the inspiration of a bus or train tend to fall when a sudden start is made? It is
activity. But if one allows ambition to drive one to attempt things because his feet move on, but his head stays still. [2014-I]
which are beyond one's own personal capacity, then unhappiness 281. The passage says that early instruments for measuring
will result. If one imagines that one can do everything better than earthquakes were:
other people, then envy and jealousy, those twin monsters, will (a) faulty in design
come to sadden one's days. But if one concentrates one's attention (b) expensive
upon developing one's own special capacities, the things one is (c) not sturdy
best at, then one does not worry over much if other people are (d) not sensitive enough
more successful. [2014-I] 282. Why was it necessary to invent instruments to observe an
276. Which one of the following alternatives brings out the eqarthquake ?
meaning of 'to have a bearing upon' clearly? (a) Because an earthquake comes like a thief in the night.
(a) to have an effect on (b) To make people alert about earthquakes during their
(b) to carry the weight on oneself conscious as well as unconscious hours.
(c) to put up with (c) To prove that we are technically advanced.
(d) to decrease friction (d) To experiment with the control of man over nature.
277. Which one of 'the following statements is correct? 283. A simple device which consisted of rods that stood up on
(a) There is a close relationship between ambition and end like ninepins was replaced by a more sophisticated one
activity. because it failed :
(b) Ambition and activity belong to two different areas. (a) to measure a gentle earthquake.
(c) Ambition is useless. (b) to measure a severe earthquake.
(d) Activity is responsible for ambition. (c) to record the direction of the earthquake.
278. The statement 'if one allows ambition to drive one to attempt (d) to record the facts with a pen on paper.
things which are beyond one's own personal capacity, then 284. The everyday observation referred to in the passage relates to:
unhappiness will result, means that: (a) a moving bus or train.
(b) the sudden start of a bus.
(a) One must always try to do less than than one’s
(c) the tendency of a standing person to fall when a bus
capacity.
or train moves suddenly.
(b) One must always try to do more than one's capacity. (d) people standing in a bus or train.
(c) Ambition must be consistent with one's capacity. 285. The early seismometers adopted the idea that in order to
(d) There should be no ambition at all. record the earthquake, it is :
279. Which one of the following statements best reflects the (a) the pen that should move just as it moves when we
underlying tone of the passage ? write on paper.
(a) One must do everything as well as one can. (b) the pen that should stay still and the paper should
(b) One must try to be better than others. move.
(c) One must continuously worry about others. (c) both pen and paper that should move.
(d) One must try beyond one's capacity to get results. (d) neither pen nor paper that should move.
280. Which one of the following statements can be assumed to
be true ? DIRECTIONS (Qs. 286-301): In this section, there are four short
(a) It is good to imagine oneself better than others. passages. After each passage, you will find a few questions each
(b) One should not imagine oneself always to be better based on what is stated or implied in the passage. First read a
than others. passage and then answer the questions following that passage.
(c) All persons have equal capacity. PASSAGE-I
(d) One should have more ambition than others. During the summer I was introduced to the game of cricket, and I
PASSAGE-IV felt my inherent foreignness for the first time. The ball is far too
An earthquake comes like a thief in the night, without hard for my taste. Even during my last games at the school, angry
warning. It was necessary, therefore, to invent instruments that
spectators would shout, "Butter fingers !" But I smiled. Everyone
neither slumbered nor slept. Some devices were quite simple.
One, for instance, consisted of rods of various lengths and knew in their hearts that 'I was going to drop the ball anyway,
thicknesses which would stand up on end like ninepins. When a and nobody expected me to be able to play the game. [2014-II]
shock came it shook the rigid table upon which these stood. If it 286. The author first played cricket
were gentle, only the more unstable rods fell. If it were severe, (a) as a child in his own country
they all fell. Thus the rods by falling and by the direction in which (b) when he was a school boy
they fell, recorded for the slumbering scientist, the strength of a (c) when he was a tourist
shock that was too weak to waken him and the direction from (d) when he returned home after his studies
which it came. But, instruments far more delicate than that were 287. "felt my inherent foreignness" means
needed if any really serious advance was to be made. The ideal to (a) felt very strange
be aimed at was to devise an instrument that could record with a (b) felt very interested and excited
pen on paper the movements, of the ground or of the table, as the (c) enjoyed learning new games
quake passed by. While I write my pen moves but the paper (d) fely my superiority over others
keeps still. With practice, no doubt, I could, in time, learn to write 288. Spectators would shout "Butter fingers" when the author
by holding the pen still while the paper moved. That sounds a was playing because
silly suggestion, but that was precisely the idea adopted in some (a) he liked butter
of the early instruments (seismometers) for recording earthquake (b) his fingers were like those of a lady
waves. But when table, penholder and paper are all moving how (c) he often dropped the ball
is it possible to write legibly? The key to a solution of that problem (d) he was very good at the game
lay in an everyday observation. Why does a person standing in
Reading Comprehension B-185
289. 'Spectator' means 296. Exchange of goods and services becomes necessary
(a) glasses (b) onlooker because
(c) watchman (d) player (a) man is a social being
PASSAGE-II (b) reciprocity is the law of life
How can you improve your reading speed? By taking off the (c) trade and commerce are means of progress
brakes. You wouldn't think of driving a car with the brake on. Yet (d) we cannot produce everything we need ourselves.
as a reader you probably have several brakes slowing you down. PASSAGE - IV
One very common brake is regressing—looking back every now Soil scientists have shown that the soil teems with millions of
and then at something already read. It is like stepping backwards living things, many of them useful, others harmful. The living
every few metres as you walk-hardly the way to move ahead things which are useful include earthworms and various kinds of
quickly. Regression may arise from a lack of confidence, bacteria. Earthworms loosen the soil and so enable air and water
vocabulary deficiency, or actually missing a word or phrase. It to enter it. Bacteria, which are microscopic living things break
makes a long sentence seem even more complex as the eyes down dead plants and animals and make humus, or take nitrogen
frequently regress. Eye movement photographs of 12,000 readers from the air and change it into substances that plants use. The
in America showed that university students regress an average living things that do harm include other bacteria and fungi which
of 15 times in reading only 100 words. The average student of cause diseases. Other harmful things are pests such as wire worms
class four was found to look back 20 times. In short, regression which feed on the roots of grass and other plants. While the
consumes one-sixth of your precious reading time. Release this farmer can usually keep weeds in check by careful cultivation,
brake and enjoy a spurt in reading speed. [2014-II] this alone may not protect his crops from insects, pests and
290. In the context of the passage, what does 'regression' mean ? diseases. Nowadays, however, he is much better able to control
(a) Lack of desire to improve the reading speed these enemies. He may plant specially resistant types of seeds or
(b) Looking back at what is already read he may keep the pests and diseases-in check with chemicals.
(c) Lack of proper understanding of what one reads With better seeds farmers have been able to increase their crop
(d) Comparing the reading speed of school and university yields. They can grow crops that ripen more quickly and have a
students stronger resistance to disease, frost or drought. [2014-II]
291. In order to be a good reader you should 297. Scientists who study soil believe that
(a) regress whenever necessary (a) all insects and bacteria are harmful
(b) be like a careful driver (b) only microscopic living things are useful
(c) not look back frequently while reading (c) only earthworms are useful
(d) test your vocabulary frequently (d) not all worms and bacteria are harmful
292. According to the author reading with regression is like 298. The living things that do harm
(a) driving with poor quality brakes (a) break down plants and animals
(b) stepping backwards while walking (b) use up the nitrogen from the air
(c) using several brakes in order to slow down (c) cause disease in the plants
(d) making sudden spurts in reading speed (d) loosen up the soil from air and water
299. Farmers are always careful
PASSAGE-III
(a) to control insects and fungi that attack plants
Even in the most primitive societies the great majority of people (b) to encourage pests in the soil
satisfy a large part of their material needs by exchanging goods (c) to eliminate all bacteria from the soil
and services. Very few people indeed can make for themselves (d) to foster all kinds of worms in the earth
everything they need – all their food, their clothes, their housing, 300. Nowadays it is possible to reduce the loss caused by pests
their tools. Ever since men started living in communities,'they and harmful bacteria
have been satisfying their needs by means of specialization and (a) with the use of chemical fertilisers
exchange; increasingly each individual has concentrated on what (b) throught the development of resistant seeds
he can do best, and has produced more of the special goods or (c) by using weeds as killers
services in which he has concentrated, than he can consume (d) by controlling earthworms
himself. The surplus he has exchanged with other members of the 301. The farmers today can also select seeds
community, acquiring, in exchange the things he needs that others (a) of slow ripening variety
have produced. [2014-II] (b) resistant to frost and drought
293. Very few people can satisfy their needs today by (c) for economy in costs
(a) providing things for themselves (d) of lower resistance to disease
(b) exchanging goods and services DIRECTIONS (QS. 302-324) : In this section you have five short
(c) concentrating on what they can do best passages. After each passage, you will find some questions based
(d) individual specialization on the passage. First, read a passage and answer the questions
294. Exchange of goods becomes possible only when based on it. You are required to select your answers based on
(a) there is no specialization the contents of the passage and opinion of the author only.
(b) goods are produced in surplus [2015-I]
(c) primitive societies become modern PASSAGE-I
(d) individuals make things for themselves
A little man beside me was turning over the pages of a magazine
295. Specialization and exchange began when men started
quickly and nervously. Opposite me there was a young mother
(a) big industries who was trying to restrain her son from making a noise. The boy
(b) concentrating on their work had obviously grown weary of waiting. He had placed an ashtray
(c) producing things for individual use on the floor and was making aeroplane-noises as he waved a
(d) living in communities pencil in his hands. Near him, an old man was fast asleep, snoring
EBD_7367
B-186 Reading Comprehension
quickly to himself and the boy's mother was afraid that sooner or (c) the tigress actually covered the distance within the
later her son would wake the gentleman up. half-hour
302. The noise was made by (d) there was a possibility of the tigress covering the
(a) the old man (b) the aeroplane distance within the half - hour
(c) the little man (d) the boy 311. The author says, “Some of us would not reach camp” because
303. The person who was the least disturbed was the (a) it was two miles away
(a) observer (b) son (b) the tigress would kill some of them
(c) old man (d) little man (c) the path is not suitable for walking
304. The factor common to all the people was that they were all (d) the ground was scattered over with great rocks
(a) watching a film 312. The author found it difficult to decide the question because.
(b) waiting for something (a) he was afraid
(c) looking at the little boy's playfulness (b) the tigress was only a mile away
(d) reading magazine (c) the ground between them was densely wooded
305. Among those present the one who appeared to be the most (d) there was uncertainty about the reaction of the tigress
bored was the to his call
(a) child (b) little man 313. The time available to the author for shooting the tigress
(c) old man (d) mother
was
PASSAGE - II
A man has two blacksmiths for his neighbours. Their names were (a) the whole day
Pengu and Shengu. The man was greatly troubled by the noise of (b) one night
their hammers. He decided to talk to them. The next day he called (c) a few hours
both of them and offered `100 each, if they found new huts for (d) thirty minutes
them selves. They took the money and agreed to find new huts 314. When the author says ‘all would be well’, he means
for themselves. The next morning he woke up again to the sound (a) that they would be able to hide themselves in the
of their hammers. He went out to see why the blacksmiths hadn't heavy jungle
found new huts and he discovered that Pengu and Shengu had (b) that the tigress would run away to the deep ravines
kept their promise. They had exchanged their huts. (c) that they would be able to shoot her down without
306. The man was troubled because difficulty
(a) the blacksmiths always fought with each others (d) that they would be able to return in daylight
(b) the blacksmiths' hammers made a lot of noise PASSAGE - IV
(c) he was afraid of blacksmiths After lunch, I felt at a loose end and roamed about the little
(d) the blacksmiths did not do their work properly flat. It suited us well enough when mother was with me, but now
307. The man gave them money because I was by myself it was too large and I'd moved the dining room
(a) the blacksmiths were poor table into my bedroom. That was now the only room I used ; it
(b) the blacksmiths had asked him for money had all the furniture I needed; a brass bedstead, a dressing table,
(c) he did not want them to make a noise some cane chairs whose seats had more or less caved in, a
(d) he wanted them to find new huts wardrobe with a tarnished mirror. The rest of the flat was never
308. The man went out of his house because used, so I didn't trouble to look after it. [2015-I]
(a) he wanted to fight with the blacksmiths 315. The flat did not really suit him any more because
(b) he wanted to ask the blacksmiths to stop the noise (a) the rooms were too small
(c) he wanted to find out why they hadn't found new (b) he was living on his own now
huts (c) his mother needed too much rooms
(d) he wanted his money back from the blacksmiths (d) the flat itself was too little
309. The man came to know that 316. He did not look after the rest of the flat because
(a) the blacksmiths were not in their huts (a) he did not use it
(b) the blacksmiths had exchanged huts (b) the bedroom was much too large
(c) the blacksmiths were going away (c) he needed only the brass bedstead
(d) the blacksmiths had not kept their promise (d) he had too much furniture
PASSAGE - III 317. “.....now I was by myself it was too large”.
The tigress was a mile away and the ground between her The word it here refers to
and us was densely wooded. scattered over with great rocks and (a) the dining room table
cut up by a number of deep ravines, but she could cover the (b) the dining room
distance well within the half - hour — if she wanted to. The (c) the bedroom
question I had to decide was, whether or not I should try to call (d) the flat
her. If I called and she heard me, and came while it was still daylight 318. From the passage we learn that the writer was
and gave me a chance to shoot her, all would be well; on the other (a) scared of living alone in the flat
hand, if she came and did not give me a shot, some of us would (b) dissatisfied with the flat
not reach camp, for we had nearly two miles to go and the path (c) satisfied with the space in his bedroom
the whole way ran through heavy jungle. [2015-I] (d) an eccentric person
310. According to the author 319. “After lunch I felt at a loose end” means
(a) the tigress wanted to cover the distance within the (a) he had nothing specific to do
half-hour (b) had a rope with a loose end
(b) the tigress did not wish to cover the distance within (c) had much work to do
the-half-hour (d) had a feeling of anxiety
Reading Comprehension B-187
PASSAGE-V (b) a famous painting
The overwhelming vote given by the greater part of the public (c) the artist's mistress
has so far been in favour of films which pass the time easily and (d) an art technique
satisfy that part of our imagination which depends on the more 327. The truth about the "Mona Lisa" is that it is a study in
obvious kind of daydreams. We make up for what we secretly (a) feminine psychology (b) facial expression
regard as our deficiencies by watching the stimulating adventures (c) feminine form (d) modelling
of the other people who are stronger, more effective, or more 328. The painter was able to produce that strange smile on Mona
beautiful than we are. The conventional stars act out our Lisa's face by
daydreams for us in a constant succession of exciting situations (a) delicate changes on the surface of cheeks below the
set in the open spaces, in the jungles or in the underworld of eyes
great cities which abounds in crime and violence. We would not (b) using bright colours
dare to be in such situations but the situations are very exciting (c) using a painting knife
to watch since our youth is being spent in day-to-day routine of (d) looking constantly at a smiling model while painting.
school, office or home. [2015-I]
229. The author of the above passage has examined 'Mona Lisa'
320. According to the passage, most of us prefer films which
(a) overwhelm our imagination from
(b) depict our times (a) an idealistic angle
(c) fulfil our secret wishes (b) an imaginary point of View
(d) appeal to our reason (c) a purely artistic angle
321. By watching thrilling adventures in films we make up for (d) a scientific and realistic standpoint
(a) the effectiveness of our desires PASSAGE-II
(b) the shortcomings in our life It is possible to give wedding presents, birthday and Christmas
(c) the stimulation of our everyday life presents, without any thought of affection at all, they can be
(d) the influence which we don't have ordered by postcard; but the unbirthday present demands the
322. Film stars present situations nicest care. It is therefore the best of all, and it is the only kind to
(a) which are familiar to us, the city dwellers which the golden rule of present-giving imperatively applies -
(b) which we have seen only in jungles
the golden rule which insists that you must never give to another
(c) which we meet everyday at work
(d) which excite us person anything that you would not rather keep: nothing that
323. Whether we admit it to ourselves or not, we are aware that does not cost you a pang to part from. It would be better if this
(a) we are weak and plain rule governed the choice also of those other three varieties of
(b) we are both powerful and handsome gifts, but they can be less exacting.
(c) we are as strong as film heroes 330. The author says that wedding, birthday and Christmas
(d) we are more beautiful than film stars Presents
324. The daily life of students, office-goers and housewives is (a) are always indicators of the giver's affection.
(a) full of new adventures (b) may not always be given with any thought of affection.
(b) the same dull repetition (c) are given only to flatter the recipient
(c) stimulating to their imagination (d) are given only to fulfil an obligation
(d) very exciting to them 331. 'They can be ordered by postcard' means that
(a) the present may only be a postcard
(b) the present would be an expensive one
DIRECTIONS (Qs. 325-344): Read the following passages and (c) the choice does not involve much care
at the end of each passage, you will find some questions based (d) the present would not be worth giving
on the passage. First, read a passage and answer the questions 332. The 'unbirthday' present is the best of all because
based on it. You are required to select your answers based on (a) it cannot be ordered by postcard
the contents of the passage and opinion of the author only. (b) it means giving expensive presents
[2015-II] (c) its choice needs the utmost care
(d) other occasions are better than birthdays for giving
PASSAGE-I
presents
Much rhapsodical nonsense has been written about the "Mona 333. A 'golden rule' is a rule which
Lisa" and her enigmatic smile, and there have been endless (a) brings profit (b) is very important
speculations as to her character and the meaning of her (c) is very difficult (d) is very easy
expression. It is all beside the mark. The truth is that the "Mona 334. The writer is of the view that one should give a present that
Lisa" is a study of modeling. Leonardo da Vinci had discovered (a) one would like to possess oneself
that the expression of smiling is much more a matter of modeling (b) one would like to get rid of
of the cheek and of the forms below the eye than of the change in (c) cannot be ordered by mail
the line of the lips. It interested him to produce a smile wholly by (d) is highly expensive and attractive
these delicate changes of surface; hence the mysterious
PASSAGE-III
expression.
325. The word rhapsodical as used in the passage means People project their mental processes into their handwriting. They
(a) plain (b) unreadable subconsciously shape and organise their letters, words and lines
(c) enthusiastic (d) uniformed in ways that directly reflect their personalities. This explains why
326. "Mona Lisa" is the name of no two handwritings are - or even can be - alike; the medium is
(a) a beautiful woman who made history in ancient Rome just too personal. Everyday observation confirms the link between
EBD_7367
188
B- Reading Comprehension

handwriting and personality, at least in an elementary way. Precise 342. When iron is heated to about 1300 degree centigrade
people construct their words with care, slowly and exactly; (a) flames turn from white to blue
dynamic people dash them off. Flamboyant people boldly cover (b) chemical reaction starts
half a page with a few words and a signature, whose size fittingly (c) oxide film is found on its surfaces
reflects their expansive sense of self. Most of us have made such (d) it turns into steel
observations. But it takes a practiced eye to discern the scores of 343. The flux is used to
variations and interpret the subtle interplay of forces at work in (a) make the metal plastic
any given handwriting. In fact in Europe, handwriting analysis (b) cool the heated metal
known as graphology, now enjoys scientific acceptance and (c) cover up any dirt
common use. (d) dissolve oxide and other impurities
335. If you are a showy and colourful person, your-handwriting 344. For fairly thick bars of metals
is likely to be (a) a vee shaped weld should be used
(a) neat and slow (b) dashing and careless (b) ordinary butt weld should be used
(c) bold and large (d) legible but small (c) a number of different types of weld may be used
336. Graphology is (d) a pressure weld may be used.
(a) the study of graphs
(b) the analysis of handwriting DIRECTIONS (Qs. 345-365) : In this section you have six short
passages after each passage, you will find some questions based
(c) a special branch of phonetics
on the passge. First, read a passage and answer the questions
(d) a graphical description of handwriting based on it. You are required to select your answers based on the
337. Handwriting analysis is contents of the passage and opinion of the author only [2016-I]
(a) not useful to us
(b) an elementary study PASSAGE-1
(c) an imprecise science To avoid the various foolish opinions to which mankind is prone,
(d) a means of studying personality no superhuman brain is required. A few simple rules will keep
338. According to the author, people are: you. not from all errors, but from silly errors.
(a) not conscious of what they write If the matter is one that can be settled by observation, make
(b) aggressive in the nature of their writing the observation yourself. Aristotle could have avoided the
(c) not conscious of the way they write mistake of thinking that women have fewer teeth than men. by
(d) not used to personal writing the simple device of asking Mrs. Aristotle to keep her mouth
339. The fact that handwriting is related to personality open while he counted. Thinking that you know, when in fact
(a) has been noticed by most people you do not, is a bad mistake to which we are all prone. I believe
(b) is appreciated by dynamic people myself that hedgehogs eat black beetles, because I have been
(c) is restricted to persons who write carefully told that they do, but if I were writing a book on the habits of
(d) is known only to graphologists hedgehogs, I should not commit myself until I had seen one
enjoying this diet. Aristotle, however, was less cautious. Ancient
PASSAGE-IV and medieval writers knew all about unicorns and salamanders:
The simplest method of welding two pieces of metal together is not one of them thought it necessary to avoid dogmatic
known as pressure welding. The ends of metal are heated to a statements about them because he had never seen one of them .
white heat - for iron, the welding temperature should be about 345. The writer believes that
l300°C - in a flame. At this temperature the metal becomes plastic. (a) most people could avoid making foolish mistakes if
The ends are then pressed or hammered together, and the joint is they were clever
smoothed off. Care must be taken to ensure that the surfaces are (b) through observation we could avoid making many
thoroughly clean first, for dirt will weaken the weld. Moreover, mistakes
the heating of iron or steel to a high temperature cause oxidation, (c) Aristotle made many mistakes because he was not
and a film of oxide is formed on the heated surfaces. For this observant
reason, a flux is applied to the heated metal. At welding heat, the (d) All errors are caused by our own error in thinking
flux melts, and the oxide particles are dissolved in it together with 346. With reference to the passage, which one of the following
any other impurities which may be present. The metal surfaces is the correct statement ?
are pressed together, and the flux is squeezed out from the centre (a) Aristotle was able to avoid the mistake of thinking
of the weld. A number of different types of weld may be used, but that women have fewer teeth than men
for fairly thick bars of metals, a vee-shaped weld should normally (b) Aristotle thought women have fewer teeth than men
be employed. It is rather stronger than the ordinary butt weld. (c) Aristotle proved that women have fewer teeth bv
340. The simplest way of welding two pieces of Metal together is counting his wife's teeth
(a) heating the metal (d) Aristotle may have thought that women have fewer
(b) holding it in a flame teeth because he never had a wife
(c) coating the metal with plastic 347. The writer says that if he was writing a book on hedgehogs
(d) hammering heated pieces (a) he would maintain that they eat black beetles because
he had been told so
341. Unless the surfaces are cleaned first (b) he would first observe their eating habits
(a) the metal will not take white heat (c) he would think it unnecessary to verify that they are
(b) the resulting weld will be weak black beetles
(c) the joint will be rough (d) he would make the statement that they ate black beetles
(d) the metal will be less plastic and later verify it
Reading Comprehension B-189
348. The writer is of the opinion that PASSAGE-4
(a) unicorns and salamanders were observed by ancient We are tempted to assume that technological progress is real
and medieval writers but were unknown to modern progress and that material success is the criterion of civilization.
writers. If the Eastern people become fascinated by machines and
(b) ancient and medieval writers wrote authoritatively techniques and use them, as Western nations do, to build huge
about unicorns and salamanders though they had never industrial organizations and large military establishments, they
seen them will get involved in power politics and drift into the danger of
(c) unicorns and salamanders do not exist death. Scientific and technological civilization brings great
(d) only those who had observed the habits of unicorns opportunities and great rewards but also great risks and
and salamanders wrote about them temptations. Science and technology are neither good nor bad.
349. A 'dogmatic statement' in the context means a statement They are not to be tabooed but tamed and assigned their proper
which is place. They become dangerous only if they become idols.
(a) convincing (b) proved 355. According to the author, people think that real progress lies in
(c) unquestionable (d) doubtful (a) material success and technological growth
PASSAGE-2 (b) imitating Western nations
Since I had nothing better to do. I decided to go to the market to (c) having large industries and politics power
buy a few handkerchiefs, the old ones had done vanishing trick. (d) taking risks and facing temptations
On the way I met an old friend of mine and I took him to a nearby 356. According to the author, science and technology should
restaurant for tea and snacks. Afterwards I went to the shop and be
(a) tabooed and eliminated from life
selected a dozen handkerchiefs. I pulled out my purse to make
(b) used in a controlled and careful manner
the payment, and discovered that it was empty: I then realized
(c) encouraged and liberally used
that it was not my purse, it was a different purse altogether. How
(d) made compulsory in education
that happened is still a source of wonder to me and I refuse to
357. From the passage one gathers that the Eastern people must
believe that it was the work of my good old friend, for it was his
(a) appreciate scientific achievements
purse that I held in my hand.
(b) build huge industrial organizations
350. The man could not buy the handkerchiefs because
(c) avoid being controlled by machines and techniques
(a) he did not like the handkerchiefs
of industrial production
(b) his friend did not allow him to buy them (d) be fascinated by machines
(c) the shop did not have any handkerchiefs 358. According to the author, science and technology are
(d) he had no money in the purse (a) totally harmless
351. When he tried to take out the purse, he discovered that (b) extremely dangerous
(a) it was not there (c) to be treated as idols
(b) it was lost (d) useful, if they are not worshipped blindly
(c) it was a new purse
(d) it was his friend's purse PASSAGE-5
PASSAGE-3 It is not luck but labour that makes men. Luck, says an American
writer, is ever waiting for something to turn up; labour with keen
A profound terror, increased still by the darkness, the silence eyes and strong will always turns up something, Luck lies in bed
and his waking images, froze his heart within him. He almost felt and wishes the postman would bring him news of a legacy: labour
his hair stand on end, when by straining his eyes to their utmost, turns out at six and with busy pen and ringing hammer lays the
he perceived through the shadows two faint yellow lights. At foundation of competence. Luck whines, labour watches. Luck
first he attributed these gradually to distinguish the objects around relies on chance; labour on character. Luck slips downwards to
him in the cave. and he beheld a huge animal lying but two steps self-indulgence; labour strides upwards and aspires to
from him. independence. The conviction, therefore, is extending that
352. The opening of the passage suggests that diligence is the mother of good luck; in other words, that a man's
(a) darkness, silence and waking images added to his success in life will be proportionate to his efforts, to his industry,
already being in profound terror to his attention to small things.
(b) a profound terror increased the waking images in his 359. Which one of the following statements sums up the meaning
frozen heart of the passage ?
(c) the person was frightened by darkness and silence (a) Luck waits without exertion but labour exerts without
(d) a profound terror was caused in him by the silence and waiting
darkness of the night (b) Luck waits and complains without working while labour
353. When he perceived through the shadows two faint lights. achieves success although it complains
(a) he experienced a great strain (c) Luck often ends in defeat but labour produces luck
(b) he felt his hair stand upright (d) Luck is self-indulgent but labour is selfless
(c) his eyes felt strained to their utmost 360. Which one of the following statements is true about the
(d) his pupils dilated passage ?
354. The person in the story (a) Luck is necessary for success
(a) imagined that he saw an animal (b) Success depends on hard work and attention to details
(b) could not recognize the animal (c) Expectation of good luck always meets with
(c) saw the animal by chance disappointment
(d) expected to see the animal (d) Success is exactly proportionate to hard work only
EBD_7367
190
B- Reading Comprehension

361. Labour turns out at six and with busy pen and ringing milestones which mankind passes on its way to something else.
hammer lays the foundation fo competence. This statement But with works of art it is not so. The place which they occupy in
means the estimation of succeeding ages and the power which they
(a) hard work of all kinds makes people efficient and skilled exercise over men's spirits are as great as they were in the age
(b) the labour lays the foundation of the building which produced them; indeed, their power tends to increase with
(c) the writer and the labourer are the true eyes of the time, as they came to be better understood.
society 366. The power of art can be judged through
(d) there is no worker who works so hard as the labourer (a) its influence of a few individuals.
who begins his day at six in the morning (b) its influence on the people over the years.
(c) the greatness of great artists.
PASSAGE-6 (d) the opinions of great thinkers.
The avowed purpose of the exact sciences is to establish complete 367. The statement 'Newton's theory of gravitation has been
intellectual control over experience in terms of precise rules which superseded by Einstein's theory of relativity" suggests that
can be formally set out and empirically tested. Could that ideal be (a) the theory of relativity has nothing to do with the
fully achieved, all truth and all error could henceforth be ascribed theory of gravitation.
to an exact theory of the universe, while we who accept this (b) the theory of relativity is new in comparison to the
theory would be relieved of any occasion for exercising our theory of gravitation.
personal judgement. We should only have to follow the rules (c) the theory of relativity is an improvement over the
faithfully. Classical mechanics approaches this ideal so closely theory of gravitation.
that it is often thought to have achieved it. But this leaves out of (d) the theory of relativity has suppressed the theory of
account the element of personal judgement involved in applying gravitation.
368. The achievements of generals, politicians and statement
the formulae of mechanics to the facts of experience.
have been compared to milestones by the author because
362. The purpose of the exact sciences is to (a) they are inscribed on the milestones.
(a) form opinions about our experience (b) they have contemporary relevance.
(b) formulate principles which will help us to exercise our (c) they have topical and historical interest.
personal judgement (d) they are strong and lasting stones.
(c) assert our intellectual superiority 369. How is a work of art different from the work of a scientist?
(d) make formal and testable rules which can help verify (a) A work of art is as permanent as the work of a scientist.
experience (b) The influence of a work of art increases from age to
363. An exact theory of the universe is age unlike the work of a scientist which diminishes in
(a) not desirable (b) improbable course of time.
(c) possible (d) yet to be made (c) A work of art has no material value like the work of a
364. In exact sciences scientist.
(a) personal judgements are set aside in favour of a (d) A work of art is an expression of creative power while
mechanical theory the work of a scientist is not.
(b) one does not find answers to all questions and PASSAGE - 2
problems Most disputes about whether or not men are stronger than women
(c) one reposes faith i n actual experience are meaningless because the word 'strong' may mean many things.
(d) one interprets the universe according to one's wish Most men can surpass most women in lifting heavy weights, in
365. ClassicaI mechanics striking an object, in running, jumping or doing heavy physical
(a) has formulated precise rules based on experience labour. But most women live longer than most men, they have a
(b) has gained intellectual control over the world better chance of resisting disease, they can beat men at operations
(c) has formulated an exact theory of the universe requiring finger dexterity and the ability to work accurately under
monotonous conditions. So it would be legitimate to argue that
(d) just falls short of achieving intellectual control over women are stronger than men. The truth is that each gender can
experience surpass the other in certain kinds of activities. To say that one is
DIRECTIONS (Qs. 366-385): In this section, you have five short stronger than the other is to indulge in futile arguments.
passages. After each passage, you will find same question based 370. Which one of the following statements best reflects the
on the passage. First, read a passage and answer the questions main contention of the author?
based on it. You are required to select your answers based on (a) In most cases men are stronger than women.
the contents of the passage and opinion of the author only. (b) Since women are healthier than men they are also
[2016-II] stronger.
(c) In some activities men are stronger than women and
Passage - l in some others women are stronger than men.
One of the most important things to notice about the power of art (d) Men and women are equally strong.
is the way in which great works continue to exert their influence 371. The author says that any dispute about whether or not men
through the ages. Scientific discoveries which are of major are stronger than women is meaningless, because
importance at the time when they are made are superseded. Thus, (a) it is an already established fact that men are Stronger
Newton's theory of gravitation has been superseded by Einstein's than women.
theory of relativity. Hence the work of great scientists has value (b) the word 'stronger' can be interpreted in various ways.
in stages on the way to a goal which supersedes them. Broadly (c) it is difficult to assess the comparative strength and
speaking, the achievements of generals, politicians, and women.
statesment have an importance only in their own time. Hence (d) it is a dispute that might harm the man-woman
these people and their acts, great as they may have been are like relationship in our society.
Reading Comprehension B-191
372. The author says it would be legitimate to argue that women PASSAGE - 4
are stronger than men, because Just as some men like to play football or cricket, so some men like
(a) the author believes in the superior strength of women. to climb mountains. This is often very difficult to do, for mountains
(b) the author is not committed to any opinion. are not just big hills. Paths are usually very steep. Some
(c) in some of the activities woman do give an impression mountainsides are straight up and down, so that it may take many
that they are stronger than men. hours to climb as little as one hundred feet. There is always the
(d) in fact women are inferior to men in every respect. danger that you may fall off and be killed or injured. Men talk
373. From the passage, which of the following statements is about conquering a mountain. It is a wonderful feeling to reach
most likely to be correct? the top of a mountain after climbing for hours and may be even
(a) Women live longer than men because they can resist for days. You look down and see the whole country below you.
diseases better than men. You feel Godlike. Two Italian prisoners of war escaped from a
(b) Monotenous living conditions make women stronger prison camp in Kenya during the war. They did not try to get back
than men. to their own country, for they knew that was impossible. Instead
(c) All women are incapable of running, jumping and they climbed to the top of Mount Kenya, and then they came
doing physical labour because they are not strong. down again and gave themselves up. They had wanted to get
(d) Statistically speaking, most women live longer than that feeling of freedom that one has, after climbing a difficult
most men. mountain.
Passage - 3 378. Some men like to climb mountains because
In national no less than in individual life there are no watertight (a) they do not like to play football or cricket.
compartments. No sharp lines can be drawn to mark off" the (b) they know the trick of climbing.
political from the moral, the social from the economic regions of (c) they want to have a wonderful feeling.
life. Politicians often talk as though one has only to introduce (d) they like to face danger.
certain political and economic changes for paradise to descend 379. To climb a mountain is often difficult because
on earth, forgetful of the fact that the efficiency of an institution (a) mountains are big hills.
depends on the way it is worked, which itself is determined by (b) it consumes more time.
the character and wisdom of the men who work it. (c) prisoners often escape from camps and settle there.
374. Which one of the following statements most clearly (d) paths are steep and uneven.
suggests the central theme of the passage? 380. It is a wonderful feeling ............ 'It' refers to
(a) Political and economic changes can solve all the (a) the steep path (b) the prisoner
problems facing the nation. (c) the mountain (d) mountaineering
(b) There is no difference between the political, moral, 381. Two Italian prisoners escaped the camp and climbed to the
social and economic regions of life. top of Mount Kenya to
(c) It is not the institutions that are important but the (a) escape to Italy.
character and wisdom of the people who manage them. (b) come down and give up.
(d) National progress depends solely on the efficient (c) gain fame as mountaineers.
running of our institutions. (d) get the feeling of freedom.
375. Which one of the following phrases best helps to bring out
the precise contextual meaning of 'watertight compartments"? PASSAGE - 5
(a) Activities of life unaffected by public opinion. Most of the people who appear most often and most gloriously
(b) Spheres of life where no liberty of opinion is tolerated in the history books are great conquerors and generals and
(c) Ways of life peculiar to each nation and each section soldiers, whereas the people who really helped civilization forward
of society. are often never mentioned at all we do not know who first set a
(d) Spheres of life which are independent and broken leg, or launched a seaworthy boat, or calculated the length
unconnected with one another. of the year, or manoeuvred a field; but we know all about the
376. Which one of the following statements most correctly reflects killers and destroyers People think a great deal of them, so much
the attitude of the author towards politicians' opinions? so that on all the highest pillars in the great cities of the world
(a) The author totally disbelieves what the politicians you will find the figure of a conqueror or a general or a soldier.
say. And I think most people believed that the greatest countries are
(b) The author believes what the politicians say. those that have beaten in battle the greatest number of other
countries and ruled over them as conquerors.
(c) The author is sceptical about the claims of the
382. People who are glorified often in history books are those
politicians.
(a) who contributed to the public health.
(d) The author thinks that the opinions of the politicians
(b) who contributed to the technical knowledge of man.
are contradictory.
(c) who made calendars.
377. Which one of the following statements most correctly
(d) who fought and won wars.
indicates the implication of the phrase 'paradise to descend
383. The words "the people who really helped civilization
on earth'?
forward" suggest that conquerors, generals and soldiers
(a) A world of perfect economic, political and social well-
(a) contributed a great deal to civilization.
being.
(b) contributed only towards civilization.
(b) A world ruled by religious persons.
(c) were least interested in the progress of civilization.
(c) A world of total liberty and equality.
(d) contributed little to civilization.
(d) A world in which nobody needs to labour.
EBD_7367
192
B- Reading Comprehension

384. We will find the figure of a conqueror or a general or a 390. The writer likes travelling on the motorcycle. What is the
soldier on all the highest pillars in great cities because most likely reason for this ?
(a) they sacrificed their lives for the benefit of humanity. (a) The motorcycle has no windows.
(b) people have exaggerated notions about their (b) The motorcycle does not go as fast as a car.
achievements. (c) As the traveller is used to cars, travelling by
(c) they had a deep concern for the welfare of humanity. motorcycle is a change.
(d) Travelling by motorcycle, the writer feels that he is
(d) they built most cities.
part of the scenery.
385. The passage implies that the greatest countries are those 391. Which of the following statements is closest to the truth ?
that (a) The writer does not like TV as it gives a narrow view
(a) have conquered many countries and ruled over them. of things.
(b) are very large in their size. (b) The writer likes TV but he does not like watching it
(c) have the largest population. from car windows.
(d) are civilized (c) The writer does not like TV because the picture is in a
frame.
DIRECTIONS (Qs. 386-407) : In this section, you have six short (d) The writer does not like TV because the programmes
passages. After each passage, you will find some questions based are boring.
on the passage. First, read a passage and answer the questions 392. "In a car you are always in a compartment, and because
based on it. You are required to select your answers based on you are used to it you do not realise that ........ ." In this
the contents of the passage and opinion of the author only. sentence, 'it' refers to
[2017-I] (a) travelling in a car.
(b) always being in a compartment, e.g. one's room, office.
PASSAGE – 1 (c) seeing the scenery through the window frame.
When Jonathan (the seagull) came, it was well after dark, and he (d) seeing so much TV at home.
floated in moonlight on the surface of the ocean. His wings were 393. In the last sentence, the writer talks of a 'sense of presence'.
ragged bars of lead, but the weight of failure was even heavier on He is referring to the presence of
his back. He wished, feebly, that the weight would be just enough (a) his own self as part of the scene.
to drag him gently down to the bottom, and end it all. But soon he (b) the time that is now passing.
came back to normal. He pushed wearily away from the dark water (c) the scene and the beauty.
and few towards the land, grateful for what he had learned about (d) senses with which one feels.
work-saving low-altitude flying. 394. The word 'overwhelming' means
386. The word 'wearily' means (a) very strong (b) unavoidable
(c) interesting (d) humorous
(a) tireless (b) exhausted
PASSAGE – 3
(c) sadly (d) unconscious I was lying down in a dark, lonely compartment of the speeding
387. The seagull suffered because train, trying to sleep. But, quite unusually, sleep eluded me. A
(a) he had tried to do something that other seagulls had vague uneasiness gripped me. It was pitch dark outside. A few
not done. points of light flashed by as we sped through a small station and
(b) probably he had been attacked by a stronger bird. in the dim light I thought I saw a hand gripping the bars of my
(c) probably he had been attacked by some strong window. Once again the train was swallowed up by the
creature in the sea. impenetrable darkness. My heart pounded. My mouth was
(d) he had swooned and fallen into the water. parched. I could not get up. I do not know how long I remained
388. 'His wings were ragged bars of lead' means that thus before the train began to slow down. The reassuring bright
(a) his wings were damaged and supported by bars of lights of the station we were entering revealed no intruder. I
lead. breathed again.
(b) his wings were damaged and therefore very heavy. 395. The narrator could not sleep because
(c) he had rags and bars of lead on his wings. (a) he usually found it difficult to fall asleep.
(b) he could not find a place to lie down.
(d) his wings were broken like pieces of lead. (c) he was disturbed by some unspecified thoughts.
389. The lesson that he had learnt that day was about (d) the people near him were disturbing' him.
(a) not fighting with stronger birds. 396. In the dim light he saw
(b) flying carrying bars of lead on his wings. (a) someone trying to climb into the train.
(c) diving too deep into the sea. (b) someone clinging to the bars of the window.
(d) flying at low altitudes. (c) someone was attempting to steal his bag.
PASSAGE – 2 (d) someone standing outside the window.
Vacationing on a motorcycle, you see things in a way that is 397. Which of the following words best describes the condition
completely different from any other. In a car you are always in a of the traveller ?
(a) Cautious (b) Imaginative
compartment, and because you are used to it you do not realise
(c) Observant (d) Nervous
that through that car window everything you see is just more TV. PASSAGE–4
You are a passive observer and it is all moving by you boringly in I was abruptly awakened by a noisy scuffle. The sun, a mere
a frame. On a motorcycle, however, the frame is gone. You are fringe over the horizon, immediately chased away the grey half-
completely in contact with it all. You are in the scene, not just darkness. I was too sleepy to notice what was happening. Yuri
watching it anymore, and the sense of presence is overwhelming. was rolling over on the ground. I ran up to him but was struck
Reading Comprehension B-193
dumb. With his right hand he was holding a cobra by the neck. those who disparaged it. He said that the Eiffel Tower was a
Two sharp fangs showed from its jaws. The battle was over in a monument of man's folly, not of his wisdom. Tobacco, he argued,
few minutes. A hollow hissing and convulsive jerks were then was the worst of all intoxicants, inasmuch as a man addicted to it
only reminders of a just-ended tussle. The catcher half-opened was tempted to commit crimes which a drunkard never dared to
the lid of the box and calmly put the quarry in. do; liquor made a man mad, but tobacco clouded his intellect and
398. When the writer saw Yuri holding a cobra by the neck, he made him build castles in the air. The Eiffel Tower was one of the
was 'struck dumb'. This means that he was creations of a man under such influence. There is no art about the
(a) extremely delighted. (b) very much helpless. Eiffel Tower. In no way can it be said to have contributed to the
(c) rather surprised. (d) absolutely shocked. real beauty of the Exhibition. Men flocked to see it and ascended
399. From the passage, Yuri appears to be a man who is it as it was a novelty and of unique dimensions. It was the toy of
(a) calm and courageous. (b) cunning and crafty. the Exhibition. So long as we are children we are attracted by
(c) noisy and dangerous. (d) active and jumpy. toys, and the Tower was a good demonstration of the fact that we
400. With reference to the passage, the following assumptions are children attracted by trinkets. That may be claimed to be the
have been made : purpose served by the Eiffel Tower.
1. The incident took place early in the morning. 405. Why did Tolstoy disparage Eiffel Tower ?
2. Yuri threw the snake away. 1. Man was foolish to build it.
Which of these assumptions is/are correct ? 2. Huge man-made structures did not appeal to him.
(a) 1 only (b) 2 only 3. Men flocked to see it.
(c) Both 1 and 2 (d) Neither 1 nor 2 Which of the statements given above is/are correct ?
PASSAGE – 5 (a) 1 only (b) 1 and 2 only
Urbanization and industrialization have often resulted in whole (c) 1 and 3 only (d) 2 and 3 only
areas of forests being cleared to gain new land and to obtain 406. Why did Tolstoy believe that tobacco was the Worst of all
timber for the various building projects. Large areas of fields and intoxicants ?
forests have disappeared to make way for concrete jungles many (a) Man lost his intellectual abilities under the influence
of which are fitted with huge plants and chimney stacks. Industrial of tobacco.
growth has necessitated the increased demand for fuel oil to run (b) Tobacco kept man in a state of inebriation.
the machines and in doing so produces industrial gases and fumes (c) People who commit crimes are invariably addicted to
which belch through the chimney and pollute the atmosphere. tobacco.
The most evident elements in the contamination of the (d) Statements (a) and (b) above are correct in this context.
atmosphere are dust, sulphur dioxide, carbon monoxide and nitrous 407. Why did men flock to the Eiffel Tower ?
oxide. (a) Men were attracted to the castles built in the air.
401. The writer expresses the belief that (b) Men lost their wisdom under the influence of
(a) there is plenty of scope for further industrialization. intoxicants.
(b) unplanned growth of industry has done more harm (c) Men were attracted to childish things.
than good. (d) Men were attracted to things of no value.
(c) the change from rural to urban growth is a change for
the better. DIRECTIONS (Qs. 408-428) : In this section you have few short
(d) the timber obtained from the forests has been passages. After each passage, you will find some items based on
beneficially used. the passage. First, read a passage and answer the items based
402. The effect on forest areas produced by the activity described on it. You are required to select your answers based on the
in the first sentence is called contents of the passage and opinion of the author only. [2017]
(a) devastation (b) deforestation
(c) disfiguration (d) devaluation PASSAGE
403. The results of industrial development, according to the For many years, ship captains navigating the waters of Antarctica
writer, are have been intrigued by sightings of emerald icebergs. Scientists
(a) urbanization. have now explained their mystery. There icebergs are turned
(b) no shortage of fuel oil. upside down. Icebergs are blocks of ice that have broken off
(c) greater availability of domestic gas. huge slabs of frozen snow called ice shelves. Their green
(d) greater fuel consumption and pollution. appearance results from sea water that has frozen at the bottom
404. The phrase 'concrete jungle' in the paragraph refers to the over hundreds of years. The frozen sea water has dissolved
factories and houses built as a result of urbanization and organic matter which givens it a yellow tone and the fresh water
industrialization. This phrase suggests that the author ‘ice shelf’ above has a blue tinge. When the iceberg turns upside
(a) regrets that fields and forests have been replaced by down, it appears green through the visual mix of yellow with the
city buildings. blue from below.
(b) believes that too much cement has been used in 408. What is the meaning of ‘intrigued’?
building factories. (a) Surprised (b) Fascinated
(c) disapproves of modern industrial expansion. (c) Muffled (d) Repulsed
(d) would like to go back to life in the jungle. 409. What are ice shelves?
PASSAGE – 6 (a) They are huge pieces of chunks of ice
I must say a word about the Eiffel Tower. I do not know what (b) They are frozen sea water
purpose it serves today. But I then heard it greatly disparaged as (c) They are pieces of ice which look like shelves
well as praised. I remember that Tolstoy was the chief among (d) They are huge pieces of ice which are very old
EBD_7367
194
B- Reading Comprehension

410. What are icebergs? they would prefer to stay in the home rather than work in the
(a) Huge chunks of ice floating on water factory. The majority of women do not escape from the traditional
(b) Frozen sea water feminine world. Their jobs at the factory do not relieve them of
(c) Green ice housekeeping burdens; they get from neither society nor their
(d) Green yellow water below and blue above husbands, the assistance they need to become in concrete fact
411. When the iceberg turns upside down, it appears the equals of men.
(a) green (b) yellow 417. Which of the following helps women most to achieve
(c) blue (d) white equality with men?
(a) The right to vote (b) Civil liberties
PASSAGE
(c) A job (d) Wealthy husbands
In its simple form, science has helped man to protect himself from 418. Why does the writer talk about the maidservant in the hotel
Nature and to overcome natural obstacles to movement. But with lobby?
the advance of science, a situation has arisen in which Nature (a) The servants of today will one day be freed from their
need to be protected from man. He has used Nature’s own gifts, rich masters
not only of metal but even tha human brain, to attack Nature. (b) A servant can become as rich as Rockefeller or Birla
Forests are being destroyed not only to satisfy need but to (c) Even with a low paid job women can achieve equality
provide luxuries. The evil effects of deforestation are already (d) Economic independence is necessary for women’s
making themselves clearly felt in climatic changes and soil erosion. liberation
Man has at last begun to learn that he has to protect if he wants 419. In which paragraph does the writer say that it is revealed
Nature to protect him. that some women would not like to work in the factory?
(a) In paragraph four (b) In paragraph three
412. The use of science in its simple form has helped man to
(c) In paragraph two (d) In paragraph one
(a) do such things as building shelter and make carts,
420. “These liberties” in the first paragraph refer to
boats, etc.
(a) The right to vote, not to obey and right to a job
(b) make bombs and missiles (b) The right to vote and not to obey
(c) build factories using machinery (c) The rights of servants to disobey their master and the
(d) make planes right of the master to punish them
413. Nature now needs to be protected from man because (d) Women’s right to vote and earn money
(a) nature has become weak
(b) man is rapidly destroying Nature PASSAGE
(c) man is cruel to animals During the past three generations the diseases affecting western
societies have undergone dramatic changes. Polio, diphtheria,
(d) man has become irrational
tuberculosis, commonly known as TB, are vanishing; one injection
414. Forests are being destroyed in order to
of an antibiotic often cures deadly diseases such as pneumonia
(a) provide land for agriculture or syphilis; and so many mass killes have come under control
(b) provide wood for fuel that two-thirds of all death are now associated with the diseases
(c) kill dangerous animals of old age. Those who die young are more often than not victims
(d) provide necessities as well as needless comforts and of accidents, violence or suicide.
pleasures These changes in health status are generally equated with
415. The evil effect of destroying Nature instead of using it is the decrease in suffering and attributed to more or better medical
seen in care. Almost everyone believes that at least one of his friends
(a) the fall in production of our factories would not be alive and well except for the skill of a doctor. But
(b) the fall in our standard of living there is in fact no evidence of any direct relationship between
(c) the unfavourable changes in climate this change in the pattern or nature of sicknesses on the one
(d) frequent occurrence of epidemics hand and the so-called progress of medicine on the other hand.
416. Climatic changes and soil erosion are results of These changes are the results of political technological changes.
(a) scientific developments (b) nuclear explosion They are not related to the activities that require the preparation
(c) natural calamity (d) deforestation and status of doctors or the costly equipment in which doctors
PASSAGE take pride. In addition, an increase in the number of new diseases
in the last fifteen years are themselves the result for medical
According to the civil laws of most countries obedience is no
intervention. They are doctor-made or iatrogenic.
longer the duty of a wife; every woman has the political right to
vote; but these liberties remain theoretical as long as she does 421. In the western societies, the occurrence of polio, diphtheria
not have economic freedom. A woman supported by a man is not and tuberculosis has
liberated from the male. It is through gainful employment that (a) increased (b) completely stopped
woman has travelled most of the distance that separated her from (c) decreased (d) continued without changes
the male; and nothing else can guarantee her liberty in practice. 422. More death are now associated with old age than in the
past because
I once heard a maidservant declare, while cleaning the stone floor (a) iatrogenic diseases are spreading faster now
of a hotel lobby, “I never asked anybody for anything; I succeeded (b) deadly diseases affecting the young have been well
all by myself.” She was as proud of her self-sufficiency as a controlled
Rockefeller, Ford or Birla. (c) accidents, violence and suicide that killed many
However, the mere combination of the right to vote and a job youths in the past are now under control
does not mean complete liberation : working, today, is not a liberty. (d) political and technological changes now take better
A recent study of women workers in a car factory shows that care of the young than the old
Reading Comprehension B-195
423. The writer probably is arguing for problems such as poverty, under-nourishment, unemployment
(a) stopping the practice of western medicines completely and excessive fragmentation of land. Indisputably, this country
(b) stopping the use of costly equipment and medicines has been facing a population explosion of crisis dimensions. It
(c) rethinking about the successes and failures of the has largely diluted the fruits of the remarkable economic progress
western medicines that the nation has made during the last four decades or so. The
(d) giving greater attention to new, iatrogenic diseases entire battle against poverty is thwarted by the rapid increase in
than to the old diseases such as polio, diphtheria and the population. The tragedy is that while over-population
pneumonia accentuates poverty, the country’s stark poverty itself is in many
PASSAGE areas a major cause of over-population.
Poverty is a complex problem. It is far more than an economical 429. What is the irony behind the over-population of India?
condition. We measure it usually in terms of income but forget (a) Over-population gives birth to poverty, which
that poverty embraces a whole range of circumstances, including (poverty) itself is the cause of over-population
lack of access to information and to basic services like nutritional (b) Under nourishment and unemployment are outcomes
diet, health care and education. It results into a loss of cultural of flawed economic progress
identity and destroys traditional knowledge. Poor people become (c) Fragmentation of land is leading to over-population
marginalised and suffer from exploitation and loss of human (d) Fruits of the remarkable economic progress are trickling
dignity. down to the poor
424. Which of the following sentences comes close to the 430. What is the general tone of the passage?
meaning of the sentence, “Poverty embraces a whole range (a) funny/humorous (b) sombre
of circumstances.” (c) didactic (d) tragic
(a) There are lot of angles to poverty 431. What, in the author’s view, severely affects the economic
(b) They are several section in the society which are poor growth of our country?
(c) There are several types of poverty (a) poverty (b) illiteracy
(d) Poverty is solely an economic issue (c) over-population (d) None of the above
425. What way do you think ‘lack of access to information’ affects 432. What, according to the author, is the biggest reason behind
poor people? over-population?
(a) They don’t get information about how to improve their (a) under-nourishment
conditions (b) unemployment
(b) They didn’t get newspapers to read at all (c) excessive fragmentation of land
(c) They can’t go to school and read books (d) poverty
(d) They don’t get information about schemes of getting 433. “It has largely diluted the fruits of the remarkable economic
rich progress”. Find antonym of the underlined word
426. Why do you think ‘cultural identify’ is important? (a) coalesced (b) compounded
(a) A sense of cultural identify gives people self-respect (c) cheapened (d) consolidated
and confidence PASSAGE
(b) Cultural identify defines the character of poor people To eat and not be eaten – that’s the imperative of a caterpillar’s
(c) It is important to have cultural identify to get jobs existence. The leaf roller reduces its risks of being picked off by
(d) It is useful to have cultural identity because it brings predators by silking together a temporary shelter in which to feed
your success and rest. Adopting a different line of defense, the jelly slug
427. Which of the following sentences comes closest to the extrudes a sticky translucent coating that may foul the mouth-
sentence ‘Poor people become marginalised’? parts of marauding ants. For its part, the aquatic larva, by its
(a) They are not given any benefit of any government watery element, fashions a portable hideout from fragments of
schemes aquatic leaves. Cutting a serpentine trail as it feeds on tender
(b) They are ignored by the rich people young leaves, the minute citrus leaf miner spends its entire larval
(c) They are the most ignored elements of the society life inside its host plant, thus keeping its appetizing body safely
(d) They are the most disposed elements of the society under wraps.
428. What do you think is the tone of the passage? 434. Which varieties of caterpillars ‘build’ shelters to protect
(a) Objective but querulous themselves?
(b) Descriptive and impassioned (a) Leaf roller and aquatic larva
(c) Argumentative and critical (b) Leaf roller and jelly slug
(d) Objective and critical (c) Jelly slug and aquatic larva
(d) Jelly slug and citrus leaf miner
DIRECTIONS (Qs. 429-448): In this section you have few short 435. Which one of the following caterpillars produces a sticky
passages. After each passage, you will find some items based on covering?
the passage. First, read a passage and answer the items based (a) Leaf roller (b) Jelly slug
on it. You are required to select your answers based on the (c) Aquatic larva (d) Citrus leaf miner
contents of the passage and opinion of the author only. 436. Which one of the following pairs of words in the passage
[2018-1] describes enemies of the caterpillar?
(a) Serpentine and host
PASSAGE (b) Predator and marauding
Over-population is the most pressing of India’s numerous and (c) Serpentine and marauding
multi-faceted problems. In fact it has caused equally complex (d) Predator and host
EBD_7367
196
B- Reading Comprehension

437. Which one of the following makes itself unpalatable? (d) brought up in an environment unfavourable to the
(a) Leaf roller (b) Jelly slug growth of the mental life
(c) Aquatic larva (d) Leaf miner 443. The title that best expresses the central idea of the passage
438. The main idea of the passage is that caterpillars is
(a) like to eat a lot (a) difference between the Occident and the Orient
(b) have to protect themselves while feeding (b) impact of Industrialization on our civilization
(c) are good to eat (c) advantages of Industrialization
(d) are not good to eat (d) disadvantages of Industrialization
PASSAGE PASSAGE
I have always opposed the idea of dividing the world into the In Delhi, it was forbidden by the law, at one time, to take a Dog
Orient and the Occident. It is, however, the tremendous industrial into a public vehicle. One day a lady, accompanied by a pet dog,
growth that has made the West what it is. I think the difference, entered a bus. Wishing to evade the law, she placed her tiny dog
say, between India and Europe in the 12th or 13th century would in her dress pocket. It so happened that the person next to this
not have been very great. Differences have been intensified by lady was a pick-pocket; and during the journey the carefully placed
this process of industrialization which has promoted material well- his hand into her pocket in search of her purse. Great was the
being tremendously and which is destroying the life of the mind, horror to find instead a pair of sharp teeth inserted into his fingers.
which is in a process of deterioration, chiefly because the His exclamation of pain and surprise drew the attention of other
environment that has been created by it does not give time or passengers to him.
opportunity to individuals to think. If the life of the mind is not 444. Once the law in Delhi did not permit the people to
encouraged, then inevitably civilization collapses. (a) carry dogs into private vehicles
439. The words “the Orient and the Occident” mean (b) board a bus without ticket
(a) the West and the East respectively (c) carry dogs into a public vehicle
(b) the East and the West respectively (d) carry animals with them
(c) the North and the South respectively 445. In order to evade the law, the lady
(d) the South and the North respectively (a) hid the dogs under the seat
440. The author believes that the difference between India and (b) got off the bus
Europe in the 12th or 13th century was not very great (c) gave the dog to a fellow passenger
because (d) put the dog in her pocket
(a) Indians and Europeans mixed freely 446. The pick-pocket travelling with the lady
(b) Indians imitated the European way of living (a) reported the matter to the conductor
(c) Europeans imitated the Indian way of living (b) put his hand in her pocket
(d) Industrialization had not yet taken place (c) took out the dog
441. In the opinion of the author, Industrialization is (d) asked the lady to get off
(a) an absolute blessing 447. Which one of the following correctly expresses the meaning
(b) an absolute curse of “wishing to evade the law”?
(c) neither a blessing nor a curse (a) Wish to avoid following the law
(d) more of a curse than a blessing (b) Desire to follow the law blindly
442. The author says that the mental life of the world is in a (c) Reluctance to break the law
process of deterioration because the modern generation is (d) Wish to change the law
(a) endowed with low mental powers 448. Why did the pick-pocket exclaim with pain?
(b) too lazy to exert its mental powers (a) He was hit by the lady
(c) taught that physical activities are more important than (b) He was caught by the fellow-passengers
mental (c) He was bitten by the dog
(d) He fell of the bus
Reading Comprehension B-197

HINTS & SOLUTIONS


1. (d) The author is talking about the basic essentials of life, 31. (c) The author says that others are ready to tease and
food, shelter, cloth, and warmth. laugh at our attempts.
2. (c) The author is trying to suggest that increased use of 32. (a) The author says we fail in our attempts because we
technology implies unnecessary comfort an d set high ambitations and never have time to carry them
happiness for mankind and it is complementary to a out.
raised standard of living. 33. (b) The word inveterate means having a particular habit
3. (d) The increased use to technology suggests man's that is long-established and unlikely to change.
interest for more and more work. 34. (c) The author implies that despite our repeated failures
4. (b) The author suggests that Man will be working shorter we still try one more time.
and shorter hours in his paid employment. 35. (b) The word formidable means inspiring fear or respect
5. (b) The author talks of dangerous ideas which he says and here the for middable list refers to the long list
are born out of the enjoyment of freedom. which was frigtening.
6. (d) The author warns that popular violence is always the 36. (d) The passage seems to be a part of someone's personal
outcome of a deep popular dissatisfaction with the letter as it is addressed at many places by the word
government. 'you'.
7. (c) The author is trying to highlight the developed laziness 37. (a) The tone of the author sounds quite official and
that makes us incapable of social action. serious.
8. (a) Burke is highlighting the violence against injustice. 38. (b) The writer is talking in context of government service.
39. (c) The writer asks his reader to accept 'an official rank'.
9. (a) According to the author a typical unhappy man is one 40. (a) The writer implies significant worldly items.
who has been deprived of some normal satisfaction in 41. (b) The word drowsy means half-asleep or sleepy.
youth. 42. (a) The author talks about the leisure and peace he was
10. (c) The author refers one-sided direction to those activities experiencing.
which lead to happiness. 43. (d) The author mentions "In a moment the sole porter
11. (a) Drinking helps in a momentary cessation of emerged from his midday nap".
unhappiness or to forget their dissatisfaction. 44. (b) The author mentions a reference to the porter by
12. (c) According to author, becoming less alive implies living "operated a signal that clanked noisily into position".
in a make believe world. 45. (c) The word placid means pleasantly calm or peaceful;
13. (d) The author says we need to overlook some things, unruffled; tranquil; serenely quiet or undisturbed.
thus not punish each and every offence. 46. (c) The author sounds excited as he says "I was
14. (d) The author says we need to overlook some certain exceedingly surprised".
things. 47. (a) The word apparition means "anything that appears,
15. (c) The ancients said that an unforgiving temper was not especially something remarkable or startling", thus best
to be commended. describing the author's feelings.
16. (a) The author is highlighting the spirit of forgiveness. 48. (d) The author says "I went to it again to see if there were
17. (d) The author narrates the story in the context of Europe. any more".
18. (c) Buck up is an expression for the word cheer up. 49. (d) The author says "I was exceedingly surprised".
19. (a) The Indian friend was being hopeless because the 50. (c) The author was looking around to see where the foot
holiday was over. prints came from.
20. (b) The author describes the typical English character. 51. (c) The author's sentence "But he could not find a needle.
21. (a) The author is trying to show the contrast between the He fell asleep again" describes the man's reaction to
Indian and Englishman. his problem.
22. (b) The word appalling means shockingly large number. 52. (a) The man wanted to sew the hole because he didn't
23. (d) Genocide of the Jews was the culmination of a decade want to be scolded by his wife in the morning.
of Nazi policy, under the rule of Adolf Hitler. 53. (c) The man was searching the needle to sew up the hole.
24. (b) The other solution requires patience and tolerance to 54. (b) The smell of burnt cigarette awoke the man and burnt
put up with people. his only shirt.
25. (c) The author speaks in democratic tone. 55. (c) As the man fell asleep, the cigarette fell from his mouth.
26. (a) Nature here denotes a beautiful landscape, full of green 56. (c) The speaker does not mention anything about the
vegetation – the countryside. evidence in the passage.
27. (c) The author is highlighting the nature's beauty. 57. (c) Whatever others said about him he never lost faith in
28. (b) The author mentions the impact of modernization and goodness and he was sure that the truth must come
that we should be more careful not to destroy nature out one day.
while modernizing. 58. (d) In the dark dungeon he always waited for the angel to
29. (b) The word struggle here refers to the efforts required come and plead for him.
to save the nature. 59. (c) The truth must prevail means truth always wins in the
30. (c) If we preserve nature, it will ensure future existence. end.
EBD_7367
198
B- Reading Comprehension
60. (d) The man was scribbling down some odd mathematical 89. (d) The older people are psychologically more inclined
notations. towards their past.
61. (c) The man caught author's attention because he would 90. (a) The author mentions the old comfortable picture of a
stare at every person and then scribble down some friendly universe governed by spiritual values.
mathematical notion. As the man was sitting next to 91. (d) Religious skepticism rose immediately after the age of
the author, he caught his attention. the rise of science.
62. (d) The author found it quite amusing when the man was 92. (b) Braille lost his sight accidentally as a child.
trying to analyse a man's face, not as a single unit but 93. (c) In those days, the few books that were available for
thousands squares put together. blind people were printed in big, raised type; the letters
63. (a) The passage shows the inquisitiveness or curiosity of used were those of the ordinary alphabet.
the author. 94. (a) Braille evolved a system, which made use of only six
64. (d) In an age of science and technology, specialization dots in all.
becomes necessary. 95. (c) Reading and writing for the blind have thus become
65. (c) The central theme of the passage is that the aim of enormously simplified by Braille’s system.
education is to make the youth capable of independent 96. (c) The author says that our ancestors blended with one
thought and expression. another thus we are a result of a blended culture.
66. (a) The major warning suggested in the passage is that 97. (c) The author is not interested whether the Aryans were
University education should not be concerned with indigenous to India or were unwelcome intruder.
technical details rather it should embrace humanism. 98. (c) The author wishes to have the freedom to blend other
67. (c) The firewood had been gathered/ collected from the cultures with our own.
mountain streams. 99. (d) The author insists on learning English and other world
68. (b) The parents were quite glad seeing their children languages in addition to the mother tongue.
enjoying the fireside. 100. (d) As the passage talks about sky being full with rusting
69. (b) The oldest daughter is described to be a picture of wings and the streaks of red and golden in the sky,
happiness. which implies times of sunrise and the sky being full
70. (d) She glanced at him when the unpleasant feeling with birds that is time before sunrise and birds. The
passed. answer should include both 1 and 4, thus the correct
answer is (d).
71. (c) The unpleasant feeling passed when she realized her
101. (c) The word 'panting' means, 'not being able to breathe
sense of shame. properly'; the passage talks about the struggle of Jean,
72. (d) She was ashamed because she had never thanked him who was really too tired with the swimming and
for his help to her father. This thought had occurred to climbing his way on the hill towards his home.
her before also, that's why she felt ashamed. 102. (c) As the passage talks about professional equipment, a
73. (b) The traveller could not feel the breeze as he was down dozen cowrie shells, a square piece of cloth with
the ridge. obscure mystic charts on it, a note book, and a bundle
74. (c) The traveller carried with him a sling. of Palmyra writing; all these equipment are not used
75. (c) The traveler was traveling during Spring time. by a vendor, snake charmer or priest, hence the answer
76. (b) The author tries to highlight our misconception about Astrologer.
common birds. 103. (b) The author talks about attracting people's attention,
77. (b) Our ideas about birds are derived from the most as the protagonist was trying to lure possible clients.
common types. 104. (d) The astrologer is trying to catch people's attention by
78. (d) The common wild duck quacks like the mallard. displaying all his work aids.
79. (d) The boy answered the question when the teacher 105. (c) The author describes the whole incident in a very
asked the question again and again and persisted to satirical manner, hence the tone is very ironic.
answer. 106. (d) The passage highlights the importance of punctuality
80. (c) Since the boy was repeating the same mistake this with Washington's perspective and talks about the
made the teacher think that the boy was doing it incidence on same topic with his secretary.
intentionally. 107. (c) The passage highlights the importance of punctuality
81. (a) The teacher was trying to control his anger as the for achieving success in life.
boy's parents had high hopes with the boy. 108. (b) The passage talks about the importance of liberal
82. (c) Mr. Higson was in his unusual self with unshaven, in education and it's role in resolving fundamental issues.
slippers and braces and smoking 109. (a) The author is highlighting the role of liberal education
83. (b) The author mentions the atmosphere of the shop being and its importance in present day.
heavy with bitter odour and filled with newspapers. 110. (b) The synonym for word 'specifically' is 'particularly'.
84. (c) The author praises village life as there is certain dignity, 111. (b) The author talks about the great and varied concerns
and kindliness, and love for other men. of the humankind which are not daily concerns but a
85. (b) Civilization has mainly destroyed the family affection bigger picture which the mankind together is facing as
and pastoral virtues. a challenge.
86. (c) The author emphasizes to accept old age as a fact of 112. (d) The author satirically criticizes the laid back attitude
life. of the modern generation towards amusements.
87. (b) The author talks about the psychological problems 113. (d) The author is criticizing the modern day generation's
faced by older people. attitude towards socializing thus appears a social critic.
88. (d) The author is emphasizing the importance of living in 114. (d) The author tries to highlight the behaviour of any male
the present and not giving too much importance to the species to win over the female species using the
past. Jackdaw's example.
Reading Comprehension B-199
115. (c) Th e passage high lights Jackdaw's purposeful 144. (b) The Bible
behaviour of showing off himself to attract the female 145. (d) Without critical discrimination but in the light of their
species. humanistic culture.
116. (d) In the end author describes the importance of holding 146. (c) Had a gradual decline in our time
a nesting place by a Jackdaw to attract the female. 147. (c) Worthless ideas.
117. (d) The author describes the purposeful behaviour of the 148. (d) when they came unsolicited
Jackdaw to get involved in a conflict to attract the 149. (c) He could find the worth of a story with a little effort.
female. 150. (b) her story was rejected unread
118. (c) The author is highlighting the facts about Jackdaw's 151. (c) witty
love life and his behaviour to attract a female. 152. (b) in order to publish them.
119. (a) The author talks about the importance of equal trust 153. (d) Reflection of the intellect
shown to all people on earth for reconciliation. 154. (b) a man of character refuses to be influenced by outside
120. (d) The author also tells not to humiliate the members of compulsions against his will.
those nations whose leaders have committed inhuman 155. (a) Good thoughts lead to the control of the sense organs.
crimes. 156. (c) attempt to destroy man's character
121. (d) The author is talking about the equality in treating 157. (b) the attainment of perfect character is the result of a
people from different native lands. long process of metal discipline.
122. (d) As the author is highlighting the importance of equal 158. (c) it gives rise to many other problems.
treatment for all people on earth, irrespective of their 159. (d) our economic planning
origin or races the answer is (d). 160. (b) is likely to become less satisfactory.
123. (d) The speaker is motivating his fellowmen to fight for 161. (a) is less than satisfactory.
their country is a patriot urging defence of his 162. (a) more provision for cloth than accommodation.
motherland. 163. (a) She was pale and emaciated.
124. (c) As the speaker talks about the New World to step 164. (b) She was frightened.
forth and rescue, he's expecting help from other parts 165. (d) Her inability to go near the fire.
of the world to arrive. 166. (c) The blue marks on her body.
125. (a) The speaker is motivating his fellow men to continue 167. (a) Addicts
168. (c) They need large sums of money to buy drugs.
highlighting the never give up attitude.
169. (d) Driving addicts to crime
126. (d) The speaker's intention is to motivate his fellow 170. (a) Very high
countrymen to fight for their mother land, thus 171. (d) Neither 1 nor 2
repetitively reinforcing this attitude. 172. (a) Providing things for themselves
127. (d) As the Captain and his crew were lost on unknown 173. (b) The goods are produced in surplus
islands; he sent his men in search of water. 174. (d) Living in communities
128. (b) The savage brandished their spears to frighten the 175. (d) We cannot produce everything we need ourselves.
crew 176. (b) The dog was sturdy and short tempered.
129. (d) As the inhabitants of the islands had spears and clubs, 177. (d) The dog bit him more than he bit other in the family.
this points that they were primitive tribes. 178. (c) The writer's dog had bitten at least forty people.
130. (b) The purpose of Mandela's talk was to help people 179 (c) The girl could not understand it
oppose oppression. 180. (a) 1 only (the governess taught the same lesson several
131. (c) The passage clearly describes the struggle of black time)
people with an unfavourable government. 181. (a) The dullness of the girl.
132. (b) As the commander had ordered lights out by 7 p.m. he 182. (a) The inability of the governess to endure further the
went to check whether his orders had been followed girl's failure to answer.
or not. 183. (d) The gradual acceptance of vaccination as a preventive
133. (d) The commander entered his son's tent because he against small pox
wanted to punish any soldier who disobeyed his order. 184. (b) Some of the vaccines used were of a poor hygienic
134. (c) The son was eager to tell his mother about his own standard.
deeds and thus was writing a letter to her. 185. (b) Fear of the terrible disease drove them to take the risk
135. (d) During the hot summer days author and Jack were of vaccination.
lazing around the house and yards. 186. (c) Build up a defence against small pox germs.
136. (b) They were planning for some adventurous trip to an 187. (b) Only 2 (there is no outbreaks of smallpox nowadays).
unknown place. 188. (d) By the word prosperous, the author means that the
137. (c) Their mother had already packed all the good clothes voyage was uneventful as the ship stopped only at
for their upcoming trip. two ports and was nothing to talk about.
138. (c) The plate camera was one of the ingenious artefacts 189. (d) On the ship, the author never left the ship till they
made by Jack. came into the downs.
139. (d) The author mentions Father and Jack's trip to Ropley 190. (a) In the context of the passage, provisions means food.
to see and rent a cottage there. The author writes that the ship stopped at one or two
140. (d) Mr. Sharma's friend heard the noise of a fight. ports for food and fresh water.
141. (a) Mr. Sharma was astonished to see the Tigress. 191. (b) For the payment of the author's freight, the captain
142. (a) Mr. Sharma was concerned about his wife and daughter refused to accept any money not even a single farthing.
and was trying to protect them. 192. (c) Since the captain did not take a single penny from the
143. (d) Literature and history are inseparably linked together author and was very soft to him, it shows his
in the classics and the bible. friendliness and kindness towards the author.
EBD_7367
200
B- Reading Comprehension

193. (d) Unless the children are taught differently, they think 217. (a) The passage shows that the girl was clever as she
that the sun and moon are moving round the earth. gave relevant clues to save her mother and was brave
194. (c) The early ideas of the man were wrong because they as she did not lose hope and was helpful throughout.
did not have any scientific knowledge as there were 218. (d) The girl helped the police to trace the car by honking
no telescopes or instruments to study the motion of the car's horn.
earth and stars. 219. (c) The writer depicts the importance and utility of portable
195. (d) Since the sun has been described as lamp in the telephones.
passage, answer is (d). 220. (c) The sentence means that the girl visited many property
196. (a) Lack of scientific knowledge was the main cause of dealers over the last few days without any success.
wrong ideas. 221. (b) The girl was an independent freelance writer who is
197. (c) The words 'The absence of sensation in his feet left not particularly employed in any company.
him unrelated to the earth' 222. (c) Single jobless women always face difficulty in renting
198. (d) As a result of the cold from snow, his hands and fingers an accommodation.
were freezing and could neither be bend nor had any 223. (c) Martin played little tricks because his son was
sensation. entertained and got pleasure from them.
199. (c) His hands could not clutch depicts how frozen his 224. (b) The paper mask is that of a hissing dragon face with
hands and fingers are that they cannot be moved and his tongue out.
felt. 225. (d) the father and son rolled with laughter after the father
200. (d) George's absence from the town was not noticed by removed the mask from his face.
the station master as he called out George asking if he 226. (c) We see in the given passage that Galileo made different
was going away. telescopes because he was not satisfied with the
201. (a) George was expecting a big welcome because he outcome. His previous instrument was too small. This
thought that he had achieved a lot in a short span of means that option c which talks about his instruments
time. not being powerful enough should be the reason
202. (c) George's expectations of people greeting him when he behind Galileo making many telescopes.
227. (a) When Galileo saw the moon his heart leaped with joy.
reaches his town shows his pride that he has achieved
Thus option a which says that he was overjoyed
a lot in business.
should be the correct answer.
203. (c) George's success was clearly visible in his being
228. (d) the author wants to say that his father had a strong
unbearably conceited and proud of himself. desire to teach and this desire was without any vanity
204. (b) The passage highlights how with modern technology, but full of innocence. This means that his father was
serious life hazards are being accompanied. passionate about teaching and loved it. Thus option
205. (b) The harmful effects of modern technology are d should be the correct answer as to what author wants
widespread and long lasting. to say.
206. (a) Only first assumption is true. Second is in correct as 229. (b) From the passage we get to know of an instance where
conservation of flora and fauna can be done with his father started teaching even at railway carriages
efficient management. which made the author embarrassed. Thus, to question
207. (d) Both the statements are incorrect and incomplete. as to what made him embarrassed option b should be
208. (c) In order to achieve high food productions, the answer, "even at odd places".
monocultures and other modern farming methods are 230. (c) The last sentence of passage conveys that for people
used. These is a widespread transmission of animal of his father's time, education was a golden gateway
diseases now-a-days. of enchanted realms of mind. This means that education
209. (b) Mr. Tagde did not sit down because he was in a defiant was a means to explore and sensitize the mind.
mood (meaning bold resistant) and stood for a 231. (a) Over all in the passage, we notice that the author
student's expulsion from the school for a prank. describes his father's passion very lovingly and with
210. (b) He did not wanted to take the report back because it pride. He glorifies his character in a profound manner.
was an accurate report based on the poor and Thus option a that he loved and admired his father is
damaging conduct of the student. the correct answer.
211. (d) The Principal was unhappy because he did not want 232. (d) the author was looking for all types of marks left by
to take the extreme step of expelling student from the the tiger's prey a deer as it is mentioned in the first line
school. of the passage.
212. (d) The wind was very sharp and was cutting the skin like 233. (c) In the last sentence of the passage, it is revealed that
a knife on the bitterly cold night. mothi was running to save his life from the tiger which
213. (b) The younger woman dressed in seal skin (fur coat) had come face to face with him.
was carrying the dog as a fashion accessory. It is 234. (b) we already know that the author was looking for the
usually carried by women of high stature. marks left by the tiger's prey, a deer. So the prey only
214. (d) The conductor was unfriendly and malicious with a ill means a kill depicting the deer.
will. 235. (a) The word used to describe the atmosphere before
215. (b) The sentence describes the unbearable cold night and mothi screamed was "silent". Silent also means quiet.
the sharp winds that were blowing at that moment. 236. (b) Ibbotson was reluctant to come with the narrator
216. (c) The little girl said that he she could see the airplanes because he thought that ten nights on a stretch would
and the sky. The airplanes can be seen only in the be too much for the narrator. Too much to bear also
airport premises and not outside it. hints at his getting tired.
Reading Comprehension B-201
237. (a) The correct answer is option (a) narrator clearly 253. (a) Option (b), (c), and (d) are quite positive in idea
mentions that he hopes to shoot the man eater, i.e. whereas the author is talking about the negative
Leopard. perspective of extreme nationalism. How it becomes a
238. (c) It is hinted in the passage that farmers were pressurized curse for the nation when people think only about
to cultivate indigo and forsake cultivating other crops. their own selves rather than others. Thus option (a) is
Thus it means that they were forced to do it. the correct answer.
239. (b) In the passage the first sentence talks of Britishers' 254. (d) In the first few sentences it has been suggested that
intention and their compelling farmers to take up indigo nationalism becomes a curse for the nation when
cultivation. This was their very intention to purchase nationalism turns narrow and fanatical. Thus it is clear
land in Bengal. that option (d) the answer.
240. (a) It is clearly evident from the passage as to why the 255. (b) In the second sentence, the author hints that in
indigo cultivation was profitable for Britishers. It was fanatical situation religion can lead men astray. This
because they held monopoly over it. That means they proves that religion sometimes can cause its misuse
had no competitors and only they could export it. by some men. Thus option (b) is the answer.
241. (b) From the passage we get the idea that rhinos are 256. (c) In the very first line it is clear that brown and his men
aggressive beasts and they rushed to attack the sat around the fire. Thus option (c) is true.
narrator while smelling his scent. This means that 257. (d) Option (a) is the correct answer as it is suggested in
rhinos attack humans on spotting them by their smell. the last sentence that the man did not know about the
242. (b) In the passage, it was the first time the narrator had danger and thus when he started walking down the
come across rhinos and he had a violent encounter. hill, he vanished, means he slipped.
Thus naturally he was frightened. 258. (a) The sentences ‘He didn’t think… in the dark’ implies
243. (c) The author could not take photos of rhino because it that he was bold and adventurous. Hence, option (a)
is evident from the passage that he was too frightened is correct.
to even do that. And in that course his mind went 259. (c) The people were sitting beside the fire and eating
blank and it did not occur to him to take photos. hungrily. They were huddling round a fire means being
244. (d) As we see in the passage that the young man was close together. Thus option (c) is the answer.
disappointed in the morning when he had come in 260. (d) In the passage the author talks about not shooting
casual dress and had not received required attention
the elephant because he feels it is a heavy and useful
from the salesmen. So he came back dressed in suit as
machinery. This phrase denotes that that the author
he wanted attention from the salesmen. Thus we see
considers it to be an important resource or property.
that option (d) is the best explanation to this question.
245. (c) Salesmen generally give attention to rich customers. Thus option (d) is the answer.
This thing is proved by the fact that the salesman had 261. (b) The author denotes in the passage that killing a
given him attention when he had arrived well dressed working elephant is like destroying a useful machine.
in the shop. Thus option (c) is the correct answer. Here the work done by elephant is compared to a heavy
246. (a) From the passage, we get the idea that young man had machinery. Thus option (b) is the correct answer.
already made up his mind not to buy anything as 262. (a) The author specifically compares the elephant to a
respect and attention was what he was asking for. Thus cow here because although an elephant is wild animal,
we can say that option (a) is the answer. it was as quiet as a cow at that moment. Thus option
247. (b) It is indicated in the passage that after the customer (a) is the answer.
was not buying anything the salesman had drawn the 263. (b) In this item the author says in the second sentence
attention of the manager towards the young man. This that he was very fond of the old soldier and inspired
is the reason why manager intervened. Thus option by his adventurous stories. Thus option (b) is the
(b) is the correct answer. answer.
248. (c) When the manger had asked the man, what he wanted, 264. (d) The author said that story of the soldier in which he
he replied by saying that he only came to assert his walked 200 miles was his favourite because he had
respect back. Thus, in this question, the correct option escaped from Japanese prison of war camp. He told it
would be (c). to the man many a times Thus, option (c) is the correct
249. (c) The prisoner had been punished for robbery. This is answer it shown he is a strong soldier.
suggested in the last line of the passage, where the 265. (d) It has been mentioned by the author that he liked the
prisoner himself says that the person he robbed is still story of Japanese prison of war the most. Thus option
a millionaire. Thus option (c) is the answer. (d) is the answer.
250. (b) It has been suggested in the passage that the robber 266. (b) The passage discusses the author's life that though
is in the prison for three years since the time when he he had to face so many troubles, he was satisfied with
committed robbery. Thus option (b) three years earlier his life. Thus option (b) is the answer.
is the answer. 267. (c) In the passage we notice that the soldier was bitten by
251. (b) In the passage, it denotes that the warden had come a snake and he had cut his toe but the toe turned
to unlock the padlock. The prisoner wanted to deceive septic because of which he had to cut his leg. Thus
the warden and get out of the jail. Thus it is clear that option (c) is the correct answer.
option (b) is the answer. 268. (d) In the passage we find that crowd lost its patience
252. (b) Of all the adversities that the prisoner had to deal when the speaker was not able to reach the venue as
with, mental strain of those black days was the worst. he was ill.
He mentioned it separately. Thus option (b) is the 269. (c) Assuaging means reducing or abating thus option c
answer. is the answer.
EBD_7367
202
B- Reading Comprehension

270. (c) We see in the passage that people shouting and 295. (d) Specialisation and exchange began when men started
heckled when the speaker did not arrive. living in communities.
271. (c) When the teaching was associated with punishment, it 296. (d) Exchange of goods and services is very important
instilled fear in the minds of the students. because we cannot produce and/or specialize in
272. (a) When the teaching was associated with reward, it gave producing everything we need.
importance to only the best student and the slow pupil 297. (d) Scientists who study soil believe that not all worms
was left behind. and bacteria are harmful. Some are useful as they helps
273. (b) The system which appealed to the competitive spirit in in mixing up of nitrogen and air in the soil, while some
the pupils was largely based on marks. help break down the decayed matter.
274. (c) In the jungle, the snake has the added advantage over 298. (c) The harmful pests and bacteria causes diseases in the
man. He is not obstructed by the thorns, trees and plants.
bushes as faced by the man. 299. (a) While farming, the farmers makes sure to avoid the
275. (d) Movement of other beings causes small vibrations in weeds and pests that attack plants.
the ground which the snake uses to decipher the danger 300. (b) With the advent in farming methods and technology,
276. (a) Bearing upon means having an effect on. resistant seeds which fight weeds and pests are easily
277. (a) There is a close relationship between ambition and available that help farmers in controlling the loss caused
activity. Ambition is the inspiration for activity. by them.
278. (c) Ambition must be consistent with one's capacities. If 301. (b) Nowadays, farmers can grow crops that ripen quickly
the capacities do not match with the ambition, then it and are resistant to frost, drought, disease.
results in failure. 302. (d) The noise was made by the young boy who was sitting
279. (a) One must try to do their best in everything they do so with his mother in front of the author.
as to achieve good results. 303. (c) Of all the people in the scene, the least bothered was
280. (b) One should not imagine oneself always to do be better the old man who was snoring to himself.
than the others. Others should be treated as equal and 304. (b) The people in the scenario have a common factor that
one must concentrate on one's capacities instead of they were waiting for something and were engaged in
competing with others. doing different tasks to pass time.
281. (d) The early instruments made were not delicate and were 305. (a) The little boy with the lowest patience level had gone
sensitive enough. weary of waiting and was the most bored. He was
282. (b) Earthquake comes with a lot of destruction and takes a making noises to get entertained.
toll on infrastructure and people's lives. An instrument 306. (b) The man was troubled by the noise made by the
to observe an earthquake is the need of the hour so as continuous beating of hammers of the blacksmiths.
to alert people about earthquakes beforehand during 307. (d) The man gave the blacksmiths `100 each as a bribe so
their conscious as well as unconcious hours. that they find new huts and he doesn't get disturbed.
283. (d) A device was needed that could record the parameters 308. (c) The man after giving the blacksmiths money to find
with a pen and paper. new huts was shocked to listen to the hammer noise
284. (c) While travelling in a bus or train, a standing person the next day again and went outside to find why they
has a tendency to fall when a bus or train moves hadn't found new huts.
suddenly. 309. (b) The man got to know that the blacksmiths have found
285. (b) According to the early seismometers, the pen should new home and kept their promise as they both
stay still and the paper should move. exchanged their huts.
286. (b) The author first played cricket when he was a school 310. (d) The tigress was miles away and according to how fast
boy. she runs, it is a possibility that she would cover the
287. (a) The author explains how he felt strange to the game of distance in half an hour.
cricket when he played it for the first time. 311. (b) The author says that some of us won't reach the camp
288. (c) The words butter fingers describes that how the means that the tigress would kill few of them if she
author's always dropped the ball. would hear them.
289. (b) Spectator means a person who watches at a show, game, 312. (d) The author found it difficult to decide the question
or other event this the correct option is onlookers. because there was uncertainty whether or not she will
290. (b) According to the author, regressing back means looking hear them.
back at the text that has already been read. 313. (a) The author had the time of whole day to kill the tigress
but before the night they should be back to their camps.
291. (c) In order to be a good reader, we must not look back
314. (c) The author says that if the tigress comes in the daytime
frequently and instead grasp quickly what we are
after hearing his voice, he would be able to shoot her
reading to avoid regression during the reading and
down without any difficulty.
enjoying a spurt in reading speed.
315. (b) The flat did not suit him much because unlike before,
292. (b) The author describes regression as walking back a few
he was staying alone without his mother in the house.
meters while walking i.e looking back at the text already
316. (a) The author had put all the items and furniture of his
read again and again.
utility into one room and never looked after the rest of
293. (c) In today's time, very few people can satisfy their needs the flat because he did not use it.
all by themselves. 317. (d) The word 'it' refers to the flat.
294. (b) Exchange of goods is possible when we produce a good 318. (c) The author was satisfied with the space in his bedroom
in surplus such that apart from our personal use, we and made sure it had all the things of his utility.
can share it. 319. (a) Be at a loose end means to have nothing to do.
Reading Comprehension B-203
320. (c) According to the passage, most of us prefer to watch 375. (a) The contextual meaning of the phrase 'watertight
films that satisfy and/or fulfil our secret wishes. compartments' is 'the activities of life unaffected by
321. (b) By watching thrilling adventures, we make up for our public opinion'.
shortcomings. 376. (d) The author says that the politicians often talk about
322. (d) Film stars present situations that are uncommon to our introducing certain political and economic changes.
daily life and which excite us. However, the author contradicts their view as the
323. (a) We somehow admit to the fact that we are weak and efficiency of an institution depends upon other factors
plain and we try to find adventure by watching movie as well. Hence, option (d) is the correct answer.
stars. 377. (a) In order to make earth a paradise to live in, everything
324. (b) The daily life of school, officegoers and home-makers from political, moral, social and economic well being
make up for the same old repetition of routine. should be perfectly synchronized with one another.
325. (c) 'Rhapsodical' means enthusiastic expression of feeling. Hence, option (a) is the correct answer.
326. (b) 'Mona lisa' is the name of a famous painting by Italian 378. (c) It is clearly mentioned in the passage that men talk
artist Leonardo da Vinci. about the wonderful feeling that they experience after
327. (d) The truth about the 'Mona Lisa' is that it is a study in reaching the top of mountain after climbing for hours
modeling. and sometimes even days. Hence, the correct answer
328. (a) 329. (d) 330. (b) 331. (c) 332. (c) is option (c).
333. (b) 334. (a) 355. (c) 379. (d) The passage mentions that it is difficult to climb
336. (b) Graphology is the study of handwriting. mountains as paths sides usually very steep. Moreover,
337. (d) 338. (c) 339. (d) 340. (d) some mountain sides are straight up and down. Hence,
341. (b) 342. (c) 343. (d) 344. (a) option (d) is the correct answer.
345. (b) 346. (b) 347. (b) 348. (d) 380. (d) In the sentence, ' It' refers to the act of climbing a
349. (c) dogmatic means inclined to lay down principles as mountain i.e. mountaineering. Hence, option (d) is the
undeniably true or unquestionable. Therefore in the correct answer.
given context dogmatic statement means a statement 391. (d) The last few lines of the passage mentions that two
which is unquestionable. Italian prisoners escaped the camp and climbed to the
350. (d) 351. (d) 352. (a) 353. (b) 354. (a) top of Mount Kenya to get the feeling of freedom.
355. (a) 356. (b) 357. (c) 358. (d) 359. (a) Hence, option (d) is the correct answer.
360. (b) 361. (b) 362. (d) 363. (d) 364. (c) 382. (d) It is clearly mentioned in the paragraph that great
365. (c) conquerors, generals and soldiers are glorified often
366. (b) It is very clearly mentioned in the passage how power in history books. Hence, option (d) is the correct answer.
of out has exerted its influence through the ages. Hence, 383. (d) From the passage, it can be inferred that the war fighters
option (b) is the correct answer. i.e. the conquerors, generals and soldiers contributed
367. (d) The statement suggests that the theory of relativity more to the destruction and killing and less to
has suppressed the theory of gravitation i.e. the theory civilization. Hence, option (d) is the best choice to
of gravitation has been replaced by the theory of pick.
relativity. Hence option (d) is the correct answer. 384. (b) The author points out in the passage that people think
368. (b) The passage clearly states that the achievements of a great deal of the conquerors, generals or soldiers, so
these people had importance only in their own time. much so that on all the hightest pillars in the great
Hence option (b) is correct answer. cities of the world, one will find the figure of one of
369. (b) The last few lines of the passage clearly suggest that these. Hence option (b) is the correct answer.
the power of art increases with time and is better 385. (a) In the passage, it is started that most people believe
understood by people. Hence, option (b) is the correct that the greatest countries are those, who have
choice. defeated other countries in battle and ruled over them.
370. (c) It is clearly mentioned in the passage how each gender Hence, option (a) is the correct answer.
can surpass the other in certain kinds of activities. 386. (b) The word ‘wearily’ means exhausted.
Hence, option (c) is the correct answer. 387. (d) The answer is given in the second and third sentences
371. (b) The very first line of the passage states that the word of the passage.
'stronger' can be interpreted in many ways. Hence, 388. (b) ‘This wings were ragged bars of lead’ means his wings
option (b) is the correct answer. were damaged and therefore very heavy.
372. (c) The author, in the passage, talks about how in certain 389. (d) The answer is given in the last sentences of the
activities, women are stronger than men. Most woman passage.
live longer than most men, they have better chance of 390. (d) The answer is given in last two sentences of the
resisting disease and work accurately under passage.
monotonous conditions etc. Hence, option (c) is the 391. (a) It can be inferred from the passage.
correct answer. 392. (a) ‘It’ here, refers to ‘travelling in a car’.
373. (d) Many instances are mentioned where women are 393. (a) By ‘sense of presence’, the author refers to his own
stronger than men. The instances appears to be based self as a part of the scene.
on statistics. Hence, option (d) in the correct answer. 394. (a) The word ‘over whelming’ means ‘very strong’.
374. (c) It is clearly suggested in the passage that the efficiency 395. (c) The answer is given by third sentence of the passage.
of the institution depends on the way it is worked and 396. (b) The answer is given in sixth sentence of the passage.
the character and wisdom of the men who work it. 397. (d) It can be inferred from the passage that the traveller
Hence, option (c) is the correct answer. was nervous.
EBD_7367
204
B- Reading Comprehension

398. (d) ‘Struck dumb’ is an expression to indicate a state of 426. (a) Cultural identiy is important because a sense of
absolute shock or surprise. cultural identity gives people self-respect and
399. (a) It can be inferred after reading the passage that Yuri confidence.
was calm and courageous. 427. (c) ‘Poor people become margenalised’ can be explained
400. (a) Only the first assumption has been made in the first in other words as ‘they are the most ignored elements
and second sentences of the passage. of the society’.
401. (b) After reading the passage, it is clear that the writer 428. (d) The tone of the passage is objective and critical.
has expressed his viewpoint about how development 429. (a) It is clearly mentioned in the last line of the passage
has done more harm to environment than benefits to that over-population accentuates poverty, and the
mankind.
country's stark poverty itself is in many areas a major
402. (b) The action of clearing a wide area of trees is called
cause of over-population.
‘deforestation’ which is referred in the first sentence
of the passage. 430. (b) The tone of the passage is sombre as the passage
403. (d) Greater fuel consumption and pollution are some of conveys a feeling of deep seriousness and sadness.
the outcomes of industrial development according to 431. (c) It is clearly mentioned in the passage that over-
the writer. population has largely diluted the fruits of the
404. (a) The answer is given in first, second and third remarkable economic progress of the country.
sentences of the passage. 432. (d) It is clearly stated in the last line of the passage.
405. (c) Only sentences 1 and 3 can be inferred after reading 433. (d) Dilute means to weaken or make thinner. Consolidated
the passage. It is nowhere mentioned in the passage will be the correct antonym as it means to make stronger
that huge man-made structure did not appeal Tolstoy. or more solid.
406. (a) The answer is given in fifth sentence of the passage. 434. (a) This fact is clearly mentioned in the passage.
407. (c) The answer is given in last three sentences of the 435. (b) It is clearly mentioned in the third line of the
passage. passage.
408. (b) ‘Intrigued’ means fascinated. 436. (b) It is clearly mentioned in the first and third line of the
409. (a) Ice shelves are huge pieces of chunks of ice. passage.
410. (a) Icebergs are huge chunks of ice floating on water. 437. (b) It is mentioned in the third line of the passage that
411. (a) When the iceberg turns upside down, it appears green. jelly slug extrudes a sticky translucent coating that
412. (a) The use of science has helped man in building shelter may foul the mouth-parts of marauding ants.
and making carts, boats, etc.
438. (b) The main idea of the passage is that caterpillars are
413. (b) Nature now needs to be protected because man is
prone to attack thus, they try various things to protect
rapidly destroying nature.
414. (d) Forest are being destroyed in order to provide themselves.
necessities as well as needless comforts and pleasures. 439. (b) 'Orient' means the countries of the East and 'Occident'
415. (c) The evil effect of destroying nature is seen in the means the countries of the West hence, option (b) is
unfavourable changes in climate. the correct answer.
416. (d) Climatic changes and soil erosion are results of 440. (d) It is clearly mentioned in the passage that there was
deforestation. not much difference between Indian and Europe in the
417. (b) ‘Civil liberties’ helps women most achieve equality 12th and 13th century but, the differences got
with men. intensified by the process of Industrialisation.
418. (d) The author talks about the maid servant in the hotel 441. (b) The second half of the passage clearly states that
lobby to show that Economic independence is Industrialisation has proved to be a curse as "it is
necessary for women’s liberation. destroying the life of the mind, which is in process
419. (a) In paragraph four, the writer says that some women ..............civilisation collapse".
would not like to work in the factory. 442. (d) The last few lines of the passage clearly state this.
420. (b) ‘These liberties’ in the first paragraph refers to the 443. (b) The author, in the passage, has tried to explain the
right of women to vote and not to obey. effect of Industrialisation on our civilization thus,
421. (c) In the western societies, the occurrence of polio, option (b) i.e. impact of Industrialisation on our
diphtheria and tuberculosis has ‘decreased’. civilization is the central idea of the passage.
422. (b) More death are now associated with old age than in
444. (c) It is clearly mentioned in the very first line of the
the past because deadly diseases affecting the young
passage.
have been well controlled.
423. (c) The author probably is arguing for rethinking about 445. (d) It is mentioned in the third line of the passage.
the successes and failures of the western medicines. 446. (b) It is mentioned in the fourth line of the passage
424. (a) There are lots of angle ‘to poverty’ comes close to the 447. (a) Evade means to escape or avoid and according to the
meaning of the sentence ‘poverty embraces a whole passage, the lady wished to avoid or ignore the law
range of circumstance’. and boarded the bus with her dog. Option (a) is the
425. (a) ‘Lack of access to information’ affects poor people a correct explanation of the phrase 'wishing to evade
lot as it does not provide them the information about the law'.
how to improve their condition. 448. (c) It is mentioned in the second last line of the passage.
Section-C : General Studies

C HA P T E R
PHYSICS
34
1. Which one of the following Indian scientists was elected as a 10. Assertion (A): In a pressure cooker food is cooked above
fellow of the Royal Society of London for his contribution in boiling point. [2008 -II]
the field of Plant Physiology and Physics ? [2007 -II] Reason (R): Boiling point of water increases as the pressure
(a) Ashutosh Mukherjee increases.
(b) Jagdish Chandra Bose (a) Both A and R are true and R is the correct explanation
(c) Prafulla Chandra Ray of A
(d) P C Mahalanodis (b) Both A and R are true, but R is not the correct
2. Assertion (A): Steam is more harmful for human body than explanation of A
the boiling water in case of burn. [2008 -II] (c) A is true, but R is false
Reason (R) : Boiling water contains more heat than steam. (d) A is false, but R is true
(a) Both A and R are true and R is the correct explanation 11. The phenomenon of mirage occurs due to which one of the
of A following? [2008 -II]
(b) Both A and R are true, but R is not the correct (a) Polarisation of light
explanation of A (b) Dispersion of light
(c) A is true, but R is false (c) Diffraction of light
(d) A is false, but R is true (d) Total internal reflection of light
3. Why is it difficult to see through fog? [2008 -II] 12. Which one of the following common devices works on the
(a) Rays of light suffer total internal reflection from the basis of the principle of mutual induction? [2008 -II]
fog droplets (a) Tubelight (b) Transformer
(b) Rays of light are scattered by the fog droplets (c) Photodiode (d) LED
(c) Fog droplets absorb light 13. Television signal cannot be received generally beyond a
(d) The refractive index of fog is extremely high particular distance due to [2009 -I]
4. The density of water varies with temperature which helps (a) curvature of the Earth
the aquatic animals to live in cold water. At what temperature (b) weakness of antenna
is the density of water maximum? [2008 -II] (c) weakness of signal
(a) 1°C (b) 2°C (d) absorption of signal in air
(c) 3°C (d) 4°C 14. If a small raindrop falls through air [2009 -I]
5. Who gave the first evidence of the big-bang theory? (a) its velocity goes on increasing
[2008 -II] (b) its velocity goes on decreasing
(a) Edwin Hubble (b) Albert Einstein (c) its velocity goes on increasing for sometime and then
(c) S Chandrasekhar (d) Stephen Hawking becomes constant
6. A ray of light is incident on a plane mirror at an angle 30° (d) it falls with constant speed for sometime and then its
with the normal at the point of incidence. The ray will be velocity increases
deviated from its incidence direction by what angle? 15. Consider the following satements [2009 -I]
[2008 -II] X-rays
(a) 30° (b) 60° 1. can pass through aluminum.
(c) 120° (d) None of these 2. can be deflected by magnetic field.
7. What would be the best choice for window material to 3. move with a velocity less than the velocity of ultraviolet
keep the outside heat away? [2008 -II] ray in vacuum.
(a) Single-pane glass Which of the statements given above is/are correct?
(b) Double-pane glass without a gap in between (a) 1,2 and 3 (b) Only 1
(c) Double-pane glass with water filled in between (c) 2 and 3 (d) 1 and 2
(d) Double-pane glass with air in between 16. Which one of the following is the permissible level of noise
8. Bar is a unit of which one of the following? [2008 -II] in a silent zone at day time? [2009 -I]
(a) Force (b) Energy (a) 50 dB (b) 60 dB
(c) Pressure (d) Frequency (c) 65 dB (d) 75 dB
9. On which one of the following conservative laws, does a 17. By which one of the following, an old written material
rocket work? [2008 -II] which cannot be read easily, can be read [2009 -I]
(a) Mass (b) Energy (a) g-rays (b) X-rays
(c) Linear momentum (d) Angular momentum (c) IR-rays (d) Radiofrequency waves
EBD_7367
2
C- Physics
18. Earthquake (shock) waves are [2009 -II] 29. Raw mangoes shrivel when pickled in brine. The
(a) infrasonic waves (b) ultrasonic waves phenomenon is associated with [2010 -II]
(c) seismic (d) infrared waves (a) osmosis
19. Laser is a device to produce [2009 -II] (b) reverse osmosis
(a) a beam of white light (b) coherent light (c) increase of surface tension of fluid
(c) microwaves (d) X-rays (d) decrease of surface tension of fluid
20. A perfect black body has the unique characteristic feature 30. The apparent weight of a steel sphere immersed in various
as [2009 -II] liquids is measured using a spring balance. The greatest
(a) a good absorber only reading is obtained for the liquid [2010 -II]
(b) a good radiator only (a) having the smallest density
(c) a good absorber and a good radiator (b) having the largest density
(d) neither a radiator nor an absorber (c) in which the sphere was submerged deepest
21. Fat can be separated from milk in a cream separation because (d) having the greatest volume
of [2009 -II] 31. If an object having mass of 1 kg is subjected to a force of 1
(a) cohesive force (b) gravitational force N it moves with [2010 -II]
(c) centrifugal force (d) centripetal force (a) a speed of 1 m/s
22. Which one of the following is not result of surface (b) a speed of 1 km/s
tension? [2009 -II]
(c) an acceleration of 10 m/s2
(a) Vapour formation above the liquid surface
(d) an acceleration of 1 m/s2
(b) Convex shape of liquid meniscus
(c) Liquid rising in a capillary 32. An athlete diving off a high springboard can perform a
(d) Spherical shape of mercury fallen on the floor variety of exercises in the air before entering the water
23. Hair of a shaving brush cling together when the brush is below. Which one of the following parameters will remain
removed from water due to [2010 -I] constant during the fall? [2010 -II]
(a) viscosity (b) surface tension (a) The athlete's linear momentum
(c) friction (d) elasticity (b) The athlete's moment of inertia
24. Consider the following statements [2010 -I] (c) The athlete's kinetic energy
1. Clear sky appears blue due to poor scattering of blue (d) The athlete's angular momentum
wavelength of visible light. 33. The pressure exerted on the ground by a man is greatest
2. Red part of light shows more scattering than blue light [2010 -II]
in the atmosphere. (a) when the lies down in the ground
3. In the absence of atmosphere, there would be no (b) when the stands on the toes of one foot
scattering of light and sky will look black. (c) when the stands with both foot flat on the ground
Which of the statements given above is/are correct? (d) all of the above yield the same pressure
(a) Only 1 (b) 1 and 2 34. A passenger in a moving train tosses a five rupee coin. If
(c) Only 3 (d) All of these the coin falls behind him, then the train must be moving
25. The most familiar form of radiant energy in sunlight that with a uniform [2010 -II]
causes tanning and sunburning of human skin, is called (a) acceleration (b) deceleration
[2010 -I] (c) speed (d) velocity
(a) ultraviolet radiation 35. As the sunlight passes through the atmosphere, the rays
(b) visible radiation are scattered by tiny particles of dust, pollen, soot and
(c) infrared radiation other minute particulate matters present there. However,
(d) microwave radiation when we look up, the sky appears blue during mid-day
26. A boy throws four stones of same shape, size and weight because [2010 -II]
with equal speed at different initial angles with the horizontal (a) blue light is scattered most
line. If the angles are 15°, 30°, 45° and 60°, at which angle (b) blue light is absorbed most
the stone will cover the maximum horizontally? [2010 -I] (c) blue light is reflected most
(a) 15° (b) 30° (d) ultraviolet and yellow component of sunlight combine
(c) 45° (d) 60° 36. The force acting on a particle executing simple harmonic
27. Which one of the following statement is true? [2010 -II] motion is [2010 -II]
(a) Temperatures differing 25° on the fahrenheit (F) scale (a) directly proportional to the displacement and is directed
must differ by 45° on the celsius (C) scale away from the mean position
(b) 0°F corresponds to – 32°C (b) inversely proportional to the displacement and is
(c) Temperatures which differ by 10° on the celsius scale
directed towards the mean position
must differ by 18° on the Fahrenheit scale
(c) directly proportional to the displacement and is directed
(d) Water at 90°C is warmer than water at 202°F
towards the mean position
28. A diffraction pattern is obtained using a beam of red light.
Which one among the following will be the outcome, if the (d) inversely proportional to the displacement and is
red light is replaced by blue light? [2010 -II] directed away from the mean position
(a) Bands disappear 37. Which one of the following statements is correct?
(b) Diffraction pattern becomes broader and further apart [2010 -II]
(c) Diffraction pattern becomes narrower and crowded (a) The angle of contact of water with glass is acute while
together that of mercury with glass is obtuse
(d) No change (b) The angle of contact of water with glass is obtuse,
while that of mercury with glass is acute
Physics 3
C-

(c) Both the angle of contact of water with glass and that Select the correct answer using the codes given below
of mercury with glass are acute (a) Only 4 (b) 1 and 4
(d) None of the above (c) 2 and 3 (d) 2, 3 and 4
38. Why are inner lining of hot water geysers made up of 48. A body has a mass of 6 kg on the Earth; when measured on
copper? [2010 -II] the Moon, its mass would be [2011 -II]
(a) Copper has low heat capacity (a) nearly 1 kg (b) less than 1 kg
(b) Copper has high electrical conductivity (c) less than 6 kg (d) 6 kg
(c) Copper does not react with steam 49. When a ball drops onto the floor it bounces. Why does it
(d) Copper is good conductor of both heat and electricity bounce? [2011 -II]
39. Half portion of a rectangular piece of ice is wrapped with a (a) Newton's third law implies that for every action (drop)
white piece of cloth while the other half with a black one. In there is a reaction (bounce)
this context, which one among the following statements is (b) The floor exerts a force on the ball during the impact
correct? [2011 -I] (c) The floor is perfectly rigid
(a) Ice melts more easily under black wrap (d) The floor heats up on impact
(b) Ice melts more easily under white wrap 50. If the door of a running refrigerator in a closed room is kept
(c) No ice melts at all under the black wrap open, what will be the net effect on the room? [2011 -II]
(d) No ice melts at all under the white wrap (a) It will cool the room
40. In scuba-diving, while ascending towards the water surface, (b) It will heat the room
there is a danger of bursting the lungs. It is because of (c) It will make no difference on the average
[2011 -I] (d) It will make the temperature go up and down
(a) Archimedes' principle 51. The focal length of convex lens is
(b) Boyle's law (a) the same for all colours
(c) Gay-Lussac's law of combining volumes (b) shorter for blue light than for red
(d) Graham's law of diffusion (c) shorter for red light than for blue
41. The blackboard seems black because it [2011 -I] (d) maximum for yellow light
(a) reflects every colour 52. When you pull out the plug connected to an electrical
(b) does not reflect any colour appliance, you often observe a spark. To which property
(c) absorbs black colour of the appliance is this related? [2011 -II]
(d) reflects black colour (a) Resistance (b) Inductance
42. Mr X was advised by an architect to make outer walls of (c) Capacitance (d) Wattage
his house with hollow bricks. The correct reason is that 53. The time period of a simple pendulum having a spherical
such walls [2011 -I] wooden bob is 2 s. If the bob is replaced by a metallic one
(a) make the building stronger twice as heavy, the time period will be [2013 -II]
(b) help keeping inside cooler in summer and warmer in (a) more than 2 s (b) 2s
winter (c) 1 s (d) less than 1 s
(c) prevent seepage of moisture from outside 54. A liquid is kept in a regular cylindrical vessel upto a certain
(d) protect the building from lightning height. If this vessel is replaced by another cylindrical vessel
43. If a ship moves from freshwater into seawater, it will having half the area of cross-section of the bottom, the
[2011 -I] pressure on the bottom will [2013 -II]
(a) sink completely (b) sink a little bit (a) remain unaffected
(c) rise a little higher (d) remain unaffected (b) be reduced to half the earlier pressure
44. Viewfinders, used in automobiles to locate the position of (c) be increased to twice the earlier pressure
the vehicles behind, are made of [2011 -I] (d) be reduced to one-fourth the earlier pressure
(a) plane mirror (b) concave mirror 55. In SONAR, we use [2013 -II]
(c) convex mirror (d) parabolic mirror (a) ultrasonic waves (b) infrasonic waves
45. The cat can survive fall from a height much more than (c) radio waves (d) audible sound waves
human or any other animal. It is because the cat 56. Two identical piano wires have same fundamental frequency
[2011 -II] when kept under the same tension.
(a) can immediately adjust itself to land on all four paws What will happen if tension of one of the wire is slightly
and bend the legs to absorb the impact of falling increased and both the wires are made to vibrate
(b) has elastic bones simultaneously? [2013 -II]
(c) has thick and elastic skin (a) Noise (b) Beats
(d) also gets injury equally with other animals, but has (c) Resonance (d) Non-linear effects
tremendous endurance, resistance and speedy 57. After rising a short distance the smooth column of smoke
recovery from a cigarette breaks up into an irregular and random
46. Fire fly gives us cold light by virtue of the phenomenon of pattern. In a similar fashion, a stream of fluid flowing past
[2011 -II] an obstacle breaks up into eddies and vortices which give
(a) fluorescence (b) phosphorescence the flow irregular velocity components transverse to the
(c) chemiluminescence (d) effervescence flow direction. Other examples include the wakes left in
47. Transformer is a kind of appliance that can [2011 -II] water by moving ships the sound produced by whistling
1. increase power and by wind instruments. These examples are the results
2. increase voltage of [2013 -II]
3. decrease voltage (a) laminar flow of air (b) streamline flow of air
4. measure current and voltage (c) turbulent flow of air (d) viscous flow at low speed
EBD_7367
4
C- Physics

58. Which one among the following correctly defines a unit (c) a blunt knife decreases the pressure for a given force
magnetic pole in SI units? [2013 -II] (d) a blunt knife decreases the area of intersection
It is the pole which when placed in air at a distance of 66. When deep sea fishes are brought to the surface of the
(a) 1 foot from an equal and a similar poled repels it with a sea, their bodies burst. This is because the blood in their
force of 1 pound bodies flows at very [2013 -II]
(b) 1 m from an equal and a similar pole repels it with a (a) high speed (b) high pressure
force of 1 N (c) low speed (d) low pressure
(c) 1 cm from an equal and a similar pole repels it with a 67. A bus travels at a speed of 50 km/h to go from its origin of
force of 1 dyne
its destination at a distance of 300 km and travels at a
(d) 1 metre from an equal and a similar pole repels it with a
force of 1 N/m2 speed of 60 km/h to return to the origin. What is the average
59. Before X-ray examination (coloured X-ray) of the stomach, speed of the bus? [2013 -II]
patients are given suitable salt of barium because (a) 54.55 km/h (b) 55 km/h
[2013 -II] (c) 55.55 km/h (d) 54 km/h
(a) barium salts are white in colour and this helps stomach 68. A device, which is used in our TV set, computer,radio set
to appear clearly for storing the electric charge, is [2013 -II]
(b) barium is a good absorber of X-rays and helps stomach (a) resistor (b) inductor
to appear clearly (c) capacitor (d) conductor
(c) barium salts are easily available 69. Which type/types of pen uses/use capillary action in
(d) barium allows X-rays to pass through the stomach
addition to gravity for flow of ink? [2013 -II]
60. The best colours for a sun umbrella will be [2013 -II]
(a) black on top and red on inside (a) Fountain pen
(b) black on top and white on inside (b) Ballpoint pen
(c) red on top and black on inside (c) Gel pen
(d) white on top and black on inside (d) Both ballpoint and gel pens
61. When ice melts, its [2013 -II] 70. If two conducting spheres are separately charged and then
(a) volume decreases brought in contact [2013 -II]
(b) volume increases (a) the total energy of the two spheres is conserved
(c) volume and mass both decrease (b) the total charge on the two spheres is conserved
(d) volume decreases while mass increases (c) both the total energy and the total charge are conserved
62. In step-down transformer, the AC output gives the (d) the final potential is always the mean of the original
[2013 -II]
potential of the two spheres
(a) current more than the input current
(b) current less than the input current 71. The gas used in a refrigerator is [2013 -II]
(c) current equal to the input current (a) cooled down on flowing
(d) voltage more than the input voltage (b) heated up on flowing
63. A rectifier is an electronic device used to convert (c) cooled down when compressed
[2013 -II] (d) cooled down when expanded
(a) AC voltage into DC voltage 72. The celsius temperature is a/an [2013 -II]
(b) DC voltage into AC voltage (a) relative temperature (b) absolute temperature
(c) sinusoidal pulse into square pulse (c) specific temperature (d) approximate temperature
(d) None of the above 73. Light Emitting Diode (LED) converts [2013 -II]
DIRECTION (Q. 64) : The following question consist of two (a) light energy into electrical energy
statements, Statement I and Statement II. You are to examine (b) electrical energy into light energy
these two statements carefully and select the answers to these (c) thermal energy into light energy
items using the codes given below. (d) mechanical energy into electrical energy
Codes 74. Dual Energy X-ray Absorptiometry (DEXA) is used to
(a) Both the statements are true and statement II is the measure [2013 -II]
correct explanation of statement I (a) spread of solid tumour
(b) Both the statements are true, but statement II is not the (b) bone density
correct explanation of statement I (c) ulcerous growth in stomach
(c) Statement I is true, but statement II is false (d) extent of brain haemorrhage
(d) Statement I is false, but statement II is true 75. The main source of energy in sun is [2013 -II]
64. Statement I: Pulling a lawn roller is easier than pushing it. (a) nuclear fusion (b) nuclear fission
Statement II: Pushing increases the apparent weight and (c) chemical reaction (d) mechanical energy
hence the force of friction. [2013 -II]
76. By what mechanism does scent spread all over the room if
65. It is difficult to cut things with a blunt knife because
[2013 -II] the lid is opened? [2013 -II]
(a) the pressure exerted by knife for a given force increases (a) Pressure in the bottle
with increase in bluntness (b) Compression from the bottle
(b) a sharp edge decreases the pressure exerted by knife (c) Diffusion
for a given force (d) Osmosis
Physics 5
C-

77. Which one among the following is not a correct statement? (c) Newton's third law of motion
[2013 -II] (d) Law of conservation of energy
(a) Cathode rays are negatively charged particles 87. Two layers of a cloth of equal thickness provide warmer
(b) Cathode rays are produced from all the gases covering than a single layer of cloth with double the
(c) Electrons are basic constituents of all the atoms thickness. Why? [2014 - I]
(a) Because of the air encapsulated between two layers
(d) Hydrogen ions do not contain any proton
(b) Since effective thickness of two layers is more
78. Which one among the following colours has the highest (c) Fabric of the cloth plays the role
wavelength? [2013 -II] (d) Weaving of the cloth plays the role
(a) Violet (b) Green 88. A mobile phone charger is [2014 - I]
(c) Yellow (d) Red (a) an inverter
79. Dispersion process forms spectrum due to white light falling (b) a UPS
on a prism. The light wave with shortest wavelength (c) a step-down transformer
[2013 -II] (d) a step-up transformer
(a) refracts the most 89. No matter how far you stand from a mirror, your image
(b) does not change the path appears erect. The mirror is likely to be [2014 - I]
(c) refracts the least (a) either plane or convex (b) plane only
(c) concave (d) convex only
(d) is reflected by the side of the prism
90. The position, relative size and nature of the image formed by
80. Magnetic, electrostatic and gravitational forces come under a concave lens for an object placed at infinity are respectively
the category of [2013 -II] [2014 - I]
(a) non-contact forces (b) contact forces (a) at focus, diminished and virtual
(c) frictional forces (d) non-frictional forces (b) at focus, diminished and real
81. A ray of white light strikes the surface of an object. If all the (c) between focus and optical centre, diminished and virtual
colours are reflected the surface would appear [2013 -II] (d) between focus and optical centre, magnified and real
(a) black (b) white 91. The upper and lower portions in common type of bi-focal
(c) grey (d) opaque lenses are respectively [2014-II]
82. Motion of an oscillating liquid column in a U-tube is (a) concave and convex
(a) periodic but not simple harmonic [2013 -II] (b) convex and concave
(c) both concave of different focal lengths
(b) non-periodic
(d) both convex of different focal lengths
(c) simple harmonic and time period is independent of the 92. Tungsten is used for the construction of filament in electric
density of the liquid bulb because of its [2014-II]
(d) simple harmonic and time period depends on the density (a) high specific resistance
of the liquid (b) low specific resistance
83. Dirty cloths containing grease and oil stains are cleaned (c) high light emitting power
by adding detergents to water. Stains are removed because (d) high melting point
detergent [2013 -II] 93. Inactive Nitrogen and Argon gases are usually used in
(a) reduces drastically the surface tension between water electric bulbs in order to [2014-II]
and oil (a) increase the intensity of light emitted
(b) increases the surface tension between water and oil (b) increase the life of the filament
(c) make the emitted light coloured
(c) increases the viscosity of water and oil
(d) make the production of bulb economical
(d) decreases the viscosity in detergent mixed water 94. In the phenomenon of dispersion of light, the light wave of
84. You are asked to jog in a circular track of radius 35 m. Right shortest wavelength is [2014-II]
one complete round on the circular track, your displacement (a) accelerated and refracted the most
and distance covered by you respectively [2014 - I] (b) slowed down and refracted the most
(a) zero and 220 m (b) 220 m and zero (c) accelerated and refracted the least
(c) zero and 110 m (d) 110 m and 220 m (d) slowed down and refracted the least
85. When an incandescent electric bulb glows [2014 - I] 95. An oscilloscope is an instrument which allows us to see
(a) the electric energy is completely converted into light waves produced by [2014-II]
(b) the electric energy is partly converted into light energy (a) Visible light (b) X-rays
and partly into heat energy (c) Sound (d) Gamma rays
96. The distribution of electrons into different orbits of an atom,
(c) the light energy is converted into electric energy
as suggested by Bohr; is [2014-II]
(d) the electric energy is converted into magnetic energy (a) 2 electrons in the K-orbit, 6 electrons in the L-orbit, 18
86. In cricket match, while catching a fast moving ball, a fielder electrons in the M-orbit
in the ground gradually pulls his hands backwards with (b) 2 electrons in the K-orbit, 8 electrons in the L-orbit, 32
the moving ball to reduce the velocity to zero. The act electrons in the M-orbit
represents [2014 - I] (c) 2 electrons in the K-orbit, 8 electrons in the L-orbit, 18
(a) Newton's first law of motion electrons in the M-orbit
(b) Newton's second law of motion (d) 2 electrons in the K-orbit, 8 electrons in the L-orbit, 16
electrons in the M-orbit
EBD_7367
6
C- Physics
97. Carbon or Graphite rods are used in atomic reactors as (c) Gravitational force has a very short range.
moderators for sustained nuclear chain reaction through (d) Gravitational force is a long range force, while the other
nuclear fission process. In this process [2014-II] two are short range forces.
(a) the neutrons are made fast 104. Creation of something from nothing is against the law of
(b) the protons are made fast (a) constant proportions
(c) the neutrons are made slow (b) conservation of mass-energy
(d) the protons are made slow
98. (c) multiple proportions
(d) conservation of momentum
105. An electron and a proton are circulating with same speed in
distance

circular paths of equal radius. Which one among the


following will happen, if the mass of a proton is about 2,000
times that of an electron ?
(a) The centripetal force required by the electron is about
2,000 times more than that required by the proton
(b) The centripetal force required by the proton is about
time 2,000 times more than that required by the electron
The distance-time graph for an object is shown above. (c) No centripetal force is required for any charged particle
Which one of the following statements holds true for this (d) Equal centripetal force acts on both the particles as
object ? [2014-II] they rotate in the same circular path
(a) The object is moving with uniform speed 106. An object is raised to a height of 3 m from the ground. It is
(b) The object is at rest then allowed to fall on to a table 1 m high from ground level.
(c) The object is having non-linear motion In this context, which one among the following statements
(d) The object is moving with non-uniform speed is correct ?
99. For a harmonic oscillator, the graph between momentum p and (a) Its potential energy decreases by two-thirds its original
displacement q would come out as [2014-II] value of total energy.
(a) a straight line (b) a parabola (b) Its potential energy decreases by one-third its original
(c) a circle (d) an ellipse value of total energy.
DIRECTION (Q. 100): The following question consist of two (c) Its kinetic energy increases by two-thirds, while
statements, Statement I and Statement II. You are to examine potential energy increases by one-third.
these two statements carefully and select the answers to these (d) Its kinetic energy increases by one-third, while
items using the code given below : potential energy decreases by one-third.
Code : 107. Two pieces of conductor of same material and of equal length
(a) Both the statements are individually true and are connected in series with a cell. One of the two pieces has
Statement II is the correct explanation of Statement I. cross-sectional area double that of the other. Which one of
(b) Both the statements are individually true, but the following statements is correct in this regard ?
Statement II is not the correct explanation of (a) The thicker one will allow stronger current to pass
Statement I. through it.
(c) Statement I is true, but Statement II is false. (b) The thinner one would allow stronger current to pass
(d) Statement I is false, but statement II is true. through it.
100. Statement I : When a gun is fired it recoils, i.e., it pushes (c) Same amount of electric current would pass through
back, with much less velocity than the velocity of the bullet. both the pieces producing more heat in the thicker
Statement II : Velocity of the recoiling gun is less because one.
the gun is much heavier than the bullet. [2015-I] (d) Same amount of electric current would pass through
101. In an observation, a-particles, b-particles and g-rays have both the pieces producing more heat in the thinner
same energies. Their penetrating power in a given medium one.
in increasing order will be 108. A wire-bound standard resistor used manganin or
(a) a, b, g (b) b, g, a constantan. It is because
(c) a, g, b (d) b, a, g (a) these alloys are cheap and easily available
102. A person standing 1 m in front of a plane mirror approaches
(b) they have high resistivity
the mirror by 40 cm. The new distance between the person
(c) they have low resistivity
and his image in the plane mirror is
(a) 60 cm (b) 1.2 m (d) they have resistivity which almost remains unchanged
(c) 1.4 m (d) 2.0 m with temperature
103. In respect of the difference of the gravitational force from 109. Which one among the following is used in making lead
electric and magnetic forces, which one of the following pencils ?
statements is true ? (a) Charcoal (b) Graphite
(a) Gravitational force is stronger than the other two. (c) Coke (d) Carbon black
(b) Gravitational force is attractive only, whereas the 110. Sodium metal should be stored in
electric and the magnetic forces are attractive as well (a) Alcohol (b) Kerosene oil
as repulsive. (c) Water (d) Hydrochloric acid
Physics C-7
111. A body is falling freely under the action of gravity alone in 120. Which one of the following statements about bar magnet is
vacuum. Which one of the following remains constant correct ? [2016-I]
during the fall? [2015-II] (a) The pole strength of the north-pole of a bar magnet is
(a) Potential energy (b) Kinetic energy larger than that of the south-pole
(c) Total linear momentum (d) Total mechanical energy (b) When a piece of bar magnet is bisected perpendicular
112. X-rays are [2015-II] to its axis, the north and south pole get separated
(a) deflected by an electric field but not by a magnetic (c) When a piece of bar magnet is bisected perpendicular
field to its axis two new bar magnets are formed
(b) deflected by a magnetic field but not by an electric (d) The poles of a bar magnet are unequal in magnitude
field and opposite in nature
(c) deflected by both a magnetic field and an electric field 121. A person rings a metallic bell near a strong concrete wall.
(d) not deflected by an electric field or a magnetic field He hears the echo after 0.3 s. if the sound moves with a
113. The focal length of the lens of a normal human eye is about speed of 340 m/s, how far is the wall from him? [2016-I]
[2015-II] (a) 102 m (b) 11 m
(a) 25 cm (b) 1 m (c) 51 m (d) 30 m
(c) 2.5 mm (d) 2.5 cm 122. The rate of change of momentum of a body equal of the
114. Newton's laws of motion do not hold good for objects resultant : [2016-I]
[2015-II] (a) energy (b) power
(a) at rest (c) force (d) impulse
(b) moving slowly 123. The SI unit of mechanical power is : [2016-I]
(c) moving with high velocity (a) Joule (b) Watt
(d) moving with velocity comparable to velocity of light (c) Newton-Second (d) Joule-Second
115. Which one of the following statements is not correct? 124. The spring constant of a spring depends on its [2016-II]
[2015-II] (a) length only
(a) Weight of a body is different on different planets. (b) material only
(b) Mass of a body on the earth, on the moon and in (c) length and its diameter
empty space is the same. (d) thickness, its diameter and its material
(c) Weigh tlessness of a body occurs when the 125. In a bipolar junction transistor [2016-II]
gravitational forces acting on it are counter-balanced. (a) all the three regions (the emitter, , the base and the
(d) Weight and mass of a body are equal at sea level on collector) have equal concentrations of impurity
the surface of the earth. (b) the emitter has the least concentration of impurity
116. A brick is thrown vertically from an aircraft flying two (c) the collector has the least concentration of impurity
kilometres above the earth. The brick will fall with a (d) the base has the least concentration of impurity
[2015-II] 126. The pressure exerted by a 760 mm column of mercury at 0°C
(a) constant speed is known as [2016-II]
(b) constant velocity (a) 1 pascal (b) 1 atmosphere
(c) constant acceleration (c) 1 bar (d) 1 poise
(d) constant speed for some time then with constant 127. Which one of the following physical quantities has the
acceleration as it nears the earth same dimensions as that of Planck's constant h? [2016-II]
117. The outside rearview mirror of modern automobiles is marked (a) Linear momentum (b) Angular momentum
with warning "objects in mirror are closer than they appear". (c) Force (d) Potential energy
Such mirrors are [2015-II] 128. In total internal reflection, the light travels from [2016-II]
(a) plane mirrors (a) rarer to denser medium and it occurs with no loss of
(b) concave mirrors with very large focal lengths intensity
(c) concave mirrors with very small focal lengths (b) denser to rarer medium and it occurs with no loss of
(d) convex mirrors intensity
118. Which one of the following statements is not correct ? (c) rarer to denser medium and it occurs with loss of
[2016-I] intensity
(a) Sounds wave in gases are longitudinal in nature (d) denser to rarer medium and it occurs with loss of
(b) Sounds wave having frequency below 20 Hz are known intensity
as ultrasonic waves 129. The energy of a photon, whose momentum is 10 MeV/ c,
(c) Sounds waves having higher amplitudes are louder where c is the speed of light, is given by [2016-II]
(d) Sound waves with high audible frequencies are sharp (a) 10 MeV (b) 100 MeV
119. A myopic person has a power of 1.25 dioptre, what is the (c) 1 MeV (d) 0.1 MeV
focal length and nature of his lens ? [2016-I] 130. Liquids and gases never show [2016-II]
(a) 50 cm and convex lens (a) diamagnetic property
(b) 80 cm and convex lens (b) paramagnetic property
(c) 50 cm and concave lens (c) ferromagnetic property
(d) 80 cm and concave lens (d) electromagnetic property
EBD_7367
C-8 Physics

131. Light travels in a straight line (rectilinear propagation of (a) C1 > C2 (b) C2 > C1
light). This statement does hold if the medium of travel for (c) C1Cra = C2Crg (d) C1 = C2
light is [2016-II] 140. A photon of X-ray has energy of 1 keV. A photon of visible
(a) of variable refractive index radiation has energy of 3 eV. In this context, which one of
(b) made up of slabs of different refractive indices the following statements is not correct? [2017-I]
(c) homogeneous and transparent (a) The wavelength of X-ray photon is less than the
(d) inhomogeneous and transparent wavelength of visible radiation photon.
132. When a piece of pure silicon is doped with aluminium, then (b) Both the photons have different energies.
(a) the conductivity of the doped silicon piece will remain (c) The speeds of both the photons in vacuum are
the same [2017-I] different.
(b) the doped silicon piece will become n-type (d) The frequency of X-ray photon is higher than the
(c) the doped silicon piece will become p~type frequency of visible radiation photon.
(d) the resistivity of the doped silicon piece will increase 141. The optical phenomenon that is primarily responsible for
133. Suppose voltage V is applied across a resistance R, The the observation of rainbow on a rainy day is [2017-I]
power dissipated in the resistance is P. Now the same voltage (a) diffraction (b) interference
V is applied across a parallel combination of three equal (c) dispersion (d) reflection
resistors each of resistance R. Then the power dissipated 142. Which one of the following statements is not correct?
in the second case will be [2017-I] [2017-I]
(a) P (b) 3P (c) P/3 (d) 2P/3 (a) The cathode rays originate from cathode and proceed
134. Which one of the following is not a semiconductor? towards the anode in a cathode-ray discharge tube.
[2017-I] (b) The television picture tubes are nothing but cathode-
(a) Silicon (b) Germanium ray tubes.
(c) Quartz (d) Gallium arsenide (c) The cathode rays themselves are not visible.
135. A parallel-plate capacitor, with air in between the plates, (d) The characteristics of cathode rays depend upon the
has capacitance C. Now the space between the two plates nature of the gas present in the cathode-ray tube.
of the capacitor is filled with a dielectric of dielectric constant 143. The wires are made having same length l and area of cross-
7. Then the value of the capacitance will become [2017-I] section A. Wire 1 is made of copper and wire 2 is made of
C aluminium. It is given that the electrical conductivity of
(a) C (b) copper is more than that of aluminium. In this context, which
7
(c) 7C (d) 14C one of the following statements is correct? [2017-II]
136. For which one of the following does the centre of mass lie (a) The resistance of wire 1 will be higher than that of wire
2.
outside the body? [2017-I]
(b) The resistance of wire 2 will be higher than that of wire
(a) A fountain pen (b) A cricket ball
1.
(c) A ring (d) A book
(c) The resistance of both the wires will be the same.
137. Consider the electromagnetic radiations having
(d) If same current is flown through both the wires, the
wavelengths 200 nm, 500nm and 1000 nm. Which
power dissipated in both the wires will be the same.
wavelength(s) of the following can make visual sensation
144. A ray of light is incident on a plane mirror at an angle of 40°
to a human eye? [2017-I]
with respect to surface normal. When it gets reflected from
(a) 200 nm and 500 nm (b) 500 nm and 1000 nm
the mirror, it undergoes a deviation of [2017-II]
(c) 500 nm only (d) 200 nm and 1000 nm
(a) 40° (b) 100°
138. A copper wire of radius r and length l has a resistance of R. (c) 90° (d) 80°
A second copper wire with radius 2r and length l is taken 145. Infrared, visible and ultraviolet radiations/light have different
and the two wires are joined in a parallel combination. The properties. Which one of the following statements related
resultant resistance of the parallel combination of the two to these radiations/light is not correct? [2017-II]
wires will be [2017-I] (a) The wavelength of infrared is more than that of
5 ultraviolet radiation.
(a) 5R (b) R
4 (b) The wavelength of ultraviolet is smaller than that of
4 R visible light.
(c) R (d) (c) The photon energy of visible light is more than that of
5 5
139. A parallel-plate capacitor of capacitance C1 is made using infrared light.
two gold plates. Another parallel-plate capacitor of (d) The photon energy of ultraviolet is lesser than that of
capacitance C2 is made using two aluminium plates with visible light.
same plate separation, and all the four plates are of same 146. After using for some time, big transformers get heated up.
area. If rg and ra are respectively the electrical resistivities This is due to the fact that [2017-II]
of gold and aluminium, then which one of the following 1. current produces heat in the transformers
relations is correct? [2017-I] 2. hysteresis loss occurs in the transformers
3. liquid used for cooling gets heated
Physics C-9

Select the correct answer using the code given below. 156. Sound waves cannot travel through a [2018-I]
(a) 1 only (b) 2 and 3 only (a) copper wire placed in air
(c) 1 and 2 only (d) 1, 2 and 3 (b) silver slab placed in air
147. A person is standing on a frictionless horizontal ground. (c) glass prism placed in water
How can he move by a certain distance on this ground? (d) wooden hollow pipe placed in vacuum
[2017-II] 157. Which one of the following is the value of one nanometer?
(a) By sneezing (b) By jumping [2018-I]
(c) By running (d) By rolling (a) 10–7 cm (b) 10–6 cm
148. Which one of the following types of glasses is used for (c) 10–4 cm (d) 10–3 cm
making optical instruments? [2017-II] 158. Consider the following statements : [2018-I]
(a) Pyrex glass (b) Soft glass 1. There is no net moment on a body which is in
(c) Hard glass (d) Flint glass equilibrium.
149. Joule-Thomson process is extremely useful and economical 2. The momentum of a body is always conserved.
for attaining low temperature. The process can be 3. The kinetic energy of an object is always
categorized as [2017-II] conserved.
(b) isobaric process Which of the statements given above is/are correct?
(b) isoenthalpic process (a) 1, 2 and 3 (b) 2 and 3 only
(c) adiabatic process (c) 1 and 2 only (d) 1 only
(d) isochoric process 159. Working of safety fuses depends upon [2018-I]
150. Ultrasonic waves are produced by making use of 1. magnetic effect of the current
[2017-II] 2. chemical effect of the current
(a) ferromagnetic material (b) ferrimagnetic material 3. magnitude of the current
(c) piezoelectric material (d) pyroelectric material 4. heating effect of the current
151. A person thrown an object on a horizontal frictionless plane Select the correct answer using the code given below.
surface. It is noticed that there are two forces acting on this (a) 1, 2, 3 and 4 (b) 1, 2 and 3 only
object– (i) gravitational pull and (ii) normal reaction of the (c) 3 and 4 only (d) 4 only
surface. According to the third law of motion, the net 160. Which of the following represents a relation for ‘heat lost =
resultant force is zero. Which one of the following can be heat gained’? [2018-I]
said for the motion of the object? [2017-II] (a) Principle of thermal equilibrium
(a) The object will move with acceleration. (b) Principle of colors
(b) The object will move with deceleration. (c) Principle of calorimetry
(c) The object will move with constant speed, but varying (d) Principle of vaporization
direction. 161. Two metallic wires made from copper have same length but
(d) The object will move with constant velocity. the radius of wire 1 is half of that of wire 2. The resistance of
152. Desalination of seawater is done by using reverse osmosis. wire 1 is R. If both the wires are joined together in series, the
The pressure applied to the solution is [2017-II] total resistance becomes [2018-I]
(a) larger than osmotic pressure R
(b) smaller than osmotic pressure (a) 2R (b)
2
(c) equal to osmotic pressure
5 3
(d) equal to atmospheric pressure (c) R (d) R
153. Which one of the following elements is used as a timekeeper 4 4
162. When the Sun is near the horizon during the morning or
in atomic clocks? [2018-I]
evening, it appears reddish. The phenomenon that is
(a) Potassium (b) Caesium
responsible for this observation is [2018-I]
(c) Calcium (d) Magnesium
(a) reflection of light (b) refraction of light
154. Why is argon gas used along with tungsten wire in an
(c) dispersion of light (d) scattering of light
electric bulb? [2018-I]
163. A wire of copper having length l and area of cross-section
(a) To increase the life of the bulb
A is taken and a current I is flown through it. The power
(b) To reduce the consumption of electricity
dissipated in the wire is P. If we take an aluminium wire
(c) To make the emitted light colored
having same dimensions and pass the same current through
(d) To reduce the cost of the bulb
it, the power dissipated will be [2018-I]
155. Which one of the following is the correct relation
(a) P (b) < P (c) > P (d) 2P
between the Kelvin temperature (T) and the Celsius
164. The pressure of a fluid varies with depth h as P = P0 + rgh,
temperature (tc)? [2018-I]
where r is the fluid density. This expression is associated
(a) These are two independent temperature scales
with [2018-I]
(b) T = tc
(a) Pascal’s law (b) Newton’s law
(c) T = tc – 273.15
(c) Bernoulli’s principle (d) Archimede’s principle
(d) T = tc + 273.15
EBD_7367
C- 10 Physics

HINTS & SOLUTIONS


1. (b) Jagdish Chandra Bose was elected as a fellow of the 20. (c) A perfect black body emits and absorbs all colours of
Royal Society of London for his contribution in the light. A good absorber is a good emitter.
field of Plant Physiology and Physics. 21. (c) Centrifugal force separates fat from milk.
2. (c) Steam burns are more serious than burns from boiling 22. (a)
water even though both are at 100°C becasue when 23. (b) When brush is taken out, the water film is formed
steam condenses it give up very high heat energy i.e. between the hairs and they contract due to surface
additional 22.6 × 105 joule of heat (latent heat of tension.
vapourisation) compared to boiling water. 24. (c) The appearance of blue colour of sky is due to
3. (b) The rays of light are scattered by the fog droplets due scattering of light which is not possible in absence of
to which it become difficult to see through fog. atmosphere.
4. (d) Density of water is maximum at 4°C. 25. (a) The tanning and sun burning of human skin is due to
5. (a) Edwin Hubble gave the first evidence of the big-bang the harmful ultraviolet radiation from the sun.
theory.
6. (b) The given situation can be shown as u 2 sin 2q
26. (c) Horizontal range R =
N g
A B For maximum horizontal range q should be 45°
u2
Rmax =
g
[Q sin 2q = sin 90° = 1]
27. (c) By the relation in different temperature scales
30° 30° C-0 F - 32 K - 273 R - 0
= = =
5 9 5 4
ir We can calculate that temperature which differ by 10°
on the celsius scale must differ by 18° on the fahrenheit
O scale.
By law of reflection, angle of incidence i is equal to
28. (c) Diffraction pattern becomes narrower and crowded
angle of reflection r
together.
Total deviation of ray = 30° + 30° = 60°
29. (a) Raw mangoes shrivel when pickled in brine due to
7. (d) To keep the outside heat away, windows of double-
osmosis i.e. flow of water from higher concentration
pane glass with air in between is the best choice
to lower concentration through a semi-permeable
because air is the bad conductor of heat.
membrane.
8. (c) 1 Bar = 105 Pa. Both bar and Pa are the unit of pressure.
30. (a) When a body immersed in a liquid, then the apparent
9. (c) Rocket works on the principle of conservation of linear
weight of the body depends upon the densities of the
momentum.
liquids and volume of the portion of the solid immersed
10. (a) In a pressure cooker while cooking pressure increases
in liquid. So, the apparent weight of a steel sphere will
due to which boiling point of water also increases as
be greatest for the liquid with the smallest density.
PµT.
11. (d) Total internal reflection is responsible for mirage F
31. (d) As we know, F = ma or a =
formation in deserts. m
12. (b) Transformer is based on the principle of mutual 1N
induction. \a= = 1 m/s2
1kg
13. (a) Curvature of the earth does not allow the television 32. (d) When a athlete jumps from the spring board, he curls
signals to be received beyond a particular distance. his body by rolling his arms and legs in. By doing so,
14. (c) Velocity of raindrop increases until it reaches the he decreases moment of inertia of his body and hence
terminal velocity. angular speed increases to conserve the angular
15. (b) X-rays cannot be deflected by magnetic field as they momentum, as I1 w1 = I2 w2 = constant.
consists of no charge. 33. (b) Pressure is normal force per unit area, therefore, for
X-rays and UV rays are electromagnetic wave, so they lesser value of area pressure is greatest.
move with same velocity in vacuum i.e. 3 × 108 m/s. 34. (a) As the passenger tosses the coin, it goes up and is in
16. (a) The permissible level is 50 dB. motion with the initial speed of the train but if the
17. (c) Infrared rays helps in reading an old written material. train is accelerated, so its speed increases. But the
18. (c) An Earthquake is the result of a sudden release of coin is in the air with initial speed of train, so the train
energy in the Earth's crust that creates seismic waves. slightly moves forward from the coin with the person
19. (b) A laser is a device that emits coherent light through a and coin falls behind him. It happens when train is
process called stimulated emission. moving with accelerated motion.
Physics C-11
52. (a) As the plug is pulled out, the value of resistance in
1
35. (a) As, scattering µ 4 the circuit decreases and hence more current tries to
l flow through the circuit. Pulling out the plug breaks
Since, wavelength of blue colour is least among three the circuit and to complete the circuit for the flow of
primary colours red, green and blue, so violet colour electricity a spark is observed.
scattered most. The scattered light in the sky contains 53. (b) Time period of a simple pendulum
blue colour in plenty and therefore sky appears blue.
36. (c) The restoring force acting on the particle is always l
T = 2p depends on its length 'l ' and acceleration
proportional to the displacement of the particle from g
the equilibrium position and is always directed due to gravity 'g' and independent of mass of the bob.
towards the equilibrium position. 54. (a) Pressure exerted by the fluid column depends on height
37. (a) For ordinary water and glass it is about 8° (acute angle) (h), density r and acceleration due to gravity (g).
and for mercury and glass it is about 135° (obtuse P = hrg
angle). i.e. independent of area of cross-section of the vessel.
38. (d) Copper is a good conductor of both heat an electricity 55. (a) Sonar (sound navigation and ranging) is a technique
due to this it is used in all such appliance where quick that uses sound propagation to navigate (usually
heat transfer is required. under water, as in submarine navigation), communicate
39. (a) Ice under black wrap will melt easily because dark with or detect objects on or under the surface of the
colour (black) absorbs the most of light energy falling water, such as the vessels as ultrasonic waves which
on it. is used having very high frequency > 20,000 Hz.
40. (b) As scuba driver ascends towards the water surface, 56. (b)
water pressure decreases, so according to Boyle's law 57. (c) Turbulent flow is accompanied by random, irregular,
air in his lungs expands to occupy a greater volume. local circular currents called vortices.
Which increase the chances of bursting the lungs. 58. (b)
41. (b) Blackboard absorbs all the colours of light falling on 59. (b) Barium is a good absorber of X-rays and helps
it and does not reflect any colour due to which it stomach to appear clearly.
appears black. 60. (d) As white colour reflects all the components of white
42. (b) The hollow bricks provide thermal insulations; the air light and black colour absorbs all the components of
in hollow bricks, does not allow outside heat or cold white light. So, the best colours for a sun umbrella,
in the house to go out or come in the house. So, it white on top and black on inside.
keeps house cool in summer and warm in winter. 61. (a) When ice melts its density increases i.e. its volume
43. (c) When a body is immersed in a fluid, the fluid exerts an decreases. Mass is an invariant quantity.
upward force on it, called buoyant force. 62. (a) In step-down transformer Np > Ns, Ep > Es, Ip < Is
and buoyant force µ density of fluid.
Here N, E and I used for no. of turns, emf and current
As the density of sea water is higher than that of the
fresh water. respectively.
63. (a) A rectifier is used to convert AC voltage into DC
So extra buoyant force and ship will rise a little higher.
voltage and invertor converts DC voltage into AC
44. (c) In automobiles, convex mirror is used as a rear view
mirror because the image of the vehicle formed is small voltage.
64. (a) It is easier to move any body by pulling it than by
in size due to this the field of view is increased and
pushing it. A body starts to move when the applied
the driver of automobile can see the traffic over large
area. force is more than the frictional force. This frictional
force is proportional to the weight of the body under
45. (a) As the cat falls from a height, it bends the legs to
which it acts. When the lawn roller is pushed by the
absorb the impact of falling and immediately adjust
itself. force (F), the vertical component of the pushing force
now increases the weight (w = mg) of the roller which
46. (c) Fire fly gives us cold light due to chemiluminescence
in turn results in increase of normal reaction and hence
i.e. light produced by chemical reaction.
47. (c) Transformer is an appliance that can increase or the friction is more.
decrease voltage. force
48. (d) Mass of an object does not change when an objects 65. (c) Pressure =
area
location changes. It is the weight of the body (w = mg) Bluntness of knife increases area, hence decreases
which changes with value of 'g' at that place. the pressure for a given force.
49. (b) When a ball drops on to the floor then floor exerts a 66. (b) When deep sea fishes are brought to the surface of
force for small time which is equal to change in the sea, their bodies burst, this is because the blood
momentum of the ball. in their bodies flows at very high pressure.
50. (b) It will heat the room because the heat removed from
the inside is released into the room, counter acting 300 + 300
total distance 300 300
any cooling, plus the energy that is used to operate 67. (a) Average speed = = +
the compressor eventually winds up being heat, total time 50 60
therefore increasing the room temperature.
51. (b) The focal length of a convex lens is shorter for blue æ distance ö
çQ Time = ÷
light than for red. è speed ø
EBD_7367
12
C- Physics

(Circumference of the circular track)


300 + 300 600
= = = 54.55 km/h
6+5 11
68. (c) Capacitor (condenser) is used to store energy in an
electric field. Capacitance of a capacitor C, defined as
the ratio of charge ± Q on each conductor to the
35 m
voltage V between them.
Q
C=
V
69. (a) Fountain pen uses capillary action in addition to 22 22
gravity for flow of ink. = 2´ ´ 35 = 70 ´ = 220m
7 7
70. (b) If two conducting spheres are separately charged and
brought in contact, then the total charge on the two 85. (b) When current flows through the filament of
spheres is conserved. This is the law of conservation incandescent electric bulb, it gets heated up. Soon it
of charge. becomes white hot and starts emitting light.
71. (c) 86. (b) We know from Newton’s second law of motion
72. (b) The degree celsius ( ° C) can refer to absolute dP
Fext = when dt is more Fext will be less.
temperature. dt
73. (b) Light Emitting Diode (LED) is semiconductor device
The greater the rate of change of momentum, the
– a forward bias p-n junction diode which converts
greater is the force and vice-versa.
electrical energy into light energy.
87. (a) Two layers of a cloth are warmer because air trapped
74. (b) Dual Energy X-ray Absorptiometry (DEXA) is used
between the layers acts as thermal insulator restricting
to measure the density or strength of bones.
the heat transfer.
75. (a) The sun is the star at the centre of the solar system. It
88. (c) A mobile phone charger is basically a step-down
generates its energy by nuclear fusion of hydrogen
transformer which converts high voltage to low voltage.
nuclei into helium.
89. (a) Image formed by a plane and a convex mirror is always
2 erect. Concave mirror forms image both erect and
1H + 1H 2 + 1H2 ®2 He4 +1H 1 + 0n1 + 21.6 Mev
inverted depends on the position of the object from
76. (c) Diffusion refers to the process by which molecules the mirror.
intermingle as a result of their kinetic energy of random 90. (a) From the ray diagram, it is very clear that the image of
motion. the object placed at infinity from a concave lens is
77. (d) Hydrogen ion has one proton i.e. , 1H1. Atomic virtual diminished and at the focus of the lens.
number = no. of proton =1 for atom like hydrogen.
78. (d) Different colours in increasing order of their
wavelength: violet, indigo, blue, green, yellow, orange Rays from
and red. the object
79. (a) The light wave with shortest wavelength i.e. violet placed at F
refracts the most. infinity
80. (a) All of these forces doesn’t need any physical contact Concave lens
to exist hence, non-contact forces.
81. (b) When an object reflects all the colours of a white 91. (a) The upper and lower portions in common type of
light, then it would appear white. White light is the bifocal lenses are concave and convex respectively.
combination of all seven colours (VIBGYOR) of light. 92. (d) Tungsten has the highest melting point and lowest
vapour pressure of all metals.
82. (c) Motion of an oscillating liquid column in a U-tube is
93. (b) The inert gas within the bulb prevents the filament
called simple harmonic motion with time period from evaporating.Thus it increases the life of the
independent of on the density of the liquid. filament.
83. (a) One end of hairpin shaped detergents attracted to 94. (b) Short wavelength is slowed down and refracted the
water and other to molecules of oil or grease that most
95. (c) Oscilloscope is an instrument which allows us to see
reduces drastically the surface tension between water
waves produced by sound. The microphone can then
and oil. pick up the sound and convert it to an electrical signal
84. (a) Displacement = Shortest distance between initial and which can be displayed on the oscilloscope.
final point = 0 96. (c) Maximum n of electrons in K,L, M and N is 2, 8, 18 and
Distance travelled by you = 2pr 32 respectively.
Physics C-13
97. (c) Moderators like carbon and Graphite are used for 114. (d) Newton's laws of motion do not hold good for objects
slowing down the neutrons in a nuclear reactor. moving with velocity comparable to velocity of light
Neutrons from fission have very high speeds and
must be slowed greatly by "moderation" to maintain because it does not follow it on this level.
the chain reaction. 115. (d) Mass is the quantity of matter contained by the object.
98. (b) 99. (d) 100. (a) Weight is the force of gravity acting on a body. Mass
101. (a) g - rays are the most penetrating rays is constant anywhere in the universe but weight
b - particles are moderately penetrating and depends upon the quantity of gravity in a particular
a - particles are least penetrating.
102. (b) The distance between person and his image is 60 cm × 2 place.
= 1.2 m 116. (c) An object falling from height will show constant
103. (b) Gravitational force is attractive only whereas the velocity so the acceleration will be constant.
electric and magnetic forces are attractive as well as 117. (d) In convex mirrors, the image is smaller than the object.
repulsive. This mirror provides a wider scope of view so it
104. (b) Creation of something from nothing is against the law becomes easy for vehicles use.
of conservation of mass energy. 118. (b) The sound waves having frequency below 20 Hz are
inaudible. Ultrasonic waves are those waves which
mv 2 have the frequency of above 20,000 Hz.
105. (b) Centripetal force =
r 119. (d) The nature of the lens used to correct myopic eye is a
Given that, concave lens.
Mass of proton = 2000 × mass of electron
Radius and speed is same for both proton and electron 1 1 1 100
\D= = f (m etre) = = =
\ Centripetal force a mass f D 1.25 125
Since mass of proton is 2000 times that of mass of = 0.80 metre = 80 cm
electron, hence the centripetal force required by proton 120. (c) If any magnet is divided into two parts, every part will
is about 2000 times more than that required by the be a new magnet.
electron. 121. (c) Let the distance is x metre.
106. (a) 107. (d) Distance = Velocity × Time
108. (d) Manganin is a trademarked name for an alloy of x = 340 × 0.3 = 102 metre (This is the total distance)
typically 86% copper, 12% manganese, and 2% nickel.
102
Constantan is a copper-nickel alloy usually consisting \ The distance of wall is = 51 metre.
of 55% copper and 45% nickel. Its main feature is its 2
resistivity which is constant over a wide range of 122. (c) Newton's 2nd Law - The rate of Change of momentum
temperatures. Other alloys with similarly low always acts in the direction of resultant force acting
temperature coefficients are manganin. on a body Þ (F = ma) (where m = mass; a =
109. (b) Graphite is used in making lead pencils due to its ability aceeleration)
to leave marks on paper and other objects. Pencil lead 123. (b) Watt is the SI unit of mechanical power.
is most commonly a mix of powdered graphite and clay. 124. (d) F = – kx; k = spring constant and it measures stiffness
110. (b) Sodium metal should be stored in kerosene oil. Sodium or softness of spring.
is a very reactive metal. It is kept in kerosene to prevent Hence. K depends on length, radius and material of
it from coming in contact with oxygen and moisture. If the spring.
this happens, it will react with the moisture present in 125. (a) A Bipolar Junction Transistor, or BJT, is a solid-state
air and form sodium hydroxide. This is a strongly device in which the current flow between two terminals
exothermic reaction, and lot of heat is generated. Thus (the collector and the emitter) is controlled by the
sodium is kept under kerosene. amount of current that flows through a third terminal
111. (d) Mechanical energy is the ability of an object to do (the base).
work. This energy is equal to the sum of kinetic and 126. (b) 76cm Hg = 760 mmHg = 1 atm = 101325 Pa
potential energy, it is always constant. (mmHg = milimeters of mercury)
112. (d) X-rays are electro magnetic radiations. It has (atm = atmosphere)
penetrating ability, so it is used in radiography, airport (Pa = Pascal = N/m2, SI unit for pressure)
railway security and CT scanning. It is not affected by 127. (b) The dimension of Plank's contant can be obtained from
electric and magnetic fields. any of the equations in which it appears, e.g. h = E/n,
113. (a) The focal length of the lens of a normal human eye is l = h/p, etc.
about 2.5 cm. The eye sets focus on a thing by Planck's Constant h = 6.626176 × 10–34 joule-second
adjusting the eye ball. The lens flexes its curvature to = 6.626176 × 10–34 m2 kg/s
focus on the objects.
EBD_7367
14
C- Physics

[h] = [M L2 T–1] _____________(1) the body. The centre of mass of a ring is at its centre
Angular momentum l = r × p which is outside its body.
[l] = [r] [p] 137. (c) Visual sensation or the perceptual experience of
= [L] [M L T–1] seeing an object is related to the visible spectrum,
= [M L2 T–1] _____________(2) which is the region of the electromagnetic radiation
Compare Eqs. (1) and (2). See that both the Planck's that is visible to the human eye. Electromagnetic
constant and angular momentum have the same radiation in this range of wavelengths (390-700 nm) is
dimensions. called visible light. 200 and 1000 nm do not lie in the
128. (b) visible range of wavelength.
129. (a) As per the formula Energy of Photon is always equal l R
to the multiple of the Momentum and Speed of light. 138. (d) R = P The new resistance will be and resultant
A 4
Therefore; P=E/C Where P is Momentum, E is Energy
and C is Speed of Light R
resistance will be
10 MeV/C=E/C ;{P (Momentum) = 10MeV/C; as given 5
in Question} 139. (d) Capacitance is defined as the ability of a system to
E= 10 MeV store electric charge and can be calculated by the
130. (c) Liquids and gases show diamagnetic, paramagnetic formula, C = Q/V, where ‘Q’ is the charge stored on
and electromagnetic property, but it never show each plate and ‘V’ is the applied electric potential
ferromagnetic property. across the plates capacitors. Capacitance does not
131. (c) depend on the electrical resistivities of the material of
132. (c) Aluminium is a p-type dopant, which means that when capacitors.
a semiconductor (silicon in this case) is doped with 140. (c) Electromagnetic radiation consists of photons which
aluminium, it creates a hole (positively charged carrier) move at speed of light within a vacuum.
by accepting an electron from the silicon atom. 141. (c) Rainbow can be seen on a rainy day due to the
Electrical conductivity of semiconductors increase and phenomena of dispersion, which is an optical process
resistivity remains same. involving splitting of white light into its constituent
133. (b) Power (P) across a resistor = V2/R; In the first case, colors while passing through a lighter to heavier
P1 = V2/R1 medium. On a rainy day, white light gets dispersed
In the second case, equivalent resistant of parallel into its seven colors while passing through rain drops.
combination = R/3 142. (d) Cathode ray is a beam of electrons emitted from the
Power in second case (P2) = (V2)/(R/3) = 3V2/R, So, cathode of a vacuum tube. It is invisible. Cathode ray
P1/P2 = 1/3 or P2 = 3P1 tubes are also found in televisions and computer
134. (c) Semiconductors are solid substances that show monitors. Since cathode ray is a beam of electrons
electrical conductivity either with the addition of an and all electrons are identical, charge to mass ratio is
impurity or because of temperature effects. Silicon, same for all the gases and does not depend on the
Germanium, and Galium Arsenide are semiconductors nature of gas.
as they show electrical conductivity depending on 143. (b) The resistance of wire 2 will be higher than that of
the addition of impurity and temperature. Quartz is an wire 1.
insulator. 144. (d) the ray of light will undergo a deviation of 80 degrees.
135. (c) 7C 145. (d) The photon energy of ultraviolet is lesser than that of
visible light.
K Îo A 146. (d) After using for some time, big transformers get heated
Capacitance =
d up. This is due to the fact that current produces heat
[A = Area of each plate. in the transformers, hysteresis loss occurs in the
Îo = Permitivit of free space. transformers, liquid used for cooling gets heated.
147. (a) Sneezing and farting would move a person standing
d = distance between plate.
on the frictionless ground. Any expulsion of fluids
k1 = dielectric constant.]
should propel the man into motion. Sneezing would
Since, K1 = hence new capacitance will be 7C.
result him moving in the direction his back faces and
136. (c) The centre of mass of a body is the point where the
farting would get him forward. The basic principle that
whole mass of a body is con sider ed to be
governance his movement is, in the horizontal plane
concentrated. It may lie within or outside the body of
is that no external force acts on the body so his
an object. Fountain pen, cricket ball and book are all
momentum can’t change.
compact body and their centre of gravity lie within
Physics C-15

148. (c) Hard glass is used for making hard glass laboratory 158. (c) Kinetic energy (KE) is the energy of motion, and kinetic
apparatus. energy is not always conserved in a collision. Kinetic
149. (c) In thermodynamics, the Joule–Thomson effect energy has the equation (1/2)mv2.
describes the temperature change of a real gas or liquid 159. (c) The speed at which a fuse blows depends on how
when it is forced through a valve or porous plug while much current flows through it and the material of which
keeping them insulated so that no heat is exchanged the fuse is made. It therefore depends on magnitude
with the environment. This procedure is called a and heating effect of the current.
throttling process or Joule–Thomson process. Since 160. (c) When the two bodies ar e isolated from their
there is no heat being exchanged it is adiabatic in surroundings, the amount of heat lost by the hot body
nature. must be equal to the amount of heat gained by the
150. (c) Ultrasonic waves are generated by a transducer that cold body. The same principle is used in calorimetry.
includes a piezoelectric crystal that converts electrical As heat is energy in transit, this principle is really just
energy (electric current) to mechanical energy (sound conservation of energy.
waves). 161. (c) Use the equation R = rho * l / A, Where
151. (d) The object will move with constant velocity. R = resistance; l = length; A = cross sectional area, rho
152. (a) Desalination by RO requires the use of an osmotic = resistivity.
membrane. The higher the applied pressure above the
Assume the wires are circular cross section. Wire 1
osmotic pressure, the higher the rate of fresh water
has resistance R. Wire 2 is the same length, but twice
transports across the membranes.
the radius, so 4 times the cross sectional area. So wire
153. (b) The principle of operation of an atomic clock is based 2 has a resistance of R/4.
on atomic physics. It uses the microwave signal that
So total resistance of 2 in series = R + R/4 = 5/4 * R
electrons in atoms emit when they change energy
162. (d) According to Rayleigh''s law, the intensity of scattered
levels. An atomic clock uses an electron transition
light varies inversely as the fourth power of its
frequency in the microwave, optical, or ultraviolet
wavelength. Sunlight consists of seven colours. Of
region of the electromagnetic spectrum of atoms as a
these, red has the maximum wavelength. During sunrise
frequency standard for its timekeeping element.
and sunset, the rays have to travel a larger part of the
Caesium-based atomic clocks use the electromagnetic
atmosphere because they are very close to the horizon.
transitions of caesium-133 atoms as a reference point.
Therefore, light other than red is mostly scattered away.
Caesium is a chemical element with symbol Cs and
Most of the red light, which is the least scattered,
atomic number 55. It is a soft, silvery-gold alkali metal.
enters our eyes. Hence, the sun and the sky appear
154. (a) Argon gas is used in fluorescent and incandescent
red.
light bulbs to stop the oxygen in the light bulbs from
corroding the hot tungsten filament. The use of argon 163. (c) A wire of copper having length l and area of cross
in light bulbs prevents the evaporation of the tungsten section A is taken and a current I is flown through it.
filaments, which results in increased light bulb life. The power dissipated in the wire is P. If we take an
155. (d) Temperature in Kelvin = Temperature in Celsius scale aluminium wire having same dimensions and pass
+ 273.15 the same current through it, the power dissipated
156. (d) Sound cannot travel through a vacuum. A vacuum is will be > P.
an area without any air, like space. So sound cannot 164. (a) Pascal's principle, also called Pascal's law, in fluid (gas
travel through space because there is no matter for or liquid) mechanics, statement that, in a fluid at rest in
the vibrations to work in. a closed container, a pressure change in one part is
157. (a), 1 nanometer = 0.00000010 centimeters (cm) or transmitted without loss to every portion of the fluid
10–7 cm and to the walls of the container.
EBD_7367
16
C- Chemistry

C HA P T E R
CHEMISTRY
35
1. Match List I with List II and select the correct answer using 10. Which of the following isotopes of carbon is/are used in
the codes given below the Lists. [2007 -II] carbon dating? [2007 -II]
(a) 6C12 only (b) 6C13 only
Lis t I Lis t II
(c) 6C14 only (d) 6C12 and 6C14
(Mineral) (Indus tries in which largely us ed) 11. An a-particle consists of which of the following?
A . Limes tone 1 Cement [2007 -II]
B. Copp er 2 Electrical goods (a) 2 protons and 2 neutrons
(b) 1 proton and 1 electron
C. Bauxite 3 M anufacture of aeroplanes
(c) 2 protons and 4 neutrons
D. Manganes e 4 Steel (d) 1 proton and 1 neutron
Codes 12. Which one of the following substances is made from natural
A B C D raw materials? [2007 -II]
(a) 3 4 1 2 (a) Rayon (b) Nylon
(b) 1 2 3 4 (c) Polyester (d) Polystyrene
(c) 3 2 1 4 13. Which one of the following metals is less reactive than
(d) 1 4 3 2 hydrogen? [2007 -II]
2. The flavour of apple is mainly due to which one of the (a) Barium (b) Copper
following? [2007 -II] (c) Lead (d) Magnesium
(a) Formalin (b) Benzene 14. Consider the following statements [2007 -II]
(c) Ethanol (d) Benzaldehyde 1. Nitric acid is used in the production of fertilizers.
3. Nail varnish remover generally contains which one of the 2. Sulphuric acid is used in the production of explosives.
following? [2007 -II] Which of the statements given above is/are correct?
(a) Vinegar (b) Benzene (a) Only 1 (b) Only 2
(c) Methyl alcohol (d) Acetone (c) Both 1 and 2 (d) Neither 1 nor 2
4. In which categories did Marie Curie win her two different 15. What is the purpose of adding baking soda to dough?
Nobel prizes? [2007 -II] [2007 -II]
(a) Physics and Chemistry (a) To generate moisture
(b) Chemistry and Medicine (b) To give a good flavour
(c) Physics and Medicine (c) To give good colour
(d) Chemistry and Peace (d) To generate carbon dioxide
5. 'Freon' used as refrigerants is chemically known as 16. As which one of the following, does carbon occur in its
[2007 -II] purest form in nature? [2008 -I]
(a) chlorinated hydrocarbon (a) Carbon black (b) Graphite
(b) fluorinated hydrocarbon (c) Diamond (d) Coal
(c) chlorofluoro hydrocarbon 17. What is the Jeweller's rouge? [2008 -I]
(d) fluorinated aromatic compound (a) Ferric oxide (b) Ferrous oxide
6. German silver is an alloy of [2007 -II]
(c) Ferrous carbonate (d) Ferric carbonate
(a) gold and silver (b) copper and silver
18. 'Misch metal' is widely used in the manufacture of which of
(c) copper, zinc and silver (d) copper, zinc and nickel
the following? [2008 -I]
7. The tracking of people by trained dogs is based on the
recognisation of which of the following compounds in the (a) Material of car brake (b) Smoke detectors
sweat from feet? [2007 -II] (c) Cigarette lighters (d) Emergency lights
(a) Carboxylic acids (b) Uric acid 19. What is the pH value of pure water? [2008 -I]
(c) Sugar (d) Salt (a) 1 (b) 6
8. Which of the following metals are present in haemoglobin (c) 7 (d) 10
and chlorophyll, respectively? [2007 -II] 20. Which one of the following is an element?
(a) Fe and Mg (b) Fe and Zn (a) Topaz (b) Diamond
(c) Mg and Zn (d) Zn and Mg (c) Ruby (d) Sapphire
9. Which one of the following is involved for desalination of 21. Which one of the following substances is used in the
sea water? [2007 -II] manufacture of safety matches? [2008 -II]
(a) Reverse osmosis (a) Red phosphorus
(b) Simple osmosis (b) White phosphorus
(c) Use of sodium aluminium silicate as zeolite (c) Phosphorus trioxide (P203)
(d) Use of ion selective electrodes (d) Black phosphorus
Chemistry C-17
22. Which one of the following is correct? Butter is 35. Aqua-regia used by alchemists to separate silver and gold
[2008 -II] is a mixture of [2009 -I]
(a) a supercooled oil (b) an emulsion (a) hydrochloric acid (concentrated) and nitric acid
(c) a molecular solid (d) None of these (concentrated)
23. Which one of the following is the softest? [2008 -II] (b) hydrochloric acid (concentrated) and sulphuric
(a) Sodium (b) Aluminium acid (concentrated)
(c) Iron (d) Copper (c) nitric acid (concentrated) an d sulphuric acid
24. Which one of the following properties changes with (concentrated)
valency? [2008 -II] (d) hydrochloric acid (dilute) and sulphuric acid (dilute)
(a) Atomic weight (b) Equivalent weight 36. Nail polish remover contains [2009 -I]
(c) Molecular weight (d) Density (a) acetone (b) benzene
25. Which of the following has maximum density? (c) formaldehyde (d) acetic acid
[2008 -II] 37. Statement I : Soaps do not form lather with water containing
(a) Chloroform (b) Water salts of calcium and magnesium. [2009 -I]
(c) Benzene (d) Ice Statement II : Calcium and magnesium salts of long chain
26. Which one of the following is used in the preparation of fatty acids are insoluble in water.
antiseptic solution? [2008 -II] (a) Both the statements are individually true and
(a) Potassium nitrate (b) Iodine Statement II is the correct explanation of Statement I.
(c) Iodine chloride (d) Potassium chloride (b) Both the statements are individually true but Statement
27. Which one of the following is not a chemical change? II is not the correct explanation of Statement I.
[2008 -II] (c) Statement I is true, but Statement II is false.
(a) Burning of coal in air (d) Statement I is false, but Statement II is true.
(b) Fermentation of sugar cane juice 38. Which one of the following is associated with the formation
(c) Crystallisation of table salt from sea water of brown air in traffic congested cities? [2009 -I]
(d) Cracking of petroleum (a) Sulphur dioxide (b) Nitrogen oxide
28. Statement I : Addition of water to an aqueous solution of (c) Carbon dioxide (d) Carbon monoxide
HC1 decreases the pH. [2008 -II] 39. Which one of the following petroleum refinery products
Statement II : Addition of water suppresses the ionisation has the lowest boiling point? [2009 -I]
of HC1. (a) Kerosene (b) Diesel
(a) Both the statements are individually true and Statement (c) Gasoline (d) Lubricating oil
II is the correct explanation of Statement I 40. Which one of the following is used as a mordant in dyeing
(b) Both the statements are individually true but Statement and tanning industry? [2009 -I]
II is not the correct explanation of Statement I (a) Magnesium oxide
(c) Statement I is true, but Statement II is false (b) Magnesium carbonate
(d) Statement I is false, but Statement II is true (c) Magnesium chloride
29. Which one of the following is the secondary source of light (d) Magnesium sulphate
in a fluorescent lamp? [2008 -II] 41. Which of the following statements about the commonly
(a) Neon gas (b) Argon gas used automobile battery are true? [2009 -II]
(c) Mercury vapour (d) Fluorescent coating I. It is usually a lead-acid battery.
30. Which one of the following is correct? Setting of plaster of II. It has six cells with a potential of 2 V each.
Paris is [2008 -II] III. Its cells work as galvanic cells while discharging
(a) dehydration power.
(b) oxidation with atmospheric oxygen IV. Its cells work as electrolytic cells while recharging.
(c) hydration leading to another hydrate Select the correct answer using the codes given below
(d) combination with atmospheric C02 (a) I, II, III and IV (b) I, II and III
31. Which one of the following is heavy water used in nuclear (c) II and IV (d) III and IV
reactor? [2009 -I] 42. The light emitted by firefly is due to [2009 -II]
(a) Water having molecular weight 18 u (a) a radioactive substance
(b) Water having molecular weight 20 u (b) chemiluminiscence process
(c) Water at 4°C but having molecular weight 19 u (c) a photoelectric process
(d) Water below the ice in a frozen sea (d) burning of phosphorus
32. The rusting of iron nail [2009 -I] 43. Which one of the following polymeric materials is used for
(a) decreases its weight making bullet proof jacket? [2009 -II]
(b) increases its weight (a) Nylon-6, 6 (b) Rayon
(c) does not affect weight but iron is oxidised (c) Kevlar (d) Dacron
(d) does not affect weight but iron is reduced 44. The coil in a heater is made of [2010 -I]
33. Which one of the following when dissolved in H2O gives (a) nichrome (b) tungsten
hissing sound? [2009 -I] (c) copper (d) iron
(a) Limestone (b) Slaked lime 45. Which among the following elements is abundant on the
(c) Sodalime (d) Quicklime lunar surface and holds the potential to put an end to the
34. Commercial vulcanisation of rubber involves [2009 -I] energy crisis of the earth? [2010 -I]
(a) sulphur (b) carbon (a) Helium-I (b) Helium-ll
(c) phosphorus (d) selenium (c) Helium-Ill (d) Helium-IV
EBD_7367
18
C- Chemistry

46. If a limestone piece is dipped in water, a bubble evolves. (c) dissolving the dirt
The bubbling is due to [2010 -I] (d) forming some aggregates of themselves and take away
(a) hydrogen (b) oxygen the dirt in the core of the aggregates
(c) water vapour (d) carbon dioxide 53. Which one of the following is not a mixture?
47. Which one of the following chemicals is commonly (a) Tootpaste (b) Toilet soap
used by farmers to destroy weeds? [2010 -I] (c) Baking soda (d) Vinegar
(a) DDT (b) Malathion 54. Which one of the following reducing agents can also act as
(c) Methyl bromide (d) 2, 4-D an oxidising agent? [2010 -II]
48. Some statements about the benefits of organic farming are (a) H2 (b) H2S
given below. Indicate whether they are true or false using (c) SO2 (d) HI
the codes given below the statements : [2010 -I] 55. Consider the following statements regarding the properties
1. It reduces CO2 emission. and uses of glass wool. [2010 -II]
2. It does not lead to toxic effect. I. Glass wool has tensile strength greater than steel.
3. It improves the water-retention capacity of the soil. II. Glass wool is fire proof.
Codes III. Glass wool has high electrical conductivity and
(a) 1 2 3 absorbs moisture.
False True False IV. Glass wool is used to prepare fibre glass.
(b) 1 2 3 Which of the statements given above are correct?
True False False (a) I and II (b) I, II and IV
(c) 1 2 3 (c) II and IV (d) I, III and IV
False True True 56. Which one among the following has been producing/can
(d) 1 2 3 produce light by a chemical change? [2010 -II]
False True True (a) Sun (b) Moon
49. Match List I with List II and select the correct answer using (c) Electric bulb (d) Lightning and thunder
the codes given below the Lists. [2010 -I] 57. A woman desires to clean the surface of her gold ornaments
by a chemical approach. For this she requires to use
Lis t I Lis t II [2010 -II]
(Item) (Toxic s u bs tance) (a) aqua-regia
A. CFL lamp 1 Nitro gen oxid es (b) concentrated H2SO4
(c) concentrated NaOH
B. A utomo bile battery 2 Ph thalates (d) sodium thiosulphate solution
C. Poly mer 3 Lead
DIRECTIONS (Qs. 58-59) : The following questions consist of
D. Dies el eng ine 4 M ercury two statements. Statement I and Statement II. You are to examine
Codes these two statements carefully and select the answers to these
A B C D A B C D items using the codes given below.
(a) 4 2 3 1 (b) 4 3 2 1 (a) Both the statements are individually true and Statement II
(c) 1 2 3 4 (d) 1 3 2 4 is the correct explanation of Statement I.
50. Match List I with List II and select the correct answer using (b) Both the statements are individually true but Statement II is
the codes given below the Lists. [2010 -I] not the correct explanation of Statement I.
List I List II (c) Statement I is true, but Statement II is false.
(Alloy) (Constituent) (d) Statement I is false, but Statement II is true.
58. Statement I : During the setting of cement, the structure
A. Solder 1 Iron and carbon has to be cooled by spraying water. [2010 -II]
B. Brass 2 Copper and zinc Statement II : The constituents of cement undergo hydration
C. Bronze 3 Copper and tin during setting of cement and it is an exothermic reaction.
59. Statement I: Superphosphate of lime can be assimilated by
D. Steel 4 Lead and tin plants. [2010 -II]
Codes Statement II: Superphosphate of lime is soluble in water.
A B C D A B C D 60. Which one of the following is not a periodic property i.e.,
(a) 1 2 3 4 (b) 4 2 3 1 does not show any trend on moving from one side to the
(c) 1 3 2 4 (d) 4 3 2 1 other in the periodic table? [2010 -II]
51. The polymeric fibre used as a substitute for wool in making (a) Atomic size (b) Valency
synthetic blankets, sweaters, etc., is [2010 -I] (c) Radioactivity (d) Electronegativity
(a) nylon (b) teflon 61. The law enforcement agencies use a chemical test to
(c) orlon (d) bakelite approximate a person's blood alcohol level. The person
52. The cleaning of dirty clothes by soaps and detergents is undergoing the test blows into the mouthpiece of a bag
due to a type of molecules called surfactants, which are containing sodium dichromate solution in acidic medium. A
present in soaps and detergents. The surfactant molecules chemical reaction with ethanol changes the colour of the
remove the dirt by [2010 -I] solution from [2010 -II]
(a) making the cloth slippery (a) orange to green (b) orange to colourless
(b) producing some gases between the dirt and the cloth (c) yellow to orange (d) colourless to orange
Chemistry C-19
62. Scuba divers are at risk due to high concentration of 74. Match List I with List II and select the correct answer using
dissolved gases while breathing air at high pressure under the codes given below the lists.
water. The tanks used by Scuba divers are filled with
[2010 -II] Lis t I Lis t II
(a) air diluted with helium (A cid ) (Sou rce)
(b) O2 A . Lactic acid 1 Tamarind
(c) N2
(d) a mixture of N2 and helium B. Tartaric acid 2 Orang e
63. Which one of the following is not needed in a nuclear C. Oxalic acid 3 Tomato
fission reactor? [2010 -II]
D. Citric acid 4 So ur curd
(a) Moderator (b) Coolant
(c) Accelerator (d) Control device Codes
64. When items or jewellery made of metals such as copper or A B C D A B C D
nickel are placed in a solution having a salt of gold, a thin (a) 2 3 1 4 (b) 2 1 3 4
film of gold is deposited by [2011 - I] (c) 4 3 1 2 (d) 4 1 3 2
(a) cooling to below 0°C 75. One of the occupational health hazards commonly faced by
(b) heating above 100°C the workers of ceramics, pottery and glass industry is
(c) passing an electric current [2011 - II]
(d) just keeping it for 10 min (a) stone formation in gall bladder
65. Heavy water implies [2011 - I] (b) melanoma
(a) water which is used in heavy industries such as thermal (c) silicosis
power plants (d) stone formation in kidney
(b) water which contains SO42- and CI– of calcium and 76. Sacrificial anode protects iron or ships, underground
magnesium pipelines etc from rusting, a process known as cathodic
(c) deuterated water protection. Which one of the following metals cannot be
(d) water which has maximum density used as a sacrificial anode? [2011 - II]
66. Which one among the following is a sin smelling agent (a) Tin (b) Zinc
added to LPG cylinder to help the detection of gas leakage? (c) Magnesium (d) Aluminium
[2011 - I] 77. Sodium thiosulphate (Na 2S 2O 3) solution is used in
(a) Ethanol (b) Thioethanol photography to [2011 - II]
(c) Methane (d) Chloroform (a) remove reduced silver
67. A body is charged negatively. It implies that [2011 - I] (b) reduce silver bromide (AgBr) grain to silver
(a) it has lost some of its protons (c) remove undecomposed AgBr as a soluble silver
(b) it has acquired some electrons from outside thiosulphate complex
(c) it has lost some of its electrons (d) convert the metallic silver to silver salt
(d) None of the above
68. Which one among the following metals is used for making DIRECTIONS (Qs.78-80) : The following questions consist of
boats because it does not corrode by sea water? two statements. Statement I and Statement II. You are to examine
[2011 - I] these two statements carefully and select the answers to these
(a) Tungsten (b) Nickel items using the codes given below.
(c) Antimony (d) Titanium (a) Both the statements are individually true and statement II is
69. Contact lenses are made from [2011 - I] the correct explanation of Statement I.
(a) polyvinyl chloride (b) polystyrene (b) Both the statements are individually true but Statement II is
(c) lucite (d) teflon not the correct explanation of Statement I.
70. Water is a good coolant and is used to cool the engines of (c) Statement I is true, but Statement II is false.
cars, buses, trucks etc. It is because water has a (d) Statement I is false, but Statement II is true
[2011 - I] 78. Statement I : Oxides of sulphur and nitrogen present in
(a) high specific heat (b) low surface tension
high concentration in air are dissolved in rain drops.
(c) high boiling point (d) teflon
Statement II : Oxyacids of sulphur and nitrogen make rain
71. A close bottle containing water at room temperature was
taken to the Moon and then the lid is opened. The water water acidic. [2011 - II]
will [2011 - I] 79. Statement I : On mixing with water, plaster of Paris hardens.
(a) freeze Statement II : By combining with water, plaster of Paris is
(b) boil converted into gypsum. CaS04 -2Hp Gypsum
(c) decompose into oxygen and hydrogen [2011 - II]
(d) not change at all 80. Statement I : All liquids are conductors of electricity.
72. Which one among the following substances evolves heat Statement II : Under the condition of low pressure and high
when dissolved in water? [2011 - I] voltage, liquids can be made conducting. [2011 - II]
(a) Potassium nitrate (b) Sodium chloride 81. Gypsum (CaSO4 . 2H2O) is added to clinker during cement
(c) Glucose (d) Calcium oxide manufacturing to [2011 - II]
73. Which one among the following would expand the most on (a) decrease the rate of setting of cement
being heated. [2011 - I] (b) bind the particles of calcium silicate
(a) water (b) Alcohol (c) facilitate the formation of colloidal gel
(c) Glass (d) Air (d) impart strength to cement
EBD_7367
20
C- Chemistry
82. Which one among the following will you put into pure water 95. Soaps cannot be used in acidic condition because they
in order to pass electric current through it? [2011 - II] lose their cleansing effect due to formation of insoluble
(a) Kerosene (c) Mustard oil [2012 - II]
(c) Lemon juice (d) Sugar (a) esters (b) alcohols
83. From which one among the following water sources, the (c) hydrocarbons (d) long chain fatty acids
water is likely to be contaminated with fluoride?[2011 - II] 96. In an atomic explosion, release of large amount of energy is
(a) Ground water (b) River water due to conversion of [2012 - II]
(c) Pond water (d) Rain water (a) chemical energy into nuclear energy
84. What is the main constituent of a pearl? [2011 - II] (b) nuclear energy into heat
(a) Calcium carbonate and magnesium carbonate (c) mass into energy
(b) Calcium sulphate only (d) chemical energy into heat
(c) Calcium oxide and calcium sulphate
(d) Calcium carbonate only DIRECTION (Q. 97) : The following question consist of two
85. Age of fossil may be found out by determining the ratio of statements. Statement I and Statement II. You are to examine
two isotopes of carbon. The isotopes are [2012 - I] these two statements carefully and select the answer to this
(a) C-12 and C-13 (b) C-13 and C-14 question using the codes given below.
(c) C-12 and C-14 (d) C-12 and carbon black (a) Both the statements are individually true and Statement II is
86. Which one among the following nontoxic gases helps in the correct explanation of Statement I
formation of enzymes which ripen fruit? [2012 - I] (b) Both the statements are individually true but Statement II is
(a) Acetylene (b) Ethane not the correct explanation of Statement I
(c) Methane (d) Carbon dioxide (c) Statement I is true, but Statement II is false
87. Vermicompost is an/a [2012 - I] (d) Statement I is false, but Statement II is true
(a) inorganic fertilizer (b) toxic substance 97. Statement I: All compounds contain more than one element.
(c) organic bio fertilizer (d) synthetic fertilizer [2012 - II]
88. The macro nutrients provided by inorganic fertilizer are Statement II : All compounds are heterogeneous mixtures.
[2012 - I] 98. The gas which turns lime water milky is [2012 - II]
(a) carbon, iron and boron (a) carbon dioxide (b) carbon monoxide
(b) magnesium, manganese and sulphur (c) ammonia (d) nitrogen dioxide
(c) magnesium, zinc and iron 99. On the labels of the bottles, some soft drinks are claimed to
(d) nitrogen, phosphorus and potassium be acidity regulators. They regulate acidity using
89. Which one among the following statements about an atom [2012 - II]
is not correct? [2012 - I] (a) carbon dioxide (b) bicarbonate salts
(a) Atoms always combine to form molecules (c) Both (a) and (b) (d) carbon dioxide and lime
(b) Atoms are the basic units from which molecules and 100. Antacids are commonly used to get rid of acidity in the
ions are formed stomach. A commonly used antacid is [2012 - II]
(c) Atoms are always neutral in nature (a) sodium hydrogen phthalate
(d) Atoms aggregate in large numbers to form the matter (b) magnesium hydroxide
that we can see, feel and touch (c) calcium hydroxide
90. Which one of the following reactions is the main cause of (d) manganese acetate
the energy radiation from the Sun? [2012 - I] 101. Iodised salt is a [2012 - II]
(a) Fusion reaction (b) Fission reaction (a) mixture of potassium iodide and common salt
(c) Chemical reaction (d) Diffusion reaction (b) mixture of molecular iodide and common salt
91. Which one among the following statements is correct? (c) compound formed by combination of potassium iodide
[2012 - I] and common salt
(a) All bases are alkalis (d) compound formed by molecular iodine and common
(b) None of the bases is alkali salt
(c) There are no more bases except the alkalis 102. Match the following lists. [2013 - I]
(d) All alkalis are bases but all bases are not alkalis
92. The pH of fresh ground water slightly decreases upon List I List II
exposure to air because [2012 - I] (Agent) (Disease)
(a) carbon dioxide from air is dissolved in the water
(b) oxygen from air is dissolved in the water A. Arsenic 1 Fluorosis
(c) the dissolved carbon dioxide of the ground water B. Fluoride 2 Melanosis
escapes into air C. Dust 3 Presbycusis
(d) the dissolved oxygen of the ground water escapes
into air D. Noise 4 Sillicosis
93. Which one among the following polymers is used for making Codes
bulletproof material? [2012 - I] A B C D A B C D
(a) Polyvinyl chloride (b) Polystyrene (a) 3 1 4 2 (b) 3 4 1 2
(c) Polyethylene (d) Polyamide (c) 2 1 4 3 (d) 2 4 1 3
94. The elements of a group in the periodic tabic [2012 - I] 103. Which one among the following gases readily combines
(a) have similar chemical properties with the haemoglobin of the blood? [2013 - I]
(b) have consecutive atomic numbers (a) Methane (b) Nitrogen dioxide
(c) are isobars (c) Carbon monoxide (d) Sulphur dioxide
(d) are isotopes
Chemistry C-21
104. Which of the following gases in the atmosphere is/are 114. A liquid initially contracts when cooled down to 4°C but on
responsible for acid rains? [2013 - I] further cooling down to 0°C, it expands. [2013 - II]
1. Oxides of sulphur 2. Oxides of nitrogen The liquid is
3. Oxides of carbon (a) alcohol (b) water
Select the correct answer using the codes given below (c) molten iron (d) mercury
(a) 1 and 2 (b) 1 and 3
115. What are the elements which are liquids at room temperature
(c) Only 2 (d) 1, 2 and 3
105. Which one among the following metals is used in fireworks and standard pressure? [2013 - II]
to make a brilliant white light? [2013 - I] 1. Helium 2. Mercury
(a) Sodium (b) Magnesium 3. Chlorine 4. Bromine
(c) Aluminium (d) Silver Select the correct answer using the codes given below
106. Why hard water does not give lather with soap? (a) 2 and 3 (b) 2, 3 and 4
[2013 - I] (c) 2 and 4 (d) 1 and 3
(a) Hard water contains calcium and magnesium ions which 116. A compound that is a white solid which absorbs water
form precipitate with soap vapour from the air is [2013 - II]
(b) Hard water contains sulphate and chloride ions which (a) sodium nitrate (b) calcium chloride
form precipitate (c) sodium carbonate (d) calcium sulphate
(c) pH of hard water is high
(d) pH of hard water is less 117. What type of mixture is smoke? [2013 - II]
107. What is the role of positive catalyst in a chemical reaction? (a) Solid mixed with a gas
[2013 - I] (b) Gas mixed with a gas
(a) It increases the rate of reaction (c) Liquid mixed with a gas
(b) It decreases the rate of reaction (d) Gas mixed with a liquid and a solid
(c) It increases the yield of the products 118. Which one of the following gases is supporter of
(d) It provides better purity of the products combustion? [2014 - II]
108. Which allotropy of carbon is in rigid three- dimensional (a) Hydrogen (b) Nitrogen
structure? [2013 - I] (c) Carbon dioxide (d) Oxygen
(a) Graphite (b) Fullerene 119. Iron sheet kept in moist air covered with rust. Rust is
(c) Diamond (d) Carbon black
[2014 - II]
109. Food cans are coated with tin but not with zinc because
[2013 - I] (a) an element
(a) zinc is costlier than tin (b) a compound
(b) zinc has a higher melting point than tin (c) a mixture of iron and dust
(c) zinc is more reactive than tin (d) a mixture of iron, oxygen and water
(d) tin is more reactive than zinc 120. A metal screw top on a glass bottle which appears to be
110. Which one among the following is responsible for the stuck could be opened by using the fact that [2014 - II]
expansion of water in the ocean? [2013 - I] (a) the metal expands more than the glass when both are
(a) Carbon dioxide (b) Nitrogen dioxide heated
(c) Carbon monoxide (d) Sulphur dioxide (b) the metal and glass expand identically when heated
DIRECTION (Q. 111) : The following questions consist of two (c) the metal shrinks when heated
statements. Statement I and Statement II. You are to examine (d) Both metal and glass shrink when cooled
these two statements carefully and select the answers to these 121. Which of the following are the two main constituents of
items using the codes given below. granite? [2014 - II]
(a) Both the statements are individually true and statement II is (a) Iron and silica
the correct explanation of Statement I. (b) Iron and silver
(b) Both the statements are individually true but Statement II is (c) Silica and aluminium
not the correct explanation of Statement I. (d) Iron oxide and potassium
(c) Statement I is true, but Statement II is false. 122. Consider the following statements. [2014 - II]
(d) Statement I is false, but Statement II is true Statement I : Clay layers are poor aquifers.
111. Consider the following Statements. [2013 - I] Statement II : The inter-particle space of clay minerals is
Statement I Chlorine radicals CI* initiate the chain reaction the least.
for ozone depletion. Select the correct answer using the codes given below :
Statement II Gaseous hypochlorous acid and chlorine are (a) Both the statements are individually true and Statement
photolysed by sunlight. II is the correct explanation of Statement I
112. Which element forms the highest number of compounds in (b) Both the statements are individually true but Statement
the periodic table? [2013 - II]
(a) Carbon (b) Oxygen II is not the correct explanation of Statement I
(c) Silicon (d) Sulphur (c) Statement I is true but Statement II is false
113. NaOH + HC1 -> NaCl + H2O in the given chemical reaction (d) Statement I is false but Statement II is true
[2013 - II] 123. Addition of ethylene dibromide to petrol [2014 - II]
(a) sodium is oxidised and oxygen is reduced (a) increases the octane number of fuel
(b) sodium is oxidised and chlorine is reduced (b) helps elimination of lead oxide
(c) sodium and hydrogen are oxidised (c) removes the sulphur compound in petrol
(d) None of them are oxidised or reduced (d) serves as a substitute of tetraethyl lead
EBD_7367
22
C- Chemistry

124. Nitric oxide pollution can lead to all of the following, except 135. Which of the following is not correct about Baking Soda ?
[2014 - II] [2014-II]
(a) leaf spotting in plants (a) It is used in soda acid fire extinguisher
(b) bronchitis related respiratory problems in human (b) It is added for faster cooking.
(c) production of corrosive gases through photochemical (c) It is a corrosive base
reaction (d) It neutralizes excess acid in the stomach
(d) silicosis in human 136. Chromium oxide in paints makes the colour of paint
125. Which of the following statements is/are correct? [2014-II]
[2014 - II] (a) Green (b) White
1. Amnion contains fluid. (c) Red (d) Blue
2. Ultrasound scan can detect the sex of an embryo. DIRECTION (Q. 137): The following question consist of two
Select the correct answer using the codes given below statements, Statement I and Statement II. You are to examine
(a) Only 1 (b) Only 2 these two statements carefully and select the answers to the
(c) Both 1 and 2 (d) Neither 1 nor 2 question using the code given below :
126. Which of the following solutions will not change the colour Code :
of blue litmus paper to red? [2014 - II] (a) Both the statements are individually true and Statement II
1. Acid solution is the correct explanation of Statement I
2. Base solution (b) Both the statements are individually true but Statement II is
3. Common salt solution not the correct explanation of Statement I
Select the correct answer using the codes given below (c) Statement I is true but Statement II is false
(a) 1 and 3 (b) 2 and 3 (d) Statement I is false but Statement II is true
(c) Only 1 (d) Only 2 137. Statement I : During indigestion, milk of magnesia is taken
127. Date of manufacture of food items fried in oil should be to get rid of pain in the stomach.
checked before buying because oils become rancid due to Statement II : Milk of magnesia is a base and it neutralizes
[2014 - II] the excess acid in the stomach. [2014-II]
138. When hard water is evaporated completely, the white solid
(a) oxidation (b) reduction
remains in the container. It may be due to the presence of
(c) hydrogenation (d) decrease in viscosity 1. Carbonates of Ca and Mg [2015-I]
128. Bagasse, a by-product of sugar manufacturing industry, is 2. Sulphates of Ca and Mg
used for the production of [2014-II] 3. Chlorides of Ca and Mg
(a) glass (b) paper Select the correct answer using the code given below :
(c) rubber (d) cement (a) 1 and 2 only (b) 1, 2 and 3
129. The main constituent of Gobar gas is [2014-II] (c) 3 only (d) 1 and 3 only
(a) Ethane (b) Methane 139. Which one among the following compounds has same
(c) Propane (d) Acetylene equivalent weight and molecular weight ? [2015-I]
130. Which of the following is a good lubricant ? [2014-II] (a) H2SO4 (b) CaCl2
(a) Diamond powder (b) Graphite powder (c) Na2SO4 (d) NaCl
(c) Molten carbon (d) Alloy of carbon and iron 140. A metallic plate sticks firmly on the mouth of a water vessel
131. In Tritium (T), the number of protons (P) and neutrons (N) made from another metal. By way of heating, one can detach
respectively are [2014-II] the plate from the vessel. This is because heat expands
(a) 1 P and 1 N (b) l P and 2 N (a) the vessel only [2015-I]
(c) l Pand 3 N (d) 2 P and l N (b) both the vessel and the plate equally
132. When carbon dioxide is passed through lime water, the (c) the vessel more than the plate
(d) the vessel and contracts the plate
solution turns milky, but, on prolonged passage, the 141. Electricity is produced through dry cell from [2015-I]
solution turns clear. This is because [2014-II] (a) chemical energy (b) thermal energy
(a) the calcium carbonate formed initially is converted to (c) mechanical energy (d) nuclear energy
soluble calcium bicarbonate on passage of more 142. Which one among the following fuels is used in gas
carbon dioxide welding? [2015-I]
(b) the reaction is reversible and lime water is regenerated (a) L P G (b) Ethylene
(c) the calcium bicarbonate formed initially is converted (c) Methane (d) Acetylene
to soluble calcium carbonate on passage of more 143. Which one among the following is a micronutrient present
carbon dioxide in soil for various crops ? [2015-I]
(d) the initially formed insoluble compound is soluble in (a) Calcium (b) Manganese
carbonic acid . (c) Magnesium (d) Potassium
133. The form of carbon known as graphite [2014-II] 144. Match List I with List II and select the correct answer using
(a) is harder than diamond the code given below the Lists : [2015-I]
(b) contains a higher percentage of carbon than diamond List I List II
(c) is a better electrical conductor than diamond (Air Pollutant) (Effect)
(d) has equal carbon-to-carbon distances in all directions A. Chlorofluorocarbon 1. Acid rain
134. Which one among the following is an example of chemical B. Sulphur dioxide 2. Depletion in ozone layer
in the atmosphere
change ? [2014-II] C. Lead compound 3. Harmful for human
(a) The melting of an ice cube nervous system
(b) The boiling of gasoline D. Carbon dioxide 4. Topmost contribution to
(c) The frying of an egg greenhouse effect
(d) Attraction of an iron nail to a magnet
Chemistry C-23
Code : 153. Which one of the following physical quantities is the same
A B C D for molecules of all gases at a given temperature?
(a) 4 3 1 2 [2015-II]
(b) 4 1 3 2 (a) Speed (b) Mass
(c) 2 1 3 4 (c) Kinetic energy (d) Momentum
(d) 2 3 1 4 154. Which one of the following statements is correct? The
145. Which one among the following metals is prominently used velocity of sound : [2016-I]
in mobile phone batteries? [2015-I] (a) Does not depend upon the nature of media
(a) Copper (b) Zinc (b) is maximum in gases and minimum in liquids
(c) Nickel (d) Lithium (c) is maximum in solids and minimum in liquids
146. Which one among the following processes is not part of a (d) is maximum in solids and minimum in gases
chemical weathering ? [2015-I] 155. A piece of ice, 100 g in mass is kept at 0°C. The amount of
(a) Hydration (b) Exfoliation heat is requires to melt at 0°C is (take latent heat of melting
(c) Oxidation (d) Solution of ice to be 333.6 j / g) : [2016-I]
147. Red phosphorus is used in the manufacture of safety (a) 750.6 J (b) 83.4 J
matches. This is due to the fact that [2015-II] (c) 33360 J (d) 3.336 J
(a) it shows phosphorescence 156. Which one of the following is an example of chemical change?
(b) at ordinary temperature, it is less reactive than other (a) Burning of paper [2016-I]
varieties of phosphorus (b) Magnetization of soft iron
(c) it cannot be converted to white phosphorus on heating (c) Dissolution of cane sugar in water
(d) it does not react with halogen on heating (d) Preparation of ice cubes from water
148. Match List-I with List-II and select the correct answer using 157. Which one of the following statements is not correct ?
the code given below the Lists : [2015-II] [2016-I]
List-I List-II (a) Hydrogen is an element
(Exponent) (Law) (b) Hydrogen is the lightest element
A. John Dalton 1. Law of definite (c) Hydrogen has no isotopes
proportion by volume (d) Hydrogen and oxygen form an explosive mixture
B. Joseph Proust 2. Law of multiple 158. Which one of the following statements is not correct ?
proportion [2016-I]
C. Antoine Lavoisier 3. Law of definite (a) Atoms of different elements may have same mass
proportion by weight numbers
D. Joseph Louis Gay- 4. Law of conservation (b) Atoms of an element may have different mass numbers
Lussac of mass (c) All the atoms of an element have same number of
Code : protons
A B C D (d) All the atoms of an element will always have same
(a) 2 3 4 1 number of neutrons.
(b) 2 4 3 1 159. The synthetic rubber has replaced natural rubber for
(c) 1 4 3 2 domestic and industrial purposes. Which one of the
(d) 1 3 4 2 following is the main reason behind that? [2016-I]
149. Which of the following is not gaseous air pollutant? (a) Natural rubber is unable to meet the growing demand
[2015-II] of different industries
(a) Oxides of Sulphur (b) Oxides of nitrogen (b) Natural rubber is grown in tropical countries only
(c) Hydrocarbon (d) Smoke (c) Raw material for synthetic rubber is easily available
150. Which one of the following statement is not correct? (d) Nutural rubber is not durable
[2015-II] 160. The handle of pressure cookers is made of plastic because
(a) Water starts boiling when its vapour pressure becomes it should be made non-conductor of heat. The plastic used
equal to atmospheric pressure. there is the first man-made plastic, which is : [2016-I]
(b) Water is known as universal solvent. (a) Polythene (b) Terylene
(c) Permanent hardness of water is due to presence of (c) Nylon (d) Bakelite
MgCl2, CaCl2, MgSO4 and CaSO4. 161. Methyl lsocyanate gas, which was involved in the disaster
(d) Density of ice is greater than that of water. in Bhopal in December 1984, was used in the Union Carbide
151. Which one of the following is not true for diamond? factory for production of : [2016-I]
[2015-II] (a) Dyes (b) Detergents
(a) Each carbon atom is linked to four other carbon atoms (c) Explosives (d) Pesticides
(b) Three-dimensional network structure of carbon atoms 162. Two systems are said to be in thermal equilibrium if and
is formed only if : [2016-I]
(c) It is used as an abrasive for sharpening hard tools (a) there can be a heat flow between them even if they are
(d) It can be used as a lubricant. at different temperatures
152. The atomic theory of matter was first proposed by (b) there cannot be a heat flow between them even if they
[2015-II] are at different temperatures
(a) John Dalton (b) Rutherford (c) there is no heat flow between them
(c) J. J. Thomson (d) Niels Bohr (d) their temperatures are slightly different
EBD_7367
24
C- Chemistry

163. German silver is used to make decorative articles, coinage 171. Which one of the following statements is correct?[2016-II]
metal, ornaments, The name is given because : [2016-I] (a) The oxidation number for hydrogen is always zero.
(a) it is an alloy of copper and contains silver as one of its (b) The oxidation number for hydrogen is always +1.
components (c) The oxidation number for hydrogen is always -1.
(b) Germans were the first to use silver (d) Hydrogen can have more than one oxidation number.
(c) Its appearance is like silver 172. In case of a standard hydrogen electrode [2016-II]
(d) It is an alloy of silver (a) absolute electrode potential is not zero
164. Which one of the following is not an allotrope of carbon? (b) absolute electrode potential is zero
[2016-II]
(c) both absolute and standard electrode potential values
(a) Coal (b) Diamond
are zero
(c) Graphite (d) Graphene
(d) electrode potential is zero only at 25°C
165. In paper manufacturing, degumming of the raw material is
done using [2016-II] 173. Which one of the following is a physical change?[2017-I]
(a) sulphuric acid (b) bleaching powder (a) Burning of coal
(c) caustic soda (d) nitric acid (b) Burning of wood
166. Dolomite powder is applied in some agricultural lands. The (c) Heating of a platinum crucible
purpose of applying it is to [2016-II] (d) Heating of potassium chlorate
(a) increase the pH of the soil 174. The pH value of a sample of multiple-distilled water is
(c) lower the pH of the soil [2017-I]
(c) increase the phosphorus content of the soil (a) zero (b) 14
(d) increase the nitrogen content of the soil (c) very near to zero (d) very near to seven
167. Excessive use of which of the following fertilizers may be 175. Which one of the following is the most characteristic
responsible for the presence of a toxic substance in property of an element? [2017-I]
groundwater? [2016-II] (a) Density (b) Boiling point
(a) Nitrogen (b) Phosphate only (c) Mass number (d) Atomic number
(c) Potassium only (d) Phosphate and potassium 176. There are two elements-calcium (atomic number 20) and
168. Which one of the following elements does not form solid argon (atomic number 18). The mass number of both the
hydrogen carbonate? [2016-II] elements is 40. They are therefore known as [2017-I]
(a) Sodium (b) Potassium (a) isotones (b) isochores
(c) Caesium (d) Lithium (c) isobars (d) isotopes
169. In the gamma decay of a nucleus [2016-II] 177. 'Plum Pudding Model' for an atom was proposed by
(a) the mass number of the nucleus changes whereas its [2017-I]
atomic number does not change
(a) Antoine Lavoisier (b) Robert Boyle
(b) the mass number of the nucleus does not change
(c) Ernest Rutherford (d) J. J. Thomson
whereas its atomic number changes
178. What is the number of atoms in 46 g of sodium-23
(c) both the mass number and the atomic number of the
nucleus change [N = Avogadro constant)? [2017-I]
(d) neither the mass number nor the atomic number of the (a) N/2 (b) N
nucleus changes (c) 2N (d) 23N
170. Match List-I with List-II and select the correct answer using 179. What is the maximum number of states of matter?
the code given below the Lists : [2016-II] [2017-I]
List-I List-II (a) Three (b) Four
(Element) (Property/ Use} (c) Five (d) Variable
A. Mg 1. Gives red colour to flame 180. The chemical properties of an element depend upon
B. Ca 2. Sulphate compound (a) the number of isotopes of the element
used in medicine to (b) the mass number of the element
examine the alimentary (c) the total number of neutrons in the element
canal of a patient (d) the number of electrons in the outermost shell of the
C. Sr 3. Traps the energy of sunlight element
in photosynthesis 181. The molecular mass of sulphuric acid is 98. If 49 g of the
D. Ba 4. Control of muscle contraction acid is dissolved in water to make one litre of solution, what
Code : will be the strength of the acid? [2017-I]
(a) A B C D (a) Two normal (b) One normal
2 4 l 3 (c) 0.5 normal (d) Four normal
(b) A B C D 182. Pearl is a hard object produced within the soft tissues of a
2 l 4 3 mollusk. Which one of the following is the main constituent
(c) A B C D
of pearl? [2017-II]
3 4 1 2
(a) Calcium carbonate (b) Calcium oxide
(d) A B C D
(c) Calcium nitrate (d) Calcium sulphate
3 l 4 2
Chemistry C-25

183. An electron and a proton starting from rest are accelerated Which of the above statements about operational principles
through a potential difference of 1000 V. Which one of the of a nuclear reactor and a nuclear bomb is/are correct?
following statements in this regard is correct? [2017-II] (a) 1 and 3 (b) 2 and 3
(a) The kinetic energy of both the particles will be different. (c) 4 only (d) 1 and 4
(b) The speed of the electron will be higher than that of 190. Tincture of iodine is an antiseptic for fresh wounds. It is a
the proton. dilute solution of elemental iodine, which does not contain
(c) The speed of the proton will be higher than that of the [2017-II]
electron. (a) water (b) acetone
(d) The speed of the electron and the proton will be equal. (c) alcohol (d) potassium iodide
184. In the reaction between hydrogen sulphate ion and water 191. Which one of the following gases dissolves in water to
give acidic solution? [2018-I]
HSO -4 + H 2O ® H3O+ + SO42 - (a) Carbon dioxide (b) Oxygen
the water acts as [2017-II] (c) Nitrogen (d) Hydrogen
(a) an acid (b) a base 192. Consider the following chemical reaction : [2018-I]
(c) a salt (d) an inert medium
185. How many hydrogen atoms are contained in 1.50 g of aFe2O3 (s ) + bCO ( g ) ® cFe (s ) + dCO2
glucose (C6H12O6)? In the balanced chemical equation of the above, which of
(a) 3.01 × 1022 (b) 1.20 × 1023 the following will be the values of the coefficients
(c) 2.40 × 10 23 (d) 6.02 × 1022 a, b, c and d respectively?
186. The paste of a white material in water is used to maintain a (a) 3, 2, 3, 1 (b) 1, 3, 2, 3
fractured bone fixed in place. The white material used is (c) 2, 3, 3, 1 (d) 3, 3, 2, 1
called [2017-II] 193. Bright light is found to emit from photographer’s flashgun.
(a) bleaching powder (b) plaster of Paris This brightness is due to the presence of which one of the
(c) powder of zinc oxide (d) lime powder following noble gases? [2018-I]
187. An emulsion consists of [2017-II] (a) Argon (b) Xenon
(a) one liquid and one solid (c) Neon (d) Helium
(b) one liquid and one gas 194. Which one of the following is not a characteristic of a
(c) two liquids compound? [2018-I]
(d) two solids (a) Composition is variable.
188. Which of the following radioactive substances (b) All particles of compound are of only one type.
enters/enter the human body through food chain and (c) Particles of compound have two or more elements.
causes/cause many physiological disorders? [2017-II] (d) Its constituents cannot be separated by simple
(a) Strontium-90 (b) Iodine-131 physical methods.
(c) Cesium-137 (d) All of the above 195. Which of the following substances cause temporary
189. Consider the following statements : [2017-II] hardness in water? [2018-I]
1. The chain reaction process is used in nuclear 1. Mg (HCO3)2 2. Ca (HCO3)2
bombs to release a vast amount of energy, but in 3. CaCl2 4. MgSO4
nuclear reactors, there is no chain reaction. Select the correct answer using the code given below.
2. In a nuclear reactor, the reaction is controlled while (a) 3 and 4 (b) 2 and 3
in nuclear bombs, the reaction is uncontrolled. (c) 1 and 4 (d) 1 and 2
3. In a nuclear reactor, all operating reactors are 196. Which one of the following elements will be an isobar of
‘critical’, while there is no question of ‘criticality’ calcium if the atomic number of calcium is 20 and its mass
in case of a nuclear bomb. number is 40? [2018-I]
4. Nuclear reactors do not use moderators, while (a) Element with 20 protons and 18 neutrons
nuclear bombs use them. (b) Element with 18 protons and 19 neutrons
(c) Element with 20 protons and 19 neutrons
(d) Element with 18 protons and 22 neutrons
EBD_7367
C- 26 Chemistry

HINTS & SOLUTIONS


1. (b) Limestone (CaCO3) is the main raw material used in 14. (c) Nitric acid is used in the production of fertilizers. The
the manufacture of cement. Copper is good conductor principal chemical produced form nitric acids
of elecricity hence, it is used in electrical goods. ammonium nitrate, sulphuric acid is used in the
Bauxite is an ore of Al. It is used in manufacture of production of explosives like TNT, nitroglycerine, gun
aeroplanes. Manganese is used in steel industry to cotton, etc.
form manganese steel. 15. (d) Baking soda has sodium bicarbonate as the chief
2. (c) The flavour of apples is mainly due to the ethanol constituent. It decomposes on heating giving carbon
(C2H5OH). dioxide. This causes dough, cakes, biscuits etc. to
3. (d) Nail varnish remover generally contains acetone expand and become light.
(CH3COCH3). 16. (a) Carbon occurs in its purest form of carbon black in
4. (a) Marie Curie win her two different Noble prizes in two nature.
different category i.e., Physics and Chemistry for the 17. (a) A very fine powder of ferric oxide is known as
discovery of radium and polonium. "jeweler's rouge", "red rouge", or simply rouge. It is
5. (c) Chlorofluoro carbon (CF2Cl2) is also known as freon. used to put the final polish on metallic jewellery and
It is used as refrigerants in refrigerators and air lenses, and historically as a cosmetic.
conditions. It is also used as propellant in aerosols 18. (c) 'Misch metal' alloyed with iron is used in the cigarettes
and foams. lighters. Misch metal is an alloy of cerium (50%),
6. (d) German silver is an alloy of copper (25-50%), zinc lanthanum (25%), neodymium (18%), praseodymium
(25-35%) and nickel (10-35%). It is used in utensils (5%) and other rare metals.
and resistance wire. 19. (c) Pure water is neutral in nature hence, the pH value of
7. (a) The tracking of people by trained dogs is based on the pure water is 7.
recognisation of carboxylic acid in the sweat from feet. 20. (b) Diamond is an element. It is an allotrope of carbon.
8. (a) Fe and Mg metals are present in haemoglobin and While ruby, topaz and sapphire are minerals (i.e.,
chlorophyll respectively. compounds). These are the valuable gemstones.
21. (a) Red phosphorus is mostly used in the manufacture of
9. (a) Reverse osmosis method is used to obtain pure water
safety matches.
from water containing a salt or for desalination of sea 22. (d) Butter is an example of a colloidal dispersion of a liquid
water. in a solid i.e., gel.
10. (c) The isotope of carbon 6C14 is used in radio carbon 23. (a) Alkali metals such as lithium, sodium and potassium
dating. are soft metals. These can be easily cut with a knife.
11. (a) An a-particle is identical with helium nuclei. It consists 24. (b) Only equivalent weight changes with valency. Atomic
of 2 protons and 2 neutrons and represented by 2He4. weight, molecular weight and density does not change
12. (a) Rayon is made from naturally occuring cellulose. with valency.
Rayon is also called artificial silk.
13. (b) Atomic weight
Equivalent weight of the element =
Valency
Most 25. (b) Water has the maximum density among these because
reactive chloroform, benzene and ice float on water.
26. (b) Iodine is a powerful antiseptic. It is used as a tincture
of iodine which is 2-3% iodine solution in alcohol-
water.
27. (c) A chemical change a new substance is formed. During
crystallisation of table salt from sea water no new
substance is formed hence, it is not a chemical change.
28. (c) Addition of water to an aqueous solution of HCl
decreases the pH because on dilution the number of
H+ ions (from water) increases. Now since
Least [H+] = 10–pH \ As [H+] increases pH decreases.
reactive 29. (d) Fluorescent coating on the glass is the secondary
source of light in a fluorescent lamp.
30. (c) Setting of plaster of Paris is the hydration leading to
another hydrate i.e., gypsum.
C H added for comparison 1 1 Setting
CaSO4 . H 2O+ 1 H 2O ¾¾¾¾ ® CaSO 4 × 2H 2O
2 2 Gypsum
Reactivity Series of Metals Plaster of Paris
In the reactivity series of metals, copper is placed 31. (b) Heavy water (D2O) is used in nuclear reactor as a
below the hydrogen. Hence, it is less reactive than moderator. The deuterium is an isotope of hydrogen.
hydrogen. On the other hand barium, lead and Its atomic mass is 2. Hence its molecular weight is
magnesium are placed above the hydrogen hence, 20u (2 + 18).
these are more reactive than hydrogen.
Chemistry C-27
32. (b) The rusting of iron nail takes place in the presence of 44. (a) The coil in a heater is made up of nichrome because
air (oxygen) and water (moisture). It increases the its melting point and resistivity are high.
weight of iron nail due to the formation of rust. 45. (c) Helium-III is abundant on the lunar surface and holds
4Fe + 3O2 + xH2O ® 2Fe2O3.xH 2O the potential to put an end to the energy crisis of the
Rust Earth.
33. (d) On adding water, quicklime (CaO) gives a hissing 46. (d) When limestone piece is dipped in water, the bubbles
sound and forms calcium hydroxide commonly known evolve due to evolution of carbon dioxide.
as slaked lime. CaCO3 + H2O ® Ca (OH)2 + CO2
CaO + H2O ® C Ca (OH2) + 15000 calories 47. (d) 2, 4-D (2, 4-dichlorophenoxy acetic acid) is used to
34. (a) Vulcanisation is a process in which natural rubber is destroy weeds.
treated with 3-5% sulphur. It introduces sulphur 48. (c) Organic farming is the form of agriculture that relies
bridges between polymer chains thereby increasing on crop rotation, green manure, compost, biological
its tensile strength, elasticity and resistance to pest control and mechanical cultivation to maintain
abrasion. soil productivity and control pests. It does not reduce
35. (a) Aqua-regia is a mixture of concentrated nitric acid CO2 emission. It does not lead to toxic effect. It
and concentrated hydrochloric acid in the ratio 1 : 3 improves the water-retention capacity of the soil.
respectively. It is a very powerful oxidising mixture. 49. (b) CFL is a type of fluorescent lamp which contains
36. (a) Acetone is used as a solvent. It is an ingredient of mercury.
nail polish remover. Automobile battery contains toxic substance lead
37. (a) Soaps do not form lather with hard water which (Pb).
contains salts of calcium and magnesium because Polymers contain toxic substance phthalates.
calcium and magnesium salts of long chain fatty acids Diesel engine emits the harmful nitrogen oxides by
are insoluble in water. the internal combustion of fuel.
38. (b) Nitrogen dioxide (NO2) is a reddish brown gas. It has 50. (b) (Alloy) (Constituent)
pungent smell and considered as air pollutant. In the Solder Lead and tin
presence of sunlight hydrocarbons and oxides of Nitrogen Brass Copper and zinc
react to form ozone and PAN (peroxyAcetyl Nitrate). Smog Bronze Copper and tin
is formed in traffic congested cities. Hytrocarbon + Steel Iron and carbon
51. (c) Polyacrylonitrile or orlon is used as a substitute for
Nitrogen oxides ¾¾¾® Sunlight
O3 + PAN+ aldehyde wool in making synthetic blankets, sweaters etc. It is
39. (c) a polymer of acrylonitrile or vinyl cyanide.
Product Boiling Point (°C) 52. (d) The cleaning of dirty clothes by soaps and detergents
(a) Kerosene 200-300° is due to a type of molecules called surfactants. The
(b) Diesel above 300° surfactant molecules remove the dirt by forming some
(c) Gasoline 25-75°C aggregates of themselves and take away the dirt in
(d) Lubricating oil >350°C the core of aggregates. The spherical aggregate of
surfactant molecules in water is called a micelle.
40. (d) Magnesium sulphate (MgSO4) is used as a mordant 53. (c) Sodium bicarbonate also known as baking soda, is
in dyeing and tanning industry. the chemical compound with the formula NaHCO3.
41. (a) Automobile batteries are usually lead-acid type, and
54. (c) SO2 can act as an oxidising agent as well as a reducing
are made of six galvanic cells in series to provide a 12-
agent.
volt system. Each cell provides 2.1 volts for a total of
It oxidises H2S to S
12.6 volts at full charge.
Lead-acid batteries are made up of plates of lead and 2H2S + SO2 ¾¾ ® 2H2O + S
separate plates of lead dioxide, which are submerged It reduces ferric sulphate to ferrous sulphate
into anelectrolyte solution of about 38% sulfuric acid
and 62% water. This causes a chemical reaction that Fe2 (SO4)3 + SO2 + 2H2O ¾¾ ® 2FeSO4 + 2H2SO4
releases electrons, allowing them to flow through 55. (b) Glass wool is an insulating material, obtained from
conductors to produce electricity. As the battery fibre glass arranged into a texture similar to wool.
discharges, the acid of the electrolyte reacts with the 56. (a) In a chemical change a new substance is formed. Sun
materials of the plates, changing their surface to lead has been producing light by nuclear fusion and it is a
sulfate. When the battery is recharged, the chemical chemical change.
reaction is reversed: the lead sulfate reforms into lead
dioxide and lead. With the plates restored to their 41 H1 ¾¾ ® 2 He 4 + 2 +1e 0 + g + Energy
original condition, the process may now be repeated. 57 (a) Aqua-regia is a mixture of concentrated nitric acid
42. (b) Light production in fireflies is due to a type of and concentrated hydrochloric acid in the ratio of 1 : 3.
chemical reaction called bioluminescence. This It is a very powerful oxidising mixture and hence, used
process occurs in specialised light-emitting organs, to clean the surface of gold ornaments.
usually on a firefly's lower abdomen. The enzyme 58. (a) During the setting of cement, the structure has to be
luciferase acts on the luciferin, in the presence of cooled by spraying water because setting of cement
magnesium ions, ATP, and oxygen to produce light. is an exothermic process and involves hydration of
43. (c) Kevlar, a polymeric material is used for making bullet calcium aluminates and calcium silicates.
proof jacket.
EBD_7367
28
C- Chemistry

59. (a) Superphosphate of lime can be easily assimilated by hence, they are used as a sacrificial anode while tin is
plants because it is soluble in water. less reactive than iron hence, it cannot be used as a
60. (c) Radioactivity is not a periodic property. It is a nuclear sacrificial anode.
property. Atomic size, valency and electronegativity 77. (c) Sodium thiosulphate (Na2S2O3) solution is used as
are periodic properties because they show a trend on fixer in photography, it removes undercomposed AgBr
moving from one side to the other in the periodic table. as a soluble silver thiosulphate complex.
61. (a) On reaction with ethanol the colour of the solution AgBr + 2Na 2S2O3 ® Na 3[Ag(S2O3 ) 2 ]+ NaBr
changes from, orange to green. Sodiumargento
62. (a) The tanks used by Scuba divers are filled with air thiosulphate
diluted with helium (i.e., He-O2 mixture) because unlike 78. (a) Oxides of sulphur and nitrogen present in high
nitrogen, helium is not soluble in blood even under concentration in air dissolve in rain drops and form
high pressure. oxyacids of sulphur and nitrogen. These oxyacids are
63. (c) Substances like graphite and heavy water are used as highly acidic in nature and therefore, they make rain
moderator to slow down the speed of neutrons. Cd or water acidic.
B are used as control rods to control rate of fission by
absorbing neutrons. 4NO2 + 2H2O + O2 ¾¾
® 4HNO3
Coolant like heavy water or molten sodium is used to
2SO2 + 2H2O + O2 ¾¾ ® 2H2SO4
carry away the heat.
64. (c) The process of depositing a thin layer of one metal The pH of acid rain is 4 - 5
over the surface of other metal by the process of 79. (a) On mixing with water, plaster of Paris hardens because
electrolysis is known as electroplating. Hence, when it takes up the water of crystallisation again and thus,
items or jewellery made of metals such as copper or converts back into the dihydrate i.e., gypsum.
nickel are placed in a solution having a salt of gold, a 1 1
thin film of gold is deposited by passing an electric CaSO4 . H 2 O + 1 H 2O ® CaSO 4 .2H 2O
2 2 Gypsum
current. Plaster of Paris
65. (c) Heavy water or deuterium oxide (D2O) is an oxide of 80. (d) All liquids are not conductors of electricity. Only the
deuterium which is an isotope of hydrogen. liquids which contain free ions or electrons, can
66. (b) Ethyl mercaptan or thioethanol (C2H5SH) is added conduct electricity.
during filling of LPG cylinders. It is a strong smelling 81. (a) Gypsum (CaSO4 × 2H2O) is added to clinker during
compound and help in the detection of gas leakage. cement manufacturing to decrease the rate of setting
67. (b) Atoms are neutral. When an atom loses electron, it of cement so, that it gets sufficiently hardens.
becomes positively charged and when an atom gains 82. (c) In lemon juice citric acid is present. Hence, acids are
electron, it becomes negatively charged. Hence, a conductors of electricity. The aqueous solution of
body is charged negatively when it has acquired some lime juice conducts electricity due to the presence of
electrons from outside. ions in it.
68. (d) Titanium (Ti) metal is used for making boats because 83. (a) Ground water is polluted by fluoride rich rocks which
it does not corrode by the sea water. contain fluoride in large quantity.
69. (c) Contact lenses are made from lucite. It is a polymer of 84. (d) Pearl consists of approximately 85% calcium
methyl methacrylate. The other names of lucite are carbonate.
plexiglass, acrylite and perspex. 85. (c) The method of determining the age of an object
70. (a) Water has a high specific heat. containing organic material by using the properties
71. (a) The water will freeze because on Moon the of radioactive isotope of carbon is known as
temperature is below the freezing point i.e., very low. Radiocarbon dating or carbon dating. Radioactive
72. (d) When calcium oxide or quicklime (CaO) is dissolved isotopes have unstable nuclei that break down i.e.,
in water, calcium hydroxide or limewater is obtained decay and form other elements. C-12 and C-14 are the
and there is a release of heat. two isotopes of carbon which is used to determine
CaO + H 2 O ® Ca(OH) 2 + Heat the age of fossil.
Quick lim e Lime water 86. (a) Acetylene is the nontoxic gases that help in formation
73. (d) Air would expand the most on being heated. of enzymes which riper fruit. Ripening is a process in
74. (d) Lactic acid occurs in sour curd. fruits causes them to become more palatable.
Tartaric acid occurs in tamarind. 87. (c) Vermicompost is the product or process of composting
Oxalic acid occurs in tomato. utilising various species of worms, usually red
Citric acid occurs in all citrus fruits like orange, lemon wigglers, white worms and earthworms to create a
etc. heterogeneous mixture of decomposing vegetables
75. (c) Silicosis is a chronic lung disease caused by breathing or food waste, bedding material and vernicast.
in silica dust. Silicosis (silico tuberculosis) occurs in Containing water soluble nutrients, vermicompost is
workers whose occupation is related to pottery, an excellent, nutrient rich organic fertilizer and soil
ceramic, and glass industry. conditioner.
76. (a) The method of protection of iron from corrosion by 88. (b) The macro nutrients provided by inorganic fertilizers
using a more reactive metal is known as sacrificial are magnesium, manganese and sulphur.
protection or cathodic protection. Zinc, magnesium 89. (d) Except (d) all statements are correct.
and aluminium metals are more reactive than iron 90. (a)
Chemistry 29
C-

91. (d) All Alkalis are bases but all bases are not alkalis · ·
Hv
because all bases are not soluble in water but alkalis HOCl(g) ¾¾¾
® O H(g) + C l(g)
like soap are water soluble bases.
·
92. (a) The pH of fresh ground water slightly decreases upon Hv
Cl 2 (g) ¾¾¾® 2C l(g)
exposure to air because when it comes to air, it absorbs
the carbon dioxide (CO2), and for carbonic acid (H2 CO3) The following reaction display how Cl atoms have an
which is a weak acid i.e. less pH. ozone destrry cycle.
93. (c) In making of bullet proof material. Polyethylene ·
polymers is used because it is cheap, flexible, durable ® C F2 Cl + Cl·
CF2 Cl2 ¾¾
and chemically resistant. · ·
94. (a) The periodic table is a tabular arrangement of the C l + O3 ¾¾
® Cl O+ O 2
elements organised on a basis of their properties.
· ·
95. (d) Soaps cannot be used in acidic condition as they are 2ClO + O 2 ¾¾ ® 2Cl + 2O 2
converted into free fatty acid which form a scum.
96. (a) 112. (a) Carbon (C) forms a large number of compounds due
97. (a) A heterogeneous mixture is a mixture of more than to its two characteristics properties.
two elements. (i) Catenation (ability to link with other carbon atoms)
98. (a) The carbon dioxide reacts with the lime water to form (ii) Tetra valency (valency of carbon is 4)
calcium carbonate that is insoluble. 113. (d) It is a simple displacement reaction in which Hydrogen
is displaced by Sodium metal, results in the formation
CaO + CO 2 ¾¾ ® CaCO 3 ¯ This substance precipitate of sodium chloride (NaCl). Also the oxidation state of
out of solution making the lime water cloudy. each atom is same in both sides.
99. (b) In soft drinks bicarbonate salts acts as acidity 114. (b) Water shows maximum density at 4°C. At freezing point
regulators. Bicarbonate of soda is effective at it expands and density decreases.
regulating the pH levels of other substances; it ensures 115. (c) Mercury (Hg), a metal is a liquid at room temperature
that the substance is neither too alkaline nor too acidic. and standard pressure.
100. (b) Generally are weak bases. Antacid which naturalises Bromine (Br 2) a non metal is a liquid at room
stomach acidity, a commonly used antacid is temperature and standard pressure.
magnesium hydroxide. Magnesium hydroxide is 116. (d) A compound calcium sulphate (CaSO4) which is a
common component of antacids and laxatives. white solid is highly hygroscopic in nature. It has a
101. (c) Iodised salt is a compound formed by combination of very remarkable property of setting into a hard, solid
potassium iodide and common salt. mass on wetting with water.
102. (c) 117. (d) Smoke is a collection of airborne solid and liquid
103. (c) Carbon monoxide readily combines with the particulates and gases emitted when a material
haemoglobin of the blood. undergoes combustion or pyrolysis, together with
104. (a) Oxides of sulphur and nitrogen are responsible for entrapment of liquid droplets or solid particulates in a
acid rain. Sulphur dioxide and nitrogen oxide, react flowing gas.
with the water molecules in the atmosphere to produce 118. (d) Oxygen is supporter of combustion. Though it is not
acids. combustible. Combustion is an oxidation process and
105. (b) Magnesium burns a very bright white, so it is used to this cannot occur without the presence of oxygen.
add white sparks or improve the overall brilliance of a 119. (b) Rust is an iron oxide formed by the redox reaction of
firework. iron and oxygen in the presence of water or air moisture
106. (a) The main cause of hardness of water is presence of
4Fe + 2O2 + xH 2O ¾¾
® 2Fe 2O3.xH 2 O
Ca or Mg ions. When hard water containing these
From air Hydrated iron oxide (rust)
ions is treated with soap solution it reacts to form
white crudy ppt known as scum. Hence rust is a compound.
107. (c) Catalysts are chemical substances used in chemical 120. (a) Both metal and glass expand when heated. Here glass
reactions that increases or decreases the rate of is a non-metal. Metal expands more than non-metal
reaction without taking part in reaction positive due to temperature change.
catalysts speed up the reactions, where as negative 121. (c) Silica and aluminium are the two main constituents of
catalysts slow them down. granite. Granite contains 70-77% of silica and 11-13%
108. (c) Carbon atoms is diamond form a rigid, three of aluminium in the form of (Al2O3).
dimensional structure with each carbon atom bonded 122. (a) An aquifer is an underground layer of water bearing
to four other carbon atoms. rock. Water bearing rocks are permeable i.e., they have
109. (c) Food cans are coated with tin instead of zinc because opening that liquids and gases can pass through. But
zinc is more reactive than tin. Tin is only reactive to clay layers are poor aquifers. Clay minerals are dense,
powerful acids such as tri-sodium phosphate, while impermeable material and act as an 'aquifuge' i.e., a
zinc is reactive towards acids. layer of material that is almost impenetrable to water.
110. (a) Carbon dioxide is responsible for the expansion of 123. (b) It helps in elimination of lead oxide.
water in the ocean. 124. (d) Silicosis in human is lung disease caused by inhalation
111. (a) Hypochlorous acid and chlorine on photochemical of crystalline sillica-dust.
decomposition gives nascent chlorine which further 125. (c) Amnion is a membrane, filled with fluid, closely covers
deplete ozone layer the embryo.
EBD_7367
30
C- Chemistry

The sex of the embryo may be determined by 142. (d) Acetylene is the only commercial fuel gas that can be
ultrasound as early as 11 weeks gestation. used for both cutting and welding applications.
126. (b) The blue litmus paper turns red in acidic solution and 143. (b) There are about seven nutrients essential to plant
blue in basic solution. The neutral solution does not growth and health that are only needed in very small
affect the litmus paper. Now since common salt solution quantities. These are Boron ,Chlorine, Copper, Iron,
is neutral hence do not show any colour change on Manganese, Molybdenum and Zinc.
litmus paper. 144. (c) The correct match is as follows:
127. (a) Oil become rancid due to oxidation. Oil contains fatty Chloroflurocarbon- Depletion in ozone layer in the
acid which react in presence of oxygen and become atmosphere
rancid. Rancid oil is a major source of destructive free Sulphur dioxide- Acid rain
radicals in our diet. Lead compound- Harmful for human nervous system
128. (b) Bagasse is sugarcane fibre waste left after juice Carbon dioxide- Topmost contribution to greenhouse
extraction.It is used for the production of paper. effect
129. (b) Biogas is primarily methane (CH4) and carbon dioxide 145. (d) Lithium is also a highly reactive element, meaning that
(CO2) and may have small amounts of hydrogen a lot of energy can be stored in its atomic bonds. This
sulphide (H2S), moisture and siloxanes. translates into a very high energy density for lithium-
130. (b) Graphite and graphite powder are a naturally occurring ion batteries. It is used in mobile phone, laptops etc.
form of carbon which serves as an industrial lubricant. 146. (b) Exfoliation involves the removal of the oldest dead
131. (b) Tritium is a radioactive isotope of hydrogen. The skin cells on the skin's outermost surface and has
nucleus of tritium contains one proton and two been used for many years to help maintain healthy
neutrons. skin. It can be achieved through mechanical or
132. (a) Lime water is actually Ca(OH)2. When it reacts with chemical means.
CO2, insoluble CaCO3 is formed which turns solution 147. (b) Phosphorus is of the two types - White and red. White
milky. When excess CO2 is added, CaCO3 reacts with phosphorus is highly reactive and catches fire at
water and CO2 to form Calcium bicarbonate which is normal room temperature but red phosphorus does
soluble. That is why the solution clears out. not show such nature.
133. (c) Graphite is a very good conductor, while diamond has 148. (a) Exponent - Law
a very low electrical conductivity.
John Dalton - Law of multiple
134. (c) Frying an egg involves a chemical change. The heat in
the frying process gives energy to the egg's molecules proportion
and brings about a permanent change in the substance. Joseph Proust - Law of definite
135. (c) Rest of the statements are true. proportion by weight
136. (a) Chromium oxide in paints makes the colour of paint Antoine Lavoisier - Law of Conservation of
green. mass
137. (a) Milk of magnesia (magnesium hydroxide) is most Joseph Louis Gay Lussac - Law of definite
commonly found as a liquid. The magnesium in the proportion by volume.
liquid can both neutralize excess stomach acid to treat 149. (c) Hydrocarbon is related with petro chemicals such as
indigestion. crude oil and petrolium. Environmental pollution by
138. (b) The white solid remains in the container when hard
crude oil is generally caused during disposal
water is evaporated completely is mainly due to the
presence of chlorides and carbonates of Ca and Mg operations and spilling in water bodies like- oceans,
and also due to presence of traces of sulphates of Ca seas and rivers.
and Mg. 150. (d) Ice is less dense in comparison to liquid water. Actually
139. (d) NaCl has same molecular weight and equivalent weight when water freezes, water (H2O) molecules form a
crystalline structure by Hydrogen bonds. These bonds
Molecular weight of salt
Equivalent weight of salt = Total charge on cation cause molecules to push apart lowering the density of
water (H2O)
Molecular weight of NaCl = 23 + 35.5 = 58.5
58.5 151. (d) The structure of diamond.
\ Equivalent weight of NaCl = = 58.5
1
140. (c) Due to more expansion of vassel, plate comes out
easily.
141. (a) Electricity is produced through dry cell from chemical
energy. The cells from which electric energy is derived
by irreversible chemical action are called primary cells.
The primary cell is capable of providing an EMF when In diamond each carbon (C) atom shares electrons
its constituent's two electrodes and a suitable (e-) with four other carbon atoms forming four single
electrolyte are assembled together. The three main bond. shape of structure appears three dimensional
primary cells are the Daniel cell, the Leclanche cell, network structure. It is very hard structure being used
and the dry cell. None of these cells can be recharged in cutting and sharpening tools. It cannot be used as
electrically.
lubricant.
Chemistry C-31
152. (a) John Dalton (1766-1844), an English chemist is known leached through soil into groundwater. The heavy use
for the work in the development of modern atomic of nitrogenous fertilizers in cropping systems is the
theory. He was awarded the Royal medal for this. largest contributor to anthropogenic nitrogen in
153. (c) Kinetic energy is the same for molecules of all gases at groundwater worldwide.
a given temperature. 168. (d) Due to the small size of lithium, it will polarize the large
154. (d) The Sound propagation is facilitated by the collisions anion oxygen, the C–OC–O bond will weaken and Li–
between the particles of medium. The inter-molecular OLi–O bond will strengthen, they will split, and they
distance in solids is very small whereas in gases it is will not be in the solid state.
very high. Therefore the velocity of sound is maximum 169. (d) Gamma decay, type of radioactivity in which some
in solids and minimum in gases. unstable atomic nuclei dissipate excess energy by a
155. (c) q = m D Hf spontaneous electromagnetic process.
Where, m = mass, q = heat energy In the radioactive substances, gamma decay occurs
D Hf = heat of fusion after the alpha and the beta decay. As the radioactive
q = (100g) × 333.6 J/g = 33360 J substance has already released some of its particles in
156. (a) Burning of paper is a chemical change because it is the alpha and the beta decay it simply wants to release
irreversible. energy burst of electromagnetic radiations. The nuclei
157. (c) Hydrogen has three naturally occuring isotopes has comes to the stable state and releases energy in
denoted by 1H, 2H and 3H. the form of gamma rays. In the gamma decay as no
158. (c) Atoms of the same element always have the same particles either protons or nucleons are released so
number of protons (is it had a different number of the element before the radioactive gamma decay and
protons it would be a different element.) after the gamma decay remains same.
159. (c) Natural rubber, coming from latex of Hevea brasiliensis 170. (c) Magnesium (Mg) is main elements of Chlorophyll,
containing traces of impurities like protein, dirt, etc. found in plants which trap energy of sunlight in
while synthetic rubber is made by the polymerization photosynthesis.
of a variety of petroleum-based precursors. Calcium (Ca) and Magnesium (Mg) work together to
160. (d) Bakelite was invented by Leo Baekeland in New York control muscle contraction. Osteoporosis is caused
in 1907. It is the first man made plastic. due to the deficiency in our body.
161. (d) Methyl Isocyanate gas (MIC) is used in preparation Strontium (Sr) is a soft, silver-yellow, alkaline-earth
of pesticides. metal. It has three allotropic crystalline forms and in
162. (c) Two systems are said to be in thermal equilibrium if is its physical and chemical properties it is similar to
no heat transfer takes place as both the systems have calcium and barium. It gives red colour to flame.
same temperature. Barium (Ba) Sulphate compound is used in medicine
163. (c) German silver is a mixture of copper, zinc and nickel. It to examine the elementary canal of patient.
is an alloy metal. It is used to make pots and toys due 171. (d) The oxidation state of hydrogen is +1 in its compounds,
to hardness toughness and resistance to corrosion. except for metal hydrides, such as NaH, LiH, etc., in
164. (d) Eight allotropes of carbon: a) Diamond, b) Graphite, c) which the oxidation state for H is –1.
Lonsdaleite, d) C60 (Buckminsterfullerene or 172. (d) A Standard Hydrogen Electrode (SHE) is an electrode
buckyball), e) C540, f) C70, g) Amorphous carbon, and that scientists use for reference on all half-cell potential
h) single-walled carbon nanotube or buckytube. reactions. The value of the standard electrode potential
Hence, (d) is the correct answer. is zero at 25°C, which forms the basis one needs to
165. (b) Bleaching of wood pulp is the chemical processing of calculate cell potentials using different electrodes or
wood pulp to lighten its color and whiten the pulp. different concentrations.
The primary product of wood pulp is paper, for which 173. (c) Physical changes lead to change in the form of a
whiteness (similar to, but distinct from brightness) is substance, without any change in chemical
an important characteristic.[1] These processes and composition. Coal is composed of carbon, oxygen,
chemistry are also applicable to the bleaching of non- and hydrogen. Wood is composed of cellulose,
wood pulps, such as those made from bamboo or kenaf. hemicellulose and lignin. Burning of wood, coal and
166. (a) Dolomite is used in agriculture to raise the pH of overly potassium chlorate leads to chemical reaction as their
acidic soil and provide nutrients for plants. components are chemically active. Platinum being an
It contains calcium carbonate and magnesium inert element does not show chemical reaction and
carbonate, two basic pH-raising ingredients that show only physical change such as rise in temperature.
counteract the acids in soil and peat. 174. (d) Multiple-distilled water is considered to be pure. pH
167. (a) Nitrate can also enter the groundwater via excessive of pure water is 7 as the number of hydroxyl ions and
use of fertilizers, including manure spreading. The hydrogen ions in pure water is balanced. A strong
excessive use of nitrogen-containing fertilizers (be acid completely ionizes when dissolved in water and
they synthetic or natural) is particularly damaging, as provides hydrogen ions. Strong acid has pH of 0.
much of the nitrogen that is not taken up by plants is Similarly, a strong base/alkali also ionizes completely
transformed into nitrate which is easily leached. in water and provides hydroxyl ions. Strong base has
The nutrients, especially nitrates, in fertilizers can pH of 14.
cause problems for natural habitats and for human 175. (d) Atomic number, which equals to the number of protons
health if they are washed off soil into watercourses or in the nucleus of an atom, is the characteristic property
EBD_7367
32
C- Chemistry

of an element and determines its chemical properties From the above equation it is obvious that velocity of
and position in the periodic table. Mass number is the particle is inversely proportional to the mass of particle
total number of protons and neutrons in the nucleus so, electron will gain more speed as it has lower mass
of an element. Density is the mass per unit volume of than that of proton. Also, as the energy is fully kinetic
a substance. Boiling point is the temperature at which so electron will have greater energy as a result of
a substance boils. greater velocity.
176. (c) Calcium and argon are isobars because they have same 184. (b) A substance that accepts a proton is a base. In the
atomic mass. Two elements with same neutron number, above reaction, water is acting as a base.
but different proton number are called isotones. Each 185. (d) There are 12 atoms of hydrogen per molecule of
of two or more forms of the same element with equal glucose, which has a molar mass of 180g/mol. Hence
numbers of protons but different numbers of neutrons the number of moles of hydrogen = 12 × 1.50 ÷ 180 =
is called isotope. Isochore is the graphic 0.10mol. Hence the number of hydrogen atoms is found
representation showing lines or surface of constant by multiplying this by the Avogadro Constant, 6.02 ×
volume of a physical system. 10²³. This gives you 6.02 × 10²² atoms.
177. (d) ‘Plum Pudding Model,’ the scientific model of atom 186. (b) Plaster of paris, quick-setting gypsum plaster
was proposed by J. J. Thomson in which he said that consisting of a fine white powder (calcium sulfate
an atom is made up of electrons surrounded by hemihydrate), which hardens when moistened and
positive charge to balance the negative charges of allowed to dry. Known since ancient times, plaster of
electrons. Negatively charged electrons are called paris is so called because of its preparation from the
‘plums’ surrounded by positively charged particles
abundant gypsum found near Paris.
‘pudding.’
187. (c) An emulsion is a mixture of two or more liquids that
178. (c) Atomic mass of Sodium = 23 gram = 1 mole of Sodium
are normally immiscible.
No. of moles in 46 grams of Sodium = 2;
1 Avogadro’s Number (N) = 1 mole of substance 188. (d) Strontium-90, Iodine-131, Cesium-137 enters/enter the
Therefore, 2 moles of Sodium = 2N human body through food chain and causes/cause
179. (c) States of matter can be defined as the forms a many physiological disorders
substance can occur, and are of five types: solid, 189. (b) In a nuclear reactor, the reaction is controlled, while in
liquid, gas, plasma and Bose-Einstein condensates nuclear bombs, the reaction is uncontrolled. In a
(BEC). Each form differs from the other mainly in nuclear reactor, all operating reactors are ‘critical’,
structures and densities. Plasma are highly charged while there is no question of ‘critically’ in case of a
particles with high kinetic energy. BEC is a state of nuclear bomb.
matter of a dilute gas of bosons cooled close to 190. (b) Tincture of iodine, iodine tincture, or weak iodine
absolute zero. solution is an antiseptic. It is usually 2–7% elemental
180. (d) The period of an element in the periodic table depends iodine, along with potassium iodide or sodium iodide,
on the total number of electron shells in the atom of dissolved in a mixture of ethanol and water. Tincture
that element. Chemical property of an element depends solutions are characterized by the presence of alcohol.
on the valence electrons which participate in bond 191. (a) Carbon dioxide (CO2) gas dissolved in water can cause
formation. Isotopes are two or more forms of the same water to become acidic.
element with equal numbers of protons but different CO2 + H2O ® H2CO3 (Carbonic acid)
numbers of neutrons. Total number of protons and 192. (b) It is a double replacement reaction. Fe2 O3 (s) +
neutrons is mass number. Chemical property is 3 CO(g) = 2 Fe(l) + 3 CO2(g).
independent of neutrons and protons. 193. (b) Xenon is used in fluorescent bulbs, flash bulbs and
181. (b) Normality is a way of expressing concentration of a lasers. Xenon emits an instant, intense light when
solution and can be calculated by the formula present in discharge tubes. This property of xenon is
Normality (N) = No. of gram equivalent of solute/ utilized in high-speed electronic flash bulbs used by
volume of solution (in liter) photographers.
Gram equivalent weight of H2SO4 = Molecular weight/ 194. (b) A compound is a substance that results from a
No. of Hydrogen atom = 98/2 = 49 gm combination of two or more different chemical element
No. of gram equivalent in 49 gm of H2SO4 = 1 So, s, in such a way that the atoms of the different elements
Normality in 1 liter of solution = 1/1 = 1 N are held together by chemical bonds that are difficult
182. (a) A pearl is a hard object produced within the soft tissue to break.
of a living shelled mollusk. Just like the shell of a 195. (d) Temporary hardness is due to the presence of calcium
mollusk, a pearl is composed of calcium carbonate hydrogencarbonate Ca(HCO3)2(aq) and magnesium
(mainly aragonite or a mixture of aragonite and calcite) hydrogencarbonate Mg(HCO3)2(aq). Both calcium
in minute crystalline form, which has been deposited hydrogencarbonate and magnesium hydrogen
in concentric layers. The ideal pearl is perfectly round carbonate decompose when heated. The original
and smooth, but many other shapes, known as
insoluble carbonate is reformed. This happens when
baroque pearls, can occur.
water is boiled.
183. (b) When accelerated from rest the energy is fully kinetic
196. (d) Element with 18 protons and 22 neutrons will be the
so
isobar of calcium if the atomic number of calcium is 2 0
KE= charge*potential difference
i.e 1/2 Mv2=Q.V and its mass number is 40.
Biology 33
C-

C HA P T E R
BIOLOGY
36
1. Consider the following [2007 - I] Select the correct answer using the code given below
Enzymes and protein can be correlated with each other in (a) 1 and 3 (b) 2 and 4
the following ways (c) 1, 2 and 4 (d) All of these
1. All proteins are enzyme. 12. In the context of ecology and environment, what does the
2. All enzymes are protein. Red Data Book pertain to? [2007 - II]
3. All enzymes are not protein. (a) Details of harmful levels of various pollutants
4. All proteins are not enzyme. (b) A complete list of all endangered plants and animals
Which of the above are correct? (c) A description of the consequences of nuclear
(a) 1 and 3 (b) 2 and 4 holocaust
(c) 1, 2 and 3 (d) 2, 3 and 4 (d) A description of the sociological and psychological
2. Consider the following statements in respect of thyroid consequence of genetically modified plants and
gland. [2007 - I] animals
1. It is situated in the neck. 13. Which of the following is not a bird? [2007 - II]
2. It is vital to maintain of normal body temperature. (a) Bat (b) Emu
Which of the statements given above is/are correct? (c) Kiwi (d) Ostrich
(a) Only 1 (b) Only 2 14. Match the following lists. [2007- II]
(c) Both 1 and 2 (d) Neither 1 nor 2 Lis t I Lis t II
3. Which one of the following is not a constituent of biogas?
[2007 - II] (S cientis t) (W ork )
(a) Methane (b) Carbon dioxide A . F G Ban tin g 1. Vaccination fo r s mall p ox
(c) Hydrogen (d) Nitrogen dioxide B. J Lis ter 2. Germ theo ry
4. A mother of blood group O has a group O child. What
C. Lou is Pas teu r 3. Us e o f carbo lic acid as an
could be the blood group of father of the child?[2007 - II]
(a) Only O (b) A or B or O antis eptic
(c) A or B (d) Only AB D. E Jenn er 4. Dis co very o f in s ulin
5. What does sphygmomanometer measure? [2007 - II] Codes
(a) Blood pressure A B C D
(b) Velocity of fluids (a) 4 3 2 1
(c) Temperature (b) 4 2 1 3
(d) Curvature of spherical surfaces (c) 3 4 2 1
6. What does water gas comprise of? [2007 - II] (d) 1 4 3 2
(a) Carbon monoxide and hydrogen 15. Which of the following part of blood carry out the function
(b) Carbon dioxide and hvdrogen of body defence? [2008 - I]
(c) Carbon monoxide and methane
(a) Red blood cells (b) White blood cells
(d) Carbon dioxide and methane
7. Which one of the following causes the chikungunya (c) Platelets (d) Haemoglobins
disease? [2007 - II] 16. Match the following
(a) Bacteria (b) Helminthic worm Lis t I Lis t II
(c) Protozoan (d) Virus (Dis eas e) (Part of Human Body Affected)
8. Which one of the following vitamins is abundant in guava A . Con jun ctiv itis 1. Ey es
fruits? [2007 - II]
(a) Vitamin-A (b) Vitamin-B12 B. Dermatitis 2. Join ts
(c) Vitamin-C (d) Vitamin-D C. Gou t 3. Skin
9. Which one of the following vitamins helps in clotting of D. M enin gitis 4. Sp inal cord
blood? [2007 - II]
(a) Vitamin-A (b) Vitamin-B6 Codes
(c) Vitamin-D (d) Vitamin-K A B C D
10. Which one of the following glands in the human body (a) 2 4 1 3
stores iodine? [2007 - II] (b) 1 3 2 4
(a) Parathyroid (b) Thyroid (c) 2 3 1 4
(c) Pituitary (d) Adrenal (d) 1 4 2 3
11. Which of the following diseases are preventable by vaccine? 17. Which one of the following is not an insect borne disease?
[2007 - II] [2008 - I]
1. Tetanus 2. Polio (a) Beri-beri (b) Kala-azar
3. Leprosy 4. Pertusis (c) Malaria (d) Plague
EBD_7367
34
C- Biology

18. Which of the following gases is released from rice fields in 30. What does airbag, used in safety of car driver, contain?
the most prominent quantities? [2008 - I] [2008 - II]
(a) Carbon dioxide (b) Methane (a) Sodium bicarbonate (b) Sodium azide
(c) Carbon monoxide (d) Sulphur dioxide (c) Sodium nitrite (d) Sodium peroxide
19. Consider the following statements. [2008 - I] 31. The characterisitic odour of garlic is due to which one of
1. Cigarette smoking exposes a person to benzene. the following? [2008 - II]
2. Benzene is a known carcinogen. (a) Chlorine-containing compounds
Which of the statements given above is/are correct? (b) Fluorine-containing compounds
(a) Only 1 (b) Only 2 (c) Nitrogen-containing compounds
(c) Both 1 and 2 (d) Neither 1 nor 2 (d) Sulphur-containing compounds
20. Which one of the following glands produces the growth 32. Which of the following plants is not capable of
hormone (somatotrophin)? [2008 - I] manufacturing its own food? [2008 - II]
(a) Algae (b) Mushroom
(a) Adrenal (b) Pancreas
(c) Carrot (d) Cabbage
(c) Pituitary (d) Thyroid
33. Which one of the following is considered as the easily
21. In the human body, Cowper's glands form a part of which digestible source of protein? [2008 - II]
one of the following system? [2008 - I] (a) Egg albumin (b) Soyabean
(a) Digestive system (b) Endocrine system (c) Fish flesh (d) Red meat
(c) Reproductive system (d) Nervous system 34. In normal adult human, what is the rate of heart beat per
22. Cloves, used as a spice, are derived from which of the minute? [2008 - II]
following plant parts? [2008 - I] (a) 72-80 (b) 70-75
(a) Seeds (b) Fruits (c) 80-97 (d) 82-87
(c) Flower buds (d) Young leaves 35. The terms lubb and dubb relates to which one of the
23. Which one among the following kinds of organisms resides following? [2008 - II]
in the roots of pulse plants to do nitrogen fixation? (a) Heart (b) Eyes
[2008 - I] (c) Teeth (d) Lungs
(a) Bacteria (b) Fungi 36. In human body, what is the number of cervical vertebrae?
(c) Protozoa (d) Virus [2008 - II]
24. The persons working in textile factories such as carpet (a) 5 (b) 7
weavers are exposed to which of the following occupational (c) 8 (d) 12
diseases? [2008 - II] 37. Which of the following is a rich source of energy?
(a) Asbestosis [2008 - II]
(b) Asthma and tuberculosis (a) Protein (b) Lipid
(c) Silicosis (c) Carbohydrate (d) Vitamin
(d) Siderosis 38. By using which one of the following techniques, is DNA
25. Dim-vision in the evening and night results from the fingerprinting done? [2008 - II]
deficiency of which one of the following? [2008 - II] (a) ELISA (b) RIA
(a) Vitamin-A (b) Vitamin-E (c) Northern Blotting (d) Southern Blotting
(c) Vitamin-B12 (d) Vitamin-C 39. Which chamber of human heart pumps fully oxygenated
26. Which of the following nutrients is not a structural blood to aorta and thence to the body? [2008 - II]
(a) Right auricle (b) Left auricle
component of the plant? [2008 - II]
(c) Right ventricle (d) Left ventricle
(a) Nitrogen (b) Calcium
40. Which one of the following is a major constituent of biogas?
(c) Phosphorus (d) Potassium [2008 - II]
27. Assertion (A): Red blood cells burst when placed in water. (a) Carbon dioxide (b) Methane
Reason (R): Due to osmosis, water enters into red blood (c) Hydrogen (d) Nitrogen dioxide
cells. [2008 - II] 41. Which one of the following is responsible for converting
(a) Both A and R are true and R is the correct explanation milk into curd? [2008 - II]
of A (a) Fungi (b) Bacteria
(b) Both A and R are true, but R is not the correct (c) Virus (d) None of these
explanation of A 42. Sweating during exercise indicates operation of which one
(c) A is true, but R is false of the following processes in the human body?
(d) A is false, but R is true [2008 - II]
28. Which one of the following is correct? A concrete wall (a) Enthalpy (b) Phagocytosis
generally? [2008 - II] (c) Homeostasis (d) Osmoregulation
(a) only reflects sound 43. Malaria in the human body is caused by which one of the
(b) only absorbs sound following organisms? [2008 - II]
(c) only transmits sound (a) Bacteria (b) Virus
(d) absorbs and transmits sound (c) Mosquito (d) Protozoan
29. Which colour of heat radiation represents the highest 44. Which one of the following diseases is caused by virus?
temperature? [2008 - II] [2008 - II]
(a) Blood red (b) Dark cherry (a) Tuberculosis (b) Typhoid
(c) Salmon (d) White (c) Influenza (d) Diphtheria
Biology C-35
45. Between which one of the following sets of blood groups, 53. The genetically engineered 'Golden Rice' is rich in which of
is the blood transfusion possible? [2008 - II] the following? [2009 - I]
(a) A and O (b) B and A (a) Vitamin-A and nicotinic acid
(c) A and AB (d) AB and O (b) b-carotene and folic acid
46. In human beings, the opening of the stomach into the small (c) b-carotene and iron
intestine is called [2009 - I] (d) Vitamin-A and niacin
(a) caecum (b) ileum 54. Bryophytes are photosynthetic but do not have vascular
(c) oesophagus (d) pylorus tissue and true roots. This feature enables them to resemble
47. Match the following [2009 - I] with which of the following? [2009 - I]
List-I List-II (a) Fungi (b) Algae
(c) Pteridophytes (d) Angiosperms
(Vitamin) (Chemica l Compound) 55. Which one of the following is a freeliving bacterium that
A. Vitamin-A 1. Thiamine helps in nitrogen fixation in soil? [2009 - I]
B. Vitamin-B 1 2. Retinol (a) Azotobacter (b) Anabaena
(c) Azolla (d) Nostoc
C. Vitamin-C 3. Ascorbic acid 56. Which one of the following is an example of vestigial organ
D. Vitamin-E 4. Tocopherol in man? [2009 - I]
Codes (a) Jaw apparatus (b) Ear muscles
A B C D (c) Canine teeth (d) Humerus
(a) 4 1 3 2 57. Which one of the following characteristics is common
(b) 2 3 1 4 among parrot, platypus and kangaroo? [2009 - I]
(c) 4 3 1 2 (a) Oviparity (b) Toothless jaws
(d) 2 1 3 4 (c) Homothermy (d) Functional postanal tail
48. Which one of the following is considered as the drug of 58. Which one of the following is present in chlorophyll which
last resort for human beings? [2009 - I] gives a green colour to plant leaves? [2009 - II]
(a) Penicillin (b) Tetracycline (a) Calcium (b) Magnesium
(c) Chloramphenicol (d) Streptomycin (c) Iron (d) Manganese
49. Match the following [2009 - I] 59. Which one of the following is responsible for the stimulating
effect of tea? [2009 - II]
Lis t I Lis t II (a) Tannin (b) Steroid
(Medicinal Product) (S ource) (c) Alkaloid (d) Flavonoid
60. Consider the follwing statement about bioremediation.
A . Quin ine 1. Po p py plant 1. It may be defined as any process that uses
B. M orph in e 2. Bacteriu m microorganisms or their enzymes to return the
C. Pen icillin 3. Cin cho na bark environment altered by contaminants to its original
condition.
D. Tetracy clin e 4. Fu n gu s
2. Bioremediation may be employed in order to attack
Codes specific contaminants, such as chlorinated pesticides
A B C D that are degraded by bacteria. [2009 - II]
(a) 3 4 1 2 Which of the statements given above is/are correct?
(b) 2 3 1 4 (a) Only 1 (b) Only 2
(c) 3 1 4 2 (c) 1 and 2 (d) Neither 1 nor 2
(d) 2 1 3 4 61. The branches of this tree root themselves like new trees
50. Wavelengths of which of the following colour of the visible over a large area. The roots then give rise to more trunks
spectrum of light are maximally absorbed by green plants? and branches. Because of this characteristic and its
[2009 - I] longevity, this tree is considered immortal and is an integral
(a) Green and Yeilow (b) Red and Blue part of the myths and legends of India. Which tree is this?
(c) Green and Red (d) Blue and Yellow [2009 - II]
51. Which one of the following plants is preferred for mixed (a) Banyan (b) Neem
cropping in order to enhance the bioavailability of nitrogen? (c) Tamarind(Imli) (d) Peepal
[2009 - I] 62. Assertion (A): The safety air bags fitted in some cars inflate
(a) Wheat (b) Gram during head-on impact of the car. [2009 - II]
(c) Maize (d) Barley Reason (R): The inflation is due to pumping of air into the
52. The plant dye Henna imparts orange-red colour to skin and balloon during the impact.
hairs due to its reaction with which of the following? (a) Both A and R are true and R is the correct explanation
[2009 - I] of A
(a) Proteins and amino acids (b) Both A and R are true, but R is not the correct
(b) Lipids explanation of A
(c) Carbohydrates (c) A ts true, but R is false
(d) Nucleic acids (d) A is false, but R is true
EBD_7367
36
C- Biology

63. Match the following [2009 - II] Codes


A B C D
Lis t I Lis t II (a) 2 1 3 4
Ag ent of Trans mis s ion Dis eas e Trans mitted (b) 2 3 1 4
A . A n op heles mo s qu ito 1. Kala-azar (c) 4 3 1 2
(d) 4 1 3 2
B. Culex mos q uito 2. Den gu e 74. Which one of the following statement regarding potato is
C. A edes 3. M alaria correct? [2009 - II]
(a) It is a root
D. San dfly 4. Filaria
(b) It is a normal stem
Codes (c) It is a modified stem
A B C D (d) It is a modified root
(a) 3 2 4 1 75. Golden fibre refers to [2009 - II]
(b) 1 4 2 3 (a) hemp (b) cotton
(c) 1 2 4 3 (c) jute (d) nylon
(d) 3 4 2 1 76. Which one of the following is commonly used as a
64. Primary source of vitamin-D for human beings is flavouring agent during the preparation of noodles?
[2009-II] (a) Saffron (b) Cinnamon [2009 - II]
(a) citrus fruits (b) green vegetables (c) Olive oil (d) Ajinomoto
(c) yeast (d) sun 77. Which one of the following plants is popularly grown along
65. Anaemia is a common health problem especially in women. the road for absorbing vehicular pollutants?
Which one of the following deficiencies is most frequently (a) Nerium (b) Neem [2010 - I]
responsible for anaemia in India? (c) Bougainvillea (d) Calotropis
(a) Calcium (b) Iron [2009 - II]
78. Match the following [2010 - I]
(c) Iodine (d) Zinc
66. Which one of the following is considered normal blood List I List II
pressure in man? [2009 - II]
(a) 120/80 mm water (b) 120/80 mm blood A. Wine 1. Barley
(c) 120/80 mm mercury (d) 120/80 mm air B. Beer 2. Sugarcane juice
67. Anthrax is a disease of human and cattle with a potential for C Whisky 3. Grapes
\
biological warfare. It is caused by [2009 - II]
(a) bacterium (b) virus D. Rum 4. Molasses
(c) protozoan (d) fungus Codes
68. Which one of the following animals breathe through the A B C D
skin? [2009 - II] (a) 2 1 4 3
(a) Fish (b) Pigeon (b) 3 4 1 2
(c) Frog (d) Cockroach (c) 3 1 4 2
69. For which one among the following diseases vaccine is not
(d) 2 4 1 3
yet available? [2009 - II]
(a) Tetanus (b) Malaria 79. A milkman puts banana leaf in milk jar, because banana leaf
(c) Measles (d) Mumps [2010 - I]
70. Which one of the following is not a feature of eutrophic (a) gives a fresh flavour to milk
lakes? [2009 - II] (b) makes the milk acidic and resistant to yeast
(a) Blooms are frequent in eutrophic lakes (c) makes the milk basic and resistant to yeast
(b) Plant nutrient flux is high (d) increases the whiteness of milk
(c) Primary productivity is low 80. Which one of the following is not biodegradable?
(d) Dominated by blue green algae (a) Woollen mat [2010 - I]
71. Quinine is a drug used in the treatment of malaria. From (b) Silver foil
which part of the plant it is obtained? [2009 - II] (c) Leather bag
(a) Roots (b) Stem (d) Jute basket
(c) Bark (d) Leaves 81. Tips of leaves in grasses and common garden plants show
72. Which among the following oils has the maximum protein water drops in early morning hours. This water accumulation
content? [2009 - II] is obtained from [2010 - I]
(a) Castor oil (b) Sunflower oil (a) atmosphere (b) stomata
(c) Soybean oil (d) Safflower oil (c) vascular bundles (d) hydathodes
73. Match the following [2009 - II] 82. If excess fertilizer is applied to a plant without water, the
List I List II plant will [2010 - I]
(a) be stunted in growth
(Mineral) (Major Source) (b) develop modifications
A. Iron 1. Banana, date (c) die due to plasmolysis
(d) remain unaffected
B. Potassium 2. Palak 83. Itching due to insect bite is caused by [2010 - I]
C. Iodine 3. Iodized common salt (a) formic acid (b) acetic acid
D. Calcium 4. Milk, egg (c) lactic acid (d) maleic acid
Biology C-37
84. Which one among the following is a major source of sugar? 2. Cold-blooded animals require much less energy to
[2010 - I] survive than warm-blooded animals.
(a) Watermelon (b) Beetroot Which of the statements given above is/are correct?
(c) Sugarcane (d) Date (a) Only 1 (b) Only 2
85. The vitamin(s), which is/are generally excreted in urine, is/ (c) Both 1 and 2 (d) Neither 1 nor 2
are [2010 - I] 95. A deficiency of which one of the following minerals is most
(a) viamin-A (b) vitamin-B likely to lead to an immunodeficiency? [2010 - II]
(c) viamin-E (d) vitamin-D and K (a) Calcium (b) Zinc
86. Which one among the following plants cannot be multiplied (c) Lead (d) Copper
by cuttings? [2010 - I] 96. What is the most conspicuous salient feature of people
(a) Rose (b) Bryophyllum with 'Progeria'? [2010 - II]
(c) Banana (d) Marigold (a) More hair on body
87. Consider the following statements [2010 - I]
(b) Less immunity to opportunistic infections
1. A person with myopia can see distant objects distinctly
but cannot see nearby objects clearly. (c) Faster rate of ageing
2. A person with hypermetropia cannot see distant (d) Surfer from infertility
objects clearly. 97. Human body's main organ of balance is located in
3. A person with presbyopia can see nearby objects [2010 - II]
without corrective glasses. (a) inner part of ear
Which of the statements given above is/are not correct? (b) middle part of ear
(a) 1, 2 and 3 (b) 1 and 2 (c) front part of brain
(c) 1 and 3 (d) Only 3 (d) top part of vertebral column
88. Consider the following statements [2010 - I] 98. Which of the following statements correctly describe the
1. Iodine is necessary for the thyroid gland to make properties of hormones? [2010 - II]
adrenaline. 1. They are steroids, proteins, peptides or amino acids
2. Iodine deficiency leads to goitre in human beings. derivatives.
3. Iodine is secreted by pancreas and helps in regulating 2. They are not produced by body organs and are mostly
cholesterol level. taken as supplements.
Which of the statements given above is/are correct? 3. They do not influence the working of those organs
(a) 1, 2 and 3 (b) 1 and 2 which have secreted them.
(c) 1 and 3 (d) Only 2 4. They act as co-enzymes and help enzymes to perform
89. Which among the following are the major reasons behind their function.
preferring eucalyptus tree in the planned forestation Select the correct answer using the code given below
process? [2010 - I] (a) 1 and 4 (b) 2, 3 and 4
1. Plantation grows very fast.
(c) 1 and 3 (d) 1. 2, 3 and 4
2. Plantation makes the soil more fertile.
3. Wood from eucalyptus tree is easily converted into 99. Which among the following statement about biofertilizers
the pulp for paper industry. are correct? [2010 - II]
Select the correct answer using the codes given below. 1. Azotobacter is one of the nitrogen fixing bacteria used
(a) 1 and 2 (b) 1 and 3 as a biofertilizer.
(c) 2 and 3 (d) All of these 2. They have to be applied to the leaves of the plant
90. Which one of the following plants is used for green only.
manuring in India? [2010 - I] 3. They alter the chemical composition of the soil.
(a) Wheat (b) Sunnhemp 4. They can be used along with organic fertilizers.
(c) Cotton (d) Rice Select the correct answer using the code given below
91. Chemically silk fibres are predominantly [2010 - II] (a) 1 and 4 (b) 1, 2 and 4
(a) Protein (c) 2 and 3 (d) 1, 2, 3 and 4
(b) Carbohydrate 100. Why are pregnant women recommended substantial intake
(c) Complex lipid of green leafy vegetables in their diet, especially in the 1st
(d) Mixture of polysaccharide and fat trimester? [2010 - II]
92. Which among the following are the most important raw (a) They are a rich source of chlorophyll
materials for the manufacturing of soap? [2010 - II] (b) They are a rich source of lecithin
(a) Fats and caustic alkali (c) They are a rich source of folic acid which is required
(b) Fats and potash for DNA synthesis
(c) Fats and acid (d) They are a rich source of essential fatty acids required
(d) Vegetable oil and potash for cell anabolism
93. Mosquito can be a vector for the following disease except 101. Cattle are capable of digesting cellulose present in the grass
[2010- II]
and/or fodder that they eat. This ability is attributed to the
(a) yellow fever (b) dengue fever
(c) filariasis (d) kala-azar [2010- II]
94. Consider the following statements [2010 - II] (a) presence of cellulose degrading bacteria in the rumen
1. Warm-blooded animals can remain active in cold (b) production of cellulose by the cattle rumen
environment in which cold-blooded animals can hardly (c) acids present in the rumen
move. (d) prolonged retention of cellulose in the rumen
EBD_7367
38
C- Biology

102. Which one among the following animal tissues transport 111. In dry regions, the leaf size of a tree becomes smaller. It is so
hormones and heat and maintains water balance? to [2011 - I]
[2010 - II] (a) reduce metabolism
(a) Connective tissue (b) Muscular tissue (b) reduce transpiration
(c) Blood (d) Nervous tissue (c) maintain natural growth
103. Which among the following is the correct increasing order (d) protect plant from animals
of pH found in human body? [2010 - II] 112. Bats can fly in dark because they [2011 - I]
(a) Gastric juice, saliva, blood (a) have strong wings
(b) Blood, saliva, gastric juice (b) have sharp eyes
(c) Saliva, blood, gastric juice (c) produce ultrasonic waves
(d) Gastric juice, blood, saliva (d) are nocturnal
113. White blood cells act [2011 - I]
104. Match the following [2010 - II] (a) as a defence against infection
Lis t I Lis t II (b) as source of energy
(c) as a clotting agent
(S ug ar) (S ource) (d) as a medium for oxygen transport from lungs to tissues
A. Cellu los e 1 Ho ney 114. Rain water collected after 30 to 40 minutes of raining is not
B. Pituitary 2 Sug arcane suitable for drinking because it [2011 - I]
(a) contains bacteria and dirt
C. M alto s e 3 Cotton wo ol (b) contains dissolved toxic chemicals
D. Sucros e 4 Starch (c) is deficient in minerals
(d) is acidic
Codes 115. Which one among the following elements/ions is essential
A B C D in small quantities for the development of healthy teeth but
(a) 3 1 4 2 causes mottling of the teeth if consumed in higher
(b) 3 4 1 2 quantities? [2011 - I]
(c) 2 1 4 3 (a) Iron (b) Chloride
(d) 2 4 1 3 (c) Fluoride (d) Potassium
105. Due to contraction of eyeball, a long-sighted eye can see 116. Which one among the following is not correct about Down's
only [2011 - I] syndrome? [2011 - I]
(a) farther objects which is corrected by using convex (a) It is a genetic disorder
lens (b) Effected individual has early ageing
(b) farther objects which is corrected by using concave (c) Effected person has mental retardation
lens (d) Effected person has furrowed tongue with open mouth
(c) nearer objects which is corrected by using convex lens 117. Dog bite can cause rabies. Which among the following other
(d) nearer objects which is corrected by using concave animals can also cause rabies? [2011 - I]
lens (a) Donkey (b) Bat
106. Dead organs are generally stored in formalin. (c) Horse (d) Crocodile
Formalin is beacuse it acts as a disinfectant which is due to 118. Which one among the following group of items contain
the presence of formaldehyde [2011 - I] only biodegradable items? [2011 - I]
(a) aqueous ferrous sulphate
(b) aqueous formaldehyde (a) Wood, Grass, Plastic
(c) aqueous formic acid (b) Wood, Grass, Leather
(d) aqueous feme alum (c) Fruit peels, Lime juice, China clay cup
107. Insects that can transmit diseases to human are referred to (d) Lime juice, Grass, Polystyrene cup
as [2011 - I] 119. When we eat something we like, our mouth waters. This is
(a) carriers (b) reservoirs actually not water but fluid secreted from [2011 - I]
(c) vectors (d) incubators (a) nasal glands (b) oval epithelium
108. Photochemical smog occurs in [2011 - I] (c) salivary glands (d) tongue
(a) cool and humid climate 120. Why do you feel cool under a tree but not so under a tin
(b) warm, dry and sunny climate shed on a sunny day? [2011 - I]
(c) cool, dry and sunny climater (a) The greenness of the tree gives the cool feeling
(d) warm and humid climate (b) Photosynthesis absorbs heat
109. Which of the following diseases are transmitted from one (c) The leaves convert water vapours into water which is
person to another? [2011 - I] a heat-absorbing process
1. AIDS 2. Cirrhosis (d) The leaves give out water which vapourises absorbing
3. Hepatitis B 4. Syphilis some heat as latent heat
Select the correct answer using the code given below 121. Polar bears are carnivores and prey on many arctic birds
(a) 2, 3 and 4 (b) 1, 3 and 4
and fishes. However, under natural conditions, no one
(c) 1 and 2 (d) All of these
found polar bears predating any penguin. This is because
110. Which one among the following industries produces the
most non-biodegradable wastes? [2011 - I] [2011- II]
(a) Thermal power plants (a) penguins have chemical substance in their muscles
(b) Food processing units which is toxic to polar bears
(c) Textile mills (b) penguins are gregarious and always move in groups.
(d) Paper mills Therefore, a polar bear cannot approach them
Biology C-39
(c) polar bears and penguins never coexist under natural 133. Which one among the following vitamins is necessary for
conditions. The former lives in the North Pole while blood clotting? [2013 - II]
the latter lives in the South Pole (a) Vitamin A (b) Vitamin D
(d) polar bears and penguins display symbiotic (c) Vitamin K (d) Vitamin C
relationships and they help each other for their 134. The pH of human blood is normally around [2013 - II]
existence in the ice-cold ecosystem (a) 4.5-5.5 (b) 5.5-6.5
122. To suspect HIV/AIDS in a young individual, which one (c) 7.5-8.0 (d) 8.5-9.0
among the following symptoms is mostly associated with? 135. The crew and passengers of a flying aircraft suffer generally
[2011 - II] from chronic obstructive pulmonary disease due to the effect
(a) Long standing jaundice and chronic liver disease of [2013- II]
(b) Severe anaemia (a) solar radiation (b) ozone concentration
(c) Chronic diarrhoea (c) nitrogen oxide (d) particulate pollutant
(d) Severe persistent headache 136. Consider the following statements regarding antibiotics
[2013 - II]
123. Development of goitre (enlarged thyroid gland) is mainly 1. They are used to destroy disease-causing bacteria.
due to deficiency of [2011 - II] 2. They can be applied to the skin, swallowed or injected
(a) sodium (b) iodine to fight micro-organisms inside the body.
(c) calcium (d) iron 3. They are effective against disease-causing viruses.
124. Endosulfan, which has been in news these days, is 4. The first antibiotic to be discovered was Tetracycline.
[2011 - II] Which of the statement(s) given above is/are correct?
(a) a pesticide (b) a fertiliser (a) 1 and 2 (b) 1, 2 and 4
(c) a sulpha drug (d) an antibiotic (c) 2, 3 and 4 (d) Only 1
125. The anti-malarial drug quinine is made from a plant. The 137. The gastrointestinal hormones namely secretin and
plant is [2011 - II] cholecystokinin secreted by duodenal epithelium activate
(a) neem (b) eucalyptus respectively which organs to discharge their secretions?
(c) cinnamon (d) cinchona [2013 - II]
126. Cutting and peeling of onions brings tears to the eyes (a) Pancreas and gall bladder
because of the presence of [2011 - II] (b) Gall bladder and stomach
(a) sulphur in the cell (b) carbon in the cell (c) Pancreas and stomach
(d) Stomach and small intestine
(c) fat in the cell (d) amino acid in the cell 138. Which one among the following statements about blood
127. If we sprinkle common salt on an Earthworm, it dies due to transfusion is correct? [2013 - II]
[2011 - II] Blood group B can give blood to
(a) osmotic shock (b) respiratory failure (a) blood Group B and receive from Group AB
(c) toxic effect of salt (d) closure of pores of skin (b) blood Groups B and AB and receive from Group B
128. It has been observed that astronauts lose substantial (c) blood Groups B and AB and receive from Group A
quantity of calcium through urine during space flight. This (d) blood Group O and receive from Group B
is due to [2011 - II] 139. What would happen if human blood becomes acidic (low pH)?
(a) hyper gravity [2013 - II]
(b) microgravtty (a) Oxygen carrying capacity of haemoglobin is increased
(c) intake of dehydrated food tablet (b) Oxygen carrying capacity of haemoglobin is decreased
(d) low temperature ip cosmos (c) RBC count increases
129. Which one among the following will be absorbed fastest (d) RBC count decreases
140. Cancer is more common in older people because
through the wall of digestive system? [2011 - II] [2013 - II]
(a) Black coffee as a hot beverage (a) their immune systems have degenerated
(b) DDT taken as a poison (b) the supply of certain hormones declines with age
(c) Raw alcohol taken as a drink (c) their bodies are unable to adjust to the changing
(d) Ice-cream as a dessert environment
130. The ultimate cause of water movement in a plant stem (d) they have accumulated more mutations
against gravity is [2013 - II] 141. Human blood is a viscous fluid. This viscosity is due to
(a) osmosis (b) transpiration [2013- II]
(c) photosynthesis (d) diffusion (a) proteins in blood (b) platelets in plasma
131. Which one among the following statements is correct? (c) sodium in serum (d) RBC and WBC in blood
[2013 - II] 142. In humans, which one among the following with reference
(a) All proteins are enzymes to breathing is correct? [2013 - II]
(a) During inhalation, diaphragm relaxes
(b) All enzymes are proteins (b) During exhalation, thorax cavity expands
(c) None of the enzymes is protein (c) During inhalation, intra-pleural pressure becomes more
(d) None of the proteins is enzyme negative
132. The fossil of Archaeopteryx represents the evidence of (d) Unlike inhalation, normal exhalation is an active
origin of [2013- II] process
(a) birds from reptiles 143. Which one among the following gases readily combines
(b) mammals from reptiles with the haemoglobin of the blood? [2013 - II]
(c) reptiles from amphibians (a) Methane (b) Nitrogen dioxide
(d) mammals from birds (c) Carbon monoxide (d) Sulphur dioxide
EBD_7367
40
C- Biology

144. Blood does not coagulate inside the body due to the 1. If the roots of the tree develop a disease, then voles
presence of [2013 - II] and foxes will starve and not the owls.
(a) haemoglobin (b) heparin 2. Population of voles is dependent on wood lice
(c) fibrin (d) plasma population through food web.
145. Vaseline was applied to both surfaces of the leaves of a 3. If owl population declines, it will indirectly affect wood
plant. Which of the following process/ processes would be lice population.
affected? [2013 - II] 4. If barks of tree are affected due to a disease, then
1. Photosynthesis 2. Respiration reduction of wood lice will affect the shrew population
3. Transpiration forcing owls to eat more voles.
Select the correct answer using the codes given below Which of the above predictions is/are correct?
(a) 1 and 3 (b) Only 2 (a) 1,2, and 4 (b) Only 3
(c) 2 and 3 (d) All of these (c) 1 and 2 (d) Only 2
146. A person feeds on rice and vegetable made up of potato 154. Deficiency of which of the following elements is responsible
only. He is likely to suffer from deficiency of [2013 - II] for weakening of bones? [2014 - I]
(a) carbohydrate and vitamins 1. Calcium 2. Phosphorus
(b) proteins 3. Nitrogen 4. Carbon
(c) carbohydrate and proteins Select the correct answer using the codes given below
(a) 1 and 2 (b) Only 1
(d) proteins and fats
(c) 1, 2 and 3 (d) Only 4
147. Which one among the following groups of animals
155. Which of the following statements is/are correct?
maintains constant body temperature in changing 1. Amnion contains fluid. [2014 - I]
environmental conditions? [2013 - II] 2. Ultrasound scan can detect the sex of an embryo.
(a) Birds (b) Fishes Select the correct answer using the codes given below
(c) Amphibians (d) Reptiles (a) Only 1 (b) Only 2
148. Which organelle in the cell, other than nucleus, contains (c) Both 1 and 2 (d) Neither 1 nor 2
DNA? [2013 - II] 156. Among the following animals, choose the one having three
(a) Endoplasmic reticulum pairs of legs. [2014 - I]
(b) Golgi apparatus (a) Spider (b) Scorpion
(c) Lysosome (c) Bug (d) Mite
(d) Mitochondria 157. Which of the following statements is/are correct regarding
149. Which one among the following groups of items contains fats? [2014 - I]
only biodegradable items? [2013 - II] 1. Fats are needed for the formation of cell membrane.
(a) Paper, grass, glass 2. Fats help the body to absorb calcium from food.
(b) Wood, flower, iron-scrap
3. Fats are required to repair damaged tissue.
(c) Sewage, plastic, leather
4. Body cannot release energy in fats as quickly as the
(d) Cow-dung, paddy-husk, vegetable wastes
energy in carbohydrates.
150. Two strands of DNA are held together by [2013 - II]
Select the correct answer using the codes given below
(a) hydrogen bonds (b) covalent bonds
(a) 1 and 4 (b) Only 1
(c) electrostatic force (d) Van der Waals' forces
(c) 2 and 4 (d) 3 and 4
151. Which one of the following elements is present in green
pigment of leaf? [2014 - I] 158. People suffering from 'anorexia nervosa' [2014 - I]
(a) Magnesium (b) Phosphorus (a) develop paralysis
(c) Iron (d) Calcium (b) show poor reflex
152. Match the following [2014 - I] (c) cannot speak properly
(d) eat very little and fear gaining weight
List I List II 159. Which of the following structures of a plant is responsible
(Gland) (Hormone) for transpiration? [2014 - I]
(a) Xylem (b) Root
A. Pancreas 1 Cortisol
(c) Stomata (d) Bark
B. Pituitary 2 Vitamin D 160. Which of the following does not possess a specialised
C. Adrenal 3 Thyroid stimulating hormone conducting tissue for transport of water and other
substances in plants? [2014 - I]
D. Kidneys 4 Glucagon (a) Marchantia (b) Marsilea
Codes (c) Cycas (d) Fern
A B C D 161. Which one among the following is the generic name of the
(a) 4 3 1 2 causal organism of Elephantiasis ? [2014-II]
(b) 4 1 3 2 (a) Filaria (b) Microfilaria
(c) 2 1 3 4 (c) Wuchereria bancrofti (d) Culex pipiens
(d) 2 3 1 4 162. Which one among the following statements is not correct ?
153. In a forest, animals like voles and woodlice feed on plant [2014-II]
roots and barks respectively. Among the other carnivores, (a) Pulses are rich in proteins
foxes, shrews and owls are present in that forest. Following (b) Milk is a rich source of Vitamin A
predictions are made by a group of observers who have (c) Cereals are very poor source of carbohydrates
visited the forest. [2014 - I] (d) Vegetables are rich source of minerals
Biology C-41
163. Which of the statements given below are correct? 170. Which one among the following cattle breed produces
[2014-II] highest amount of milk ? [2014-II]
1. A person having blood group 'A' can donate blood to (a) Brown Swiss (b) Holstein
persons having blood group 'A' and blood group 'AB'. (c) Dutch belted (d) Blaarkop
2. A person having blood group 'AB' can donate blood 171. Which one among the following statements is correct ?
to persons having blood groups 'A', 'B', 'AB' or 'O'. [2015-I]
3. A person with blood group 'O' can donate blood to (a) Prokaryotic cells possess nucleus.
persons having any blood group. (b) Cell membrane is present both in plant and animal cells.
4. A person with blood group 'O' can receive blood from (c) Mitochondria and chloroplasts are not found in
the person of any of the blood groups. eukaryotic cells.
Select the correct answer using the code given below : (d) Ribosomes are present in eukaryotic cells only.
(a) 1, 2, 3 and 4 172. Which one among the following statements is not true for
(b) l and 2 only Mammals ? [2015-I]
(c) 3 and 4 only (a) They possess hair on the body.
(d) 1 and 3 only (b) Some of them lay eggs.
164. Which one among the following is the correct pathway for (c) Their heart is three chambered
the elimination of urine ? [2014-II] (d) Some are aquatic.
(a) Kidneys, Ureters, Bladder, Urethra 173. In human digestive system, the process of digestion starts in
(b) Kidneys, Urethra, Bladder, Ureters [2015-I]
(c) Urethra, Ureters, Bladder, Kidneys (a) Oesophagus (b) Buccal cavity
(d) Bladder, Ureters, Kidneys, Urethra (c) Duodenum (d) Stomach
165. Which of the following parts are found in both plant and 174. Which one of the following diseases in humans can spread
animal cells ? [2014-II] through air ? [2015-I]
(a) Cell membrane, Chloroplast, Vacuole (a) Dengue (b) Tuberculosis
(b) Cell wall, Nucleus, Vacuole (c) HIV-AIDS (d) Goitre
(c) Cell membrane, Cytoplasm, Nucleus 175. Which one among the following structures or cells is not
(d) Cell wall, Chloroplast, Cytoplasm present in connective tissues ? [2015-I]
166. Which of the following statements is/are correct ? (a) Chondrocytes (b) Axon
[2014-II] (c) Collagen fibre (d) Lymphocytes
1. Coronary artery supplies blood to heart muscles. 176. Genetic screening is [2015-II]
2. Pulmonary vein supplies blood to lungs. (a) analysis of DNA to check the presence of a particular
3. Hepatic artery supplies blood to kidneys. gene in a person
4. Renal vein supplies blood to kidneys. (b) analysis of gene in a population
Select the correct answer using the code given below: (c) pedigree analysis
(a) l, 2 and 3 (b) 1 and 2 only (d) screening of infertility in parents
(c) 2 and 4 (d) 1, 3 and 4 177. Who among the following is considered as the father of
167. Which one of the following types of pesticides is genetic engineering? [2015-II]
convenient to control stored grain pests ? [2014-II] (a) Philip Drinker (b) Paul Berg
(a) Systemic pesticides (b) Fumigants (c) Thomas Addison (d) Alpheuss Packard Jr.
(c) Contact poisons (d) Stomach poisons 178. Which of the following cause(s) variation in the genetic
168. Match List I with List II and select the correct answer using material of progeny? [2015-II]
the code given below the Lists: [2014-II] 1. Sexual reproduction 2. Asexual reproduction
List I List II 3. Mutations 4. Epigenetic changes
(Animal) (Phyla) Select the correct answer using the code given below.
A. Ascaris 1. Mammalia (a) 2 only (b) 1, 2 and 3
B. Malarial parasite 2. Arthropoda (c) 1, 3 and 4 (d) 1 and 3 only
C. Housefly 3. Nematoda 179. Cobalt is associated with [2015-II]
D. Cow 4. Protozoa (a) growth hormone (b) vitamin B12
Code : (c) hemoglobin (d) intestinal enzymes
A B C D 180. Absorption of water in the human body can be found in
(a) 3 4 2 1 [2015-II]
(b) 3 2 4 1 1. renal tubule in kidney 2. hepatic cells in liver
(c) 1 2 4 3 3. large intestine 4. pancreatic duct
(d) 1 4 2 3 Select the correct answer using the code given below.
169. Which one among the following pairs is not correctly (a) 1, 2 and 3 (b) 1 and 3 only
matched ? [2014-II] (c) 2 and 4 (d) 3 only
(a) Sandal wood plant : Partial root parasite
181. Measles is a disease caused by [2015-II]
(b) Cuscuta : Parasite
(c) Nepenthes : Carnivorous (a) bacteria (b) virus
(d) Mushrooms : Autotroph (c) protozoa (d) worm
EBD_7367
42
C- Biology

182. Neutrophils and lymphocytes originate from [2015-II] 193. Which one of the following hormones contains peptide
(a) kidney tubule (b) spleen chain ? [2016-I]
(c) bone marrow (d) lymph node (a) Oxytocin (b) Corticotropin
183. The mandate of the scheme entitled 'Directly Observed (c) Insulin (d) Cortisone
Treatment, Short-Course (DOTS)' launched by WHO is to 194. Which one of the following is considered as 'good
ensure that [2015-II] cholesterol' with reference to individuals facing the risk of
(a) doctors treat patients with medicine for a short duration cardio-vascular diseases and hypertension? [2016-I]
(b) doctors do not start treatment without a trial
(a) High Density Lipoprotein (HDL)
(c) patients complete their course of drug
(d) patients voluntarily take vaccines (b) Low Density Lipoprotein(LDL)
184. Penicillin inhibits synthesis of bacterial [2015-II] (c) Triglyceride
(a) cell wall (b) protein (d) Fatty acids
(c) RNA (d) DNA 195. Which of the following pairs of vitamin and disease is / are
185. Most antibiotics target bacterial parasites interfering with
correctly matched ? [2016-I]
various factors of growth of metabolism such as
1. synthesis of cell wall [2015-II] 1. Vitamin A A Rickets
2. bacterial protein Synthesis 2. Vitamin B1 B Beribri
3. synthesis of nuclear membrane 3. Vitamin C C Scurvy
4. mitochondrial function Select the correct answer using the code given below:
Select the correct answer using the code given below
(a) 2 only (b) 2 and 3 only
(a) 1, 2 and 3 (b) 1 and 4
(c) 2 and 3 only (d) 1 and 2 only (c) 1 and 3 only (d) 1, 2 and 3
186. Plants contain a variety of sterols like stigmasterol, 196. The Germplasm is required for the propagation of plants
ergosterol, sitosterol etc. which very closely resemble and animals, Germplasm is the : [2016-I]
cholesterol. These plant sterols are referred as: [2016-I] 1. genetic resources
(a) Phytosterols (b) Caleiferols 2. seeds or tissues for breeding
(c) Ergocaleiferols (d) Lumisterols 3. egg and sperm repository
187. In artificial insemination (AI) process. Which of the 4. a germ cell's determining zone
following is/are introduced into the uterus of the female ? Select the correct answer using the code given below
[2016-I] (a) 1 only (b) 1, 2 and 3
(a) Egg only (b) Fertilized egg (c) 2 and 3 only (d) 2 and 4
(c) Sperm only (d) Egg and sperm 197. Dengue virus is known to cause low platelet count in blood
188. Genetically modified (GM) crops contain modified genetic of patient by : [2016-I]
material due to : [2016-I] 1. interfering in the process of platelet production in bone
1. introduction of new DNA marrow
2. removal of existing DNA 2. infecting endothelial cells
3. introduction of RNA 3. binding with platelets
4. introduction of new traits 4. accumulating platelets in intestine
Select the correct answer using the code given below :
Select the correct answer using the code given below:
(a) 1 and 2 only (b) 1, 2 and 3
(a) 1 and 2 only (b) 1 and 3 only
(c) 3 and 4 (d) 1,2 and 4
(c) 3 and 4 (d) 1, 2 and 3
189. Which one of the following vitamins has a role in blood
clotting ? [2016-I] 198. Which one of the following is most sensitive to
(a) Vitamin A (b) Vitamin B environmental change? [2016-II]
(c) Vitamin D (d) Vitamin K (a) Amphibian (b) Reptile
190. Vitamin B12 deficiency causes pernicious anemia, Animals (c) Mammal (d) Insect
cannot synthesize vitamin B12. Humans must obtain all their 199. The HIV virus weakens the immunity of a person because it
vitamin B12 from their diet. The complexing metal ion in destroys [2016-II]
vitamin B12 is : [2016-I] (a) mast cells (b) platelets
(a) Mg2+ (Magnesium ion)(b) Fe2+ (Iron ion) (c) erythrocytes (d) lymphocytes
(c) Co3+(Cobalt ion) (d) Zn 2+(Zine ion) 200. Which one of the following air pollutants combines with
191. Glucose is a source of energy. Which one of the following the haemoglobin of human blood and reduces its oxygen-
types of molecule is glucose ? [2016-I] carrying capacity, leading to suffocation and may cause
(a) Carbohydrate (b) Protein even death? [2016-II]
(c) Fat (d) Nucleic acid (a) Chlorofluorocarbon (b) Fly ash
192. The living content of cell is called protoplasm. It is (c) Carbon monoxide (d) Sulphur dioxide
composed of : [2016-I] 201. An irregular mode of reproduction resulting in the
(a) Cytoplasm only development of an embryo without fertilization is called
(b) Cytoplasm and nucleoplasm 1. Parthenogenesis [2016-II]
(c) Nucleoplasm only 2. Apogamy
(d) Cytoplasm, nucleoplasm and other organelles 3. Sporophytic budding
Biology C-43

Select the correct answer using the code given below. (b) controlled cell division and over- production of genetic
(a) 1 only (b) 2 only material occur
(c) 2 and 3 only (d) 1, 2 and 3 (c) RNA is mutated and produced in less amount
202. Deficiency of fluoride leads to which one of the following (d) DNA is mutated and produced in less amount
health problems? [2016-II] 212. Most viruses that infect plants possess [2017-I]
(a) Tooth caries (b) Mottling of tooth (a) single-stranded DNA
(c) Bending of bones (d) Stiffening of joints (b) single-stranded RNA
203. Which one of the following statements is not correct?[2016-II] (c) double-stranded DNA and RNA
(a) The response of the ear to sound of increasing (d) double-stranded RNA only
intensity is approximately logarithmic. 213. Syngamy results in formation of [2017-I]
(b) The sensitivity of the human ear does not vary with (a) haploid zygote (b) diploid zygote
the frequency of the sound. (c) non-motile male gametes (d) motile male gametes
(c) When two or more waves traverse the same medium, 214. Bleeding of gums, falling of teeth, fragile bones and delayed
the displacement of any element of the medium is the wound healing occur due to the deficiency of which one of
algebraic sum of the displacements due to each wave. the following vitamins? [2017-I]
(d) Longitudinal waves can travel in all media-solids, (a) Vitamin C (b) Vitamin K
liquids and gases. (c) Vitamin D (d) Vitamin B
204. Which one of the following hormones is essential for the 215. The mammalian heart is myogenic and it is regulated by
uptake of glucose by cells in the human body? [2016-II] nerves. The heartbeat originates from [2017-I]
(a) GH (b) TSH (a) sinoatrial node (b) QRS wave
(c) Insulin (d) Cortisol (c) T wave (d) hepatic portal system
205. Which of the following cells do not contain nucleus? 216. The plant growth regulators are small, simple molecules of
[2016-II] diverse chemical composition. They are [2017-I]
1. Root hair cells 2. Red blood cells (a) carbohydrates, fats and proteins
3. Platelets 4. Monocytes (b) indole compounds, adenine derivatives, carotenoids
Select the correct answer using the code given below. and terpenes
(a) 1 only (b) 2 and 3 only (c) fatty acids, glucose and vitamins
(c) l, 2 and 4 (d) 2, 3 and 4 (d) vitamin C, vitamin D and glucose
206. Biological catalysts are protein molecules which 217. Which one of the following proteins gives lustrous shiny
1. speed up a chemical reaction [2016-II] appearance to silk fibre? [2017-II]
2. remain unchanged after reaction (a) Fibrin (b) Sericin
3. function optimally at 37°C (c) Collagen (d) Nectin
4. do not have an enzymatic activity 218. Blue Baby Syndrome is caused by the contamination of
Select the correct answer using the code given below. [2017-II]
(a) 1 only
(c) 2 and 3 only
(b) 1, 2 and 3
(d) 1,3 and 4 ( )
-
(a) nitrite NO 2 ( )
2-
(b) sulphite SO3
207. 'Altitude sickness' is caused at high altitude due to
nitrate ( NO ) sulphate ( SO )
- 2-
(a) high partial pressure of oxygen [2016-II] (c) 3 (d) 4
(b) low partial pressure of oxygen
(c) low level of haemoglobin 219. Match List-I with List-II and select the correct answer using
(d) high partial pressure of carbon dioxide the code given below the Lists : [2017-II]
208. The elemental composition of an adult human body by mass List-I List-II
is [2016-II] (Cell Organelle) (Function)
(a) C > O > H > N (b) O > C > H > N A. Mitochondria 1. Photosynthesis
(c) N > C > H > O (d) N > C > O > H B. Chloroplast 2. Protein synthesis
209. Polynucleotide chain of DNA contains [2017-I] C. Ribosomes 3. Intracellular digestion
(a) a nitrogenous base, deoxyribose sugar and phosphate D. Lysosomes 4. ATP formation
group Codes :
(b) a nitrogenous base, ribose sugar and phosphate group A B C D
(c) deoxyribose sugar, ribose sugar and phosphate group (a) 3 1 2 4
(d) a nitrogenous base and phosphate group only (b) 3 2 1 4
210. The process of copying genetic information from one strand (c) 4 1 2 3
of DNA into RNA is termed as [2017-I] (d) 4 2 1 3
(a) translation (b) transcription 220. Human insulin molecule is composed of one a-chain having
(c) replication (d) mutation 21 amino acids and one b-chain having 30 amino acids.
211. In human body, the cell growth and differentiation are highly How many functional insulin genes occur in adult humans?
controlled and regulated, but in cancer cells [2017-I] [2017-II]
(a) there is breakdown of these regulatory mechanisms (a) One (b) Two
leading to formation of benign and malignant tumors (c) Three (d) Four
EBD_7367
44
C- Biology

221. A biological community in its environment such as a pond, 228. Statement I :


an ocean, a forest, even an aquarium is known as Bioaccumulation is a process of progressive accumulation
[2017-II] of heavy metals and pesticides in an organism.
(a) biome (b) community Statement II :
(c) abiotic environment (d) ecosystem
Large fishes of the pond are found to have higher
DIRECTION (Q. 222-): The following five (5) items consist of concentration of pesticides than planktons of the same
two statements, Statement I and Statement II. Examine these two pond.
statements carefully and select the correct answer using the 229. Which one of the following is a true fish as per the biological
code given below. system of classification? [2018-I]
Codes: [2017-II] (a) Silver fish (b) Jelly fish
(a) Both the statements are individually true and Statement II (c) Cuttle fish (d) Flying fish
is the correct explanation of Statement I 230. In which one of the following types of connective tissues
(b) Both the statements are individually true but Statement II is
in animals does fat get stored? [2018-I]
not the correct explanation of Statement I
(a) Adipocyte (b) Chondrocyte
(c) Statement I is true but Statement II is false
(d) Statement I is false but Statement II is true (c) Osteocyte (d) Reticulocyte
222. Statement I : 231. Which one of the following pairs about organ/part that helps
Skin cancer is generally caused by the ultraviolet radiation. in locomotion is not correctly matched? [2018-I]
Statement II : (a) Euglena : Flagellum
Stratosphere allows ultraviolet radiation to enter the earth (b) Paramecium : Cilia
from the sun. (c) Nereis : Pseudopodia
223. The foul smell of urine of a healthy man having healthy (d) Starfish : Tubefeet
food, when spilled on floor, is mainly due to the bacterial 232. Lysosome is formed from which of the following cell
decomposition of [2017-II]
organelles? [2018-I]
(a) urea into sulphur dioxide
(b) sugar into carbon dioxide (a) Nucleus (b) Endoplasmic reticulum
(c) lipids into methane (c) Golgi bodies (d) Ribosomes
(d) urea into ammonia 233. A protein is synthesized in the endoplasmic reticulum bound
224. Which one of the following polymers does not contain ribosomes and it targets to the inner thylakoid space of
glucose units? [2017-II] chloroplast. How many double-layered membrane layers it
(a) Glycogen (b) Starch has to pass to reach its destination? [2018-I]
(c) Cellulose (d) Rubber
(a) 2 (b) 3
225. Which one of the following elements is involved in the
control of water content of the blood? [2018-I] (c) 4 (d) 5
(a) Potassium (b) Lithium 234. Which one of the following statements about lymph is
(c) Rubidium (d) Caesium correct? [2018-I]
226. Which one of the following elements is essential for the (a) Lymph is formed due to leakage of blood through
formation of chlorophyll in green plants? [2018-I] capillaries.
(a) Calcium (b) Iron (b) Lymph contains blood cells such as RBC.
(c) Magnesium (d) Potassium (c) Lymph is also circulated by the blood circulating heart.
DIRECTIONS (QS. 227-228): The following items consists of (d) Lymph only transports hormones.
two statements, Statement I and Statement II. Examine these two 235. Which of the following classes of animals has/have three-
statements carefully and select the correct answer using the chambered heart? [2018-I]
code given below. (a) Pisces and Amphibia (b) Amphibia and Reptilia
227. Statement I : [2018-I] (c) Reptilia only (d) Amphibia only
A person may suffer from tuberculosis if she/he frequently 236. Accumulation of which one of the following in the muscles
visits crowded place. of spinters leads to cramp? [2018-I]
Statement II : (a) Lactic acid (b) Ethanol
Bacteria of tuberculosis spread through droplets by
(c) Pyruvic acid (d) Glucose
sneezing or coughing.
Biology C-45

HINTS & SOLUTIONS


1. (b) All enzymes are protein by composition but all proteins 23. (a) Rhizobium (a kind of bacteria) lives in plant roots of
are not enzymes. pulse plant (legumes) to fix nitrogen.
2. (c) Thyroid gland is one of the largest endocrine glands 24. (b) Asthma is a chronic lung disease. It inflames and
which is present in the neck. It is responsible to control narrows the air ways. Tuberculosis is a lung disease
body’s metabolism, temperature, appetite and muscle created by Mycobacterium tuberculosis. Workers in
strength, etc. textile factories and carpet manufacturing units are the
3. (d) Nitrogen dioxide (NO2) is not a component of biogas. common sufferess of asthma and tuberculosis.
4. (b) The blood group of father of the child could be A or B 25. (a) Deficiency of vitamin-A (retinol) creates night
or O. blindness. It also creates keratomalacia, xerosis cornea
5. (a) Sphygmomanometer is an instrument to measure the and Bitot’s spot. High content of vitamin-A is obtained
blood pressure. It is made up of an inflatable cuff to from liver and butter.
restrict the blood flow and a mercury or manometer to 26. (d) Potassium ions(K + ) are responsible for protein
measure pressure. synthesis, opening and closing of stomata in plants.
6. (a) Water gas is a synthetic gas, having CO and H2. The
27. (a) 28. (d)
gas is produced by passing steam over a red hot
hydrocarbon fuel as coke. 29. (d) Colour temperature is a measurement relating the colour
7. (d) Chikungunya is caused by chikungunya virus which of the light radiated by an object to its temperature. No
is an insect borne virus of genus Alphavirus. matter whatever the temperature is the hottest would
Symptoms show high fever, maculopapular rash, only radiate to blue-white light radiation.
headache, etc. 30. (b) Airbag used for safety and protection of car driver
8. (c) Vitamin-C is enough in guava fruits. Vitamin-C helps possesses sodium azide.
in strengthening of immunity, serves as an antioxidant 31. (d) The odour of garlic is due to allyl methyl sulphide.
and provides relief in hypertension too. 32. (b) Mushrooms are not able to manufacture their own
9. (d) Vitamin-K adds in blood clotting. Vitamin-K acts as an food. A mushroom is the fleshy, spores having fruiting
essential cofactor for factor-II, VII, IX, X and also for body of fungi. It is heterotrophic.
proteins Z, C and S. 33. (a) Egg white are good easy way to add protein to any
10. (b) Thyroid gland in human body contains iodine. food. Egg albumin is a class of simple, water soluble
Deficiency of iodine creates goitre disease. Which is proteins that are present in egg white.
observed by the enlargement of larynx. 34. (a) In the normal human adult, heart beats at the rate of
11. (b) Polio and pertusis are caused by virus. So, these can about 72-80 per minute in the resting state.
be controlled by vaccines. 35. (a) The term lub and dub is concernes with heart. Lubb is
12. (b) Red Data Book is related with a complete list of all the first sound in the heart produced by the closure of
endangered species. atrioventricular values (tricuspid and bicuspid) at the
13. (a) Bat is mammal not a bird. beginning of ventricular systole. Dub is the second
14. (a) sound produced by the closure of semilunar value
15. (b) White blood cells (WBC) bring out the function of towards the last point of ventricular systole.
body defence. 36. (b) In vertebrates, cervical vertebrae are immediately
16. (b) inferior to the skull. They are smallest of true vertebrae
17. (a) Beri-beri is not insect borne disease. It is caused by and their number in human body is 7.
the deficiency of vitamin-B1 or thiamine. 37. (c) Carbohydrates is an important nutrient in human. It
18. (b) Methane (CH4) is produced in rice fields in the most is a good and rich source of energy.
prominent quantities. Because when organic matter 38. (d) DNA fingerprinting includes the extraction and
decomposes without O2 in the water logged rice paddies, identification of the base pair pattern of an individual’s
bacteria in the water produce methane. DNA. The identification process of specific DNA
19. (c) Benzene (C6H6) is cause of bone marrow failure. sequences is carried out by Southern blotting.
20. (c) Somatotrophin is produced by the anterior pituitary. It 39. (d) The human heart possesses four chambers, two
is a peptide hormone that induces growth, cell superior atria and two inferior ventricles. The atria are
reproduction and regeneration. getting chambers and ventricles are providing
21. (c) Cowper’s gland is related with reproductive system. chambers. Left ventricle pumps fully oxygenated blood
Cowper’s gland is the bulbourethal gland found in to aorta then to the body. Right atrium gets
human males. They are found in pair and secrete deoxygenated blood from superior vena cava.
viscous secretion called pre ejaculate that helps in 40. (b) Biogas is typically a gas evolved by the biological
coitus. breakdown of organic wastes in absence of O2. It is
22. (c) Cloves are the flower buds of a tree in the family used as biofuel and made up basically of CH4
Myrtaceae. The tree is evergreen growing, contains (50-75%), CO2 (25-50%) N2 (0 -10%), H2 (0-1%), H2s
large leaves and flowers grouped in terminal clusters. (0-3%) and O2 (0-1%).
EBD_7367
46
C- Biology

41. (b) Curd is made due to the chemical reaction between 59. (c) The stimulating nature of tea is due to alkaloid caffeine
lactic acid bacteria and casein. found in tea. Caffeine is bitter due to white crystalline
42. (c) Homeostatic regulation includes sweating. It is the xanthine alkaloid.
main process through which the body gets coolness. 60. (c) Both statements are true
43. (d) Malaria is a mosquito borne disease of humans and other 61. (a) Banyan (Ficus benghalensis) tree has aerial roots. From
animals caused by Plasmodium protozoan. Severe disease the tip of these roots new plant can evolve. In
is largely caused by Plasmodium falciparum whereas Hinduism, the leaf of the Banyan tree is said to be the
mild forms are due to P vivax, P oval and P malariae. place for Lord Krishna.
44. (c) Influenza is caused by virus and all other three 62. (b) 63. (d)
diseases are bacterial. Influenza, generally called flu, 64. (d) Sun is the primary source of vitamin-D. The body can
is an infectious disease caused by RNA viruses of synthesise vitamin-D itself in presence of sunlight.
family Orthomyxoviridae. Vitamin-D is soluble in fat and assists the body to
45. (c) O is the universal donor, but can obtain blood only absorb calcium. Its deficiency can cause osteoporosis.
from O so option (a) and (d) are not possible. 65. (b) Iron is the main element in haemoglobin. So its
Regarding (b), B can’t provide blood to A because of deficiency creates anaemia. Women show careless-
presence of antibodies against A antigen. So, (c) is ness about their nutrient requirements, so they face
the only condition of possible blood transfusion as the problem generally.
AB is the universal recipient. 66. (c) Systolic pressure in an adult man is 120 mm Hg
46. (d) The stomach shows division into two parts fundic whereas diastolic pressure is 80 mm Hg normally. High
and pyloric region. The pyloric region enters small or low blood pressure can create serious conditions in
intestine through pyloric valve of pylorus. humans.
47. (d) Vitamin-A-Retinol, Vitamin-B1 -Thiamine, Vitamin-C- 67. (a) Anthrax is an infectious disease occurring by bacteria
Ascorbic acid, Vitamin-E-Tocopherol. called Bacillus anthracis. Anthrax spores can be
evolved in vitro and used as a biological weapon.
48. (c) Chloramphenicol affects bacteria which are resistant
Anthrax can’t directly spread from one individual to
to all other medicine. Chloramphenicol is a prototypical
another but can cause infection through spores.
broad spectrum bacteria killing drug.
68. (c) Frog takes respiration in water through the moist skin.
49. (c) Cinchona bark creates quinine which is an anti-malarial Whereas on the Earth respiration occurs through lungs
drug. Poppy plant creates morphine it is an opium Fish has gills, Cockroach has spiracles and pigeon
alkaloid, which is a strong analgesic and also has possesses lungs to respire.
sedative and soothing effect. Fungus and bacterium 69. (b) Malaria is a protozoan infection caused by the genus
are the sources of antibiotics penicillin and tetracycline plasmodium in humans. No vaccine is yet available
respectively. for this illness.
50. (b) Green plants absorb red and blue colour maximally. 70. (c) Eutrophication is the movement of trophic status of
51. (b) The root nodules of gram plant contain symbiotic aquatic body (lake, pond) towards the direction of
bacteria which is responsible for fixing nitrogen. So, increasing biomass by adding natural or artificial
gram is preferred for mixed farming. substances. So, its primary productivity is clearly high.
52. (a) Henna (Lawsonia inermis) is a flowering plant. It is 71. (c) Quinine is a natural white crystalline alkaloid. It has
used to dye skin, hair, fingernails, leather and wood.
antipyretic (fever-reducing), antimalarial, analgesic,
Colouring feature of henna is due to lawsone, an
and anti-inflammatory qualities. It is present in the bark
organic compound that shows affinity for proteins and
of cinchona tree.
amino acids in animals.
72. (c) Soyabean oil & seed is a rich source of protein.
53. (c) Golden rice is rich in b -carotene and iron. It is a variety 73. (a) Palak (Spinach) has 5% non-haeme iron. One banana
of Oryza sativa evolved through process of bio possesses 422 mg potassium. One cup of dates
technology to synthesize b -carotene, a precursor of pro- possesses 464 mg potassium. One cup milk has 276
vitamin -A. mg calcium. One egg possesses 30 mg calcium. One
54. (b) Bryophytes do not have vascular bundles (xylem & kg packet of iodized common salt has 15 mg iodine.
phloem) and roots so they show similarity with algae. 74. (c) It is a modified stem to store food synthesised by
55. (a) Azotobacter is a genus of usually motile and free living plants.
bacteria that builds thick walled cysts. They are able 75. (c) Golden fibre denotes to jute because of its colour. Jute
to fix nitrogen in soil. is grown as cash crop. Jute is used for green manure
56. (b) The ear muscles are vestigial organs in man. Vestigial and has much higher CO2 assimilation rate.
organs are not functional in organisms. These are not 76. (d) Ajinomoto is Chinese salt used for preparing noodles.
active in related animals but were functional in their It is used as a flavouring agent.
ancestors. There are more than 90 vestigial organs in 77. (b) Bougainvillea is popularly planted along the road for
human beings. absorption of vehicular pollutants.
57. (c) The parrot, platypus and kangaroo fall in the group of 78. (c) Wine is an alcoholic beverage, produced from fermented
warm blooded animals. They show homothermy. juice, generally of grapes, Beer is the oldest alcoholic
58. (b) Chlorophyll is a tetrapyrrole ring system that contains beverage manufactured by brewing and fermentaion
the magnesium ion. The tetrapyrole ring system of cereal grains (e.g., barley). Whisky is a type of
chelates magnesium illustrating a conjugated double distilled alcoholic beverage produced from molasses.
bond. This bond gives the light absorption feature to Rum is also a distilled alcoholic beverage made from
chlorophyll and provides it green colour. sugarcane’s juice.
Biology C-47
79. (c) A milkman puts banana leaf in milkpot because leaf 97. (a) The internal ear of human beings consists of three
can make the milk basic and avoid yeast formation parts viz. cochlea, utricle and sacculus The cochlea
(fermentation). Banana leaf has tryptophan which helps in hearing whereas utricle and sacculus help to
creates basicity. maintain the body balance.
80. (b) Silver foil is manufactured from metal that is non 98. (a) The hormones are secreted by endocrine glands of
biodegrable. Woollen mat, leather bag and jute basket body. They can behave as coenzymes, e.g., thiamine
are biodegradable because they are manufactured from pyrophosphate can help in the transfer of aldehye
natural materials. group. Enzymes can be protein or steroid in
81. (b) Tips of leaves in grasses and common garden plants composition.
exhibit water droplets in early morning hours because 99. (a) Azotobactor and Rhizobium are known as nitrogen
of a atmospheric phenomenon of dew formation. Dew flxing bacteria that are utilized as biofertilizers. The
is produced due to condensation of atmospheric biofertilizers can be applied with organic fertilizers.
moisture. 100. (c) A pregnant woman requires an additional 400mg of
82. (c) The excess fertilizer creates exosmosis in the plant folic acid daily during the first trimester of pregnancy.
tissue causing plasmolysis. Beacuse fertilizer produces Green vegetables are rich in folic acid because they
hypotonic solution. It may cause cellular damage to have folate.
plant leading to plant death. 101. (a) Mammalian digestive system do not secrete enzymes
83. (a) Insect bite creates formic acid that causes itching. to digest cellulose, but the cellulose digesting bacteria
84. (c) Sugarcane is a main raw material for the production of present in the rumen of cattles can digest the cellulose
sugar. Sugar producing countries are Cuba, Brazil, India existing in fodder.
and China. 102. (c) Blood is a fluid connective tissue. It carries harmones
85. (b) Vitamin-B is water soluble. Its over-dose is excreted from endocrine glands to target organs. It carries heat
out in urine, vitamin A, D, E and K are fat soluble and set homeostasis. It also aids in osmoregulation
therefore, they are not excreted in urine. by carrying fluid to the kidney and skin, etc.
86. (c) The banana develops from stem. The aerial parts of 103. (a) The pH of gastric juice is not more than 4.5 pH of saliva
banana exhibit only leaves. Therefore, banana can’t is 6-7.4, whereas the pH of blood is equal to 7.4
be multiplied by cuttings. 104. (a) Cotton wool is acquired from cotton plant. It consists
87. (a) All the three statements are not correct. Myopia refers of cellulose. Fructose is fruit sugar present in honey
long-sightedness, hypermetropia to short-sightedness and other fruits. Starch is a polysaccharide. It breaks
and presbyopia is a disorder in which the eye shows a down into maltose during digestion whereas sucrose
progressively diminished ability to focus on near object is common sugar. It is a disaccharide having glucose
with the passage of age. ans fructose and is acquired from sugarcane.
88. (d) lodine deficiency causes goitre in human beings. 105. (a) The contraction of eyeball brings about in a long-
Iodine is the main component for the production of sighted eye. To see the nearby objects, a convex lense
thyroxine from thyroid gland and the main reason of is used.
goitre is hypo or hyperthyroidism. 106. (b) Formalin contains 40% aqueous solution of
89. (b) Eucalyptus growth is very fast and the stem is raw formaldehyde. Dead organs are preserved in 10%
material used for pulp and paper industry. formalin solution.
90. (b) Sunnhemp (crotoloria juncea) is a plant which is 107. (c) Insects that transmit diseases to humans are known
utilized for green manuring in India. It is largely as vectors. Insects may carry infection mechanically
cultivated for green manure and its leaves are fed as on feet or mouth parts, by passage through the
high protion supplement to other poorer feeds. digestive tract but without the insect being infected
91. (a) Silk fibres consist of protein like wool. It is produced or by becoming an intermediate host with some part of
by pupa of silk insect. the parasite’s life cycle occuring in the insect tissue.
92. (a) The major raw materials for soap production are fat 108. (b) Photochemical smog takes place in warm, dry and
and alkali. Other materials like water, softner, perfume, sunny climate.The word smog means ‘smoke’ and ‘fog’
etc, may also be mixed if required. means ‘smoky fog’. Its main components are created
93. (d) Kala-azar is spread by sandfly of genus phlebotomus from the action of sunlight on unsaturated
hydrocarbons and nitrogen oxide produced by
argentipse of yellow fever and dengue fever is spread
automobiles and factories.
by Ades mosquito whereas filariasis is spread by
109. (b) Cirrhosis is a only disorder which is caused due to
Anopheles mosquito. excessive drink of alcohol or hepatitis. All the other
94. (c) The warm blooded animals produce heat by converting three disorders are infectious diseases and transmitted
their food to energy and maintain a constant body from one peron to another.
temperature thus may be active even in cold 110. (a) The most non biodegradable wastes are produced by
environment whereas cold blooded animals hibernate thermal power plants. They utilize radioactive elements
and need much less energy to survive. as raw material and thus evolves radioactive wastes
95. (a) The deficiency of calcium can cause irritability, anger which takes a very long period (hundreds of years) for
tendency and impaired immune function. losing its radioactivity completely.
96. (c) Progeria (pro means prior; geria means ageing) causes 111. (b) The small leaves possess no or very few stomata thus
early ageing and the death of child occurs at the age the loss of water by transpiration is decreased and
of 12-13 years. It is a progressive genetic disease and protects the plant from dehydration.
created due to the mutation in the LMNA gene. 112. (c) Bats can fly in dark because they transmit ultrasonic
waves. The bats have very poor vision. It cannot see
EBD_7367
48
C- Biology
in dark. It produces ultrasonic waves striking with the 128. (b) Exposure to microgravity in the environment of space
obstacles present on its path. The bat receives its echo induces astronauts to lose calcium from bones. This
and changes its path to protect itself from collision. loss takes place due to absence of ‘Earth gravity’ which
113. (a) White blood cells destroy foreign proteins (bodies) disrupts the process of bone maintenance. The calcium
e.g pathogens by the process of phagocytosis and in is excreted out with urine.
this way protects the body from infection. 129. (c) Raw alcohol after being drunk will be absorbed fastest
114. (d) After raining for 30-40 minutes, the pollutants present through the wall of digestive tract. Alcohol is easily
in atmosphere come down with raining water that raise diffusible across the plasma membrance. It does not
its pH upto 5.6 making it acidic in nature. need any digestion before absorption in the alimentary
115. (c) Fluoride is essential in small quantity for the protection canal. Therefore, its absorption begins from mouth,
of the teeth but in high quantity it causes fluorosis whereas the other substances are absorbed in the later
deforming the teeth. parts of the alimentary canal.
116. (b) Down’s syndrome is characterised by an extra 130. (b) Transpiration is a process like evaporation part of the
chromosome 21. It is also known as trisomy 21. All the water cycle and it is the loss of water vapour from
given characters are observed in Down’s syndrome, different parts of plants, particularly in leaves but also
except option (b) only early ageing which is the in stems, flowers and roots.
symptom of progeria disease. 131. (b) All enzymes are proteins which catalyse various
117. (b) Some bats are sanguivorous feeding on animal blood. reactions occuring in cells. Therefore all enzymes are
When it sucks the blood of an animal which is suffering proteins, but not all proteins are enzymes.
from rabies, it can cause rabies if it bites another animal 132. (a) Archeopteryx is a genus of theropad dinosaur that is
or man. closely related to bird. It demonstrates the evidence
118. (b) All the three items are organic matter and produced of origin of birds from reptiles.
by living organisms thus are biodegradable wood is 133. (c) Vitamin K is a group of structurally similar to fat-
obtained from trees and leather is obtained from the soluble vitamins that are essential for the synthesis of
hides of animals. Glass is acquired naturally or by certain proteins needed for blood coagulation or
melting silicates. clotting.
119. (c) Salivary glands secrete amylase enzyme (ptyalin) and 134. (c) In chemistry, pH measures the acidity or basicity of an
water to moist the food and to digest starch. When aqueous solution. pH is normally measured in a range
we see, smell or eat something we like, the salivary of 0-14.
glands are stimulated and secrete saliva in buccal cavity pH value of some common material
thus watering the mouth.
120. (d) The transpiration takes place from the leaves surface Material pH value
all the time. The water taken out from leaves absorbs 1. Sea water 8.4
heat from the surrounding atmosphere to vapourise 2. Human blood 7.4
so the area under a tree becomes cool.
3. Milk 6.4
121. (c) Polar bears and penguins live in different polar regions
i.e. polar bears live in the North Pole while the penguins 4. Human urine 6.0
live in Antarctica (the Southern Hemisphere). 5. Alcohol 2.8
122. (d) After 4-8 weeks of infection by HIV virus to a healthy
135. (c) Due to lower atmospheric presure, N2 gets mixed with
individual, the acute phase of HIV infection begins.
O2 to form such oxide which is inhaled by the
About 30-60% people develop symptoms like severe
passengers or crew.
persistant headache with sore throat and mild fever 136. (a) Antibiotics are very effective medicines that treat
etc. bacterial infections. They are ineffective against viral
123. (b) The cause of goitre is normally due to lack of iodine. infection as viruses have different genetic makeup as
Iodine plays an important role in the synthesis of compared to bacteria.
thyroxine hormone from thyroid gland. 137. (a) Secretin and cholecystokinnin activate pancreas and
124. (a) Endosulfan is an organochlorine pesticide and acaricide gall bladder to discharge their secretions.
which is used. Endosulfan became a highly controversial 138. (b) Blood group B can provide blood to blood Groups B
agrichemical due to large number of victims in kerala. and AB and gets from Group B and Group’O.
125. (d) The cinchona plant has antimalarial alkaloids 139. (b) If human blood becomes acidic (low pH), The amount
cinchonidine and quinine. Quinine is obtained from of CO2 is increased and O2 carrying capacity of
the bark of this plant. haemoglobin is reduced.
126. (a) Cutting and peeling of onions bring tears to the eyes 140. (d) Cancer is more common in older people because they
because of the presence of sulphur in the cell. have accumulation of more mutations.
127. (a) The sprinkled salt dissolves in the water present on 141. (a) Blood is a viscous fluid in human being. This viscosity
the moist skin of Earthworm and makes concentrated is due to presence of different proteins in blood.
solution. The body fluid, being hypotonic, diffuses 142. (a) During inhalation, the diaphragm contracts and flattens
out and the worm dies due to osmotic shock (water enlarging the volume of the thoracic cavity. Thus,
loss-dehydration). reduction in intra-thoracic pressure takes place.
Biology C-49
143. (c) Carbon monoxide (CO) readily gets combined with the and an irrational fear of weight gain, and a distorted
haemoglobin of the blood causing harmful effect. body self-perception.
144. (b) Heparin is an anticoagulant (blood thinner) that 159. (c) Stomata are minute pores that are used for gas
prevents the blood clotting. It is applied to cure and exchange. Air containing carbon dioxide and oxygen
prevent blood clots in the veins, arteries, or lungs. enter the plant through these pores where it is used
145. (d) All the above process will get stopped if vaseline is in photosynthesis and respiration. Also water vapour
applied on both the surfaces of the leaf. Reason is that evaporates into the atmosphere through these pores
stomata will be blocked, all these processes requires by a process called transpiration.
the direct contact between the leaf and atmosphere. 160. (a) Marchantia belongs to group bryophytes. In
146. (b) The person will suffer from deficiency of protein bryophytes plants absorb water through general body
because potato has very little amonut of protein. surface and other substances also. Bryophytes lack
147. (a) Birds are warm blooded animals maintaining a constant xylem and phloem Marsilea and fern belong to group
body temperature through the process of homeostasis. pteridophytes having xylem tissue for transport of
Fishes, amphibians and reptiles are cold blooded water. Cycas belongs to gymnosperm also having
animals. xylem.
148. (d) Mitochondrial DNA is the DNA located in organelles 161. (c) Elephantiasis is a symptom of a variety of diseases,
where parts of a person's body swell to massive
called mitochondria. In eukaryotic cells it converts
proportions. It is caused by a number of parasitic
chemical energy from food into a Adenosine
worms, particularly Wuchereria bancrofti.
Triphosphate (ATP) that cells can use. 162. (c) Cereals are good source of carbohydrate.
149. (d) A biodegradable product gets decomposition safely 163. (d) A person with AB blood group can receive blood from
and quickly, through biological process into the raw any group but cannot donate blood to any group other
materials and disappear into the environment. Cow- than AB. O individual can receive blood only from a
dung, padddy-husk and vegetable wastes, are group of O individual but can donate blood to all
biodegradable products. groups.
150. (a) Two strand of DNA are held together by h ydr og en - 164. (a) The kidneys filter metabolic wastes, excess ions, and
bond formed between the bases of two strand. Adenine chemicals from the blood to form urine. The ureters
pairs with Thymine by two Hydrogen-bond and are a pair of tubes that carry urine from the kidneys to
Guanine pairs with cytosine by three Hydrogen-bond. the urinary bladder. The urethra is the tube through
151. (a) Magnesium is found in green pigment of leaf. The which urine passes from the bladder to the exterior of
green pigment is called chlorophyll molecule. the body.
Chlorophyll is chemically a tetrapyrrole structure in 165. (c) Cell Membrane, Cytoplasm and Nucleus are found in
which Mg++ exists in the centre. both plant and animal cells.
152. (a) Pancreas secretes glucagon hormone, pituitary 166. (b) The coronary arteries deliver blood to the heart muscle,
secretes thyroid stimulating hormone. Adrenal glands providing a continuous supply of oxygen and
secrete cortisol and kidney synthesises or converts nutrients needed for it to stay healthy and function
biological inactive 25-hydroxychole calciferol (vitamin normally. The pulmonary veins receive oxygenated
D-3) into biological active 1,25 dihydroxychole blood from the lungs and drain into the left atrium of
calciferol (vitamin D-3). the heart.The Hepatic artery supplies oxygen-rich
153. (d) If owl population decreases, it will indirectly affect blood to the liver, pylorus, pancreas, and duodenum.
wood lice population. If owl population will decrease, The renal arteries carry a large portion of total blood
the population of wood lice increases. Reason is that flow to the kidneys.
owl is not present to eat the wood lice population. 167. (b) Fumigants are used to penetrate and remain in the
154. (a) Deficiency of calcium causes weakening of bones container for long enough to kill all stages of insects
because bone consists of calcium and phosphate both. present in stores grains.
155. (c) Amnion is a membrane filled with fluid which covers 168. (a) The correct match is as follows:
the embryo. Ascaris - Nematoda
The sex of growing embryo (foetus) may be determined Malarial parasite - Protozoa
by ultrasound as early as 11 weeks pregnancy. Housefly - Arthropoda
156. (c) Among the given options , bug possesses only three Cow - Mammalia
pairs of legs. All the insects in phylum Arthropoda 169. (d) Mushroom is not autotroph.
have three pairs of legs in given option only bug 170. (b) Holstein Friesians are world's highest-production dairy
belongs to class-insecta. animals. They are found mostly in Netherlands/
Germany.
157. (a) Fats are essential for the formation of all membranes 171. (b) A prokaryote is a single-celled organism that lacks a
Fats are present in the form of phospholipid. membrane-bound nucleus (karyon). Plant and animal
Phospholipid are the same as triglycerides except that cells are both eukaryotic cells, so they have several
one of the three fatty acid units is replaced by a features in common, such as the presence of a cell
molecule having phosphorus. Body can not release membrane, mitochondria and endoplasmic reticulum.
energy in fats as quickly as the energy in Chloroplasts are found in plant cells which is
carbohydrates because fat first will oxidise into eukaryotic. Ribosomes are found in both prokaryotes
carbohydrate to evolve energy. and eukaryotes.
158. (d) Anorexia nervosa is an eating disorder characterised 172. (c) Characteristics of mammals:
by improper food restriction, inappropriate taking meal • Vertebrates (which means they have a backbone
habits or rituals, obsession with having a slim body, or spine).
EBD_7367
50
C- Biology
• Regulate their own body temperate which allows 186. (a) Phytosterols (plant sterols) are a family of molecules
them to live in almost every climate on Earth. related to cholesterol. Structurally, these are very
• Have hair on their bodies. similar to cholesterol except that they always contain
• Produce milk to feed their babies. some substitutions at the C24 position on the sterol
• Four chambered heart: two atria and two ventricles
(bird and mammal) side chain.
173. (b) In human digestive system the process of digestion 187. (c) The Artificial Insemination (AI) is the process in which
starts in buccal cavity. The buccal cavity is more a thin, flexible tube (Catheter) is used to put sperm
commonly known as the mouth, and it is the beginning artificially into a woman's cervix.
of the digestive system for humans and animals alike. 188. (d) Introduction of new traits to the plant which does not
It starts with the lips and ends with the throat, occur naturally in the species introduction of new DNA
covering the oral cavity, the tongue, and the jaw in and removal of existing DNA are the processes by
between. which Genetically Modified crops are produced.
174. (b) The TB bacteria are put into the air when a person
189. (d) Vitamin K is essential for the functioning of several
with TB disease of the lungs or throat coughs,
sneezes, speaks, or sings. People nearby may breathe proteins involved in physiological processes that
in these bacteria and become infected. encompass, but are not limited to, the regulation of
175. (b) An axon is also known as a nerve fibre. It is a long, blood clotting.
slender projection of a nerve cell, or neuron that 190. (c) Cobalt is the active centre of coenzymes called
typically conducts electrical impulses away from the cobalamins, the most common example of which is
neuron's cell body. Each nerve is a cordlike structure vitamin B12.
that contains many axons and also called nerve fibres. 191. (a) Glucose is a sugar with the molecular formula C6H12O6.
Within a nerve, each axon is surrounded by a layer of
The suffix "-ose" is a chemical classifier, denoting a
connective tissue called the endoneurium.
176. (a) Genetic screening is a process through which analysis Carbohydrate.
of gene is performed to find out defective gene causing 192. (d) Protoplasm is the whole content of a cell enclosed
a specific disorder in a person. within the cell membrane, including both the cytoplasm
177. (b) Paul Berg (1926), an American biochemist is considered and nucleus.
as the father of genetic engineering. He evolved a 193. (c) Insulin is a peptide hormone produced by beta cells in
technique to join DNAs from two different organisms. the pancreas and by Brockmann body in some teleost
178. (c) Epigenetic Changes, mutations (due to change in DNA fish.
structure) and sexual reproduction (due to change 194. (a) Good Cholesterol refers to HDL (High Density
during meiosis process) cause variation in the genetic Lipoprotein), Lipoproteins are complex particles
material of off spring. composed of multiple type of proteins.
179. (b) Cobalt is an element taking part in the composition of 195. (b) Vitamin A - Night blindness
Vitamin B12. It plays an important role in production of Vitamin B1 - Beriberi
blood cells. Vitamin C - Scurvy
180. (b) Large intestine is the last part of alimentary canal 196. (b) Germplasm is a biosubstance (eg. - seeds, tissue pollen
where extra water is absorbed to prepare the waste as grains, sperms & eggs etc.) which is used as genetic
a solid stool. material for animal & plant breeding for preservation
Renal tubule in kidney is responsible for absorption and research purpose.
of water to produce proper concentration of urine. 197. (d) Dengue virus after infection, interferes the production
181. (b) Measles is a viral disease which is also called rubeola. of platelets in bone marrow, infects endothelial cells
It is a contagious disease spreading through contact and bind with platelets.
sneezing and coughing etc. 198. (a) Environmental changes affecting all regions the world
182. (c) Bone marrow is located within the bone cavities. It is and influence all organisms. Amphibians are one of
composed of stromal cells and parenchymal cells. the most sensitive groups to change, whether that is
Neutrophils and lymphocytes are blood cells which caused by habitat loss, invasive species, disease, trade
are produced within bone marrow. or climate change. Nearly 33% of the amphibian species
183. (c) DOTS (Directly Observed Treatment, Short course) of the world are categorized as threatened on the Red
launched by WHO is a therapy for treatment of TB List. Furthermore, given their complex life cycles and
patient. other traits, amphibians are often recognized as
184. (b) Penicillin is very effective medicine against bacterial indicators of ecosystem health.
infection. During bacterium division, it prevents the 199. (d) The human immunodeficiency virus (HIV) seeks and
cell wall formation. Thus it helps in prevention of destroys CD4+ cells, a type of T lymphocyte (T cell).
bacterial growth. T cells are critical to the immune system. They're
185. (d) Mostly antibiotics inhibit the synthesis of cell wall responsible for warding off diseases and most
and protein synthesis in bacteria. Due to this reason, infections, including viral infections.
bacterial infection is controlled by this type of 200. (c) Carbon monoxide (CO) has strong affinity to combine/
medicines. bind with haemoglobin in blood to form
Biology C-51

carboxyhaemoglobin, which reduces oxygen carrying nitrogen (3.2%), calcium (1.5%), and phosphorus
capacity of blood to cause headache, n ausea, (1.2%).
asphyxiation and fainting (low supply of O2 to body 209. (a) DNA is made up of many nucleotides joining together,
though one may inhale normally). Against normal level called polynucleotide chain of DNA. Each
of 0.5% CO in blood, severe pollution may lead to 8 to polynucleotide is made up of three components: (i)
9% CO. nitrogenous base [such as adenine (A), guanine (G),
201. (d) Parthenogenesis, a reproductive strategy that involves cytocine (C) and thymine (T)], (ii) deoxyribose sugar,
development of a female (rarely a male) gamete (sex and (iii) phosphate group.
cell) without fertilization. It occurs commonly among 210. (b) The process by which the genetic information in a
lower plants and invertebrate animals (particularly strand of DNA is copied into a new molecule of
rotifers, aphids, ants, wasps, and bees) and rarely messenger RNA (mRNA) is termed as transcription. It
among higher vertebrates. is carried out by an enzyme called RNA polymerase
Apogamy is a type of reproduction, occurring in some and a number of accessory proteins called
ferns, in which the sporophyte develops from the transcription factors.
gametophyte without fusion of gametes 211. (a) In cancer, there is unwanted cell proliferation leading
Adventive embryony, also called sporophytic to uncontrolled cell division. Some cancers may spread
apomixis, sporophytic budding, or nucellar embryony: into other tissues. Mutation is the main cause of
Here there may be a megagametophyte in the ovule, cancer at genetic level. Mutation causing agents are
but the embryos do not arise from the cells of the called carcinogens, such as UV radiation, tobacco and
gametophyte; they arise from cells of nucellus or the tobacco smoke, pesticides, etc.
integument. 212. (b) In general, plant viruses have single-stranded RNA.
202. (a) Fluorine deficiency is a disorder which may cause Animal viruses have either single or double-stranded
increased dental caries (or tooth decay, is the RNA or double-stranded DNA. Bacterial viruses
breakdown of dental tissues by the acidic products usually have double stranded DNA.
released by the "bacterial fermentation of dietary 213. (b) Syngamy is the process of fusion of two cells or their
carbohydrates.") and possibly osteoporosis (a bone nucleus during reproduction. More particularly, it can
disorder which leads to a decrease in bone mass, and be said that syngamy is fusion of one male gamete
an increase in bone fragility), due to a lack of fluoride with egg cell and leads to formation of diploid zygote.
in the diet. 214. (a) Vitamin C is needed to produce skin, tendons,
203. (b) The human ear can respond to minute pressure ligaments, and blood vessel. It is also needed in repair
variations in the air if they are in the audible frequency of bones and teeth. Deficiency of vitamin C results in
range, roughly 20 Hz - 20 kHz. bleeding of gums, falling of teeth, weak bones and
204. (c) Insulin is a hormone made by the pancreas that allows delay in wound healing.
your body to use sugar (glucose) from carbohydrates 215. (a) Heart beat originates from sinoatrial node, a group of
in the food that you eat for energy or to store glucose cells situated in the upper part of the right atrium wall.
for future use. Insulin helps keeps your blood sugar The electrical impulses causing the heart beat are
level from getting too high (hyperglycemia) or too low generated in sinoatrial node. Hepatic portal system
(hypoglycemia). comprises hepatic portal vein and veinlets. T wave
205. (b) Prokaryotes (meaning 'before nucleus') are cells depicts repolarization of heart ventricles. QRS wave
without a definite nucleus. The DNA floats in the are three waves seen on a electrocardiogram.
cytoplasm and they divide through mitosis. RBC and 216. (b) Plant growth regulators or plant hormones
platelets present in eukaryotes also do not have (phytohormones) are chemicals that control plant
nucleus. growth, e.g. auxins, gibberellins, cytokinins, abscisic
206 (b) Enzymes are soluble protein molecules that can speed acid and ethylene. Auxin is composed of indole-3-
up chemical reactions in cells. These reactions include acetic acid. Cytokinins are derivatives of the
respiration, photosynthesis and making new proteins. nitrogenous purine base adenine. Some plant
For this reason enzymes are sometimes called hormones are also composed of derivatives of
biological catalysts. These enzymes functions carotenoids and terpenes.
optimally at 37° C and it remains unchanged after 217. (a) Silk is a fibre produced by the silkworm in production
reaction. of its cocoon. It consists mainly of two proteins, fibroin
207. (b) Altitude sickness is a disorder caused at high altitude and sericin. Silk consists of 70–80% fibroin and
due to low partial pressure of oxygen. Symptoms of 20–30% sericin; fibroin being the structural center of
altitude sickness include weakness, sleepiness, and the silk, and sericin being the gum coating the fibres
lack of appetite. and allowing them to stick to each other.
208 (b) The elemental composition of an adult human body 218. (c) Blue baby syndrome is a condition some babies are
by mass is O>C>H>N respectively. 99% of the mass born with or develop early in life. It’s characterized by
of the human bodies are made up of just six elements: an overall skin color with a blue or purple tinge, called
oxygen (65%), carbon (18%), hydrogen (9.5%), cyanosis. This condition stems from nitrate poisoning.
EBD_7367
52
C- Biology

It is can happen in babies who are fed infant formula 228. (b) Bioaccumulation is defined as the increase in
mixed with well water or homemade baby food made concentration of a substance(s) in an organism or a
with nitrate-rich foods, like spinach or beets. part of that organism. Toxic substances are lipophilic
219. (c) A-4, B-1, C-2, D-3 is the correct answer. ATP supplies or fat-loving, the reason why these substances are
energy to the cell. It is formed by the oxidation of deposited and concentrated in the fat tissues of the
glucose in the mitochondria. Since, mitochondria organisms. The affected organism has a higher
synthesize energy rich compounds (ATP) so, known concentration of the substance than the concentration
as power house of the cell. Ribosomes are dense, in the organism's surrounding environment. The toxic
spherical and granular particles which occur freely in substances are very slowly metabolized or excreted
the matrix (cytosol) or remain attached to the ER. It so if the organism keeps on consuming prey or food
plays an important part in the synthesis of proteins. contaminated with toxic substances, the concentration
Lysosomes are simply tiny spherical sac-like of the substance will further increase in its body,
structures evenly distributed in the cytoplasm. Its cells hence, bioaccumulation results. When a certain
digest foreign proteins, bacteria and viruses. So, it is threshold level is reached, measured in parts per million
a kind of garbage disposable system of the cell. (ppm), symptoms due to the type of toxin are
220. (a) Human Insulin protein consists of 51 amino acids. manifested.
Insulin is formed as a precursor protein preproinsulin. 229. (d) Fish are the gill-bearing aquatic craniate animals that
This is coded by the INS gene. In some animals there lack limbs with digits. Fish, the member of the Animalia
are two insulin genes or two genes that code for Kingdom is classified into Phylum Chordata and
insulin. In most animals, including humans, a single Vertebrata Subphylum. Fishes poses notochord,
gene is present. tubular nerve chord, paired gills, segmentation of the
221. (a) A biome is a community of plants and animals that body parts, post anal tail, ventral heart, and an
have common characteristics for the environment they endoskeleton to be the member of the Chordata. In
exist in. They can be found over a range of continents. order to be a vertebrate, it poses backbone. This back
Biomes are distinct biological communities that have bone supports and protects the spinal cord. All the
formed in response to a shared physical climate. species of the fish found in the world are classified
222. (b) is the correct answer. into the following three groups.
223. (d) Fresh urine has little to no odor. But if urine sits for a • Agnatha - jawless fish
time — as can happen when a person is wearing • Chrondrichthyes - cartilaginous fish
protective underwear or an incontinence pad — the • Osteichthyes - bony fish
urine will begin to smell like ammonia. The smell is Flying fish is a bony fish.
caused by bacteria that break down urea, a substance 230. (a) Adipocytes are cells that derive from multipotential
that is excreted in urine. mesenchymal cells; their main role is to store energy
224. (d) Starch is a polymer of a-glucose and consists of two in the form of lipids to cushion and insulate the body
components-amylose and amylopectin. Glycogen is a from the outer environment.
polymer of 1000 glucose units. Cellulose is a linear 231. (c) Nereis possess setae and parapodia for locomotion.
polymer of b-D-glucose. Natural rubber is an addition 232. (c) Lysosomes are spherical membranous sacs of
polymer that is obtained as a milky white fluid known enzymes. Lysosome enzymes are made by proteins
as latex from a tropical rubber tree. Natural rubber is from the endoplasmic reticulum and enclosed within
from the monomer isoprene (2-methyl-1,3-butadiene), vesicles by the Golgi apparatus. Lysosomes are formed
which is a conjugated diene hydrocarbon by budding from the Golgi complex.
225. (a) Kidney directly controls water content of blood. 233. (b)
Kidneys purify the blood by removing toxins from it, 234. (a)
and in doing this water is excreted along with these 235. (b) Three chambered hearts with two atria and one
toxins. Potassium and sodium elements are involved ventricle is present in amphibian and reptiles. The 3
in the process. chambered heart contains only one ventricle in which
226. (c) Magnesium is essential for the formation of oxygen poor and oxygen rich intermingle, but not
chlorophyll in green plants. Without magnesium, entirely. In amphibians, the pulmonary respiration is
chlorophyll cannot capture sun energy needed for improved because they absorbs oxygen through skin
photosynthesis. In short, magnesium is required to also.
give leaves their green color. 236. (a) Muscle cramping is a common problem encountered
227. (d) Tuberculosis (TB) is caused by a type of bacterium by athletes and non-athletes alike. Lactic acid is formed
called Mycobacterium tuberculosis. It's spread when and accumulated in the muscle under conditions of
a person with active TB disease in their lungs coughs high energy demand, rapid fluctuations of the energy
or sneezes and someone else inhales the expelled requirement and insufficient supply of O2. It causes
droplets, which contain TB bacteria. muscles to get cramps.
C HA P T E R
HISTORY
37
1. Among the following which Mughal Emperor introduced 10. In which one of the following cities, was the East India
the policy of Sulh-i-kul? [2007-II] Association founded by Dadabhai Naoroji in 1866?
(a) Babar (b) Humayun [2008-I]
(c) Akbar (d) Shahjahan (a) Paris (b) London
2. The Sun Temple of Konark was built by Narasimhadeva I. (c) New York (d) Tokyo
To which dynasty did he belong to? [2007-II] 11. Who among the following had founded the Theosophical
(a) Somavamsi dynasty Society in the United States of America? [2008-I]
(b) Imperial Ganga dynasty (a) Swami Dayanand Saraswati
(c) Suryavansi Gajapti dynasty (b) Madam Blavatsky
(d) Bhoi dynasty (c) Madam Cama
3. Consider the following statements [2007-II] (d) Lala Hardayal
1. At the time of independence, the Government of India 12. Who among the following was not a party to the coalition
followed the calender based on Saka era. that fought against the English in the Battle of Buxar?
2. The National Calendar commenced on Chaitra 1 Saka, [2008-I, II]
1879 corresponding to March 22, 1957 AD. (a) Mir Kasim (b) Mir Jafar
Which ofthe statements given above is/are correct? (c) Shuja-ud-daulah (d) Shah Alam II
13. Which one of the following is the correct chronological
(a) Only 1 (b) Only 2
order of the freedom movements of India? [2008-I]
(c) Both 1 and 2 (d) Neither 1 nor 2
(a) Quit India Movement, Non-Cooperation Movement,
4. Who among the following was the Governor-General of Civil Disobedience Movement
India immediately preceeding Chakravarthi Raja (b) Non-cooperation Movement, Civil Disobedience
gopalachari? Movement, Quit India Movement
(a) The Marquess of Linlithgow [2007-II] (c) Quit India Movement, Civil Disobedience Movement,
(b) Lord Mountbatten Non, Cooperation Movement
(c) Lord Wavell (d) Non-Cooperation Movement–Quit India Movement–
(d) Lord Chelmsford Civil Disobedience Movement
5. With which one of the following did the Civil Disobedience 14. ‘The Musalman’, the handwritten daily newspaper in
Movement in India commerce? [2008-I] circulation since 1927, is published from which one of the
(a) The Dandi March following places? [2008-I]
(b) No Tax campaign by Sardar Patel (a) Chennai (b) Hyderabad
(c) The unfurling of the National flag by Nehru (c) Mysore (d) Lucknow
(d) Mahatma Gandhi’s going on fast 15. The ‘Arthasastra’ is a treatise on which one of the following?
6. After 1857, which of the following announced, at a Darbar [2008-I]
at Allahabad, the assumption of the Government of India (a) Economics
by the Sovereign of Great Britain? [2008-I] (b) Environment
(a) Lord Canning (b) Sir John Lawrence (c) Political Philosophy
(c) Lord Mayc (d) Lord Northbrok (d) Religion in Administration
7. Arrange the following in correct chronological order 16. Arrange the following in chronological order [2008-I]
1. Third Carnatic War [2008-I] 1. Partition of Bengal
2. First Burmese War 2. Chauri-Chaura Incident
3. First Mysore War 3. First Round Table Conference
4. Second Afghan War Codes :
Codes : (a) 1, 2, 3 (b) 3, 2, 1
(a) 1, 4, 3, 2 (b) 1, 3, 2, 4 (c) 1, 3, 2 (d) 2, 1, 3
(c) 2, 4, 1, 3 (d) 3, 1, 2, 4 17. Who among the following was the Viceroy of India at the
8. Who among the following was the first Governor General time of the formation of Indian National Congress?
of India? [2008-I] [2008-I]
(a) Lord Amherst (b) Lord William Bentinck (a) Lord Mayo (b) Lord Ripon
(c) Sir Charles Metcalfe (d) Robert Clive (c) Lord Dufferin (d) Lord Lansdowne
9. In which one of the following sessions was the Indian 18. Separate electorates for Muslims in India were introduced
National Congress split into moderates and extremists? by which one of the following Acts? [2008-I]
[2008-I] (a) Government of India Act of 1909
(a) Nagpur (b) Allahabad (b) Indian Councils Act of 1892
(c) Surat (d) Calcutta (c) Rowlatt Act of 1919
(d) Government of India Act of 1935
EBD_7367
54
C- History

19. Match the following [2008-I] Codes :


List I List II A B C D
(Session of Indian National (Venue) (a) 1 2 4 3
Congress) (b) 3 4 2 1
A. 1st session 1. Allahabad (c) 1 4 2 3
B. 2nd session 2. Madras (d) 3 2 4 1
C. 3rd session 3. Calcutta 29. Who among the following was thrice elected President of
D. 4th session 4. Bombay the Indian National Congress? [2008-II]
(a) Dadabhai Naoroji
Codes :
(b) Surendranath Banerjee
A B C D A B C D (c) Gopal Krishna Gokhl
(a) 4 3 2 1 (b) 1 3 2 4 (d) Shankaran Nair
(c) 4 2 3 1 (d) 1 2 3 4 30. Who among the following has started the Public Works
20. The Treaty of Amritsar was concluded between Maharaja Department in India in AD 1848? [2008-II]
Ranjit Singh and who of the following? [2008-I] (a) Lord William Bentinck (b) Lord Dalhousie
(a) Lord Cornwallis (b) Lord Dalhousie (c) Lord Wellesley (d) Lord Cornwallis
(c) Lord Hastings (d) Lord Minto 31. Under which one of the following systems of assessment,
21. The Buddha delivered his first sermon, known as ‘Turning the British Government collected revenue directly from the
of the wheel of law’ at [2008-I] farmers? [2008-II]
(a) Sanchi (b) Sarnath (a) Zamindari (b) Ryotwari
(c) Sravasti (d) Bodh Gaya (c) Annawari (d) Desaiwari
22. Whose philosophy is called the Advaita? [2008-II] 32. What is the correct sequence of the following events?
(a) Ramanujacharya (b) Shankaracharya 1. Rowlatt Act [2008-II]
(c) Nagarjuna (d) Vasumitra 2. Gandhi-lrwin Pact
23. Who among the following was the founder of the Muslim 3. Morley-Minto Reforms
League? [2008-II] 4. Illbert Bill
Select the correct answer using the codes given below
(a) Muhammad Ali Jinnah (a) 1, 2, 4, 3 (b) 3, 4, 1, 2
(b) Shaukat Ali (c) 4, 1, 3, 2 (d) 4, 3, 1, 2
(c) Nawab Salimullah 33. Who among the following was not a member of the Cabinet
(d) Aga Khan Mission? [2008-II]
24. In Buddhism, what does Patimokkha stand for? (a) Sir Stafford Cripps (b) AV Alexander
[2008-II] (c) Radcliffe (d) Pathick Lawrence
(a) A description of Mahayana Buddhism 34. Which one of the following was the original name of Tansen,
(b) A description of Hinayana Buddhism the famous musician in the court of Akbar? [2008-II]
(c) The rules of the Sangha (a) Mahananda Pande (b) Lal Kalwant
(d) The questions of king Menander (c) Baz Bahadur (d) Ramtanu Pande
25. In which of the following years was the first Railway line 35. Who among the following has issued the coin rupee for the
first time? [2008-II]
between Bombay and Thane laid? [2008-II]
(a) Muhammad bin Tughluq
(a) 1853 (b) 1854 (b) Ala-ud-din Khilji
(c) 1856 (d) 1858 (c) Sher Shah
26. What is the correct sequence of the following events? (d) Akbar
1. Bardoli Satyagraha [2008-II] 36. In which of the following years was the All India Trade
2. Rajkot Satyagraha Union Congress formed in Bombay? [2008-II]
3. Champaran Satyagraha (a) 1918 (b) 1919
4. Nagpur Satyagraha (c) 1920 (d) 1921
Codes : 37. Who among the following was elected as the President of
(a) 1, 2, 4, 3 (b) 4, 3, 1, 2 All India Khilafat Conference met at Delhi in 1919?
(c) 3, 1, 4, 2 (d) 3, 4, 1, 2 [2008-II]
27. Who among the following Urdu poets was invited to the (a) Motilal Nehru (b) Mahatma Gandhi
(c) M A Jinnah (d) Shaukat Ali
Second and Third Round Table [2008-II] 38. Who among the following was not a member of the
Conference? Constituent Assembly? [2008-II]
(a) Faiz Ahmad (b) Josh Malihabadi (a) Sardar Vallabhbhai Patel
(c) Muhammad Iqbal (d) Firaq Gorakhpuri (b) Acharya JB Kriplani
28. Match the following [2008-II] (c) Lok Nayak Jayprakash
List I List II (d) K M Munshi
(Person) (Associated in Formation of) 39. Who among the following had moved the objectives
A. GK Gokhaie 1. Servants of resolution which formed the basis of the Preamble of the
Indian Society Constitution of India in the Constituent Assembly on
B. MM Malaviya 2. Banaras December 13, 1946? [2008-II]
Hindu University (a) Dr B R Ambedkar
C. C Rajagopalachari 3. Free India Society (b) Dr Rajendra Prasad
(c) Sardar Vallabhbhai Patel
D. VD Savarkar 4. Swatantra Party
(d) Pandit Jawaharlal Nehru
History C-55
40. The head of a district (Ahara), the principal coordinator of Which of the statements given above is/are correct?
the revenue and the officer incharge of general and military (a) Only 1 (b) Only 2
functions in his jurisdiction during Mauryan empire was (c) Both 1 and 2 (d) Neither 1 nor 2
known as [2008-II] 49. Fa-Hien’s mission to India was to [2009-I]
(a) Krori (b) Rajuka (a) learn about the administrative system of the Gupta
(c) Foujdar (d) Chirastadar kings
41. ‘A Forgotten Empire’, written by the renowned historian (b) understand the social position of women during the
Robert Sewell is about which one of the following empires? Gupta period
[2008-II] (c) visit the Buddhist institutions and to collect copies of
(a) Kushan empire (b) Mauryan empire Buddhist manuscripts
(c) Vijayanagar empire (d) Mughal empire (d) get full knowledge about the condition of peasants
42. The Doctrines of ‘Non-Violence’ and ‘Civil Disobedience’ during the period of Gupta kings
associated with Mahatma Gandhi were influenced by the 50. Which British military officer defeated Tipu Sultan in India,
works of [2009-I] Napolean Bonaparte in Europe and eventually, became the
(a) Churchill-lrwin-Tolstoy Duke of Wellington? [2009-I]
(b) Ruskin-Tolstoy-Thoreau (a) Arthur Wellesley (b) Robert Clive
(c) Thoreau-Humen-Shaw (c) Warren Hastings (d) Richard Wellesley
(d) Cripps-Tolstov-Howes 51. Consider the following statements [2009-I]
43. The Ashtadhyayi of Panini, the Mahabhasya of Patanjali 1. Dayanand Saraswati founded the Arya Samaj in 1875.
and the Kashika Vritti of Jayaditya deal with [2009-I] 2. The Arya Samaj repudiated the authority of the caste
(a) Principles of Law (b) Principles of Phonetics system.
(c) Principles of Grammar (d) Principles of Linguistics 3. Dayanand Saraswati was born in the Brahman family.
44. Match the following [2009-I] Which of the statements given above are correct?
List I List II (a) 1, 2 and 3 (b) 1 and 2
(Person) (Satyagraha) (c) 2 and 3 (d) 1 and 3
A. Raj Kumar Shukla 1. Kheda Satyagraha 52. Consider the following statements about National Calendar
B. Ambalal Sarabhai 2. Ahmedabad Mill Strike
of India [2009-I]
C. Indulal Yagnik 3. Bardoli Satyagraha
1. The National Calendar based on the Saka era with
D. Vallabhbhai Patel 4. Champaran Satyagraha
Chaitra as its first month.
Codes :
2. Dates of the National Calendar have a permanent
A B C D A B C D
correspondence with dates of the Gregorian calendar.
(a) 3 1 2 4 (b) 4 1 2 3
(c) 4 2 1 3 (d) 3 2 1 4 Which of the statements given above is/are correct?
45. Consider the following statements relating to Raja (a) Only 1 (b) Only 2
Rammohan Roy [2009-I] (c) Both 1 and 2 (d) Neither 1 nor 2.
1. He knew that the spread of Western education was 53. Assertion (A) First War of independence broke out in India
necessary to develop a national and scientific attitude in 1857, soon after the departure of Lord Dalhousie from
in the Indian society. India.
2. He played a pioneering role in the religious and social Reason (R) Lord Dalhousie’s annexationist policy had
reform movements of 19th century, Bengal. caused great discontent. [2009-I]
Which of the above statements is/are correct? Codes :
(a) Only 1 (b) Only 2 (a) Both A and R are true and R is the correct explanation
(c) Both 1 and 2 (d) Neither 1 nor 2 of A
46. Sheikh Moin-ud-din, Bakhtiyar Kaki and Farid-ud-din Ganj- (b) Both A and R are true, but R is not the correct
i-Shakar were [2009-I] explanation of A
(a) prominent military leaders of the Sultanate period (c) A is true, but R is false
(b) prominent painters from the Sultanate period (d) A is false, but R is true
(c) prominent chisti saints 54. Which one of the following statement about Brihadeswara
(d) prominent poets from the courts of the Sultanate period temple at Tanjavur, is not correct? [2009-I]
47. Which of the following statements on Gandhian movements (a) The temple is a splendid example of Chola architecture
is not correct? [2009-I] (b) It was built by emperor Rajaraja
(a) Mahatma Gandhi was in favour of mass movement (c) The temple is constructed of granite
(b) Gandhian movements were non-violent in nature (d) The temple is a monument dedicated to Lord Vishnu
(c) In Gandhian movements, leadership had no role 55. Consider the following statements about Swami
(d) Mahatma Gandhi was in favour of passive resistance Vivekananda [2009-I]
48. Consider the following statements relating to Sher Shah 1. He said that Vedanta was the religion of all.
[2009-I] 2. He believed in reviving all the best traditions of
1. During Sher Shah’s region, the village Panchayat and Hinduism.
Zamindars were not allowed to deal with civil and 3. He was impressed by the status of women in the West.
criminal cases at the local level. Which of the statements given above are correct?
2. Sher Shah set-up army cantonments in different parts (a) 1, 2 and 3 (b) 1 and 2
of the empire and a strong garrison was posted in (c) 2 and 3 (d) 1 and 3
each of them.
EBD_7367
56
C- History

56. Which among the following was not one of the provisions 63. The Nehru-Liaquat Pact between India and Pakistan was
of the ‘Communal Award’? [2009-I] signed in 1950 to resolve the issue of [2009-II]
(a) Member of the depressed classes were assigned (a) the protection of minorities
reserved seats and separate electorates (b) the accession of princely states
(b) Separate electorates for the Muslims (c) the border disputes
(c) Separate electorates for the Europeans and the Sikhs (d) the problem of refugees
(d) The separate electorates were to lapse at the end of 10 64. October 26, 1947 is an important date in the Indian history,
years
because of [2009-II]
57. Which of the following statements about Annie Besant are
(a) Maharaja Hari Singh’s signing on Instrument of
correct? [2009-I]
1. She founded the Central Hindu College at Benaras. Accession
2. She organised the Home Rule League. (b) ceasefire with Pakistan
3. She introduced the Theosophical Movement in India. (c) merger of Sind
Select the correct answer using the codes given below (d) declaration of war over India by Pakistan
(a) 1, 2 and 3 (b) 1 and 2 65. Who drafted the Constitution of Muslim League, ‘The Green
(c) 2 and 3 (d) 1 and 3 Book’? [2009-II]
58. Which one of the following inscriptions mentions Pulakesin (a) Rahamat Ali
II’s military success against Harshavardhana? [2009-II] (b) Muhammad Iqbal
(a) Allahabad Pillar inscription (c) Muhammad Ali Jinnah
(b) Aihole inscription (d) Maulana Muhammad Ali Jauhar
(c) Damodarpur Copperplate inscription 66. ‘Use of white marble, long legs and slender frames, human
(d) Bilsad inscription beings as central characters and prominence of kings,
59. Which one of the following statements regarding Harappan princess and palaces’ were the characteristic features of
civilisation is correct? [2009-II] which one of the following ancient art forms of India?
(a) The standard Harappan seals were made of clay
[2009-II]
(b) The inhabitants of Harappa had neither knowledge of
(a) Amaravati School of Art
copper nor bronze
(c) The Harrapan civilisation was rural based (b) Gandhara School of Art
(d) The inhabitants of Harappan grew and used cotton (c) Mathura School of Art
60. After the death of Raja Rammohan Roy, the Brahmo Samaj (d) Pahari School of Art
split into two sections; the Brahmo Samaj of India and the 67. Which two of the following plays did Kalidasa wrote before
Adi Brahmo Samaj. Who were the leaders of the two writing Abhigyanashakuntalam? [2009-II]
sections, respectively? [2009-II] 1. Vikramorvashiyam
(a) Keshab Chandra Sen and Debendranath Tagore 2. Malavikagnimitram
(b) Radhakanta Deb and Debendranath Tagore 3. Swapnavasavadattam
(c) Keshab Chandra Sen and Radhakanta Deb 4. Kadambari
(d) Debendranath Tagore and Radhakanta Deb Select the correct answer using the codes given below
61. The ‘dual government’ recommended by Lord Clive (a) 1 and 2 (b) 1 and 3
provided that the [2009-II] (c) 1 and 4 (d) 3 and 4
(a) criminal justice would be left to the Nawabi officials, 68. Consider the following statements and identify the person
while civil and fiscal matters would be controlled by referred to therein with the help of the codes given below
the company
During his stay in England, he endeavoured to educate the
(b) company will look after fiscal matters and all the rest
would be dealt by the Indian rulers British people about their responsibilities as rulers of India.
(c) Indian rulers will deal with all the matters of He delivered speeches and published articles to support
administration under the supervision of a company his opposition to the unjust and oppressive regime of the
official British Raj. In 1867, he helped to establish the East India
(d) Indian rulers will be only titular heads and all the powers Association of which he became the Honorary Secretary.
shall be directly dealt by the company [2009-II]
62. Consider the following statements relating to the famous (a) Pherozeshah Mehta (b) Mary Carpenter
Muzaffarpur murders (1908) [2009-II] (c) Dadabhai Naoroji (d) Anand Mohan Bose
1. The bomb, which was hurled at their carriage of Mrs 69. Which one of the following commissions/ committees was
Pringle and her daughter was actually intended for Mr appointed by the British Government to investigate into
Kingsford, the District Judge of Muzaffarpur. the massacre in Jallianwala Bagh? [2009-II]
2. The revolutionaries wanted to kill Mr Kingsford, (a) Welby Commission (b) Hunter Committee
because he had inflicted severe punishments on (c) Simon Commission (d) Butler Committee
Swadeshi activists. 70. Which one of the following statements related to the Boston
3. Khudiram and Prafulla Chaki had to pay the penalty Tea Party on December 16, 1773 during the American War
for their action by death.
of Independence is correct? [2009-II]
Which of the statements given above is/are correct?
(a) Only 1 (b) Only 2 (a) The revolutionaries stealthily entered into the ships
(c) 2 and 3 (d) All of these and threw all the chests of tea into the water
History C-57
(b) The revolutionaries hosted a Tea Party in the honour 78. Which of the following statements regarding Mahatma
of Charles Townshend, the British Chancellor of the Gandhi’s philosophy of Satyagraha is/are correct?
Exchequer in order to place their grievances before [2010-I]
him 1. Truth and non-violence are its two vital ingredients.
(c) It marked a celebration when Lord North, the successor 2. The follower of Satyagraha would resist evil but not
of Townshend, repealed some of the duties imposed hate the evil doer.
by Townshend
3. The Satyagrahi would, if necessary, inflict suffering
(d) It was a protest against the Quebec Act
71. Which of the following was/were the main feature (s) of the on himself, and also the evil doer.
Government of India Act, 1919? [2009-II] Select the correct answer using the codes given below
1. Introduction of separate electorates for Muslims. (a) 1, 2 and 3 (b) 2 and 3
2. Devolution of legislative authority by the centre to (c) Only 1 (d) 1 and 2
the provinces. 79. Which one among the following prompted Rabindranath
3. Expansion and reconstitution of central and provincial Tagore to surrender his title of ‘Sir’? [2010-I]
legislatures. (a) The passing of the Rowlatt Act
Select the correct answer using the codes given below (b) The passing of the Act of 1919
(a) Only 2 (b) 1 and 3 (c) To support Mahatma Gandhi’s Satyagraha Movement
(c) Only 3 (d) 2 and 3 (d) To protest against the massacre at Jallianwala Bagh
72. ‘Dyarchy’ was first introduced in India under [2009-II] and the imposition of martial law in Punjab
(a) Morley-Minto reforms
80. Consider the following statements relating to Indigo Revolt
(b) Montford reforms
(c) Simon Commision plan [2010-I]
(d) Government of India Act, 1935 1. The Indigo Revolt is particularly memorable for the
73. When Lord Mountbatten became the first Governor-General role played by the intelligentsia to uphold the cause
of India, who among the following became the Governor- of the movement.
General for Pakistan? [2009-II] 2. After the revolt, the colonial authorities set-up the
(a) Lord Mountbatten (b) Muhammad Ali Jinnah Indigo Commission to enquire into the grievances of
(c) Liaquat Ali Khan (d) Shaukat Ali the Indigo cultivators.
74. Consider the following paragraph Which of the statements given above is/are correct?
He was seriously injured in police lathi charge in Lahore (a) Only 1 (b) Only 2
during demonstrations against Simon Commission, for (c) Both 1 and 2 (d) Neither 1 nor 2
which he subsequently died in November, 1928. Later on, 81. Prior to 1813, which among the following measures, was
the British officer who was responsible for the lathi charge
not adopted by the British to exploit the Indians
on him, was shot dead by Bhagat Singh and Rajguru.
The revolutionary referred to in the above paragraph is economically ? [2010-I]
[2009-II] (a) Monopolising the trade of raw goods wherever
(a) Pandit Govind Ballabh Pant possible and selling them at high rates
(b) Lala Lajpat Rai (b) Forcing Indiancraftsman to produce quality products
(c) Mangal Singh in fixed quantity and at fixed price
(d) Motilal Nehru (c) Free Trade Policy
75. Which among the following are the five principles of (d) Elimination of Indian traders from competition by every
peaceful co-existence of Panchsheel as outlined by means
Jawaharlal Nehru? [2010-I] 82. Which one of the following statements regarding the Revolt
(a) Mutual respect, military collaboration, economic of 1857 is not correct? [2010-I]
collaboration, de-colonisation and territorial integrity (a) The revolt was supported whole heartedly by the
(b) Mutual respect for each other territorial integrity and
educated, elite Indian everywhere
sovereignty, non-aggression, non-interference,
equality and mutual benefit and peaceful co-existence (b) The diverse elements which took part in the revolt
(c) Peaceful co-existence, economic collaboration cultural were united by their hatred of British rule
interaction, territorial integrity and decolonisation (c) The role of Hindu-Muslim unity in the revolt was
(d) Disarmament, peaceful co-existence, economic acknowledged by many, including British officials
collaboration, cultural interaction and territorial (d) The revolt did not spread to South India
integrity 83. Consider the following statements aboul Vijayanagara
76. The Russian revolutionaries derived their ideology from empire [2010-I]
the doctrines of [2010-I] 1. Vijayanagara was noted for its markets dealing in
(a) Lenin and Stalin (b) Marx and Lenin spices, textiles and precious stones.
(c) Marx and Engels (d) Lenin and Engels 2. Krishnadeva Raya’s rule was characterised by the
77. Which one among the following sums up Marx’s view about strain within the imperial structure,
history? [2010-I]
(a) History is a record of the wars between various people 3. The Amara-Nayakas were military commanders who
(b) History is a succession of struggle between the were given territories to govern by the Royas.
oppressor .and the oppressed classes Which ofthe statements given above is/are correct?
(c) History is a faithful record of the past events (a) Only 3 (b) 1 and 2
(d) None of the above (c) 1 and 3 (d) All of these
EBD_7367
58
C- History

84. Consider the following statements about the famous Which of the statements give above is/are correct?
traveller, Ibn Batuta [2010-I] (a) Only 1 (b) 2 and 3
1. He was a Moroccan traveller. (c) 1 and 3 (d) All of these
2. He narrated his experiences while travelling the sub 91. Which of the following statements relating to the Non-
continent in the 14th century in Kitab-ul-Hind. Alignment Movement is/are not correct? [2010-I]
3. He was sent as the envoy of Sultan of Delhi to China. 1. Non-alignment came to symbolise the struggle of India
Which of the statements given above is/are correct? and other newly independent nations to retain and
(a) 1, 2 and 3 (b) 1 and 3 strengthen their independence from colonialism and
(c) Only 3 (d) 1 and 2 imperialism.
85. Which one of the following was not a result of British 2. Non-alignment advanced the process of democrati-
colonial rule in India? [2010-I] sation of international relations.
(a) Ruin of Indian agriculture 3. Military alliances formed a major part of non-alignment.
(b) Ruin of Indian industries Select the correct answer using the codes given below
(c) Ruin of Indian trade (a) 1 and 2 (b) 2 and 3
(d) Ruin of Indian feudalism (c) Only 3 (d) Only 1
86. Consider the following events in the history of British India 92. To which among the following the residuary powers were
[2010-I] conferred by the federation established by the act of 1935?
1. Santhal Rebellion [2010-I]
2. Indigo Revolt (a) Federal legislature
3. Sanyasi and Fakir Rebellion (b) Provincial legislatures
4. Birsa Munda Rebellion (c) Governor-General
Which one of the following is a correct chronological (d) Provincial Governors
sequence of the above events starting with the earliest? 93. Which one among the following was not a reform measure
(a) 3, 2, 1, 4 (b) 1, 4, 2, 3 carried out by Mahmud Gawan of Bahmani kingdom?
(c) 3, 1, 2, 4 (d) 2, 1, 4, 3 [2010-II]
87. Statement I : Bal Gangadhar Tilak sought to diffuse the (a) The kingdom was divided into eight provnices or
spirit of nationalism among the masses. Tarafs
Statement II : Tilak organised a corps of volunteers to help
(b) Nobles were paid salaries and were asked to maintain
the famine stricken people in the Deccan. [2010-I]
contingents of horses
Codes :
(c) A tract of land, Khalisa, was set apart for the expenses
(a) Both the statements are true and Statement II is the
of the Tarafdar
correct explanation of Statement I
(b) Both the statements are true, but Statement II is not (d) Lands were measured and land taxes were fixed on
the correct explanation of Statement I that basis
(c) Satement I is true, but Statement II is false 94. Consider the following statement and identify the author of
(d) Statement I is false, but Statement II is true the statement using the codes given below [2010-II]
88. Consider the following statements about the Non- I shall work for an India in which the poorest shall feel that
Cooperation Movement [2010-I] it is their country, in whose making they have an effective
1. The movement was a mixture of nationalism, middle voice, an India in which there shall be no high class and low
class politics, religion, feudalism, agrarian discontent class of people, an India in which all communities shall live
and working class agitation. in perfect harmony.
2. The movement was much greater in intensity than any (a) Baba Saheb Ambedkar (b) Mahatma Gandhi
other political agitation which came before it. (c) Gurudev Tagore (d) Pandit Nehru
3. The movement helped to foster Hindu-Muslim unity. 95. Which one among the following statements regarding the
Which of the statements given above is/are correct? social and religious reform ideas of Raja Rammohan Roy is
(a) Only 2 (b) 2 and 3 not correct? [2010-II]
(c) Only 3 (d) 1 and 3 (a) His efforts led to the formation of Brahmo Samaj in
89. Which of the following statements above Ryotwari 1828
settlement is/are correct? [2010-I] (b) He considered different religions as embodiments of
1. It recognised the cultivators as the owner of land. universal theism
2. It was a temporary settlement. (c) His Vedantic monism was strengthened after 1815
3. It was introduced later than the permanent settlement. since an exposure to Christian unitarianism
Select the correct answer using the codes given below (d) He paid attention exclusively to the problems/issues
(a) 1 and 2 (b) 2 and 3 of the emerging middle class of India
(c) Only 1 (d) All of these 96. Which one among the following was part of Gandhi-lrwin
90. Consider the following statements about Dr Ram Manohar Agreement of 1931? [2010-II]
Lohia [2010-I] (a) Stern action against policemen guilty of brutal assault
1. He believed the Satyagraha without constructive work on Satyagrahis
is like a sentence without a verb. (b) Nehru to represent Congress at the Round Table
2. He wrote his PhD thesis paper on the topic of Salt Conference of 1931
Satyagraha, focusing on Mahatma Gandhi’s socio- (c) Deletion of communal question from the agenda of
economic theory. the conference
3. He recognised that caste, more than class, was the (d) Release of political prisoners excepting those guilty
huge stumbling block to India’s progress. of arson and violence
History C-59
97. Match the following [2010-II] 103. Consider the following statement and identify with the help
List I List II of the codes given below, the person who made the
(Event) (Place/Person) statement
A. Tarmashirin Khan’s 1. Vijayanagara ‘It would be quite impossible for a few hundred British to
invasion of India administer against the active opposition of the whole of
B. Visit of Farnao 2. Zainul Abidin the politically minded fo the population. [2011-I]
Martinz Evangelho, (a) Jawaharlal Nehru in April, 1947 in an address to
the Portuguese factor Congressmen at Lucknow
C. Visit of the traveller 3. Champaner (b) Clement Attlee January, 1947 in a private letter
Nuniz addressed to Ernest Bevin
D. Network of canals 4. Muhammad Tughlaq
(c) Lord Mountbatten in December, 1946 in a note given
in Kashmir valley
Codes : to British Parliament
A B C D A B C D (d) Viceroy Wavell in January, 1946 in a letter to Secretary
(a) 2 1 3 4 (b) 2 3 1 4 of State
(c) 4 3 1 2 (d) 4 1 3 2 104. Consider the following passage and identify the three tribal
98. Which one of the following statements about the teachings principalities referred to there, in using the codes given
of Kabir is not correct? [2010-II] below
(a) He was not against pilgrimage and idol worship In the early history of the far South in India, three tribal
(b) He believed in universal love principalities are mentioned in Ashokan inscriptions of the
(c) He emphasised on one God and the spread of 3rd century BC and in Kharavela inscription of the 1st
devotionalism century BC. [2011-I]
(d) He did not consider it necessary to abandon the normal (a) Vakatakas, Cholas and Satvahanas
life of a householder (b) Cholas, Pandyas and Cheras
99. Which one among the following statements about Ashokan (c) Ikshvakus, Vakatakas and Pandyas
edicts is correct? [2010-II] (d) Pallavas, Cholas and Pandyas
(a) The Pillar edicts were located in all parts of the empire 105. Industrial Revolution in Europe mainly emerged due to
(b) The edicts give details of his personal concerns but 1. locating the production process in the countryside.
are silent on events of the empire 2. declining of the guilds because of non-farming
(c) The subject of inscribed matter on Rock edicts differs production coming under a single roof (the factory).
completely with that of the Pillar edicts 3. growing role of merchant capitalists in the production
(d) The Greek or Aramaic edicts are versions or process. [2011-I]
translations of the texts used in other edicts
Select the correct answer using the codes given below
100. Which one among the following pairs is correctly matched?
[2011-I] (a) 1, 2 and 3 (b) 2 and 3
(a) The Second Battle : Defeat of Jaichand of Kannauj (c) 1 and 3 (d) Only 2
of Tarain by Muhammad of Ghori 106. In addition to Macaulay’s Minutes on Education, another
(b) The First Battle of : Defeat of Sikander Lodhi by landmark draft is also attributed to him. Identify the draft
Panipat Babur from the following. [2011-I]
(c) The Battle of Chausa : Defeat of Humayun by Sher (a) Draft of Indian Penal Code
Shah (b) Draft of Indian Forest Policy
(d) The Battle of Khanwa : Defeat of Rana Pratap by (c) Draft of the Zamindari Abolition Act
Akbar (d) Draft of the Maritime Trade Policy
107. Which of the following statements regarding the American
DIRECTIONS (Qs. 101-102) : The following questions consist Revolution is/are correct? [2011-I]
of two statements, Statement I and Statement II. You are to
1. The American Revolution was a conflict between
examine these two statments carefully and select the answers to
these items using the codes given below [2011-I] British settlers and native Americans.
2. The Americans refused to pay taxes imposed by the
Codes British Parliament in which the Americans had no
(a) Both the statements are true and Statement II is the correct representation.
explanation of Statement I Select the correct answer using the codes given below
(b) Both the statements are true, but Statement II is not the (a) Only 1 (b) Only 2
correct explanation of Statment I
(c) 1 and 2 (d) Neither 1 nor 2
(c) Statement I is true, but Statement II is false
(d) Statement II is false, but Statement II is true 108. Which of the following statements about Fourth Anglo-
101. Statement I: In North-Western India, the Civil Disobedience Mysore War are correct? [2011-I]
Movement took a mass character under the leadership of 1. The Madras Council suggested a policy of rigorous
Khan Abdul Ghaffar Khan. and intense attack on Mysore.
Statement II : The Nehru Report (1928) had argued that the 2. Lord Wellesley tried to revive the triple alliance.
‘next immediate step’ for India must be dominion status. 3. Tipu sent emissaries to Arabia, Versailles, Mauritius
102. Statement I : Gandhiji failed to realise that the Khilafat was and Kabul enlisting support against the English.
an extra-territorial issue. 4. The war was of a very short duration though decisive.
Statement II : The cause of Khilafat was discredited by Select the correct answer using the codes given below
1923, as Mustafa Kamal Pasha set-up a secular republican (a) 2, 3 and 4 (b) 1, 3 and 4
Government in Turkey. (c) 2 and 4 (d) 1 and 3
EBD_7367
60
C- History

109. Which of the statements given below about Vivekananda 116. Which one among the following was not a possible reason
are correct? [2011-II] for the success of Nadir Shah’s military compaign in Delhi?
1. He believed that Vedanta was fully rational. [2011-II]
2. He criticised his contrymen for having lost touch with (a) Weak Mughal emperor
the outside world. (b) Lack of strong defence in the North-West frontier
3. He condemned the caste system. (c) Late preparation for the defence of Delhi
(d) Use of superior military technology by the invading
4. He considered the Veda to be infallible. army
Select the correct answer using the codes given below 117. What was the ‘privy purse’ in the context of the history of
(a) 1, 2 and 3 (b) 2, 3 and 4 modern India? [2011-II]
(c) 1, 3 and 4 (d) 1 and 2 (a) A purse given privately by one organisation to another
110. In 1856, Awadh would not have been annexed with the (b) A purse given by the Government of India to
British empire if the Nawab of Awadh had [2011-II] dignitaries for services rendered
(a) allied with the British (c) A grant given by the Government of India to the
(b) not refused to introduce reforms as suggested by the erstwhile Prince of India
British (d) A gift given by an erstwhile Prince of india to the
(c) fought against the British Government of India
(d) a natural heir 118. The Constituent Assembly of India was choosen on the
basis of the provincial elections of 1946. With the withdrawal
111. Which one among the following is correct about the of the Muslim League from the Constituent Assembly, it
Doctrine of Lapse? [2011-II] turned out that majority of the assembly members were also
(a) It did not allow the Indian rulers to adopt any heir members of the Congress. Under that circumstance, how
(b) It did not allow an adopted heir to rule a state after the was the Constituent Assembly given a broader social basis?
death of the ruler [2011-II]
(c) It made the annexation of Indian State compulsory (a) By nominating independent members from various
after the death of a ruler minority groups
(d) It made the annexation of Indian State compulsory if (b) By nominating independent members from various
the adoption of heir had not been approved by the caste and religious groups
British authorities (c) By nominating independent members of different
112. Which one among the following was the result of the First castes, religious groups and women and also by taking
Anglo-Maratha War of 1775–1982? [2011-II] in representatives of the princely states and asking
for written submission from the public at large
(a) The British won the war (d) By taking in representatives of the princely states and
(b) The Marathas won the war asking for written submission from the public at large
(c) There was no victory for either side 119. The first effort at drafting a Dominion Status Constitution
(d) It helped Haider Ali to gather strength because the for India was made in response to the [2011-II]
British and Marathas were engaged in a mutual war (a) Minto-Morley reforms
113. Which one among the following was a reason for which the (b) Montague-Chelmsford reforms
French could not succeed in India in the 18th century? (c) Simon Commission
[2011-II] (d) First Round Table Conference
(a) They sided with the weak Indian sides such as Chanda 120. The Iron Pillar near Qutub Minar draws attention of
Sahib and Muzaffar Jang scientists due to its [2011-II]
(b) Dupleix was called back at a crucial time (a) antiquity (b) glitter
(c) hardness (d) rustlessness
(c) They conspired against the Indian powers 121. Statement I Annie Besant worked together with the
(d) Their trading company was heavily dependent on the Congress and the Muslim League during the Home Rule
French Government Movement.
114. Which of the following was/were reason/reasons for the Statement II Annie Besant felt that this was necessary to
success of European trading companies in South India, get the support of the masses for the Home Rule Movement.
during the 17th century? [2011-II] [2011-II]
1. The presence of the Mughals in the South was not as Codes :
much as in the North. (a) Both the statements are true and Statement II is the
2. The Vijayanagara kingdom had been over-thrown in correct explanation of Statement I
the late 16th century. (b) Both the statements are true, but Statement II is not
3. The South had many small and weak states. Select the the correct explanation of Statment I
correct answer using the codes given below (c) Statement I is true, but Statement II is false
(d) Statement II is false, but Statement II is true
(a) 1, 2 and 3 (b) 1 and 2 122. Which one among the following is not a characteristic of
(c) 2 and 3 (d) Only 1 Rig-Vedic Aryans? [2011-II]
115. Which one among the following was not true about the (a) They were acquainted with horses, chariots and the
Kerala king, Marthanda Verma? [2011-II] use of bronze
(a) He ruled over Travancore (b) They were acquainted with the use of iron
(b) He subdued the feudatories (c) They were acquainted with the cow, which formed the
(c) He gave heavy bribes to the European officers to most important form of wealth
maintain peace (d) They were acquainted with the use of copper and the
(d) He organised a strong modern army modern ploughshare
History 61
C-

123. The earliest Buddhist literature which deal with the stories Which of the statements given above is/are correct?
of the various birth of Buddha are [2011-II] (a) Only 1 (b) Only 2
(a) Vinaya pitakas (b) Sutta pitakas (c) Both 1 and 2 (d) Neither 1 nor 2
(c) Abhidamma pitakas (d) Jatakas 130. Which of the statements given below about the Champaran
124. The Jainas believe the Jainism is the outcome of the Satyagraha is/are correct? [2011-II]
teachings of 24 tirthankaras. In the light of this statement, 1. It was related to Indigo plantations.
which one among the following is correct of Vardhamana
2. It started because the European planters oppressed
Mahavira? [2011-II]
(a) He was the first tirthankara and the founder of Jainism the Zamindars.
(b) He was the 23rd tirthankara, the first 22 tirthankaras Select the correct answer using the codes given below
being considered legendary (a) Only 1 (b) Only 2
(c) He was the last and 24th tirthankara, who was not (c) Both 1 and 2 (d) Neither 1 nor 2
considered as the founder of the new faith but as a 131. The Viceregal Lodge at Shimla is a well-known ancient
reformer of the existing religious sect monument. Which of the following statements about the
(d) He was not one of the 24 tirthankaras monument are correct? [2012-I]
125. Gandhiji led the Indian Nationalist Movement from the front 1. The Lodge was built by 17th Viceroy, Earl Dufferin.
and his leadership was motivated by a wider philosophy, 2. The present shape of the building was given by Earl
he nurtured throughout the course of the movement. Which of Marquis of Lansdowne.
one among the following was a continuous movement 3. It is famous for holding three meetings before
based on this philosophy and not a specific movement? Independence of India including the Cabinet Mission.
[2011-II] Select the correct answer using the codes given below
(a) Non-Cooperation Movement
(b) Swadeshi Movement (a) 1, 2 and 3 (b) 2 and 3
(c) Quit India Movement (c) 1 and 3 (d) 1 and 2
(d) Civil Disobedience Movement 132. Which of the following statements about Permanent
126. Which of the following statements regarding Permanent Settlement are correct? [2012-I]
Settlement is/are correct? [2011-II] 1. It conferred proprietary rights to peasants.
1. The Permanent Settlement was introduced in parts of 2. The Zamindars were recognised as the proprietors of
the Madras and Bombay Presidencies. land.
2. The Permanent Settlement created a new class of 3. The government permanently fixed the land revenue
landlords with hereditary rights on land. demand.
3. The landlords created by the Permanent Settlement 4. The Zamindars acted as the middlemen between the
could never be removed under any circumstance. peasants and the government.
Select the correct answer using the codes given below Select the correct answer using the codes given below
(a) Only 1 (b) Only 2 (a) 2, 3 and 4 (b) 3 and 4
(c) 2 and 3 (d) 1, 2 and 3
(c) 1 and 4 (d) 1 and 3
127. Which among the following was the most immediate factor
for the spread of Swadeshi and boycott of foreign goods 133. Which one among the following was not a demand of the
during the first decade of the last century? [2011-II] Prarthana Samaj? [2012-I]
(a) Curzon’s design to curtail the sphere of local self- (a) Women education
government (b) Widow remarriage
(b) Curzon’s attempt to control the universities (c) Raising the age of marriage for boys and girls
(c) Curzon’s partition of Bengal (d) Abolition of untouchability
(d) Curzon’s plan to curb the growing popularity of the 134. The Name of Ram Prasad Bismil is associated with
Indian National Congress [2012-I]
128. Which one among the following statements appropriately (a) Kanpur Conspiracy Case
defines the term ‘Drain Theory’ as propounded by Dadabhai (b) Alipore Conspiracy Case
Naoroji in his work ‘Poverty and Un–British Rule in India’? (c) Kakori Conspiracy Case
[2011-II] (d) Meerut Conspiracy Case
(a) That a part of India’s national wealth or total annual
135. Match the following [2012-I]
product was being exported to Britain for which India
got no material returns List I List II
(b) That the resources of India were being utilised in the (Battle) (Treaty)
interest of Britain A. The Third Carnatic 1. Treaty of Salbai
(c) That the British industrialists were being given a War
opportunity to invest in India under the protection of B. The Third Mysore 2. Treaty of Lahore
the imperial power War
(d) That the British goods were being imported to India C. The First Maratha War 3. Treaty of Paris
making the country poorer day by day D. The First Anglo 4. Treaty of Srirangapatam
129. Consider the following statements about the Ahmedabad Sikh War
Mill Strike of 1918 [2011-II] Codes :
1. It was related to a dispute between the workers and A B C D A B C D
the European mill owners regarding hours of work. (a) 2 1 4 3 (b) 2 4 1 3
2. Gandhiji advised the workers to go on strike. (c) 3 4 1 2 (d) 3 1 4 2
EBD_7367
62
C- History

136. Who among the following scholars were contemporary of 145. Who among the following Governor Generals formed the
Kanishka? [2012-I] Triple Alliance against Tipu Sultan? [2012-I]
1. Ashvaghosa 2. Nagarjuna (a) Warren Hastings (b) Lord Cornwallis
3. Vasumitra 4. Chanakya (c) Lord Wellesley (d) Lord William Bentinck
Select the correct answer using the codes given below 146. Which one among the following statements is correct?
(a) 1 and 2 (b) 3 and 4 [2012-I]
(c) 2 and 4 (d) 1, 2 and 3 (a) The Revolt of 1857 was not supported by the Nizam of
137. Ashokan inscriptions of Mansehra and Shahbazgadhi are Hyderabad
written in [2012-I] (b) Dinabandhu Mitra was the author of the book,
(a) Prakrit language, Kharoshthi script Unhappy India
(b) Prakrit language, Brahmi script (c) The Sindhias of Gwalior gave shelter to the Rani of
(c) Prakrit-Aramaic language, Brahmi script Jhansi
(d) Aramaic language, Kharoshthi script (d) Mangal Pandey led the Sepoys March to Delhi
138. Which one among the following statements regarding Jorwe 147. Which one among the following principles was not
Culture is not correct? [2012-I] propagated by the Theosophical Society? [2012-I]
(a) Pravara-Godavari Valley was the nuclear zone of Jorwe (a) Belief in Karma and Rebirth
Culture (b) Belief in Universal Brotherhood and Humanity
(b) The main sites of Jorwe Culture are Daimabad, (c) Belief in Vedantic Philosophy
Inamgaon, Jorwe and Nevasa (d) Belief in the Eradication of Untouchability
(c) At Daimabad, one gets archaeological evidence of 148. Which one among the following statements regarding Bal
symbolic burial Gangadhar Tilak is not correct? [2012-I]
(d) Practically all Jorwe settlement in the Northern Deccan (a) He propounded the theory of the Arctic Home of the
were suddenly deserted Aryans
139. Match the following [2012-I] (b) He founded the Anti-Cow Killing Society
List 1 List II (c) He set-up the Home Rule League at Poona
A. lyothere Thass 1. Satyashodhak Samaj
(d) He supported the Age of Consent Bill
B. Jyotiba Phule 2. Dravida Kazhagam
149. The site of Harappa is located on the bank of river
C. John Rathinam 3. Self Respect Movement
[2012-I]
D. E V Ramaswami 4. Dravida Mahajana Sabha
(a) Saraswati (b) Indus
Naickar
(c) Beas (d) Ravi
Codes :
150. The suppression of Indian language newspapers under the
A B C D A B C D
(a) 4 2 1 3 (b) 3 1 2 4 Vernacular Press Act of 1878 was caused by the criticism of
(c) 4 1 2 3 (d) 3 2 1 4 [2012-II]
140. Identify the correct sequence of the following events of (a) lavish lifestyle of the English officials
Indian history (starting with the earliest) [2012-I] (b) ill-treatment given to Indigo workers by their English
1. The Doctrine of Lapse 2. The Subsidiary Alliance masters
3. The Treaty of Lahore 4. T Pitt’s India Act (c) inhuman approach of English officials towards the
Select the correct answer using the codes given below victims of the famine of 1876-77
(a) 4, 2, 3, 1 (b) 1, 2, 3, 4 (d) misuse of religious places of India by English officials
(c) 2, 1, 4, 3 (d) 3, 2, 1, 4 151. Dalhousie has been regarded as the maker of modern India
141. Which one among the following states was first annexed because he brought about reforms and made a beginning in
by Lord Dalhousie under the Doctrine of Lapse? many fields. Which one among the following was not one
[2012-I] of his schemes of reforms? [2012-II]
(a) Nagpur (b) Jhansi (a) Educational reforms
(c) Sambalpur (d) Satara (b) Construction of railways and introduction of telegraph
142. Which one among the following Indus cities was known and postal services
for water management? [2012-I] (c) Establishment of a public works department
(a) Lothal (b) Mohenjodaro (d) Factories Act to improve the condition of Indian labour
(c) Harappa (d) Dholavira 152. Consider the following statements about the European
143. Who among the following Chinese travellers visited the travellers to India [2012-II]
Kingdoms of Harsrhavardhana and Kumar Bhaskar Varma? 1. Sir Thomas Roe, the Representative of the East India
[2012-I] Company, was granted the permission by Jahangir to
(a) I–Tsing (b) Fa–Hien open a factory at Surat.
(c) Hiuen Tsang (d) Sun Shuyun 2. Captain Hawkins was driven out from Agra by the
144. The Cabinet Mission Plan for India envisaged a Mughals at the instigation of Portuguese.
[2012-I] 3. Father Monserrate travelled with Akbar on his journey
(a) Federation to Kashmir.
(b) Confederation Which of the statement(s) given above is/are correct?
(c) Unitary form of Government (a) 2 and 3 (b) Only 2
(d) Union of States (c) 1 and 2 (d) 1 and 3
History C-63
153. Industrial Revolution, in its early phase from C 1760 to C. 161. Which one among the following books was authored by a
1850, was largely dependent for its success on lady of the Mughal Royal House? [2012-II]
[2012-II] (a) Akbar Nama (b) Babur Nama
(a) machine power that replaced the labour on a large scale (c) Humayun Nama (d) Badshah Nama
(b) elaborate road network for transportation of industrial
162. Which one among the following statements is not true of
products
(c) network of electric power stations the jajmani system? [2012-II]
(d) organisation of labour under factory system (a) It was a non-market exchange system
154. Which one among the following pairs is not properly (b) It was practised in many villages and regions during
matched? [2012-II] the pre-colonial period
(a) Megasthenes : Indica (c) It was introduced under pressure from the colonial
(b) Ashvaghasa : Bddha charita regime
(c) Panini : Mahabhashya (d) It was incorporated into wider networks of exchange
(d) Vishakhadatta : Mudrarakshasa
through which agricultural products and other goods
155. The Dhamma propagated by Ashoka was [2012-II]
(a) the tenets of Buddhism circulated
(b) a mixture of the philosophies of Ajivikas and 163. Which one among the following depicts the correct meaning
Charvakas of the term Jins-i-Kamil concerning crops in Mughal India?
(c) a system of morals consistent with the tenets of most [2012-II]
of the sects of the time (a) Paddy crop (b) Cash crop
(d) the religious policy of the state (c) Coarse crop (d) Crop grown, in the arid
156. What was the Kutagarashala literally, a hut with a pointed 164. Consider the following statements about Sufism
roof ? [2012-II]
[2012-II]
(a) A place where animals were kept
(b) A place where intellectual debates among Buddhist 1. The Sufism were critical of the dogmatic definitions
mendicants took place and scholastic methods of interpreting the Quran and
(c) A place where weapons were stored Sunna (traditions of the prophet) adopted by the
(d) A place to sleep theologianc.
157. Which among the following materials were used for minting 2. The Sufis sought an interpretation of the Quran on
coins during the rule of the Mauryas? [2012-II] the basis of their personal experience.
(a) Gold and Silver (b) Silver and Copper Which of the statement(s) given above is/are correct?
(c) Copper and Bronze (d) Gold and Copper (a) Only 1 (b) Only 2
158. Which one among the following was the immediate cause
of attack by Nawab Siraj-ud-Daula on Calcutta in 1756? (c) Both 1 and 2 (d) Neither 1 nor 2
[2012-II] 165. Which among the following statements regarding the Gupta
(a) Refusal of the English Company to pay the overdue Dynasty is/are correct? [2012-II]
trade tax 1. The Kumaramatyas were the most important of the
(b) The English conspired against the Nawab with a view and they were appointed directly by the king in the
to depose him from the Throne home provinces.
(c) Siraj-ud-Daula wanted to drive out the English from Bengal 2. The village headmen lost importance and of the
(d) Refusal of the English to demolish the fortification of transactions began to be effected without their
Calcutta
159. Which one among the following correctly describes consent.
Gandhiji’s attitude towards the Swarajist leaders? Select the correct answer using the codes given below
[2012-II] (a) Only 1 (b) Only 2
(a) He was not opposed to their entry into council (c) Both 1 and 2 (d) Neither 1 nor
(b) He had full trust in their bona fides and considered 166. Which one among the following is not a function of Mir
them most valued and respected leaders Bakshi, the Head of the Military Department as well as of
(c) He was not in favour of maintaining warm personal the nobility under Mughal rule? [2012-II]
relations with them (a) He made recommendations for appointment to
(d) He was neutral to government’s offensive against the
Mansabs to the emperor
Swarajists and did not defend them
160. Silk routes are a good example of vibrant pre-modern trade (b) He collected reports of intelligence and information
and cultural links between distant parts of the world. Which agencies of the empire and presented them to the
one among the following is not true of silk routes? emperor at the court
[2012-II] (c) He was responsible for all incomes and expenditures
(a) Historians have identified several silk route over land and held control over Khalisa, Jagir and Inam lands
and sea (d) He was responsible for the security of foreign travellers
(b) Silk routes have linked Asia with Europe and Northern on the highways of the empire
Africa 167. The first Indian ruler to be shown in images as wearing a
(c) Silk routes existed before the Christian era and thrived
almost upto fifteenth century dress akin to trousers is [2012-II]
(d) As a result of silk route trade, precious metals like (a) Chandragupta (b) Ashoka
gold and silver, flowed from Asia to Europe (c) Kanishka (d) Samudragupta
EBD_7367
64
C- History

168. Consider the following statements about Shivaji’s military 177. Statement I: Sculptures of the Gandhara School stylistically are
acumen [2012-II] typically linked to the Greco-Roman and the Parthian art of Iran.
1. He was a master in guerrilla tactics and swift cavalry Statement II: The earliest stone Buddha images in the Swat
warfare. valley pre-dated the Kushana period, which suggests that
2. He had built a series of fortified strongholds on the certain iconographic conventions were already well-
table mountains of the Western Deccan. established in the pre-Kushana period.
Which of the statement(s) given above is/are correct? 178. Statement I: The Permanent Zamindari Settlement of land
(a) Only 1 (b) Only 2 created a new class of landlords.
(c) Both 1 and 2 (d) Neither 1 nor 2 Statement II: The new class of landlords became strong
169. Who among the following cultures were the first to paint political allies of the British and were interested in the
their pottery? [2012-II] continuance of British dominion.
(a) Mesolithic (b) Chalcolithic 179. Which of the following was/were not related to Buddha’s
(c) Neolithic (d) Iron age life? [2013-I]
170. The polity of the Harappan people, as derived from the 1. Kanthaka 2. Alara Kalama
material evidence, was [2012-II] 3. Channa 4. Goshala Maskariputra
(a) secular-federalist (b) theocratic-federalist Select the correct answer using the codes given below
(c) oligarchic (d) theocratic-unitary (a) Only 1 (b) Only 4
171. Several nationalist leaders in India wrote commentaries on (c) 1 and 2 (d) 3 and 4
the Bhagvad Geeta to argue the case for an ethical 180. What was Ziyarat in the language of the Sufis?
foundation to Indian nationalism, who among the following [2013-I]
is an exception to it? [2012-II] (a) Pilgrimage to the tombs of Sufi saints for seeking barkat
(a) Sri Aurobindo (b) Mahatma Gandhi (spiritual grace)
(c) Bal Gangadhar Tilak (d) Ram Manohar Lohia (b) Reciting divine name
172. Who among the following was not associated with the (c) Offering free kitchens run on futuh (unasked for
foundation of the All India Trade Union Congress? charity)
[2013-I] (d) Setting up of auqaf (charitable trusts)
(a) NM Joshi (b) Lajpat Rai 181. British colonialism in India saw the emergence of new cities.
(c) VVGiri (d) Joseph Baptista Calcutta, now Kolkata, was one of the first cities. Which of
173. Which chemical was an important symbol in our struggle the following villages were, amalgamated to form the city of
for freedom? [2013-I] Calcutta? [2013-I]
(a) Glucose (b) Fertilizer (a) Midnapur, Chittagong, Burdwan
(c) Medicine (d) Sodium chloride (b) 24-Parganas, Kalikata, Thakurgaon
(c) Sutanuti, Kalikata, Gobindapur
DIRECTIONS (Qs. 174-178): The following questions consist (d) Midnapur, Thakurgaon, Gobindapur
of two statements, StatementI and Statement 11. You are to
182. Several socio-political organisations were formed in the 19th
examine these two statements carefully and select the answers and 20th centuries in India, Anjuman-e-Khawatin-e-Islam
to these questions using the codes given below founded in the year 1914 was [2013-I]
Codes [2013-I] (a) All India Muslim Ladies Conference
(a) Both the statements are true and Statement II is the correct (b) A radical wing of the All India Muslim League
explanation of Statement I (c) All India Muslim Student’s Conference
(b) Both the statements are true, but Statement II is not the (d) All India Islamic Conference
correct explanation of Statement I 183. Ibn Batuta’s work, Rihla, completed in 1355, is [2013-I]
(c) Statement I is true, but Statement II is false (a) an autobiography
(d) Statement I is false, but Statement II is true (b) an account of the Delhi Sultans from Aibak to
174. Statement I: There was great exodus of Jaina monks under Ghiyasuddin Tughlaq
the leadership of Bhadrabahu to the Deccan following (c) a religious text
severe famine in the Ganga valley towards the end of (d) an account of trade with Morocco
Chandragupta’s reign. 184. Between 1309 and 1311, Malik Kafur led two campaigns in
Statement II: Chandragupta Maurya joined the Jaina order South India. The significance of the expeditions lies in it
as a monk. that [2013-I]
175. Statement I: Mahavira initially joined a group of ascetics 1. they reflected a high degree of boldness and spirit of
called Nirgranthas. adventure on the part of the Delhi rulers.
Statement II: The sect was founded 200 years earlier by 2. the invaders returned to Delhi with untold wealth.
Parsva. 3. they provided fresh geographical knowledge.
176. Statement I: Lomash Rishi and Sudama caves in the Barabar 4. Alauddin promoted Malik Kafur to the rank of Malik-
hills, modelled on wooden architectural prototypes are naib or Vice-Regent of the empire.
examples of the earliest cave architecture in India. Select the correct answer using the codes given below
Statement II: Barabar hills caves were dedicated by (a) 1 and 3 (b) 1, 2 and 4
Chandragupta Maurya to Ajivika monks. (c) 2 and 4 (d) All of the above
History C-65
185. The fortification of Calcutta by the British in 1756 was 193. Which of the following did not constitute part of the army
regarded by the Nawab of Bengal, Siraj-ud-Daulah, as reforms of Sher Shah? [2013-II]
(a) growth of large-scale British trade [2013-I] (a) Keeping a large army at the centre
(b) an attack upon his sovereignty (b) Introduction of a swiftly moving artillery
(c) insecurity of the British in India (c) Cash payment to soldiers
(d) Introduction of the practice of branding horses
(d) British control over Bengal
194. Rajtarangini, authored by Kalhan, describes the history of
186. The highly polished monolithic Ashokan Pillars were carved [2013-II]
out of single pieces of a buff-coloured sandstone, usually (a) Gujarat (b) Bengal
mined from the quarries of [2013-I] (c) Kashmir (d) Punjab
(a) Chunar near Mirzapur
(b) Lauriya in Nandangarh DIRECTIONS (Qs. 195-199) : The following questions consists
(c) Sarnath near Varanasi of two Statements, Statement I and Statement II. You are to
(d) Udayagiri near Bhubaneshwar examine these two statements carefully and select the answers
187. In the Gupta age, Varahamihira wrote the famous book, to these items using the codes given below Codes [2013-II]
‘Brihat Samhita’. It was a treatise on [2013-II] (a) Both the statemtns are true and Statement II is the correct
(a) astronomy explanation of Statement I
(b) statecraft (b) Both the statements are true, but Statement II is not the
(c) ayurvedic system of medicine. correct explanation of Statement I.
(d) economics (c) Statement I is true, but Statement II is false.
188. Which one among the following thinkers argued that (d) Statement I is false, but Statement II is true.
Maratha rule in general and Shivaji in particular represented 195. Statement I: Kali age reflects the presence of deep social
early nationalist cons-ciousness in India? crisis characterised by varnasankara i. e., intermixture of
(a) Pandita Ramabai [2013-II] varnas or social orders.
(b) MG Ranade Statement II: The Vaisyas and Sudras (peasants, artisans
(c) Bipin Chandra Pal and labourers) either refused to perform producing
(d) Gopal Krishna Gokhale functions or pay taxes or supply necessary labour for
189. Consider the following statements about the causes of economic production resulting in weakening of Brahminical
success of the American Revolution [2013-II] social order and social tension.
1. the remoteness of the American continent and British 196. Statement I: The social institutions of caste in India
ignorance of the American continent led to the success underwent major changes in the colonial period.
of the Americans. Statement II: Caste, in contemporary society is more a
2. the fierce spirit of liberty drove the Americans to product of ancient Indian tradition than of colonialism.
success. 197. Statement I: The Russian Revolution of 1917 inspired the
3. the American military forces were superior to the Indian Working Class Movement.
British. Statement II: The Non-Cooperation Movement (1921-22)
Which of the statements given above is/are correct? saw the involvement of the Indian Working Class.
(a) Only 1 (b) 1 and 2 198. Statement I: In India tribal movements of 19th century
(c) Only 2 (d) 1 and 3 resulted out of the process of land displacements and the
190. Consider the following introduction of forest laws.
‘India would be far more reliable as a base for operations. Statement II: The Indian national movement resolved the
Moreover, the prospect of a settlement will be greatly problems faced by the tribals.
enhanced by the disappearance of Gandhi, who had for 199. Statement I: The economy of India in the 19th century came
years torpedoed every attempt at a settlement.’ to a state of ruin under English East India Company.
The above statement was made by the British in the context Statement II: English East India Company’s acquisition of
of [2013-II] Diwani right led to the miseries of the peasants and those
(a) Kheda Satyagraha associated with the traditional handicrafts industry of India.
(b) Civil Disobedience Movement 200. Sangam literature formed a very important source for the
(c) Quit India Movement reconstruction of the history of South India. It was written
(d) Non-Cooperation Movement in [2013-II]
191. The Mansabdari system of the Mughals was a complex (a) Tamil (b) Kannada
system. Its efficient functioning depended upon (c) Telugu (d) Malayalam
[2013-II] 201. Which of the following statements is/are not correct about
1. the practice of offering the title of ‘Mansabdar’ to Bhakti tradition in South India? [2013-II]
military personnel only. 1. Earliest Bhakti Movements in India were led by Alvar
2. proper functioning of the dagh (branding) system. and Nayanar saints.
3. proper functioning of the jagirdari system. 2. Nalayira Divyaprabandham, frequently described as
Select the correct answer using the codes given below Tamil Veda is an anthology of compositions by the
(a) Only 1 (b) 1 and 3 Alvars.
(c) 2 and 3 (d) All of these 3. Karaikkal Ammaiyar, women Alvar saints, supported
192. Among the precious stones, the most extensive foreign patriarchal norms.
trade during the Gupta age was that of [2013-II] Select the correct answer using the codes given below
(a) diamonds (b) ruby (a) 1 and 2 (b) Only 3
(c) pearl (d) sapphire (c) Only 2 (d) All of these
EBD_7367
66
C- History

202. Which one among the following was the primary reason 212. Consider the following statements regarding Indian
behind the failure of the Young Bengal Movement in Bengal? Feudalism in the early medieval period [2014-I]
[2013-II] 1. The revenue assignments were called Bhoga.
(a) It did not appeal to educated people 2. The hereditary chiefs neither collected revenues nor
(b) Its economic programme was not popular assumed administrative powers.
(c) It was too radical Which of the statements given above is/are correct?
(d) It did not have good leaders (a) Only 1 (b) Only 2
203. Who among the following first used the word ‘Swarajya’ in (c) Both 1 and 2 (d) Neither 1 nor 2
its political sense and accepted Hindi as the national 213. Which one of the following statements about ancient Indian
language of India? [2013-II] Mahajanapadas is correct? [2014-I]
(a) All Mahajanapadas were oligarchies where power was
(a) Rammohan Roy (b) Swami Vivekananda
exercised by a group of people
(c) Mahatma Gandhi (d) Bal Gangadhar Tilak (b) All Mahajanapadas were located in Eastern India
204. Consider the following statements about Satyagraha (c) No army was maintained by the Mahajanapadas
1. It was first experimented by Gandhiji in South Africa. (d) Buddhist and Jaina texts list sixteen Mahajanapadas
2. It was first experimented in India at Champaran 214. Consider the following statements [2014-I]
[2013-II] 1. Battle of Buxar provided the key to the English to
Which of the statements given above is/are correct? establish their rule in India.
(a) Only 1 (b) Only 2 2. The Treaty of Allahabad, concluded in 1765, enabled
(c) Both 1 and 2 (d) Neither 1 nor 2 the British to establish their rule in Bengal.
205. Which one among the following events was not associated Which of the stattements given above is/are correct?
with French Revolution? [2013-II] (a) Only 1 (b) Only 2
(a) Calling of the Estates General (c) Both 1 and 2 (d) Neither 1 nor 2
(b) Guillotine 215. Which one of the following statements about Rig Veda is
(c) Battle of Concord not correct? [2014-I]
(d) Tennis Court Oath (a) Deities were worshipped through prayer and sacrificial
206. Which one among the following events was associated rituals
with American War of Independence? [2013-II] (b) The Gods are presented as powerful, who could be
(a) Tennis Court Oath (b) Boston Tea Party made to intervene in the world of men via the
(c) Fall of Bastille (d) Reign of Terror performance of sacrifices
207. Which one among the following statements about Civil (c) The Gods were supposed to partake of the offerings
Disobedience Movement is correct? [2013-II] as they were consumed by the fire
(a) It started with Gandhiji’s march to Champaran (d) The sacrifices were performed in the temples
(b) Under Gandhi–lrwin agreement Congress agreed to 216. The only inscribed stone portrait of Emperor Ashoka has
give up Civil Disobedience Movement been found at [2014-I]
(c) The British Government was quite soft towards the (a) Sanchi (b) Amaravati
movement from the beginning (c) Kanaganahalli (d) Ajanta
(d) There was no violence during the movement 217. The social ideals of Mahatma Gandhi were first put forth in
208. Consider the following statements [2014-I] [2014-I]
1. The Jains believed that Mahavira had twenty-three (a) Hind Swaraj
predecessors. (b) An Authobiography–The Story of My Experiments
2. Parshvanatha was twenty-third Tirthankara. with Truth
3. Rishava was immediate successor of Mahavira. Which (c) History of the Satyagraha in South Africa
of the statements given above is/are correct? (d) The Bhagavad Geeta According to Gandhi
(a) 1 and 2 (b) 2 and 3 218. Consider the following statements about Syed Ahmed Khan,
(c) Only 2 (d) Only 3 the founder of Muhammadan Anglo-Oriental College,
209. Consider the following statements about Sher Shah’s Aligarh [2014-I]
administration [2014-I] 1. He was a staunch supporter of Indian National
1. He divided his empire into Sarkars, which were further Congress.
subdivided into Parganas. 2. Muhammadan Anglo-Oriental College was set-up with
2. The Sarkars and the Parganas were directly administered the objective of promoting learning of Islamic
by Sher Shah without the help of any other officials. education among the Muslims.
Which of the statements given above is/are correct? Which of the statements given above is/are correct?
(a) Only 1 (b) Only 2 (a) Only 1 (b) Only 2
(c) Both 1 and 2 (d) Neither 1 nor 2 (c) Both 1 and 2 (d) Neither 1 nor 2
210. The University of Nalanda was set-up by which Gupta ruler? 219. Consider the following statements [2014-I]
[2014-I] 1. B G Tilak founded the Home Rule League in April 1916,
(a) Kumaragupta II (b) Kumaragupta I in Maharashtra.
(c) Chandragupta II (d) Samudragupta 2. N C Kelkar was not associated with Home Rule
211. The Fourth Buddhist Council was held in Kashmir under Movement.
the leadership of [2014-I] Which of the statements given above is/are correct?
(a) Bindusara (b) Ashoka (a) Only 1 (b) Only 2
(c) Kunal (d) Kanishka (c) Both 1 and 2 (d) Neither 1 nor 2
History C-67
220. Consider the following statements about the First Session Codes :
of the Indian National Congress [2014-I] A B C D A B C D
1. It was held in Bombay in 1885. (a) 2 4 1 3 (b) 3 1 4 2
2. Surendranath Banerji could not attend the session due (c) 3 4 1 2 (d) 2 1 4 3
to the simultaneous session of the Indian National 228. Consider the following statements from Kalhana’s
Conference. Rajatarangini
Which of the statements given above is/are correct? 1. The common people ate rice and Utpala-saka
(a) Only 1 (b) Only 2
(a wild.vegetable of.bitter taste).
(c) Both 1 and 2 (d) Neither 1 nor 2
221. The Society of Jesus, whose followers were called Jesuits, 2. Harsha introduced into Kashmir a general dress
was set-up by [2014-I] befitting a king which included a long coat.
(a) Martin Luther (b) Ulrich Zwingli Which of the statements given above is/are correct?
(c) Erasmus (d) Ignatius Loyola [2014-I]
222. Who among the following was not associated with the (a) Only 1 (b) Only 2
activities of the Theosophical Society? [2014-I] (c) Both1and 2 (d) Neither 1 nor 2
(a) Madame H P Blavatsky 229. Marco Polo's trip to India (1271 AD) earned much fame in
(b) Mr A O Hume Europe on account of [2014-II]
(c) Col H S Olcott (a) his having discovered a safe route to India
(d) Mrs Annie Besant (b) his having established amicable relations with many
223. Which among the following was the reason of the Kings of India
resignations of the Indian Ministers in all the provinces in (c) his account of commercial, religious and social
the year 1939? [2014-I] conditions in the East
(a) The Governors refused to act as constitutional heads (d) All of the above
(b) The Centre did not provide the required financial help 230. Which of the statements given below about the Mughal
to provinces
(c) The Governor-General converted Indian administration rule in India is false ? [2014-II]
from federal to unitary one because of the beginning (a) Peasant communities were a united and homogeneous
of the World War II group
(d) India was declared a party to the World War II without (b) There was an abundance of food grain
the consent of the provincial government (c) The State encouraged these crops that brought in
224. Match the following [2014-I] more revenue
List I List II (d) Most regions produced two crops in a year
(Temple) (Town) 231. The Ghadar party, formed in the USA, was determined to
A. Kailasanathar 1. Bhuvneshwar start a revolt in India. Which among the following provinces
B. Lingaraj 2. Khajuraho did the party choose to begin its armed revolt ? [2014-II]
C. Kandariya Mahadey 3. Mount Abu (a) Punjab (b) Bengal
D. Dilwara 4. Kanchipuram (c) United Provinces (d) Bihar
Codes: 232. The ruins of the Vijayanagara at Hampi were brought to
A B C D A B C D light in 1800 by [2014-II]
(a) 4 2 1 3 (b) 4 1 2 3 (a) Colonel Colin Mackenzie
(c) 3 1 2 4 (d) 3 2 1 4
(b) Sir John Shore
225. Consider the following statements concerning the initial
phase of the Industrial Revolution in England. [2014-I] (c) Andrew Fraser
1. England was fortunate in that coal and iron ore were (d) John Marshall
plentifully available to be used in industry. 233. Consider the following statements about colonial economy
2. Until the 18th century, there was a scarcity of usable of Vietnam (Indo-China) : [2014-II]
iron. 1. The colonial economy in Vietnam was primarily based
Which of the statements given above is/are correct? on rice cultivation and rubber plantations.
(a) Only 1 (b) Only 2 2. All the rubber plantations in Vietnam were owned and
(c) Both 1 and 2 (d) Neither 1 nor 2 controlled by a small Vietnamese elite.
226. Which one of the following was a temple built by the Chola 3. Indentured Vietnamese labour was widely used in the
Kings? [2014-I] rubber plantations.
(a) Brihadiswara Temple, Tanjavur 4. Indentured labourers worked on the basis of contracts
(b) Meenakshi Temple, Madurai that did not specify any rights of labourers but gave
(c) Srirangam Temple, Thiruchirapalli immense power to the employers.
(d) Durga Temple, Aihole Which of the statements given above is/are correct?
227. Match the following
(a) 1, 3 and 4 (b) 1 and 4 only
List I List II
(Text) (Author) (c) 2 and 3 (d) 1 only
A. Kitab-al-Hind 1. Ibn Battuta 234. Which of the four linguistic regions in South India remained
B. Rehla 2. Al–Biruni unaffected by the Non-Cooperation Movement (1921-22) ?
C. Humayun Nama 3. Lohori [2014-II]
D. Badshah Nama 4. Gulbadan Begum (a) Kerala (b) Tamil Nadu
[2014-I] (c) Andhra Pradesh (d) Karnataka
EBD_7367
68
C- History

235. Which of the following statements about the penetration 241. Statement I: The annexation of Awadh by Lord Dalhousie
of English into Bengal is/are correct? [2014-II] in 1856 adversely affected the financial conditions of the
1. Job Charnock arrived in Sutanati in August 1690 and sepoys.
laid the foundation of Calcutta which later became Statement II: The sepoys had to pay higher taxes on the
the heart of the British Indian Empire. land where their family members stayed in Awadh.
2. The French East India Company built a fort near the 242. Which of the following characteristic(s) describes the nature
Fort William in Calcutta. of religion according to the Rig Veda? [2015-I]
Select the correct answer using the code given below: 1. Rig Vedic religion can be described a naturalistic
(a) 1 only (b) 2 only polytheism.
(c) Both 1 and 2 (d) Neither 1 nor 2 2. There are striking similarities between the Rig Vedic
236. Under the forceful thrust of British rule, a rapid religion and the ideas in the Iranian Avesta.
transformation of the Indian economy took place. In this 3. Vedic sacrifices were conducted in the house of the
context, which of the following statements is/ are correct? priest who was called yajaman.
[2014-II] 4. Vedic sacrifices were of two kind – those conducted
1. Indian economy was transformed into a colonial by the household and those that required rituals
economy in the 19th century whose structure was specialists.
determined by Britain's fast developing industrial Select the correct answer using the code give below :
economy. (a) 3 only (b) 1 and 2 only
2. The influx of cheap Indian products into England gave (c) 1, 2, 3 and 4 (d) 1, 2 and 4 only
a great blow to English textile industries. 243. Which of the following statement(s) about caste movements
3. The 19th century saw the collapse of the traditional in early 20th century Kerala is/are false ? [2015-I]
Indian village economy and fresh economic alignment 1. Kerala's first modern novel Indulekh attacked the
along commercial lines. social dominance of Nambudiri Brahmans in Kerala.
Select the correct answer using the code given below : 2. C.V. Raman Pillai's novel Marthanda Varma was
(a) 1 and 3 (b) 1 only written against the exploitation of peasants by Nair
(c) 2 only (d) l and 2 landlords.
237. Which of the following statements is/ are correct? 3. Sree Narayana Guru was one of the founders of the
[2014-II] Sree Narayana Dharma Paripalana Yogam that was
1. Early Buddhist literature is generally composed of the engaged in the upliftment of the Ezhavas in Kerala.
canonical text. 4. Dr. Palpu, the first Ezhava graduals was one of the
2. The Buddhist schools classify their canonical founders of the S Narayana Dharma Paripalana Yog
literature as only the Pitakas. that was engaged in the upliftment the Ezhavas in
Select the correct answer using. the code given below: Kerala.
(a) 1 only (b) 2 only Select the correct answer using the code given below :
(c) Both 1 and 2 (d) Neither 1 nor 2 (a) 1 only (b) 1 and 3
DIRECTIONS (Qs. 238-241) : The following questions consist (c) 2 only (d) 2 and 4
of two statements, Statement I and Statement II. You are to 244. Which of the following statements about Mahatma Gandhi's
examine these two statements carefully and select the answers South African experience (1893 - 1914) is/are true ?
to these items using the code given below : [2014-II] [2015-I]
1. Muslim merchants were actively involved in Gandhian
Codes :
(a) Both the statements are individually true and Statement II political movements in South Africa.
is the correct explanation of Statement I 2. In 1906, Gandhi led a campaign in Cape Town against
(b) Both the statements are individually true but Statement II is the ordinance of compulsory registration and passes
not the correct explanation of Statement I for Indians.
(c) Statement I is true but Statement II is false 3. Gandhi began his political career with struggles
(d) Statement I is false but Statement II is true against the imposition excessive taxes on Indians in
238. Statement I: The twelfth century witnessed the emergence Cape Town.
of a new movement in Karnataka led by a Brah mana named Select the correct answer using the code give below :
Basavanna. (a) 1 and 2 only (b) 3 only
Statement II: The Lingayats worshipped Shiva in his (c) 1, 2 and 3 (d) 1 only
manifestation as a Linga. 245. Which of the following sets of newspapers reflected the
239. Statement II: Ram Mohan Roy in his famous work Gift to concerns of educated Indian Muslims during the Khilafat
Monotheism put forward weighty arguments against belief Movement ? [2015-I]
in many Gods and for the worship of a single God. (a) Comrade and Hamdard
Statement II: Ram Mohan Roy in his Precepts of Jesus (b) Comrade and Hindustan Times
tried to separate the moral and philosophic message of the (c) Zamindar and Muslim Voice
New Testament. (d) Comrade, Hamdard, Zamindar and Al Hilal
240. Statement I: The Bethune School, founded in Calcutta in 246. Which of the following statements about the social reformer,
1849 was the first fruit of the powerful movement for women's Raja Rammohun Roy, is false ? [2015-I]
education that arose in the 1840s, and 1850s. (a) Rammohun Roy belonged to the gentry class whose
Statement II: The first step in giving modern education to power had been diminished because of the imposition
girls was undertaken by Vidyasagar in 1800. of the Permanent Settlement.
History C-69
(b) He studied both Vedantic Monism and Christian Select the correct answer using the code given below :
Unitarianism. (a) 1 only (b) 1, 2 and 3
(c) He translated the Upanishads into Bengali. (c) 2 only (d) 2 and 3 only
(d) His first organization was the Atmiya Sabha, founded 252. Which of the following statements about the musical culture
in Calcutta in 1815. in 18th and 19th century South India is/are correct ?
247. Which of the following characteristics about the state of [2015-I]
Travancore in 18th century Kerala is/are correct ? 1. Musical developments were spearheaded by the Arcot
[2015-I] court.
1. Travancore was ruled by Marthanda Varma from 1729 2. Tanjavur replaced Madras as the cultural capital of
to 1758. classical music in the second half of the nineteenth
2. Travancore built a strong army and defeated the Dutch century.
in 1741. 3. Three great composers, Tyagaraja, Diksitar and Syama
3. Travancore was an important centre of learning. Sastri experimented with the kriti form to set the
Select the correct answer using the code given below : foundations for modern Carnatic music.
(a) 1 and 2 only (b) 2 only Select the correct answer using the code given below :
(c) 1, 2 and 3 (d) 1 only (a) 1 and 3 only (b) 2 only
248. Which of the following is not a central tenet of Socialism ? (c) 3 only (d) 1, 2 and 3
[2015-I] 253. Which of the following statements about visual culture in
(a) Historical Materialism 18th and early 19th century North India is/are correct ?
(b) Dialectical Materialism [2015-I]
(c) Alienation and Class Struggle 1. Painters from Patna and Murshidabad flocked to
(d) Individual Freedom Calcutta and produced water colours in the English
249. Which of the following is/are the characteristic(s) of the mode.
Sannyasi and Fakir uprisings ? [2015-I] 2. Landscape and portraiture became extremely important
1. These uprisings refer to a series of skirmishes between at this time.
the English East India Company and a group of 3. The artists Zayan-al-Din, Bhawani Das and Ram Das
sannyasis and fakirs. were adopted by the English East India Company to
2. One reason for the uprising was the ban on free produce albums of Indian birds and animals.
movement of the sannyasis along pilgrimage routes. 4. While Indians were influenced by European artistic
3. In the course of the uprisings in 1773, Warren Hastings styles, European artists did not visit regional courts.
issued a proclamation banishing all sannyasis from Select the correct answer using the code given below :
Bengal and Bihar. (a) 1, 2 and 3 (b) 2 and 3 only
4. Are contemporaneous with the Non-Cooperation (c) 1, 3 and 4 (d) 4 only
Movement. 254. Match List I with List II and select the correct answer using
Select the correct answer using the code given below : the code given below the Lists : [2015-I]
(a) 1 only (b) 1 and 3 only List I List II
(c) 1, 2 and 3 (d) 2 and 4 only (Editor) (Journal / Newspaper)
250. Which of the following features of the State of Arcot in 18th A. S.A. Dange 1. Labour-Kisan Gazette
century South India are correct ? [2015-I] B. Muzaffar Ahmed 2. Inquilab
1. The founders of the dynasty that ruled Arcot were C. Ghulam Hussain 3. Navayug
Daud Khan Panni and Sa'adatullah Khan. D. M. Singaravelu 4. The Socialist
2. Arcot became the site of a protracted struggle between Code :
the English and Dutch East India Companies from the (a) A-4; B-3; C-2; D-1 (b) A-4; B-2; C-3; D-1
1740s. (c) A-1; B-2; C-3; D-4 (d) A-1; B-3; C-2; D-4
3. Decentralization was a key feature of the State of Arcot 255. Which of the following was/were connected primarily to
in the 18th century. the communist ideology ? [2015-I]
4. The other major State to emerge in South India at this 1. Kirti Kisan Party
time was Mysore. 2. Labour Swaraj Party
Select the correct answer using the code given below : Select the correct answer using the code given below:
(a) 1 and 2 only (b) 1, 2 and 4 (a) 1 only (b) 2 only
(c) 3 and 4 only (d) 2 and 4 only (c) Both 1 and 2 (d) Neither 1 nor 2
256. The interest of the British Government of India in
251. Which of the following statements about the musical culture
Afghanistan in the nineteenth century came about in order
in 18th and 19th century North India is/are not correct:
to [2015-I]
[2015-I]
(a) make use of the natural resources of Afghanistan.
1. The period was marked by the growing eminence of
(b) ensure that the Russian empire did not have an
Sadarang Neamat Khan who introduced the khayal
influence over Afghanistan.
form.
(c) increase the reach of the British Empire.
2. A large number of musicians moved out of regional
(d) establish a monopoly over the markets of Afghanistan.
centres to Delhi where they hoped they would receive
257. The Tungbhadra river provided sustenance of which
more employment and patronage. empire? [2015-I]
3. The period was marked by the formation of specific
(a) Chola (b) Vijayanagara
region based gharanas.
(c) Vakataka (d) Pandya
EBD_7367
70
C- History

258. Which empire did Niccolo de Conti, Abdur Razzaq, Afanasy 263. The Rihla was written in [2015-II]
Nikitin and Fernao Nuniz visit ? [2015-I] (a) Arabic in the Fourteenth Century by Ibn Battuta
(a) The empire of Kannauj (b) Vijayanagara empire (b) Persian in the Fifteenth Century by Abdur Razzaq
(c) Hoysala empire (d) Rashtrakuta empire
(c) Persian in the Thirteenth Century by Ibn Battuta
259. Match List-I with List-II and select the correct answer (d) Italian in the Thirteenth Century by Marco Polo
using the code given below the Lists : [2015-II] 264. The 'Sur Sagar' is [2015-II]
List-I List-II (a) a medieval treatise on music composed by Tansen
(Steel mill) (State) (b) a poetic work of Surdas
A. Kalinganagar 1. West Bengal
B. Vijayanagar 2. Tamil Nadu (c) memories of Bahadur Shah Zafar
C. Salem 3. Odisha (d) None of the above
D. Durgapur 4. Karnataka 265. Who among the following was not a painter at Akbar's
Code : Court? [2015-II]
A B C D (a) Daswanth (b) Abdus Samad
(a) 1 4 2 3
(c) Kalyan Das (d) Basawan
(b) 1 2 4 3
(c) 3 4 2 1 266. After the death of Shivaji, there was a fight for succession
(d) 3 2 4 1 between [2015-II]
260. Consider the following statements about Alauddin Khalji's (a) Shambhaji and the widow of Shivaji
market policy : [2015-II] (b) Shambhaji and Bajirao
1. He placed markets under the control of a high officer (c) Rajaram and Shambhaji
called 'Shahna' for strictly controlling the shopkeepers
(d) None of them
and prices.
2. In order to ensure a regular supply of cheap food 267. Match List-I with List-II and select the correct answer using
grains, he ordered the land revenue from Doab region the code given below the Lists : [2015-II]
to be paid directly to the State. List-I List-II
Which of the statements given above is/are correct? (Author) (Work)
(a) 1 only (b) 2 only A. Somadeva l. Malavikagnimitra
(c) Both 1 and 2 (d) Neither 1 nor 2 B. Kalidasa 2. Kathasaritsagara
261. Match List-I with List-II and select the correct answer using C. Bhasa 3. Chaurapanchasika
the code given below the Lists : [2015-II] D. Bilhana 4. Svapnavasavadatta
List-I List-II Code :
(Term) (Meaning) A B C D
A. Mihrab 1. Stepped pulpit (a) 2 1 4 3
B. Mimbar 2. Direction towards the (b) 3 4 1 2
Kaba for prayer (c) 2 4 1 3
C. Khutba 3. Arch (d) 3 1 4 2
D. Kibla 4. Sermon 268. Patanjali was [2015-II]
Code : (a) a philosopher of the 'Yogachara' school
A B C D (b) the author of a book on Ayurveda
(a) 2 1 4 3 (c) a philosopher of the 'Madhyamika' school
(b) 2 4 1 3 (d) the author of a commentary on Panini's Sanskrit
(c) 3 1 4 2 grammar
(d) 3 4 l 2 269. Match List-I with List-II and select the correct answer using
262. Match List-I with List-II and select the correct answer using the code given below the Lists : [2015-II]
the code given below the Lists : [2015-II]
List-I List-II
List-I List-II
(King) (Region)
(Person) (Work)
A. Shashanka 1. Assam
A. Uddanda 1. Sudhanidhi
B. Kharavela 2. Maharashtra
B. Sayana 2. Mallikamaruta
C. Simuka 3. Orissa
C. Yadavaprakasa 3. Malatimadhava
D. Bhaskara Varman 4. Bengal
D. Bhavabhuti 4. Vaijayanti
Code :
Code:
A B C D
A B C D
(a) 4 2 3 1
(a) 2 1 4 3
(b) 1 3 2 4
(b) 3 4 1 2
(c) 4 3 2 1
(c) 2 4 1 3
(d) 3 1 4 2 (d) 1 2 3 4
History C-71

270. In ancient India, the 'Yaudheyas' were [2015-II] 2. He fought in the famous Battle of Chinhat.
(a) a sect of the Buddhism (b) a sect of the Jainism 3. He was killed by British troops under the command of
(c) a republican tribe (d) Vassals of the Cholas Henry Lawrence.
271. The Dashakumaracharita or Tales of Ten Princes' was Select the correct answer using the code given below.
composed by [2015-II] (a) 1 only (b) 1 and 3
(a) Bhatti (b) Banabhatta (c) 2 and 3 (d) 1 and 2
(c) Bhasa (d) Dandin 276. Who among the following is the author of the autobiography
272. Which of the following statements about the Hastings Plan entitled Aamar Katha (1913)? [2015-II]
of 1772 is/ are correct? [2015-II] (a) Satyajit Ray
1. Each district was to have a civil and a criminal Court. (b) Rassundari Devi
2. The Judges were helped by native assessors who were (c) Ganesh Chandra Ghosh
skilled in Hindu and Islamic laws. (d) Binodini Dasi
3. The Sadar Diwani Adalat was mainly meant to settle 277. Name the French revolutionary who wrote Declaration of
mercantile cases exceeding ` 10,000 in value. the Rights of Woman and Female Citizen. [2015-II]
4. These Courts did not put into place any procedural (a) Olympe de Gouges (b) Nancy Ruhling
improvements. (c) Maximilien Robespierre (d) Mary Wollstonecraft
Select the correct answer using the code given below. 278. Consider the following statements : [2016-I]
1. In Hind Swaraj, Mahatma Gandhi formulates a
(a) 1 and 2 (b) 3 and 4 conception of part life for the individual as well as the
(c) 2 and 4 (d) 2 only society.
273. Which of the following statements about the formation of 2. Hind Swaraj was the outcome of the experience of
the Indian National Congress are true? [2015-II] Gandhi’s prolonged struggle against colonial raj in
1. The Indian National Congress was formed at a national India
convention held in Calcutta in December 1885 under Which of the statement given above is / are correct?
the Presidency of Motilal Nehru. (a) 1 only (b) 2 only
(c) Both 1 and 2 (d) Neither1 nor 2
2. The Safety Valve Theory regarding the formation of
279. Who among the following archaeologists was the first to
the Indian National Congress emerged from a idenity similarities between a pre - Harappan culture and
biography of A. O. Hume written by William the mature Harappan culture? [2016-I]
Wedderbrn. (a) Amalananda Ghosh (b) Rakhaldas Banerji
3. An early decision was that the President would be (c) Dava Ram Sahni (d) Sir John Marshall
from the same region where the session was to be 280. Which one of the following is the common element between
held. the kailasanatha Temple at Ellora and the Shore Temple at
4. W. C. Banerjee was the first President of the Indian Mamallapuram? [2016-I]
National Congress. (a) Both are examples of Nagara architecture
(b) Both are carved out from solid rocks
Select the correct answer using the code given below.
(c) Both are Gupta period temples
(a) 1, 2 and 4 (b) 2 and 3 (d) Both were built under the patronage of Pallava Kings
(c) 2 and 4 only (d) 1 and 3 281. Which of the following is / are not depicted in the Rajput
274. Which of the following statements about Jyotirao Phule's paintings? [2016-I]
Satyashodhak Samaj Movement in Maharashtra is / are true? 1. The stories of Krishna 2. Ragas and Raginis
[2015-II] 3. The deeds of Hamza 4. The deeds of Babur
1. The Satyashodhak Samaj was set up in 1873. Select the correct answer using the code given below:
(a) 1, 2 and 3 (b) 2, 3 and 4
2. Phule argued that Brahmins were the progeny of 'alien'
(c) 3 and 4 only (d) 4 only
Aryans.
282. Consider the following statements [2016-I]
3. Phule's focus on the Kunbi peasantry in the 1880s and 1. Abhinavagupta wrote a comprehensive treatise called
1890s led to a privileging of Maratha identity. the Tantraloka which systematically presents the
Select the correct answer using the code given below. teachings of the kula and Trika systems
(a) 1 and 2 only (b) 2 only 2. The Samartehchakaha by Haribhadra Suri written in
(c) 1 and 3 only (d) 1, 2 and 3 Gujarat around the eighth century is technically not a
275. Which of the following statements about Maulvi tantric work but is saturated with tantric ideas and
practices
Ahmadullah Shah, who played an important part in the
Which of the statements given above is / are correct?
Revolt of 1857, is/ are correct? [2015-II]
(a) 1 only (b) 2 only
1. He was popularly known as Danka Shah or the Maulvi (c) Both 1 and 2 (d) Neither 1 nor 2
with a drum.
EBD_7367
72
C- History

283. Consider the following statements: [2016-I] 291. The Agrahara in early India was : [2016-I]
1. The inscriptions on the pillar at Rummindei give vivid (a) The name of a village or land granted to Brahmins
details or Ashok’s Dhamma (b) the garland of flowers of Agar
2. the Nigalisagar inscription records the fact of Ashoka (c) the grant of land to officers and soldiers
having visited the Konagamana stupa (d) land or village settled by Vaishya farmers
Which of the statements given above is / are correct? 292. Shishu, Kishor and Tarun are the schemes of : [2016-I]
(a) 1 only (b) 2 only (a) Regional Rural Banks.
(c) Both 1 and 2 (d) Neither 1 nor 2 (b) Micro Units Development & Refinance Agency Ltd
284. Which of the following statements is /are true ? [2016-I] .(MUDRA).
1. Faxian’s ‘Gaoseng Faxian zhuani was the earliest first - (c) Small Industries Development Bank of India.
hand Chinese account of Buddhist sites and practices (d) Industrial Developement Bank of India.
in India 293. Which one among the following was not an attribute of
2. Faxian was only 25 years old at the times of writing the text Samudragupta described in Prayag Prashasti ? [2016-I]
3. Faxian’s main aim in coming to India was to obtain and (a) Sharp and polished intellect
take back texts containing monastic rules (b) Accomplished sculptor
Select the correct answer using the code given below: (c) Fine musical performances
(a) 1, 2 and 3 (b) 2 only (d) Poetical talent of genius
(c) 1 and 3 only (d) 3 only 294. Kamandaka’s Nitisara is a contribution to : [2016-I]
285. Which of the following statements about the Vijayanagara (a) Logic and Philosopy
Empire is / are true? [2016-I] (b) Mathematics
1. The kings claimed to rule on behalf of the God (c) Political morality
Virupaksha
(d) Grammar
2. Rulers used the title “Hindu Suratrana”
3. All Royal orders were signed in Kannada, Sanskrit 295. The Lilavati of Bhaskara is a standard text on : [2016-I]
and Tamil (a) Mathematics (b) Surgery
4. Royal Portrait sculpture was now displayed in temples (c) Poetics (d) Linguistics
Select the correct answer using the code given below 296. The followers of Gorakhnath were called : [2016-I]
(a) 4 only (b) 1 and 2 only (a) Jogis
(c) 1, 2 and 3 (d) 1, 2 and 4 (b) Nath-Panthis
286. Tqta in medieval India meant: [2016-I] (c) Tantriks
(a) land assigned to religious personnel for spiritual (d) Sanyasis
purposes 297. Match List-I with List-II and select the correct answer using
(b) land revenue from different territorial units assigned the code given below the Lists : [2016-II]
to army officers List-I List-II
(c) charity for educational and cultural activities (Power Plant) (State)
(d) the rights of the zamindar A. Satpura 1. Maharashtra
287. Which one of the following books was not illustrated with B. Dhuvaran 2. Uttarakhand
paintings in Akbar’s court? [2016-I] C. Tanakpur 3. Gujarat
(a) Hamzanama (b) Razmnama D. Dabhol 4. Madhya Pradesh
(c) Baburnama (d) Tarikh - i -Alfi Code :
288. Which kingdom did the temple of Hazara Rama belong to ? (a) A B C D
[2016-I] 4 3 2 1
(a) Avadh (b) Travancore (b) A B C D
(c) Vijayanagara (d) Ahom 1 3 2 4
289. Consider the following statement about votive inscriptions (c) A B C D
in the second century BC ? [2016-I] 1 2 3 4
1. They records gifts made to religious institutions (d) A B C D
2. They tell us about the idea of transference of the 4 2 3 1
meritorious results of the action of one person DIRECTIONS (Qs. 298-300) : The following three (3) items
Which of the statements given above is are correct? consist of two statements, Statement I and Statement II. Examine
(a) 1 only (b) 2 only these two statements carefully and select the correct answer
(c) Both 1and 2 (d) Neither 1 nor 2 Using the code given below.
290. Which one among the following statements about the coins Code : [2016-II]
of the Gupta rulers is correct? [2016-I] (a) Both the statements are individually true and
(a) The obverse and reverse, both had only the king's
portrait and date Statement II is the correct explanation of Statement I
(b) The obverse an reverse, both had only an image of a (b) Both the statements are individually true but
deity and date Statement II is not the correct explanation of
(c) The obverse generally had king's portrait and reverse Statement I
had an image of a deity or a motif (c) Statement I is true but Statement II is false
(d) The obverse generally had king 's portrait and reverse (d) Statement I is false but Statement II is true
always had a date
History C-73

298. Statement I : [2016-II] 304. Which of the following statements about the philosopher
The Pahi-kashta peasants were non-resident cultivators Shankara is /are true ? [2016-II]
cultivating lands on a contractual basis. 1. Shankara espoused a form of Vedanta called Advaita.
Statement II : 2. He elaborated on the philosophy of Gaudapada.
The Pahi-kashta peasants worked under the temptation of 3. Shankara tried to demonstrate that the Upanishads
favourable terms of revenue or the compulsion of economic and Brahmasutras contain a unified, systematic
distress. philosophy.
299. Statement I : [2016-II] 4. Shankara founded the Amanaya Mathas.
The Zamindars were an exploitative class in Mughal India. Select the correct answer using the code given below.
Statement II : (a) 1 and 2 only (b) 1, 2 and 3
The Zamindars often received the support of the peasantry (c) 3 and 4 (d) 4 only
in a large number of agrarian uprisings in North India in the 305. Which of the following statements about the Deccan Riots
seventeenth century. of 1875 is/are true? [2016-II]
300. Statement I : [2016-II] 1. The Deccan Riots resulted in protection for peasants
King Ashoka abolished capital punishment and disbanded through the Deccan Agriculturalists Relief Act of 1879.
his army. 2. The riots did not spread to the whole of Maharashtra
Statement II : because of prompt suppression by the British.
After Kalinga War, Ashoka was remorseful and became a 3. The British were unable to contain the riots and they
Buddhist. spread throughout Maharashtra.
301. Which one of the following sources tells us about women 4. The cotton boom in the Deccan that had been caused
protesting against the infidelity of their husbands or the by the artificial demand generated by the American
neglect of the wife and children by the male head of the Civil War caused the impoverishment of the peasants.
household? [2016-II] Select the correct answer using the code given below.
(a) The Kitab-ul-Hind (a) 1 only (b) 1 and 3
(b) Documents belonging to the Village Panchayats of (c) 2 and 4 only (d) 1, 2 and 4
Rajasthan, Gujarat and Maharashtra 306. Why are the Gypsies regarded as of Indian origin?
(c) Sculptures from Mandor 1. They follow Indian religious practices. [2016-II]
(d) The Ain-i-Akbari of Abul Fazl 2. Their language, called Romani, is an Indo-Aryan
302. Match List-I with List-II and select the correct answer using language.
the code given below the Lists : [2016-II] 3. They believe that they came from India.
List-I List-II 4. Epigraphic evidence locates their original home in Uttar
(Historian) (Book) Pradesh.
A. Sumit Sarkar 1. The Rise and Growth Select the correct answer using the code given below.
of Economic (a) 1 and 4 only (b) 2 only
Nationalism in India (c) 1, 2 and 4 (d) 2 and 3
B. Shahid Amin 2. A Rule of Property 307. The term Upari refers to which one of the following?
for Bengal (a) A category of proprietary tenure under the Mughal rule
C. Ranajit Guha 3. The Swadeshi (b) A category of tenancy tenure under the Maratha rule
Movement in Bengal, (c) A soldier in the Maratha army
1903-1908 (d) A village headman in the Mughal period
D. Bipan Chandra 4. Event, Metaphor, 308. The 'water frame' of Richard Arkwright was a device for
Memory-Chauri [2016-II]
Chaura, 1922~1 992 (a) producing a new type of painting
Code: (b) irrigating fields for rice cultivation
(a) A B C D (c) producing strong threads of yarn
3 4 2 1 (d) the faster movement of steamships
(b) A B C D 309. The 'four great needs' of the people as identified by the
3 2 4 1 Guomindang were [2016-II]
(c) A B C D (a) clothing, food, housing and transportation
1 2 4 3 (b) education, food, housing and health care
(d) A B C D (c) food, housing, education and employment
1 4 2 3 (d) employment, housing, education and health care
303. What form of Shiva is most prominent in the Brihadeshvara 310. Which one of the following was the earlier name of Tokyo?
Temple built by the Chola dynasty? [2016-II] [2016-II]
(a) Harihara (b) Bhairava (a) Osaka (b) Kyoto
(c) Rudra (d) Tripurantaka (c) Samurai (d) Edo
EBD_7367
74
C- History

311. The epic, Silappadikaram refers to the [2016-II] 317. Which of the following statements about the Shiromani
Gurudwara Prabandhak Committee (SGPC) are correct?
(a) story of Rama [2017-I]
(b) Jaina elements in the storyline 1. It began as the political wing of the Singh Sabhas in
(c) culture of Sri Lankan Buddhists the late 19th century.
(d) cult of shakti worship 2. It was formed in 1920 as part of the upcoming Akali
312. Rakshasa type of marriage is [2016-II] movement.
(a) marriage by purchase 3. It was founded to reclaim control of the Sikh shrines
(b) marriage by capture from the government manipulated loyalist committees.
(c) marriage by giving dowry 4. It formed the Akali Dal to coordinate groups (Jathas)
(d) marriage by seduction to reclaim control of the shrines.
313. Consider the following statements about Harappan cities : Select the correct answer using the code given below.
[2016-II] (a) l, 3 and 4 (b) 2, 3 and 4
1. Roads were not always absolutely straight and did (c) 2 and 4 only (d) 2 and 3 only
not always cross one another at right angles. 318. Which of the following statements with regard to the speech
2. A striking feature is the uniformity in the average size of Mahatma Gandhi at the opening of the Banaras Hindu
of the bricks for houses and for city walls. University are correct? [2017-I]
Which of the statements given above is/are correct? 1. He charged the Indian elite with a lack of concern for
(a) 1 only (b) 2 only the labouring poor.
(c) Both 1 and 2 (d) Neither 1 nor 2 2. He asserted that our salvation can come only through
314. Which of the following statements about the Ilbert Bill (1883) the farmers.
is/are correct? [2017-I] 3. He highlighted the plight of the untouchables.
1. It proposed to grant limited criminal jurisdiction to 4. He promised to take up the cause of the mill owners of
native officials. Ahmedabad.
2. It proposed to grant complete civil and criminal Select the correct answer using the code given below.
jurisdiction to native officials. (a) l, 2, 3 and 4 (b) l and 2 only
3. The proposed Bill generated opposition from (c) 1, 2 and 3 only (d) 3 and 4 only
England's European subjects in India. 319. Match List-I with List-II and select the correct answer using
4. In spite of opposition to the Bill, it was passed without the code given below the Lists : [2017-I]
any modifications. List-I List-ll
Select the correct answer using the code given below. (Act/ Event) (Year)
(a) 1 and 2 (b) 2 only A. Rowlatt Act 1. 1922
(c) 1 and 3 (d) 3 and 4 B. Salt March 2. 1931
315. Consider the following statements about the different C. Chauri Chaura Incident 3. 1930
meanings of 'Swaraj' as articulated by Mahatma Gandhi : D. Second Round Table 4. 1919
[2017-I] Conference
1. Swaraj is intimately linked with Ahimsa (non-violence) Code:
and Satyagraha (adherence to truth). (a) A B C D
2. Swaraj has two senses-one ' political and one beyond 4 3 1 2
the realm of politics. (b) A B C D
3. Swaraj is something that requires time and patience to 3 1 4 2
acquire. (c) A B C D
4. With determination, Swaraj could be obtained easily 4 3 2 1
and quickly. (d) A B C D
Which of the statements given above is/are correct? 3 4 2 1
(a) 1 only (b) 1 and 2 only 320. Consider the following statements about the Salt March :
(c) 3 and 4 (d) 1, 2 and 3 [2017-I]
316. What was /were the formative influence(s) on the philosophy 1. The Salt March was deliberately ignored by the
of Mahatma Gandhi? [2017-I] European media.
1. Gandhiji was influenced by the 18th century Pranami 2. The Salt March was widely covered by the American
sect that advocated the unity of faiths. and European press.
2. Gandhiji was influenced by the theosophists. 3. The Salt March was the first nationalist activity in
which women participated in large numbers.
3. Gandhiji was an admirer of the writings of Romantics
4. The Salt March impressed upon the British the urgent
like Wordsworth.
need to devolve more power to Indians.
Select the correct answer using the code given below.
Which of the statements given above are correct?
(a) 1 and 2 only (b) 2 and 3 only
(a) 1, 2 and 4 (b) 2, 3 and 4
(c) 1, 2 and 3 (d) 1 only
(c) 3 and 4 only (d) 2 and 3 only
History C-75

321. Which of the following statements about the social base of 328. Which one of the following statements about the All India
the Arya Samaj in British India is/are correct? [2017-I] Depressed Classes Association is not correct? [2017-I]
1. It was located mainly in Punjab and western Uttar (a) The All India Depressed Classes Association was
Pradesh. formed in Nagpur with M. C. Rajah as its first elected
2. It mainly comprised the trading castes. President.
3. It was much more limited than that of the Brahmo Samaj. (b) The All India Depressed Classes Association was not
Select the correct answer using the code given below. attended by Dr. B. R. Ambedkar in 1926.
(a) 1 and 2 only (b) 2 and 3 only (c) Ambedkar resigned from the All India Depressed
(c) 1 only (d) 1, 2 and 3 Classes Association and formed his own All India
Depressed Classes Congress in 1930.
322. Who among the following is the author of the book, A
(d) The All India Depressed Classes Association favoured
Comparison between Women and Men? [2017-I]
Ambedkar's demand for separate electorate for the
(a) Pandita Ramabai (b) Sarojini Naidu depressed classes.
(c) Tarabai Shinde (d) Rameshwari Nehru 329. Which one of the following statements about the Justice
323. During the mid 19th century Industrial Revolution, the Party is not correct? [2017-I]
average life span of workers in Manchester was [2017-I] (a) It clamoured for the same kind of separate communal
(a) 17 years (b) 30 years representation for the Non-Brahmins as had been
(c) 55 years (d) 62 years granted to the Muslims by the Morley-Minto Reforms.
324. George Washington was made the Commander in Chief of (b) It was patronized mainly by richer landowning and
the American forces [2017-I] urban middle class Non-Brahmins.
(a) in December 1773, after the Boston Tea Party (c) It succeeded in getting the provision for 28 reserved
(b) at the First Continental Congress in September 1774 seats for the Non-Brahmins in the Montagu-
(c) at the Second Continental Congress in 1775 Chelmsford Reforms.
(d) by the Continental Congress at the Declaration of (d) It supported the call given by the Congress to boycott
Independence on 4th July, 1776 elections in 1920.
325. Alexander Kerensky was [2017-I] 330. The Ryotwari experiment in land revenue was started by
(a) the head of the Provisional Government in Russia [2017-I]
before the October Revolution (a) Henry Dundas
. (b) a close confidant of Lenin, with whose help the Czar (b) Alexander Reed
(c) David Ricardo
was dethroned
(d) Mountstuart Elphinstone
(c) the head of the Czar's army
(d) an advisor of Joseph Stalin DIRECTIONS (Qs. 331-332): The following five (5) items consist
DIRECTIONS (Qs. 326-327 : The following two (2) items consist of two statements, Statement I and Statement II. Examine these
of two statements, Statement I and Statement II. Examine these two statements carefully and select the correct answer using the
two statements carefully and select the correct answer using the code given below. [2017-I]
code given below. [2017-I] Codes:
Codes: (a) Both the statements are individually true and
(a) Both the statements are individually true and Statement II is the correct explanation of Statement I
Statement II is the correct explanation of Statement I (b) Both the statements are individually true
(b) Both the statements are individually true but Statement but Statement II is not the correct explanation of
II is not the correct explanation of Statement I Statement I
(c) Statement 1 is true but Statement II is false (c) Statement I is true but Statement II is false
(d) Statement I is false but Statement II is true
(d) Statement 1 is false but Statement II is true
331. Statement I : [2017-II]
326. Statement I : [2017-I]
The Communists left the All India Trade Union Congress in
The passing of the Coercive Acts made reconciliation
1931.
between Britain and her American colonies virtually Statement II :
impossible. By 1928, the Communists were no longer working with the
Statement II : mainstream national movement.
The British Parliament, having issued the Stamp Act in 1765, 332. Statement I : [2017-II]
repealed it later. The early Aryans, who were essentially pastoral, did not
327. Statement I : [2017-I] develop any political structure which could measure up to
The Poona Pact provided for 151 reserved seats for the a State in either ancient or modern sense.
scheduled castes to be elected by a joint, not separate, Statement II :
electorate. Kingship was the same as tribal chiefship; the term Rajan
Statement II : being used for tribal chief who was primarily a military leader
Dr. B. R. Ambedkar withdrew from active politics for almost and who ruled over his people and not over any specified
a decade when the Communal Award was revoked. area.
EBD_7367
76
C- History

333. Which one of the following peasant struggles was an Which one of the following is the correct chronological
outcome of British opium policy? [2017-II] order of building the above monuments?
(a) Phulaguri Dhawa (1861) (a) 3, 1, 4, 2 (b) 3, 1, 2, 4
(b) Birsaite Ulgulan (1899-1900) (c) 1, 3, 2, 4 (d) 1, 3, 4, 2
(c) Pabna Revolt (1873) 340. Match List-I with List-II and select the correct answer using
(d) Maratha Peasant Uprising (1875) the code given below the Lists : [2017-II]
334. Which one of the following statements about the Sayyid List-I List-II
brothers during the period of Later Mughals is not true? (Type of Land) (Meaning)
[2017-II] A. Urvara 1. Land watered by a
(a) They brought Jahandar Shah to power.
river
(b) They wielded administrative power.
B. Maru 2. Fertile land
(c) They followed a tolerant religious policy.
(d) They reached an agreement with King Shahu. C. Nadimatrika 3. Land watered by
335. Which of the following statements with regard to the rain
heterodox sect between sixth and fourth century BC are D. Devamatrika 4. Desert land
correct? [2017-II] Code :
1. The Jain ideas were already being circulated in the A B C D
seventh century BC by Parshva. (a) 2 1 4 3
2. Although Buddhism, and to a lesser extent Jainism, (b) 3 4 1 2
took account of the changes in material life and (c) 3 1 4 2
reacted against orthodoxy, neither of these sought (d) 2 4 1 3
to abolish the caste system. 341. Who was the editor of the journal Indian Social Reformer
3. The first female disciple of mahavira is said to have that was started in 1890? [2017-II]
been a captured slave woman. (a) K. T. Telang (b) Veeresalingam
4. Buddha held that nuns could attain spiritual (c) N. G. Chandavarkar (d) K. N. Natarajan
liberation just like a monk and granted them an 342. Which twelfth century Sanskrit scholar was first responsible
equal status in the mendicant order. for the compilation of ‘Nibandhas’ or digests of epic and
Select the correct answer using the code given below. Puranic texts? [2017-II]
(a) 1, 2, 3 and 4 (b) 1, 2 and 3 only (a) Harsha (b) Govinda chandra
(c) 1, 2 and 4 only (d) 3 and 4 only
(c) Lakshmidhara (d) Kalidasa
336. Which one of the following combinations of year and event
343. Which river is praised in the fifth century Tamil epic,
concerning the French Revolution is correctly matched?
Silappadikaram? [2017-II]
[2017-II]
(a) 1789 : Nepoleonic Code (a) Cauvery (b) Godavari
(b) 1791 : Tennis Court Oath (c) Saraswati (d) Ganges
(c) 1792 : National Convention 344. Which one of the following statements about the Harappan
(d) 1804 : New Constitution of France Culture is not correct? [2017-II]
337. Who among the following is the author of the book, The (a) It witnessed the first cities in the subcontinent.
Social Contract? [2017-II] (b) It marks the first use of script, written from right to left.
(a) Voltaire (b) Hobbes (c) It marks the earliest known use of iron as a medium for
(c) Locke (d) Rousseau the art of sculpting.
338. Due to which of the following factors, the Industrial (d) It marks the earliest known use of stone as a medium
Revolution took place in England in the eighteenth century? for the art of sculpting.
[2017-II] 345. Harshacharita has references to various presents sent by
1. The discovery of coal and iron deposits a ruler named Bhaskara to Harshavardhana. Bhaskara
2. The discovery of steam power belonged to [2017-II]
3. The introduction of railways (a) Haryanka Dynasty of Magadha
4. The regular supply of raw materials (b) Varman Dynasty of Assam
Select the correct answer using the code given below. (c) Nanda Dynasty of North India
(a) 1 and 2 only (d) None of the above
(b) 2, 3 and 4 only
346. Which of the following was/were founded by Raja Ram
(c) 1 and 3 only
Mohan Roy? [2017-II]
(d) 1, 2, 3 and 4
1. Atmiya Sabha 2. Brahmo Samaj
339. Consider the following : [2017-II]
1. Tughlaqabad Fort 3. Prarthana Samaj 4. Arya Samaj
2. Bada Gumbad in Lodhi Garden Select the correct answer using the code given below.
3. Qutub Minar (a) 1, 2 and 3 (b) 2 only
4. Fatehpur Sikri (c) 1 and 2 only (d) 1, 3 and 4
History C-77

347. Match List-I with List-II and select the correct answer using 352. Which of the following statements about the Non-
the code given below the Lists : [2017-II] Cooperation Movement is/are correct? [2018-I]
List-I List-II 1. It was marked by significant participation of
(Harappan Site) (Modern Name) peasants from Karnataka.
A. Dholavira 1. Saurashtra 2. It was marked by non-Brahmin lower caste
B. Rakhigarhi 2. Hisar participation in Madras and Maharashtra.
C. Bhirrana 3. Kadir Island 3. It was marked by the lack of labour unrest in places
D. Bhogavo 4. Haryana like Assam, Bengal and Madras.
Code : 4. It was badly shaken by the Chauri Chaura incident
A B C D in 1922 after which Gandhiji decided to continue
(a) 1 4 2 3
with the movement on a much smaller scale.
(b) 1 2 4 3
Select the correct answer using the code given below.
(c) 3 2 4 1
(a) 1 only (b) 1, 2 and 4
(d) 3 4 2 1
(c) 2 and 3 (d) 2 only
348. Which one of the following statements about the Gupta
353. Which of the following statements about the usage of the
period in Indian History is not correct? [2017-II]
(a) Sanskrit language and literature, after centuries of term ‘barbarian’ is/are correct? [2018-I]
evolution, reached what has been described as a level 1. It is derived from the Greek word ‘barbaros’ which
of classical excellence through royal patronage. means a non-Greek.
(b) The status of women was redefined. They were entitled 2. Romans used the term for the Germanic tribes, the
to formal education and hence there were women Gauls and the Huns.
teachers, philosophers and doctors. Early marriage was Select the correct answer using the code given below.
prohibited by law and they were given the right to (a) 1 only (b) 2 only
property. (c) Both 1 and 2 (d) Neither 1 nor 2
(c) Decentralization of administrative authority was 354. Which one of the following statements is not correct?
impacted by increased grants of land and villages with [2018-I]
fiscal and administrative immunities to priests and (a) Al-Biruni identifies the Sufi doctrine of divine love as self-
temples. annihilation with parallel passages from Bhagavad Gita.
(d) Land grants paved the way for feudal developments (b) According to Al-Biruni, Sufi theories of Soul were
and emergence of serfdom in India, resulting in the similar to those in Patanjali’s Yoga Sutra.
depression of the peasantry. (c) The Hatha Yogic treatise Amrita Kunda had lasting
349. Verses ascribed to poet-saint Kabir have been compiled in impact on Sufism.
which of the following traditions? [2018-I] (d) Hujwiri’s conversation with the Yogis shows that he
1. Bijak in Varanasi was impressed with their theory of the division of the
2. Kabir Granthavali in Rajasthan human body.
3. Adi Granth Sahib 355. Consider the following statement : [2018-I]
Select the correct answer using the code given below.
“So much is wrung from the peasants, that even dry bread
(a) 1 and 2 only (b) 1, 2 and 3
is scarcely left to fill their stomachs.”
(c) 2 and 3 only (d) 3 only
Who among the following European travellers had made
350. Around twelfth century, Sufi Silsilas began to crystallize in
the above statement about the condition of peasantry in
different parts of the Islamic world to signify [2018-I]
the Mughal Empire?
1. continuous link between the master and disciple
(a) Francisco Pelsaert
2. unbroken spiritual genealogy to the Prophet
Muhammad (b) Francois Bernier
3. the transmission of spiritual power and blessings (c) Jean-Baptiste Tavernier
to devotees (d) Niccolao Manucci
Select the correct answer using the code given below. 356. What is the name of the literary genre developed by the
(a) 1 and 2 only (b) 2 only Khojas who are a branch of the Ismaili sect? [2018-I]
(c) 1 and 3 only (d) 1, 2 and 3 (a) Ginan (b) Ziyarat
351. In the 10th Mandala of the Rigveda, which one of the (c) Raag (d) Shahada
following hymns reflects upon the marraige ceremonies? 357. Who was/were the 10th century composer(s) of the Nalayira
[2018-I] Divya Prabandham? [2018-I]
(a) Surya Sukta (b) Purusha Sukta (a) Alvars (b) Nayanars
(c) Dana Stutis (d) Urna Sutra (c) Appar (d) Sambandar
EBD_7367
78
C- History

358. Consider the following statement : [2018-I] 364. Who deciphered the Brahmi and Kharoshthi scripts?
“A sound body means one which bends itself to the spirit [2018-I]
and is always a ready instrument at its service”. (a) Piyadassi
The above statement is attributed to (b) Colin Mackenzie
(a) Sardar Patel (b) Winston Churchill (c) Alexander Cunningham
(c) Mahatma Gandhi (d) Baden-Powell (d) James Prinsep
359. Who among the following was the founder of Phoenix 365. Which of the following is/are the feature(s) of the
Settlement? [2018-I] Brahmadeya Grants during c 600–1200 AD? [2018-I]
(a) Mahatma Gandhi (b) B. R. Ambedkar 1. Their creation meant a renunciation of actual or
(c) Rabindranath Tagore (d) Swami Vivekananda potential sources of revenue by the State.
360. Name the platform used for ritual purposes by the kings of 2. These grants could vary from a small plot to several
the Vijayanagara Empire. [2018-I] villages.
(a) Mahanavami Dibba (b) Lotus Mahal 3. Most grants were made in unsettled areas.
(c) Hazara Rama (d) Virupaksha Select the correct answer using the code given below.
361. The idea of ‘Farr-i Izadi’, on which the Mughal kingship (a) 1 only (b) 2 and 3 only
was based, was first developed by which one of the (c) 1 and 2 only (d) 1, 2 and 3
following Sufi saints? [2018-I] 366. Who is the author of Manimekalai? [2018-I]
(a) Shihabuddin Suhrawardi (a) Kovalan (b) Sathanar
(b) Nizamuddin Auliya (c) Ilango Adigal (d) Tirutakkatevar
(c) Ibn al-Arabi 367. Which of the following statements about Sir Syed Ahmad
(d) Bayazid Bistami Khan is/are correct? [2018-I]
362. Which Buddhist text contains an account of the Mauryan 1. He argued that India was a federation of ethnic
Emperor Ashoka? [2018-I] communities based on common descent.
(a) Vinaya Pitaka (b) Sutta Pitaka 2. His philosophy was very similar to that of the
(c) Abhidhamma Pitaka (d) Mahavamsa Indian National Congress.
363. Which one of the following statements about Buddhist 3. He imagined India as a Nation State based on
Stupas in India is not correct? [2018-I] individual citizen’s rights.
(a) Ashoka played an important role in popularizing the 4. The curriculum at the Mohammedan Anglo-
Stupa cult.(b) They were repositories of Oriental College blended Muslim theology and
relics of Buddha and other monks. European empiricism.
(c) They were located in rural areas. Select the correct answer using the code given below.
(d) They were located close to trade routes. (a) 1 only (b) 2 and 3
(c) 3 only (d) 1 and 4
History C-79

HINTS & SOLUTIONS


1. (c) Akbar ruled with a social and religious toleration that 13. (b) The Non-Cooperation Movement, 1920-22
was relative, not absolute, and was based on his Civil Disobedience Movement in India, 1930-34)
concept of sulh-i-kul (for the general good of all people) The Quit India Movement, 1942
which built on his liberal views of religion. Akbar took 14. (d) The Musalman is the oldest Urdulanguage daily
the Sufi mystic notion of sulh-i-kul and transformed it newspaper published from Chennai in India. The
to become a principle denoting amity within a culturally newspaper was founded by Syed Azmatullah in 1927.
The Musalman is possibly the only handwritten
pluralistic India.
newspaper in the world.
2. (b) Narasimhadeva I (AD 1238-1264) was the king of Ganga 15. (c) The Arthasastra is a treatise on Political philosophy.
dynasty. The book, written in Sanskrit, discusses theories and
3. (b) The Indian National Calendar, sometimes called the principles of governing a state. The meaning of
Saka calendar, is the official civil calendar in use in Arthashastra is 'Science of Polity'. It is written by
India. Saka Calendar begins on 22 March every year Kautilya.
except in leap years when it starts on 23 March. The 16. (a) The partition of Bangal took place on16 October 1905.
Kushana emperor Kanishka is credited with the The Chauri Chaura incident occurred at Chauri Chaura
initiation of the Saka era on his accession to the throne in the Gorakhpur district of the United Province, British
in 78 A D. India on 5 February 1922.The first Round table
4. (b) Lord Mountbatten was preceded by C Rajgopalachari. conference was opened officially by King George V
Lord Mountbatten was the last Viceroy of India (1947) on November 12, 1930 in London.
and the first Governor-General of the independent 17. (c) Lord Dufferin was the viceroy of India at the time of
the formation of Indian national Congress.
Dominion of India (1947-48). 18. (a) Government of India Act 1909 introduced separate and
5. (a) Under the leadersh ip of Gandhiji, the Civil discriminatory electorates for Muslims. This was for
Disobedience Movement was launched in AD 1930. It the first time that, electorate for returning to the
began with the Dandi March. On 12 March 1930, Gandiji representatives to the councils was decided on the
with some of his followers left the Sabarmati Ashram basis of class & community. For the central council,
at Ahmedabad and made their way towards Dandi, a one more category Muslims was added. This was for
village on the west coast of India. Here, Gandhiji the first time that the seats in the legislative bodies
protested against the Salt Law (salt was a monopoly were reserved on the basis of religion for Muslims.
of the government and no one was allowed to make This is called Communal representation.
salt) by making salt himself and throwing up a challenge Sessions of Indian National Congress.
to the British government. 19. (a)
6. (a) After 1857 Lord Canning announced at a Darbar at Sessions Place Date & year President
Allahabad, the assumption of the Govt. of India by st Bombay Dec. 28-30, 1885 Womesh Chandra
1 Session
the sovereign of great Britain. Bonerjee
7. (b) 1. Third Carnatic War (1756–1763)
2 Session Calcutta Dec. 27-30, 1886 Dadabhai Naoroji
nd
2. The First Anglo–Mysore War –(1767–1769)
3. The First Anglo–Burmese War–(1824–1826) 3 Session Madrass
rd Dec. 27-30, 1887 Badruddin Tyabji
4. The Second Anglo–Afghan War (1878–1880) th
4 Session Allahabad Dec. 26-29, 1888 George Yule
8. (b) Lord William Bentinck was the first governor General
of India.
20. (d) The Treaty of Amritsar of 1809 was an agreement
9. (c) The 23rd Session (1907) of the Congress was held at
between the British East India Company and Ranjit
Surat.In the session, there was an open clash between Singh, the Sikh leader who founded the Sikh empire.
the Moderates and the Extremists and ultimately it led The treaty settled Indo-Sikh relations for a generation.
to a split in the Congress. The immediate occasion was the French threat to
10. (b) The "Grand Old Man of India" Dadabhai Naoroji northwestern India, following Napoleon's Treaty of
initiated establishment of East India Association at Tilsit with Russia (1807) and Ranjit Singh's attempt to
London. bring the Cis-Sutlej states under his control. The British
11. (b) The Theosophical Society was formed by Helena wanted a defensive treaty against the French and
Petrovna Blavatsky, Henry Steel Olcott, William Quan control of Punjab to the Sutlej River. Although this
Judge and others in November 1875 in New York. The was not a defensive treaty, it did fix the frontier of
lands controlled by Ranjit Singh broadly along the
aim of the society was to promote spiritual principles line of the Sutlej River.
and search for truth known as Theosophy. 21. (b) Buddha delivered his first sermon at Sarnath. Sarnath
12. (b) It was a significant battle fought between the forces is a city located in the north-east of Varanasi near the
under the command of the British East India Company confluence of the Ganges and the Gomati rivers in
on the one side and the combined army of Mir Kasim, Uttar Pradesh,
the Nawab of Bangal, Nawab of Awadh and Shah Alam 22. (b) Shankaracharya philosophy is called Advaita. The
ll, the Mughal Emperor. Mir Jafar was a puppet of Advaita Vedanta focuses on the basic concepts as
British East India Company. Brahman, atman, vidya (knowledge), avidya (ignorance),
maya, karma and moksha.
EBD_7367
80
C- History

23. (c) The All India Muslim League, a political organization president. Upto 1945 Congressmen, Socialists,
was founded in 1906 by Nawab of Dhaka Salimullah.Its Communists worked in the AITUC which was the central
mainpurpose was to safeguard the political rights of trade union organisation of workers of India.It is the
Muslims in India. oldest trade union federation in India .
24. (c) Patimokkha stands for the rules of the Sangha. Buddha 37. (b) Gandhiji was elected President of the All-India Khilafat
Dhamma and Sangha are the three Jewels of Buddhism. Conference which met at Delhi on November 23, 1919.
25. (a) The country's first railway, built by the Great Indian They decided to withdraw all cooperation from the
Peninsula Railway (GIPR), opened in 1853 between government if their demands were not met.
Bombay and Thane. 38. (c) The Constitution of India was drafted by the
26. (d) • Champaran Satyagrah - 1917 constituent assembly and it was set up under the
• Nagpur Satyagrah - 1923 cabinet Mission plan on 16 May 1946. The members
• Bardoli Satyagrah - 1928 of the constituent assembly were elected by the
• Rajkot Satyagrah- 1939 Provincial assemblies by method of single transferable
27. (c) Muhammad Iqbal was invited to the second and third vote system of proportional representations.Members
round table conference. of the committee:, Sardar Vallabhbhai Patel, K. M.
28. (a) • G.K Gokhale established The Servants of India Munshi,Acharya J.B kriplani . Lok Nayak Jai Prakash
Society in Pune, Maharashtra, on June 12, 1905. was not the member of the constituent assembly.
• Pandit Madan Mohan Malaviya founded Banaras 39. (d) The historic Objective Resolution was moved by
Hindu University (BHU) at Varanasi in 1916. Jawaharlal Nehru on 13 December 1946, and was
• The Swatantra Party was founded by C. adopted unanimously on 22 January 1947.
Rajagopalachari, was Indian conservative political 40. (b) In District administration each district was administered
party that existed from 1959 to 1974. by 3 officers viz. Pradeshika, Rajuka, & Yukta.
• Vinayak Damodar Savarkar founded Free India Pradeshika was senior and Rajuka was subordinate.
Society(student society). Yukta was subordinate to both of them. The district
29. (a) Dadabhai Naoroji administration was under the charge of Rajukas, whose
2nd Session Calcutta Dec. 27-30, 1886 Dadabhai position and functions are similar to modern collectors.
Naoroji He was assisted by Yuktas or subordinate officials.
9th Session Lahore Dec. 27-30, 1893 Dadabhai The rajjuka was responsible for surveying and
Naoroji assessing the land, fixing its rent and record keeping
22nd Session Calcutta Dec. 26-29, 1906 Dadabhai besides judicial functions.
Naoroji 41. (c) Robert Sewell was a civil servant of Madras
30. (b) A separate Public Works Department was established presidency. He was Keeper of the Record Office of
by Lord Dalhousie. The main works of this department Madras .He authored 'A Forgotten Empire-
wereto construct roads, bridges and government Vijayanagar', acontribution to the History of India.
buildings. 42. (b) The Doctrine of Non-Violence and Civil Disobedience
31. (b) Ryotwari System was introduced by Thomas Munro associated with Mahatma Gandhi were influenced by
in 1820. In this System, the ownership rights were the works of Ruskin, Tolstoy and Thoreau.
handed over to the peasants. British Government 43. (c) These books deal with principles of grammar.
collected taxes directly from the peasants. 44. (c) Gandhi ji launched Champaran Satyagrah on the
32. (d) repeated request of a local leader, Raj kumar Shukla,
• Illbert Bill- 1883 Ambalal Sarabhai was an industrialist of Ahmedabad .
• Marley Minto Reforms- 1909 He gave both personal and financial support to the
• Rowlatt Act- 1919 freedom struggle in Ahmedabad.
• Gandhi Irwin pact- 1931 Indulal Kanaiyalal Yagnik was an independence
33. (c) Cabinet Mission was composed of three Cabinet activist and a leader of the All India Kisan Sabha. He
Ministers of England participated in the Kheda Satyagraha(In 1918,) led by
• Sir Pethick Lawrence, Secretary of State for India. Gandhiji.
Vallabhbhai Jhaverbhai Patel was a barrister and leader
• Sir Stafford Cripps, President of the Board of Trade,
of Indian Independence. He started Bardoli Satyagrah
• A .V Alexander, the First Lord of the Admiralty
in Gujarat against the raised tax of 30%.
• Sir Cyril Radcliffe was appointed to chair two
45. (c) Raja Ram Mohan Roy was an Indian socio-educational
Boundary Commissions in June 1947 -one for
reformer who was also known as 'Maker of Modern
Bengal and one for Punjab.
India' and 'Father of Modern India'. He was born on
34. (d) Tansen, who was one of the nine jewels or navaratnas
May 22, 1774 into a Bengali Hindu family. He was the
in the court of Emperor Akbar, was born in a Hindu
founder of the Brahmo Samaj, one of the first Indian
family at Behat near Gwalior in the Madhya Pradesh
socio-religious reform movements. He advocated the
state. Father of Tansen was Makarand Pande, who
study of English, Science, Western Medicine and
named him Ramtanu Pandey. Technology. He was given the title 'Raja' by the Mughal
35. (c) The first Rupee (Silver coin) was introduced by Sher Emperor.
Shah and was called "Rupia" which remained in use 46. (c) Qutbuddin Bakhtiar Kaki was disciple of Moinuddin
throughout the Mughal rule. Chishti. His most famous disciple wasFariduddin
36. (c) All India Trade Union Congress (AITUC) was founded Ganjshakar.He was a renowned Muslim Sufi mystic,
on 31 October 1920 with Lala Lajpat Rai as its first
saint and scholar of the Chishti Order .
History C-81
47. (c) Gandhi Ji was the preeminent leader of Indian • The system of separate electorates for the
independence movement in British-ruled India. He led minorities was to be retained.
various movements such as Champaran and Kheda • The Muslims, wherever they were in minority, were
Satyagraha,Khilafat movement, Non-cooperation to be granted a weightage.
movement, Quit India Movement etc. • Except NWFP, 3 % seats for women were to be
48. (b) Sh er Shah's biggest achievement was the r e- reserved in all provinces.
establishment of law and order across his empire. The • The depressed, dalits or the untouchables were
village headmen and zamindars were made responsible to be declared as minorities.
for losses due to robberies. If they were not able to • Allocation was to be made to labours, landlords,
catch the robbers, they were themselves punished as traders and industrialists.
criminals. At the local level, village panchayats and 57. (a) Annie Besant (1847-1933) was the second President
zamindars settled disputes and punished the guilty. of The Theosophical Society from 1907 to 1933. She
Above them was the qazi in each state. founded Central Hindu collegeat Banaras. Besant
49. (c) Fa-hien was a Chinese monk who travelled India during launched the All India Home Rule League along with
the age of Chandra Gupta Vikramaditya. He travelled Lokmanya Tilak in 1916.
during C 399-414. Fa-hien wanted to go to India to 58. (b) Pulakesin II (610-642 CE) is the most famous ruler of
search for the treatises of the Vinaya Pitaka, the the Chalukya dynasty.The Aihole inscription describes
monastic rules of BUDDHISM. Travelling through how the mighty Harsha lost his harsha (joy) when he
Central Asia and Northwest India, Fa-hien reached suffered the ignominy of defeat. Pulakesin entered into
northern India and then visited the holy Buddhist sites a treaty with Harsha, with the Narmada River
located in the Ganges valley: Kapilavastu, the designated as the border between the Chalukya Empire
birthplace of Buddha; Bodhgaya, the site of Buddha's and that of Harshavardhana.
enlightenment; Sarnath, where Buddha preached his 59. (d)
first sermon, and Kushinagara, the place of Buddha's 60. (a) The conflict between Sen and Debendranath Tagore
was broadened in 1865, when Tagore allowed the
nirvana.
Brahmos conducting services to wear their sacred
50. (d) Lord Wellesley defeated Tipu Sultan in India and
threads. Sen objected this decision and withdrew from
Napolean Bonaparte in Europe.
the Brahmo Samaj along with his followers. Keshab
51. (a) Dayanand Saraswati founded the Arya Samaj which Chandra Sen and his disciples established the Brahmo
was an organization for teaching the true Vedic religion Samaj of India on 11 th November, 1866 and
and also a social reform movement. He fought for equal Debendranath Tagore grouped themselves into the
rights for all and emphasized on social upliftment of Adi (original) Brahmo Samaj.
women and the so-called lower caste. He denounced 61. (b) Robert Clive established the military and political
superstitions and condemned practices such as animal supremacy of the East India Company in Bengal. Under
sacrifices. the dual Government company will look after fiscal
52. (c) The Saka calendar used as the official civil calendar in matters and all the rest would be dealt by the Indian
the country is the National Calendar of India. The Saka rulers.
calendar, often referred as the Hindu calendar is 62. (d) On the evening of 30 April 1908, Khudiram threw a
originally named as Saka Samvat. There are 12 months bomb on the carriage of Kingsford but it was occupied
in Saka Calendar which are named as Vaisakha,Jyestha not by Kingsford but the wife and daughter of barrister
,Asadha,Sravana, Bhadrapada, Asvina , Kartika Pringle Kennedy were killed instead of Kingsford.
Margasirsa , Pausa, Magh, Phalgura,Chaitra. 63. (a) The Nehru-Liaquat pact was signed between India and
53. (a) The Doctrine of Lapse was introduced by Lord Pakistan in 1950 to resolve the issues of protection of
Dalhousie. The states annexed by the application of minorities.
this doctrine of lapse were Satara, Jaitpur and 64. (a) On 26th October, 1947, Maharaja Hari Singh signed on
Sambalpur, Baghat, Udaipur, Jhansi and Nagpur.Due instrument of accession. Therefore this date became
to this annexation policy discontent simmered among an important day in the history of India.
many sections of Indian society and the largely 65. (c) Muhammad Ali Jinnah drafted the constitution of
indigenous armed forces; these rallied behind the Muslim league 'The green Book'.
deposed dynasties during the Indian rebellion of 1857 66. (a) Amaravati School of Art : In Amaravati, situated in
also known as the Sepoy Mutiny of 1857. the eastern Deccan, Andhra Pradesh, a different type
54. (d) The Brihadeshwara temple, called the Big Temple, is of art form evolved and flourished for nearly six
dedicated to Lord Shiva. It was built by the great Chola centuries commencing from 200-100 BC. Patronized first
King Raja Raja 1 (985 -1012 A.D). It is an outstanding by the Satavahanas and later by the Ikshvakus and
example of Chola architecture. Recognizing its unique also by other groups
architectural excellence, UNESCO has declared it a • The lotus and the purnakumbha motifs are typical
World Heritage Monument. of Amaravati Art expressing auspiciousness and
55. (a) All statements are correct. abundance.
56. (d) On August 16, 1932, the British Prime Minister • White Marble was used in this art and the themes
McDonald announced the Communal Award. The were Buddha's life and Jatakas tales.
Communal Award was basically a proposal on minority • The curly hair of Buddha is a feature that is
representation.Important proposals were as follows: influenced by the Greeks
• The existing seats of the provincial legislatures • The Palaces of Kings, Princes, etc. have got
were to be doubled. prominence
EBD_7367
82
C- History

67. (a) assassination of Scott. But, the revolutionaries,


68. (c) Dadabhai Naoroji was a Parsi intellectualand Indian mistaking J.P. Saunders, an Assistant Superintendent
political and social leader. His book Poverty and Un- of Police, as Scott, killed him instead.
British Rule in India brought attention to the draining 75. (b). The Five Principles of Peaceful Coexistence are a set
of India's wealth into Britain. of principles to govern relations between states. The
69. (b) On 13 April 1919, a crowd of nonviolentprotesters, agreement was signed between India and China on 29
along with Baishakhi pilgrims, had gathered in the April 1954.The five principles of the panchsheel are :
Jallianwala Bagh garden in Amritsar, Punjab to protest • Mutual respect for each other's territorial integrity
against the arrest of two nationalist leaders, Dr. and sovereignty.
Satyapal, Dr. Saifuddin Kitchlew, On the orders of • Mutual non-aggression.
Brigadier-General Reginald Dyer, the army fired on the • Mutual non-interference in each other's internal
crowd for ten minutes, there were 370 dead and 1200 affairs.
wounded. • Equality and cooperation for mutual benefit.
70. (a) On the night of December 16, 1773, Samuel Adams and • Peaceful co-existence.
the Sons of Liberty boarded three ships in the Boston 76. (c) The German philosopher Karl Marx became one of the
harbour and threw 342 chests of tea overboard. This most influential thinkers of the 20th century. Marx
resulted in the passage of the punitive Coercive Acts believed that all historical change was caused by a
in 1774 and pushed the two sides closer to war. series of class struggles between the bourgeoisie
71. (d) Some of the features of Government of India Act 'haves' and the proletariat 'have-nots'. The work of
1919(Montague Chelmsford Reforms) are as follows: Karl Marx caught the attention of members of the
• It introduced Diarchy system. intelligentsia. The first Russian to translate Marx's
• The Government of India Act of 1919, made a works was George Plekhanov. In 1883 he founded the
provision for classification of the central and first Marxist organisation in Russia. He was called the
provincial subjects. father of Russian Marxism.Marks and Engels wrote
• The Indian executive comprised of the Governor several books together.
General and his council. 77. (b) Karl Marx was a German philosopher, economist,
• This act made the central legislature bicameral. sociologist, journalist, and revolutionary socialist.His
The first house which was central legislature, with famous books are The Communist Manifesto(1848)
145 members (out of which 104 elected and 41 and Das Kapital (1867-1894). According to him,History
nominated) was called central Legislative is a succession of struggle between the oppressor
Assembly and second called with 60 members (out and the oppressed classes.
of which 33 elected and 27 nominated) was called 78. (d) Principles of Satyagrah
Council of States. The term of the assembly was • Nonviolence (ahimsa)
fixed 3 years and council 5 years. • Truth - this includes honesty, but goes beyond it
• The act provided for the establishment of a Public to mean living fully in accord with and in devotion
Service Commission in India for the first time. to that which is true
72. (a) Dyarchy, a system of double government was • Not stealing
introduced by the Government of India Act (1919) for • Chastity (brahmacharya) - this includes sexual
the provinces of British India. Dyarchy was introduced chastity, but also the subordination of other
as a constitutional reform by Edwin Samuel Montagu sensual desires to the primary devotion to truth
(secretary of state for India, 1917-22) and Lord • Non-possession (not the same as poverty)
Chelmsford (viceroy of India, 1916-21). • Body-labour or bread-labour
73. (b) Mohammed Ali Jinnah was Indian Muslim politician, • Control of the palate
founder and first governor-general (1947-48) of • Fearlessness
Pakistan. As the first Governor-General of Pakistan, • Equal respect for all religions
Jinnah worked to establish the new nation's • Economic strategy such as boycott of exported
government and policies, and to help settle the millions goods (swadeshi)
of Muslim migrants who had emigrated from the new 79. (d) Rabindranath Tagore was Asia's first Nobel
nation of India to Pakistan after the partition. He is Laureatewho won the Nobel Prize for Literature in 1913
revered in Pakistan as Quaid-i-Azam. for his creation of Gitanjali. The Jallianwala Bagh
74. (b) In 1928, the British government set up the Commission, Massacre happened in Amritsaron 13 April 1919 in
headed by Sir John Simon, to report on the political which Brigadier-General Reginald Dyer ordered
situation in India. The Indian political parties British, Indian Army soldiers to open fire on an
boycotted the Commission, because it did not include unarmed gathering of men, women and children.
a single Indian in its membership, and it met with Thousands of people were killed in this massacre.
country-wide protests. When the Commission visited Tagore renounced the knighthood that he had
Lahore on 30 October 1928, Lajpat Rai led silent march received in 1915.
in protest against it. The superintendent of police, 80. (c) The Indigo revolt of Bengal was directed against British
James A. Scott, ordered the police to lathicharge. He planters who forced peasants to take advances and
became seriously injured and died on 17 November sign fraudulent contracts which forced the peasants
1928.To take avenge of the death of Lala Lajpat Rai, to grow Indigo under terms which were the least
Bhagat Singh and his associates plotted the profitable to them. Digambar Biswas and Bishnu
History C-83
Biswas who organised the peasants into a counter 2. Farmers could use, sell, mortgage, bequeath, and
force to deal with the planters lathiyals (armed lease the land as long as they paid their taxes. In
retainers). In April 1860 all the cultivators of the Barasat other words Ryotwari system gave a proprietary
subdivision and in the districts of Pabna and Nadia right upon the landholders.
resorted to strike. They refused to sow any indigo. It 3. IF they did not pay taxes, they were evicted
led to the appointment of an Indigo Commission in 4. Taxes were only fixed in a temporary settlement
1860 by the government by which some of the abuses for a period of thirty years and then revised.
of Indigo cultivation was removed. 5. Government had retained the right to enhance land
81. (c) British did not adopt Free trade policy to exploit the revenue whenever it wanted
Indians economically. 6. Provided measures for revenue relief during
82. (a) The Indian Rebellion of 1857 began as a mutiny of famines but they were seldom applied in real life
sepoys of the East India Company's army on 10th May situation.
1857, in the cantonment of the town of Meerut. 90. (d) Ram Manohar Lohia- An Indian politician and activist
83. (b) For effective administration the vast territory of the who was a prominent figure in socialist politics and in
Rajya was divided as vassal states and provinces ruled the movement toward Indian independence. In 1934
directly by the king's direct representatives. While the Lohia became actively involved in the Congress
vassal states were administered by the Nayakas or Socialist Party (CSP), Lohia wrote his PhD thesis paper
Samantas, the territories directly under the king were on the topic of Salt Taxation in India,focusing on
called Rajyas, Mandalas and sometimes Chavadis. Gandhi's socio-economic theory. "Where caste
84. (b) Kitab-Ul-Hind was written by Al Biruni . Ibn Batuta is prevails, opportunity and ability are restricted to ever-
known for his extensive travels, accounts of which narrowing circles of the people",these words from Ram
were published in the Rihla . In 1332, Ibn Battuta Manohar truly project what he thought of ever existent
decided to go to India. He was greeted openheartedly caste system in India.
by the Sultan of Delhi. There he was given the job of a 91. (c) The Non-Aligned Movement (NAM) is a group of
judge. He stayed in India for a period of 8 years and states which are not formally aligned with or against
then left for China. any major power bloc. The organization was founded
85. (d) Ruin of Indian Feudalism was not a result of British in Belgrade in 1961.
colonial rule in India. The principles of The Non-Aligned movement were:
86. (a) Santhal Rebellion (January 3, 1856 ) was a native • Mutual respect for each other's territorial integrity
rebellion in present day Jharkhand in eastern India and sovereignty
against both the British colonial authority and upper • Mutual non-aggression
caste zamindari system by the Santalpeople. • Mutual non-interference in domestic affairs
The Indigo revolt (or Nilbidroha) was a peasant • Equality and mutual benefit
movement and subsequent uprising of indigo farmers • Peaceful co-existence
against the indigo planters that arose in Bengal in 1859. 92. (c) Governor General was the sole authority to decide and
Sanyasi means Ascetics and Sadhus. In this rebellion, confer such powers upon central and state legislature.
the Sanyasis or Fakirs including the Hindu and Muslim 93. (c) Mahmud Gawan was a prime minister in the Bahamani
ascetics stood up against the oppressive tax collection Sultanate of Deccan about twenty three years from
post Bengal famine of 1770. 1458 to 1481 A.D.
Birsa was an Indian tribal freedom fighter and a folk Gawan introduced the following administrative
hero, who belonged to the Munda tribe. (1875-1900) reforms:
87. (b) (i) The existing four provinces were divided into two
88. (b) Noncooperation movement, (September 1920-February each and created eight 'Tarafs'.
1922), was organized by Gandhiji to induce the British (ii) He reduced the powers of the governors of these
government of India to grant self-government, or 'Tarafs' (Provinces).
swaraj, to India. Noncooperation was agreed to by the (iii) In each of these eight 'Tarafs' some areas were
Indian National Congress at Calcutta in September kept reserve for the control of the Central
1920 and launched that December. The movement was Government and for their administrative control,
to be nonviolent and to consist of the resignations of officials were appointed by the Centre.
titles; the boycott of government educational (iv) Orders were issued by him for placing one fort in
institutions, the courts, government services, foreign each province under the charge of the governor
goods, and elections; and the eventual refusal to pay and others under the control of military
taxes. commanders appointed by the Sultan.
89. (d) The Ryotwari system, instituted in some parts of British (v) He put a check on the powers of the nobles.
India, was one of the two main systems used to collect (vi) He introduced a fair system of justice.
revenues from the cultivators of agricultural land. 94. (b)
Features of Ryotwari System 95. (d) Reforms of Raja Ram Mohan Roy:
1. Government claimed the property rights to all the • Crusaded against social evils like sati, polygamy
land, but allotted it to the cultivators on the and child marriage etc.
condition that they pay taxes. In other words, it • Demanded property inheritance rights for women.
established a direct relation between the • Supported induction of western learning into
landholder and the government. Indian education.
EBD_7367
84
C- History

He is best known for his efforts to abolish the practice 103. (b) This statement was made by Clement Attlee.He was a
of sati, the Hindu funeral practice in which the widow British Labour Partyleader and prime minister of Britain
was compelled to sacrifice herself on her husband's during the period of 1945-51.He was known for the
funeral pyre in some parts of the then Bengal. granting of independence to India.
96. (d) The proposed conditions of Gandhi-Irwin pact were 104. (b) The Pillar Edicts are divided into:
as follows: Major Rock Edicts: 14 Edicts (termed 1st to 14th) and
• Discontinuation of the civil disobedience 2 separate ones found in Odisha Minor Rock
movement by the Indian National Congress Inscriptions:
• Participation by the Indian National Congress in Minor Rock Edicts, the Queen's Edict, Barabar Caves
the Round Table Conference inscriptions and the Kandahar bilingual inscription.
In these Edicts there were three South Indian states
• Withdrawal of all ordinances issued by the British
described as Cholas, Pandayas, Cheras.
Government imposing curbs on the activities of
105. (b) Causes of the Revolution
the Indian National Congress • The end of medieval structures changed economic
• Withdrawal of all prosecutions relating to several relationships and allowed for change.
types of offenses except those involving violence • A higher population because of less disease and
• Release of prisoners arrested for participating in lower infant mortality allows for a larger industrial
the civil disobedience movement workforce.
• Removal of the tax on salt, which allowed the • The Agricultural Revolution frees people from the
Indians to produce, trade, and sell salt legally and soil, allowing - or driving - them into cities and
for their own private use manufacturing.
97. (c) Option c is correct answer. • Proportionally large amounts of spare capital were
98. (a) Kabir did not believe in idol worship. He was also available for investment.
against the performance of rituals and superstitions • Inventions and the scientific revolution allowed
or pilgrimage to the so-called holy places. for new technology to increase and cheapen
99. (c) production.
100. (b) First Battle of Panipat (1526) was fough between two • Colonial trade networks allowed the import of
mega-powers- Babur, then ruler of Kabul and Ibrahim materials and the export of manufactured goods.
Lodhi, king of Delhi Sultanate. It was fought near • The presence of all the required resources closed
Panipat (present day Haryana). Babur won the battle together, such as coal near iron.
and established the Mughal Empire.Second Battle of • Culture of hard work, risk taking and the
Panipat (1556) was fought between Akbar (Ruler of development of ideas.
Mughal Dynasty) and Muhammad Adil Shah (ruler of • Demand for goods.
106. (a) The Indian Penal code was drafted in 1860 on the
Pashtan Suri Dynasty), along with his Prime Minister
recommendations of first law commission of India
Hemu. Third Battle of Panipat (1761) was fought established in 1834 under the Charter Act of 1833 under
between the Afghans and the Marathas. The battle the Chairmanship of Thomas Babington Macaulay. It
lasted for two months which ultimately resulted in the came into force in British India in 1862. Indian Penal
defeat of Marathas and end of their dominance in India. Code (IPC) is the main criminal code of India. It is a
101. (b) The Civil Disobedience Movement was formed in the comprehensive code intended to cover all substantive
year 1930 and was one of the most important phases aspects of criminal law.
in the Indian National Movement. The main ideology 107. (c) The American Revolutionary War (1775-1783), was an
behind the Civil Disobedience Movement was to defy armed conflict between the Kingdom of Great Britain
the laws made by the British. The British government and thirteen of its former North American colonies,
was pressurized by the Indian National Congress to which declared themselves the independent in 1776.
accept the Nehru Report (1928) as it is. The Calcutta 108. (a) The Fourth Anglo-Mysore War (1798-1799) was a war
Session of the INC held in 1928 warned the British in South India between the Kingdom of Mysore and
government that it would start a Civil Disobedience the British East India Company under the Earl of
Movement if India was not granted the dominion Mornington. In the battle, Tipu Sultan was shot and
status. 2nd March 1930 is remembered as one of the killed.
important days in Indian history as the Civil 109. (c) Vivekananda did not criticise his country men for
Disobedience Movement was launched on that day. having last touch with the outside world.
The launch triggered off with a Dandi Salt March where 110. (b) Lord Dalhousie was keen on annexing the kingdom of
the British Salt was broken. Awadh. But the task presented certain difficulties
102. (a) The Government created commissions and because the Nawab of Awadh had been a British ally
since the Battle of Buxar and had been most obedient
commissions on one another with an undeclared
to the British over the years. Lord Dalhousie hit upon
motive of creating mutual apprehensions and
the idea of alleviating the plight of the people of Awadh
mistrusts. It was the time when the top leaders including and accusing Nawab Wajid Ali on the ground of
Gandhi were failed to understand the political misgovernance and annexed his state in 1856.
implications of his extensions of support to the cause 111. (d) Doctrine of lapse, in Indian history, formula devised
of Khilafat. Practically, the leaders of Khilafat needed by Lord Dalhousie, governor-general of India (1848-
support of Gandhiji only for a defined particular 56), to deal with questions of succession to Hindu
purpose. Indian states.
History C-85
According to 'Doctrine of Lapse', if any ruler of a 123. (d) Jatakas are the stories that tell about the previous lives
protected state died without a natural heir, the states' of the Buddha, in both human and animal form.
authority will pass to the English East India Company. 124. (c) Mahavira was a teacher of the religion of Jainism. He
Annexation in the absence of a natural or adopted heir lived in India. His followers believed that he was the
was enforced in the cases of Satara (1848), Jaitpur and 24th in a line of great teachers. These teachers were
Sambalpur (1849), Baghat (1850), Chota Udaipur (1852), called tirthankaras.
Jhansi (1853), and Nagpur (1854). 125. (b) Gandhi's vision of a free India was not a nation-state
112. (c) The First Anglo-Maratha War (1775-1782) was fought but a confederation of self-governing, self-reliant, self-
between the British East India company and Maratha employed people living in village communities,
Empire in India. The war began with the Treaty of Surat deriving their right livelihood from the products of
their homesteads.
and ended with the Treaty of Salbai.
126. (b) As per the Permanent Settlement system, the
113. (d) Jean-Baptiste Colbert, founded the French East India Zamindars who formerly collected revenues were
Company in 1664 with the intention of rivalling the "recognized" as Landlords and the ownership of the
success of the English and the Dutch in India.Their Land was made hereditary. This means that now
trading company was heavily dependent on the French onwards there would be no auctioning. The son of
government. Zamindar would be a Zamindar.
114. (a) All statements are true. 127. (c) The British tried to curb Bengali influence on the
115. (c) Anizham Thirunal Veerabaala Marthanda Varma was nationalist movement and also introduced a new form
king of Travancore from 1729 until his death in 1758. of division based on religion to create challenges for
He was the only Indian King to beat the European the Indian National Congress, which was slowly
armed force at the 1741 Battle of Colachel against the becoming the main opponent to British rule.
Dutch. 128. (a) Dadabhai Naoroji was the first man to say that internal
116. (d) Nadir Shah, was the founder of the Afsharid dynasty. factors were not the reasons of poverty in India but
Nadir Shah led his army which consisted of 80,000 poverty was caused by the colonial rule that was
armymen. He placed 3,000 of his army men in the front draining the wealth and prosperity of India. In 1867,
as a clear line of defence against the Mughal army. Dadabhai Naoroji put forward the 'drain of wealth'
Nadir Shah advanced towards India in 1738. At the theory in which he stated that the Britain was
battle of Karnal on 13 February 1739, Muhammad Shah completely draining India. He mentioned this theory
was defeated and surrendered. The whole city of Delhi in his book Poverty and Un-British Rule in India.
was destroyed, looted, plundered and ruined by the 129. (b) In February March 1918, there was a situation of
army of Nadir Shah. Nadir Shah took with him the conflict between the Gujarat Mill owners and workers
on the question of Plague Bonus of 1917. The Mill
Peacock throne built by Shah Jahan. He also took the
Owners wanted to withdraw the bonus while the
legendary "Koh-i-noor" diamond.
workers demanded a 50% wage hike. The Mill Owners
117. (c) Privy Purse in India was a payment that was made to were willing to give only 20% wage hike. In March
the royal families of the formerprincely states of India. 1918, under the leadership of Gandhi, there was a strike
The Privy Purse was created as part of the agreements in the cotton mills. In this strike Gandhi used the
made by them to merge with Union of India in the year weapon of Hunger strike. The result was that the strike
1947. was successful and the workers got a 35% wage
118. (c) For the first time in 1934,the idea of a Constituent increase.
Assembly for India was put forward by M. N. Roy. 130. (a) The Champaran Satyagraha of 1917 was Mahatma
Under the cabinet mission plan of 1946, elections were Gandhi's first Satyagraha. The peasants (bhumihars)
held for the first time for constituent Assembly. The of the Champaran and other areas of North Bihar were
Constitution of India was drafted by the constituent growing the Indigo under the tinakathia system. Under
assembly and it was set up under the cabinet Mission the tinakathia system the peasants were bound to plant
plan on 16 May 1946. The members of the constituent 3 out of 20 parts of his land with indigo for his landlord.
assembly were elected by the Provincial assemblies One local peasant leader Rajkumar Shukla had invited
by method of single transferable vote system of Mahatma Gandhi to visit Champaran. The very
proportional representation. important outcome of the movement was the enactment
119. (c) Simon Commission was appointed under the of Champaran Agrarian Act assented by Governor
chairmanship of Sir John Simonin November 1927 by General of India on 1st May, 1918.
the British government to report on the working of 131. (c) The Viceregal Lodge, also known as Rashtrapati Niwas,
the Indian constitution established by the Government is located on the Observatory Hills of Shimla, Himachal
of India Act of 1919. The commission consisted of Pradesh,It was built during the regime of Lord Dufferin.
Its construction started in 1880 and was completed in
seven members. None of the Indians was appointed in 1888.The cabinet Mission 1946 was held in the The
the commission. Viceregal Lodge. The Planting of gardens were done
120. (d) The ironpillar near Qutub Minar highlights ancient by Marquis Lansdowne.
India's achievements in metallurgy. The pillar is made 132. (a) On the basis of the report submitted by Sir John Shore
of 98 per cent wrought iron and has stood 1,600 years Cornwallis introduced land revenue settlement for ten
without rusting or decomposing. years with Zamindars in 1789 but the same was made
121. (a) permanent in 1793 by the proclamation of Cornwallis
122. (b) Rig Vedic Aryans were not acquainted to the use of and it was known as Permanent Land Revenue
Iron. settlement.
EBD_7367
86
C- History
The basic features of the settlement were as follows: 145. (b) Triple Alliance consisted of British, Nizam and
1. The Zamindars were made hereditary owners of Marathas against Tipu sultan in 1791-92.
the land under their possession. They and their 146. (a) The Nizam-ul-Mulk of Hyderabad was a monarch of
successors exercised total control over lands. the Hyderabad State, did not support the revolt of
2. The Zamindars could sell and purchase lands. 1857.
3. The state had no direct contact with the peasants. 147. (d) The Theosophical Society was officially formed in New
4. The company's share in the revenue was fixed York City, United States, in November 1875 by Helena
permanently with the Zamindars. Petrovna Blavatsky, Henry Steel Olcott, William Quan
133. (d) Prarthana Samaj was founded by M.G. Ranade and Judge and others.
R.G. Bhandarkar in 1867 with an aim to make people 148. (d) Considered as Father of Indian National Movement;
believe in one God and worship only one God. The
Founded "Deccan Education Society" to impart quality
main reformers were the intellectuals who advocate
reforms of the social system of the Hindus. He never education to India's youth; was a member of the
demanded abolition of untouchability. Municipal Council of Pune, Bombay Legislature, and
134. (c) Ram Prasad Bismil was the famous freedom fighter an elected 'Fellow' of the Bombay University; formed
who was involved in the historic Kakori train robbery. Home Rule League in 1916 to attain the goal of Swaraj.
He was born in 1897 at Shahjahanpur,Uttar Pradesh. The Age of Consent Act, 1891 was a legislation enacted
On 9th August, 1925, Ram Prasad Bismil along with in British India on 19 March 1891 which raised the age
his fellow followers looted the money of the British of consent for sexual intercourse for all girls, married
government from the train while it was passing through or unmarried, from ten to twelve years in all
Kakori, Lucknow. Except Chandrashekhar Azad, all jurisdictions, its violation subject to criminal
other members of the group were arrested. Ram Prasad prosecution as rape.It was opposed by Hindu
Bismil along with others was given capital punishment. nationalists including Bal Gangadhar Tilak.
This great freedom fighter of India was executed on 149. (d) The site of Harappa was located on the banks of river
19th December, 1927. Ravi.
135. (c) The third Carnatik war- Treaty of Paris (1763) 150. (c) The Vernacular Press Act in 1878 was passed by Lord
The Third mysore war- Treaty of Srirangapatnam(1792) Lytton and repealed by Lord Ripon. The suppression
The first Maratha war- Treaty of salbai(1782) of Indian language newspapers under the Vernacular
The first Anglo sikh war- Treaty of Lahore (1846) Press Act of 1878 was caused by the Criticism of
136. (d) Kanishka, was the emperor of the Kushan dynasty in inhuman approach of English officials towards the
127-151 A.D;famous for his military, political, and victims of the famine of 1876-77 and the reactionary
spiritual achievements. Some of the scholars in the policy of Lytton.
Court of Kanishka were Parsva, Vasumitra, Asvaghosa, 151. (d) Factory act to improve the condition of Indian labour
Nagarjuna, Charaka and Mathara..
137. (d) Mansehra and Shahbazgadhi inscriptions of ashoka was not an initiativeof reforms of Dalhousie.It is
are written in Aramaic language, Kharoshthi script. associated with lord Ripon.
Shahbaz Garhi is an historic site located in Mardan 152. (d) Sir Thomas Roe was sent by James I in 1615. He
District in the North West Frontier Province of Pakistan. reached Surat in September 1615 and proceeded to
Mansehra is located in Mansehra District, Khyber- Ajmer where Jahangir was. He remained at the court of
Pakhtunkhwa province of Pakistan. Jahangir from 1615 to 1618.Father Monserrate travelled
138. (c) During Jorwe culture, one can get archaeological with Akbar on his journey to Kashmir.
evidence of burial system at Daimabad. 153. (d) Industrial Revolution in its early phase (from 1760 c to
139. (c) 1850 c) was largely dependent for its success on
140. (a) organization of labour under factory system.
141. (d) Satara was first annexed by lord Dalhousie under the 154. (c) Ashtadhyayi, (Sanskrit grammar) written in the 6th to
Doctrine of lapse. The Doctrine of Lapse was an 5th century BCE by the Indian grammarian Panini
annexation policy purportedly devised by Lord 155. (c) The word Dhamma is the Prakrit version of the Sanskrit
Dalhousie, who was the Governor General for the East word Dharma. The scholars believe that the Dhamma
India Company in India between 1848 and 1856. By which Ashoka tried to propagate was not simply the
applying the doctrine of lapse, Dalhousie annexed the teachings of Buddhism, but it contained the noblest
States of Satara (1848 A.D.), Jaitpur (1849 A.D.), ideas of humanism which are the essence of all
Sambhalpur (1849 A.D.), Bahat (1850 A.D.), Udaipur religions.
(1852 A.D.), Jhansi (1853 A.D.), and Nagpur (1854 156. (b) Kutagarashala is a place where intellectual debates
A.D.). among Buddhist mendicants took place.
142. (d) Dholavira was known for water management.
157. (b) During the rule of Maurya Dynasty Silver and Copper
143. (c) Hieun Tsang was a Chinese Buddhist monk who visited
the kingdom of Harshvardhan and Kumar Bhaskar were used for minting coins.
Varma. 158. (d) His charges against the company were mainly
144. (d) The cabinet Mission plan for India envisaged a Union threefold. Firstly, that they strengthened the
of State. Cabinet Mission of 1946 to India aimed to fortification around the Fort William without any
discuss and plan for the transfer of power from the intimation and approval; secondly, that they grossly
British Government to Indian leadership, providing abused the trade privileges granted to them by the
India with independence. The mission consisted of Mughal rulers, which caused heavy loss of customs
Lord Pethick-Lawrence, the Secretary of State for India, duties for the government; and thirdly, that they gave
Sir Stafford Cripps. shelter to some of his officers, for example Krishnadas,
History C-87
son of Rajballav, who fled Dhaka after misappropriating 169. (b) Chalcolithic Cultures were the first to paint their
government funds. Hence, when the East India pottery.
Company started further enhancement of military 170. (d) The polity of the Harappan people as derived from the
preparedness at Fort William in Calcutta, Siraj asked material evidence was a theocratic-unitary.
them to stop. The Company did not heed his directives, 171. (d) Bal Gangadhar Tilak interpreted Gita as the scripture
so Siraj-ud Daulah retaliated and captured Kolkata of Karmayoga. Ram Manohar Lohia did not write any
(Shortly renamed as Alinagar) from the British in June commeutaries on Geeta.
1756. 172. (c) V.V GIRI was the fourth president of India from 24
159. (b) Gandhiji had full trust in their bonafides and considered August 1969 to 24 August 1974.He became president
them most valued and respected leaders. of the All India Trade Union Congress twice but was
160. (d) The Silk Route or Silk Road refers to a network of not the founding member of AITUC.
ancient trade routes connecting Asia, Europe and 173. (d) Sodium Cloride (Salt) was an important symbol in our
Africa. Extending more than 6,500 kms, the Silk Route struggle for freedom, because Mahatma Gandhi tried
was majorly used to transport Chinese Silk to Europe to break salt law by organizing dandi March.
through Central Asia from 2nd Century BC. The ancient 174. (a) The important occasion of Chandragupta Maurya's
Silk Road helped to integrated the old Chinese, Indian, reign was the forecast of a twelve years famine. At
Persian, Arabian, ancient Greek and Roman cultures that time Acharya Bhadrabahu was the head of the
and promoted the exchange of the Western and
Jain church. According to the Jain text Rajvaliya Katha,
Oriental civilizations. Half of the Silk Road, which
the king saw sixteen dreams. He requested Bhadrabahu
winded along between Xi'an to the east bank of the
Mediterranean, was located in Xinjiang. Xinjiang was to interpret the dreams. The latter explained all the
a place where the ancient Western and Oriental cultures dreams and according to the last one, he predicted a
met and many famous historical people visited. It was famine of twelve years. Chandragupta, who was a
the world's first information superhighway. follower of Bhadrabahu, gave up his throne. He finally
161. (c) Gulbadan Begum (daughter of Babur) was the author ended his life following the Jain fashion.
of Humayun Nama. 175. (b) Mahavira was not the founder and author of Jain
162. (d) Jajmani system is a socio-economic institution of the religion but simply a reformer. This sect of the
pre-industrial self-subsistent village economy. The Nirgranthas was an important sect at the rise of
term 'jajmani' is derived from a Vedic term 'Yajman' means Buddhism. This may be inferred from the fact that they
a patron who employs the services of a Brahmin for are frequently mentioned in the pitakas as opponents
the performance of sacrifices or other religious of Buddha and his disciples. This conclusion is further
ceremonies. The jajmani system is essentially based supported by another fact. Makkhali Gosala, a
on caste system. contemporary of Buddha and Mahavira divided
Advantages of jajmani system mankind into six classes, of these the third class
The occupation being hereditary, it provides security contained the Nirgranthas. Hence, statement II is not
of occupation to the kamin. the correct explanation of statement I.
It provides economic security as the 'jajman takes care 176. (b) Both statements are true however, statement II is not
of all the needs of kamin. the correct explanation of statement I.The Barabar
It strengthens the relationship between jajman and Caves are the oldest surviving rock-cut caves in India.
kamin(Praja), which is more personal than economic. They are located in the Bela Ganj Block of Gaya district,
163. (b) In Mughal period Cash crops were called jins-i-ala. Bihar. The caves were used by ascetics from the Ajivika
164. (c) Sufismismystical Islamic belief and practice in which sect. Lomas Rishi caves are the arch-like shape facade
Muslims seek to find the truth of divine love and of Lomas Rishi Caves, imitate the contemporary timber
knowledge through direct personal experience of God. architecture. Sudama caves were dedicated by
165. (a) The village headman became more important in Gupta Mauryan Emperor, Ashoka in 261 BC.
period and no land transaction could be effected 177. (b) Both the statements are true but statement II is not the
without the headman's consent. correct explanation of statement I.Gandhara School
166. (c) The responsibility for all finance and management of was based on Greco-Roman norms . It is also known
Jagir and Inam Lands rested with Wazir. as Graeco-Buddhist School of art. The Buddhist
167. (c) Kanishka was the first Indian ruler who has been sculpture produced by the Kushans in the Swat valley
shown in images as wearing a dress akin to trousers. was referred to as Gandharan, and was influenced by
168. (c) Shivaji trained his soldiers in the guerilla warfare. The Graeco-Roman elements . The ancient kingdoms of
guerilla warfare was very suitable in the geographical Udyana (Swat) and Gandhara (Peshawar ) were formed
location of most territories in Maharashtra. He believed by the Kushans, who were of Chinese origin and took
in the surprise attacks on the enemy, killed or looted it over the area in the first century BC . Under the king
and disappeared into the forest. which leveraged Kanishka, who ruled from around 100 AD, Buddhism
strategic factors like geography, speed, and surprise flourished in Gandhara; at one point there were 1,400
and focused pinpoint attacks to defeat his larger and monasteries in the lower Swat Valley alone.
more powerful enemies.Shivaji himself constructed 178. (a) In order to keep powerful people happy and to collect
about 15-20 new forts (including key sea forts like better revenue, Lord Cornwallis introduced the
Sindhudurg), but he also rebuilt or repaired many Permanent Settlement. As per permanent system, rajas
strategically placed forts to create a chain of 300 or and taluqdars were recognized as zamindars. The
more, stretched over a thousand kilometres across the zamindars were supposed to collect the land revenue
rugged crest of the Western Ghats. from the peasants.
EBD_7367
88
C- History
179. (b) Goshala Maskariputra was the founder of Ajivika 192. (d) India exported pearls, precious stones, cloth, perfumes,
Sect.He was the contemporary of Mahavir. and even spices, indigo, silver, tin, lead, silk and horses.
remained his companion before the enlightenment of 193. (b) Sher Shah did not introduce swiftly moving artillery.
Mahavira. Sher Shah effected many military reforms in order to
180. (a) Ziyarat is used to refer to a form of pilgrimage to sites save the country from foreign invasions and internal
associated with Muhammad and his family members revolt. With the object of organising the Army, Sher
and descendants. Sites of pilgrimage include mosques, Shah began to pay the soldiers their salary in cash;
graves, battlefields, mountains, and caves. maintained their respective roles; brandished the
181. (c) Sutanuti, Gobindapur and Kalikata, the three villages horses etc.
which were merged to form the city of Kolkata (formerly 194. (c) Rajtarangini was written by Kalhan(Kashnmiri
Calcutta) in India. Job Charnock, an administrator with Brahman) in Sanskriin 12th century CE.It records the
the British East India Company is traditionally credited heritage of Kashmir, but 120 verses of Rajtarangini
with the honour of founding the city. describe the misrule prevailing in Kashmir during the
182. (a) It is founded in 1916 by Begums Rokeya. It claimed to reign of King Kalash, son of King Ananta Deva of
represent the interests of all Muslim women.Besides Kashmir.
working for the social and educational uplift of 195. (d) Statement ll is correct because vaisyas and shudras
Muslimwomen the Anjuman's main contribution was refused to perform producing functions or pay taxes
to popularize a new style of burqa pattern on the which resulted in the weakening of Brahminical social
Turkish model. order.
183. (d) Abu Abdullah Muhammad Ibn Battuta, better known 196. (b)
simply as Ibn Battuta (1304-circa 1377 AD) was a Berber 197. (a) Both the statements are true and statement ll is correct
Muslim scholar and traveler, who was born in Tangier, explanation of statement l. The Russian revolution
Morocco. He travelled almost 30 years. In that period, inspired the involvement of Indian working class in
he covered nearly the entirety of the known Islamic the Non Cooperation movement.
world and beyond: from North Africa, West Africa, 198. (c) The Indian national movement was against the British
Southern Europe and Eastern Europe.
rule in India.It resulted in our Independence from the
184. (c) Malik Kafur was the slave general of Sultan Alauddin
Britishers but it did not resolve the tribal issues.
Khilji who had won for him the Deccan territories of
199. (a) Both statements are true and statement ll is correct
Warangal, Devgiri, Madura and Dwarasamudra, etc.
Malik Kafur was originally a Hindu slave who fell into explanation of statement I Britain ruined India's
the hands of the Muslims at Cambay after the conquest previous economy and industries as a result of Britain
of Gujarat. Alauddin's general Nusrat Khan had paid forcing them to import their goods from Britain and
1,000 dinars to buy him and that is why Malik Kafur is not buy their own goods from India. Industries such
also known as the "Hazardinari. as the handmade cloth industry, clock-making, metal,
185. (b) English began to fortify Calcutta as part of the and carpentry industries, fell because cloth was made
preparations of war with the French without the of better quality for cheaper in Britain because of mass
permission of the Nawab. Siraj-ud-Daula asked both production.
the French and the English to pull down their 200. (a) The poems belonging to the Sangam literature were
fortifications and considered them an attack on the composed by Tamil poets.
Nawab’s sovereignty. The French Company complied 201. (b) Karaikal Ammaiyar was one of the three women
but the English refused to obey the orders. amongst the sixty three Nayanmars, and was one of
186. (a) The highly polished monolithic Ashoka pillar usually the greatest figures of early Tamil literature.
with small black spots were quarried from the Chunar 202. (c) Young Benga Movement was launched under the
in Mirzapur district of U.P. influence of a radical teacher Derezio. It was very radical
187. (a) Varahamihira was an Indian astronomer, mathematician, movement.
and astrologer who lived in Ujjain. He wrote Brihat 203. (d) The word Swarajya was first coined by Bal Gangadhar
Samhita which covers astrology, planetary movements, Tilak. He is known for his quote "Swarajya is my
eclipses, rainfall, clouds. birthright, and I shall have it!"
188. (b) In his book the “Rise of Marathas” M.G. Ramade 204. (c) The term Satyagraha was coined by Gandhi Ji.He used
explained about Maratha rule and its polity. it in South Africa for Indian rights. And in India it was
189. (b) The causes of the success of American revolution first experimented in Champaran District of Bihar.
include spirit of freedom of American people and less 205. (b) A guillotine is an apparatus designed for carrying out
knowledge about American continent by the Britishers executions by beheading. The device is best known
etc. for its use in France, in particular during the French
190. (c) Above statement was made in the context of Quit India Revolution.
Movement by the Britishers. 206. (b) The Tea Party was the culmination of a resistance
191. (c) Mansabdari was a system of army and civil services movement throughout British America against the Tea
introduced by Akbar in place of the Jagirdari system. Act, which had been passed by the British Parliament
The 'Mansab' is an Arabic word meaning rank or in 1773. The Boston Tea Party was a political protest
position or status of a person. During the early reign by the Sons of Liberty in Boston, on December 16,
of Akbar, the lowest grade was ten and the highest 1773. The demonstrators destroyed an entire shipment
was 5000. Akbar introduced the rank of 'Zat' and 'Sawar' of tea sent by the East India Company, in defiance of
in the Mansabdari system. the Tea Act of May 10, 1773.
History C-89
207. (b) The Gandhi-Irwin Pact was signed by Mahatma 220. (c) The first session of INC was held in Bombay. Womesh
Gandhi and Lord Irwin on 5 March 1931 before the Chandra Bonnerjee was the first President of the INC.
second Round Table Conference in London.One of It was held from 28-31 December 1885, and was
the proposed conditions is Discontinuation of the civil attended by 72 delegates.
disobedience movement by the Indian National 221. (d) Ignatius of Loyola founded the society.He composed
Congress. the Spiritual Exercises to help others follow the
208. (a) Rishabha- 1st Tirthankara out of 24. Parshvanatha- teachings of Jesus Christ.
23rd tirthankara. Mahavira 24th tirthankara 222. (b) The Theosophical Society was officially formed by
209. (a) Sher Shah divided the entire Kingdom in 47 divisions Helena Petrovna Blavatsky, Henry Steel Olcott, William
and called them "Sarkars". These Sarkars were divided
Quan Judge in New York in November 1875. Allan
into Parganas. Every Pargana was under a Shikqdar,
who looked into the law and order of his Pargana. Octavian Hume was one of the founders of the Indian
210. (b) University of Nalanda was set up by Gupta king National Congress.
Kumargupta I. 223. (d) At the outbreak of the World War II, the Viceroy
211. (d) The Fourth Buddhist Council was held at Kundalvana, proclaimed India's involvement without prior
Kashmir in 72 AD under the patronage of Kushan king consultations with the main political parties. When
Kanishka and the president of this council was Congress demanded an immediate transfer of power
Vasumitra, with Asvaghosa as his deputy. This council in return for cooperation of the war efforts, the British
distinctly divided the Buddhism into 2 sects- Mahayan government refused. As a result Congress resigned
& Hinayan. from power in all the provinces.
212. (a) Bhoga included taxes of fruits, wood, foodgrains etc 224. (b) The Kanchi Kailasanathar temple is dedicated to the
to the king. They were collected by hereditary chiefs. Lord Shiva. It is the oldest structure in Kanchipuram,
213. (d) According to Angunttara Nikaya(Budhist text) and located in Tamil Nadu.'Lingaraj' temple is dedicated to
Bhagavati sutra (jain text) there were 16 mahajanapadas Lord Shiva. It is also said that the temple was
in ancient India. constructed by Jajati Keshari, the SomaVanshi King.It
214. (c) The Battle of Buxar was fought between British East is located in Bhubaneswar, Orissa. Kandariya Mahadev
India Company and the combined forces of Mir Qasim, Temple is located in Khajuraho in Madhya Pradesh,
the Nawab of Bengal, Shuja-ud-Daulah, the Nawab of Khajuraho was once the religious capital of the
Awadh and Shah Alam II, the Mughal Emperor on 23rd
Chandela Rajputs. Dilwara Jain Templesare located at
October, 1764. Shah Alam II, signed the Treaty of
Allahabad that secured Diwani Rights for the Mount Abu,Rajasthan and were built by Vastupal
Company to collect and manage the revenues of almost Tejpal.These temples were built between 11th to 13th
100,000,000 acres (400,000 km2) of real estate, which century AD. Dilwara Temple complex consists of five
helped establish the English rule. major sections or temples devoted to five Jain
215. (d) The mode of worship was worship of the elements like trithankaras(saints): Shri Mahaveer Swami Temple, Shri
fire and rivers, worship of heroic gods like Indra, ParshavNath Temple, Shri Rishabdaoji Temple, Shri
chanting of hymns and performance of sacrifices. They NemiNathJi Temple.
were not associated with temples. 225. (a) The widespread availability of coal and iron was a
216. (c) Ashoka's only inscribed stone portrait was found at significant factor in Europe's Industrial Revolution of
kanaganahalli, near Bhima river In Gulbarg district, the late 18th and early 19th centuries.
Karnataka. 226. (c) Brihadeshwara Temple (Raja Rajeswara Temple) was
217. (a) Hind Swaraj- Gandhi ji's views on Indian Home Rule builtby emperor Raja Raja Chola I in 1010 AD.
My Experience with Truth- Autobiography of Gandhiji Meenakshi Temple located on the southern bank of
He deployed satyagraha in the Indian independence the Vaigai River ( Madurai, Tamil Nadu). It is dedicated
movement and also during his earlier struggles in South to Parvati known as Meenakshi.Srirangam temple is
Africa for Indian rights and social Justice. one of the most illustrious Vaishnava temples in South
Bhagavad Gita- Spiritual Lives of Common people. India located on an island on the bank of Cauvery
218. (d) He was not supporter of Indian national Congress.He
river.The Durga temple is located in Aihole, Karnataka.
was one of those early pioneers who recognized the
critical role of education for the empowerment of the It was built by Chalukyas between the 7th and the 8th
poor and backward Muslim community. In more than century.
one ways, Sir Syed was one of the greatest social 227. (d) Al-Biruni wrote the book Kitab-Al-Hind in 11th century
reformers and a great national builder of modern India. in Turkish language.Ibn Batuta is known for his
So with the aim of promoting social and Economic extensive travels, accounts of which were published
development of Indian Muslim community, Syed in the Rihla (lit. "Journey"). Over a period of thirty
Ahmad Khan founded the Muhammadan Anglo years, Ibn Battuta visited most of the known Islamic
Oriental College (Now Aligarh Muslim University) in world as well as many non-Muslim lands.Gulbadan
1875. Begum, the daughter of Mughal emperor Babar, wrote
219. (a) B G Tilak setup the Home Rule League at the Bombay the Humayun Nama, the account of the life of her half-
Provincial conference held at Belgaum in April 1916. brother, Humayun. Abdul Hamid Lahori a court
Narasimha Chintaman Kelkar (N. C. Kelkar) was historian of Shah Jahan wrote the book Padshahnama
associated with Shikshana Prasarak Mandali in Pune also referred as Badshahnama. In this book Shah
established in 1904 Jahan's first twenty years of reign were described.
EBD_7367
90
C- History

228. (c) Rajatarangini points out that the scarcity of rice 'Precepts of Jesus' in which he tried to separate the
invariably resulted in disastrous famine. This clearly moral and philosophic message of the New Testament,
proves that rice was the principal food of the people which he praised, from its miracle stories.
and Utpala Saka was a kind of wildgrowing herb of 240. (b) The first steps in giving a modern education to girls
bitter taste generally taken by common people. were taken by the missionaries in 1821, but these efforts
Kalhana's Rajatarangini is the most famous historical were marred by the emphasis on Christian religious
poem which records the oldest and fullest history of education. Ishwar Chandra Vidyasagar was born on
the legendary kings of Kashmir as well as gives 26th September, 1820.
accounts of the Kashmiri kings of the historical period. 241. (a) The annexation of Awadh by Lord Dalhousie on the
Harshvardhan introduced a general dress befitting a pretext of maladministration left thousands of nobles,
king which includes a long coat. officials, retainers and soldiers jobless. Moreover, the
229. (d) annexation of Awadh adversely affected the sepoy's
230. (a) Peasant Communities were heterogeneous group on purse. He had to pay higher taxes on the land his family
the basis of caste and other caste like distinctions. held in Awadh.
231. (a) The Ghadar Party was an organization founded by 242. (c) Even though monotheism characterizes some of the
Punjabi Indians in the United States and Canada with hymns of Rig Veda, naturalistic polytheism and
the aim of securing India's independence from British monism can be discerned in the religion of the hymns
rule. After the outbreak of World War I, Ghadar party of Rig Veda. Like the rigvedic Aryans the ancient
members returned to Punjab to agitate for rebellion Iranians worshipped gods like Mitr,Vayu and
alongside the Babbar Akali Movement. Verutraghna. They also wore the scared thread and
232. (a) The ruins were brought to light by an engineer and worshipped fire etc. Initially sacrifices were simple,
antiquarian named Colonel Colin Mackenzie (Surveyor house hold duties performed by the oldest male
General of India) in 1800 AD. member of the house yajmana who acted as priest
233. (a) Rubber plantation in Vietnam was owned and and the oldest female member. yajmana patni(wife of
controlled by French and a small Vietnamese elite. the householder) who assisted in the act. The
agnihotra was a simple domestic sacrifice to be
234. (d) Most of the South Indians participated in the movement
performed daily by the head of a Dvija household in
except Karnataka which remained unaffected.
the morning and the evening. Other involved the
235. (a) Sutanuti was one of the three villages which were
participation of many different ritual specialists along
merged to form the city of Kolkata (formerly Calcutta)
with their assistants.
in India. Job Charnock, an administrator with the British 243. (c) MarthandaVarma is written by C.V. Raman Pillai which
East India Company is traditionally credited with the was published in 1891. It is presented as a historical
honour of founding the city. Fort William was romance recounting the history of Venad (Travancore)
established to protect the East India Company's trade during the final period of Rajah Rama Varma's reign
in the city of Calcutta, the principal town of the Bengal and subsequently to the accession of Marthanda
Presidency. In 1756, with the possibility of conflict Varma.
with Frenchforces, the British began building up the 244. (a) In 1893,Mahatma Gandhi accepted an offer from a firm
fort's strengths and defences. of Muslims to represent them legally in Pretoria,
236. (a) capital of Transvaal in the Union of South Africa. They
237. (c) Canonical texts are those that are linked to Gautama were actively involved in Gandhian political
Buddha in one way or another. Canonical texts movements in South Africa. Gandhi led a campaign in
comprise three different kind of texts - the sutras Cape Town against the ordinance on compulsory
(discourses), vinaya (discipline) and abhidharma registration and passes for Indians.
(analytical texts),together they are called the 'Three 245. (d) Educated muslims were the editors of Comrade,
Baskets' or Tipitaka (in Pali language) and Tripitaka Hamdard,Zamindar and Al Hilal. Maulana Mohammad
(in Sanskrit). Ali Jauhar launched the Urdu weekly Hamdard and in
238. (b) Basavanna was an Indian philosopher, statesman, English The Comrade in 1911. The founding editor of
Kannada poet and a social reformer in Karnataka. He this newspaper was Zafar Ali Khan, a poet, intellectual,
fought against the practice of the caste system which writer, Muslim nationalist and supporter of All India
discriminated against people based on their birth, Muslim League's Pakistan Movement. The Al-Hilal
gender and other rituals in Hinduism. Basava used was a weekly Urdu language newspaper established
Ishtalinga, an image of the Siva Linga, to eradicate by Maulana Abul Kalam Azad.
untouchability, to establish equality among all human 246. (a)
beings and as a means to attain spiritual enlightenment. 247. (c) Marthanda Varma was king of Travancore from 1729
Lingayats worshipped Shiva in the form of linga to 1758.The Travancore-Dutch War was fought
universal God or Ishtalinga. between the Dutch East India Company (VOC) and
239. (b) In 1809, he wrote his famous work 'Gift to Monotheists' the Indian kingdom of Travancore, culminating in the
in Persian, in which he put forward weighty arguments Battle of Colachel in 1741.Travancore became the
against belief in many gods and for the worship of a second most prosperous princely state in British India
single God. Ram Mohan Roy insisted on applying with reputed achievements in education, political
rationalism to Christianity too, particularly to the administration, public work and social reforms.
elements of blind faith in it. In 1820, he published his 248. (d) Individual freedom is not central tenet of Socialism.
History C-91

249. (c) The immediate cause of the rebellion was the 259. (c) Steel mill - State
restrictions, imposed on the pilgrims visiting the holy Kalinganagar - Odisha
places. The Sanyasis raided the English factories and Vijayanagar - Karnataka
collected contributions from the towns, leading to a Salem - Tamil Nadu
series of conflicts between the large bands of Durgapur - West Bengal
Sanyasis and the British forces. The rebellion started 260. (c) Alauddin Khalji was the second ruler of khalji dynasty
1750 onwards but took a violent turn since 1773 when (1296-1316 reigning period). He is famous for market
Warren Hastings assumed as the Governor-General reforms and price control. Actually he did so for
of Bengal. In 1773,he issued a proclamation banishing maintaining a large army. By controlling the items price,
all sannyasis from Bengal and Bihar. he became able to pay salary to soldiers.
250. (c) 251. (b) 261. (a) Mihrab - Direction towards the Kaba for prayer.
252. (a) Nawab Mohammed Abdul Ail(Prince of Arcot) is a Mimbar - Stepped pulpit
patron of music,art and literature. The musical trinity Khutba - Sermon
consists of Dikshitar, Tyagaraja and Syama Sastri Kibla - Arch
although, unlike the Telugu compositions of the 262. (a) Person Work
others, his compositions are predominantly in Uddanda - Mallikamaruta
Sanskrit. He also had composed some of his Kritis in Sayana - Sudhanidhi
Manipravalam. Yadavaprakasa - Vaijayanti
253. (a) About 30 British portrait painters trained in oil Bhavabhuti - Malatimadhava
paintings and 28 miniaturists travelled to India 263. (a) Ibn Battuta (1304-1377) was a Muslim scholar and
between 1770 and 1825 in search of commissions. traveller. He was born in Morocco. The book, Rihla is
Amongst the earliest European artists who visited account of his travelling throughout his life in different
India were John Zoffany, William Hodges, tilly Kettle, countries.
William and Thomas Daniells, Emily Eden and others. 264. (d) Surdas (1479-1586) was a saint, poet and musician,
254. (a) S.A Dange- The Socialist famous for devotional songs for Lord krishna, Sur
Muzaffar Ahmad- Navayug Sagar is a great composition (a poetic work) and two
Ghulam Hussian- Inquilab other works - Sur Sarawali and Sahitya Lahiri.
M. Singaravelu- Labour- Kisan Gazette 265. (c) Akbar (1556-1605) took interest in paintings. His court
255. (c) After Russian Revolution many left groups and trade had some famous painters as - Daswanth, Abdus
unions were formed in India on the ideologies of Samad, Basawan and Mir Sayyid Ali etc.
Marxism or communism. Labour Swaraj Party, 266. (c) Chhatrapati Shivaji (1630-1680) had two heirs for
Congress Labour Party, KirtiKisan Party, Labour Kisan throne. The eldest son Sambhaji was irresponsible
Party of Hindustan were based on communism person, towards subjects. So younger son Rajaram
ideology. By 1928, all these provincial parties were was crowned.
knitted into an all India organization named Workers 267. (a) Author Work
and Peasants Party (WPP). Somadeva - Kathasaritsagara
256. (b) The British became the major power in the Indian sub- Kalidasa - Malavikagnimitra
continent after the Treaty of Paris (1763)and began to Bhasa - Svapnavasavasdatta
show interest in Afghanistan as early as their 1809 Bilhana - Chaurapanchasika
treaty with Shuja Shah Durrani. It was the threat of 268. (d) Patanjali (2nd BC), an author of a commentary on
the expanding Russian Empire beginning to push for Panini's Ashtadhyayi was a great philosopher. He
an advantage in the Afghanistan region that placed compiled a famous book - Yogashutra.
pressure on British India, in what became known as 269. (c) King Region
the "Great Game". The Great Game set in motion the Shashanka - Bengal
confrontation of the British and Russian empires, Kharavela - Orissa
whose spheres of influence moved steadily closer to Simuka - Maharashtra
one another until they met in Afghanistan. Bhaskara - Assam
257. (b) River Tungabhadra was the lifeline of Vijayanagar 270. (c) Yaudheyas were related with Johiya Kshatriyas
Kingdom. It provided sustenance to the empire. residing near Muttan (Presently in Pakistan).
258. (b) Niccolo de conti(Venetian merchant ), Abdur 271. (d) Dandin (8th century), a Sanskrit grammarian was a
Razzaq(Persia), Afanacy Nikitin(Russian merchant ) great writer and poet. His works are related with prose
and Fernao Nuniz(Portuguese traveller) visited romance. His book Dashakumaracharita is a
Vijaynagar empire. composition of tale of ten princes.
EBD_7367
92
C- History

272. (a) The Governor General of India, Waren Hasting 285. (d)
(1772-1774) proposed a judicial plan - 286. (b) In medieval India, Iqta was a type of land revenue.
(i) Each district will have a civil and criminal court.
287. (c)
(ii) The judges will be assisted by native experts in Hindu
and Islamic laws. 288. (c) The temple of Hazara Rama belongs to Vijaynagara
273. (c) Indian National Congress was established on 28 Kingdom. It was built in the 15th century. by Devaraya II.
December, 1885 by A.O. Hume, a British Civil Servant 289. (a) Votive inscriptions (2nd century BC) describes about
to create a platform for civil and political discussion. offerings given to super natural forces to get their
Womesh Chander Benerji (1844 –1906), the president favour.
of Indian National Congress, presided over the first 290. (c) The Gupta coinage started with a remarkable series in
session of the Indian National Congress held at Gold issued by Chandragupta I, the third ruler of the
Bombay in 1885. dynasty.
274. (b) Jyotirao Phule (11 April 1827- 28 Nov 1890) on social 291. (a) An Agrahara is the name given to the Brahmin quarter
reformer, writer and theologist organised an of a heterogenous village or to any village inhabited
organisation - Satyashodhak Samaj Movement, He by Brahmins.
opined that Brahmins are descendent of alien Aryans.
292. (b) The primary product of PM's 'Pradhan Mantri Mudra
275. (d) Maulvi Ahmadullah Shah was a talukdar in Faizabad
Yojana' will be providing refinance for lending to micro
(UP). His talook was confiscated by the British
businesses. The interventions have been named
government. He was an important character in the
'Shishu', 'Kishor', and ' Tarun' to signify the stage of
battle of 1857. He was famous as Danka shah in public.
growth/development.
He took part in the battle of Chinhat. He fought in the
famous Battle of Chinhat in which the British forces 293. (b) According to Prayag Prashasti, Samudragupta was a
under Henry Lawrence were defeated great warrior as well as a great musician and poet.
276. (d) Binodini Dasi (1862-1941) was a renowned actress in 294. (c) Kamandaka's Nitisara is related to the subjects like
Kolkata. Her autobiography- Amarkatha was political science, etc.
published in 1913, this book reveals the contemporary 295. (a) The Lilavati is India Mathematician Bhaskara - II's
sketch of Bengali society. treatise on mathematics, written in 1150.
277. (a) Olympe de Gouges (7 May 1748 - 3 Nov 1793), a 296. (a) The followers of Guru Gorakshanath (also known as
French playwright and political activist, was famous Gorakhnath) were called 'yogis' (Jogis).
for writing in favour of rights of women. She opposed
297. (a) Satpura Thermal Power Plant is located in Betul district
male dominant society.
of Madhya Pradesh, India.
278. (a) Hind Swaraj or Indian Home Rule is a book written by Dhuvaran Thermal Power Station is a 220 MW coal
Mahatma Gandhi in which he expresses his views on plant in Gujarat state, India, owned by Gujarat State
Swaraj, modern civilisation, Religion, Nationality, Electricity Corporation.
Patriotism, Unity etc. Tanakpur Hydro Power Plant India is located at
279. (a) Amalananda Ghosh was the first archaeologist to Tanakpur near Banbassa, District Champawat,
identify similarities between a pre-harappan culture Uttarakhand, India.
and the mature Harappan culture. Dabhol Power Station is located at Ratnagiri district in
280. (b) Kailasanatha Temple at Ellora and the Shore temple at Maharashtra, India, about 160 kilometres south of
Mamallapuram are carved out from solid rocks. Mumbai.
281. (c) 'The deeds of Hamza" and 'The deeds of Babar' are not 298. (a) Sources of the 17th century refer to two kinds of
depicted in the Rajput Paintings. peasants, Khud-Kashta and Pahi-Kashta. Khud-
Kashta were the resident of the village in which they
282. (c) Tantraloka was written by Abhinavagupta. Which
held their land. Pahi-Kashta were nonresident
presents the teachings of the Kula and Trika systems.
cultivators, who belonged to some other villages, but
283. (b) The inscriptions on the pillar at Rummindei give vivid cultivated land elsewhere on contractual basis. They
details of Ashoka's conversion and the Nigalisagar worked under the temptation of favourable terms of
inscription records the fact that Ashoka had visited revenue or the compulsion of economic distress.
the Kongamana stupa. 299. (c) Zamindars during the Mughal era were petty
284. (c) Gaoseng Faxian Zhuan a Chinese Buddist monk (5th landholders in the villages, descendants of old ruling
Century) wrote a travelogue describing contemperary families who retained small portions of their ancestral
Buddist effect over Indian society. lands. They generally made collection from the
History C-93

individual peasants at rates fixed by custom or by information have confirmed the Indian origin of the
themselves and paid a fixed tax to the Government. Romas. Studies have supported the hypothesis that
The difference between his collections and the amount Romani shared a common origin with the Indo-Aryan
he paid to the state was his personal income. Thus languages of Northern India.ashtra because of prompt
Zamindars became exploitative. Due to this exploitative suppression by the British. In 1879, the Agriculturists
nature they did not receive any support of the Relief Act was passed which ensured that the farmers
peasantry in a large number of agrarian uprisings in could not be arrested and imprisoned if they were
North India in the seventeenth century. unable to pay their debts.
300. (c) King Ashoka never abolished capital punishment. 307. (b) Upari refers to peasants who cultivated the rented
Asoka adopted Buddhism in 9th year of his reign after lands in villages.
winning Kalinga. He was inspired by Nigrodha, a boy 308. (c) It was a water powered spinning frame device used for
monk. Later, he came in contact with Moggaliputta production of threads.
Tissa. Later his brother Tissa, queen Karuvaki also 309. (a) Food,clothing,housing and transportation were the
adopted Buddhism. In the Bhabru edict Asoka says four great needs of the people as identified by the
that he has full faith in Buddha, Sangha and Dhamma. Guomindang.
301. (b) Documents belonging to village panchayats from 310. (d) The earlier name of Tokyo was Edo which literally
Western India, Rajasthan, Gujarat and Maharashtra means estuary. In 1868 the name was changed to
tell us about the wives protested against the infidelity Tokyo.
of their husbands or the neglect of the wife and children 311. (b) Silappadikaram refers to the existence of Jaina
by the male head of the household. monasteries near city boundaries of Madurai. It also
The protesting women through these record petitions mentioned Kovalan and Kannagi's-the central
sent to the village panchayat asked for redressal and characters inclination towards the Jaina Path. Kavundi
justice. who accompanied them to Madurai was a Jaina
302. (a) The correct combination is, Bhikshuni.
Sumit Sarkar- The Swadeshi Movement in Bengal, 1903- 312. (b) Rakshasa vivah is marriage by capture or forceful
1908 abduction of bride.
Shahid Amin- Event, Metaphor, Memory: Chauri 313. (c) Although the basic town planning in Harappan cities
Chaura, 1922-1992 were followed but there were some significant
Ranajit Guha- A Rule of property for Bengal departures from the established norms. Like Banwali
Bipan Chandra- The Rise and Growth of Economic which is one of the towns of Harappan civilization,
Nationalism in India lacked the gridiron pattern of planning and the roads
303. (d) Most prominent form of Shiva in the Brihadeshvara were neither always straight nor do they necessarily
temple is Tripurantaka. Shiva's Tripurantaka form in cut each other at right angles.
different postures is depicted in sculptures. Standardized ratio of Brick size was the common feature
304. (b) Shankara founded the Sringeri, Puri, Badrikashrama of all Harappan cities.
314. (c) Ilregional intergovernmentalbert bill was introduced
and Dwaraka mathas at the four corners of the country.
in 1883 by Viceroy Ripon, to allow Indian judges and
305. (d) The Deccan uprising began at Supa village in the
magistrates the jurisdiction to try British offenders in
district of Poona. The farmers attacked places where
criminal cases. After opposition from Europeans in
many moneylenders and sahukars lived and burnt
Britain as well as India, the bill was withdrawn and
account books and looted grain shops. The farmers'
reintroduced after amendment saying that a jury of
main motive was to destroy the account books of the
50% Europeans was required if an Indian judge was
moneylenders.
to face a European.
The riots did not spread to the whole of Mahar
315. (d) Swaraj movement was initiated by Mahatma Gandhi
306. (b) It is believed that Gypsies or Romani people in Europe
which emphasized on self-rule or self-governance and
are of Indian descent. Romas are believed to be
referred to Gandhi’s concept Indian independence
descendants of specific groups from north-west India, from British rule. The movement had two meanings,
They are known differently as Tsigan in Russia, political and non-political. It was political in the sense
Bulgaria and Romania, Gitano in Spain, Tshingan in that it opposed being ruled by foreigners, and non-
Turkey and Greece, Zigeuner in Germany, Tsyiganes political in the sense that every person has right to be
or Manus in France, Tatara in Sweden, and Gypsies in ruled by himself or herself. Swaraj is a virtue which
Great Britain. Both linguistic evidence and genetic need time and patience to achieve.
EBD_7367
94
C- History

316. (c) Gandhi’s autobiography “My Experiments With 323. (a) In the first half of the 19th century Industrial
Truth,” mentions the Pranami sect and according to Revolution, poor nutrition, poor sanitation, lack of
him, the sect derived the best of both the Quran and proper medical care resulted in poor public health for
the Gita. Gandhi had interest in theosophy, and the majority of English people, including workers. In
studied books by theosophists such as Madame 1849, 10,000 workers died of cholera in Manchester.
Blavatsky and Annie Besant. Gandhi’s ideas were The average life span was 25 years or below.
close to Romanticism. He admired writings of poet 324. (c) George Washington was appointed as Commander in
William Wordsworth as they advocated nonviolence. Chief of the Continental Army at the Second
317. (b) Shiromani Gurdwara Parbandhak Committee (SGPC) Continental Congress on June 19, 1775. He assumed
was formed in November 15, 1920, for the management the office of the Continental Army in Cambridge on
of gurudwaras, Sikh places of worship in three states July 3, 1775. He resigned as Commander in Chief on
of Punjab, Haryana, and Himachal Pradesh and union December 23, 1783.
territory of Chandigarh. Shiromani Akali Dal began a 325. (a) Alexander Fyodoravich Kerensky served as head of
non-violent struggle against the British government the Russian Provisional government from July 17 to
for the control of the Gurdwaras. Jathas are separate October 17. Kerensky was eventually overthrown by
and independent religious orders that was formed to the Bolsheviks during the October Revolution. He
gain control over local gurdwaras. was forced to leave Russia and lived in exite until his
318. (b) Mahatma Gandhi gave his speech at the opening of death.
Banaras Hindu University in 1915 where he highlighted 326. (b) The Coercive Acts included a series of laws passed
his concerns over rich people’s lack of concern for by the British Parliament in 1774, including Boston
the poor and role of farmers. He said that farmers play Port Act, the Massachusetts Government Act, the
an important role in the society and acts as saviour Administration of Justice Act, and the Quartering Act of
for others. 1774, which were meant to punish American colonies
319. (a) Rowlatt Act, passed by Imperial Legislative Council for throwing a large tea shipment into Boston Harbor.
in Delhi on March 18, 1919, allowed certain political The Stamp Act of 1765 was passed by Parliament of
cases to be tried without juries and permitted arrest Great Britain to impose a direct tax on the colonies
of suspects without trial. Salt March (Dandi March), of British America. It was later repealed on March 18,
initiated by Mahatma Gandhi on March 12, 1930, 1766 after protest.
against British Salt Tax. Chauri Chaura incident 327. (c) The Poona Pact was an agreement between Dr.
(February 4,1922) involved clash between police and Babasaheb Ambedkar and Mahatma Gandhi signed
Indian protestors in British India. Second Round Table on 24 September 1932 at Yerwada Central
conference (1931) was held to discuss constitutional Jail in Pune, India, which asserted that there will be a
reforms. single Hindu electorate, with scheduled castes having
320. (d) Salt March (March 12, 1930) was initiated by Mahatma seats reserved within it. The Communal Award was
Gandhi against British Salt Tax and to promote salt made by the British Prime Minister Ramsay
production from sea water. The movement was widely MacDonald on 16 August 1932 allowing separate
covered by Indian, European, and American electorates in India, and it was supported by
newspapers and also saw participation of large number Ambedkar.
of women. The event shook the British, and for the 328. (d) All India Depressed Classes Association was formed
first time, they realized the power of Indians. In 1926 by M.C. Rajah for the welfare of depressed
321. (b) Arya Samaj, an Indian religious movement based on classes. Ambedkar did not attend the association and
Vedas and founded by Dayananda Saraswati on April was elected one of its vice-presidents.
7, 1875, was based in Punjab and western Uttar
Ambedkar became the President of the first All India
Pradesh states like, Aligarh, Varanasi, etc. It was
Depressed Classes Congress in 1930.
against caste system and advocated equality of men
329. (d) The Justice Party, a political party formed in
and women. Brahmo Samaj, founded by Raja
the Madras Presidency of British India in 1917 by T.
Rammohan Roy, was limited since it was popular only
M. Nair and P. Theagaraya Chetty to demand more
among elite and educated society.
representation for non-Brahmins in government. After
322. (c) The book A Comparison between Women and Men is
negotiations between Justice Party, Congress and the
written by Tarabai Shinde. It highlights the history of
British Government, 28 (3 urban and 25 rural) of the 63
women and gender relations in colonial India and also
general seats were reserved for non-Brahmins. In 1920,
explores changes in colonial society and their
Justice Party won 63 of the 98 seats.
implications for women.
History C-95

330. (b) The Ryotwari, a system to collect revenues from the 342. (a) Harsha was first responsible for the compilation of
cultivators of agricultural land, was initiated by ‘Nibandhas’ or digests of epic and Puranic texts.
Alexander Reed in Baramahal in 1792 and was 343. (a) Tamil epic Silappadikaram is an encyclopaedia of art
continued by Thomas Munro in Madras, Bombay, and music of the ancient Tamils. Ilango Adikal, author
parts of Assam and Coorgh provinces of British India. of Silappadikaram has described in detail the two great
Ryotwari was better than Zamindari System. rivers Kaveri and Vaigai that run through Choza and
331. (c) is the correct answer. Pandyan territories.
332. (a) is the correct answer. 344. (c) Harappan Culture doesn’t marks the earliest known
333. (a) Peasants of a remote and decrepit village, Phulaguri, use of iron as a medium for the art of sculpting.
345. (b) Bhaskaravarman belonged to Varman dynasty of
in central Assam’s Nagaon district had dealt the first
Assam. He is known for his alliance with
blow to the British in the Northeast in 1861. The revolt,
Harshavardhana against Shashanka, the first major
known as ‘Phulaguri dhawa’, in which a British official
ruler of Bengal (Karnasuvarna).
was killed and several police officers were injured,
346. (c) Atmiya Sabha was a philosophical discussion circle
was triggered by a ban imposed on opium cultivation
in India. The association was started by Ram Mohan
and a proposed taxation on betel leaf and nut.
Roy in 1815 in Kolkata. Brahmo Samaj, the theistic
334. (a) They brought Jahandar Shah to power. movement within Hinduism, founded in Calcutta in
335. (c) The jain ideals were already being circulated in the 1828 by Ram Mohun Roy. Prarthana Samaj was
seventh century BC by Parshava. Although Buddhism, founded by Atmaram Pandurang in 1867 when Kesab
and to a lesser extent Jainism, took account of the Chandra sen visited Maharashtra, with an aim to make
changes in material life and reacted against orthodoxy, people believe in one God and worship only one God.
neither of these sought to abolish the caste system. Arya samaj was founded by the sannyasi (ascetic)
Buddha held that nuns could attain spiritual liberation Dayananda Saraswati on 7 April 1875.
just like a monk and granted them an equal status in 347. (c) Dholavira is an archaeological site at Khadirbet in
the mendicant order. Bhachau Taluka of Kutch District, in the state of
336. (c) The Napoleonic Code is the French civil code Gujarat in western India, which has taken its name
established under Napoléon I in 1804. Tennis Court from a modern-day village 1 kilometre south of it.
Oath was taken on 20 June 1789. New Constitution of Rakhigarhi, or Rakhi Garhi, is a village in Hisar District
France was created in 1791. in the state of Haryana in India, situated 150 kilometers
337. (d) The Social Contract, by Jean-Jacques Rousseau, is a to the northwest of Delhi. Bhirrana, also Bhirdana and
1762 book in which Rousseau theorized about the best Birhana, is a small village located in Fatehabad District,
way to establish a political community in the face of in the Indian state of Haryana. Bhogavo River is a
the problems of commercial society, which he had river in Gujarat, India.
already identified in his Discourse on Inequality 348. (b) The status of women was redefined. They were entitled
(1754). to formal education and hence there were women
338. (d) All are correct. teachers, philosophers and doctors. Early marriage
was prohibited by law and they were given the right
339. (c) The Tughlaqabad Fort was built by the founder of the
to property.
Tughlaq Dynasty, Ghiyas-ud-din-Tughlaq in 1321.
349. (b) All are correct.
The Bara Gumbad was constructed in 1490 CE, during
the reign of the Lodhi dynasty. Qutab-ud-din Aibak, 350. (d) Sufi silsilas began to crystallise in different parts of
the Islamic world around the twelfth century. The word
the first Muslim ruler of Delhi, commenced the
silsila literally means a chain, signifying a continuous
construction of the Qutab Minar in 1200 AD, but could
link between master and disciple, stretching as an
only finish the basement. His successor, Iltutmush,
unbroken spiritual genealogy to the Prophet
added three more storeys, and in 1368, Firoz Shah
Muhammad. It was through this channel that spiritual
Tughlak constructed the fifth and the last storey.
power and blessings were transmitted to devotees.
Fatehpur Sikri was founded in 1569 by the great
Special rituals of initiation were developed in which
Mughal emperor Akbar. initiates took an oath of allegiance, wore a patched
340. (d) Urvara means fertile. Nadimatrika land is a land which garment, and shaved their hair.
depends on irrigation. Devamatrika land is a land which 351. (a) The tenth mandala of the Rigveda has 191 hymns.
depends on rainfall. Maru is a soil destitute of water. 10.85 is a marriage hymn, evoking the marriage of Sury?,
341. (d) KN Natarajan was the editor of the Indian Social daughter of Surya (the Sun), another form of Ushas,
Reformer. the prototypical bride.
EBD_7367
96
C- History

352. (b) Most of the South Indians participated in the non- the king who then became the source of spiritual
cooperation movement except Karnataka, which guidance for his subjects.
remained unaffected. There were some non-Brahman 362. (d) The Mahavamsa spoke in glowing terms of an Indian
lower-caste participation in Madras and Maharashtra. king Ashoka who had ordained his own son and
353. (c) The term barbarian originates from the Greek: daughter and sent them to Sri Lanka to spread the
Barbaros, which in turn originates from the Buddha's message.
incomprehensible languages of early Anatolian 363. (d) Ashoka was responsible for building many stupas all
nations that were heard by the Greeks as "bar..bar. In over northern India and the other territories under the
Ancient Rome, the Romans used the term towards non- Mauryan Dynasty in areas now known as Nepal,
Romans such as the Germanics, Celts, Gauls, Iberians, Pakistan, Bangladesh, and Afghanistan. Ashoka also
Thracians, Illyrians, Berbers, Parthians, and Sarmatians.
built stupas in regions where the people might have
354. (d) Hujwiri's conversation with the Yogis shows that he difficulty reaching the stupas that contained the
was impressed with their theory of the division of the Buddha's ashes. Stupas were also built in rural areas.
human body.
364. (d) The script was deciphered in 1837 by James Prinsep,
355. (a) The above statement was made by Pelsaert, a Dutch
an archaeologist, philologist, and official of the East
traveler in response to the poverty that has infiltrated
India Company
deeply into the lives of the people of Muhgal, India.
365. (b) Brahmadeya represented the grant of land either in a
The statement was made when he visited the
subcontinent in the early decades of the 17th century single plot or whole villages donated to Brahamans
and saw how miserably the people lived. For this abject by making them land-owners or land-controllers. The
poverty, Pelsaert held the State responsible. gift of land were mostly selected around the irrigation
356. (a) The Khojas are a group of diverse people who facilities such as tanks or lakes and were supposed to
converted to Islam in India and Pakistan. be operable to fulfill the needs of the donees.
Brahmadeya helped the expansion of agrarian economy
357. (a) The Nalayira Divya Prabandham is a collection of 4,000
Tamil verses (Naalayiram in Tamil means 'four and the emergence of urban settlements.
thousand') composed by the 12 Alvars, and was 366. (b) Seethalai Saathanaar is the author of the book
compiled in its present form by Nathamuni during the 'Manimekalai'. The book is a poem in 30 cantos. Its
9th - 10th centuries. story is a sequel of an epic 'Silappatikaram' that tells
358. (c) "A sound body means one which bends itself to the the story of the conversion to Buddhism of the
spirit and is always a ready instrument at its service." daughter of Kovalan and Madhavi.
This statement is attributed to Mahatma Gandhi. 367. (d) Sir Syed Ahmad Khan, was an Indian Muslim
359. (a) In 1904, Mohandas Gandhi, who had resided in nearby pragmatist, Islamic reformist, philosopher of nineteenth
Durban since 1893, established a small village-like century British India and the Father of Two Nation
settlement, Phoenix Settlement, on the north-western Theory. Born into a family with strong ties with Mughal
outskirts of Inanda. court, Syed studied the Quran and sciences within the
360. (a) Mahanavami Dibba is a structure in the Royal Centre court. Syed heavily critiqued the Indian National
in Vijaynagar Empire. The name come from the shape Congress. Sir Syed pioneered modern education for
of the platform and the Mahanavami festival which the Muslim community in India by founding the
was observed here. Muhammedan Anglo-Oriental College, which later
361. (a) Iranian Sufi Shihabuddin Suhrawardi developed the developed into the Aligarh Muslim University. The
idea of farr-i izadi. According to this idea, there was a curriculum at the Mohammedan Anglo-Oriental College
hierarchy in which the Divine Light was transmitted to blended Muslim theology and European empiricism.
CHAPTER
GEOGRAPHY
38
1. Among the following forest types in India, which one 10. River Luni originates near Pushkar and drains into which
occupies the largest area? [2007-I] one of the following? [2007-II]
(a) Tropical wet evergreen forest (a) Rann of Kachchh (b) Arbian Sea
(b) Tropical moist deciduous forest (c) Gulf of Cambay (d) Lake Sambhar
(c) Sub-tropical dry evergreen forest 11. The Great Barrier Reef is located in which country?
(d) Montane wet temperate forest [2007-II]
2. The National Forestry Action Plan aims to bring 33% of (a) Chile (b) Australia
land area in India under tree cover by which year? [2007-I] (c) USA (d) Indonesia
(a) 2008 (b) 2012 12. Which one of the following pairs is not correctly matched?
(c) 2016 (d) 2020 [2007-II]
3. The reservoir GB Pant Sagar is located on which river?
[2007-I] Headquarters of Railways Zone City
(a) Betwa (b) Ghaghara A. South-Western Hubli
(c) Kosi (d) Rihand B. North-Western Jaipur
4. Which one of the following national parks is located near
Chamoli? [2007-I] C. Western Mumbai
(a) Dudhwa National Park D. Central Jabalpur
(b) Great Himalayan Park
(c) Jim Corbett National Park 13. Which one of the following is correct? The Palghat Gap
(d) Nanda Devi National Park serves inland communications from [2007-II]
5. Which one of the following cities is located on the banks of (a) Madurai to Thiruvananthapuram
the River St Lawrence? [2007-I] (b) Chennai to Kochi
(a) Chicago (b) Detroit (c) Pune to Mumbai
(c) Montreal (d) Vancouver (d) Bangaluru to Mangalore
6. Match the following [2007-II] 14. Which one of the following states has built the famous
Gandhi Sagar across the Chambal River? [2007-II]
Lis t I Lis t II (a) Rajasthan (b) Maharashtra
(Centre of Economic Activity) (Country) (c) Uttar Pradesh (d) Madhya Pradesh
A . Ed mo nton 1. A u s tralia 15. What is 'Operation Kolleru' that was recently in the news?
[2007-II]
B. Kimberley 2. So u th A frica (a) A massive river linkage project
3. Cardiff 3. Un ited Kin gd om (b) A project to improve a wetland
4. Cairn s 4. Canada (c) A project to supply drinking water to a mega city
(d) A rural drinking water supply scheme of a southern state
Codes 16. Which country among the following has the largest cultivated
A B C D area of cotton in the world? [2007-II]
(a) 1 2 3 4 (a) Egypt (b) India
(b) 4 3 2 1 (c) Pakistan (d) USA
(c) 1 3 2 4 17. Consider the following statements [2007-II]
(d) 4 2 3 1 1. Dakshin Gangotri is India's second scientific manned
7. Which of the following straits is not in Asia? [2007-II] station in Antarctica after Maitri.
(a) Malacca Strait (b) Bass Strait 2. The National Centre for Antarctic and Ocean Research
(c) Formosa Strait (d) Molucca Strait is situated at Goa.
8. What is the other name for the equatorial rain farests? Which of the statements given above is/are correct?
[2007-II] (a) Only 1 (b) Only 2
(a) Lianos (b) Campos (c) Both 1 and 2 (d) Neither 1 nor 2
(c) Gran Chaco (d) Selvas 18. In the Cauvery River water dispute, which one of the
9. Which of the following seas/gulfs are connected by the Suez following groups of states are concerned? [2007-II]
Canal? [2007-II] (a) Kerala and Karnataka only
(a) The Mediterranean Sea and the Red Sea (b) Karnataka, Andhra Pradesh and Maharashtra
(b) Gulf of Oman and the Red Sea (c) Kerala, Karnataka, Tamil Nadu and Union Terrtory of
(c) The Mediterranean Sea and the Gulf of Oman Puducherry
(d) Persian Gulf and the Arabian Sea (d) Kerala, Goa, Karnataka and Tamil Nadu
EBD_7367
C- 98 Geography

19. Which one of the following districts does not have a gold (a) Christopher Columbus (b) John Cabot
field? [2007-II] (c) Hernando Cortez (d) De Leon
(a) Anantpur (b) Kolar 29. What is mixed farming? [2008-I]
(c) Raichur (d) Vishakhapatnam (a) Growing of several crops in a planned way
20. What is the approximate life expectancy for males in India (b) Growing rabi as well as kharif crops
on the basis of data for the year 2006? [2007-II] (c) Growing several crops and also rearing animals
(a) 56 years (b) 58 years (d) Growing of fruits as well as vegetables
(c) 64 years (d) 68 years 30. Match the following [2008-I]
21. Which one of the following is the capital of Cambodia? Listl List II
[2007-II]
(a) Ho Chi Minh city (b) Hanoi (Celestial Body) (Roman God)
(c) Semarang (d) Phnom Penh A. Mercury 1. God of the Underworld and Death
22. Match the following [2007-II] B. Pluto 2. God of Commerce. Eloquence and Skill
List l List II C. Mars 3. God of the Sea
(Ethnic Community) (State) D. Neptune 4. God of War
A. Apatani 1. Arunachal Pradesh Codes
A B C D
B. Bhutia 2. Madhya Pradesh (a) 3 1 4 2
C. Gond 3. Manipu (b) 2 4 1 3
D. Meiti 4 Sikkim (c) 3 4 1 2
(d) 2 1 4 3
Codes 31. Consider the following statements [2008-I]
A B C D 1. Dinosaurs roamed the Earth during the Mesozoic Era.
(a) 1 2 4 3 2. All Dinosaurs and other reptiles such as Ichthyosaurus
(b) 1 4 2 3 and Pterosaurus became extinct due to unknown
(c) 3 2 4 1 reasons in Triassic Period.
(d) 3 4 2 1 Which of the statement given above is/are correct?
23. Which one of the following countries is not situated in the (a) Only 1 (b) Only 2
region between the Caspian Sea and the Black Sea? (c) Both 1 and 2 (d) Neither 1 nor 2
[2007-II] 32. How many hours of daylight does the equator experience
(a) Armenia (b) Azerbaijan on September equinox? [2008-I]
(c) Georgia (d) Ukraine (a) 8 hours (b) 9 hours
24. Match the following [2007-II] (c) 10 hours (d) 12 hours
33. Consider the following statements [2008-I]
List l List II 1. Currently three waterways ham been declared as the
(Wild life) (State) national waterways of India.
2. The Sadiya-Dhubri stretch of river Brahmaputra is the
A. Wild Ass 1. Kerala longest national waterway.
B. Lion Tailed Macaque 2. Asom Macaque Which of the statement given above is/are correct?
C. Hoolock Gibbon 3. Gujarat (a) Only 1 (b) Only 2
(c) Both 1 and 2 (d) Neither 1 nor 2
A B C
(a) 1 3 2 DIRECTION (Q. 34) : The following question consists of two
(b) 2 1 3 statements one labelled as ‘Assestion (A)’ and the other as
(c) 3 1 2 ‘Reason (R)’. You are to examine these two statements carefully
(d) 3 2 1 and select the correct answer to this question using the codes
25. At which one of the following places, East-West corridor given below. [2008-I]
connecting Silchar and Porbandar and North-South corridor Codes
connecting Sri Nagar and Kanyakumari intersect each other? (a) Both A and R are true and R is the correct explanation of A
[2008-I] (b) Both A and B are true and R is not the correct explanation of A
(a) Jhansi (b) Agra (c) A is true, but R is false.
(c) Jabalpur (d) Nagpur (d) A is false, but R is true.
26. Which one of the following is included in the world list of 34. Assertion (A) The percentage of the Earth's surface covered
biosphere reserves by UNESCO? [2008-I] by India is 2.4%.
(a) Kinnaur Region (b) Spiti Valley Reason (R) On the Mercator's map of the world, India is
(c) Nallamalai Hills (d) Sunderbans shown much smaller than Canada. [2008-I]
27. Which one of the following groups of animals are primates? 35. Consider the following statements [2008-I]
[2008-I] 1. The Golden Quadrilateral connects the four major
(a) Giraffes and zebras (b) Kangaroos and koalas cities of Delhi, Mumbai, Bengaluru and Kolkata.
(c) Lemurs and lorises (d) Rabbits and hares 2. The North-South corridor will pass through Hyderabad.
28. Who among the following was the first European to encounter Which of the statements given above is/are correct?
the cocoa plant, from which chocolate is made? [2008-I] (a) Only 1 (b) Only 2
(c) Both 1 and 2 (d) Neither 1 nor 2
Geography C- 99
36. Which one of the following is the oldest oil refinery in India? 48. Which one of the following is the correct sequence of the
[2008-I] nuclear power plants of India in the increasing order of
(a) Haldia (b) Koyali their installed power generation capacity?
(c) Digboi (d) Mathura (a) Rawatbhata-Narora-Kaiga-Tarapur
37. Which of the following divides India into northern and (b) Narora-Kaiga-Rawatbhata-Tarapur
Southern parts? [2008-I] (c) Kaiga-Tarapur-Narora-Rawatbhata
(a) Equator (b) Tropic of cancer (d) Tarapur-Narora-Kaiga-Rawatbhata [2008-II]
(c) Tropic of capricorn (d) Arctic circle 49. Which one of the following has geographical position of
38. Which one of the following iron and steel plants was 0° latitude and 0° longitude? [2008-II]
established with the British collaboration? [2008-I] (a) In the South Atlantic Ocean
(a) Bhilai (b) Rourkela (b) In the Mediterranean Sea
(c) Bokaro (d) Durgapur (c) In Ghana, a West African country
39. Consider the following statements [2008-I] (d) At Greenwich Observatory in England
The South-West monsoon originates in India due to DIRECTIONS (Qs. 51-53) : The following questions consist of
1. low pressure in the Punjab plain. two statements, one labelled as 'Assertion (A)' and the other as
2. high pressure in areas South of India. 'Reason (R)'. You are to examine these two statements carefully
3. equatorial low being filled up by descending air current.
and select the correct answers to these questions using the codes
4. the Himalayas.
Which of the statements given above are correct? given below. [2008-II]
(a) 1 and 4 (b) 1 and 2 Codes
(c) 1 and 3 (d) 2 and 4 (a) Both A and R are true and R is the correct explanation of A
40. Which one of the following countries is not a UN (b) Both A and R are true, but R is not the correct explanation of A
member country? [2008-I] (c) A is true, but R is false
(a) Greece (b) Taiwan (d) A is false, but R is true
(c) Portugal (d) Australia 50. Assertion (A) The equatorial regions bulge outwards by
41. Which waterway separates India from Sri Lanka? about 21 km compared to poles.
(a) 8° Channel [2008-I] Reason (R) Earth's slow rotation reduces the effect of
(b) Palk Strait gravity around the equator.
(c) 10° Channel 51. Assertion (A) It is easier to swim in sea water compared
(d) Andaman Sea
to ordinary water.
42. Which one of the following pairs is correctly matched?
Reason (R) Density of sea water is higher than that of
Country Currency ordinary water.
A. Myanmar Baht 52. Assertion (A) In temperate cyclones, winds below from
B. Thailand Qyat the periphery towards its centre.
[2008-I] Reason (R) There is high pressure in the centre of
C. Cambodia Riel temperate cyclone.
D. Vietnam Rupiah 53. Which one of the following is correct?
43. Which one of the following is the correct order in which Great bear is a [2008-II]
the following cities are located from West to East? (a) Galaxy (b) Planet
(a) Alwar-Gorakhpur-Ranchi-Bhagalpur (c) Star (d) Constellation
(b) Bhagalpur-Ranchi-Alwar-Gorakhpur 54. Which one of the following is a global biodiversity
(c) Ranchi-Alwar-Bhagalpur-Gorakhpur hotspot in India? [2008-II]
(d) Gorakhpur-Bhagalpur-Ranchi-Alwar [2008-I] (a) Western Ghats (b) Western Himalayas
44. Which one of the following peaks is the highest? (c) Eastern Ghats (d) Northern Himalayas
(a) Nanda Devi (b) Kanchenjunga 55. In which one of the following places is the headquarters
(c) Godwin Austin (d) Nanga Parbat [2008-I] of North-Eastern Railway located? [2008-II]
45. With which one of the following countries, India shares (a) Guwahati (b) Gorakhpur
maximum length of the border? [2008-I] (c) Kolkata (d) Bhuvneshwar
(a) Bangladesh (b) Pakistan 56. The boundary between North Korea and South Korea is
(c) China (d) Nepal marked by which one of the following? [2008-II]
46. Which one of the following diagrams shows the direction
(a) 16 Parallel (b) 29 Parallel
and duration of wind? [2008-I]
(c) 38 Parallel (d) 49 Parallel
(a) Cartogram (b) Climogram
57. Match the following [2008-II]
(c) Ergograph (d) Star diagram
47. Which one of the following is correct? List I List II
The mean distance from the Sun to the Earth is called a/
(Place) (Industry)
an [2008-II]
(a) Light year A. Muzaffarnagar 1. Cotton textiles
(b) Parallactic second B. Adoni 2. Sugar
(c) Astronomical unit
(d) Angstrom C. Ranchi 3. Sports goods
D. Jalandhar 4. Engineering goods
EBD_7367
100
C- Geography
Codes Codes
A B C D A B C D
(a) 2 1 4 3 (a) 3 4 1 2
(b) 1 3 4 2 (b) 2 1 4 3
(c) 2 3 4 1 (c) 3 1 4 2
(d) 1 4 3 2 (d) 2 4 1 3
58. Which of the following tributaries of the Ganga System 70. For which one of the following is Sualkuchi famous?
flows Northwards? [2008-II] [2008-II]
(a) Kosi (b) Ghaghara (a) Bird sanctuary (b) Temple city
(c) Gandak (d) Sone (c) Silk centre (d) Hill station
59. For which one of the following is planimeter used? 71. In which one of the following countries is the volcano
[2008-II] 'Guallatiri" located? [2008-II]
(a) To know the surface area on a map (a) Peru (b) Chile
(b) To know the distance on a map (c) Tanzania (d) Ecuador
(c) To know the direction on a map 72. The Island of Honshu is located in which one of the
(d) To know the height of places on a map following countries? [2008-II]
60. The city of Nasik is situated on the bank of which one of (a) Indonesia (b) Malaysia
the following rivers? [2008-II] (c) Japan (d) South Korea
(a) Krishna (b) Mandovi 73. Which one of the following rivers flows into the Arabian
(c) Godavari (d) Tapti Sea? [2008-II]
61. Rihand Valley Project is located in which one of the following (a) Indravati (b) Godavari
states? [2008-II] (c) Cauvery (d) Narmada
(a) Odisha (b) Gujarat 74. Match the following [2008-II]
(c) Himachal Pradesh (d) Uttar Pradesh List I List II
62. Which one of the following states has the largest forest
area to its total land area? [2008-II] A. Slate 1. Igneous rock
(a) Mizoram (b) Arunachal Pradesh B. Lignite 2. Metamorpric rock
(c) Sikkim (d) Jammu and Kashmir C. Bauxite 3. Non-ferrous mineral
63. What is the correct order of occurrence of the following
places as one moves from East to West? [2008-II] D. Granite 4. Sedimentary rock
(1) Murshidabad (2) Gorakhpur Codes
(3) Bhopal (4) Bhavnagar A B C D
Select the correct answer using the codes given below (a) 1 3 3 2
(a) 2, 4, 3, 1 (b) 1, 3, 4, 2 (b) 2 4 3 1
(c) 1, 2, 3, 4 (d) 4, 3, 2, 1 (c) 2 3 4 1
64. Which one of the following Indian states does not have a (d) 1 4 3 2
common international border with Bangladesh? [2008-II] 75. Which one of the following places is famous for production
(a) Manipur (b) Paschim Banga of railways coaches? [2008-II]
(c) Tripura (d) Asom (a) Nasik (b) Kapurthala
65. 28.38° N and 77.12° E are the respective latitude and longitude (c) Kanpur (d) Kochi
of which one of the following places? [2008-II] 76. The hill station of Coonoor is located in which one of the
(a) Jaipur (b) Delhi followng states? [2008-II]
(a) Karnataka (b) Andhra Pradesh
(c) Lucknow (d) Allahabad
(c) Kerala (d) Tamil Nadu
66. Mist is a result of which one of the following [2008-II] 77. Which one of the following statements is not correct?
(a) Condensation (b) Evaporation [2008-II]
(c) Sublimation (d) Saturation (a) All meridians run in a ture North-South direction
67. In which one of the following states is Ranganathittu Bird (b) Meridians are spaced farthest apart at the equator and
Sanctuary located? [2008-II] converge to common points at the poles
(a) Tamil Nadu (b) Kerala (c) All meridians are always parallel to one another
(c) Karnataka (d) Andhra Pradesh (d) An indefinite number of meridians may be drown on a
68. Pattadakal group of monuments are located in which one of globe
the following states? [2008-II] 78. In which of the following State is Kakrapar Nuclear Power
(a) Bihar (b) Odisha Station located? [2008-II]
(c) Madhya Pradesh (d) Karnataka (a) Karnataka (b) Tamil Nadu
69. Match the following [2008-II] (c) Maharashtra (d) Gujarat
79. Madhya Pradesh shares its border with how many States?
Lis t I Lis t II [2009-I]
A . A u ran gabad 1. A urov ille (a) 3 (b) 4
B. Guwahati 2. Sarn ath (c) 5 (d) 6
80. In which one of the following places is the Forest Survey of
C. Pud uch erry 3. Ellora temples India (FSI), a national organisation engaged in forest cover
D. Varan as i 4. Nilanch al hills mapping, forest inventory and training in the field of remote
sensing and GIS, located? [2009-I]
Geography 101
C-

(a) Dehradun (b) Itanagar 89. Which one of the following lakes in India has the highest
(c) Ahmedabad (d) Aizawl water salinity? [2009-I]
81. Which one of the following is the longest national highway (a) Dal (b) Chilila
in India? [2009-I] (c) Wular (d) Sambhar
(a) NH 2 (b) NH 7 90. Match the following [2009-I]
(c) NH 8 (d) NH 31 List I List II
82. Which one of the following states is the largest producer of River Tributary
black pepper in India? [2009-I] A. Brahamaputra 1 . Musi
(a) Tamil Nadu (b) Kerala B. Krishna 2 . Tawa
(c) Karnataka (d) Andhra Pradesh
83. The shaded area in the map given below is the major C. Narmada 3 . Bhavani
producer of which one of the following? [2009-I] D. Cauveri 4 . Dikhow
Codes
A B C D
(a) 4 2 1 3
(b) 4 1 2 3
(c) 3 2 1 4
(d) 3 1 2 4
91. Which one of the following is the oldest mountain range in
India? [2009-I]
(a) Himalayas (b) Aravalli
(c) Satpura (d) Nilgiri
92. Which one of the following longitudes determines the
(a) Cotton (b) Groundnut Indian sandard time? [2009-I]
(c) Wheat (d) Mustard (a) 85.5° E (b) 86.5° E
84. Which one of the following oil fields of India is the oldest (c) 84.5° E (d) 82.5° E
and still producing oil? [2009-I] 93. Match the following [2009-I]
(a) Bombay High (b) Digboi
List I List II
(c) Ankleshwar (d) Naharkatiya
85. Which one of the following States of India is the largest (Pass) (State)
producer of lignite coal? [2009-I] A. Zoji La Pass 1. Sikkim
(a) Maharashtra (b) Gujarat B. Bara Lacha Pass 2. Uttarakhand
(c) Madhya Pradesh (d) Tamil Nadu
86. Match the following [2009-I] C. Jelep La Pass 3. Himachal Preadesh
List l List II D. Niti Pass 4. Jammu and Kashmir
(Multi purpose River Projects) (Hydel Power Station) Codes
A B C D
A. Rihand 1. Hirakund (a) 4 1 3 2
B. Gandak 2. Balmikinagar (b) 2 3 1 4
(c) 4 3 1 2
C. Chambal 3. Pipri (d) 2 1 3 4
D. Mahanadi 4. Kota 94. In which one of the following islands of India is an active
volcano found? [2009-I]
Codes (a) Car Nicobar Island
A B C D (b) Nancowry Island
(a) 3 4 2 1 (c) Barren Island
(b) 1 2 4 3 (d) Maya Bunder Island
(c) 3 2 4 1 95. Which one of the following pairs is not correctly matched?
(d) 1 4 2 3 [2009-I]
Country Currency
87. Which of the following sequences of the oil refineries of India
as they occur from South to North is correct? [2009-I] A. Brazil Reals
(a) Kochi-Mangalore-Mumbai-Koyali B. China Yuan
(b) Koyali-Mumbai-Mangalore-Kochi
(c) Kochi-Mumbai-Mangalore-Koyali C. Mexico Pesos
(d) Mangalore-Kochi-Mumbai-Koyali D. Thailand Ringgit
88. Which one of the following pairs is not correctly matched? 96. In which one of the following is higher percentage of carbon
[2009-I] found? [2009-I]
River City (a) Lignite coal (b) Peat coal
(c) Bituminous coal (d) Anthracite coal
A . Gomuti Luckn o w
97. Which one of the following is the tide produced as a
B. Saryu A yo dh ya consequence of Moon and Sun pulling the Earth in the same
C. A lakanan da Bad rin ath direction? [2009-I]
(a) Spring tide (b) Neap tide
D. Narmad a Satn a (c) High tide (d) Low tide
EBD_7367
102
C- Geography

98. Which one of the following is an object with such a strong 108. A collection of gas dust which appears as a bright ball of
gravitational field that even light cannot escape from its light in the sky with a glowing tail is called [2009-II]
surface? [2009-I] (a) Star (b) Comet
(a) Neutron Star (b) White Dwarf (c) Constellation (d) Galaxy
(c) Black hole (d) Supernova Star 109. Which of the following factors is/are responsible for high
99. Because of which one of the following factors, clouds do concentration of jute mills in the Hugli basin?
not precipitate in deserts? [2009-I] Select the correct answer using the code given below
(a) Low pressure (b) Low humidity [2009-II]
(c) High wind velocity (d) High temperature 1. Nearness to coal fields.
DIRECTIONS (Qs. 100-103) : The following questions consist 2. Convenient dry climate for spinning and weaving.
of two statements, one labelled as 'Assertion (A)' and the other (a) Only 1 (b) Only 2
as 'Reason (R)'. You are to examine these two statements carefully (c) Both 1 and 2 (d) Neither 1 nor 2
and select the correct answers to these questions using the codes 110. High spring tides occur at new Moon and full Moon because
given below. [2009-I] the [2009-II]
(a) Moon and Earth are at right angles
Codes (b) Sun and Moon are at right angles
(a) Both A and R are true and R is the correct explanation of A (c) Sun, Earth and the Moon are in straight line
(b) Both A and R are true, but R is the correct explanation of A (d) Sun and Earth are at right angles
(c) A is true, but R is false 111. Leaching is the maximum in the soil type of [2009-II]
(d) A is false, but R is true (a) Laterite (b) Red
100. Assertion (A) Water in an open pond remains cool even on (c) Regur (d) Desert
a hot summer day. 112. Match the following [2009-II]
Reason (R) Water on heating evaporates and heat energy
gets converted into latent heat. List l (Place) List II (Famous for)
101. Assertion (A) Physiological density is preferable to
arithmetic density as an index of population density. A. Balaghat 1. Oil field
Reason (R) Physiological density is based on arable land B. Katni 2. Iron ore
while arithmetic density is based on total area. C. Singrauli 3. Manganese
102. Assertion (A) Despite low evaporation and stable stratification
of the atmosphere, salinity is high in polar regions. D. Kalol 4. Bauxite
Reason (R) Sea water freezes leaving the remaining water 5. Coal
saline than before.
103. Assertion (A) Venus is the brightest object in the sky after Codes
the Sun. A B C D
Reason (R) Venus is the second planet from the Sun in our (a) 1 2 4 3
solar system. (b) 3 4 5 1
(c) 3 5 4 1
104. Consider the following regions of India
1. Western Ghats 2. Aravalli Hills (d) 1 2 5 3
3. Eastern Himalayas 113. Sal trees are the typical species of [2009-II]
Which of the above is/are biodiversity hot spot/hot spots? (a) Tropical rain forest (b) Tropical monsoon forest
[2009-I] (c) Taiga forest (d) Tundra forest
(a) Only 1 (b) 1 and 3 114. Which one of the following regions is culturally closer to
(c) 2 and 3 (d) All of the above India but ethnically closer to China? [2009-II]
105. Which one of the following is categorised as millet? (a) Central Asia (b) West Asia
[2009-I] (c) Polynesia (d) South-East Asia
(a) Wheat (b) Rice 115. Consider the following statements
(c) Sorghum (a) Maize 1. In the month of July, the Inter Tropical Convergence
106. Consider the following statements Zone is located in the Indo Gangetic plain.
1. National parks are a special category of protected areas 2. Northen Inter Tropical Convergence Zone is the
of land and sea coasts where people are an integral zone of clouds and heavy rainfall.
part of the system. Which of the statement given above is/are correct?
2. Sanctuaries are concerned with conservation of [2009-II]
particular species. (a) Only 1 (b) Only 2
3. Biosphere reserves are connected with the habitat of a (c) Both 1 and 2 (d) Neither 1 nor 2
particular wild animal. 116. The ageing index in India has increased from 14% in 1951
Which of the statements given above is/are correct? to > 21% in 2001. With respect to this which of the following
[2009-I] effects is/are correct?
(a) 1,2 and 3 (b) Only 2 1. Decline in the proportion of child population.
(c) 1 and 2 (d) 1 and 3 2. Increase in the proportion of old population.
107. Which one of the following zones of the atmosphere is rich Select the correct answer from the codes given below
in ozone gas? [2009-I] [2009-II]
(a) Mesosphere (b) Troposphere (a) Only 1 (b) Only 2
(c) Stratosphere (d) Ionosphere (c) Both 1 and 2 (d) Neither 1 nor 2
Geography 103
C-

117. Which water body separates Australia from New Zealand? 128. What is the correct sequence of the location of the following
[2009-II] sea ports of India from South to North? [2009-II]
(a) Cook Straits (b) Megallan (a) Cochin, Thiruvananthapuram, Calicut, Mangalore
(c) Tasman Sea (d) Great Barrier Reef (b) Calicut, Thiruvananthapuram, Cochin, Mangalore
118. In a significant climate responsive project, in which one of (c) Thiruvananthapuram, Cochin, Calicut, Mangalore
the following places recently an abandoned thermal power (d) Thiruvananthapuramr Calicut, Mangalore, Cochin
plant has been converted into a mega solar power enerating 129. In the map given above four petrochemical centres are
station which poised to give a huge fillip to India's renewable marked with numbers. Match them with the following four
energy ambition? [2009-II] places and select the correct answer using the code given
(a) Talcher (Odisha) (b) Namrup (Asom) below [2009-II]
(c) Jamuria (Paschim Bangal) (d)Jaisalmar (Rajasthan)
DIRECTIONS (Qs. 119-120) : The following questions consist of
two statements, one labelled as the 'Assertion (A)' and the other as
'Reason (R)'. You are to examine these two statements carefully and
select the answers to these questions using the codes given below.
[2009-I] 1
2
Codes
(a) Both A and R are true and R is the correct explanation of A 3
(b) Both A and R are true, but R is not the correct explanation of A
(c) A is true, but R is false 4
(d) A is false, but R is true
119. Assertion (A) The Himalayan meadows are suitable for A. Koyali B. Trombay
transhumance. C. Jamnagar D. Mangalore
Reason (R) In these areas transport facility to move from A B C D
one place to another is good. [2009-II] (a) 2 3 1 4
120. Assertion (A) Convectional rains occur during pre- (b) 2 1 3 4
monsoon summer in India.
(c) 4 2 3 1
Reason (R) Such rains occur due to adiabatic cooling.
(d) 4 3 2 1
[2009-II]
130. Which one of the following countries is located South of
121. Consider the following statements [2009-II]
1. Jim Corbett National Park is the oldest national park of the equator? [2009-II]
india. (a) Cameroon (b) Sudan
2. It was one of the nine tiger reserves created at the launch (c) Nigeria (d) Rwanda
of the Project Tiger in 1973. 131. Consider the following statements
3. Initially it was named as 'Hailey National Park'. 1. Suez Canal is an important link between developed
Which of the statements given above are correct? countries and developing countries.
(a) 1 and 2 (b) All of these 2. It joins the Mediterranean Sea with the Gulf of Suez
(c) 2 and 3 (d) 1 and 3 3. It is not a sea level canal.
122. What is the name of the strait where 'Adams bridge' is Which of the statement given above is/are correct?
situated? [2009-II] [2009-II]
(a) Bering Strait (b) Cook Strait (a) 1 and 2 (b) 1 and 3
(c) Palk Strait (d) Taiwan Strait (c) 2 and 3 (d) Only 1
123. Radio waves are reflected back to Earth from the 132. The channel separating the Andaman Island from the
[2009-II] Nicobar Islands is known as [2009-II]
(a) Troposphere (b) Exosphere (a) Coco channel (b) 10° channel
(c) Stratosphere (d) Ionosphere (c) Duncan passage (d) somboraro channel
124. The pacific Islands from new Guinea South East-wards to 133. Match the following [2009-II]
the Fiji Islands' group is called [2009-II]
(a) The Polynesia (b) The Mellanesia List I List II
(c) The Micronesia (d) The Autralasia (River) (Source)
125. Israel has common borders with [2009-II]
(a) Lebanon, Syria, Jordan and Egypt A. Ganga 1. Amarkantak
(b) Turkey, Syria, Jordan and Yemen B. Son 2. Gaumukh
(c) Lebanon, Syria, Turkey and Jordan
(d) Cyprus, Turkey, Jorden and Egypt C. Godavari 3. Mahabaleshwar
126. Which continent of the world does not have a desert? D. Krishna 4. Trimbakeshwer
[2009-II] Codes
(a) Australia (b) Europe A B C D
(c) Asia (d) North America (a) 1 2 4 5
127. The hill range that separates the State of Manipur from (b) 2 1 3 4
the State of Nagaland is known as [2009-II] (c) 4 3 1 2
(a) Arakan hills (b) Patkai hills (d) 2 1 4 3
(c) Barail hills (d) Manipur hills
EBD_7367
104
C- Geography

134. At high altitudes, pressure cooker is preferable for cooking, (c) approximate centre of a group of related Earthquakes
because the boiling point of water [2010-I] (d) point on the surface directly above where the rupture
(a) reduces due to higher atmospheric pressure along the fault zone occur
(b) reduces due to lower atmospheric pressure 145. Which of the following seas are enclosed?
(c) increases due to reduced gravitational force 1. Andaman Sea 2. Arab Sea
(d) reduces due to increased ozone content in the atmosphere 3. Sea of Azov 4. Bering Sea
135. Which one of the following does not have a heavy water Select the correct answer using the codes given below
plant? [2010-I] [2010-I]
(a) Narora (b) Sriharikota (a) 1 and 2 (b) 3 and 4
(c) Kakrapar (d) Kota (c) 2 and3 (d) 1 and 4
136. Which one of the following trains/railways passes through 146. Match the following [2010-I]
Germany, France, Austria, Hungary and Romania? [2010-I]
(a) Trans-Siberia (b) Cape-Cairo Listl List II
(c) Orient Express (d) Union and Central Pacific (Sphere of the (Main Constituent of
137. Which one of following projects of the NHPC (National
Hydroelectric Power Corporation) has the largest power Earth) the Sphere)
generation capacity (installed)? [2010-I] A. Lithosphere 1 Living objects
(a) Baira-Siul (b) Chamera-1 B. Hydrosphere 2 Mixture of gases
(c) Loktak (d) Salal
138. Deltas are common in India for the East-flowing river C. Atmosphere 3 Water
systems, whereas they are nearly absent on the West coast, D. Biosphere 4 Soil
because West-flowing rivers [2010-I]
(a) are few Codes
(b) have lesser water volume and carry less run-off silt A B C D A B C D
(c) originate in dry areas (a) 1 2 3 4 (b) 4 2 3 1
(d) originate largely in the Western Ghats and have short (c) 1 3 2 4 (d) 4 3 2 1
distance to cover to the sea 147. Bolivar is the monetary unit of [2010-I]
139. Which of the following statements with regard to rainfall (a) Venezuela (b) Brazil
in India is/are correct? (c) Bolivia (d) Belarus
1. Most of the rainfall in India is due to the South- 148. The time at Cairo is 2 hours ahead of Greenwich. Hence, it is
West monsoon. located at [2010-I]
2. In South India, rainfall decreases away from the
(a) 30°W longitude (b) 30°E longitude
Eastern Chats.
Select the correct answer using the code given below (c) 28°E longitude (d) 28°W longitude
[2010-I] 149. Which is the correct arrangement of the following rivers
(a) Only 1 (b) Only 2 from North to South? [2010-I]
(c) Both 1 and 2 (d) Neither 1 nor 2 (a) Godavari, Penner, Cauveri, Periyar
140. Tsunamis are not caused by [2010-I] (b) Penner, Godavari, Periyar, Cauveri
(a) hurricanes (b) Earthquakes (c) Godavari, Cauveri, Penner, Periyar
(c) undersea landslides (d) volcanic eruptions (d) Cauveri, Godavari, Periyar, Penner
141. If the movement of the Earth's crust or a major climatic 150. Consider the following features of a border district of India
change makes an old stream young, it is called [2010-I] and identify the district using the code given below Situated
(a) consequent stream (b) rejuvenation at the height of 10000 feet, which is enchanted with scenic
(c) subsequent stream (d) aggradation beauty, the district is surrounded by two nations in the North
142. The jet streams are [2010-I] and South-West. River Nyamjangchu runs through it. Potato,
(a) wind systems with a pronounced seasonal reverse at
maize and millet are the main crops of the district. Besides
a direction
(b) winds blowing from the subtropical high-pressure belts monasteries, tourist attractions of the district and its
towards the subpolar low-pressure belts surrounding places include Sella Pass, PTSO, War memorial,
(c) narrow meandering bands of swift winds which blow Jaswant Garh etc. Losar is one of the prominent festivals of
in the midlatitudes near the tropopause and encircle the district. [2010-I]
the globe (a) Lohit (b) Tawang
(d) winds blowing from the subpolar low-pressure belts (c) Kangra (d) West Sikkim
towards the subtropical high-pressure belts 151. Which among the following Rajdhani trains covers the
143. The concept of sustainable development relates to longest distance? [2010-I]
[2010-I] (a) 2433 Chennai Central (b) 2431 Trivandrum Central
(a) consumption levels (b) exhaustible resources (c) 2435 Dibrugarh Town (d) 2429 BangluruCityJunction
(c) social equity (d) intergenerational equity 152. Which one of the following places is not located on the
144. An Earthquake epicentre is the [2010-I]
bank of river Ganga? [2010-I]
(a) point where the seismograph is located
(b) point within the Earth where the movement along the (a) Uttarkashi (b) Kanpur
fault occurs (c) Fatehpur (d) Bhagalpur
Geography C-105
153. On the above map of India, the shaded districts are those Select the correct answer using the code given below
in which a particular tribal group constitutes more than 50% (a) 1, 2 and 3 (b) 1 and 2
of the tribal population. What is that particular tribal group? (c) 2 and 3 (d) 1 and 3
[2010-I] 161. Which among the following states of India have common
borders with Pakistan? [2010-II]
(a) Jammu and Kashmir, Himachal Pradesh, Punjab and
Rajasthan
(b) Punjab, Jammu and Kashmir, Rajasthan and Gujarat
(c) Jammu and Kashmir, Punjab, Haryana and Rajasthan
(d) Puniab, Himachal Pradesh, Gujarat and Rajasthan
162. Which of the following factors influence ocean currents?
1. Rotation of the Earth. 2. Air pressure and wind.
3. Ocean water density 4. Revolution of the Earth
Select the correct answer using the codes given below
(a) Gonds (b) Bhils [2010-II]
(c) Nagas (d) Santhals (a) 1 and 2 (b) 1, 2 and 3
154. Which one of the following is not associated with (c) 1 and 4 (d) 2, 3 and 4
monsoon climate in India?

You might also like